Banco Kap. - CK 2

You might also like

Download as docx, pdf, or txt
Download as docx, pdf, or txt
You are on page 1of 1099

MASTER THE BOARDS CK 2

Table of Contents
Cirugia............................................................................................................................................2
Psiquiatria...................................................................................................................................105
Ginecologia.................................................................................................................................172
Pediatria......................................................................................................................................290
Interna.........................................................................................................................................464
Diagnostic.................................................................................................................................1364
Simulacros................................................................................................................................1403
Cirugia
A 79-year-old man is brought to the emergency department from a nursing home because of
altered mental status of recent onset. He has deteriorated significantly in the last 3 weeks. His
past medical history is significant for hypertension, osteoarthritis, and frequent falls that led to
his move into a nursing home. Current medications include amlodipine and aspirin daily. His
temperature is 38.5°C (101.3°F), pulse is 90/min, respirations are 18/min, and blood pressure is
100/50 mm Hg. The patient is oriented only to person. Mucous membranes are moist.
Cardiopulmonary examination shows clear lungs, normal S1 and S2, regular rhythm, and no
murmurs. Neurologic examination shows delayed tendon reflexes. Laboratory studies show:
WBC 18,500 mm3
Neutrophils 85%
Lymphocytes 5%
Hemoglobin 13 g/dL
Hematocrit 39%
Platelets 300,000 mm3
Na+ 145 mEq/L
K+ 4.6 mEq/L
Cl –
110 mEq/L
HCO3– 23 mEq/L
BUN 25 mg/dL
Creatinine 0.5 mg/dL
Glucose 60 mg/dL
Urine culture shows Escherichia coli sensitive to quinolones, and IV ciprofloxacin is started. His
fever subsides and his white blood cell count improves, although his mental status does not; he
remains oriented only to person. Which of the following is the most appropriate next step in
diagnosis?
 Correct Answer Image A.CT scan of the head
 Incorrect Answer ImageB.Duplex ultrasound of the legs
 Incorrect Answer ImageC.Stop all home medications
 Incorrect Answer ImageD.Ultrasound of the kidneys and bladder
 Incorrect Answer ImageE.Water restriction

A 2-year-old boy is shot in the arm in a drive-by shooting. His brachial artery is partially
transected, and there is copious bleeding. Emergency medical technicians are able to control the
site of bleeding by local pressure, and the child stops losing blood. However, he is hypotensive
and tachycardic. Fluid resuscitation is urgently needed, but several attempts at starting peripheral
intravenous lines are unsuccessful. Which of the following is the best alternative route in this
situation?
 Incorrect Answer ImageA.Central line via subclavian vein
 Inmri sca Answer ImageB.Gastric fluid administration via nasogastric tube
 Correct Answer Image C.Intraosseous cannulation in the proximal tibia
 Incorrect Answer ImageD.Percutaneous femoral vein cannulation
 Incorrect Answer ImageE.Saphenous vein cutdown

A 71-year-old man who underwent a right hemicolectomy 36 hours ago complains of pain at the
site of his Foley catheter. His temperature is 37.8°C (100.0°F), pulse is 88/min, respirations are
22/min, and blood pressure is 123/79 mm Hg. Urinalysis returned as negative for leukocyte
esterase, white blood cells, and nitrates. There has been no change in his vital signs. His
abdomen is mildly distended and soft, with a clean dressing in place. On physical examination of
his penis, the Foley catheter is located in the meatus. The glans penis is exposed, edematous, and
dusky with a ring of tight foreskin around the portion of the penis proximal to the glans. The
entire penis is tender to palpation. Which of the following is the best course of management?
 Incorrect Answer ImageA.Change the Foley catheter
 Incorrect Answer ImageB.Cover the patient with broad-spectrum antibiotics
 Incorrect Answer ImageC.Elevate the penis on a towel and apply an ice-pack
 Correct Answer Image D.Manually reduce the patient's foreskin
 Incorrect Answer ImageE.Order topical lidocaine to be applied to the penis

An obese 40-year-old woman has made multiple attempts to lose weight for the past 5 years with
no lasting success. She says she can’t resist sweets. She has, at times, lost up to 18 kg (40 lb),
which she has always regained. Her body mass index is 42 kg/m2. She is moderately
hypertensive and is on a diuretic and an angiotensin-converting enzyme inhibitor. She has
osteoarthritis of both knees. She also has severe obstructive sleep apnea and uses continuous
positive airway pressure during sleep. What is the best option for her to achieve long-term
weight loss and morbidity reduction? 
 Incorrect Answer ImageA.Appetite suppressive medication
 Incorrect Answer ImageB.Counseling aimed at reducing the underlying cause of her
overeating
 Incorrect Answer ImageC.Laparoscopic gastric band
 Incorrect Answer ImageD.Low-fat diet
 Correct Answer Image E.Sleeve gastrectomy

A 33-year-old woman is involved in a high-speed automobile collision. She arrives at the


emergency department gasping for breath. Her lips are cyanotic and she has flared nostrils. There
are bruises over both sides of the chest and tenderness to palpation of the chest wall. Her blood
pressure is 60/45 mm Hg and pulse is 160/min and feeble. Her neck and forehead veins are
distended. She is diaphoretic, and subcutaneous emphysema is present in her lower neck and
upper chest. Her left hemithorax has no breath sounds and is hyper-resonant to percussion with
tracheal deviation to the right. Which of the following are the most likely diagnosis and etiology?
 Incorrect Answer ImageA.Air embolism from tracheobronchial injuries
 Incorrect Answer ImageB.Flail chest caused by multiple rib fractures
 Incorrect Answer ImageC.Massive intrapleural bleeding from torn intercostal vessels
 Incorrect Answer ImageD.Massive mediastinal bleeding from a ruptured aorta
 Correct Answer Image E.Tension pneumothorax likely caused by a lung puncture from
broken ribs

A 67-year-old man was treated for a polymicrobial urinary tract infection (UTI) 2 weeks ago and
is now seen in follow up. He complains of the recent onset of the passage of "bubbles" with his
urine. His medical history is significant for diverticulitis approximately 1 year ago. He denies
any recent weight loss, blood in the stool, or a recent change in bowel habits. Physical
examination is normal and stools are guaiac-negative. What is the most likely diagnosis?
 Incorrect Answer ImageA.Bladder cancer
 Correct Answer Image B.Colovesical fistula
 Incorrect Answer ImageC.Emphysematous cystitis
 Incorrect Answer ImageD.Enterovesical fistula
 Incorrect Answer ImageE.Polycystic kidney disease

A 64-year-old homeless woman is brought to the emergency department because of a tender,


swollen right breast. The patient was in the emergency room two weeks ago with the same issue
and was given antibiotics. She states that she is back because the condition is not improving. On
physical examination, the right breast is erythematous, tender, and noted to have diffuse
thickening and dimpling of the affected skin. In addition, the right axilla contains multiple
immobile, hard masses. Core biopsy specimens of the breast reveal highly undifferentiated
infiltrating ductal carcinoma, and assays for estrogen and progesterone receptors are negative.
Which of the following is the most appropriate next step in management?
 Correct Answer Image A.Neoadjuvant chemotherapy followed by mastectomy
 Incorrect Answer ImageB.Palliative mastectomy
 Incorrect Answer ImageC.Prolonged course of antibiotics
 Incorrect Answer ImageD.Radical mastectomy with extended lymph node dissection
 Incorrect Answer ImageE.Tamoxifen therapy

A 45-year-old man who weighs 65 kg (143 lbs) has sustained second- and third-degree burns
over both of his lower extremities as a result of his pants catching fire while lighting the
barbeque. Physical examination shows that virtually all of the skin has been burnt off. Most of
the affected area appears to be a third-degree burn, with some areas of second-degree burns.
Pulses are palpable with regular rate and rhythm in the bilateral dorsalis pedis arteries. Plans are
made to wash all the burned areas in the operating room, but in addition to that, which of the
following should be done during the first hour of his treatment?
 Incorrect Answer ImageA.Administer 1 liter of hypotonic normal saline
 Correct Answer Image B.Administer 1 liter of IV Ringer's lactate
 Incorrect Answer ImageC.Intubate his trachea and place him on a respirator
 Incorrect Answer ImageD.Perform bilateral escharotomies
 Incorrect Answer ImageE.Start parenteral nutrition

A 33-year-old woman is found to have a palpable thyroid nodule during a routine maintenance
examination. A sonogram confirms the presence of a solid, 1.5-cm nodule in the right lobe of the
thyroid gland without any suspicious lymph nodes in the neck. Fine-needle aspirate (FNA)
cytology shows a follicular tumor that is otherwise unspecified. At surgery, a frozen section
shows follicular carcinoma. No enlarged jugular and paratracheal lymph nodes are identified.
Which of the following is the most appropriate treatment?
 Incorrect Answer ImageA.Enucleation of the tumor
 Incorrect Answer ImageB.Right thyroid lobectomy
 Incorrect Answer ImageC.Subtotal thyroidectomy
 Correct Answer Image D.Total thyroidectomy plus postoperative radioactive iodine
 Incorrect Answer ImageE.Total thyroidectomy, radical neck dissection, and postoperative
radioactive iodine

A 29-year-old woman is brought to the emergency department after being in a motor vehicle
collision. Emergency medical personnel report she was a front-seat passenger and was wearing
her seat belt. The car had been traveling on the highway at approximately 55 mph when another
car swerved in front of it, resulting in a head-to-side collision. In the emergency department, the
patient appears ill and complains of severe epigastric pain. Her pulse is 105/min, respirations are
20/min, and blood pressure is 138/62 mm Hg. Glasgow coma scale score is 14. Physical
examination shows abdominal distension and tenderness to palpation in all 4 quadrants.
Ecchymosis is seen across her abdomen in the pattern of the seat belt. Which of the following is
the most appropriate next step in the management of this patient?
 Incorrect Answer ImageA.Chest radiograph
 Correct Answer Image B.CT scan of the abdomen and pelvis
 Incorrect Answer ImageC.Diagnostic peritoneal lavage
 Incorrect Answer ImageD.Exploratory laparotomy
 Incorrect Answer ImageE.Focused abdominal sonography for trauma (FAST) exam

A 32-year-old woman comes to the office because of bilateral hand numbness and tingling. She
states that the symptoms began approximately 9 months ago and have gotten worse. Overall, the
symptoms are worse at night and when she is working. She also states that her symptoms are
affecting her work performance as a computer programmer. Her past medical history is
unremarkable. Physical examination shows tingling of the fingers when the bases of the wrists
are tapped. There is mild weakness of right thumb abduction. No atrophy is seen. Deep tendon
reflexes are normal. Wrist radiographs are unremarkable. Which of the following is the most
appropriate next step in management?
 Incorrect Answer ImageA.CT of the wrist
 Correct Answer Image B.Electromyography and nerve conduction studies
 Incorrect Answer ImageC.MRI of the brain
 Incorrect Answer ImageD.MRI of the cervical spine
 Incorrect Answer ImageE.Surgical exploration

A 62-year-old woman comes to the outpatient clinic because of a new breast mass. On physical
examination, she has a hard mass deep to the nipple and areola of her left breast. The mass
occupies most of the breast, but the breast is freely movable from the chest wall. There is no
dimpling or ulceration of the skin over the mass, and careful palpation of the axilla is negative
for lymphadenopathy. A mammogram is negative for a mass in the right breast. An ultrasound-
guided core needle biopsy is performed which shows infiltrating ductal carcinoma. Chest
radiograph and liver function tests are normal. She has no symptoms suggestive of brain or bone
metastasis. Which of the following should be offered to this woman?
 Incorrect Answer ImageA.Lumpectomy only
 Incorrect Answer ImageB.Lumpectomy with sentinel lymph node biopsy and
postoperative radiation
 Correct Answer Image C.Mastectomy and sentinel lymph node biopsy
 Incorrect Answer ImageD.Radical mastectomy with complete axillary dissection
 Incorrect Answer ImageE.Simple mastectomy

A 37-year-old woman undergoes a lumpectomy and axillary node sampling for a 3 cm


infiltrating ductal carcinoma in the upper outer quadrant of her left breast that was diagnosed by
core biopsies. The pathology report of the surgical specimen is received 3 days after the
operation. It indicates that all margins around the tumor are clear and that all 4 of the removed
sentinel nodes have metastatic tumor. The tumor is reported to be estrogen and progesterone
receptor-negative. Which of the following should further therapy most likely include? 
 Incorrect Answer ImageA.Conversion to modified radical mastectomy
 Correct Answer Image B.Radiation therapy and chemotherapy
 Incorrect Answer ImageC.Radiation therapy and tamoxifen
 Incorrect Answer ImageD.Radiation therapy only
 Incorrect Answer ImageE.Tamoxifen only

A 68-year-old man is brought to the emergency department after he was in a high-speed


automobile accident where his car hit a tree head-on. He was wearing his seat belt and his airbag
deployed. He is alert and complains of chest and mild back pain. His past medical history is only
significant for hypertension and hypercholesterolemia. He has no allergies. His temperature is
37.0ºC (98.6ºF), pulse is 70/min, respirations are 22/min, and blood pressure is 100/70 mm Hg.
Physical examination shows multiple rib fractures, possible spinal fracture, and a large bruise
over his left flank. A series of radiographs are ordered, including the chest, spine, and pelvis.
Chest x-ray shows a widened mediastinum. Which of the following is the best next step in
management?
 Incorrect Answer ImageA.Aortogram
 Correct Answer Image B.CT angiogram of the chest
 Incorrect Answer ImageC.Emergency thoracotomy
 Incorrect Answer ImageD.MRI scan of the chest
 Incorrect Answer ImageE.Transesophageal echocardiogram (TEE)

A 65-year-old man is brought to the emergency department after slipping on the ice and striking
his head on the pavement. He was briefly unconscious but awoke promptly and was able to get
up with the help of onlookers. He complains of a throbbing sensation in the left side of his head.
Although he has no apparent injuries, he is noted to have left temporal tenderness on palpation.
No open wound, soft-tissue edema, or hematoma is observed. On neurologic examination, the
pupils are equal and reactive to light bilaterally. Cranial nerves are intact. He has no extremity
motor or sensory deficits, and gait is normal when he uses the cane (which he uses because of
severe osteoarthritis). There is some limited short-term memory, but he freely admits that this is
not new. CT scan of the head is pending. While waiting for the stretcher, he suddenly loses
alertness and becomes unresponsive. Although hemodynamically stable, he ultimately needs to
be intubated. Which of the following is the most appropriate next step in management?
 Incorrect Answer ImageA.Antiepileptic medication
 Incorrect Answer ImageB.Further imaging with MRI scan
 Incorrect Answer ImageC.Observation
 Incorrect Answer ImageD.Pharmacologic blood-pressure control
 Correct Answer Image E.Surgical evacuation of an intracranial hematoma

A 65-year-old woman is admitted to the hospital for constant, severe abdominal pain that has
worsened over the previous week. She has no other associated symptoms, such as nausea or
vomiting. She has had a constant desire to urinate, but when she tries, only a small amount of
bloody urine is discharged. The patient has a 90 pack-year history of smoking, although she
claims to have quit 6 months ago. A bladder ultrasound in the emergency department shows a
mass located at the bladder neck and an estimated 700 mL of urine in the bladder. Which of the
following is most likely to be detected on imaging the patient's genitourinary system? 
 Incorrect Answer ImageA.Air-fluid levels
 Correct Answer Image B.Bilateral hydronephrosis
 Incorrect Answer ImageC.Bladder dyskinesis
 Incorrect Answer ImageD.Unilateral hydronephrosis
 Incorrect Answer ImageE.Ureteral stenosis

A 60-year-old man who works as a police officer comes to the physician because of intense,
disabling, sharp heel pain every time his foot strikes the ground. The pain is worse in the
mornings, preventing him from putting any weight on the heel. On physical examination, pulse is
87/min, respirations are 14/min, blood pressure is 132/90 mm Hg, and BMI is 30 kg/m2. He
shows exquisite tenderness to direct palpation over the anteromedial aspect of the heel.
Furthermore, the pain is exacerbated when the toes are dorsiflexed. X-ray films appear normal.
Which of the following is the most likely diagnosis?
 Incorrect Answer ImageA.Calcaneal stress fracture
 Incorrect Answer ImageB.Fracture of the posterolateral talar tubercle
 Incorrect Answer ImageC.Osteoarthritis of the ankle joint
 Correct Answer Image D.Plantar fasciitis
 Incorrect Answer ImageE.Posterior Achilles tendon bursitis

A 58-year-old man is brought to the emergency room via ambulance because of chest pain. He
suddenly felt a dull, crushing chest pain that radiated to his left arm. He immediately dialed 911.
On physical examination, he appears moribund and is sweating profusely. An electrocardiogram
shows ST-segment elevation and the patient is transferred to the cardiac catheterization
laboratory for a coronary angiogram and subsequent percutaneous coronary intervention with
stent placement. Twenty-four hours later, the patient is sitting in the cardiac care unit when his
blood pressure drops to 95/55 mm Hg. His pulse is 118/min and respirations are 30/min.
Cardiovascular examination shows difficulty in hearing the heart sounds and (click media file).
Lung sounds are clear. The blood pressure is re-checked with a manual sphygmomanometer,
which varies between 110/55 mm Hg and 95/55 mm Hg. Which of the following is the most
likely diagnosis?
 Correct Answer Image A.Cardiac tamponade
 Incorrect Answer ImageB.Complete heart block
 Incorrect Answer ImageC.Congestive heart failure
 Incorrect Answer ImageD.Right ventricular infarct
 Incorrect Answer ImageE.Valvular rupture

A 58-year-old man comes to the emergency department because of a sudden onset of severe leg
pain and warmth for 4 hours. He has a history of obesity, type 2 diabetes mellitus, and coronary
artery disease. His temperature is 39.4°C (102.9°F), pulse is 100/min, respirations are 15/min,
and blood pressure is 160/95 mm Hg. Examination shows erythema over the right lower
extremity. The range of motion is limited by pain. There is extreme tenderness to palpation over
the entire right lower extremity. Hemoglobin A1c level is 11.2%. Which of the following is most
likely the definitive cure for this patient's condition?
 Incorrect Answer ImageA.Broad-spectrum IV antibiotics
 Incorrect Answer ImageB.Fine-needle aspiration for culture
 Incorrect Answer ImageC.IV saline and insulin drip
 Incorrect Answer ImageD.Oral antibiotics and observation
 Correct Answer Image E.Surgical debridement

An 85-year-old woman with diabetes who is on dialysis for chronic renal failure develops
bilateral orbital and facial pain, headache, and sudden loss of vision in the right eye. When
examined 2 days later, her right eyelid is red and swollen. She also has a complete loss of vision
and motion of the right eye and partial loss of vision of the left eye. The mucosa of the nasal
passages is swollen and necrotic with black discoloration. MRI scan shows soft-tissue swelling
of the nasal mucosa, sinuses, and orbital tissues, with no evidence of cavernous sinus thrombosis
or retro-orbital cellulitis. A biopsy specimen of the nasal and sinus mucosa shows thrombosed
vessels and multiple broad, nonseptate hyphae with right-angle branches. Which of the following
is the most appropriate next step in management? 
 Correct Answer Image A.Amphotericin B and surgical debridement
 Incorrect Answer ImageB.Corticosteroids
 Incorrect Answer ImageC.Craniotomy with orbital decompression
 Incorrect Answer ImageD.Fluconazole
 Incorrect Answer ImageE.Urgent sinus decompression

A 63-year-old man is admitted to the hospital for lower extremity cellulitis. His past medical
history is notable for ischemic heart disease and congestive heart failure, with ejection fraction
25%. Despite several days of antibiotics, his condition fails to improve. One morning, he is
found to be somnolent and lethargic. His blood pressure is 85/55 mmHg and the extremities are
cold and clammy. The patient is admitted to the ICU, where he is administered IV fluids,
phenylephrine, and his antibiotics are adjusted. Over the next day and a half, his blood pressure
stabilizes; however, he develops diarrhea and diffuse abdominal pain. CT scan of the abdomen is
ordered, which demonstrates small-bowel thickening and foci of intramural gas. He is also noted
to have a lactic acidosis. What is the most likely diagnosis? 
 Incorrect Answer ImageA.Antibiotic-associated diarrhea
 Correct Answer Image B.Nonocclusive mesenteric ischemia
 Incorrect Answer ImageC.Superior mesenteric artery (SMA) embolus
 Incorrect Answer ImageD.Thrombosis of the celiac axis
 Incorrect Answer ImageE.Venous thrombosis

A 60-year-old man with a history of diabetes and hypertension comes to the emergency
department because of 2 days of lower abdominal pain. The pain is mostly on the left side and
has not migrated during the previous 48 hours. He has no appetite and has had 2 episodes of non-
bloody diarrhea during this period. His physical examination is notable for a fever of 39.4°C
(102.9°F), pulse of 123/min, and blood pressure of 105/64 mm Hg. His abdominal examination
is significant for left lower quadrant tenderness to light palpation and for voluntary guarding. His
laboratory studies show a white blood cell count of 19,000/mm3 with 88% neutrophils. CT scan
of the abdomen and pelvis is obtained and reveals significant sigmoid diverticulitis with a large
left lower quadrant phlegmon and at least several abscesses (all less than 3 cm) within that large
inflammatory mass. After fluid resuscitation and IV antibiotics, what would be the most
appropriate next step in management?
 Incorrect Answer ImageA.Begin dopamine infusion
 Incorrect Answer ImageB.CT-guided percutaneous drainage of the abscesses
 Incorrect Answer ImageC.Discharge home on oral antibiotics
 Incorrect Answer ImageD.Laparotomy and Hartmann procedure
 Correct Answer Image E.Monitor closely and continue broad-spectrum antibiotics

A 63-year-old man comes to the clinic because he has noticed blood in his urine. Although the
bleeding is usually painless, there are times when he suffers from painful urination and an
overwhelming urgency to urinate. Additionally, he reports that the frequency of his urination has
increased to the point that, on some days, he uses the bathroom many times per hour. When the
bleeding occurs, it occurs throughout the urine flow. His past medical history is unremarkable.
He denies any occupational exposures, but he is a heavy smoker, with a 60 pack-year smoking
history (2 packs/day for 30 years). His temperature is 37.0°C (98.6°F), pulse is 80/min,
respirations are 20/min, and blood pressure is 139/85 mm Hg. Abdominal examination reveals
no evidence of guarding or rebound tenderness. Digital rectal examination is normal. Urinalysis
reveals 3+ hematuria but is negative for protein, glucose, leukocyte esterase, and nitrates.
Microscopic examination shows many red blood cells, but no abnormal sediment or bacteria.
Which of the following is the most appropriate next diagnostic step?
 Incorrect Answer ImageA.CT scan of the pelvis
 Correct Answer Image B.Cystoscopy
 Incorrect Answer ImageC.Transvesical needle biopsy
 Incorrect Answer ImageD.Ultrasound
 Incorrect Answer ImageE.Urine flow cytometry

A 25-year-old man comes to the clinic after being struck on the right ear by a fist. He reports
right ear pain, decreased hearing on the right side, and describes a feeling of "fullness" in that
ear. He denies any loss of consciousness or other injuries. He also denies any dizziness or
imbalance, headaches, or visual changes. His past medical history is otherwise unremarkable.
Physical examination shows a healthy-appearing man in no acute distress. The patient's right
external ear appears mildly ecchymotic, but there are no lacerations and no evidence of a
hematoma. All cranial nerves, including the facial nerve, appear intact bilaterally. He has grossly
intact hearing on both sides, although he reports a decrease in hearing on the right side.
Otoscopic examination shows a central perforation of the right tympanic membrane of
approximately 25%. Although there is some erythema and ecchymosis of the tympanic
membrane, there is no evidence of active bleeding or otorrhea. A Weber tuning fork test would
be expected to reveal which of the following findings? 
 Incorrect Answer ImageA.Complete deafness of the right ear
 Incorrect Answer ImageB.Lateralization of sound to the left
 Correct Answer Image C.Lateralization of sound to the right
 Incorrect Answer ImageD.Moderate right-sided sensorineural hearing loss
 Incorrect Answer ImageE.The tuning fork heard equally in each ear

A 71-year-old farmer from West Texas has a non-healing, punched-out, 2-cm ulcer with heaped-
up borders over his left temple. The ulcer has been slowly growing over the past 3 years. There
are no enlarged lymph nodes in the head and neck. He comes to the office now for evaluation.
Which of the following is the most appropriate next step in diagnosis?
 Incorrect Answer ImageA.Full-thickness biopsy from the center of the lesion
 Correct Answer Image B.Full-thickness biopsy that includes the edge of the lesion
 Incorrect Answer ImageC.Resection with 1 cm circumferential margins
 Incorrect Answer ImageD.Response to a trial of radiation therapy
 Incorrect Answer ImageE.Scrapings and culture of the ulcer base

A 14-year-old boy slides down a banister and crashes into a large knob at its base, striking his
genital area. He presents to the emergency department with acute testicular pain and a significant
enlargement of the scrotum to approximately the size of a grapefruit. The scrotal skin appears
ecchymotic. He is able to void normally, and there is no blood in the urine. Rectal examination is
unremarkable. Findings from which of the following would be best to determine further
therapy? 
 Incorrect Answer ImageA.Aspiration of scrotal contents
 Incorrect Answer ImageB.Retrograde cystogram
 Incorrect Answer ImageC.Retrograde urethrogram
 Correct Answer Image D.Scrotal ultrasound
 Incorrect Answer ImageE.Surgical exploration of the scrotum

Two weeks after receiving an allogeneic bone marrow transplant for treatment of acute
myelogenous leukemia, a 45-year-old man develops fever, intractable diarrhea, a generalized
maculopapular rash, and a nonproductive cough. Chest x-ray shows bilateral interstitial
infiltrates. The patient dies of overwhelming sepsis and multiple-organ failure. Autopsy
investigations reveal cytomegalovirus pneumonia and extensive single-cell necrosis in the
intestinal epithelium and skin. This complication of bone marrow transplantation is mediated by
which of the following cells? 
 Incorrect Answer ImageA.B lymphocytes of bone marrow graft
 Incorrect Answer ImageB.Leukemic cells
 Incorrect Answer ImageC.Natural killer cells of recipient
 Correct Answer Image D.T lymphocytes of bone marrow graft
 Incorrect Answer ImageE.T lymphocytes of recipient
A thin, 7-year-old boy lost control of the bicycle he was riding and fell to the ground, sustaining
a deep abdominal contusion as he landed on the handlebar. He is evaluated at the emergency
department and found to be stable and relatively asymptomatic except for some localized
abdominal pain in the epigastrium where there is some bruising from the handlebar impact. A CT
scan without contrast is negative. The boy is sent home, but the next day he returns with diffuse,
constant abdominal pain. He is lying on the stretcher without moving, and his abdominal
examination reveals generalized tenderness and muscle guarding. There is a deeply ecchymotic
area where he was hit by the handlebar. His temperature is 37ºC (98.6ºF), pulse is 110/min,
respirations are 28/min, and blood pressure is 110/80 mm Hg. Laboratory studies show a
hemoglobin of 14 g/dL, leukocyte count of 9,500/mm3, serum amylase level of 550 U/L, and
serum lipase level of 260 U/L. What is the most appropriate next step in management?
 Incorrect Answer ImageA.Abdominal ultrasound
 Incorrect Answer ImageB.Diagnostic peritoneal lavage
 Incorrect Answer ImageC.Focused abdominal sonography for trauma (FAST scan)
 Correct Answer Image D.Repeat CT scan of the abdomen and pelvis with contrast
 Incorrect Answer ImageE.Splenectomy
 Incorrect Answer ImageF.Urgent laparoscopy

A 19-year-old man is brought to the emergency department after sustaining multiple injuries in a
high-speed automobile collision. There is a pneumothorax on the left side, for which he has a
chest tube placed. Over the next several days, the left chest tube has a large, persistent air leak,
and daily chest x-rays show that his collapsed left lung is not expanding. The patient is not on a
respirator. All of his other injuries have been treated appropriately. Which of the following is the
most likely cause of these findings? 
 Incorrect Answer ImageA.Air embolism
 Incorrect Answer ImageB.Injury of the lung parenchyma
 Correct Answer Image C.Injury to a major bronchus
 Incorrect Answer ImageD.Insufficient suction being applied to the chest tube
 Incorrect Answer ImageE.Tension pneumothorax

A 61-year-old male comes to the emergency department with colicky abdominal pain,
obstipation, and vomiting of 3 days' duration. On physical examination, his abdomen is
moderately distended and he has high-pitched, hyperactive bowel sounds and a tender,
discolored, 5-cm groin mass. On direct questioning, he explains that he has had that bulge for
many years but has always been able to "push it back in" when he lies down. For the past 3 days,
however, he has been unable to do so. He has a temperature of 38.9ºC (102ºF) and a white blood
cell count of 15,500/mm3. Which of the following is the most appropriate next step in
management at this time?
 Incorrect Answer ImageA.A sonogram of the mass
 Correct Answer Image B.Emergent surgical intervention
 Incorrect Answer ImageC.Manual reduction of the hernia, followed by a period of
observation
 Incorrect Answer ImageD.Nasogastric decompression and IV fluids
 Incorrect Answer ImageE.Urgent sigmoidoscopy
An 8-year-old boy falls on his right hand with the arm extended, and he breaks his elbow by
hyperextension. X-ray films show a supracondylar fracture of the humerus. The fracture is
reduced and immobilized. Which of the following has the greatest associated risk with this type
of fracture? 
 Incorrect Answer ImageA.Growth plate damage
 Incorrect Answer ImageB.Instability that requires open reduction and internal fixation
 Incorrect Answer ImageC.Insufficient remodeling
 Incorrect Answer ImageD.Malunion
 Correct Answer Image E.Vascular and nerve injuries

A 58-year-old woman with diabetes is brought to the emergency department via ambulance
because of chest pain. She was playing with her grandchildren when she suddenly felt a dull pain
that radiated to her left arm. She reports marked shortness and breath and nausea. Her
temperature is 37.8ºC (100.4ºF), pulse is 128/min, and blood pressure is 168/94 mm Hg. She
appears in great distress and is pale and sweating profusely. She is breathing 100% oxygen via a
facemask. She becomes chest pain-free on aspirin, intravenous heparin, beta-blockers,
intravenous nitrates, and eptifibatide. After subsequently improving, she is transferred to the
cardiac care unit and then the stepdown unit. She makes an uneventful recovery over the next 5
days. Which of the following is the next best step in management?
 Incorrect Answer ImageA.Coronary angiography
 Incorrect Answer ImageB.Coronary artery bypass grafting
 Incorrect Answer ImageC.Discharge on aspirin, metoprolol, atorvastatin, and enalapril
 Incorrect Answer ImageD.Maximal stress test
 Incorrect Answer ImageE.Percutaneous coronary intervention
 Correct Answer Image F.Submaximal stress test

A 50-year-old obese man is brought to the emergency department by paramedics after his wife
found him on the floor at home. The wife reports that the patient had a "cold" for the past week.
She informs you that her husband is a diabetic and has high blood pressure, though she can't
remember which medications he takes. His temperature is 36.6ºC (97.8ºF), pulse is 102/min,
respirations are 18/min, blood pressure is 105/65 mm Hg, and urinary output for the past 30
minutes is 50 mL. On physical examination, the patient is unresponsive to touch or voice. Head
and neck examination shows low jugular venous pressure, dry oral mucosa, and decreased skin
turgor. The patient is given a 1 L bolus of normal saline. Laboratory studies show: 
Na+ 142 mEq/L
K+ 3.2 mEq/L
Cl −
106 mEq/L
HCO3- 16 mEq/L
Glucose  1260 mg/dL
BUN  35 mg/dL
Creatinine  2.8 mg/dL
Which of the following is the most appropriate next step in management? 
 Incorrect Answer ImageA.Continue normal saline
 Correct Answer Image B.Start half normal saline together with potassium replacement IV
 Incorrect Answer ImageC.Start normal saline together with IV dobutamine
 Incorrect Answer ImageD.Start normal saline together with IV regular insulin
 Incorrect Answer ImageE.Start normal saline together with potassium replacement IV

A 62-year-old man comes to the emergency department with a large, swollen, tender ankle. He
had been running to catch a bus when he fell on his inverted foot and twisted his ankle. Physical
examination shows swelling and abnormal angulation of the ankle. AP, lateral, and mortise x-
rays show displaced fractures of both malleoli. Which of the following would be the preferred
form of treatment? 
 Incorrect Answer ImageA.Closed reduction and casting
 Correct Answer Image B.Closed reduction and splinting, then open reduction and internal
fixation
 Incorrect Answer ImageC.Fusion of the ankle joint
 Incorrect Answer ImageD.Immediate, emergency open reduction and internal fixation
 Incorrect Answer ImageE.Replacement with a metal prosthesis
 Incorrect Answer ImageF.Skeletal traction

A 74-year-old man comes to his primary care physician for an annual examination. The patient
reports that he has had significant problems with maintaining an erection. He is able to achieve
an erection, but he is not able to sustain it for sufficient time to complete intercourse. His past
medical history is significant for hypertension, coronary artery disease, congestive heart failure,
osteoarthritis, and gastroesophageal reflux disease. He takes atenolol, isosorbide dinitrate,
lisinopril, furosemide, ibuprofen, and omeprazole. He denies tobacco and alcohol use. His
temperature is 36.8°C (98.3°F), pulse is 72/min, respirations are 16/min, and blood pressure is
124/72 mm Hg. His physical examination is unremarkable. Complete blood count, basic
metabolic panel, lipid profile, and serum testosterone are within normal limits. Which of the
following is the most appropriate recommendation for this patient?
 Incorrect Answer ImageA.Discontinue omeprazole
 Incorrect Answer ImageB.Penile implant
 Incorrect Answer ImageC.Sildenafil
 Incorrect Answer ImageD.Testosterone
 Correct Answer Image E.Vacuum device
 Incorrect Answer ImageF.Yohimbine

A 58-year-old woman undergoes a radical hysterectomy for endometrial cancer. The


intraoperative time is 5 hours, and her blood loss is estimated to be 1.8 L. She is extubated and
transferred to the recovery room, where she slowly wakes up. She is then brought to the surgical
floor, where her urine output is documented by the nurse to be decreasing. Over the last 4 hours,
she has made 30, 22, 14, and 6 mL of urine, respectively. Her blood pressure is also noted to be
decreasing; the last reading was 106/60 mm Hg. Her pulse is 104/min. Which of the following is
the most appropriate next step in management?
 Incorrect Answer ImageA.Draw a complete blood count
 Correct Answer Image B.Give a fluid challenge of 500 mL of IV fluid over 10 minutes
 Incorrect Answer ImageC.Insert a central line to measure central venous pressure
 Incorrect Answer ImageD.Irrigate the Foley catheter
 Incorrect Answer ImageE.Transfuse packed red blood cells
While working at a bookbinding shop, a young man suffers a traumatic amputation of his index
finger. The finger is cleanly severed at its base. The patient and the finger are brought to a first
aid station, from which both are to be transported to a highly specialized medical center where
replantation can be done. What is the correct way to prepare and transport the severed finger? 
 Incorrect Answer ImageA.Dry the finger of any traces of blood and place it in a cooler
filled with crushed ice
 Incorrect Answer ImageB.Freeze the finger as quickly as possible, and transport it
immersed in liquid nitrogen
 Incorrect Answer ImageC.Immerse the finger in cold alcohol for the entire trip
 Incorrect Answer ImageD.Paint the finger with antiseptic solution and place it on a bed
of dry ice
 Correct Answer Image E.Wrap the finger in a moist gauze, put it in a plastic bag, and
place the bag on a bed of ice

A 59-year-old man comes to the physician for a routine health maintenance examination. He
feels well. A discrete, hard, 1.5-cm nodule is felt in his prostate during the rectal examination. A
rectal examination performed one year ago showed no abnormalities. His prostatic-specific
antigen (PSA) level 3 months ago was 3.5 ng/mL. He denies any family history of prostate
cancer. Which of the following is the most appropriate next step in management? 
 Incorrect Answer ImageA.Follow-up rectal exam in 3 months
 Correct Answer Image B.Needle biopsy of the prostate
 Incorrect Answer ImageC.Repeat determination of PSA
 Incorrect Answer ImageD.Sonogram of the prostate
 Incorrect Answer ImageE.Trial of antibiotics

A 33-year-old woman develops bloody nipple discharge from the right breast. It occurs
intermittently over the course of 6 weeks. There is no family history of breast cancer. The patient
denies any trauma, recent pregnancy, prior similar episodes, pain, or fevers. On physical
examination, both breasts are normal in appearance. There are no palpable masses in either
breast. A very small amount of blood can be manually expressed from the right nipple.
Discharge is sent for cytology and results show the presence of few inflammatory cells and
multiple RBCs. The patient is sent for mammography and ultrasound of both breasts, which
indicate no masses. Which of the following is the best next step in management? 
 Incorrect Answer ImageA.Breast MRI
 Correct Answer Image B.Galactography
 Incorrect Answer ImageC.Large-needle core biopsy
 Incorrect Answer ImageD.Perform fine-needle aspiration (FNA) biopsy
 Incorrect Answer ImageE.Repeat cytology of the discharge

A 72-year-old man of Norwegian ancestry comes to the physician because of an inability to


completely open his right hand. He reports that his hand can no longer be placed flat on a table.
The problem has developed gradually over many years. He has no pain and no neurologic
abnormalities, and to the extent that the deformity allows, he can move his fingers at will.
Physical examination shows an inability to completely extend the fingers, and also shows the
presence of palpable fascial nodules along the course of the flexor tendons, mostly related to the
4th and 5thdigits. Which of the following is the most likely diagnosis?
 Incorrect Answer ImageA.De Quervain tenosynovitis
 Correct Answer Image B.Dupuytren contracture
 Incorrect Answer ImageC.Rheumatoid arthritis
 Incorrect Answer ImageD.Scleroderma
 Incorrect Answer ImageE.Stenosing tenosynovitis

A 51-year-old man is an unrestrained front-seat passenger in a motor vehicle collision. Physical


examination shows multiple facial lacerations and a shortened, adducted, and internally rotated
right lower extremity. His temperature is 37.7ºC (99.9ºF), pulse is 110/min, respirations are
16/min, and blood pressure is 155/75 mm Hg. Neurologic tests are normal and a CT scan of the
head, chest, and abdomen are normal. Which of the following additional imaging studies is
indicated?
 Incorrect Answer ImageA.CT angiogram of the aorta
 Incorrect Answer ImageB.MRI scan of both knees
 Correct Answer Image C.X-ray of both hips
 Incorrect Answer ImageD.X-ray of the lumbar spine
 Incorrect Answer ImageE.X-ray of the skull

A 29-year-old man comes to the clinic with a 2-day history of severe left-sided scrotal pain and
swelling. He is sexually active and has multiple sexual partners. He has no history of sexually
transmitted diseases. His temperature is 38.2°C (100.8°F), blood pressure is 120/70 mm Hg.
Physical examination shows unilateral scrotal tenderness and swelling. Pain is relieved on lifting
the testes. Which of the following is the most likely diagnosis? 
 Correct Answer Image A.Epididymitis
 Incorrect Answer ImageB.Scrotal hematoma
 Incorrect Answer ImageC.Strangulated hernia
 Incorrect Answer ImageD.Testicular torsion
 Incorrect Answer ImageE.Varicocele

A 22-year-old man is involved in a motorcycle accident and sustains serious injuries. He is


brought by ambulance to the emergency department, where he is noted to have injuries to the
head, chest, pelvis, and legs. His pulse is 110/minute, with a blood pressure of 100/60 mm Hg. In
addition to bruises along his body, blood is noted at the urethral meatus. A pelvic radiograph
shows significant separation of the pubic symphysis with a fracture of the pelvis. CT scan shows
active extravasation of arterial contrast in the pelvis. The patient is taken to the interventional
radiology suite, where several embolization coils are placed in branches of the internal iliac
artery. He concurrently receives 2 units of packed red blood cells. His pulse improves to 90/min
and his blood pressure improves to 125/75 mm Hg. Which of the following is the most
appropriate next step in the management of his potential urethral injury?
 Incorrect Answer ImageA.IV pyelogram
 Incorrect Answer ImageB.No further management is required, as the injury heals
spontaneously in most cases
 Incorrect Answer ImageC.Placement of a Foley catheter
 Correct Answer Image D.Retrograde urethrogram
 Incorrect Answer ImageE.Surgical exploration
A 68-year-old man is brought to the emergency department via ambulance after calling 911
because of dull, central chest pain and tightness that began while he was driving his car home
from work. The patient also feels short of breath. He states that he has had similar episodes in the
past but not as severe. He has noticed that his chest pain has become progressively worse over
the past 2 weeks, especially when walking to his car. He states that he now parks closer to his
office to avoid the walk. He is a 25 pack-year smoker and drinks approximately 20 ounces of
alcohol a week. His temperature is 37.4ºC (99.3ºF), pulse is 115/min, and blood pressure is
150/96 mm Hg. On physical examination, the patient is sweating profusely and appears to be in a
great degree of discomfort. He is currently breathing 100% oxygen via a facemask. There is no
evidence of jugular venous distension. Cardiovascular examination shows distant heart sounds
that are regular in rhythm. The electrocardiogram shows no ST-segment elevation. The patient is
started on aspirin, enoxaparin, IV metoprolol, morphine, and nitrates. CK-MB and troponins are
significantly elevated. The patient is sent to the cardiac catheterization laboratory, and a coronary
angiogram is performed showing 60% occlusion of the left main coronary artery. Which of the
following is the next best step in management?
 Incorrect Answer ImageA.Administer IV diazepam
 Correct Answer Image B.Coronary artery bypass grafting
 Incorrect Answer ImageC.Dobutamine echocardiography
 Incorrect Answer ImageD.Percutaneous coronary intervention
 Incorrect Answer ImageE.Prescribe atorvastatin
 Incorrect Answer ImageF.Prescribe enalapril
 Incorrect Answer ImageG.Stress echocardiography
 Incorrect Answer ImageH.Stress thallium scan

A 57-year-old woman comes to the emergency department because of abdominal pain. She states
that the pain began a few hours previously; it is sharp, constant, non-radiating, and does not
change with position. Her temperature is 38.5ºC (101.3ºF), pulse is 104/min, and blood pressure
is 121/77 mm Hg. On physical examination, she is diaphoretic, and her abdomen is non-
distended with decreased bowel sounds. There is tenderness with guarding in the left lower
quadrant. Pelvic examination reveals discomfort in the left lower quadrant and no cervical
motion tenderness. CBC shows a white blood cell count of 15,000/mm3. What of the following
diagnostic tests is the next best step in management to establish the diagnosis?
 Incorrect Answer ImageA.Barium enema
 Correct Answer Image B.CT scan of the abdomen and pelvis
 Incorrect Answer ImageC.Flexible sigmoidoscopy
 Incorrect Answer ImageD.Pelvic ultrasound
 Incorrect Answer ImageE.Renal ultrasound

A 34-year-old unconscious man is brought to the emergency department 5 hours after a motor
vehicle accident in which he was ejected from the car and hit his head on the pavement. He was
treated for a head laceration on the scene but then lost consciousness 3 hours later. His medical
history is unremarkable, and he takes no medications. His pulse is 42/min, respirations are
12/min and irregular, and blood pressure is 160/96 mm Hg. Neurologic examination shows that
the patient is unresponsive to verbal or painful stimulation. Eye examination shows a fixed and
dilated right pupil. The patient is in a cervical spine collar placed by the paramedics prior to
arrival. The patient is intubated. Plain radiograph films of the lateral cervical spine show no
acute abnormalities with visualization of C7–T1. The CT scan is shown. The patient remains
unresponsive to verbal and painful stimuli and his right pupil remains fixed and dilated. Which
of the following is the most appropriate next step in management?
 Incorrect Answer ImageA.CT scan repeated in 8 hours
 Correct Answer Image B.Emergent craniotomy
 Incorrect Answer ImageC.Hypertonic saline drip in the ICU
 Incorrect Answer ImageD.Hyperventilation to PCO2 <25
 Incorrect Answer ImageE.IV steroids
 Incorrect Answer ImageF.Ventricular shunt placement

A 48-year-old man is brought to the emergency room via ambulance because of chest pain. He
suddenly felt a dull, crushing chest pain that radiated to his jaw and left arm. He immediately
dialed 911 as he thought he was "having a heart attack." On physical examination, he appears
moribund and is sweating profusely. Cardiovascular examination shows muffled heart sounds
that are regular in rate and rhythm. An electrocardiogram shows ST-segment elevation, and the
patient is transferred to the cardiac catheterization laboratory for a coronary angiogram and
subsequent percutaneous coronary intervention with stent placement. Twenty-four hours later,
the patient is sitting in the cardiac care unit when his blood pressure drops to 105/55 mm Hg. His
pulse is 120/min. Cardiovascular examination again shows muffled heart sounds with no
murmurs present. There is jugular venous distension. The blood pressure is re-checked with a
manual sphygmomanometer, and the blood pressure varies between 110/55 mm Hg and 95/55
mm Hg. Which of the following is the next best step in the management?
 Incorrect Answer ImageA.Give atropine
 Incorrect Answer ImageB.Intra-aortic counterpulsation balloon placement
 Incorrect Answer ImageC.Intravenous fluids
 Correct Answer Image D.Pericardiocentesis
 Incorrect Answer ImageE.Transfer the patient to the cardiac catheterization laboratory

During a routine medical checkup, a healthy 58-year-old woman is found to have a serum
calcium level of 11.8 mg/dL. Repeated testing confirms values between 10.9 and 12.2 mg/dL.
She is also found to have elevated concentrations of parathyroid hormone. She is asymptomatic,
has no pertinent family history, and has no evidence of renal stones or bone disease. She is
offered the option of elective parathyroidectomy, but she declines and elects to have close
medical follow-up. While doing so, it would be advisable for her to be placed on which of the
following therapies? 
 Correct Answer Image A.Estrogen-progestin replacement
 Incorrect Answer ImageB.Long-term calcitonin
 Incorrect Answer ImageC.Low calcium intake
 Incorrect Answer ImageD.Low-dose thiazides
 Incorrect Answer ImageE.Phosphate restriction

A 55-year-old woman with a history of oxygen-dependent COPD comes to the emergency


department with worsening shortness of breath and chest pain. The chest pain is described as
mostly right-sided and constant but worse with deep inspiration. The pain and shortness of breath
awoke her from sleep in the middle of the night and she called emergency medical services to
bring her to the emergency department. Her temperature is 37.0ºC (98.6ºF), pulse is 133/min,
respirations are 28/min, and blood pressure is 170/70 mm Hg. Oxygen saturation is 83% on 6 L
of oxygen by nasal cannula. She is in moderate respiratory distress and is using accessory
muscles to breathe. The lungs have decreased breath sounds on the right side more than on the
left. There are scattered wheezes on the left. The remainder of her physical examination is
normal. A stat chest x-ray is shown. Which of the following is the most appropriate next step in
management?
 Incorrect Answer ImageA.Albuterol, ipratropium bromide, oxygen, antibiotics, and
steroids
 Incorrect Answer ImageB.Aspirin, heparin, oxygen, metoprolol, and nitrates
 Incorrect Answer ImageC.Heparin infusion
 Incorrect Answer ImageD.Left chest tube
 Correct Answer Image E.Right chest tube

A 65-year-old woman stands on a chair to reach a cupboard and falls on her outstretched hand.
She comes to the clinic with a deformed and painful wrist. Neurologic examination is normal.
The x-ray is shown. Which of the following is the most appropriate management?
 Incorrect Answer ImageA.Closed reduction and long arm cast
 Correct Answer Image B.Closed reduction and short arm cast
 Incorrect Answer ImageC.Intramedullary rod
 Incorrect Answer ImageD.Open reduction and internal fixation
 Incorrect Answer ImageE.Skeletal traction

A 20-year-old man comes to the clinic because of a newly discovered "bump" on his right
testicle. He denies pain or tenderness of the testicle and states that he has no other complaints.
Examination reveals a firm, palpable mass on the right testicle. Ultrasound confirms the presence
of a heterogenous hypoechoic testicular mass, and he subsequently undergoes orchiectomy.
Histologic analysis demonstrates sheets of small, poorly differentiated cells with scant cytoplasm
and crowded nuclei. Numerous mitotic figures and zones of necrosis are also noted. Staging MRI
scan demonstrates enlarged retroperitoneal lymph nodes, and laboratory studies show an
increased serum alpha-fetoprotein. What is the most likely diagnosis? 
 Incorrect Answer ImageA.Choriocarcinoma
 Correct Answer Image B.Embryonal carcinoma
 Incorrect Answer ImageC.Seminoma
 Incorrect Answer ImageD.Teratoma
 Incorrect Answer ImageE.Yolk-sac carcinoma

A 47-year-old woman with chronic alcohol abuse develops protracted vomiting, aspiration, and
right lower lobe pneumonia. Her family history is positive for multiple sclerosis. She is
hospitalized in the evening, and initial laboratory studies show a prothrombin time of 22 seconds,
a serum sodium concentration of 123 mEq/L, and a white blood cell count of 17,000/mm3.
During the next 12 hours, she is placed on antibiotics, receives an IV drip of 5% saline, and is
administered several ampules of vitamin K. By the next morning, her prothrombin time is 18
seconds, her serum sodium concentration is 141 mEq/L, and her white blood cell count has
decreased to 12,000/mm3. On morning rounds, however, she is found to be unresponsive. Further
examination shows that she is actually conscious and oriented but is unable to move her
extremities or to speak. She can respond to commands only by moving her eyes up and down and
by blinking. Although complete neurologic examination was not done at admission, at that time
she was awake and alert and had no obvious neurologic complaints or deficits. An MRI of the
brain will most likely show which of the following findings?
 Correct Answer Image A.Bright patches within the central basis of the pons
 Incorrect Answer ImageB.Demyelinating lesions scattered over both hemispheres
 Incorrect Answer ImageC.Diffuse intracranial hemorrhage
 Incorrect Answer ImageD.Thrombosis of the basilar artery
 Incorrect Answer ImageE.Tonsillar herniation

A 21-year-old man sustains a gunshot wound to the base of his neck. He was shot point-blank
with a .38-caliber revolver. The entrance wound is above the left clavicle, below the level of the
cricoid cartilage, and just lateral to the sternocleidomastoid muscle. The exit wound is just above
the spinous process of the right scapula. He denies dysphagia, dysphonia, or dyspnea. Vital signs
are within normal limits. He is awake and alert, talking with a normal tone of voice, has normal
breath sounds bilaterally, and is neurologically intact. X-ray films of the neck and chest taken in
the emergency department show some air in the tissues of the lower neck but are otherwise
nondiagnostic. Which of the following is the most appropriate next step in management?
 Correct Answer Image A.Angiogram and esophagogram
 Incorrect Answer ImageB.CT scan of the lower neck and upper chest
 Incorrect Answer ImageC.Immediate surgical exploration of the lower neck through a
collar incision
 Incorrect Answer ImageD.Immediate surgical exploration of the upper chest through a
median sternotomy
 Incorrect Answer ImageE.Observation for several hours

A previously healthy 29-year-old man comes to the physician because of worsening pain and
swelling in his right arm over the past 2 months. He notices that the symptoms worsen when he
raises his arm. He doesn't smoke, and he drinks alcohol socially. He took up heavy weight-lifting
approximately 1 year ago, and he uses several supplements for weight gain and muscle-building.
Vitals are within normal limits. He is well-developed. Head, neck, lung, cardiac, and abdominal
examinations show no abnormalities. Trace pitting edema is noted in the right arm. His radial
pulse is present. He is instructed to raise his arm above his head. After 1 minute, the swelling
increases markedly in his right arm. Which of the following is most likely the source of the
patient's symptoms? 
 Correct Answer Image A.Anterior scalene muscle
 Incorrect Answer ImageB.Cervical rib
 Incorrect Answer ImageC.Pancoast tumor
 Incorrect Answer ImageD.Subclavian steal syndrome
 Incorrect Answer ImageE.Subclavian vein thrombosis

A healthy 37-year-old firefighter is injured while rescuing a young child from a house fire. The
firefighter is hemodynamically stable and oxygenating well in the emergency department. On
physical examination he is noted to have first-degree burns to the chest and abdomen, and
second-degree burns to the forearms and upper arms. Topical silver sulfadiazine is applied to the
burns and they are loosely dressed with Vaseline gauze. He is started on IV fluids, and a Foley
catheter is inserted. Initially he does not complain of pain; however, the following morning, he
complains of significant forearm pain, worse on the right. The dressings are removed and the
right forearm is noted to have significant edema. He is able to grasp with 5/5 strength but has
decreased sensation in his fifth digit. Both radial and ulnar pulses are faintly palpable. Which of
the following is the next best step in this patient’s management? 
 Incorrect Answer ImageA.Change topical wound care to sulfur mafenide
 Incorrect Answer ImageB.Decrease the rate of IV fluid to decrease edema
 Incorrect Answer ImageC.Intubate the patient for airway protection secondary to airway
edema
 Correct Answer Image D.Perform forearm escharotomy
 Incorrect Answer ImageE.Splint the forearm in extension position

A 48-year-old man comes to the emergency department after a motor vehicle accident. He was a
restrained driver in a high speed crash with airbag deployment. He is conscious and complains of
abdominal pain. His pulse is 102/min, respirations are 18/min, and blood pressure is 123/86 mm
Hg. On physical examination, multiple lacerations and bruises are seen all over the body, but
there is no obvious deformity. There is tenderness and instability of the pelvic girdle. On rectal
examination, there is no blood. The prostate is palpable but is mobile and able to be pushed
upward. Which of the following is the most appropriate next step in management? 
 Incorrect Answer ImageA.Cystoscopy
 Incorrect Answer ImageB.Insertion of Foley catheter
 Correct Answer Image C.Retrograde urethrogram
 Incorrect Answer ImageD.Sonogram of the bladder
 Incorrect Answer ImageE.Suprapubic cystostomy

A 68-year-old man presents with progressive edema affecting his right lower extremity. He first
noticed it approximately 2 years ago, but it has become much worse in the last 6 months. He has
a history of coronary artery disease and has had multiple catheterizations and stents placed. He
has been on oral antibiotics for 10 days for cellulitis of his right leg. His temperature is 37ºC
(98.6ºF), pulse is 105/min, respirations are 12/min, and blood pressure is 122/78 mm Hg. Right
lower extremity examination shows pitting edema from the groin to the toes, prominent varicose
veins, and an area of chronic cellulitis above the medial malleolus. There is an audible bruit and
a palpable thrill over the right groin. Pulses in the foot are palpable. His left lower extremity
exam is normal. What is the most likely diagnosis?
 Incorrect Answer ImageA.Abnormal venous valves
 Incorrect Answer ImageB.Antibiotic-resistant cellulitis
 Correct Answer Image C.Arteriovenous fistula
 Incorrect Answer ImageD.Congestive heart failure
 Incorrect Answer ImageE.Deep venous thrombosis (DVT)

A 69-year-old man who has a history of smoking, drinking, and has poor dentition comes to the
emergency department because of a hard, fixed, 4 cm mass in his left neck. The mass is just
medial to and in front of the sternocleidomastoid muscle, at the level of the upper notch of the
thyroid cartilage. It has been there for at least 6 months, and it is growing. Which of the
following is the most appropriate next step in diagnosis?
 Incorrect Answer ImageA.Open excisional biopsy of the mass
 Incorrect Answer ImageB.Open incisional biopsy of the mass
 Correct Answer Image C.Panendoscopy (triple endoscopy) and mucosal biopsies
 Incorrect Answer ImageD.Radionuclide scan of the thyroid gland
 Incorrect Answer ImageE.Sputum cytology and CT scan of the lungs

An 81-year-old man with Alzheimer’s disease who lives in a nursing home undergoes surgery
for a fractured femoral neck. On the fifth postoperative day, his abdomen is grossly distended
and tense but non-tender with occasional bowel sounds. The rectal vault is empty on digital
examination, and there is no evidence of occult blood. Vital signs are normal. X-ray films show a
few distended loops of small bowel and a very distended colon. The cecum measures 9 cm in
diameter, and the entire colon is equally distended, including the sigmoid and rectum. No stool is
seen on x-ray. Which of the following is the most likely diagnosis?
 Incorrect Answer ImageA.Fecal impaction
 Correct Answer Image B.Ogilvie syndrome
 Incorrect Answer ImageC.Paralytic ileus
 Incorrect Answer ImageD.Small bowel obstruction
 Incorrect Answer ImageE.Volvulus of the sigmoid colon

A 50-year-old obese white man is brought to the emergency department by the paramedics after
his wife found him on the floor of their apartment. She informs you that her husband is a diabetic
and also has "high blood pressure," but she can't remember which medications her husband
takes. On physical examination, the patient is unresponsive to touch or voice. His temperature is
36.6ºC (97.8ºF), pulse is 102/min, respirations are 18/min, and blood pressure is 110/65 mm Hg.
Urine output for the past 30 minutes is 50 mL. Physical examination reveals low jugular venous
pressure, dry oral mucosa, and decreased skin turgor. Laboratory studies show:
Na+ 142 mEq/L
K +
5.4 mEq/L
Cl− 106 mEq/L
HCO3- 16 mEq/L
Glucose  1260 mg/dL
BUN  35 mg/dL
Creatinine  2.8 mg/dL
Once the patient is stable and his glucose and electrolytes are normalized, which of the following
medications most likely caused his hospitalization and needs to be discontinued? 
 Incorrect Answer ImageA.Clozapine
 Incorrect Answer ImageB.Cocaine
 Correct Answer Image C.Hydrochlorothiazide
 Incorrect Answer ImageD.Lisinopril
 Incorrect Answer ImageE.Metformin
 Incorrect Answer ImageF.Metoprolol

A 29-year-old man comes to the outpatient orthopedic surgery clinic because of persistent right
leg pain. He reports "achy" pain at the hip radiating down his femur to the knee. It is worse on
exertion, especially with jogging more so than climbing stairs, but it is also present at rest. It is
usually relieved with over-the-counter nonsteroidal anti-inflammatory medications. His past
medical history is significant for Crohn's disease in remission for 2 years on 6-mercaptopurine.
His last flare-up was successfully managed with 2 weeks of oral prednisone and did not require
hospitalization. Family history is noncontributory. He does not smoke cigarettes but he drinks
alcohol 2 or 3 times per week. He denies any illicit drug use. Physical examination shows limited
range of motion of the right hip, especially with external rotation. A plain x-ray of the right hip
shows a dulling of the femoral head but no other pathology. Which of the following is the next
step in diagnosis?
 Incorrect Answer ImageA.Bone scan
 Incorrect Answer ImageB.CT scan
 Correct Answer Image C.MRI scan
 Incorrect Answer ImageD.PET scan
 Incorrect Answer ImageE.Ultrasound

A 21-year-old man is brought to the emergency department after reportedly sustaining a gunshot
wound to his right lower back. He is alert and oriented but complaining of back pain. His
temperature is 36°C (96.8°F), pulse 98/min, and blood pressure 126/68 mm Hg. He is breathing
without difficulty and has bilateral breath sounds with an oxygen saturation of 99% on room air.
On physical examination, a wound is seen in the right lower back, just inferior to the subcostal
margin, 4 cm from the posterior axillary line. It is oozing bright-red blood. The abdomen is
nondistended, and the patient denies tenderness to palpation. Two 18-gauge IV lines are inserted,
and laboratory studies are sent. A Foley catheter is placed and yields clear urine. A chest x-ray
film reveals no pneumothorax or bony abnormalities, and the bullet is not visualized. Which of
the following is the most appropriate next step in management?
 Correct Answer ImageA .Cross-table x-ray
 Incorrect Answer ImageB.CT scan of the abdomen
 Incorrect Answer ImageC.CT scan of the chest
 Incorrect Answer ImageD.Exploratory laparotomy
 Incorrect Answer ImageE.IV pyelogram

A 71-year-old man is brought to the operating room for an elective repair of a growing
abdominal aortic aneurysm (AAA) with an endovascular stent. The aneurysm has been followed
closely for 3 years but has grown 1 cm over the past year, to 5.8 cm. The operation is
uncomplicated and the patient is extubated and brought to the surgical ICU postoperatively for
management. He remains hemodynamically stable and his lower extremities remain warm and
well perfused. On postoperative day 1 his creatinine increases from 1.1 mg/dL to 1.3 mg/dL, but
his urine output remains approximately 40 mL/h. On postoperative day 2, he complains of
abdominal pain and has an episode of bloody diarrhea. A stat complete blood count shows that
hemoglobin has dropped from 10 g/dL to 9 g/dL, and white blood cell count has elevated from
12,000/mm3 in the morning to 15,000/mm3. Which of the following is the next best step in
management?
 Incorrect Answer ImageA.CT scan of the abdomen and pelvis
 Incorrect Answer ImageB.Repeat laboratory studies in 6 to 8 hours
 Incorrect Answer ImageC.Send Clostridium difficile toxin assay and start oral
metronidazole
 Incorrect Answer ImageD.Transfuse 2 units of whole blood stat
 Correct Answer Image E.Urgent colonoscopy
A 71-year-old man comes to the physician because of problems with urination. He reports that
over the past 2 weeks his weak urine stream has become weaker. Over the last 24 hours, he has
been unable to urinate at all, and it is becoming extremely uncomfortable. He has a history of
benign prostatic hyperplasia (BPH), for which he takes tamsulosin. There is no history of
dysuria, hematuria, fever, or chills. He takes diltiazem for hypertension and had a
cholecystectomy 20 years ago. He denies any use of over-the-counter medications and has never
had surgery. His temperature is 37.4ºC (99.3ºF), pulse is 86/min, and blood pressure is 164/88
mmHg. He is in moderate distress. The abdomen is soft with lower abdominal distension and
suprapubic discomfort on palpation and percussion. On digital rectal examination, the prostate is
estimated to be 50 grams in size, nontender, and without nodularity. When an attempt is made to
place a Foley urinary catheter, resistance is met and no urine comes out. Blood then begins to
ooze from the urethral meatus. Which of the following is the best next step in management?
 Incorrect Answer ImageA.Insert bilateral percutaneous nephrostomy tubes
 Incorrect Answer ImageB.Perform retrograde urethrogram
 Correct Answer Image C.Perform suprapubic tube placement
 Incorrect Answer ImageD.Plan emergency ileal conduit creation
 Incorrect Answer ImageE.Schedule emergent transurethral resection of the prostate

A 35-year-old man falls on his hand and comes to the urgent care clinic because of wrist pain.
He states that he was not able to break the fall and that the heel of his hand took the brunt of his
full weight as it hit the pavement. On physical examination, he is distinctly tender to palpation
over the anatomic snuffbox. Anteroposterior and lateral x-ray films are negative. Which of the
following are the most likely diagnosis and most appropriate next step in the management of this
patient?
 Incorrect Answer ImageA.De Quervain tenosynovitis; steroid injections
 Incorrect Answer ImageB.Displaced scaphoid fracture; open reduction and internal
fixation
 Incorrect Answer ImageC.Ligamentous injury; Ace bandage and analgesics
 Incorrect Answer ImageD.No fracture; reassurance
 Correct Answer Image E.Scaphoid bone fracture; thumb spica cast

A 57-year-old man comes to the office for his periodic health maintenance examination. He has
no major health issues or complaints. He has no weight loss, fever, or night sweats. His heart has
a regular rate and rhythm, lungs are clear, and abdomen is soft, nontender, and nondistended
with no palpable masses. His penis is circumcised, and his testes are descended bilaterally and
are normal in size and contour. On rectal examination, his prostate is nontender and minimally
enlarged, and there is a discrete nodule over the left base. His prostate-specific antigen (PSA) is
5.8 ng/mL (high). What is the next best step in management? 
 Incorrect Answer ImageA.Check a serum PSA
 Incorrect Answer ImageB.Initiate flutamide
 Incorrect Answer ImageC.Initiate terazosin
 Incorrect Answer ImageD.Pelvic CT scan
 Incorrect Answer ImageE.Prostatectomy
 Correct Answer Image F.Transrectal ultrasonography-guided prostate biopsy
A 72-year-old man is brought to the emergency department after sustaining a fall. His past
medical history is significant for coronary artery disease, chronic kidney disease, and
osteoarthritis. His temperature is 37.3°C (99.2°F), the pulse is 124/min, and blood pressure is
120/72 mm Hg. On physical examination, he has severe right hip tenderness that is worse on
abduction and external rotation, with significant soft-tissue edema and ecchymosis but no open
wounds. Which of the following is the most appropriate initial step in management?
 Incorrect Answer ImageA.CT scan of the pelvis
 Incorrect Answer ImageB.MRI scan of the pelvis
 Incorrect Answer ImageC.Ultrasound of the deep venous system
 Incorrect Answer ImageD.Urgent operative intervention
 Correct Answer Image E.X-rays of the right hip and femur

A 42-year-old obese woman comes to the urgent care clinic because of fever and epigastric
abdominal pain for 12 hours. She reports intermittent postprandial abdominal pain that has been
increasing in frequency over the past few months. The pain is described as crampy and severe at
times but it usually does not persist longer than an hour. It has been occasionally associated with
nausea but no vomiting, and sometimes radiates to the back. The patient’s pain worsened last
night after she ate two slices of pizza, and she became severely nauseated. Finally, after the
woman noted a fever of 38.8ºC (101.8ºF), her daughter persuaded her to come to the clinic.
Physical examination of this patient is most likely to show which of the following findings?
 Incorrect Answer ImageA.Absence of bowel sounds
 Incorrect Answer ImageB.Hyperactive bowel sounds
 Correct Answer Image C.Inspiratory splinting with palpation of the right upper quadrant
 Incorrect Answer ImageD.Scleral icterus
 Incorrect Answer ImageE.Tenderness over the right costovertebral angle

A 72-year-old man is scheduled to have elective sigmoid resection for diverticular disease. He
has a history of myocardial infarction (MI) 2 years ago. He does not have angina currently but
lives a sedentary life because he becomes "out of breath" with exertion. Physical examination
shows regular heart sounds without a murmur and normal breath sounds bilaterally with rales at
the bases. Jugular venous congestion is appreciated. He has a hemoglobin level of 10.0 g/dL. If
surgery is indeed needed, which of the following should be done prior to the operation?
 Correct Answer Image A.Echocardiogram
 Incorrect Answer ImageB.Blood transfusion
 Incorrect Answer ImageC.Intensive respiratory therapy
 Incorrect Answer ImageD.Pulmonary function studies
 Incorrect Answer ImageE.Wait 6 months before performing surgery

A 72-year-old woman with diabetes, hypertension, coronary artery disease, and a history of
myocardial infarction 2 years ago is admitted to the hospital after sustaining a fall and injuring
her hip. She is in excruciating pain and lies on the stretcher with the affected leg appearing
shorter and externally rotated. X-ray shows a displaced femoral neck fracture. Which of the
following is the next best step in the management of this patient?
 Incorrect Answer ImageA.Complete hip replacement
 Incorrect Answer ImageB.External fixation
 Incorrect Answer ImageC.Open reduction and internal fixation
 Correct Answer Image D.Replacement of femoral head with metal prosthesis
 Incorrect Answer ImageE.Skeletal traction

A 58-year-old man has been having annual prostatic-specific antigen (PSA) determinations since
age 50. His value last year was 2.8 ng/mL. This year's result is 8 ng/mL. Digital rectal
examination shows an area of induration within the right lobe of the prostate. He voids five
times/day and two times/night. He has a good force of urinary stream and denies urinary
dribbling. A transrectal biopsy is positive for adenocarcinoma of the prostate. A staging CT scan
does not show any metastatic disease. Which of the following would be the most appropriate
treatment?
 Incorrect Answer ImageA.Cryotherapy
 Incorrect Answer ImageB.External beam radiation therapy
 Incorrect Answer ImageC.Microwave ablation
 Correct Answer Image D.Radical prostatectomy
 Incorrect Answer ImageE.Transurethral microwave therapy (TUMT)
 Incorrect Answer ImageF.Transurethral resection of the prostate (TURP)

A 14-year-old boy dives into the shallow end of a swimming pool and hits his head on the
concrete bottom, losing consciousness. After he is rescued, he is evaluated by paramedics, where
physical examination shows a complete lack of neurologic function below the neck. The boy is
still breathing on his own, but he cannot move or feel his arms and legs. The paramedics
carefully immobilize his neck for transportation to the hospital, and they alert the emergency
department of his impending arrival. Which of the following is the most appropriate next step in
management?
 Correct Answer Image A.Continuous cardiac and respiratory monitoring
 Incorrect Answer ImageB.Hyperbaric oxygenation
 Incorrect Answer ImageC.IV antibiotics
 Incorrect Answer ImageD.IV diuretics
 Incorrect Answer ImageE.Surgical decompression of the cord

A 19-year-old man is shot with a .45-caliber revolver in the lower abdomen just above the pubis.
The entrance wound is at the midline, and there is no exit wound. His pulse is 86/min and blood
pressure is 120/74 mm Hg after receiving 1 L of lactated Ringer's intravenously. He has localized
tenderness around the entrance wound on abdominal examination. Digital rectal examination and
proctoscopic examination are unremarkable, but he has gross hematuria upon placement of the
urinary catheter. X-rays show the bullet embedded in the sacral promontory, to the right of the
midline. Which of the following is the most appropriate next step in management?
 Incorrect Answer ImageA.CT scan of the abdomen and pelvis
 Incorrect Answer ImageB.Focused assessment with sonography for trauma (FAST scan)
 Correct Answer Image C.Exploratory laparotomy
 Incorrect Answer ImageD.Laparoscopy with bladder repair
 Incorrect Answer ImageE.Placement of a suprapubic catheter

A 59-year-old man comes to the clinic because of urinary urgency and frequency for the past 6
weeks. He also states that he has noticed the passage of small bubbles of air with his urine and
that the urine has become foul-smelling with traces of mucus. He denies any prior urinary
symptoms or disease. He has had nocturia once per night, and occasionally he has felt that his
urine stream was becoming weaker. His bowel habits are unchanged, and he denies blood per
rectum, diarrhea, and constipation. His medical history is significant for type 2 diabetes. His
surgical history is negative. He drinks alcohol socially and does not smoke. On physical
examination, he is afebrile with normal vital signs and is not in any distress. His abdomen is soft
and nondistended; bowel sounds are normal. There is mild tenderness to deep palpation in the
left lower quadrant. Genital examination is normal without any urethral discharge. On rectal
examination, his sphincter tone is good, and his prostate is small without nodules or tenderness.
His stool is guaiac-negative. His leukocyte count is 12,500/mm3 with a normal differential.
Urinalysis sent after catheterization shows many leukocytes, many bacteria, and is positive for
nitrates. Urine culture returns with multiple organisms. Which of the following is the most
appropriate next step in management?
 Incorrect Answer ImageA.A course of broad-spectrum oral antibiotics
 Incorrect Answer ImageB.Referral for an exploratory laparotomy
 Incorrect Answer ImageC.Resubmit a urine culture
 Correct Answer Image D.Send the patient for CT scan of the abdomen and pelvis with
contrast
 Incorrect Answer ImageE.Send the patient for urine cytology

A 56-year-old diabetic man is brought to the emergency department via ambulance because of
shortness of breath. For the past 6 hours, he has experienced dyspnea at rest and has been
coughing up blood-tinged sputum. He denies chest pain. His temperature is 37.2ºC (98.9ºF),
pulse is 124/min, respirations are 22/min, and blood pressure is 156/94 mm Hg. The patient
appears to be in distress and is unable to complete full sentences. Physical examination shows
jugular venous distention and pedal edema. There is no chest wall tenderness. Cardiopulmonary
examination shows fine crepitations throughout the chest. A third heart sound is present. There
are no murmurs or rubs. An electrocardiogram is normal. Which of the following is the next best
step in management?
 Correct Answer Image A.Administer supplemental oxygen
 Incorrect Answer ImageB.Dobutamine
 Incorrect Answer ImageC.Dopamine
 Incorrect Answer ImageD.Echocardiography
 Incorrect Answer ImageE.Hydralazine
 Incorrect Answer ImageF.Intravenous loop diuretic
 Incorrect Answer ImageG.Morphine
 Incorrect Answer ImageH.MUGA scan
 Incorrect Answer ImageI.Nitrates
 Incorrect Answer ImageJ.Synchronized cardioversion
 Incorrect Answer ImageK.Verapamil

A 62-year-old man has had gastroesophageal reflux disease for several years. The diagnosis was
originally made by 24 hour pH monitoring, and he was placed on proton-pump inhibitors. He has
been poorly compliant with medical management, taking his medication when he has pain but
not when feeling well. He recently underwent an endoscopy, and biopsy specimens now show
severe peptic esophagitis with Barrett’s esophagus and low-grade dysplasia. Additional studies
show normal esophageal motility, with low pressure in the lower esophageal sphincter and
normal gastric emptying. Which of the following is the most appropriate treatment at this time? 
 Incorrect Answer ImageA.Heller myotomy of the lower esophageal sphincter
 Correct Answer Image B.Laparoscopic Nissen fundoplication
 Incorrect Answer ImageC.Resection of the lower esophagus
 Incorrect Answer ImageD.Total esophagectomy
 Incorrect Answer ImageE.Vagotomy

An overweight 42-year-old man comes to the physician because of persistent pain, swelling, and
limping which began while playing tennis the previous day. In the middle of the game, he heard
a loud "pop" and he fell to the ground, clutching his left ankle. He limped off the court with pain
and swelling in the back of the lower left leg. He can put weight on the foot with no exacerbation
of the pain, but the motion of taking a step is painful. Which of the following is the most likely
finding on physical examination?
 Incorrect Answer ImageA.Tapping on the calcaneus is extremely painful
 Incorrect Answer ImageB.The left ankle joint can be abducted farther out than the right
ankle
 Incorrect Answer ImageC.The left ankle joint can be adducted farther in than the right
ankle
 Correct Answer Image D.There is an absence of plantar flexion on squeezing of the
gastrocnemius muscle with the patient in the prone position
 Incorrect Answer ImageE.There are crepitations and grating on direct palpation over
either malleolus

A 42-year-old woman is thrown from the vehicle during a rollover car accident. The car
subsequently lands on her. Physical examination in the emergency department shows a pelvic
fracture, which is confirmed by a portable x-ray film done as the patient is being resuscitated.
Her pulse is 160/min and barely palpable, and initial blood pressure is 50/30 mm Hg. Thirty
minutes later, after 2 L Ringer's lactate and 2 units of packed red blood cells have been infused,
her pulse is 130/min and blood pressure is only 70/50 mm Hg. An ultrasound shows no intra-
abdominal bleeding and a diagnostic peritoneal lavage confirms no blood in the abdomen. Rectal
and vaginal examinations show no injury to those organs. There is no blood in the patient's urine.
Which of the following is the most appropriate next step in management?
 Incorrect Answer ImageA.Angiographic embolization of the torn veins
 Incorrect Answer ImageB.Exploratory laparotomy with pelvic dissection and hemostasis
 Correct Answer Image C.External fixation of the pelvis
 Incorrect Answer ImageD.Open reduction and internal fixation of the pelvis
 Incorrect Answer ImageE.Packing of the vagina and rectum

A 63-year-old man comes to the emergency department with mid-abdominal moderate-to-severe


pain that is associated with nausea, anorexia, and vomiting for the past 24 hours. The patient has
a history of deep venous thrombosis and had a pulmonary embolus in the distant past, but he is
not currently on anticoagulants. His temperature is 98.6ºF (37ºC), pulse is 102/min, and blood
pressure is 95/60 mm Hg. On physical examination, a distended and tympanic abdomen is noted,
with minimal tenderness and no rigidity. Abdominal x-rays show mildly distended loops of small
bowel and distension of the right colon, up to the middle of the transverse colon. Stool is positive
for occult blood. Which of the following is the most appropriate next diagnostic step?
 Incorrect Answer ImageA.Colonoscopy
 Correct Answer Image B.CT scan of the abdomen with IV contrast
 Incorrect Answer ImageC.Doppler ultrasound
 Incorrect Answer ImageD.Laparoscopy
 Incorrect Answer ImageE.Upper GI endoscopy

A 68-year-old man has had difficulty urinating for the past 2 years. Now, however, he finds that
he will often "leak" a small amount of urine, particularly when he coughs, sneezes, or laughs. He
also finds that frequently he urinates on himself at night and has to make multiple trips to the
bathroom. When he does urinate voluntarily, he has urinary hesitancy with a weak urinary stream
and dribbling. Physical examination reveals a palpable mass from the umbilicus to the pelvic
brim. Digital rectal examination reveals a 50 gram prostate with a right-sided area of induration.
A Foley catheter is inserted, resulting in the evacuation of 900 mL of straw-colored urine. Which
of the following is also an appropriate initial treatment? 
 Incorrect Answer ImageA.5α-reductase inhibitor
 Incorrect Answer ImageB.Alpha-adrenergic agent
 Correct Answer Image C.Alpha-antagonist agent
 Incorrect Answer ImageD.Transurethral microwave therapy (TUMT)
 Incorrect Answer ImageE.Transurethral resection of the prostate (TURP)

A 31-year-old man undergoes a small bowel resection for a jejunoileal fistula from Crohn’s
disease. Ten hours later, he develops a fever, confusion, and severe abdominal pain. He vomits
and passes loose stools multiple times. His past medical history is significant for type 2 diabetes
mellitus and Crohn's disease for several years. Current medications include metformin,
mesalamine, and methylprednisolone. On physical examination his temperature is 38.9ºC
(102.0ºF), pulse is 122/min, respirations are 18/min, and blood pressure is 100/52 mm Hg.
Laboratory studies show:
Hemoglobin  14 g/dL
Leucocytes  11,400/mm3
Neutrophils-segmented  42%
Eosinophils  5%
Lymphocytes  50%
Serum sodium  130 mEq/L
Serum potassium  5 mEq/L
Serum glucose  60 mg/dL
Chest x-ray is normal. Which of the following is the most likely cause of this patient's
condition? 
 Incorrect Answer ImageA.Abdominal abscess
 Correct Answer Image B.Adrenal insufficiency
 Incorrect Answer ImageC.Hypoglycemia
 Incorrect Answer ImageD.Hyponatremia
 Incorrect Answer ImageE.Intra-abdominal bleed
A 62-year-old man is admitted to the intensive care unit after suffering an acute myocardial
infarction while undergoing coronary angioplasty. An emergent triple coronary bypass surgery
was performed, but an ECG and cardiac enzymes showed significant myocardial damage. Eight
hours later, his pulse is 110/min, blood pressure is 85/65 mm Hg, and central venous pressure is
26 mm Hg. He appears pale and diaphoretic, and his urinary output is 18 mL/h. An x-ray film of
the chest shows pulmonary congestion compatible with pulmonary edema. Further studies show
lactic acidosis and a cardiac index of 1.9 L/min per square meter, a wedge pressure of 29 mm
Hg, and high pulmonary and systemic peripheral resistance. An ultrasound of the pericardial sac
is unremarkable. Which of the following is the most appropriate treatment at this time? 
 Incorrect Answer ImageA.Additional IV fluids
 Incorrect Answer ImageB.Antibiotics and high-dose steroids
 Correct Answer Image C.Inotropic drugs
 Incorrect Answer ImageD.Reoperation
 Incorrect Answer ImageE.Vasodilators alone

A 75-year-old woman with a past medical history significant for paroxysmal atrial fibrillation,
hypertension, and osteoarthritis comes to the emergency department with bright red blood per
rectum. Her medications include warfarin, nifedipine, metoprolol, and ibuprofen as needed. She
reports that the first incident occurred the previous night, approximately 12 hours earlier, with 3
subsequent episodes. She denies chest pain, lightheadedness, and shortness of breath. She has no
abdominal pain. Her temperature is 37.5°C (99.5°F), pulse is 110/minute, respirations are
14/min, and blood pressure is 101/72 mm Hg. Abdominal examination is unremarkable, but
rectal exam is notable for frank heme-positive stool. No masses or hemorrhoids are palpated.
Laboratory studies reveal a serum potassium level of 3.1, creatinine 1.4, hemoglobin 6.7, and
INR 8.5. Which of the following is the best option to reverse her coagulopathy?
 Incorrect Answer ImageA.Activated factor VII
 Incorrect Answer ImageB.Administer protamine sulfate
 Incorrect Answer ImageC.Cryoprecipitate
 Incorrect Answer ImageD.Phytonadione (Vitamin K)
 Correct Answer Image E.Plasma infusion (fresh frozen plasma)

A 25-year-old man is stabbed once in the right chest. The entrance wound is on the midaxillary
line, at the level of the fifth intercostal space. He arrives at the emergency department
moderately short of breath, but he is fully awake and alert and is talking with a normal tone of
voice. His pulse is 82/min, respirations are 18/min, blood pressure is 130/75 mm Hg, and oxygen
saturation is 97% on room air. Physical examination of the chest shows the wound, which is not
visibly "sucking air," and demonstrates no breath sounds on the right side, which is tympanitic to
percussion. There is no evidence of tracheal deviation or distension of forehead or neck veins.
Which of the following is the next step in management?
 Correct Answer Image A.Cover the wound with regular dressing and order a chest x-ray
 Incorrect Answer ImageB.Cover the wound with Vaseline gauze, taped on 3 sides
 Incorrect Answer ImageC.Endotracheal intubation
 Incorrect Answer ImageD.Insert a chest tube at the right pleural base
 Incorrect Answer ImageE.Insert an 18-gauge needle into the right pleural space at the
second intercostal space
An obese, 40-year-old Hispanic woman reports repeated episodes of right upper quadrant
abdominal pain. The pain is brought about by the ingestion of fatty foods and radiates to the right
shoulder and around to the back, and is sometimes accompanied by nausea and occasional
vomiting. The patient has no pain at this time but is anxious to avoid further episodes. She is
afebrile, and her physical examination is unremarkable. Which of the following is the most
appropriate next step in management?
 Incorrect Answer ImageA.Antibiotics, IV fluids, and nothing by mouth
 Incorrect Answer ImageB.CT scan of the abdomen
 Incorrect Answer ImageC.Endoscopic retrograde cholangiopancreatography (ERCP)
 Incorrect Answer ImageD.Magnetic resonance cholangiopancreatography (MRCP)
 Correct Answer Image E.Sonogram of the biliary tract and gallbladder

A 79-year-old man with a history of mitral stenosis is brought to the emergency department
because of mid-abdominal colicky pain with nausea, anorexia, and vomiting for the past 48
hours. On physical examination, his temperature is 38.9ºC (102ºF), pulse is 122/min and
irregularly irregular, and blood pressure is 85/60 mm Hg. A mild diastolic murmur is heard at the
apex. He has a distended abdomen with diffuse, exquisite tenderness. There is trace blood on the
rectal examination. X-ray film shows a distended small bowel and distended right colon up to the
middle of the transverse colon. He has a metabolic acidosis. Which of the following is the most
appropriate next step?
 Incorrect Answer ImageA.Abdominal ultrasound with Doppler flow
 Incorrect Answer ImageB.Colonoscopy
 Incorrect Answer ImageC.CT scan of the abdomen
 Correct Answer Image D.Exploratory laparotomy
 Incorrect Answer ImageE.Upper gastrointestinal endoscopy

A 72-year-old man undergoes a right hemicolectomy for colon cancer discovered on routine
colonoscopy screening. His past medical history is remarkable for hypertension and diabetes.
During the operation, he was noted by the anesthesiologist to be tachycardic to 145/min, but his
blood pressure remained stable, as did his urine output. Post-operatively, he is profoundly
hypotensive with a blood pressure of 90/60 mm Hg, and he has negligible urine output. A
pulmonary artery catheter is placed, which shows a cardiac index of 1.6 L/min/m2 (normal range
is 2.5 to 4 L/min/m2) and a systemic vascular resistance of 1,900 (dynes x sec)/cm5 (normal range
is 1,000-1,300 [dyne x sec]/cm5). Which of the following agents would be the most appropriate
first-line therapy?
 Incorrect Answer ImageA.Amiodarone
 Incorrect Answer ImageB.Atropine
 Correct Answer Image C.Dobutamine
 Incorrect Answer ImageD.Epinephrine
 Incorrect Answer ImageE.Lidocaine

A 51-year-old man comes to the physician because of weight loss and difficulty eating. His
physician has been counseling the patient for many years to stop drinking alcohol and smoking
tobacco, but the man has continued to do both regularly. The patient states that he noticed he was
having some difficulty swallowing his dinner over the past couple of months. Initially, he had
trouble with meats, but now it seems that even mashed potatoes and some soups are not "going
down smoothly." He has also begun losing weight, at least 11.3 kg (25 lb) since his last visit 6
months ago. On examination, the patient appears weak and his skin is pale. There are no palpable
lymph nodes. His abdomen is soft with normal bowel sounds and without palpable mass or
tenderness. Stool is guaiac-negative. Which of the following is the most likely diagnosis?
 Incorrect Answer ImageA.Achalasia
 Incorrect Answer ImageB.Adenocarcinoma of the colon
 Incorrect Answer ImageC.Adenocarcinoma of the esophagus
 Incorrect Answer ImageD.Gastric carcinoma
 Correct Answer Image E.Squamous cell carcinoma of the esophagus

A 23-year-old man is brought to the emergency department after sustaining a gunshot wound to
the right chest. In the ambulance, he was resuscitated with 1 L of normal saline. On his arrival,
he is unresponsive. His pulse is 124/min and his blood pressure is 70/40 mm Hg. Physical
examination shows an entrance wound in the right chest wall. He has absent breath sounds on the
right side. He is rapidly intubated for airway protection and preparations are made for insertion
of a chest tube. Which of the following is the most appropriate next step in the management of
this patient?
 Correct Answer Image A.Insertion of a right femoral venous introducer catheter
 Incorrect Answer ImageB.Insertion of a right femoral venous triple lumen catheter
 Incorrect Answer ImageC.Insertion of a right internal jugular venous introducer catheter
 Incorrect Answer ImageD.Insertion of a right lower extremity intraosseous line
 Incorrect Answer ImageE.Insertion of bilateral 18-gauge peripheral IV catheters

During a screening colonoscopy, a 70-year-old man is discovered to have ascending colon


cancer. As part of his preoperative evaluation, he receives an abdominal CT scan to evaluate for
metastatic disease. Although no lesions consistent with metastatic disease are seen on CT scan,
the gallbladder has significant intramural calcification. The calcified walls appear thickened,
although there is no pericholecystic fluid present. No gallstones are noted. The man denies any
complaints of abdominal pain and reports feeling healthy. Vital signs and physical examination,
including a careful abdominal examination, are within normal limits. Which of the following is
an appropriate treatment for this patient's calcified gallbladder?
 Incorrect Answer ImageA.Biopsy of gallbladder wall at the time of his colectomy
 Incorrect Answer ImageB.Biliary stenting prior to colectomy
 Correct Answer Image C.Cholecystectomy at the time of his colectomy
 Incorrect Answer ImageD.Reassurance that the finding is inconsequential
 Incorrect Answer ImageE.Repeat CT scans at 6 months and 1 year

A 62-year-old man undergoes exploratory laparotomy and repair of a bleeding duodenal ulcer. A
central line and a Foley catheter are inserted before the surgery. IV cefazolin and gentamicin are
also administered. He is stable throughout surgery, and he makes 400 mL of urine during the 3-
hour procedure. In the recovery room, he is agitated and thrashing until he receives appropriate
analgesics. He is then transferred to the surgical floor, where his nurse notices that he has had no
urine output for 3 hours. The central venous pressure is 12 mm Hg and his blood pressure and
pulse are stable. Which of the following is the most likely etiology of this patient's lack of urine
output?
 Incorrect Answer ImageA.Acute nephrotoxicity from preoperative antibiotics
 Incorrect Answer ImageB.Decreased renal perfusion
 Correct Answer Image C.Kinked Foley catheter
 Incorrect Answer ImageD.Ligation of the ureter
 Incorrect Answer ImageE.Renal vein thrombosis

A 76-year-old man is recovering from an emergency coronary artery bypass graft (CABG) in the
cardiac care unit. His past medical history is significant for diabetes mellitus, hypertension,
hyperlipidemia, and chronic tobacco use. He came to the emergency department with acute-onset
chest pain and was found to have an ST-elevation myocardial infarction. Urgent cardiac
angiography revealed multiple vessel disease. Hemodynamic instability prompted the insertion
of an intra-aortic balloon pump and emergent CABG. He required extensive vasopressor and
ventilator support for 4 days postoperatively but was gradually weaned and extubated on
postoperative day 5. On postoperative day 7, he reports abdominal pain and vomits once. He is
noted to be febrile to 38.8°C (101.8°F). Laboratory studies show WBC count 16,000/mm3,
hemoglobin 9.2 g/dL, and creatinine level 1.8 mg/dL. An abdominal x-ray film is unremarkable.
Ultrasound of the right upper quadrant reveals normal intra- and extrahepatic bile ducts and
dilated gallbladder with no stones but a thickened wall. There is also a small amount of fluid
surrounding the gallbladder. What is the next best step in the management of this patient?
 Incorrect Answer ImageA.CT scan of the abdomen and pelvis
 Incorrect Answer ImageB.Endoscopic retrograde cholangiopancreatography
 Incorrect Answer ImageC.Laparoscopic cholecystectomy
 Incorrect Answer ImageD.Open cholecystectomy
 Correct Answer Image E.Percutaneous cholecystostomy tube placement

A 32-year-old man is struck by a taxi that is going approximately 30 miles per hour. He is
brought to the emergency department where he is noted to be awake, talking, and in no
respiratory distress, but tachycardic to 110/min with blood pressure 136/68 mm Hg. On physical
examination, extreme tenderness of the right calf is noted, with significant right leg edema and a
thready right dorsalis pedis and posterior tibial pulse as compared with the left lower extremity.
X-ray confirms a tibial-fibular fracture. Which of the following is the most appropriate next step
in management?
 Incorrect Answer ImageA.Anticoagulation with IV heparin
 Correct Answer Image B.CT angiography of the lower extremity
 Incorrect Answer ImageC.Immediate surgical exploration
 Incorrect Answer ImageD.Observation in the ICU
 Incorrect Answer ImageE.Open reduction and internal fixation of the fractures

A 36-year-old man is brought to the emergency department after being assaulted in an alleyway.
He was beaten with a blunt object and received multiple blows to the face, head, and chest. He
lost consciousness at the scene. He is arousable upon arrival in the emergency department, and
his Glasgow coma scale score (GCS) is 10. His pulse is 110/min, respirations are 28/min, and
blood pressure is 122/76 mm Hg. Oxygen saturation on room air is 92%. Physical examination
shows that he sustained multiple facial fractures. He also has soft-tissue swelling over his vertex
and over his face. Trachea is midline. There is exquisite lateral thoracic tenderness with no
crepitus. Lungs are clear bilaterally with diminished breath sounds on the left. Cardiovascular
and abdominal examinations are unremarkable. Computed tomography of the head and face
shows subarachnoid blood, a nondepressed skull fracture, and multiple right orbital fractures,
including a medial blow-out fracture and a lateral orbital wall fracture. A right retro-orbital
hematoma and right globe proptosis with tenting at the junction of the optic nerve and globe are
identified. Which of the following is the most appropriate next step in management?
 Incorrect Answer ImageA.Admit for observation and repeat CT scan in 2 hours
 Incorrect Answer ImageB.Cranial burr holes
 Incorrect Answer ImageC.Orotracheal intubation
 Incorrect Answer ImageD.Placement of a left tube thoracostomy (chest tube)
 Correct Answer Image E.Stat ophthalmologic consult

A 57-year-old man with a history of alcohol abuse is brought to the emergency department
because of severe epigastric abdominal pain, nausea, and vomiting for 2 days. The pain is
constant and radiates through to the back. His temperature is 38.8ºC (101.8ºF), pulse is 100/min,
and respirations are 22/min. On physical examination he is noted to have bruising along the
flanks. He is admitted to the ICU where he is treated for several days with mechanical
ventilation, intravenous fluids, and antibiotics. Eventually he improves and is transferred to the
medical floor. Three days after leaving the unit and approximately 2 weeks after the onset of his
illness, his condition worsens again. His temperature is 40.1ºC (104.2ºF). Laboratory studies
show:
Hematologic
Hemoglobin 13 g/dL
Hematocrit 43%
Leukocyte count 14,500/mm3
Segmented neutrophils 85%
Lymphocytes 10%
Basophils 0.5%
Monocytes 1%
Eosinophils 3%
Urinalysis shows:
pH 6.5
WBC 2-3/hpf
RBC 0/hpf
Nitrites Negative
Casts Negative
A CT scan of the abdomen shows inflammatory changes of the pancreas with areas that do not
enhance with IV contrast. These recent findings most likely represent the development of which
of the following?
 Incorrect Answer ImageA.Cholecystitis
 Correct Answer Image B.Infected pancreatic necrosis
 Incorrect Answer ImageC.Pancreatic abscess
 Incorrect Answer ImageD.Pancreatic pseudocyst
 Incorrect Answer ImageE.Systemic inflammatory response syndrome (SIRS)

A 27-year-old man is brought to the emergency department because he sustained penetrating


injuries of the chest and abdomen when he was repeatedly stabbed with a long ice pick. At the
time of admission, he has a right pneumothorax, for which a chest tube is placed before he
undergoes general anesthesia for an exploratory laparotomy. The operation reveals no intra-
abdominal injuries and is terminated sooner than had been anticipated. The patient remains
intubated, waiting for the anesthetic to wear off. Because he is not moving enough air, he is
placed on a ventilator. He then suddenly goes into cardiac arrest and dies. Throughout this entire
time he has been hemodynamically stable and never had any signs of hypotension or
arrhythmias. Which of the following is the most likely cause of his cardiac arrest?
 Correct Answer Image A.Air embolism
 Incorrect Answer ImageB.Fat embolism
 Incorrect Answer ImageC.Myocardial infarction
 Incorrect Answer ImageD.Pulmonary embolus
 Incorrect Answer ImageE.Tension pneumothorax

A 23-year-old male college student is brought to the emergency department because of chest
pain. The pain had a sudden onset this morning after repeated episodes of bilious, nonbloody
vomiting that the patient attributes to his eating "spoiled Chinese takeout" the night before. The
pain is not worse with breathing but does radiate to the left shoulder. It has not remitted in the
hour since the initial event. He has no significant past medical history. His family history and
social history are noncontributory. He is febrile and tachypneic. Breath sounds are normal
bilaterally. Palpation at the base of his neck is suggestive of subcutaneous air. The abdomen is
slightly distended and his epigastric region is tender to palpation. Initial laboratory studies are
sent for analysis. Which of the following is the next step in management?
 Incorrect Answer ImageA.Barium swallow study
 Correct Answer Image B.Gastrografin swallow study
 Incorrect Answer ImageC.Nasogastric tube placement
 Incorrect Answer ImageD.Ultrasound of the abdomen
 Incorrect Answer ImageE.Upper endoscopy

A 62-year-old man is brought to the emergency department after sustaining a fall. His medical
history is significant for hypertension, dyslipidemia, and migraines. He is hemodynamically
stable. On physical examination, he is found to have severe right hip tenderness that is worse on
abduction and external rotation. X-rays confirm a femoral fracture. He is taken to the operating
room where open reduction and internal fixation are performed. The operation is uneventful. For
several days postoperatively, however, he is in significant pain with a limited range of motion,
resulting in the need for bed rest and heavy narcotic use. Which of the following is the most
appropriate means of prevention of a deep venous thrombosis in this patient?
 Incorrect Answer ImageA.Aspirin, 81 mg daily
 Correct Answer Image B.Low molecular-weight heparin, 40 mg subcutaneously daily
 Incorrect Answer ImageC.Passive range-of-motion exercises
 Incorrect Answer ImageD.Sequential compression devices
 Incorrect Answer ImageE.Unfractionated heparin, 5,000 units subcutaneously twice daily

A 55-year-old woman comes to the emergency department because of a 3-day history of chills,
spiking fever, and right-sided abdominal pain. She reports that she underwent a laparoscopic
cholecystectomy for acute cholecystitis at another hospital three months ago. Four days after
surgery she became deeply jaundiced and required a second operation, an open laparotomy, but
she does not know exactly what was found or done. Her temperature is 38.7°C (101.7°F), pulse
is 110/min, and blood pressure is 110/60 mm Hg. Today, she is jaundiced and has moderate
tenderness in her right upper quadrant (RUQ) without peritonitis. Her laboratory studies show
the following: 
Total bilirubin 8 mg/dL
Indirect bilirubin 1 mg/dL
ALT 56 mg/dL
AST 52 mg/dL
Alkaline phosphatase 1,052 mg/dL
Ultrasound shows dilated intrahepatic ducts, but the common duct could not be visualized.
Which of the following is the most likely diagnosis at this time? 
 Incorrect Answer ImageA.Biloma
 Incorrect Answer ImageB.Common bile duct stone
 Correct Answer Image C.Common bile duct stricture
 Incorrect Answer ImageD.Sclerosing cholangitis
 Incorrect Answer ImageE.Subphrenic abscess

A 53-year-old man is brought to the emergency department after a motor vehicle accident with
blunt abdominal trauma. During surgery, the patient is found to have a perforation of the small
bowel, and he undergoes resection and anastomosis of the affected area. The patient has an
uneventful postoperative course until the eighth postoperative day when he develops a fever of
38.7°C (101.7°F). He has been tolerating oral intake since the fifth postoperative day, and at this
time his physical examination is unremarkable except for some mild tenderness in the
periumbilical area. The wound is also unremarkable. Laboratory studies show: 
Hemoglobin  13 g/dL
Hematocrit  43%
WBC 12,400/mm3
Segmented neutrophils  85%
Lymphocytes  10%
Basophils  0.5%
Monocytes  1%
Eosinophils  3%
Aside from blood and urine cultures, what is the most appropriate next step in management? 
 Incorrect Answer ImageA.Begin broad-spectrum antibiotics
 Correct Answer Image B.CT scan of the abdomen and pelvis
 Incorrect Answer ImageC.Encourage use of incentive spirometer
 Incorrect Answer ImageD.Open the abdominal wound
 Incorrect Answer ImageE.Return to the OR for re-exploration

A 54-year-old obese man comes to the physician because of burning retrosternal pain and
heartburn that is brought about by bending over, wearing a tight belt, or lying flat in bed at night.
He gets symptomatic relief from over-the-counter antacids. The problem has been present for
many years and seems to be progressing. He does not smoke cigarettes, and he occasionally
drinks alcohol. Which of the following is the most appropriate next step in management? 
 Incorrect Answer ImageA.Barium swallow
 Incorrect Answer ImageB.Cardiac enzymes and electrocardiogram
 Correct Answer Image C.Endoscopy and biopsies
 Incorrect Answer ImageD.Hospitalization for intravenous proton pump inhibitors
 Incorrect Answer ImageE.Laparoscopic Nissen fundoplication

A 66-year-old woman complains of severe chest pain and shortness of breath. She underwent
laparotomy for pancreatic cancer approximately 1 week ago. When examined today, she is found
to be anxious, diaphoretic, and tachycardic. Her pulse is 112/min, respirations are 24/min, blood
pressure is 90/55 mm Hg, and oxygen saturation is 88% on room air. Her BMI is 31 kg/m2. She
has a loud S2 click with prominent distended veins in the neck and forehead. Arterial blood gas
shows hypoxemia and hypocapnia. Chest x-ray shows atelectatic changes near the bases. ECG
shows sinus tachycardia. The nurses place her on supplemental oxygen by face mask with
improved oxygen saturation to 98%. Her tachypnea and anxiety subsequently improve as well.
Which of the following is the most appropriate next step?
 Incorrect Answer ImageA.Cardiac enzymes
 Correct Answer Image B.CT pulmonary angiography
 Incorrect Answer ImageC.Intubation and mechanical ventilation
 Incorrect Answer ImageD.Pulmonary angiography, catheter-directed
 Incorrect Answer ImageE.Ventilation-perfusion lung scan

A 24-year-old woman who recently gave birth comes to the physician because of ongoing rectal
pain for the past several weeks. She notes extreme pain when defecating, and there is bright red
blood in her stools and on the toilet paper. She also feels pain with coughing, and it is relieved
slightly by standing. She is nursing, and reports being constipated since her delivery. Her past
medical history is unremarkable and she takes no medications. On examination, there is a 1 cm,
thin, fleshy tag in the posterior midline at the anal verge, and she is too tender to perform a rectal
exam. What is the most likely diagnosis? 
 Correct Answer Image A.Anal fissure
 Incorrect Answer ImageB.Infarcted perianal skin tag
 Incorrect Answer ImageC.Perianal abscess
 Incorrect Answer ImageD.Prolapsed internal hemorrhoid
 Incorrect Answer ImageE.Thrombosed external hemorrhoid

A 62-year-old woman had an abdominal hysterectomy and salpingo-oophorectomy 3 days ago.


She had an indwelling bladder catheter during the procedure, which was removed in the recovery
room. She has been voiding voluntarily since then. She also had compression pneumatic
stockings on both lower extremities during the operation. She began ambulating on the first
postoperative day and has been as active as possible under the circumstances. She also has had
good adherence to a prescribed program of incentive spirometry. On the evening of the third
postoperative day, she spikes a fever with a temperature to 39.4ºC (103ºF). Which of the
following is the most likely source of the fever?
 Incorrect Answer ImageA.Atelectasis
 Incorrect Answer ImageB.Deep venous thrombosis
 Incorrect Answer ImageC.Intra-abdominal abscess
 Correct Answer Image D.Urinary tract infection
 Incorrect Answer ImageE.Wound infection
A 31-year-old woman crashes her car into a bridge abutment. She sustains multiple injuries,
including upper and lower extremity fractures. When she is brought to the emergency
department, she is fully awake and alert. She reports that she was not wearing a seat belt, and she
distinctly remembers hitting her abdomen against the steering wheel. Her pulse is 120/min and
blood pressure is 80/50 mm Hg. After rapid administration of 2 liters of intravenous fluids, her
blood pressure is 84/52 mm Hg. Physical examination shows that she has a rigid and tender
abdomen. There is severe tenderness when external pressure is applied to her abdomen and then
suddenly released. She has no bowel sounds. Which of the following is the most appropriate next
step in evaluating her potential intra-abdominal injuries?
 Incorrect Answer ImageA.Continued clinical observation
 Incorrect Answer ImageB.CT scan of the abdomen
 Incorrect Answer ImageC.Diagnostic peritoneal lavage
 Correct Answer Image D.Exploratory laparotomy
 Incorrect Answer ImageE.Focused assessment with sonography for trauma (FAST) scan

In the course of a robbery, a 30-year-old woman is stabbed repeatedly. On arrival at the


emergency department, she is shivering and asks for a blanket and a drink of water. She is noted
to be pale and perspiring. Her blood pressure is 72/50 mm Hg and her pulse is 130/min. The neck
and forehead veins are large and distended. A quick initial survey reveals entry wounds in the
left chest and upper abdomen. She has bilateral breath sounds and a scaphoid, nontender
abdomen. As IV infusions of Ringer's lactate are started, her systolic blood pressure drops
further to 40 mm Hg, no distal pulses can be felt, and she loses consciousness. Her central
venous pressure at that time is 28 cm H2O. Which of the following is the most appropriate next
step in management?
 Incorrect Answer ImageA.Bilateral chest tubes
 Incorrect Answer ImageB.Chest x-ray to direct further therapy
 Incorrect Answer ImageC.Diagnostic peritoneal lavage
 Correct Answer Image D.Evacuation of the pericardial sac
 Incorrect Answer ImageE.Laparotomy in the emergency department to clamp the aorta

A 19-year-old man comes to the emergency department after being shot in the abdomen. He is
hemodynamically unstable. During emergent exploratory laparotomy, more than 2 L of blood are
found in the abdomen on entry. All 4 quadrants are packed. When the packing is removed, blood
is noted to be coming from the right upper quadrant. A Pringle maneuver (atraumatic clamping
of the hepatoduodenal ligament) is used, but blood continues to flow into the field, and the
patient remains hypotensive. What anatomic structure is likely injured?
 Correct Answer Image A.Hepatic veins
 Incorrect Answer ImageB.Left hepatic artery
 Incorrect Answer ImageC.Portal vein
 Incorrect Answer ImageD.Right hepatic artery
 Incorrect Answer ImageE.Splenic artery

A 56-year-old woman who smokes 1 pack of cigarettes per day undergoes a right hemicolectomy
for colon cancer. A stapled anastomosis is performed and no intraoperative complications are
encountered. On the night of the surgery, the physician is called to evaluate the patient because
of her temperature of 38.9ºC (102.0ºF). The patient is comfortable except for normal incisional
pain. Her nasogastric tube is in place, pulse is 102/min, respirations are 16/min, and blood
pressure is 123/86 mm Hg. Oxygen saturation is 93% on 2 L of nasal cannula oxygen. The
surgical dressing is clean, the Foley catheter is draining clear urine, and there is no lower
extremity edema or tenderness. Sequential compression devices have been implemented since
the time of surgery. Which of the following is the most appropriate next step in management?
 Incorrect Answer ImageA.Change the Foley catheter
 Correct Answer Image B.Encourage the use of the incentive spirometer
 Incorrect Answer ImageC.Order a CT pulmonary angiogram of the chest
 Incorrect Answer ImageD.Explore the surgical incision
 Incorrect Answer ImageE.Order a lower extremity Doppler study

A 61-year-old man is found to have a large polyp in the cecum on screening colonoscopy. He
undergoes a right hemicolectomy, and the pathology reveals a 2 cm adenocarcinoma extending
into but not through the muscularis propria. One of 13 lymph nodes contains a microscopic focus
of tumor. He recovers uneventfully and is discharged home on postoperative day 6. He is seen
back in the office 2 weeks postoperatively and is recovering well. What is the next best step in
the management of this patient’s condition?
 Correct Answer Image A.5-FU based chemotherapy
 Incorrect Answer ImageB.Carcinoembryonic Antigen (CEA) measurement in 1 year
 Incorrect Answer ImageC.Colonoscopy in 3 months
 Incorrect Answer ImageD.PET scan
 Incorrect Answer ImageE.Radiation therapy to the tumor site

A 22-year-old woman is brought to the emergency department. She was trapped in an


underground subway train for 1 hour after a fire in the carriage in which she was riding. She and
her fellow passengers were able to move to an unaffected carriage and were subsequently
rescued. The patient is screaming in pain and has extensive second-degree burns to both upper
and lower limbs. There is also a patch of second-degree burns on her back. When asked whether
she feels pain anywhere else, she says in a hoarse voice that she twisted her right ankle when
jumping from the carriage. Her temperature is 37.3°C (99.1ºF), pulse is 98/min, and blood
pressure is 146/92 mm Hg. Chest examination shows heart sounds that are regular in rate,
rhythm, and character. There is an absence of an inspiratory wheeze bilaterally.
Electrocardiogram is normal in rate and rhythm. The patient is administered 100% oxygen.
Which of the following is the most appropriate next step in management?
 Incorrect Answer ImageA.Administer H2-receptor blockers
 Incorrect Answer ImageB.Bronchoscopy
 Incorrect Answer ImageC.Chest x-ray
 Incorrect Answer ImageD.Cover the burns with Vaseline gauze
 Incorrect Answer ImageE.Determine the body surface area burned and administer fluids
 Incorrect Answer ImageF.Immunize against tetanus
 Incorrect Answer ImageG.Insert a Foley catheter
 Correct Answer Image H.Intubate
 Incorrect Answer ImageI.Measure amylase
 Incorrect Answer ImageJ.Measure carboxyhemoglobin levels
 Incorrect Answer ImageK.X-ray the right ankle
A 74-year-old otherwise healthy man complains of persistent bloody soiling of his underwear.
He has a history of a procedure done for "a problem with his anus" over 15 years ago. He does
not remember the details, other than the fact that he was in a lot of pain and the surgeon told him
he had a bad infection. On physical examination, there is a 3 cm scar on the left side of his anus
and there appears to be a small bud of granulation tissue at the end of the scar. A subcutaneous
cord is palpated from the scar to the anus. Which of the following is the most likely diagnosis?
 Incorrect Answer ImageA.Crohn's disease
 Correct Answer Image B.Fistula-in-ano
 Incorrect Answer ImageC.Perianal abscess
 Incorrect Answer ImageD.Pilonidal sinus
 Incorrect Answer ImageE.Squamous cell carcinoma of the anus

A 36-year-old woman is involved in a head-on collision with a tractor-trailer, and paramedics


find her unconscious, strapped in her seat belt. They remove her from the car, insert an
endotracheal tube, and begin an IV line of Ringer's lactate solution. On arrival in the emergency
department, the patient is still unconscious. Her pulse is 40/min and her blood pressure is 160/90
mm Hg. Pupils are fixed and dilated bilaterally. There is no response to painful stimuli. The only
other injuries are a right-sided pneumothorax and a separated right shoulder, which have been
appropriately managed. There are no obvious injuries to the face or scalp. A CT scan of the head
shows diffuse blurring of the gray-white matter interface, diffuse cerebral edema, and multiple
small, punctate hemorrhages. What is the most appropriate next step in the management of the
patient's neurologic state?
 Correct Answer Image A.Attempt to decrease intracranial pressure without surgery
 Incorrect Answer ImageB.Craniotomy to control arterial bleeding
 Incorrect Answer ImageC.Craniotomy to evacuate venous bleeding
 Incorrect Answer ImageD.Perform MRI scan of the brain
 Incorrect Answer ImageE.Place the patient in Trendelenburg position to increase blood
flow to the brain

A 28-year-old woman is brought to the emergency department by her boyfriend when she began
describing severe, vague, right upper quadrant abdominal pain while at home. She denies recent
trauma or chance of pregnancy as she is compliant with taking her birth control pills. Her pulse is
121/min and her blood pressure is 81/42 mm Hg. Her abdomen is mildly distended and diffusely
tender. Hemoglobin is 8.1 g/dL. Abdominal radiograph films show no evidence of
pneumoperitoneum. Which of the following is the most likely source of this patient's condition?
 Incorrect Answer ImageA.Acute cholecystitis
 Incorrect Answer ImageB.Ascending cholangitis
 Incorrect Answer ImageC.Ectopic pregnancy
 Incorrect Answer ImageD.Perforated duodenal ulcer
 Correct Answer Image E.Ruptured hepatic adenoma

A patient who is in shock due to multiple traumatic injuries is taken to the operating room for an
exploratory laparotomy that lasts 3.5 hours. Multiple blood transfusions are administered, and
several liters of Ringer's lactate are infused during surgery. On the second postoperative day, the
patient develops confusion, agitation, lower extremity edema, and abdominal distension. The
abdominal sutures are cutting through the skin around them. The patient's creatinine level has
doubled since the surgery. Which of the following treatments at the time of the initial surgical
intervention could have prevented these complications? 
 Incorrect Answer ImageA.Approximate the skin only, using towel clips
 Incorrect Answer ImageB.Close the abdomen with heavy retention sutures
 Incorrect Answer ImageC.Give diuretics and close the abdomen in the usual fashion
 Incorrect Answer ImageD.Leave the abdomen and its contents open to air until
postoperative day 3
 Correct Answer Image E.Provide temporary abdominal closure with a negative pressure
device

A 56-year-old man has been having bloody bowel movements on and off for the past several
weeks. He reports that the blood is bright red and it coats the outside of the stools. There is also
blood on the toilet paper. He has no pain. Further questioning shows that he has been constipated
for the past 2 months and that the caliber of his stools is thinner. Which of the following is the
best step in management at this time?
 Incorrect Answer ImageA.Advise him to take fiber supplementation and stool softeners
 Correct Answer Image B.Colonoscopy
 Incorrect Answer ImageC.Corticosteroid suppositories
 Incorrect Answer ImageD.CT scan of the pelvis
 Incorrect Answer ImageE.Treat his internal hemorrhoids with banding

A 44-year-old woman is recovering from a mild episode of acute ascending cholangitis


secondary to choledocholithiasis. When seen initially, she had a spiking fever, leukocytosis, and
a very high alkaline phosphatase level. All these findings subsided rapidly after she was placed
on IV antibiotics. At that time, a sonogram of the right upper quadrant showed the presence of
gallstones in the gallbladder, but the diameter of the biliary ducts was normal. Because she is
suspected of having passed a common duct stone, plans to do an endoscopic retrograde
cholangiopancreatogram (ERCP) were canceled. While awaiting elective cholecystectomy, she
again developed a fever and leukocytosis, and her liver function test showed a minimal elevated
bilirubin level of 2.5 mg/dL and an alkaline phosphatase level of 115 U/L. A repeat sonogram
shows no changes in her bile ducts, but it does show a 6-cm abscess in the right lobe of the liver.
Which of the following is the most appropriate treatment for this new development? 
 Incorrect Answer ImageA.ERCP and biliary drainage
 Incorrect Answer ImageB.Long-term IV antibiotics
 Incorrect Answer ImageC.Metronidazole
 Incorrect Answer ImageD.Open surgical drainage
 Correct Answer Image E.Percutaneous drainage

A 61-year-old man comes to the physician because of spots of blood on the toilet paper on
completing defecation. He notices mild anal discomfort and swelling just after bowel movements
that resolve quickly when he stands up. His symptoms have been intermittent for the past 2
months but have progressed over the past 2 days. He has been travelling and his diet has been
poor lately, resulting in some constipation. He underwent colonoscopy 1 year ago which showed
only diverticulosis. Abdominal and digital rectal examinations show no abnormalities. Anoscopy
shows dilated and erythematous hemorrhoids bulging into the lumen, but they do not prolapse
out of the anal canal. Flexible proctosigmoidoscopic examination shows no other pathology.
Which of the following is the most appropriate next step in management?
 Correct Answer Image A.Advise him to drink adequate fluids and take fiber
supplementation
 Incorrect Answer ImageB.Biopsy of the hemorrhoids to exclude carcinoma
 Incorrect Answer ImageC.Hemorrhoidectomy
 Incorrect Answer ImageD.Topical hydrocortisone cream
 Incorrect Answer ImageE.Urgent colonoscopy

A 36-year-old man is brought to the emergency department on a backboard and with a cervical
collar after being struck by a car while crossing the street. He is agitated and confused, and he
smells of alcohol. Breath sounds are not heard over the right hemithorax, so a chest tube is
placed. He is intubated for airway protection and is hemodynamically stable. He has no other
injuries on primary or secondary survey. CT findings include a fracture of the right transverse
process of his T2 vertebrae and a small residual right hemothorax. CT scan of the head is notable
for a right frontotemporal contusion with a small subarachnoid hemorrhage and multiple
nondisplaced calvarial fractures. He remains stable and is brought to the surgical ICU for
monitoring. His chest tube drains minimal serosanguineous fluid, then ceases. The following day
he becomes bradycardic to approximately 40/min, with blood pressure 160/100 mm Hg. He
becomes agitated easily and is not overbreathing the ventilator. Pupils are symmetric but
sluggish. A second CT scan is scheduled. What is the next step in management?
 Incorrect Answer ImageA.Administration of atropine
 Correct Answer Image B.Administration of mannitol
 Incorrect Answer ImageC.Emergent craniotomy
 Incorrect Answer ImageD.Removal of the hard cervical collar
 Incorrect Answer ImageE.Ventilator weaning and extubation

A 55-year-old man comes to the urgent care clinic complaining of groin pain. He reports the pain
as intense, constant, and dull. He also thinks he noticed a new bulge in his groin. Past medical
and surgical history is significant for hypertension. His temperature is 38.0ºC (100.4ºF), pulse
98/min, respirations 16/min, and blood pressure 138/80 mm Hg. Examination of the groin shows
mild right-sided erythema and a mildly tender mass can be felt when the examiner's finger is
placed in the internal inguinal ring. The size and presence of the mass are unaffected by cough or
strain. The remainder of the examination is otherwise unremarkable. Which of the following is
the most appropriate next step in the management of this patient?
 Incorrect Answer ImageA.Elective surgical repair
 Incorrect Answer ImageB.Emergent puncture and drainage of the inguinal abscess
 Correct Answer Image C.Emergent surgical repair
 Incorrect Answer ImageD.Reassurance, ice compresses, and rest
 Incorrect Answer ImageE.Ultrasonography of the mass

A 67-year-old restrained driver is involved in a motor vehicle accident. He is stabilized by


paramedics at the scene and is brought to the trauma center on a backboard with a cervical collar
in place. He reports left-sided chest pain with shortness of breath. His pulse is 87/min,
respirations are 24/min, blood pressure is 110/56 mm Hg, and oxygen saturation is 91% on room
air. Physical examination shows a seat-belt abrasion over the left hemithorax, with decreased
breath sounds and gurgling sounds over the left chest. His abdomen is nondistended and soft,
without peritoneal signs. His pelvis is stable, but there is deformity of the left lower extremity
consistent with a femur fracture. Initial chest x-ray shows elevation of the left hemidiaphragm; a
second chest x-ray is pending. Which of the following is the most likely diagnosis?
 Incorrect Answer ImageA.Fat embolus
 Incorrect Answer ImageB.Flail chest
 Incorrect Answer ImageC.Hemothorax
 Incorrect Answer ImageD.Tension pneumothorax
 Correct Answer Image E.Traumatic diaphragmatic rupture
 Incorrect Answer ImageF.Traumatic phrenic nerve injury

An 18-year-old man is brought to the emergency department after sustaining a fall from a 3-story
building. He landed on his right leg and heard a snap. He felt excruciating pain in his thigh and
noticed that it appeared deformed. His past medical history is unremarkable, and he does not
receive any medications. On physical examination, his pulse is 95/min and blood pressure is
152/78 mm Hg. Head, neck, cardiovascular, pulmonary, and abdominal examinations are
unremarkable. There is extreme tenderness to movement and palpation of the right thigh, and
there is marked swelling over the region. The compartmental pressure of the right thigh is 15 mm
Hg. The popliteal and pedal pulses on the right are barely palpable. Corresponding left-sided
pulses are 2+. Radiograph film shows a comminuted fracture through the right femoral diaphysis
with lateral angulation. Laboratory studies show:
White blood cells  6,700/mm3
Hematocrit  42% 
Platelets  289,000/mm3
INR  1.0 
Partial thromboplastin time 20 seconds
Which of the following is the next step in management? 
 Correct Answer Image A.CT angiography and splinting
 Incorrect Answer ImageB.Emergent fasciotomy
 Incorrect Answer ImageC.Reduction of the fracture
 Incorrect Answer ImageD.Resuscitation with packed red blood cells
 Incorrect Answer ImageE.Vigorous hydration

A 21-year-old man walks into the emergency department after he is stabbed in the right chest
during an altercation in front of the hospital. He is short of breath, and this persists even after he
is placed on a stretcher and given supplemental oxygen. His pulse is 102/min and blood pressure
is 88/54 mm Hg. No injuries are identified over the head and neck. Auscultation over the right
chest reveals no breath sounds. The right chest is also dull to percussion. There are adequate
heart and breath sounds over the left chest. There are abrasions over the anterior abdominal wall.
A chest tube is placed into the right pleural space and 1,600 mL of blood is drained. Which of
the following is the most appropriate next step in management? 
 Incorrect Answer ImageA.Admit to the hospital for observation
 Incorrect Answer ImageB.Order a CT scan of the abdomen and pelvis
 Incorrect Answer ImageC.Order a CT scan of the chest
 Incorrect Answer ImageD.Order plain film radiographs of the chest
 Correct Answer Image E.Perform thoracotomy

After a grand mal seizure, a 32-year-old epileptic woman finds it too painful to move her right
shoulder. She comes to the emergency department where an anteroposterior x-ray film of her
shoulder is performed, which is negative. She is administered pain medication and discharged
home. The next day she has the same pain and is unable to move her arm. She returns to the
emergency department holding her arm close to her body, with her hand resting on the anterior
chest wall. Which of the following is the most likely diagnosis?
 Incorrect Answer ImageA.Acromioclavicular separation
 Incorrect Answer ImageB.Anterior dislocation of the shoulder
 Incorrect Answer ImageC.Articular cartilage crushing
 Correct Answer Image D.Posterior dislocation of the shoulder
 Incorrect Answer ImageE.Torn teres major and minor muscles

A 33-year-old man comes to his physician with a mass in his scrotum that he first noticed a week
ago. The mass is painless, and he is not sure how long it has been present. His past medical
history is unremarkable. He does not have a family history of cancer. On physical examination,
he is afebrile and normotensive. Head and neck, chest, cardiovascular, abdominal, and peripheral
examinations are normal. Palpation of the scrotum reveals a smooth, painless, non-indurated
mass just above his right testicle. The lesion transilluminates when a light is shined through it.
The testicles are normal. A scrotal ultrasound reveals a cystic lesion without internal echoes in
the right epididymis. Which of the following is the next best step in management?
 Incorrect Answer ImageA.Aspiration of the cyst
 Incorrect Answer ImageB.Follow up ultrasound in 3 months
 Correct Answer Image C.Reassurance that it is benign and should resolve
 Incorrect Answer ImageD.Staging CT scan of the chest and abdomen
 Incorrect Answer ImageE.Surgical excision

A 71-year-old woman is brought to the physician by her distressed daughter. The daughter
reports that 3 days ago, her mother began to report right upper quadrant abdominal pain. The
woman did not want to eat and "took to her bed sick." The daughter recalls the woman reported
chills, nausea, and some vomiting. Physical examination shows an obtunded, hypotensive, and
obviously very sick elderly woman. The woman has severe pain to deep palpation in the right
upper quadrant with guarding and rebound tenderness. Her temperature is 40°C (104°F), and
laboratory analysis shows a white blood cell count of 22,000/mm3 with multiple immature forms,
a bilirubin level of 5 mg/dL, and alkaline phosphatase level of 840 U/L. Her serum amylase level
is normal. An emergency sonogram shows multiple stones in the gallbladder, normal thickness
of the gallbladder wall without pericholecystic fluid, dilated intrahepatic ducts, and common bile
duct diameter of 2.1 cm. The sonographer cannot identify stones in the common bile duct. In
addition to IV fluids and antibiotics, which of the following is the most appropriate next step in
management?
 Incorrect Answer ImageA.Elective cholecystectomy
 Incorrect Answer ImageB.Emergency cholecystectomy
 Correct Answer Image C.Emergency decompression of the common bile duct
 Incorrect Answer ImageD.Emergency percutaneous cholecystostomy
 Incorrect Answer ImageE.Emergency surgical removal of all stones from the common
bile duct

A 59-year-old man comes to the physician because of a painless sore on the tip of his penis. The
lesion has been present for almost 3 months, but he has been too embarrassed to seek treatment.
Initially, the lesion was very small with a rough surface; however, the lesion has begun to
enlarge and has slowly developed into a painless ulcer. He denies any prior similar lesions. He
has no history of sexually transmitted diseases. He is divorced, has had 2 sexual partners in the
past year, and rarely uses condoms. He has had no sexual contact since the sore initially
manifested. He denies any fever, chills, weight loss, urinary urgency, frequency, or dysuria.
There are small, palpable lymph nodes in his inguinal area bilaterally. His penis is
uncircumcised, and the foreskin retracts easily to reveal a 1.5 cm, nontender, ulcerated lesion on
the glans of the penis. The surface is red and granular. The testes are palpable and without
masses. There are no hernias. The rectal examination is normal. Venereal disease research
laboratory (VDRL) test is negative. Urinalysis shows no red blood cells, no white blood cells,
and no leukocyte esterase or nitrites. Which of the following is the most appropriate next step in
management?
 Incorrect Answer ImageA.Have the patient undergo a circumcision
 Incorrect Answer ImageB.Prescribe a course of acyclovir
 Incorrect Answer ImageC.Prescribe an antifungal cream for the lesion
 Correct Answer Image D.Refer the patient for a biopsy of the lesion
 Incorrect Answer ImageE.Treat the patient for gonorrhea and chlamydia
 Incorrect Answer ImageF.Vaccinate the patient for human papillomavirus (HPV)

A 2-year-old boy diagnosed with hydrocephalus shortly after birth has a ventriculoperitoneal
(VP) shunt that was placed at 3 months of age. The family has been instructed in the proper care
of the shunt, but over the last 3 days, the child has become irritable, complains of headache, and
has developed nausea and vomiting. This morning, the family noted impairment of the boy's
upward gaze. On examination, his temperature is 37.8ºC (100.0ºF), pulse is 68/min, respirations
are 18/min, and blood pressure is 111/62 mm Hg. Physical examination shows no focal
neurologic deficit. Which of the following is the most likely explanation for this boy's condition?
 Incorrect Answer ImageA.Intracranial hemorrhage
 Correct Answer Image B.Obstruction of the shunt
 Incorrect Answer ImageC.Shunt infection
 Incorrect Answer ImageD.The child has outgrown the shunt
 Incorrect Answer ImageE.The child no longer needs the shunt

A 27-year-old man is brought to the emergency department because of severe left knee pain. He
was playing in a community league football game after being tackled from behind during a
community league football game because of severe left knee pain. Physical examination shows a
markedly swollen left knee. The lower leg can be pulled anteriorly with the knee flexed at 90
degrees. A similar finding is elicited with the knee flexed at 30 degrees by grasping the thigh
with one hand and pulling the lower leg anteriorly with the other. MRI scan of the knee confirms
the suspected diagnosis. Which of the following is the most appropriate management for this
patient's condition?
 Correct Answer Image A.Anterior cruciate ligament surgical reconstruction
 Incorrect Answer ImageB.Arthroscopic meniscectomy
 Incorrect Answer ImageC.Hinged cast for collateral ligament injury
 Incorrect Answer ImageD.Immobilization with a knee brace and rehabilitation
 Incorrect Answer ImageE.Physical therapy for the posterior cruciate ligament tear

A 54-year-old man is admitted to the hospital because of protracted vomiting and progressive
abdominal distention. The symptoms began 5 days ago, and he has not had a bowel movement or
passed flatus since then. He underwent an exploratory laparotomy 5 years ago after a gunshot
wound to the abdomen. He has no other pertinent past medical history. Physical examination
shows hyperactive bowel sounds and some abdominal discomfort but no tenderness or rigidity.
His abdominal x-ray shows dilated loops of small bowel, multiple air-fluid levels, and no free air
under the diaphragm. Nasogastric suctioning and intravenous fluids are initiated. After 6 hours,
he develops a fever, leukocytosis, and abdominal pain. There is severe tenderness when external
pressure is applied to his abdomen and then suddenly released. Which of the following is the
most appropriate next step in management?
 Incorrect Answer ImageA.Antibiotics and serial examinations
 Incorrect Answer ImageB.CT scan of the abdomen
 Correct Answer Image C.Exploratory laparotomy
 Incorrect Answer ImageD.Upper gastrointestinal endoscopy and introduction of a long
intestinal tube
 Incorrect Answer ImageE.Upper gastrointestinal series

A 22-year-old man is being prepared for an inguinal hernia repair. Shortly after a spinal
anesthetic is placed, his blood pressure decreases to 75/20 mm Hg. His central venous pressure is
near 0. He is otherwise healthy and has no history of blood pressure abnormalities. Which of the
following should be included in his therapy? 
 Incorrect Answer ImageA.Intra-aortic balloon pump
 Incorrect Answer ImageB.Pain medication and oxygen
 Correct Answer Image C.Vasoconstrictors
 Incorrect Answer ImageD.Vasodilators
 Incorrect Answer ImageE.Whole blood transfusion

A 72-year-old woman is admitted to the hospital with an acute myocardial infarction. She is
quickly brought to the cardiac angiography suite, where she is found to have severe 3-vessel
disease. She is taken to the operating room for an emergent coronary artery bypass graft. She
recovers well and is extubated on the first postoperative day. Her chest tubes are removed on
postoperative day 2, and she is started on a clear liquid diet. On postoperative day 3, she
develops abdominal pain and acute abdominal distention. She is made NPO and a nasogastric
tube is inserted with only minimal bilious output. Fluids and electrolytes are kept within normal
parameters, and an abdominal x-ray film is obtained. It shows only minimally dilated loops of
small bowel but diffusely dilated large bowel, with the transverse colon measuring 10 cm. No
free air under the diaphragm is visualized. A bedside proctosigmoidoscopic examination does
not reveal an obstructing carcinoma and easily visualizes the distended part of the colon. Some
air is suctioned out, and she improves. However, the following day she is distended again, and
the abdominal x-ray is unchanged. What is the next step in management? 
 Correct Answer Image A.Administration of IV neostigmine
 Incorrect Answer ImageB.Advancement of the nasogastric tube
 Incorrect Answer ImageC.CT scan of the abdomen and pelvis with oral contrast
 Incorrect Answer ImageD.Exploratory laparotomy
 Incorrect Answer ImageE.Urgent colonoscopy

An 89-year-old woman is brought to the emergency department via ambulance because of chest
pain. Six hours previously, she began to experience a dull, "crushing" chest pain that radiated to
her left arm. She also reports shortness of breath and describes feeling very nauseous. Her
temperature is 37.8ºC (100ºF), pulse is 128/min, and blood pressure is 154/92 mm Hg. On
physical examination, the patient appears moribund. She is breathing 100% oxygen via a
facemask. She becomes chest pain-free on aspirin, intravenous heparin, beta-blockers,
intravenous nitrates, and eptifibatide. A few days after admission to the cardiac care unit, her
blood pressure drops to 102/60 mm Hg and her pulse is 126/min. Auscultation of her chest
shows scattered rales and wheezes. Chest palpation shows a systolic thrill. Cardiovascular
examination shows a pansystolic murmur heard best at the apex and radiating to the axilla.
Which of the following is the next best step in management?
 Incorrect Answer ImageA.Coronary angiography
 Incorrect Answer ImageB.Give beta-blockers
 Incorrect Answer ImageC.Give streptokinase
 Incorrect Answer ImageD.Give tissue plasminogen activator
 Incorrect Answer ImageE.Intravenous fluids
 Incorrect Answer ImageF.Pericardiocentesis with pericardial window
 Correct Answer Image G.Transfer the patient to the operating room

 21-year-old man celebrating his team's victory while standing on his seat at a sporting event
slips and falls in such a way that he is straddling the seatback when he lands. There is immediate
pain in his genital area. Physical examination shows his vital signs to be stable, but his bladder is
palpable to the level of the umbilicus; there is no evidence of ecchymosis in the flank or back.
Genital examination reveals gross blood at the urethral meatus. There is a butterfly pattern of
ecchymosis over the scrotum and perineum. Testicles are palpable and mildly tender but
descended appropriately within the scrotum. The prostate is normal in location, size, and
consistency. The patient feels the need to urinate but is only able to generate a few drops of fresh
blood from the urethral meatus. Which of the following is the most appropriate next step in
management? 
 Incorrect Answer ImageA.Obtain a scrotal ultrasound
 Correct Answer Image B.Order a retrograde urethrogram
 Incorrect Answer ImageC.Perform a cystoscopy
 Incorrect Answer ImageD.Place a Foley catheter
 Incorrect Answer ImageE.Schedule a CT scan of the abdomen and pelvis

A 66-year-old woman with no significant past medical history comes to the emergency
department because of right upper quadrant pain, nausea, and vomiting for 2 days. She denies
any fever, chills, or diarrhea. Physical examination shows mild tenderness in the right upper
quadrant (RUQ) but no rebound or guarding. Laboratory studies are within normal limits, and an
abdominal ultrasound shows gallstones but no other abnormalities. She is administered pain
medication, which alleviates the symptoms, and she is discharged to follow up with her family
physician. A week later, she returns to the emergency department because of right upper
quadrant pain, fever, and chills. Her temperature is 40.0ºC (104.1ºF), pulse is 112/min,
respirations are 18/min, and oxygen saturation is 96% on room air. Physical examination shows
scleral icterus and there is moderate RUQ tenderness with no masses or guarding. Laboratory
studies show:
Hematocrit 39%
Leukocyte count 21,500/mm3
Segmented neutrophils 90%
Lymphocytes  10%
Basophils 0.5%
Monocytes 1%
Eosinophils 3%
Aspartate aminotransferase (AST)  126 U/L
Alanine aminotransferase (ALT) 221 U/L
Alkaline phosphatase 512 U/L
Amylase  110 U/L
Creatinine  1.1 mg/dL
Urea nitrogen 15 mg/dL
Total bilirubin  7.2 mg/dL
Direct bilirubin  4.2 mg/dL
Indirect bilirubin  3 mg/dL
Lactate dehydrogenase  250 U/dL
Which of the following is the most likely diagnosis?
 Incorrect Answer ImageA.Acute cholecystitis
 Incorrect Answer ImageB.Acute infectious hepatitis
 Incorrect Answer ImageC.Carcinoma of the pancreas
 Correct Answer Image D.Cholangitis
 Incorrect Answer ImageE.Peptic ulcer disease

A 2-year-old boy is brought to the emergency department by his parents because of limping. His
parents report that he was well until yesterday when he fell in the bathtub. Since then, he has
been limping and refusing to bear weight on his right leg. A radiograph of the right femur shows
a displaced transverse fracture of the shaft of the femur without soft tissue changes. The child's
leg is immobilized and stabilized. Complete blood count and coagulation studies are within
normal limits. Which of the following is the most appropriate initial step in management?
 Incorrect Answer ImageA.Application of traction
 Incorrect Answer ImageB.Obtain a genetics consult for osteogenesis imperfecta
 Correct Answer Image C.Obtain a skeletal survey
 Incorrect Answer ImageD.Perform closed reduction and external fixation
 Incorrect Answer ImageE.Perform open reduction and internal fixation
 Incorrect Answer ImageF.Placement of a spica cast

A 12-year-old boy hits a crack on the sidewalk and is thrown from his bicycle, landing on the
curb of the street and injuring his right shoulder and arm. When he first comes to the emergency
department, he is noted to have extensive abrasions and ecchymosis over the lateral aspect of his
right upper arm and shoulder, exquisite tenderness, and his range of motion is limited by pain.
He does not complain of any distal paresthesias. Radial pulse is noted to be 2+. Radiograph films
of the shoulder and arm reveal a fracture of the junction of the middle and distal third of the
humerus. The orthopedic team reduces the fracture and casts the arm. While waiting for a second
x-ray, the boy complains of numbness in his thumb and is unable to extend his wrist. His radial
pulse is still 2+. What is the next step in management?
 Incorrect Answer ImageA.Immediate operative exploration
 Incorrect Answer ImageB.MRI of the arm
 Correct Answer Image C.Re-manipulation by the orthopedic team
 Incorrect Answer ImageD.Wait for the repeat x-ray
 Incorrect Answer ImageE.Warm compress and elevation of the hand

A 55-year-old woman falls in the shower and hurts her right shoulder. She comes to the
emergency department with her arm held close to her body and the forearm rotated outward as if
she were going to shake hands. She is in pain and will not move the arm from that position. Her
right shoulder looks "square" in comparison with the rounded, unhurt opposite side, and there is
numbness in a small area of the right shoulder over the deltoid muscle. Which of the following is
the most likely diagnosis? 
 Incorrect Answer ImageA.Acromioclavicular separation
 Correct Answer Image B.Anterior dislocation of the shoulder
 Incorrect Answer ImageC.Colles fracture
 Incorrect Answer ImageD.Fracture of the upper end of the humeral shaft
 Incorrect Answer ImageE.Posterior dislocation of the shoulder

A 58-year-old man comes to the emergency department because for the last 4 days, he has
suffered spiking fevers and chills, low back pain, perineal pain, urinary hesitancy, a weak urinary
stream, and a "burning" pain when he urinates. He describes his urine as dark and cloudy. He
also endorses diffuse myalgias and fatigue during the last 4 days. His past medical history
includes impaired glucose tolerance, borderline hypertension, and distant history of depression.
He has never had a sexually transmitted disease and reports having sex with one partner during
the last year. The patient’s temperature is 38.3°C (100.9°F), pulse is 78/min, respirations are
20/min, and blood pressure is 138/92 mm Hg. Abdominal examination shows some mild
suprapubic tenderness. Rectal examination shows a tender, boggy prostate. Urine is sent for
culture and Gram stain, and the results are pending. Which of the following is the most
appropriate next step in management?
 Incorrect Answer ImageA.A single dose of intramuscular ceftriaxone
 Incorrect Answer ImageB.Cephalexin for 10 to 14 days
 Correct Answer Image C.Ciprofloxacin for 4 weeks
 Incorrect Answer ImageD.Foley catheter placement
 Incorrect Answer ImageE.Vigorous prostatic massage to express pus

A 55-year-old woman undergoes a laparoscopic gastric bypass (LGB) for obesity. At her 3
month follow-up, she reports flushing, headaches, and palpitations after eating, accompanied by
diarrhea and abdominal cramping. Her weight loss has been appropriate, although she says that
she has been eating more sweets lately. She has not had any fevers or vomiting. What is the most
likely diagnosis?
 Incorrect Answer ImageA.Afferent loop syndrome
 Incorrect Answer ImageB.Anastomotic stricture
 Incorrect Answer ImageC.C. Difficile colitis
 Correct Answer Image D.Dumping syndrome
 Incorrect Answer ImageE.Lactose intolerance

 57-year-old woman with a history of hypertension is brought to the emergency department


because of the acute onset of headache and loss of balance for 2 hours. She is conscious and
oriented to person, place, and time. She feels nauseated and cannot walk or stand without help.
Her blood pressure is 190/110 mm Hg. Neurologic examination shows paralysis of conjugate
lateral gaze to the right side. There is no decrease in muscle strength or sensory loss. Her gait is
markedly abnormal. A CT scan of the head shows an intracerebellar hematoma. Which of the
following is the most appropriate next step in management?
 Incorrect Answer ImageA.Anticoagulant treatment
 Correct Answer Image B.Immediate surgical evacuation
 Incorrect Answer ImageC.Lumbar puncture
 Incorrect Answer ImageD.MRI of the brain
 Incorrect Answer ImageE.Supportive medical treatment

A 29-year-old woman comes to the emergency department because of pain along the radial side
of the wrist and the first dorsal compartment. She states that the pain is often caused by wrist
flexion and simultaneous thumb extension that she assumes to carry the head of her baby. On
physical examination, the pain is reproduced by asking her to hold her thumb inside her closed
fist, and then forcing the wrist into ulnar deviation. Which of the following is the most likely
diagnosis? 
 Incorrect Answer ImageA.Acute and chronic bursitis
 Incorrect Answer ImageB.Carpal tunnel syndrome
 Incorrect Answer ImageC.Hairline unrecognized fracture of the carpal navicular
(scaphoid) bone
 Incorrect Answer ImageD.Palmar fascial contracture
 Correct Answer Image E.Tenosynovitis of the abductor or extensor tendons of the thumb

A 38-year-old man has an 8-year history of intermittent abdominal cramps and bloody diarrhea.
He has not seen a physician for this because he has maintained reasonably good control with
dietary measures and over-the-counter (OTC)medication. Because of a recent relapse of diarrhea,
he took over-the-counter loperamide 1 day ago. He comes to the emergency department because
he has had severe abdominal pain for the last 12 hours. His temperature is 40.0ºC (104.0ºF),
blood pressure is 135/79 mm Hg, pulse is 100/min, and respirations are 18/min. He looks ill and
toxic. His abdomen is tender, particularly in the epigastric area, and he has muscle guarding and
rebound tenderness. Laboratory studies show:
Hemoglobin 13g/dL
Hematocrit 43%
Leukocyte count  14,500/mm3
Segmented neutrophils 85%
Lymphocytes 10%
Basophils  0.5%
Monocytes  1%
Eosinophils  3%
X-ray shows a massively distended transverse colon. There is gas within the wall of the colon
but no free air under the diaphragm. Which of the following is the most likely diagnosis for his
current condition?
 Incorrect Answer ImageA.Colonic ileus
 Incorrect Answer ImageB.Fistulization between the colon and small bowel
 Incorrect Answer ImageC.Infectious colitis
 Incorrect Answer ImageD.Pneumatosis cystoides intestinalis
 Correct Answer Image E.Toxic megacolon

A 25-year-old man undergoes an elective inguinal hernia repair. The operation initially proceeds
according to plan. The first incisions are made without complication, and the surgical mesh is
placed appropriately. There is minimal blood loss and no technical complications.
Approximately 30 minutes after the start of the operation, however, the patient develops muscle
rigidity. Some cyanosis and mottling are noted on the man's extremities. The anesthesiologist
reports that the patient has a temperature of 39.4ºC (102.9ºF) and pulse is 128/min. Before
starting the operation, the patient's temperature had been 37.0ºC (98.6ºF) and pulse was 70/min.
Past medical history is significant for schizoaffective disorder treated with haloperidol and
fluoxetine for 4 years. Which of the following is the most appropriate next step in management?
 Incorrect Answer ImageA.Administer cyproheptadine
 Correct Answer Image B.Administer dantrolene
 Incorrect Answer ImageC.Administer dexamethasone
 Incorrect Answer ImageD.Discontinue haloperidol and start risperidone
 Incorrect Answer ImageE.Removal of mesh
An otherwise healthy 24-year-old man who is undergoing a supraclavicular node biopsy under
local anesthesia suddenly dies from cardiac arrest after a hissing sound is heard by staff carrying
out the procedure. At the time of the event, the target node was under traction, and the final cut
was being made blindly behind the node to free it completely. The patient was inhaling at that
moment. He had been placed in a semi-sitting position to improve surgical access to the node.
Which of the following most likely caused the patient's death?
 Incorrect Answer ImageA.Arterial injury with air embolization
 Correct Answer Image B.Major vein injury with air embolism
 Incorrect Answer ImageC.Sudden pneumothorax with lung collapse
 Incorrect Answer ImageD.Sympathetic discharge
 Incorrect Answer ImageE.Tracheal injury

A 45-year-old man is an unrestrained, front-seat passenger in a car when it crashes; he sustains


closed comminuted fractures of both femoral shafts. Shortly after admission, he develops a pulse
of 110/min, a blood pressure of 80/50 mm Hg, and becomes pale, cold, and clammy. He denies
any difficulty breathing. The rest of the physical examination and x-rays of the chest and pelvis
are unremarkable. Focused assessment with sonography for trauma (FAST) examination
performed in the emergency department is likewise negative. Which of the following most likely
explains the patient's low blood pressure?
 Correct Answer Image A.Blood loss at the fracture sites
 Incorrect Answer ImageB.Fat embolism
 Incorrect Answer ImageC.Intracranial bleeding
 Incorrect Answer ImageD.Neurogenic shock
 Incorrect Answer ImageE.Pericardial tamponade

A 52-year-old man comes to the physician for a routine health maintenance examination. Aside
from being overweight, he has had no major medical problems. Recently he has noticed a small
lump in his left groin that he thinks gets larger after being on his feet for long periods of time. He
states that the mass is uncomfortable from time to time but for the most part is only rarely
noticeable. His temperature is 37.0ºC (98.6ºF), blood pressure is 110/60 mm Hg, pulse is 94/min,
and respirations are 14/min. When the patient performs a Valsalva maneuver, a small mass can
be felt pressing against the lateral aspect of the examiner's finger when the finger is invaginated
in the external inguinal ring. Testicular examination shows no abnormalities, and physical
examination is otherwise unremarkable. Which of the following is the most likely etiology of the
patient's current condition?
 Incorrect Answer ImageA.Acute inflammation of an inguinal lymph node
 Incorrect Answer ImageB.Deep (internal) inguinal ring defect
 Incorrect Answer ImageC.Epididymitis
 Incorrect Answer ImageD.Femoral ring defect
 Correct Answer Image E.Posterior wall defect of the inguinal canal
 Incorrect Answer ImageF.Varicocele
A 35-year-old woman comes to the physician because of persistent pain and swelling at the base
of her right thumb for 2 weeks. She has had minimal relief with ibuprofen. Two weeks ago, she
underwent an open reduction and internal fixation with placement of a K-wire for a comminuted
fracture at the base of the first metacarpal joint. Her leukocyte count is 12,300/mm3. A
radiograph of the right hand shows intact alignment without hardware loosening or failure.
Which of the following is the most appropriate next step in management? 
 Incorrect Answer ImageA.Bone biopsy
 Incorrect Answer ImageB.Computed tomography
 Incorrect Answer ImageC.Magnetic resonance imaging
 Correct Answer Image D.Nuclear triphasic bone scan
 Incorrect Answer ImageE.Thumb amputation

A 61-year-old woman is brought to the emergency department complaining of lower extremity


weakness and worsening neck pain. She is imaged with an MRI scan of the neck, which shows
chronic cervical stenosis with acute inflammatory changes consistent with edema. She is
admitted to the hospital and started on methylprednisolone. The following day her symptoms are
markedly improved. Three days later, however, she complains of sharp epigastric pain and
vomits once. Routine laboratory studies are sent, and an AP upright chest x-ray is ordered that
shows a small rim of free air under the diaphragm. What is the next step in management? 
 Incorrect Answer ImageA.CT scan of the abdomen
 Correct Answer Image B.Exploratory laparotomy
 Incorrect Answer ImageC.Insertion of a nasogastric tube
 Incorrect Answer ImageD.Triple therapy with tetracycline, metronidazole, and a proton-
pump inhibitor
 Incorrect Answer ImageE.Upper GI endoscopy

A 74-year-old woman undergoes a left total knee replacement for severe osteoarthritis. She has a
past medical history significant for type 1 diabetes mellitus and glaucoma. She continues with
daily NPH insulin and has good control of her blood glucose. She also takes oxycodone, which
was administered to control her postoperative pain. She is planning to start a physical therapy
rehabilitation program at the local hospital. Which of the following medications is most
indicated? 
 Incorrect Answer ImageA.ACE inhibitor
 Incorrect Answer ImageB.Aspirin
 Correct Answer Image C.Low molecular-weight heparin
 Incorrect Answer ImageD.Nonsteroidal anti-inflammatory agent
 Incorrect Answer ImageE.Tissue-type plasminogen activator (t-PA)

A 66-year-old man comes to the urgent care clinic with progressive jaundice, which he first
noticed 6 weeks ago. He has no significant past medical history and takes no medications. He has
smoked 1 pack of cigarettes a day for the past 30 years and drinks a glass of wine each evening.
On examination, he is afebrile and appears deeply jaundiced. His abdomen is nontender. He has
a total bilirubin level of 22 mg/dL, with direct (conjugated) bilirubin level 16 mg/dL.
Transaminase levels are minimally elevated, whereas alkaline phosphatase levels are
approximately 6 times the upper limit of normal. Sonogram shows dilated intrahepatic ducts,
dilated extrahepatic ducts, and a very distended, thin-walled gallbladder without stones. Which
of the following is the most likely diagnosis? 
 Correct Answer Image A.Carcinoma of the pancreas
 Incorrect Answer ImageB.Chronic pancreatitis
 Incorrect Answer ImageC.Common bile duct stone
 Incorrect Answer ImageD.Sclerosing cholangitis
 Incorrect Answer ImageE.Structure of bile duct

An 8-year-old boy is brought in via ambulance after losing consciousness after falling off his
skateboard. Prior to the fall, he was last seen by his mother when he was skating along the
driveway. It is unclear how long he was unconscious. Paramedics arrived at the scene and found
him with a normal pulse and blood pressure. Pulse oximetry showed 95% saturation without
supplemental oxygen. His fingerstick glucose in the field was 101 mg/dL. On initial
examination, he is not arousable to any stimuli. His pupils are reactive, neurological examination
is nonfocal, and the Babinski reflex is negative. Cardiovascular, pulmonary, and abdominal
exams are normal. There are several abrasions on his forearms and legs, and soft-tissue swelling
at his occiput. Laboratory studies show:
White blood cells 5,600/mm3
Hematocrit 42%
Platelets 224,000/mm3
Sodium 139 mEq/L
Potassium 4.2 mEq/L
Chloride 105 mEq/L
Bicarbonate 24 mEq/L
Urea nitrogen 14 mg/dL
Creatinine 1.0 mg/dL
Glucose 115 mg/dL
Calcium 8.8 mg/dL
Phosphate 2.9 mg/dL
Magnesium 2.0 mg/dL
Prothrombin time 12 seconds
Partial thromboplastin time 25 seconds
Computed tomography of the head shows a hypodense lesion within the pons, with a hyperdense
focus within the basilar artery consistent with thrombus. Which of the following is the most
appropriate next step in management?
 Correct Answer Image A.CT angiogram of the head/neck
 Incorrect Answer ImageB.Hypercoagulable workup
 Incorrect Answer ImageC.Lumbar puncture
 Incorrect Answer ImageD.MRI of the brain
 Incorrect Answer ImageE.Transthoracic echocardiogram
A 69-year-old woman comes to the emergency department because of a 6-hour history of
abdominal pain and fever. She has been having intermittent right upper quadrant pain for the past
few months, but today the pain has been constant and worse than usual. Past medical history
includes severe chronic obstructive pulmonary disease for which she is on home oxygen,
pulmonary hypertension, and severe obesity. She has not had any prior surgery. Her temperature
is 38.5ºC (101.4ºF), blood pressure is 145/80 mm Hg, pulse is 115/min, and oxygen saturation is
91% on 4 L of oxygen by way of nasal cannula. Physical examination shows diffuse wheezes
bilaterally and tenderness in the right upper quadrant with palpation. Ultrasound shows multiple
large and small gallstones and a markedly thickened gallbladder wall with pericholecystic fluid.
The patient is admitted and placed on IV fluids and antibiotics, and subsequently improves. She
is felt to be a poor operative candidate because of her comorbidities and is sent home. A month
later, she comes to the emergency department with bilious vomiting and constipation. Upright
abdominal radiograph shows multiple dilated loops of small bowel and air in the biliary tree. The
colon is not dilated. No air is visible within the gallbladder wall on ultrasound. Which of the
following is the most likely diagnosis of the patient’s current condition?
 Incorrect Answer ImageA.Adhesive small bowel obstruction (SBO)
 Incorrect Answer ImageB.Emphysematous cholecystitis with ileus
 Incorrect Answer ImageC.Gallbladder carcinoma
 Correct Answer Image D.Gallstone ileus
 Incorrect Answer ImageE.Incarcerated femoral hernia

Eight hours after undergoing a transsphenoidal hypophysectomy for a prolactinoma, a 24-year-


old woman becomes lethargic, confused, and eventually comatose. The anesthesia and the
operation had proceeded without any apparent problems. Physical examination is unremarkable,
and the pupils are of equal size and reactive to light. A review of her records shows that she has
not been administered narcotics, and the only medications received are IV fluids, 5% dextrose in
half-normal saline at 100 mL per hour. Her pulse is 94/min, blood pressure is 98/72 mm Hg, and
urinary output has averaged 600 mL per hour since the operation. Which of the following studies
is most likely to establish the diagnosis?
 Incorrect Answer ImageA.Blood sugar
 Incorrect Answer ImageB.Blood urea nitrogen (BUN)
 Incorrect Answer ImageC.CT scan of the head
 Incorrect Answer ImageD.Lumbar puncture
 Correct Answer Image E.Serum sodium concentration

 41-year-old gas station attendant is shot once with a 0.38 caliber revolver. The entry wound is in
the left midclavicular line, 2 inches below the nipple. There is no exit wound. He is
hemodynamically stable. Physical examination reveals diffuse abdominal discomfort. A chest x-
ray shows a small pneumothorax on the left and demonstrates the bullet to be lodged in the left
paraspinal muscles. In addition to the appropriate treatment for the pneumothorax, which of the
following is the most appropriate intervention?
 Incorrect Answer ImageA.Barium swallow
 Incorrect Answer ImageB.Bronchoscopy
 Incorrect Answer ImageC.CT scan of the abdomen
 Incorrect Answer ImageD.Extraction of the bullet via left thoracotomy
 Incorrect Answer ImageE.Extraction of the bullet via local back exploration
 Correct Answer Image F.Exploratory laparotomy

A 25-year-old man is shot with a .22-caliber revolver. The entrance wound is in the anteromedial
aspect of the upper thigh, while the exit wound is approximately 6 cm lower, in the posterolateral
aspect of the thigh. He has a large, expanding hematoma in the upper inner thigh. There are no
palpable pulses in the foot. The bone is intact by physical examination and x-ray film. Which of
the following is the most appropriate next step in management?
 Incorrect Answer ImageA.Arteriogram
 Incorrect Answer ImageB.CT angiography
 Incorrect Answer ImageC.Doppler studies
 Incorrect Answer ImageD.Embolectomy
 Correct Answer Image E.Surgical exploration

A 49-year-old obese man comes to the physician for a follow-up visit. He has a longstanding
history of gastroesophageal reflux (GERD) symptoms that initially resolved with over-the-
counter antacids. After persistent symptoms, his primary physician places him on a proton-pump
inhibitor (PPI). He receives some relief with this medical therapy, but his symptoms progress
over the subsequent 8 weeks. He then undergoes an endoscopy that shows severe peptic
esophagitis with no dysplastic changes. Twenty-four-hour pH monitoring confirms the diagnosis
of esophageal reflux. Manometry, gastric emptying study, and a barium swallow are all normal
except for a lax lower esophageal sphincter. No reflux is seen on the barium swallow. Which of
the following should be offered to this patient?
 Incorrect Answer ImageA.Continue PPI at a higher dose and follow up in 6 weeks
 Correct Answer Image B.Nissen fundoplication
 Incorrect Answer ImageC.Pyloromyotomy
 Incorrect Answer ImageD.Repeat the barium swallow
 Incorrect Answer ImageE.Truncal vagotomy

An unrestrained front-seat passenger is ejected through the windshield as the car crashes into a
telephone pole at 30 miles per hour. He arrives in the emergency department strapped to a
headboard, with sandbags on both sides of his neck. He has multiple facial lacerations, but is
otherwise awake, alert, and hemodynamically stable. He reports severe neck pain. Physical
examination shows severe tenderness to palpation over a specific spot at the midline, in the upper
part of the back of the neck. Neurologic examination shows no abnormalities. Which of the
following is the most appropriate next step in management?
 Incorrect Answer ImageA.Cervical spine traction
 Incorrect Answer ImageB.Clinical observation
 Correct Answer Image C.CT scan of the cervical spine
 Incorrect Answer ImageD.Emergent neurosurgery consultation
 Incorrect Answer ImageE.MRI of the cervical spine
A 53-year-old man with a history of hypertension is brought to the emergency department by his
coworkers because he vomited bright red blood after lunch. He is in no apparent distress and
denies nausea. His vital signs are normal and stable. Abdominal examination shows hypoactive
bowel sounds, and the abdomen is soft, non-distended, and there is minimal epigastric tenderness
to deep palpation. He admits a history of moderate alcohol consumption. A careful examination
of his nose and mouth reveal no obvious bleeding sources. A gastric lavage via nasogastric (NG)
tube quickly evacuates bright red blood and begins to return clear fluid. Two large-bore IV lines
are placed and laboratory studies are sent for analysis. What is the most appropriate next step in
management?
 Incorrect Answer ImageA.Arteriogram
 Incorrect Answer ImageB.Insertion of a Blakemore tube
 Incorrect Answer ImageC.Nuclear medicine-tagged red blood cell scan
 Incorrect Answer ImageD.Transfusion of packed red blood cells
 Correct Answer ImageE.Upper GI endoscopy

A 68-year-old man is brought to the emergency department with excruciating back pain that
began suddenly 45 minutes ago. The pain is constant and is not exacerbated by coughing. There
was no preceding back discomfort. The patient is diaphoretic and has a systolic blood pressure of
90 mm Hg. There is an 8-cm pulsatile mass deep in his epigastrium, above the umbilicus. A
chest x-ray film is unremarkable. Two years ago, he was diagnosed with prostatic cancer and
was treated with orchiectomy and radiation. At that time he had high blood pressure, for which
he declined treatment. Which of the following is the most likely diagnosis? 
 Incorrect Answer ImageA.Dissecting thoracic aortic aneurysm
 Incorrect Answer ImageB.Fracture of lumbar pedicles with cord compression
 Incorrect Answer ImageC.Herniated disc
 Incorrect Answer ImageD.Metastatic prostate cancer to the lumbar spine
 Correct Answer ImageE.Rupturing abdominal aortic aneurysm

A 28-year-old woman in month 7 of pregnancy suddenly develops upper abdominal pain and
collapses. She wakes up within a few minutes, but over the ensuing hour, she becomes extremely
pale, diaphoretic, tachycardic, and progressively weaker. Her family calls an ambulance. Upon
arrival, emergency medical technicians find that she has a blood pressure of 75/55 mm Hg and is
developing abdominal distention. They transport her rapidly to a nearby hospital where she has
an undetectable blood pressure and is barely responsive. A focused assessment with sonography
(FAST) scan identifies free intra-abdominal fluid. She is brought to the operating room where
she is found to have a hemoperitoneum, and she dies during surgery. The history provided by her
relatives indicates that this was her second pregnancy, that she had never taken oral
contraceptives, had prenatal care for the present pregnancy, and had last been seen by her
obstetrician 2 weeks before this event. There was no reported history of trauma, and she had not
experienced any labor pains. Which of the following is the most likely cause of this patient's
intra-abdominal bleeding?
 Incorrect Answer ImageA.Ectopic pregnancy
 Incorrect Answer ImageB.Ruptured abdominal aortic aneurysm
 Incorrect Answer ImageC.Ruptured hepatic adenoma
 Incorrect Answer ImageD.Ruptured uterus
 Correct Answer ImageE.Ruptured splenic artery aneurysm

A 38-year-old woman comes to the emergency department complaining of severe epigastric pain
that radiates to her back. The pain woke her up this morning and she vomited 3 times. She has
not been hungry since the onset of the pain. The pain is not associated with movement and has
not been relieved by ibuprofen. She denies alcohol use. She notes a history of transient, self-
limited right upper quadrant pain after eating. She denies hematemesis, melena, hematochezia,
diarrhea, or constipation. She takes no medications. Her past medical history is notable for
obstructive sleep apnea secondary to obesity. She has not had any surgeries. She denies smoking
or drug use. On physical examination her temperature is 37.5ºC (99.5ºF), blood pressure 120/59
mm Hg, pulse 105/min, and oxygen saturation 98% on 2 L of oxygen by nasal cannula. She is
anicteric, has dry mucous membranes and marked epigastric tenderness without rebound or
guarding. Laboratory values are:
White blood cells  14,000/mm3
Hematocrit 39%
Platelets 555,000/mm3
Sodium 137 mEq/L
Potassium 3.2 mEq/L
Chloride 105 mEq/L
Bicarbonate 21 mEq/L
Urea nitrogen 19 mg/dL
Creatinine 1.1 mg/dL
Glucose 120 mg/dL
Calcium 8.9 mg/dL
Triglycerides 200 mg/dL 
AST 89 IU/L 
ALT 77 IU/L 
Alkaline phosphatase 200 IU/L 
Amylase 2,200 IU/L 
Lipase 155 IU/L
Total Bilirubin 0.9 mg/dL
Abdominal ultrasound demonstrates a mildly dilated common bile duct and trace fluid around
the gallbladder. No gallstones are seen. The pancreas is not visualized due to overlying gas.
Which of the following is the next step in management?
 Incorrect Answer ImageA.Endoscopic retrograde cholangiopancreatography (ERCP)
 Correct Answer ImageB.Initiation of intravenous fluids, pain medication, and admission
to the hospital
 Incorrect Answer ImageC.Laparoscopic cholecystectomy
 Incorrect Answer ImageD.Magnetic Resonance Cholangiopancreatography (MRCP)
 Incorrect Answer ImageE.Placement of a nasogastric (NG) tube
A 59-year-old man sustains blunt trauma in an automobile collision, resulting in multiple intra-
abdominal injuries. Surgery for the repair of these injuries takes several hours and requires
multiple blood transfusions and infusions of Ringer's lactate. Before all the operative steps are
completed, the patient develops a significant inability for coagulation, a core body temperature
<34ºC (93.2ºF), and refractory acidosis. The anesthesiologists administer fresh frozen plasma
and platelet packs. Which of the following is the most appropriate next step in management?
 Incorrect Answer ImageA.Abort the operation and leave the abdomen open, covering the
bowel with mesh
 Incorrect Answer ImageB.Attempt hemostasis by liberal use of electrocoagulation
 Incorrect Answer ImageC.Complete the operation as soon as possible and do a formal
abdominal closure
 Correct Answer ImageD.Pack the bleeding surfaces and close the abdomen temporarily
with towel clips
 Incorrect Answer ImageE.Wash the abdomen with warm saline and continue to operate

A 69-year-old woman comes to the physician for a periodic health maintenance examination.
She has no complaints and has no significant past medical history. She takes 1,500 mg of
calcium daily, a multivitamin, and daily aspirin. She exercises regularly, eats a fairly healthy
diet, and does not smoke cigarettes. She drinks a glass of wine with dinner each evening and
smokes marijuana occasionally with a friend who has glaucoma. Breast, digital rectal, and pelvic
examinations are unremarkable. A mammogram 6 months ago was normal. A Pap smear done 4
years ago was normal. Which screening examination would you recommend for her at this time?
 Incorrect Answer ImageA.Blood work for CA-125
 Incorrect Answer ImageB.Breast MRI
 Incorrect Answer ImageC.Chest CT (low dose)
 Correct Answer ImageD.Colonoscopy
 Incorrect Answer ImageE.Pap smear

A 17-year-old boy is brought to the emergency department from a high-school football game on
a backboard with a cervical collar in place. He was tackled while running and was thrown
backward to the ground. He was unconscious for a few minutes, and then woke up
spontaneously. He does not recall the event, but he complains of right shoulder pain. On
examination, he is alert and oriented to person, place, and time, and remembers the ambulance
ride to the hospital. He has no focal neurologic deficits. His right clavicle is exquisitely tender
with a palpable defect laterally. He has no motor or sensory deficits in his right upper extremity
and his radial pulse is palpable. Chest x-ray shows a fracture of the distal third of the clavicle but
no rib fracture or pneumothorax. What is the next step in management? 
 Incorrect Answer ImageA.CT scan of the head with IV contrast
 Correct Answer ImageB.CT scan of the head without IV contrast
 Incorrect Answer ImageC.Operative fixation of the clavicle
 Incorrect Answer ImageD.Repeat chest x-ray in 4 hours to rule out pneumothorax
 Incorrect Answer ImageE.Shoulder sling
A 39-year-old man is brought to the operating room because of acute appendicitis. His appendix
is inflamed and scarred in the retrocecal position. The surgeons must release the cecum and
ascending colon from the lateral abdominal wall to mobilize the appendix and deliver the
terminal ileum into the wound to adequately ligate its base. His abdominal cavity is copiously
irrigated and the wound is closed in layers. The patient is kept nothing-by-mouth (NPO)
postoperatively, and one dose of antibiotics is administered. He is given clear liquids to drink on
postoperative day 1, which he tolerates. On morning rounds of day 2, he has not passed any
flatus, and he feels distended with mild diffuse abdominal pain. On examination, he has minimal
bowel sounds with a soft, distended, tympanitic abdomen. An abdominal x-ray shows
moderately distended loops of small bowel and colon, as well as a small amount of free
intraperitoneal air. Which of the following is the most appropriate next step in management?
 Incorrect Answer ImageA.Begin broad-spectrum antibiotics
 Incorrect Answer ImageB.Colonoscopy with decompression
 Correct Answer ImageC.Nothing-by-mouth (NPO)
 Incorrect Answer ImageD.Place a nasogastric (NG) tube
 Incorrect Answer ImageE.Urgent return to the operating room

A 61-year-old woman comes to the emergency department because of 3 days of nausea, loss of
appetite, colicky abdominal pain, and protracted vomiting. She denies similar symptoms in the
past and recent weight loss. She has not had any bowel movements or passed gas over the last 2
days. Her past medical history is significant for hypertension, osteoporosis, and cholelithiasis.
She had an abdominal hysterectomy for a fibroid uterus 10 years ago, and a right total knee
arthroplasty 2 years ago. She takes metoprolol, calcium supplementation, and hormone
replacement therapy, and she has no known drug allergies. On physical examination, she is
afebrile, her pulse is 104/min, and blood pressure is 142/86 mm Hg. Cardiac examination is
normal and lungs are clear to auscultation. Click "Play Media" to see her abdominal examination
findings. She has an old, well-healed, midline infraumbilical incision. Laboratory studies show:
Hematocrit 33%
Hemoglobin 11 g/dL
WBC 12,400/mm3
Neutrophils 65%
Lymphocytes 15%
AST 24 U/L
ALT 22 U/L
Bilirubin 1.0 mg/dL
BUN 36 mg/dL
Creatinine 1.5 mg/dL
What is the most appropriate next step in the management of this patient? 
 Correct Answer ImageA.Abdominal x-ray
 Incorrect Answer ImageB.Bowel preparation and colonoscopy
 Incorrect Answer ImageC.CT scan of the abdomen
 Incorrect Answer ImageD.Hepatobiliary iminodiacetic acid (HIDA) scan
 Incorrect Answer ImageE.Right upper quadrant ultrasound
A 72-year-old woman is admitted to the hospital with worsening constipation, weight loss, and
fatigue. She has had difficulty moving her bowels for approximately 2 months and reports that
her stools have become flat and are sometimes coated with bright red blood. In addition, she has
lost 6.8 kg (15 lb) during this period, though her appetite and oral intake have not decreased. Her
past medical history is significant for diabetes mellitus controlled with oral hypoglycemics. She
has not had a prior colonoscopy, but she has not had any prior history of gastrointestinal issues.
On physical examination, she is awake and alert, hemodynamically stable, afebrile, and in mild
distress. The abdomen is mildly distended with hypoactive bowel sounds, and she experiences
mild discomfort on palpation in the left lower quadrant. She has no masses, rebound tenderness,
or guarding, however. Rectal examination is notable for heme-positive stool but no palpable
masses. Laboratory studies are significant for a hematocrit of 21%. What is the most likely
diagnosis?
 Correct Answer ImageA.Cancer of the left side of the colon
 Incorrect Answer ImageB.Cancer of the right side of the colon
 Incorrect Answer ImageC.Diverticulitis
 Incorrect Answer ImageD.Internal hemorrhoids
 Incorrect Answer ImageE.Squamous cell carcinoma of the anus

A 45-year-old man is brought emergency department with a Glasgow Coma Scale (GCS) score
of 5, strapped to a headboard with sandbags on both sides of his head after being hit by a car
while crossing the street. The paramedics report that he was unconscious at the site of the
accident. Primary survey shows stable vital signs, and his pupils are of equal size and reactive to
light. He has bilateral periorbital ecchymosis and there is a sizable hematoma behind his right ear
with clear fluid dripping from the ear canal. Which of the following is the next best step in
management?
 Incorrect Answer ImageA.Administer antibiotics to prevent meningitis
 Incorrect Answer ImageB.CT scan of the head and neck STAT since his vital signs are
stable
 Correct Answer ImageC.Intubation using fiberoptic bronchoscope and neck stabilization
 Incorrect Answer ImageD.Placement of nasogastric tube to prevent aspiration prior to
intubation
 Incorrect Answer ImageE.Tracheostomy for airway management

A 73-year-old man with coronary artery disease, hypertension, and hypercholesterolemia is


brought to the emergency department because of bright red blood per rectum. He is tachycardic
at 110/min and hypotensive with a blood pressure of 100/40 mm Hg. He is started on IV fluids,
and blood is sent for typing, cross-matching, and a complete blood count (CBC). The
gastroenterology team is contacted ,and colonoscopy is planned for the morning. Blood pressure
improves to 110/60 mm Hg after 2 L of crystalloid fluids, and hemoglobin and hematocrit are
7.5g/dL and 23%, respectively. Blood transfusion is initiated, during which he complains of back
pain and is noted to be febrile to 39°C (102.3°F), tachycardic at 132/min, and diaphoretic. An
ECG is normal, as are troponin levels. Repeat CBC shows hemoglobin and hematocrit to be 6
g/dL and 19%, respectively. Urine from the Foley catheter looks reddish. What is the next best
step in management? 
 Incorrect Answer ImageA.Acetaminophen administration
 Correct Answer ImageB.Discontinue the blood transfusion
 Incorrect Answer ImageC.Intravenous antibiotic administration
 Incorrect Answer ImageD.Intravenous corticosteroid administration
 Incorrect Answer ImageE.Intravenous diphenhydramine administration

A 61-year-old man comes to his primary care physician because he noticed a neck mass
approximately 3 months ago, which has been growing since then. His past medical history is
significant for hypertension, well-controlled with a calcium-channel blocker, and osteoarthritis,
managed with nonsteroidal anti-inflammatory drugs. He has a 40 pack-year smoking history and
quit drinking recently, after many years of frequent alcohol abuse. He denies IV drug use. He is
married and is a retired police officer. He denies fevers, chills, night sweats, shortness of breath,
palpitations, or changes in his appetite or weight. He is afebrile with normal vital signs. Head
and neck examination is significant for a palpable mass approximately 3 cm in diameter and just
medial to the sternocleidomastoid muscle at the level of the hyoid. The mass is nontender, firm,
and nonfluctuant. No other masses are palpated in the neck or supraclavicular area. Visual
examination of the oropharynx shows no visible lesions. He undergoes a fine-needle aspiration
of the mass. Which of the following is the most likely histologic finding?
 Incorrect Answer ImageA.Lymphoma
 Incorrect Answer ImageB.Normal thyroid tissue
 Incorrect Answer ImageC.Papillary thyroid carcinoma
 Incorrect Answer ImageD.Small-cell carcinoma of the lung
 Correct Answer ImageE.Squamous cell carcinoma

A 36-year-old woman undergoes a minor orthopedic procedure to correct a deformity of her


forefoot. Upon waking from the anesthetic, she vomits repeatedly. Although she has orders to
resume oral intake whenever she can, it is also ordered that her IV line be kept open with a drip
of 5% dextrose in water (D5W). Over the ensuing 48 hours, she continues to be nauseous, and
her IV intake of D5W averages 3 to 4 liters per day. On day 3 she becomes lethargic, has a
generalized tonic-clonic seizure, and goes into a coma. Laboratory studies show a serum sodium
concentration of 118 mEq/L, and a CT scan of the head reveals severe generalized cerebral
edema. Which of the following is the most likely mechanism by which the cerebral edema
occurred?
 Incorrect Answer ImageA.Active transport of water into the cells
 Incorrect Answer ImageB.Hydrostatic pressure gradients at the capillary membrane
 Incorrect Answer ImageC.Passive movement of sodium out of cells
 Correct Answer ImageD.Passive movement of water into the cells
 Incorrect Answer ImageE.Vascular oncotic pressure gradients at the capillary membrane

An 88-year-old Caucasian man is brought to the urology clinic by his children for a routine
checkup. He has an extensive past medical history including hypertension, coronary artery
disease with 2 previous myocardial infarctions, diabetes mellitus, peripheral vascular disease,
hypothyroidism, and colon cancer. He has had frequent hospitalizations. He had a coronary
artery bypass 10 years ago and a cardiac angioplasty 2 years ago. Repeat coronary artery bypass
graft at that time was considered, but it was determined that he was a poor surgical candidate
because of his cardiac ejection fraction of 15%. He also had a right below-knee amputation and a
left lower extremity artery bypass 8 years ago. On examination today, his temperature is 36.7°C
(98.0°F), blood pressure is 147/88 mmHg, pulse is 76/min, and respirations are 18/min. He is
frail. On rectal examination, his sphincter tone is fair and the prostate is moderately enlarged,
firm, and has a hard, nontender 1 cm nodule over the apex of the right lobe. Stool is guaiac-
negative. When questioned at the end of the examination, the patient denies any bone pain or
difficulty with urination. Which of the following is the most appropriate next step in his
management?
 Incorrect Answer ImageA.Determination of serum prostate specific antigen (PSA)
 Incorrect Answer ImageB.He should be referred for radiation therapy
 Incorrect Answer ImageC.Hormone therapy to block androgen activity
 Incorrect Answer ImageD.Prostate biopsy
 Correct Answer ImageE.Radiologic testing for staging purposes

A 22-year-old man is brought to the emergency department via ambulance. The man was in a
motorcycle accident and has numerous abrasions and lacerations and a femoral fracture. After
the patient is stabilized and his fracture is placed in traction, the rest of his body is examined for
injuries. He has a laceration across his nose and nasolabial folds, a laceration on the lateral aspect
of his penis, a large, oozing wound on his right index finger, and an actively bleeding scalp
laceration. Additionally, his left external auricle has a large laceration. While waiting for the
attending orthopedic surgeon to arrive, a medical student on trauma service is encouraged to
clean, anesthetize, and suture the man's numerous lacerations. The student plans to use a 2%
lidocaine solution with epinephrine at a concentration of 1:100,000. This method of local
anesthesia and hemostasis is most appropriate for the patient's wounds in which of the following
areas? 
 Incorrect Answer ImageA.Ear
 Incorrect Answer ImageB.Finger
 Incorrect Answer ImageC.Nose
 Incorrect Answer ImageD.Penis
 Correct Answer ImageE.Scalp

A 37-year-old, 70 kg (154 lb) man is brought to the emergency department with 60% body
surface area (BSA) third-degree burns. He is started on a Ringer's lactate solution of 500
mL/hour. Over the next 3 hours, his urinary output is recorded as 15 mL, 22 mL, and 10 mL.
Physical examination shows extensive third-degree burns over the chest, back, and thighs. His
temperature is 37.7ºC (100.0ºF), blood pressure 100/70 mm Hg, pulse 105/min, oxygen
saturation 98%, and central venous pressure 2 cm H2O. Initial urinalysis studies show:
Appearance  Dark yellow
Specific gravity 1.045 
pH 6.5
WBC  2-3/hpf
RBC Negative
Nitrites Negative
Casts Negative
Urine Na+ 10 mEq/L
Urine dipstick for glucose, bilirubin, and hemoglobin is negative. A serum creatinine level is 1.3
mg/dL and BUN is 30 mg/dL. Which of the following is the most appropriate next step in
management?
 Incorrect Answer ImageA.Administer furosemide 40 mg IV
 Incorrect Answer ImageB.Administer normal saline plus NaHCO3
 Correct Answer ImageC.Double the rate of fluid administration
 Incorrect Answer ImageD.Observe, and repeat laboratory studies in 3 hours
 Incorrect Answer ImageE.Order nephrology consult

A 9-year-old girl is brought to the physician by her mother for evaluation of a mass in the child's
neck. The mother reports that a small swelling has been present since the child was an infant, but
that it has increased in size over the past few months. She also mentions that the mass increases
in size during upper respiratory tract infections. She denies any history of drainage. The child
had an unremarkable birth history and is otherwise presently healthy. The child is currently
afebrile with stable vital signs and is in no acute distress. There is no evidence of stridor.
Examination of the neck shows a 2- to 3-cm midline neck mass at approximately the level of the
thyroid cartilage. The mass is mobile, nontender, well-circumscribed, and somewhat fluctuant.
The mass elevates when the child is asked to swallow. The remainder of the head and neck
examination is within normal limits. Which of the following is the most likely diagnosis?
 Incorrect Answer ImageA.Cystic hygroma
 Incorrect Answer ImageB.Dermoid cyst
 Incorrect Answer ImageC.Lymphadenitis
 Incorrect Answer ImageD.Thymic cyst
 Correct Answer ImageE.Thyroglossal duct cyst

A 46-year-old man is brought to the emergency department after a motor vehicle accident that
occurred 30 minutes ago. His medical history is unremarkable. He appears ill. His temperature is
36.9ºC (98.4ºF), pulse is 120/min, and blood pressure is 70/40 mm Hg. Oxygen saturation is
90% on 2 L of oxygen by face mask. Cardiac examination shows distant heart sounds, an S1 and
S2, and sinus tachycardia. Lung examination shows decreased breath sounds over the left
hemithorax. A radiograph of the chest is shown. Which of the following is the most appropriate
next step in management?
 Incorrect Answer ImageA.CT angiogram of the chest
 Incorrect Answer ImageB.Emergency department thoracotomy
 Incorrect Answer ImageC.Exploratory laparotomy
 Correct Answer ImageD.Left tube thoracostomy
 Incorrect Answer ImageE.Observation and repeat the x-ray in 1 hour
A 58-year-old man undergoes liver transplantation. The procedure goes well without any noted
complications. Ten days later, however, the patient's levels of gamma-glutamyl transferase
(GGT), alkaline phosphatase, and bilirubin begin to increase. Which of the following is the most
appropriate next step to establish the diagnosis?
 Incorrect Answer ImageA.Liver biopsy and determination of portal pressures
 Incorrect Answer ImageB.Liver biopsy and more detailed liver function tests
 Incorrect Answer ImageC.Liver biopsy and trial of steroid boluses
 Incorrect Answer ImageD.Measurement of preformed antibody levels
 Correct Answer ImageE.Ultrasound of the biliary tract and Doppler studies of the
anastomosed vessels

A 26-year-old man with no significant past medical history is brought to the emergency
department complaining of 2 days' history of abdominal pain. The pain began in the epigastric
area and migrated to the right lower quadrant. Currently, his temperature is 39.1°C (102.4°F),
pulse is 108/min, and blood pressure is 110/68 mm Hg. Physical examination is significant for
abdominal tenderness, which is the worst in the right lower quadrant. His white blood cell count
comes back elevated at 16,000/mm3 with 92% neutrophils. He is taken to the operating room
with a diagnosis of acute appendicitis, and he receives a single preoperative dose of antibiotics
prior to undergoing a laparoscopic appendectomy. The findings at operation include an
erythematous, injected appendix that is intact and resected in the traditional manner. On
postoperative day 1, he has a temperature of 37.8°C (100°F). Which of the following durations
of antibiotic pharmacotherapy is most appropriate for this patient?
 Incorrect Answer ImageA.Forty-eight hours
 Correct Answer ImageB.None are indicated at this time
 Incorrect Answer ImageC.Ten days
 Incorrect Answer ImageD.Twenty-four hours
 Incorrect Answer ImageE.Until he is afebrile for 24 hour

A 26-year-old man with Crohn disease is brought to the emergency department by his girlfriend
after 2 days of significant abdominal pain. The pain is colicky in nature, worst in his left lower
quadrant and left flank, and radiates medially to his scrotum and inner thigh. It started suddenly
and has not remitted over 48 hours. It is not associated with nausea or vomiting, but the patient
has had a single episode of hematuria. He has a low-grade fever of 38.1°C (100.6°F) and normal
vital signs. He is in obvious distress. An abdominal examination is surprisingly benign for
someone who is obviously in great pain. He has normal bowel sounds, very mild left-sided
tenderness to palpation, and no distention. He has had Crohn disease for several years. He is
extremely anxious about his condition and wants to avoid surgery at all costs. Which of the
following is the most appropriate next step in management?
 Incorrect Answer ImageA.Barium enema
 Incorrect Answer ImageB.Bowel prep and colonoscopy
 Incorrect Answer ImageC.CT scan with IV contrast
 Correct Answer ImageD.CT scan without IV contrast
 Incorrect Answer ImageE.MRI scan of the abdomen
Psiquiatria
A 24-year-old male college freshman comes to the physician for a routine checkup after his first
week of finals. During the interview, the patient discloses that this past week has been “crazy.”
He begins by describing how “dangerously fast” he has been driving. When asked to be more
specific, the patient apologizes for not hearing you and comments on the paintings in the
examination room. He admits that his attention has been lacking because he is “thinking so
much.” Still, he says that he has made frequent trips to the library and has been able to “stay up
all night to study without a problem,” which he feels compensates for his inattention during the
last week of lectures. He states confidently that he probably “aced” each examination. He
digresses to describe the intricate architecture of the library, confused about why he only recently
noticed it. Physical examination is unremarkable except for tachycardia and dilated pupils.
Which of the following is the most likely diagnosis?
 Incorrect Answer ImageA.Adult-onset attention deficit/hyperactivity disorder (ADHD)
 Incorrect Answer ImageB.Bipolar I disorder
 Incorrect Answer ImageC.Bipolar II disorder
 Correct Answer ImageD.Stimulant intoxication
 Incorrect Answer ImageE.Cyclothymic disorder
 Incorrect Answer ImageF.Delusional disorder, grandiose type

A 24-year-old male college freshman comes to the physician for a routine checkup after his first
week of finals. During the interview, the patient discloses that this past week has been “crazy.”
He begins by describing how “dangerously fast” he has been driving. When asked to be more
specific, the patient apologizes for not hearing you and comments on the paintings in the
examination room. He admits that his attention has been lacking because he is “thinking so
much.” Still, he says that he has made frequent trips to the library and has been able to “stay up
all night to study without a problem,” which he feels compensates for his inattention during the
last week of lectures. He states confidently that he probably “aced” each examination. He
digresses to describe the intricate architecture of the library, confused about why he only recently
noticed it. Physical examination is unremarkable except for tachycardia and dilated pupils.
Which of the following is the most likely diagnosis?
 Incorrect Answer ImageA.Adult-onset attention deficit/hyperactivity disorder (ADHD)
 Incorrect Answer ImageB.Bipolar I disorder
 Incorrect Answer ImageC.Bipolar II disorder
 Correct Answer ImageD.Stimulant intoxication
 Incorrect Answer ImageE.Cyclothymic disorder
 Incorrect Answer ImageF.Delusional disorder, grandiose type

A 20-year-old man comes to the psychiatrist because of referral by his parole officer. The patient
was formerly arrested and incarcerated for drug possession and armed robbery and was released
on parole last week. The patient laughs about the harm he caused to his victims, shares an
inconsistent employment history, and reveals numerous apartment evictions and an ability to
“always fool the landlords.” The patient also tells the physician that he does not “care who [the
physician] is” and will “mess him up” if he “cuts into his brain.” In an attempt to threaten the
physician, the patient proceeds to describe physical fights in which he was involved. The
psychiatrist asks how long the patient remembers having legal problems, to which the patient
responds, “My whole life!” There is no evidence for mania during the course of the interview.
Which of the following factors in the patient’s history best support the patient’s diagnosis?
 Incorrect Answer ImageA.Confirmation of at least 5 police arrests after the age of 18
years
 Correct Answer ImageB.Evidence of conduct disorder
 Incorrect Answer ImageC.Evidence of inappropriate sexual behaviors
 Incorrect Answer ImageD.Evidence of oppositional defiant disorder
 Incorrect Answer ImageE.Pattern of unstable interpersonal relationships

An 18-year-old Caucasian woman is brought to the emergency department by her parents, who
are concerned about her recent weight loss. She has become a picky eater, avoiding foods that
are high in calories, she exercises 2 hours per day, and she is constantly saying that she fears
food and "looks fat." Her height is 64 inches (163 cm) and her weight has dropped from 138 lb
(62.6 kg) to 110 lb (49.9 kg) over the past 4 months. She now has a BMI of 18.7. She has not
gotten her menstrual period in the past 3 months. She denies binge eating or purging symptoms.
Vital signs, physical examination, CBC, and electrolyte panel are within normal limits. What is
the most appropriate next step in management?
 Incorrect Answer ImageA.Hospitalization
 Correct Answer ImageB.Recommend a behaviorally-based outpatient treatment
 Incorrect Answer ImageC.Recommend long-term psychodynamic psychotherapy
 Incorrect Answer ImageD.Start clonazepam
 Incorrect Answer ImageE.Start fluoxetine

A 17-year-old girl comes to the physician with her mother, who states that she doesn't know
what to do with her daughter. Ever since the daughter started modeling, she has become
concerned about her weight. The daughter states that she has a fear of gaining weight and that
whenever she eats a meal she forces herself to vomit afterward. She has not had her menstrual
period for 5 months. She denies use of laxatives or diuretics. On examination, she is thin and has
fine hair all over her body. She has scars on her knuckles and poor dentition. Her blood pressure
is 78/46 mm Hg, pulse is 54/min and regular, and BMI is 16.5 kg/m2. Which of the following is
the most appropriate next step in management?
 Correct Answer ImageA.Admit the patient to the hospital to reestablish weight and
correct metabolic abnormalities
 Incorrect Answer ImageB.Arrange a consultation with a nutritionist
 Incorrect Answer ImageC.Refer the patient for psychodynamic psychotherapy
 Incorrect Answer ImageD.Start the patient on medication to increase her appetite
 Incorrect Answer ImageE.Start the patient on sertraline

A 45-year-old woman comes to her psychiatrist for her routine biweekly appointment 2 months
after being hospitalized for an episode of severe, recurrent major depressive disorder with
psychotic features. During her hospitalization, she was started on medications which she
continued daily as an outpatient. At today's appointment, she reports to her physician that she has
missed her menstrual period for 2 months and reports tenderness in her breasts. Which of the
following medications is most likely responsible for these symptoms?
 Incorrect Answer ImageA.Olanzapine
 Incorrect Answer ImageB.Prolactinoma
 Incorrect Answer ImageC.Quetiapine
 Correct Answer ImageD.Risperidone
 Incorrect Answer ImageE.Sertraline

A 20-year-old man is brought to the physician by his parents. They tell the doctor that they are
concerned that their son is an “alcoholic” and want him to check himself in for treatment. In a
discussion about his drinking habits, the young man brags that compared to in the past, he can
now drink more before he gets drunk. Which of the following statements is most likely to be true
about this patient?
 Incorrect Answer ImageA.He can be diagnosed with alcohol intoxication
 Incorrect Answer ImageB.He can be diagnosed with alcohol use disorder
 Incorrect Answer ImageC.He will have withdrawal symptoms on cessation of alcohol
 Incorrect Answer ImageD.The amount of alcohol he will be able to drink before getting
“drunk” will increase with age
 Correct Answer ImageE.There is not sufficient information to determine if this patient
has alcohol use disorder

A 63-year-old man is brought to the clinic by his wife, because of concerns for early stages of
Alzheimer disease. The patient has been having memory disturbances lately and his wife reports
that he is “not as sharp intellectually as he used to be.” Cognitive testing reveals that the patient
has some deficiencies in memory and calculation. To evaluate the patient's abstract thinking,
which of the following aspects of the mini-mental status exam (MMSE) is most appropriate? 
 Incorrect Answer ImageA.Copy a figure
 Incorrect Answer ImageB.Count backward from 100 in increments of 7
 Incorrect Answer ImageC.Give the current date
 Correct Answer ImageD.Interpret a proverb
 Incorrect Answer ImageE.Name the past five presidents

A 19-year-old woman is brought to the emergency department by her friends after a week-long
history of “bizarre behavior.” The friends say that she has been “talking a mile a minute” and
stating that she has special powers from God. The patient has not slept in the past 3 days and yet
has been “running all over the place and spending to the limit on her credit cards.” On mental
status examination, the patient is distracted and shows pressured speech and flight of ideas.
When asked about her mood, the patient replies that she has never felt this good before. She
denies any suicidal or homicidal intent, ideation, or plan. She notes that she has never been
depressed in her life. Physical examination is unremarkable. There is no history of substance use
disorder and urinary toxicology is negative. Which of the following is the most likely diagnosis? 
 Correct Answer ImageA.Bipolar I disorder
 Incorrect Answer ImageB.Bipolar II disorder
 Incorrect Answer ImageC.Cyclothymic disorder
 Incorrect Answer ImageD.Schizophrenia
 Incorrect Answer ImageE.Schizoaffective disorder

A 63-year-old man with schizoaffective disorder is brought to the hospital by his health aide
because of a 3-day history of severe weakness in his legs, an unsteady gait, and diarrhea. His
hands are shaking and he seems mildly confused. He was recently hospitalized because of a
manic episode during which time he was also diagnosed with chronic renal insufficiency,
hypertension, and obesity. His medications were changed and he is now on a combination of
lithium, valproate, and quetiapine. Nifedipine has been added to his antihypertensive medication.
The patient’s symptoms are most likely to be due to which of the following? 
 Incorrect Answer ImageA.Alcohol use disorder
 Incorrect Answer ImageB.Gastroenteritis
 Incorrect Answer ImageC.Increased blood valproate level
 Correct Answer ImageD.Lithium intoxication
 Incorrect Answer ImageE.Total renal failure

A 31-year-old man is brought to the emergency department by a neighbor. The neighbor relates
that the patient knocked on her door complaining that he was having auditory hallucinations and
tremors, along with associated nausea and vomiting. The patient states that he feels very anxious.
He appears to be obtunded and, on mini-mental status exam (MMSE), he scores 22 of 30. Which
of the following substances is this patient most likely withdrawing from?
 Correct Answer ImageA.Alprazolam
 Incorrect Answer ImageB.Amphetamine
 Incorrect Answer ImageC.Cocaine
 Incorrect Answer ImageD.Heroin
 Incorrect Answer ImageE.Nicotine

A 44-year-old HIV-positive man is being treated in the hospital for Pneumocystis


carinii pneumonia. He has a history of treatment-resistant depression and is being treated with
amitriptyline, which has been continued in the hospital. While in the hospital, he becomes
delirious and begins to experience visual hallucinations. The psychiatry consultation team starts
him on olanzapine. The patient's delirium gradually resolves, but prior to discharge he complains
that his urine stream has become weak and his bladder feels full. On examination, the physician
confirms lower abdominal distention. Which of the following is the most effective treatment for
his current condition? 
 Incorrect Answer ImageA.Benztropine
 Correct Answer ImageB.Bethanechol
 Incorrect Answer ImageC.Furosemide
 Incorrect Answer ImageD.Propranolol
 Incorrect Answer ImageE.Physostigmine

A 24-year-old Caucasian man is brought to the emergency department. He was picked up by the
police after yelling and cursing at another driver whom he believed cut him off inappropriately
on the highway. He has been stopped by the police in the past and has been on probation for
multiple driving violations. The young man describes a history of aggressive behavior and states,
“I tend to overreact.” He reports a history of physical fights that have resulted in injuries to
himself and others. He also reports that his mother told him he once had a seizure when he had a
high fever as a child. Physical examination and laboratory studies are unremarkable. The patient
works in a factory as a production assistant and has done well at this job for the past 3 years.
Which of the following is the most appropriate diagnosis for this patient?
 Incorrect Answer ImageA.Antisocial personality disorder
 Incorrect Answer ImageB.Brief psychotic disorder
 Correct Answer ImageC.Intermittent explosive disorder
 Incorrect Answer ImageD.Oppositional defiant disorder
 Incorrect Answer ImageE.Temporal lobe seizures

A 20-year-old man is brought to the emergency department by his mother with nausea and
vomiting. She reports he took 50 tablets of 325-mg acetaminophen 6 hours ago in a suicide
attempt. His temperature is 37.0ºC (98.6ºF), blood pressure is 135/80 mm Hg, pulse is 100/min,
and respirations are 20/min. On physical examination he has mild abdominal tenderness. Serum
acetaminophen concentration is within toxic range, AST is 25 IU/L, ALT 32 IU/L, and alkaline
phosphatase is 56 IU/L. Which of the following is the most appropriate next step in
management? 
 Correct Answer ImageA.Acetylcysteine treatment
 Incorrect Answer ImageB.Activated charcoal treatment
 Incorrect Answer ImageC.Admit and monitor for liver function abnormalities
 Incorrect Answer ImageD.Gastric lavage
 Incorrect Answer ImageE.Induced vomiting

A 32-year-old single mother of two with no prior psychiatric history, is brought to the hospital
by a friend after stating that she wanted “to just disappear and never come back.” The patient
tells the physician that she has been in bed since approximately 1 week ago when she discovered
that her ex-husband, who was providing child support, had committed fraud and had lost all of
his money. She felt desperate when she realized that she didn't have any money to pay the rent
and buy food for the children. The patient tearfully admits that she has been selfish and really
hasn't thought of her children. Which of the following is the most likely diagnosis? 
 Incorrect Answer ImageA.Acute stress disorder
 Correct Answer ImageB.Adjustment disorder
 Incorrect Answer ImageC.Brief psychotic disorder
 Incorrect Answer ImageD.Post-traumatic stress disorder (PTSD)
 Incorrect Answer ImageE.Major depressive disorder

A 39-year-old woman comes to her physician because of depressed mood, lack of energy,
feelings of hopelessness and guilt, with a diminished ability to concentrate and a decreased
appetite for the past 5 weeks. She has never had similar symptoms in the past and has never been
hospitalized. She has a past history of episodes of elevated mood and energy with increased
irritability lasting around one week, but denies feelings of grandiosity or delusions. Physical
examination of the patient is within normal limits. Which of the following is the most likely
diagnosis?
 Incorrect Answer ImageA.Bipolar I disorder
 Correct Answer ImageB.Bipolar II disorder
 Incorrect Answer ImageC.Cyclothymic disorder
 Incorrect Answer ImageD.Major depressive disorder
 Incorrect Answer ImageE.Substance use disorder

A 47-year-old Hispanic woman who has no past psychiatric history comes to the physician
because of poor sleep, decreased appetite, and sadness. These symptoms began 3 months ago
when she discovered that her husband was having an affair. Her husband apologized to her and
promised to end the affair. Despite this, the woman reports feeling betrayed and has feelings of
being a bad wife and mother to her three children. She also reports that she has been more
reluctant to see friends and do her usual hobbies in the past few months and wonders if her life is
worth living like this. What is the most appropriate next step in the management of this patient? 
 Incorrect Answer ImageA.Admit her to the inpatient unit
 Incorrect Answer ImageB.Arrange for couples therapy
 Incorrect Answer ImageC.Arrange for individual therapy
 Correct Answer ImageD.Assess for suicidal thoughts
 Incorrect Answer ImageE.Prescribe antidepressant medication

A 25-year-old African American man is brought to the physician by his parents for follow up.
His parents report a 3-year history of odd behavior and poor functioning. The parents tell the
doctor that he dropped out of community college and has been unable to get a job. He has trouble
doing things for himself, such as cleaning his room or preparing meals. They are worried that he
has been hearing voices and at times seems to be talking to himself. On mental status
examination, the patient’s speech is hard to understand and his thoughts are tangential. His affect
is flat and mildly inappropriate. He denies paranoid or suicidal ideation and medical
examination, laboratory studies, and toxicology are unremarkable. Which of the following is the
most likely diagnosis? 
 Incorrect Answer ImageA.Autism spectrum disorder
 Incorrect Answer ImageB.Intellectual disability
 Incorrect Answer ImageC.Major neurocognitive disorder
 Correct Answer ImageD.Schizophrenia
 Incorrect Answer ImageE.Schizophreniform disorder

A 24-year-old student is brought to the emergency department because of a sudden change in


behavior. His friends report that he has been acting "weird" and confused and has been talking
about "flying above the floor" since being at a party the night before. The student laughs loudly
without reason and then bursts into tears. He appears hot and has uncoordinated movements. On
physical examination, he has horizontal nystagmus, ataxia, and muscular rigidity, and becomes
very agitated and combative during the examination. Which of the following agents should be
administered at this time to treat his agitation? 
 Incorrect Answer ImageA.Chlorpromazine
 Incorrect Answer ImageB.Flumazenil
 Incorrect Answer ImageC.Haloperidol
 Correct Answer ImageD.Lorazepam

 Incorrect Answer ImageE.Naltrexone

A 26-year-old single, unemployed Caucasian man who has a history of schizophrenia comes to
the physician for evaluation. He reports that his previous psychiatrist prescribed risperidone, 2
mg/d, but he does not take it regularly. He states that it makes him tired and occasionally stiff,
and he does not think he needs to take medication on a daily basis. He is mildly disorganized and
somewhat disheveled but denies acute psychotic symptoms or thoughts of self-harm. Which of
the following is the most appropriate next step in management?
 Incorrect Answer ImageA.Hospitalize the patient
 Incorrect Answer ImageB.Increase risperidone to 4 mg/d
 Incorrect Answer ImageC.Switch to aripiprazole
 Correct Answer ImageD.Switch to depot medication
 Incorrect Answer ImageE.Switch to olanzapine

A 26-year-old medical student is arrested for threatening a neighbor with a knife. The student is
brought to the hospital because he tried to slit his wrists on the way to the police station. When
his roommate arrives he states that the patient has been doing “weird things” over the past two
months and has been “acting as if he had no emotions.” The patient is socially withdrawn and
has been overheard saying that the FBI is giving him tips on patient care and that “people from
the sky” are telling him to do “bad things.” He says he does not want to be a doctor anymore. His
face is expressionless and unresponsive. His physical examination is otherwise typical.
Toxicologic screening is negative. Which of the following is the most likely diagnosis?
 Incorrect Answer ImageA.Bipolar I disorder
 Incorrect Answer ImageB.Brief psychotic disorder
 Incorrect Answer ImageC.Delusional disorder
 Incorrect Answer ImageD.Schizophrenia
 Correct Answer ImageE.Schizophreniform disorder

A 17-year-old adolescent male is brought to the hospital by police after being stopped for driving
too slowly. He denies any use of alcohol but seems slow in his behavior and reactions. He is
laughing inappropriately and has a dry mouth. His gait is somewhat slow and his coordination is
impaired. His eyes are red, which he explains as being due to pollen allergy. Which of the
following will most likely appear on a urine drug screen?
 Incorrect Answer ImageA.Amphetamines
 Incorrect Answer ImageB.Benzodiazepines
 Incorrect Answer ImageC.Cocaine
 Incorrect Answer ImageD.Opiates
 Correct Answer ImageE.Tetrahydrocannabinol

A 35-year-old man is brought to the emergency clinic by his mother because of an episode of
slurred speech associated with the uncomfortable sensation that his tongue is thick and
protruding from his mouth. The episode started suddenly 30 minutes ago. The patient is noted to
be holding onto his tongue with his thumb and forefinger. His mother reports that the patient has
had schizophrenia for 10 years and consistently takes medications prescribed by his psychiatrist.
Several days ago, however, the patient ran out of one of his medications but has continued to
take the others. Administration of which of the following should be included in the initial step in
the management of this patient?
 Incorrect Answer ImageA.Alprazolam
 Correct Answer ImageB.Benztropine
 Incorrect Answer ImageC.Haloperidol
 Incorrect Answer ImageD.Lorazepam
 Incorrect Answer ImageE.Olanzapine

A 35-year-old woman comes to the physician for a routine health maintenance examination. She
has no physical complaints, but she is concerned that she hardly ever feels happy. She says that
she has basically been “down” for at least 3 years. She rarely goes out with friends, and basically
keeps to herself at work. She states that her work performance has been stable but uninspired and
that she usually feels tired and “blah.” She denies suicidality and physical examination and
laboratory studies are unremarkable. Which of the following is the most likely diagnosis?
 Incorrect Answer ImageA.Antisocial personality disorder
 Incorrect Answer ImageB.Cyclothymic disorder
 Incorrect Answer ImageC.Major depressive disorder
 Correct Answer ImageD.Persistent depressive disorder
 Incorrect Answer ImageE.Schizoaffective disorder

After experiencing a very rough landing on a commercial flight, a successful, 27-year-old


businesswoman develops a fear of flying. Now, she is paralyzed with fear and unable to travel
for business. Her physician gave her a prescription for lorazepam to take just before flying but
she said that it did not help, and she was still too frightened to fly. She returns to the physician
and asks if there is anything else that she can do to reduce her fear because her promotion at
work depends on her ability to travel. Which of the following is the most commonly used
treatment for this disorder?
 Incorrect Answer ImageA.Aversive conditioning
 Correct Answer ImageB.Exposure therapy
 Incorrect Answer ImageC.Fluoxetine
 Incorrect Answer ImageD.Propranolol
 Incorrect Answer ImageE.Supportive therapy

A 57-year-old man comes to the physician because of "depression." He states that he has been
tearful and has experienced a 15-pound weight loss in the months following the death of his
wife. Which of the following is the most appropriate initial question to open the interview? 
 Incorrect Answer ImageA."Do you ever feel suicidal?"
 Incorrect Answer ImageB.“Do you ever hear things that others cannot hear?”
 Incorrect Answer ImageC.“Please tell me about your sleep and appetite?”
 Correct Answer ImageD."What brings you here today?"
 Incorrect Answer ImageE."Why are you depressed?"

A medical student is interviewing a 78-year-old man who was brought from a nursing home to
the emergency department. The student has been talking to the man for almost 30 minutes and
has obtained very little information. Every time he asks a question, the patient starts talking and
goes into unnecessary details, eventually answering the question but only after he has told his
entire story to the medical student. The student is becoming frustrated with the patient because
there are two other patients waiting to be interviewed. This patient's speech is an example of
which of the following? 
 Correct Answer ImageA.Circumstantiality
 Incorrect Answer ImageB.Flight of ideas
 Incorrect Answer ImageC.Loose associations
 Incorrect Answer ImageD.Tangentiality
 Incorrect Answer ImageE.Word salad

A 52-year-old African American woman comes to the physician because of outbursts of crying
spells, sleep problems, and memory problems. She says she would like to be with her son, who
died 5 weeks ago, even though she denies suicidal ideation, intent, or plan. She expresses
extreme anger at God and questions her religious faith. She states that she has little interest in
activities that used to bring pleasure. Which of the following is the most appropriate next step in
the management of this patient?
 Correct Answer ImageA.No treatment necessary at this time
 Incorrect Answer ImageB.Observe for a neurocognitive disorder
 Incorrect Answer ImageC.Prescribe sertraline
 Incorrect Answer ImageD.Recommend a psychiatric consultation
 Incorrect Answer ImageE.Refer patient for psychotherapy

A 22-year-old college student comes to the physician because of feelings of worthlessness and
sadness, and restless sleep. She admits to missing classes because she “just can't seem to get out
of bed,” and is “indifferent” about her plans following graduation. She reports that she has felt
this way for a couple of weeks, but she has recently been crying more frequently. She denies
suicidal plans but says that she has had “strange” visions of a macabre nature that suggest to her
that she “should” kill herself. Additionally, she confides that she “knows” that her high-school
academic board is trying to locate her at college and revoke her high-school diploma for a paper
she secretly plagiarized, though she is unable to provide evidence for this belief. She feels
extremely guilty about attending college, given her dishonesty. She has even changed her
appearance so that she will not be “recognized by investigators from her high-school academic
board.” She was given medications to control her symptoms and seemed to show improvement.
However, several months later she reported weight gain of more than 9 kg (20 lb). Which of the
following medications most likely caused this significant weight gain?
 Incorrect Answer ImageA.Aripiprazole
 Incorrect Answer ImageB.Haloperidol
 Incorrect Answer ImageC.Lurasidone
 Correct Answer ImageD.Olanzapine
 Incorrect Answer ImageE.Risperidone

A 42-year-old man comes to the physician complaining of a 2-month history of a depressed


mood, difficulty sleeping, and decreased appetite. He says his work performance is also
suffering. When discussing medical therapy, the patient tells his physician that he is extremely
concerned about his sexual performance. He notes how recently he has been having some sexual
difficulties with his wife, which he attributes to “getting older.” Which of the following would be
the best choice for initial treatment of this patient?
 Correct Answer ImageA.Bupropion
 Incorrect Answer ImageB.Fluoxetine
 Incorrect Answer ImageC.Imipramine
 Incorrect Answer ImageD.Sildenafil
 Incorrect Answer ImageE.Sertraline

A 20-year-old woman has a history of repeated hospital admissions for sudden episodes of
intense nausea. No known etiology has been found for this nausea despite intensive metabolic,
infectious, and endocrine evaluations. The patient has also had repeated imaging studies of her
abdomen and one exploratory laparotomy. She comes to the emergency department complaining
of similar nausea and is admitted to the hospital for evaluation. Now on the inpatient floor, the
patient seems relaxed and comfortable. Imaging and laboratory studies are unremarkable. When
told about the typical findings, the patient asks what tests or surgeries are scheduled next, but
expresses no desire to be released from the hospital. Which of the following is the most likely
diagnosis?
 Incorrect Answer ImageA.Delusional disorder
 Correct Answer ImageB.Factitious disorder
 Incorrect Answer ImageC.Illness anxiety disorder
 Incorrect Answer ImageD.Malingering
 Incorrect Answer ImageE.Somatic symptom disorder

A 68-year-old woman with hypertension comes to the physician because of fatigue for the past 6
weeks. She says that even though she has recently lost about 7 lb (3.2 kg), she does not want to
eat. She is also worried because every time she is watching TV or somebody is talking to her, she
finds her mind "wandering" and cannot stay focused on what the person is saying. She notes that
it takes her 2-3 hours to fall asleep, and yet she wakes up early in the morning. She thinks all
these difficulties are due to her "advanced age," and she “just feels like staying home.” She tells
the physician, "Nothing can be done, Doctor." The last visit to her primary care physician was 3
months ago. At that time her blood pressure was 150/90 mm Hg and the doctor put her on an
antihypertensive. Which of the following agents is her primary care physician most likely to
have prescribed for this patient?
 Incorrect Answer ImageA.Captopril
 Incorrect Answer ImageB.Furosemide
 Incorrect Answer ImageC.Minoxidil
 Correct Answer ImageD.Propranolol
 Incorrect Answer ImageE.Verapamil

An 82-year-old woman, whose husband passed away 6 weeks ago after a long illness, is admitted
to the hospital for worsening of her cardiac problems. During the hospital stay, a psychiatrist is
invited to consult at the request of the patient's daughters. The daughters think their mother is
depressed because she is sad and cries all the time. Her family is also concerned because she
does not want to eat and has lost about 4 lb (1.8 kg) during this time period. She often talks about
her dead husband and states that she has heard his voice several times. She blames herself for not
having gone with him on his last visit to his sister, and she starts crying. She states, “I have some
good days here and there. I’m just so lonely.” Which of the following is the most likely
diagnosis? 
 Incorrect Answer ImageA.Depression secondary to general medical condition
 Incorrect Answer ImageB.Major depressive disorder
 Correct Answer ImageC.Normal grief reaction
 Incorrect Answer ImageD.Persistent depressive disorder
 Incorrect Answer ImageE.Psychosis not otherwise specified

A woman comes to the clinic with her 13-year-old grandson, stating that he has had behavioral
problems for the past 4 years. She says that she has been trying to handle this issue on her own,
but that he is getting out of control. Her grandson is living with her now because his mother is in
a drug rehabilitation center. She states that the boy “answers back to” and argues with her and
with teachers and adult neighbors. In school, he often blurts out answers to questions before the
questions are completed and he is hyperactive and impulsive. He deliberately argues to annoy
people. He has been removed from the school bus for lying about another student and
challenging the driver. Recently, the grandmother caught the boy stealing from her wallet; when
confronted he showed no remorse. He is about to be expelled from school since he punched his
classmate in the face. Which of the following is the most likely diagnosis?
 Incorrect Answer ImageA.Attention deficit/hyperactivity disorder (ADHD)
 Incorrect Answer ImageB.Antisocial personality disorder
 Correct Answer ImageC.Conduct disorder
 Incorrect Answer ImageD.Intermittent explosive disorder
 Incorrect Answer ImageE.Oppositional defiant disorder

A 40-year-old man is brought to the emergency department just after the small plane he was
piloting crashed into the ocean. The man has survived the crash with cuts and a broken arm, but
he claims he has no idea how the crash happened or how he escaped the plane. He is also unable
to explain how he got his life jacket on. His physical examination is significant only for minor
lacerations and a fractured right humerus, and he has no alteration in consciousness. A CT scan
is normal. Which of the following is the most likely diagnosis? 
 Incorrect Answer ImageA.Acute stress disorder
 Correct Answer ImageB.Dissociative amnesia
 Incorrect Answer ImageC.Dissociative amnesia with fugue
 Incorrect Answer ImageD.Factitious disorder
 Incorrect Answer ImageE.Post-traumatic stress disorder

A 72-year-old African American woman is being readied for discharge from the hospital 2 weeks
after a stroke that affected her right occipital cortex. Because the patient has had severe
insomnia, decreased appetite, and crying spells since the stroke, a psychiatric consultation is
scheduled. After determining that the patient's symptoms are due to depression and not to
delirium, which of the following medications would be most appropriate to treat her depressive
symptoms and severe insomnia?
 Incorrect Answer ImageA.Amitriptyline
 Incorrect Answer ImageB.Methylphenidate
 Incorrect Answer ImageC.Nortriptyline
 Incorrect Answer ImageD.Phenelzine
 Correct Answer ImageE.Trazodone

A 27-year-old man with schizophrenia comes to the physician for a follow-up visit. He is alert,
oriented, and well-related, and his thought process is completely organized. He is not
complaining of auditory hallucinations and does not seem to be responding to internal stimuli.
He denies any suicidal or homicidal intent, ideation, or plan. He says he has difficulty staying in
his chair during the interview and constantly feels an urge to move around. His past medical
history is significant for pneumonia 1 month ago which resulted in delirium which led to
hospitalization. During the hospitalization, his dose of risperidone was changed. Which of the
following is the most appropriate next step in management? 
 Incorrect Answer ImageA.Add an antidepressant
 Incorrect Answer ImageB.Add haloperidol to control psychotic agitation
 Correct Answer ImageC.Decrease the dose of risperidone
 Incorrect Answer ImageD.Increase the dose of risperidone because of incomplete
treatment response
 Incorrect Answer ImageE.Switch the patient to haloperidol

Most weekdays, a 36-year-old man stands on the same subway platform at 7 A.M. because
across the tracks, a group of high school girls gathers at that time daily. If no one else is on the
platform, the man opens his coat and exposes his genitals to the girls hoping to shock and
frighten them. This behavior is most characteristic of which of the following disorders? 
 Correct Answer ImageA.Exhibitionistic disorder
 Incorrect Answer ImageB.Fetishistic disorder
 Incorrect Answer ImageC.Frotteuristic disorder
 Incorrect Answer ImageD.Sexual masochism disorder
 Incorrect Answer ImageE.Sexual sadism disorder

A 19-year-old woman comes to her physician because she often feels tired and sleeps throughout
the course of the day. She says that she sleeps for about 7 hours each night, and has been told
that she snores while sleeping. A few weeks ago, she collapsed to the floor when she met her
father for the first time after he had a stroke. She remained awake during this episode. She notes
that she sometimes finds that she cannot move for a few minutes when she first wakes up. She
has no significant medical history and denies substance use. Her vital signs are within normal
limits. Her physical examination is normal except for obesity. Which of the following is the most
appropriate management for this patient’s condition?
 Incorrect Answer ImageA.Continuous positive airway pressure (CPAP)
 Incorrect Answer ImageB.Ethosuximide
 Correct Answer ImageC.Modafinil
 Incorrect Answer ImageD.No treatment is required at this time
 Incorrect Answer ImageE.Temazepam

A 29-year-old woman was climbing the stairs in her new home 3 months ago when the staircase
collapsed. She was taken to the hospital, diagnosed, and treated for a fractured left femur. Today
at her follow-up appointment, she reports nightmares and flashbacks and is afraid to go to sleep
as a result of the accident. During the interview, she is tearful and afraid that her fear of falling is
preventing her from fully participating in her rehabilitation. Which of the following is the most
appropriate next step in management?
 Incorrect Answer ImageA.24-hour suicide watch
 Incorrect Answer ImageB.Insight-oriented psychotherapy
 Incorrect Answer ImageC.Lorazepam
 Incorrect Answer ImageD.No intervention
 Correct Answer ImageE.Sertraline

A 28-year-old man comes to the emergency department because of a 3-month history of fatigue,
depression, insomnia, guilty-feelings, and weight loss. A Mini-Mental Status examination
shows that he has some mild cognitive defects, scoring 24 of 30. On physical examination, the
patient is noted to have oral thrush and lymphadenopathy. Which of the following is the most
appropriate next step in management?
 Incorrect Answer ImageA.Administer fluoxetine
 Incorrect Answer ImageB.Apply miconazole gel
 Correct Answer ImageC.Perform enzyme-linked immunosorbent assay (ELISA)
 Incorrect Answer ImageD.Perform a urine drug screen
 Incorrect Answer ImageE.Perform a head CT without contrast
A 45-year-old Caucasian woman comes to the physician for a routine health maintenance
examination. Her physician notices that she has lost approximately 12 kg (25 lb) over the past
year. When asked about her weight loss, she states that she has not had much of an appetite since
her daughter left to go to college about 8 months ago. In describing this, she is somewhat tearful
and complains of loneliness, poor memory, decreased concentration and feeling tired. Although
she appears very sad, she denies feeling depressed and states that she simply needs to become
more socially active to distract herself from her worries, which keep her up at night. Her physical
examination is unremarkable except for the noticeable weight loss. Her height is 168 cm (5ft 6
in) and her current weight is 47.6 kg (105 lb). Which of the following is the most likely
diagnosis?
 Incorrect Answer ImageA.Adjustment disorder with depressive symptoms
 Incorrect Answer ImageB.Bereavement
 Incorrect Answer ImageC.Dependent personality disorderIncorrect Answer Image
 Incorrect Answer ImageD.Persistent depressive disorder
 Correct Answer ImageE.Major depressive disorder

A 37-year-old IV heroin abuser, who is HIV-positive and has many legal and social problems,
has been admitted to the hospital for treatment of pneumonia. During his stay, he intermittently
refuses to undergo procedures or cooperate with doctors but insists on going out to smoke with
his visitors. On one occasion, a visitor was caught injecting drugs into the patient’s IV infusion.
When confronted, the patient throws a tray at a member of the staff and accuses him of being
unprofessional. Which of the following is the most likely diagnosis? 
 Correct Answer ImageA.Antisocial personality disorderCorrect Answer Image
 Incorrect Answer ImageB.Borderline personality disorder
 Incorrect Answer ImageC.Conduct disorder
 Incorrect Answer ImageD.Major depressive disorder with anxious distress
 Incorrect Answer ImageE.Paranoid personality disorder

A 35-year-old woman comes to the physician for evaluation. She reports that the apartment
building in which she lives had a serious fire approximately one year ago, and she narrowly
escaped. Many other tenants were injured and had to be taken to the hospital for medical care.
Since that time, the woman has had nightmares about being trapped in her apartment and unable
to escape. She cannot concentrate at work because she often thinks of the fire, and she feels
fearful when she sees someone light a match. She has not been participating in her usual hobbies
and feels detached from her friends and family. She reports difficulty falling asleep because she
is afraid she will have more nightmares about the fire. She also reports a poor appetite and a
“short fuse” with coworkers and friends. Which of the following is the most appropriate
intervention for this patient?
 Incorrect Answer ImageA.Flooding
 Incorrect Answer ImageB.Long-term psychodynamic psychotherapy
 Incorrect Answer ImageC.Psychological debriefing
 Incorrect Answer ImageD.Start alprazolam
 Correct Answer ImageE.Start sertraline
A 23-year-old woman is brought to the emergency department by ambulance at midnight after
her roommate found her using a paper clip to make lacerations on her wrists. The patient had
been seen in the same emergency department twice during the past 2 weeks with a complaint of
feeling very depressed and, at times, suicidal. She has a history of one drug overdose on
children's vitamins. At both previous visits to the emergency department, she was prescribed
benzodiazepines for anxiety and sent home with an outpatient psychiatric clinic referral. On this
presentation, she states that if she is not admitted to the hospital she will go out and find a bridge
to jump off. Which of the following is the most appropriate initial step in management?
 Incorrect Answer ImageA.Admit the patient to the medicine unit immediately
 Correct Answer ImageB.Admit the patient to the psychiatric unit immediately
 Incorrect Answer ImageC.Explain to the patient that her attempts to mutilate herself and
overdose on vitamins are unlikely to result in death
 Incorrect Answer ImageD.Explain to the patient that she needs psychotherapy as soon as
possible
 Incorrect Answer ImageE.Set up a new outpatient psychiatric clinic appointment as soon
as possible
A 45-year-old Caucasian woman comes to the emergency department because of recent onset of
anxiety and depression. The patient says that she worries about everything and has felt very
depressed lately. She reports lack of energy, difficulty sleeping, and a decreased appetite. She
reports difficulties in her relationship with her husband whom she depends on financially. Her
husband does not talk to her very much and often leaves home for long periods. She states that
he is not violent and she denies physical abuse or threats by him. She has 6-year-old twin sons
who were recently diagnosed with autism spectrum disorder and require a lot of attention, and
she is feeling overwhelmed with her family and financial problems. She appears to be an
adequate historian and is cooperative during the interview, but appears slightly disheveled. Her
mood and affect are depressed and anxious. She becomes tearful during the interview. Her flow
of thought is coherent, and her thought content reveals feelings of low self-esteem. There is
absence of delusions and hallucinations. The patient denies suicidal ideas. She is oriented to date,
place, and person with intact recent and remote memory. The patient denies any previous
medical or psychiatric problems. Her vital signs and physical examination are unremarkable.
Which of the following is the most appropriate next step in management?
 Incorrect Answer ImageA.Admit the patient to the hospital
 Incorrect Answer ImageB.Call for a psychiatric consult to evaluate for major depressive
disorder
 Incorrect Answer ImageC.Call for a social worker consult to assist with management
 Incorrect Answer ImageD.Discharge the patient from the emergency department with
antidepressant medication
 Incorrect Answer ImageE.Discharge the patient from the emergency department with
anxiolytic medication
 Correct Answer ImageF.Discharge the patient from the emergency department with an
appointment for outpatient mental health followup
 Incorrect Answer ImageG.Report the possibility of spousal abuse to social services and
inform the patient of available help
During a trip out of town, a 25-year-old woman is involved in a minor motor vehicle accident
and is brought to the emergency department. She has no injuries, aside from minor abrasions.
During the observation period, the physician notices that she appears anxious and confused.
Physical examination shows she is disoriented, tremulous, and her heart rate and blood pressure
are elevated. During the initial intake, the patient reported that the only medication she uses is
alprazolam, which is prescribed for her to take "as needed" for anxiety attacks. She clarifies that
she has been taking alprazolam daily for one month and ran out early before embarking on her
trip. Her friends arrive in the emergency department, clearly supportive and willing to help her in
any way needed. Which of the following is the most appropriate next step in management?
 Correct Answer ImageA.Admit her to the medical unit and treat for delirium
 Incorrect Answer ImageB.Admit her to the psychiatry unit and treat the anxiety attack
 Incorrect Answer ImageC.Discharge her to her friends without refilling her alprazolam
and advise her to return to the hospital if her condition worsens
 Incorrect Answer ImageD.Discharge her to her friends with a one week prescription for
alprazolam and advise her to return to the hospital if her condition worsens
 Incorrect Answer ImageE.Recommend discharge to an outpatient substance use disorder
treatment program

 30-year-old woman experiences the sudden onset of a headache, dizziness, and tingling in her
left arm while standing on the subway platform. She is overwhelmed by fear and feels like she is
going to die. She has difficulty catching her breath and is sweating profusely. Over the next few
weeks, she has several similar episodes in which she feels that she is about to lose her mind. She
develops intense fear when she needs to go alone to any place that is more than a few blocks
from her home. She has gone to the emergency department twice because of similar episodes.
During these visits, her cardiac enzymes were normal, ECG showed tachycardia with no QRS or
ST variations, and CBC showed mild anemia. Her father died suddenly from a stroke 5 months
ago. Which of the following is the most likely diagnosis? 
 Correct Answer ImageA.Anxiety disorder
 Incorrect Answer ImageB.Dissociative disorder
 Incorrect Answer ImageC.Personality disorder
 Incorrect Answer ImageD.Somatic symptom disorder
 Incorrect Answer ImageE.Vascular neurocognitive disorder

A 51-year-old man is dying from end-stage Huntington disease. He is currently on a medical


floor and is receiving comfort care measures. He is obtunded, his respirations are becoming
agonal, and his blood carbon dioxide levels are rising steadily. The patient’s advance directive
states: "No intubation under any circumstances, no CPR, no intensive care unit–level treatment,
no assisted breathing devices, and no heroic measures." The nurses have been issued DNR/DNI
(Do Not Resuscitate/Do Not Intubate) orders for the patient. The patient's son wishes to discuss
the patient's situation. The son has recently heard of a new herbal treatment and thinks his father
would be interested in trying it. He begs the physician to intubate his father because this will
hopefully buy some time for the son to procure the herbal medication as well as speak to the
patient, should he unexpectedly become less obtunded. He also admits he just can't seem to "let
Dad go," and wants "to talk to him one more time." The patient's daughter, the only other family
member, agrees with the son's wishes. Which of the following is the most appropriate next step?
 Incorrect Answer ImageA.Contact the ethics committee and intubate temporarily until
recommendations are available
 Incorrect Answer ImageB.Follow the son’s request as he is the next of kin
 Incorrect Answer ImageC.Have the patient sign power of attorney papers, then comply
with family wishes
 Incorrect Answer ImageD.Initiate BiPap, because new information may change the
patient's wishes and void the DNR/DNI
 Correct Answer ImageE.Make no change in the treatment plan

 45-year-old woman comes to the physician because of blurred vision. She states that this
symptom started about 2 days ago. She denies any past history of significant medical or
neurologic problems. She does state that several months ago she began to have depressive
symptoms and several days ago she went to a psychiatrist who put her on medication for
depression. She admits to drinking more water over the last several days due to a dry mouth. She
also complains of dizziness when she stands up from lying or sitting. Her temperature is 37ºC
(98.6ºF), blood pressure lying down is 135/75 mm Hg, blood pressure standing is 110/64 mm
Hg, pulse lying down is 84/min, pulse standing is 95/min, and respiratory rate is 16/min.
Physical examination is unremarkable except for mild pupillary dilation which is the likely cause
of her blurred vision. Which of the following antidepressant medications most likely accounts for
this patient's symptoms? 
 Incorrect Answer ImageA.Bupropion
 Incorrect Answer ImageB.Citalopram
 Correct Answer ImageC.Imipramine
 Incorrect Answer ImageD.Vortioxetine
 Incorrect Answer ImageE.Sertraline

A 30-year-old woman, a model, has a 7-year history of binging and purging. She also exercises
excessively, often for 4 hours/day, 6 days a week. She does not eat publicly, and she hides food.
She spends hours in front of the mirror looking at her body. She is not currently suicidal and
does not have a history of suicidal ideation. Her blood pressure is 127/77 mm Hg, pulse is
78/min, and respirations are 17/min. Her BMI is 24 kg/m2. The rest of her physical examination
is normal and laboratory studies are unremarkable. The patient has been referred to an eating
disorders program for behavioral therapy. Which of the following is the most appropriate next
step in management?
 Incorrect Answer ImageA.Bupropion
 Correct Answer ImageB.Fluoxetine
 Incorrect Answer ImageC.Hospitalization
 Incorrect Answer ImageD.Nutritional counseling
 Incorrect Answer ImageE.Licensed personal trainer
A 50-year-old man comes to the emergency department because of bilateral tremor in his hands,
diaphoresis, anxiety, headache, and the sensation that “my skin is crawling.” His medical history
is significant for hypertension, for which he takes hydrochlorothiazide and enalapril. He also has
bipolar disorder and anxiety for which he takes three medications prescribed by his psychiatrist.
He ran out of these three medications 3 days ago. His temperature is 37.0ºC (98.6ºF), blood
pressure is 150/100 mm Hg, pulse is 115/min, and respirations are 20/min. Physical examination
is notable for diaphoresis and tremulousness. Administration of which of the following is the
most appropriate initial step in this patient's care? 
 Incorrect Answer ImageA.Clonidine
 Incorrect Answer ImageB.Haloperidol
 Incorrect Answer ImageC.Hydrochlorothiazide
 Correct Answer ImageD.Lorazepam
 Incorrect Answer ImageE.Prochlorperazine

A 31-year-old woman with schizophrenia is brought to the emergency department by the police
because she was found in a park, actively hallucinating and talking to herself. She is cooperative
but irritable. She denies command hallucinations and is not violent or suicidal. She has been
living in a personal care home for the past 3 months and had been stable on her medication. She
tells the physician that she ran out of her medication several days ago and was unable to get in
touch with her case manager. She notes that she left the personal care home yesterday and did
not return. Which of the following is the most appropriate next step in management? 
 Incorrect Answer ImageA.Admit her to the psychiatry inpatient service for further
treatment
 Correct Answer ImageB.Contact her case manager to ensure support and follow-up
 Incorrect Answer ImageC.Give her a new prescription and discharge her to the personal
care home
 Incorrect Answer ImageD.Suggest that she join a support group for people with
schizophrenia
 Incorrect Answer ImageE.Suggest that she take a haloperidol decanoate injection so that
she can stay “medicated” for a month

A 22-year-old white woman comes to a psychiatrist's office after having been referred by her
family physician. She reports no history of mental problems but tells the psychiatrist that she is
concerned about hair loss. Her mother says that the patient has been combing her hair frequently
and checking her image in the mirror. Over the past 6 months, her family physician has
examined her several times but found no evidence of hair loss. He has run multiple tests, but
there is no evidence of any abnormality. The woman states that she is embarrassed and tries to
avoid going out because everyone "will notice my hair loss” and ask questions. She denies
pulling out her hair and there is no evidence that she does so. Despite this problem, she is doing
well in her job. Which of the following is the most likely diagnosis? 
 Incorrect Answer ImageA.Alopecia areata
 Correct Answer ImageB.Body dysmorphic disorder
 Incorrect Answer ImageC.Brief psychotic disorder
 Incorrect Answer ImageD.Bulimia nervosa
 Incorrect Answer ImageE.Trichotillomania

A 54-year-old woman comes to the emergency department with profuse vaginal bleeding. She
has been followed by a gynecologist for a “fibroid uterus” and has had multiple episodes of small
amounts of vaginal bleeding at irregular times. She states that she has never had this much blood
loss before. A complete blood cell count, approximately 1 year ago, showed her hematocrit to be
42%. A spun hematocrit sent from the emergency department today returns at 20%. As 4 units of
typed and cross-matched packed red blood cells are ordered, she states that she is a Jehovah's
Witness and does not want any type of blood transfusion. She knows that she requires emergency
surgery and that she will most likely lose more blood during the operation. Because her past
medical history is significant for low hematocrit, coronary artery disease, and chronic obstructive
pulmonary disease, blood loss during surgery will put this patient at increased risk for death.
Which of the following is the most appropriate next step in management?
 Incorrect Answer ImageA.Consult with a clergyman regarding the patient's choice before
surgery
 Incorrect Answer ImageB.Consult with a risk management lawyer before proceeding
with surgery
 Correct Answer ImageC.Discuss with the patient how her history and complications can
lead to the increased risk of death during surgery and ask her to repeat in her own words her
understanding of this risk
 Incorrect Answer ImageD.Insist that she have a preoperative blood transfusion before
you will do the surgery
 Incorrect Answer ImageE.Proceed with the operation and transfuse intraoperatively if it
becomes necessary during surgery

A 36-year-old woman comes to her gynecologist because of a 3-month history of amenorrhea.


Until this time, her menstrual periods had been regular. She also complains of decreased sex
drive, worsening over the past couple of months. The patient denies any other symptoms. She
started seeing a psychiatrist 5 months ago after a brief hospitalization during which she was
diagnosed with major depressive disorder, severe, with psychotic features. She was placed on
medication in the hospital and continues to take it. She currently is not experiencing depressive
or psychotic symptoms. She has no other significant medical history. Which of the following
medications is most likely responsible for the patient's current symptoms?
 Incorrect Answer ImageA.Fluoxetine
 Incorrect Answer ImageB.Quetiapine
 Correct Answer ImageC.Risperidone
 Incorrect Answer ImageD.Trazodone
 Incorrect Answer ImageE.Venlafaxine

A 29-year-old woman comes to her new primary care physician requesting medication to help
with her recent spells of anxiety and depression. She wishes to be started on a medication that
will not cause too much sedation. Examination of her medical records shows two previous
hospitalizations for minor laceration repair on her left arm and treatment for an acetaminophen
overdose. For both of these admissions, she was under close observation. She is otherwise
healthy and does not smoke cigarettes or drink alcohol. She is not currently taking any
medication. Given this history, which of the following antidepressant medications would most
likely be contraindicated for this patient?
 Correct Answer ImageA.Amitriptyline
 Incorrect Answer ImageB.Buspirone
 Incorrect Answer ImageC.Fluoxetine
 Incorrect Answer ImageD.Paroxetine
 Incorrect Answer ImageE.Sertraline

A 51-year-old man comes to the emergency department several days after being discharged from
the hospital where he had an extensive workup for abdominal pain that was negative. He now
reports similar abdominal pain, and when the physician tries to reassure him that his physical
findings and laboratory studies are normal, he accuses the medical team of "not doing enough."
He insists on seeing a surgeon but agrees to a psychiatric consultation first. The psychiatrist
describes the patient's mental status as unremarkable with no evidence of suicidal or homicidal
ideation. The interview with the patient, however, reveals that 6 weeks ago, after 24 years of
marriage, his wife died in a car accident. Which of the following is the most appropriate initial
step in management?
 Incorrect Answer ImageA.Admit the patient to psychiatry service for further evaluation
 Correct Answer ImageB.Ask about the patient's feelings about the recent loss of his wife
 Incorrect Answer ImageC.Evaluate the patient for factitious disorder
 Incorrect Answer ImageD.Prescribe an antidepressant
 Incorrect Answer ImageE.Reassure the patient that his pain is most likely stress-related

A 52-year-old woman is brought to the emergency department because of changes in movement.


She started having slowed down, stiff movements over the past 24 hours. She is currently
undergoing treatment with chemotherapy after surgery for breast cancer and just started taking
prochlorperazine for nausea yesterday. She has no other medical conditions. Her physical
examination, laboratory studies, and vital signs are all within normal limits. Which of the
following is the most appropriate pharmacotherapy?
 Incorrect Answer ImageA.Alprazolam
 Correct Answer ImageB.Benztropine
 Incorrect Answer ImageC.Haloperidol
 Incorrect Answer ImageD.Methylphenidate
 Incorrect Answer ImageE.Valproic acid

A 32-year-old man is in twice-weekly insight-oriented psychotherapy with a psychiatrist.


Recently, the patient has been exploring his thoughts and feelings around his wife's complaint
that he is too restricted and inhibited in their sexual activity. The patient admits he finds his wife
sexually attractive and wishes to be more sexually available for her, but finds himself
maintaining a restricted stance. Which of the following defense mechanisms would best describe
this patient’s behavior in his sexual relationship with his wife? 
 Incorrect Answer ImageA.Projection
 Correct Answer ImageB.Reaction formation
 Incorrect Answer ImageC.Sexualization
 Incorrect Answer ImageD.Somatization
 Incorrect Answer ImageE.Sublimation

A 31-year-old man with schizoaffective disorder comes with his family to the psychiatrist for
follow-up. He has an unremarkable medical history and has been stable and compliant with his
medication. He is very concerned, however, about his 15 lb weight gain in the past 3 months,
despite trying to watch his diet and increase his exercise. His wife tells the physician that she
heard about a new medication that is available for the treatment of schizophrenia and is
seemingly weight-neutral. She requests that her husband be switched to this new drug. Which of
the following antipsychotic medications is least likely to cause weight gain?
 Incorrect Answer ImageA.Clozapine
 Incorrect Answer ImageB.Olanzapine
 Incorrect Answer ImageC.Quetiapine
 Incorrect Answer ImageD.Risperidone
 Correct Answer ImageE.Ziprasidone

A 36-year-old man with a history of hypertension, hypercholesterolemia, and bipolar disorder


comes to the physician because of a 2-week history of facial twitching and tremors of his hands.
He has also had nausea and diarrhea for the last few days. He has had to take a leave from work
due to his symptoms. He was diagnosed as having hypertension and hypercholesterolemia 6
years ago. His bipolar disorder was diagnosed 1 year ago and his symptoms have responded well
to therapy. Physical examination shows a temperature of 36.2ºC (97.1ºF), blood pressure of
130/70 mm Hg, respiratory rate of 14/min, and heart rate of 50/min. Pupils are equal, round, and
reactive to light. Nystagmus is present. Fundus examination shows no papilledema. There is no
neck rigidity or lymphadenopathy. Lungs are clear to auscultation, with no wheezes or crackles.
Cardiovascular examination shows normal S1 and S2 sounds with no rub or gallop. Abdomen is
soft, nontender, and nondistended. Bowel sounds are active. Irregular coarse tremors are seen in
both upper extremities. Motor strength is normal. There are no sensory deficits. Laboratory
studies show:
Hgb  14.2 g/dL
WBC  12,700/mm3
Platelets  320,000/mm3
Sodium  138 mEq/L
Potassium  4.4 mEq/L
Chloride  102 mEq/L
Bicarbonate  26 mEq/L
BUN  21 mg/dL 
Creatinine  0.9 mg/dL 
Glucose  80 mg/dL
Calcium  9.6 mg/dL
CT scan of the head is normal. Which of the following is the most appropriate diagnostic test? 
 Incorrect Answer ImageA.Fasting lipid profile
 Incorrect Answer ImageB.Glycosylated hemoglobin
 Correct Answer ImageC.Lithium level
 Incorrect Answer ImageD.Nerve conduction studies
 Incorrect Answer ImageE.No further tests are needed
 Incorrect Answer ImageF.Repeat CT scan of the head

A 49-year-old man comes to a psychiatrist for depressive symptoms. He talks about his past
psychiatric problems including alcohol use disorder. He discusses how taking disulfiram has
helped him abstain from alcohol. He once tried to drink while taking disulfiram and ended up
being terribly sick. Which of the following best describes this treatment of alcoholism?
 Correct Answer ImageA.Conditioned avoidance
 Incorrect Answer ImageB.Extinction
 Incorrect Answer ImageC.Flooding
 Incorrect Answer ImageD.Positive reinforcement
 Incorrect Answer ImageE.Reciprocal inhibition

A 64-year-old man is undergoing chemotherapy and has occasional nausea and vomiting, which
are treated with IV prochlorperazine. After several days of therapy, the patient complains that he
feels very restless and agitated and he cannot stop moving his legs. He has never experienced
these symptoms before and is concerned that they were caused by his recent therapy. Which of
the following medications would best treat his symptoms?
 Incorrect Answer ImageA.Chlorpromazine
 Incorrect Answer ImageB.Haloperidol
 Correct Answer ImageC.Lorazepam
 Incorrect Answer ImageD.Paroxetine
 Incorrect Answer ImageE.Sertraline

An 8-year-old boy is brought to the physician for new-onset enuresis. A prior workup to
determine a medical cause was negative. In conversation, it seems that the enuresis started
following a parental argument which led to a brief separation. The boy wets himself at least
twice a week and feels upset about it, refusing to go to friends’ houses for sleepovers. Which of
the following is the most effective first step in the management of enuresis in this child?
 Correct Answer ImageA.Behavioral therapy
 Incorrect Answer ImageB.Desmopressin acetate (DDAVP)
 Incorrect Answer ImageC.Imipramine
 Incorrect Answer ImageD.Insight-oriented psychotherapy
 Incorrect Answer ImageE.Interpersonal therapy

A 28-year-old man is in psychotherapy because he is depressed after his girlfriend left him. In
the initial evaluation sessions, he talks about his accomplishments, and later on about his anger
related to the recent breakup. He complains of restlessness and insomnia. He tells the therapist
that the girlfriend was the most sought after woman in their law school class and says, “She
broke up with me because she could not handle my success.” He tells the therapist that he does
not understand why he feels depressed as he “knows he will be an amazing lawyer” and “he
deserves open access to his professors’ office hours.” When asked about his friendship with
classmates, he says he has few friends because “most of them are envious of my intelligence”.
Which of the following is the most likely diagnosis?
 Incorrect Answer ImageA.Antisocial personality disorder
 Incorrect Answer ImageB.Bipolar disorder
 Incorrect Answer ImageC.Histrionic personality disorder
 Incorrect Answer ImageD.Major depressive disorder
 Correct Answer ImageE.Narcissistic personality disorder

A 50-year-old woman comes to the physician to discuss “a personal matter.” The patient tells the
doctor that she is having a sexual problem. She says that while she has had orgasms in the past,
she cannot reach orgasm with her current sexual partner, a man she has been seeing for a few
weeks. Her history shows that over the last four months, she has had episodes of depressed mood
and lack of interest in activities for which she recently saw a psychiatrist and was put on
medication. She has a past history of high blood pressure and has recently started on a diet and
exercise program to address an elevated lipid panel. She has been taking her prescribed
medications regularly and says her depression and blood pressure are well-managed. Which of
the following medications is most likely to be related to this woman’s sexual problem?
 Incorrect Answer ImageA.Alcohol
 Incorrect Answer ImageB.Angiotensin-converting enzyme (ACE) inhibitor 

 Correct Answer ImageC.Antidepressant


 Incorrect Answer ImageD.Benzodiazepine
 Incorrect Answer ImageE.Beta-blocker

A 65-year-old woman comes to the physician because of poor memory. After her husband died 6
months ago the woman notes that she decreased her church involvement and had less interest in
her hobbies. She reports feeling tired and worried and has difficulty with attention and memory.
She has a history of a major depressive episode approximately 10 years ago, which required
hospitalization. She was treated with an antidepressant for 2 years until it was tapered and
discontinued. She currently denies depressed mood although her shoulders are hunched forward
and she has a sad expression. She states that her mother died of Alzheimer disease and that she
finds her memory loss particularly worrisome in light of this family history. Which of the
following is most likely to characterize this patient if she has pseudodementia of depression
rather than true dementia?
 Incorrect Answer ImageA.The patient appears unconcerned about the memory loss
 Incorrect Answer ImageB.The patient has more difficulty with recent memory than
remote memory
 Correct Answer ImageC.The patient seems concerned about the memory loss
 Incorrect Answer ImageD.The patient tends to conceal the memory difficulty
 Incorrect Answer ImageE.The patient tries hard to answer questions about orientation and
registration

The mother of a 35-year-old man calls her primary care physician for advice on a drug treatment
program for her son. She states that he has lived with her since the loss of his job and apartment
4 months ago. She reports that he goes out on weekends and uses drugs heavily. When he returns
home, usually on Monday mornings, he sleeps for the entire day and seems depressed. He is also
very irritable and anxious over the course of the next day. She believes that he uses the drug
throughout the week, but with more intense binges on the weekends and then becomes depressed
when he does not use the drug. On several occasions, he has called her from the local psychiatric
emergency center after admitting himself for suicidal ideation days after the use of the drug.
Which of the following drugs is most likely responsible for this man's withdrawal state?
 Incorrect Answer ImageA.Alcohol
 Incorrect Answer ImageB.Benzodiazepines
 Incorrect Answer ImageC.Cannabis
 Correct Answer ImageD.Cocaine
 Incorrect Answer ImageE.Heroin

An 8-year-old boy is brought to the physician by his mother and father. The mother describes
several episodes of sleep troubles over the past few weeks. Her son has awakened screaming
within a few hours of going to sleep, sits upright in bed, and looks frightened. She has tried to
comfort him at these times, but he does not respond to her questions. The episodes last
approximately 20 minutes. In the morning, the child wakes up very tired and says that he doesn't
remember the episode. Physical examination and laboratory studies are within normal limits.
Which of the following is the most likely diagnosis? 
 Incorrect Answer ImageA.Narcolepsy
 Incorrect Answer ImageB.Nightmare disorder
 Incorrect Answer ImageC.Seizure disorder
 Correct Answer ImageD.Sleep terror disorder
 Incorrect Answer ImageE.Sleepwalking disorde

Three days after a successful elective cosmetic surgery, a 42-year-old woman has a seizure. She
is confused, agitated, diaphoretic, and has a tremor and a temperature of 100.4ºF (38ºC). She
states that she was agitated because she could hear people in the corridor saying bad things about
her when there was no one there. She has a history of gastritis. Which of the following aspects of
the patient's history would be most likely to point to the diagnosis?
 Correct Answer ImageA.Alcohol use
 Incorrect Answer ImageB.Family history of malignant hyperthermia
 Incorrect Answer ImageC.History of seizure disorder
 Incorrect Answer ImageD.Past LSD use
 Incorrect Answer ImageE.Schizophrenia
A 42-year-old married man comes to the physician for a routine health maintenance examination.
He reports that he was diagnosed in his late teens with schizophrenia, after experiencing
persecutory delusions, auditory hallucinations, and poor school and occupational functioning. He
reports that he has a number of stressors at this time, including the death of his father and
problems with his wife. He has a family psychiatric history significant for major depressive
disorder on his mother's side of the family. He is currently stable on aripiprazole, 15 mg/d, and
he works in a factory. Which of the following features of this patient is associated with a poor
prognosis? 
 Incorrect Answer ImageA.Family history of a mood disorder
 Incorrect Answer ImageB.History of positive symptoms
 Correct Answer ImageC.Onset of symptoms at an early age
 Incorrect Answer ImageD.Precipitating stressors at the start of psychotic symptoms
 Incorrect Answer ImageE.Quick onset of symptoms
 Incorrect Answer ImageF.Working in a factory

A 46-year-old African American man who has a history of schizoaffective disorder is admitted
for acute psychosis. He has auditory hallucinations, paranoia, depression with suicidal ideation,
anhedonia, blunted affect, increased appetite, a 10 lb (4.5 kg) weight gain over the past month,
and increasing somnolence. The patient is stabilized in the hospital with haloperidol. Which of
the following is the best atypical antipsychotic medication to prescribe for this patient for long-
term management?
 Incorrect Answer ImageA.Clozapine
 Incorrect Answer ImageB.Olanzapine
 Incorrect Answer ImageC.Quetiapine
 Incorrect Answer ImageD.Risperidone
 Correct Answer ImageE.Ziprasidone

 mother brings her 9-year-old son to the physician. She tells the doctor that the boy has presented
several symptoms since the age of 6. He does not pay attention to his teachers or parents and has
trouble in school and at home because of his inability to stay quiet and still. Last week the
teacher referred him to the school psychologist for counseling and management of a learning
disorder. The mother is concerned also about her 4-year-old who is also having difficulty
interacting with other children at the local daycare, is unable to sit through cartoons and meals,
and is increasingly irritable. What is the most appropriate statement for the physician to say to
the mother concerning her child's problem?
 Incorrect Answer ImageA.“It is a benign problem and is self-limited to childhood”
 Incorrect Answer ImageB.“The child has a 90% risk of developing antisocial personality
disorder”
 Incorrect Answer ImageC.“The child has an increased risk of developing schizophrenia”
 Incorrect Answer ImageD.“The child’s problem is related to the ingestion of sugar and
artificial coloring agents”
 Correct Answer ImageE.“The child’s problem is likely to improve as he gets older”
An 80-year-old woman comes to the physician because of difficulty sleeping for the past few
months. She reports taking 45 to 60 minutes to fall asleep, frequent nighttime awakenings to use
the bathroom and stretch her legs, and sleeping on average 5 to 6 hours per night. She finds that
she needs to take a short nap during the day. She reports that her husband of 55 years died 1 year
ago, and she misses him very much. She reports that she lives alone and has some mild memory
loss which does not interfere with her function. She denies changes in her appetite or weight. She
reports an active social life with friends from her retirement community and her family. Her
physical examination is unremarkable. What is the most likely diagnosis? 
 Correct Answer ImageA.Age-related sleep changes
 Incorrect Answer ImageB.Early Alzheimer disease
 Incorrect Answer ImageC.Insomnia disorder
 Incorrect Answer ImageD.Major depressive disorder
 Incorrect Answer ImageE.Restless legs syndrome

A 32-year-old woman comes to the physician because she cannot enjoy her social life. She states
that over the past 7 months she has experienced extreme fatigue, muscle tension, and irritability.
She has difficulties falling asleep, partly because she is unable to control thoughts that something
bad is going to happen to her husband and children. In the past month, she has had episodes of
shortness of breath, dizziness, and restlessness, and has avoided going to work because she
“couldn't take the stress.” Her physical examination and laboratory studies, as well as her
electrocardiogram, are unremarkable. Which of the following is the most appropriate
pharmacotherapy?
 Incorrect Answer ImageA.Alprazolam
 Incorrect Answer ImageB.Clomipramine
 Incorrect Answer ImageC.Isocarboxazid
 Incorrect Answer ImageD.Propranolol

 Correct Answer ImageE.Venlafaxine

A 26-year-old graduate student comes to the physician because she feels depressed, suicidal, and
tense, and admits to "hurting herself" for many years. She denies the intention to kill herself.
Rather, she explains that usually, it is therapeutic to feel the pain she inflicts on herself, that it
helps her release tension. She has several scars on her arms that were caused by superficial cuts
made with a razor blade. She has two new superficial cuts from the night before. She believes
that her current "crisis state" is related to a recent breakup with a man. She has had several
stormy relationships with men that typically end poorly. However, she is usually unhappy when
she is alone, so she goes directly into another relationship. Often she feels empty and bored, yet
does not have the initiative to change things about herself. She frequently uses drugs to elevate
her mood. She attends her classes regularly and usually can prepare for examinations. Which of
the following is most likely to be a comorbid problem for this patient? 
 Incorrect Answer ImageA.Adjustment disorder
 Incorrect Answer ImageB.Antisocial personality disorder
 Incorrect Answer ImageC.Dependent personality disorder
 Correct Answer ImageD.Substance use disorder
 Incorrect Answer ImageE.Narcissistic personality disorder

A 7-year-old boy is brought to the physician because his teacher requested a formal evaluation of
the child's behavior. The boy likes going to school and does his homework assignments
regularly. The teacher has noticed that the child talks a lot in school but can be quiet when
necessary. When the teacher asks questions, he sometimes shouts out the answer, without
waiting to be called on. His parents note that he can play for hours without tiring and has been
very active ever since they can remember. He regularly sits still while watching his favorite TV
shows and plays uninterruptedly for hours with his 4-year-old brother. Which of the following is
the best description of this child? 
 Incorrect Answer ImageA.Attention deficit/hyperactivity disorder (ADHD)
 Incorrect Answer ImageB.Autism spectrum disorder (ASD)
 Incorrect Answer ImageC.Conduct disorder
 Correct Answer ImageD.Normal
 Incorrect Answer ImageE.Oppositional defiant disorder

A 9-year-old boy is brought to the physician because of poor school performance. The parents
stress that the child is very hard-working, and go on to add that the boy may actually "work too
hard." He will turn work in to the teacher only if the margins of his writing are perfectly aligned.
Additionally, the boy makes a point of washing his hands after every paragraph that he writes.
Standardized testing shows that he is above average intelligence and has no specific weaknesses
in any academic discipline. He has many friends and is well-behaved and friendly in social and
academic settings. On mental status examination, he is a well-related and friendly child who
appears his stated age. He appears happy and denies any suicidal or homicidal intent, ideation, or
plan. When asked why he must perform his schoolwork in this manner, he replies, ”I don‘t know
why. I know the other kids don’t, but I just have to." There is no evidence of auditory
hallucinations or disturbances in his thought process. Which of the following is the most
appropriate pharmacologic therapy?
 Incorrect Answer ImageA.Clonazepam

 Correct Answer ImageB.Fluoxetine


 Incorrect Answer ImageC.Imipramine
 Incorrect Answer ImageD.Isocarboxazid
 Incorrect Answer ImageE.Methylphenidate

A 59-year-old African American woman, who has been HIV-positive for the past 12 years, is
brought to the emergency department after an episode of combativeness at home. Her family
reports that over the past couple of days she has been talking “out of her head,” stating that her
daughter-in-law has been putting poison into the food. She is admitted to the hospital but refuses
to eat or take medication, stating that the physicians are also against her. Her last CD4 count,
done 6 months ago, was 85/mm3. Physical and neurologic examinations are unremarkable. The
current workup for HIV-related infections is negative, and laboratory tests are otherwise
unremarkable. On Folstein's Mini-Mental Status Examination she scores 21/30, missing on
delayed recall, three-stage command, writing, and copying, as well as orientation to year. Which
of the following is the most likely diagnosis?
 Incorrect Answer ImageA.Brief psychotic disorder
 Incorrect Answer ImageB.Delirium
 Incorrect Answer ImageC.Delusional disorder
 Correct Answer ImageD.Neurocognitive disorder due to HIV infection
 Incorrect Answer ImageE.Schizophrenia

A 42-year-old man with a history of bipolar disorder, gastrointestinal reflux disease (GERD),
and chronic obstructive pulmonary disease (COPD) comes to the physician because of nausea,
vomiting, diarrhea, coarse tremors, and muscle weakness. Two years prior to this, the patient was
diagnosed with bipolar disorder, and lithium was started. His medications now include lithium,
pantoprazole, albuterol and ipratropium inhalers. He has a 20-pack-year history of smoking. His
pulse is 80/min and blood pressure is 120/60 mm Hg. Occasional muscular fasciculations are
seen. Pupils are equal, round, and reactive. Lungs are clear to auscultation. Cardiovascular
examination shows a normal S1 and S2 with no rub or gallops. Abdomen is soft, nontender, and
non-distended. Bowel sounds are present. Extremities show no pitting edema. Laboratory studies
show:
Hb 13 mg/dL
Leukocytes 10,000/mm3
Creatinine 0.9 mg/dL
Urea 12 mg/dL
Na 138 mEq/L
K 4 mEq/L
Lithium 1.5 mEq/L 
Which of the following is the most appropriate next step in the management of this patient? 
 Incorrect Answer ImageA.Administer propranolol
 Correct Answer ImageB.Decrease the dosage of lithium
 Incorrect Answer ImageC.Hemodialysis
 Incorrect Answer ImageD.Intravenous fluids only
 Incorrect Answer ImageE.Observation
 Incorrect Answer ImageF.Repeat lithium levels in 24 hours

A 28-year-old pregnant woman is admitted to the labor and delivery floor because of
hypertension and mild edema. During the hospital stay, she frequently argues with the day staff,
insists on going outside to smoke cigarettes, and refuses to obey the hospital rules. She gets
along well with the chief resident and states that he is the only one who understands her. She also
states that the rest of the staff does not like her, so she avoids calling them for her needs. The
staff can no longer handle her attitude and they complain to the head nurse about the resident
being too permissive. The patient then gets more outraged at the nurses, accusing them of being
rigid, cold, and punitive. This patient’s behavior is most consistent with which of the following
personality disorders? 
 Incorrect Answer ImageA.Antisocial personality disorder
 Incorrect Answer ImageB.Avoidant personality disorder
 Correct Answer ImageC.Borderline personality disorder
 Incorrect Answer ImageD.Histrionic personality disorder
 Incorrect Answer ImageE.Schizotypal personality disorder

A 21-year-old man drops out of college because of poor performance. On questioning by the
physician, he explains that he has been doing poorly because he has not been going to class. He
has been missing class because he is afraid of having another episode of sudden discomfort in
which he develops a fear of losing control. He had four such attacks of anxiety in the last month.
He became so afraid that he could not concentrate in class and is no longer willing to go to class
because he fears that he would not be able to get out of the classroom if he had an episode.
Additionally, he has become increasingly uncomfortable with being outside of his house without
the presence of someone else. Which of the following is the most likely diagnosis? 
 Incorrect Answer ImageA.ADHD (attention-deficit hyperactivity disorder)
 Correct Answer ImageB.Agoraphobia
 Incorrect Answer ImageC.Conversion disorder
 Incorrect Answer ImageD.Social anxiety disorder
 Incorrect Answer ImageE.Somatic symptom disorder

 35-year-old woman is brought to the emergency department because she is stiff and unable to
swallow or talk. She is confused and tremulous, and her family is concerned because she had a
high fever the previous night and they think she has developed an infection. Her past medical
history is significant for bipolar disorder for 10 years. She was recently given an antipsychotic in
the hospital for an acute manic episode and, at the same time, was started on lithium. In the past
couple of days, her family reports that she has appeared "strange." Her temperature is 39.5ºC
(103.1ºF), blood pressure is 160/108 mm Hg, pulse is 124/min, and respirations are 24/min. She
appears tremulous, diaphoretic, and confused. She has increased tone in her neck and
extremities. Her leukocyte count is 21,200/mm3 and the serum creatinine phosphokinase is 5,238
U/L. A thorough evaluation for infection is negative. Which of the following should be the next
step in management?
 Incorrect Answer ImageA.An antibiotic for meningitis
 Correct Answer ImageB.Dantrolene for neuroleptic malignant syndrome
 Incorrect Answer ImageC.Dialysis for lithium toxicity
 Incorrect Answer ImageD.Halothane for malignant hyperthermia
 Incorrect Answer ImageE.Succinylcholine for malignant hyperthermia

A suspected victim of sexual assault is brought to the emergency department. She appears visibly
shaken and it takes the emergency physician several minutes to develop rapport with her. When
she finally begins to describe the attack, she abruptly stops talking and does not say anything for
several seconds. After the physician waits for a few more seconds, which of the following is the
most appropriate statement to facilitate further history-taking?
 Incorrect Answer ImageA."Are you suicidal?"
 Correct Answer ImageB."Please continue."
 Incorrect Answer ImageC."We both know that what you're going through is serious."
 Incorrect Answer ImageD."You seem reluctant to talk about this."
 Incorrect Answer ImageE."You're too ashamed to speak to me now, aren't you?"

A 28-year-old college student comes to the emergency department because of skin rashes that he
developed 2 days ago. The patient was taken to a psychiatrist 5 weeks ago because he was very
loud and talkative. He stated that prior to that psychiatry visit, he had not slept for the previous 4
days because he had been making frequent trips to the library, but had been able to stay up all
night to study without a problem. He knew he was going to excel in his exams, because he
recently had gained some power from God. The psychiatrist started him on an anticonvulsant
that the patient was told would help with his mood swings. His past history is significant for
chickenpox at the age of 10. On examination today, he is noted to have atypical targetoid lesions
with focal confluence and detachment of the epithelium over about 10% of his body surface.
Which of the following medications did this patient most likely take?
 Incorrect Answer ImageA.Clonazepam
 Incorrect Answer ImageB.Gabapentin
 Correct Answer ImageC.Lamotrigine
 Incorrect Answer ImageD.Lithium
 Incorrect Answer ImageE.Valproate

A 73-year-old Caucasian man with benign prostatic hyperplasia and no past psychiatric history
comes to the physician for a routine health maintenance examination. His physical examination
and routine laboratory studies are unremarkable. The patient's wife died approximately 2 months
ago after an extended course of colon cancer, and since then he has been “sad and lonely.” He
misses his wife and states “Sometimes I wish I was with her.” He is troubled by the feeling that
he can actually hear his wife calling out his name when he is alone in the house that the couple
shared. The patient has recently resumed meeting his friends for coffee in the morning and has
renewed his magazine subscriptions. Which of the following is the most appropriate next step in
management?
 Incorrect Answer ImageA.Hospitalize the patient because he is suicidal
 Incorrect Answer ImageB.Prescribe a short course of antipsychotic medication
 Correct Answer ImageC.Reassure the patient that he is experiencing a normal grief
reaction
 Incorrect Answer ImageD.Recommend that the patient have neuropsychological testing
 Incorrect Answer ImageE.Send the patient for a psychiatric evaluation

A woman with severe depression attends weekly psychotherapy sessions and takes fluoxetine.
She is doing well and is beginning to feel better, but she becomes furious when her psychiatrist,
who is about to leave for vacation, is not willing to reveal where he is going. She angrily says
that he is just like her father, who always had his own "own life" outside the family. Which of
the following best describes this patient's reaction?
 Incorrect Answer ImageA.Acting out
 Incorrect Answer ImageB.Countertransference
 Incorrect Answer ImageC.Identification
 Incorrect Answer ImageD.Projection
 Correct Answer ImageE.Transference

A 38-year-old woman comes to the physician because of a 3-month history of fatigue and
insomnia. She describes feeling tired most of the day, having difficulty falling asleep as well as
waking up too early, poor appetite, frequent crying spells, poor concentration, and a recent loss
of interest in her hobbies. Her past medical history is significant for a seizure disorder since
childhood. Physical examination and routine laboratory studies, including thyroid-stimulating
hormone, are within normal limits. Which of the following pharmacologic interventions is most
likely contraindicated in this patient?
 Incorrect Answer ImageA.Alprazolam
 Correct Answer ImageB.Bupropion
 Incorrect Answer ImageC.Imipramine
 Incorrect Answer ImageD.Paroxetine
 Incorrect Answer ImageE.Phenelzine

A 34-year-old man is brought to the physician by a friend because he has been very loud and
talkative over the past 2 weeks. The man states that he has not slept for the past 2 days because
he has been working to establish a catering service, for which he has been buying kitchen
utensils, baking supplies, and recipe books. He has also been “sampling wines from around the
world” so that he can become a wine expert for his new business. Physical examination is
unremarkable. Laboratory studies show:
Hgb 14.2 g/dL
WBC 8,700/mm3
Platelets 320,000/mm3
Sodium  138 mEq/L
Potassium  4.4 mEq/L
Chloride  102 mEq/L
Bicarbonate  26 mEq/L
BUN  40 mg/dL
Creatinine  2.5 mg/dL
Glucose  80 mg/dL
Calcium  9.6 mg/dL
Which of the following is the most appropriate treatment for this man's condition? 
 Incorrect Answer ImageA.Buspirone
 Incorrect Answer ImageB.Carbamazepine
 Incorrect Answer ImageC.Lamotrigine

 Incorrect Answer ImageD.Lithium


 Correct Answer ImageE.Valproate
A 20-year-old woman suddenly develops the loss of voluntary movement of her right upper
extremity after witnessing a terrible accident. She adamantly states that she is unable to move her
arm. She has full strength in all extremities, except her right upper extremity. Reflexes are
symmetric and 2+ in all extremities, including the right upper extremity. Muscle tone is also
symmetric in all extremities. Laboratory studies, including B12 and folate levels, and thyroid
function tests, are normal. MRI of the head and cervical spine are unremarkable. Which of the
following is the most likely diagnosis?
 Incorrect Answer ImageA.Body dysmorphic disorder
 Correct Answer ImageB.Conversion disorder
 Incorrect Answer ImageC.Illness anxiety disorder
 Incorrect Answer ImageD.Factitious disorder
 Incorrect Answer ImageE.Malingering

A 52-year-old male executive was laid off from his job as a mortgage banker. He had been with
the company for 20 years and was shocked and angered at being let go. He remained calm and
stoic when his boss gave him the news and thanked him for a wonderful career. He arrived home
that evening; his wife asked him about his day and his children asked him for help with their
homework. He criticized his wife for doing a poor job making dinner, threw his food away
before tasting it, and told his children they need to do their own homework. Which of the
following defense mechanisms is this man using? 
 Incorrect Answer ImageA.Acting out
 Correct Answer ImageB.Displacement
 Incorrect Answer ImageC.Rationalization
 Incorrect Answer ImageD.Somatization
 Incorrect Answer ImageE.Sublimation

A 41-year-old man with a long history of schizophrenia has been on the inpatient unit for almost
2 weeks. This is his third admission in the past 6 months, and each time he seems to be less
responsive to treatment. In the past, he has been on typical antipsychotics. Recently he has been
taking an atypical agent, risperidone, with limited success. His family is supportive and makes
sure he takes his medication. Given his most recent poor response, which of the following is the
most appropriate next step in treatment? 
 Correct Answer ImageA.Clozapine
 Incorrect Answer ImageB.Droperidol
 Incorrect Answer ImageC.Electroconvulsive treatment (ECT)
 Incorrect Answer ImageD.Lamotrigine
 Incorrect Answer ImageE.Paliperidone

A 41-year-old man with a long history of schizophrenia has been on the inpatient unit for almost
2 weeks. This is his third admission in the past 6 months, and each time he seems to be less
responsive to treatment. In the past, he has been on typical antipsychotics. Recently he has been
taking an atypical agent, risperidone, with limited success. His family is supportive and makes
sure he takes his medication. Given his most recent poor response, which of the following is the
most appropriate next step in treatment? 
 Correct Answer ImageA.Clozapine
 Incorrect Answer ImageB.Droperidol
 Incorrect Answer ImageC.Electroconvulsive treatment (ECT)
 Incorrect Answer ImageD.Lamotrigine
 Incorrect Answer ImageE.Paliperidone

An 86-year-old Caucasian woman is brought to the physician by her daughter. The patient has a
past medical history of gout, hypothyroidism, and hyperlipidemia. Her daughter is concerned
about her because the patient has been forgetting the names of people she has recently met and
restaurants she has recently visited and has reported misplacing items such as her keys. The
patient lives alone and is able to take care of her daily needs, such as cooking, cleaning, paying
the bills, and daily hygiene. She denies difficulty with attention or orientation. She has been
progressively falling asleep earlier and awakening earlier but denies trouble with sleep. She
denies suicidal ideation, feelings of guilt, or difficulty with appetite. Which of the following is
the most likely explanation for this woman’s forgetfulness?
 Correct Answer ImageA.Age-related cognitive change
 Incorrect Answer ImageB.Alzheimer disease
 Incorrect Answer ImageC.Delirium
 Incorrect Answer ImageD.Major depressive disorder
 Incorrect Answer ImageE.Major neurocognitive disorder due to vascular disease

A 29-year-old Caucasian man is brought to the emergency department by a friend who is


concerned about his recent behavior. The patient is alert and reports increased energy, poor
sleep, and a "great mood" for 2 days. His blood pressure is 130/90 mm Hg and his pulse is
100/min. On physical examination, he is restless, his mouth is dry, his pupils are dilated, and he
has coordination difficulties. Which of the following is the most likely cause of these findings? 
 Incorrect Answer ImageA.Acute manic episode
 Correct Answer ImageB.Amphetamine use
 Incorrect Answer ImageC.Cannabis use
 Incorrect Answer ImageD.Inhalant use
 Incorrect Answer ImageE.Phencyclidine (PCP) use

A 63-year-old white man who recently retired after working as a plumber for over 30 years
comes to the physician, saying that he has been feeling “very down” lately and has had a
decreased appetite and a loss of interest in activities that used to give him pleasure. He is a
smoker, drinks no alcohol, and is being treated by his family physician for moderate essential
hypertension. His physical examination is unchanged from his previous visits. Which of the
following is the most appropriate next step in management? 
 Incorrect Answer ImageA.Discuss activities that will help him enjoy his retirement
 Incorrect Answer ImageB.Order a thyroid-stimulating hormone (TSH) level
 Incorrect Answer ImageC.Prescribe an antidepressant
 Correct Answer ImageD.Review the patient's medication history
 Incorrect Answer ImageE.Schedule a psychiatry consult

At the request of his teacher, a 10-year-old boy is brought to the physician by his parents. The
parents tell the doctor that during the past few months, the teacher has noticed that the child has
become more fidgety in class and at times coughs repeatedly. He also makes sudden arm
movements to touch his peers in the middle of conversations. He also blinks often, and this has
recently worsened. The boy says he has a feeling of tightness that is relieved by arm extension,
and a burning feeling in his eyes before an eye blink. The main reason for the teacher calling the
parents and recommending a medical evaluation is that the boy's sudden screams distract and
frighten the other students in class. He seems very upset when the screaming occurs and admits
that he is aware that other children make fun of him and avoid him because of it. He states that
he tries, but is unable to control it. He is successful academically and is very neat, and there are
no other behavioral or emotional problems. Which of the following is most closely associated
with this patient's condition? 
 Incorrect Answer ImageA.Absence seizure
 Correct Answer ImageB.Attention deficit/hyperactivity disorder (ADHD)
 Incorrect Answer ImageC.Autism spectrum disorder (ASD)
 Incorrect Answer ImageD.Huntington disease
 Incorrect Answer ImageE.Oppositional defiant disorder

A 63-year-old man with diabetes presents to the hospital with 10/10 chest pain radiating to his
left arm. An electrocardiogram reveals an ST-elevation myocardial infarction, and he is sent to
the catheterization lab. A stenosis in the mid-left anterior descending artery is stented. After the
procedure the patient develops hypotension, and has to be intubated and sent to the cardiac
intensive care unit. A bedside echocardiogram shows an ejection fraction of 10%, and he is
found to have pulmonary edema. The decision is made to place an intra-aortic balloon pump,
which stabilizes his blood pressure. Over the next week, there is difficulty weaning him from the
ventilator. His family decides to continue the current level of care and is by his bedside well after
visiting hours are over in the intensive care unit. One evening, the patient's son is seen berating
the nurse and then asks to speak to the doctor. The son tells the doctor that the nurse is not
keeping the patient's blood pressure well-titrated, and he fears that his father will have another
heart attack. He feels that he needs to monitor the nurse's activities all night to feel comfortable.
The nurse feels that her job is being compromised and that she cannot take care of the patient
adequately with his son constantly contradicting her and challenging her decisions. What is the
most appropriate response to the son? 
 Incorrect Answer ImageA.“Do not worry. The medical staff knows what the appropriate
blood pressure is for your father.”
 Correct Answer ImageB."Tell me more about your frustrations with your father's care."
 Incorrect Answer ImageC."Try to be patient with the nursing staff. They are competent
professionals who are trying their best."
 Incorrect Answer ImageD."You really need to leave the intensive care unit when visiting
hours are over."
 Incorrect Answer ImageE."Your father is very sick and could die any minute. I
recommend that you spend his last hours peacefully with him."

A 21-year-old male college student comes to the physician for psychiatric evaluation. He reports
that for the past 2 years he has had recurrent urges to touch unfamiliar women inappropriately.
He waits for rush hour when the subways are crowded to go home after school. He often rubs his
genitals against the thighs and buttocks of women in the crowded subway car and gets sexual
pleasure from this. He is disturbed by this behavior but cannot stop it and is afraid he will get
caught. Which of the following is the most appropriate diagnosis?
 Incorrect Answer ImageA.Exhibitionistic disorder
 Incorrect Answer ImageB.Fetishistic disorder
 Correct Answer ImageC.Frotteuristic disorder
 Incorrect Answer ImageD.Sexual masochism disorder
 Incorrect Answer ImageE.Sexual sadism disorder

A 35-year-old woman is referred by her dermatologist to a psychiatrist. For the past several
years, she has had a need to wash her hands after shaking hands with someone. She experiences
intrusive thoughts about the transfer of germs and getting sick when shaking hands, which causes
her anxiety. Since she is in the public relations business, hiding her problem has become
increasingly difficult. She knows that if she does not wash her hands after a handshake, she
becomes nervous and tense. Which of the following is the most appropriate treatment?
 Incorrect Answer ImageA.Buspirone
 Incorrect Answer ImageB.Clonazepam
 Incorrect Answer ImageC.Fenfluramine
 Correct Answer ImageD.Fluvoxamine
 Incorrect Answer ImageE.Risperidone

An 18-year-old girl with no known past medical history comes to the university health center
because of fatigue. Since the student transferred to this school last month, she says that she feels
as if she has barely enough energy to finish her classwork, has been having tension headaches,
and has been feeling anxious about how these problems may affect her academic progress.
Physical examination and laboratory studies, including complete blood count, thyroid studies,
serum chemistries, and testing for mononucleosis, are all within normal limits. When discussing
the results of these studies she admits to feeling homesick, and states, “I‘m worried that I’m not
good enough to make it here.” Sleep and appetite are normal. Despite these problems, she has
made many friends and goes out most weekends. She does not drink or use any illicit drugs.
Which of the following is the most likely diagnosis? 
 Correct Answer ImageA.Adjustment disorder
 Incorrect Answer ImageB.Bipolar disorder
 Incorrect Answer ImageC.Generalized anxiety disorder
 Incorrect Answer ImageD.Major depressive disorder
 Incorrect Answer ImageE.Persistent depressive disorder
A 25-year-old law school student comes to the physician for a follow-up visit 2 weeks after
discharge from the psychiatric inpatient unit where she was diagnosed and treated for her first
episode of schizophrenia. She is compliant with her medication (olanzapine) and seems to be
tolerating it well. She no longer seems to be having psychotic symptoms, but she also is not
socializing with friends or relatives, broke up with her boyfriend approximately a week ago, and
is not interested in returning to school. Her parents are concerned that she may have a hard time
dealing with her illness. Which of the following is the most appropriate next step in
management? 
 Correct Answer ImageA.Assess the patient's suicide risk
 Incorrect Answer ImageB.Check the patient's thyroid stimulating hormone (TSH) level
 Incorrect Answer ImageC.Ensure compliance with medication
 Incorrect Answer ImageD.Provide education about disease and medication
 Incorrect Answer ImageE.Suggest that the patient go back to school and resume daily
activities

A 49-year-old man is recuperating in the hospital after having sustained a recent cerebrovascular
accident that damaged part of his right temporal lobe. His wife and children are at his bedside
day and night. They want to be active participants in his rehabilitation so they specifically ask
the attending physician what they can expect from him behaviorally in the future. Once the
patient has recuperated from the immediate effects of his stroke, to which of the following
psychiatric disorders will he be most predisposed?
 Incorrect Answer ImageA.Conversion disorder
 Correct Answer ImageB.Major depressive disorder
 Incorrect Answer ImageC.Narcolepsy
 Incorrect Answer ImageD.Schizophrenia
 Incorrect Answer ImageE.Substance use disorder

A 19-year-old student is admitted to the psychiatry inpatient unit with the working diagnosis of
psychosis. Because of combativeness and threats to others, he is put in seclusion. At first, he
refuses to take medication by mouth. However, after attacking a nurse, he is given an
antipsychotic medication intramuscularly as a chemical restraint for the safety of himself and
others on 2 occasions. After 12 hours, he develops a painful contraction of a group of muscles in
the neck and twitching of the mouth and face. Which of the following is the most appropriate
treatment?
 Incorrect Answer ImageA.Clozapine
 Incorrect Answer ImageB.Dantrolene
 Incorrect Answer ImageC.Diazepam
 Correct Answer ImageD.Diphenhydramine
 Incorrect Answer ImageE.Hydralazine

A 23-year-old man is brought to the emergency department for evaluation of altered mental
status and fever. His friend is with him and says that the man has been healthy recently but saw a
doctor several days ago for complaints of "feeling really awful." He was started on a new
medication, but the friend does not know the name of it. He has a temperature of 40ºC (104.1ºF).
Physical examination shows the patient is obtunded and unable to respond coherently to
questioning. He has rigidity in all of his extremities. Laboratory studies show his creatinine
phosphokinase level is 8,000 U/L. Which of the following is the most likely diagnosis?
 Incorrect Answer ImageA.Acute dystonic reaction
 Incorrect Answer ImageB.Bacterial meningitis
 Incorrect Answer ImageC.Metabolic encephalopathy
 Correct Answer ImageD.Neuroleptic malignant syndrome
 Incorrect Answer ImageE.Serotonin syndrome

 32-year-old man who works as a salesman and has a history of panic disorder returns to his
psychiatrist after several weeks of treatment with paroxetine. The patient tells the doctor that
while he has noted significant improvement in his panic disorder symptoms, he still has anxiety
when he is put into social situations in which he has to speak in public or become the center of
attention. He has been taking the paroxetine as prescribed and has not missed any doses. Given
that he had some improvement on his current medication regimen, which of the following would
be the most appropriate next step in management? 
 Incorrect Answer ImageA.Benzodiazepines
 Correct Answer ImageB.Cognitive behavior therapy
 Incorrect Answer ImageC.Medication change
 Incorrect Answer ImageD.Psychoanalysis
 Incorrect Answer ImageE.Seeing the patient more often for supportive psychotherapy

A 22-year-old woman with anorexia nervosa and a history of depression moves to a new city and
goes to a local psychiatrist to establish care in her new location. She tells the doctor that she has
been on the same dose of bupropion since she was 18 years old, and that her depression has been
stable, but that she has lost 4.5 kg (10 lb) over the last month and weighs only 41 kg (90 lb) now
with a 165 cm (5 ft 5 in). The physician is concerned and explains that there is a complication
that she must be aware of if she is going to continue with this medication. Which of the
following complications is the physician most likely to be concerned about?
 Incorrect Answer ImageA.Cardiac arrhythmias
 Incorrect Answer ImageB.Decreased renal function
 Incorrect Answer ImageC.Illness anxiety disorder
 Incorrect Answer ImageD.Osteoporosis
 Correct Answer ImageE.Seizures

A 21-year-old comes to the physician because he feels very anxious around people and fears that
something will happen to him because he has noticed other people watching him and talking
about him. He has difficulty focusing during class, thinking that his peers are watching his every
move. He has recently dropped out of classes and taken a medical leave of absence. The student
vehemently denies using any drugs because he heard stories that they make people "go crazy."
These specific symptoms have been present over the past 3 months. He usually stays at home in
his room and has recently started covering the mirrors in the house. He occasionally mumbles
things to himself. The student denies any other symptoms, and his physical examination and
laboratory studies are within normal limits. During the interview, his responses are short but
organized and goal-directed and he has a restricted affect. Which of the following is the most
likely diagnosis? 
 Incorrect Answer ImageA.Avoidant personality disorder
 Incorrect Answer ImageB.Brief psychotic disorder
 Incorrect Answer ImageC.Delusional disorder
 Incorrect Answer ImageD.Paranoid personality disorder
 Incorrect Answer ImageE.Schizophrenia
 Correct Answer ImageF.Schizophreniform disorder

An 8-year-old girl is admitted to the neurologic unit for the evaluation of fainting spells that have
been occurring for the past few weeks. The neurologist wants to have the patient monitored in
the hospital for the possibility of a seizure disorder. The physical examination is unremarkable.
Laboratory studies show mildly low serum glucose. After 2 days of video-EEG monitoring, no
seizures are found, no fainting spells have occurred, and the neurologist comes by to order the
discharge. The girl's mother is very upset and insists on a more extensive workup in the hospital
to make sure that nothing is wrong with her child. She threatens to call the hospital legal
department if she does not get her way. What is the most likely diagnosis? 
 Incorrect Answer ImageA.Diabetes mellitus
 Correct Answer ImageB.Factitious disorder imposed on another
 Incorrect Answer ImageC.Hypoglycemia
 Incorrect Answer ImageD.Malingering

 Incorrect Answer ImageE.Undiagnosed seizure disorder

A 35-year-old man comes to the emergency department because of a cough. His PO2 is found to
be 83 mm Hg and he is hospitalized. On the second day in the hospital, the patient develops
generalized muscle aches and abdominal cramps with loose stools. He denies abdominal bloating
and does not have blood or mucus in his stool. He reports chills but does not have a fever. He
also has a clear nasal discharge. Examination is unremarkable except for dilated pupils. Which of
the following is the most appropriate agent for treating this man's autonomic symptoms?
 Incorrect Answer ImageA.Alprazolam
 Incorrect Answer ImageB.Clonazepam
 Correct Answer ImageC.Diphenhydramine
 Incorrect Answer ImageD.Flumazenil
 Incorrect Answer ImageE.Methylphenidate

A 26-year-old man comes to the physician because he has been more and more afraid to leave
his house because he has been having frequent episodes of sudden discomfort without warning.
During these episodes he develops a fear of losing control. He is worried that he will have an
episode at some time while he is in public. He has no other medical conditions. Physical
examination is unremarkable. Which of the following is the most appropriate initial
pharmacotherapy for this patient?
1.
 Incorrect Answer ImageA.Buspirone
 Incorrect Answer ImageB.Chlorpromazine
 Correct Answer ImageC.Citalopram
 Incorrect Answer ImageD.Mirtazapine
 Incorrect Answer ImageE.Trifluoperazine

A 20-year-old Caucasian man is brought to the emergency room by 2 friends on a Sunday


morning after a fraternity party the night before. His friends note that since the party he has been
very belligerent, agitated, and loud. They suspect that he may have used drugs at the party. His
temperature is 37ºC (98.6ºF), blood pressure is 145/95 mm Hg, pulse is 105/min, and
respirations are 20/min. Physical examination shows slurred speech, unsteady gait, and
nystagmus. The patient appears to be responding to auditory hallucinations. Which of the
following most likely caused this patient’s symptoms?
 Incorrect Answer ImageA.Alcohol withdrawal
 Incorrect Answer ImageB.Cocaine intoxication
 Incorrect Answer ImageC.Diazepam withdrawal
 Incorrect Answer ImageD.Opioid withdrawal
 Correct Answer ImageE.Phencyclidine intoxication

Over the past five months, a 48-year-old single woman has had a significant change in behavior.
She started accusing her sister of poisoning her to get the entire piece of property they were
going to split as an inheritance. She then began accusing her sister, who works in a government
agency, of plotting against her and getting government officials involved. She started writing
numerous accusatory letters to government officials and finally hired a private detective to
"investigate and collect the evidence." In the meantime, she has been working for years as a
personal assistant in a law firm. Mental status examination shows organized thoughts, an
appropriate affect, and no hallucinations. She has no personal or family history of medical or
psychiatric disorders. Which of the following is the most likely diagnosis? 
 Incorrect Answer ImageA.Brief psychotic disorder
 Correct Answer ImageB.Delusional disorder
 Incorrect Answer ImageC.Paranoid personality disorder
 Incorrect Answer ImageD.Schizophrenia
 Incorrect Answer ImageE.Schizophreniform disorder

A 3-year-old boy is brought to the office by his parents for a routine health maintenance
examination. The mother states that the child’s behavior has not changed significantly since his
last visit 6 months ago. She notes that she has finally adjusted to the fact that her son never
returns the hug when she hugs him. When around other children, the child prefers to play by
himself. On physical examination, the boy is non-cooperative and utters unintelligible mumbles
when approached. The physician fails to establish any eye contact with him. The child has had
three episodes of otitis media since birth. Which of the following is the most likely diagnosis? 
 Correct Answer ImageA.Autism spectrum disorder (ASD)
 Incorrect Answer ImageB.Hearing loss due to repeated otitis media
 Incorrect Answer ImageC.Intellectual disability
 Incorrect Answer ImageD.Normal shyness
 Incorrect Answer ImageE.Rett syndrome

A 34-year-old divorced man comes to the physician for consultation. He recently moved from
another state where he was being treated with sertraline, 150 mg/d for a major depressive
episode. He had no prior psychiatric history. He has been on sertraline for 2 months. He
complains of occasional nausea that does not affect his life, but he is otherwise doing well on the
medication, with marked relief from his symptoms of depression. He is concerned about taking
too much medication and asks your opinion. Which of the following is the most appropriate
management of this patient's medication? 
 Incorrect Answer ImageA.Add prochlorperazine to the treatment regimen
 Incorrect Answer ImageB.Change medication to bupropion secondary to nausea
 Correct Answer ImageC.Continue sertraline for 6 more months
 Incorrect Answer ImageD.Continue sertraline for the rest of the patient’s life
 Incorrect Answer ImageE.Gradually taper the patient off sertraline

A medicine consultation is requested on a 32-year-old woman with schizophrenia who has been
a patient in a closed psychiatric unit for the past week. Several days after the patient's admission,
she developed polyuria, vomiting, stupor, diarrhea, and restlessness. She is currently taking
risperidone, 6 mg given at bedtime, but no other medications. Physical examination is
unremarkable. Which of the following is the most likely diagnosis?
 Incorrect Answer ImageA.Acute dystonic reaction
 Incorrect Answer ImageB.Anticholinergic crisis
 Incorrect Answer ImageC.Serotonin syndrome
 Incorrect Answer ImageD.Tardive dyskinesia
 Correct Answer ImageE.Water intoxication

A 45-year-old man who has schizophrenia is brought to the clinic by his mother. She reports that
he has been followed at the clinic for many years, treated with different antipsychotic agents, and
is currently on risperidone, 4 mg/d, and fluoxetine, 20 mg/d. His psychotic symptoms are well
managed on risperidone. She is concerned about abnormal movements that he has developed in
the past few months. These include facial grimacing, tongue thrusting, and rolling movements of
his fingers and toes. When she asks her son about them, he says that he cannot stop them. Which
of the following is the most likely diagnosis for this condition? 
 Incorrect Answer ImageA.Akathisia
 Incorrect Answer ImageB.Dystonia
 Incorrect Answer ImageC.Parkinsonism
 Incorrect Answer ImageD.Serotonin syndrome
 Correct Answer ImageE.Tardive dyskinesia

A 45-year-old woman with bipolar I disorder that is managed with medication comes to the
physician because of the worsening of her depressive symptoms and weight gain. She had been
stable for a few years but lately has started to feel tired and slow. Her skin has been drier than
usual and her husband has noticed a change in her voice. She is concerned that her intense stress
is causing her hair to fall out. This patient's condition is most likely due to which of the
following?
 Incorrect Answer ImageA.Anemia
 Incorrect Answer ImageB.Iodine deficiency
 Correct Answer ImageC.Lithium use
 Incorrect Answer ImageD.Valproate use
 Incorrect Answer ImageE.Risperidone use

A 30-year-old woman comes to the physician because of feeling sad and tired for about 2
months. She describes difficulty falling asleep at night, decreased appetite with a 15 lb weight
loss, diminished interest in her hobbies, and poor concentration. Physical examination and
routine laboratory studies, including thyroid-stimulating hormone, are within normal limits. The
patient and her physician discuss initiating treatment with a selective serotonin reuptake inhibitor
(SSRI). Which of the following would be the most important additional information to gather
before starting the SSRI?
 Incorrect Answer ImageA.Family history of alcoholism or drug dependence
 Incorrect Answer ImageB.Family history of depressive episodes
 Correct Answer ImageC.Personal history of hypomanic or manic episodes
 Incorrect Answer ImageD.Personal history of kidney diseases
 Incorrect Answer ImageE.Personal history of obsessions or compulsions

An 18-year-old man is brought to the emergency department by his parents after they found him
in the garage 30 minutes ago in a lethargic state. His parents noted that when they found him
there were many opened, unmarked canisters next to him. The physician is unable to obtain any
more information from the patient. His temperature is 36.8ºC (98.2ºF), blood pressure is 90/50
mm Hg, pulse is 100/min, and respirations are 30/min. Physical examination shows diffuse
abdominal tenderness to palpation. Laboratory studies show:
Na+ 142 mEq/L
K+ 3.0 mEq/L
Cl -
94 mEq/L
HCO3- 6 mEq/L
BUN 12 mg/dL
Creatinine 1.5 mg/dL
Glucose 60 mg/dL
pH 7.25
PaCO2 25 mmHg
PaO2 85 mmHg
Urinalysis shows:
Color Dark yellow
pH 4.5
WBC 2-5 hpf
RBC 3-4 hpf
Ketones Negative
Glucose Negative
Nitrites Negative
Crystals Envelope-shaped crystals
The patient is treated with a trial of naloxone and glucose but his mental status does not improve.
Which of the following is the most appropriate next step in management for this patient? 
 Incorrect Answer ImageA.Ethanol 800 mg/kg loading dose
 Correct Answer ImageB.Fomepizole 15 mg/kg loading dose
 Incorrect Answer ImageC.Flumazenil 0.2 mg
 Incorrect Answer ImageD.Serum toxin levels
 Incorrect Answer ImageE.Urgent dialysis
Ginecologia
A 32-year-old woman, gravida 1, para 0, at 33 weeks’ gestation is brought to the emergency
department by her husband because of decreased consciousness. Her pregnancy has been
uneventful up to this point. She mentioned vague right upper quadrant pain to him approximately
3–5 days ago. Today, her husband noted that her eyes appeared "yellow." Her past medical
history is unremarkable, and she does not take any medications outside of her prenatal
supplements. The couple traveled throughout India during the second trimester of her pregnancy.
Her temperature is 38°C (100.5°F), blood pressure is 110/50 mm Hg, pulse is 99/min, and
respirations are 14/min. Physical examination shows an obtunded patient with scleral icterus.
The lungs are clear on auscultation and no heart murmurs are noted. She winces when her right
upper quadrant is palpated. Ultrasonography of her right upper quadrant shows a normal
gallbladder and a heterogeneous liver, with no ductal dilatation. Laboratory studies show:
Alanine aminotransferase (ALT) 1,200 IU/L
Aspartate aminotransferase (AST) 1,230 IU/L
Alkaline phosphatase 400 IU/L
Alanine aminotransferase (ALT) 1,200 IU/L
Total bilirubin 5.1 mg/dL
Conjugated bilirubin 4 mg/dL
Hepatitis A, B, and C serologies are negative. However, reverse transcriptase polymerase chain
reaction demonstrates detectable hepatitis E virus in her blood. When discussing the diagnosis
with the patient’s husband, which of the following statements is most appropriate? 
 Correct Answer ImageA.Mortality is significantly higher in pregnant patients
 Incorrect Answer ImageB.Person-to-person transmission is common in pregnant patients
 Incorrect Answer ImageC.The patient’s presentation is more common in the first
trimester
 Incorrect Answer ImageD.There is a high chance that the infection will be transmitted to
the fetus
 Incorrect Answer ImageE.Transmission is via sexual intercourse

A 23-year-old woman, gravida 2, para 1 at 6 weeks' gestation, comes to the emergency


department because of progressive lower abdominal pain and fever over the last 2 days. Past
medical history is significant for 2 episodes of gonorrhea and 1 episode of chlamydia. Her
temperature is 38.9ºC (102.1ºF), blood pressure is 110/76 mm Hg, pulse is 102/min, and
respirations are 12/min. Abdominal examination shows significant lower abdominal tenderness.
Pelvic examination shows a mucopurulent cervical discharge, and bimanual examination elicits
both cervical motion and adnexal tenderness. Complete blood count shows leukocyte count of
18,000/mm3. Pelvic ultrasound shows a 6-week intrauterine gestation with no adnexal findings.
Which of the following is the most appropriate management?
 Incorrect Answer ImageA.Intramuscular ceftriaxone, oral doxycycline, and discharge
home
 Incorrect Answer ImageB.Intravenous cefotetan and doxycycline and hospital admission
 Correct Answer ImageC.Intravenous cefoxitin, azithromycin 1 gm p.o., and hospital
admission
 Incorrect Answer ImageD.Laparoscopy
 Incorrect Answer ImageE.No treatment is necessary

A 29-year-old woman, gravida 2, para 1, at 38 weeks' gestation comes to the labor and delivery
ward with frequent painful contractions that have been progressive over the last 6 hours. She has
a history of asthma for which she uses an albuterol inhaler. The patient has a documented history
of anaphylactic reaction after administration of penicillin. Physical examination shows regular
uterine contractions every 2 minutes. The cervix is 5 cm dilated and 100% effaced. Presentation
is vertex at -1 station. Her prenatal urine culture shows 100,000 colony-forming units/mL of
group B Streptococcus; sensitivities were not performed. Which of the following is the most
appropriate pharmacotherapy?
 Incorrect Answer ImageA.Cefazolin
 Incorrect Answer ImageB.Clindamycin
 Incorrect Answer ImageC.Erythromycin
 Incorrect Answer ImageD.Penicillin G
 Correct Answer ImageE.Vancomycin
A 27-year-old Hispanic woman, gravida 4, para 1, aborta 3, comes to the physician for
contraceptive counseling. She had been taking oral contraceptive pills but she finds it difficult to
take a pill every day. She is sexually active with her husband, and they use condoms
inconsistently. Her last menstrual period was 3 weeks ago. Her last Pap smear 4 months ago
showed low-grade squamous intraepithelial lesion (LSIL). At the time no HPV testing was done.
She takes no medications. Pelvic examination shows a normal-appearing vulva and vagina; the
cervix is erythematous. The uterus is small and there are no adnexal masses. Urine human
chorionic gonadotropin (hCG) test is negative. Which of the following is the most appropriate
next step in the management of this patient?
 Incorrect Answer ImageA.Biopsy of the lesion
 Correct Answer ImageB.Colposcopy with endocervical curettage
 Incorrect Answer ImageC.Colposcopy with loop electrosurgical excision procedure
(LEEP)
 Incorrect Answer ImageD.HPV-DNA testing
 Incorrect Answer ImageE.Repeat cotesting in 12 months

A 34-year-old woman, gravida 3, para 2, at 32 weeks' gestation comes to the labor and delivery
department because of painless vaginal spotting that occurred 4 hours ago. Her past medical
history includes two cesarean sections for failure to progress, but is otherwise unremarkable. Her
temperature is 37ºC (98.6ºF), blood pressure is 117/67 mm Hg, pulse is 90/min, and respirations
are 12/min. Pelvic examination shows no blood on the perineum. Transabdominal ultrasound
shows hypervascularity in the uterine myometrium extending to the serosa under a low anterior
placenta. Fetal development and heart rate are normal for gestational age. This patient is at
greatest risk for which of the following?
 Incorrect Answer ImageA.Fetal demise
 Incorrect Answer ImageB.Large gestational age infant
 Correct Answer ImageC.Massive hemorrhage
 Incorrect Answer ImageD.No increased risk of poor outcome
 Incorrect Answer ImageE.Urinary tract infections

A 34-year-old woman, gravida 6, para 5, at 39 weeks' gestation comes to the emergency


department in active labor. She denies any vaginal discharge, bleeding, or gush of fluid.
Contractions started 12 hours ago and now occur every 4-5 minutes. She has no history of
serious illness. Physical examination shows the cervix is 8 cm dilated and 60% effaced; the
vertex is -1 station. Amniotomy is performed to augment the labor. Fetal heart rate monitoring
shows the baseline to be 140/min with gradual decreases down to 115/min, lasting 35-40 seconds
each, that coincide with uterine contractions. What is the most likely cause of the fetal heart rate
changes?
 Correct Answer ImageA.Fetal head compression
 Incorrect Answer ImageB.Insufficient placental blood flow
 Incorrect Answer ImageC.Intrauterine infection
 Incorrect Answer ImageD.Oligohydramnios
 Incorrect Answer ImageE.Umbilical cord compression

A 24-year-old woman, gravida 1, para 0, at 34 weeks’ gestation is brought to the emergency


department after a motor vehicle accident. Her husband was dropping her off at work when they
got rear-ended. They were both wearing seat belts. The husband drove them to the emergency
department after reporting the accident. The patient reports mild back pain but she is primarily
very worried about her baby. Her vital signs at the emergency department are stable. Her
obstetrician is contacted, who states that her pregnancy has been unremarkable and her blood
type is O positive. Physical examination shows mild lower back pain and bruises on her lower
abdomen and right shoulder where her seat belts restrained her. Pelvic examination shows a
closed posterior cervix. There is no vaginal bleeding or pooling. An obstetric sonogram is normal
and the fetal heart rate is 164/min. Which of the following is the most appropriate next step in
management?
 Correct Answer ImageA.Electronic fetal monitoring
 Incorrect Answer ImageB.Follow up with her obstetrician next morning
 Incorrect Answer ImageC.Induction of labor
 Incorrect Answer ImageD.Observation in the emergency department
 Incorrect Answer ImageE.Prepare for cesarean section

A 17-year-old girl comes to the physician because she has not yet had a menstrual period. Past
medical history is significant for type 1 diabetes and some visual problems. She does not enjoy
eating a wide variety of food because of her inability to smell. Past surgical history is significant
for a cleft palate that was repaired in childhood. She takes no medications and has no allergies to
medications. Physical examination shows absent breast development and a hypoestrogenic vulva
and vagina. Urine hCG is negative. Which of the following is the most likely diagnosis? 
 Incorrect Answer ImageA.Anorexia nervosa
 Incorrect Answer ImageB.Complete androgen insensitivity syndrome
 Correct Answer ImageC.Kallmann syndrome
 Incorrect Answer ImageD.Polycystic ovarian syndrome
 Incorrect Answer ImageE.Pregnancy

A 31-year-old woman comes to the physician because she has not had a menstrual period for 7
months. Until that time she had had normal cycles. She has had weakness, fatigue, constipation,
weight gain, and depression for the past year. She has felt cold when other people are
comfortable. Her temperature is 36.7°C (98.0°F), blood pressure is 120/70 mm Hg, pulse is
56/min, and respirations are 10/min. Examination shows brittle hair. Deep tendon reflexes are
delayed. Laboratory studies show thyroid stimulating hormone level of 20 µU/mL and prolactin
level of 41 ng/mL. Urine human chorionic gonadotropin (hCG) is negative. Which of the
following is the most likely mechanism of this patient's amenorrhea?
 Incorrect Answer ImageA.Increased prolactin levels causing high TRH, which suppresses
GnRH
 Correct Answer ImageB.Increased TRH causing an increased prolactin, suppressing
GnRH
 Incorrect Answer ImageC.Prolactin suppressing dopamine causing high TRH and TSH
 Incorrect Answer ImageD.Triiodothyronine (T3) and thyroxine (T4) directly affecting
GnRH secretion from the hypothalamus
 Incorrect Answer ImageE.TSH directly affecting GnRH secretion from the hypothalamus

A 25-year-old patient comes to the physician for a routine health maintenance examination. She
has normal menstrual periods every 30 days. She was 15 years old when she first began having
intercourse and has had 8 lifetime sexual partners. She uses condoms for contraception. She has
multiple sclerosis. This condition has required her to use a wheelchair for the past 4 years, which
makes pelvic examination somewhat difficult for her. She smokes 1 pack of cigarettes per day.
She has had normal, annual Pap smears since age 21. Given her difficulty with the pelvic
examination, she inquires as to how often she needs to have a Pap smear performed. Which of
the following is the most appropriate recommendation?
 Incorrect Answer ImageA.A Pap smear is not necessary
 Incorrect Answer ImageB.Every 6 to 9 months
 Incorrect Answer ImageC.Every year
 Correct Answer ImageD.Every 3 years
 Incorrect Answer ImageE.Every 5 years

A 15-year-old girl comes to the physician because of a 2-day history of vaginal discharge and
burning on urination. She is sexually active, and she and her partner use condoms inconsistently.
Pelvic examination is shown. There is mild cervical motion tenderness. A urine culture shows no
growth, and a urine pregnancy test is negative. Cultures of cervical discharge grow Neisseria
gonorrhoea.
In addition to treatment for the gonorrhea infection, the patient should also receive treatment for
which of the following microorganisms?
 Incorrect Answer ImageA.Gram-negative diplococcus
 Correct Answer ImageB.Gram-negative intracellular bacterium
 Incorrect Answer ImageC.Gram-negative rods
 Incorrect Answer ImageD.Gram-positive cocci
 Incorrect Answer ImageE.Gram-positive rods

A 62-year-old woman comes to the physician because of progressive involuntary urine leakage
over the last 2 years. She has had episodes during which she feels an immediate need to void and
is not always able to make it to the restroom in time, requiring the use of adult diapers. This need
to urinate occurs at all times of the day and night, and she often awakens with the need to
urinate. She has a history of acute-angle glaucoma requiring surgical intervention and untreated
hypertension. Which of the following is the most appropriate therapy?
 Incorrect Answer ImageA.Imipramine
 Incorrect Answer ImageB.Oxybutynin
 Correct Answer ImageC.Pelvic floor exercises and bladder training
 Incorrect Answer ImageD.Surgery
 Incorrect Answer ImageE.Tamsulosin
A 58-year-old woman comes to the physician because of a 6-month history of a progressive,
pruritic rash on her breast. She has tried over-the-counter antibiotic ointment, antifungal
ointment, and steroid ointment, none of which have helped. Examination shows a bright-red,
sharply demarcated, oozing and crusting rash with excoriations surrounding the involved area.
The rash involves only the areola of her right breast. There are no breast masses and no axillary
lymphadenopathy. Examination is shown. The left breast shows no abnormalities. The rest of her
physical examination shows no other similar lesions. Which of the following is the most likely
diagnosis?
 Incorrect Answer ImageA.Acute mastitis
 Incorrect Answer ImageB.Ductal carcinoma
 Incorrect Answer ImageC.Eczema
 Incorrect Answer ImageD.Inflammatory carcinoma
 Correct Answer ImageE.Paget disease of the breas

A 28-year-old man and his 25-year-old wife, gravida 0, para 0, come to the physician because of
a 14-month history of inability to conceive, despite regular, unprotected intercourse. The man
has had 3 episodes of difficulty maintaining an erection, which he attributes to a stressful
lifestyle and alcohol consumption. The woman has had regular menstrual periods for 10 years
but had irregular menstrual periods during her teenage years. She had an episode of pelvic
inflammatory disease 9 years ago which was treated with ceftriaxone and doxycycline. Physical
examination shows no abnormalities in both patients. Semen analysis shows:
Normal Values
Ejaculate volume  1.2 mL  >1.5 mL 
pH  7.3  >7.2 
Sperm 12 million/mL  >15 million/mL 
concentration 
Total Motility  50%  >40% 
Progressive 38%  >32% 
motility 
Total Sperm 39.4 million spermatozoa 39 million spermatozoa
Number  per ejaculate  per ejaculate 
Morphology  >4% normal forms  >4% normal forms 
Which of the following is the most appropriate next step in management? 
 Incorrect Answer ImageA.Discuss treatment options
 Incorrect Answer ImageB.Hysterosalpingogram
 Incorrect Answer ImageC.Laparoscopy for evaluation of endometriosis
 Incorrect Answer ImageD.Night tumescence test
 Incorrect Answer ImageE.Ratio of luteinizing hormone to follicle-stimulating hormone
 Correct Answer ImageF.Repeat semen analysis

A 41-year-old woman comes to the physician because she thinks she is going through
menopause. She has been periodically missing periods over the last 9 months and is experiencing
hot flashes and vaginal dryness. She has no medical problems except for mild osteopenia. She
takes calcium and vitamin D. Her family history is significant for breast cancer in her mother and
maternal grandmother. There is also a strong family history of osteoporosis. Because of her
family history, the patient declines hormone replacement therapy and inquires about raloxifene
therapy. When counseling this patient about pharmacotherapy with raloxifene, which of the
following is the most appropriate statement? 
 Correct Answer ImageA.It can be used for osteoporosis prevention and treatment
 Incorrect Answer ImageB.It causes endometrial hyperplasia if not used with progesterone
 Incorrect Answer ImageC.It does not increase the risk of thromboembolic disease
 Incorrect Answer ImageD.It increases high-density lipid concentrations
 Incorrect Answer ImageE.It may provide relief for menopausal symptoms

A 32-year-old woman, gravida 2, para 1, comes to the emergency department because of a large
gush of fluid that soaked through her pants. Her prenatal course is significant for being Rh-
negative, for which she received RhoGAM at 28 weeks’ gestation. She is admitted and delivers a
healthy infant boy after a long and difficult delivery complicated by uterine atony and mild
postpartum hemorrhage. There is suspicion for fetomaternal hemorrhage. Which of the following
is the most appropriate next step in management?
 Incorrect Answer ImageA.Administer 300 mcg RhoGAM intramuscularly
 Incorrect Answer ImageB.Clinically estimate hemorrhage and adjust Rho(D) immune
globulin dosage accordingly
 Incorrect Answer ImageC.No further management is required
 Incorrect Answer ImageD.Perform a Kleihauer-Betke stain
 Correct Answer ImageE.Perform a rosette test

A 39-year-old woman, gravida 1, para 0, at 40 weeks' gestation comes to the labor and delivery
ward with contractions. Her prenatal course has been unremarkable. She has hypothyroidism, for
which she takes levothyroxine. She has no known drug allergies. Her prenatal laboratory studies
show:
Blood type A-negative
Hepatitis B surface antigen Negative
Rubella Immune
Rapid plasma reagin Nonreactive
HIV Negative
50 gram one-hour glucose screen 100 mg/dL (normal <130 mg/dL)
The patient delivers a 3.12 kg (6 lb 14 oz) female fetus with Apgar scores of 9 and 9. The fetal
blood type is found to be A-positive. The patient plans to breastfeed her infant. Which of the
following is the most appropriate next step in management? 
 Incorrect Answer ImageA.Administer hepatitis B immune globulin and hepatitis B
vaccine
 Correct Answer ImageB.Administer RhoGAM
 Incorrect Answer ImageC.Administer rubella immunization
 Incorrect Answer ImageD.Start metoclopramide
 Incorrect Answer ImageE.Start the combined oral contraceptive pill
A 34-year-old primigravid woman delivered a healthy, term male infant 48 hours ago via
uncomplicated vaginal delivery. The woman’s temperature is 37ºC (98.6ºF), blood pressure is
120/76 mm Hg, pulse is 67/min, and respirations are 12/min. She has no complaints, is tolerating
oral intake, and passed her first bowel movement this morning. Upon discharge, the patient
should be told to expect which of the following regarding postpartum lochia?
 Incorrect Answer ImageA.Discharge should last for approximately 1 more week
 Incorrect Answer ImageB.Discharge should last for approximately 12 weeks
 Correct Answer ImageC.Discharge should progress from bright red to pinkish-brown to
yellowish-white
 Incorrect Answer ImageD.Discharge should progress from pinkish-brown to yellowish-
white to greenish
 Incorrect Answer ImageE.Duration of lochia is correlated to lactation

A 22-year-old woman comes to the emergency department because of lower right abdominal
pain for 1 day. She also has mild nausea. She denies vomiting, diarrhea, constipation, or dysuria.
She has not noticed any blood in her urine or stool. She had a similar episode of abdominal pain
1 year ago, which was less severe. Her last menstrual period was 2 weeks ago. Her medical
history is unremarkable, and she takes no medications. She is sexually active, and she and her
partner use condoms inconsistently. Her temperature is 37.4°C (99°F), blood pressure is 110/65
mm Hg, and pulse is 92/min. There is moderate tenderness to deep palpation in the right lower
quadrant. There is no rebound tenderness or guarding. Pelvic examination shows no
abnormalities. Laboratory studies show:
White blood cells  10,300/mm3
Hematocrit 38%
Platelets  227,000/mm3
Neutrophils 60%
Amylase  31 U/L
Lipase 37 U/L
Erythrocyte sedimentation rate  10 mm/hour
Which of the following is the most likely diagnosis? 
 Incorrect Answer ImageA.Acute appendicitis
 Incorrect Answer ImageB.Cecal diverticulitis
 Correct Answer ImageC.Corpus luteum cyst or oophoritis
 Incorrect Answer ImageD.Pancreatitis
 Incorrect Answer ImageE.Pelvic inflammatory disease
 Incorrect Answer ImageF.Chocolate cyst

A 32-year-old woman comes to the physician because she has not had a period in 8 months. She
had menarche at age 13 years and has had normal periods since then. She also has concerns
about occasional milky nipple discharge. She has no medical problems and takes no medications.
She is particularly concerned because she would like to become pregnant as soon as possible.
Physical examination shows a whitish nipple discharge bilaterally, but the rest of the
examination is unremarkable. Urine beta-hCG level is negative. Thyroid-stimulating hormone
(TSH) level is normal. Prolactin is elevated. Which of the following is the most appropriate next
step in management? 
 Incorrect Answer ImageA.Estrogen-progesterone challenge test
 Correct Answer ImageB.MRI scan of the brain
 Incorrect Answer ImageC.Neurosurgical consult
 Incorrect Answer ImageD.Prescribe bromocriptine
 Incorrect Answer ImageE.Progesterone challenge test

A 36-year-old woman comes to the physician because of a 3-month history of dyspareunia and
vaginal spotting. She is a sex worker and she uses condoms inconsistently. She has had 3
episodes of Chlamydia infection and 2 episodes of gonorrhea, all of which have been treated.
Pelvic examination shows several 4-mm condylomas on the outer right labia majora and a
normal-appearing vagina, uterus, and rectum. The cervix is erythematous with an 8-mm friable
lesion on the transformation zone. A Pap smear shows a high-grade squamous intraepithelial
lesion (HSIL). Which of the following is the most appropriate next step in management?
 Incorrect Answer ImageA.Acyclovir
 Incorrect Answer ImageB.Dilation and curettage (D&C)
 Correct Answer ImageC.Office colposcopy and biopsy
 Incorrect Answer ImageD.Office colposcopy and loop electrosurgical excision procedure
(LEEP)
 Incorrect Answer ImageE.Electrofulguration of the condylomas
 Incorrect Answer ImageF.Repeat Pap smear

A 45-year-old woman comes to the physician because of a 2-year history of progressively heavy
and painful menses. She denies dyschezia, dyspareunia, or irregularities in her menstrual cycle.
Her past medical history is significant for hypothyroidism, for which she takes levothyroxine.
She is allergic to sulfa drugs. Her temperature is 37.1°C (98.8°F), pulse is 80/min, respirations
are 12/min, and blood pressure is 130/80 mm Hg. Pelvic examination shows a soft, enlarged,
tender uterus consistent in size with 14 weeks of gestation. The remainder of the physical
examination shows no abnormalities. Urine β-hCG is negative. Pelvic ultrasound shows a
diffusely enlarged uterus. Pelvic MRI shows an enlarged, globular uterus with cystic areas in the
myometrial wall. Which of the following is the most likely diagnosis?
 Correct Answer ImageA.Adenomyosis
 Incorrect Answer ImageB.Ectopic pregnancy
 Incorrect Answer ImageC.Hydatidiform mole
 Incorrect Answer ImageD.Leiomyoma
 Incorrect Answer ImageE.Leiomyosarcoma

A 37-year-old woman comes to the physician for a follow-up visit to discuss an abnormal Pap
smear result. The Pap smear taken 1 week ago shows atypical squamous cells of undetermined
significance (ASCUS) and human papillomavirus (HPV) DNA testing is negative. Her last 2 Pap
smears were normal. Six months ago she had cervical mucopurulent discharge treated with
ceftriaxone and doxycycline. She has a 1-year history of hypertension. Current medications
include hydrochlorothiazide and oral contraceptive pills. She does not smoke. Which of the
following is the most appropriate next step in management?
 Incorrect Answer ImageA.Ectocervical biopsy
 Incorrect Answer ImageB.Endocervical curettage
 Incorrect Answer ImageC.Repeat Pap smear in 1 year
 Incorrect Answer ImageD.Repeat Pap smear in 4–6 months, and if normal, again in 12
months
 Correct Answer ImageE.Repeat co-testing with cervical cytology and HPV testing in 3
years

A 33-year-old woman comes to the physician because of anxiety and irritability over the past
year. She says that approximately 10 days before the onset of her menses, she goes from "feeling
fine" to "feeling awful." She has frequent angry outbursts toward her husband and children, feels
depressed and irritable, and cries easily. She also develops severe breast tenderness during this
time. These symptoms resolve soon after the start of her menses and she feels fine until the
symptoms start over again as the next menses approach. She feels that she can't go on like this
anymore, because her job and family relations are suffering greatly. She has no other medical
problems, takes no medications, and her physical examination is entirely normal. Which of the
following is the most appropriate next step in management? 
 Incorrect Answer ImageA.Alprazolam
 Correct Answer ImageB.Fluoxetine
 Incorrect Answer ImageC.Ibuprofen
 Incorrect Answer ImageD.Progesterone
 Incorrect Answer ImageE.Spironolactone

Ginecologia

A 62-year-old woman comes to the physician because of intermittent vaginal bleeding over the
last month. Her last menstrual period was 11 years ago, and she states that she had no bleeding
between menopause and the onset of her current symptoms. She has hypertension and type 2
diabetes mellitus. Physical examination shows a mildly obese woman in no apparent distress.
Pelvic examination shows no abnormalities. An endometrial biopsy is performed that shows
grade I endometrial adenocarcinoma. Which of the following is the most appropriate next step in
management?
 Incorrect Answer ImageA.Anti-estrogen therapy
 Incorrect Answer ImageB.Cone biopsy
 Incorrect Answer ImageC.Dilation and curettage (D&C)
 Correct Answer ImageD.Hysterectomy
 Incorrect Answer ImageE.Hysteroscopy
 Incorrect Answer ImageF.Radiation therapy

A 38-year-old Ashkenazi Jewish woman comes to the physician for an annual gynecologic
examination. She has no chronic medical problems and takes no medications. She has had 3
cesarean deliveries and a bilateral tubal ligation. The patient has smoked 1/2 pack of cigarettes
daily since at 17, has a beer with dinner twice a week, and does not use illicit drugs. She lives
with her husband and kids and feels safe at home. The patient’s maternal grandmother and a
maternal first cousin were diagnosed with breast cancer in their late 50s. Vital signs are within
normal limits. A pelvic examination is normal. A clinical breast examination shows uniformly
dense breast and no masses, skin changes, or lymphadenopathy. Aside from recommending
smoking cessation, which of the following is the most appropriate next step in the management?
 Correct Answer ImageA.BRCA1/2 testing
 Incorrect Answer ImageB.Breast MRI
 Incorrect Answer ImageC.Breast self-examination
 Incorrect Answer ImageD.Breast ultrasound
 Incorrect Answer ImageE.Mammogram

A 29-year-old woman comes to the clinic because of irregular menstrual periods. The patient has
taken combination oral contraceptives since menarche at age 13 due to dysmenorrhea. While
taking the oral contraceptives, the patient’s periods occurred every 28 days and consisted of 4
days of light bleeding and mild cramping. The patient stopped taking the oral contraceptives 12
months ago in order to conceive. Since then she has had 2 menstrual periods. Blood pressure is
135/88 mm Hg, pulse is 72/min, and body mass index is 32 kg/m2. Physical examination shows
dark coarse hairs on the upper lip and chest. Breast and pelvic examination are normal. This
patient is at greatest risk for which of the following conditions?
 Correct Answer ImageA.Diabetes mellitus
 Incorrect Answer ImageB.Epithelial cervical cancer
 Incorrect Answer ImageC.Ovarian torsion
 Incorrect Answer ImageD.Primary ovarian insufficiency
 Incorrect Answer ImageE.Tubal factor infertility

A 28-year-old Caucasian woman, gravida 1, para 0, at 8 weeks' gestation comes to the physician
for an initial prenatal visit. She had some spotting a few weeks ago, which has now resolved.
Transvaginal ultrasound, performed when she had the spotting, showed a 6-week intrauterine
pregnancy. She has no medical problems and no surgical history. She takes no medications and
has no known drug allergies. Her family history is unremarkable. Her husband is also Caucasian
and has no medical problems. Physical examination shows a nontender uterus consistent in size
with 8 weeks of gestation. This patient should be offered screening for which of the following
disorders?
 Incorrect Answer ImageA.Achondroplasia
 Correct Answer ImageB.Cystic fibrosis
 Incorrect Answer ImageC.Marfan syndrome
 Incorrect Answer ImageD.Myotonic dystrophy
 Incorrect Answer ImageE.Neurofibromatosis

A 19-year-old woman comes to the physician because she missed several of her birth control
pills in the past 2 months and a home urine pregnancy test was positive. She cannot remember
when she had her last period. She is feeling well and has no complaints. She lives with her
mother and 2 younger siblings. She does not smoke, drink alcohol, or use illicit drugs. Her
temperature is 37ºC (98.6ºF), pulse is 72/min, respirations are 16/min, and blood pressure is
104/78 mm Hg. Physical examination shows tender breasts, a gravid uterus, a blue-tinged cervix,
and is otherwise unremarkable. Pelvic ultrasonography shows an intrauterine pregnancy of 12
weeks’ gestation. Prenatal laboratory studies, including a Pap smear and urine and cervical
cultures, are obtained. Urinalysis shows moderate bacteriuria with positive leukocyte esterase
and positive nitrites. She is started on prenatal vitamins with folate. Which of the following is the
most appropriate next step in management?
 Incorrect Answer ImageA.Levonorgestrel

 Incorrect Answer ImageB.Miconazole

 Correct Answer ImageC.Nitrofurantoin

 Incorrect Answer ImageD.No additional management is necessary

 Incorrect Answer ImageE.Trimethoprim/sulfamethoxazole (TMP-SMX)

A 16-year-old girl comes to the physician because she has never had a period. She has no
medical problems, has never had surgery, and takes no medications. Physical examination shows
that she is a tall female with long extremities. Breast development is normal with pale areolas.
Pelvic examination is significant for no pubic hair and a short, blind-ended vaginal pouch.
Because of her condition, this patient is at risk for malignancy from which of the following?
 Incorrect Answer ImageA.Chromosomal anomaly
 Incorrect Answer ImageB.Excessive testosterone production
 Correct Answer ImageC.Location of the gonads
 Incorrect Answer ImageD.Unopposed estrogen
 Incorrect Answer ImageE.Unopposed progesterone

A 67-year-old woman comes to the physician because of dysuria and frequent urination. She has
hypertension for which she takes a beta-blocker, but no other medical problems. She states that
she is not sexually active. She does not smoke and drinks cranberry juice daily. Her temperature
is 37ºC (98.6ºF), blood pressure is 134/84 mm Hg, pulse is 84/min, and respirations are 12/min.
Physical examination shows mild suprapubic tenderness and genital atrophy but is otherwise
unremarkable. Urinalysis shows 50 to 100 leukocytes/high-powered field (hpf) and 5 to 10
erythrocytes/hpf. Which of the following is the most likely predisposing factor for her condition?
 Incorrect Answer ImageA.Cardiac disease
 Incorrect Answer ImageB.Cranberry juice ingestion
 Correct Answer ImageC.Hypoestrogenism
 Incorrect Answer ImageD.Nephrolithiasis
 Incorrect Answer ImageE.Sexual intercourse

A 22-year-old woman, gravida 2, para 1, at 39 weeks’ gestation comes to labor and delivery with
painful uterine contractions and rupture of membranes that happened 4 hours ago. Her past
medical history is remarkable for a prior cesarean section, performed for breech presentation 2
years ago. She desires to undergo a trial of labor after cesarean delivery. Her temperature is 37ºC
(98.6ºF), blood pressure is 120/76 mm Hg, pulse is 72/min, and respirations are 14/min. Physical
examination shows her cervix to be 4 cm dilated and 60% effaced; the vertex is at -1 station. The
fetal heart rate is 140/min with moderate variability, accelerations, and no decelerations. An
intrauterine pressure catheter shows contractions every 3 to 4 minutes. The physician is called 2
hours later for a sudden increase in abdominal pain. Maternal blood pressure is 90/65 mm Hg
and pulse is 124/min. The fetal heart rate drops up to 70/min. Pelvic examination shows a
moderate amount of blood in the vagina, the cervix is 8 cm dilated and 100% effaced, with loss
of station of the fetal head. The intrauterine pressure catheter shows a decreased intensity of
uterine contractions. Which of the following is the most appropriate next step in management?
 Correct Answer ImageA.Emergent laparotomy
 Incorrect Answer ImageB.Emergent forceps vaginal delivery
 Incorrect Answer ImageC.Emergent obstetric ultrasound
 Incorrect Answer ImageD.Intravenous magnesium sulfate
 Incorrect Answer ImageE.Intravenous oxytocin

A 28-year-old woman, gravida 1, para 0, at 15 weeks' gestation comes for a routine prenatal
examination in November. She has had no nausea or vomiting. She underwent surgery for an
open leg fracture at age 15 years. Her medical history is otherwise unremarkable. She has
received all of the childhood immunizations that were offered to her. Her last immunization was
at the age of 16 years but she does not recall what it was for. Current medications include
acetaminophen for sporadic headaches. She does not smoke cigarettes, drink alcohol, or use
illicit drugs. Which of the following is the most appropriate immunization to administer at this
visit? 
 Incorrect Answer ImageA.Hepatitis A and hepatitis B vaccines
 Incorrect Answer ImageB.Hepatitis B vaccine and tetanus toxoid (Td)
 Correct Answer ImageC.Inactivated influenza vaccine
 Incorrect Answer ImageD.Inactivated influenza vaccine and pneumococcal
polysaccharide vaccine
 Incorrect Answer ImageE.Inactivated influenza vaccine and tetanus toxoid (Td)
 Incorrect Answer ImageF.Inactivated influenza vaccine, MMR, and varicella
 Incorrect Answer ImageG.Live attenuated influenza vaccine and tetanus toxoid (Td)
 Incorrect Answer ImageH.Measles, mumps, and rubella vaccine (MMR)

A 31-year-old woman, gravida 1, para 1, comes to the physician 10 days after an uncomplicated
vaginal delivery because of sadness and fatigue. The patient is breastfeeding her infant who
awakens every 2-3 hours at night to nurse. Over the past week, she has had frequent crying spells
and has been very anxious about whether she will be a good mother. She has had no suicidal or
homicidal thoughts. The patient’s pregnancy was uncomplicated. She is taking daily oral iron
supplementation and has no chronic medical problems. The patient lives with her husband who
returned to work several days after her delivery. Vital signs are within normal limits. Physical
examination shows an alert, oriented, and tearful patient. Which of the following is the most
appropriate treatment for this patient?
 Incorrect Answer ImageA.Electroconvulsive therapy
 Incorrect Answer ImageB.Lithium
 Correct Answer ImageC.No treatment necessary
 Incorrect Answer ImageD.Psychotherapy
 Incorrect Answer ImageE.Sertraline
A 52-year-old nulliparous woman comes to the physician for an annual gynecologic
examination. The patient is feeling well and has no complaints. Her last menstrual period was at
age 50. She has a history of hypertension managed with a calcium channel blocker and recently
completed progesterone treatment for endometrial hyperplasia. The patient has 2 glasses of wine
with dinner each night and has smoked a pack of cigarettes a day since age 17. Her sister was
recently diagnosed with metastatic breast cancer at age 55 and her mother died from breast
cancer at age 59. BMI is 17 kg/m2. Her pulse is 88/min and blood pressure is 124/76 mm Hg.
Pelvic and breast examination are within normal limits. A Papanicolaou test is normal. Which of
the following is the most appropriate next step in management?
 Incorrect Answer ImageA.Anastrozole therapy
 Incorrect Answer ImageB.Bilateral mastectomy
 Incorrect Answer ImageC.Bilateral salpingoophorectomy
 Correct Answer ImageD.Raloxifene therapy
 Incorrect Answer ImageE.Tamoxifen therapy

A 75-year-old woman comes to the physician because of abdominal distension. She states that
she always feels bloated and that she gets full quickly when eating. She has hypertension for
which she takes an angiotensin-converting enzyme (ACE) inhibitor. She does not have any other
medical problems. Abdominal examination shows abdominal distension and a positive fluid
wave. Pelvic examination shows a large, nontender right adnexal mass. An abdominal and pelvic
CT scan shows bilateral ovarian masses, ascites, and omental caking. CA-125 level is
significantly elevated. Serum alpha-fetoprotein (AFP) and human chorionic gonadotropin (hCG)
are negative. Which of the following is the most likely diagnosis?
 Incorrect Answer ImageA.Choriocarcinoma
 Incorrect Answer ImageB.Cystic teratoma
 Incorrect Answer ImageC.Embryonal carcinoma
 Correct Answer ImageD.Epithelial ovarian cancer
 Incorrect Answer ImageE.Sertoli stromal cell tumor

A 22-year-old woman comes to the physician because of burning during urination and vulvar
itching over the past 3 days. She is sexually active with one partner, and they use condoms
inconsistently. Vital signs are normal. Abdominal and costovertebral examination shows no
tenderness. Pelvic examination shows mild vulvar erythema and a thick, white discharge on the
vaginal walls. The cervix is normal-appearing and nontender. Microscopy of the vaginal
discharge shows epithelial cells with multiple adherent bacteria as well as budding yeast and
hyphae. The addition of potassium hydroxide (KOH) to the slide produces an amine odor. The
pH of the discharge is 5.0. Urinalysis is within normal limits. Vaginal cultures and nucleic acid
amplification testing (NAATs) for N. gonorrhoeae and C. trachomatis are sent. Which of the
following is the most appropriate pharmacotherapy for this patient? 
 Incorrect Answer ImageA.Await results of vaginal culture
 Incorrect Answer ImageB.Metronidazole prescription for her sexual partner
 Incorrect Answer ImageC.Oral ciprofloxacin
 Incorrect Answer ImageD.Oral fluconazole
 Correct Answer ImageE.Oral fluconazole and tinidazole
 Incorrect Answer ImageF.Oral metronidazole
A 24-year-old woman comes to the emergency department because of abdominal pain and
vaginal spotting. She states that the pain started yesterday morning and is continuous but mild. It
is located in the left lower quadrant and nothing that she does seems to make it better or worse.
She has depression, for which she takes sertraline. She has never had any surgeries. Her
temperature is 37.0ºC (98.6ºF), blood pressure is 110/70 mm Hg, pulse is 84/min, and
respirations are 12/min. Abdominal examination shows mild abdominal tenderness in the left
lower quadrant. Speculum exam shows scant dark blood in the vaginal vault with a closed
cervical os. The remainder of the physical examination is normal. Laboratory evaluation shows:
Urine β-hCG Positive
Serum β-hCG 420 mIU (non-pregnant female reference range <5
mIU/mL)
Leukocyte 9,000/mm3
count
Hematocrit 38%
Platelets 230,000/mm3
Blood type A-positive
Pelvic ultrasound shows no intrauterine or ectopic pregnancy and is subsequently read as normal.
Which of the following is the most appropriate next step in management? 
 Incorrect Answer ImageA.Administer methotrexate
 Incorrect Answer ImageB.Administer RhoGAM
 Incorrect Answer ImageC.Perform dilation and curettage
 Incorrect Answer ImageD.Perform laparoscopy
 Correct Answer ImageE.Recheck serum β-hCG in 2 days

A 27-year-old woman returns from her honeymoon in Hawaii and comes to the physician
because of urinary frequency and dysuria for 2 days. She has no history of urinary tract infection.
She denies fever or chills but does report a vague suprapubic pressure that is constant and only
partially relieved with acetaminophen. Her temperature is 37.0ºC (98.6ºF), blood pressure is
100/60 mm Hg, pulse is 63/min, and respirations are 18/min. Physical examination is
unremarkable except for mild suprapubic tenderness with no rebound or guarding. A
photomicrograph of the urine Gram stain is shown. Which of the following is the most
appropriate next step in management?
 Incorrect Answer ImageA.No treatment until cultures are obtained
 Incorrect Answer ImageB.Reassurance and recommendation of plenty of fluids, such as
cranberry juice
 Incorrect Answer ImageC.Seven-day course of an antistaphylococcal penicillin
 Incorrect Answer ImageD.Seven-day course of IV ciprofloxacin
 Correct Answer ImageE.Three-day course of oral trimethoprim/sulfamethoxazole

A 41-year-old woman comes to the physician because of a 4-day history of a painful rash over
her vulva. She has a 6-year history of periodic episodes of malaise and fever, together with a
painful rash. She has had approximately 8 to 10 episodes per year. Each outbreak is preceded by
a burning sensation over her vulva. Which of the following is the mechanism of action of the
most appropriate pharmacotherapy for this patient? 
 Incorrect Answer ImageA.Altering cell membrane permeability
 Incorrect Answer ImageB.Inhibition of bacterial cell wall synthesis
 Correct Answer ImageC.Inhibition of DNA synthesis
 Incorrect Answer ImageD.Inhibition of folic acid synthesis
 Incorrect Answer ImageE.Inhibition of protease
 Incorrect Answer ImageF.Inhibition of replication by blocking reverse transcriptase

A 19-year-old woman, gravida 1, para 1, is seen by the physician 2 hours after delivery of a non-
viable fetus at 35 weeks' gestation because of increased vaginal bleeding and bleeding from her
IV site. She had been diagnosed with loss of a fetus measuring 31 weeks' gestation by ultrasound
after coming to the labor and delivery department with uterine cramping. She underwent an
induction of labor and delivered the non-viable fetus without complications. Her temperature is
37ºC (98.6ºF), blood pressure is 110/62 mm Hg, pulse is 105/min, and respirations are 12/min.
Pelvic examination shows watery blood coming from the cervical os. No cervical or vaginal
lacerations are seen. Laboratory studies show:
Hemoglobin 7.2 g/dL
Leukocyte count 8,100/mm3
Platelets 102,000/mm3
Prothrombin time  26 sec
Partial thromboplastin time 31 sec
(activated)
Fibrinogen 98 mg/dL (normal 200–400
mg/dL)
Which of the following is the most appropriate next step in management?
 Incorrect Answer ImageA.Administer misoprostol
 Incorrect Answer ImageB.Administer oxytocin
 Correct Answer ImageC.Immediate transfusion with packed red blood cells (PRBCs) and
fresh frozen plasma (FFP)
 Incorrect Answer ImageD.Take her to the operating room for immediate hysterectomy
 Incorrect Answer ImageE.Uterine massage

A 34-year-old woman, gravida 3, para 3, comes to the physician because of an area on her left
breast that is red, swollen, and painful. She thinks she has had fevers and chills over the last 2
days but has not taken her temperature. Four weeks ago, she had a normal spontaneous vaginal
delivery of a healthy male infant. She is breastfeeding successfully and had a postpartum tubal
ligation for birth control. The infant is doing well. She has occasional migraine headaches and no
other medical problems. She takes no medications and is allergic to sulfa drugs. She has no
family history of breast cancer. Her temperature is 37.9ºC (100.2ºF). Physical examination shows
significant tenderness in an area on her left breast, which is surrounded by erythema and edema.
The area is not fluctuant. The remainder of her physical examination shows no abnormalities.
Which of the following is the most appropriate next step in management? 
 Incorrect Answer ImageA.Continue with expectant management
 Incorrect Answer ImageB.Perform incision and drainage
 Correct Answer ImageC.Prescribe dicloxacillin
 Incorrect Answer ImageD.Prescribe levofloxacin
 Incorrect Answer ImageE.Prescribe penicillin
A 29-year-old woman, gravida 2, para 2, comes to the physician because of urinary incontinence
for 5 months. She explains that she loses small amounts of urine when she picks up her children
and when she sneezes. She delivered an infant boy 16 months ago via normal vaginal delivery, at
which time she suffered a second-degree vaginal laceration. Her medical history is
unremarkable. She takes no medications. Which of the following is the most appropriate next
step in management? 
 Incorrect Answer ImageA.Alpha 1-adrenergic agonist
 Incorrect Answer ImageB.Alpha 1-adrenergic antagonist
 Incorrect Answer ImageC.Acetylcholine agonist
 Incorrect Answer ImageD.Kegel exercises
 Correct Answer ImageE.Pelvic examination
 Incorrect Answer ImageF.Q-tip test
 Incorrect Answer ImageG.Residual volume measurement by catheterization
 Incorrect Answer ImageH.Residual volume measurement by ultrasound

A 42-year-old woman comes to the emergency department because of several weeks of


worsening left lower quadrant pain. Over the past few days, the pain has been associated with
nausea and vomiting. The patient takes no medications, has no chronic medical problems, and
has had no prior surgery. Temperature is 37.0℃ (98.6℉), blood pressure is 134/82 mm Hg, and
pulse is 120/min. Speculum examination is normal. Bimanual pelvic examination reveals a
tender left adnexal mass. Urine pregnancy test is negative. TSH is 0.1 μU/mL and free thyroxine
is 2.4 ng/dL (reference range 0.9-1.7 ng/dL). Pelvic ultrasound shows a 5-cm complex cyst in the
left adnexa. Which of the following is the most likely diagnosis?
 Incorrect Answer ImageA.Choriocarcinoma
 Incorrect Answer ImageB.Dysgerminoma
 Incorrect Answer ImageC.Epithelial carcinoma
 Correct Answer ImageD.Mature teratoma
 Incorrect Answer ImageE.Sertoli-Leydig cell tumor

A 32-year-old woman, gravida 3, para 2, at 34 weeks' gestation comes to the emergency


department because of severe headache, nausea, vomiting, and pain in the epigastric area for the
past 48 hours. She works in a daycare center but does not recall being around any sick children.
She also denies eating anything unusual and denies having diarrhea. The pregnancy has been
uncomplicated and she does not have any other medical history. Her temperature is 37.5ºC
(99.5ºF), blood pressure is 160/105 mm Hg, pulse is 105/min, and respirations are 12/min.
Physical examination shows mild tenderness to palpation in the right upper quadrant and a fundal
height of 35 cm; chest auscultation shows clear lungs and no cardiac murmurs or rubs. The fetal
heart rate is 140/min. Laboratory studies show:
Hemoglobin 9.0 mg/dL
Hematocrit 29%
Platelets 75,000 cells/mm3
Leukocyte count  8,000 cells/mm3
Peripheral blood smear Schistocytes
Urea nitrogen, serum 20 mg/dL
Creatinine 1.0 mg/dL
Total bilirubin 4.0 mg/dL
Lactate dehydrogenase 650 U/L
Alanine aminotransferase (ALT) 150 U/L
Aspartate aminotransferase (AST) 165 U/L
Prothrombin time 15 seconds
Partial thromboplastin time (activated) 40 seconds
Urine dipstick 3+ protein
Which of the following is the most likely diagnosis? 
 Incorrect Answer ImageA.Acute cholecystitis
 Incorrect Answer ImageB.Acute fatty liver of pregnancy
 Incorrect Answer ImageC.Acute hepatitis A infection
 Correct Answer ImageD.HELLP syndrome
 Incorrect Answer ImageE.Idiopathic thrombocytopenic purpura

A 34-year-old woman, gravida 3, para 2, at 34 weeks' gestation is brought to the emergency


department by her husband because of confusion. She began experiencing a severe headache,
nausea, and vomiting 3 days ago. This morning she was pacing the living room and not
responding to his voice. She has had regular prenatal care and a previously uncomplicated
pregnancy. She works in a daycare center and had perfect mental function prior to this event. Her
temperature is 37.5ºC (99.5ºF), blood pressure is 160/105 mm Hg, pulse is 105/min, and
respirations are 12/min. Physical examination shows mild tenderness to palpation in the right
upper quadrant, fundal height 35 cm; chest auscultation shows clear lungs and no cardiac
murmurs or rubs. The fetal heart rate is 150/min. Serum laboratory results show:
Hemoglobin 10.0 mg/dL
Hematocrit 29%
Platelets 40,000 cells/mm3
Leukocyte count  15,000 cells/mm3
Glucose 50 mg/dL
BUN 50 mg/dL
Creatinine 2.5 mg/dL
Total bilirubin 8.0 mg/dL
LDH 900 U/L
ALT 250 U/L
AST 365 U/L
PT 25 seconds
PTT 60 seconds
Spot dipstick 3+ proteinuria
Which of the following is the most likely diagnosis? 
 Incorrect Answer ImageA.Acute cholecystitis
 Correct Answer ImageB.Acute fatty liver of pregnancy
 Incorrect Answer ImageC.Acute hepatitis A infection
 Incorrect Answer ImageD.HELLP syndrome
 Incorrect Answer ImageE.Idiopathic thrombocytopenic purpura

A 32-year-old woman comes to the emergency department because of pain and swelling in her
left calf over the last day. She has a history of 3 deep venous thromboses in the last 2 years. The
first 2 episodes involved the right lower extremity and the third episode involved the left lower
extremity. She has had 2 pregnancies in the last 2 years, which she miscarried at 12 and 14
weeks' gestation, respectively. She was prescribed warfarin after her most recent deep venous
thrombosis, but she recently ran out of the medication. She is admitted to the hospital for
treatment. Which of the following tests is the most appropriate next step in the diagnosis of her
underlying condition? 
 Incorrect Answer ImageA.ANA
 Correct Answer ImageB.Anticardiolipin antibodies
 Incorrect Answer ImageC.Factor VIII level
 Incorrect Answer ImageD.Factor IX level
 Incorrect Answer ImageE.Hepatitis profile
 Incorrect Answer ImageF.von Willebrand activity

A 32-year-old woman at 36 weeks’ gestation comes to the physician for intractable itching. The
itching began on her palms and soles, has progressed proximally, and has gone on for
approximately 5 days. Diphenhydramine has offered minimal relief. Her pregnancy up to this
point has been uneventful, and she has had no complications. Physical examination shows no
abnormalities except for excoriations on her skin from scratching. Laboratory studies show a
total bilirubin of 3.3 mg/dL, significantly elevated bile acids, and an alkaline phosphatase of 500
U/L. Which of the following is the most appropriate next step in management? 
 Incorrect Answer ImageA.Computed tomography of the abdomen
 Correct Answer ImageB.Delivery of the fetus
 Incorrect Answer ImageC.Liver biopsy
 Incorrect Answer ImageD.Reassurance
 Incorrect Answer ImageE.Supplementation with B vitamins

A 26-year-old woman comes to the emergency department because of the sudden onset of lower
abdominal pain that started 12 hours ago. The pain has been accompanied by a small amount of
vaginal bleeding. She has also been nauseated and had one episode of vomiting. She has not had
any diarrhea, constipation, or dysuria. Her last menstrual period was 7 weeks ago. Her
temperature is 36.6ºC (98.0ºF) and respirations are 16/min. Orthostatic vital signs show a blood
pressure of 120/80 mm Hg and pulse of 90/min while sitting, and a blood pressure of 90/60 mm
Hg and a pulse of 120/min while standing. Physical examination shows dry oral mucosa, and
right lower quadrant and suprapubic tenderness to palpation. Pelvic examination shows cervical
motion tenderness and a right-sided adnexal mass. There is no cervical discharge. Laboratory
studies show:
Hemoglobin 11.7 g/dL
Leukocyte count 16,000/mm3
Platelets 270,000/mm3
Sodium 132 mEq/L
Potassium 4.0 mEq/L
Chloride 92 mEq/L
Bicarbonate 28 mEq/L
Urea nitrogen 20 mg/dL
Creatinine 1.0 mg/dL
Glucose 100 mg/dL
Calcium 9.2 mg/dL
Serum β-hCG 1,700 U/L
Aspartate aminotransferase (AST) 19 U/L
Alanine aminotransferase (ALT) 21 U/L
Which of the following is the most likely diagnosis? 
 Incorrect Answer ImageA.Acute cholecystitis
 Incorrect Answer ImageB.Acute pancreatitis
 Correct Answer ImageC.Ectopic pregnancy
 Incorrect Answer ImageD.Ovarian cyst
 Incorrect Answer ImageE.Pelvic inflammatory disease

A 19-year-old woman comes to the physician because she has not had a menstrual period. She
experienced normal breast development through puberty but has yet to have a period. She has no
other concerns. She has no medical problems. Physical examination shows the patient to be tall
with long arms and big hands. The breasts are normal-appearing except that the nipples are
immature and the areolae are pale. Pelvic examination shows scant pubic hair with a blind-ended
vaginal pouch. Which of the following is the most likely diagnosis? 
 Incorrect Answer ImageA.Asherman syndrome
 Correct Answer ImageB.Complete androgen insensitivity syndrome
 Incorrect Answer ImageC.Kallmann syndrome
 Incorrect Answer ImageD.Polycystic ovarian syndrome
 Incorrect Answer ImageE.Turner syndrome

A 32-year-old Hispanic woman, gravida 2, para 1, at 38 weeks' gestation by last menstrual


period and first-trimester ultrasound comes to the physician for prenatal care. She has had
occasional headaches. She does not have right upper quadrant pain, visual changes, uterine
contractions, vaginal bleeding, or leakage of fluid. She says there have been decreased fetal
movements during the last week. Her history is significant for traveling to the United States
when she was 28 weeks pregnant with poor follow-up since then. Her prenatal records from
Mexico show she has chronic hypertension but is otherwise healthy. She was previously taking
labetalol; however, she ran out of the medication 5 weeks ago. Blood pressure is 152/94 mm Hg,
and the fundal height is 31 cm. Urine dipstick is negative for proteinuria. An ultrasound with
fetal biometry shows a fetus in vertex presentation with a head circumference consistent with 34
weeks of gestation and an abdominal circumference consistent with 32 weeks of gestation. A
detailed fetal anatomic survey did not find any congenital anomalies. Only 1 fetal breathing
movement of 35 seconds was noted, but there was no noted extremity flexion or movement. The
amniotic fluid index is 4 cm. A non-stress test shows a baseline fetal heart rate of 140/min with
moderate variability. No accelerations or decelerations are present. Which of the following is the
most appropriate next step in management?
 Incorrect Answer ImageA.Administer steroids for fetal lung maturity
 Incorrect Answer ImageB.Admission to the hospital for magnesium sulfate infusion
 Incorrect Answer ImageC.Immediate admission to the hospital for emergent cesarean
section
 Correct Answer ImageD.Immediate admission to the hospital for induction of labor
 Incorrect Answer ImageE.Perform a biophysical profile
 Incorrect Answer ImageF.Restart labetalol and follow up in 3 days
A 43-year-old primigravid woman at 10 weeks' gestation comes to the physician for a prenatal
visit. She is feeling well except for some occasional nausea. She denies vaginal bleeding,
abdominal pain, dysuria, frequency, or urgency. She has asthma for which she occasionally uses
an inhaler. Physical examination is normal for a woman at 10 weeks’ gestation. Urine dipstick is
positive for nitrites and leukocyte esterase and a urine culture shows 100,000 colony-forming
units per milliliter ofEscherichia coli; antibiotic sensitivities are unavailable. Which of the
following is the most appropriate pharmacotherapy?
 Incorrect Answer ImageA.Doxycycline
 Incorrect Answer ImageB.Levofloxacin
 Correct Answer ImageC.Nitrofurantoin
 Incorrect Answer ImageD.No treatment indicated
 Incorrect Answer ImageE.Trimethoprim-sulfamethoxazole

A 34-year-old woman comes to the physician because of lower abdominal pain which occurs
during intercourse and urination. She has to urinate more frequently than usual and never feels
she has completely emptied her bladder. The patient has had no fevers or chills. She has a history
of HIV infection which is well-controlled with combination antiretroviral therapy. The patient
has no known drug allergies. She is sexually active with 2 partners and uses condoms most of the
time. Temperature is 37.0ºC (98.6ºF), blood pressure is 110/70 mm Hg, and pulse is 96/min. A
speculum examination shows no discharge or lesions. A bimanual pelvic examination shows
suprapubic tenderness. Urinalysis is positive for leukocyte esterase and nitrites. Which of the
following is the most appropriate initial pharmacotherapy?
 Incorrect Answer ImageA.Intramuscular ceftriaxone and oral doxycycline
 Incorrect Answer ImageB.Intravenous levofloxacin and vancomycin
 Incorrect Answer ImageC.Intravesical lidocaine and sodium bicarbonate
 Correct Answer ImageD.Oral trimethoprim and sulfamethoxazole
 Incorrect Answer ImageE.Vaginal clindamycin and probiotics

A 47-year-old woman comes to the physician because of multiple episodes of urinary


incontinence each day. The episodes follow an overwhelming need to urinate that is so strong
that the patient is unable to reach the bathroom in time, and as a result she is afraid to leave her
home. She has had no burning with urination or blood in her urine. The patient has a history of
hypertension for which she takes an angiotensin converting enzyme inhibitor, and narrow-angle
glaucoma for which she takes a topical prostaglandin. She has smoked 1 pack of cigarettes daily
for 23 years and drinks 2 cups of caffeinated coffee a day. BMI is 38 kg/m2. Blood pressure is
128/88 mm Hg. Bimanual and speculum examinations are normal. Urinalysis is within normal
limits and urine culture is negative. Which of the following is the most appropriate next step in
management?
 Correct Answer ImageA.Bladder training and kegel exercises
 Incorrect Answer ImageB.Pessary placement and caffeine restriction
 Incorrect Answer ImageC.Self-catheterization and weight loss
 Incorrect Answer ImageD.Tolterodine and smoking cessation
 Incorrect Answer ImageE.Vaginal estrogen and water restriction

A 24-year-old woman, gravida 1, para 0, comes to the emergency department 7 days after an
elective abortion with suction curettage because of abdominal pain, fever, nausea, and vomiting.
She also reports a foul-smelling vaginal discharge and heavy bleeding. She was given a
prescription for doxycycline, but she developed a rash the second day and stopped taking the
medication. She has a history of chlamydia infection 5 years ago that was treated. Her
temperature is 39.4ºC (102.9ºF), pulse is 120/min, respirations are 14/min, and blood pressure is
82/50 mm Hg. She appears to be in moderate distress. Abdominal examination shows
generalized abdominal tenderness with guarding and rebound. Pelvic examination shows
minimal dark red blood with foul-smelling discharge. There is no cervical motion tenderness.
Transvaginal ultrasound reveals an endometrial thickness of 7 mm with no obvious intrauterine
mass. The results of further tests are pending. Which of the following is the most likely
diagnosis?
 Incorrect Answer ImageA.Endometritis and sepsis with retained products of conception

 Correct Answer ImageB.Endometritis and sepsis without retained products of conception

 Incorrect Answer ImageC.Hematometra

 Incorrect Answer ImageD.Isolated endometritis

 Incorrect Answer ImageE.Pelvic inflammatory disease

An 18-year-old woman comes to the physician because of extreme irritability prior to her period.
During these times, she is tearful and unable to concentrate. Her appetite increases and she
craves chocolate and sweets. She also has headache, dizziness, breast tenderness, abdominal
bloating, and gastrointestinal symptoms. The symptoms resolve with menses. This has happened
each month for the past 12 months. Which of the following is the most common physical
symptom of this patient's condition? 
 Correct Answer ImageA.Abdominal bloating
 Incorrect Answer ImageB.Dizziness
 Incorrect Answer ImageC.Food cravings
 Incorrect Answer ImageD.Headache
 Incorrect Answer ImageE.Increased appetite
 Incorrect Answer ImageF.Labile mood

A 61-year-old woman comes to the physician because of a 2-week history of vaginal bleeding.
Her last menstrual period was 8 years ago. She has had no bleeding since then, until this most
recent episode. Her past medical history is significant for hypertension, for which she takes an
ACE inhibitor. Her last Pap smear was a year ago, which was normal. She had an appendectomy
at 21 years of age. She has no known drug allergies. Her temperature is 37.1°C (98.8°F), pulse is
80/min, respirations are 12/min and blood pressure is 130/80 mm Hg. Physical examination
shows a mildly obese abdomen with no abdominal tenderness. Pelvic examination shows a
uterus consistent in size with 10 weeks of gestation. The remainder of the physical examination
is unremarkable. Which of the following is the most appropriate next step in management?
 Incorrect Answer ImageA.Initiate hormone replacement therapy
 Incorrect Answer ImageB.Perform a colposcopy
 Correct Answer ImageC.Perform an endometrial biopsy
 Incorrect Answer ImageD.Schedule a hysterectomy
 Incorrect Answer ImageE.Start the patient on the oral contraceptive pill

A 21-year-old woman comes to the physician for an annual examination. She has been sexually
active since age 19 and has not had a Pap smear or pelvic examination. She is currently sexually
active with 2 male partners and 1 female partner, and reports that she uses condoms
intermittently. She has no medical problems and takes no medications. Her examination is
unremarkable. Her Pap smear shows no atypical cells and has satisfactory epithelial cells but has
an absence of endocervical cells. The Pap smear is otherwise within normal limits. Which of the
following is the most appropriate next step in management?
 Incorrect Answer ImageA.Colposcopy with colposcopically-directed biopsies
 Incorrect Answer ImageB.Cryotherapy on the cervix
 Incorrect Answer ImageC.Laparoscopy with laparoscopically-directed biopsies
 Correct Answer ImageD.Repeat Pap smear in 3 years
 Incorrect Answer ImageE.Repeat Pap smear in 6 months

A 61-year-old woman comes to the physician because of pelvic fullness and progressive,
involuntary loss of urine for 1 year. The urine loss is usually during the day, but recently she has
lost urine while asleep as well. She has had a 4.5-kg (10-lb) weight gain over the past year. She
has a 15-year history of type 2 diabetes mellitus poorly controlled with insulin. Physical
examination shows poor sensation to light touch on both heels. Pelvic examination shows no
abnormalities. Ultrasonography of the bladder after micturition shows increased residual volume.
Which of the following is the most likely diagnosis?
 Incorrect Answer ImageA.Hypertonic bladder
 Correct Answer ImageB.Hypotonic bladder
 Incorrect Answer ImageC.Menopausal hormonal changes
 Incorrect Answer ImageD.Stress incontinence
 Incorrect Answer ImageE.Urethral diverticulum

A 22-year-old woman, gravida 4, para 3, at 38 weeks' gestation, comes to the labor and delivery
ward because of a gush of transvaginal secretion. Speculum examination shows a pool of fluid
that is Nitrazine-positive and forms ferns when viewed under the microscope. The fetal heart rate
is 150/min and reactive. An ultrasound shows the fetus in the breech position. A cesarean
delivery is performed. During the operation, the physician, who has received routine
immunizations for healthcare providers, is stuck with a needle that had been used on the patient.
Which of the following is this physician at greatest risk of contracting?
 Incorrect Answer ImageA.Group B streptococci bacteremia
 Incorrect Answer ImageB.Hepatitis B
 Correct Answer ImageC.Hepatitis C
 Incorrect Answer ImageD.HIV
 Incorrect Answer ImageE.Human T-lymphotropic retrovirus I (HTLV I)

A 21-year-old woman is brought to the emergency department because of fever, nausea, and
lower abdominal pain that has worsened over the last 2 days. Her medical history is significant
for a gonorrhea infection 2 years ago. She has never had surgery. She takes no medications and
is allergic to penicillin. Her temperature is 38.3ºC (101ºF), blood pressure is 116/78 mm Hg,
pulse is 112/min, and respirations are 12/min. Abdominal examination shows moderate lower
abdominal tenderness. Pelvic examination with a speculum shows purulent discharge coming
from the cervical os. The patient has bilateral adnexal tenderness, worse on the right. Pelvic
ultrasound shows a 3-cm right ovarian mass showing a single, cystic region. Urine β-hCG is
negative. Which of the following is the most appropriate next step in management?
 Correct Answer ImageA.Admit the patient for intravenous antibiotics
 Incorrect Answer ImageB.Ceftriaxone and oral doxycycline for 14 days
 Incorrect Answer ImageC.Outpatient treatment with clindamycin and gentamicin
 Incorrect Answer ImageD.Schedule for laparoscopic visualization of right adnexa
 Incorrect Answer ImageE.Single-dose oral azithromycin and cefixime

A 45-year-old woman comes to the physician because of a 1-year history of involuntary loss of
urine after experiencing a sudden need to urinate. She denies episodes of incontinence with
coughing or sneezing. She has never been pregnant. Pelvic examination shows a normal-
appearing vulva, vagina, and cervix. There is no prolapse, and bimanual examination shows a
normal size uterus with no masses. Neurologic examination shows normal pain, temperature, and
vibratory sensation and 5/5 motor strength in all extremities. Her reflexes are 2+ in the upper and
lower extremities. Fasting serum glucose is 95 mg/dL and hemoglobin A1c is 5.2%. Urinalysis is
within normal limits. Urine culture shows no growth of bacteria. Which of the following is the
most appropriate next step in diagnosis?
 Correct Answer ImageA.Complete history of lifestyle, medications, and diet
 Incorrect Answer ImageB.CSF analysis for oligoclonal bands
 Incorrect Answer ImageC.Cystourethroscopy
 Incorrect Answer ImageD.Fluoroscopy
 Incorrect Answer ImageE.Postvoid residual volume
 Incorrect Answer ImageF.Provocative stress testing

A 32-year-old woman, gravida 2, para 1, at 26 weeks’ gestation comes to the physician for a
routine prenatal visit. Fetal movement has been normal. The patient has had no abdominal pain,
vaginal bleeding, or leakage of fluid. The patient’s routine prenatal laboratory testing and
ultrasounds have been normal. Her blood type is O, Rh positive. Her first pregnancy ended with
a spontaneous vaginal delivery of a male infant at 39 weeks’ gestation who developed jaundice
due to hemolysis and was treated with antibiotics for Group B Streptococcus sepsis. The patient
takes a daily prenatal vitamin. She has no chronic medical problems and has had no previous
surgery. She works as an elementary school teacher and lives with her husband and son. Vital
signs are within normal limits. The fetal heart rate is 155/min. A symphysis-fundal height
measurement is 25 cm. A urinalysis dipstick shows no glucose and trace protein. Which of the
following is the most appropriate next step in the management of this patient?
 Correct Answer ImageA.1-hour glucose challenge test
 Incorrect Answer ImageB.24-hour urine for total protein
 Incorrect Answer ImageC.Rectovaginal bacterial culture
 Incorrect Answer ImageD.Red blood cell antibody screen
 Incorrect Answer ImageE.Repeat sexually transmitted infection screening

A 28-year-old primigravid woman at 41 weeks’ gestation comes to the emergency department


because of contraction pains. She is admitted to the hospital due to labor, and has an
uncomplicated vaginal delivery of a healthy 3750-g (8-lb 4-oz) male infant. Apgar scores are 9
and 9 at 1 and 5 minutes, respectively. The woman is Rh-negative and received anti-D immune
globulin (Rh IgG) at 10 weeks’ gestation due to vaginal bleeding and again at 28 weeks’
gestation. Which of the following is the most appropriate next step in the management of this
patient?
 Incorrect Answer ImageA.Administer Rho(D) immune globulin
 Incorrect Answer ImageB.Perform Coombs test
 Incorrect Answer ImageC.Perform Kleihauer-Betke test
 Correct Answer ImageD.Perform neonatal blood type
 Incorrect Answer ImageE.Perform rosette test

A 54-year-old woman comes to the physician because of hot flashes that have been progressive
over the last year since her total abdominal hysterectomy and bilateral salpingo-oophorectomy
for menometrorrhagia. Pathology from the surgery showed low-grade endometrial hyperplasia.
She has no medical problems and takes no medications. Her family history is unremarkable
except for a strong family history of osteoporosis. She states that the hot flashes have become
absolutely debilitating for her and she wants to take something that will give her the best chance
of stopping them. Which of the following is the most appropriate pharmacotherapy? 
 Incorrect Answer ImageA.Escitalopram
 Correct Answer ImageB.Estrogen alone
 Incorrect Answer ImageC.Estrogen combined with progesterone
 Incorrect Answer ImageD.Progesterone alone
 Incorrect Answer ImageE.Raloxifene

A 24-year-old woman comes to the physician for an initial prenatal visit. Her last menstrual
period was 7 weeks ago and a home urine pregnancy test was positive. She has had no bleeding
or abdominal pain. She reports increased fatigue and some mild nausea and vomiting. She takes
prenatal vitamins and has no known drug allergies. Physical examination shows a diastolic
murmur. The uterus is appropriate in size and nontender. Which of the following is the most
appropriate next step in management of this patient's murmur?
 Correct Answer ImageA.Echocardiography
 Incorrect Answer ImageB.Electrocardiogram (ECG)
 Incorrect Answer ImageC.Exercise ECG test
 Incorrect Answer ImageD.Iron supplementation
 Incorrect Answer ImageE.Reassurance

A 23-year-old woman comes to the physician because of a swelling in her vulva that has been
progressive over the last 3 days. The swelling is not painful, but it is uncomfortable when she
jogs. She has asthma for which she uses an albuterol inhaler, but no other medical problems.
Pelvic examination shows a cystic mass 4 cm in diameter near the hymen by the patient's left
labia minora. The mass is nontender and there is no associated erythema. The mass is freely
mobile. The rest of the pelvic examination is unremarkable. Which of the following is the most
likely diagnosis? 
 Correct Answer ImageA.Bartholin cyst
 Incorrect Answer ImageB.Condyloma lata
 Incorrect Answer ImageC.Granuloma inguinale
 Incorrect Answer ImageD.Hematocolpos
 Incorrect Answer ImageE.Vulvar cancer

A 43-year-old woman comes to the physician because of a 6-month history of irregular periods.
She also has whitish nipple discharge from both breasts. Her last menstrual period was 3 months
ago. She is sexually active with her husband, and they use condoms consistently. She denies
vigorous exercise. She does not smoke or drink alcohol. She takes no medications. Her
temperature is 37°C (98.6°F) and her blood pressure is 130/75 mm Hg. Breast examination
shows no masses, and a moderate amount of whitish discharge is expressed from her nipples.
Pelvic examination shows no abnormalities. Urine human chorionic gonadotropin (β-hCG) test is
negative. Serum prolactin level is 40 ng/mL (normal, <20 ng/mL) and serum TSH level is 3
mIU/L. Further laboratory testing is most likely to show which of the following?
 Incorrect Answer ImageA.High antidiuretic hormone (ADH)
 Incorrect Answer ImageB.High luteinizing hormone (LH)
 Incorrect Answer ImageC.Low adrenocorticotropic hormone (ACTH)
 Correct Answer ImageD.Low follicle-stimulating hormone (FSH)
 Incorrect Answer ImageE.Low growth hormone (GH)

A 23-year-old primigravid woman at 28 weeks' gestation comes to the physician for a routine
prenatal visit. Fetal movement has been normal. The pregnancy has been complicated by
persistent nausea and vomiting which has been partially responsive to oral antiemetics and, as a
result, the patient has inadequate weight gain. The patient’s mother has diabetes mellitus and her
father has sickle-cell anemia. Vital signs are within normal limits. The fetal heart rate is 140/min.
Third trimester routine laboratory studies shows:
1-hour glucose test 125 mg/dL
Hematocrit 30%
Hemoglobin 11.2 g/dL
Mean corpuscular volume 87 fL
Platelets 154,000/mm3
Which of the following is the most likely cause for this patient’s laboratory test results?
 Incorrect Answer ImageA.Alloimmunization
 Correct Answer ImageB.Expanded plasma volume
 Incorrect Answer ImageC.Inadequate folate intake
 Incorrect Answer ImageD.Inadequate iron intake
 Incorrect Answer ImageE.Mutation of beta-globin gene

A 35-year-old woman, gravida 2, para 1, at 18 weeks’ gestation is brought to the emergency


department after a motor vehicle accident where she collided with an oncoming tractor-trailer.
She was found to be unconscious at the scene. Upon arrival her temperature is 37.2ºC (99.0ºF),
blood pressure is 110/55 mm Hg, pulse is 124/min, and respirations are 12/min. Pulse oximetry
on room air shows an oxygen saturation of 96%. She is conscious, although oriented only in
person, not time or place. Physical examination shows bruises over the patient's forehead with
tenderness to palpation over left frontal and parietal bones, chest auscultation shows clear lungs
and no heart murmurs or rubs, the abdomen is moderately tender throughout with no localized
tenderness or rebound. There is no vaginal bleeding, fluid leakage, or uterine tenderness. Fetal
heart rate is 150/min with moderate variability. Which of the following is the most appropriate
next step in management?
 Correct Answer ImageA.CT scan of the head without contrast
 Incorrect Answer ImageB.CT scan of the abdomen with contrast
 Incorrect Answer ImageC.Emergent cesarean delivery
 Incorrect Answer ImageD.Fetal heart rate monitoring for 24 hours
 Incorrect Answer ImageE.MRI of the brain
 Incorrect Answer ImageF.Pelvic ultrasonography

A 22-year-old woman, gravida 1, para 0, at 12 weeks’ gestation comes to the


emergency department because of vaginal bleeding. She has had an uneventful pregnancy so far.
This morning, while in the bathroom, she noticed a moderate amount of vaginal bleeding. There
were no associated contractions, pain, or abdominal cramping. Her blood pressure is 128/72 mm
Hg, pulse is 72/min, respirations are 20/min, and temperature is 37ºC (98.6ºF). Speculum
examination shows a small amount of clotted blood in the vaginal vault and surrounding the os
with no active bleeding or abnormal tissue; the cervix is closed and the uterus is consistent in
size with 12 weeks of gestation. Which of the following is the most likely diagnosis?
 Incorrect Answer ImageA.Complete abortion
 Incorrect Answer ImageB.Incomplete abortion
 Incorrect Answer ImageC.Inevitable abortion
 Incorrect Answer ImageD.Missed abortion
 Correct Answer ImageE.Threatened abortion

A 29-year-old woman previously diagnosed with fibrocystic disease of the breast comes to the
physician for follow-up of a lump in her right breast. The patient first noticed the lump 4 months
ago. It was aspirated at that time and cytology was negative, but the cyst recurred approximately
1 month later. The cyst was re-aspirated 2 months ago and again the cytology was negative. The
lump has now recurred. Examination shows a mass at the 10 o'clock position, approximately 4
cm from the areola. Ultrasonography shows a cystic lesion. Which of the following is the most
appropriate next step in management?
 Correct Answer ImageA.Excisional biopsy
 Incorrect Answer ImageB.Mammography in 6 months
 Incorrect Answer ImageC.Repeat FNA in 2 months
 Incorrect Answer ImageD.Tamoxifen therapy
 Incorrect Answer ImageE.Ultrasound in 6 months

A 25-year-old woman comes to the physician because of progressive pain and burning with
urination over the last 2 days. She denies fever or chills and has never had these symptoms
before. She has hypothyroidism for which she takes thyroid hormone replacement. She has no
other medical or surgical history. Her temperature is 37°C (98.6°F), blood pressure is 122/78 mm
Hg, pulse is 72/min, and respirations are 14/min. Physical examination is unremarkable,
including a normal pelvic examination. KOH and normal saline "wet prep" performed on her
vaginal discharge is negative. Urinalysis shows numerous white blood cells. Which of the
following is the most likely pathogen? 
 Correct Answer ImageA.Escherichia coli
 Incorrect Answer ImageB.Neisseria gonorrhoeae
 Incorrect Answer ImageC.Pseudomonas species
 Incorrect Answer ImageD.Staphylococcus saprophyticus
 Incorrect Answer ImageE.Trichomonas vaginalis

A 36-year-old woman, gravida 4, para 3, aborta 1, comes to the physician because of irregular
and intermittent vaginal bleeding for 5 months. She denies pain. She uses 6–8 pads per day
between menstrual periods. Prior to these symptoms, she had regular menstrual cycles since age
14. Last year she underwent tubal ligation confirmed by a hysterosalpingogram which showed
normal anatomy. She had an appendectomy at age 22. Her temperature is 37.2ºC (99.0ºF), blood
pressure is 115/75 mm Hg, pulse is 86/min, and respirations are 14/min. Pelvic examination
shows a normal-appearing vulva, vagina, and cervix. There is dark blood in the vagina. The
uterus is non-enlarged and nontender, and there are no adnexal masses. Pelvic ultrasonography
shows no visible abnormalities. Laboratory studies show:
Hemoglobin  9.6 g/dL
Hematocrit  30%
Leukocyte count  9,700/mm3
Neutrophils  60%
Lymphocytes  24%
Platelets  220,000/mm3
β-hCG  3 mIU/mL (negative, <5
mIU/mL)
Prothrombin time  12 seconds
Partial thromboplastin time 35 seconds
(activated) 
Follicle-stimulating hormone  28 mIU/mL (normal, 5-30
mIU/mL)
Luteinizing hormone  25 mIU/mL (normal, 5-30
mIU/mL)
Which of the following is the most likely diagnosis? 
 Correct Answer ImageA.Anovulatory uterine bleeding
 Incorrect Answer ImageB.Adenomyosis
 Incorrect Answer ImageC.Endometrial cancer
 Incorrect Answer ImageD.Leiomyomas
 Incorrect Answer ImageE.Vaginal atrophy

A 22-year-old woman, gravida 1, para 0, at 39 weeks’ gestation is brought to the emergency


department in active labor. She has had regular painful contractions since last night and her
membranes ruptured 8 hours ago. Her pregnancy course was unremarkable, but she hasn’t been
in her physician’s office for the last two months. She has a past medical history significant for a
mitral valve prolapse with regurgitation that was demonstrated on echocardiography. She takes
no medications and has no allergies. Her temperature is 37.2ºC (98.9ºF), blood pressure is 120/70
mm Hg, pulse is 80/min, and respirations are 14/min. Physical examination shows that her cervix
is 4 cm dilated and 50% effaced; vertex is at -1 station. The fetal heart rate is 150/min with
moderate variability. Which of the following is the most appropriate management of this patient?
 Incorrect Answer ImageA.Administer IV antibiotics 30 minutes prior to the delivery
 Incorrect Answer ImageB.Administer IV antibiotics 6 hours after the delivery
 Incorrect Answer ImageC.Administer IV antibiotics after the cord is clamped
 Incorrect Answer ImageD.Administer IV antibiotics throughout labor
 Correct Answer ImageE.Antibiotic prophylaxis is not necessary

A 36-year-old woman, gravida 1, para 0, at 36 weeks' gestation comes to the physician for a
prenatal visit. She is experiencing good fetal movement and denies loss of fluid, bleeding from
the vagina, or contractions. She has no complaints. She has mitral stenosis, mild persistent
asthma for which she uses an inhaled steroid daily and an albuterol inhaler as needed, and genital
herpes outbreaks approximately once a year. At her last visit, she was found to be positive for
Group B Streptococcus. For which of the following conditions would she most likely benefit
from a forceps-assisted vaginal delivery?
 Incorrect Answer ImageA.Asthma
 Incorrect Answer ImageB.Group B Streptococcus (GBS) colonization
 Incorrect Answer ImageC.Herpes
 Correct Answer ImageD.Mitral stenosis
 Incorrect Answer ImageE.This patient would not benefit from a forceps-assisted vaginal
delivery

A 33-year-old woman comes to the physician for her first prenatal visit. Her last menstrual
period was 7 weeks ago. She has had no bleeding or abdominal pain. She has no medical
problems and takes no medications. She has no family history of congenital anomalies. Her
husband is 55 years old. He is in good health and also has no family history of birth defects. The
patient is concerned that her husband's age may place their fetus at increased risk of a
chromosomal anomaly. She wishes to know the paternal age above which amniocentesis or
chorionic villus sampling should be considered. Which of the following is most appropriate? 
 Incorrect Answer ImageA.Age >30
 Incorrect Answer ImageB.Age >35
 Incorrect Answer ImageC.Age >40
 Incorrect Answer ImageD.Age >45
 Correct Answer ImageE.There is no age cutoff for paternal risk

A 65-year-old woman comes to the physician because of vaginal bleeding over the last month.
Her last menstrual period was at age 50 years and she denies bleeding since. She has no medical
problems and takes no medications. She is not sexually active. Pelvic examination shows normal
appearing vaginal mucosa and cervix. An endometrial biopsy is performed. The patient has
moderate cramping during and after the procedure but feels well enough to go home. Later that
night, with her abdominal pain worsening, the patient comes to the emergency department. An
ultrasound is performed that shows a normal uterus and ovaries with a complex fluid collection
posterior to the uterus. Which of the following is the most likely diagnosis? 
 Incorrect Answer ImageA.Bowel perforation
 Incorrect Answer ImageB.Endometrial cancer
 Incorrect Answer ImageC.Endometritis
 Incorrect Answer ImageD.Tubo-ovarian abscess
 Correct Answer ImageE.Uterine perforation
A 21-year-old woman, gravida 2, para 1, at 22 weeks' gestation comes to the physician because
of a 2-day history of progressive malodorous vaginal discharge. Her prenatal course has been
unremarkable during this pregnancy. Her prior pregnancy was complicated by preterm labor and
delivery at 31 weeks' gestation. Pelvic examination shows a grayish vaginal discharge. A strong
amine odor is released when KOH is applied to a sample of the discharge. Examination of a
normal saline preparation shows numerous granular-appearing vaginal epithelial cells that are
coated with coccobacillary organisms. Which of the following is the most appropriate
pharmacotherapy? 
 Incorrect Answer ImageA.Intramuscular penicillin
 Incorrect Answer ImageB.No treatment is needed
 Correct Answer ImageC.Oral metronidazole
 Incorrect Answer ImageD.Oral penicillin
 Incorrect Answer ImageE.Topical metronidazole

A 74-year-old woman comes to the physician because of progressive itching and irritation on her
vulva for the past 2 years. She has been seen by several doctors during this time and has been
treated with antifungal creams and topical steroids, which have not provided any relief. Her past
medical history is significant for hypertension, for which she takes an angiotensin-converting
enzyme (ACE) inhibitor. Her past obstetric history is significant for two successful cesarean
deliveries in her thirties. She has no known drug allergies. Her temperature is 37.1°C (98.8°F),
pulse is 80/min, respirations are 12/min, and blood pressure is 130/80 mm Hg. Pelvic
examination shows numerous excoriations with scarring on the labia majora. Speculum
examination shows a normal vaginal discharge. Microscopic examination of the vaginal
discharge shows absent clue cells, trichomonads and hyphae. The remainder of the physical
examination is normal. Which of the following is the most appropriate next step in management?
 Incorrect Answer ImageA.Clotrimazole cream
 Incorrect Answer ImageB.Metronidazole gel
 Incorrect Answer ImageC.Re-examination in 1 year
 Correct Answer ImageD.Vulvar biopsy
 Incorrect Answer ImageE.Vulvectomy

A 29-year-old woman, gravida 1, para 0, comes to the emergency department because of vaginal
bleeding and lower abdominal pain. The symptoms started approximately 12 hours ago and have
progressively worsened. Since the onset of the pain, the patient has passed several clots. Her last
menstrual period was 6 weeks ago. She takes no medications, has no chronic medical problems,
and has had no prior surgery. Blood pressure is 106/54 mm Hg and pulse is 104/min. Speculum
examination shows a scant amount of blood in the posterior vaginal fornix and no active
bleeding. Pelvic examination shows a 1 cm dilated cervix, a normal uterus, and no adnexal
masses or tenderness. The quantitative beta-hCG is 2,564 mIU/mL. Hemoglobin is 10.8 g/dL.
Transvaginal ultrasound shows an empty endometrial cavity. Which of the following is the most
likely cause of this patient’s condition?
 Incorrect Answer ImageA.Cervical insufficiency
 Correct Answer ImageB.Chromosomal abnormalities
 Incorrect Answer ImageC.Dispermic fertilization
 Incorrect Answer ImageD.Ectropion of the cervix
 Incorrect Answer ImageE.Extrauterine embryonic implantation
A 38-year-old woman, gravida 3, para 2, at 10 weeks' gestation comes to the physician for
prenatal care. She is feeling well with only some occasional nausea. Her past obstetric history is
notable for 2 spontaneous term vaginal deliveries following uncomplicated pregnancies. Her past
medical history is significant for hypertension, for which she takes nifedipine. She has never had
surgery and has no known drug allergies. Her initial blood pressure is 144/92 mm Hg; it is
140/90 mm Hg when repeated 5 minutes later. The remainder of her physical examination is
normal for a woman at 10 weeks' gestation. Two weeks later she returns for a blood pressure
check and is found to have a blood pressure of 142/92 mm Hg. Which of the following is the
most appropriate next step in management?
 Correct Answer ImageA.Continue to monitor blood pressures
 Incorrect Answer ImageB.Start the patient on alpha-methyldopa
 Incorrect Answer ImageC.Start the patient on atenolol
 Incorrect Answer ImageD.Start the patient on captopril
 Incorrect Answer ImageE.Start the patient on labetalol

A 34-year-old woman, gravida 2, para 1, at 26 weeks' gestation comes to the emergency


department because of painless vaginal bleeding that started 1 hour ago. She was having sexual
intercourse with her husband when she saw a small amount of bright red blood on the sheets. Her
past medical history is unremarkable. Her temperature is 37ºC (98.6ºF), blood pressure is 110/67
mm Hg, pulse is 87/min, and respirations are 12/min. The fetal heart rate is 150/min. Physical
examination shows fundal height of 26 cm, nontender uterus, and blood on the patient’s
perineum and underwear. Which of the following is the most appropriate next step in
management?
 Correct Answer ImageA.Abdominal ultrasonography
 Incorrect Answer ImageB.CT scan of the abdomen
 Incorrect Answer ImageC.Digital examination
 Incorrect Answer ImageD.Immediate caesarean delivery
 Incorrect Answer ImageE.Vaginal ultrasonography

A 30-year-old woman comes to the physician because of a 3-year history of infertility. She has
not used contraception with her husband for the past 3 years. She has had irregular vaginal
spotting during this time, ever since she stopped taking her oral contraceptive pills. She waxes
her upper lip, chin, chest, and back every week. The patient weighs 77 kg (170 lb) and is 160 cm
(5 ft 3 in) tall. Vital signs are within normal limits. Skin examination of the axilla is shown.
There is also mild papulopustular facial acne. Pelvic examination shows normal external
genitalia, a small, normal-appearing cervix, and a small uterus. Which of the following findings
is most likely in this patient?
 Incorrect Answer ImageA.Low plasma cortisol
 Correct Answer ImageB.Low sex hormone-binding globulin
 Incorrect Answer ImageC.Luteinizing hormone:follicle-stimulating hormone ratio of 1:2
 Incorrect Answer ImageD.Single ovarian cyst
 Incorrect Answer ImageE.Small ovaries

A 38-year-old woman comes to the physician because of a burning sensation during urination
and vaginal discharge for 2 days. She has no urinary frequency or urgency. The patient had a
spontaneous vaginal delivery 3 months ago and has been exclusively breastfeeding her infant.
She has been sexually active for the past 6 weeks with her husband and has used only an
intrauterine device for contraception. Vital signs are within normal limits. External genital
examination shows excoriations on the labia majora. Sterile speculum examination shows the
vaginal mucosa to be erythematous and covered with a thick, white discharge. Testing of the
discharge shows a pH of 4.0. Which of the following is the most likely diagnosis?
 Incorrect Answer ImageA.Bacterial vaginosis
 Correct Answer ImageB.Candida vulvovaginitis
 Incorrect Answer ImageC.Chlamydia cervicitis
 Incorrect Answer ImageD.Trichomoniasis
 Incorrect Answer ImageE.Vaginal atrophy

A 54-year-old woman comes to the physician because of occasional vaginal bleeding over the
last 2 months. She underwent menopause 2 years ago. At her last visit 6 months ago, she was
prescribed tamoxifen for breast cancer prevention because of a strong family history of breast
cancer. She has no history of major medical illness and takes no medications. Pelvic examination
shows a small amount of blood at the cervical os. Bimanual examination discloses no masses. An
endometrial biopsy specimen shows atypical hyperplasia. Which of the following is the most
appropriate next step in management?
 Correct Answer ImageA.Discontinue the tamoxifen
 Incorrect Answer ImageB.Increase the tamoxifen dose
 Incorrect Answer ImageC.Repeat the endometrial biopsy in 6 months
 Incorrect Answer ImageD.Schedule a pelvic ultrasound
 Incorrect Answer ImageE.Switch the patient to estrogen

A 22-year-old woman, gravida 1, para 0, at 8 weeks' gestation comes to the physician for her
first prenatal visit. She has had some nausea but no other complaints. She has had no bleeding
per vagina or abdominal pain. She had an ovarian cystectomy at age 18, but no other medical or
surgical problems. She takes no medications and has no known drug allergies. Physical
examination is unremarkable except for a non-tender uterus consistent in size with an 8-week
gestation. The patient wants information on vitamin supplementation during pregnancy. Which
of the following represents the amount of vitamin A supplementation this patient should take
daily?
 Incorrect Answer ImageA.10,000 IU
 Incorrect Answer ImageB.25,000 IU
 Incorrect Answer ImageC.50,000 IU
 Incorrect Answer ImageD.100,000 IU
 Correct Answer ImageE.Vitamin A supplementation during pregnancy is not
recommended

A 22-year-old woman, gravida 1, para 0, at 39 weeks’ gestation comes to the labor and delivery
ward because of painful contractions every 2 minutes. She has had no gush of fluid and no
bleeding from the vagina. Her prenatal course was unremarkable. She takes no medications and
has no known drug allergies. Physical examination shows that her cervix is 6 cm dilated and
100% effaced; the vertex is at 0 station. The fetal heart rate is 150/min and the tracing is reactive.
The patient desires an epidural for pain relief. Which of the following should be given orally
shortly before the epidural is placed?
 Correct Answer ImageA.Antacid
 Incorrect Answer ImageB.Antibiotic
 Incorrect Answer ImageC.Aspirin
 Incorrect Answer ImageD.Clear liquid meal
 Incorrect Answer ImageE.Regular meal

A 26-year-old woman, gravida 3, para 2, at 39 weeks’ gestation comes to the emergency


department in active labor after a large gush of fluid 2 hours ago. She has had no prenatal care
and no significant medical history. Physical examination shows the cervix is 3 cm dilated and
50% effaced; the vertex is -2 station. Fetal heart rate monitoring shows a baseline of 150/min
with moderate variability and no decelerations. Repeat evaluation in 4 hours shows the cervix is
9 cm dilated and 100% effaced; the vertex is 0 station. Fetal heart rate monitoring shows abrupt
decreases in fetal heart rate of 20/min below the baseline that last for 15–20 seconds with no
relation to uterine contractions. Which of the following is the most likely cause of the fetal heart
rate pattern?
 Incorrect Answer ImageA.Fetal head compression
 Incorrect Answer ImageB.Intrauterine infection
 Incorrect Answer ImageC.Normal fetal well-being
 Correct Answer ImageD.Umbilical cord compression
 Incorrect Answer ImageE.Uteroplacental insufficiency

A 36-year-old woman, gravida 2, para 2, comes to the physician because of a 3-year history of
progressively worsening lower abdominal pressure. She reports a history of heavy and regular
menses during this time. She has no history of medical illnesses. Her past obstetric history is
significant for 2 normal spontaneous vaginal deliveries. Current medications include ibuprofen,
which provides partial relief of her symptoms. Her temperature is 37.1°C (98.8°F), pulse is
80/min, respirations are 12/min, and blood pressure is 130/80 mm Hg. Pelvic examination shows
an enlarged, nontender uterus with an irregular contour. The remainder of the physical
examination is unremarkable. Laboratory studies show:
Hematocrit 30%
Leukocyte count 8,500/mm3
Platelets 210,000/mm3
Urine β-hCG Negative
Pelvic ultrasound shows an enlarged uterus with 2 intramural 6-cm fibroids. The patient requests
relief from symptoms; however, she is not sure if she is done with childbearing. Which of the
following is the most appropriate next step in management?
 Incorrect Answer ImageA.Expectant management
 Incorrect Answer ImageB.Hysterectomy
 Incorrect Answer ImageC.Hysteroscopy
 Correct Answer ImageD.Myomectomy
 Incorrect Answer ImageE.Oral contraceptive pills (OCPs)

A 21-year-old woman comes to the physician because of several days of malodorous vaginal
discharge and dysuria. She takes no medications and has no chronic medical problems. Vital
signs are within normal limits. Physical examination shows normal external genitalia. Speculum
examination shows a greenish, frothy, malodorous vaginal discharge and petechial lesions on the
cervix. Bimanual examination is normal. Microscopic evaluation of the vaginal discharge is most
likely to show which of the following?
 Incorrect Answer ImageA.𝛃-hemolytic gram-positive cocci in pairs and chains
 Incorrect Answer ImageB.Gram-negative cocci in pairs
 Correct Answer ImageC.Motile, flagellated, unicellular organisms
 Incorrect Answer ImageD.Pseudohyphae, hyphae, or buds
 Incorrect Answer ImageE.Squamous cells coated with coccobacilli

A 52-year-old woman comes to the physician because of hot flashes that have been occurring
over the last year and occur several times per day. Her last menstrual period was 1 year ago. Her
hot flashes awaken her at night and interfere with her sleep. She has difficulty getting through
her workday. She was prescribed low-dose paroxetine for these symptoms 4 months ago but she
does not report any improvement. She has no history of major medical illnesses. She has no
family history of cardiovascular disease, breast cancer, or dementia. She does not smoke
cigarettes. Physical examination is unremarkable. After a discussion about the risks and benefits
associated with hormone replacement therapy, she is given a prescription for estrogen and
medroxyprogesterone acetate. Which of the following is decreased by the addition of a
progestin? 
 Incorrect Answer ImageA.Breast cancer risk
 Incorrect Answer ImageB.Breast pain
 Correct Answer ImageC.Endometrial cancer risk
 Incorrect Answer ImageD.Mood changes
 Incorrect Answer ImageE.Weight gain

A 28-year-old woman, gravida 2, para 1, at 37 weeks’ gestation delivers dichorionic/diamniotic


twins via vaginal delivery after a 2-day induction of labor for preeclampsia. Shortly after the
placenta delivers, there is profuse vaginal bleeding. Her blood pressure is 95/40 mm Hg and
pulse is 120/min. Bimanual pelvic examination shows the uterus to be soft and flaccid. The
perineum, vagina, and cervix are examined and found to be intact. Bimanual uterine massage is
performed, however, the bleeding persists. Which of the following is the most appropriate next
step in management?
 Incorrect Answer ImageA.Intrauterine balloon tamponade
 Incorrect Answer ImageB.Methylergonovine injection
 Correct Answer ImageC.Oxytocin infusion
 Incorrect Answer ImageD.Tranexamic acid infusion
 Incorrect Answer ImageE.Uterine artery embolization

A 44-year-old woman, gravida 4, para 3, at 8 weeks' gestation comes to the physician for her
first prenatal visit. She has mild nausea and vomiting but no other complaints. Her obstetric
history is significant for 3 full-term, normal vaginal deliveries of normal infants. She has no
medical or surgical history and takes no medications. Physical examination reveals a uterus
consistent in size with 8-week gestation and is otherwise unremarkable. Given her age, she
wishes to have chromosomal testing of the fetus performed. Compared with amniocentesis,
chorionic villus sampling includes a greater risk for which of the following?
 Incorrect Answer ImageA.Accessory placental lobe
 Correct Answer ImageB.Fetal limb defects
 Incorrect Answer ImageC.Intellectual disability in the fetus
 Incorrect Answer ImageD.Membrane rupture
 Incorrect Answer ImageE.Third-trimester elective abortion

A 27-year-old woman, gravida 1, para 0, at 39 weeks' gestation comes to the labor and delivery
department with a gush of fluid and regular contractions. Physical examination shows that she
has grossly ruptured membranes, she is contracting every 2 minutes, and her cervix is 4 cm
dilated and 60% effaced. The fetal heart rate is 140/min with moderate variability. She is
admitted to labor and delivery, and over the following 4 hours she progresses to 9 cm dilation.
Over the past hour, the fetal heart rate has increased from a baseline of 140/min to a baseline of
160/min. There is minimal variability and moderate to severe variable decelerations are seen
with each contraction. The patient has been placed on her left side and supplemental oxygen is in
place. Which of the following is the most appropriate next step in management?
 Incorrect Answer ImageA.Forceps delivery
 Incorrect Answer ImageB.Intrauterine pressure catheter placement
 Incorrect Answer ImageC.Oxytocin administration
 Incorrect Answer ImageD.Scalp lead placement
 Correct Answer ImageE.Urgent cesarean section

A 48-year-old woman is referred to the breast subspecialty clinic by her primary care physician.
She has had pain and swelling in her right breast for six weeks. She was prescribed an antibiotic
course 2 weeks ago but her symptoms have not improved. She has not had any trauma to the
breast and no fever during this time. She has a 15-year-old daughter who was born via a cesarean
section, after which she had a tubal ligation. She has no family history of cancer. Her vital signs
are within normal limits. Physical examination shows the right breast noticeably larger in size
than the left breast with erythematous and warm-to-touch skin. However, there is no discreetly
palpable mass. The left breast appears normal and no masses are felt. Which of the following is
the most likely diagnosis?
 Incorrect Answer ImageA.Acute mastitis
 Correct Answer ImageB.Inflammatory cancer of the breast
 Incorrect Answer ImageC.Normal menopausal involutionary changes
 Incorrect Answer ImageD.Paget's disease of the breast
 Incorrect Answer ImageE.Pyogenic breast abscess

A 21-year-old woman comes to the physician because of "bumps" on her vulva that she has just
recently noticed. These bumps do not cause any symptoms, but she wants to know what they are
and wants them removed. She has no medical problems, takes no medications, and has no
allergies to medications. She smokes one half pack of cigarettes per day. She is sexually active
with 3 partners. Pelvic examination shows numerous cauliflower-like lesions on the labia majora.
Which of the following is the most appropriate next step in management?
 Incorrect Answer ImageA.Acyclovir
 Incorrect Answer ImageB.Cone biopsy
 Correct Answer ImageC.Cryotherapy
 Incorrect Answer ImageD.Penicillin
 Incorrect Answer ImageE.Vulvectomy

A 37-year-old woman, gravida 3, para 2, comes to her physician for follow-up on her ectopic
pregnancy. She was diagnosed with an ectopic pregnancy 7 days ago and given methotrexate.
She now presents with abdominal pain that started this morning. Examination is significant for
moderate left lower quadrant tenderness. Laboratory analysis shows that her beta-hCG value has
doubled over the past week. Transvaginal ultrasound shows that the ectopic pregnancy is roughly
the same size but there is an increased amount of fluid in the pelvis. Which of the following is
the most appropriate next step in management? 
 Incorrect Answer ImageA.Expectant management
 Incorrect Answer ImageB.Hysterectomy
 Correct Answer ImageC.Laparoscopy
 Incorrect Answer ImageD.Oophorectomy
 Incorrect Answer ImageE.Repeat methotrexate

A 21-year-old woman, gravida 2, para 1, at 28 weeks' gestation, comes to the physician because
of spotting after intercourse and a foul-smelling vaginal discharge. Her prenatal course has been
uncomplicated, and she has no medical problems. Speculum examination shows inflammation of
the cervix with a mucopurulent cervical discharge. A gonorrhea and Chlamydia test is
performed, which comes back positive for Chlamydia. Which of the following is the most
appropriate pharmacotherapy? 
 Correct Answer ImageA.Azithromycin
 Incorrect Answer ImageB.Doxycycline
 Incorrect Answer ImageC.Levofloxacin
 Incorrect Answer ImageD.Penicillin
 Incorrect Answer ImageE.Streptomycin

A 38-year-old woman, gravida 3, para 2, at 32 weeks' gestation comes to the physician because
of an episode of vaginal bleeding. She states that this morning she has passed 2 quarter-sized
blood clots from her vagina. Otherwise, she feels well. The baby has been moving normally and
she has had no contractions or gush of fluid from the vagina. Her obstetrical history is significant
for 2 previous low-transverse cesarean deliveries for nonreassuring fetal heart rate tracings.
Ultrasound examination shows a complete placenta previa. Which of the following conditions is
this patient at the highest risk of developing?
 Incorrect Answer ImageA.Dystocia
 Incorrect Answer ImageB.Intrauterine fetal demise (IUFD)
 Correct Answer ImageC.Placenta accreta
 Incorrect Answer ImageD.Preeclampsia
 Incorrect Answer ImageE.Uterine rupture

A 23-year-old woman, gravida 1, para 0, at 32 weeks' gestation is brought to the emergency


department after a major motor vehicle accident with severe, intermittent abdominal pain. She
was involved in a head-on collision with an oncoming drunk driver. Her blood pressure is 97/70
mm Hg, pulse is 119/min, and respirations are 15/min. Physical examination shows an awake,
alert woman in moderate pain and blood on the perineum with moderate uterine tenderness. The
fetal heart rate is 160/min with minimal variability. Which of the following is the most likely
diagnosis?
 Incorrect Answer ImageA.Mild placental abruption
 Correct Answer ImageB.Moderate placental abruption
 Incorrect Answer ImageC.Placenta previa
 Incorrect Answer ImageD.Severe placental abruption
 Incorrect Answer ImageE.Uterine rupture

A 60-year-old woman comes to the physician for an annual well-woman examination. She is
concerned about her risk for cancer. She had her last menstrual period at age 55 and has had no
vaginal bleeding since then. She has no medical problems and has never had surgery. There is no
family history of cancer. She takes no medications and has no allergies to medications. She
reports a 20 pack-year smoking history but quit approximately 20 years ago. Physical
examination shows no abnormalities. From which of the following cancers is the patient most at
risk for dying?
 Incorrect Answer ImageA.Breast cancer
 Incorrect Answer ImageB.Cervical cancer
 Incorrect Answer ImageC.Endometrial cancer
 Correct Answer ImageD.Lung cancer
 Incorrect Answer ImageE.Ovarian cancer

A 33-year-old nulligravid woman comes to the physician because she has not had a menstrual
period for 8 months. She had menarche at the age of 12 years and used to have regular 28-day
cycles up to this point. She has no history of major medical illness and takes no medications.
Physical examination shows no abnormalities. Urine human chorionic gonadotropin (hCG) and
serum beta-hCG levels are negative. Serum thyroid stimulating hormone, follicle stimulating
hormone, and prolactin levels are also normal. The patient is administered a 10-day course of
medroxyprogesterone acetate. After completing the 10-day course, she has a heavy menstrual
period. Which of the following is the most likely cause of this patient's response to progesterone?
 Incorrect Answer ImageA.Asherman syndrome
 Correct Answer ImageB.Endogenous estrogen production
 Incorrect Answer ImageC.Endometrial carcinoma
 Incorrect Answer ImageD.Menopause
 Incorrect Answer ImageE.Pregnancy

A 22-year-old woman, gravida 3, para 2, at 22 weeks' gestation comes to the physician because
of a painless ulcer near her vagina, which she noticed 2 days ago. The patient has no history of
medical problems and takes no medications. She is allergic to penicillin, which causes
anaphylaxis. Physical examination shows a uterus consistent in size with a 22-week gestation
and a 1-cm, raised, nontender lesion on the distal portion of the vagina. Laboratory testing shows
a positive rapid plasma reagin (RPR) test. A microhemagglutination assay for Treponema
pallidum(MHA-TP) is also read as positive. Which of the following is the most appropriate next
step in management?
 Incorrect Answer ImageA.Administer erythromycin
 Incorrect Answer ImageB.Administer levofloxacin
 Incorrect Answer ImageC.Administer metronidazole
 Incorrect Answer ImageD.Administer tetracycline
 Correct Answer ImageE.Desensitize the patient and then administer penicillin

A 27-year-old woman comes to the physician for contraception management. The patient is
taking calcium and vitamin D due to a dual-energy x-ray absorptiometry (DEXA scan) which
showed a Z score of -2.2, and which was performed during treatment for anorexia nervosa. Since
being treated for anorexia nervosa, menses have occurred every 4-8 weeks and have consisted of
3-8 days of heavy bleeding accompanied by severe cramps. The patient has smoked ½ pack of
cigarettes per day since age 15 and does not drink or use illicit drugs. She is 170 cm (5 ft 7
inches) tall and weighs 55 kg (121 lb); BMI is 19 kg/m2. Blood pressure is 104/58 mmHg. Pelvic
examination is within normal limits. Which of the following is the most appropriate
contraceptive choice for this patient?
 Correct Answer ImageA.Combined estrogen-progestin oral contraceptives
 Incorrect Answer ImageB.Copper-containing intrauterine device
 Incorrect Answer ImageC.Etonogestrel subcutaneous implant
 Incorrect Answer ImageD.Intramuscular medroxyprogesterone acetate
 Incorrect Answer ImageE.Levonorgestrel-containing intrauterine device

A 24-year-old woman, gravida 3, para 1, aborta 1, at 34 weeks' gestation comes to the physician
for her first prenatal visit. Her pregnancy and the delivery of her first child were uncomplicated,
her menstrual periods are regular, and she denies any history of serious illness. Her temperature
is 37.2ºC (98.9ºF), blood pressure is 155/95 mm Hg, pulse is 70/min, and respirations are
16/min. Repeat examination 10 minutes later shows her blood pressure is 158/98 mm Hg.
Physical examination shows fundal height of 30 cm, the fetal heart rate is 146/min. Urine
dipstick is negative for protein. Ultrasound examination with fetal biometry is indicated. Which
of the following ultrasound findings is the most likely to be seen in this patient?
 Incorrect Answer ImageA.Large head and large abdominal circumference
 Incorrect Answer ImageB.Normal head and normal abdominal circumference
 Correct Answer ImageC.Normal head and small abdominal circumference
 Incorrect Answer ImageD.Small head and normal abdominal circumference
 Incorrect Answer ImageE.Small head and small abdominal circumference

A 16-year-old nulligravid girl comes to the emergency department because of heavy vaginal
bleeding. She says she normally has heavy periods every 2 to 3 months, but this period has been
unusually heavy with the passage of large clots. She has no medical problems, no history of
bleeding diathesis, and takes no medications. Her temperature is 37.0ºC (98.6ºF), blood pressure
is 110/70 mm Hg, pulse is 96/min, and respirations are 12/min. Pelvic examination shows a
moderate amount of blood in the vagina and a closed cervix. Vaginal ultrasound shows a normal
uterus and adnexa. Hematocrit is 30%. Urine beta-hCG is negative. Which of the following is the
most appropriate next step in management?
 Correct Answer ImageA.Cyclic progestins
 Incorrect Answer ImageB.Dilation and curettage
 Incorrect Answer ImageC.Endometrial ablation
 Incorrect Answer ImageD.Hysterectomy
 Incorrect Answer ImageE.No further steps are necessary
A 32-year-old woman, gravida 1, para 0, at 36 weeks’ gestation comes to the labor and delivery
department in active labor. She has had no prenatal care. She appears to be malnourished. After a
relatively short labor, she delivers a male infant who is assigned Apgar scores of 5 and 7 at 1 and
5 minutes, respectively. In the intermediate care nursery, the baby is crying constantly and has
marked tremors. The length, weight, and head circumference are all less than the third percentile.
Physical examination of the neonate shows a smooth philtrum, a thin and smooth vermilion
border, and short palpebral fissures. He also has flexion contractures of both elbows and fifth
finger clinodactyly. He subsequently has a seizure, which is controlled with phenobarbital, and a
CT of the brain shows absence of the corpus callosum. Which of the following is the most likely
diagnosis for this infant?
 Correct Answer ImageA.Fetal alcohol syndrome
 Incorrect Answer ImageB.Fetal valproate syndrome
 Incorrect Answer ImageC.Hyperthyroidism
 Incorrect Answer ImageD.Hypocalcemia
 Incorrect Answer ImageE.Neonatal abstinence syndrome

A 26-year-old woman comes to the physician because of a 3-month history of amenorrhea. She
has also had fatigue, decreased bowel movements, and a 10 pound weight gain. She has had
stiffness and cramping of her lower extremities for the past 6 weeks. Her blood pressure is
110/75 mm Hg and pulse is 56/min. Examination shows pale and dry skin. Pelvic examination
shows normal external genitalia, a normal-appearing cervix, and a small uterus. There are no
adnexal masses. Urine β-hCG is negative. Which of the following is the most likely cause of this
patient's irregular vaginal bleeding?
 Correct Answer ImageA.Anovulation
 Incorrect Answer ImageB.Endometrial hyperplasia
 Incorrect Answer ImageC.Endometriosis
 Incorrect Answer ImageD.Leiomyomata
 Incorrect Answer ImageE.Proliferative phase defect

A 27-year-old woman comes to the physician for a preconception visit. She states that she has
been feeling well over the past year. She has a normal menstrual period every 30 days. She has a
history of migraine headaches, but these have not bothered her in the past year. She had a left
ovarian cystectomy at age 23 years and no other surgeries. She takes no medications and is
allergic to penicillin. She lives with her husband and works as a lawyer. Physical examination,
including breast and pelvic examination, is normal. Which of the following is the most
appropriate next step in management? 
 Correct Answer ImageA.Folic acid 0.4 mg/day
 Incorrect Answer ImageB.Folic acid 4.0 mg/day
 Incorrect Answer ImageC.No supplementation is needed at this time
 Incorrect Answer ImageD.Vitamin A 25,000 IU/day
 Incorrect Answer ImageE.Vitamin A 250,000 IU/day

A 20-year-old woman comes to the physician because of left lower quadrant pain for 2 months
that is now getting worse. She has had no changes in bowel or bladder function. She has had no
fever, chills, nausea, vomiting, or diarrhea. The pain is intermittent and sometimes feels like a
dull pressure. Pelvic examination shows a left adnexal mass that is mildly tender. Urine β-hCG is
negative. Pelvic ultrasonography shows a 7-cm complex left adnexal mass with features
consistent with a benign cystic teratoma. Which of the following is the most appropriate next
step in management?
 Incorrect Answer ImageA.Hysteroscopy
 Incorrect Answer ImageB.Oral contraceptives
 Incorrect Answer ImageC.Repeat pelvic examination in 1 year
 Incorrect Answer ImageD.Repeat pelvic ultrasound in 6 weeks
 Correct Answer ImageE.Surgical removal of the cyst

A 31-year-old primigravid woman at 24 weeks' gestation comes to the emergency department


because she experienced a large gush of fluid after sneezing. She has had no vaginal bleeding or
contractions. Fetal movement has been normal. Her pregnancy has been uncomplicated. The
patient takes a daily prenatal vitamin, has no medical problems, and has had no previous surgery.
Vital signs are within normal limits. The fetal heart rate is 145/min by Doppler ultrasound.
Speculum examination shows a closed cervix. There is a small pool of clear fluid in the posterior
vaginal fornix. A swab of the fluid is smeared on a slide. Microscopic examination of the dried
slide is shown. Which of the following is the most likely diagnosis?
 Incorrect Answer ImageA.Leukorrhea of pregnancy
 Correct Answer ImageB.Prelabor rupture of membranes
 Incorrect Answer ImageC.Stress urinary incontinence
 Incorrect Answer ImageD.Trichomoniasis
 Incorrect Answer ImageE.Vulvovaginal candidiasis

A 34-year-old woman, gravida 0, para 0, comes to the physician because of painful menses for
the past 12 months. She mentions that the pain started 2–3 days before her periods and is only
mildly relieved with acetaminophen. She also reports painful bowel movements, but there is no
associated hematochezia or melena. She has been unable to conceive despite regular, unprotected
intercourse. She had been on depot medroxyprogesterone acetate (DMPA) for 8 years, but
stopped the injections 15 months ago and has had regular periods since then. Her past medical
history is unremarkable. She is not taking any regular medications. Pelvic examination shows a
normal-appearing vulva, vagina, and cervix. There are no adnexal masses. Rectovaginal
examination shows uterosacral ligament nodularity with tenderness on palpation. Laboratory
studies show:
Hb  10.3 g/dL
Hct  30.5%
MCV  74 µm3
WBC  9,300 mm3
Neutrophils  56%
Bands  3%
Platelets  205,000 mm3
ESR  68 mm/h
Which of the following is the most common anatomic site affected by this patient's condition? 
 Incorrect Answer ImageA.Bladder
 Correct Answer ImageB.Ovaries
 Incorrect Answer ImageC.Parietal peritoneum
 Incorrect Answer ImageD.Peritoneal organs
 Incorrect Answer ImageE.Rectum
 Incorrect Answer ImageF.Uterosacral ligaments

A 35-year-old woman, gravida 3, para 2, at 39 weeks' gestation comes to the emergency


department in active labor. Past obstetric history is significant for two normal spontaneous
vaginal deliveries at term. Physical examination shows the cervix to be 4 cm dilated and 50%
effaced; the vertex is at -1 station. The patient is contracting every 4 minutes. Over the next 2
hours the patient progresses to 5 cm dilation. An epidural anesthesia is administered to help ease
the pain. Artificial rupture of membranes is performed, demonstrating copious clear fluid. Four
hours later the patient is still at 5 cm dilation and the contractions have spaced out to every 10
minutes. Which of the following is the most appropriate next step in management?
 Incorrect Answer ImageA.Expectant management
 Incorrect Answer ImageB.Cesarean delivery
 Incorrect Answer ImageC.Forceps-assisted vaginal delivery
 Correct Answer ImageD.Oxytocin
 Incorrect Answer ImageE.Vacuum-assisted vaginal delivery

A 51-year-old woman comes to the physician due to vaginal dryness and hot flashes that have
disturbed her sleep nightly since her last menstrual period, approximately a year ago. The patient
has hypertension which is controlled with a calcium channel blocker. She had a left
oophorectomy for ovarian torsion at age 29 years. The patient’s paternal aunt and a maternal
cousin had breast cancer at age 81 and 72, respectively. Vital signs are within normal limits. A
speculum examination shows the vaginal mucosa to be pale and without rugae. Which of the
following is the most appropriate indication for the use of hormone replacement therapy in this
patient?
 Incorrect Answer ImageA.Prevention of breast cancer
 Incorrect Answer ImageB.Prevention of cardiac disease
 Incorrect Answer ImageC.Prevention of deep venous thrombosis
 Incorrect Answer ImageD.Treatment of genitourinary syndrome of menopause
 Correct Answer ImageE.Treatment of hot flashes

A 31-year-old woman, gravida 1, para 0, at 10 weeks’ gestation comes to the physician to initiate
prenatal care. She has had no nausea, abdominal pain, or bleeding. She takes a daily prenatal
vitamin, has no medical problems, and has had no prior surgery. Vital signs are within normal
limits. Pelvic examination shows a slightly enlarged uterus and no tenderness. Routine prenatal
laboratory studies are normal. A urinalysis is positive for nitrites and bacteria, and a urine culture
shows >100,000 colony-forming units/mL of gram-negative rods. Which of the following is the
most appropriate next step in management?
 Incorrect Answer ImageA.Conservative management only
 Incorrect Answer ImageB.Five days of ciprofloxacin therapy
 Incorrect Answer ImageC.Five days of nitrofurantoin therapy
 Incorrect Answer ImageD.Seven days of amoxicillin therapy
 Correct Answer ImageE.Seven days of cephalexin therapy

A 32-year-old woman at 10 weeks' gestation comes to the physician for the initial prenatal visit.
During this visit, she is found to have a palpable mass in the upper outer quadrant of her left
breast. A mammogram shows a single spiculated 3-cm mass in the upper outer quadrant of her
left breast. A core biopsy shows infiltrating ductal carcinoma. The woman wants to keep her
baby and receive treatment for her cancer. Which of the following would be the best course of
action at this time?
 Incorrect Answer ImageA.Chemotherapy now, defer surgery until after delivery
 Incorrect Answer ImageB.Lumpectomy and axillary sampling, followed by radiotherapy
in 6 weeks
 Correct Answer ImageC.Modified radical mastectomy now, defer systemic therapy until
later
 Incorrect Answer ImageD.Radiation therapy now, defer surgery until after delivery
 Incorrect Answer ImageE.Therapeutic abortion and modified radical mastectomy

A 32-year-old woman, gravida 3, para 0, at 29 weeks' gestation comes to the physician for a
prenatal visit. She has no complaints. She had a prophylactic cerclage placed at 12 weeks'
gestation because of her history of two consecutive 19-week spontaneous abortions. These
spontaneous abortions were both characterized by painless cervical dilation, with the membranes
found bulging into the vagina on examination. Transvaginal ultrasound now shows her cervix to
be long and closed with no evidence of funneling. Which of the following is the most appropriate
time to remove the cerclage from this patient? 
 Incorrect Answer ImageA.At 30–32 weeks
 Incorrect Answer ImageB.At 32–34 weeks
 Incorrect Answer ImageC.At 34–36 weeks
 Correct Answer ImageD.At 36–38 weeks
 Incorrect Answer ImageE.At 38–40 weeks
A 32-year-old woman, gravida 1, para 0, at 37 weeks’ gestation comes to the emergency
department because of a sudden gush of fluid which soaked her underwear and pants one hour
ago. Fetal movement has been normal. The patient has had no pain or bleeding. She was
diagnosed with HIV during her initial prenatal visit and has been taking antiretroviral therapy
since her diagnosis. Her last viral load, at 36 weeks’ gestation, was 1145 copies/mL. Vital signs
are within normal limits. Cardiotocography shows no uterine contractions and a fetal heart rate
of 150/min, moderate variability, accelerations, and no decelerations. A sterile speculum
examination confirms membrane rupture. A digital cervical examination shows the cervix to be
firm, closed, long, and posterior. Ultrasound shows anhydramnios (amniotic fluid index = 0) and
a fetus in cephalic presentation. In addition to ziduvodine, which of the following is the most
appropriate next step in management?
 Incorrect Answer ImageA.Amnioinfusion
 Incorrect Answer ImageB.Ampicillin and azithromycin
 Correct Answer ImageC.Cesarean delivery
 Incorrect Answer ImageD.Labor induction with misoprostol
 Incorrect Answer ImageE.Labor induction with oxytocin

A 58-year-old postmenopausal woman comes to the physician because she noticed a lump in her
left breast during a breast self-examination 2 days ago. She has had yearly mammograms, all of
which have been normal. She has not had any weight loss, bone pain, or anorexia. She has
hypertension and osteoporosis, for which she takes hydrochlorothiazide and vitamin D
supplements. Physical examination shows a palpable nodule in the upper outer quadrant of the
left breast. No other masses or nodules are felt. Diagnostic mammography shows a spiculated 8-
mm density in the same quadrant. Ultrasound-guided biopsy specimen demonstrates an invasive
ductal carcinoma. Immunohistochemistry shows that the tumor is estrogen receptor-positive,
with a histologic grade of 2 out of 3. The most appropriate treatment strategy includes which of
the following?
 Incorrect Answer ImageA.Bilateral simple mastectomy
 Incorrect Answer ImageB.Lumpectomy only
 Correct Answer ImageC.Lumpectomy, sentinel lymph node biopsy, radiation therapy,
and tamoxifen
 Incorrect Answer ImageD.Lumpectomy, sentinel lymph node biopsy, and tamoxifen
 Incorrect Answer ImageE.Tamoxifen only
A 32-year-old woman, gravida 2, para 1, at 37 weeks’ gestation comes to labor and delivery
because of a large gush of fluid 1 hour ago while shopping. She does not have uterine
contractions. Her prenatal course has been unremarkable and cultures at 36 weeks' gestation were
negative for group B β-hemolytic Streptococcus. Pelvic examination with a sterile speculum
shows fluid in the posterior fornix that causes nitrazine paper to turn blue when applied. The
cervix appears closed and thick. Microscopic examination of fluid allowed to air-dry shows a
ferning pattern. The fetal heart rate is 140/min and fetal monitoring shows accelerations of up to
15 beats above baseline. Which of the following is the most appropriate next step in
management?
 Incorrect Answer ImageA.Administer betamethasone to improve fetal lung maturity
 Incorrect Answer ImageB.Expectantly manage as an outpatient given her unfavorable
cervix
 Correct Answer ImageC.Induction of labor with oxytocin
 Incorrect Answer ImageD.Start penicillin to prevent infection with group B streptococcus
 Incorrect Answer ImageE.Start prophylactic broad-spectrum intravenous antibiotics with
ampicillin and gentamicin

A 32-year-old woman, gravida 2, para 2 at 33 weeks' gestation is brought to the labor and
delivery department after she began to have generalized jerking movements of the extremities
and then became unresponsive 10 minutes ago. She has been in the hospital on the antepartum
service for the last 2 months because of severe preeclampsia. She is receiving only a prenatal
vitamin. Her temperature is 37.2ºC (99.0ºF), blood pressure is 145/95 mm Hg, pulse is 78/min,
and respirations are 13/min. Recurrent seizure prophylaxis is initiated, and an induction of labor
is begun. The nurse returns to check on the patient 4 hours later to find the patient minimally
responsive. Blood pressure is 90/50 mm Hg, pulse is 55/min, and respirations are 6/min. Physical
examination shows the skin to be warm, pink, and diaphoretic. The anesthesia team emergently
intubates the patient because of respiratory failure.
Which of the following is the most likely cause of the patient's respiratory failure? 
 Incorrect Answer ImageA.Hypercalcemia
 Incorrect Answer ImageB.Hyperkalemia
 Correct Answer ImageC.Hypermagnesemia
 Incorrect Answer ImageD.Hypernatremia
 Incorrect Answer ImageE.Hypocalcemia
 Incorrect Answer ImageF.Hypomagnesemia
 Incorrect Answer ImageG.Hyponatremia
A 32-year-old woman, gravida 2, para 2 at 33 weeks' gestation is brought to the labor and
delivery department after she began to have generalized jerking movements of the extremities
and then became unresponsive 10 minutes ago. She has been in the hospital on the antepartum
service for the last 2 months because of severe preeclampsia. She is receiving only a prenatal
vitamin. Her temperature is 37.2ºC (99.0ºF), blood pressure is 145/95 mm Hg, pulse is 78/min,
and respirations are 13/min. Recurrent seizure prophylaxis is initiated, and an induction of labor
is begun. The nurse returns to check on the patient 4 hours later to find the patient minimally
responsive. Blood pressure is 90/50 mm Hg, pulse is 55/min, and respirations are 6/min. Physical
examination shows the skin to be warm, pink, and diaphoretic. The anesthesia team emergently
intubates the patient because of respiratory failure.
After discontinuing the magnesium, which of the following is the most appropriate next step in
management? 
 Correct Answer ImageA.Calcium gluconate
 Incorrect Answer ImageB.Diazepam
 Incorrect Answer ImageC.Ephedrine
 Incorrect Answer ImageD.Naloxone
 Incorrect Answer ImageE.Phenytoin

A 31-year-old nulliparous woman comes to the physician because she is worried she might be
pregnant. She had intercourse one day ago with her boyfriend and the condom broke. Her last
menstrual period was 2 weeks ago. The patient has been in a monogamous relationship with her
boyfriend for the past year. She has used condoms exclusively for pregnancy prevention and is
interested in a second form of contraception. The patient takes a triptan as needed for migraines
with auras. Vital signs are within normal limits. A bimanual pelvic examination is normal. Urine
pregnancy test is negative. Which of the following is the most appropriate contraception for this
patient at this time?
 Incorrect Answer ImageA.Combination oral contraceptives
 Correct Answer ImageB.Copper-containing intrauterine device
 Incorrect Answer ImageC.Etonogestrel contraceptive implant
 Incorrect Answer ImageD.Hormonal contraceptive vaginal ring
 Incorrect Answer ImageE.Levonorgestrel-containing intrauterine device

A 32-year-old woman, gravida 2, para 0, comes to the physician for a prenatal checkup. She
reports abdominal distension, breast tenderness, and nausea without vomiting. She does not have
any vaginal bleeding or discharge. Her last menstrual period was 10 weeks ago, and prior to this,
she had regular 28-day cycles. Her past medical history is significant for two previous
pregnancies, both of which were terminated early via elective abortion because of “bad timing.”
She and her husband have been actively trying to get pregnant for the last year and a half.
Transvaginal ultrasound shows a normal, thin endometrial lining with no intrauterine pregnancy.
Quantitative serum β-hCG is 0.5 mIU/mL (≤ 5 mIU/mL negative for pregnancy). A repeat β-
hCG performed 1 week later is unchanged; however, her symptoms persist. Which of the
following is the most likely diagnosis? 
 Incorrect Answer ImageA.Complete spontaneous abortion
 Incorrect Answer ImageB.Ectopic pregnancy
 Incorrect Answer ImageC.Missed abortion
 Incorrect Answer ImageD.Molar pregnancy
 Correct Answer ImageE.Pseudocyesis

A 58-year-old post-menopausal woman comes to the physician because of vulvar itching and
burning for the past 2 months. She has slight pain in the vaginal introitus after intercourse but no
vaginal bleeding. She is sexually active with 3 male partners and uses condoms inconsistently.
Past medical history is significant for allergic rhinitis, rheumatoid arthritis and sexually
transmitted diseases (20 years ago). Current medications include loratadine, ibuprofen and
methotrexate. Pelvic examination shows a 1-cm white, flat and glistening area on the right vulvar
region with diffuse vulvar skin atrophy and excoriations. Which of the following is the most
likely diagnosis and most appropriate initial pharmacotherapy?
Diagnosis Treatment

A. Atopic dermatitis Tacrolimus cream

B. Candidiasis Oral fluconazole

C. Candidiasis Vaginal clotrimazole cream

D. Lichen sclerosus Clobetasol cream

E. Lichen sclerosus Dexamethasone cream

F. Lichen simplex chronicus Fluorinated corticosteroid cream


 Incorrect Answer ImageA.A.
 Incorrect Answer ImageB.B.
 Incorrect Answer ImageC.C.
 Correct Answer ImageD.D.
 Incorrect Answer ImageE.E.
 Incorrect Answer ImageF.F.

A 32-year-old woman, gravida 3, para 2, at 37 weeks' gestation comes to the physician for a
prenatal visit. She has no current complaints. Her past medical history is significant for hepatitis
C infection, which she acquired through a needle stick injury at work as a nurse. She is hepatitis
B and HIV negative. She takes no medications and has no allergies to medications. Her prenatal
course has been uncomplicated. She wants to know whether she can have contact with the baby
or breastfeed given her hepatitis C status. Which of the following is the most appropriate
response? 
 Incorrect Answer ImageA.Casual contact with the baby is prohibited
 Incorrect Answer ImageB.Complete isolation is not needed but breastfeeding is
prohibited
 Incorrect Answer ImageC.The patient should be completely isolated from the baby
 Correct Answer ImageD.There is no evidence that breastfeeding increases HCV
transmission
 Incorrect Answer ImageE.There is strong evidence that breastfeeding increases HCV
transmission
An 18-year-old woman, gravida 1, para 0, at 35 weeks' gestation comes to the physician because
of a foul-smelling vaginal discharge for 7 days. She has had no fever, frequency, urgency,
painful urination, or blood in her urine. She is sexually active with a new male partner. She
reports normal fetal movements and denies contractions. Her temperature is 36.9°C (98.4°F),
blood pressure is 110/75 mm Hg, and pulse is 75/min. The fetal heart rate is 140/min. Pelvic
examination shows an erythematous cervix with punctate hemorrhages and a frothy, green
discharge. There is no cervical motion tenderness. Gonorrhea and chlamydia nucleic acid
amplification tests were negative 3 days ago. Which of the following is the most appropriate next
step in diagnosis?
 Incorrect Answer ImageA.Cervical cytology
 Incorrect Answer ImageB.Vaginal culture
 Correct Answer ImageC.Vaginal discharge microscopy
 Incorrect Answer ImageD.Vaginal nucleic acid amplification testing
 Incorrect Answer ImageE.Vaginal pH testing
 Incorrect Answer ImageF.Vaginal rapid antigen testing

A 36-year-old woman, gravida 4, para 3, aborta 1, comes to the physician because of painless
and irregular intermenstrual vaginal bleeding for the last 4 months. She uses 6 to 8 pads per day
between her menstrual periods. Previously, her menstrual cycles occured every 28 days and
lasted for 7 days. Menarche was at age 14 years. She does not report any weight changes, visual
disturbances, or galactorrhea. She underwent tubal ligation last year that was confirmed by
hysterosalpingogram and an appendectomy at age 22. Her last pap smear was done 3 months ago
and was normal. She does not smoke, drink alcohol or use any recreational drugs. Her
temperature is 37.2ºC (99.0ºF), pulse is 86/min, respirations are 14/min, and blood pressure is
115/75 mm Hg. Chest and abdominal examination is unremarkable. Pelvic examination shows
no vulvar, vaginal, or cervical lesions. There is dark blood in the vagina. Bimanual examination
shows a symmetric, nonenlarged, and nontender uterus with normal adnexa. Laboratory studies
show:
Hb 9.6 g/dL
Hct 30%
WBC  9,700/mm3
Neutrophils 60%
Lymphocytes 24%
Platelets 220,000/mm3
Beta-hCG  3 mIU/mL (negative, <5 mIU/mL)
PT  12 s
PTT  35 s
TSH 3.0 μU/mL
FSH 28 mIU/mL (normal, 5–30 mIU/mL)
LH 25 mIU/mL (normal, 5–30 mIU/mL)
Endometrial biopsy shows weakly proliferative endometrium with no evidence of hyperplasia or
atypia. Based on the patient's history and laboratory findings, what would be the best initial
treatment? 
 Incorrect Answer ImageA.Dilation and curettage
 Incorrect Answer ImageB.Endometrial ablation
 Incorrect Answer ImageC.High dose IV estrogen
 Correct Answer ImageD.Low dose combination oral contraceptive pills (OCPs)
 Incorrect Answer ImageE.Transfusion

A 35-year-old woman, gravida 2, para 1, at 38 weeks' gestation comes to labor and delivery
because of painful contractions and vaginal bleeding. Her temperature is 36.6°C (98°F), blood
pressure is 170/115 mm Hg, pulse is 96/min, and respirations are 16/min. Fetal heart rate is
130/min with deep, late decelerations and no accelerations. Soon after initial assessment, the
patient reports pelvic pain and becomes confused. Repeat vital signs show blood pressure is
80/40 mm Hg. The patient is intubated, 2 L of Ringer's lactate is infused, and an emergency
Cesarean section is performed. The infant is then transferred to the neonatal intensive care unit.
However, the mother remains hypotensive to 60/40 mm Hg with a pulse of 158/min. Which of
the following is the most appropriate next step in management?
 Incorrect Answer ImageA.Administration of intravenous narcotics
 Incorrect Answer ImageB.Emergency hysterectomy
 Incorrect Answer ImageC.Emergency hysterectomy with bilateral salpingo-
oophorectomy
 Incorrect Answer ImageD.Rapid infusion of 2 L of normal saline
 Correct Answer ImageE.Transfusion of packed red blood cells

A 43-year-old nulliparous woman at 37 weeks' gestation comes to the emergency department


because of abdominal pain, nausea, vomiting, and fatigue. Fetal movement has been decreased
and there have been no contractions, leakage of fluid, or vaginal bleeding. The patient conceived
via in vitro fertilization with a donor egg. Her pregnancy has been otherwise uncomplicated. The
patient takes a daily prenatal vitamin, has no chronic medical problems, and has had no previous
surgery. Her blood pressure is 142/86 mm Hg. The fetal heart rate tracing shows a baseline of
140/min, moderate variability, accelerations, and no decelerations. Abdominal examination
shows epigastric tenderness, no masses, and no rebound or guarding. A urine dipstick shows 3+
proteinuria. Laboratory studies show:
Lactate dehydrogenase (LDH) 829 U/L
Hemoglobin 7.8 g/dL
Platelets 67,000/mm3
Aspartate aminotransferase (AST) 220 U/L
Alanine aminotransferase (ALT) 316 U/L
Ultrasound shows the fetus is in cephalic presentation. Which of the following is the most
appropriate next step in management?
 Incorrect Answer ImageA.Betamethasone sodium phosphate
 Incorrect Answer ImageB.Cesarean delivery
 Incorrect Answer ImageC.Induction of labor
 Correct Answer ImageD.Magnesium sulfate
 Incorrect Answer ImageE.Platelet transfusion

A 64-year-old woman is postoperative day 4 after undergoing a total abdominal hysterectomy


and bilateral salpingo-oophorectomy for uterine prolapse. On postoperative day 1, a complete
blood count shows: 
Leukocytes 10,000/mm3
Hct 36%
Platelets 245,000/mm3
By postoperative day 2, the patient is alert and able to ambulate without difficulty. She has no
complaints. She has not taken in nutrition orally and is receiving IV fluids. She is voiding
without difficulty and has passed flatus. She has failed to have a bowel movement. Her
temperature is 37.6ºC (99.6ºF), blood pressure is 124/72 mm Hg, pulse is 86/min, and
respirations are 12/min. Examination shows her abdomen to be soft, nontender, and
nondistended. The Jackson-Pratt tubes were removed on postoperative day 3 and she has
minimal serosanguineous discharge on the dressing. The rest of the examination is unremarkable.
Which of the following is a reason for keeping this patient hospitalized for a longer period of
time? 
 Incorrect Answer ImageA.Absent bowel movement
 Correct Answer ImageB.Absent oral intake
 Incorrect Answer ImageC.Drainage indicating infection
 Incorrect Answer ImageD.Febrile
 Incorrect Answer ImageE.Laboratory values

A 37-year-old woman, gravida 1, para 0, at 41 weeks' gestation is admitted to the hospital for
induction of labor. The patient’s pregnancy has been uncomplicated. She takes a daily prenatal
vitamin, has no chronic medical problems, and has had no previous surgery. Vital signs are
within normal limits. The fetal heart tracing shows a baseline of 140/min, moderate variability,
accelerations, and no decelerations. A digital cervical examination shows the cervix to be 3 cm
dilated and 80% effaced. A bedside ultrasound confirms cephalic presentation. An oxytocin
infusion is initiated. Three hours later the patient is reevaluated after spontaneous membrane
rupture. The fetal heart tracing shows a baseline of 140/min, minimal variability, no
accelerations, and gradual decreases in the fetal heart rate to a nadir of 130/min which occur after
each contraction. The cervical examination is unchanged. Which of the following is the most
likely etiology of the findings seen on this patient’s fetal heart rate tracing?
 Incorrect Answer ImageA.Fetal anemia
 Incorrect Answer ImageB.Fetal head compression
 Incorrect Answer ImageC.Umbilical cord compression
 Incorrect Answer ImageD.Umbilical cord prolapse
 Correct Answer ImageE.Uteroplacental insufficiency

A 15-year-old girl, gravida 1, para 0, at 31 weeks’ gestation comes to the emergency department
because she had a large gush of fluid 2 hours ago while watching television. She does not have
any uterine contractions. Her prenatal course is remarkable for smoking a half-pack of cigarettes
per day despite counseling on cessation. Pelvic examination with a sterile speculum shows
pooling of clear fluid in the posterior fornix. The fluid causes nitrazine paper to turn blue.
Examination under the light microscope shows a ferning pattern. Fetal heart tones are in the 150s
on an external tocodynamometer, and an abdominal ultrasound shows oligohydramnios with a
fetus in the cephalic presentation. The pooled fluid is sent to the laboratory for evaluation of fetal
lung maturity, and it shows a lecithin to sphingomyelin ratio of 1.5:1. Which of the following is
the best next step in management? 
 Correct Answer ImageA.Administer IM betamethasone
 Incorrect Answer ImageB.Administer IV prophylactic antibiotics
 Incorrect Answer ImageC.Expedite delivery with IV oxytocin
 Incorrect Answer ImageD.Immediate delivery via Caesarean section
 Incorrect Answer ImageE.Insert an intrauterine catheter and begin an amnioinfusion
A 29-year-old woman comes to the emergency department because of abdominal distension and
shortness of breath that has been progressive over the last 2 days. Approximately 1 week ago,
she underwent fertility treatment with ovulation induction and oocyte retrieval. She has a history
of polycystic ovary syndrome but no other medical problems. She has no known drug allergies.
Her temperature is 37ºC (98.6ºF), blood pressure is 80/40 mm Hg, pulse is 130/min, and
respirations are 28/min. Physical examination shows crackles at the lung bases bilaterally and a
distended, nontender abdomen with a fluid wave. Ultrasound demonstrates bilaterally enlarged
ovaries (each >10 cm) and free fluid in the abdomen. Urine β-hCG is negative. Which of the
following is the most likely diagnosis?
 Incorrect Answer ImageA.Ectopic pregnancy
 Incorrect Answer ImageB.Hemorrhagic ovarian cyst
 Correct Answer ImageC.Ovarian hyperstimulation syndrome
 Incorrect Answer ImageD.Ovarian torsion
 Incorrect Answer ImageE.Tubo-ovarian abscess

A 28-year-old woman, gravida 1, para 0, at 34 weeks' gestation comes to the emergency


department because of decreased fetal movement. She has had no vaginal bleeding, leakage of
fluid, or contractions. Blood pressure is 118/76 mm Hg. A nonstress test shows a fetal heart rate
baseline of 140/min, moderate variability, accelerations, and no decelerations. An ultrasound
shows a fetus in frank breech presentation, normal fetal movement, tone, and breathing, and a
normal amniotic fluid volume. Which of the following is the best next step in management?
 Incorrect Answer ImageA.Cesarean delivery
 Correct Answer ImageB.Expectant management
 Incorrect Answer ImageC.External cephalic version
 Incorrect Answer ImageD.Induction of labor
 Incorrect Answer ImageE.Internal podalic version

A 24-year-old woman, gravida 1, para 0, at 36 weeks' gestation comes to the physician for a
prenatal visit. The pregnancy has been uncomplicated and she has received routine prenatal care.
She has no medical problems. She takes a prenatal vitamin and does not use recreational drugs.
Vital signs and physical examination are within normal limits for a woman in the third trimester.
Which of the following is the most appropriate next step in management? 
 Incorrect Answer ImageA.Blood test for hepatitis B surface antigen
 Incorrect Answer ImageB.Oral glucose tolerance test
 Incorrect Answer ImageC.Papanicolaou (Pap) cervical-smear
 Incorrect Answer ImageD.Vaginal swab for bacterial vaginosis
 Correct Answer ImageE.Vaginal swab for group B Streptococcus

A 34-year-old woman comes to the physician because of painful bowel movements over the last
year. She often finds herself "wincing" during defecation and sometimes is constipated, which
she associates with a fear of having the pain. She also has pain during intercourse, especially
with deep penetration. Her menstrual periods are regular and occur every 28 days. She has
painful cramps beginning a day or 2 before her period and ending on the second day of her flow.
She has no chronic medical conditions and takes no medications. She has never had any sexually
transmitted diseases and has never been pregnant. Her mother has Crohn's disease, and her father
has prostate cancer. Physical examination is unremarkable. Rectal examination shows brown,
guaiac-negative stool. Leukocyte count is 4,000/mm3, and erythrocyte sedimentation rate is 15
mm/h. Which of the following will most likely establish the diagnosis in this patient? 
 Incorrect Answer ImageA.Barium enema
 Incorrect Answer ImageB.Colonoscopy with biopsies
 Incorrect Answer ImageC.CT scan of the abdomen and pelvis
 Incorrect Answer ImageD.Hysterosalpingography
 Incorrect Answer ImageE.Hysteroscopy
 Correct Answer ImageF.Laparoscopy
 Incorrect Answer ImageG.Ultrasonography of the pelvis

A 39-year-old woman, gravida 2, para 1, at 10 weeks' gestation comes to the physician to initiate
prenatal care. She has some fatigue and mild nausea and no pain or bleeding. The patient’s first
pregnancy, 2 years ago, ended with a term vaginal delivery of a 7 lb female diagnosed neonatally
with Down syndrome. The patient has no chronic medical problems and has had no previous
surgery. Vital signs are within normal limits. Physical examination is normal. Bedside ultrasound
shows a 10-week intrauterine pregnancy with a normal fetal heartbeat. The patient does not want
another infant with Down syndrome and has decided that she would terminate the pregnancy for
chromosomal abnormalities. Which of the following is the most appropriate initial test for
aneuploidy in this patient?
 Incorrect Answer ImageA.Cell-free DNA testing
 Correct Answer ImageB.Chorionic villus sampling
 Incorrect Answer ImageC.First-trimester combined screening
 Incorrect Answer ImageD.Genetic amniocentesis
 Incorrect Answer ImageE.Quadruple marker test

A 24-year-old woman, gravida 2, para 1, at 24 weeks' gestation comes to the physician for a
prenatal visit. She does not have any vaginal discharge or bleeding, contractions, headache, or
leg swelling, and her fetus is moving normally. She gave birth to a healthy infant girl 2 years ago
following an uncomplicated pregnancy. She has no significant past medical history. Her weight
today is 65.8 kg (145 lb) (last visit 1 month ago was 62 kg [137 lb]). Her blood pressure is
150/97 mm Hg, pulse is 87/min, and respirations are 12/min. Physical examination shows a
fundal height of 24 cm with a fetal heart rate of 150/min. There is no abdominal tenderness and
her lungs are clear to auscultation. A repeat blood pressure taken 30 minutes later is 145/91 mm
Hg. Spot urine dipstick reveals 2+ proteinuria. Serum laboratory testing shows:
Na+ 136 mEq/dL 
K+ 3.6 mEq/dL 
Cl −
100 mEq/dL 
HCO3− 24 mEq/L 
BUN  55 mg/dL 
Creatinine  0.8 mg/dL 
White blood cells  9,000/mm3
Hemoglobin  10.0 mg/dL 
Hematocrit  31% 
Platelets  175,000/mm3
AST  32 U/L
ALT 28 U/L 
Which of the following is the most likely diagnosis? 
 Incorrect Answer ImageA.Eclampsia
 Incorrect Answer ImageB.Gestational hypertension
 Incorrect Answer ImageC.HELLP syndrome
 Correct Answer ImageD.Preeclampsia without severe features
 Incorrect Answer ImageE.Preeclampsia with severe features

A 14-year-old girl is brought to the physician by her parents because of mild left lower
abdominal pain for the past 12 hours. She describes the pain as crampy and sharp. She
experienced a similar episode of pain one month ago, which lasted for less than a day. She began
menstruating 2 years ago and has noticed some cramping lower abdominal pain during the first 2
days of her menses. Her last menstrual period was 2 weeks ago and lasted 5 days. She denies
sexual activity and has not had any vaginal discharge. Her bowel movements have been normal
and she reports no urinary frequency, urgency, or burning with urination. Which of the following
is the most likely diagnosis?
 Incorrect Answer ImageA.Dysmenorrhea
 Incorrect Answer ImageB.Ectopic pregnancy
 Correct Answer ImageC.Mittelschmerz
 Incorrect Answer ImageD.Ovarian torsion

 Incorrect Answer ImageE.Pelvic inflammatory disease

A 31-year-old woman is evaluated in the hospital 1 day after an uncomplicated spontaneous


vaginal delivery of a healthy male infant. The infant’s blood type is A, Rh-negative. The patient
feels well and has no symptoms of depression or sadness. Breastfeeding has been going well.
Vital signs are within normal limits. Physical examination shows the uterine fundus to be firm,
nontender and palpable at the umbilicus. There is 2+ pitting edema to the shins bilaterally.
Review of the patient’s prenatal laboratory studies shows:
Blood type O, Rh-negative
Hepatitis B surface antigen Negative
Hepatitis B surface antibody Positive
Rubella IgG Negative
Which of the following is the most appropriate next step in management?
 Incorrect Answer ImageA.Enoxaparin prophylaxis
 Incorrect Answer ImageB.Hepatitis B immune globulin
 Incorrect Answer ImageC.Hepatitis B vaccine
 Correct Answer ImageD.Measles-mumps-rubella vaccine
 Incorrect Answer ImageE.Rho(D) immune globulin

A 39-year-old woman, gravida 2, para 1, at 30 weeks’ gestation comes to the physician for a
prenatal visit. She has no contractions, loss of fluid, or vaginal bleeding, and the fetus is active.
The patient's gestational age was determined by a 7-week ultrasound. Her prenatal course has
been unremarkable. The fetal heart rate is 150/min and fundal height is 27 centimeters, which is
the same as 4 weeks ago. Which of the following is the most likely cause of the patient’s fundal
height measurement?
 Incorrect Answer ImageA.Inaccurate dating
 Correct Answer ImageB.Intrauterine growth restriction
 Incorrect Answer ImageC.Premature labor
 Incorrect Answer ImageD.Twin gestation
 Incorrect Answer ImageE.Uterine cancer

A 19-year-old nulligravid woman comes to the emergency department because of severe left
lower quadrant pain that has been intermittent over the last 3 days. This afternoon she reports it
became persistent and severe and was accompanied by nausea and vomiting. Her temperature is
37°C (98.6°F), blood pressure is 122/78 mm Hg, pulse is 72/min, and respirations are 14/min.
Physical examination shows left lower quadrant tenderness and a tender left adnexal mass
accompanied by abdominal guarding. Urine β-hCG is negative. Pelvic ultrasound shows a 7-cm
left ovarian complex mass. Which of the following is the most appropriate next step in
management?
 Incorrect Answer ImageA.Expectant management
 Incorrect Answer ImageB.Follow-up ultrasound in 6 weeks
 Incorrect Answer ImageC.IV antibiotics
 Correct Answer ImageD.Laparoscopy
 Incorrect Answer ImageE.Oophorectomy

A 35-year-old woman, gravida 4, para 3, at 38 weeks’ gestation comes to the labor and delivery
ward after a gush of clear fluid from the vagina. After the gush, she has had increasing
contractions. Sterile speculum examination shows a pool of clear fluid in the vagina that is
nitrazine-positive. Cervical examination shows that the patient is 5 cm dilated, with the fetal face
in a mentum anterior face presentation. External uterine monitoring shows that the patient is
contracting every 2 minutes, and external fetal monitoring shows that the fetal heart rate is
140/min and the tracing is reactive. Which of the following is the most appropriate next step in
management?
 Incorrect Answer ImageA.Cesarean section
 Correct Answer ImageB.Expectant management
 Incorrect Answer ImageC.Forceps delivery
 Incorrect Answer ImageD.Oxytocin augmentation
 Incorrect Answer ImageE.Vacuum delivery

A 32-year-old woman, gravida 2, para 1, at 38 weeks' gestation comes to the emergency


department because of worsening contractions. The patient’s pregnancy has been uncomplicated.
Her prior pregnancy ended with a cesarean delivery at 37 weeks’ gestation due to placenta
previa. She takes a daily prenatal vitamin and has no medical problems. Vital signs are within
normal limits. The fetal heart rate is 145/min with moderate variability, accelerations, and no
decelerations. Digital cervical examination shows the cervix to be 5 cm dilated and 75% effaced
with the fetal head at -3 station. The patient is admitted and managed expectantly. Epidural
anesthesia is administered. A few minutes later there is a large gush of clear fluid from the
vagina. The fetal heart rate is 70/min for 4 minutes. Digital examination shows the cervix to be 8
cm dilated and 90% effaced with the fetal head at -3 station. There is a soft pulsating mass in
front of the fetal head. Which of the following is the most likely diagnosis?
 Incorrect Answer ImageA.Abruptio placentae
 Incorrect Answer ImageB.Dehiscence of the uterine scar
 Incorrect Answer ImageC.Placenta previa
 Correct Answer ImageD.Umbilical cord prolapse
 Incorrect Answer ImageE.Vasa previa

A 41-year-old woman, gravida 4, para 3, at term is admitted to the labor and delivery ward with
regular contractions every 2 minutes. Examination shows that her membranes are grossly
ruptured and that her cervix is 5 cm dilated. Over the following 3 hours, she progresses to full
dilation and +2 station. Fetal bradycardia develops, and the decision is made to proceed with
vacuum-assisted vaginal delivery. A 3,402 g (7 lb 8 oz) boy is delivered. Apgar scores are 8 and
9 at 1 and 5 minutes, respectively. Which of the following best represents an advantage of
vacuum extraction over the forceps for expediting delivery?
 Incorrect Answer ImageA.Vacuum can be used at higher stations
 Incorrect Answer ImageB.Vacuum can be used for fetuses in breech presentation
 Incorrect Answer ImageC.Vacuum can be used in face presentations
 Incorrect Answer ImageD.Vacuum can be used with intact membranes
 Correct Answer ImageE.Vacuum does not occupy space next to the fetal head
A 42-year-old woman, gravida 1, para 1, comes to the physician because of nausea, vomiting,
and abdominal fullness for several weeks. She had an uncomplicated term vaginal delivery of a
male fetus weighing 3,500 g 12 weeks ago. She has had vaginal bleeding since delivery and uses
two to three maxi pads per day. She has no other medical problems. Her temperature is 37ºC
(98.6ºF), blood pressure is 118/76 mm Hg, pulse is 67/min, and respirations are 12/min. Physical
examination shows a nontender and slightly distended abdomen. Pelvic examination shows scant
dark to bright red blood in the vagina without purulent discharge, palpable bilateral pelvic
masses, an enlarged uterus consistent in size with a 10-week gestation, and no cervical motion
tenderness. Urine β-hCG is positive. Which of the following is the most appropriate next step in
management?
 Incorrect Answer ImageA.Methotrexate
 Correct Answer ImageB.Quantitative β-hCG
 Incorrect Answer ImageC.Radiotherapy
 Incorrect Answer ImageD.Serum CA-125 levels
 Incorrect Answer ImageE.Suction curettage

A 39-year-old woman, gravida 3, para 2, at 18 weeks' gestation, comes to the physician for the
results from her amniocentesis. The amniocentesis was offered because of advanced maternal
age and paternal family history of Down syndrome. Her obstetric history is significant for two
full-term cesarean deliveries. She has no medical problems. She takes prenatal vitamins and has
no known drug allergies. She is informed that her amniocentesis shows that her fetus has trisomy
21 (Down syndrome). Which of the following is the most likely cause of Down syndrome in this
patient?
 Incorrect Answer ImageA.Inheritance of a chromosome with added genetic material from
chromosome 21
 Correct Answer ImageB.Nondisjunction during maternal meiosis I
 Incorrect Answer ImageC.Nondisjunction during maternal meiosis II
 Incorrect Answer ImageD.Nondisjunction during mitosis
 Incorrect Answer ImageE.Nondisjunction during paternal meiosis II

A 30-year-old woman, gravida 3, para 2, at 38 weeks' gestation comes to the labor and delivery
department because of regular contractions every 3 minutes for the past hour. She has had two
uncomplicated term vaginal deliveries. She has received routine prenatal care and was diagnosed
with gestational diabetes that was difficult to control during her pregnancy. The first and second
stages of labor are prolonged, and she delivers a 4,250-g (9 lb 6 oz) male infant. After delivery of
the placenta, the estimated blood loss is 600 mL. Intravenous access is obtained for volume
resuscitation and fundal massage is performed. Despite these interventions, the mother continues
to bleed 45 minutes after delivery. Which of the following is the most likely finding on physical
examination?
 Incorrect Answer ImageA.Inversion of the uterine fundus
 Incorrect Answer ImageB.Mucosal bleeding and bruising
 Incorrect Answer ImageC.Rupture of the uterus
 Correct Answer ImageD.Spongy, soft uterus
 Incorrect Answer ImageE.Vaginal laceration

A 27-year-old primigravid woman at 11 weeks' gestation comes to the physician because she has
nausea each morning and vomits approximately once per week. She is tolerating some solid food
and liquids. She has a history of severe depression for which she takes sertraline. She has no
surgical history. She is allergic to penicillin. Her blood pressure is 112/72 mm Hg. Urine dip
shows trace protein and no glucose. Fetal pulse is in the 150s. Lab values show: 
Blood type A-negative
Rubella Nonimmune
Hepatitis B surface antigen Negative
RPR Reactive
Gonorrhea Negative
Chlamydia Negative
Hemoglobin electrophoresis AA
Which of the following is the most appropriate next step in management? 
 Incorrect Answer ImageA.Admission for penicillin desensitization
 Correct Answer ImageB.Check microhemagglutination assay for antibodies to T.
pallidum
 Incorrect Answer ImageC.Electrophoresis of hemoglobin from the father
 Incorrect Answer ImageD.Hepatitis B immune globulin (HBIG) administration
 Incorrect Answer ImageE.MMR vaccine
 Incorrect Answer ImageF.RhoGAM administration

A 24-year-old woman, gravida 1, para 0, at 9 weeks' gestation by last menstrual period who has a
twin intrauterine pregnancy comes to the physician because of nausea and vomiting that has
occurred over the last 3 days. She has tried eating soda crackers, carbonated beverages, and
ginger lollipops, but she has not been able to decrease the nausea. She vomits at least twice per
day and is nauseated throughout. Her pre-pregnancy weight was 54.5 kg (120 lb). She now
weighs 50.8 kg (112 lb). Her temperature is 99.0ºF (37.2ºC), blood pressure is 114/67 mm Hg,
pulse is 67/min, and respirations are 12/min. Ultrasound examination shows a twin pregnancy
with good cardiac activity consistent with a 9-week gestation. Physical examination shows mild
epigastric tenderness and is otherwise unremarkable. Laboratory studies show:
Hgb  14.2 g/dL 
WBC  8,000/mm3
Platelets  270,000/mm3
Sodium  136 mEq/L 
Potassium  4.0 mEq/L 
Chloride  101 mEq/L 
Bicarbonate  28 mEq/L 
BUN  18 mg/dL 
Creatinine  0.8 mg/dL 
Glucose  100 mg/dL 
Calcium  9.2 mg/dL 
AST  28 U/L
ALT 32 U/L
Thyroid studies are normal. Which of the following is the most appropriate next step in
management? 
 Incorrect Answer ImageA.Admit the patient for intravenous nutrition
 Incorrect Answer ImageB.Continue current measures
 Incorrect Answer ImageC.Corticosteroids
 Incorrect Answer ImageD.Ondansetron
 Correct Answer ImageE.Pyridoxine

A 32-year-old woman, gravida 3, para 2, at 14 weeks' gestation comes to the physician for a
routine prenatal visit in the month of October. The patient has had no pain or vaginal bleeding.
She has not yet felt the baby move. She takes a daily prenatal vitamin, has no chronic medical
problems, and has had no prior surgery. Vital signs are within normal limits. The fetal heart rate
is 150/min by Doppler ultrasound. Routine prenatal laboratory studies show:
Blood type A, Rh-negative
Antibody screen Negative
Rubella IgG Negative
Varicella IgG Negative
Hepatitis B surface antigen Negative
HIV Negative
The blood types of her first 2 children are unknown. Which of the following is the most
appropriate next step in management?
 Correct Answer ImageA.Influenza vaccine
 Incorrect Answer ImageB.Measles-mumps-rubella vaccine
 Incorrect Answer ImageC.Tetanus-diphtheria-acellular pertussis vaccine
 Incorrect Answer ImageD.Rho(D) immune globulin
 Incorrect Answer ImageE.Varicella-zoster immune globulin
A 35-year-old woman, gravida 1, para 0, at 16 weeks' gestation comes to the physician because
of nausea and palpitations. She also feels flushed at times throughout the day and has difficulty
falling asleep. Her past medical history is unremarkable. Her temperature is 37ºC (98.6ºF), blood
pressure is 120/76 mm Hg, pulse is 90/min and regular, and respirations are 12/min.
Ultrasonography shows a singleton gestation of approximately 16 weeks with good cardiac
activity. Serum laboratory studies show:
Sodium 135 mEq/L
Potassium 4.5 mEq/L
Chloride 105 mEq/L
Bicarbonate 25 mEq/L
Urea nitrogen 10 mg/dL
Creatinine 1.0 mg/dL
Thyroid-stimulating hormone 0.1 mU/L
Hemoglobin 10.5 mg/dL
Hematocrit 33%
Leukocyte count 5,000/mm3
Platelet count 160,000/mm3
Alanine aminotransferase (ALT) 12 U/L
Total bilirubin 0.8 mg/dL
Which of the following is the most likely diagnosis?
 Incorrect Answer ImageA.Hyperemesis gravidarum
 Correct Answer ImageB.Hyperthyroidism
 Incorrect Answer ImageC.Normal pregnancy course
 Incorrect Answer ImageD.Paroxysmal supraventricular tachycardia
 Incorrect Answer ImageE.HELLP syndrome

A healthy 64-year-old woman has a mammogram that shows a 1.5 cm spiculated mass in the
inner upper quadrant of the right breast with calcifications. Biopsy shows carcinoma cells and
staging workup shows no metastatic disease. After a lumpectomy and sentinel lymph node
biopsy, the tumor is found to be a low-grade invasive ductal carcinoma, 1.8 cm in size, with no
lymph nodes positive for tumor. The tumor does not over-express human epidermal growth
factor receptor 2 (HER2), and is estrogen receptor (ER) negative and progesterone receptor (PR)
negative (“triple-negative”). Which of the tumor features suggests a poorer prognosis for this
patient?
 Incorrect Answer ImageA.Axillary lymph node status
 Incorrect Answer ImageB.Calcification of the tumor
 Correct Answer ImageC.“Triple-negative” status
 Incorrect Answer ImageD.Tumor grade
 Incorrect Answer ImageE.Tumor size of 1.8 cm

A 22-year-old woman comes to the emergency room because of vaginal discharge and postcoital
spotting for 3 days. Her last menstrual period was 8 weeks ago. The patient takes no
medications, has no chronic medical problems, and has not had prior surgery. She has been
sexually active with a new male partner for the last 2 months and has used condoms
inconsistently. Vital signs are within normal limits. Bimanual pelvic examination shows a
slightly enlarged, nontender uterus and no adnexal masses. A speculum examination shows a
thick yellowish opaque discharge coming from the cervix. Touching the cervix with a swab
causes it to bleed. Pelvic ultrasound shows an 8-week intrauterine gestation with a normal fetal
heartbeat. Which of the following is the most appropriate pharmacotherapy?
 Incorrect Answer ImageA.Cefotaxime and doxycycline
 Correct Answer ImageB.Ceftriaxone and azithromycin
 Incorrect Answer ImageC.Clindamycin and gentamicin
 Incorrect Answer ImageD.Levofloxacin and metronidazole
 Incorrect Answer ImageE.Nystatin and triamcinolone

A 62-year-old woman comes to the physician for an annual examination. She also has questions
about the side effects of tamoxifen, which she has been taking for the last year for the prevention
of breast cancer because of a strong family history. She takes no other medications. She has no
other medical or surgical history. Breast examination shows no masses, lymphadenopathy, or
skin changes. Which of the following is this patient most likely to develop as a result of
tamoxifen therapy?
 Incorrect Answer ImageA.Aplastic anemia
 Incorrect Answer ImageB.Elevated LDL cholesterol
 Correct Answer ImageC.Endometrial cancer
 Incorrect Answer ImageD.Myocardial infarction
 Incorrect Answer ImageE.Osteoporosis

A 21-year-old woman comes to the emergency department with diffuse lower abdominal pain for
2 days. She also has fevers and chills and thinks most of her symptoms began shortly after her
last menstrual period, which is described as normal. She has had 3 sexual partners over the last 3
months and her partners usually use condoms. She takes an oral contraceptive pill and has no
allergies. Her temperature is 38.3ºC (101.0ºF), blood pressure is 120/80 mm Hg, pulse is
115/min, and respirations are 24/min. Physical examination shows mild right upper quadrant
tenderness. There is no worsened pain or inspiratory arrest on deep subcostal palpation. The
lower abdomen is diffusely tender without rebound tenderness or guarding. Which of the
following is the most appropriate next step in management?
 Incorrect Answer ImageA.Abdominal ultrasound
 Correct Answer ImageB.Bimanual pelvic examination
 Incorrect Answer ImageC.Complete blood count
 Incorrect Answer ImageD.CT scan of the abdomen
 Incorrect Answer ImageE.Urine pregnancy test

A 41-year-old woman, gravida 4, para 0, aborta 4, comes to the physician for evaluation. The
patient has a history of 4 spontaneous abortions all at less than 12 weeks’ gestation. Menarche
was at age 13. Menses typically occur every 29 days and consist of 5 days of heavy bleeding and
cramping. The patient has no chronic medical problems and takes no medications. She drinks a
glass of wine with dinner once a week and does not smoke cigarettes or use illicit drugs. Vital
signs are within normal limits. A pelvic examination is normal. Which of the following is the
most appropriate next step in the management?
 Incorrect Answer ImageA.Endometrial biopsy
 Correct Answer ImageB.Saline infusion sonohysterogram
 Incorrect Answer ImageC.Semen analysis of patient’s partner
 Incorrect Answer ImageD.Serum homocysteine level
 Incorrect Answer ImageE.Ultrasonographic antral follicle count

A 34-year-old woman, gravida 2, para 1, at 37 weeks' gestation comes to the labor and delivery
department because of a large gush of clear fluid 30 minutes ago. Her pregnancy history is
significant for being RhD-negative with the father confirmed to be RhD-positive. Her antibody
screen was negative and she received the standard dose of anti-D immune globulin (Rhogam) at
28 weeks' gestation. Physical examination shows spontaneously ruptured membranes and the
cervix is 2-cm dilated. She is admitted and oxytocin infusion is administered for labor
augmentation. After a 16-hour labor course, she delivers a 3,400-g male neonate. Because of the
prolonged labor and a difficult delivery, there is a concern for fetomaternal hemorrhage. Which
of the following is the most appropriate next step in management?
 Incorrect Answer ImageA.Administer a 300 mcg dose of Rhogam
 Incorrect Answer ImageB.Clinically estimate fetomaternal hemorrhage and administer
300 mcg dose of Rhogam per 30 mL of fetal whole blood
 Incorrect Answer ImageC.Do not administer Rhogam because she has already received
adequate prophylaxis
 Incorrect Answer ImageD.Perform a Kleihauer-Betke test on maternal blood
 Correct Answer ImageE.Perform a rosette test on maternal blood

A 34-year-old woman, gravida 6, para 5, at 39 weeks’ gestation comes to labor and delivery
because of contractions lasting 6 hours. She reports that she had a large gush of fluid 15 minutes
ago. She has no other medical conditions. Examination of the cervix shows she is completely
dilated and a single fetal leg and foot is seen in the vaginal canal extending through the introitus.
Which of the following is the most likely diagnosis?
 Incorrect Answer ImageA.Cephalic presentation
 Incorrect Answer ImageB.Complete breech presentation
 Incorrect Answer ImageC.Compound presentation
 Incorrect Answer ImageD.Frank breech presentation
 Correct Answer ImageE.Incomplete breech presentation

A 32-year-old woman, gravida 2, para 1, at 28 weeks' gestation comes to the physician for a
prenatal visit. She is concerned that her baby is not moving as much as usual over the last 48
hours. She has a history of chronic hypertension for which she takes methyldopa. Her
temperature is 37ºC (98.6ºF), blood pressure is 118/78 mm Hg, pulse is 87/min, and respirations
are 14/min. Cardiovascular examination is unremarkable and there is no peripheral edema. Her
fundal height is 27 cm (2 weeks ago it was 25 cm). The fetal heart rate is 140/min. Which of the
following is the most appropriate next step in management?
 Incorrect Answer ImageA.Admit the patient to labor and delivery for 24 hours of
monitoring
 Incorrect Answer ImageB.Fetal biometry ultrasound
 Incorrect Answer ImageC.Induction of labor
 Correct Answer ImageD.Nonstress test
 Incorrect Answer ImageE.Reassurance and follow-up in a week
 Incorrect Answer ImageF.Umbilical artery blood flow ultrasonography

A 21-year-old woman comes to the physician because of a 3-month history of progressive


abdominal pain. She has a normal appetite. Examination shows mild right lower quadrant
abdominal tenderness. Pelvic examination shows mild right adnexal enlargement and tenderness.
Urine β-hCG is negative. Pelvic ultrasound shows a 5-cm, heterogeneous hyperechoic lesion
with cystic areas in the right adnexa; hair and calcifications are seen within the cystic areas. This
patient is at greatest risk of morbidity from which of the following?
 Incorrect Answer ImageA.Hemolytic anemia
 Incorrect Answer ImageB.Infection
 Incorrect Answer ImageC.Malignant degeneration
 Incorrect Answer ImageD.Rupture
 Correct Answer ImageE.Torsion

A 27-year-old woman comes to the physician because of vaginal discharge and itching that has
been progressive over the last 3 days. The discharge seems to be faintly malodorous. Her past
medical history is significant for candidiasis and bacterial vaginosis. She takes fluoxetine for
depression. She has no drug allergies. Pelvic examination with a speculum shows some greenish-
whitish discharge. There is a faint amount of vaginal erythema. The bimanual examination is
unremarkable. Which of the following is the most appropriate next step in diagnosis? 
 Incorrect Answer ImageA.Colposcopy
 Incorrect Answer ImageB.Gram stain of the vaginal discharge
 Incorrect Answer ImageC.Pelvic computed tomography (CT) scan
 Incorrect Answer ImageD.Pelvic ultrasound
 Correct Answer ImageE.Wet mount of the discharge

A 43-year-old woman, gravida 1, para 0, at 10 weeks' gestation comes to the physician for a
prenatal visit. She is feeling well except for some occasional nausea. She has no vaginal
discharge or bleeding, abdominal pain, dysuria, frequency, or urgency. Medications include folic
acid and a multivitamin. Her temperature is 36.7°C (98.1°F), pulse is 60/min, and blood pressure
is 105/70 mm Hg. Physical examination is unremarkable. Urine dipstick is positive for nitrites
and leukocyte esterase and a midstream urine culture shows 100,000 colony forming units per
milliliter of Escherichia coli. Which of the following is the most appropriate next step in
management?
 Incorrect Answer ImageA.Intravenous antibiotics
 Correct Answer ImageB.Oral antibiotics
 Incorrect Answer ImageC.Reassurance
 Incorrect Answer ImageD.Renal ultrasound
 Incorrect Answer ImageE.Repeat urine culture in a month

A 27-year-old woman comes to the physician because of a large mass in her left breast. She has
recently immigrated to the United States from El Salvador. The mass has been present for 7
years and has grown slowly to its present size. She has no history of major medical illness and
takes no medications. Her mother was diagnosed with breast cancer at the age of 60. Physical
examination shows a mass in the left breast 14 x 12 x 9-cm in size. It is firm, nontender, rubbery,
and completely movable. It is not attached to the overlying skin or the chest wall. There are no
palpable axillary nodes and no skin or nipple changes. Which of the following is the most likely
diagnosis?
 Incorrect Answer ImageA.Chronic cystic mastitis
 Incorrect Answer ImageB.Inflammatory breast cancer
 Incorrect Answer ImageC.Intraductal papilloma
 Incorrect Answer ImageD.Mammary duct ectasia
 Correct Answer ImageE.Phyllodes tumor

A 33-year-old woman, gravida 3, para 2, at 36 weeks' gestation comes to the physician for a
prenatal visit. She states that the fetus is moving regularly and she denies bleeding, loss of fluid,
or contractions. Her pregnancy has been uncomplicated thus far except for a urine culture
positive for group B beta-hemolytic Streptococcus in the first trimester. Her past obstetric history
is significant for 2 term vaginal deliveries. One of her children developed group
B Streptococcus (GBS) sepsis but is now doing well. She has no known drug allergies. Her blood
pressure is 114/78 mm Hg. Physical examination shows the fetal heart rate is 150/min and fundal
height is 38 cm. Urine dipstick shows no protein or glucose. Which of the following is the most
appropriate next step in management?
 Incorrect Answer ImageA.Obtain a culture for GBS at this visit
 Incorrect Answer ImageB.Offer the patient cesarean delivery at 39 weeks
 Incorrect Answer ImageC.Treat the patient with penicillin at this visit
 Incorrect Answer ImageD.Treat the patient with penicillin from 38 weeks until delivery
 Correct Answer ImageE.Treat the patient with penicillin in labor

A 28-year-old woman, gravida 1, para 0, at 8 weeks’ gestation comes to the physician for her
first prenatal visit. She has no complaints, and examination is consistent with an 8-week
pregnancy. She is concerned, however, because she is a carrier of the fragile X mutation. Her
husband is also known to be a carrier. This is a highly desired pregnancy, and she wants to know
if there is a way to determine whether the fetus is affected. Which of the following is the most
appropriate next step in management?
 Correct Answer ImageA.Second trimester amniocentesis
 Incorrect Answer ImageB.MRI of the fetus
 Incorrect Answer ImageC.Termination of the pregnancy
 Incorrect Answer ImageD.Testing of the parents
 Incorrect Answer ImageE.There is nothing to offer this couple
A 26-year-old woman, gravida 1, para 0, at 12 weeks' gestation comes to the emergency
department because of worsening pain and swelling in her right thigh for 2 days. She has no
other medical conditions and she has no history of thrombi. Ultrasonography of the lower
extremity venous system shows evidence of a proximal thrombus in the right leg. She is started
on low-molecular-weight heparin injections. Which of the following is an advantage to using
low-molecular-weight heparin compared with unfractionated heparin in this patient?
 Incorrect Answer ImageA.Low-molecular-weight heparin has a shorter half-life
 Incorrect Answer ImageB.Low-molecular-weight heparin is less expensive than
unfractionated heparin
 Incorrect Answer ImageC.Low-molecular-weight heparin is less likely to cause birth
defects
 Correct Answer ImageD.Low-molecular-weight heparin is less likely to cause
thrombocytopenia
 Incorrect Answer ImageE.Low-molecular-weight heparin is less likely to cross the
placenta

A 42-year-old woman, gravida 1, para 0, at 42 weeks’ gestation is admitted to the hospital for
induction of labor. She feels well. Pregnancy was uncomplicated. Pelvic examination shows
closed and uneffaced cervix. The fetal heart rate is 140/min with moderate variability.
Intravaginal prostaglandins are administered for cervical ripening. Four hours later, the cervix is
2 cm dilated and 60% effaced and oxytocin infusion is started. Once a regular contraction pattern
is established, amniotomy is performed. Immediate painless vaginal bleeding occurs. The fetal
heart rate rapidly drops up to 80/min. Which of the following is the most likely diagnosis?
 Incorrect Answer ImageA.Placenta accreta
 Incorrect Answer ImageB.Placenta increta
 Incorrect Answer ImageC.Placental abruption
 Incorrect Answer ImageD.Placenta percreta
 Correct Answer ImageE.Vasa previa

A 36-year-old woman, gravida 3, para 2, at 33 weeks' gestation comes to the physician for a
prenatal visit. She has some fatigue but no other complaints. Her current pregnancy has been
complicated by a Group B Streptococcus urine infection at 16 weeks. Her past obstetric history is
significant for a primary, classic cesarean delivery 5 years ago for a non-reassuring fetal tracing.
Two years ago, she had a repeat cesarean delivery. Past surgical history is significant for an
appendectomy 10 years ago. Which of the following is the major contraindication to a vaginal
birth after cesarean (VBAC) in this patient? 
 Correct Answer ImageA.Classic uterine scar
 Incorrect Answer ImageB.Group B Streptococcus urine infection
 Incorrect Answer ImageC.Previous appendectomy
 Incorrect Answer ImageD.Prior cesarean delivery for non-reassuring fetal heart tracing
 Incorrect Answer ImageE.Two prior cesarean deliveries
A 33-year-old woman, gravida 2, para 2, at 34 weeks' gestation comes to the emergency
department because of progressively worsening vaginal bleeding and shortness of breath for 12
hours. She was diagnosed with intrauterine fetal demise 2 weeks ago. She declined an induction
of labor at that time. Her temperature is 37.0ºC (98.6ºF), blood pressure is 100/65 mm Hg, pulse
is 110/min, and respirations are 16/min. Oxygen saturation is 86% on room air. Physical
examination shows scattered petechiae. Serum laboratory studies show:
Sodium 136 mEq/dL
Potassium 3.6 mEq/dL
Chloride 100 mEq/dL
Bicarbonate 18 mEq/L
Urea nitrogen 55 mg/dL
Creatinine 1.8 mg/dL
Leukocyte count 9,000/mm3
Hemoglobin 11.0 mg/dL
Hematocrit 33%
Platelets 50,000/mm3
Prothrombin time 25 seconds
Partial thromboplastin time 50 seconds
(activated)
Fibrinogen 100 mg/dL (normal 175–433
mg/dL)
D-dimer positive
A spiral CT scan of the chest shows a pulmonary embolism in the right lung. Which of the
following is the most likely cause of this patient's condition? 
 Incorrect Answer ImageA.Endotoxin production with monocyte activation
 Correct Answer ImageB.Release of thromboplastic substances from placental tissue
 Incorrect Answer ImageC.Trauma causing release of thromboplastic substances
 Incorrect Answer ImageD.Tumor release of thromboplastic substances
 Incorrect Answer ImageE.Widespread antigen-antibody complexes

A 31-year-old woman, gravida 3, para 2, at 9 weeks' gestation comes to the physician because of
vaginal bleeding for the last 2 days. The bleeding was particularly heavy over the last 24 hours.
Two weeks ago she came to the physician to start her prenatal care and an ultrasound performed
at that time showed a uterine pregnancy consistent with 7 weeks' gestation. Her obstetric history
is significant for 2 term vaginal deliveries. She has no medical problems and has never had
surgery. Her temperature is 37.0ºC (98.6ºF), blood pressure is 112/78 mm Hg, pulse is 82/min,
and respirations are 12/min. Physical examination shows a nontender abdomen, scant blood in
the vaginal vault with a closed cervix, and a nonenlarged uterus with no adnexal masses or
tenderness. Ultrasonography shows no intrauterine pregnancy and no remaining products of
conception. Which of the following is the most appropriate next step in management?
 Incorrect Answer ImageA.Check β-hCG titer in 2 days
 Incorrect Answer ImageB.Continue routine prenatal care
 Correct Answer ImageC.Offer counseling to the patient
 Incorrect Answer ImageD.Perform emergency dilation and curettage
 Incorrect Answer ImageE.Schedule dilation and curettage

A 39-year-old woman, gravida 4, para 3, comes to the physician for a prenatal visit. Her last
menstrual period was 8 weeks ago. She has had no abdominal pain or vaginal bleeding. Her past
medical history is unremarkable other than an allergic reaction to penicillin that caused a rash in
childhood. She does not take any medications. Her temperature is 36.7ºC (98.0ºF), blood
pressure is 110/65 mm Hg, pulse is 70/min, and respirations are 12/min. Physical examination
shows a nontender uterus consistent in size with an 8-week gestation. Prenatal screening
laboratory tests are sent. The rapid plasma reagin (RPR) test returns positive. A confirmatory
microhemagglutination assay for antibodies to Treponema pallidum (MHA-TP) test is sent and
results are also positive. Which of the following is the most appropriate pharmacotherapy?
 Incorrect Answer ImageA.Doxycycline
 Incorrect Answer ImageB.Erythromycin
 Incorrect Answer ImageC.Metronidazole
 Correct Answer ImageD.Penicillin
 Incorrect Answer ImageE.Tetracycline

A 37-year-old primigravid woman at 15 weeks' gestation comes to the physician to initiate


prenatal care. The patient has had no pain or bleeding. She takes a daily prenatal vitamin, has no
chronic medical problems, and has had no prior surgery. Vital signs are within normal limits.
Cell-free DNA testing is screen positive for trisomy 21. Bedside ultrasound shows a 15-week
intrauterine gestation with a normal fetal heart rate. Which of the following is the most
appropriate next step in management?
 Incorrect Answer ImageA.Chorionic villus sampling
 Incorrect Answer ImageB.Fetal anatomy ultrasound
 Correct Answer ImageC.Genetic amniocentesis
 Incorrect Answer ImageD.Quadruple marker test
 Incorrect Answer ImageE.Serum integrated test

A 23-year-old primigravid woman comes to the physician because of vaginal bleeding. Her last
menstrual period was 6 weeks ago. She has no other symptoms. Physical examination shows a
uterus consistent in size with 10-week gestation, but is otherwise unremarkable. Pelvic
ultrasound shows a snowstorm pattern consistent with a complete mole. Serum beta-hCG is
markedly elevated over normal pregnancy values. A chest x-ray is negative. A dilation and
evacuation is performed and the pathologic diagnosis is complete hydatidiform mole. Which of
the following is the most appropriate next step in management?
 Incorrect Answer ImageA.Dactinomycin
 Incorrect Answer ImageB.Evaluation in 1 year
 Correct Answer ImageC.Follow beta-hCG levels to 0
 Incorrect Answer ImageD.Hysterectomy
 Incorrect Answer ImageE.Methotrexate
A 39-year-old woman, gravida 3, para 2, at 40 weeks' gestation comes to the labor and delivery
ward after a gush of fluid with regular, painful contractions every two minutes. She is found to
have rupture of the membranes and a cervix that is 5 cm dilated, a fetus in vertex presentation,
and a reassuring fetal heart rate tracing. She is admitted to the labor and delivery ward. Two
hours later she states that she feels hot and sweaty. Temperature is 38.3ºC (101ºF). She has mild
uterine tenderness. Her cervix is now 8 cm dilated and the fetal heart tracing is reassuring. Which
of the following is the most appropriate management of this patient? 
 Incorrect Answer ImageA.Administer antibiotics to the mother after vaginal delivery
 Correct Answer ImageB.Administer antibiotics to the mother now and allow vaginal
delivery
 Incorrect Answer ImageC.Perform cesarean delivery
 Incorrect Answer ImageD.Perform cesarean delivery and then administer antibiotics to
the mother
 Incorrect Answer ImageE.Perform intra-amniotic injection of antibiotics

A 22-year-old woman, gravida 2, para 1, at 28 weeks' gestation comes to the physician for a
prenatal visit. She states that the fetus is moving very well and she has no bleeding, contractions,
or loss of fluid. She had nausea and vomiting throughout the first trimester, but her pregnancy
has been otherwise uncomplicated. Her obstetric history is significant for a term vaginal delivery
after a normal prenatal course. She has mild intermittent asthma, for which she occasionally uses
an inhaler, and has no other medical problems. Her blood pressure is 106/66 mm Hg and pulse is
78/min. Physical examination shows the fetal heart rate is 150/min and her fundal height is 29
cm. Laboratory studies show a 50 gram 1-hour glucose challenge test is 148 mg/dL and urine
dipstick is negative for protein and glucose. Which of the following is the most appropriate next
step in management?
 Incorrect Answer ImageA.Induce the patient for diabetes
 Incorrect Answer ImageB.Reassure the patient that the result is normal
 Correct Answer ImageC.Schedule a 100 gram, 3-hour oral glucose tolerance test
 Incorrect Answer ImageD.Start the patient on metformin
 Incorrect Answer ImageE.Start the patient on insulin

A 29-year-old woman, gravida 2, para 1, comes to the physician at 12 weeks' gestation for her
second prenatal visit. She is feeling well except for some mild nausea. Her past obstetric history
is significant for a term vaginal delivery following an uncomplicated pregnancy 3 years ago. She
has no medical problems and has never had surgery. She takes prenatal vitamins and is allergic
to penicillin. Her blood pressure is 100/70 mm Hg and weight is 54.4 kg (120 lb). The fetal pulse
is in the 160s. A review of her prenatal laboratory studies shows that her Pap smear is reported as
high-grade squamous intraepithelial lesion (HSIL). Which of the following is the most
appropriate next step in management? 
 Correct Answer ImageA.Perform colposcopy
 Incorrect Answer ImageB.Perform endocervical curettage
 Incorrect Answer ImageC.Recommend termination of pregnancy
 Incorrect Answer ImageD.Repeat the Pap smear after delivery
 Incorrect Answer ImageE.Schedule the patient for a cone biopsy
A 26-year-old woman, gravida 4, para 0, at 28 weeks' gestation comes to the physician for a
prenatal care visit. She says that she sometimes feels the fetus moving, but not at other times.
She has had no bleeding, loss of fluid, or contractions. Her past obstetric history is significant for
spontaneous abortions at 9, 12, and 15 weeks of gestation. She has no past medical history and
has never had surgery. This pregnancy has been uncomplicated thus far. Physical examination
shows no detectable fetal heart rate, and she is sent immediately for an ultrasound. The
ultrasound shows a 23-week fetal demise. After discussion, the patient states that she desires an
autopsy to be performed on the fetus and that she does not want expectant management. Which
of the following is the most appropriate next step in management?
 Incorrect Answer ImageA.Continue routine prenatal care
 Incorrect Answer ImageB.Schedule cesarean delivery
 Incorrect Answer ImageC.Schedule dilation and evacuation
 Correct Answer ImageD.Schedule induction of labor
 Incorrect Answer ImageE.Schedule repeat ultrasound

A 26-year-old woman, gravida 1, para 0, at 35 weeks' gestation comes to the labor and delivery
department because of painful uterine contractions and a gush of fluid. Sterile speculum
examination shows a pool of clear fluid in the vagina that is nitrazine positive. When the fluid is
allowed to dry on a slide and examined under the microscope, an arborizing pattern is seen.
Cervical examination shows the patient to be 4 cm dilated, 100% effaced, and the vertex is at 0
station. Fetal fingers can be felt alongside the fetal head. External uterine monitoring shows
contractions every 2 minutes. External fetal monitoring shows a fetal heart rate of 130/min and
the tracing is reactive. Which of the following is the most appropriate next step in management?
 Incorrect Answer ImageA.Cesarean section
 Correct Answer ImageB.Expectant management
 Incorrect Answer ImageC.Forceps delivery
 Incorrect Answer ImageD.Oxytocin augmentation
 Incorrect Answer ImageE.Vacuum delivery

A 22-year-old woman, gravida 3, para 1, aborta 1, at 40 weeks' gestation comes to the emergency
department with intense abdominal pain for the past 8 hours. Gestational age was confirmed by
ultrasound at 18 weeks’ gestation, and her pregnancy has been uneventful so far. In the
emergency department, she has contractions every 3 minutes, lasting 50 seconds each. Pelvic
examination shows the cervix as 70% effaced, 6 cm dilated, and the vertex at zero station. She is
transferred to labor and delivery, where an epidural anesthesia is administered to help ease the
pain. Four hours later, a physical exam shows that the cervix is still 70% effaced and 6 cm
dilated, and the vertex is at zero station. The fetal heart rate tracing is reassuring. Which of the
following is the most likely diagnosis?
 Correct Answer ImageA.Active phase arrest
 Incorrect Answer ImageB.Prolonged active phase
 Incorrect Answer ImageC.Prolonged latent phase
 Incorrect Answer ImageD.Prolonged second stage of labor
 Incorrect Answer ImageE.Prolonged third stage of labor
 Incorrect Answer ImageF.Second stage arrest

A 23-year-old woman, gravida 1, para 0, at 33 weeks' gestation comes to the physician for a
routine prenatal visit. She states that the fetus is moving well and that she has had no
contractions, bleeding from the vagina, or leakage of fluid. She has had no headaches, visual
changes, or abdominal pain. Her pregnancy has been uncomplicated up to this point. She has no
medical problems. She takes prenatal vitamins and has no known drug allergies. Her blood
pressure is 150/94 mm Hg and 148/92 mm Hg when repeated. Urine dipstick shows 2+
proteinuria. After 24 hours of observation in the hospital, she remains asymptomatic; 24-hour
urine shows 410 mg protein, and blood pressures continue to be in the 140-150/90-100 mm Hg
range. Complete blood count and liver function tests are within normal limits. Anatomic
ultrasound of the fetus shows a vertex fetus in the 75th percentile for weight with a biophysical
profile of 10/10. Which of the following is the most appropriate next step in management?
 Correct Answer ImageA.Admit to the hospital and manage expectantly
 Incorrect Answer ImageB.Discharge to her home and continue routine prenatal care
 Incorrect Answer ImageC.Initiate immediate induction of labor
 Incorrect Answer ImageD.Perform emergent cesarean delivery
 Incorrect Answer ImageE.Schedule cesarean delivery for 35 weeks' gestation

A 39-year-old African American woman, gravida 6, para 5, at 34 weeks' gestation comes to the
emergency department because of vaginal bleeding that started 1 hour ago while she was
shopping in the mall. She does not have any pain, contractions, or leakage of clear fluid. Her first
trimester ultrasound showed a dichorionic diamniotic twin pregnancy. She has regular prenatal
care, smokes six cigarettes a day, and uses cocaine occasionally. Aside from the tobacco and
cocaine use, her pregnancy has been uncomplicated. Today, her temperature is 37.2ºC (99.0ºF),
blood pressure is 91/60 mm Hg, pulse is 110/min, and respirations are 12/min. Physical
examination shows that her pants and underwear are soaked with blood. Two fetal heart rates are
auscultated with a baseline of 140/min. Two large-bore intravenous catheters are placed and a
bolus of 500 mL of Ringer's lactate is administered, followed by infusion at a rate of 200 mL/hr.
Which of the following is the most appropriate next step in management?
 Correct Answer ImageA.Abdominal ultrasound
 Incorrect Answer ImageB.Bimanual examination
 Incorrect Answer ImageC.Cesarean delivery
 Incorrect Answer ImageD.Speculum examination
 Incorrect Answer ImageE.Transvaginal ultrasound

A 35-year-old woman, gravida 3, para 2, at 32 weeks' gestation comes to the emergency


department because of a severe headache that has not been relieved with acetaminophen at home.
She states that the headache has been accompanied by "floating spots" in her vision. She has a
history of preeclampsia in her first pregnancy, which required delivery at 37 weeks. Her
temperature is 37ºC (98.6ºF), blood pressure is 164/104 mm Hg, pulse is 87/min, and
respirations are 13/min. Physical examination shows 2+ pedal edema, a fundal height of 32 cm,
and mild epigastric tenderness. Urinalysis shows 4+ proteinuria, negative leukocyte esterase,
negative blood, and negative glucose. Serum laboratory studies show:
Sodium  136 mEq/dL 
Potassium  3.6 mEq/dL 
Chloride  100 mEq/dL 
Bicarbonate  18 mEq/L 
Urea nitrogen  55 mg/dL 
Creatinine  0.8 mg/dL 
Leukocyte count  9,000/mm3
Hemoglobin  10.0 mg/dL 
Hematocrit  31% 
Platelets  175,000/mm3
Alanine aminotransferase (ALT)  28 U/L
Aspartate aminotransferase (AST) 32 U/L
After the initial evaluation, the patient begins to have tonic-clonic seizure activity. The seizure
resolves after 2 minutes without treatment. Which of the following is the most appropriate next
step in management?
 Incorrect Answer ImageA.Diazepam 10 mg every 10 minutes for three doses
 Incorrect Answer ImageB.Immediate delivery via Caesarean section
 Incorrect Answer ImageC.Induction of labor
 Correct Answer ImageD.Magnesium sulfate 6-g load over 15 minutes
 Incorrect Answer ImageE.Phenytoin 15 mg/kg loading dose

A 27-year-old woman, gravida 3, para 0, aborta 2, at 7 weeks' gestation comes to the physician
because of abdominal pain and vaginal bleeding with blood clots since last night. Her pregnancy
was confirmed 2 weeks ago with a urine pregnancy test. Her menstrual periods are irregular. She
had one spontaneous abortion at 8 weeks’ gestation 3 years ago, and was hospitalized for an
ectopic pregnancy that was treated with methotrexate two years ago. Today, her temperature is
36.7ºC (98.0ºF), blood pressure is 115/75 mm Hg, pulse is 75/min, and respirations are 12/min.
On pelvic examination, the uterus is not palpable; the cervix is closed and there is a small
amount of blood in the vaginal vault. Transvaginal ultrasound shows no signs of intrauterine
pregnancy. Serum laboratory studies show her blood type is AB+. Which of the following is the
most appropriate next step in management?
 Incorrect Answer ImageA.Diagnostic laparoscopy
 Incorrect Answer ImageB.Dilation and curettage
 Incorrect Answer ImageC.Misoprostol
 Correct Answer ImageD.Observation and serial beta-hCG measurements
 Incorrect Answer ImageE.RhoGAM

A 24-year-old woman, gravida 1, para 0, at 35 weeks' gestation comes to the physician because
of decreased fetal movement over the last 12 hours. She has a history of chronic hypertension
secondary to moderate systemic lupus erythematosus. Her hypertension has been well-controlled
with methyldopa until this time. She takes no other medications. Her blood pressure is 132/82
mm Hg, pulse is 86/min, and respirations are 16/min. Fundal height is 35 cm (34 cm one week
ago). Fetal heart rate averages 150/min. A nonstress test is performed and fails to reveal
accelerations in the fetal heart rate greater than 15/min over 20 minutes. Vibroacoustic
stimulation is applied and there are still no accelerations. Which of the following is the most
appropriate next step in management?
 Incorrect Answer ImageA.Admit the patient to labor and delivery for 24 hours of
monitoring
 Correct Answer ImageB.Biophysical profile
 Incorrect Answer ImageC.Fetal biometry ultrasound
 Incorrect Answer ImageD.Induction of labor
 Incorrect Answer ImageE.Reassurance and follow-up in a week
 Incorrect Answer ImageF.Umbilical artery blood flow ultrasonography

A 23-year-old woman, gravida 3, para 2, at 42 weeks' gestation comes to the physician because
of decreased fetal movements over the last 48 hours. She has only had 3 prenatal visits during
this pregnancy, all of which revealed normal findings. Her past medical history is positive for
cocaine use, and she states that she last used 2 weeks ago. Her temperature is 37.2ºC (98.9ºF),
blood pressure is 140/90 mm Hg, pulse is 90/min, and respirations are 16/min. Physical
examination shows fundal height of 41 cm; a closed, posterior, and firm cervix. A modified
biophysical profile shows variable decelerations with fetal heart rate at 160/min and an amniotic
fluid index of 3. She is given oxygen and moved to the left lateral decubitus position for 15
minutes. Which of the following is the most likely cause of the fetal heart tracings?
 Incorrect Answer ImageA.Head compression
 Incorrect Answer ImageB.Hypertension
 Correct Answer ImageC.Low amniotic fluid index
 Incorrect Answer ImageD.Post-term pregnancy
 Incorrect Answer ImageE.Use of illicit drugs

A 25-year-old woman, gravida 2, para 1, at 22 weeks' gestation, comes to the physician with
complaints of burning with urination and frequent urination. Her prenatal course has been
uncomplicated except for a urinary tract infection (UTI) with E. coli at 12 weeks' gestation,
which was treated at that time. Physical examination is unremarkable. Urine culture
demonstrates >100,000 colony-forming units per milliliter of E. coli. Which of the following is
the most appropriate next step in management? 
 Incorrect Answer ImageA.Intravenous antibiotics
 Incorrect Answer ImageB.Intravenous pyelography
 Incorrect Answer ImageC.Reassurance
 Incorrect Answer ImageD.Renal ultrasound
 Correct Answer ImageE.Suppressive daily antibiotics

A 34-year-old woman, gravida 2, para 1, at 32 weeks' gestation with a twin pregnancy comes to
labor and delivery because of painless bright red vaginal bleeding for the past 2 hours. She
denies uterine contractions. Her pregnancy has been uneventful thus far. Her temperature is 37ºC
(98.6ºF), blood pressure is 117/67 mm Hg, pulse is 110/min, and respirations are 12/min. Pelvic
examination shows bright red blood on the perineum. Fetal heart rates are 140/min and 144/min
with moderate variability. Which of the following is the most likely cause of the vaginal
bleeding?
 Correct Answer ImageA.Avulsion of anchoring villi in the lower uterine segment
 Incorrect Answer ImageB.Ruptured fetal vessels traversing the cervical os
 Incorrect Answer ImageC.Separation of the placenta from the upper uterine segment
 Incorrect Answer ImageD.Villus invasion into the myometrium
 Incorrect Answer ImageE.Villus invasion into the urinary bladder

A 34-year-old woman, gravida 1, para 0, at 30 weeks gestation comes to the labor and delivery
department with regular contractions every 6 minutes. Her prenatal course was significant for
type 1 diabetes, which she has had for 16 years. Over the course of 1 hour, she continues to
contract, and her cervix advances from closed and long to a fingertip of dilation with some
effacement. The patient is started on magnesium sulfate, penicillin, and betamethasone. Which of
the following is the most likely side effect from the administration of betamethasone to this
patient?
 Incorrect Answer ImageA.Decreased childhood intelligence
 Correct Answer ImageB.Increased maternal insulin requirement
 Incorrect Answer ImageC.Maternal infection
 Incorrect Answer ImageD.Neonatal adrenal suppression
 Incorrect Answer ImageE.Neonatal infection

A 29-year-old woman, gravida 2, para 1, at 36 weeks' gestation, comes to the physician for a
prenatal visit. Her pregnancy was complicated by a primary outbreak of genital herpes at 26
weeks' gestation. She is now feeling well and with no complaints. The fetus is moving well and
she has had no bleeding, loss of fluid, or contractions. Her obstetric history is notable for a term
vaginal delivery 2 years ago following an uncomplicated prenatal course. Her blood pressure is
100/60 mm Hg, and her urine dipstick test is negative. Physical examination shows a fetal heart
rate in the 150s and a fundal height of 37 cm. Pelvic examination shows normal external
genitalia without evidence of herpetic lesions. Which of the following is the most appropriate
next step in management? 
 Correct Answer ImageA.Offer acyclovir prophylaxis
 Incorrect Answer ImageB.Recommend cesarean delivery at 39 weeks
 Incorrect Answer ImageC.Recommend immediate cesarean delivery
 Incorrect Answer ImageD.See the patient for a follow-up visit in 1 week
 Incorrect Answer ImageE.Send the patient to labor and delivery for induction

A 30-year-old woman, gravida 2, para 1, at 7 weeks' gestation comes to the physician for an
initial prenatal visit. She has some nausea and also complains of feeling weak. Her obstetric
history is significant for a term vaginal delivery following a normal prenatal course 3 years ago.
She had a cholecystectomy at age 27 years and has no other medical history. She takes prenatal
vitamins and has no known drug allergies. Initial physical examination is normal for a woman at
7 weeks' gestation. Laboratory studies are normal except for a thyroid-stimulating hormone of
10.8 mU/L and free thyroxine (free T4) of 0.76 ng/dL (normal 0.8–2.8 ng/dL). Which of the
following is the most appropriate next step in management?
 Incorrect Answer ImageA.Continue routine prenatal care
 Incorrect Answer ImageB.Recheck thyroid function tests in the second trimester
 Incorrect Answer ImageC.Start methimazole therapy
 Incorrect Answer ImageD.Start propylthiouracil therapy
 Correct Answer ImageE.Start thyroid hormone therapy

A 39-year-old woman, gravida 3, para 2, at 40 weeks’ gestation is brought to the emergency


department for spontaneous membrane rupture and labor. At the time of admission, the fetal
heart tracing shows a baseline of 140/min, moderate long term variability, spontaneous
accelerations, and no decelerations. The tocodynamometer shows contractions every 3 minutes.
Digital cervical examination shows the cervix to be 7 cm dilated and 100% effaced with the fetal
head at -1 station. An epidural is placed and the patient is managed expectantly. Two hours after
admission the patient is reevaluated. Vital signs are within normal limits. The fetal heart rate
tracing shows a baseline of 140/min, with minimal variability and shallow decelerations after
each uterine contraction. The tocodynamometer shows contractions every 3 minutes. Digital
cervical examination shows the cervix to be unchanged. Which of the following is the most
appropriate next step in management?
 Correct Answer ImageA.Left lateral positioning
 Incorrect Answer ImageB.Operative vaginal delivery
 Incorrect Answer ImageC.Oxytocin administration
 Incorrect Answer ImageD.Saline amnioinfusion
 Incorrect Answer ImageE.Urgent cesarean delivery

A 24-year-old woman, gravida 1, para 0, at 35 weeks' gestation comes to the emergency


department after a motor vehicle accident. The patient was in the passenger seat when her car
rear-ended another car. She was wearing a seatbelt and experienced some abdominal trauma. The
airbags deployed. Since the accident, the patient has had mild contractions every 5 to 10 minutes.
Fetal movement has been normal. There has been no leakage of fluid or vaginal bleeding. The
patient’s pregnancy was previously uncomplicated. She takes a daily prenatal vitamin, has no
chronic medical problems, and has had no prior surgery. Vital signs are within normal limits.
The fetal heart rate is 140/min. Physical examination shows mild lower back pain and bruises on
her lower abdomen. Speculum examination shows no vaginal bleeding. Digital cervical
examination shows the cervix to be long, closed and posterior. Hemoglobin is 11.4 mg/dL and
blood type is O+. Ultrasound shows the fetus to be in frank breech presentation. Which of the
following is the most appropriate next step in management?
 Incorrect Answer ImageA.Biophysical profile
 Correct Answer ImageB.Electronic fetal monitoring
 Incorrect Answer ImageC.External cephalic version
 Incorrect Answer ImageD.Kleihauer-Betke test
 Incorrect Answer ImageE.Magnesium sulfate
 Incorrect Answer ImageF.Terbutaline sulfate
A 29-year-old woman, gravida 2, para 1, at 36 weeks' gestation comes to the physician to initiate
prenatal care. Fetal movement has been normal. The patient has had no bleeding, pain, or
leakage of fluid. The patient’s prior pregnancy 1 year ago was uncomplicated and ended at 38
weeks’ gestation with a normal vaginal delivery of a healthy 3,200 g (7 lb) male infant. She takes
a daily prenatal vitamin and has no chronic medical problems. A fasting blood glucose prior to
the current pregnancy was within normal limits. Blood pressure is 106/78 mm Hg and BMI is 34
kg/m2. The fetal heart rate is 150/min. A dipstick urinalysis is positive for glucose and negative
for protein. Routine prenatal laboratory testing is performed including a 50 g one-hour glucose
screen which shows a plasma glucose level of 215 mg/dL. For which of the following
complications is this patient most likely at risk?
 Incorrect Answer ImageA.Agenesis of the fetal sacrum
 Incorrect Answer ImageB.Fetal cardiac defect
 Incorrect Answer ImageC.Fetal growth restriction
 Correct Answer ImageD.Neonatal clavicular fracture
 Incorrect Answer ImageE.Neonatal hyperglycemia

A 29-year-old woman, gravida 2, para 1, at 39 weeks' gestation comes to the emergency


department because of decreased fetal movement. The patient has had no contractions, vaginal
bleeding, or leakage of fluid. Her pregnancy has been uncomplicated. Her prior pregnancy, 3
years ago, ended with a normal spontaneous vaginal delivery of a 4677-g (10-lb 5-oz) healthy
boy. She has no medical problems and has had no previous surgery. Vital signs are within
normal limits. The fetal heart rate at baseline is 150/min. Digital examination shows the cervix to
be long, closed, and posterior. Ultrasound shows a fetus in cephalic presentation, an estimated
fetal weight of 5120-g (11-lb 4.6-oz), and a single deepest pocket of amniotic fluid of 1.1 cm.
Which of the following is the most appropriate next step in management?
 Incorrect Answer ImageA.Amniotomy and amnioinfusion
 Correct Answer ImageB.Cesarean delivery
 Incorrect Answer ImageC.Intravenous oxytocin
 Incorrect Answer ImageD.Outpatient expectant management
 Incorrect Answer ImageE.Vaginal misoprostol

A 31-year-old woman, gravida 1, para 0, at 36-weeks' gestation with twins comes to the
physician for a routine prenatal visit. The patient has had no contractions, bleeding from the
vagina, or loss of fluid and the babies are moving well. Based on today's ultrasound, the first
fetus is in vertex presentation with an estimated fetal weight of 2,450 g (5 lb 6 oz). The second
fetus is in breech presentation with an estimated fetal weight of 2,300 g (5 lb 1 oz). The patient
wants to know if she should have a vaginal or cesarean delivery. Which of the following is the
most appropriate counseling for this patient?
 Correct Answer ImageA.Both vaginal delivery and cesarean delivery are acceptable
 Incorrect Answer ImageB.Cesarean delivery is mandated because the fetuses are >2,000
g
 Incorrect Answer ImageC.Cesarean delivery is mandated because the second twin is
breech
 Incorrect Answer ImageD.Vaginal delivery is mandated because the fetuses are >2,000 g
 Incorrect Answer ImageE.Vaginal delivery is mandated because the first twin is vertex

A 35-year-old woman, gravida 3, para 2, at 38 weeks' gestation is admitted to the hospital in


labor. The patient received no prenatal care. Her first 2 pregnancies ended with uncomplicated
vaginal deliveries of normal male infants. She quickly delivers a female infant who weighs 2,270
g (5 lb), which is <10th percentile. Apgars are 8 and 8 at 1 and 5 minutes, respectively. Physical
examination of the infant shows upslanting palpebral fissures, epicanthic folds, low-set ears,
brachycephaly, and poor muscle tone. Which of the following is the most likely cause of the
infant’s condition?
 Correct Answer ImageA.Meiotic nondisjunction
 Incorrect Answer ImageB.Missense mutation
 Incorrect Answer ImageC.Partial deletion of chromosome 5
 Incorrect Answer ImageD.Ring chromosome X formation
 Incorrect Answer ImageE.Trinucleotide repeat expansion

A 26-year-old woman, gravida 1, para 0, at 16 weeks' gestation comes to the physician because
of palpitations and anxiety. She has had no complications in the pregnancy thus far. Her
temperature is 37.0ºC (98.6ºF), blood pressure is 110/80 mm Hg, pulse is 83/min, and
respirations are 18/min. Physical examination shows no tremor, exophthalmos, or myxedema.
Her thyroid gland is palpable but not enlarged. Serum laboratory studies show:
Thyroid-stimulating 3.0 µU/L (reference range 0.2–3.0 µU/L for
hormone second trimester)
Thyroxine (T4) High
Free thyroxine (T4) Normal
Which of the following is the most appropriate next step in management?
 Incorrect Answer ImageA.Methimazole
 Incorrect Answer ImageB.Propylthiouracil
 Incorrect Answer ImageC.Radioactive iodine
 Correct Answer ImageD.Reassurance
 Incorrect Answer ImageE.Subtotal thyroidectomy

A 21-year-old woman, gravida 1, para 0, at 39 weeks' gestation comes to the labor and delivery
ward with painful contractions every 3 minutes. Her prenatal course was unremarkable.
Examination shows her cervix to be 3 cm dilated and 90% effaced. External fetal monitoring
shows a fetal heart rate of 150/min with moderate variability. Five hours later, cervical
examination shows that the patient is 9 cm dilated and 100% effaced and the vertex is at -1
station. The fetal heart rate tracing shows moderate variable decelerations with each contraction,
absent variability, and baseline fetal heart rate of 100/min. There is no improvement after
administration of maternal oxygen, an IV fluid bolus, and maternal repositioning. There are no
fetal heart rate accelerations noted with scalp stimulation. Which of the following is the most
appropriate next step in management?
 Correct Answer ImageA.Cesarean delivery
 Incorrect Answer ImageB.Episiotomy
 Incorrect Answer ImageC.Expectant management
 Incorrect Answer ImageD.Forceps-assisted vaginal delivery
 Incorrect Answer ImageE.Vacuum-assisted vaginal delivery

A 41-year-old woman, gravida 1, para 0, at 32 weeks' gestation comes to the emergency room
because of a headache, abdominal pain, and spots in her vision. The patient became pregnant via
in vitro fertilization and has a dichorionic, diamniotic twin gestation. She takes acetaminophen as
needed for migraine headaches and a daily prenatal vitamin. She has had no prior surgery. Her
blood pressure is 174/104 mm Hg and pulse is 87/min. The fetal heart rate is 150/min. Physical
examination shows 2+ pedal edema and mild epigastric tenderness. A dipstick urinalysis shows
3+ proteinuria. Laboratory studies show:
Hemoglobin  11.1 mg/dL 
Platelet count  135,000/mm3 
Creatinine  0.8 mg/dL
Aspartate aminotransferase (AST)  62 U/L
Alanine aminotransferase (ALT) 78 U/L 
Peripheral blood smear shows no schistocytes or spherocytes. Which of the following is the most
likely diagnosis?
 Incorrect Answer ImageA.Gestational hypertension
 Incorrect Answer ImageB.HELLP syndrome
 Incorrect Answer ImageC.Migraine headache with aura
 Incorrect Answer ImageD.Pheochromocytoma
 Correct Answer ImageE.Preeclampsia

A 22-year-old woman in active labor progresses to 7 cm dilation and then has an arrest of labor.
She subsequently undergoes a primary cesarean section. On postoperative day 2, her temperature
is 39.1ºC (102.4ºF), blood pressure is 110/70 mm Hg, pulse is 90/min, and respirations are
14/min. The lungs are clear to auscultation bilaterally. Her abdomen is moderately tender. The
dressing is clean, dry, and intact. The incision is approximated with no evidence of erythema.
Pelvic examination shows uterine tenderness. Which of the following is the most appropriate
pharmacotherapy?
 Incorrect Answer ImageA.Ampicillin
 Incorrect Answer ImageB.Ampicillin-gentamicin
 Correct Answer ImageC.Clindamycin-gentamicin
 Incorrect Answer ImageD.Clindamycin-metronidazole
 Incorrect Answer ImageE.Metronidazole
Pediatria
A premature male neonate weighing 1,190 g (2 lb 10 oz) is found to have a machine-like heart
murmur. He is diagnosed with a patent ductus arteriosus on echocardiogram and is successfully
treated with indomethacin. Oral intake is started on the third day of life, and shortly thereafter the
infant develops feeding intolerance and abdominal distention. By day 4 he has a rapidly
decreasing platelet count. Abdominal radiograph shows distended loops of bowel throughout the
abdomen, and there is intramural air in some of the loops. He does not have air in the biliary tree
or pneumoperitoneum. Which of the following is the most likely diagnosis?
 Incorrect Answer ImageA.Intussusception
 Incorrect Answer ImageB.Malrotation with Ladd bands
 Incorrect Answer ImageC.Meconium ileus
 Incorrect Answer ImageD.Mesenteric embolus
 Correct Answer ImageE.Necrotizing enterocolitis

A 2-year-old girl is brought to the emergency department because she looked "unwell" and has
not wanted to get out of bed for 24 hours. She had a fever 1 week ago associated with bloody
diarrhea, from which she had recovered. However, 2 days ago she developed reduced appetite
and lethargy. She has had only 2 wet diapers in the last 36 hours. She was previously a well child
with normal growth and development. Examination shows an irritable, pale child with petechiae
and edema. Her blood pressure is 102/74 mm Hg, pulse is 136/min, and respirations are 26/min.
Lung examination shows crackles bilaterally and her abdomen is diffusely tender but with no
guarding or rigidity. Laboratory studies show:
Hemoglobin 7.1 g/dL
White cells 22 x 109/L 
Differential Polymorphs 38%, lymphocytes 52%,
monocytes 8%
Platelets 80 x 109/L 
Blood film Fragmented red blood cells
Prothrombin time 12.1 sec
Partial thromboplastin 32 sec
time
Serum sodium 132 mEq/L
Serum potassium 6.3 mEq/L
Serum chloride 106 mEq/L
Serum bicarbonate 16 mEq/L
Blood urea nitrogen 32 mg/dL
Serum creatinine 3.1 mg/dL
Which of the following is the most appropriate next step in management? 
 Incorrect Answer ImageA.Administer a normal saline bolus of 20 mg/L/k
 Incorrect Answer ImageB.Administer methylprednisolone
 Correct Answer ImageC.Begin peritoneal dialysis
 Incorrect Answer ImageD.Begin plasmapheresis
 Incorrect Answer ImageE.Transfuse red blood cells

A 5-month-old girl is brought to the physician for a well-child visit. She has been doing well
until 1 week ago when she suddenly became fussy and began drooling excessively. Her feedings
have also been affected and she stops in the middle of breastfeeding and starts furiously rubbing
her gums over anything within reach. In the office, she is smiling and playing with plastic rings
that she repeatedly chews on. Her bib is soaked with saliva. Her vital signs are within normal
limits and she is in the fiftieth percentile for weight and height. Physical examination shows a
bluish discoloration of the gums in several areas. The rest of the physical examination shows no
abnormalities. The physician explains to the parents that the child's current symptoms will likely
resolve as soon as her first teeth erupt. Which of the following teeth are most likely to erupt first?
 Correct Answer ImageA.Mandibular central incisors
 Incorrect Answer ImageB.Mandibular lateral incisors
 Incorrect Answer ImageC.Maxillary central incisors
 Incorrect Answer ImageD.Maxillary first molars
 Incorrect Answer ImageE.Maxillary lateral incisors

A 17-year-old boy is brought to the physician because of a cough for 2 weeks. He has had two
episodes of non-bloody vomiting after coughing. He had a runny nose, fever, sneezing, and a
mild cough 3 weeks ago. He denies sore throat or difficulty breathing. His childhood
immunizations are incomplete because his parents did not want him vaccinated. He is allergic to
macrolides. Physical examination shows no abnormalities. A radiograph of the chest is within
normal limits. A nasopharyngeal swab is pending. Which of the following is the most
appropriate next step in management? 
 Incorrect Answer ImageA.Amoxicillin
 Incorrect Answer ImageB.Azithromycin
 Incorrect Answer ImageC.Clarithromycin
 Incorrect Answer ImageD.Supportive therapy with antitussive medications
 Correct Answer ImageE.Trimethoprim-sulfamethoxazole

A 6-month-old boy is brought to the emergency department by his mother, who states that when
she picked him up from the babysitter he was "not acting right." The babysitter stated that he was
sleeping more and was fussy. His temperature is 37.8°C (99.9°F), pulse is 140/min, and
respirations are 36/min. Physical examination shows a 4-cm ecchymosis on the right cheek and
the child appears stuporous. Fundoscopic examination shows retinal hemorrhages. The
remainder of the physical examination is unremarkable. A CT scan of the head, skeletal survey,
chemistry panel, and complete blood count are ordered. Which of the following is the most
appropriate next step in management?
 Incorrect Answer ImageA.Ammonia level
 Correct Answer ImageB.Coagulation studies
 Incorrect Answer ImageC.Immediate hospitalization
 Incorrect Answer ImageD.Report to Child Protection Services
 Incorrect Answer ImageE.Thyroid studies

A 10-month old infant is brought to the physician because of a seizure one hour ago. His parents
report that he was in his usual state of health until this morning. He has not been able to feed all
day and has been unusually somnolent. He vomited twice in the past 2 hours. The parents do not
recall any traumatic event, however, the mother states that she is constantly frustrated with her
son and has to discipline him regularly. The child has not had any recent illnesses and has been
afebrile. His past medical history is significant for chickenpox 3 months ago that recovered
within 3 weeks. He was born at 34 weeks' gestation and is up to date on his immunizations. He
has been meeting his developmental milestones and has not had any sick contacts. On physical
examination, his temperature is 37.2ºC (99.0ºF), blood pressure is 90/50 mm Hg, and pulse is
110/min. He is somnolent and responds minimally to voice, palpation, or toys. His neurologic
examination shows no focal lesions. Cardiovascular, pulmonary, and abdominal examinations
are normal. Which of the following is the most appropriate next step in management?
 Incorrect Answer ImageA.Check serum ammonia levels
 Incorrect Answer ImageB.Check serum electrolytes and glucose
 Incorrect Answer ImageC.Obtain an CT of the head
 Correct Answer ImageD.Perform a fundoscopic examination
 Incorrect Answer ImageE.Perform a lumbar puncture
A 5-year-old girl is brought to the physician by her mother because of the development of
axillary hair and breast budding for 2 months. She began to have vaginal bleeding 1 day ago. She
had a tibial fracture 1 year ago. She takes no medications. She is in the 60th percentile for both
height and weight. Breast and axillary and pubic hair development are Tanner stage 2. Skin
examination shows a café au lait spot. Pelvic examination shows blood at the introitus. Which of
the following is the most likely diagnosis?
 Incorrect Answer ImageA.Granulosa cell tumor
 Incorrect Answer ImageB.Incomplete isosexual precocious puberty
 Incorrect Answer ImageC.Neurofibromatosis type I
 Correct Answer ImageD.McCune-Albright syndrome
 Incorrect Answer ImageE.Tuberous sclerosis

A 19-month old boy is brought to the physician for a pre-operative evaluation for an inguinal
hernia repair. The child is developmentally meeting all his milestones. He is on the 50th percentile
for weight and 40th percentile for height. He has a flat nasal bridge with bilateral epicanthal folds
obscuring the medial canthus. Both of his eyes appear to deviate inward while gazing forward.
This inward deviation worsens on lateral gaze, both to the left and to the right. The corneal light
reflex, cover, and cover/uncover tests are normal. The remainder of the physical examination
shows no abnormalities except for an easily reducible left inguinal mass. Which of the following
is the next best step in management?
 Incorrect Answer ImageA.Eye patching
 Correct Answer ImageB.Reassure the parents that their child is normal
 Incorrect Answer ImageC.Refer the child to a geneticist
 Incorrect Answer ImageD.Refer the child to a neurologist
 Incorrect Answer ImageE.Refer the child to an ophthalmologist

A 15-year-old boy with tall stature is brought to the physician because of decreased exercise
tolerance over the past 2 months. He used to play two consecutive games of basketball with no
problem, but he now can play only one game. On physical examination, his weight is 52 kg (115
lb), which is at the 25th percentile for age and sex, and his height is 190 cm (6 ft 3 in), which is
greater than the 99th percentile for age and sex. His upper-to-lower segment ratio is less than 1.
His arm span is 191 cm. He has a pectus deformity, his joints are mildly hyperextensible, and he
has positive wrist and thumb signs. He is also scheduled for an ophthalmologic examination as
he is having visual problems. Over this patient’s lifetime, which of the following is the most
likely murmur to present on cardiac auscultation?
 Correct Answer ImageA.Diastolic high-pitched blowing murmur at the upper to mid-left
sternal border
 Incorrect Answer ImageB.Mid-diastolic rumbling murmur radiating to the apex, preceded
by an opening snap
 Incorrect Answer ImageC.Pansystolic, high-pitched, apical murmur radiating to the axilla
 Incorrect Answer ImageD.Systolic ejection murmur at the upper left sternal border
preceded by an ejection click
 Incorrect Answer ImageE.Systolic ejection murmur at the upper right sternal border
preceded by an ejection click
A 10-year-old girl is brought to the physician because of fever and chills for 5 days. She had
been complaining of a headache and feeling tired for the past 10 days. Her past medical history is
significant for frequent streptococcal throat infections over the past 2 years and for a new-onset
heart murmur that was detected at her last well-child visit 2 months ago. She is currently on no
medication other than acetaminophen. Otherwise, she has been wearing dental braces for the past
year and had a primary tooth extracted 2 weeks ago in an attempt to liberate space for permanent
teeth. On physical examination, the patient is diaphoretic and in moderate distress Her
temperature is 38ºC (101.2ºF), pulse is 110/min, respirations are 24/min, and blood pressure is
100/62 mm Hg. Cardiac auscultation confirms a grade 2/6 blowing, holosystolic murmur best
heard at the apex in the lateral decubitus position and that radiates to the left axilla. Which of the
following microorganisms is the most likely cause of this patient's current condition?
 Incorrect Answer ImageA.Candida species
 Incorrect Answer ImageB.Group B-hemolytic Streptococcus
 Incorrect Answer ImageC.Pseudomonas aeruginosa
 Incorrect Answer ImageD.Staphylococcus aureus 
 Incorrect Answer ImageE.Streptococcus pneumoniae
 Incorrect Answer ImageF.Streptococcus pyogenes
 Correct Answer ImageG.Streptococcus viridans

A 22-day-old newborn born at 29 weeks’ gestation has been on increasing nasogastric feedings,
which have been tolerated very well. The rest of his maintenance fluid has been provided by total
parenteral nutrition via a central venous catheter. He had been intubated during the first week of
life for surfactant therapy due to respiratory distress syndrome and successful treatment of a
patent ductus arteriosus with indomethacin. Today, his NICU nurse alerted the neonatologist that
the baby had bloody gastric residuals in his stomach prior to the next feeding and has abdominal
distension. His pulse is 180/min, respirations are 62/min, blood pressure is 48/30 mm Hg, and
oxygen saturation is 89% on room air. Examination shows coarse breath sounds bilaterally and a
distended abdomen with absent bowel sounds. Rectal examination shows dark stool that is heme
positive. Laboratory studies show:
CBC
16,600 WBC with 20% lymphocytes
60% PMNs
8% immature PMNs
10% monocytes
2% eosinophils
Platelets 105,000
Hgb 8.6 g/dL
Serum glucose 28 mg/dL
Electrolytes
Na+ 130 meq/L
K+ 5.5 meq/L
Cl -
93 meq/L
CO2 17 meq/L
Based on the most likely diagnosis with respect to the change in this baby’s condition, which of
the following would be an expected radiographic finding? 
 Incorrect Answer ImageA.Dilated rectosigmoid with line of demarcation with normal
caliber proximal bowel
 Incorrect Answer ImageB.Epigastric double-bubble sign with no distal bowel gas
 Incorrect Answer ImageC.Intraluminal ground glass appearance of small bowel in right
lower quadrant
 Correct Answer ImageD.Intramural intestinal gas in the distal ileum and proximal colon
 Incorrect Answer ImageE.Intraperitoneal calcifications

A 4-year-old boy is found unconscious in the kitchen by his mother. She came in from the living
room and found him barely arousable. On arrival in the emergency department, the boy
withdraws to painful stimuli but is obtunded. According to his mother, he has no significant past
medical history and takes no medications. The mother reports that when she kissed her son while
trying to arouse him, she tasted a sweet substance around the boy's lips. He is afebrile and blood
pressure is 83/52 mm Hg. He is obviously tachypneic, although he is saturating 99% on room air.
Head examination shows reactive pupils. Lungs are clear to auscultation. There are no abnormal
heart sounds and his abdomen is nontender. Peripheral examination shows no rashes or cyanosis.
Laboratory studies show:
Blood chemistry
White blood cells 4,500/mm3
Hematocrit 41%
Platelets 225,000/mm3
Sodium 137 mEq/L
Potassium 3.2 mEq/L
Chloride 105 mEq/L
Bicarbonate 13 mEq/L
Blood urea nitrogen 35 mg/dL
Creatinine 2.2 mg/dL
Glucose 89 mg/dL
Serum osmolality 340 mEq/L
Arterial pH 7.22
Urinalysis
10–15 WBC/HPF
15–20 RBC/HPF
Numerous calcium oxalate crystals
Which of the following is the most likely etiology of this patient's presentation? 
 Incorrect Answer ImageA.Cough syrup ingestion
 Incorrect Answer ImageB.Diabetic ketoacidosis
 Correct Answer ImageC.Ethylene glycol ingestion
 Incorrect Answer ImageD.Lactic acidosis
 Incorrect Answer ImageE.Methanol ingestion
 Incorrect Answer ImageF.Salicylate ingestion
 Incorrect Answer ImageG.Uremia

A worried 19-year-old single mother calls the physician because her 5-day-old son has
developed red eyes, tearing, and a yellow, sticky discharge that prevents him from opening his
eyes after sleeping until she cleans it off. The baby was born at term via vaginal delivery and his
Apgar scores were 9 and 9 at 1 and 5 minutes, respectively. The physical examination at the time
of birth was normal. The mother and son were discharged after 24 hours and everything was
going well until this happened. This is her first child and she does not know whether specific
treatment is required or the condition will resolve on its own. The physician advises the mother
to bring the newborn in and a couple of hours later they arrive at the office. On physical
examination, the neonate does not seem to be in distress. He has bilateral prominent tearing,
conjunctival injection, substantial lid edema, and a purulent discharge. A Gram stain of the
purulent material shows gram-negative diplococci. Which of the following is the most
appropriate treatment for this patient?
 Correct Answer ImageA.Systemic ceftriaxone
 Incorrect Answer ImageB.Systemic erythromycin
 Incorrect Answer ImageC.Topical erythromycin
 Incorrect Answer ImageD.Topical silver nitrate
 Incorrect Answer ImageE.Topical vidarabine

A 5-year-old boy is brought to the pediatrician's office 8 hours after he fell from his bicycle and
struck his head against the sidewalk. There were no witnesses to the incident. The patient denies
loss of consciousness. The child is otherwise healthy, up-to-date on his immunizations, and takes
no medications. On physical examination his vital signs are normal. There is a 5 × 4 cm abrasion
on the forehead. He is alert and oriented to date, place, and self. His motor and sensory
examinations are normal and reflexes are normal. Which of the following is the most appropriate
next step in management?
 Incorrect Answer ImageA.Admit overnight for observation
 Correct Answer ImageB.Instruct parents to observe neurological status for 24 hours
 Incorrect Answer ImageC.Obtain a head computerized tomography scan
 Incorrect Answer ImageD.Obtain a skull radiograph and discharge if normal
 Incorrect Answer ImageE.Obtain a skull radiograph, observe for 24 hours and discharge

A 14-year-old boy is brought to the physician by his parents because of short stature. He is
otherwise healthy and has had no major medical illnesses. He eats three meals a day consisting
of vegetables, protein, and healthy fats and exercises regularly. The developmental screening and
psychosocial assessments disclose no alarming findings. His father is 160 cm (5 ft 3 in) and his
mother is 147 cm (4 ft 10 in). No particular diseases or problems run in his family among first
and second-degree relatives. His height and weight have been consistently at the fifth percentile
since early childhood. Physical examination, including orthopedic and neurological, shows no
abnormalities and he is at sexual maturity rating (SMR) stage 3. Which of the following is the
most likely finding to be seen in this patient? 
 Correct Answer ImageA.Bone age equal to chronological age
 Incorrect Answer ImageB.Bone age greater than chronological age
 Incorrect Answer ImageC.Bone age less than chronological age
 Incorrect Answer ImageD.Decreased growth hormone (GH) level
 Incorrect Answer ImageE.Increased thyroid stimulating hormone (TSH)

A 7-year-old girl is brought to the office because of fever, headache, and malaise that has been
increasing over the previous 7 days. She had been away at her grandparents' house in the
countryside for vacation and had come down with a sore throat and fever about 4 weeks ago. Her
grandmother had given her some over-the-counter cold medication and kept her in bed for a few
days, but when she had not been getting better, they had brought her back to the city. Her mother
noted a faint red rash that has been waxing and waning on her chest, back, and shoulders. The
girl's past medical history is unremarkable and she has not been on any medications recently. On
physical examination, she is in moderate distress with a temperature of 39.4°C (102.9°F), pulse
of 120/min, and respirations of 30/min. She reports of chest discomfort and repeatedly coughs
during the examination. Deep breaths cause moderate chest pain, so her breathing is shallow. She
also has difficulty swallowing. The tonsils are edematous, erythematous, and covered with
purulent exudate. Her ankles are swollen, erythematous, and with limited range of motion. The
rash her mother described is present only on the right shoulder in the form of an annular
erythematous plaque, 5 cm in diameter. A rapid strep test done in the office is positive. Which of
the following complications is the most likely to develop in this patient? 
 Incorrect Answer ImageA.Aortic valve disease
 Incorrect Answer ImageB.Complications are unlikely to happen in this patient
 Correct Answer ImageC.Mitral valve disease
 Incorrect Answer ImageD.Pulmonary valve disease
 Incorrect Answer ImageE.Tricuspid valve disease

A 6-month-old boy is brought to the physician by his mother because of a 1-week history of
fussiness, decreased oral intake, and fevers up to 38.9°C (102.0°F). He has had red eyes, runny
nose, and a cough for the last 4 days. He has had no recent travel. His temperature is 38.3°C
(101.0°F). Examination shows a maculopapular, blanching rash on his face and neck. There is a
dusky sore with surrounding erythema on the buccal mucosa. Which of the following is the most
appropriate diagnostic study? 
 Incorrect Answer ImageA.Biopsy and histologic diagnosis of the rash
 Incorrect Answer ImageB.Culture from respiratory secretions
 Correct Answer ImageC.Disease-specific IgG and IgM blood titers
 Incorrect Answer ImageD.Disease-specific IgM blood titers
 Incorrect Answer ImageE.Posterior pharynx and tonsillar swab with rapid antigen
detection test

A 17-year-old girl is referred for evaluation because of persistent headaches for the past 3
months. While driving during the past 2 weeks she hit a lamp post on two separate occasions.
She started menstruating 2 years ago but now has amenorrhea. On physical examination, she has
fully developed breasts but minimal axillary and pubic hair. Neurological examination shows
bilateral visual field defects but is otherwise normal. A brain MRI and laboratory tests are
ordered. Which of the following is the most likely laboratory finding?
 Incorrect Answer ImageA.Elevated levels of androgens (testosterone and DHEA)
 Incorrect Answer ImageB.Elevated levels of estrogens
 Correct Answer ImageC.Elevated prolactin
 Incorrect Answer ImageD.Low estrogens
 Incorrect Answer ImageE.Low prolactin

A 10-year-old girl is brought to the physician by her parents. She was recently diagnosed with
generalized tonic-clonic epilepsy during a clinic visit that her grandmother took her to. The girl
is now accompanied by both of her parents who are concerned with the diagnosis and are seeking
advice regarding what they should do when the child has a seizure. Which of the following
suggestions is appropriate? 
 Incorrect Answer ImageA.Call an ambulance immediately as soon as seizure begins
 Incorrect Answer ImageB.Do not allow the child to return to her activities after recovery
 Incorrect Answer ImageC.Put something in the child's mouth at the onset of seizure
 Correct Answer ImageD.Try to place the child on her side during the seizure
 Incorrect Answer ImageE.Try to restrain the child during the seizure

A 9-year-old girl is brought to the physician by her father because of "an itchy rash" on the scalp
and neck for the past 10 days. The rash has not improved with frequent application of topical
steroid cream for a week. She has a history of eczema since she was an infant. She appears in no
distress but she scratches the back of her head frequently. Her temperature is 36.9°C (98.4°F).
Examination shows erythematous, scaly patches and mild lichenification on the posterior neck.
The suboccipital lymph nodes are enlarged and non-tender. There are multiple small, oval,
whitish flecks firmly attached on the hair shafts close to the scalp. Which of the following is the
most likely cause of this patient's skin lesions? 
 Incorrect Answer ImageA. Dermacentor andersoni 
 Correct Answer ImageB. Pediculus humanus capitis 
 Incorrect Answer ImageC. Pediculus humanus corporis 
 Incorrect Answer ImageD. Phthirus pubis 
 Incorrect Answer ImageE. Sarcoptes scabiei 

A 7-year-old girl is brought to the physician by her parents because of a 2-month history of
progressive skin infection that started on her face and is spreading to the rest of her body. The
skin lesions have not responded to antibiotic cream. Her medical history is unremarkable. She
takes no medications. She is up-to-date with her immunizations. Her temperature is 36.9°C
(98.4°F). Examination is shown. There are multiple lesions with yellow crusting on the trunk, in
the axilla, and on the extensor extremities. The lesions are minimally itchy. Which of the
following is the most likely diagnosis? 
 Incorrect Answer ImageA.Bullous pemphigoid
 Correct Answer ImageB.Impetigo
 Incorrect Answer ImageC.Poison ivy
 Incorrect Answer ImageD.Scabies
 Incorrect Answer ImageE.Tinea corporis
 Incorrect Answer ImageF.Toxic epidermal necrolysis

A 5-day-old boy who was born at home is being evaluated for bruising and gastrointestinal
bleeding. Laboratory studies show:
Elevated thromboplastin time and prothrombin time
Serum bilirubin 4.7 mg/dL
Alanine aminotransferase 20 mg/dL
Platelet count 330,000/mm3
Hemoglobin 9 g/dL
His mother has Factor V Leiden mutation. Which of the following is the most likely cause of the
boy's bleeding?
 Incorrect Answer ImageA.Factor VIII deficiency
 Incorrect Answer ImageB.Factor IX deficiency
 Incorrect Answer ImageC.Idiopathic thrombocytopenic purpura
 Incorrect Answer ImageD.Liver disease
 Correct Answer ImageE.Vitamin K deficiency

A 5-year-old boy suddenly begins coughing while eating peanuts at home. When he is brought to
the emergency department, the parents state that he has been choking and gagging for the past 15
minutes. He is awake and is able to give his name, but he continues to cough. On physical
examination, he is afebrile, respirations are 30/min, pulse is 100/min, and blood pressure is
100/68 mm Hg in his right arm while seated. He is pink and well-perfused and has no stridor,
wheezing, retractions, or accessory muscle use. Which of the following is the most appropriate
initial step in management? 
 Correct Answer ImageA.Allow the patient to clear the foreign object by spontaneous
coughing
 Incorrect Answer ImageB.Clear the oropharynx with multiple blind sweeps with a finger
 Incorrect Answer ImageC.Intubate the patient
 Incorrect Answer ImageD.Perform emergency tracheostomy and take the patient to
surgery
 Incorrect Answer ImageE.Position the patient and perform back blows
 Incorrect Answer ImageF.Stand behind the patient and perform abdominal thrusts

An 18-year-old Caucasian man comes to the physician for a follow-up visit. He’s a new patient
and came in 4 days ago with severe fatigue and dyspnea on exertion and was found to have a
hematocrit of 22%. At that time he was admitted to the local hospital for evaluation and given 2
units of packed red blood cells, increasing his hematocrit to 28%. His past medical history prior
to these events is remarkable for a blood disease and multiple hospitalizations in his hometown
of Naples, Italy. Physical examination shows conjunctival pallor and splenomegaly. He still
reports some mild shortness of breath, but nothing like the previous episode. Laboratory studies
at the time of admission showed:
Hemoglobin 7.5 g/dL
Hematocrit 23%
MCV 59 µm3
Reticulocytes 4.3%
Serum iron 160 µg/dL
TIBC 230 µg/dL (240-450 µg/dL)
Ferritin 80 ng/mL
Which of the following is the most likely cause of this patient's anemia? 
 Incorrect Answer ImageA.Alpha-thalassemia minor
 Incorrect Answer ImageB.Anemia of chronic disease
 Correct Answer ImageC.Beta-thalassemia
 Incorrect Answer ImageD.G6PD deficiency
 Incorrect Answer ImageE.Iron deficiency anemia

A 5-year-old boy is brought to the physician by his mother because he has refused to stand or
walk for 2 days. He walked without assistance at 14 months of age, he rode a tricycle at the age
of 3 years, and he recently started riding a bicycle with training wheels. He had an upper
respiratory infection which ended about one week ago and has otherwise been well. His
temperature is 37.1°C (98.8°F). Examination shows inability to bear weight. There is tenderness
to palpation over the left hip joint. The remainder of the examination shows no abnormalities.
Laboratory studies show: 
Hematocrit  43%
White blood 7,500/mm3 with 60% lymphocytes 
cells  28% neutrophils
10% monocytes
2% eosinophils
Platelets  429,000/mm3
ESR 12 mm/hr
Hip ultrasound Right hip normal; left hip normal other than a very
small effusion
He is prescribed ibuprofen and rest. The pain resolves in 3 days and there are no other problems.
Which of the following is the most likely diagnosis? 
 Incorrect Answer ImageA.Femoral neck fracture secondary to child abuse
 Incorrect Answer ImageB.Legg-Calvé-Perthes disease
 Incorrect Answer ImageC.Osteomyelitis of the femoral neck
 Correct Answer ImageD.Post-viral synovitis
 Incorrect Answer ImageE.Slipped capital-femoral epiphysis

A 3-year-old girl is brought to the physician by her parents because of newly developed skin
lesions that appeared on her neck and in the armpits over the last several weeks. She had just
started going to daycare when the teachers pointed out the "pimples." She is otherwise healthy
and has no medical problems except for mild childhood eczema that is well-controlled with daily
emollients and over-the-counter cortisone preparations. On physical examination, she is pleasant
and joyfully playing with a toy. She is well developed and well-nourished. On her lateral neck
and in both axillae, extending down the sides of her trunk, there are a total of approximately 20
pink, discreet, dome-shaped, pearly papules with dimpling on the top. The individual lesions
measure 2 to 5 mm. Some of them have an erythematous halo and are crusted. Which of the
following is the most likely diagnosis?
 Incorrect Answer ImageA.Atopic dermatitis
 Incorrect Answer ImageB.Erythema toxicum
 Incorrect Answer ImageC.Herpes simplex
 Correct Answer ImageD.Molluscum contagiosum
 Incorrect Answer ImageE.Varicella
 Incorrect Answer ImageF.Verruca plana
 Incorrect Answer ImageG.Verruca vulgaris

A previously healthy 4-year-old girl is brought to the physician by her parents because of a 2-
week history of non-productive cough, rhinorrhea, and generalized malaise. Her mother explains
that the child now experiences several severe minute-long coughing spells that are followed by
vomiting. The girl's mother admits to smoking in the house. The patient has no history of any
chronic medical conditions and has been taking an herbal remedy for the cough. She has received
a few immunizations, but her mother is unsure of which ones. Her temperature is 38.2ºC (100ºF),
pulse is 130/min, respirations are 32/min, and blood pressure 90/58 mm Hg. Physical
examination shows petechiae on her upper chest, neck, and face; mild conjunctival erythema, a
normal oropharynx without exudate or erythema, and no cervical lymphadenopathy. Equal
bronchial breath sounds are heard bilaterally upon auscultation of the chest and there is adequate
inspiratory and expiratory effort without subcostal or intercostal retractions. Which of the
following would most likely have prevented this condition?
 Incorrect Answer ImageA.Antiviral drugs
 Correct Answer ImageB.Immunization
 Incorrect Answer ImageC.Reduction of smoking in the home
 Incorrect Answer ImageD.Regular use of bronchodilators
 Incorrect Answer ImageE.Vitamin supplementation

A 3-year-old boy is brought to the emergency department by his parents because of a 2-day
history of low-grade fever and a rash that spread over most of his body. His medical history is
unremarkable, and he takes no medications. He has no allergies. His temperature is 38.2°C
(100.8°F). Examination shows a scarlatiniform rash over most of the trunk, arms, legs, and part
of the face, with thick crusting around the mouth. The conjunctivae are injected, and oral mucosa
is normal-appearing. There are radial fissures around the eyes, mouth, and nose. There is a
furuncle on the left buttock. The skin is tender to palpation, and tangential pressure on the skin
causes sloughing. Which of the following diseases most likely has a similar pathogenesis to this
patient's condition?
 Incorrect Answer ImageA.Acute generalized exanthematous pustulosis
 Correct Answer ImageB.Bullous impetigo
 Incorrect Answer ImageC.Epidermolysis bullosa acquisita
 Incorrect Answer ImageD.Stevens-Johnson syndrome
 Incorrect Answer ImageE.Toxic epidermal necrolysis
A 3-year-old boy is brought to the emergency department because of weakness and irritability
after spending the whole afternoon in the playground. Although his mother reports that he is
never as energetic as other kids his age, the patient is significantly weaker today, thus prompting
the emergency visit. His immunizations are up to date. Developmental milestones are globally
delayed. His height is less than the fifth percentile and weight is between fifth and tenth
percentiles for his age. His rectal temperature is 35.9ºC (96.6ºF), blood pressure is 86/44 mm Hg,
pulse is 108/min, and respirations are 24/min. The patient is lethargic with dry lips and mucosa
and decreased skin turgor. Tapping on the left cheek produces twitching on the left part of the
face. The remainder of the physical examination shows no abnormalities. Laboratory studies
show:
Sodium (Na+) 130 mEq/L
Potassium (K+) 3.2 mEq/L
Bicarbonate (HCO3-) 30 mEq/L
Calcium (Ca2+) 7.8 mg/dL
This condition is most similar to the effect of which of the following diuretics?
 Incorrect Answer ImageA.Amiloride
 Incorrect Answer ImageB.Eplerenone
 Correct Answer ImageC.Ethacrynic acid
 Incorrect Answer ImageD.Metolazone
 Incorrect Answer ImageE.Spironolactone
A 3-year-old boy is brought to the emergency department because of weakness and irritability
after spending the whole afternoon in the playground. Although his mother reports that he is
never as energetic as other kids his age, the patient is significantly weaker today, thus prompting
the emergency visit. His immunizations are up to date. Developmental milestones are globally
delayed. His height is less than the fifth percentile and weight is between fifth and tenth
percentiles for his age. His rectal temperature is 35.9ºC (96.6ºF), blood pressure is 86/44 mm Hg,
pulse is 108/min, and respirations are 24/min. The patient is lethargic with dry lips and mucosa
and decreased skin turgor. Tapping on the left cheek produces twitching on the left part of the
face. The remainder of the physical examination shows no abnormalities. Laboratory studies
show:
Sodium (Na+) 130 mEq/L
Potassium (K+) 3.2 mEq/L
Bicarbonate (HCO3 ) -
30 mEq/L
Calcium (Ca2+) 7.8 mg/dL
Which of the following is the most appropriate pharmacotherapy for this patient? 
 Incorrect Answer ImageA.Bumetanide and ibuprofen
 Incorrect Answer ImageB.Hydrochlorothiazide and calcium citrate
 Incorrect Answer ImageC.Magnesium oxide and chlorthalidone
 Incorrect Answer ImageD.Potassium chloride and indomethacin
 Correct Answer ImageE.Spironolactone and naproxen sodium

A 9-year-old boy is brought to the physician by his mother because of a significant decline in
functioning and increased social withdrawal. The mother reports that the boy always has had
difficulty going to school and is timid and anxious when speaking in front of others. She reports
that when the child was 7 years old he refused to go to school unless she came with him. The
boy’s problem with going to school worsened one year ago when he started at a new school after
the family relocated. After a month in the new school, the mother brought the child to a
pediatrician because of recurrent episodes of abdominal pain, nausea, and tiredness. These
episodes usually occurred on Sunday nights and caused repeated absences from school. All
physical examinations have been normal, but the child’s physical symptoms persist. Which of
the following is the most likely diagnosis?
 Incorrect Answer ImageA.Adjustment disorder
 Incorrect Answer ImageB.Conversion disorder
 Incorrect Answer ImageC.Factitious disorder
 Incorrect Answer ImageD.Illness anxiety disorder
 Correct Answer ImageE.Separation anxiety disorder

A 9-month-old girl with Down syndrome is brought to the physician for follow-up of a
respiratory tract infection. This is the third time in 3 months that she has been treated for
pneumonia. She has difficulty feeding and is not gaining weight. She frequently pauses during
eating and has to calm down to be able to continue. She also has difficulty breathing when she
cries. The parents have noticed that she started crawling over the past week but gets tired easily
and seems to have an aversion to activity of any sort. Her medical history is significant for
multiple respiratory tract infections, pneumonia, and poor growth. On physical examination, the
patient is below the fifth percentile for weight and height. Her lips are mildly cyanotic. She has a
hyperinflated thorax and a bulging precordium. Auscultation of the heart reveals that the second
heart sound is widely split with no respiratory variations. There is a systolic ejection murmur at
the upper left sternal border and a diastolic rumble at the lower left sternal border. Chest
radiography shows a grossly enlarged heart with a prominent pulmonary artery and increased
pulmonary vascular markings. Which of the following is the most likely diagnosis? 
 Incorrect Answer ImageA.Atrial septal defect
 Correct Answer ImageB.Endocardial cushion defect
 Incorrect Answer ImageC.Mitral regurgitation
 Incorrect Answer ImageD.Tetralogy of Fallot
 Incorrect Answer ImageE.Ventricular septal defect

An 11-year-old girl is brought to the physician by her parents who are concerned about her
increased urinary frequency, fatigue, and weight loss. The child is otherwise healthy and has not
had any diarrhea, nausea, or vomiting. She is in the 50th percentile for height and weight. She
has a 7-year-old brother who is healthy. Serum glucose level is 353 mg/dL, and urine dipstick is
positive for glucose and ketones. In addition to discussing the diagnosis and initial management,
the pediatrician discusses the possibility of other diseases associated with their daughter's
diagnosis. Which of the following statements can be made at this time? 
 Correct Answer ImageA.Genetic risk for certain other autoimmune diseases overlaps and
includes genes within the major histocompatibility complex (MHC) human leukocyte antigens
(HLA)
 Incorrect Answer ImageB.Initial testing should include anti-21-hydroxylase antibodies
 Incorrect Answer ImageC.Initial testing should include antiparietal and anti-intrinsic
factor (IF) antibodies.
 Incorrect Answer ImageD.The most commonly associated autoimmune disease is
juvenile idiopathic arthritis disease
 Incorrect Answer ImageE.The most commonly associated immunoendocrinopathy is
autoimmune polyendocrine syndrome type I

A 2-year-old girl is brought to her pediatrician because of excessive fatigue over the past week.
She had a mild upper respiratory illness, but recovered completely 3 weeks ago. Over the past 2
days the child has been pale and inactive. Her physical examination is unremarkable l except for
a pulse of 100/min and a grade II systolic ejection murmur over the left lower costal margin. This
murmur had not been previously appreciated. Laboratory studies show:
Hb 8.5 g/dL
MCV Normal
WBC 9,000 cells/mm3
Reticulocytes 0.5% of RBCs
HbF Normal for age
Adenosine deaminase level Normal
What is the next appropriate step in management for this patient? 
 Correct Answer ImageA.Follow Hb until resolution of the problem
 Incorrect Answer ImageB.Perform a bone marrow aspiration for diagnosis
 Incorrect Answer ImageC.Refer the child to a hematologist for workup of leukemia
 Incorrect Answer ImageD.Start the child on an oral corticosteroid
 Incorrect Answer ImageE.Transfuse with packed red blood cells
A 14-year-old boy is brought to the physician by his parents because of difficulty staying awake
in school. He states that he always feels tired and needs to take a nap every day before dinner.
His parents report that he is in bed by 9:00 p.m. on school nights, and gets up at 6:30 a.m. On
non-school nights he is in bed at 10:00 p.m. and awakens at 9:00 a.m. His weight is 77.1 kg (170
lb), and his height is 1.6 m (5 ft 3 in); BMI is 30.1 kg/m2. Physical examination shows tonsillar
enlargement and nasal intonation. Which of the following therapies will most likely result in
immediate symptomatic improvement in this patient? 
 Correct Answer ImageA.Adenotonsillectomy
 Incorrect Answer ImageB.Nasal continuous positive pressure during sleep
 Incorrect Answer ImageC.Nasal decongestants and topical steroids
 Incorrect Answer ImageD.Supplemental oxygen during sleep
 Incorrect Answer ImageE.Uvulopalatopharyngoplasty
 Incorrect Answer ImageF.Weight loss

A 7-year-old boy is brought to the physician because of red eyes, swollen lips, ulcers in the
mouth, fever, and progressive rash for 2 days. He was treated for Mycoplasma
pneumoniae pneumonitis with erythromycin 10 days ago. After receiving erythromycin, his fever
and cough resolved, but a day prior to the current symptoms his fever returned. His temperature
is 38.2°C (100.8°F) and pulse is 85/min. Examination shows erythematous, tender macules on
his face, upper chest, and arms. There are central vesicles and bullae with areas of skin on his
face and upper trunk that are denuded. There are also ulcers inside the mouth. Eye examination
shows bilateral conjunctivitis, and slit-lamp examination shows leukocytes between the cornea
and the lens. Which of the following is the most likely diagnosis?
 Incorrect Answer ImageA.Bullous pemphigoid
 Incorrect Answer ImageB.Epidermolysis bullosa
 Incorrect Answer ImageC.Erythema multiforme
 Incorrect Answer ImageD.Pemphigus vulgaris
 Correct Answer ImageE.Stevens-Johnson syndrome

A 6-month-old male is brought to the physician by his mother for a well-child visit. He was born
at term without any complications and has been well. He has no history of serious illness, takes
no medications, and is up to date on his immunizations. A bruit is heard on auscultation of the
right upper quadrant of the abdomen. An ultrasound of the liver shows a mass consistent with an
infantile hepatic hemangioma. Which of the following is most consistent with this finding?
 Incorrect Answer ImageA.It is associated with hypercoagulation
 Incorrect Answer ImageB.It is likely to continue growing throughout childhood and
adolescence
 Incorrect Answer ImageC.It is often associated with thrombocytosis
 Correct Answer ImageD.It may be associated with congestive heart failure
 Incorrect Answer ImageE.The first line of therapy for a symptomatic lesion is interferon

A 3-year-old boy is brought to the physician by his parents because of progressively worsening
weakness for the last 3 months. He has been having difficulty running, jumping, and walking
upstairs, and he now uses hand support to stand up from a seated position. His height is in the
60th percentile and his weight in the 70th percentile with a normal growth velocity. Physical
examination shows hip waddle and enlargement of the bilateral gastrocnemius muscles. He is
able to ambulate, but his muscle strength is diminished symmetrically. Which of the following is
the most likely mechanism of action of this disease? 
 Incorrect Answer ImageA.Inflammatory myopathy
 Incorrect Answer ImageB.Muscle phosphorylase deficiency
 Incorrect Answer ImageC.Mutations of genes coding for proteins of the nuclear envelope
 Correct Answer ImageD.Mutation of the gene coding for a plasma membrane protein of
muscle fibers
 Incorrect Answer ImageE.Thyroid hormone deficiency

A 6-week-old female is brought to the physician for a routine newborn examination. She was
born at 40 weeks gestation via uncomplicated vaginal delivery after an external cephalic version
successfully corrected her breech presentation. Apgars were 9 and 9 at 1 and 5 minutes,
respectively. The infant has had no issues since birth and has been feeding and growing
appropriately. The family asks the physician to check her hips because her older sibling has
significant disability from developmental dysplasia of the hip that was not properly diagnosed.
On physical examination, the physician cannot completely abduct the infant's thighs when her
hips and knees are flexed, but a click or a snap cannot be elicited when the physician tries to
manually dislocate or reduce the femoral heads. The infant's legs are of the same length, and
gluteal folds are symmetric. Which of the following is the most appropriate course of action? 
 Correct Answer ImageA.Do ultrasonography of her hips
 Incorrect Answer ImageB.Obtain x-ray films of her hips
 Incorrect Answer ImageC.Reassure the parents
 Incorrect Answer ImageD.Recheck the infant at 6 months of age
 Incorrect Answer ImageE.Secure the infant in a Pavlik harness for 4 weeks

A 4-year-old boy is being evaluated for an injury to the forearm. His mother states she did not
see how it happened, nor did she hear any noise from a fall. He has had bone fractures on two
other occasions since the age of 2. Physical examination reveals a swollen left forearm. Mild
scoliosis, bowed femur, and multiple bruises are also noted. No swelling or tenderness in the
other joints is found. When asked about the bruises, the mother remarks, "They don't bother him
because they always appear here and there with minor pressure." The boy's height is below the
5thpercentile. HEENT examination is unremarkable, except for bilateral hearing aids. The
remainder of the exam is normal. X-ray reveals fracture of the left radius with no bony
abnormalities. Which of the following is the most likely diagnosis? 
 Incorrect Answer ImageA.Achondroplasia
 Incorrect Answer ImageB.Battered child syndrome
 Incorrect Answer ImageC.Constitutional delay of growth
 Incorrect Answer ImageD.Familial short stature
 Correct Answer ImageE.Osteogenesis imperfecta
 Incorrect Answer ImageF.Rickets
 Incorrect Answer ImageG.Scurvy

A 14-year-old boy is brought to the emergency department after being hit by an automobile
while he was walking across the street. On arrival, he is conscious and complains of shortness of
breath and chest pain. Physical examination shows an ecchymotic area over his right chest, and
palpation suggests that he has multiple broken ribs on that side. Within a matter of minutes, he
becomes hypotensive, and large distended veins are noted on his neck and forehead, concomitant
with increasing respiratory distress. He has an asymmetric chest rise. Breath sounds are absent
on the right side and decreased on the left. His trachea is deviated to the left. Which of the
following is the most appropriate initial step in the management of this patient?
 Incorrect Answer ImageA.Arterial blood gas (ABG)
 Incorrect Answer ImageB.Intravenous rapid bolus of normal saline
 Correct Answer ImageC.Needle decompression in the right second intercostal space

 Incorrect Answer ImageD.Pericardiocentesis


 Incorrect Answer ImageE.Plain radiography of the chest

A 12-year-old boy is brought to the office by his mother for evaluation of microscopic
hematuria. The boy had a routine physical examination done in his school that showed 10–12
RBCs/hpf. He has been healthy all through his childhood. In the last few months, however, his
mother reports that he does not listen to her and does not pay attention to what she says. His
teachers have similar complaints. The child often watches TV with the volume high. Family
history is positive for the boy's uncle who had renal failure and has been on dialysis for the last 2
years. He is also deaf. His blood pressure is 120/70 mm Hg, pulse is 100/min, and respirations
are 12/min. He is afebrile, and the rest of his examination is unremarkable. Laboratory studies
show:
Sodium 138 mEq/L
Potassium 4.2 mEq/L
Chloride 101 mEq/L
Bicarbonate 26 mEq/L
BUN 18 mg/dL
Creatinine 0.6 mg/dL
Glucose 84 mEq/L
Urinalysis shows: 
Specific gravity 1.010
pH 6.0
Blood 1+
Protein 1+
Ketones Negative
Glucose Negative
RBCs 10–12/hpf
WBCs 0/hpf
Epithelial cells Negative
Casts Negative
Crystals Negative
CT of the abdomen is normal. His mother is told that he is in need of further evaluation. Which
of the following findings would be pathognomonic for this disease? 
 Correct Answer ImageA.Anterior lenticonus
 Incorrect Answer ImageB.Corneal erosions
 Incorrect Answer ImageC.Glomerular basement membrane thickening
 Incorrect Answer ImageD.Macular flecks
 Incorrect Answer ImageE.Nephrotic syndrome
A 14-year-old boy is brought to the physician because of decreased exercise tolerance. He is up
to date on all of his childhood immunizations and has been generally healthy until now. His
blood pressure is 114/74 mm Hg in the right arm and 120/80 mm Hg in the right leg. He is noted
to have a grade 3/6 systolic ejection murmur best heard at the left mid sternal border and a grade
2/6 mid-diastolic murmur at the lower left sternal border. The first heart sound is normal. The
second heart sound is widely split and fixed. A right ventricular impulse is palpated. Pulses are
equally palpable in all four extremities. On a chest x-ray, the pulmonary artery segment is
enlarged, and pulmonary vascular markings are increased. An electrocardiogram shows right
axis deviation. Which of the following is the most likely diagnosis?
 Incorrect Answer ImageA.Aortic stenosis
 Correct Answer ImageB.Atrial septal defect
 Incorrect Answer ImageC.Coarctation of the aorta
 Incorrect Answer ImageD.Patent ductus arteriosus
 Incorrect Answer ImageE.Ventricular septal defect

A 5-year-old boy is brought to the physician during springtime because of a malodorous


discharge coming from his right nostril. The patient has had no prior history of nasal allergies,
adenotonsillitis, or upper airway obstructive symptoms. His parents report that the child
developed frequent sneezing and mucus discharge mixed with blood on occasions for the last 2
weeks. They thought it was a common cold. He has had no fevers. Physical examination shows
decreased air entry through the right nostril with associated unilateral purulent discharge. Which
of the following is the most likely diagnosis?
 Incorrect Answer ImageA.Allergic rhinitis
 Incorrect Answer ImageB.Juvenile angiofibroma
 Correct Answer ImageC.Nasal foreign body
 Incorrect Answer ImageD.Nasal polyp
 Incorrect Answer ImageE.Vasomotor rhinitis

An infant born at term has ambiguous genitalia on initial inspection at the time of birth. There
are prominent labioscrotal folds without palpable gonads bilaterally as well as non-distinct
phallic enlargement. A pelvic ultrasound is performed and shows a normal uterus and ovaries, as
well as normal kidneys and bladder. Chromosomal studies show a 46,XX karyotype. Parents
declined further medical investigations including the newborn screen and were discharged home.
At 3 weeks of age, the infant presents to the emergency department with hypovolemic shock.
Routine electrolyte studies show hyponatremia and hyperkalemia, and an intravenous bolus of
isotonic sodium chloride solution is given emergently. Measuring which of the following is most
likely to confirm the diagnosis?
 Incorrect Answer ImageA.5-Alpha reductase level
 Correct Answer ImageB.17-Hydroxyprogesterone level
 Incorrect Answer ImageC.CT scan of the abdomen
 Incorrect Answer ImageD.Maternal androgen level
 Incorrect Answer ImageE.Serum estrogen level
 Incorrect Answer ImageF.Serum LH levels
 Incorrect Answer ImageG.Testosterone: DHT ratio
A 9-year-old girl is brought to the clinic because of abdominal pain and a rash. Four days ago her
mother noticed a red rash on the top of the girl's feet, which has now spread upward to the entire
lower extremities. The rash was initially small red dots but has now become patchy, with larger
lesions that are more purple-appearing. The child subsequently developed periumbilical,
cramping abdominal pain. Review of systems is positive only for a respiratory viral illness 3
weeks ago. Her temperature is 37.7ºC (99.8ºF), pulse is 96/min, and respirations are 18/min.
Examination of her face shows pallor. Abdominal examination is remarkable for tenderness to
palpation near the umbilicus, but the abdomen is otherwise soft, with no rebound or guarding,
and no organomegaly. Raised, palpable purpuric lesions are present on the thighs and down to
the feet. The ankles are tender and edematous bilaterally. Which of the following is this patient's
laboratory studies most likely to show?
 Incorrect Answer ImageA.Fecal leukocytosis
 Correct Answer ImageB.Red blood cells in the urine
 Incorrect Answer ImageC.Thrombocytopenia
 Incorrect Answer ImageD.White blood cell casts in the urine
 Incorrect Answer ImageE.Yeast in the urine

A 14-year-old girl is brought to the physician by her mother because of concerns about a possible
eating disorder. Over the past 2 months, the patient has had a 6.8 kg (15-lb.) weight loss. She
occasionally eats two dinners, but her parents do not know if she purges. The girl denies any
problems with her self-image and wishes she could keep her weight up, as she feels her weight
loss is limiting her ability as an athlete. She has had increased weakness and fatigue over the past
month and increased frequency of urination without dysuria, even waking at night to void. She
has had no major medical illnesses and takes no medications. Examination shows a thin but well-
appearing teenager. Which of the following is most likely to confirm the patient's condition?
 Correct Answer ImageA.Antiglutamic acid decarboxylase antibodies
 Incorrect Answer ImageB.Elevated fasting cholesterol and triglyceride levels
 Incorrect Answer ImageC.Parotid gland hypertrophy and loss of tooth enamel
 Incorrect Answer ImageD.Retinal vessel proliferation
 Incorrect Answer ImageE.Significant proteinuria on a urine dipstick

A 3-year-old boy has been followed by his pediatrician since birth. During the first few months
of life, the child had poor head control when pulled to sit. The boy's early developmental
milestones occurred at the appropriate ages, except he first walked without help at 16 months of
age. For the past year, he has been showing some hip girdle weakness and falls frequently. The
mother reports that over the past few months when the child has been playing on the floor, he has
had trouble getting up. To confirm her suspicions, the pediatrician requests a serum creatine
kinase level, which is reported to be 30,000 IU/L (normal <160 IU/L). Which of the following
findings is the most appropriate next step to establish the diagnosis?
 Incorrect Answer ImageA.Electromyography showing myopathic features
 Incorrect Answer ImageB.Increase in other serum lysosomal enzymes
 Incorrect Answer ImageC.Muscle biopsy showing myopathic changes
 Incorrect Answer ImageD.Nerve conduction velocity showing significant slowing
 Correct Answer ImageE.Polymerase chain reaction of serum sample showing a specific
genetic defect
An 8-year-old boy is brought to the pediatrician by his mother because she is concerned he is not
sufficiently physically active. He does not enjoy sports and is minimally active in physical
education class at school. When the boy is questioned privately, he admits that he is not active
because the other children make fun of how he runs. On physical examination, he is slim and
noted to have bulkier gastrocnemius and soleus muscles on the left lower extremity and slightly
limited abduction at the right hip, though he denies pain on passive flexion. Evaluation of his gait
shows a persistently externally rotated femur that appears mildly painful. An x-ray appointment
is arranged. Which of the following is the most likely diagnosis? 
 Incorrect Answer ImageA.Femoral osteosarcoma
 Correct Answer ImageB.Legg-Calvé-Perthes disease
 Incorrect Answer ImageC.Osteoarthritis
 Incorrect Answer ImageD.Polymyositis
 Incorrect Answer ImageE.Slipped capital femoral epiphysis

A 9-month-old infant is brought to the physician by her parents because of failure to gain weight.
Her length and weight are both below the 5th percentile for her age. The patient's chart indicates
that at the age of 6 months, her length and weight were at the 50th percentile. Her mother reports
that she returned to work when the infant was 6 months old, and the grandmother has assumed
the care of the infant. The infant consumes 6- to 8-oz of iron-fortified, cow's milk-based formula
four times per day and she sleeps from 9:00 PM to 6:00 AM every night. She has 4 to 6 wet
diapers per day and stool is described as normal 1 to 3 times per day. There are no other
problems noted, and her vital signs and physical examination are normal. Which of the following
is the best initial step in the management of this infant? 
 Correct Answer ImageA.Ask how the formula is mixed
 Incorrect Answer ImageB.Obtain a detailed family history for lactose intolerance
 Incorrect Answer ImageC.Obtain a stool specimen
 Incorrect Answer ImageD.Obtain a sweat chloride test
 Incorrect Answer ImageE.Obtain a urinalysis

A 5-month-old boy has a urine output of less than 0.1 mL/kg/h shortly after undergoing major
surgery. On examination, he has generalized edema. His pulse is 180/min, respirations are
20/min, and blood pressure is 70/40 mm Hg. His blood urea nitrogen is 38 mg/dL and serum
creatinine is 1.4 mg/dL. Initial urinalysis shows a specific gravity of 1.008 and 2+ protein.
Microscopic examination of the urine sample shows 1 WBC per high-power field (HPF), 18
RBCs per HPF, and 5 granular casts per HPF. His fractional excretion of sodium is 3.2%. Which
of the following is the most appropriate next step in diagnosis?
 Incorrect Answer ImageA.CT of the abdomen and pelvis with contrast
 Incorrect Answer ImageB.Cystourethrography
 Incorrect Answer ImageC.Intravenous pyelography
 Incorrect Answer ImageD.Renal biopsy
 Correct Answer ImageE.Renal ultrasonography

A newborn girl born at 37 weeks' gestation is admitted to the neonatal intensive care unit because
of abdominal distention with no apparent external anal opening. Further evaluation shows a
significant distance between a blind rectal pouch and where the anus would be externally. There
is no perineal or vaginal fistula present and urinalysis is negative for meconium. Prior to
performing a diverting colostomy surgery, the infant must be evaluated for the presence of
additional malformations. Which of the following is most likely to be present?
 Incorrect Answer ImageA.Malrotation of the intestines
 Correct Answer ImageB.Renal agenesis with a presacral lipoma
 Incorrect Answer ImageC.Supranumerary toes and fingers
 Incorrect Answer ImageD.Tethered cord
 Incorrect Answer ImageE.Tetralogy of Fallot

A 9-year-old girl with a history of atopic dermatitis is brought to the physician because of
worsening pruritus and a rash for 1 week. Current medications include topical corticosteroids and
bland emollients. Her father has herpes labialis. Her temperature is 38.2°C (100.8°F). Physical
examination shows eczematous patches on her cheeks, lateral neck, trunk, and antecubital fossae.
There are erythematous superimposed vesicles and small, "punched-out" ulcerations. The
vesicles have spread beyond the edges of the eczematous patches onto uninvolved skin. Which
of the following is the most likely diagnosis? 
 Incorrect Answer ImageA.Dermatitis herpetiformis
 Incorrect Answer ImageB.Erythema multiforme
 Incorrect Answer ImageC.Herpes gestationis
 Correct Answer ImageD.Kaposi varicelliform eruption
 Incorrect Answer ImageE.Varicella-zoster reactivation

A female infant is born at 39 weeks’ gestation to a healthy 32-year-old gravida 1, para 0, woman
via uncomplicated vaginal delivery. The newborn is taken to the neonatal intensive care unit
because of respiratory distress. A review of the mother's medical records shows that she did not
receive routine prenatal care. The newborn’s respirations are 65/min and oxygen saturation
shows 70% on room air. She appears lethargic and cyanotic. Physical examination shows
grunting with nasal flaring. Lung auscultation shows decreased air entry over the left lung field.
An arterial blood gas shows PaO2 of 55 mmHg. An x-ray of the chest is shown. Which of the
following is the most appropriate next step in management?
 Incorrect Answer ImageA.Bag-and mask-ventilation
 Incorrect Answer ImageB.Chest tube placement immediately
 Incorrect Answer ImageC.Extracorporeal membrane oxygenation
 Incorrect Answer ImageD.High-pressure ventilation
 Correct Answer ImageE.Nasogastric suction
 Incorrect Answer ImageF.Surfactant
 Incorrect Answer ImageG.Surgery immediately

A 10-year-old boy is brought to the emergency department by his mother because of bleeding
from his nose that started 1 day ago. She put some ice packs on his nose, but the bleeding did not
stop. The mother states that a few weeks ago, her son had a viral upper respiratory tract infection
and has just recovered from it. She reports that he was afforded some relief from acetaminophen
administration during his viral illness. He is currently not on any medications. Her son has
otherwise been healthy with no bleeding problems, nor is there any family history of bleeding
disorders. His vital signs are within normal limits. Examination of the nasal mucosa reveals mild
bleeding but no polyps or masses. Petechiae are present on both lower extremities. There is no
adenopathy or hepatosplenomegaly. Laboratory studies show:
Hgb  13.2 g/dL
WBC  6,000/mm3 with a normal differential 
Platelets  8,000/mm3
Peripheral smear shows low platelet count and some large platelets. No schistocytes are seen.
Which of the following is the most likely diagnosis? 
 Incorrect Answer ImageA.Acetaminophen-induced thrombocytopenia
 Incorrect Answer ImageB.Chronic liver disease
 Incorrect Answer ImageC.Disseminated intravascular coagulation
 Incorrect Answer ImageD.Hemolytic-uremic syndrome
 Correct Answer ImageE.Idiopathic thrombocytopenic purpura
 Incorrect Answer ImageF.Thrombotic thrombocytopenic purpura

A 17-year-old girl is brought to the emergency department because of a 2-day history of a


painful and swollen right ankle. She also has had a fever for 2 days, with temperatures up to
38.9ºC (102ºF). Her parents decided to bring her to the hospital tonight because she has been
unable to sleep due to the pain. Her temperature is 38.8ºC (101.8ºF). On physical examination,
her right ankle is markedly swollen, brightly erythematous and very painful to touch. She is
unwilling to move the ankle or put weight on it. Aspirated fluid under ultrasound guidance from
the joint shows a white blood count of 55,000/ mm3 with increased protein and decreased
glucose. Which of the following would be diagnostic of this condition?
 Incorrect Answer ImageA.Aspirated joint fluid showing deposits of calcium
pyrophosphate crystals
 Incorrect Answer ImageB.Aspirated joint fluid showing deposits of monosodium urate
crystals
 Correct Answer ImageC.Aspirated joint fluid showing gram-positive cocci in clusters
 Incorrect Answer ImageD.Blood for rheumatoid factor (RF) and anti-cyclic citrullinated
peptide (anti-CCP) positive
 Incorrect Answer ImageE.Blood for Western immunoblotting showing 5 of 10 IgG bands
and 2 of 3 IgG bands positive for B. burgdorferi

A young mother brings her 18-month-old son to the physician for a well-child visit. The mother
recalls that a child in his daycare program had to be sent home 9 days ago because of "pink eye,"
but none of the other children seem to have been affected. Physical examination shows stable
vital signs. He has appropriate growth curves for both height and weight. The baby has a white
reflex in the left eye with no evidence of discharge. Which of the following is the most
appropriate course of action?
 Incorrect Answer ImageA.Provide antibacterial eye drops and follow up in 2 weeks
 Incorrect Answer ImageB.Provide genetic counseling
 Incorrect Answer ImageC.Routine followup
 Correct Answer ImageD.Seek an emergency ophthalmologic consultation
 Incorrect Answer ImageE.Seek a surgery consultation for the next available appointment

A male infant is brought the pediatrician for a 2-month well-child examination. The baby has
been breastfeeding and has not had any problems. At this visit, however, the mother reports that
he has not been feeding as well as previously. She states that she is going back to work and
would like to switch to formula feeding. The pediatrician tells her that this is a reasonable plan.
She brings the infant back in 3 weeks because he is still not feeding well despite the change to
the formula feeding. At this visit, the pediatrician finds generalized hypotonia, absent deep
tendon reflexes, and fasciculations of the tongue. Which of the following findings is
characteristic of this disorder?
 Incorrect Answer ImageA.Abnormal motor nerve conduction
 Incorrect Answer ImageB.Absence of deletions or mutations in the SMN1 gene
 Incorrect Answer ImageC.A large increase in serum creatine kinase level
 Incorrect Answer ImageD.Decremental response to repetitive stimulation on
electromyography
 Correct Answer ImageE.Muscle biopsy showing a pattern of perineural denervation

A 14-year-old boy is referred to a pediatric nephrologist for evaluation and management of renal
stones. He has had kidney stones since early childhood and has had several emergency
department visits over the years for flank pain and hematuria. He has also passed stones on
several occasions. Surgical removal of the stones was required for two different episodes. The
stones were analyzed and a diagnosis was made. He has been unable to comply with prescribed
treatment. He denies any current fever, chills, or weight loss. His chart shows no other
significant medical condition, except for the usual childhood illnesses. He had an appendectomy
when he was 10-years-old. He does not take any medications except for ibuprofen during
episodes of flank pain. He denies use of any herbal medications and eats a regular diet. There is
no other family history of kidney stones. Physical examination is unremarkable. Urinalysis is
performed, which shows the presence of hexagonal crystals. A plain anterior and lateral
radiograph of the abdomen shows bilateral renal stones that are only slightly radiopaque. What
would be the most appropriate test to establish the etiology of the patient's clinical presentation? 
 Incorrect Answer ImageA.24-hour urine for creatinine

 Correct Answer ImageB.24-hour urine for cystine

 Incorrect Answer ImageC.24-hour urine for uric acid

 Incorrect Answer ImageD.Computerized tomographic (CT) scan of the abdomen

 Incorrect Answer ImageE.Intact parathyroid hormone (PTH) level

 Incorrect Answer ImageF.Serum uric acid

A 16-year-old boy comes to your office as a new patient. He just moved in with his grandparents
and, prior to this, had minimal medical care. In your office, he has a recorded blood pressure in
the right upper extremity of 180/110 mm Hg. The boy has not had many medical problems in the
past and takes no medication. He is not engaged in any strenuous sports activity because all of
his free time is devoted to academic endeavors. On physical examination, the patient's upper
body is well developed but his legs and his arms are disproportionately thin. His body
temperature, respiratory rate, and pulse are within normal limits. Femoral pulse is noted to be
delayed. On auscultation, a short systolic murmur is appreciated in the upper mid-back. Which of
the following is most likely to be present on a chest radiograph?
 Incorrect Answer ImageA.Boot-shaped heart
 Incorrect Answer ImageB.Enlarged right atrium and ventricle
 Incorrect Answer ImageC.Prominent pulmonary artery
 Correct Answer ImageD.Erosion of posterior ribs on chest x-ray
 Incorrect Answer ImageE.The chest radiograph will most likely be normal

An infant is born via spontaneous vaginal delivery at 35 weeks’ gestation and becomes irritable,
tremulous, and inconsolable at 24 hours after birth. The infant’s mother did not have routine
prenatal care and has a history of polysubstance use. The infant weighs 1,900 grams (4 lb 3 oz),
length is 38 cm (15 in), and head circumference is 30 cm. Her cry is high-pitched, and there are
periods of apnea, myoclonic jerks, mottling of the skin, increased wakefulness, and fist-sucking.
Physical examination shows no dysmorphic facial features. Temperature is 38ºC (100.4ºF),
respirations are 60/min, and pulse is 160/min. Over the next 12 hours, the infant develops
diarrhea. Which of the following substances was the newborn most likely exposed to in utero?
 Incorrect Answer ImageA.Alcohol
 Incorrect Answer ImageB.Barbiturates
 Incorrect Answer ImageC.Cocaine
 Incorrect Answer ImageD.Marijuana
 Correct Answer ImageE.Opioids

A 4-day-old newborn born at 39 weeks' gestation to a G1P1 mother is brought to the physician
because of yellow discoloration of the skin. The newborn's mother reports that she feels that the
infant has been breastfeeding well. He has had three wet diapers per day and has had one stool
since discharge. Weight measurement shows that he has lost 14% of his birth weight.
Hemoglobin is 16 g/dL, serum total bilirubin is 15 mg/dL, and direct bilirubin is 1.1 mg/dL. A
review of the medical records shows the mother's blood type is A+ and the infant is O+. Which
of the following is the most likely diagnosis?
 Incorrect Answer ImageA.Breast milk jaundice
 Incorrect Answer ImageB.Crigler-Najjar syndrome
 Correct Answer ImageC.Inadequate caloric intake
 Incorrect Answer ImageD.Isoimmunization
 Incorrect Answer ImageE.Polycythemia

A newborn is found to have a translucent cystic structure covered by a thin membrane that arises
posteriorly from the back at the level of the lower lumbosacral vertebrae. Through the covering,
neural tissue is visualized. Examination of the lower extremities shows flaccid paralysis, absence
of deep tendon reflexes, and no response to pinprick. The baby also has bilateral clubfeet. The
baby is taken to the neonatal intensive care unit and the lesion is covered with sterile saline-
soaked gauze. An MRI of the brain would most likely show which of the following? 
 Incorrect Answer ImageA.Arachnoid cyst in the posterior fossa
 Incorrect Answer ImageB.Cerebellar vermian aplasia
 Incorrect Answer ImageC.Subependymal nodules
 Incorrect Answer ImageD.Superior cerebellar peduncle elongation
 Correct Answer ImageE.Tonsillar herniation

A 5-year-old boy is brought to the physician by his parents who are concerned about his poor
performance in school, and increasing irritability and recent severe headaches. He has also been
constipated, has had little appetite, and has been complaining of crampy abdominal pain. He has
otherwise been healthy, lives in an old house, and has a history of allergic rhinitis. While
examining the patient, he vomits twice and is lethargic. His temperature is 37.7ºC (99.8ºF), blood
pressure is 100/60 mm Hg, pulse is 120/min, and respirations are 18/min. Conjunctiva are pale
and there is papilledema on fundoscopic examination. Cardiac examination shows a 2/6 systolic
ejection murmur and normal rate and rhythm. His lungs are clear to auscultation. His abdomen is
diffusely tender on deep palpation.
Laboratory studies show:
Hgb  11 g/dL
Hct  38%
MCV  75 fL
MCH  24 pg/cell
Peripheral smear  basophilic stippling on RBC
Reticulocyte count  2%
Lead levels  74 µg/dL
Which of the following is the next best step in the management of this patient? 
 Incorrect Answer ImageA.Admit to the hospital and administer activated charcoal
 Incorrect Answer ImageB.Admit to the hospital and administer deferoxamine
 Correct Answer ImageC.Admit to the hospital and administer parenteral EDTA +
dimercaprol
 Incorrect Answer ImageD.Admit to the hospital and administer EDTA
 Incorrect Answer ImageE.As an outpatient, administer oral succimer

A 10-year-old boy is brought to the physician because of severe pain in the great toe that started
last night. He has a history of global developmental delay and intellectual disability since
infancy, multiple episodes of nephrolithiasis, and progressive renal failure for which he has been
on dialysis for the past 4 years. Since he was 1-year-old, he has had choreoathetoid, dystonic
movements. His speech is unintelligible. Physical examination of the great toe shows a grossly
inflamed, red, hot, tender metatarsophalangeal joint. Bite marks on the fingers and forearms are
also noted. Neurologic examination shows spasticity in all four limbs. Which of the following
would most likely confirm the diagnosis in this patient? 
 Incorrect Answer ImageA.24-hour urinary uric acid excretion
 Incorrect Answer ImageB.Blood lead level
 Incorrect Answer ImageC.Blood uric acid level
 Correct Answer ImageD.Measurement of HPRT enzyme activity in blood
 Incorrect Answer ImageE.Skeletal survey for suspected child abuse
 Incorrect Answer ImageF.Urine for mucopolysaccharides

A couple wants to become pregnant with their second child. Their first child, a boy, was born at
term after an uncomplicated pregnancy and delivery, but was diagnosed with 21-hydroxylase
deficiency at 2 weeks of life after experiencing a salt-wasting crisis. After the birth of their first
child, the parents received genetic counseling and are now being seen by a perinatologist. They
understand that there is a 1 in 4 chance of their second child having 21-hydroxylase deficiency,
but they really want another child. Which of the following is the most appropriate step in
management after conception has been confirmed?
 Incorrect Answer ImageA.Examine the baby after birth for ambiguous genitalia and, if
present, admit to the neonatal intensive care unit for further evaluation
 Incorrect Answer ImageB.Follow the pregnancy with high-resolution ultrasonography for
any evidence of abnormal genital development and then discuss options with the parents
 Incorrect Answer ImageC.Perform an amniocentesis at 16 to 18 weeks of pregnancy to
test for the disorder and if present, institute antenatal therapy
 Incorrect Answer ImageD.Perform chorionic villus sampling between 10 and 12 weeks to
test for the disorder, and then institute steroid treatment on the mother
 Correct Answer ImageE.Start the administration of dexamethasone to the mother no later
than the 6th week of pregnancy, and then perform chorionic villus sampling between 10 and 12
weeks

A 16-year-old boy is admitted to pediatric intensive care after sustaining head trauma in a motor
vehicle accident. He is on mechanical ventilation and is heavily sedated and paralyzed. He is
currently receiving maintenance intravenous fluids through a centrally placed catheter. He also
has an arterial catheter in his right radial artery. His serum sodium has been decreasing over the
past 2 days and is now 121 mEq/L. The physician determines that his hyponatremia is due to the
syndrome of inappropriate antidiuretic hormone secretion (SIADH). Which of the following is
most consistent with this diagnosis?
 Correct Answer ImageA.High intravascular volume and high urine sodium
 Incorrect Answer ImageB.Low intravascular volume and low urine sodium
 Incorrect Answer ImageC.Low intravascular volume and very high urine sodium
 Incorrect Answer ImageD.Normal intravascular volume and high urine sodium
 Incorrect Answer ImageE.Normal intravascular volume and normal urine sodium

A 4-week-old boy is brought to the emergency department by his parents because of vomiting
for 1 week. The parents state that the infant vomits after each feeding but he is always hungry.
The emesis is consistent with feedings and nonbilious in color. He is afebrile. Physical
examination shows a sunken fontanelle and poor capillary refill. Heart and lung examinations
show no abnormalities. Abdominal examination shows visible gastric peristaltic waves and a
firm, mobile 2-cm mass above and to the right of the umbilicus in the mid-epigastrium beneath
the liver edge. The stool is guaiac-negative. Laboratory studies are consistent with the typical
findings of this condition. A nasogastric tube and intravenous line are placed. Which of the
following is the most appropriate next step in management?
 Incorrect Answer ImageA.Correct hyperchloremic, hypokalemic metabolic acidosis
 Correct Answer ImageB.Correct hypochloremic, hypokalemic metabolic alkalosis
 Incorrect Answer ImageC.Order an abdominal ultrasound
 Incorrect Answer ImageD.Perform pyloromyotomy
 Incorrect Answer ImageE.Schedule endoscopy

A 12-year-old girl comes to the adolescent health clinic because of vaginal discharge. The
discharge started approximately 2 months ago. It is thin, whitish in color, and there is no odor.
The patient denies itching, burning, or pain. She started breast development at age 9 and her
pubertal development has proceeded normally to this point. She has not had her first menses and
is not sexually active. She has no medical problems. Physical examination is normal for her age.
Microscopic examination of the discharge shows no evidence of granular-appearing vaginal
epithelial cells or motile organisms. Which of the following is the most likely diagnosis? 
 Incorrect Answer ImageA.Bacterial vaginosis
 Incorrect Answer ImageB.Candida vulvovaginitis
 Correct Answer ImageC.Physiologic leukorrhea
 Incorrect Answer ImageD.Syphilis
 Incorrect Answer ImageE.Trichomoniasis

A 15-year-old boy is brought to the physician because of a 3-month history of episodic tingling
of his hands, followed by blue discoloration, then red discoloration. The entire episode lasts
approximately 5 minutes and usually occurs when he is cold. He has had occasional headaches.
He denies fever, weight loss, weakness, rash, arthritis, or shortness of breath. He has a history of
exercise-induced asthma treated with albuterol. His father has a history of hypertension. He plays
rugby for his school team. He does not smoke cigarettes, drink alcohol, or use illicit substances.
His temperature is 37.2°C (99°F), pulse is 70/min, respirations are 12/min, and blood pressure is
110/70 mm Hg. His oxygen saturation is 99% on room air. Physical examination shows no
abnormalities. Which of the following is the most appropriate next step in the management of
this patient?
 Incorrect Answer ImageA.Antinuclear antibody level
 Incorrect Answer ImageB.C3 and C4 complement levels
 Correct Answer ImageC.Nailfold microscopy
 Incorrect Answer ImageD.Observation
 Incorrect Answer ImageE.Rheumatoid factor level
 Incorrect Answer ImageF.Urine drug screen

A 12-year-old girl is brought to the physician for a health maintenance examination. She is doing
well and has no significant medical problems except for frequent nosebleeds. The nosebleeds
typically "begin out of nowhere" and with no discernible trigger. They occur from either side of
her nose. The other day, she was walking home from school and suddenly started bleeding.
When she arrived home, her shirt and pants were stained with blood. The patient's
immunizations are up to date. She has no known environmental allergies or allergies to
medication. She has no other history of easy bruising or bleeding. She is in the 70th percentile
for height and weight and her vital signs are within normal limits. Examination of the nasal
passages shows moist, pink mucosa with a hemorrhagic crust on the anterior nasal septum on the
right side and a small, superficial ulceration on the left anterior nasal septum. Which of the
following is the most likely cause of the patient's recurrent epistaxis?
 Incorrect Answer ImageA.Acquired hemophilia
 Incorrect Answer ImageB.Christmas disease
 Correct Answer ImageC.Chronic nonallergic or allergic rhinitis
 Incorrect Answer ImageD.Chronic sinusitis
 Incorrect Answer ImageE.Juvenile nasopharyngeal angiofibroma

A 4-month-old girl is brought to the physician by her mother because of an itchy rash for 1 week.
The mother states that the infant has been rubbing and scratching it, especially at night. She has
been on a soy formula because of intolerance to cow's milk formula from birth. Her temperature
is 36.9°C (98.4°F). Examination shows excoriated areas of erythema on the forehead and cheeks.
There are also bilateral red, excoriated plaques on the elbows. Which of the following is the most
appropriate long-term maintenance therapy in the treatment of this condition?
 Incorrect Answer ImageA.Antihistamines
 Correct Answer ImageB.Cutaneous hydration with emollients
 Incorrect Answer ImageC.Phototherapy
 Incorrect Answer ImageD.Topical glucocorticoids
 Incorrect Answer ImageE.Topical immunomodulator
 Incorrect Answer ImageF.Topical tar preparations

A newborn male has a unilateral undescended testicle at birth. He was born at 39 weeks gestation
with an uncomplicated prenatal course, labor, and vaginal delivery. Apgar scores were 9 and 9 at
1 and 5 minutes, respectively. The newborn examination performed by his pediatrician shows no
other abnormalities. The testicle is not palpable in the upper scrotum or inguinal canal. Which of
the following is the most appropriate statement when the pediatrician reports this finding to the
parents?
 Incorrect Answer ImageA."Both testes should be removed due to the significant increase
in the likelihood of development of a germ cell tumor in either testicle."
 Correct Answer ImageB."If the testicle does not descend by 4 months, surgery should be
performed to locate it and attempt an orchiopexy no later than 9–15 months of age."
 Incorrect Answer ImageC."If the testicle does not descend by age 1 year, surgery should
be performed to locate it and attempt an orchiopexy at 2–3 years of age, when surgery is easier."
 Incorrect Answer ImageD."The chance of infertility due to this condition is 75%."
 Incorrect Answer ImageE."There are expected pathological changes present already in
the undescended testicle, and, therefore, it needs to be removed."
A 4,196 g (9 lb, 4 oz) male infant is delivered at 38 weeks' gestation via vaginal delivery to a 31-
year-old woman, gravida 1, para 1, who has diabetes. The delivery was complicated by shoulder
dystocia. Initial laboratory studies show a serum glucose level of 20 mg/dL and a hematocrit of
65%. Which of the following is most likely associated with this infant's condition?
 Incorrect Answer ImageA.Aniridia
 Incorrect Answer ImageB.Macroglossia
 Incorrect Answer ImageC.Omphalocele
 Incorrect Answer ImageD.Single palmar crease
 Correct Answer ImageE.Small left colon

A 6-month-old boy is brought to the emergency department because of a three-day history of


cough, congestion and low grade fever. The mother states that the baby has not been feeding well
and has used only two diapers over the past 24 hours. Physical examination shows a pale infant
in respiratory distress. Temperature is 37.8ºC (100.1ºF), pulse is 170/min, respirations are 75/min
and oxygen saturation is 88% on room air. The patient exhibits nasal flaring and subcostal and
intercostal retractions. Lung examination shows diffuse wheezing. Cardiac examination shows a
regular but tachycardic rhythm without murmur. The central capillary refill is 4 seconds. The
remainder of the examination is normal. Which of the following is the most appropriate initial
step in management?
 Incorrect Answer ImageA.Albuterol nebulizer treatment
 Incorrect Answer ImageB.Bolus of intravenous fluids
 Incorrect Answer ImageC.Chest x-ray film
 Incorrect Answer ImageD.Intravenous steroids
 Correct Answer ImageE.Oxygen therapy

A 2-year-old girl is brought to the physician by her grandmother for a well-child examination.
The grandmother reports that the girl frequently has "the cold." She has not had any
immunizations since last year. She stands independently but cannot walk without assistance. The
girl lives with her grandmother because her mother is in and out of drug rehabilitation programs,
and her father is estranged. One year ago she was at the 5th percentile for height and weight; she
is currently at the 4th percentile. Her temperature is 37.8ºC (100.1ºF) and her pulse is 160/min.
Examination of the oral mucosa shows erythema with multiple white, adherent plaques that ooze
blood when scraped with a tongue depressor. Otoscopy shows an erythematous left tympanic
membrane that is dull and bulging. The submandibular, cervical, axillary, and inguinal lymph
nodes are enlarged and firm. Which of the following is the most likely diagnosis? 
 Correct Answer ImageA.Acquired immune deficiency syndrome
 Incorrect Answer ImageB.Disseminated herpes simplex virus infection
 Incorrect Answer ImageC.Infectious mononucleosis
 Incorrect Answer ImageD.Streptococcal pharyngitis
 Incorrect Answer ImageE.Systemic candidiasis

A frantic mother telephones a pediatrician's office. The woman's 7-year-old daughter was
playing under the sink and accidentally spilled a strong, corrosive alkaline drain cleaner all over
her arms and legs. On the phone, the nurse can hear the child screaming in the background. The
physician's office and nearest hospital are a few miles away from the woman's home. Which of
the following is the most appropriate instruction to give the mother? 
 Incorrect Answer ImageA.Cover the burned areas with triple antibiotic ointment until the
child can be seen at the office
 Correct Answer ImageB.Get the child into the shower right away and keep the water
running over her for 30 minutes before bringing her to the emergency department
 Incorrect Answer ImageC.Get the child to the emergency department as soon as possible
 Incorrect Answer ImageD.Wash the burned areas with diluted vinegar and bring the child
to the office
 Incorrect Answer ImageE.Wrap the burned areas in sterile dressings before bringing the
child to the emergency department

An 11-year-old boy is brought to the physician because of the new onset of headaches. He tells
the physician that his head hurts all over, but is worse over the front and back. The headaches
started approximately 1 month ago and were controlled with ibuprofen at first, but are now
worsening. They tend to start in the morning after he gets out of bed and last through the
afternoon. This prevents him from concentrating in school. He had a headache today, but it
ended approximately 2 hours ago. Physical examination shows slight ataxia but is otherwise
normal. Which of the following is the most appropriate next step in management?
 Incorrect Answer ImageA.Obtain an electroencephalogram
 Correct Answer ImageB.Obtain MRI of the brain
 Incorrect Answer ImageC.Refer the patient for behavior management
 Incorrect Answer ImageD.Refer the patient for biofeedback and self-hypnosis exercises
 Incorrect Answer ImageE.Start the patient on sumatriptan
A 2-month-old male infant is brought to the physician because of a 1-week history of low-grade
fever and persistent cough that occurs in short, abrupt bursts. He had an eye infection at 1 week
of age that resolved with antibiotic eye drops. He appears in no distress. His temperature is
37.2ºC (99ºF), pulse is 140/min, and respirations are 75/min. An x-ray film of the chest shows
hyperinflation with fine reticular opacities. Laboratory studies show: 
Hematocrit  42%
Hemoglobin  13.2 g/dL
Leukocyte count 10,000/mm3
Neutrophils  50%
Lymphocytes  35%
Monocytes 5%
Eosinophils  10%
Which of the following is the most likely cause of this infant's condition? 
 Incorrect Answer ImageA.Aspiration of food
 Correct Answer ImageB.Chlamydia trachomatis
 Incorrect Answer ImageC.Mycoplasma pneumoniae
 Incorrect Answer ImageD.Staphylococcus aureus
 Incorrect Answer ImageE.Streptococcus pneumoniae

A 6-year-old girl with acute lymphocytic leukemia (ALL) is admitted to the hospital because of a
rash and fever. Her mother reports that the girl has had a fever and runny nose for the past 6
days, and a rash developed on the child's face 3 days ago. Her temperature is 37.5ºC (99.5ºF),
blood pressure is 110/60 mm Hg, pulse is 90/min, and respirations are 16/min. Examination
shows 10 to 15 small, translucent vesicles on an erythematous base. There are pustules with
central umbilication, erythematous macules, and crusted lesions on the face, upper chest, and
abdomen. Which of the following is the most serious possible complication of this patient's
condition? 
 Incorrect Answer ImageA.Arthritis
 Incorrect Answer ImageB.Cellulitis
 Incorrect Answer ImageC.Endocarditis
 Incorrect Answer ImageD.Hepatitis
 Correct Answer ImageE.Pneumonia

The mother of a newborn is concerned because he has not had a wet diaper during the first 24
hours of his life. The boy is the third child for this 26-year-old mother and 29-year-old father.
His siblings have no significant history. Prenatal care was adequate and the mother says that the
infant's kidneys were "a little bit swollen" on prenatal ultrasound. There was no birth trauma and
no abnormalities were seen on initial examination at birth. Currently the infant appears
comfortable and there is no fever. Significant findings on physical examination include a
palpable mass in the suprapubic area. The remainder of the genitourinary examination is normal.
Which of the following is the most appropriate next step in the management of this infant? 
 Incorrect Answer ImageA.Obtain a renal-bladder ultrasound
 Incorrect Answer ImageB.Obtain a surgery consult
 Incorrect Answer ImageC.Order a retrograde urethrogram (RUG)
 Incorrect Answer ImageD.Perform voiding cystourethrogram (VCUG)
 Correct Answer ImageE.Place a urethral catheter

A 4-year-old boy has severe pain in both of his legs. On physical examination, his temperature is
37.7ºC (99.8ºF), blood pressure is 108/68 mm Hg, pulse is 96/min, and respirations are 17/min.
He is noted to have marked pallor on his lips and palpebral conjunctiva. Numerous purpura and
petechiae are noted on his skin. His spleen is palpable 3 cm below his left costal margin.
Laboratory studies show a white blood cell count of 1600/mm3; hemoglobin 6.1 g/dL; and
platelets 36,000/mm3. Which of the following diagnoses is most consistent with these findings?
 Correct Answer ImageA.Acute lymphocytic leukemia
 Incorrect Answer ImageB.Aplastic anemia
 Incorrect Answer ImageC.Henoch-Schönlein purpura
 Incorrect Answer ImageD.Immune thrombocytopenic purpura
 Incorrect Answer ImageE.Thrombotic thrombocytopenic purpura

A 15-year-old boy is brought to the physician because of recurrent acne. He reports washing his
face twice daily with a mild soap but admits to eating large amounts of chocolate and spicy food.
Examination shows mild to moderate acne, mostly consisting of open comedones, some closed
comedones, and a few pustules on the forehead and cheeks. The remainder of the examination is
unremarkable. Which of the following is the most appropriate recommendation? 
 Incorrect Answer ImageA.Avoidance of chocolate and spicy foods
 Incorrect Answer ImageB.Frequent face washing with strong soap
 Incorrect Answer ImageC.Oral treatment with isotretinoin
 Correct Answer ImageD.Topical application of tretinoin
 Incorrect Answer ImageE.Treatment with oral minocycline

A 5-day-old boy is brought to the pediatrician for a newborn examination because his mother
delivered him at home. The examination is completely normal except for asymmetry of the
scrotum. On the right, the scrotum is smooth and distended and a tense mass is palpated within it.
There is a slight uniform bluish discoloration to the scrotum. The baby remains calm during the
examination. Compression of the scrotal contents completely reduces the mass. The physician
notes that shining a penlight to the posterior aspect of the scrotum allows the light to pass
through diffusely. Which of the following is the most likely diagnosis?
 Incorrect Answer ImageA.Hydrocele
 Correct Answer ImageB.Hydrocele with inguinal hernia
 Incorrect Answer ImageC.Neonatal testicular torsion
 Incorrect Answer ImageD.Varicocele
 Incorrect Answer ImageE.Yolk sac tumor

A 3-year-old boy is brought to the physician for a well-child visit. The parents state that he has
been well, but has lately lost his appetite and seems a little moody. They attribute this to the fact
that they moved in with the mother's parents approximately 6 months ago because of financial
issues and the boy has yet to blend into the new environment. In addition, her parent's house is
old and stuffy and needs renovating, which they started doing 2 months ago by deplastering the
walls in the boy's room and repainting. The child is in no acute distress but seems a little
apathetic. He is in the 20th percentile for weight and the 30th percentile for height. Laboratory
studies show that the hemoglobin concentration is 7.0 g/dL. Which of the following is the next
best step in management?
 Incorrect Answer ImageA.Administer calcium disodium edetate
 Incorrect Answer ImageB.Blood transfusion
 Incorrect Answer ImageC.Iron supplementation
 Incorrect Answer ImageD.Reassure the parents that improvement will ensue as the child
adjusts to the new environment
 Correct Answer ImageE.Remove the child from his current environment

A 4-year-old girl is brought to the physician by her mother because of foul-smelling vaginal
discharge for several weeks. Three days ago, the child began scratching the vulva and
complaining of burning with urination. She is otherwise healthy and has never had a similar
problem. Examination shows normal structural anatomy without evidence of atrophy. The
medial aspects of the labia majora are erythematous and excoriated. There is a mucous discharge
with streaks of admixed blood. Which of the following is the most likely diagnosis? 
 Incorrect Answer ImageA.Lichen sclerosis
 Incorrect Answer ImageB.Pelvic inflammatory disease (PID)
 Incorrect Answer ImageC.Sarcoma botryoides (embryonal rhabdomyosarcoma)
 Incorrect Answer ImageD.Sexual abuse
 Correct Answer ImageE.Vaginal foreign body

A mother brings her 18-month-old daughter to the emergency department 20 minutes after the
child had an episode of choking and gasping while eating potato chips. The mother doesn't think
the child went blue during the episode, and the child returned to playing with her toys shortly
after the episode. On examination, the child has an intermittent cough but is otherwise playful
and in no apparent acute distress. Physical examination is normal, except for an expiratory
wheeze noted in the right lung fields. Blood pressure is 95/58 mm Hg, respirations are 27/min,
pulse is 107/min, and O2 saturation is 97% on room air. Chest radiograph appears normal, but
expiratory films are unable to be performed due to her age. Which of the following is the most
appropriate next step in management?
 Incorrect Answer ImageA.Blind finger sweeps
 Incorrect Answer ImageB.Bronchodilator administration
 Incorrect Answer ImageC.Endotracheal intubation
 Incorrect Answer ImageD.Expectant management
 Incorrect Answer ImageE.Flexible bronchoscopy
 Correct Answer ImageF.Rigid bronchoscopy

A 3-day-old newborn girl develops a fever of 38.2ºC (101.7ºF). She was born at term via
uncomplicated spontaneous vaginal delivery. Apgar scores were 7 and 9 at 1 and 5 minutes,
respectively. After delivery, her mother remained hospitalized because of fever, myalgia, fatigue,
and abdominal discomfort. The infant is promptly transferred to the neonatal intensive care unit
for a sepsis workup and prophylactic antibiotics. Within the next several hours, she appears
much sicker and has weak pulses with delayed capillary refill, tachycardia, and respiratory
distress. She requires endotracheal intubation and mechanical ventilation. On repeat
examination, her pulse is 180/min. Cardiac examination shows distant heart sounds with a full
gallop rhythm and a grade 3/6 decrescendo systolic murmur over the apex. Her mean systolic
pressure is 30 mm Hg from the umbilical artery. Laboratory studies show:
Arterial blood gas on 80% oxygen
Hepatic transaminases Mildly elevated
pH 7.18
PCO2 40 mm Hg
PO2 50 mm Hg (FiO2 1.0) 
Electrolytes
Na+ 137 mEq/L
K+ 3.9 mEq/L
Cl –
100 mEq/L
HCO3– 10 mEq/L
Chest x-ray shows a large heart with prominent vascular markings. ECG shows sinus
tachycardia, reduced QRS voltage, and ST-segment and T-wave abnormalities. Which of the
following is the most likely etiology? 
 Incorrect Answer ImageA.Beta-hemolytic streptococci Group A
 Incorrect Answer ImageB.Beta-hemolytic streptococci Group B
 Correct Answer ImageC.Coxsackievirus B
 Incorrect Answer ImageD.Endocardial fibroelastosis
 Incorrect Answer ImageE.Hepatitis B virus
 Incorrect Answer ImageF.Staphylococcus aureus

A newborn male infant born to a diabetic mother is brought to the neonatal intensive care unit.
He was born at 42 weeks’ gestation and weighs 5 kg (11 lb). His initial glucose was 10 mg/dL.
An umbilical venous catheter is inserted, the baby is given a bolus of glucose, and a glucose
infusion is started. His respirations are 80/min. His oxygen saturations are 90% on room air. He
is started on 50% hood oxygen and pulse oximeter reading increases to 100%. An umbilical
artery catheter is then placed. His mean blood pressure is 30 mm Hg. A chest radiograph shows
prominent perihilar vessels, fluid in the fissures, and darkened, slightly hyperexpanded lung
fields. Which of the following findings would most likely be seen on echocardiography with this
presentation?
 Correct Answer ImageA.Asymmetric septal hypertrophy
 Incorrect Answer ImageB.Atrial septal defect
 Incorrect Answer ImageC.Hypoplastic left heart
 Incorrect Answer ImageD.Transposition of the great arteries
 Incorrect Answer ImageE.Ventricular septal defect

A 2-year-old boy is brought to the physician by his father because of a progressively worsening
productive cough for the past week. This is his third visit to the physician in 1 year for similar
symptoms. His parents also report several episodes of diarrhea since birth. His temperature is
37.0ºC (98.6ºF), blood pressure is 109/72 mm Hg, pulse is 90/min, and respirations are 16/min.
His height and weight are in the fifth percentile. On physical examination, fine crackles are heard
in the lungs bilaterally, and he has mild clubbing of his fingers. A chest x-ray shows poorly
defined bronchovascular markings and a CT shows diffuse bronchiectasis. Which of the
following is the most likely diagnosis?
 Incorrect Answer ImageA.Allergic bronchopulmonary aspergillosis
 Correct Answer ImageB.Cystic fibrosis
 Incorrect Answer ImageC.Foreign body aspiration
 Incorrect Answer ImageD.Mycoplasma pneumoniae infection
 Incorrect Answer ImageE.Primary ciliary dyskinesia

A 14-year-old boy is brought to the emergency department by his father. He was playing a game
of ice hockey that afternoon and sustained a direct hit to the head by the puck at full speed. He
immediately lost consciousness and fell to the ice. The father and the coach were able to arouse
him with smelling salts and help him to his father's car to go to the local hospital. He seemed fine
at first, lucid and alert to the point that his dad almost turned the car back toward the ice rink, but
then he started being confused and nauseated all of a sudden, and about 20 minutes after the
initial blackout, he lost consciousness again. On arrival at the emergency department, he is
unconscious but grimaces and retracts to painful stimuli. The pupils are reactive and symmetrical
bilaterally. Vital signs are within normal limits and head CT shows a lenticular extracerebral
hemorrhage. Which of the following is the most likely diagnosis? 
 Incorrect Answer ImageA.Cerebral concussion
 Incorrect Answer ImageB.Cerebral contusion
 Correct Answer ImageC.Epidural hematoma
 Incorrect Answer ImageD.Subarachnoid hemorrhage
 Incorrect Answer ImageE.Subdural hematoma

A 15-year-old girl comes to the physician for a well-child examination. Menarche was at age 11,
and her last menstrual period was 8 days ago. She has a history of asthma controlled with an
albuterol inhaler. She underwent an appendectomy at age 9. She has been sexually active since
age 14, and she and her partner use condoms inconsistently. She has had no fever, abdominal
pain, shortness of breath, or cough. Examination, including pelvic examination, shows no
abnormalities. Which of the following diagnostic studies is most appropriate to perform in this
patient?
 Incorrect Answer ImageA.Dark field microscopy
 Incorrect Answer ImageB.Giemsa stain
 Incorrect Answer ImageC.Oxygen saturation
 Incorrect Answer ImageD.Pap smear
 Incorrect Answer ImageE.Peak flow measurement
 Correct Answer ImageF.Polymerase chain reaction test for N. gonorrhea and Chlamydia
 Incorrect Answer ImageG.Pulmonary function testing
 Incorrect Answer ImageH.Tzanck smear

A 4-year-old boy is brought to the physician because of weakness and malaise for the past
month. His mother has also noticed that while bathing him he feels unusually stiff. He refuses to
eat as much as he used to, and he has not been gaining any weight. He also complains of feeling
dizzy and of belly pain. His past medical history is unremarkable. He has had all his childhood
immunizations. On physical examination, he is afebrile and his blood pressure is 80/48 mm Hg.
He is found to have orthostatic hypotension. Examination of other systems shows no
abnormalities. Laboratory studies show:
White blood cells 6,800/mm3
Hematocrit 46%
Platelets 252,000/mm3
Sodium 130 mEq/L
Potassium 5.6 mEq/L
Chloride 97 mEq/L
Bicarbonate 20 mEq/L
Blood urea nitrogen 22 mg/dL
Creatinine 0.6 mg/dL
Glucose 45 mg/dL
Cortisol level is low. An assay for fatty acids of at least 22 carbons demonstrates a high level in
the child's blood. Which of the following statements is true for this diagnosis?
 Incorrect Answer ImageA.Glycerol trierucate with glyceryl trioleate (Lorenzo's oil) has
been shown to slow the progression of symptoms once they have begun
 Correct Answer ImageB.It is an X-linked disorder
 Incorrect Answer ImageC.Magnetic resonance imaging of the brain is usually normal
 Incorrect Answer ImageD.Renal insufficiency may necessitate the use of hemodialysis
 Incorrect Answer ImageE.The responsible gene codes for a mitochondrial membrane
protein

A 12-year-old boy is brought to his pediatrician for a routine health maintenance visit. He has
been well except for occasional attacks of asthma and has met all developmental milestones. His
immunizations are up to date. He occasionally uses albuterol for his asthma. Physical
examination is remarkable for a blood pressure of 150/90 mm Hg in both arms. Which of the
following is the most likely cause of his hypertension?
 Incorrect Answer ImageA.Albuterol usage
 Incorrect Answer ImageB.Chronic lung disease
 Incorrect Answer ImageC.Coarctation of the aorta
 Incorrect Answer ImageD.Congenital heart disease
 Correct Answer ImageE.Renal disease

A 4-year-old boy is brought to the emergency department by his mother immediately after
sustaining a deep cut to his right arm while playing outside. His medical history is unremarkable.
He takes no medications, and his immunizations are up to date. Examination shows a 4-cm
wound that has dirt in it. The wound is thoroughly cleaned and debrided, then sutured. Which of
the following is the most appropriate next step in management? 
 Incorrect Answer ImageA.DTaP
 Correct Answer ImageB.No immunization or immunoglobulin
 Incorrect Answer ImageC.Tetanus-diphtheria toxoid (Td)
 Incorrect Answer ImageD.Tetanus-diphtheria toxoid and tetanus immune globulin
 Incorrect Answer ImageE.Tetanus immune globulin (TIG)
 Incorrect Answer ImageF.Tetanus toxoid

A 17-year-old boy comes to the physician for a routine annual examination. In the last year, he
has gained 9 kg (20 lb). Over the past 3 years, he has been trending upward on his body mass
index (BMI) curve and is now over the 95th percentile for age and sex. His temperature is 37.2ºC
(99.0ºF), blood pressure is 158/90 mm Hg using a large, appropriate cuff, pulse is 74/min, and
respirations are 16/min. The blood pressure is taken two more times over the next hour and the
reading is confirmed. His mother has a similar body habitus and is being treated for type 2
diabetes. The pediatrician also obtains the following fasting laboratory studies:
Triglycerides 200 mg/dL
Total cholesterol 240 mg/dL
Serum glucose 150 mg/dL 
Based on these findings, which of the following would most likely develop in this patient? 
 Incorrect Answer ImageA.Acute pancreatitis
 Incorrect Answer ImageB.Cardiomyopathy
 Incorrect Answer ImageC.Cushing disease
 Incorrect Answer ImageD.Hypothalamic dysfunction
 Correct Answer ImageE.Nonalcoholic fatty liver disease

A 4-year-old boy is brought to the physician by his mother because the child has been ignoring
her for 3 months. The boy pays no attention when she asks him a question, especially when she
stands on his left side. He has had 4 ear infections during the past year. His last ear infection was
4 months ago. He plays well with other children and speaks in full sentences. His temperature is
37.1°C (98.8°F). The patient appears comfortable at rest. When the physician asks the boy to
bring a toy to her, he immediately complies. Pneumatic otoscopy examination shows bilaterally
dull and retracted tympanic membranes with decreased tympanic membrane movement, greater
on the left. Which of the following is the most appropriate next step in management? 
 Incorrect Answer ImageA.Adenoidectomy
 Correct Answer ImageB.Audiometry
 Incorrect Answer ImageC.Myringotomy
 Incorrect Answer ImageD.Prophylactic antibiotics
 Incorrect Answer ImageE.Tonsillectomy

A 2-year-old boy is brought to the physician by his parents because of a 3-day history of diarrhea
and vomiting. He is able to tolerate small amounts of fluids, although he has been vomiting once
or twice per day and has frequent loose stools. His parents state that he has been having normal
urine output. His temperature is 37.0ºC (98.6ºF), blood pressure is 88/42 mmHg, pulse is 100
/min, and respirations are 20/min. Examination shows dry mucous membranes, good skin turgor,
and capillary refill of 2 seconds. The following serum basic metabolic panel was performed: 
Na+ 137 meq/L
K+ 3.8 meq/L
Cl–
100 meq/L
HCO3– 18 meq/L
Glucose 65 mg/dL
BUN 28 mg/dL
Cr 0.4 mg/dL
Which of the following is the next best step in management? 
 Incorrect Answer ImageA.Hospitalization and IV fluids
 Incorrect Answer ImageB.Hospitalization, IV fluids, and limited oral intake
 Incorrect Answer ImageC.Oral rehydration therapy at home followed by a clear liquid
diet for 24 hours
 Correct Answer ImageD.Oral rehydration therapy at home followed by a normal diet
 Incorrect Answer ImageE.Oral rehydration therapy at home followed by a restrictive
bland diet

A 16-year-old girl is brought to the emergency department after being found unresponsive by her
parents. Her father reports that she has been very depressed over the past few weeks after
breaking up with her boyfriend. The father describes the daughter as being isolated and spending
much of her time alone in her room. When she failed to get up for school, the father went into
her locked bedroom and found her face down and barely breathing on her bed. The father
brought the empty bottles of each of the following medications found next to the patient:
clonazepam, diphenhydramine, oxycodone, and olanzapine. He also states there was an empty
bottle of vodka found next to his daughter. An EMS team gave her a narcotic antagonist, IV
dextrose solution, and IV fluids. In addition, her trachea was intubated for airway protection
before she was brought to the emergency department. On physical examination, her pupils are 8
mm bilaterally and minimally reactive. Her abdomen is markedly distended with decreased
bowel sounds. Which of the following ingested substances is most likely to threaten this patient's
life within the next 24 hours? 
 Incorrect Answer ImageA.Benzodiazepines
 Correct Answer ImageB.Diphenhydramine
 Incorrect Answer ImageC.Ethanol
 Incorrect Answer ImageD.Olanzapine
 Incorrect Answer ImageE.Opiates

A 2,000 g (4.4 lb) female infant is born at term to a 16-year-old girl, gravida 1, para 0, via
spontaneous vaginal delivery. The mother has had no prenatal care. She did not smoke or drink
alcohol during her pregnancy. Apgar scores are 5 and 6 at 1 and 5 minutes, respectively. Eye
examination shows bilateral white pupillary reflexes. Cardiac examination shows an apical
heave, a loud machine-like continuous murmur with a thrill at the upper left sternal border and
bounding peripheral pulses. A chest x-ray shows a large heart and increased pulmonary vascular
markings. Which of the following maternal infections is most likely the cause of the infant's
condition?
 Incorrect Answer ImageA.Herpes simplex
 Incorrect Answer ImageB.Human papillomavirus
 Correct Answer ImageC.Rubella
 Incorrect Answer ImageD.Syphilis
 Incorrect Answer ImageE.Varicella

An 11-year-old boy is brought to the physician for a routine health screening examination before
entering 6th grade. His parents report that he has no complaints and is generally in very good
health. He has had a recent growth spurt that eliminated the marginal obesity present on the prior
physical examination, and he is now in the 80th percentile for both height and weight. The
patient's vital signs are within normal limits and the only abnormality is a slight discrepancy in
shoulder level. The right shoulder seems to be about half an inch higher than the left shoulder
when he stands up. Which of the following is the most appropriate way to initially screen this
patient for scoliosis?
 Correct Answer ImageA.Adams forward bend
 Incorrect Answer ImageB.Myelogram of the spinal canal
 Incorrect Answer ImageC.Neurosurgical evaluation
 Incorrect Answer ImageD.Pulmonary function testing
 Incorrect Answer ImageE.Radiographs of the spine

A previously healthy 11-year-old boy is brought to the physician because of a fever and earache
for 5 days. He felt nauseated and has been retching; he had not vomited until this morning, and
he saw streaks of bright red blood in the emesis. There is no abdominal pain. Physical
examination and laboratory studies show no abnormalities. Which of the following is the most
likely cause of the hematemesis?
 Incorrect Answer ImageA.Esophageal varices
 Incorrect Answer ImageB.Esophagitis
 Correct Answer ImageC.Mallory-Weiss tear
 Incorrect Answer ImageD.Meckel’s diverticulum
 Incorrect Answer ImageE.Peptic ulcer disease

A 24-month-old toddler is brought to the physician by his parents for a routine well-child
examination. The toddler has no history of developmental delay, however, his mother is
concerned because her neighbor's son "seems more advanced" in performing motor functions.
The patient was born at 40 weeks’ gestation via an uncomplicated vaginal delivery. He has no
history of chronic medical conditions and takes no medications. Physical examination is
unremarkable. Which of the following motor milestones is most consistent with this child’s age?
 Incorrect Answer ImageA.Building a tower of two cubes
 Incorrect Answer ImageB.Copying a closed circle
 Correct Answer ImageC.Pick up objects while standing without losing balance
 Incorrect Answer ImageD.Scribbling
 Incorrect Answer ImageE.Walking backward

A 12-year-old boy is brought to the emergency department because of a 2-month history of


recurrent headaches and deteriorating school performance. Neurologic examination is
unremarkable except for bitemporal deficits in his visual field. CT of the head shows
calcifications in the suprasellar region. MRI of the brain shows a multicystic tumor displacing
the optic chiasm. The patient undergoes surgical resection. During the procedure, the surgeon
notices that the cystic cavities contain a viscous yellow fluid. Which of the following is the most
likely diagnosis?
 Correct Answer ImageA.Craniopharyngioma
 Incorrect Answer ImageB.Cysticercosis
 Incorrect Answer ImageC.Ganglioglioma
 Incorrect Answer ImageD.Pilocytic astrocytoma
 Incorrect Answer ImageE.Pituitary adenoma

A 13-year old boy is being evaluated by a developmental pediatrician and child psychiatrist.
After appropriate testing, he is found to have significantly subaverage intellectual functioning
and impairment of adaptive functioning. He has had learning problems throughout elementary
school and has been held back 1 year. Based on a thorough history from the parents and hours of
observation and examination, he is found to have impairments in social interaction and
communication and repetitive, stereotyped patterns of behaviors from early childhood. On
physical examination, his height is at the 97th percentile; there is increased distance from the top
of his forehead to his chin; he has large, cupped ears and a large, square jaw; and testicular
volume is significantly greater than to be expected for a child of his age and sexual maturity
rating. Which of the following is the most likely mechanism that produced his underlying
problem?
 Correct Answer ImageA.Amplification of a polymorphic repeat sequence in the
untranslated region of a specific gene
 Incorrect Answer ImageB.Deletion of the maternally derived imprinted segment of
chromosome 15
 Incorrect Answer ImageC.Deletion of the paternally derived imprinted segment of
chromosome 15
 Incorrect Answer ImageD.Digenic inheritance
 Incorrect Answer ImageE.Maternal uniparental disomy for chromosome 15
 Incorrect Answer ImageF.Paternal uniparental disomy for chromosome 15

 Incorrect Answer ImageG.Y-linked inheritance from the father

A 6-year-old boy is brought to the emergency department by his parents because of abdominal
pain for the past 6 hours. At home, he vomited twice and initially complained only of
periumbilical pain since morning, but now he also has pain in the right side of his abdomen. He
has never had similar pain in the past. His past medical history is significant for an episode of
painless bleeding per rectum at age 2 years when they lived in Mexico. His temperature is 38.0ºC
(100.4ºF), pulse is 120/min, respirations are 20/min, and blood pressure is 128/86 mm Hg.
Physical examination shows rebound tenderness in the right lower quadrant and abdominal wall
rigidity. Sharp pain is elicited by a voluntary cough. Laboratory studies show:
WBC count  15,000/mm3
Neutrophils  70%
Lymphocytes  30%
Eosinophils  3%
Basophils  0.5%
Monocytes  5%
Which of the following is the most likely diagnosis? 
 Incorrect Answer ImageA.Ascariasis
 Incorrect Answer ImageB.Constipation
 Incorrect Answer ImageC.Intestinal obstruction
 Correct Answer ImageD.Meckel diverticulitis
 Incorrect Answer ImageE.Right-sided colonic diverticulitis
 Incorrect Answer ImageF.Yersinia enterocolitis

A previously healthy 13-year-old girl is brought to the emergency department with an acute
onset of red urine after she played a vigorous soccer match in the morning on a hot August day.
There is no family history of hematuria. Her physical examination is unremarkable. Urinalysis
shows:
Color Red
Blood +4
pH 6.2
Specific gravity 1.024
Glucose Negative
Protein Trace
Red blood cells None
White blood cells None
Nitrite Negative
Leukoesterase Negative
Which of the following is the most likely explanation of the red urine?
 Incorrect Answer ImageA.Glomerulonephritis
 Incorrect Answer ImageB.Hematuria
 Incorrect Answer ImageC.Ingestion of food coloring
 Correct Answer ImageD.Myoglobinuria
 Incorrect Answer ImageE.Presence of urates
 Incorrect Answer ImageF.Trauma
 Incorrect Answer ImageG.Urinary tract infection

A 4-year-old girl is brought to the physician because of new-onset pain and swelling in her left
knee for 1 week. She has been complaining of knee pain every morning for the past 7 days. It is
worst when she wakes up because she can’t get up from bed to walk, but within an hour or two it
improves and she is fine for the rest of the day. Her parents tried giving her acetaminophen but it
did not seem to help. She had just recovered from an upper respiratory tract illness and was
doing well except for being a little tired and sleeping longer. The parents deny any trauma or
injury to her knee. She has no significant past medical history and takes no medication. On
physical examination, the patient does not seem to be in any acute distress. Her vital signs are
within normal limits and her blood pressure is 90/60 mm Hg. Physical examination shows a
mildly edematous and warm left knee with decreased range of motion. Erythrocyte
sedimentation rate (ESR) is 57 mm/hr in the first hour. A complete blood count and urinalysis
are unremarkable. Radiography of the knee does not show any destructive changes.
Ophthalmologic examination shows bilateral asymptomatic nongranulomatous uveitis. Which of
the following findings would indicate a favorable prognosis in this patient?
 Correct Answer ImageA.Antinuclear antibody positive
 Incorrect Answer ImageB.Fever
 Incorrect Answer ImageC.Hepatosplenomegaly
 Incorrect Answer ImageD.HLA B27 positive
 Incorrect Answer ImageE.Rheumatoid factor positive
A 1-year-old boy is brought to the emergency department because of recurrent productive cough
and congestion for the past 6 months. His weight and height are at the 5th percentile. His
temperature is 37.0ºC (98.6ºF) and pulse oximetry is 96% on room air. Physical examination
shows purulent rhinorrhea and nasal polyps. Auscultation of the heart shows a regular rate and
rhythm, with the point of maximal impulse felt on the right side of the chest. Which of the
following is the most likely diagnosis? 
 Incorrect Answer ImageA.Cystic fibrosis
 Incorrect Answer ImageB.Human immunodeficiency virus infection
 Correct Answer ImageC.Primary ciliary dyskinesia
 Incorrect Answer ImageD.Severe combined immunodeficiency
 Incorrect Answer ImageE.X-linked agammaglobulinemia
A 9-year-old boy is brought by his mother to the physician's office because he complains of
feeling tired for the last 2 days. He had numerous episodes of diarrhea 3 days ago. His mother
also notes that he has been passing less urine today, even though he was drinking fluids. The
mother reports that he has received all vaccines appropriate for his age. On physical examination,
he appears pale, with sunken eyes and dry oral mucosa. Pulse is 120/min, respiratory rate is
16/min, temperature is 37.8°C (100°F), and blood pressure is 90/55 mm Hg. Lungs are clear to
auscultation. Cardiovascular examination is significant for tachycardia. Abdominal examination
shows diffuse tenderness with no guarding or rigidity. Extremities show no pitting edema. Lab
workup shows:
Hgb 8.4 g/dL
WBC 6,000/mm3
Platelets 50,000/mm3
Sodium 136 mEq/L
Potassium 4.7 mEq/L
Chloride 111 mEq/L
Bicarbonate 17 mEq/L
BUN 30 mg/dL
Creatinine 2.1 mg/dL
Glucose 80 mg/dL
Calcium 9.2 mg/dL
LDH 1,200 U/L (normal: 45–90 U/L)
PT Normal
aPTT Normal
Urinalysis reveals plenty of RBCs. Which of the following is the most likely diagnosis?
 Incorrect Answer ImageA.Acute glomerulonephritis
 Incorrect Answer ImageB.Disseminated intravascular coagulation
 Correct Answer ImageC.Hemolytic uremic syndrome
 Incorrect Answer ImageD.Henoch-Schönlein purpura
 Incorrect Answer ImageE.Volume depletion from infectious diarrhea

A 5-year-old boy is brought to the emergency room by his parents because of swelling of his
ankles and feet and around his eyes. When asked about the events leading up to this, the parents
state that about 2 weeks ago, he had a fever and diarrhea for 3 days. The edema is confirmed on
examination. The boy's vital signs are within normal limits and there are no other abnormal
physical findings. Urine studies show 4+ protein and fatty casts. Serum laboratory studies show:
Na 133 mEq/L
K 3.9 mEq/L
Cl 100 mEq/L
CO2 22 mEq/L
Cr 0.7 mg/dL
BUN 15 mg/dL
Total protein 3.7 g/dL
What is the next best step in management of this patient?
 Incorrect Answer ImageA.Admit for intravenous albumin and diuretic infusions
 Incorrect Answer ImageB.Admit for management of renal insufficiency
 Incorrect Answer ImageC.Follow up in the morning with his pediatrician for a repeat
urine sample
 Incorrect Answer ImageD.Refer to a nephrologist for a renal biopsy
 Correct Answer ImageE.Start a 24-hour urine collection for protein and start oral
prednisone

Parents bring a 2-month old baby to the pediatrician because of colic pain. The baby cries a lot in
pain and often bends himself forward at the level of the trunk. The parents state that the baby
burps within about 5 minutes after a feed. The pediatrician notes the presence of three
characteristically shaped, hypopigmented lesions on the baby's skin that are more apparent with
ultraviolet light. Examination of all other systems reveals nothing significant. Which of the
following would be the most appropriate treatment after confirming the diagnosis? 
 Incorrect Answer ImageA.Acetaminophen
 Correct Answer ImageB.Adrenocorticotropic hormone
 Incorrect Answer ImageC.Ketoconazole ointment
 Incorrect Answer ImageD.Ranitidine
 Incorrect Answer ImageE.Thickened feeds with positioning

A 4-year-old girl is brought to a new pediatrician for a preschool physical exam. The parents
state that they are new to this area and report a detailed history of the child as developmentally
and physically healthy. She is at the 5th percentile for height and 10th percentile for weight. On
physical examination she is a well-appearing girl with no facial deformities. Her skin is pale,
though she has several hyperpigmented patches on the abdomen and anogenital areas. She has
rudimentary thumbs bilaterally. The remainder of the physical exam is benign. She speaks in
sentences, and is able to draw a circle, square, and cross. Which of the following is the most
likely diagnosis?
 Incorrect Answer ImageA.Bloom syndrome
 Incorrect Answer ImageB.Diamond-Blackfan anemia
 Correct Answer ImageC.Fanconi anemia
 Incorrect Answer ImageD.Fanconi-Bickel syndrome
 Incorrect Answer ImageE.Thrombocytopenia and absent radii (TAR) syndrome

A 2-year-old girl is brought to the emergency department by her parents because of a severe
cough with sputum, fever, and rapid breathing. The cough began 1 month ago with sputum and
has become persistent. She has a history of prolonged neonatal jaundice and two episodes of
bronchiolitis at ages 7 and 13 months. Her temperature is 39°C (102.4°F), blood pressure is
90/64 mm Hg, pulse is 90/min, respirations are 60/min, and oxygen saturation on room air is
88%. Physical examination is pertinent for nasal polyps and coarse breath sounds that are heard
on auscultation of the chest. Chest radiograph shows hyperinflation with patchy consolidations.
She is below the fifth percentile for both height and weight. After establishing ABCs and
providing respiratory support, which of the following is the next best step in management?
 Incorrect Answer ImageA.Genetic testing
 Incorrect Answer ImageB.Histopathologic analysis of the nasal polyps
 Correct Answer ImageC.IV ceftazidime with tobramycin
 Incorrect Answer ImageD.IV levofloxacin
 Incorrect Answer ImageE.Oral trimethoprim-sulfamethoxazole
 Incorrect Answer ImageF.Pilocarpine iontophoresis test

A previously healthy 3-week-old girl is brought to the emergency department by her mother
because of irritability, restlessness, vomiting, and diarrhea for 3 hours. During this time, the
infant has had incessant crying and poor feeding. She is bottle-fed and has been well prior to this.
She is crying and in moderate respiratory distress. Her temperature is 38ºC (101ºF). Examination
shows grunting and retractions. There is decreased muscle tone. Head and neck examination
shows a full fontanelle and normal neck flexion. There is no rash. Lumbar puncture and
cerebrospinal fluid analysis show: 
Leukocyte count 1,200/mm3
Segmented neutrophils 70%
Lymphocytes 30%
Protein 120 mg/dL
Glucose 25 mg/dL
Gram stain Gram-positive rods
Serum glucose 60 mg/dL
Which of the following is the most likely cause of this patient's condition? 
 Incorrect Answer ImageA.Clostridium botulinum
 Incorrect Answer ImageB.Clostridium tetani
 Incorrect Answer ImageC.Escherichia coli
 Incorrect Answer ImageD.Haemophilus influenzae
 Correct Answer ImageE.Listeria monocytogenes
 Incorrect Answer ImageF.Streptococcus agalactiae
 Incorrect Answer ImageG.Streptococcus pneumoniae

A 15-year-old boy is brought to the physician by his parents because of concern about multiple
stretch marks that have developed on his chest, shoulders, and thighs over the previous 3 to 4
years. He plays center on his high school basketball team and is annoyed by the appearance of
the skin lesions. His medical history is remarkable for myopia; otherwise, he has been well. His
height is 190 cm (>97% for age), BMI is 17 kg/m2 (<10% for age), upper to lower segment ratio
is 0.88, and his arm span is 195 cm. He has multiple striae in the pectoral, deltoid, and thigh
regions. He has a slight protuberance of his sternum, positive thumb and wrist signs, and mild
kyphoscoliosis. Which of the following is the most likely cause of his condition? 
 Incorrect Answer ImageA.Accumulation of mineralized tissue in skin, retina, and blood
vessels
 Incorrect Answer ImageB.Cystathionine beta-synthase deficiency
 Correct Answer ImageC.Mutation in the gene encoding for fibrillin-1
 Incorrect Answer ImageD.Quantitative defect in fibrillar collagen
 Incorrect Answer ImageE.Structural or quantitative defect in type I collagen

A male infant who was born at 25 weeks’ gestation continues to be treated in the neonatal
intensive care unit (NICU) and is now at 36 weeks’ postmenstrual age. He is on nasal continuous
positive airway pressure (CPAP) of 7 cm H2O and an inspired oxygen concentration (FiO2) of
0.40. His temperature is 37ºC (98.6ºF), pulse is 120/minute, respirations are 30/min, and blood
pressure is 60/37 mm Hg. He has had poor weight gain on nasogastric feedings with provision of
increased calories. On examination, he has mild intercostal retractions, diffuse fine crackles over
his lung fields, increased anterior-posterior thoracic diameter and occasional scattered wheezes.
His most recent arterial blood gas (on the above CPAP and oxygen) and serum chemistries are:
pH 7.50
PCO2 55 mm Hg
PO2 58 mm Hg
Sodium 130 mEq/L 
Chloride 92 mEq/L
Potassium 3.1 mEq/L
Bicarbonate 35 mEq/L
Ionized calcium 3.5 mg/dL
Urinary chloride 12 mEq/L 
Which of the following is the explanation for these findings?
 Incorrect Answer ImageA.11-β hydroxylase deficiency
 Incorrect Answer ImageB.Bartter syndrome 

 Correct Answer ImageC.Chronic furosemide therapy


 Incorrect Answer ImageD.Chronic spironolactone therapy
 Incorrect Answer ImageE.Cystic fibrosis
 Incorrect Answer ImageF.Renin-secreting tumor

A 6-year-old boy is brought to the physician because of right knee pain and swelling. The child
is currently taking amoxicillin for his third episode of pneumonia during the past two months.
The mother also reports decreased appetite in the child, and states that he only "eats a little bit of
his food because he gets full very quickly". His temperature is 39.4ºC (101.1ºF), pulse is
120/min, and respirations are 20/min. The child appears acutely ill and pale. Cardiac examination
shows a systolic ejection murmur. There are rales over the right lung base. The liver and spleen
are palpated 3 and 2 cm below the right and left costal margins, respectively. There are petechiae
on the lower extremities. The patient is confused with no focal neurologic deficits. Laboratory
studies show: 
Hemoglobin  9 g/dL
Platelets  90,000/mm3
White blood cells  4,500/mm3
Peripheral smear Atypical lymphocytes
Bone marrow biopsy 30% blasts
Myeloperoxidase negative
The diagnosis is made and immediate treatment is initiated. Several weeks later the child arrives
at the emergency department with confusion and headache. Physical examination shows
papilledema. Which of the following would have prevented this child's new complication? 
 Incorrect Answer ImageA.Bone marrow transplantation

 Incorrect Answer ImageB.Combination of chemo and radiotherapy

 Incorrect Answer ImageC.High dose chemotherapy

 Incorrect Answer ImageD.High dose prednisone


 Correct Answer ImageE.Intrathecal methotrexate

 Incorrect Answer ImageF.Radiotherapy

A 5-year-old girl is brought to her pediatrician by her mother. She came in 5 days ago with what
was diagnosed as pharyngitis and treated with oral cefaclor, which she was given on two other
occasions in the past year. She now has urticarial-like plaques distributed diffusely across her
trunk and extremities, mild swelling and erythema of her hands, swelling of her knees, enlarged
anterior cervical and epitrochlear nodes, and an axillary temperature of 38.2ºC (100.8ºF). Which
of the following is the most likely cause of her presentation?
 Incorrect Answer ImageA.Acute annular urticaria
 Incorrect Answer ImageB.Connective tissue disease
 Incorrect Answer ImageC.Drug reaction with eosinophilia and systemic symptoms
(DRESS)
 Incorrect Answer ImageD.Kawasaki disease
 Correct Answer ImageE.Serum sickness-like reaction

A 4-year-old boy has a 5-day history of fever, sore throat, and increased irritability. His
temperature is 40.2ºC (104.3ºF), blood pressure is 98/68 mm Hg, pulse is 112/min, and
respirations are 24/min. On physical examination, he is noted to have an enlarged, mildly tender
right-sided cervical lymph node, an erythematous pharynx without exudate, and bilateral
conjunctival injection. Examination of his tongue shows hypertrophy of the papillae in a bright
red background. His palms and soles are erythematous. There is a polymorphous macular rash
generalized on his trunk. Which of the following is the most appropriate pharmacotherapy?
 Incorrect Answer ImageA.Amoxicillin
 Incorrect Answer ImageB.Aspirin and corticosteroid
 Correct Answer ImageC.Aspirin and IV immune globulin
 Incorrect Answer ImageD.Corticosteroid
 Incorrect Answer ImageE.IV immune globulin

A 6-year-old boy whose family recently immigrated to the United States from Cambodia is
brought to the university pediatric clinic because of a rash for several days. Through the
translator, the mother reports that the rash is not itchy. The boy was sick about 2 weeks ago and
was reluctant to eat or drink anything. Physical examination shows multiple large, erythematous,
serpiginous, macular lesions with pale centers most prominent on the boy's lower extremities and
trunk and less so on the arms. There are no lesions on the face. He also has a swollen, red, and
tender right knee. When questioned about this via the translator, the mother states that the knee
swelling is new, but about 6 days ago, his right wrist looked similar to this and was painful for a
few days, but then improved. In order to confirm the most likely diagnosis, which of the
following must also be present in this child?
 Incorrect Answer ImageA.Anemia
 Correct Answer ImageB.Elevated anti-DNase B titer
 Incorrect Answer ImageC.Elevated erythrocyte sedimentation rate
 Incorrect Answer ImageD.Fever
 Incorrect Answer ImageE.Prolonged PR interval
A 1-day-old neonate appears dusky in the newborn nursery during feeding. Oxygen is
immediately administered by nasal cannula. Shortly afterward, she develops tachypnea. Her
pulse is 180/min, respirations are 60/min, and blood pressure from the right upper arm is 50/30
mm Hg. An echocardiogram is consistent with hypoplastic left heart syndrome. Physical
examination is most likely to show which of the following sets of findings in this patient?
 Incorrect Answer ImageA.Continuous ductal murmur, bounding pulses
 Incorrect Answer ImageB.Continuous ductal murmur, poor peripheral pulses
 Incorrect Answer ImageC.Holosystolic murmur, poor peripheral pulses, quiet second
heart sound
 Incorrect Answer ImageD.No murmur, precordial hyperactivity, quiet second heart sound
 Correct Answer ImageE.Precordial hyperactivity, loud second heart sound, weak pulse

A 4-month-old boy is brought to the emergency department by his parents because of poor
feeding and irritability for the past 2 weeks. The mother states that he vomited several times in
the past few days, and she also noticed reduced activity in the child. He was born at 26 weeks'
gestation and developed seizures soon after birth. He is up to date on all vaccinations. Physical
examination shows a tense anterior fontanelle and visibly distended scalp veins. Both eyes
deviate downward and the upper lids are retracted. Which of the following is the most likely
underlying cause of this patient's clinical presentation? 
 Incorrect Answer ImageA.Arnold-Chiari malformation
 Correct Answer ImageB.Germinal matrix hemorrhage
 Incorrect Answer ImageC.Kernicterus
 Incorrect Answer ImageD.Periventricular leukomalacia
 Incorrect Answer ImageE.Ulegyria

A 12-year-old boy is brought to the physician for the first time for a school physical
examination. The boy's parents state that he has never had any significant illnesses, takes no
medications, and has no known allergies. The boy is in sixth grade and does very well in school.
He spends most of his free time reading and working on the computer. He and his family eat a lot
of fast food. Both parents appear to be overweight. On examination, the boy’s height is 147 cm
(4 ft 10 in) and his weight is 76.6 kg (169 lb). The pediatrician calculates the boy's body mass
index (BMI) to be greater than the 95th percentile for boys his age. Blood pressure in the left arm
while seated is 135/88 mm Hg. Blood pressure repeated later in the visit is the same. He has
appropriate blood pressures in his other three extremities. He has purple striae on his abdomen
and darker pigmented skin in his axillae, back of his neck, and inguinal regions. Which of the
following is the next step in the management of this patient?
 Incorrect Answer ImageA.Begin an oral angiotensin-converting enzyme inhibitor
 Incorrect Answer ImageB.Begin an oral diuretic
 Correct Answer ImageC.Obtain fasting plasma glucose
 Incorrect Answer ImageD.Obtain morning serum cortisol levels
 Incorrect Answer ImageE.Place the boy on a low-calorie diet with a monitored exercise
program
 Incorrect Answer ImageF.Reschedule for follow-up blood pressure checks on two
separate visits
A 5-year-old boy is brought to the physician because of difficulty in school. According to the
teacher, he often "blanks" out with tears in his eyes. This has been affecting his classroom
performance. The child is otherwise normal. His speech and comprehension are within normal
limits. He is able to play with friends without any obvious difficulties. There is no history of
convulsions, loss of consciousness, or bladder or bowel incontinence. He has been otherwise
healthy and is not on any medications. The physical examination is unremarkable. An EEG
shows a generalized 3-Hz spike and slow-wave discharges. Which of the following is the most
appropriate treatment?
 Incorrect Answer ImageA.Carbamazepine
 Correct Answer ImageB.Ethosuximide
 Incorrect Answer ImageC.Ethosuximide and valproic acid
 Incorrect Answer ImageD.Methylphenidate
 Incorrect Answer ImageE.Vigabatrin

An 18 month year-old girl is being evaluated following a nosebleed which was so severe as to
require nasal packing and transfusion of platelet concentrates. When a blood sample had been
drawn in the emergency room for serum chemistry studies, the local hospital laboratory had
noted that the clot that formed was unusual in that it failed to retract. Peripheral blood smear
obtained by finger puncture showed an appropriate number of normal-sized platelets, all of
which were individual, without clumping. The child's parents are cousins, but neither the parents
nor any of the grandparents have bleeding disorders. Special platelet studies show that the child's
platelets fail to aggregate with any physiologic aggregating agent, including a high concentration
of exogenous ADP. Which of the following is the most likely diagnosis?
 Incorrect Answer ImageA.Bernard-Soulier syndrome
 Incorrect Answer ImageB.Chediak-Higashi syndrome
 Correct Answer ImageC.Glanzmann thrombasthenia
 Incorrect Answer ImageD.May-Hegglin anomaly
 Incorrect Answer ImageE.Von Willebrand disease

A 1-year-old boy is brought to the physician by his father for a well-child visit. His father states
that he is worried that his son is smaller than he should be for his age. The child was born at
term, his birth weight was 3.4 kg (7.4 lb), and birth length was 50.8 cm (20 in). His weight and
height at this visit are 9.1 kg (22 lb) and 71.1 cm (28 in), respectively, which is at the
25th percentile. The father reports that the child is currently on a regular table food diet and has a
good appetite. He takes 24 ounces of a lactose-based, cow-milk protein formula per day. There is
nothing concerning in his interim history and his physical examination is unremarkable. Which
of the following is the most appropriate statement regarding this child’s growth?
 Incorrect Answer ImageA."He has a typical growth velocity, but he has pathological
short stature."
 Correct Answer ImageB."He is growing typically. His birth weight has approximately
tripled by 12 months and his height has increased an average amount from birth for a healthy
full-term infant."
 Incorrect Answer ImageC."He is not growing typically. His birth weight and height
should each quadruple by one year."
 Incorrect Answer ImageD."He is not growing typically. His height is within a typical
range for his age, but his weight/height ratio is less than the 5th percentile, so he is
malnourished."
 Incorrect Answer ImageE."He is not growing typically. His weight is within the typical
range for his age, but he has an abnormal growth velocity."

A 5-year-old girl is brought to the physician because of mild abdominal pain. The pain started in
the past week and is intermittent and diffuse. Urination and stooling have been normal. Her past
medical history is significant for the finding of a horseshoe kidney after she was diagnosed and
treated for a urinary tract infection at age 11 months. The child is well appearing. Her
temperature is 37.2ºC (98.0ºF), blood pressure is 130/80 mm Hg, pulse is 74/min, and
respirations are 16/min. Physical examination is significant for an 8-cm non-localized abdominal
mass. Laboratory studies show:
Alpha fetoprotein normal range
Hgb 9 g/dL with normocytic indices
Platelet count 490,000
U/A 30 RBCs/hpf
Urinary HVA and VMA negative
Which of the following is the most likely cause of these findings? 
 Incorrect Answer ImageA.Congenital mesoblastic nephroma
 Incorrect Answer ImageB.Hepatoblastoma
 Correct Answer ImageC.Nephroblastoma
 Incorrect Answer ImageD.Neuroblastoma
 Incorrect Answer ImageE.Renal cell carcinoma
 Incorrect Answer ImageF.Renal clear cell sarcoma

A 6-year-old girl is brought to the physician by her mother for a routine well-child examination.
Her mother reports that she has frequently missed school because of persistent rhinorrhea and
intermittent cough. She has also been snoring at night. She has no significant past medical
history and only takes over-the-counter cold remedies with minimal improvement in her
symptoms. Physical examination shows a horizontal crease over the nasal bridge and clear
rhinorrhea. The nasal mucosa appears boggy and turbinates are edematous with a bluish
discoloration. Which of the following is the most appropriate treatment for this patient?
 Incorrect Answer ImageA.Cromolyn nasal solution
 Incorrect Answer ImageB.Intranasal decongestants
 Correct Answer ImageC.Intranasal steroids
 Incorrect Answer ImageD.Oral antihistamines
 Incorrect Answer ImageE.Oral decongestants

A 3-year-old toilet trained girl is brought to the physician because of increased urinary
frequency, dysuria, and itching on urination. Her urinalysis is consistent with a urinary tract
infection. Last year, she was diagnosed with vesicoureteral reflux, Grade 2. This is her second
urinary tract infection in the past year, despite adequate antibiotic coverage. After treating the
current infection, which of the following is the most appropriate next step in management?
 Incorrect Answer ImageA.Antireflux surgery
 Incorrect Answer ImageB.Discontinue the prophylactic antibiotic regimen
 Incorrect Answer ImageC.Increase the dose of antibiotics
 Incorrect Answer ImageD.Prescribe an anticholinergic
 Correct Answer ImageE.Start a trial of timed voiding

A 12-year-old girl is brought to the physician because of frequent episodes of shortness of breath
and wheezing. The patient first developed these episodes last spring after she moved with her
family from the Midwest to Florida. The patient is otherwise healthy and she feels fine between
these episodes. During one previous episode, she was brought to the emergency department and
was successfully treated with albuterol. Currently, she does not take any medications. The
patient’s mother said that her daughter has an allergy to cats. Her temperature is 36.6ºC (98.2ºF),
pulse is 108/min, respirations are 24/min, and blood pressure is 120/65 mm Hg. Physical
examination of the lungs shows diffuse wheezing bilaterally. The rest of her physical
examination is unremarkable. Atopic asthma is suspected. Which of the following diagnostic
studies is necessary to establish a diagnosis of asthma in this case?
 Incorrect Answer ImageA.Histamine challenge test
 Incorrect Answer ImageB.Methacholine challenge test
 Correct Answer ImageC.No test is necessary
 Incorrect Answer ImageD.Pulmonary function testing with bronchodilators
 Incorrect Answer ImageE.Thermal challenge test

A 3-year-old girl is being evaluated for developmental delay. Her parents report that the
pregnancy was uneventful and that the delivery was without complications. The girl reached all
milestones typically during her first year. Length, weight, and head circumference were all well
within the normal range during the first year. During her second year, the pediatrician noticed
deceleration of head growth, and her parents noticed some odd hand-wringing movements, and
social deficits such as disinterest in play, and lack of eye contact. Her movements became poorly
coordinated and she showed periods of rapid breathing. Which of the following statements is true
about this patient's condition?
 Incorrect Answer ImageA.It has an autosomal recessive pattern of inheritance
 Incorrect Answer ImageB.It is associated with a known enzyme deficiency
 Incorrect Answer ImageC.It is associated with average intelligence in adulthood
 Incorrect Answer ImageD.It is associated with a worsening of social deficits in adulthood
 Correct Answer ImageE.It is seen almost exclusively in females

A 4-year-old boy falls from the jungle gym at preschool. He sustains minor abrasions and
contusions and is taken care of by the school nurse. His parents take him that same afternoon to
his regular pediatrician and demand "a thorough check-up" for possible internal injuries. The
pediatrician complies, and a complete physical examination is normal. His hemoglobin is 14
g/dL, and a urinalysis shows the presence of microhematuria. There are no additional signs of
bleeding. Which of the following is the most appropriate next step in management? 
 Incorrect Answer ImageA.Cystoscopy
 Incorrect Answer ImageB.Retrograde urethrogram
 Incorrect Answer ImageC.Spiral CT of the pelvis and abdomen
 Correct Answer ImageD.Ultrasound of kidneys, ureter, and bladder
 Incorrect Answer ImageE.Voiding cystourethrogram (VCUG)
A 32-year-old woman, gravida 4, para 3, at 39 weeks' gestation is brought to the emergency
department because of painful contractions. Her prenatal course has been unremarkable. Physical
examination shows a 5-cm dilated cervix, 100% effacement, and the fetal heart rate is in the 130s
and reactive. Meperidine is given for pain control. Two hours later, a 3,345g (7 lb, 6 oz) infant is
delivered. The Apgar score is 3 at 1 minute and the infant is making little respiratory effort.
Which of the following is the next immediate step in management?
 Incorrect Answer ImageA.Administer epinephrine
 Incorrect Answer ImageB.Administer naloxone
 Correct Answer ImageC.Bag and mask ventilation
 Incorrect Answer ImageD.Establish an umbilical vein line
 Incorrect Answer ImageE.Start chest compressions

A 2-day-old boy born at 34 weeks gestation is transferred to the neonatal intensive care unit
because of respiratory distress, increased irritability, and poor feeding. His mother received
routine prenatal care and tested negative for group B streptococcus. His temperature is 37.0°C
(98.6°F), pulse is 160/min, respirations are 60/min, blood pressure is 72/40 mm Hg, and oxygen
saturation 78% on room air. Physical examination shows central cyanosis. Lungs are clear. Heart
rate is regular with a grade 3/6 holosystolic murmur best heard at the lower left sternal border
and radiating upward. The second heart sound is single. Abdominal examination shows a non-
distended, non-tender abdomen without hepatosplenomegaly. Meconium was passed on the first
day of life. An x-ray of the chest shows a narrow, straight mediastinum, a normal-sized
unusually-shaped heart, and slightly increased pulmonary vasculature. No consolidations are
noted. Laboratory studies are shown:
White blood cells 5,600/mm3
Hematocrit 45%
Platelets 235,000/mm3
Oxygen (100%) by face mask is started but fails to improve oxygenation. Which of the following
is most likely to be seen on echocardiogram? 
 Incorrect Answer ImageA.Aortic stenosis
 Incorrect Answer ImageB.Coarctation of the aorta
 Incorrect Answer ImageC.Ebstein anomaly
 Correct Answer ImageD.Transposition of the great arteries
 Incorrect Answer ImageE.Truncus arteriosus

An 8-year-old boy is brought to the physician for a routine health maintenance examination. His
mother states that he has had difficulty reading and concentrating in his second-grade class.
Physical examination shows 7 hyperpigmented cutaneous lesions ranging from light to dark
brown on the body, as well as 2 small, rubbery, rough masses above the orbit that have a slightly
darker pigmentation than the surrounding skin and have distinct borders that are greater than 5
mm in diameter. There are also numerous 2-3 mm hyperpigmented areas in the axillary and groin
areas. His past medical history is significant for two seizures 4 and 6 months ago. His mother has
lesions similar to the ones that he has. Which of the following is the most likely complication of
the underlying condition? 
 Correct Answer ImageA.Bony dysplasia
 Incorrect Answer ImageB.Cardiac rhabdomyoma
 Incorrect Answer ImageC.Cerebral calcification
 Incorrect Answer ImageD.Diabetes mellitus
 Incorrect Answer ImageE.Malignant melanoma

An 8-year-old boy is brought to the physician by his mother because of progressive limping for
several weeks. The mother states that her son was very active before this and spent all of his free
time outdoors playing soccer or football with his friends from the neighborhood. Then, about 3
weeks earlier, he developed a limp in his right leg that has progressively gotten worse. His
parents had first thought he had strained a muscle in one of his activities, but it seemed to be
worsening as time went by. Over the last week the boy has started complaining of mild pain in
his groin and hip that radiates to the thigh. It is exacerbated with activity and relieved by rest. On
physical examination, the boy has an obvious limp of the right leg and complains of pain with
active range of motion. A hip radiograph shows a widened articular space and abnormalities in
the radiopacity of the femoral head. Which of the following is the most likely diagnosis? 
 Incorrect Answer ImageA.Acute slipped capital femoral epiphysis
 Incorrect Answer ImageB.Chronic slipped capital femoral epiphysis
 Incorrect Answer ImageC.Coxarthrosis
 Incorrect Answer ImageD.Developmental dysplasia of the hip
 Correct Answer ImageE.Legg-Calvé-Perthes disease

A 16-year-old girl comes to the emergency department because of severe right-sided abdominal
pain. Her last menstrual period was 2 weeks ago and lasted 4 days. She appears to be in moderate
distress. Her temperature is 38.5ºC (101.3ºF), blood pressure is 128/75 mm Hg, pulse is 80/min,
and respirations are 22/min. The abdomen is diffusely tender, especially the lower abdomen.
Rectal examination shows guaiac-negative brown stool and exquisite tenderness of the lateral
wall of the rectum. Pelvic examination shows cervical tenderness and fullness of the right
adnexa. A urine pregnancy test is negative. Which of the following is the most likely diagnosis?
 Incorrect Answer ImageA.Ectopic pregnancy
 Incorrect Answer ImageB.Endometriosis
 Incorrect Answer ImageC.Ovarian cyst
 Correct Answer ImageD.Pelvic inflammatory disease
 Incorrect Answer ImageE.Pyelonephritis

A 3-year-old boy is brought to the physician by his mother because of refusal to walk for 2 days.
He had fever and ear pain 10 days ago. He was prescribed amoxicillin suspension but refused to
take it because of its bad taste. The ear pain resolved after 4 days without taking the amoxicillin.
He appears in mild distress. His temperature is 37°C (98.6°F), blood pressure is 105/70 mm Hg,
and pulse is 90/min. He refuses to stand on his right leg because of pain. Examination shows
swelling over the anterior right thigh, and there is tenderness to palpation of the femur. His
leukocyte count is 15,000/mm3. An x-ray film of the right leg shows soft-tissue swelling in the
middle third of the right leg. Which of the following is the most likely cause of this patient's
condition? 
 Incorrect Answer ImageA.Group B streptococci
 Incorrect Answer ImageB.Pasteurella multocida
 Incorrect Answer ImageC.Pseudomonas aeruginosa
 Incorrect Answer ImageD.Salmonella
 Correct Answer ImageE.Staphylococcus aureus
An 11-day-old infant is brought to the physician because of a 1-day history of fever and poor
feeding. He was born at term via vaginal delivery to a 24-year-old woman, gravida 1, para 0,
who was GBS-negative at the time of delivery. His temperature is 38.2ºC (100.8ºF). Lumbar
puncture and cerebrospinal fluid analysis show:
Leukocytes  6,000/mm3
Neutrophils  68% 
Lymphocytes  30% 
Bands  2% 
Protein  200 mg/dL 
Glucose  35 mg/dL 
An MRI scan of the brain is shown. Which of the following is the most likely cause of these
findings?
 Correct Answer ImageA.Citrobacter koseri
 Incorrect Answer ImageB.Escherichia coli
 Incorrect Answer ImageC.Klebsiella pneumoniae
 Incorrect Answer ImageD.Listeria monocytogenes
 Incorrect Answer ImageE.Streptococcus agalactiae

An 8-day-old boy is brought to the emergency department by his parents because of a left-sided
neck mass. The child was born at 39 weeks' gestation by way of spontaneous vaginal delivery
after an uncomplicated pregnancy. The parents say they did not notice the mass before his
discharge from the hospital. They report that he has been tolerating his feedings without
problems and has had no episodes of respiratory distress. He is afebrile. He does not appear to be
in acute respiratory distress. His head appears to stay rotated to the right at rest. Palpation of the
neck shows a firm, nontender, well-circumscribed mass. A complete blood count shows no
abnormalities. An MRI scan of the brain shows a dense, nonenhancing enlargement within the
body of the left sternocleidomastoid muscle. Which of the following is the most appropriate next
step in management?
 Correct Answer ImageA.Conservative management for likely congenital torticollis
 Incorrect Answer ImageB.Excision of the enlarged lymph node for cure of mycobacterial
lymphadenitis
 Incorrect Answer ImageC.Incision and drainage of left neck branchial cyst
 Incorrect Answer ImageD.Initiation of chemotherapy for treatment of a
rhabdomyosarcoma
 Incorrect Answer ImageE.Treat the infectious condition with intravenous antibiotics

Two worried parents rush into the emergency department carrying their 3-year-old daughter.
They tell the physician that she ingested multiple acetaminophen pills less than an hour ago
while sitting at a desk and putting the tablets in her mouth. The mother immediately removed the
pills that were still in the child's mouth and then rushed her to the hospital. On the way, she
started vomiting. The mother does not know how many pills her daughter took because the bottle
was not full, but she saw her put at least 4 tablets in her mouth and believes she may have
ingested up to 10 pills . She removes the bottle from her pocket and spills the contents on the
counter. There are a total of 36 out of an original 100 of 325 mg acetaminophen pills. The child
is crying on the examining table. She appears frightened and diaphoretic, and repeats several
times that she wants to throw up. Her pulse is 120/min and respirations 24/min. Which of the
following is the most appropriate next step in the management?
 Correct Answer ImageA.Activated charcoal and measurement of plasma concentration of
acetaminophen 3 hours later
 Incorrect Answer ImageB.Dextrose and naloxone intravenously
 Incorrect Answer ImageC.Measurement of plasma concentration of acetaminophen
 Incorrect Answer ImageD.N-acetylcysteine intravenously
 Incorrect Answer ImageE.N-acetylcysteine orally

A 12-year-old girl with a history of asthma is brought to the emergency department by her
parents because of wheezing and difficulty breathing for the past 4 hours. Her parents report that
she has been using an albuterol inhaler more frequently without any improvement in her
symptoms. She has nocturnal symptoms twice weekly and has been getting short of breath easily.
She has never been intubated but has been brought to the emergency department 3 times in the
past 12 months for acute asthma exacerbations. In the emergency department, she is treated for
her respiratory distress and she responds to therapy. The patient is ready for discharge. Which of
the following is the most appropriate next step in pharmacotherapy?
 Correct Answer ImageA.Begin a daily low-dose inhaled corticosteroid and continue the
short-acting beta agonist as needed for exacerbations
 Incorrect Answer ImageB.Begin nebulized cromolyn sodium two times daily and
continue the short-acting beta agonist as needed for exacerbations
 Incorrect Answer ImageC.Change from a short-acting beta agonist to a combination
inhaler of a beta agonist and an anticholinergic agent to be used as needed for exacerbations
 Incorrect Answer ImageD.Continue the same dose of the beta agonist as needed for
exacerbations but add monthly injections of omalizumab
 Incorrect Answer ImageE.Increase the number of actuations of the beta agonist inhaler
for each use during an exacerbation

A 4-year-old girl is brought to the physician because of a 4-month history of limping, swelling of
the left knee, and pain in the right ankle. Her parents note that she has difficulty extending her
left knee fully. There is no history of trauma. She has trouble walking in the morning due to
stiffness and complains of pain later in the day. Physical examination is normal, except for a left
knee with joint swelling and limitation of flexion. She complains of mild discomfort with passive
motion of the right ankle. Her antinuclear antibody serology is 1:120. Which of the following is
the most appropriate screening test to prevent morbidity from disease? 
 Incorrect Answer ImageA.Bone scan
 Incorrect Answer ImageB.Complete blood count with differential and platelet count
 Incorrect Answer ImageC.Erythrocyte sedimentation rate
 Incorrect Answer ImageD.MRI of the knee
 Correct Answer ImageE.Slit-lamp examination
 Incorrect Answer ImageF.Urinalysis

A 10-year-old boy is brought to the emergency department because of a severe headache,


vomiting, and photophobia for 7 hours. His pediatrician had given him an intramuscular injection
of ceftriaxone for the past 2 days. He is in moderate distress. His temperature is 39.5ºC
(103.1ºF), pulse is 105/min, and respirations are 28/min. There is nuchal rigidity. Flexion of the
neck while supine produces involuntary flexion of the knee and hips, and there is pain on
extension of the leg while the hip is flexed at 90º. Of note, his immunizations are up to date. A
lumbar puncture is performed and shows cell count of 2,100/mm3 of which 78% are neutrophils,
15% lymphocytes, and 7% monocytes; glucose of 35 mg/dL (with a serum glucose of 60 mg/dl);
protein 140 mg/dl; and Gram stain and culture are negative. Latex agglutination is negative.
Which of the following is the most appropriate pharmacotherapy?
 Incorrect Answer ImageA.Intravenous ampicillin and gentamicin
 Incorrect Answer ImageB.Intravenous ceftazidime and tobramycin
 Incorrect Answer ImageC.Intravenous trimethoprim-sulfamethoxazole
 Correct Answer ImageD.Intravenous vancomycin and cefotaxime
 Incorrect Answer ImageE.Intravenous vancomycin, ceftriaxone, and ampicillin

A 3-week-old girl is brought to the physician by her parents because of problems with
constipation. The mother states that the infant can go up to 4 days without a bowel movement,
and when she does go, the stool is very hard. Prenatal care was appropriate and the infant was
born at 39 weeks' gestation without any difficulties except for a delayed first bowel movement 2
days after birth. Otherwise, her first 3 weeks of life have been uneventful. Examination shows a
distended, soft, and nontender abdomen. Immediately after the rectal examination, the infant
passes an explosive bowel movement with flatus. After the bowel movement, her abdomen is
much less distended. Which of the following tests will confirm the diagnosis? 
 Incorrect Answer ImageA.Anorectal manometry
 Incorrect Answer ImageB.Barium enema
 Correct Answer ImageC.Full-thickness biopsy of the rectal mucosa
 Incorrect Answer ImageD.Plain films of the abdomen
 Incorrect Answer ImageE.Prescribe laxatives and dietary change
 Incorrect Answer ImageF.Sweat chloride test

A 6-week-old boy is brought to the physician by his parents because of a 5-day history of a red
rash in the diaper area. He has had no fever, lethargy, or difficulty with feeding. His medical
history is unremarkable. His temperature is 37.1°C (98.8°F) and pulse is 110/min. Examination
shows erythematous, scaly patches on the buttocks, perineum, and lower abdomen. The skin
creases are normal-appearing. The lungs are clear to auscultation. Cardiac examination shows a
normal S1 and S2, and no murmurs are heard. Which of the following is the most likely
diagnosis?
 Incorrect Answer ImageA.Candidiasis
 Correct Answer ImageB.Irritant contact dermatitis
 Incorrect Answer ImageC.Langerhans cell histiocytosis
 Incorrect Answer ImageD.Psoriasis
 Incorrect Answer ImageE.Seborrheic dermatitis

A 4-week-old infant is brought to the emergency department with tachycardia, tachypnea, and
poor weight gain. His arterial blood gas shows a pH of 7.34, a PaCO2 of 41 mm Hg, and a
PaO2 of 74 mm Hg. A chest radiograph shows cardiomegaly. Echocardiography shows a
structurally normal heart, left ventricular dilatation, a left ventricular ejection fraction of 20%,
and mild mitral and tricuspid regurgitation. Intravenous administration of which of the following
medications is the most appropriate initial step in the management of this patient?
 Incorrect Answer ImageA.Angiotensin-converting enzyme inhibitor
 Incorrect Answer ImageB.Corticosteroid
 Incorrect Answer ImageC.Digoxin
 Incorrect Answer ImageD.Epinephrine
 Correct Answer ImageE.Furosemide

A newborn girl experienced a seizure in the newborn nursery and was immediately transferred to
the neonatal intensive care unit (NICU). The seizure stopped spontaneously after 45 seconds;
however, the baby had a subsequent seizure shortly after NICU admission. This second seizure
was terminated with intravenous diazepam. The mother had an uneventful pregnancy except for
a urinary tract infection that was treated during the 30th week of gestation. The prenatal and
labor-and-delivery records show that a pediatrician was called to the delivery because of severe
late decelerations and a cord pH of 6.9. The baby responded to bag-and-mask ventilation with
100% oxygen. Apgar scores were 1 and 6 at 1 and 5 minutes, respectively. Because the baby
appeared to be in good condition at 15 minutes, she was sent to the newborn nursery. Which of
the following is the most likely cause of the seizures?
 Incorrect Answer ImageA.Cerebral malformation
 Incorrect Answer ImageB.Hypoglycemia
 Correct Answer ImageC.Hypoxic-ischemic encephalopathy
 Incorrect Answer ImageD.Organic acidemia
 Incorrect Answer ImageE.Sepsis

A 14-year-old boy is being seen by his pediatrician because of breast enlargement and nipple
discharge. His mother states that over the past few months he has developed an asymmetric
breast enlargement, which started on the left and then on the right side. He has mild tenderness
on both sides. His review of systems is unremarkable, except for occasional headaches that are
usually relieved with ibuprofen, although at times he needs to lie down. He takes no medications,
drugs, or herbal preparations. The mother states that as an adolescent, her brother also had breast
enlargement that was symmetric and he had no discharge. On examination, the boy is at the
sexual maturity rating stage 4 of development. He does have bilateral breast tissue present that is
about 3 cm in diameter and concentric with respect to the areola. The left side is slightly larger
than the right and both are mildly tender to palpation. There are no overlying skin abnormalities,
fixation, or regional lymphadenopathy. In addition, a small milky discharge is present bilaterally.
Testicular examination is normal and appropriate for the sexual maturity rating. The thyroid
gland is not palpable, there are no abdominal masses and no hepatosplenomegaly. The remainder
of his examination is unremarkable. Which of the following is the next best step in management?
 Correct Answer ImageA.Obtain a serum prolactin level
 Incorrect Answer ImageB.Order a testicular ultrasound
 Incorrect Answer ImageC.Reassure the patient and his mother that this is physiologic
pubertal gynecomastia and no further workup or therapy is required
 Incorrect Answer ImageD.Refer to a surgeon for endoscopic transaxial removal of
glandular tissue
 Incorrect Answer ImageE.Search for an exogenous source of estrogens
 Incorrect Answer ImageF.Start treatment with an anti-estrogen drug
A 2-day-old newborn girl who was born at 38 weeks' gestation is brought to the emergency
department because of progressively worsening lethargy and poor feeding. In the emergency
department, the newborn is in severe respiratory distress and appears cyanotic. Physical
examination shows grunting with nasal flaring and tachypnea. Normal S1 and S2 with a regular
rate and rhythm are heard upon auscultation of the chest. Lung sounds are also normal. The
newborn's breathing and color improve when she cries but deep inspirations are ineffective.
Direct laryngoscopy is performed and shows no abnormalities. Which of the following will most
likely confirm the diagnosis?
 Correct Answer ImageA.CT scan with contrast of the head
 Incorrect Answer ImageB.Insert a catheter through the nose
 Incorrect Answer ImageC.Rhinography
 Incorrect Answer ImageD.Obtain an arterial blood gas
 Incorrect Answer ImageE.Obtain an echocardiogram

An 8-year-old girl is brought to the physician by her mother because of patchy hair loss and
scalp itching for 3 weeks. The mother reports that the girl has had extra dandruff for the past 2
months, followed by hair thinning in patches. Symptoms did not improve with anti-dandruff
shampoo. She has had no fever or weight loss. Her medical history is unremarkable.
Examination shows multiple circular patches with hairs broken off at the level of the scalp. There
is scaling at the edge of the patches. The occipital lymph nodes are enlarged and tender. The
remainder of the examination shows no abnormalities. Which of the following is the most likely
diagnosis? 
 Incorrect Answer ImageA.Psoriasis
 Incorrect Answer ImageB.Seborrheic dermatitis
 Incorrect Answer ImageC.Telogen effluvium
 Correct Answer ImageD.Tinea capitis
 Incorrect Answer ImageE.Trichotillomania

A 4-year-old boy is brought to the physician because of acute knee pain and swelling after he hit
his knee on the edge of a banister. There was no immediate pain, but swelling was present. He
denies fever, malaise, or myalgias. Past medical history is unremarkable, and he takes no
medications. On physical examination, there is a large left-knee joint effusion. There is no laxity
with passive movement, nor does flexion or extension obviously exacerbate the pain.
Arthrocentesis shows frank blood. Laboratory studies show:
White blood cells 5,600/mm3
Hematocrit 41%
Platelets 198,000/mm3
INR 1.0 (normal <1.1)
Partial thromboplastin time 60 seconds (normal 25–35)
Bleeding time 4 minutes (normal 2–7 minutes)
Factor VIII level 0.05 IU/mL (normal: >0.3 IU/mL)
Which of the following is the most appropriate diagnostic test?
 Incorrect Answer ImageA.Bone marrow biopsy
 Incorrect Answer ImageB.No further testing is necessary
 Incorrect Answer ImageC.Peripheral blood smear
 Correct Answer ImageD.Plasma mixing study
 Incorrect Answer ImageE.Ristocetin cofactor activity

A 6-month-old girl has had multiple infections, including thrush, CMV, and pneumonia, as well
as persistent diarrhea since 3 weeks of age. She has had all of the recommended immunizations
for her age. Her brother died of pneumonia at age 2 years. On examination, there is no evidence
of any lymph nodes or tonsils. Laboratory studies show a leukocyte count of 500/mm3. T-
lymphocytes, B-lymphocytes, and natural killer cells are markedly decreased. Radiography
shows flaring of the anterior ribs and dysplasia of the costochondral junctions and vertebral
bodies. Which of the following is the most likely diagnosis?
 Correct Answer ImageA.Adenosine deaminase deficiency
 Incorrect Answer ImageB.Bruton agammaglobulinemia
 Incorrect Answer ImageC.DiGeorge syndrome
 Incorrect Answer ImageD.Hyper IgM syndrome
 Incorrect Answer ImageE.X-linked severe combined immunodeficiency

A 5-year-old boy is brought to the physician because his parents have noticed swelling around
his eyes for the past 3 days. The child was previously well and was last seen for an annual well-
child visit 1 month ago when his weight was 18.5 kg (41 lb). He has been urinating less than
usual, but he is otherwise well. No change in the urine color was noticed and there was no fever,
diarrhea, or vomiting. Examination shows an alert and active child with a blood pressure of
98/66 mm Hg. There is periorbital edema as well as swelling of the ankles. Lung and abdominal
examinations are normal and there is no skin rash. Laboratory studies show:
Hemoglobin 13.2 mg/dL
Serum:
Bilirubin (total) 0.8 mg/dL
Bilirubin (direct) 0.3 mg/dL
Creatinine 0.5 mg/dL
Total cholesterol 250 mg/dL
Urea nitrogen 10 mg/dL
Urine:
Protein 4+
RBC 1–2/hpf
WBC 1/hpf
Which of the following is the most appropriate next step in management?
 Incorrect Answer ImageA.Albumin infusion
 Incorrect Answer ImageB.Cyclophosphamide
 Incorrect Answer ImageC.Furosemide
 Incorrect Answer ImageD.High-protein diet
 Correct Answer ImageE.Prednisone

A 2-year-old girl is seen by her pediatrician for fever of 101.4º F and “fussiness” for the past two
days. Her mother noticed nausea, two vomiting episodes and crying during urination the past two
days. The child was born 2 weeks premature, is up to date with her immunizations, and has
reached all her developmental milestones. On physical examination, her temperature is 39.0ºC
(102.3ºF), blood pressure is 101/60 mm Hg, and pulse is 110/min. Physical examination is
unremarkable. She has no rash, petechiae, or edema. Her abdominal examination is normal. She
cries when her left flank is percussed. Laboratory studies show:
White blood cells 14,000/mm3
Hematocrit 42%
Urinalysis 20-25 white blood cells/high power field, many
bacteria 
The child is admitted to the hospital for intravenous hydration and antibiotics. Which of the
following is the next step in the management of this patient? 
 Incorrect Answer ImageA.Immunodeficiency workup
 Incorrect Answer ImageB.Intravenous pyelogram
 Incorrect Answer ImageC.Nephrostomy tube placement
 Incorrect Answer ImageD.Renal scintigraphy
 Correct Answer ImageE.Renal ultrasound
 Incorrect Answer ImageF.Voiding cystourethrogram

A 28-year-old primigravid woman at 39 weeks' gestation comes to the emergency department


after a spontaneous rupture of membranes. The fluid is found to be nitrazine positive. She has
had a routine prenatal course and is VDRL negative, hepatitis negative, rubella immune, and
with unknown group B streptococci status. A 3.2 kg (7 lb) boy is born and Apgar scores are 8
and 9 at 1 and 5 minutes, respectively. An hour later the newborn's temperature is 37ºC (98.6ºF),
blood pressure is 60/35 mm Hg, pulse is 160/min, and respirations are 80/min. Physical
examination shows nasal flaring and expiratory grunting. Breath sounds are equal and clear
bilaterally. Mild to moderate chest retractions are also noted. Chest x-ray shows hyperinflation,
perihilar linear densities, and interfissural fluid without evidence of consolidation. Arterial blood
gas shows pH 7.35, PCO2 55, PO2 70, and oxygen saturation 85% on room air. Low-flow
supplemental oxygen is started with marked improvement of the newborn's symptoms after 12
hours. Which of the following is the most likely cause of this infant's respiratory distress?
 Incorrect Answer ImageA.Air leak syndrome
 Incorrect Answer ImageB.Aspiration of meconium
 Incorrect Answer ImageC.Bacterial infection of the lung parenchyma
 Incorrect Answer ImageD.Congenital cardiac malformation
 Correct Answer ImageE.Delayed resorption of fetal lung fluid
 Incorrect Answer ImageF.Relative deficiency of surfactant

A 16-year-old boy is brought to the physician for a routine examination. He has no history of any
major medical illnesses and takes no medications. To view the physical examination, click on the
"Play Media" button. The remainder of the physical examination shows no abnormalities. Which
of the following is the most appropriate next step in patient care? 
 Incorrect Answer ImageA.Aspirin
 Incorrect Answer ImageB.Chest x-ray
 Incorrect Answer ImageC.Echocardiography
 Incorrect Answer ImageD.Electrocardiogram
 Correct Answer ImageE.Reassurance

A newborn is transferred to the neonatal intensive care unit for observation and evaluation
because of the need for resuscitation in the delivery room. On physical examination, the baby has
low-set ears, a small mandible, a short philtrum, and a depressed nasal bridge. There is also a
cleft palate present. On cardiac examination, there is a loud, single second heart sound and a
grade IV systolic ejection murmur along the left sternal border with a loud, blowing diastolic
murmur. A chest radiograph shows a right-sided aortic arch, a large heart, and pulmonary edema.
The thymus is not seen. Which of the following would confirm the underlying cause of the
cardiac defect? 
 Incorrect Answer ImageA.Cardiac catheterization
 Incorrect Answer ImageB.Echocardiography
 Incorrect Answer ImageC.Karyotype
 Correct Answer ImageD.Polymerase chain reaction-based genotyping
 Incorrect Answer ImageE.Serum calcium level

A 13-year-old boy is brought to the physician by his parents because of limping and persistent
knee pain for several weeks. Since the age of 3 he has had a known diagnosis of moderate
asthma, treated chronically with inhaled steroids and salmeterol. He has been taking oral steroids
for the last 10 days for a recent asthma attack. He sits on the examining table with the sole of the
foot on the affected side pointing to the other leg. Physical examination is normal for the knee
but shows limited hip motion. As the affected hip is flexed, the leg goes into external rotation
and cannot be rotated internally. His pulse is 72/min, respirations are 17/min, blood pressure is
132/85 mm Hg, and BMI is 31 kg/m2. Which of the following is the most likely diagnosis?
 Incorrect Answer ImageA.Corticosteroid-induced avascular necrosis of the femoral head
 Incorrect Answer ImageB.Legg-Calve-Perthes disease
 Incorrect Answer ImageC.Osteogenic sarcoma of the lower femur
 Correct Answer ImageD.Slipped capital femoral epiphysis
 Incorrect Answer ImageE.Tibial torsion with foot inversion

A 16-year-old girl is brought to the emergency department by her parents because of a


progressive itchy rash and swollen lips for 3 hours since waking up in the morning. She injured
her ankle 1 day ago while kickboxing. Last night she took 2 ibuprofen tablets for pain before
going to bed. She denies difficulty breathing, abdominal pain, and any change in bowel
movements. Her medical history is unremarkable. She takes no medications. Her blood pressure
is 115/70 mm Hg and pulse is 82/min. Examination shows mild edema of the upper lip and
multiple 0.5 cm to 5 cm raised, erythematous patches with a pale center on her trunk and
extremities. Bilateral wheezes are heard on auscultation of the lungs. Which of the following is
the most likely diagnosis?
 Incorrect Answer ImageA.Erythema multiforme
 Incorrect Answer ImageB.Exercise-induced anaphylaxis
 Correct Answer ImageC.Urticaria
 Incorrect Answer ImageD.Urticaria pigmentosa
 Incorrect Answer ImageE.Urticarial vasculitis

A 16-year-old girl is brought to the emergency department by ambulance after she was extracted
from a burning vehicle involved in an accident on the local highway. She was the only person in
her car, which burst into flames after it was rear-ended at high speed. She swerved off the road
and collided with the side railing. When the ambulance arrived, she was unconscious in the
driver's seat. On arrival at the emergency department, the patient is in moderate respiratory
distress. Her pulse is 120/min, respirations are 30/min, and blood pressure is 80/40 mm Hg. After
securing the patient's airway and administering oxygen and intravenous fluids, the physician
evaluates the extent of the sustained burn injury. There is diffuse erythema and edema of the
patient's face, and most of her scalp hair is scorched with some blistering of the underlying skin.
Both arms show diffuse erythema, edema, and areas of extensive blistering. The rest of her body
shows no significant burns. Which of the following is the estimated body surface area of the
burn?
 Incorrect Answer ImageA.9%
 Incorrect Answer ImageB.18%
 Correct Answer ImageC.27%
 Incorrect Answer ImageD.36%
 Incorrect Answer ImageE.45%

A term newborn is delivered vaginally following a breech presentation. The pediatric orthopedic
surgeon is called to the newborn nursery because on physical examination, the Barlow test is
positive for bilateral subluxation of the hips. There is decreased abduction of both hips. Aside
from breech presentation, which of the following infants are most at risk for this condition? 
 Incorrect Answer ImageA.African American infants
 Correct Answer ImageB.Female infants
 Incorrect Answer ImageC.Infants of mothers with preeclampsia
 Incorrect Answer ImageD.Premature infants
 Incorrect Answer ImageE.Second-born infants

A 5-year-old boy is brought to the emergency department because of a 2-day history of fever,
anorexia, loose stools, and yellow skin color. He attends a large daycare center. His temperature
is 38.1ºC (100.7ºF), blood pressure is 88/56 mm Hg, pulse is 74/min, and respirations are
15/min. Initial laboratory evaluation shows a total bilirubin of 1.8 mg/dL and alanine
aminotransferase of 764 U/L. Neither of his parents nor any of his siblings are sick. Which of the
following is the most appropriate next step in diagnosis?
 Incorrect Answer ImageA.Hepatitis B surface antigen in serum
 Incorrect Answer ImageB.IgG for hepatitis A in serum
 Incorrect Answer ImageC.IgG for hepatitis B surface antigen in serum
 Correct Answer ImageD.IgM for hepatitis A in serum
 Incorrect Answer ImageE.Stool culture for hepatitis A

A 6-year-old boy is brought to the emergency department because of a 1-day history of a painful
and swollen right arm. He was bitten and scratched on the right arm by the family's cat 2 days
ago. His medical history is unremarkable. He has no allergies to medications. His temperature is
39.6ºC (103.2ºF). Examination of the right forearm shows two puncture wounds and one
laceration. The right forearm is erythematous, edematous, and tender. Which of the following is
the most appropriate pharmacotherapy? 
 Correct Answer ImageA.Amoxicillin-clavulanate
 Incorrect Answer ImageB.Ampicillin
 Incorrect Answer ImageC.Clindamycin
 Incorrect Answer ImageD.Doxycycline
 Incorrect Answer ImageE.Trimethoprim-sulfamethoxazole
An 8-year-old boy is brought to the emergency department with decreased mental status. His
mother states that he has been drinking and urinating more frequently over the past several
weeks. He was hard to wake up this morning and reported abdominal pain. Physical examination
shows an afebrile drowsy male with tachypnea and mild tachycardia. Mucous membranes are dry
and the lips are cracked. The abdomen is mildly tender to palpation diffusely but there is no
rebound or guarding. Laboratory studies show a glucose level of 560 mg/dL and a potassium of
4.9 mEq/L. Arterial blood gas is pH 7.18. Urinalysis is positive for ketones and glucose. Chest x-
ray is normal. Two hours after the initiation of treatment, the physician adds potassium to the
patient's IV fluids. Which of the following best explains the physician's therapeutic decision?
 Incorrect Answer ImageA.Acidosis causes extracellular depletion of potassium
 Incorrect Answer ImageB.Hyperglycemia causes potassium to shift to the extracellular
space
 Incorrect Answer ImageC.Hyperkalemia will protect the patient against dysrhythmias
 Correct Answer ImageD.Hypokalemia will result as acidosis is corrected
 Incorrect Answer ImageE.Potassium should not have been added to the IV fluids

A 5-year-old boy is brought to the physician due to recurrent leg pains. The boy has been
complaining for several weeks about pain deep in both lower legs, usually occurring soon after
going to bed. The pain is severe enough to interrupt sleep and make the child cry in the middle of
the night. He has relief after taking an analgesic, such as ibuprofen. His parents deny any recent
injury, daytime pain, weight loss, or fever in the child. He does not limp, and normal patterns of
activity are maintained. Physical examination is unremarkable. Which of the following is the
most appropriate next step in management?
 Incorrect Answer ImageA.Complete blood count with ESR
 Incorrect Answer ImageB.CT of the lower extremities
 Correct Answer ImageC.Massage and reassurance
 Incorrect Answer ImageD.Orthopedic consult
 Incorrect Answer ImageE.X-ray of the lower extremities

A newborn who was born at 39 weeks' gestation becomes cyanotic during feeding. His mother
reports that he turned pink when he began crying. A review of the mother's past medical history
shows that she had routine prenatal care. There is no history of maternal drug or alcohol abuse
during pregnancy and no family history of congenital defects. Physical examination is pertinent
for low-set, abnormally shaped ears, and small external genitalia. Funduscopic examination
shows a sharply defined cleft in the left optic disc. Which of the following is the next best step in
management?
 Incorrect Answer ImageA.Barium studies
 Correct Answer ImageB.Echocardiography
 Incorrect Answer ImageC.Fiberoptic bronchoscopy
 Incorrect Answer ImageD.Karyotype for trisomy 21
 Incorrect Answer ImageE.Renal ultrasonography
 Incorrect Answer ImageF.Skeletal survey

A 3-week-old baby girl was born to a primigravid 19-year-old with no prenatal care. After
delivery, she had a 2-day follow-up appointment made at the university hospital pediatric clinic,
which she did not keep. She now brings her baby back to the clinic because of severe vomiting
that started a week ago and is increasing in frequency. She was born at full term and Apgar
scores were 8 and 9 at 1 and 5 minutes, respectively. She weighed 2,950 g (6.5 lb) and was 41
cm (16 in) long. According to the mother, the first 2 weeks after the baby's birth went well and
there were no problems with feeding or sleeping. The baby has a light complexion and an
eczematous rash on her cheeks and forehead and has constant tremors of her extremities. A
musty odor is emitted from the infant as well. Which of the following is most likely to be
elevated?
 Incorrect Answer ImageA.Blood level of branched-chain amino acids
 Incorrect Answer ImageB.Blood level of glucose
 Correct Answer ImageC.Blood level of phenylalanine
 Incorrect Answer ImageD.Serum tyrosine
 Incorrect Answer ImageE.Urine homogentisic acid

A 12-year-old boy is brought to the physician because of recurrent knee pain. His mother reports
that for the last 12 to 18 months when he comes home from school he refuses to exert himself in
any way because his knees hurt from physical education class at school. At first, she believed he
was simply being lazy, but finally decided to have him examined. On examination, the boy is in
the 50th percentile of height and the 75th percentile of weight. He appears to be well and in no
apparent distress. No significant motor or sensory deficits are noted in the lower extremities, and
all deep tendon reflexes are intact. Range of motion in all joints is normal. However, on flexion
of the knee joint, he complains of pain in his proximal shin, and there is tenderness to palpation
over the tibial tubercle. What is the most appropriate management of this condition?
 Incorrect Answer ImageA.Excision of the tibial tubercle
 Incorrect Answer ImageB.Intra-articular steroid injection
 Incorrect Answer ImageC.Regular use of nonsteroidal anti-inflammatory drugs
 Correct Answer ImageD.Rest and ice after physical activity
 Incorrect Answer ImageE.Rest, ice, and crutches for 6 weeks

A 20-month-old boy is brought to the physician by his mother because of a 2-day history of non-
productive cough and other cold symptoms. He has also been hoarse. The mother states that he
“sounds like a seal.” His temperature is 38.1ºC (100.6ºF), pulse is 90/min, and respirations are
36/min. Physical examination shows a normal S1 and S2 without any murmurs, gallops, or rubs.
Inspiratory stridor is heard on auscultation of the chest, and there are no retractions or accessory
muscle use. Which of the following is the most likely causal organism to account for these
findings? 
 Correct Answer ImageA.Adenoviridae
 Incorrect Answer ImageB.Corynebacterium diphtheriae
 Incorrect Answer ImageC.Moraxella catarrhalis
 Incorrect Answer ImageD.Mycoplasma pneumoniae
 Incorrect Answer ImageE.Streptococcus pyogenes

A 7-year-old child is brought in for evaluation because he has developed diplopia for the last 2
weeks. He states that 3 months ago he felt that his neck was getting stiffer, but for the last 2
weeks his neck has been bent to one side. Examination shows right-sided facial weakness, which
includes the forehead. MRI shows patchy T2 hyperintensity, enhancement, and marked
enlargement of the pons, displacing but not occluding the fourth ventricle. Which of the
following is the most likely diagnosis?
 Incorrect Answer ImageA.Benign paroxysmal torticollis
 Correct Answer ImageB.Brainstem glioma
 Incorrect Answer ImageC.Craniopharyngioma
 Incorrect Answer ImageD.Medulloblastoma
 Incorrect Answer ImageE.Meningioma

A 6-month-old infant is brought to his pediatrician for a post-neonatal intensive care unit (NICU)
discharge evaluation at the beginning of November. He was discharged 1 week ago. Gestational
age at birth was 27 weeks. He was treated with surfactant for respiratory distress syndrome and
indomethacin for a patent ductus arteriosus, which successfully resulted in its closure. He was on
oxygen from birth for the first 9 weeks of life and has been on room air since. He was not
discharged with any medications and has no other conditions. He has been gaining weight
steadily on advancing breastfeedings of expressed breast milk with additional fortification. The
mother states that she is very pleased with his progress and had a good week at home without
any problems. A pulse oximeter shows 96% saturation on room air and his examination is
normal for age. Which of the following is the most appropriate next step in management?
 Incorrect Answer ImageA.Aerosolized ribavirin
 Incorrect Answer ImageB.Inhaled beta-agonist

 Correct Answer ImageC.Monthly intramuscular palivizumab

 Incorrect Answer ImageD.Oral thiazide diuretic daily


 Incorrect Answer ImageE.Supplemental oxygen by nasal cannula

A male infant is born to a 37-year-old primigravid mother who has type 2 diabetes mellitus and
did not receive prenatal care. In the delivery room, the infant becomes cyanotic and mildly
tachypneic and is given oxygen. All attempts to remove the oxygen result in continued cyanosis.
The baby is transferred to the neonatal intensive care unit and placed in a hood with 100%
oxygen. A pulse oximetry reading shows a saturation of 68%. Hypoxemia is confirmed with an
arterial blood gas showing a PaO2 of 37 mm Hg while on the 100% oxygen. The PaCO2 is 36 mm
Hg with a pH of 7.43. Cardiac examination shows normal precordial impulse, a single, loud
second heart sound, and a soft systolic ejection murmur at the mid-left sternal border. Which of
the following is the most appropriate next step in management?
 Correct Answer ImageA.Begin a continuous intravenous infusion with prostaglandin E1
 Incorrect Answer ImageB.Insert an endotracheal tube and begin mechanical ventilation
 Incorrect Answer ImageC.Obtain a chest radiograph
 Incorrect Answer ImageD.Obtain an electrocardiogram
 Incorrect Answer ImageE.Perform a septic workup and give intravenous ampicillin and
gentamicin

A 6-week-old infant born at 32 weeks' gestation with a birth weight of 1,500 g (3 lb 5 oz) is
brought to the physician by his parents because he has had an average weight gain of only 8
g/day since birth. He takes an iron-fortified formula that is 24 kcal/oz. His calorie intake is
approximately 125 kcal/day. His stool is poorly formed, bulky, and oily appearing. Which of the
following dietary modifications will most likely result in decreased amounts of poorly formed,
bulky stools and improved weight gain?
 Incorrect Answer ImageA.Add pancreatic enzymes to the formula
 Incorrect Answer ImageB.Change to a lactose-free formula
 Incorrect Answer ImageC.Increase calorie intake to 175 kcal/day by increasing the
volume per feed
 Correct Answer ImageD.Replace the long-chain triglycerides with medium-chain
triglycerides
 Incorrect Answer ImageE.Supplement with vitamins A and E

A 9-year-old boy is brought to the emergency department by his parents 2 days after the sudden
onset of fever, headache, muscle pain, and malaise. He has also had pain and swelling on the
right side of his neck for the past day. He is in moderate distress. His temperature is 39.0ºC
(102.2ºF) and pulse is 110/min. Examination of the oral mucosa shows erythema and edema
surrounding the right parotid duct. There is a tender, firm, nonmobile mass in the distribution of
the parotid gland that extends from the lower end of the ear to the mid-neck area. Which of the
following is the most likely complication from this patient's condition? 
 Incorrect Answer ImageA.Arthritis
 Incorrect Answer ImageB.Dacryoadenitis
 Incorrect Answer ImageC.Infertility
 Correct Answer ImageD.Meningoencephalomyelitis
 Incorrect Answer ImageE.Orchitis

A 3-year-old girl is brought to the physician because of a 2-day history of fever and abdominal
pain. Her mother reports that she has had no other symptoms. Her past medical history is
significant for two febrile urinary tract infections. Physical examination shows no abnormalities.
Urine dipstick shows positive nitrites and leukocyte esterase; urinalysis shows WBCs and gram-
negative rods. After 2 days of treatment with cephalexin, she is afebrile and without pain. The
urine culture from her previous visit is now growing E. coli. Which of the following is the most
appropriate next step in management?
 Incorrect Answer ImageA.Admit to hospital
 Incorrect Answer ImageB.Discontinue antimicrobials
 Incorrect Answer ImageC.Renal scintigraphy
 Correct Answer ImageD.Voiding cystourethrogram
 Incorrect Answer ImageE.Wait until repeat cultures are negative before discontinuing
treatment

Mark

Previous

Next
Lab Values

Notes

Calculator

Reverse Color

A
Text Zoom
A 7-year-old girl is brought to the physician by her mother because of the development of pubic
hair for 6 weeks. She has had vaginal bleeding for 1 day. Her medical history is unremarkable.
She takes no medications. She is in the 50th percentile for height and 70th percentile for weight.
Which of the following is the most likely first sign of this condition? 
 Incorrect Answer ImageA.Adrenarche
 Incorrect Answer ImageB.Increase in height and weight
 Incorrect Answer ImageC.Menarche
 Incorrect Answer ImageD.Premature closure of growth plate
 Correct Answer ImageE.Thelarche

A 17-year-old boy is brought to the physician by his mother because "he has been sick for some
time and the doctors do not seem to know what is wrong with him." For the past few days they
have noticed a yellowing of his sclera. On further questioning a history of protracted diarrhea is
elicited. For approximately 3 years he has been having intermittent episodes of crampy
abdominal pain and bloody diarrhea. Today he has a bilirubin level 3.6 mg/dL with a conjugated
bilirubin level of 2.9 mg/dL, alkaline phosphatase level of 625 U/L, and mildly elevated
transaminase levels. Ultrasound of the right upper quadrant shows strictures and dilation of the
intrahepatic and extrahepatic ducts without gallstones or evidence of liver abscess. Stool cultures
are negative. This patient is at increased risk for which of the following malignancies?
 Correct Answer ImageA.Colon cancer
 Incorrect Answer ImageB.Gastric carcinoma
 Incorrect Answer ImageC.Hepatocellular carcinoma
 Incorrect Answer ImageD.Oral cancer
 Incorrect Answer ImageE.Pancreatic carcinoma

A 7-year-old boy is brought to the physician by his mother because he has continuous bleeding
since having a tooth extracted 24 hours ago. He has taken acetaminophen twice in the past 24
hours. His mother states he is a "very active child who is always getting bruises all over his
body." His paternal uncle had acute lymphoblastic leukemia. His temperature is 37.7ºC (99.8ºF),
pulse is 100/min, respirations are 16/min, and blood pressure is 90/60 mm Hg. Examination
shows bleeding from the site of the dental extraction. The lungs are clear to auscultation. A 1/6
systolic ejection murmur is heard at the left sternal border. There are petechiae bilaterally on
lower extremities and on the chest. There is a 4-cm ecchymosis on the anterior aspect of the right
knee. Laboratory studies show:
Hemoglobin  13.1 gm/dL
Platelets  300,000/mm3
White blood cells  8,600/mm3
PT  12 seconds
PTT  32 seconds
Bleeding time  10 minutes (normal, 2–7 minutes)
Platelet aggregation studies show:
Ristocetin  ++++ (normal response)
ADP  0 (no response)
Epinephrine  0 (no response)
Which of the following is the most likely diagnosis? 
 Incorrect Answer ImageA.Bernard-Soulier syndrome
 Incorrect Answer ImageB.Diamond-Blackfan syndrome
 Correct Answer ImageC.Glanzmann thrombasthenia
 Incorrect Answer ImageD.Hemophilia B (Christmas disease)
 Incorrect Answer ImageE.Immune thrombocytopenic purpura
 Incorrect Answer ImageF.von Willebrand disease

A term male infant is found to be cyanotic shortly after birth and requires endotracheal
intubation due to a weak respiratory effort. On physical examination, his blood pressure is 68/34
mm Hg (equal in all four extremities), pulse is 180/min, and respirations are 32/min. His
precordium is hyperdynamic, there is a grade 4/6 systolic ejection-type murmur along the left
sternal border, and wide, fixed splitting of the second heart sound. There is also a low-pitched
diastolic rumble heard best at the lower sternum. Chest radiography shows a large heart with a
smaller round shadow above the heart and significantly increased pulmonary vascular markings.
ECG shows right ventricular hypertrophy with peaked P waves in the right precordial leads. An
arterial blood gas on an FiO2 of 100% shows pH 7.34; PaCO2 47 mm Hg; PaO2 46 mm Hg.
Which of the following is the most likely diagnosis?
 Incorrect Answer ImageA.Atrial septal defect
 Incorrect Answer ImageB.Hypoplastic left heart syndrome
 Incorrect Answer ImageC.Patent ductus arteriosus
 Incorrect Answer ImageD.Tetralogy of Fallot
 Correct Answer ImageE.Total anomalous pulmonary venous return

A newborn infant is born at 40 weeks' gestation to a healthy 25-year-old primigravid woman via
an uncomplicated vaginal delivery. Physical examination shows upward slanting palpebral
fissures, flat occiput, speckled irises, a single transverse palmar crease, and fifth finger
clinodactyly. The baby’s length is at the 40th percentile, weight at the 50th percentile, and head
circumference at the 10th percentile. He also has general hypotonia. Based on the likely
diagnosis and its most likely cause, what is the best estimate for the likelihood of recurrence? 
 Correct Answer ImageA.1%
 Incorrect Answer ImageB.5%-7%
 Incorrect Answer ImageC.50%
 Incorrect Answer ImageD.75%
 Incorrect Answer ImageE.100%

A 12-month-old girl is brought to the physician for a well-child examination. She stands and
takes steps without assistance, and she can say "Mama" and "Dada." She has received the
following immunizations at the recommended ages: 
Age (month[s]) Immunization

1  Hepatitis B

3  Rotavirus

3  Diphtheria, tetanus, and pertussis

3 Haemophilus influenzae type b 

3  Pneumococcus

3  Poliovirus
She is at the 40th percentile for length and 55th percentile for weight. Examination shows no
abnormalities. The mother asks about possible side effects of the chickenpox vaccine. Which of
the following is the most likely side effect of the varicella vaccine? 
 Incorrect Answer ImageA.Disseminated viral infection
 Incorrect Answer ImageB.Febrile seizures
 Incorrect Answer ImageC.Meningitis
 Correct Answer ImageD.Rash
 Incorrect Answer ImageE.Thrombocytopenia

A newborn infant born at 40 weeks' gestation is found to have a small right parietal scalp lesion.
The lesion is a 0.5 cm oval plaque that is sharply demarcated from the surrounding scalp and has
no hair. It has a yellow-orange color and is slightly elevated. The physician explains to the
mother that over time, this lesion will become nodular with a wart-like appearance. A biopsy of
the lesion is planned. Which of the following is the most likely diagnosis? 
 Incorrect Answer ImageA.Congenital melanocytic nevus
 Incorrect Answer ImageB.Halo nevus
 Incorrect Answer ImageC.Melanoma
 Incorrect Answer ImageD.Neurofibroma
 Incorrect Answer ImageE.Nevus of Ota
 Correct Answer ImageF.Nevus sebaceous
A 5-year-old boy is brought to the physician by his parents because of the development of bowed
legs for 1 year. Current medications include vitamin D supplement. His mother states that her
brother and maternal uncles have had similar bone deformities. Examination shows frontal
bossing, multiple enlargements of the costochondral junction on the chest, and bowing of the
legs. An x-ray film of the wrist shows severe bone demineralization. Which of the following is
most likely to be decreased in the chronic phase of this patient's disease? 
 Incorrect Answer ImageA.Alkaline phosphatase
 Incorrect Answer ImageB.Parathyroid hormone
 Incorrect Answer ImageC.Serum calcium
 Correct Answer ImageD.Serum phosphate
 Incorrect Answer ImageE.Vitamin D

Four weeks after a camping trip, a 16-year-old boy begins to pass stools with an unusually bad
smell. He also develops anorexia and flatulence. None of his friends from the trip are ill. He is
generally healthy and does not take any medications. His temperature is 37.3ºC (99.1ºF).
Physical examination shows diffuse abdominal pain and distension and guaiac-negative stool.
The remainder of the examination is unremarkable. A microscopic image of his stool is shown
above. Which of the following is the most correct statement about his condition? 
 Correct Answer ImageA.A single dose of tinidazole has greater than 90% efficacy
 Incorrect Answer ImageB.Asymptomatic infection is rare
 Incorrect Answer ImageC.Boiling of water during the camping trip would not have killed
the infective organism
 Incorrect Answer ImageD.Infection is limited to the large intestine
 Incorrect Answer ImageE.The trophozoite is the infective form of this organism

A 7-year-old boy is brought to the emergency department 8 hours after the sudden onset of
headache, fever, vomiting, and a rose-colored skin rash that started around his ankles. He just
returned from a 10-day camping trip with his family, during which he had numerous mosquito
bites and removed three ticks. He is in moderate distress. His temperature is 39.0ºC (102.2ºF),
pulse is 110/min, and respirations are 24/min. Examination shows multiple erythematous
macules on the lower extremities and hands with petechiae interspersed between them. His ECG
is unremarkable. The platelet count is 72,000/mm3. Which of the following is the most
appropriate pharmacotherapy for this patient? 
 Incorrect Answer ImageA.Amoxicillin
 Incorrect Answer ImageB.Amoxicillin/clavulanate
 Incorrect Answer ImageC.Ceftriaxone
 Incorrect Answer ImageD.Chloramphenicol
 Correct Answer ImageE.Doxycycline
 Incorrect Answer ImageF.Penicillin G

A 3-year-old girl and her extremely concerned parents are referred to a pediatric endocrinologist
for the evaluation of short stature. The pregnancy, labor, and delivery were unremarkable and the
child's birth weight and height were initially appropriate for gestational age. After this, she had
very slow growth, but their pediatrician wanted to continue to follow her without intervention
because there was no evidence of any disease. The girl's past medical history is entirely
unremarkable. She has a good appetite and has never had any illness other than an ear infection
and the common cold. Her immunizations are up to date. Family history is completely negative
for any inherited illnesses. On physical examination, the child is obviously short but has a
completely normal physical examination, including neurologic examination. The parents are
measured and are above average for adult heights. The physician sees that on the growth curve,
the child's height began at the 50th percentile at birth but then fell to less than the 3rd percentile
and has remained there since, with periods of very little growth velocity. A bone age is obtained
and is interpreted as being less than what it should be for a 3-year old. Which of the following is
the most appropriate next step in management?
 Incorrect Answer ImageA.Administer a growth hormone stimulation test
 Incorrect Answer ImageB.Obtain blood for a karyotype
 Correct Answer ImageC.Obtain blood for insulin-like growth factor 1 (IGF-1) and its
binding protein (IGF-BP3)
 Incorrect Answer ImageD.Obtain skull radiographs
 Incorrect Answer ImageE.Reassure the parents that this is constitutional delay and that
nothing further need be done
 Incorrect Answer ImageF.Send blood for an initial screen for some etiology of chronic
disease

A 4-year-old child is brought to the physician by his parents for a routine well-child visit. He has
no significant past medical history and he is up-to-date on all vaccinations. His mother is
concerned about his development because she has been comparing him to other children in his
preschool. Physical examination is unremarkable. Which of the following skills would be
expected of a child aged 4 years?
 Incorrect Answer ImageA.Building a 10-cube staircase
 Incorrect Answer ImageB.Drawing a person with 8 to 10 parts
 Correct Answer ImageC.Drawing a square
 Incorrect Answer ImageD.Drawing a triangle
 Incorrect Answer ImageE.Repeating 5 digits

A female newborn is born at 39 weeks' gestation to a healthy 31-year-old woman via


uncomplicated vaginal delivery. Apgar scores are 7 and 8 at 1 and 5 minutes. 2 days later, she
develops jaundice, vomiting, and lethargy. Her mother reports that the infant had been breast-
feeding well until now. Physical examination shows jaundice and hepatomegaly. Eye
examination shows bilateral gray-white lens opacities that are visible without an
ophthalmoscope. A fundoscopic examination fails to show a bilateral red reflex. Which of the
following is the most likely diagnosis?
 Incorrect Answer ImageA.Congenital mumps infection
 Correct Answer ImageB.Galactosemia
 Incorrect Answer ImageC.Maple-syrup urine disease
 Incorrect Answer ImageD.Prematurity
 Incorrect Answer ImageE.Wilson disease

A mother brings her 12-year-old daughter to the physician because she is concerned that her
child has delayed physical development. The daughter began developing breasts at age 10 but
has not had her first menstrual period, which is particularly concerning to her mother. The
daughter has no medical problems and takes no medications. Examination shows developing
breasts and normal external female genitalia; the rest of the examination is unremarkable. Which
of the following is the most appropriate response by the physician to the mother’s concern?
 Incorrect Answer ImageA.Breast development at age 10 is unusually early
 Incorrect Answer ImageB.Breast development at age 10 is unusually late
 Correct Answer ImageC.Evaluation for late menses should not be started until age 15
years
 Incorrect Answer ImageD.Evaluation for late menses should start immediately
 Incorrect Answer ImageE.It is inappropriate to discuss the child's sexual development
with the mother

A 15-year-old girl is brought to the physician because of a 2-day history of pain and swelling in
her left knee. She plays soccer regularly on her school team. There is no history of trauma. On
physical examination, there is marked swelling and tenderness over her anterior tibial tuberosity.
A radiograph of her left knee shows soft tissue swelling with loss of the sharp margins of the
patella and fragmentary ossification of the tibial tubercle. Which of the following is the most
likely cause of her symptoms? 
 Incorrect Answer ImageA.Avascular necrosis of the hip
 Incorrect Answer ImageB.Legg-Calve-Perthes disease
 Incorrect Answer ImageC.Septic arthritis
 Incorrect Answer ImageD.Slipped capital femoral epiphysis
 Incorrect Answer ImageE.Stress fracture
 Correct Answer ImageF.Traction apophysitis

A 14-year-old boy is brought to the emergency department because of soreness and weakness in
his legs for the past day that has slowly spread from his calves to his thighs. He now complains
of weakness in his trunk and arms, and double vision. On further questioning, he says that he had
a diarrheal episode a week ago. On examination, he appears tired and lies on the examining
table. His temperature is 37ºC (98.6ºF), pulse is 48/min, and respirations are 22/min. Both of his
legs are diffusely tender. Neurologic examination reveals symmetric weakness. Deep tendon
reflexes are difficult to elicit in the lower extremities, and sensation is greatly diminished. Which
of the following is the best initial test to make the diagnosis? 
 Incorrect Answer ImageA.Botulinum toxin analysis in stool and blood
 Correct Answer ImageB.Cerebrospinal fluid studies
 Incorrect Answer ImageC.Creatinine phosphokinase levels
 Incorrect Answer ImageD.MRI of the spine
 Incorrect Answer ImageE.Muscle biopsy

A 3-year-old boy who has recently emigrated from Mexico with his family is brought to the
emergency department because of a high-grade temperature, difficulty breathing, difficulty
speaking, and irritability that began overnight. He has no history of chronic medical conditions
and takes no medication. He received some vaccinations in Mexico, but his mother is unsure
which ones. His temperature is 39.5ºC (103.1ºF), pulse is 165/min, respirations are 44/min, and
blood pressure is 102/62 mm Hg. On physical examination, the patient is noted to be sitting
forward on the exam table with his neck extended, mouth open, and is drooling. He has nasal
flaring and uses accessory muscles when breathing. Which of the following is the most
appropriate initial step in management?
 Incorrect Answer ImageA.Administer intramuscular dexamethasone
 Correct Answer ImageB.Endotracheal intubation and ventilation
 Incorrect Answer ImageC.Examine the oropharynx with a tongue depressor
 Incorrect Answer ImageD.Obtain an anterior and lateral neck radiograph
 Incorrect Answer ImageE.Perform a throat culture
 Incorrect Answer ImageF.Reassure the mother that this is a fleeting viral illness
 Incorrect Answer ImageG.Send a blood sample for culture and sensitivity
 Incorrect Answer ImageH.Start an intravenous line and administer broad-spectrum
antibiotics

A 15-year-old girl is referred to a pediatric cardiology clinic due to a questionable murmur on


her sports physical by her pediatrician. She has no current complaints and her review of systems
is noncontributory. Past medical history is negative for any significant illnesses or diseases and
there are no diseases or conditions that run in the family. Physical examination shows a healthy-
appearing 15-year-old girl. Her temperature is 37.2°C (99°F), pulse is 90/min, and respirations
are 20/min. The lungs are clear to auscultation. Standing exam of her spine shows some degree
of scoliosis. Cardiac examination while supine shows a very soft murmur at the apex. She is
placed in the left lateral recumbent position and a mid-systolic click is heard. The click is heard
earlier in the sitting position; with a Valsalva maneuver, a decreasing mid-to-late systolic
murmur is also heard. Which of the following represents the most likely findings on chest x-ray
(CXR), electrocardiogram (ECG), and echocardiogram (echo)?
 Incorrect Answer ImageA.CXR: Heart enlarged with prominent pulmonary vascularity;
ECG: Bifid P-waves and increased voltage of R-waves in left precordial leads; Echo: Increased
size of left atrium and ventricle and with retrograde flow across the mitral valve

 Incorrect Answer ImageB.CXR: Increased left atrium and right ventricle with venous
pulmonary congestion; ECG: Increased voltage of R-waves in right precordial leads and bifid P-
waves in the left precordial leads; Echo: Thickened mitral valve leaflets, significant decrease in
mitral valve orifice, small left ventricle and enlarged left atrium

 Correct Answer ImageC.CXR: Normal; ECG: Occasional unifocal PVCs; Echo: Mitral
valve thickening with posterior movement of the posterior mitral of the posterior valve leaflet in
mid-to-late systole

 Incorrect Answer ImageD.CXR: Normal size but prominent ascending aorta; ECG:
Increased voltage of R-waves and inverted T-waves in left precordial leads; Echo: Left
ventricular hypertrophy and thick, domed aortic valve leaflets; increased left ventricular
shortening fraction

 Incorrect Answer ImageE.CXR: Prominence of main pulmonary artery and right


ventricular enlargement; ECG: Minimal increased R-waves in the right precordial leads; Echo:
Retrograde diastolic flow from the pulmonary artery to the to the right ventricle

A 17-year-old boy comes to the physician because of a 2-week history of fever, abdominal pain,
and intermittent, non-bloody diarrhea. He has had no jaundice. He is from Mexico and is visiting
family in the United States. His temperature is 39.1°C (102.4°F), blood pressure is 116/74 mm
Hg, and pulse is 66/min. Examination shows right upper quadrant tenderness without guarding or
rebound. There is no hepatosplenomegaly. Laboratory studies show: 
Leukocyte count  12,000/mm3
Alkaline phosphatase  200 U/L
Aspartate transaminase (AST, GOT)  134 U/L
Alanine transaminase (ALT, GPT)  122 U/L
Ultrasound of the abdomen shows a subcapsular abscess in the right lobe of the liver. Which of
the following is the most appropriate next step in management? 
 Incorrect Answer ImageA.Computed tomography (CT) of the abdomen (including liver)
 Incorrect Answer ImageB.Fecal microscopy
 Incorrect Answer ImageC.Gallium scan
 Incorrect Answer ImageD.Magnetic resonance imaging (MRI) of the abdomen (including
liver)
 Incorrect Answer ImageE.Needle aspiration of liver abscess
 Incorrect Answer ImageF.Repeat ultrasound in 1 month
 Correct Answer ImageG.Serum antibody testing
A 14-year-old healthy boy is brought to the pediatrician for a routine health maintenance visit. A
routine urinalysis is required by his school for sports participation which shows 1+ proteinuria.
There is no hematuria or bacteriuria. There is no family history of renal disease. His physical
examination is unremarkable, showing no edema and his blood pressure is normal. Which of the
following is the most likely diagnosis?
 Incorrect Answer ImageA.Acute glomerulonephritis
 Incorrect Answer ImageB.IgA nephropathy
 Incorrect Answer ImageC.Minimal change disease
 Correct Answer ImageD.Orthostatic proteinuria
 Incorrect Answer ImageE.Urinary tract infection

An infant boy is born at 39 weeks' gestation by way of spontaneous vaginal delivery to a healthy,
32-year-old primigravid woman. Prenatal care was appropriate and there were no problems at the
time of delivery. Physical examination is normal, with the exception of the genital examination.
The urethral meatus is located on the ventral aspect of the penile shaft. Which of the following is
the most correct statement about this condition? 
 Incorrect Answer ImageA.A urine culture should be obtained at this time
 Incorrect Answer ImageB.A voiding cystourethrogram should be ordered
 Correct Answer ImageC.Circumcision (if desired) should be delayed
 Incorrect Answer ImageD.The infant may have a circumcision now
 Incorrect Answer ImageE.The infant may never have a circumcision

A newborn male is born at 39 weeks gestation to a 29-year-old woman, gravida 1, para 0, via
planned cesarean section because of the presence of a low meningomyelocele found on fetal
ultrasound. The infant had no problems after birth and was taken to the neonatal intensive care
unit (NICU) in the side-lying position, after covering the open cord with sterile saline-soaked
pads and gauze. Examination and further evaluation in the NICU prior to surgery for the defect
also shows high, broad forehead, epicanthal folds, a wide nasal bridge with small upturned nose,
and a long, indistinct philtrum. The infant also has a cleft palate, shortened left forearm, bilateral
cryptorchidism, and hypospadias Cardiac examination shows a large, unrestrictive ventricular
septal defect. Which of the following is the most likely agent that was used by the mother during
pregnancy?
 Incorrect Answer ImageA.Alcohol
 Incorrect Answer ImageB.Folic acid deficiency

 Incorrect Answer ImageC.Isotretinoin


 Correct Answer ImageD.Valproic acid
 Incorrect Answer ImageE.Warfarin

A 3-month-old baby boy is brought to the physician with a 2-week history of intermittent bloody
diarrhea. His mother has changed his formula twice, but the blood in the stool has recurred. His
birth history is normal, although his mother recalls that the baby’s doctor was slightly concerned
about excessive bleeding from his umbilicus. He had an ear infection at age one month and was
hospitalized for pneumonia at age 2 months. Physical examination shows raised scaly patches on
his face and extensor areas and a few petechiae on his chest. A platelet count is 25,000/mm3. If
this child survives until adolescence, he is at a particularly high risk of developing which of the
following?
 Incorrect Answer ImageA.Congestive heart failure
 Incorrect Answer ImageB.Crohn disease
 Correct Answer ImageC.Lymphoma
 Incorrect Answer ImageD.Rheumatoid arthritis
 Incorrect Answer ImageE.Wilms tumor

A 14-year-old boy is brought to the physician by his mother because of facial asymmetry and an
inability to taste things for 4 days. He had a fever, sore throat, and runny nose approximately 2
weeks ago. His temperature is 36.9°C (98.4°F). Examination is shown (see media file).
Laboratory studies, including a complete blood count and chemistry profile, are within normal
limits. Infection with which of the following is the most likely cause of this patient's condition? 
 Correct Answer ImageA.Epstein-Barr Virus
 Incorrect Answer ImageB.Group A Streptococcus
 Incorrect Answer ImageC.Human immunodeficiency virus
 Incorrect Answer ImageD.Rhinovirus
 Incorrect Answer ImageE.Measles

A 17-year-old boy is brought to the physician because of an 8-month history of low back pain
that begins at night and radiates down his legs. He has had morning stiffness. He recently quit
the swim team because of his back pain. He has had no fever, diarrhea, tenesmus, hematuria,
frequency, urgency, or painful urination. His temperature is 37.0ºC (98.6ºF). A 2/6 diastolic
murmur is heard along the left sternal border. Musculoskeletal examination shows tenderness of
the lower spine and moderate limitation of back motion. Laboratory studies show an elevated
erythrocyte sedimentation rate (ESR); rheumatoid factor is negative. An x-ray film of the lumbar
spine is shown. Which of the following is the most appropriate next step in management?
 Correct Answer ImageA.Indomethacin
 Incorrect Answer ImageB.Intra-articular corticosteroid injection
 Incorrect Answer ImageC.Low-dose oxycodone
 Incorrect Answer ImageD.Oral prednisone
 Incorrect Answer ImageE.Spinal bracing
 Incorrect Answer ImageF.Sulfasalazine

 6-year-old boy is brought to the physician by his father because of “bumps on his abdomen and
along the waistline” for 1 week. The father states that another child at school has come down
with similar bumps. There are no other known exposures or contacts. The boy has a history of
atopic dermatitis from early infancy. He takes no medications. His temperature is 36.7°C
(98.0°F). Examination is shown. The physician also notes a few discrete, smooth, dome-shaped
papules on the neck and right thigh. The groin is not affected. Which of the following is the most
likely cause of this patient’s condition?
 Incorrect Answer ImageA.Bartonella henselae 
 Incorrect Answer ImageB.Human papillomaviruses 2 and 4
 Correct Answer ImageC.Molluscum pox virus type 1
 Incorrect Answer ImageD.Parvovirus B19
 Incorrect Answer ImageE.Sarcoptes scabiei var. hominis

A 34-year-old woman with gestational diabetes goes into labor at 42 weeks’ gestation. She
delivers a girl who weighs 2,600 g (5.7 lb) and is 43 cm long (16.9 in). At 1 minute of life, the
neonate is pale with mottled, cool extremities. The mother had received insulin for her
gestational diabetes. Family history is insignificant. The neonate's pulse is 95/min. There is a
weak cough, generalized hypotonia with some flexion of the extremities, and a slow respiratory
effort. Her skin is dry with scarce lanugo. There is no significant change at her 5-minute
assessment. Detailed physical examination shows macroglossia with coarse facial features,
enlarged fontanelles, and an umbilical hernia. Which of the following is the most likely cause of
this condition?
 Incorrect Answer ImageA.Hereditary defects in thyroid hormone synthesis
 Incorrect Answer ImageB.Iodine deficiency in the mother
 Incorrect Answer ImageC.Pituitary hypothyroidism
 Correct Answer ImageD.Thyroid dysgenesis
 Incorrect Answer ImageE.Transplacental transfer of blocking antibodies
 Incorrect Answer ImageF.Transplacental transfer of insulin

A 1-week-old newborn boy is brought to the emergency department because of rapid breathing
and intermittently turning blue. He was born at 37 weeks’ gestation and has not had a follow-up
visit after being released from the newborn nursery on his 2nd day of life. Physical examination
shows tachypnea and tachycardia. Pulse oximetry in room air shows an oxygen saturation of
85% and he is placed on oxygen. Cardiac examination shows a loud systolic murmur along the
left upper sternal border and is widely radiating; there is also a loud low-pitched diastolic rumble
over the xiphoid area. A gallop rhythm is present, and S2 is widely split and does not vary with
respiration. A chest x-ray shows cardiomegaly with a prominent right atrium, right ventricle and
pulmonary artery; there is a large supracardiac shadow directly above and abutting the superior
aspect of the heart, and significantly increased pulmonary vasculature. An electrocardiogram
shows right ventricular hypertrophy and right atrial enlargement. Echocardiography is most
likely to show which of the following?
 Incorrect Answer ImageA.D-transposition of the great arteries
 Incorrect Answer ImageB.Hypoplastic left heart
 Correct Answer ImageC.Total anomalous pulmonary venous return
 Incorrect Answer ImageD.Tricuspid atresia
 Incorrect Answer ImageE.Truncus arteriosus

A 10-year-old boy is brought to the physician because of pain in his right leg for 4 months. The
pain is worse at night and is unrelenting, but it can usually be relieved with ibuprofen. According
to his parents, he is a physically active child and plays basketball and little league baseball. On
physical examination, there is localized tenderness over the anterior aspect of the proximal right
thigh. There is also mild atrophy of the affected limb, and the boy walks with a slight limp. A
radiograph of the femur shows a radiolucent nidus with surrounding reactive sclerotic bone.
Which of the following is the most likely diagnosis?
 Incorrect Answer ImageA.Brodie's abscess
 Incorrect Answer ImageB.Ewing sarcoma
 Incorrect Answer ImageC.Osteosarcoma
 Correct Answer ImageD.Osteoid osteoma
 Incorrect Answer ImageE.Stress fracture

An 8-year-old boy is brought to the emergency department by his mother because of fever and
decreased activity for 1 day. She reports that he had a headache 2 days ago. The child has not
been exposed to anybody with illness, nor is she aware of any insect bites. The family migrated
from Southeast Asia when he was 3 years old. His medical history is unremarkable and he does
not take any medications. His temperature is 38.1ºC (100.5ºF). Neurological examination shows
nuchal rigidity. The lungs are clear to auscultation and cardiac examination shows no
abnormalities. The abdomen is soft and without tenderness. There is no rash. A lumbar puncture
and cerebrospinal fluid analysis shows:
Color Cloudy, turbid
Opening pressure 120 cm
White blood cells 200/mm3, 20% PMNs
Differential 78% lymphocytes
Glucose 30 mg/dL
Protein 400 mg/dL
Gram stain No organisms
Which of the following is the most appropriate next step in management?
 Incorrect Answer ImageA.Acyclovir
 Incorrect Answer ImageB.Ceftriaxone and vancomycin
 Incorrect Answer ImageC.Isoniazid, rifampin, ethionamide, and pyrazinamide
 Correct Answer ImageD.Isoniazid, rifampin, ethionamide, pyrazinamide, and prednisone
 Incorrect Answer ImageE.Supportive therapy

A 5-month-old girl is brought to the office by her mother, who states that the girl had an episode
following feeding during which she began to breathe deeply, became blue, and then lost
consciousness. The mother states that she picked her up and held her, and the infant regained her
usual color and became alert. Physical examination shows a harsh systolic murmur. The
remainder of the physical examination is unremarkable. Which of the following is the most
likely diagnosis? 
 Incorrect Answer ImageA.Aortic stenosis
 Incorrect Answer ImageB.Coarctation of the aorta
 Incorrect Answer ImageC.Patent ductus arteriosus
 Correct Answer ImageD.Tetralogy of Fallot
 Incorrect Answer ImageE.Ventricular septal defect

A 27-year-old primigravid woman with type 2 diabetes mellitus comes to the emergency
department because of a rapidly enlarging "stomach" and the sudden onset of contractions. She is
at 34 weeks’ gestation. She has not had prenatal care since the first trimester. She had been
feeling well until several days earlier when she noticed that her abdominal girth was rapidly
increasing. She started having contractions that morning and decided to go to the hospital.
Examination shows a distended abdomen and a large palpable uterus. She is fully dilated and has
contractions 2 to 3 minutes apart. Several hours later she delivers a boy who weighs 2,500 g (5.5
lbs) and is 38 cm (14.9 in) long. His APGAR scores are 6 and 7 at 1 and 5 minutes, respectively.
Physical examination of the child shows a scaphoid abdomen and a palpable fullness of the
epigastrium. An abdominal x-ray film of the newborn shows gaseous distention of the stomach
and proximal duodenum. A nasogastric tube is placed and suction produces bilious fluid from the
stomach. Which of the following prenatal studies might have detected the most likely
abnormality?
 Incorrect Answer ImageA.Alpha-fetoprotein level
 Incorrect Answer ImageB.Antibody screens for common fetal infections
 Incorrect Answer ImageC.Hemoglobin electrophoresis
 Incorrect Answer ImageD.Human choriogonadotropin levels
 Correct Answer ImageE.Ultrasonography

An infant is brought to the physician for a well-child visit. She was born at 39 weeks' gestation,
and both pregnancy and delivery were uncomplicated. She has been feeding well and is growing
appropriately. Her vital signs are normal. On examination, she can lift her head to 90 degrees in
the prone position, her eyes follow past the midline, and she laughs, regards her own hand, and
has some awareness of the presence of her mother. Which of the following is the most likely age
of this infant?
 Incorrect Answer ImageA.2 months
 Correct Answer ImageB.4 months
 Incorrect Answer ImageC.6 months
 Incorrect Answer ImageD.12 months
 Incorrect Answer ImageE.18 months

A 16-year-old girl comes to the physician because of a 5-day history of foul-smelling vaginal
discharge. She has had no fever or abdominal pain. She is sexually active and she uses condoms
consistently with her partner. There is no abdominal tenderness. Pelvic examination shows a
normal-appearing vulva, but a thin layer of white, thin discharge on the vaginal walls and cervix
is present. A fishy odor is produced when KOH is added to the discharge. The vaginal fluid has a
pH of 5. Which of the following is the most likely finding in this patient?
 Incorrect Answer ImageA.Gram-negative diplococci
 Incorrect Answer ImageB.Lactobacilli
 Incorrect Answer ImageC.Motile, flagellated organisms
 Incorrect Answer ImageD.Pseudohyphae
 Correct Answer ImageE.Vaginal epithelial cells covered with coccobacillis

An 8-year-old girl is brought to the physician by her mother because of intermittent fever, sore
throat, malaise, and headache for the past 2 days. Her temperature is 38.4ºC (101.2ºF), pulse is
105/min, respirations are 18/min, blood pressure is 102/62 mm Hg, and oxygen saturation is 98%
on room air. Physical examination shows bibasilar rales. Chest radiograph shows significant
bilateral interstitial infiltrates. Laboratory studies show WBC count 8,500/mm3. Which of the
following is the next best step in management of this patient?
 Incorrect Answer ImageA.Administer oxygen and a nebulized beta agonist
 Incorrect Answer ImageB.Hospitalize and start IV cefotaxime
 Incorrect Answer ImageC.Perform a PCR for the organism on nasopharyngeal secretions
 Incorrect Answer ImageD.Start a course of oral amoxicillin
 Correct Answer ImageE.Start a course of oral azithromycin

A 10-year-old boy is brought to the emergency department because of difficulty walking,


headache, nausea, and vomiting for the past 3 days. Neurologic examination shows nuchal
rigidity and papilledema. A CT scan shows an infiltrating cerebellar tumor, which is located in
the midline (vermis), with plaque-like extensions onto the cerebellar surface. There is no cystic
component. The fourth ventricle is compressed, and the third and lateral ventricles are dilated.
His 3 siblings are all healthy. Which of the following is the most likely diagnosis? 
 Incorrect Answer ImageA.Ependymoma
 Incorrect Answer ImageB.Hemangioblastoma
 Correct Answer ImageC.Medulloblastoma
 Incorrect Answer ImageD.Oligodendroglioma
 Incorrect Answer ImageE.Pilocytic astrocytoma

A 4-month-old infant is brought to the emergency department by her parents because of


constipation and decreased activity for 3 days. Prior to this, she was well. The mother also says
that the infant has been feeding poorly for the past day. Her immunizations are up to date. Her
temperature is 36.8°C (98.4°F). Examination shows a weak cry and drooling. There is poor
muscle tone with generalized weakness and decreased movement. In the pull-to-sit, there is
complete head lag. Which of the following is the most likely diagnosis? 
 Incorrect Answer ImageA.Congenital hypothyroidism
 Incorrect Answer ImageB.Guillain-Barré syndrome
 Correct Answer ImageC.Infant botulism
 Incorrect Answer ImageD.Myasthenia gravis
 Incorrect Answer ImageE.Vaccine-associated poliomyelitis

A toddler is brought to the emergency department with burns on both of his buttocks. The
mother states that the child accidentally fell into a tub filling with hot water while she went to
answer the telephone. The child has previously been well with no known medical conditions. He
is afebrile and crying with a heart rate of 120/min; blood pressure is not obtainable at this time.
There is an approximate 10 cm diameter burn on each buttock. The skin of the burned area is
mottled pink and white and there are moist blebs and blisters present. It appears to be very
painful to the child. It is estimated that this area represents less than 10% of the body surface
area. A rapid assessment reveals no other apparent injuries or burns. Which of the following is
the first step in management?
 Correct Answer ImageA.Application of silver sulfadiazine cream to the burned areas
 Incorrect Answer ImageB.Early excision and grafting of the burned areas
 Incorrect Answer ImageC.Education of the parents on accident prevention
 Incorrect Answer ImageD.Intravenous fluid resuscitation
 Incorrect Answer ImageE.Referral to child protective services for suspected child abuse

A 4-year-old girl is found drinking a bottle of liquid drain cleaner and is immediately brought to
the emergency department. She appears to be very irritable and is unwilling to swallow any
liquid medication. Her mother reports that the child has been crying intensely and has had nausea
but no vomiting. Examination of the oral cavity shows no evidence of burns or ulcerations.
Lungs are clear to auscultation without crackles, wheezes, or stridor. Which of the following is
the most appropriate management?
 Incorrect Answer ImageA.Barium swallow
 Incorrect Answer ImageB.CT of the abdomen
 Correct Answer ImageC.Esophagoscopy
 Incorrect Answer ImageD.Indirect laryngoscopy
 Incorrect Answer ImageE.Intubation

A 16-year-old boy is brought to the physician because of a temperature of 38.4°C (101°F) and
low back, wrist, and knee pain. He had a sore throat 1 month earlier. Pea-sized swellings are
noted over the skin on his knees. He has a serpiginous erythematous area on his anterior trunk.
His blood and throat cultures are negative, and his CBC is unremarkable. His antistreptolysin-O
(ASO) titer is high. Which of the following is the most appropriate therapy?
 Incorrect Answer ImageA.Acetaminophen
 Incorrect Answer ImageB.Aspirin
 Incorrect Answer ImageC.Penicillin
 Correct Answer ImageD.Penicillin and aspirin
 Incorrect Answer ImageE.Supportive care

A 9-month-old boy is brought to the emergency department because of fever and a cough
productive of sputum for 3 days. He has had two episodes of otitis media, occurring at 6 and 7
months of age, and whenever he is off antibiotics, he has yellow-green nasal drainage. His
mother's brother has had repeated infections since childhood and is being treated for a disease,
but the mother does not know what it is. His temperature is 38.7°C (101.7°F) and respirations are
60/min. There are crackles and decreased breath sounds on the right. He has no visible tonsils
and no palpable lymph nodes. There is no hepatosplenomegaly. Laboratory studies show: 
IgG 80 mg/dL (normal, 723–1,685 mg/dL) 
IgA 60 mg/dL (normal, 81–463 mg/dL) 
IgM 20 mg/dL (normal, 48–271 mg/dL) 
An x-ray film of the chest is shown. Which of the following is the most likely explanation of his
underlying disease?
 Incorrect Answer ImageA.Defect in the class-switch recombination process
 Incorrect Answer ImageB.Failure of differentiation of B cells into immunoglobulin-
producing cells
 Correct Answer ImageC.Failure of pre-B cell expansion and maturation
 Incorrect Answer ImageD.Inadequate immune response to Epstein-Barr virus infections
 Incorrect Answer ImageE.Selective failure of maturation of B cells into IgA-producing
cells

A 2,040-g (4.5-lb) male newborn is delivered at 34 weeks' gestation to a 27-year-old woman,


gravida 2, para 1. Apgar scores are 4 and 5 at 1 and 5 minutes, respectively. Two weeks ago,
during the pregnancy, she had a flu-like illness, including fever, nausea, vomiting and diarrhea.
The amniotic fluid is brown and murky without meconium. The newborn appears lethargic. His
temperature is 35ºC (95ºF) and pulse is 160/min. There is grunting and apnea. Skin examination
shows small, pale nodules, and there is an erythematous rash over the body. Gram stain of a
smear from the mother's cervix shows pleomorphic gram-positive bacilli. Placental biopsy shows
granulomas and microabscesses. The infant’s complete blood count is significant for a striking
monocytosis. Which of the following is the most likely diagnosis for these findings? 
 Incorrect Answer ImageA.Congenital cytomegalovirus infection
 Incorrect Answer ImageB.Congenital rubella
 Incorrect Answer ImageC.Congenital syphilis
 Incorrect Answer ImageD.Group B Streptococcus sepsis
 Incorrect Answer ImageE.Neonatal herpes simplex infection
 Correct Answer ImageF.Neonatal listeriosis
 Incorrect Answer ImageG.Neonatal tuberculosis

A 5-year-old boy is brought to the physician because of a 1-week history of fever, nasal
congestion, runny nose, and sneezing. He has had difficulty sleeping at night, sore throat,
decreased oral intake, and one episode of non-bloody diarrhea. He began kindergarten 2 weeks
ago. His temperature is 37.5°C (99.5°F). Examination shows erythema and swelling of the nasal
mucosa as well as mild, non-tender anterior cervical lymphadenopathy. The remainder of his
examination shows no abnormalities. Which of the following is the most appropriate next step in
management? 
 Incorrect Answer ImageA.Cidofovir
 Incorrect Answer ImageB.Oseltamivir
 Incorrect Answer ImageC.Palivizumab
 Incorrect Answer ImageD.Penicillin
 Correct Answer ImageE.Supportive therapy

A health care provider examines a 2-month-old infant who had been admitted to the pediatric
intensive care unit due to tachypnea, respiratory distress, poor perfusion, and hypotension. The
infant was born at term after an uncomplicated labor and delivery. The infant is afebrile, pulse is
140/min, respirations are 60/min, and blood pressure in the right arm is 65/28 mm Hg. The infant
has a loud continuous murmur heard maximally in the second left intercostal space, along with a
thrill that obscures the first and second heart sound. There is also a low-pitched, mid-diastolic
murmur heard best at the apex, and a prominent apical impulse and heave present. The liver is
palpated 3 cm below the right costal margin. After stabilization in the ICU, which is the most
appropriate next step in management?
 Incorrect Answer ImageA.Administration of intravenous alprostadil
 Incorrect Answer ImageB.Administration of intravenous indomethacin
 Incorrect Answer ImageC.Pharmacological treatment in the ICU until condition
normalizes
 Incorrect Answer ImageD.Schedule an elective surgical closure of the septal defect
 Correct Answer ImageE.Transcatheter closure of lesion in the catheterization lab

A 2-year-old Korean boy is brought to the physician after 6 days of spiking fever. His parents
noticed swelling of his hands and feet on the third day of fever, and a red rash on his torso that
developed yesterday. The parents deny cough, nausea or vomiting, changes in his urine, or
tugging at the ears. He reports diarrhea for the past day with increasing irritability and right
upper quadrant abdominal pain. There are no known sick contacts. He has not had his 18-month
immunizations, as he had a cold at the time. On examination, the boy has a polymorphic
maculopapular rash on his torso. He has bilateral conjunctival injection without exudate, and
there is the presence of a unilateral enlarged, slightly tender cervical lymph node. His
temperature is 40ºC (104ºF), pulse is 140/min, respirations are 20/min and blood pressure is
90/65 mm Hg. He is admitted to the hospital for further evaluation and treatment. An abdominal
ultrasound is ordered, which shows hydrops of the gallbladder. Which of the following is part of
the initial evaluation of this patient?
 Incorrect Answer ImageA.Bone marrow biopsy
 Incorrect Answer ImageB.Cerebrospinal fluid analysis
 Incorrect Answer ImageC.CT scan of the brain
 Correct Answer ImageD.Echocardiography
 Incorrect Answer ImageE.Liver biopsy
A 12 year-old female is brought to the physician by her parents for evaluation of a new rash.
They state that the child has been experiencing a headache and fever for the last few days and
has been home from school. Now she has developed a rash and has started to complain of mild
pain in her fingers, wrists, and ankles. On examination, she is now afebrile, her pulse is 80/min,
respirations are 20/min, and blood pressure seated in the right arm is 112/75 mm Hg. She has
very red cheeks and a reticulated erythema of her proximal arms and upper anterior chest. She
has very mild swelling and stiffness on movement of her knees. No other abnormalities are noted
on examination. At this time, what should the physician tell the parents?
 Incorrect Answer ImageA.She needs an immediate complete blood count and reticulocyte
count
 Incorrect Answer ImageB.She needs to be hospitalized for blood transfusions
 Incorrect Answer ImageC.She needs to be hospitalized for intravenous immunoglobulin
therapy (IVIG)
 Incorrect Answer ImageD.She needs to be isolated at home until symptoms go away
 Correct Answer ImageE.She should return to school

A newborn boy born at term without any complications is discharged home within 24 hours of
delivery. The newborn is breastfed as the mother did this for her previous child. She takes the
baby to his pediatrician 2 days after delivery because of the early hospital discharge. At this time,
feedings are going well, he is not jaundiced, and he is having normal stools. The next well-child
visit is scheduled for 2 weeks of age. At this time, his cardiovascular examination shows a grade
IV murmur starting after the first heart sound and continuing into diastole. It is heard best in the
left second intercostal space and radiates to the clavicle. There is also a prominent apical
impulse, bounding pulses, and wide pulse pressures. Which of the following is the definitive
corrective procedure for this anomaly?
 Incorrect Answer ImageA.Arterial switch
 Incorrect Answer ImageB.Atrial balloon septostomy
 Incorrect Answer ImageC.Heart transplantation
 Correct Answer ImageD.Ligation
 Incorrect Answer ImageE.Placement of a patch

A 10-day-old newborn male is brought to the emergency department because of grossly bloody
stools, vomiting, and poor feeding for the past 12 hours. He was born at 34 weeks' gestation to a
healthy 29-year-old woman, gravida 1, para 1, via a normal vaginal delivery without
complications. Birth weight was 2,030 g (4 lb 8 oz). Apgar scores were 3 and 8 at 1 and 5
minutes, respectively. Passage of meconium occurred within 24 hours. He has been taking
formula from a bottle well. His temperature is 38.2ºC (100ºF), blood pressure is 60/35 mm Hg,
pulse is 170/min, and respirations are 38/min. Physical examination shows abdominal distension,
a large area of erythematous, warm, indurated abdominal wall, and decreased bowel sounds.
Abdominal x-ray film shows multiple dilated loops of bowel and gas in a linear pattern along the
thickened bowel wall. Free air is seen on lateral decubitus view. Which of the following is the
most appropriate next step in management?
 Incorrect Answer ImageA.Abdominal decompression
 Incorrect Answer ImageB.Broad-spectrum antibiotics with anaerobic coverage
 Incorrect Answer ImageC.Electrolyte repletion
 Incorrect Answer ImageD.Enteral feeding
 Incorrect Answer ImageE.Parenteral nutrition
 Incorrect Answer ImageF.Serial abdominal x-ray
 Correct Answer ImageG.Surgical resection

A 5-year-old girl from Connecticut is brought to the family clinic with a 3-day history of fever
and intermittent joint pain. She is generally healthy, but according to her mother, she had a
"cold" about 1 month ago. On physical examination, her temperature is 39.6°C (103.2°F), blood
pressure 94/60 mmHg, pulse 114/min, and respirations 22/min. Knees and elbow joints are
swollen, warm, and tender to palpation. There is a grade III/VI diastolic murmur best heard at the
apex. Multiple fine, pink macules are noted on her trunk. These macules blanch in the center.
Which of the following is the most likely diagnosis? 
 Incorrect Answer ImageA.Juvenile rheumatoid arthritis
 Incorrect Answer ImageB.Lyme disease
 Correct Answer ImageC.Rheumatic fever
 Incorrect Answer ImageD.Scarlet fever
 Incorrect Answer ImageE.Septic arthritis

A 17-year-old girl comes to the physician with her mother because she has not menstruated yet.
Her medical history is unremarkable and she takes no medication. She weighs 58 kg (128 lb) and
is 165 cm (65 in) tall. Breast, axillary, and pubic hair development are Tanner stage 4. Pelvic
examination shows normal-appearing external genitalia and a short vagina; no uterus is
palpable. There are no inguinal hernias or labial masses. Which of the following is the most
likely diagnosis? 
 Incorrect Answer ImageA.Androgen insensitivity
 Incorrect Answer ImageB.Gonadal dysgenesis
 Incorrect Answer ImageC.Imperforate hymen
 Incorrect Answer ImageD.Kallmann syndrome
 Correct Answer ImageE.Müllerian agenesis

A 5-week-old infant is brought to the physician by her parents because of a 4-week history of
noisy breathing. The parents report that the noisy breathing increases when the child is asleep
and when she cries. They also sometimes notice it with feeding, but she has been gaining weight
appropriately. In addition, she has had a low-grade fever and rhinorrhea for the past 4 days, and
the noisy breathing has subsequently worsened. Physical examination shows an infant in mild
distress with moderate inspiratory stridor. It is loudest in the supine position and lessens when
prone. Which of the following is the most likely diagnosis?
 Incorrect Answer ImageA.Bilateral choanal atresia
 Incorrect Answer ImageB.Croup
 Incorrect Answer ImageC.Foreign body obstruction
 Correct Answer ImageD.Laryngomalacia
 Incorrect Answer ImageE.Tuberculosis

A 3-month-old girl born to a couple from Nicaragua is brought to the physician for a well-child
visit. The infant appears well-nourished and her weight is at the 45th percentile of the typical
growth curve. Physical examination shows a blue-gray pigmentation over the buttocks. When
questioned further, the parents say that the pigmentation was present from the time of birth. The
rest of the physical examination is unremarkable. Which of the following is the next best step in
management?
 Incorrect Answer ImageA.File a report of suspected child abuse with state child-
protective services
 Incorrect Answer ImageB.Order CT/MRI scans to rule out dysraphism
 Incorrect Answer ImageC.Refer the patient to a dermatologist
 Incorrect Answer ImageD.Tell the parents that this is a bruise caused by trauma
 Correct Answer ImageE.Tell the parents that this is a typical finding

A 4-year-old boy is brought to the physician by his mother because of an extremely itchy rash
that has been present for 2 weeks. The rash is not relieved with topical calamine lotion. The
mother reports that the child goes to daycare and a similar rash was reported in one of the other
children. Both the mother and the patient's younger brother have a similar rash. His temperature
is 37°C (98.6°F). Examination is shown. There are multiple excoriated and crusted erythematous
papules on the elbows, abdomen, and buttocks. Which of the following is the most appropriate
next step in management?
 Incorrect Answer ImageA.Clotrimazole cream
 Incorrect Answer ImageB.Lindane cream
 Incorrect Answer ImageC.No treatment is necessary, as the disease will resolve
spontaneously
 Correct Answer ImageD.Permethrin cream
 Incorrect Answer ImageE.Precipitated sulfur in petrolatum
 Incorrect Answer ImageF.Topical steroids

A 5-year-old boy is brought to the physician for a well-child visit. He is developmentally


meeting all of his milestones, is growing well, and remains generally healthy. On physical
examination, he is a healthy appearing male, with the border of the hepatic edge 4 cm below the
right costal margin, and the spleen palpable 1–2 cm below the left costal margin. A complete
blood count shows:
WBC 3,670/mm3
Hgb 10.3 g/dL
Hematocrit 36%
Platelets 130,000/mm3
The pediatrician thinks of a possible diagnosis based on these findings and orders a skeletal
survey, which shows flaring of the distal femurs and scattered areas of osteosclerosis and lytic
lesions of the long bones. What would be the most likely cause of these findings? 
 Incorrect Answer ImageA.Alpha-mannosidosis
 Correct Answer ImageB.Gaucher disease (GD)
 Incorrect Answer ImageC.Hunter syndrome
 Incorrect Answer ImageD.Niemann-Pick disease, type A
 Incorrect Answer ImageE.Sanfilippo disease, type B

A 6 year-year-old girl is brought to the physician for a well-child visit. The family has just
immigrated to the United States from a foreign country, and, when asked about previous
immunizations, the mother states that she received them all but she lost the immunization record.
You explain that we cannot accept her statement as proof of immunizations and either she will
need positive titers or to be re-immunized for school entry. Which of the following are the
immunizations for her age that should be re-administered?
 Incorrect Answer ImageA.DTaP, rotavirus, meningococcal conjugate, Tdap, MMR,
varicella
 Incorrect Answer ImageB.DTaP, Hib, hepatitis B, MMR, PCV13, varicella
 Incorrect Answer ImageC.DTaP, Hib, hepatitis A, hepatitis B, PCV13, MMR, varicella
 Correct Answer ImageD.DTaP, hepatitis A, hepatitis B, IPV, MMR, varicella
 Incorrect Answer ImageE.Tdap, meningococcal conjugate, HPV

A 4-year-old African American boy is brought to the physician by his mother after she noticed
an area of hair loss on his scalp 3 days ago. Examination of the affected area is shown. The hair
in this area appear to be broken off right at the hair follicle, and appear as small black dots. There
is preauricular and suboccipital lymphadenopathy on the right side. Microscopy of a KOH
preparation of a hair from the border of the affected area shows chains of spores within the hair
shaft. Which of the following is the most appropriate next step in management?
 Correct Answer ImageA.Oral griseofulvin
 Incorrect Answer ImageB.Oral fluconazole
 Incorrect Answer ImageC.Oral itraconazole
 Incorrect Answer ImageD.Shampooing with 2.5% selenium sulfide
 Incorrect Answer ImageE.Topical clotrimazole

A 10-year-old boy is brought by his mother to the physician along with two younger siblings
because of a rash on his face, upper body, and arms for 1 day. He developed a sore throat, low-
grade fever, and headache 7 days ago. His medical history is unremarkable, and he is up to date
on his immunizations. Both siblings have had similar symptoms. His temperature is 37.2°C
(99.0°F). Photographs of patient’s rash and his younger siblings’ facial rashes are provided by
the mother and shown. The remainder of the examination shows no abnormalities. Which of the
following is the most likely cause of this patient's condition?
 Incorrect Answer ImageA.Group A beta-hemolytic streptococci
 Incorrect Answer ImageB.Human herpesvirus 6
 Incorrect Answer ImageC.Measles virus
 Incorrect Answer ImageD.Mumps virus
 Correct Answer ImageE.Parvovirus B19
 Incorrect Answer ImageF.Rubella virus

A 9-year-old boy is brought to the emergency room by his mother because he stated, "My heart
seems really fast and is pounding." This has been occurring for the past 3 hours. He has never
complained about this before. In fact, he has been healthy, having had only the typical acute
childhood illnesses, and has not even seen a doctor for the past 2 years. Upon auscultation, his
heart rate is about 200/min. He is easily converted to a sinus rhythm after his face is submerged
in iced saline for a few seconds. Further cardiac examination shows a grade 3/6 holosystolic
murmur over most of the anterior left chest, along with a gallop rhythm heard over the lower left
sternal border. An electrocardiogram is then performed at his normal heart rate and shows a short
P-R interval with a slow, curved upstroke of the R-wave. Which of the following is the most
likely diagnosis?
 Incorrect Answer ImageA.Atrial septal defect
 Correct Answer ImageB.Ebstein anomaly
 Incorrect Answer ImageC.Pulmonic stenosis
 Incorrect Answer ImageD.Tetralogy of Fallot
 Incorrect Answer ImageE.Tricuspid atresia
 Incorrect Answer ImageF.Tricuspid regurgitation

A 3-year-old boy is brought to the physician for a well-child visit. The child has no history of
serious illness and takes no medications. His immunizations are up to date. He is playful and
responds appropriately to conversation and interaction. Physical examination shows an
abdominal mass that extends 10 cm below the left costal margin. The remainder of the
examination shows no abnormalities. Laboratory studies show the following:
White blood cells  7,800/mm3
Hematocrit  45%
Platelets  289,000/mm3
INR  1.0
Partial thromboplastin time  17 seconds
A CT scan of the abdomen shows a 9 cm mass encasing the aorta and displacing the left kidney
inferiorly. An iodine-131-meta-iodobenzylguanidine (MIBG) scan shows increased uptake by
the mass. Which of the following is the most likely diagnosis?
 Incorrect Answer ImageA.Adrenal adenocarcinoma
 Incorrect Answer ImageB.Lymphoma
 Correct Answer ImageC.Neuroblastoma
 Incorrect Answer ImageD.Retroperitoneal sarcoma
 Incorrect Answer ImageE.Wilms tumor

A 15-year-old girl is brought to the physician by her aunt because of a rash for 2 days that started
on her face and then rapidly spread to her body. The rash does not itch. She had a low-grade
fever and sore throat 5 days ago. Her medical history is unremarkable, and she takes no
medications. She is visiting her aunt and uncle from Eastern Europe for the summer. She is
sexually active. Her temperature is 38.2ºC (100.8ºF), pulse is 90/min, and respirations are
18/min. Examination shows an erythematous macular and papular rash on the face and body.
There are punctate hemorrhagic spots on the soft palate. The retroauricular, posterior cervical,
and postoccipital lymph nodes are enlarged and slightly tender. There is moderate edema and
tenderness of the interphalangeal joints on her hands and feet. Which of the following tests is
most appropriate in this patient? 
 Incorrect Answer ImageA.Anti-Borrelia antibody titer
 Incorrect Answer ImageB.Antinuclear antibody titer
 Incorrect Answer ImageC.Rheumatoid factor titer
 Incorrect Answer ImageD.Striated muscle biopsy
 Correct Answer ImageE.Urine pregnancy test

A 14-year-old boy is brought to the emergency department because of 2 days of persistent mid-
epigastric pain radiating to his back. The pain has not changed. The boy has also had a fever with
nausea and vomiting that is worse when his temperature increases in the afternoon. He is not
sexually active. His current temperature is 39.2°C (102.5°F) and physical examination shows
marked upper abdominal tenderness with guarding and no palpable masses. Mild abdominal
distension is present with no audible bowel sounds. A complete blood count shows leukocytosis
and serum chemistry shows a normal amylase level. Which of the following is the most likely
diagnosis?
 Incorrect Answer ImageA.Fitz-Hugh-Curtis syndrome
 Incorrect Answer ImageB.Intussusception
 Correct Answer ImageC.Pancreatitis
 Incorrect Answer ImageD.Pyelonephritis
 Incorrect Answer ImageE.Wilms tumor

 10-year-old boy is brought to the emergency department because of fever, cough, and shortness
of breath for 2 days. He also complains of right-sided chest pain that is worse with inhalation.
There is no recent history of trauma and he last received vaccinations at age 6. Past medical
history is significant for a motor vehicle accident 2 years ago which necessitated abdominal
surgery with removal of the spleen due to severe bleeding. The boy also needed a cast for 4
weeks because of a wrist fracture. Temperature is 39ºC (102ºF), blood pressure is 105/65 mm
Hg, pulse is 92/min, and respirations are 36/min. Chest examination shows increased work of
breathing, decreased breath sounds and dullness to percussion on the right lung field. Chest x-ray
confirms the diagnosis. Which of the following bacterial agents is the most likely pathogen for
this patient's condition?
 Correct Answer ImageA.Haemophilus influenzae type B
 Incorrect Answer ImageB.Klebsiella pneumoniae
 Incorrect Answer ImageC.Moraxella catarrhalis
 Incorrect Answer ImageD.Neisseria meningitidis
 Incorrect Answer ImageE.Staphylococcus aureus

A 3-year-old boy is brought to the physician because of a headache and vomiting. The child's
past medical history is significant for surgery for undescended testes 1 year ago. He has also
been followed by a pediatrician for developmental delay. His blood pressure is 130/80 mm Hg.
On physical examination, he appears pale and is noted to have a large abdominal mass. Head
examination shows hypoplasia of both irises. Abdominal CT scan shows a highly vascularized
tumor in the upper pole of the right kidney. Which of the following is the most likely cause of
this presentation?
 Correct Answer ImageA.Deletion of a gene on chromosome 11
 Incorrect Answer ImageB.Fragile X syndrome
 Incorrect Answer ImageC.Neuroblastoma
 Incorrect Answer ImageD.Renal cell carcinoma
 Incorrect Answer ImageE.Translocation of chromosomes 9 and 22

A 6-year-old male is brought to the physician by his mother because of a 3-month history of
multiple episodes of sudden awakening at night. His mother states that when he wakes up
suddenly, he screams, "Go! Get away! Go!" and does not respond to his parents. His eyes are
wide open during these episodes, and he sweats heavily, breathes rapidly, looks scared, and has a
racing pulse. His parents have had to struggle to awaken him. After the episodes, he has no
memory of what happened. The boy is well adjusted to his school life, and appears to be
developing normally. Which of the following is the most likely diagnosis?
 Incorrect Answer ImageA.Acute stress disorder
 Incorrect Answer ImageB.Confusional arousals
 Correct Answer ImageC.Night terrors
 Incorrect Answer ImageD.Nightmares
 Incorrect Answer ImageE.Obstructive sleep apnea
 Incorrect Answer ImageF.Panic disorder

A 3 -year-old previously healthy boy is brought to a pediatric cardiologist after being referred to
him by his pediatrician for evaluation of a murmur. Cardiac examination shows an early systolic
ejection click best at the apex and upper right sternal border. Its intensity does not change with
respiration. This is followed by a grade 4/6 systolic ejection murmur, which radiates up the neck.
There is a thrill palpable in the suprasternal notch. He also hears a soft, early decrescendo
diastolic murmur best at the upper right sternal border. The first and second heart sounds are
normal, the apical impulse is not displaced, the peripheral pulses, and four-extremity blood
pressures are all normal. There are no lifts or heaves. An electrocardiogram and chest X-ray are
normal. The child is active and participates in normal activities without problems. The
echocardiogram will be diagnostic of which of the following conditions?
 Correct Answer ImageA.Bicuspid aortic valve
 Incorrect Answer ImageB.Congenital mitral stenosis
 Incorrect Answer ImageC.Critical aortic stenosis
 Incorrect Answer ImageD.Isolated aortic insufficiency
 Incorrect Answer ImageE.Pulmonic valve stenosis

A 17-year-old boy comes to the physician because of a lesion on his penis for the past 10 days.
He has had no fever or pain on his genitalia. He has had 2 infections with gonorrhea and genital
warts in the past 2 years. He has no allergies to medications. He is sexually active with 3 partners
and he uses condoms infrequently The lesion is non-tender and multiple enlarged lymph nodes
are present in the bilateral inguinal regions. A photograph of the penis is shown. Which of the
following is the most appropriate next step in management?
 Correct Answer ImageA.Intramuscular benzathine penicillin G
 Incorrect Answer ImageB.Intramuscular ceftriaxone
 Incorrect Answer ImageC.Intravenous crystalline penicillin G
 Incorrect Answer ImageD.Oral acyclovir
 Incorrect Answer ImageE.Oral doxycycline

A 6-year-old boy is brought to the physician by his mother because of pain in his left knee for 2
days. One week ago, he had pain in his right ankle. The mother reports that he had a red rash on
his leg about 2 months ago in association with a low-grade fever, fatigue, muscle pain, and
headache that resolved after 2 weeks. His past medical history is otherwise unremarkable. The
mother states that the family went camping and hiking in wooded areas of Wisconsin
approximately 2–1/2 months ago. His temperature is 37.1°C (98.8°F), pulse is 80/min,
respirations are 20/min, and blood pressure is 100/60 mm Hg. Examination shows edema and
mild tenderness of the left knee; range of motion is within normal limits. Which of the following
is the most appropriate next step in management?
 Incorrect Answer ImageA.Amoxicillin
 Incorrect Answer ImageB.Arthrocentesis
 Incorrect Answer ImageC.Doxycycline
 Incorrect Answer ImageD.Prednisone
 Correct Answer ImageE.Serologic testing

A 17-year-old girl comes to the physician because of a rash that has been rapidly spreading over
her neck and chest for 3 weeks. It began as a small red bump that grew larger with central
clearing. Similar lesions appeared and spread in the same way. She has had no fever, fatigue, or
hair loss. Her medical history is unremarkable, and she takes no medications. She has several
pets including a turtle, a hamster, and a kitten. Examination shows multiple annular
erythematous plaques with central clearing on the lateral neck and anterior chest; the lesions
have scales and vesicles at the border. Some of the lesions have become confluent. Which of the
following is the most appropriate next step in management?
 Incorrect Answer ImageA.Bacterial culture
 Incorrect Answer ImageB.Diascopy
 Correct Answer ImageC.Potassium hydroxide examination
 Incorrect Answer ImageD.Skin biopsy
 Incorrect Answer ImageE.Wood's light examination

A healthy 1-month-old boy born at term undergoes circumcision. Three hours later, there is a
moderate amount of blood on the diaper. Neither of the parents have a history of bleeding
diathesis. Laboratory studies show:
Hemoglobin  14.1 gm/dL
Platelets  90,000/mm3
White blood cells  4,600/mm3
Peripheral smear  Giant platelets
PT  14 seconds
PTT  36 seconds
Bleeding time  11 minutes (normal, 2-7 minutes)
Platelet aggregation studies show:
Ristocetin  0 (no response)
Ristocetin response is not corrected by the addition
of normal plasma 
ADP ++++ (normal
response)
Epinephrine  ++++ (normal
response)
Which of the following is the most likely diagnosis? 
 Correct Answer ImageA.Bernard-Soulier syndrome
 Incorrect Answer ImageB.Glanzmann thrombasthenia
 Incorrect Answer ImageC.Immune thrombocytopenic purpura
 Incorrect Answer ImageD.von Willebrand disease
 Incorrect Answer ImageE.Wiskott-Aldrich syndrome

A 3-year-old boy is admitted to the hospital with the diagnosis of aspiration pneumonia. He has
had problems eating due to swallowing abnormalities for the last year. He also has a generalized
seizure disorder, which is being treated with 2 medications, and has not had a seizure for 4
months after adjustment of the medications. His past medical history is significant for being born
at 26 weeks’ gestation with a birth weight of 760 g (1.68 lb). He was appropriate for gestational
age with respect to length, weight, and head circumference. Apgar scores were 5 and 7 at 1 and 5
minutes, respectively. He started wearing glasses 1 year ago. He has global developmental delay.
His temperature is 38.2ºC (100.8ºF), respirations are 36/min, pulse is 120/min, and blood
pressure is 100/60 mm Hg in the right arm supine. He has increased tone, rigidity, and spasticity
of all four extremities; 4+ deep tendon reflexes; and bilateral plantar extensor responses with
clonus. He also has flexion contractures of his knees, tight heel cords, and tightly adducted hips.
In addition to treatment for aspiration pneumonia, his physician ordered an MRI of the brain.
What of the following is the MRI most likely to show?
 Incorrect Answer ImageA.Agenesis of the corpus callosum
 Incorrect Answer ImageB.Basal ganglia abnormalities
 Incorrect Answer ImageC.Multicystic subcortical encephalomalacia
 Incorrect Answer ImageD.Normal anatomy
 Correct Answer ImageE.Periventricular leukomalacia

A 2-month-old male infant is brought into the emergency department by his parents because of
projectile, non-bilious vomiting after every feeding attempt. He has been having normal bowel
movements. The parents are from an underdeveloped country where there was minimal prenatal
monitoring, and their child was born 6 weeks premature as a result. On physical examination, the
infant has sunken fontanelles, dry mucous membranes, and increased skin tenting. On deep
palpation of the abdomen, there is a reducible, olive-shaped, 2 x 2 cm mass on the right upper
quadrant. Laboratory results are as follows:
Na+ 145 mEq/L
K +
3.3 mEq/L
Cl- 94 mEq/L
HCO3 -
38 mEq/L
BUN 20 mg/dL
Creatinine 1.0 mg/dL
Glucose 60 mg/dL
pH 7.50
PaCO2 40 mm Hg
PaO2 90 mm Hg
Which of the following is the most appropriate next step in management?
 Incorrect Answer ImageA.Abdominal sonogram
 Incorrect Answer ImageB.Endoscopy
 Incorrect Answer ImageC.Hydrochloric acid replacement
 Correct Answer ImageD.Intravenous 0.9% NaCl solution
 Incorrect Answer ImageE.Laparotomy with pyloromyotomy

An 11-year-old boy is brought to the physician by his father because of a fever and sore throat
for the past 3 days. His temperature is 37.7°C (100.0°F), blood pressure is 92/58 mm Hg, pulse is
84/min, and respirations are 16/min. Physical examination shows mild anterior cervical
lymphadenopathy, posterior oropharyngeal exudates, enlarged erythematous tonsils, and palatal
petechiae. In the past, he has had difficulty completing a full course of medications and he is
leaving for summer camp in 2 days. He has a history of eczema and asthma but has no allergies
to medications. After confirming the diagnosis, which of the following is the best treatment for
this patient?
 Correct Answer ImageA.A single dose of benzathine penicillin G intramuscularly
 Incorrect Answer ImageB.A single dose of ceftriaxone intramuscularly
 Incorrect Answer ImageC.A single dose of procaine penicillin G intramuscularly
 Incorrect Answer ImageD.Erythromycin orally for 5 days
 Incorrect Answer ImageE.Penicillin V orally for 5 days

A 4-year-old boy is brought to the emergency department after he and his mother were in an
automobile accident. His mother was also injured and was stabilized at the scene. She told the
paramedics that her son was riding in the front seat with a seat belt on when she slammed on the
brakes because another car went through a red light. She hit the car and he flew forward hitting
his head on the dashboard. On admission, there is bruising and edema of the right frontoparietal
area of the head. His temperature is 37ºC (98.6ºF), blood pressure is 125/82 mm Hg, pulse is
56/min, and respirations are 6/min with shallow, irregular breaths. A rapid trauma evaluation is
undertaken, and the pediatric Glasgow Coma Scale (GCS) is 4; 1 for eye opening, 1 for verbal
response, and 2 for motor response. He then has a generalized seizure. Which of the following is
the most appropriate next step in management?
 Incorrect Answer ImageA.Administer midazolam intramuscularly
 Incorrect Answer ImageB.Perform an emergency computerized tomographic (CT) scan
of the head
 Incorrect Answer ImageC.Place an intravenous line and administer dexamethasone
 Incorrect Answer ImageD.Place an intravenous line and administer lorazepam
 Correct Answer ImageE.Rapid sequence intubation

A 2-month-old infant is brought to a pediatric cardiologist. The infant was noted at birth to have
an upper left sternal border ejection murmur. At that time the infant was not cyanotic, but she
slowly developed cyanosis over the next 2 months. Electrocardiogram shows right axis deviation
and right ventricular hypertrophy. An x-ray of the chest shows a small heart with a concave main
pulmonary artery segment and diminished pulmonary blood flow. Surgical repair is
recommended. Which of the following is the most likely diagnosis?
 Incorrect Answer ImageA.Ebstein anomaly
 Incorrect Answer ImageB.Hypoplastic left ventricle
 Incorrect Answer ImageC.Isolated atrial septal defect
 Correct Answer ImageD.Tetralogy of Fallot
 Incorrect Answer ImageE.Transposition of the great arteries

A 4-year-old girl is brought to the physician by her mother because the girl recently spent the
summer with her grandfather who was diagnosed with tuberculosis one week ago. Her mother
denies that the patient has had fever, cough, or night sweats. Her medical history is
unremarkable. She was born in the United States and has not traveled outside of the country. Her
temperature is 36.8°C (98.2°F). Physical examination shows no abnormalities. A chest x-ray
shows no abnormalities. Tuberculin skin test with purified protein derivative shows 4 mm of
induration. Which of the following is the most appropriate next step in management? 
 Incorrect Answer ImageA.Ethambutol, isoniazid, pyrazinamide, and rifampin
 Incorrect Answer ImageB.Interferon-gamma release assay
 Correct Answer ImageC.Isoniazid and rifapentine
 Incorrect Answer ImageD.No further intervention
 Incorrect Answer ImageE.Repeat tuberculin skin test in 8 weeks
 Incorrect Answer ImageF.Rifampin

A 5-year-old boy is brought to the physician by his parents after they noticed two ticks on his
legs. The parents, the boy, and his two sisters returned from a 1-week camping trip 1 day ago.
The parents believe that the ticks have been on the boy's legs for less than 24 hours. None of the
other family members have found any ticks on their bodies. The boy is in no acute distress. His
temperature is 36.9°C (98.4°F). Examination shows two moderately engorged ticks on the lower
extremities. There is no erythema, edema, or tenderness. There is no rash. The ticks are carefully
removed with forceps. Which of the following is the most likely complication of infection
transmitted by the ticks? 
 Incorrect Answer ImageA.Acrodermatitis chronica atrophicans
 Incorrect Answer ImageB.Erythema ab igne
 Incorrect Answer ImageC.Erythema annulare centrifugum
 Incorrect Answer ImageD.Erythema infectiosum
 Correct Answer ImageE.Erythema migrans

A 4-year-old girl is brought to the physician by her mother because of a progressive bloody,
greenish, malodorous vaginal discharge over 3 days. The girl has no other symptoms. The
mother reports no concerns regarding abuse of the child. Pelvic examination is attempted but
impossible because of the child's absolute refusal to be examined. Several efforts at persuasion
are made but are unsuccessful. Besides the refusal to be examined, the child seems happy and
well adjusted. Which of the following is the most appropriate next step in management? 
 Incorrect Answer ImageA.Antibiotic treatment
 Correct Answer ImageB.Examination under anesthesia
 Incorrect Answer ImageC.Notification of child protection services
 Incorrect Answer ImageD.Pelvic examination with physical restraints
 Incorrect Answer ImageE.Reassurance and expectant management

A 4-week-old boy is brought to the physician because of a 5-day history of forceful vomiting. He
has been vomiting part way through every feeding. The vomitus contains mostly undigested
formula and is nonbilious. Examination shows dry oral mucosa, depressed anterior fontanelle,
and capillary refill time of 3 seconds. Abdominal examination shows a 2 cm mass in the
epigastrium. The remainder of the examination is unremarkable. Which of the following findings
(electrolytes in mEq/L) will most likely be seen?
Na Cl K HCO3 PCO2

A. 124 82 2.8 34 42 mm Hg

B. 150 115 5.5 10 30 mm Hg

C. 125 84 4.9 14 35 mm Hg

D. 140 105 4.3 23 41 mm Hg

E. 137 100 3.1 32 30 mm Hg


 Correct Answer ImageA.A.
 Incorrect Answer ImageB.B.
 Incorrect Answer ImageC.C.
 Incorrect Answer ImageD.D.
 Incorrect Answer ImageE.E.

A 20-month-old girl is brought to the physician because of 2-day history of fever and pulling at
her ear. The patient had an episode of acute otitis media 2 months ago and another one 5 months
ago. She has no allergies to medications. Her temperature is 38.9°C (102°F). The lungs are clear
to auscultation. She is coughing and there is some upper airway congestion. Pneumatic otoscopy
shows erythema of the right tympanic membrane with no mobility on pneumatic otoscopy. The
left tympanic membrane is normal-appearing. Which of the following is the most appropriate
course of pharmacotherapy? 
 Incorrect Answer ImageA.A 5-day course of oral azithromycin
 Incorrect Answer ImageB.A 5- to 7-day course of oral amoxicillin
 Correct Answer ImageC.A 10-day course of high-dose amoxicillin
 Incorrect Answer ImageD.A 10-day course of oral cefuroxime axetil
 Incorrect Answer ImageE.An intramuscular injection of ceftriaxone

A 9-year-old child is brought to the emergency department by his grandmother because he


suddenly developed a fever with shallow breathing, vomiting, and mental confusion. The child
had come to visit her over the holidays and had been perfectly well before this. He has never had
any problems with his health except for a skin condition present from birth that she describes as
"alligator skin." When asked if she had given the child any unusual medication or remedy she
admits to having applied a homemade paste to his skin which was supposed to get rid of the
heavy scale. She states that there have been several affected family members in previous
generations and this remedy has been used to treat them without ever having caused problems.
She also insists that it could not have been the cause of any illness because it was really just a
bunch of aspirin tablets crushed and mixed with petrolatum. His temperature is 39ºC (102.2ºF),
pulse is 120/min, and respirations are 35/min. He has vomited twice while in the emergency
department and appears to be dehydrated and lethargic. Blood glucose is 85 mg/dL and serum
pH is 6.9. Which of the following is the most appropriate next step in the management?
 Incorrect Answer ImageA.Activated charcoal by mouth
 Incorrect Answer ImageB.Gastric lavage
 Correct Answer ImageC.IV bicarbonate
 Incorrect Answer ImageD.IV 5% dextrose in saline
 Incorrect Answer ImageE.Oral N-acetylcysteine

A 6-year-old boy with a history of deafness is brought to the emergency department after
collapsing at home. He was running after one of his three siblings when he suddenly turned pale
and fell to the floor. He subsequently recovered in the ambulance. In the emergency room, he
appears pale. The parents are concerned and state that the boy is not taking any medication.
There was a maternal uncle who also experienced similar episodes but died at an early age. An
electrocardiogram is performed and shows marked QT prolongation and morphologic
abnormalities of the T waves. Within 1 minute, the patient again loses consciousness and the
electrocardiogram shows an irregular polymorphic ventricular tachycardia that appears to
undulate about the isoelectric line. Which of the following is the most likely diagnosis?
 Correct Answer ImageA.Jervell-Lange-Nielsen syndrome
 Incorrect Answer ImageB.Ortner syndrome
 Incorrect Answer ImageC.Romano-Ward syndrome
 Incorrect Answer ImageD.Von Hippel-Lindau syndrome
 Incorrect Answer ImageE.Wolff-Parkinson-White syndrome
A 3,402 g (7.5 lb) female newborn is delivered at 40 weeks' gestation to a 29-year-old woman,
gravida 3, para 2. Apgar scores are 3 and 4 at 1 and 5 minutes, respectively. During labor, the
fetal heart monitor showed late decelerations and loss of short- and long-term variability. The
membranes were ruptured to expedite the delivery, and the amniotic fluid contained meconium.
The infant is delivered 45 minutes later. The infant appears cyanotic and limp. Examination
shows poor tone; deep tendon reflexes are absent. Moro reflex is absent. Ten hours later, the
neonate has a seizure. Which of the following is the most likely cause of this patient's condition? 
 Correct Answer ImageA.Encephalopathy from asphyxia
 Incorrect Answer ImageB.Inborn error of metabolism
 Incorrect Answer ImageC.Respiratory distress
 Incorrect Answer ImageD.Subarachnoid hemorrhage
 Incorrect Answer ImageE.Werdnig-Hoffman disease

A few hours after delivery, a newborn male develops the inability to feed. He was born at 37
weeks' gestation to a 20-year-old G1P1 woman via vaginal delivery. Pregnancy was complicated
by polyhydramnios. While being formula-fed for the first time, he immediately begins to cough
and spit up the formula. Feeding is reattempted and the newborn now begins to froth at the nose
and mouth. An abdominal x-ray film shows a gasless abdomen. Which of the following is the
most likely diagnosis?
 Incorrect Answer ImageA.Duodenal atresia
 Correct Answer ImageB.Esophageal atresia (Type A without tracheoesophageal fistula)
 Incorrect Answer ImageC.Esophageal atresia with distal tracheoesophageal fistula (Type
C)
 Incorrect Answer ImageD.Malrotation with volvulus
 Incorrect Answer ImageE.Tracheoesophageal fistula (Type E)

A 14-year-old boy is brought to the physician because of a 2-week history of a tense lump on the
left side of his neck that has been causing increasing discomfort. He also has had fatigue and
muscle aches. His medical history is unremarkable. He is not sexually active. He has a cat at
home. His grandfather who lives nearby has a history of lymphoma and recent infection with
tuberculosis. His temperature is 37.8°C (100°F). Examination shows a tender, erythematous 3
cm mass at the left anterior cervical region. There is a line of three 5 mm papules on the left
forearm. Which of the following is the most likely cause of the findings? 
 Incorrect Answer ImageA.Actinomyces israelii
 Correct Answer ImageB.Bartonella henselae
 Incorrect Answer ImageC.Epstein-Barr virus
 Incorrect Answer ImageD.Francisella tularensis
 Incorrect Answer ImageE.Human immunodeficiency virus
 Incorrect Answer ImageF.Mycobacterium tuberculosis
 Incorrect Answer ImageG.Non-tuberculous Mycobacteria
 Incorrect Answer ImageH.Pasteurella multocida
 Incorrect Answer ImageI. Staphylococcus aureus 
 Incorrect Answer ImageJ.Toxoplasma gondii
A 16-year-old boy is brought to the emergency department by his mother after he has "a fit." His
mother says that she first saw him fall abruptly; his entire body was then stiff for a while and
then started jerking for about 5 minutes. Then he fell asleep and woke up slowly with a
headache. He also reports joint pain that is most prominent in the hands and is worst in the
morning. He has no significant past medical history and takes no medications. His temperature is
37.3ºC (99.1ºF), pulse is 80/min, respirations are 16/min, and blood pressure is 140/90 mm Hg.
Physical examination of the skin shows diffuse erythema and edema of the nose and cheeks.
When questioned about the rash, the patient states that it started when he was on vacation at the
beach 2 weeks ago and does not seem to be improving. The proximal and distal interphalangeal
joints of the hands are found to be edematous and tender on examination. On the nasal septum,
there are several shallow ulcers up to 1 cm in diameter. CBC with differential, serum
electrolytes, and urinalysis are normal. Which of the following is the most specific test to
diagnose the primary cause of this patient's symptoms?
 Incorrect Answer ImageA.CT scan of the brain
 Incorrect Answer ImageB.Drug testing for cocaine abuse
 Incorrect Answer ImageC.Electroencephalogram
 Incorrect Answer ImageD.Rheumatoid factor
 Correct Answer ImageE.Serum anti-dsDNA antibody
 Incorrect Answer ImageF.Serum antinuclear antibody

A 4,000-g male neonate develops severe cyanosis that begins within minutes of birth. Blood
drawn 1 hour after birth shows metabolic acidosis with respiratory acidosis. An x-ray of the chest
shows a narrow mediastinum, narrow heart base, and absence of the pulmonary artery. An ECG
shows no abnormalities. An echocardiogram is ordered and the report is pending. Which of the
following is the most likely diagnosis?
 Incorrect Answer ImageA.Aortic valve stenosis
 Incorrect Answer ImageB.Complete atrioventricular canal defect
 Incorrect Answer ImageC.Tetralogy of Fallot
 Correct Answer ImageD.Transposition of the great arteries
 Incorrect Answer ImageE.Underdeveloped (hypoplastic) left ventricle syndrome

A 5-year-old boy is brought to the emergency department by his parents because of a headache,
cough, myalgia, and a fever. His parents report that he was given a decongestant and a painkiller
by his grandmother who had been babysitting him this afternoon. He is otherwise healthy and his
immunizations are up to date. He is treated in the emergency department with supportive care
and is sent home. Four days later, he is brought to the emergency department again because of
lethargy, persistent vomiting, and irritability. He appears to be somnolent and becomes
combative on stimulation. Which of the following laboratory studies is most likely to assist in
diagnosing this patient? 
 Correct Answer ImageA.Serum ammonia level
 Incorrect Answer ImageB.Serum blood urea nitrogen level
 Incorrect Answer ImageC.Serum calcium level
 Incorrect Answer ImageD.Serum opiate level
 Incorrect Answer ImageE.Serum sodium level

An 8-year-old boy is brought to the pediatrician's office because of bloody bowel movements.
His mother reports that over the past 3 weeks, 2 of his stools were dark red. The boy denies any
pain or diarrhea and his mother confirms this.Family history is unremarkable. The boy is well-
appearing and in no distress. His vital signs are within normal limits; abdominal and rectal
examinations are unremarkable. Which of the following is the most appropriate next step in
diagnosis?
 Incorrect Answer ImageA.Barium enema
 Incorrect Answer ImageB.Colonoscopy
 Incorrect Answer ImageC.Tagged red blood cell scan
 Correct Answer ImageD.Technetium-99m pertechnetate scan
 Incorrect Answer ImageE.Upper GI endoscopy

A 10-year-old girl is brought to the physician by her father because of fever and pain in her right
knee for 1 day. Two weeks ago, she had a fever, a sore throat, and difficulty swallowing that
lasted 7 days and resolved with over-the-counter analgesics. She is in moderate distress. Her
temperature is 38.7ºC (101.6ºF), pulse is 110/min, and respirations are 24/min. Examination
shows an erythematous pharynx, edematous tonsils, and a yellowish-white exudate. She limps
when she walks and grimaces with every step. The right knee is swollen, erythematous, warm,
and tender to touch. Range of motion is limited by severe pain. Which of the following is the
most appropriate study to confirm the suspected diagnosis? 
 Correct Answer ImageA.Arthrocentesis
 Incorrect Answer ImageB.CBC
 Incorrect Answer ImageC.Pharyngeal culture
 Incorrect Answer ImageD.Radiograph of the knee
 Incorrect Answer ImageE.Ultrasonography

A 3-year-old child whose parents immigrated from a country with limited medical resources 6
months ago is found to have a 16 mm induration on his arm 48 hours after the administration of
purified protein derivative (PPD). He is prescribed isoniazid and sent home. Two days later, the
parents are concerned because the child is not as active as he was before. He does not want to eat
or play with his toys. His temperature is 37.1ºC (98.7ºF), pulse is 120/min, and respirations are
20/min. Cardiac examination shows a systolic ejection murmur on cardiac and his lungs are clear
to auscultation. His abdomen is soft and nontender. Skin examination shows jaundice noted on
the upper anterior thorax, and he has conjunctival icterus. He is confused on neurological
examination but does not have any focal findings. Laboratory studies show:
Hemoglobin  8 g/dL
Hematocrit  22%
Platelets  180,000/mm3
White blood cells  6,500/mm3
MCV  85 μm3
MCHC  34 pg/cell
Reticulocytes  5%
PT  10 seconds
PTT  35 seconds
Total bilirubin  3 mg/dL
Direct bilirubin  0.1 mg/dL
Haptoglobin  Low
UA  Hemoglobin +
Peripheral smear  Presence of bite cells
G6PDH level  Normal
Direct Coombs’ test  Negative
Which of the following is the most likely diagnosis? 
 Incorrect Answer ImageA.Drug-induced hemolytic anemia
 Correct Answer ImageB.Glucose-6-Phosphate dehydrogenase deficiency
 Incorrect Answer ImageC.Hereditary elliptocytosis
 Incorrect Answer ImageD.Phosphofructokinase deficiency
 Incorrect Answer ImageE.Pyruvate kinase deficiency

A male infant was born at 32 weeks’ gestational age following an uncomplicated spontaneous
vaginal delivery. The mother had regular prenatal care with negative serologies and moderate
control of pre-existing diabetes. She is group B streptococci (GBS) negative, and her blood
group is A+. Apgar scores were 7 and 9 at 1 and 5 minutes, respectively. Four hours after birth,
the child has episodes of right-sided facial and arm twitching with rapid eye movement lasting 3-
5 minutes each. His temperature is 36.8ºC (98.2ºF), pulse is 140/min, respirations are 60/min,
blood pressure is 90/50 mm Hg, and birth weight 1830 g (4 lb 0.5 oz). Physical examination,
including neurologic evaluation, is normal, and there are no dysmorphic features evident. Which
of the following is the most appropriate first step in diagnosis?
 Incorrect Answer ImageA.Blood cultures
 Incorrect Answer ImageB.Electroencephalogram
 Incorrect Answer ImageC.Lumbar puncture
 Incorrect Answer ImageD.MRI scan of the head
 Correct Answer ImageE.Serum glucose level
 Incorrect Answer ImageF.Ultrasound of the head

A 17-year-old girl with a history of Hashimoto thyroiditis is brought to the emergency


department by her parents because of yellow skin discoloration and abdominal pain for the past 3
days. Serum total bilirubin level is 8 mg/dL and direct bilirubin 5 mg/dL. Abdominal ultrasound
shows a dilated common bile duct and multiple stones within the gallbladder. Endoscopic
retrograde cholangiopancreatography (ERCP) is performed and a gallstone is extracted from the
common bile duct. Seven days later, she returns to the physician for a follow-up. She denies any
abdominal pain and her skin discoloration is greatly improved. Abdominal examination shows
no rebound tenderness or hepatosplenomegaly, but she does have mild tenderness in the right
upper quadrant. Repeat laboratory studies show a total bilirubin of 2.5 mg/dL and direct bilirubin
1.5 mg/dL. Both AST and ALT are within normal limits. Which of the following is the next best
step in management? 
 Incorrect Answer ImageA.Evaluate for autoimmune hepatitis
 Incorrect Answer ImageB.Initiate treatment with cholic acid
 Incorrect Answer ImageC.Obtain hepatic B serologic studies
 Incorrect Answer ImageD.Repeat the ERCP
 Correct Answer ImageE.Set up consultation to a pediatric surgeon regarding
cholecystectomy

A 6-year-old boy is brought to the emergency department immediately after having a seizure that
lasted 10 minutes. He has been having fevers, vomiting, and ear pain for the last week. He
developed a severe headache earlier today. Current medications include high dose amoxicillin of
90 mg/kg for 3 days. The patient appears post-ictal and lethargic. His temperature is 39.5°C
(103°F), pulse is 120/min, respirations are 26/min, and blood pressure is 128/80 mm Hg.
Otoscopy shows an erythematous left tympanic membrane with a perforation, and purulent
material in the external ear canal. Funduscopic examination shows elevation of the left optic
disc, with obscuring of blood vessels at the disc margin. Which of the following is the most
appropriate next step in management?
 Incorrect Answer ImageA.Blood cultures
 Correct Answer ImageB.CT scan of the head
 Incorrect Answer ImageC.EEG studies
 Incorrect Answer ImageD.Lumbar puncture
 Incorrect Answer ImageE.Myringotomy

A newborn girl is born at 38 weeks' gestation to a healthy 36-year-old, G3P2 woman. The
mother did not receive routine prenatal care. Physical examination of the newborn shows
hypotonia, a poor Moro reflex, low set ears, enlarged tongue, upslanting palpebral fissures, and
deflection of the fifth fingers. Which of the following is the most likely cause of this infant’s
underlying diagnosis?
 Incorrect Answer ImageA.Alcohol abuse by the mother
 Incorrect Answer ImageB.Extra copy of autosome 18
 Correct Answer ImageC.Extra copy of autosome 21
 Incorrect Answer ImageD.Missing X chromosome
 Incorrect Answer ImageE.Mutation in fibrillin gene

A 2-month-old boy is brought to the office because of chronic constipation and failure to thrive
since birth. He has had infrequent small bowel movements since birth, usually no more than once
a week and with great difficulty and crying. The mother tried changing to formula instead of
breastfeeding, hoping it would help, but this seemed to make the boy's problem worse. Her
pregnancy was uneventful and she delivered vaginally at full term without complications. He
weighed 2,900 g at birth and currently weighs 3,500 g. The infant did not pass meconium until
the third day and had several episodes of vomiting in the first few weeks of life. On physical
examination, the infant is not in distress and is in the thirtieth percentile for length and the fifth
percentile for weight. His abdomen is mildly distended and nontender to palpation. The anal
sphincter tone is increased and the rectal vault is empty of stool. Upon performing a digital
examination, there is a sudden release of dark stool. Which of the following is the next best step
in management for this patient? 
 Incorrect Answer ImageA.CT of the abdomen
 Incorrect Answer ImageB.Daily enemas

 Incorrect Answer ImageC.Dietary changes

 Incorrect Answer ImageD.Genetic counseling and sweat test

 Correct Answer ImageE.Rectal biopsy

 Incorrect Answer ImageF.Surgical bowel resection 

A 5-year-old girl is brought to the physician by her mother because of anal itching for 2 weeks.
The mother reports that the girl wakes up at night crying and scratching her buttocks and anus.
She has had no fever, weight loss, nausea, vomiting, or diarrhea. The girl recently started
kindergarten. She appears well-nourished. The abdomen is nontender. Genital examination
shows no abnormalities. Microscopy of clear scotch tape pressed against the perianal skin shows
bean-shaped eggs. Which of the following is the most appropriate pharmacotherapy? 
 Incorrect Answer ImageA.Ivermectin
 Incorrect Answer ImageB.Ketoconazole
 Correct Answer ImageC.Mebendazole
 Incorrect Answer ImageD.Metronidazole
 Incorrect Answer ImageE.Praziquantel

A 10-day-old girl is brought to the physician by her parents because of yellowish skin
discoloration, which they noted over the past 2-3 days. She was born at 39 weeks' gestation to a
29-year-old woman, gravida 1, para 1, whose blood type is A+. Birth weight was 3,600 g (7 lb
15 oz). Apgar scores were 9 and 9 at 1 and 5 minutes, respectively, and the baby and mother
were discharged after 24 hours. Nursing notes state that breastfeeding was going well for the first
day. She had her 72-hour postnatal pediatric visit, and everything was noted to be well at that
time. Weight at that visit was 3,520 g (7 lb 12 oz). Physical examination shows jaundice down to
the mid-thoracic area, and she is otherwise pink and well-perfused. Temperature is 37.5°C
(99.5°F), respirations are 30/min, pulse is 120/min, and blood pressure is 65/40 mm Hg.
Examination of the heart and lungs are normal. Liver is palpated 1 cm below the right costal
margin and a spleen tip is just palpable. There are no known blood or liver diseases on either side
of the family. Which of the following is the most likely cause of this infant's condition?
 Correct Answer ImageA.Breast milk jaundice
 Incorrect Answer ImageB.Breastfeeding jaundice
 Incorrect Answer ImageC.Glucose-6-phosphate dehydrogenase (G6PD) deficiency
 Incorrect Answer ImageD.Physiologic jaundice
 Incorrect Answer ImageE.Rh incompatibility
A 3-year-old boy is brought to the emergency department by his mother because of irritability
and lethargy. The child has had progressively worsening diarrhea for the past 3 days, which has
been treated with oral fluid rehydration. He is afebrile, blood pressure is 70/42 mm Hg, pulse is
160/min, and capillary refill is greater than 3 seconds. His weight is 20 kg (44 lb). Laboratory
studies show a serum sodium level of 163 mEq/L. An intravenous line is inserted and he is
administered 1,200 mL of normal saline over 30 minutes. He requires another 20 mL/kg normal
saline bolus to achieve hemodynamic stability. Which of the following is the best subsequent
fluid management for this patient? 
 Incorrect Answer ImageA.Give all of the fluid deficit only over 24 hours and do not
correct the maintenance
 Incorrect Answer ImageB.Give him calculated fluid deficit plus his maintenance fluids
over 24 hours
 Correct Answer ImageC.Give him calculated solute and free-water deficits with daily
maintenance slowly over 48 hours
 Incorrect Answer ImageD.Give the maintenance fluids only for 48 hours then correct the
deficit over the next 24 hours
 Incorrect Answer ImageE.Give the maintenance fluids over 24 hours and then correct the
calculated deficit over the next 24 hours

A 4-year-old boy with congenital hydrocephalus is brought to the emergency department because
of fever, headache, irritability, lethargy, photophobia, and vomiting for 2 days. He underwent
surgery for placement of a ventriculoperitoneal shunt 1 year ago. His childhood immunizations
are up to date. His temperature is 39.6ºC (103.2ºF). Examination shows nuchal rigidity. Kernig
and Brudzinski signs are present. The skin overlying the shunt tract is erythematous. A lumbar
puncture and cerebrospinal fluid analysis shows a leukocyte count of 40,000/mm3 with 85%
neutrophils, a glucose level of 48 mg/dL, and a protein level of 169 mg/dL. Which of the
following is the most likely cause of this patient's condition? 
 Incorrect Answer ImageA.Haemophilus influenzae
 Incorrect Answer ImageB.Neisseria meningitidis
 Incorrect Answer ImageC.Pseudomonas aeruginosa
 Correct Answer ImageD.Staphylococcus epidermidis
 Incorrect Answer ImageE.Streptococcus pneumoniae

A 34-year-old woman, gravida 1, para 0, at 40 weeks' gestation is brought to the emergency


department because of spontaneous rupture of membranes. In the delivery room, thin meconium
is seen within the ruptured membranes. Labor progresses normally and spontaneous vaginal
delivery occurs. Immediately upon delivery, the newborn begins to cry vigorously and appears to
have good muscle tone. Which of the following is the most appropriate next step in management
of the newborn?
 Incorrect Answer ImageA.Check the heart rate
 Correct Answer ImageB.Dry the infant while on the mother's chest
 Incorrect Answer ImageC.Intubate and perform tracheal suctioning
 Incorrect Answer ImageD.Provide bag and mask ventilation
 Incorrect Answer ImageE.Provide blow-by oxygen

A newborn girl is born at 39 weeks' gestation to an obese 40-year-old woman with a past medical
history of hypothyroidism and cocaine use for the past 5 years. Physical examination of the
newborn shows an open neural tube defect. Further evaluations show a right-sided double ureter
and a moderate size membranous ventricular septal defect. Which of the following maternal
conditions would be consistent with the findings in the newborn? 
 Incorrect Answer ImageA.Cocaine abuse
 Correct Answer ImageB.Diabetes mellitus
 Incorrect Answer ImageC.Hypothyroidism
 Incorrect Answer ImageD.Iodine deficiency
 Incorrect Answer ImageE.Syphilis

A 3-year-old boy is admitted to the hospital because of fever, cough, and shortness of breath for
10 days. He has a history of a liver abscess and a perirectal abscess, both caused
by Staphylococcus aureus. Temperature is 38.8°C (101.8°F). Examination shows
hepatosplenomegaly. There are crackles in the left lung fields. An x-ray film of the lung shows a
3-cm abscess in the left lower lobe. The abscess is aspirated. Cultures of the aspirate
grew Serratia marcescens. Which of the following is the best initial test to support the
diagnosis? 
 Incorrect Answer ImageA.Complement levels
 Incorrect Answer ImageB.Delayed-hypersensitivity skin testing
 Correct Answer ImageC.Dihydrorhodamine (DHR)-123 fluorescence
 Incorrect Answer ImageD.Nitroblue tetrazolium test
 Incorrect Answer ImageE.Quantitative immunoglobulin levels

A 5-month-old boy is brought to the emergency department by his parents because he has been
having crying spells for the past several hours. He was apparently doing fine in the morning
when he suddenly started crying. He appeared to be in severe pain. This lasted for 10 to 15
minutes, and then he became very drowsy. At first the parents thought it was colic. However,
after several episodes of similar attacks, the infant started vomiting and passed stool twice. He is
breastfed, and the mother has started adding solids over the previous 3 to 4 weeks. He has
otherwise been well. The infant appears well-nourished, pale, diaphoretic, and in moderate
distress. His temperature is 36.7ºC (98.0ºF), pulse is 100/min, and respirations are 22/min.
Physical examination of the abdomen shows mild tenderness to palpation without distention. The
right lower quadrant feels empty on palpation. An abdominal x-ray shows absence of air in the
right lower quadrant. Which of the following is the most appropriate next step management? 
 Correct Answer ImageA.Air enema
 Incorrect Answer ImageB.Immediate surgical reduction
 Incorrect Answer ImageC.Laparoscopy
 Incorrect Answer ImageD.Manual reduction
 Incorrect Answer ImageE.Observation in expectation of spontaneous resolution
A 23-year-old woman, gravida 1, para 0, at 26 weeks' gestation comes to the labor and delivery
department because of contractions for 2 hours. She states that they have been occurring every 3
to 5 minutes and that they are worsening in intensity. Her temperature is 37.1°C (98.8°F), blood
pressure is 117/62 mm Hg, pulse is 88/min, and respirations are 16/min. Physical examination
shows the cervix is 3 cm dilated and the fetus is in vertex presentation. Intravenous magnesium
sulfate and penicillin are started. Intramuscular injection of betamethasone is also given. Despite
tocolysis, in eight hours she delivers a female infant, weighing 760 grams with Apgar scores of 5
and 8. The baby is eventually discharged to home on full oral feedings at 38 weeks
postmenstrual age (i.e., 12 weeks after birth). Which of the following complications would most
likely lead to developmental delay in this child?
 Correct Answer ImageA.Intraventricular hemorrhage
 Incorrect Answer ImageB.Late-onset sepsis
 Incorrect Answer ImageC.Necrotizing enterocolitis
 Incorrect Answer ImageD.Patent ductus arteriosus
 Incorrect Answer ImageE.Respiratory distress syndrome

A 5-day-old infant is brought to the emergency department by his mother because of poor
feeding and irritability for the past 2 days; he has had irregular breathing and blue hands and feet
for the past 4 hours. He was delivered at term to a 19-year-old woman, gravida 1, para 1. The
mother received regular prenatal care beginning at approximately 10 weeks’ gestation and says
that the pregnancy was uneventful, but labor was prolonged and the membranes had ruptured 20
hours before the neonate was delivered. He had an uneventful course in the nursery and was
discharged with the mother at 36 hours of age. He appears irritable and is grunting. His weight is
3.3 kg (7 lb 4 ounces); head circumference is 35 cm (14.2 in). His temperature is 35.6ºC
(96.1ºF), pulse is 85/min, and respirations are 75/min. Examination shows a bulging anterior
fontanel. There are mild suprasternal retractions. There is cyanosis of the hands and feet. The rest
of the examination is within normal limits. Which of the following is the most likely diagnosis?
 Incorrect Answer ImageA.Congenital rubella
 Incorrect Answer ImageB.Congenital syphilis
 Incorrect Answer ImageC.Congenital toxoplasmosis
 Incorrect Answer ImageD.Cyanotic congenital heart disease
 Correct Answer ImageE.Neonatal sepsis

A 16-year-old boy is brought to the emergency department by paramedics because he collapsed


in the bathroom. His parents report that they found the toilet bowl full of blood and mucus. He
has otherwise been healthy. The patient reports severe cramping abdominal pain and a
continuous urge to defecate. His temperature is 37.7ºC (99.9ºF), blood pressure is 90/50 mm Hg,
pulse is 100/min, and respirations are 26/min. He appears to be in moderate distress and is
diaphoretic and pale. The abdomen is mildly tender to palpation. There is a palpable mass in the
right lower quadrant. Rectal examination shows rectal bleeding and bloody fecal material.
Flexible sigmoidoscopy shows the presence of ulcerations of the rectum and colon and multiple
biopsy specimens show transmural inflammation. Treatment is begun with daily steroid enemas.
Which of the following findings would establish a diagnosis of Crohn's disease in this patient?
 Incorrect Answer ImageA.Good response to systemic steroids
 Incorrect Answer ImageB.Involvement of the colon
 Correct Answer ImageC.Mouth ulcers
 Incorrect Answer ImageD.Rectal bleeding
 Incorrect Answer ImageE.Rectal disease

A 3-year-old girl with human immunodeficiency virus (HIV) is brought to the physician by her
mother before starting daycare. The child is able to speak in sentences of three to four words,
follow commands, feed herself, and draw a circle. She is currently practicing toilet-training with
her parents. Her vaccinations are up to date. Current medications include efavirenz, tenofovir,
emtricitabine, and a multivitamin. Which of the following would be the most appropriate reason
to exclude the child from attending daycare? 
 Incorrect Answer ImageA.The child exhibits aggressive behavior, such as biting and
scratching
 Incorrect Answer ImageB.The child has a history of occasional nosebleeds
 Incorrect Answer ImageC.The child is not yet toilet trained
 Incorrect Answer ImageD.The guardian does not want to disclose the HIV status of the
child
 Correct Answer ImageE.There is no reason for exclusion

 2-year-old boy is brought to the emergency department because of a 2-day history of fever up to
39.0°C (102.2°F), irritability, and redness and tenderness of the skin. The parents state that there
were two small, fluid-filled lesions that broke open with pressure on the boy's left arm 5 days
ago. His temperature is 38.8°C (101.8°F) and pulse is 125/min. Examination shows tender,
bright red erythema over his trunk, extremities, and around his eyes and mouth. It is accentuated
in the flexural areas. He has conjunctival inflammation and crusting and fissuring around the
eyes and nose. There are two bullae on the right lower extremity. There is a separation of the
epidermis with the application of lateral pressure of the skin over the abdomen with moist,
denuded skin underneath. Which of the following is the most appropriate pharmacotherapy?
 Incorrect Answer ImageA.Amoxicillin orally
 Incorrect Answer ImageB.Ampicillin intravenously
 Incorrect Answer ImageC.Ceftriaxone intramuscularly
 Incorrect Answer ImageD.Ciprofloxacin intravenously
 Incorrect Answer ImageE.Erythromycin orally
 Correct Answer ImageF.Oxacillin intravenously

An 11-month-old infant is brought to the physician by her parents because of an abdominal mass.
Physical examination shows a small bulge at the umbilicus, approximately 1 cm in diameter,
which pops out when the girl cries. The contents of the bulge can be easily reduced. It is not
painful, and the girl is otherwise asymptomatic. She is reaching all developmental milestones.
Which of the following is the most appropriate next step in management? 
 Incorrect Answer ImageA.Elective surgical repair
 Incorrect Answer ImageB.External pressure with abdominal binding
 Correct Answer ImageC.No therapy unless the mass persists beyond age 5 years
 Incorrect Answer ImageD.Observation until 2 years of age
 Incorrect Answer ImageE.Urgent surgical repair

A 4-year-old boy is brought to the physician by his parents because of a 3-week history of
progressive diarrhea, colicky abdominal pain, and painful straining while passing stool. He has
had 8 to 10 bowel movements daily over the past week and is now having frank diarrhea. The
family recently returned from a fishing trip on the Amazon River in South America. He appears
ill. His temperature is 38.9ºC (102.0ºF), blood pressure is 90/60 mm Hg, pulse is 130/min, and
respirations are 20/min. Examination shows diffuse abdominal pain with no rebound or guarding.
Stool is positive for occult blood. Microscopic examination of stool shows protozoa with
ingested erythrocytes. Which of the following is the most appropriate next step in management? 
 Correct Answer ImageA.Abdominal ultrasound
 Incorrect Answer ImageB.Barium enema
 Incorrect Answer ImageC.Colonic biopsy
 Incorrect Answer ImageD.Gallium scan
 Incorrect Answer ImageE.Magnetic resonance imaging (MRI) of the bowels

 5-month-old girl is admitted to the hospital from the emergency department. According to the
parents, she has had a fever and severe diarrhea for the past 4 days. Initially, she continued to
breastfeed but has become increasingly lethargic, is no longer interested in feeding, and has not
had a wet diaper in about a day. She had a diarrhea stool in the ED, which was watery with little
solid stool and no evidence of fecal blood. There were no fecal leukocytes on microscopic
evaluation of a stool smear. Upon arrival to the hospital floor, her temperature is 38.9ºC (102ºF),
respirations are 36/minute, pulse is 140/minute, and blood pressure in the right arm supine is
50/30 mmHg. She appears lethargic, has tenting of her skin over the abdomen, capillary refill is
4 seconds, and she has a sunken anterior fontanel and dry mucous membranes. A percutaneous
central intravenous catheter is placed and a blood sample is sent to the laboratory for a basic
metabolic panel. In addition, an arterial blood gas is performed. She is then immediately given a
bolus of 20 mL/kg of body weight 0.9% normal saline. Which of the following represents the
most likely laboratory findings in this infant?
 Incorrect Answer ImageA.pH 7.10, PaCO2 35 mmHg, HCO3– 11 meq/L, Cl– 100
meq/L, K+ 4.1 meq/L, anion gap 20
 Incorrect Answer ImageB.pH 7.15, PaCO2 30 mmHg, HCO3– 10 meq/L, Cl– 85
meq/L, K+ 6.2 meq/L, anion gap 8
 Incorrect Answer ImageC.pH 7.32, PaCO2 38 mmHg, HCO3– 18 meq/L, Cl– 98
meq/L, K+ 5.7 meq/L, anion gap 15
 Correct Answer ImageD.pH 7.30, PaCO2  29 mmHg, HCO3– 14 meq/L, Cl– 120
meq/L, K+ 2.8 meq/L, anion gap 10
 Incorrect Answer ImageE.pH 7.50, PaCO2 47 mmHg, HCO3– 34 meq/L, Cl– 84
meq/L, K+ 3.0 meq/L, anion gap 7
A 10-month-old boy is brought to the physician by his mother because of a 1-day history of a red
rash that developed suddenly on the chest and spread to the arms and face last night and through
this morning. For the past 4 days, the child had a fever up to 40ºC (104ºF), which resolved 1 day
ago. His temperature is now 37.8ºC (100ºF). Examination shows a diffuse, erythematous macular
rash on the trunk, face, and limbs. There are no pustules or excoriations. Which of the following
is the most likely cause of this child's condition? 
 Correct Answer ImageA.Human herpesvirus 6
 Incorrect Answer ImageB.Measles
 Incorrect Answer ImageC.Parvovirus B19
 Incorrect Answer ImageD.Rubella
 Incorrect Answer ImageE.Varicella

A 15-year-old girl is brought to the physician by her parents because of chronic diarrhea and
weight loss that started several months earlier. The girl had been previously healthy. She reports
loss of appetite and crampy abdominal pain associated with several loose stools daily. The
parents thought that she had started a strict diet to lose weight and encouraged her to eat healthy
food and exercise. The patient followed this advice but the symptoms did not improve. She
continued to lose weight and has had several episodes of low-grade fever. They finally decided
to have her examined when she reported noticing blood in her stool several days after her period
had ended. She admits to having recurrent achy pain in her elbows and knees ever since the
diarrhea started. Abdominal examination shows mild diffuse tenderness to deep palpation. Rectal
examination shows guaiac-positive brown stool. Laboratory studies show a hemoglobin of 11.0
g/dL and ESR is 56 mm/h. A flexible sigmoidoscopy shows diffuse superficial ulcerations and
friable mucosa of the rectum and colon that bleeds easily. Which of the following antibody titers
is most likely to be elevated?
 Incorrect Answer ImageA.Antineutrophil cytoplasmic antibody with a cytoplasmic
staining pattern (c-ANCA)
 Correct Answer ImageB.Antineutrophil cytoplasmic antibody with a perinuclear staining
pattern (p-ANCA)
 Incorrect Answer ImageC.Antinuclear antibodies
 Incorrect Answer ImageD.Anti-Ro antibodies
 Incorrect Answer ImageE.Anti-Smith antibodies

An 8-year-old boy is brought to the physician because of a 2-day history of rash that started on
his face and spread down to his chest, arms, and legs. He also has had a fever, cough, red eyes,
and a runny nose for the past 5 days. The mother reports that a child in his class at school has
similar symptoms. His temperature is 38.2ºC (100.7ºF), blood pressure is 88/56 mm Hg, pulse is
76/min, and respirations are 16/min. Examination of his oral cavity shows several red lesions
with bluish central spots on his buccal mucosa at the level of his premolars. There is a diffuse
erythematous and maculopapular rash on the face, trunk, and extremities. Which of the following
is the most likely cause of this disease? 
 Incorrect Answer ImageA.Coxsackievirus A16
 Incorrect Answer ImageB.Human herpesvirus 6
 Correct Answer ImageC. Paramyxoviridae RNA virus
 Incorrect Answer ImageD.Parvovirus B19
 Incorrect Answer ImageE.Togaviridae RNA virus

A 3-year-old boy is brought to the emergency department by his father because of fever and a
headache for 5 hours. The father and his partner adopted the boy at 18 months of age and he is
not up to date on his vaccinations. He appears lethargic. His temperature is 39.7ºC (103.5ºF).
Examination shows photophobia and mildly injected conjunctiva. There is neck stiffness and a
Kernig sign is present. His leukocyte count is 24,000/mm3 with 64% segmented neutrophils and
25% bands. A lumbar puncture is done, which shows cerebrospinal fluid pressure of 240 mm
water. Cerebrospinal fluid analysis shows decreased glucose, elevated protein, and gram-
negative pleomorphic rods. Which of the following is the most likely cause of this patient's
condition? 
 Incorrect Answer ImageA.Escherichia coli
 Incorrect Answer ImageB.Haemophilus ducreyi
 Correct Answer ImageC.Haemophilus influenzae type b
 Incorrect Answer ImageD.Listeria monocytogenes
 Incorrect Answer ImageE.Neisseria meningitidis
 Incorrect Answer ImageF.Streptococcus pneumoniae

A 5-year-old girl is brought to the physician because of a rash. Two days prior, she had the onset
of a sore throat and fever. Yesterday, she broke out in a red rash that started in the groin area and
has spread to her abdomen and arms and legs. On examination, her throat is erythematous with
enlarged exudative tonsils. There are mildly tender enlarged anterior cervical lymph nodes. She
has a diffuse, erythematous finely papular rash that feels rough to the touch and involves her
axilla, neck, trunk, and extremities. Additionally, there is circumoral pallor. Which of the
following is the best description of the etiology of this disease?
 Incorrect Answer ImageA.Gram-positive bacteria that are lancet-shaped and usually grow
in pairs, are encapsulated, and exhibit alpha-hemolysis on blood agar medium
 Incorrect Answer ImageB.Gram-positive, catalase-negative facultative anaerobes in pairs
or short chains that exhibit no hemolysis (gamma) on blood agar, and the specific species is
identified by its pattern of carbohydrate fermentation
 Incorrect Answer ImageC.Gram-positive cocci in pairs or chains that are facultative
anaerobes, contain a capsular polysaccharide, exhibit beta-hemolysis, and are resistant to a
bacitracin disc on blood agar
 Incorrect Answer ImageD.Gram-positive cocci that cannot be Lancefield-classified;
exhibit alpha-hemolysis on blood agar; and are part of the normal flora of the throat, nose, skin,
and genitourinary tract
 Correct Answer ImageE.Gram-positive cocci that produce complete hemolysis (beta) on
blood agar and exist in chains, whose growth is inhibited by a bacitracin disc, and are infected by
a bacteriophage

A couple who has been trying to have a baby for 5 years has decided to adopt a male infant from
rural China. The infant is 10 months old upon entering the U.S. His records are sparse. It is
noted, however that he has had diarrhea since age 2 months. Prior to giving her baby up for
adoption, the mother was breastfeeding; however, since his adoption at age 1 month, he has been
on a standard lactose-based, cow milk protein infant formula. He is immediately evaluated by
their chosen pediatrician. The physician writes the following in his notes: “Weight and height are
<5%, head circumference at the 10%. Infant is listless and irritable. Skin is loose and there is
decreased muscle mass. There is an eczematous-appearing, scaly rash around the mouth, fingers
and toes and wrists and ankles; there a several vesicobullous lesions in these areas. Nail
dystrophy of all nails is present. Hair is sparse and reddish-tinted. There is bilateral conjunctival
erythema. Tongue has a smooth appearance without papillae. Heart rate is 140/min and rhythm is
normal; there is a grade 2 systolic ejection murmur heard at the apex. Pulses are full and
symmetric. Abdomen is slightly distended without obvious tenderness. Bowel sounds are
decreased. The baby has bilateral cryptorchidism.” Which of the following is the most likely
cause of this infant’s medical condition? 
 Incorrect Answer ImageA.Chromium

 Incorrect Answer ImageB.Copper


 Incorrect Answer ImageC.Manganese
 Incorrect Answer ImageD.Selenium
 Correct Answer ImageE.Zinc

A 4-week-old male infant born at 27 weeks’ gestation is in the neonatal intensive care unit
receiving total parenteral nutrition (TPN) by a central intravenous catheter. Feeding via
nasogastric tube has been unsuccessful due to 2 episodes of feeding intolerance. The infant
received surfactant therapy at birth and had a successful closure of a patent ductus arteriosus
(PDA) with the use of indomethacin. He was ventilated for a total of 6 days and then had nasal
continuous positive airway pressure (nCPAP) for 10 days. He is now on nasal cannula oxygen at
an inspired oxygen concentration of 28%. He has just undergone a dilated retinal examination
that shows a demarcation line with height, width, and volume extending up and out of the plane
of the retina. Which of the following may have prevented this from occurring?
 Incorrect Answer ImageA.Discontinuing positive pressure ventilation after the first 72
hours
 Correct Answer ImageB.Keeping the arterial oxygen saturation 85%–92% for the first 4
weeks
 Incorrect Answer ImageC.Keeping the arterial oxygen saturation 97%–100% for the first
4 weeks
 Incorrect Answer ImageD.Keeping the blood gas PaCO2 30–35 mm Hg
 Incorrect Answer ImageE.Keeping the blood gas pH greater than 7.4
 Incorrect Answer ImageF.Ligating the PDA instead of using indomethacin
 Incorrect Answer ImageG.Using TPN only without attempts at enteral feedings for the
first month

A previously healthy 3-week-old boy is brought to the physician by his mother because of
irritability and restlessness for the past 6 hours. He has been crying incessantly and has been
feeding poorly. The infant is bottle-fed with a cow's milk formula. His temperature is 39.3°C
(102.8°F). He is crying during the physical examination. The fontanelle is bulging and there is
decreased muscle tone. Which of the following is the most likely cause of this patient's
symptoms? 
 Incorrect Answer ImageA.Borrelia burgdorferi
 Incorrect Answer ImageB.Cryptococcus neoformans
 Incorrect Answer ImageC.Mycobacterium tuberculosis
 Incorrect Answer ImageD.Neisseria meningitidis
 Correct Answer ImageE.Streptococcus agalactiae
 Incorrect Answer ImageF.Streptococcus pneumoniae
 Incorrect Answer ImageG.Treponema pallidum

A previously healthy 9-year-old boy is brought to the physician by his father because of a high-
grade fever, vomiting, malaise, and a sore throat for the past 2 days. He also has odynophagia
and has only been able to drink a few sips of water over the past 24 hours. Several of his
classmates have also been ill with similar symptoms. He has no significant past medical history
and takes no medications. His temperature is 39ºC (102.2ºF), blood pressure is 92/56 mm Hg,
pulse is 98/min, and respirations are 22/min. He appears to be in mild distress. Physical
examination shows tender anterior cervical lymphadenopathy, edematous tonsils with yellow-
white, thick posterior oropharyngeal exudates. Laboratory studies show WBC count 13,800/mm3,
82% neutrophils, and 11% bands. Which of the following complications is this patient most at
risk for despite adequate treatment? 
 Correct Answer ImageA.Glomerulonephritis
 Incorrect Answer ImageB.Peritonsillar abscess
 Incorrect Answer ImageC.Retropharyngeal abscess
 Incorrect Answer ImageD.Rheumatic fever
 Incorrect Answer ImageE.Scarlet fever
Interna
A 71-year-old man comes to his physician because of fatigue and shortness of breath for the past
6 months. He reports progressive dyspnea with exertion. He denies chest pain, chronic cough,
diarrhea, cramping, hematochezia, melena, orthopnea, paroxysmal nocturnal dyspnea, edema,
weight loss, or fever. He has no history of significant medical illnesses. He does not take any
medications. He occasionally drinks alcohol, but he does not smoke. His temperature is 37.1°C
(98.8°F), pulse is 98/min, respirations are 12/min, and blood pressure is 131/72 mm Hg. Oxygen
saturation is 96% on room air. On physical examination, there is pallor and bounding pulses. The
remainder of the physical examination is unremarkable. Laboratory studies show:
White blood cells 5,600/mm3
Hematocrit 29%
Platelets 225,000/mm3
Mean corpuscular volume 118 μm3
Iron 50 μg/dL
Ferritin 150 ng/mL
Serum folate 8 ng/mL
Serum B12 35 pg/mL
LDH 113 U/L
Peripheral blood smear hypersegmented neutrophils, macrocytosis
Antiparietal cell antibodies and anti-intrinsic factor antibodies are negative. A Schilling test is
performed, and a urine assay for radiolabeled oral B12 is negative. The Schilling test is repeated
with intrinsic factor supplementation four weeks later. Urine assay for oral B12 is still
negative. What is the most likely diagnosis in this patient?
 Incorrect Answer ImageA.Dietary deficiencyIncorrect Answer Image
 Correct Answer ImageB.Diphyllobothrium latum infection
 Incorrect Answer ImageC.Metastatic disease
 Incorrect Answer ImageD.Pernicious anemia
 Incorrect Answer ImageE.Ulcerative colitis

A 26-year-old woman comes to her physician because of a 1-year history of amenorrhea. She
also reports occasional discharge from both breasts. She has no history of significant medical
illnesses, and she takes no medications. She has a 5-year-old child that she breast-fed until 9
months of age. Her temperature is 37.1°C (98.8°F), pulse is 80/min, respirations are 12/min, and
blood pressure is 130/80 mm Hg. Physical examination shows diffuse enlargement of the thyroid
gland. Breast examination shows the expression of milk from both breasts. Laboratory studies
show:
Serum prolactin  160 ng/mL 
(normal, 4–30 ng/mL)
Thyroid-stimulating hormone 36 mU/L
(normal, 0.4–5 mU/L) 
Thyroxine (T4) 0.6 ng/dL
(normal, 0.8–1.8 ng/dL) 
Urine β-hCG Negative
MRI of the brain shows pituitary enlargement. Which of the following is the most appropriate
next step in management?
 Incorrect Answer ImageA.Bromocriptine
 Incorrect Answer ImageB.Cabergoline
 Incorrect Answer ImageC.Radiotherapy
 Correct Answer ImageD.Thyroxine replacement
 Incorrect Answer ImageE.Transsphenoidal surgery

A 70-year-old man comes to the physician because of increased fatigue, dyspnea, and weight
loss over the past 6 months. He denies chest pain or anorexia. He has previously been healthy but
has not seen a physician in 15 years. Review of systems is positive for a change in his stool
pattern. He has had 3 bowel movements a day instead of one, and they are looser. Family
members have commented on his pale appearance. He is currently not taking any medications.
He denies allergies. He has no history of alcohol or drug abuse. On examination, he is ill-
appearing and pale. Pulse is 120/min and blood pressure is 100/70 mm Hg. His abdomen is
nontender to palpation. Rectal examination shows stool that is positive for occult blood. His
electrocardiogram shows sinus tachycardia. His hematocrit is 20%. Which of the following is the
most appropriate next step in management?
 Incorrect Answer ImageA.Abdominal CT scan
 Correct Answer ImageB.Colonoscopy
 Incorrect Answer ImageC.Esophagogastroduodenoscopy
 Incorrect Answer ImageD.Increased fiber diet
 Incorrect Answer ImageE.Stool culture

A 24-year-old woman comes to the emergency department because of an acute onset of


abdominal cramping and hives for 12 hours. She states that 2 days ago, she had a sore throat and
started taking an over-the-counter cold medicine containing dextromethorphan,
diphenhydramine, pseudoephedrine, and ibuprofen. She also has had a generalized itch. She is in
mild distress because of her symptoms but denies shortness of breath or difficulty swallowing.
Of note, she has a history of asthma and takes doxycycline for acne. Her temperature is 36ºC
(96.8ºF), pulse is 90/min, respirations are 24/min, and blood pressure is 105/65 mm Hg. Physical
examination shows multiple 1 to 10 cm, pink-and-white urticarial papules and plaques on the
trunk and proximal extremities. Which of the following is the most likely cause of this patient's
condition?
 Incorrect Answer ImageA.Dextromethorphan
 Incorrect Answer ImageB.Diphenhydramine
 Correct Answer ImageC.Ibuprofen
 Incorrect Answer ImageD.Pseudoephedrine
 Incorrect Answer ImageE.Doxycycline

A 32-year-old man comes to his physician because of worsening nausea, polyuria, abdominal
pain, constipation, diffuse bone pain, and intermittent headaches for the past year. The patient
has a 10 pack-year history of smoking, but does not use alcohol or recreational drugs. He has no
other past medical history and takes no medications. Family history is positive for kidney stones
in his mother. Vital signs are within normal limits. Physical examination shows dry mucous
membranes. There are no palpable neck masses. The chest is clear to auscultation. There is mild
left lower quadrant abdominal pain on deep palpation. Laboratory studies show:
Complete blood count
Hemoglobin 14 g/dL
Hematocrit 50%
Leukocyte count 7,000/mm3
Platelets 250,000 mm3
Serum Chemistry
Na+ 145 mEq/L
K+ 5.0 mEq/L
Ca 2+
12.1 mg/dL
Cl- 100 mEq/L
HCO3 -
24 mEq/L
BUN 30 mg/dL
Creatinine  1.0 mg/dL
Glucose  100 mg/dL
Albumin  4 g/dL
Phosphorus  3.0 mg/dL
Parathyroid hormone  94 pg/mL (N = 10–65 pg/mL)
The chest x-ray is normal. Which of the following is the most appropriate next step in the
management of this patient? 

 Incorrect Answer ImageA.Measure serum parathyroid hormone-related peptide (PTHrP)


 Incorrect Answer ImageB.Measure serum vitamin D
 Correct Answer ImageC.Measure urinary calcium
 Incorrect Answer ImageD.Order MRI of the thyroid and parathyroid glands
 Incorrect Answer ImageE.Order Sestamibi parathyroid scintigraphy

A previously healthy 31-year-old man comes to the emergency department writhing in pain. He
awoke 1 hour ago with excruciating pain in his right lower quadrant. The pain is sharp,
intermittent, and non-radiating. He can eat and drink normally. He has no past history of
abdominal pain. He has mild seasonal rhinitis, for which he takes fluticasone as needed. His
temperature is 36.7ºC (98.0ºF), pulse is 112/min, respirations are 24/min, and blood pressure is
137/84 mm Hg. His abdomen is soft and nondistended, with normal bowel sounds. He has mild
right-sided tenderness to deep palpation, with lower quadrant greater than upper quadrant. He
has a circumcised phallus with no urethral discharge. Both testes are within the scrotum and
nontender. Rectal examination is nontender and reveals a firm, small prostate. His stool is
negative for occult blood. Laboratory studies show:
WBC 8,400/mm3
BUN 11 mg/dL
Creatinine 1.1 mg/dL
Urinalysis findings
Urine specific gravity 1.030
pH 5.0
RBC 25–30/hpf
WBC 3–4/hpf
CT scan of the abdomen and pelvis shows a 3 mm calcification at the level of the right
ureterovesical junction. Which of the following is the most appropriate management?
 Incorrect Answer ImageA.Admit to the hospital for serial abdominal examinations
 Correct Answer ImageB.Discharge home with a prescription for a nonsteroidal anti-
inflammatory agent (NSAID)
 Incorrect Answer ImageC.Open surgical removal of stone
 Incorrect Answer ImageD.Placement of a ureteral stent
 Incorrect Answer ImageE.Placement of a percutaneous nephrostomy tube

A 72-year-old man comes to his physician because he has had decreased energy and severe pain
in his back and leg, such that he has a distinct limp when walking. He denies any trauma. On
examination, he is pale and has tenderness to palpation along the vertebrae as well as pain upon
flexion and extension of his left hip. He has no other positive physical findings. A complete
blood count shows a hematocrit of 27% with marked rouleaux formation. His white blood cells
and platelets are normal in number and appearance. An x-ray of his left hip and upper leg shows
a fracture of the femoral neck. Laboratory studies show a serum calcium of 12 mg/dL; ESR of 40
mm/h; and urine dipstick positive for 3+ protein. Which of the following describes the hallmark
of the diagnosis?
 Incorrect Answer ImageA.A bone-marrow aspiration showing peritrabecular lymphoid
aggregates
 Incorrect Answer ImageB.A bone-marrow aspiration showing the Philadelphia
chromosome
 Incorrect Answer ImageC.A decrease in serum viscosity

 Correct Answer ImageD.A monoclonal spike in the beta- or gamma-globulin region on


serum protein electrophoresis
 Incorrect Answer ImageE.Hypogammaglobulinemia

A 42-year-old man comes to the physician because of a 3-day history of right elbow pain. The
pain is worse with right arm movement. His pain is not relieved by rest or ice application. He
reports having a similar pain in the past, relieved with rest and ice. He denies having either a
fever or fall. He plays tennis for 4 hours per day and gives tennis lessons. Physical examination
shows tenderness over the lateral epicondyle. Extension of the hand at the wrist against
resistance produces pain. The tendon of which of the following muscles is most likely injured in
this patient? 
 Incorrect Answer ImageA.Brachioradialis
 Correct Answer ImageB.Extensor carpi radialis brevis
 Incorrect Answer ImageC.Extensor carpi radialis longus
 Incorrect Answer ImageD.Flexor carpi radialis
 Incorrect Answer ImageE.Pronator teres

A 45-year-old man who had been diagnosed with type 1 diabetes mellitus at the age of 15 comes
to the physician for a follow-up visit. The patient has no complaints at this time and is compliant
with his medications, which include NPH insulin, regular insulin, and lisinopril. His temperature
is 37ºC (98.6ºF), blood pressure is 130/85 mm Hg, pulse is 86/min, and respirations are 12/min.
His last hemoglobin A1C was 7.2%. Physical examination shows deformity of both feet with loss
of the normal arches and abnormal orientation of the toes. No inflammation is noted.
Radiographs show deformed bones with new bone formation adjacent to the bony cortex. Several
large, bizarrely shaped osteophytes are seen at the joint margins. Which of the following is the
most likely diagnosis?
 Incorrect Answer ImageA.Avascular necrosis
 Incorrect Answer ImageB.Gout
 Incorrect Answer ImageC.Infectious arthritis
 Correct Answer ImageD.Neurogenic arthropathy
 Incorrect Answer ImageE.Osteoarthritis

 A 64-year-old man comes to the physician for follow-up of his congestive heart failure (CHF).
He has had CHF for 2 years following a myocardial infarction. He was hospitalized 4 weeks ago
because of a CHF exacerbation caused by excessive salt intake. He currently takes metoprolol,
simvastatin, and a loop diuretic. His blood pressure is 150/72 mm Hg and pulse is 52/min.
Physical examination shows a laterally displaced point of maximal impulse. There is no jugular
venous distension or dependent edema. Which of the following changes in management will
provide the greatest survival benefit for this patient?
 Incorrect Answer ImageA.Begin a calcium channel blocker
 Correct Answer ImageB.Begin an ACE inhibitor
 Incorrect Answer ImageC.Begin digoxin therapy
 Incorrect Answer ImageD.Change metoprolol to propranolol
 Incorrect Answer ImageE.Increase diuretic dose

A 52-year-old man with a 3-month history of hypertension comes to the physician because of
swelling in his hands and feet for 2 years. He has had no change in vision, chest pain, or changes
in weight. His blood pressure is 152/90 mm Hg. Examination shows macroglossia,
macrognathia, and soft-tissue swelling in the extremities. Cardiac examination shows a left
displaced point of maximal impulse. A grade 2/6 systolic murmur is heard over the left sternal
border. Serum growth hormone level is 4 ng/mL 1 hour after ingestion of 75 g of glucose (n, <1
ng/mL). MRI scan of the head shows a 1.5 cm pituitary macroadenoma. The patient does not
wish to undergo surgery. Which of the following is the most appropriate next step in
management?
 Incorrect Answer ImageA.Chlorambucil
 Incorrect Answer ImageB.Cyclophosphamide
 Incorrect Answer ImageC.Cyclosporine
 Incorrect Answer ImageD.External beam radiation
 Incorrect Answer ImageE.No further treatment
 Correct Answer ImageF.Octreotide

A 66-year-old diabetic man is brought to the emergency department via ambulance because of
shortness of breath. For the past 2 hours, he has experienced dyspnea at rest and has been
coughing up sputum. He denies chest pain. His temperature is 37.2ºC (98.9ºF), pulse is 124/min,
respirations are 22/min, and blood pressure is 156/94 mm Hg. On physical examination, the
patient is lying down and in distress. He is unable to complete full sentences. An
electrocardiogram is normal. What is the next best step in management? 
 Correct Answer ImageA.Assist the patient to a sitting position

 Incorrect Answer ImageB.Dobutamine


 Incorrect Answer ImageC.Dopamine
 Incorrect Answer ImageD.Echocardiography
 Incorrect Answer ImageE.Hydralazine
 Incorrect Answer ImageF.MUGA scan
 Incorrect Answer ImageG.Synchronized cardioversion
 Incorrect Answer ImageH.Verapamil

A 27-year-old woman comes to the emergency department because of shortness of breath. The
current symptoms began approximately 5 hours ago and have become gradually more severe.
She reports a history of mild episodic asthma but has not had an attack in more than 3 years. She
is on no medications. Her respiratory rate is 25/min. Lung examination is notable for loud
expiratory wheezing bilaterally. The expiratory phase is prolonged, and air movement is fair, but
decreased. A peak expiratory flow rate is 110 L/min. Her oxygen saturation on room air is 92%.
Which of the following is the most appropriate next step in management?
 Correct Answer ImageA.Albuterol nebulizer
 Incorrect Answer ImageB.Inhaled corticosteroids
 Incorrect Answer ImageC.Intravenous methylprednisone
 Incorrect Answer ImageD.Oral prednisone
 Incorrect Answer ImageE.Subcutaneous epinephrine

A 45-year-old woman comes to her physician for increasing edema for 1 year. The edema was
initially in both feet, but 3 weeks ago she noticed severe diffuse swelling and blood in her urine.
Her shoes no longer fit. Her pulse is 90/min and her blood pressure is 175/100 mm Hg. She has
diffuse 4+ edema to mid-thigh. Laboratory studies show: 
Sodium 136 mEq/L
Potassium 5.7 mEq/L
Chloride 100 mEq/L
Bicarbonate 19 mEq/L
Creatinine 9.6 mg/dL
Urea nitrogen 100 mg/dL
Albumin 3.1 mg/dL
Calcium 8.1 mg/dL
Phosphorus 7.5 mg/dL
Urinalysis shows 4+ protein, 2+ blood, and 5–10 RBC/HPF. ECG is normal. Renal ultrasound
shows small, echogenic kidneys. The patient is admitted to the hospital, has a dialysis catheter
placed, and hemodialysis is started on hospital day 2. Which of her problems will be least
improved by the hemodialysis?
 Incorrect Answer ImageA.Acidemia
 Incorrect Answer ImageB.Edema
 Incorrect Answer ImageC.Hyperkalemia
 Correct Answer ImageD.Hyperphosphatemia
 Incorrect Answer ImageE.Hypertension

A 61-year-old woman comes to the physician because of shortness of breath. She states that she
has had a 6-month history of progressively worsening dyspnea while climbing the stairs in her
house. She denies both chest pain and dyspnea at rest. She appears to be comfortable at rest and
has a barrel-shaped chest. Her medical history is remarkable for hypertension and chronic
obstructive pulmonary disease. Physical examination shows a prolonged expiratory phase and
wheezes on forced exhalation. Heart sounds are regular in rate and rhythm with no murmurs,
rubs, or gallops. A transthoracic echocardiogram shows diastolic left ventricular dysfunction.
Which of the following is the most appropriate next step in the management of this patient? 
 Incorrect Answer ImageA.Digoxin
 Incorrect Answer ImageB.Dobutamine
 Incorrect Answer ImageC.Propranolol
 Incorrect Answer ImageD.Transesophageal echocardiography
 Correct Answer ImageE.Verapamil

A 57-year-old man comes to the physician because of shortness of breath and progressive cough
over the past month. He denies fever or weight loss. He has a 5-year history of chronic
obstructive pulmonary disease treated with albuterol and ipratropium bromide. He works as a
construction worker and has smoked 2 packs of cigarettes daily for the past 39 years. His
temperature is 36.8ºC (98.2ºF), blood pressure is 130/70 mm Hg, pulse is 110/min, and
respirations are 25/min. Physical examination shows diffuse wheezing bilaterally. The remainder
of the examination is normal. Chest x-ray shows an opacity in the right lung. CT scan of the
chest without contrast shows a right lung mass and an enlarged paratracheal lymph node. Which
of the following is the most appropriate next step?
 Incorrect Answer ImageA.Bronchoscopy with a biopsy
 Incorrect Answer ImageB.Helical CT of the chest with contrast
 Incorrect Answer ImageC.MRI of the chest
 Correct Answer ImageD.Positron emission tomography (PET) CT of the chest with
fluorodeoxyglucose
 Incorrect Answer ImageE.Pulmonary function testing

A previously healthy 28-year-old woman comes to the emergency department because of fevers,
headache, photophobia, and mild slurred speech for 48 hours. She recently returned from a trip to
upstate New York with her friends. A day later, she noted a rash with a central area of clearing
on her forearm. The rash resolved over the next three weeks and she was feeling fine until
yesterday. Currently, her temperature is 38.0ºC (100.4ºF), blood pressure is 120/70 mm Hg,
pulse is 73/min, and respirations are 13/min. Physical examination shows mild photophobia and
nuchal rigidity. She has a mild left-sided facial droop. There is no rash present. The remainder of
her physical examination is normal. Which of the following is the most appropriate next step in
management? 
 Incorrect Answer ImageA.Administration of intravenous vancomycin
 Incorrect Answer ImageB.Administration of oral doxycycline
 Incorrect Answer ImageC.Determination of serum Lyme antibody titers
 Correct Answer ImageD.Lumbar puncture
 Incorrect Answer ImageE.Lyme polymerase chain reaction of serum

A 33-year-old man comes to the physician for a routine health maintenance examination. He
denies fever, chills, weight loss, loss of appetite, abdominal pain, diarrhea, chest pain, or cough.
He has a history of childhood asthma that resolved at the age of 6 years. He underwent an
appendectomy at age 11 years. He exercises regularly and takes no medications. His pulse is
75/min, and blood pressure is 115/80 mm Hg. His lungs are clear to auscultation. Cardiac
examination shows a normal S1 and S2 without murmurs. Laboratory studies are normal. A
tuberculin skin test with purified protein derivative (PPD) shows an induration of 11 mm in
diameter after 48 hours. Which of the following is the most appropriate interpretation of this
patient's PPD test result?
 Incorrect Answer ImageA.A tuberculin test cannot be interpreted in asymptomatic
patients
 Incorrect Answer ImageB.A tuberculin test is always equivocal if the patient does not
have a cough
 Incorrect Answer ImageC.A tuberculin test is always positive if the chest radiograph is
normal
 Incorrect Answer ImageD.The tuberculin test result is equivocal
 Correct Answer ImageE.The tuberculin test result is negative

A 19-year-old woman comes to the emergency department after a syncopal event 1 hour ago.
While working in the garden, she accidentally disturbed a wasp nest and was stung on her right
hand. She has never been stung before. The woman tried to rest at home but felt extremely ill and
called for an ambulance. She says she is dizzy and is concerned about some swelling in both of
her hands and feet. Her temperature is 37.8ºC (100.0ºF), pulse is 130/min, respirations are
22/min, and blood pressure is 83/40 mm Hg. Oxygen saturation is 97% on room air. Pulmonary
examination shows mild diffuse wheezing in all lung fields. Which of the following is the most
appropriate next step in management?
 Correct Answer ImageA.Administer epinephrine
 Incorrect Answer ImageB.Bolus patient with 500 mL of half-normal saline
 Incorrect Answer ImageC.Perform endotracheal intubation
 Incorrect Answer ImageD.Prescribe diphenhydramine and cimetidine
 Incorrect Answer ImageE.Start IV corticosteroids

A 74-year-old woman comes to the physician because of a 3-month history of bone pain. The
pain is not relieved by ibuprofen. She has a history of hypertension and coronary artery disease.
Current medications include lisinopril, metoprolol, and aspirin. She has smoked 2 packs of
cigarettes daily for 30 years. She does not drink alcohol. Her temperature is 36.8°C (98.2°F),
blood pressure is 130/85 mm Hg, and pulse is 65/min. A photograph of her hands is shown. A
prolonged expiratory phase and an S4 gallop are heard on auscultation. There is tenderness to
palpation of her distal long bones. Radiographs of the forearms and femurs show bilateral
periostosis. Which of the following is the most appropriate next step in management?
 Incorrect Answer ImageA.Bone marrow biopsy
 Correct Answer ImageB.Chest radiography
 Incorrect Answer ImageC.Increase her ibuprofen frequency and dosage
 Incorrect Answer ImageD.Joint aspiration
 Incorrect Answer ImageE.No further management is indicated

A 25-year-old man comes to the physician because of chest pain. He says that the pain is
spontaneous and intermittent. He also reports two prior episodes of fainting during exercise but
has otherwise been healthy. He reports a similar history in other family members and notes that
his father collapsed and died at age 50 while playing tennis. The patient denies use of cocaine or
other recreational drugs and takes no medications. Cardiac examination shows a systolic ejection
murmur that is loudest along the left sternal border. The remainder of the physical examination
shows no abnormalities. Echocardiography shows asymmetric septal hypertrophy without
obstruction. Which of the following interventions would most likely decrease this patient's
systolic murmur?
 Incorrect Answer ImageA.Amyl nitrite inhalation
 Incorrect Answer ImageB.Digoxin
 Correct Answer ImageC.Lying down
 Incorrect Answer ImageD.Standing up
 Incorrect Answer ImageE.Valsalva maneuver

A 60-year-old man initially develops an inflamed papule on his thumb followed by the lesion
shown below one day after hunting and skinning rabbits. Several days later, he develops a severe
illness with cough and confusion. Physical examination performed in the emergency department
shows left axillary lymphadenopathy, reduced breath sounds, rales bilaterally over the lung
bases, and splenomegaly. Blood studies show a mild leukocytosis. Which of the following is the
most likely diagnosis?
 Incorrect Answer ImageA.Actinomycosis
 Incorrect Answer ImageB.Brucellosis
 Incorrect Answer ImageC.Melioidosis
 Incorrect Answer ImageD.Plague
 Correct Answer ImageE.Tularemia

An 18-year-old man comes to the physician for a physical examination before leaving for
college. Although he has no complaints and states that he is in good health, he has noticed a
small mass in the left supraclavicular region. The mass has been present for at least the last
month and he believes it is getting larger. He denies having any pets or any recent travel. His
past medical history is significant for a tonsillectomy as a child. He is a nonsmoker. His
temperature is 37.0ºC (98.6ºF), blood pressure is 128/72 mm Hg, pulse is 60/min, and
respirations are 20/min. Examination shows a hard, 3 cm supraclavicular lymph node that is
nontender and immobile. The remainder of the examination is normal. Chest radiograph and
complete blood count are ordered. Which of the following is most likely to confirm the
diagnosis?
 Incorrect Answer ImageA.Chest CT with contrast
 Incorrect Answer ImageB.Close observation with serial measurement
 Incorrect Answer ImageC.Fine-needle aspiration of the mass
 Correct Answer ImageD.Surgical consult for excisional biopsy
 Incorrect Answer ImageE.Ultrasound of the mass

A 47-year-old man is brought to the emergency room after vomiting blood. He reports
abdominal pain and severe nausea. He has a history of drinking alcohol for the past 25 years and
has drunk non-stop for the last six days. He reports multiple episodes of severe abdominal pain
in the previous three weeks. His medical records show a history of pancreatitis and
ultrasonographic changes of liver cirrhosis. His temperature is 37.6ºC (99.6ºF), blood pressure is
98/55 mm Hg, pulse is 105/min, and respirations are 18/min. He is given oxygen and 2 large
bore IV with normal saline are started. Laboratory tests are taken and show the following: 
WBC 13,200/mm3
Neutrophils 68%
Hgb 8.6 g/dL
Hct 26%
Sodium 133 mEq/L
Potassium 3.9 mEq/L
Chloride 97 mEq/L
Amylase 625 U/L
Lipase 937 U/L
ALT 103 U/L
AST 198 U/L
Alkaline phosphatase 104 IU/L
Abdominal examination shows a mildly distended abdomen, tympanic to percussion, and painful
to palpation with no voluntary guarding or rebound. No masses are palpated, although there is
slight liver and splenic enlargement. An abdominal ultrasound shows changes consistent with
portal hypertension, and a thrombosis of the splenic vein not seen on a previous study. An upper
endoscopy is planned. Which of the following is the most likely source of his bleeding? 
 Incorrect Answer ImageA.Alcoholic gastritis
 Incorrect Answer ImageB.Esophageal varices
 Correct Answer ImageC.Gastric varices
 Incorrect Answer ImageD.Peptic ulcer disease
 Incorrect Answer ImageE.Squamous cell cancer of the oropharynx

A 21-year-old woman goes to the doctor because of itchy, watery eyes and clear nasal discharge.
Further questioning indicates that she typically has problems all year long but that these two
complaints are more pronounced during the spring and fall seasons. She denies episodes of fever,
thick nasal drainage, or facial pain. Physical examination shows bilateral conjunctival injection
and grayish discoloration of her lower eyelids. She has pale, boggy nasal mucosa with swollen
inferior turbinates. The nasal septum is midline. She is diagnosed with allergic rhinitis and given
a prescription for an intranasal glucocorticoid, oral loratadine and nasal phenylephrine spray as
needed for symptoms. She returns to the clinic two weeks later with no improvement. She is
unable to work or sleep. Which of the following is the best next step in the management of all of
her complaints? 
 Correct Answer ImageA.A 10-day course of an oral steroid
 Incorrect Answer ImageB.Continue the current regimen
 Incorrect Answer ImageC.Oral amoxicillin for 10 days
 Incorrect Answer ImageD.Oral amoxicillin-clavulanate for 10 days
 Incorrect Answer ImageE.Oxymetazoline
 Incorrect Answer ImageF.X-ray of the sinuses
A 70-year-old man with a history of chronic arthritis comes to the physician with two days of
acute right-sided abdominal pain that is cramping in nature. Physical examination is abnormal
for right knee redness and swelling. Urinalysis shows hematuria. A CT scan shows multiple
kidney stones in the right renal pelvis. He passes a kidney stone and analysis indicates it is a uric
acid stone. His serum urate level is 12 mg/dL and creatinine is 1.9 mg/dL. Aspirated fluid from
his right knee shows urate crystals. Which of the following is the most appropriate long-term
treatment for this patient?
 Correct Answer ImageA.Allopurinol
 Incorrect Answer ImageB.Corticosteroids
 Incorrect Answer ImageC.Lithotripsy
 Incorrect Answer ImageD.Probenecid
 Incorrect Answer ImageE.Ureteral stent placement

A 66-year-old man comes to the physician because of unrelenting diarrhea, fatigue, and weight
loss. The symptoms have been insidiously progressing for the past 2 months. He has also noticed
that his stools are pale, oily, bulky, and difficult to flush. He voids approximately 7 times per
day. He also complains of increasing arthralgias. He denies recent travel or recent camping trips.
His past medical history is significant for hypertension treated with hydrochlorothiazide. On
physical examination, he appears frail and in no acute distress. His blood pressure is 110/75 mm
Hg and pulse is 88/min. Head and neck examination shows mild glossitis and lymphadenopathy.
On cardiac examination, a systolic murmur is present. His lungs are clear to auscultation. He has
increased bowel sounds overall, without hepatosplenomegaly or tenderness. Peripheral
examination shows no cyanosis, clubbing, or edema. Neurologic examination shows profound
ataxia and difficulty with rapid alternating movements. Steatorrhea is confirmed on a 24-hour
stool collection. Biopsy of his small bowel demonstrates periodic acid-Schiff-stained
macrophages in the lamina propria. Stains for fungi and acid-fast bacteria are negative. He is also
found to be HIV-negative. Which of the following is the most appropriate next step in
management?
 Correct Answer ImageA.Antibiotics
 Incorrect Answer ImageB.Corticosteroids
 Incorrect Answer ImageC.Gluten avoidance
 Incorrect Answer ImageD.Lactose avoidance
 Incorrect Answer ImageE.Loperamide

A 43-year-old woman comes to the emergency department because of a productive cough of 3


days' duration and spiking fevers. She has a 10-year history of rheumatoid arthritis and takes
NSAIDs to control her disease. Her temperature is 38.6ºC (101.5ºF), pulse is 98/min, respirations
are 22/min, and blood pressure is 123/75 mm Hg. Her oxygen saturation is 94% on room air.
Physical examination shows rhonchi in the right middle and lower lobes and normal heart
sounds. Her abdomen is nontender and nondistended. The spleen is palpable 2 cm below the
costal margin. She has rheumatoid nodules on the extensor surfaces of her fingers and lower legs
bilaterally. A chest radiograph shows a right lower lobe consolidation with an effusion. Which of
the following additional findings is most likely to be seen in this patient? 
 Correct Answer ImageA.Absolute neutrophil count <1,500/mm3
 Incorrect Answer ImageB.Decreased serum immunoglobulins
 Incorrect Answer ImageC.Hypocellular bone marrow
 Incorrect Answer ImageD.Low-titer rheumatoid factor
 Incorrect Answer ImageE.Small granular lymphocytes on peripheral blood smear

A 34-year-old man comes to his physician for a routine health maintenance examination. He
reports feeling well. He has no history of significant medical illnesses. He takes no medications.
His family history is significant for polycystic kidney disease in his mother and brother. His vital
temperature is 37.1°C (98.8°F), pulse is 80/min, respirations are 12/min, and blood pressure is
145/85 mm Hg. Laboratory data show a serum creatinine of 1.2 mg/dL. CT scan of the abdomen
shows normal-sized kidneys with no cysts. However, there is a 1.7 cm hypodense nodule with
smooth rounded contours on the left adrenal gland. Which of the following is the most
appropriate next step in the work-up for this patient?
 Correct Answer ImageA.24-hour urine catecholamine and metanephrine, low-dose
dexamethasone suppression test, and serum aldosterone-to-renin ratio
 Incorrect Answer ImageB.Cortisol stimulation test
 Incorrect Answer ImageC.CT abdomen adrenal protocol (multi-phase) to better evaluate
the lesion
 Incorrect Answer ImageD.CT chest, abdomen, and pelvis to assess for metastases
 Incorrect Answer ImageE.MRI abdomen adrenal protocol (multi-phase) to better evaluate
the lesion

A 60-year-old alcoholic woman with cirrhosis comes to the physician because of a 3-month
history of abdominal pain, weakness, fatigue, and 15-lb weight loss. The pain is vague and
located over the right upper quadrant. It is not associated with meals, movement, or positional
changes. There is occasional vomiting after meals. Her last colonoscopy a year ago was normal.
On examination, she is fatigued and moves slowly. Her abdomen is non-tender, distended, with
mild shifting dullness and a palpable mass in the right upper quadrant. The patient is most likely
to have which of the following laboratory values?
 Correct Answer ImageA.Increased alpha-fetoprotein
 Incorrect Answer ImageB.Increased beta-human chorionic gonadotropin
 Incorrect Answer ImageC.Increased carcinoembryonic antigen
 Incorrect Answer ImageD.Leukocytosis
 Incorrect Answer ImageE.Serum albumin concentration of 5.6 g/dL

A 34-year-old man with a history of epilepsy comes to the physician because he is planning a
trip to Africa. He has plans to visit rural areas in addition to major cities. He has had no loss of
consciousness or migraines. His last seizure was 2 years ago. Current medications include
phenytoin. He has no allergies. He has been vaccinated against hepatitis B. In addition to
mosquito-avoidance measures, which of the following is recommended for malaria prophylaxis? 
 Incorrect Answer ImageA.Chloroquine
 Incorrect Answer ImageB.Ciprofloxacin
 Correct Answer ImageC.Doxycycline
 Incorrect Answer ImageD.No prophylaxis if he is using mosquito-avoidance measures
 Incorrect Answer ImageE.Pyrimethamine-sulfadoxine

A 60-year-old woman comes to the physician for a follow up examination 2 months after
undergoing partial parathyroidectomy for a parathyroid adenoma that caused
hyperparathyroidism. She now has had 2 weeks of increasing muscle twitching as well as a
tingling sensation around her mouth, hands and feet. Her blood pressure is 126/82 mm Hg and
pulse is 76/min. Physical examination shows a well-healed surgical neck scar. Tapping the facial
nerve anterior to the ear causes a muscle spasm of the ipsilateral facial muscles. Her serum
calcium level is 6.6 mg/dL, with phosphate 2.0 mg/dL. Which of the following is the most likely
cause of this patient's hypocalcemia?
 Correct Answer ImageA.Hungry bone syndrome
 Incorrect Answer ImageB.Pancreatitis from hyperparathyroidism
 Incorrect Answer ImageC.Pseudohypoparathyroidism
 Incorrect Answer ImageD.Removal of the wrong parathyroid gland
 Incorrect Answer ImageE.Surgical damage to the remaining parathyroid glands

A 32-year-old woman comes to the office for evaluation of hypertension detected on a public
health screen in the local park where her blood pressure was 178/94 mm Hg. She has not had any
major health problems in the past and is currently asymptomatic. She denies gross hematuria,
headaches, visual changes, chest pain, cough, hemoptysis, dyspnea, numbness, weakness, skin
rash, or arthralgias. She does not take any medications. She does not smoke, drink alcohol, or use
illicit drugs. She has never been pregnant. Her mother has diabetes and her father has
hypertension. At this visit, her blood pressure is 180/92 mm Hg in the left arm and 178/90 mm
Hg in the right arm, and pulse is 88/min. The optic fundi are normal. The neck is supple with no
JVD or lymphadenopathy. Cardiopulmonary examination shows no abnormalities. Abdominal
examination is normal. Extremities show no edema. The serum BUN is 21 mg/dL, creatinine 2.0
mg/dL. Prior BUN and creatinine measurements are not available. Complete blood count and
serum electrolytes, blood sugar, and liver function studies are normal. Urinalysis shows:
Specific gravity 1.015
pH 5.5 
Blood 1+
Protein 2+
Ketones Negative
Glucose Negative
RBCs 15–20/HPF
WBCs 0/HPF
Epithelial cells Negative
Casts 1–2 RBC casts/LPF
Crystals Negative
Anti-antistreptolysin O, ANA, hepatitis B/C, HIV titers are negative, and C3 and C4 complement
levels are normal. A 24-hour urine test collection shows 2.2 g of protein in 24 hours. Which of
the following is the most appropriate next step in management?
 Incorrect Answer ImageA.CT scan of the abdomen
 Incorrect Answer ImageB.Cystoscopy
 Correct Answer ImageC.Kidney biopsy
 Incorrect Answer ImageD.No further tests are required
 Incorrect Answer ImageE.Skin biopsy
 Incorrect Answer ImageF.Urine culture

A 30-year-old man is admitted to the intensive care unit after a motor-vehicle accident in which
he sustained a right femur fracture. On admission, his leukocyte count is 30,000/mm3. Physical
examination shows rales in the right lung base. There is no hepatosplenomegaly. Laboratory
values show hemoglobin 9 g/dL, leukocyte count 55,000/mm3 with 95% neutrophils, and
platelets 160,000/mm3. No myelocytes or metamyelocytes are noted. Leukocyte alkaline
phosphatase level is high. Which of the following is the most likely cause of the high leukocyte
count? 
 Incorrect Answer ImageA.Agnogenic myeloid metaplasia
 Incorrect Answer ImageB.Chronic lymphocytic leukemia
 Incorrect Answer ImageC.Chronic myelogenous leukemia
 Correct Answer ImageD.Leukemoid reaction
 Incorrect Answer ImageE.Polycythemia vera

A 68-year-old man comes to the emergency department because of a 3-hour history of pain in his
left foot. He has a 20-year history of hypertension, chronic renal insufficiency, and peptic ulcer
disease. Current medications include hydrochlorothiazide, nifedipine, and omeprazole. His
temperature is 37.8ºC (100ºF), blood pressure is 140/85 mm Hg, and pulse is 85/min. A
photograph of his foot is shown. Arthrocentesis shows needle-shaped, negatively birefringent
crystals. Serum creatinine level is 2.4 mg/dL. Which of the following is the most appropriate
pharmacotherapy for this patient?
 Incorrect Answer ImageA.Allopurinol
 Correct Answer ImageB.Corticosteroids
 Incorrect Answer ImageC.High-dose colchicine
 Incorrect Answer ImageD.Indomethacin
 Incorrect Answer ImageE.Probenecid

A 60-year-old woman with a 1-year history of osteoarthritis comes to the physician because of
pain in her left knee not relieved by 2,200 mg of ibuprofen per day. She denies abdominal pain
or blood in her stool. She had a 4.5-kg (10-lb) weight gain over the past year and currently
weighs 90.7 kg (200 lbs) and is 160 cm (63 in). Her temperature is 37°C (98.6°F), blood pressure
is 125/75 mm Hg, and pulse is 70/min. The left knee shows moderate joint effusion that is
diffusely tender to palpation with crepitus, and mild medial and lateral joint instability. Which of
the following is the most appropriate next step in management?
 Incorrect Answer ImageA.Add acetaminophen to her regimen
 Incorrect Answer ImageB.Intra-articular injection of steroids every 2 weeks for 6 months
 Incorrect Answer ImageC.Knee replacement
 Incorrect Answer ImageD.Switch to a different non-steroidal anti-inflammatory drug
(NSAID)
 Correct Answer ImageE.Weight loss plus resistance training

A 70-year-old woman with a history of hypertension and hypercholesterolemia comes to the


physician because of worsening stable angina. She used to smoke one pack of cigarettes daily for
35 years but quit 5 years ago. Physical examination shows a thrill at the sternal notch on
palpation, an S4 on auscultation, and a crescendo-decrescendo systolic murmur in the
precordium. The murmur decreases with a Valsalva maneuver and upon standing, and increases
while squatting. Which of the following is the most likely diagnosis?
 Incorrect Answer ImageA.Aortic regurgitation
 Correct Answer ImageB.Aortic stenosis
 Incorrect Answer ImageC.Mitral regurgitation
 Incorrect Answer ImageD.Mitral stenosis
 Incorrect Answer ImageE.Pulmonary regurgitation
 Incorrect Answer ImageF.Pulmonary stenosis

A 35-year-old man comes to the physician because of headaches and difficulty with his vision
for 6 months. He often hits the curb while driving. He has had no fever, neck stiffness, loss of
consciousness, or seizures. He takes no medications. His temperature is 37ºC (98.6°F), blood
pressure is 120/70 mm Hg, pulse is 83/min, and respirations are 14/min. Neurologic examination
shows decreased vision bilaterally in the temporal fields. Muscle strength is 5/5 in all
extremities. Reflexes and sensation are normal. MRI of the brain shows a pituitary adenoma.
Which of the following tests is most likely to be abnormal in this patient? 
 Incorrect Answer ImageA.Adrenocorticotropic hormone (ACTH)
 Incorrect Answer ImageB.Growth hormone (GH)
 Correct Answer ImageC.Prolactin
 Incorrect Answer ImageD.Somatomedin C
 Incorrect Answer ImageE.Thyroid-stimulating hormone (TSH)

A 92-year-old man with a history of pulmonary emphysema is intubated in the intensive care
unit because of progressive pneumonia that has failed to improve after 72 hours of antibiotic
therapy. Although the inspired fraction of oxygen is 100%, the patient's pO2 remains at 57 mm
Hg. Positive end-expiratory pressure (PEEP) is added and twelve hours later the patient suddenly
becomes hypotensive with oxygen saturation dropping from 92% to 61%. His blood pressure is
80/50 mm Hg and pulse is 124/min. He has distended neck veins and distant heart sounds. Which
of the following would also most likely be seen on this patient's physical examination?
 Correct Answer ImageA.Absence of breath sounds on the affected side
 Incorrect Answer ImageB.High-amplitude carotid artery upstroke
 Incorrect Answer ImageC.Pleural friction rub
 Incorrect Answer ImageD.Pulsus alternans
 Incorrect Answer ImageE.Splenomegaly

A 19-year-old man comes to the physician for a follow-up visit 48 hours after the placement of a
purified protein derivative (PPD) tuberculin skin test. His PPD test showed 7 mm of induration
when he was tested 3 weeks ago. He has had no cough, fever, night sweats, weight loss, or sick
contacts. He emigrated from India 3 years ago and has not returned to India since then. He works
as a bartender. His medical history is unremarkable, and he takes no medications. He does not
smoke cigarettes and drinks alcohol socially. His temperature is 37.1°C (98.8°F), pulse is
78/min, respirations are 16/min, and blood pressure is 118/79 mm Hg. The lungs are clear to
auscultation. Cardiac examination shows a normal S1 and S2; no murmurs are heard. The PPD
that was placed 48 hours ago shows 13 mm of induration. Which of the following is the most
likely explanation for the increase in induration as compared to the previous test?
 Incorrect Answer ImageA.Active pulmonary tuberculosis
 Correct Answer ImageB.Booster phenomenon
 Incorrect Answer ImageC.Disseminated tuberculosis
 Incorrect Answer ImageD.False positive
 Incorrect Answer ImageE.Interval development of tuberculosis since his prior test

A 28-year-old woman with a history of asthma and seasonal allergies comes to the physician
because of a 7-kg (15-lb) weight gain. She has had no change in her diet and she walks 2 miles
daily. She has been on the same medications for several months which include cetirizine,
prednisone, and inhaled fluticasone-salmeterol. The combined inhaler was added 2 months ago
due to frequent albuterol use. She has had no cough, wheeze or shortness of breath, and has not
used her PRN albuterol for 4 weeks. She is 160 cm (63 in) tall, and she weighs 86 kg (190 lb).
Her temperature is 37.1ºC (98.8ºF), blood pressure is 150/75 mm Hg, pulse is 90 /min, and
respirations are 16/min. The lungs are clear to auscultation. The remainder of the examination
shows no abnormalities. Which of the following is the most appropriate next step in
management? 
 Incorrect Answer ImageA.Discontinue inhaled budesonide and re-evaluate in 1 month
 Incorrect Answer ImageB.Discontinue prednisone and re-evaluate in 1 month
 Incorrect Answer ImageC.High-dose dexamethasone suppression test
 Incorrect Answer ImageD.Low-dose dexamethasone suppression test
 Incorrect Answer ImageE.Measure 24-hr urinary excretion of cortisol
 Incorrect Answer ImageF.Measure morning serum cortisol
 Correct Answer ImageG.Taper the dose of prednisone and re-evaluate in 1 month

A 32-year-old man comes to the physician because of a 2-week history of fecal urgency, a
sensation of incomplete rectal evacuation, and bloody diarrhea that occurs up to 12 times daily.
He has had abdominal cramps in his left lower abdomen. He has not traveled outside the country.
He has a history of Pneumocystis jiroveci pneumonia 8 months ago. He does not drink alcohol.
He has smoked a pack of cigarettes daily for 14 years. He used heroin daily for 8 years, but quit
5 years ago. Which of the following is the most likely causal organism?
 Incorrect Answer ImageA.Clostridium difficile 
 Incorrect Answer ImageB.Cryptosporidium parvum 
 Correct Answer ImageC.Cytomegalovirus
 Incorrect Answer ImageD.Entamoeba histolytica 
 Incorrect Answer ImageE.Enterotoxigenic Escherichia coli
A 31-year-old woman with HIV is brought to the emergency department by ambulance
accompanied by her husband after a seizure that lasted 2 minutes. She has had headaches and
fevers for the past 2 months. Her husband also reports that she "has not been herself lately."
Current medications include emtricitabine, tenofovir, and efavirenz. On arrival, she is alert and
oriented. Her temperature is 38.5ºC (101.3ºF). Physical examination shows no abnormalities.
Her CD4+ T lymphocyte count is 68/µL. A CT scan of the head is shown. The lesions and
symptoms respond to a 6-week course of pyrimethamine and sulfadiazine. Which of the
following is the most appropriate next step in management?
 Incorrect Answer ImageA.Azithromycin
 Incorrect Answer ImageB.Biopsy of one of the lesions
 Incorrect Answer ImageC.Change antiviral medications to emtricitabine, tenofovir, and
raltegravir
 Correct Answer ImageD.Lower the doses of pyrimethamine and sulfadiazine
 Incorrect Answer ImageE.Radiation and corticosteroids

 30-year-old woman with a history of asthma comes to the physician because of a low-grade
fever and cough productive of whitish sputum with brown-colored cords in the phlegm for 2
weeks. She has completed a 5-day course of azithromycin with no improvement. She has had no
hemoptysis, weight loss, chest pain, or leg swelling. Current medications include inhaled
salmeterol and fluticasone. She has no allergies. She has not had any recent travel. She smokes
marijuana daily and does not use any other illicit drugs. Her temperature is 37.3ºC (99.1ºF). The
lungs are clear to auscultation. Laboratory studies show: 
Leukocyte count 13,000/mm3
Segmented neutrophils 44%
Eosinophils 20%
Lymphocytes 30%
Monocytes 6% 
A radiograph of the chest is shown. Which of the following is the most likely diagnosis?
 Incorrect Answer ImageA.Acute eosinophilic pneumonia
 Correct Answer ImageB.Allergic bronchopulmonary aspergillosis
 Incorrect Answer ImageC.Churg-Strauss syndrome
 Incorrect Answer ImageD.Löffler syndrome
 Incorrect Answer ImageE.Mycoplasma pneumonia
 Incorrect Answer ImageF.Psittacosis

Four friends go on a camping trip in the desert but one of them gets separated from the group. He
is found and rescued 3 days later. When brought to the emergency department he is awake and
alert, and reports muscle weakness and cramps. His vital signs include blood pressure 100/70
mm Hg, pulse 110/min, and respirations 19/min. Physical examination shows decreased skin
turgor and dry mucous membranes. Laboratory results show Na+ 152 mEq/L, K+ 5.2 mEq/L,
and Cl− 107 mEq/L. After providing a bolus of 1 liter of normal saline, which of the following
would be the best IV maintenance fluid to rehydrate this patient?
 Correct Answer ImageA.0.45% sodium chloride
 Incorrect Answer ImageB.0.9% sodium chloride
 Incorrect Answer ImageC.3% sodium chloride
 Incorrect Answer ImageD.5% dextrose
 Incorrect Answer ImageE.Ringer's lactate

 50-year-old man comes to the emergency department because of a 2-week history of dull
abdominal pain. He reports a gnawing periumbilical pain that has no association with food
intake. He denies fever, nausea, vomiting, changes in stool color, diarrhea, and constipation. He
has no history of significant medical illnesses, and he does not take any medications. Vital signs
are normal, and physical examination shows mild tenderness to palpation over the periumbilical
region. Laboratory studies are unremarkable. A CT scan of the abdomen shows a 1.5 cm
hypodense, homogenous, right adrenal nodule with smooth, rounded contours. Which of the
following is the most appropriate next step in management of the adrenal nodule?
 Correct Answer ImageA.Assure the patient that the adrenal nodule does not appear to be
malignant
 Incorrect Answer ImageB.Fine-needle aspiration biopsy
 Incorrect Answer ImageC.Open biopsy
 Incorrect Answer ImageD.Serial CT scan of the abdomen every 6 months for the next 3
years
 Incorrect Answer ImageE.Surgery for removal of the right adrenal gland

A heterosexual 52-year-old man has had mouth dryness and an unpleasant taste for 3 weeks. He
has no other complaints. He denies pain on swallowing. He reports that he has no chronic
medical conditions, takes no medications, and that he is "surprisingly healthy" for his age
compared with his friends. He plays racquetball on the weekends and lifts weights during the
week. He drinks a glass of wine with dinner and does not smoke cigarettes. He works as a
salesman for a large computer company and often attends business conferences in Asia and
Africa. He was married for 25 years, but his wife died from breast cancer 3 years ago. He has 2
grown children who now have children of their own. He enjoys spending time with his
grandchildren and says that they are the reason he needs to stay healthy. His mother and father
both died from complications of diabetes mellitus. His temperature is 37.7ºC (99.9ºF). An image
of his mouth is shown. The lesions are easily rubbed off with a tongue depressor. The remainder
of the examination is unremarkable. Nonfasting blood glucose level is 103 mg/dL. Which of the
following is the most appropriate management?
 Incorrect Answer ImageA.Prescribe oral acyclovir and recommend HIV testing
 Incorrect Answer ImageB.Prescribe oral acyclovir and reevaluate in 2 to 4 weeks
 Incorrect Answer ImageC.Prescribe clotrimazole troches and determine fasting blood
glucose level
 Correct Answer ImageD.Prescribe clotrimazole troches and recommend HIV testing
 Incorrect Answer ImageE.Prescribe clotrimazole troches and reevaluate in 2 to 4 weeks
for HIV testing

A 60-year-old man comes to the emergency department because of a 2-day history of severe
abdominal pain. He denies nausea, vomiting, or diarrhea. He does not drink alcohol or smoke
cigarettes. He has not traveled outside of the country and has no sick household contacts.
Examination shows mild jaundice. The liver has a span of 14 cm and is tender and smooth on
palpation. Laboratory studies show: 
Hemoglobin 20.1 g/dL
Hematocrit 63%
Leukocyte count 11,800/mm3
Platelet count 476,000/mm3
Serum studies show:
Aspartate aminotransferase (AST, GOT)  412 U/L
Alanine aminotransferase (ALT, GPT)  622 U/L
Peripheral smear shows normal-appearing morphology of the cells. Which of the following is the
most likely diagnosis? 
 Incorrect Answer ImageA.Alcoholic cirrhosis
 Correct Answer ImageB.Budd-Chiari syndrome
 Incorrect Answer ImageC.Cholelithiasis
 Incorrect Answer ImageD.Chronic hepatitis B
 Incorrect Answer ImageE.Fulminant hepatitis A

A 71-year-old man comes to the physician because of a 2-month history of episodes of severe
pain over the right jaw that lasts for 60 seconds. The pain is sudden in onset, "shock-like" and
comes on after brushing his teeth or chewing tough food. He has a history of hypertension, and
he had a myocardial infarction 4 years ago. Current medications include aspirin, metoprolol,
lisinopril, and atorvastatin. His blood pressure is 132/84 mm Hg and pulse is 64/min.
Examination shows no abnormalities. Laboratory studies show:
Hemoglobin 14.2 mg/dL
Leukocyte count 7,800/mm3
Platelet count 308,000/mm3
Erythrocyte sedimentation rate 9 mm/hr
Which of the following is the most appropriate pharmacotherapy? 
 Incorrect Answer ImageA.Baclofen
 Correct Answer ImageB.Carbamazepine
 Incorrect Answer ImageC.Indomethacin
 Incorrect Answer ImageD.Prednisone
 Incorrect Answer ImageE.Sumatriptan

A 72-year-old man was admitted to the hospital 10 days ago because of a cerebrovascular
accident. His past medical history includes hypertension, coronary artery disease, and non-
insulin-dependent diabetes mellitus. The patient's neurologic condition slowly improved, but on
hospital day 8, he developed cough, shortness of breath, and fever. Chest x-ray at that time
showed an opacity in the right middle lobe. The patient was diagnosed with hospital-acquired
pneumonia, and sputum samples are obtained to assess the etiology. Gram stain showed multiple
leukocytes and gram-negative coccobacilli. He was started on levofloxacin. Despite three days of
treatment with levofloxacin, his condition has failed to improve. Culture results
show Acinetobacter baumannii. Drug sensitivity test results are pending and will be available
within the next 24 hours. Which of the following is the most appropriate next step in
management? 
 Incorrect Answer ImageA.Add cefepime to current treatment until antibiogram is
available and modify therapy accordingly
 Incorrect Answer ImageB.Continue levofloxacin until antibiogram is available and
modify therapy accordingly
 Incorrect Answer ImageC.Switch to ceftriaxone
 Incorrect Answer ImageD.Switch to ciprofloxacin
 Correct Answer ImageE.Switch to imipenem

A 20-year-old man with a history of nephrotic syndrome undergoes a renal biopsy. He came to
his physician 1 week ago with generalized edema and foamy urine without hematuria. His blood
pressure is 100/60 mm Hg. Examination shows periorbital edema, with clear lungs, a normal
cardiac examination, and 4+ edema and anasarca. His 24-hour urine contains 8 grams of protein
and his serum creatinine is 0.8 mg/dL. His urinalysis shows 4+ protein but no red cells or casts.
Electron microscopic evaluation of the renal biopsy shows that the basement membrane of the
glomeruli is morphologically normal, with no electron-dense material deposition. In the visceral
epithelial cells (podocytes), there is a uniform and diffuse effacement of foot processes. Which
of the following is the most appropriate pharmacotherapy?
 Incorrect Answer ImageA.Cyclophosphamide
 Incorrect Answer ImageB.Mycophenolate mofetil
 Incorrect Answer ImageC.Observation
 Correct Answer ImageD.Prednisone
 Incorrect Answer ImageE.Rituximab

A 50-year-old man comes to the clinic for evaluation of an elevated fasting blood glucose level,
which was discovered as part of a preoperative evaluation for bilateral carpal tunnel surgery. He
also reports a history of hypertension and nonspecific arthralgias of the knees, ankles, and hips.
A review of systems demonstrates that over the past year, he has experienced significant
headaches, excessive diaphoresis, and impotence. He states that his appearance has changed
remarkably, which he noticed while watching his friend's wedding video. He says he now looks
much older. He also noticed recent changes in his facial appearance and says he wears a larger
hat size. Current medications include hydrochlorothiazide and a daily aspirin. His pulse is
62/min and blood pressure is 155/75 mm Hg. Physical examination shows coarse skin, skin tags,
and large, fleshy hands and feet. Laboratory tests are ordered to confirm the likely diagnosis.
Which of the following is most likely to be elevated?
 Incorrect Answer ImageA.Alkaline phosphatase
 Incorrect Answer ImageB.Cortisol
 Correct Answer ImageC.IGF-1
 Incorrect Answer ImageD.Prolactin
 Incorrect Answer ImageE.Somatostatin
 Incorrect Answer ImageF.Thyroid-stimulating hormone (TSH)

A 52-year-old woman comes to the urgent care clinic with difficulty breathing. She has a history
of bronchial asthma and has suffered with this disease since an early age. For the past years she
has had to use her short-acting bronchodilator inhaler approximately 3 or 4 times per month.
Over the past 2 months, however, she has had to use the inhaler 3 times per week. She has no
other past medical history and takes no medications besides her inhaler. She denies smoking or
alcohol use. On physical examination, you note her appearance (click media file) and
auscultation shows diffuse wheezes. No abnormal heart sounds are heard on auscultation. The
remainder of her physical examination is normal. She is given a nebulizer treatment, which
improves her acute symptoms. Which of the following is the next step in management?
 Correct Answer ImageA.Add an inhaled corticosteroid to her regimen
 Incorrect Answer ImageB.Add theophylline to her regimen
 Incorrect Answer ImageC.Administer subcutaneous epinephrine
 Incorrect Answer ImageD.Change her short-acting beta agonist to a long-acting one
 Incorrect Answer ImageE.Encourage more frequent inhaler use

A 45-year-old woman comes to the physician because of progressive weakness of 4 months


duration. She has had difficulty climbing stairs and lifting objects over her head. She had a fever
up to 38.5ºC (101.3ºF). Her medical history is otherwise unremarkable. She takes no
medications. She drinks alcohol on social occasions and does not smoke cigarettes. Physical
examination shows palpable proximal muscle tenderness in the upper arm and lower extremities
without atrophy. There is periorbital edema and an erythematous rash on the eyelids. An
photograph of her hands is shown. Laboratory studies show:
Hemoglobin 13.2 g/dL
Leukocyte count  9,300/mm3
Creatine kinase 2,550 U/L
Erythrocyte sedimentation rate (ESR)  22 mm/hr
Antinuclear antibody (ANA) titer  Positive at 1:1280
Rheumatoid factor  Positive at 1:512
Which of the following is the most likely diagnosis?
 Correct Answer ImageA.Dermatomyositis
 Incorrect Answer ImageB.Mixed connective tissue disease
 Incorrect Answer ImageC.Polymyalgia rheumatica
 Incorrect Answer ImageD.Psoriatic arthritis
 Incorrect Answer ImageE.Scleroderma

A 47-year-old man comes to the physician because of pain when he urinates. He has been
urinating more often than usual and has now developed burning on urination. The pain started
vaguely in the pelvis. He denies any experiences of similar episodes in the past. His temperature
is 38.5°C (101.3°F). Blood pressure and pulse are within normal limits. His abdomen is non-
distended and is soft to palpation with no tenderness or palpable masses. Bowel sounds are
normoactive. On genital examination, there is no urethral discharge and testes are non-tender.
Digital rectal examination causes a significant amount of pain and is unable to be adequately
completed. Which of the following is the most likely diagnosis?
 Incorrect Answer ImageA.Anal fissure
 Incorrect Answer ImageB.Cystitis
 Incorrect Answer ImageC.Epididymitis
 Correct Answer ImageD.Prostatitis
 Incorrect Answer ImageE.Urethriti

A 53-year-old man with a longstanding history of alcohol abuse sees his physician for a 2 to 3-
month history of progressive malaise, increased abdominal girth, weight loss, decreased appetite,
and yellow eyes. He has not seen a physician in several years. He smokes one pack of cigarettes
per day and consumes approximately 5 to 6 beers per day. He used intravenous drugs
approximately 20 years ago but denies current use. He takes no medications. His pulse is 95/min
and his blood pressure is 101/67 mm Hg. On physical examination, he has temporal wasting,
scleral icterus, and jaundice. He has scattered spider angiomata, gynecomastia, palmar erythema,
abdominal fluid wave, a nodular liver edge extending 3 cm below the right costal margin and
splenomegaly. An abdominal ultrasound shows a small liver and ascitic fluid. Which of the
following is most likely to be found in the ascites fluid?
 Incorrect Answer ImageA.Elevated amylase
 Incorrect Answer ImageB.Elevated triglycerides
 Incorrect Answer ImageC.Lymphocytosis
 Incorrect Answer ImageD.Neutrophilia

 Correct Answer ImageE.Serum-ascites albumin gradient >1.1

A 20-year-old pregnant woman is experiencing rapidly progressive hearing loss. Several


relatives in her family developed hearing loss prior to age 25. Otoscopic examination shows
normal tympanic membranes bilaterally. Hearing testing shows a conductive hearing loss, with
one ear being affected more severely than the other. Which of the following is the most likely
diagnosis?
 Incorrect Answer ImageA.Acoustic neuroma
 Incorrect Answer ImageB.Chronic otitis media
 Incorrect Answer ImageC.Meniere disease
 Correct Answer ImageD.Otosclerosis
 Incorrect Answer ImageE.Presbycusis

A 46-year-old man comes to the physician because of an itchy rash in his groin for 3 weeks. He
experienced no relief after using an over-the-counter antifungal cream. Past medical history is
significant for type 2 diabetes mellitus controlled with diet and essential hypertension treated
with losartan. He has no known allergies to medications and does not drink or smoke. Physical
examination shows sharply demarcated, dry, brown, slightly scaly patches bilaterally in both
genitocrural folds. There are similar but smaller patches in the axillae and in the fourth
interdigital space on the feet. A Woods light examination shows coral-red fluorescence in all
three locations. Which of the following is the most likely diagnosis?
 Correct Answer ImageA.Erythrasma
 Incorrect Answer ImageB.Intertrigo
 Incorrect Answer ImageC.Inverse psoriasis
 Incorrect Answer ImageD.Seborrheic dermatitis
 Incorrect Answer ImageE.Tinea cruris

A 55-year-old man comes to the physician because he "bleeds too much" every time he brushes
his teeth. For the past three days, he has not been feeling well and has been also complaining of
headaches and chest pain. He has been trying to workout lately but as soon as he starts walking,
he feels very fatigued and has to stop. His temperature is 39ºC (102ºF), pulse is 110/min,
respirations are 20/min, and blood pressure is 130/80 mm Hg. There is a 2/6 systolic ejection
murmur heard on auscultation. Lung examination shows bilateral rhonchi. There are scattered
petechiae over the thorax and extremities. The patient is oriented to person, time, and place, but
has 4/5 strength throughout with normal reflexes. Laboratory studies show: 
Hemoglobin  7.1 gm/dL
Platelets  30,000/mm3
White blood cells  11,800/mm3
Peripheral blood numerous blasts
Bone marrow biopsy hypercellular with 35% blasts
Blast cells presence of rod-shaped structures in the cytosol 
Myeloperoxidase positive
Chest X-ray scattered infiltrations on the base, bilaterally 
Which of the following is the most likely diagnosis?
 Incorrect Answer ImageA.Acute lymphocytic leukemia
 Correct Answer ImageB.Acute myelogenous leukemia
 Incorrect Answer ImageC.Aplastic anemia
 Incorrect Answer ImageD.Chronic myelogenous leukemia
 Incorrect Answer ImageE.Myelofibrosis

A 15-year-old boy comes to the emergency department because of nasal bleeding from the left
nostril for 20 minutes. He has had no history of trauma to his nose or face. He has had episodes
of epistaxis for the past 4 months that are increasing in severity and frequency. He also has had
decreased hearing, nasal congestion, and rhinorrhea with occasional blood-tinged nasal discharge
from his left nostril. His temperature is 37°C (98.6°F), blood pressure is 110/75 mm Hg, and
pulse is 68/min. Examination with bedside nasopharyngoscopy shows a firm, grayish-red mass in
the left nasopharynx. There are no skin lesions. Which of the following is the most likely
diagnosis?
 Incorrect Answer ImageA.Inverted papilloma
 Correct Answer ImageB.Juvenile nasopharyngeal angiofibroma
 Incorrect Answer ImageC.Nasopharyngeal carcinoma
 Incorrect Answer ImageD.Pyogenic granuloma
 Incorrect Answer ImageE.Rhabdomyosarcoma

A 37-year-old man with a history of type 1 diabetes mellitus and hypertension comes to the
physician because of erectile dysfunction. He reports that two weeks ago, after heavy eating and
heavy drinking, he was unable to get an erection when attempting intercourse with his wife. He
has remained impotent since then. He does report having an erection in the morning when he
wakes up. There is no history of perineal trauma or pelvic surgery. His diabetes is well controlled
with insulin. He was recently started on gemfibrozil and metoprolol for hypertriglyceridemia and
hypertension, respectively. His testosterone level is 557 ng/dL (normal, 437–707 ng/dL). Which
of the following is the most likely underlying cause of this patient's current complaint? 
 Incorrect Answer ImageA.Atherosclerosis
 Incorrect Answer ImageB.Diabetes mellitus
 Incorrect Answer ImageC.Gemfibrozil
 Incorrect Answer ImageD.Metoprolol
 Correct Answer ImageE.Psychogenic causes

One week after an upper respiratory infection, an adult develops conjunctival hyperemia, watery
discharge, and ocular irritation. While both eyes are involved by the time a physician is
consulted, the symptoms began in one eye. On the morning of the doctor's visit, the patient had
difficulty opening his eyelids on awakening as they were "glued shut". Physical examination
demonstrates hyperemic bulbar and tarsal conjunctiva. There is a watery, non purulent discharge
noted. The right preauricular lymph node is enlarged on palpation. Which of the following
pathogens would most likely cause these symptoms?
 Correct Answer ImageA.Adenovirus
 Incorrect Answer ImageB.Herpes simplex I
 Incorrect Answer ImageC.Herpes simplex II
 Incorrect Answer ImageD.Neisseria gonorrhoeae
 Incorrect Answer ImageE.Staphylococcus aureus

A 73-year-old woman comes to the physician because of a 6-month history of an enlarging dark
skin lesion on her right temple. She had a basal cell carcinoma removed from her back 2 years
ago. Her past medical history is also significant for type 2 diabetes mellitus and essential
hypertension. Current medications include metformin, pentoxifylline, and nifedipine. Physical
examination shows a 2 x 3 cm, dark brown, waxy, keratotic plaque with a verrucous surface
punctuated by multiple pseudocysts on the right forehead. It appears to be “pasted” on her skin.
Gentle scraping of the surface causes keratotic debris to dislodge. Which of the following is the
most likely diagnosis?
 Incorrect Answer ImageA.Actinic keratosis
 Incorrect Answer ImageB.Basal cell carcinoma
 Incorrect Answer ImageC.Melanoma
 Correct Answer ImageD.Seborrheic keratosis
 Incorrect Answer ImageE.Solar lentigo

A 40-year-old woman comes to the physician for a routine health maintenance examination. Her
medical history is unremarkable and she takes no medications. She does not smoke cigarettes or
drink alcohol. She works as a librarian. She is not sexually active. Her sister had breast cancer at
age 44, and her father died of colon cancer at age 63. Her temperature is 37°C (98.6°F), pulse is
78/min, respirations are 18/min, and blood pressure is 120/68 mm Hg. Physical examination
shows no abnormalities. Which of the following screening tests is most appropriate in this
patient? 
 Incorrect Answer ImageA.CA-125 level
 Incorrect Answer ImageB.Fecal occult blood testing
 Incorrect Answer ImageC.Lipid panel
 Correct Answer ImageD.Mammography
 Incorrect Answer ImageE.Ovarian ultrasound

A 34-year-old woman with no significant medical history who is healthy without underlying
medical problems comes to the clinic with complaints of fever with a temperature up to 38.3ºC
(101.0ºF) and cough with greenish sputum production for 2 days without dyspnea. Her blood
pressure is 120/80 mm Hg, pulse is 88/min, and respirations are 18/min. There is no accessory
muscle use, conversational dyspnea, wheezes, bronchial breath sounds, rales, or egophony over
the right lower lung fields. Chest radiograph shows a right lower lobe consolidation. Leukocyte
count is 13,000/mm3. Which of the following is the most appropriate pharmacotherapy?
 Correct Answer ImageA.Amoxicillin plus azithromycin
 Incorrect Answer ImageB.Ampicillin-sulbactam plus doxycycline
 Incorrect Answer ImageC.Ceftriaxone plus azithromycin
 Incorrect Answer ImageD.Ciprofloxacin plus piperacillin-tazobactam
 Incorrect Answer ImageE.Clarithromycin

A 37-year-old woman comes to the emergency department because of difficulty with her vision.
She has noticed symptoms of "clear things floating in front of [her] eyes" and "flashing lights"
over the past 2 days. She states that she was diagnosed with HIV several years ago, but has not
seen a physician since the diagnosis. Her appetite has been poor recently, and she has noticed
significant weight loss. She is afebrile, pulse is 90/min, blood pressure is 105/62 mm Hg, and
oxygen saturation is 96% on room air. Head and neck examination shows cervical
lymphadenopathy. Her chest is clear to auscultation, and heart sounds are normal. Peripheral
examination shows poor skin turgor without cyanosis, clubbing, or edema. Which of the
following is the most appropriate next step in management?
 Correct Answer ImageA.Dilated funduscopic examination
 Incorrect Answer ImageB.Intravenous foscarnet
 Incorrect Answer ImageC.Intravenous ganciclovir
 Incorrect Answer ImageD.Magnetic resonance imaging of the brain
 Incorrect Answer ImageE.Ocular ganciclovir implantation

A 24-year-old HIV-positive man who works at a homeless shelter comes to the emergency
department because of a cough and weight loss. He is given a provisional diagnosis
of pneumocystis jiroveci pneumonia (PJP) due to a low CD4 count and lack of antimicrobial
prophylaxis. He is treated with IV trimethoprim/sulfamethoxazole and prednisone. Despite
treatment, his condition progresses. The patient is transferred to a negative-pressure room and
sputum samples are collected for acid-fast bacillus (AFB) smear analysis and culture. He is
found to have active pulmonary tuberculosis and is started on a 4-drug regimen of isoniazid,
pyrazinamide, rifampin, and ethambutol, as well as pyridoxine. After appropriate inpatient
treatment he is discharged with close follow-up planned in the outpatient clinic. The patient
returns to the clinic 2 weeks later and is prescribed highly active antiretroviral therapy (HAART)
for the treatment of his human immunodeficiency (HIV). At that time he also reports sudden
blurred or “foggy” vision and pain that is worse with movement of the right eye. Visual testing
shows a visual acuity of 20/25 OS and 20/200 OD, with intact peripheral vision. Which of the
following is the most likely cause of this patient's symptoms?
 Correct Answer ImageA.Ethambutol
 Incorrect Answer ImageB.Isoniazid
 Incorrect Answer ImageC.Pyrazinamide
 Incorrect Answer ImageD.Pyridoxine
 Incorrect Answer ImageE.Rifampin

A 70-year-old woman with aortic stenosis is admitted to the hospital with chest pain. A
myocardial infarction is ruled out by cardiac enzymes, but the patient has recurrent symptoms
while being weaned off heparin. On hospitalization day 5, she has right arm pain, absent brachial
pulse on the right, and a cold distal right arm. Her hematocrit is 34% and platelets are
30,000/mm3. Her partial thromboplastin time is 64 seconds. Which of the following is the most
likely cause of this patient's absent brachial pulse? 
 Incorrect Answer ImageA.Embolization from aortic stenosis
 Correct Answer ImageB.Heparin-induced thrombocytopenia
 Incorrect Answer ImageC.Hypercoagulable state from immobilization
 Incorrect Answer ImageD.Paradoxical embolus
 Incorrect Answer ImageE.Vasospasm of the brachial artery

A 17-year-old boy is brought to the emergency department by his parents after they found him
wandering aimlessly in the street. His parents say that 1 day ago his friends dared him to drink a
bottle of windshield wiper fluid. The patient appears ill, is unable to speak coherently, and
complains of blurry vision. His temperature is 36.6ºC (98ºF), blood pressure is 110/70 mm Hg,
pulse is 100/min, and respirations are 24/min. Physical examination shows a poorly groomed boy
who is disheveled. Breath sounds are clear bilaterally. Cardiac examination shows a normal S1
and S2 with no murmurs. The abdomen is diffusely tender to palpation with hypoactive bowel
sounds. Neurologic examination shows dilated pupils. Laboratory studies show:
Na+ 140 mEq/L
K+ 3.0 mEq/L
Cl -
100 mEq/L
HCO3- 10 mEq/L
BUN  15 mg/dL
Creatinine  1.0 mg/dL
Arterial blood gas analysis is most likely to show which of the following? 

 Incorrect Answer ImageA.


 Correct Answer ImageB.
 Incorrect Answer ImageC.
 Incorrect Answer ImageD.
 Incorrect Answer ImageE.

A 35-year-old woman comes to the physician because of a 3-day history of fever and confusion.
She has no past medical history and does not smoke or use alcohol. She takes no medications.
Her temperature is 38.5ºC (101.3ºF), pulse is 98/min, and blood pressure is 110/75 mm Hg. She
is alert and oriented to person only. It is difficult to engage her in a focused conversation. There
are scattered petechiae and ecchymosis over the chest and both lower extremities. There is no
neck stiffness and no localizing neurologic signs. Laboratory studies show:
Hematocrit 21%
Hemoglobin 7.2 gm/dL
WBC 9,000/mm3
Platelets 13,000/mm3
MCV 89 µm3
Reticulocyte count 5%
Creatinine 1.8 mg/dL
AST 24 U/L
ALT 25 U/L
LDH 648 U/L
Total bilirubin 2.2 mg/dL
Prothrombin time 12 sec
Partial thromboplastin time 30 sec
A peripheral smear shows schistocytes and decreased platelets. Which of the following is the
most likely diagnosis? 
 Incorrect Answer ImageA.Disseminated intravascular hemolysis
 Incorrect Answer ImageB.HELLP syndrome
 Incorrect Answer ImageC.Hemolytic uremic syndrome
 Incorrect Answer ImageD.Idiopathic thrombocytopenic purpura
 Correct Answer ImageE.Thrombotic thrombocytopenic purpura

A 43-year-old man is admitted to the hospital with significant shortness of breath. The patient
was previously diagnosed with interstitial pulmonary disease, based on imaging studies and
pulmonary function testing. Despite multiple tests, the exact etiology of the patient's condition is
not established. Due to his worsening condition, bronchoscopy with bronchoalveolar lavage
(BAL) is performed. BAL shows the presence of big cells with pentalaminar in-foldings of the
cell membrane (Birbeck granules). Which of the following is the most likely diagnosis in this
patient? 
 Incorrect Answer ImageA.Alveolar proteinosis
 Correct Answer ImageB.Pulmonary Langerhans cell histiocytosis
 Incorrect Answer ImageC.Sarcoidosis
 Incorrect Answer ImageD.Tuberculosis
 Incorrect Answer ImageE.Granulomatosis with polyangiitis

A 37-year-old homeless man with schizophrenia comes to the hospital because of a dry cough,
hemoptysis, weakness, fever, sweating, and weight loss. He came voluntarily, wanting to get
"something for the cough." However, after necessary studies are performed, he is admitted
because of the high possibility of having active tuberculosis. On the unit, he is started on anti-
tuberculosis medication and an antipsychotic. Initially, he is cooperative. After 2 days, however,
he shouts at the nurses and demands to be discharged immediately. Conversations with
physicians and staff to persuade him that he needs treatment are not helpful and he threatens to
sign himself out of the hospital against medical advice. Which of the following facts should most
likely lead the physician to justify involuntary commitment?
 Incorrect Answer ImageA.Patient has a potentially life-threatening medical condition
 Correct Answer ImageB.Patient has tuberculosis
 Incorrect Answer ImageC.Patient is agitated
 Incorrect Answer ImageD.Patient is homeless and unable to keep follow-up appointments
 Incorrect Answer ImageE.Patient is schizophrenic

A 39-year-old African-American man has had a 16-pound weight gain along with progressively
worsening massive lower extremity edema and fatigue over the last 2 weeks. His urine is normal
colored but foamy. He denies any recent skin or throat infections, diabetes, or carpal tunnel
syndrome. He takes no medications and does not use illicit drugs. There is no personal or family
history of cancer and no history of urinary infection. His temperature is 37.0ºC (98.6ºF), pulse is
93/min, respirations are 18/min, and blood pressure is 170/70 mm Hg. Prior to the onset of the
present illness, his BMI was 22 kg/m2. Physical examination shows massive upper and lower
extremity edema together with sacral edema. His lungs have decreased breath sounds at the bases
bilaterally. Cardiac and abdominal examinations are within normal limits. He has no retinopathy.
Fasting laboratory studies show:
Sodium 130 mEq/L
Potassium 4.5 mEq/L
Chloride 100 mEq/L
Bicarbonate 24 mEq/L
Glucose 96 mg/dL
BUN 27 mg/dL
Creatinine 1.7 mg/dL
Albumin 1.7 g/dL
LDL cholesterol 450 mg/dL
Hemoglobin 12 mg/dL
HCT 31%
WBC 7,700/mm3
Platelets 270,000/mm3
Urinalysis:
Leukocyte esterase Negative
Nitrite Negative
Protein ++++
Blood Negative
24-h protein collection 8.3 g/24 h
Tests for HIV antibody and sickle hemoglobin are negative. Biopsy of the kidney would most
likely reveal which of the following findings on light microscopy? 
 Incorrect Answer ImageA.Diffuse interstitial infiltration with inflammatory cells
(tubulointerstitial nephritis)
 Correct Answer ImageB.Glomerulosclerosis involving segments of some glomeruli with
other glomeruli appearing normal (focal segmental glomerulosclerosis)
 Incorrect Answer ImageC.Kimmelstiel-Wilson lesions
 Incorrect Answer ImageD.Normal appearance
 Incorrect Answer ImageE.Normal glomerular cellularity, thickened glomerular capillary
walls (membranous nephropathy)

A 70-year-old woman with a history of congestive heart failure, atrial fibrillation, and dementia
comes to the physician for a routine health maintenance examination. Current medications
include digoxin, warfarin, fosinopril, aspirin, and furosemide. Her blood pressure is 124/82 mm
Hg, and her pulse is 76/min and irregular. The lungs are clear to auscultation. There is 1+ pedal
edema. Her international normalized ratio (INR) is 2.3. Her husband is taking herbal medications
and she inquires about herbal supplements. Which of the following herbal supplements is most
likely to increase this patient's risk for bleeding? 
 Correct Answer ImageA.Dong quai
 Incorrect Answer ImageB.Echinacea
 Incorrect Answer ImageC.Kava kava
 Incorrect Answer ImageD.Psyllium
 Incorrect Answer ImageE.St. John's wort

A 41-year-old non-insulin dependent patient with diabetes comes to the office because of a
strange taste and pain in her mouth. She has noticed these symptoms over the past week and they
seem to be getting worse. Aside from diabetes, her past medical history includes asthma since
childhood that she is able to control with an albuterol rescue inhaler and a twice daily
maintenance steroid inhaler. Review of systems and family history are unremarkable. Her
temperature is 37.0ºC (98.6ºF), pulse 73/min, respirations 20/min, and blood pressure 128/79
mmHg. Oral examination shows white patches on her tongue and a generally erythematous
mucosa. With a tongue depressor, some of the material is easily removed, leaving a raw,
bleeding surface. Physical examination is otherwise unremarkable. Which of the following is the
most appropriate initial step in management?
 Correct Answer ImageA.Clotrimazole troche
 Incorrect Answer ImageB.Discontinuation of steroid inhaler
 Incorrect Answer ImageC.Hemoglobin A1c
 Incorrect Answer ImageD.Oral fluconazole
 Incorrect Answer ImageE.Rinse the mouth with water after using the inhaler

A 26-year-old woman has a history of type 2 diabetes controlled with sitagliptin. Her
hemoglobin A1c level is 6.3%. The woman's physician gets a call from another physician
identifying herself as the physician of the patient's brother. Apparently, the patient's brother also
has a history of type 2 diabetes that is poorly controlled. The brother's physician asks if the
sister's physician would discuss the details of the sister’s treatment so she can recommend it to
the brother. Which of the following is the most appropriate response by the sister's physician?
 Incorrect Answer ImageA.Offer to meet with the brother's physician to discuss the sister's
treatment regimen
 Incorrect Answer ImageB.Offer to send a letter to the brother's physician, detailing the
treatment plan for the sister
 Incorrect Answer ImageC.Offer to send the brother's physician literature about the
benefits and risks of the sister's drug regimen
 Incorrect Answer ImageD.Provide the details of the sister's treatment over the phone to
the brother's physician
 Correct Answer ImageE.Refuse to provide any information to the brother's physician
without the sister's permission

A 67-year-old woman comes to the physician with fever and back pain. She states that five days
ago she developed burning pain with urination, followed by fever and chills. She has a history of
hypertension well controlled with enalapril. There is no history of diabetes mellitus. Her
temperature is 38.9ºC (102ºF), pulse is 102/min, respirations are 16/min, and blood pressure is
90/70 mm Hg. The examination shows mild distress with rigors and right flank tenderness. The
rest of the exam is normal. The blood leukocyte count is 16,000/mm3. Urinalysis shows 50
leukocytes/high-power field and many bacteria. She is admitted to the hospital and given
intravenous saline and ciprofloxacin. Which of the following clinical findings indicates the need
for an abdominal CT or ultrasound study in this patient? 
 Incorrect Answer ImageA.Abdominal pain
 Incorrect Answer ImageB.Costovertebral tenderness
 Incorrect Answer ImageC.Fever
 Correct Answer ImageD.Hypotension
 Incorrect Answer ImageE.Urine white blood cell cast

A 42-year-old woman comes to the emergency department because of double vision and
difficulty keeping her right eye open for 1 day. She had a severe migraine 2 days ago. She has
had no recent fever or seizure. She has a history of type 2 diabetes mellitus and migraine
headaches. Her mother died of a myocardial infarction at age 54, and her brother has renal
failure. Examination shows a droopy right eyelid and an inability to elevate, adduct, and depress
the right eye. The right pupil also appears dilated. Examination of the left eye shows no
abnormalities. Visual acuity is normal bilaterally. Hemoglobin A1c level is 9.4%. Which of the
following is the most likely cause of her visual complaints? 
 Incorrect Answer ImageA.Cavernous sinus thrombosis
 Incorrect Answer ImageB.Diabetic microvascular infarct
 Incorrect Answer ImageC.Hyperglycemia
 Correct Answer ImageD.Intracranial aneurysm
 Incorrect Answer ImageE.Ophthalmic migraine
 

A 45-year-old woman with a longstanding history of uncontrolled hypertension comes to the


physician because of a 2-month history of worsening shortness of breath with minimal exertion.
She denies chest pain. She takes no medications and has no allergies. She has smoked one pack
of cigarettes daily for the past 15 years. Her temperature is 37.7°C (99.9°F), pulse is 75/min,
respirations are 16/min, and blood pressure is 180/65 mm Hg. Physical examination shows no
abnormalities except a prominent precordial impulse. A chest x-ray shows a prominent left
ventricle (LV). ECG shows left ventricular hypertrophy. An echocardiogram shows concentric
LV hypertrophy, without areas of hypokinesis, ejection fraction of 58% and a LV wall thickness
of 1.4 cm. Which of the following is the most likely diagnosis?
 Incorrect Answer ImageA.Acute myocardial infarction
 Incorrect Answer ImageB.Chronic obstructive pulmonary disease
 Correct Answer ImageC.Diastolic dysfunction
 Incorrect Answer ImageD.Interstitial lung disease
 Incorrect Answer ImageE.Pulmonary embolism

Infectologia

A 45-year-old man presents to the emergency department with worsening blurry vision,
difficulty speaking, and difficulty swallowing. He reports that 2 colleagues are also sick with
similar symptoms. He also has nausea, constipation, and xerostomia. He has no history of travel
and takes no medications. On physical examination he is alert and oriented. Temperature is
normal. There is bilateral ptosis and pupillary dilatation. Muscle power is reduced in the shoulder
and arm and normal in the lower extremities. Sensory examination is unremarkable. Which of
the following is the most likely organism resulting in this patient's presentation? 
 A.Campylobacter jejuni
 Correct Answer B.Clostridium botulinum
 C.Cytomegalovirus
 D.Epstein-Barr virus
 E.Mycoplasma pneumoniae

A 62-year-old man is brought to the emergency department by his son because of a 3-week
history of fever, night sweats, and a cough productive of blood-tinged sputum for 3 weeks. He
has had a 5 kg (11 lb) weight loss. Two weeks ago, he was treated for an upper respiratory
infection with amoxicillin he took for only six days. He had a positive purified protein derivative
(PPD) test 4 years ago. He drinks 8-10 cans of beer daily and has smoked a pack of cigarettes
daily for 40 years. On physical examination, his temperature is 39ºC (102.2ºF), pulse is 96/min,
respirations are 20/min, and blood pressure is 140/90 mm Hg. Physical examination shows
halitosis with egophony and vocal and tactile fremitus in the right upper lobe. Cardiac
examination shows a normal S1 and S2 without murmurs. Radiograph of the chest shows a thick-
walled cavitary lesion in the right upper lobe. Which of the following is the most appropriate
next step in management?
 A.CT scan of the chest
 B.Fiberoptic bronchoscopy to aspirate fluids
 C.Gram stain and culture of sputum
 Correct Answer D.IV clindamycin
 E.IV vancomycin

A 48-year-old woman comes to the physician because of a 2-month history of fatigue and
malaise. She takes no medications. She has smoked 1 pack of cigarettes daily for 30 years. She
underwent emergency splenectomy following a motor vehicle accident 22 years ago. During that
operation, she was transfused with multiple units of red blood cells. Laboratory studies show: 
HAV IgG Positive
Anti-HBs Positive
Anti-HBc Negative
Anti-HCV Positive
Liver biopsy specimen shows chronic inflammation with moderate fibrosis. Which of the
following is the most appropriate pharmacotherapy for this patient? 
 Incorrect Answer ImageA.Azathioprine
 Incorrect Answer ImageB.Interferon-alpha (IFN-alpha)
 Incorrect Answer ImageC.Lamivudine and peginterferon
 Correct Answer ImageD.Ledipasvir/sofosbuvir
 Incorrect Answer ImageE.Ribavirin
A 67-year-old man comes to his physician because of rectal bleeding that started 2 weeks ago.
He has no symptoms of rectal pain or change in stool consistency but has noted blood in the
toilet water and on the toilet paper. Approximately 10 years ago, he had a benign colon polyp
that was removed endoscopically. Additional medical history is positive for hyperlipidemia and
coronary artery disease. Three years ago, he underwent placement of a permanent pacemaker.
Two years ago, following a myocardial infarction, he underwent coronary angiography with
angioplasty and placement of a coronary artery stent. He has been asymptomatic since that time,
and his exercise tolerance is good. There is no evidence of congestive heart failure, and
echocardiography shows a systolic ejection fraction of 50%. He has no drug allergies, and his
medications include metoprolol, aspirin, and atorvastatin. You refer the patient to a
gastroenterologist for colonoscopy to evaluate the rectal bleeding, but before you do so, the
patient asks about antibiotic endocarditis prophylaxis. What would be the best approach to
endocarditis prophylaxis for this patient?
 Incorrect Answer ImageA.Prescribe oral amoxicillin to be taken before the procedure
 Incorrect Answer ImageB.Tell the patient that he should be given parenteral ampicillin
and gentamicin before the procedure and again 6 hours later
 Incorrect Answer ImageC.Tell the patient that he should be given parenteral ampicillin
before the procedure
 Incorrect Answer ImageD.Tell the patient that he should be given parenteral vancomycin
before the procedure
 Correct Answer ImageE.Tell the patient that he should not receive endocarditis
prophylaxis as he does not require it

A 62-year-old truck driver comes to the emergency department with significant shortness of
breath. He denies any significant medical problems. He has smoked two packs of cigarettes per
day for the last 40 years. His family history is significant for hypertension and coronary artery
disease. His temperature is 38.4ºC (101.1ºF), pulse is 112/min, respirations are 24/min, and
blood pressure 128/64 mm Hg. His oxygen saturation is 87% on room air. The physical
examination is significant for decreased breath sounds with crackles at the bases bilaterally.
Arterial blood gas (ABG) analysis and complete blood count (CBC) show:
pH 7.4
PCO2 32 mm Hg
PO2 54 mm Hg
HCO3 –
20 mEq/l
Hemoglobin  14.8 g/dL
Hematocrit  43%
WBC  4,500/mm3
Neutrophils  94%
Lymphocytes  2%
Monocytes  2%
Eosinophils  1%
Basophils  1%
Platelets  256,000/mm3
Chest x-ray is shown. What is the most definitive diagnostic study that should be obtained next?
 Incorrect Answer ImageA.Basic metabolic panel
 Incorrect Answer ImageB.Bronchoscopy with bronchoalveolar lavage
 Incorrect Answer ImageC.CT scan of the chest
 Correct Answer ImageD.Immunofluorescence of induced sputum sample
 Incorrect Answer ImageE.Liver function tests

A 29-year-old scuba diver visits his physician for ear pain and whitish discharge from his right
ear for the last 3 days. The patient is otherwise healthy. He does not smoke and does not drink
alcohol. His vitals are temperature 36.9ºC (98.2ºF), blood pressure 120/68 mm Hg, pulse 76/min,
and respirations 14/min, and oxygen saturation of 100% on room air. Physical examination
shows pain with gentle traction of the external ear. Speculum examination of the auditory canal
shows erythema and swelling of the external auditory canal. There is also white discharge and
moist debris in the canal. The tympanic membrane is difficult to visualize. There is no pain on
palpation and percussion of the mastoid bone behind the ear. The remainder of the physical
examination is unremarkable. What is the most likely causative organism? 
 Incorrect Answer ImageA.Aspergillus
 Incorrect Answer ImageB.Candida
 Incorrect Answer ImageC.Haemophilus influenzae
 Incorrect Answer ImageD.Moraxella catarrhalis
 Correct Answer ImageE.Pseudomonas aeruginosa 

A 78-year-old man with type 2 diabetes mellitus comes to the emergency department because of
a 2-day history of fever, chills, and a headache. His temperature is 39.7ºC (103.5ºF).
Examination shows nuchal rigidity and photophobia. His hemoglobin A1c level is 11.8%. Lumbar
puncture is performed and cerebrospinal fluid analysis is shown below.
CSF Analysis:
Opening Pressure 300 mm H2O
Appearance Turbid
White Blood Cell 1,500/microL, 85% neutrophils, and 15%
Count lymphocytes
Glucose 25 mg/dL
Protein 250 mg/dL
Gram stain is negative and cultures are pending. Which of the following is the most appropriate
initial pharmacotherapy?
 Incorrect Answer ImageA.Ceftriaxone
 Incorrect Answer ImageB.Ceftriaxone and vancomycin
 Correct Answer ImageC.Ceftriaxone, vancomycin, and ampicillin
 Incorrect Answer ImageD.Penicillin G
 Incorrect Answer ImageE.Vancomycin
A 33-year-old man comes to the emergency department with fevers, chills, and a productive
cough. He has a prior history of cellulitis and tricuspid valve endocarditis due to intravenous
drug use. A recent HIV test at his methadone clinic is pending. On physical examination, his
temperature is 39.1ºC (102.4ºF), pulse is 98/min, respirations are 28/min, and blood pressure is
112/62 mm Hg. Lung examination shows rhonchi in the right mid posterior lung field. There is a
soft, holosystolic murmur heard at the left lower sternal border that increases with inspiration.
Chest radiograph is shown below. Which of the following is the best pharmacotherapy for this
patient?
 Incorrect Answer ImageA.Azithromycin, IV
 Incorrect Answer ImageB.Ceftriaxone, intramuscular
 Incorrect Answer ImageC.Daptomycin, IV
 Correct Answer ImageD.Linezolid, IV
 Incorrect Answer ImageE.Trimethoprim/sulfamethoxazole, IV

A 22-year-old Spanish-speaking man comes to the emergency department because of a fever and
abdominal pain. The patient recently emigrated from Mexico and has been working at a
construction site. For the past 48 hours he has been having temperatures to 38.7ºC (101.6ºF) and
this morning developed chills and has been unable to work. On physical examination, his
abdomen is nondistended and soft. There is tenderness over the liver. No organomegaly is noted.
Ultrasound shows an abscess in the right lobe of the liver. Alkaline phosphatase and
aminotransferase levels are elevated. Which of the following is the most appropriate next step in
management? 
 Incorrect Answer ImageA.Begin therapy with levofloxacin
 Correct Answer ImageB.Begin therapy with metronidazole
 Incorrect Answer ImageC.Begin therapy with praziquantel
 Incorrect Answer ImageD.Percutaneous drainage of abscess
 Incorrect Answer ImageE.Surgical drainage and injection of ethyl alcohol

A 48-year-old Vietnamese woman is discharged from the hospital after a 2-week stay for the
treatment of pulmonary tuberculosis. She is started on a 4-drug treatment regimen pending the
sensitivity results of her sputum cultures. Her regimen consists of isoniazid, rifampin,
pyrazinamide, and ethambutol. She initially tolerates the therapy well and takes all of her
medications as prescribed. When she returns to clinic 3 weeks later for culture results, she
reports the presence of red-orange colored urine. She denies dysuria, polyuria, urgency, or
myalgias. On physical examination, she is anicteric. The abdomen is nontender and the liver is
nonpalpable. The remainder of the physical examination is normal. Which of the following drugs
is most likely to cause this side effect? 
 Incorrect Answer ImageA.Ethambutol
 Incorrect Answer ImageB.Isoniazid
 Incorrect Answer ImageC.Pyrazinamide
 Incorrect Answer ImageD.Pyridoxine
 Correct Answer ImageE.Rifampin

A previously healthy 26-year-old man comes to the physician because of a 2-day history of sore
throat, malaise, and fever. He has had no nausea, vomiting, myalgias, shortness of breath, or skin
rash. He denies recent travel. He is sexually active with one person, and he uses condoms
consistently. His temperature is 38.5ºC (101.3ºF), pulse is 100/min, respirations are 18/min, and
blood pressure is 120/80 mm Hg. Physical examination shows tonsillar swelling and exudates,
tender cervical adenopathy, and mild splenomegaly. Laboratory studies show leukocyte count
12,200/mm3 with 11% atypical lymphocytes. Which of the following is the most appropriate next
step in management?
 Correct Answer ImageA.Acetaminophen
 Incorrect Answer ImageB.Acyclovir
 Incorrect Answer ImageC.Amoxicillin
 Incorrect Answer ImageD.No treatment needed
 Incorrect Answer ImageE.Prednisone

A 74-year-old woman who lives in a nursing home is brought to the emergency department
because of progressive drowsiness for the past 6 hours. She also has had a headache for the past
day. She has a history of hypertension, type 2 diabetes mellitus, coronary artery disease, and
hypothyroidism. Examination shows the patient is somnolent, unable to follow commands, and
has neck stiffness. Her temperature is 38.7ºC (101.8ºF), blood pressure is 110/78 mm Hg, and
pulse is 102/min. The lungs are clear to auscultation. Cardiac examination shows normal S1 and
S2; no murmurs are heard. There is pain with extension of the knee while the hip is flexed. CT
scan of the head shows no mass lesions. CSF culture is pending. A lumbar puncture is
performed, and CSF analysis is shown: 
Leukocyte count  2,000/mm3
Segmented neutrophils  90% 
Total protein 250 mg/dL 
Glucose  30 mg/dL 
Gram stain  No organisms 
Which of the following is the most appropriate next step in management? 
 Incorrect Answer ImageA.Acyclovir alone
 Incorrect Answer ImageB.Amphotericin B plus flucytosine
 Incorrect Answer ImageC.Ampicillin alone
 Incorrect Answer ImageD.Ceftriaxone alone
 Incorrect Answer ImageE.Ceftriaxone and ampicillin
 Incorrect Answer ImageF.Ceftriaxone and vancomycin
 Correct Answer ImageG.Ceftriaxone, vancomycin, and ampicillin
 Incorrect Answer ImageH.Supportive care only
 Incorrect Answer ImageI.Vancomycin alone
A 36-year-old man with a history of HIV, hypertension, and diabetes mellitus comes to the
physician because of progressive weakness of 3 months' duration. He has difficulty arising from
a seated position and mild persistent headache. He denies difficulty swallowing, incontinence,
tingling, or numbness. Current medications include zidovudine, lamivudine, efavirenz,
trimethoprim-sulfamethoxazole, insulin, and lisinopril. His vital signs are normal. Muscle
strength is 4/5 in all extremities. Deep tendon reflexes are decreased with normal sensation.
Which of the following is the most likely cause of this patient's weakness?
 Incorrect Answer ImageA.Hemorrhagic stroke
 Incorrect Answer ImageB.Insulin
 Incorrect Answer ImageC.Lisinopril-associated myopathy
 Incorrect Answer ImageD.Spinal stenosis
 Incorrect Answer ImageE.Thrombotic stroke
 Correct Answer ImageF.Zidovudine-associated myopathy

A woman is visiting her neighbor who adopts stray cats and then finds them homes; she has just
taken in a new cat. The visiting woman attempts to pick up the cat, but it bites her on the hand.
She does nothing about the bite, but 3 days later requires attention at the local urgent care center.
At this time the wound appears infected. The clinician suspects infection with Pasteurella
multocida. Cultures and sensitivities of the wound are sent to the lab and results are pending.
Past medical history is positive for a "rash" during treatment for an ear infection 3 months ago,
but the patient can't remember the name of the drug. Which of the following is the best initial
treatment? 
 Incorrect Answer ImageA.Ceftazidime
 Correct Answer ImageB.Clindamycin and cefuroxime
 Incorrect Answer ImageC.Oral clindamycin
 Incorrect Answer ImageD.Oral doxycycline
 Incorrect Answer ImageE.Penicillin G

A 35-year-old man with a 10-year history of HIV infection comes to the emergency department
because of a fever, headache, and confusion for 4 days. He has been irritable for the past 4
weeks. He has had no seizures or loss of consciousness. He has a 2-year history of known
positive anti toxoplasma IgG antibody. He appears lethargic. His temperature is 38.3°C (101°F),
pulse is 92/min, and blood pressure is 115/75 mm Hg. Examination of the skin shows no
abnormalities. The neck is supple. His CD4 T lymphocyte count is 10 cells/mm3, and HIV viral
load is 450,000/mL. CT scan of the brain shows 1 small ring-enhancing mass lesion in the basal
ganglia. Which of the following is the most appropriate next step in management? 
 Incorrect Answer ImageA.Anti-toxoplasma IgM test
 Incorrect Answer ImageB.Lumbar puncture and cerebrospinal fluid studies
 Incorrect Answer ImageC.MRI scan of the brain
 Correct Answer ImageD.Pyrimethamine and sulfadiazine
 Incorrect Answer ImageE.Stereotactic brain biopsy

A 37-year-old woman has the lesion shown for 2 days. She went hiking in upstate New York
with her daughter's seventh-grade class 15 days ago and found a tick on her arm 36 hours after
arriving home. She removed the tick with a tweezer and thought that she was going to be fine.
She is now upset because she has a friend who had a similar experience and is now suffering
from chronic synovitis and memory loss. The patient is anxious and says that she is worried
because she needs to participate in a blood drive at her daughter's school. She then adds, "I have
to be around for my daughter to go on class trips, see her off to college, and be there for her
wedding. This tick can't kill me!" Her temperature is 37.0ºC (98.6ºF). Physical examination
shows no other abnormalities. At this time, which of the following is the most appropriate
statement on the part of the physician? 
 Incorrect Answer ImageA."A vaccine is widely available to individuals who commonly
visit geographic areas of high risk during peak transmission season. Maybe you should look into
this for other members of your family."
 Correct Answer ImageB."I understand that you are very concerned. We'll treat this with
an antibiotic that most patients respond to with a prompt resolution of symptoms within 4 weeks.
This disease is rarely, if ever, fatal."
 Incorrect Answer ImageC."I understand that you are very concerned. We'll treat this with
an antibiotic that most patients respond to with a prompt resolution of symptoms within 4 weeks.
When you're finished here today, you can go donate blood."
 Incorrect Answer ImageD."The tick that caused this infection was most likely the size of
common dog and cattle ticks."
 Incorrect Answer ImageE."This could have been prevented if you had removed the tick
with petroleum jelly and a hot match. Tweezers may leave the mouthparts of the tick in the skin."

A 40-year-old man with HIV is admitted to the hospital for pneumocystis jirovecipneumonia


(PJP). He appears lethargic. His pulse is 125/min, and blood pressure is 80/58 mm Hg. He is
started on IV fluids, IV trimethoprim-sulfamethoxazole, and IV dopamine drip. Laboratory
studies show:
Na+ 133 mEq/L
K +
5.9 mEq/L
Cl− 104 mEq/L
HCO3 −
25 mEq/L
BUN 22 mg/dL
Creatinine 0.9 mg/dL
Glucose 82 mg/dL
CD4 count 30/mm3 
Cortisol (AM) 2 µg/dL
Which of the following is the most likely underlying cause of this patient's electrolyte
abnormalities? 
 Incorrect Answer ImageA.Dopamine drug effect
 Incorrect Answer ImageB.Deficiency of ACTH
 Incorrect Answer ImageC.Intravenous fluids
 Correct Answer ImageD.Mycobacterium avium-intracellulare
 Incorrect Answer ImageE.PJP pneumonia

A 19-year-old college student has new-onset bloody diarrhea and abdominal pain that started
approximately 4 hours before this office visit. He has no significant past medical history and
takes no medications. He recently returned from a camping trip where he was drinking water
from fresh mountain streams. In addition to drinking the water, he often would barbecue shrimp
and hamburgers and eat potato salad that was prepared 2 days before the trip. Last night, after
arriving home, he ate his roommate's cold, leftover fried rice from a Chinese takeout restaurant.
His temperature is 39.0ºC (102.2ºF), blood pressure is 110/70 mmHg, pulse is 93/min, and
respirations are 16/min. His abdomen is soft and nontender with hyperactive bowel sounds. Stool
is positive for occult blood. Which of the following organisms is most likely responsible for this
patient's diarrhea? 
 Incorrect Answer ImageA.Bacillus cereus
 Correct Answer ImageB.Campylobacter jejuni
 Incorrect Answer ImageC.Giardia lamblia
 Incorrect Answer ImageD.Staphylococcus aureus
 Incorrect Answer ImageE.Vibrio parahaemolyticus

A 43-year-old homosexual man comes to the physician because of a small plaque in his mouth.
He denies any symptoms. His past medical history is significant for HIV infection of 3 years'
duration and borderline hypertension. He is not currently on any medication. Family history is
unremarkable. His blood pressure is 140/85 mmHg, pulse 65/min, and respirations 16/min. The
patient is in no acute distress. Oral examination shows a violaceous 1x1.3 cm infiltrated plaque
on the soft palate. Inspection of the skin is remarkable for multiple violaceous 0.5 to 2 cm
macules and patches on the lower back and legs. Palpation of the neck, axillae, inguinal, and
popliteal areas do not show any enlarged lymph nodes. Complete blood count and chest x-ray are
normal. A punch biopsy of one of the skin lesions is performed and histopathologic evaluation
confirms the vascular nature of the process. Which of the following is the most likely causative
agent of this patient's skin disease?
 Incorrect Answer ImageA.Cytomegalovirus
 Incorrect Answer ImageB.Epstein-Barr virus
 Incorrect Answer ImageC.Human herpesvirus type 6
 Correct Answer ImageD.Human herpesvirus type 8
 Incorrect Answer ImageE.Human papillomavirus type 6

A 59-year-old male truck driver comes to the hospital because of cough and conjunctivitis. The
patient describes a cough that has progressively worsened over the past 3 days, becoming more
productive of yellowish-white sputum. He had one episode of shaking chills 2 days ago with a
fever up to 38.8ºC (101.8ºF). He has smoked one pack of cigarettes per day for the past 20 years.
His past medical history is significant for rheumatoid arthritis, for which he takes a nonsteroidal
agent for pain control. On physical examination, his temperature is 37.5ºC (99.5ºF), pulse is
100/min, respirations are 13/min, and blood pressure is 140/90 mm Hg. He has coarse breath
sounds with crackles over his right base (click media file) and a normal cardiac examination.
Examination of the oral cavity shows tonsillar erythema and mild enlargement. There is
conjunctivitis bilaterally, and no adenopathy is appreciated. Examination of the extremities
shows no clubbing, cyanosis, or edema. Which of the following is the most appropriate
diagnostic study for this patient?
 Correct Answer ImageA.Chest radiograph
 Incorrect Answer ImageB.Heterophil antibody test
 Incorrect Answer ImageC.No further testing
 Incorrect Answer ImageD.Rapid strep test
 Incorrect Answer ImageE.Ventilation-perfusion scan

A 45-year-old man comes to the physician because of a new onset of progressive rash that has
been worsening daily for the last week and is accompanied by malaise, headache, and low-grade
fever. He has otherwise been in good health. He reluctantly discloses having unprotected sexual
intercourse with a "friend of a friend" approximately 5 weeks ago during a business trip.
Otherwise, he has had a single steady sexual partner for the past 10 months. Past medical history
is unremarkable and he is not taking any medications. He reports being in mild distress. His
temperature is 37.8°C (100.0°F), blood pressure is 120/85 mm Hg, and pulse is 95/min. There is
a diffuse skin eruption on the trunk, proximal extremities, and palms and soles (see photograph).
The axillary and inguinal lymph nodes are enlarged but nontender and freely movable. The
remainder of the physical examination is normal. Which of the following is the most appropriate
next step in management?
 Incorrect Answer ImageA.Dark-field microscopy of a lesional swab
 Incorrect Answer ImageB.Fluorescent treponemal antibody-absorption
 Correct Answer ImageC.Rapid plasma reagin test
 Incorrect Answer ImageD.Skin biopsy
 Incorrect Answer ImageE.Urethral swab

A 34-year-old man with a 4-year history of HIV comes to the physician because of 2 weeks of
diarrhea that has become bloody over the past week. He has 6 bowel movements daily, with
severe fecal urgency. He has had 2 episodes of fecal incontinence during the past week. He has a
history of a Pneumocystis jiroveciinfection 3 years ago. He is sexually active with one male
partner, and he and his partner use condoms consistently. His temperature is 38.2°C (100.8°F)
and blood pressure is 112/75 mm Hg. The lungs are clear to auscultation. Cardiac examination
shows normal S1 and S2 and a grade 2/6 systolic murmur is heard along the left sternal border.
Abdominal examination shows mild tenderness without masses. CD4+ T lymphocyte count is
32/mm3. Flexible sigmoidoscopy shows inflammation of the sigmoid colon and rectum with deep
ulcers and inclusion bodies. Which of the following is the most likely diagnosis?
 Incorrect Answer ImageA.Crohn's disease
 Correct Answer ImageB.Cytomegalovirus colitis
 Incorrect Answer ImageC.Entamoeba histolytica infection
 Incorrect Answer ImageD.Salmonella enteritidis infection
 Incorrect Answer ImageE.Ulcerative colitis

A concerned family comes to the physician after the grandmother, who had been living with
them, was diagnosed with active TB. She was started on appropriate treatment. The family
consists of a 45-year-old father, a 43-year-old mother, and an 8-year-old daughter. All of the
family members presently are asymptomatic. Which of the following is the most appropriate next
step in management? 
 Incorrect Answer ImageA.Obtain chest radiographs on all family members
 Correct Answer ImageB.Place protein purified derivative (PPD) test on all members and
make further studies if they are 5 mm or more
 Incorrect Answer ImageC.Place protein purified derivative (PPD) test on all members
and make further studies if they are 10 mm or more
 Incorrect Answer ImageD.Place protein purified derivative (PPD) test on all members
and make further studies if they are 15 mm or more
 Incorrect Answer ImageE.Start 4-drug treatment for all family members
 Incorrect Answer ImageF.Start prophylaxis with isoniazid for all family members

A 24-year-old man comes to the physician because of a 1-week history of fever, cough,
hemoptysis, wheezing, and shortness of breath. He developed an itchy rash on his chest and
abdomen 1 day ago. He is a farmer in the rural southeastern United States. His respirations are
30/min. Physical examination shows scattered bilateral wheezes on auscultation, urticarial rash
on the trunk and abdomen, mild abdominal distension, and diffuse abdominal tenderness to
palpation. Radiograph of the chest showed several bilateral patchy infiltrates worse in the right
lung. Laboratory studies show:
Hemoglobin  14.3 g/dL 
Leukocyte count  5,400/mm3
Segmented neutrophils  35%
Eosinophils  30% 
Lymphocytes  30% 
Monocytes 5%
Which of the following is the most appropriate pharmacotherapy? 
 Incorrect Answer ImageA.Ivermectin
 Incorrect Answer ImageB.Levamisole
 Correct Answer ImageC.Mebendazole
 Incorrect Answer ImageD.Pyrantel pamoate
 Incorrect Answer ImageE.Thiabendazole

A 30-year-old man comes to the clinic for evaluation of joint pain. For the last month, he has had
diffuse arthralgias, myalgias, low-grade fever, occasional palpitations, and fatigue. He has been
taking acetaminophen alternating with naproxen, only achieving minimal pain relief. He is
normally physically active outdoors and reported spending upward of 2 weeks at a time hiking
throughout the Northeastern United States and Western Europe. He is sexually active in a
monogamous relationship with his sexual partner over the past year. The man cannot recall any
rash or genital lesions, but he does report that he had ticks that he removed from his legs. Past
medical history is significant for a 3-week episode of a left facial droop. His temperature is
37.0ºC (98.6ºF), pulse is 50/min, respirations are 20/min, and blood pressure is 122/70 mm Hg.
Physical examination, aside from some ectopy on cardiac auscultation, is unremarkable. Which
of the following is the most likely cardiac abnormality?
 Incorrect Answer ImageA.Dilated aortic root on transthoracic echocardiogram
 Incorrect Answer ImageB.Dilated cardiomyopathy on transthoracic echocardiogram
 Correct Answer ImageC.Electrocardiogram showing atrioventricular block
 Incorrect Answer ImageD.Electrocardiogram with diffuse ST-segment elevations
 Incorrect Answer ImageE.Electrocardiogram with low-amplitude QRS complexes

A 39-year-old alcoholic man is brought to the emergency department by his wife immediately
after he had an episode of loss of consciousness and violent shaking that lasted 3 minutes. He
had a sinus infection 1 month ago. He has had a severe headache over the past 3 weeks not
relieved by over-the-counter analgesics. His temperature is 38.6ºC (101.5ºF), pulse is 116/min,
and blood pressure is 102/64 mm Hg. He is disoriented to time, person, and place. Muscle
strength is 3/5 in the left upper extremity and normal on the right side. CT scan of the head with
contrast is shown. A brain biopsy specimen and culture grows Bacteroides fragilis only. Which
of the following is the most appropriate next step in management?
 Incorrect Answer ImageA.Ceftazidime
 Incorrect Answer ImageB.Clindamycin
 Incorrect Answer ImageC.Fluconazole
 Correct Answer ImageD.Metronidazole
 Incorrect Answer ImageE.Penicillin
 Incorrect Answer ImageF.Pyrimethamine and sulfadiazine
 Incorrect Answer ImageG.Surgical excision

A 22-year-old medical student comes to the clinic complaining of severe lethargy, sore throat,
fever of 38.9ºC (102.0ºF) for the last 4 days, and a pink, generalized rash for the last 2 days. He
had been in good health before this episode. After obtaining a history and physical, it is noted
that the patient has not had any sick contacts, has been involved in a monogamous heterosexual
relationship for the last year, and has not traveled over the past year. He denies any illicit drug
use. He admits to self-treating with amoxicillin. He appears pale and exhausted. Physical
examination is significant for diffuse lymphadenopathy and inflamed erythematous tonsils. He
has a palpable and tender liver and spleen. A complete blood count shows:
Hematocrit 42%
Hemoglobin 14 g/dL
WBC 11,500/mm3
Neutrophils 35%
Lymphocytes 60%
Eosinophils 2%
Monocytes 3%
Platelets 140,000/mm3
A peripheral blood smear is significant for >10% of large lymphocytes with abundant cytoplasm.
Which of the following is the most appropriate next step? 
 Correct Answer ImageA.Heterophil antibody test
 Incorrect Answer ImageB.IgM toxoplasmosis
 Incorrect Answer ImageC.Lymph node biopsy
 Incorrect Answer ImageD.p24 antigen testing
 Incorrect Answer ImageE.Rapid streptococcal antigen testing

A 35-year-old man is taken to an emergency room by a friend after the man starts having severe
tonic spasms that cause his arms and legs to contract. The friend thinks that he is having seizures,
but there is no loss of consciousness and the man is able to describe these episodes. The patient's
difficulties began a day and a half earlier when he noticed jaw stiffness. This has since
progressed to difficulty swallowing, sore throat, stiff neck, fever, chills, and tonic spasms of his
arms and legs. The patient's speech is difficult to understand because he has trouble "making his
jaw work right." His mental status is intact. Which of the following is the most likely diagnosis?
 Incorrect Answer ImageA.Botulism
 Incorrect Answer ImageB.Diphtheria
 Incorrect Answer ImageC.Lyme disease
 Incorrect Answer ImageD.Seizure disorder
 Correct Answer ImageE.Tetanus

An 84-year-old woman is brought from home after being found by her daughter to have an
altered mental status. For the last 2 days, the woman reports a productive cough with green-
tinged sputum, fevers, shaking chills, and anorexia. Her past medical history includes diabetes
mellitus type 2, hypertension, and a myocardial infarction. Her temperature is 37.9ºC (100.2ºF),
pulse is 118/min, respirations are 26/min, and blood pressure is 145/95 is mm Hg. Oxygen
saturation is 91% while the patient is breathing room air. Examination of the chest shows
dullness to percussion and coarse inspiratory crackles at the left lung base. The remainder of her
physical examination is normal. Chest radiograph shows opacification of the left lower lobe with
air bronchograms without effusion or interstitial edema. The patient is admitted to the hospital.
Which of the following is the most appropriate single empiric antibiotic for this patient?
 Incorrect Answer ImageA.Azithromycin
 Incorrect Answer ImageB.Ceftriaxone
 Correct Answer ImageC.Moxifloxacin
 Incorrect Answer ImageD.Piperacillin/tazobactam
 Incorrect Answer ImageE.Vancomycin

A 6-year-old girl is brought to the emergency department by her parents immediately after she
was bitten on the hand by the neighbor's domestic dog. The father reports that the girl tried to pet
the dog while it was eating. The dog has been vaccinated regularly. Examination shows three
bite marks on the dorsum of the left hand with broken skin and dried blood. The wound is
cleaned and bandaged. Which of the following is the most appropriate next step for rabies
prophylaxis in this patient? 
 Incorrect Answer ImageA.Rabies immunoglobulin
 Incorrect Answer ImageB.Rabies immunoglobulin and vaccine
 Incorrect Answer ImageC.Rabies vaccine
 Incorrect Answer ImageD.The dog should be killed and the brain examined for signs of
rabies
 Correct Answer ImageE.The dog should be observed for behavioral changes suggestive
of rabies

A 43-year-old man comes to your office for evaluation of a spreading skin rash that appeared 5
days earlier. He noticed an initial red "bump" after returning from a camping trip to Vermont and
thought it was an insect bite. He also reports a headache and joint pain. His past medical and
family history is unremarkable, and he does not take medications. His temperature is 37.8°C
(100.0°F), pulse is 90/min, respirations are 18/min, and blood pressure is 110/70 mm Hg.
Physical examination of the skin shows an erythematous annular plaque 15 cm in diameter on his
right flank. It is centered by a resolving petechial macule. There is diffuse joint tenderness but no
impaired range of motion or localized inflammation of the small or large joints. An
ophthalmologic examination is normal. Which of the following vectors is most likely responsible
for this patient's disease?
 Incorrect Answer ImageA.Amblyomma americanum
 Incorrect Answer ImageB.Dermacentor andersoni
 Correct Answer ImageC.Ixodes scapularis
 Incorrect Answer ImageD.Loxosceles reclusa
 Incorrect Answer ImageE.Pediculus humanus corporis

A 24-year-old graduate student comes to the clinic complaining of pain on urination. The
discomfort began approximately 2 weeks ago and has not improved. He states that there is
purulent drainage from the tip of the penis, frequent urination, and the constant sensation to
urinate. There is no change in the caliber of his stream and no urinary incontinence. Besides an
episode of hypotension and tachycardia after a penicillin injection during childhood, he has been
healthy and denies any similar episode in the past. He recently returned from a spring-break trip
with his college friends. On physical examination, he is anxious but in no distress. His
temperature is 37.0ºC (98.6ºF), pulse is 90/min, and blood pressure is 126/81 mm Hg. Heart,
lung, and abdominal examinations are normal. His penis is circumcised, and there is a purulent
discharge at the meatus. The testes are descended, normal in contour, and without tenderness.
Prostate is smooth, firm, and without fluctuance or tenderness. Gram stain shows intracellular
gram-negative diplococci. A DNA probe for chlamydia is negative. Which of the following is the
most appropriate next step in the management of this patient?
 Incorrect Answer ImageA.Culture and sensitivity of the discharge to establish appropriate
therapy
 Correct Answer ImageB.Treat with azithromycin and gemifloxacin
 Incorrect Answer ImageC.Treat with azithromycin
 Incorrect Answer ImageD.Treat with ceftriaxone
 Incorrect Answer ImageE.Treat with ciprofloxacin
A 35-year-old man comes to the emergency department because of a severe headache, vomiting,
watery diarrhea, sore throat, muscle aches, rash, and fever to 39.9°C (103.8°F) for 2 days. Four
days ago, he sustained a deep laceration on his left arm after tripping and sliding down a hill. He
was able to stop the bleeding and cleaned the wound subsequently. He had a tetanus vaccination
four years ago. His temperature is 39.4°C (102.9°F), pulse is 110/min, and blood pressure is
86/60 mm Hg. Physical examination shows a diffuse macular erythematous rash and bilateral
conjunctival inflammation. Which of the following is the most likely causal organism?
 Incorrect Answer ImageA.Borrelia burgdorferi 
 Incorrect Answer ImageB.Clostridium perfringens 
 Incorrect Answer ImageC.Clostridium tetani 
 Incorrect Answer ImageD.Pseudomonas aeruginosa 
 Correct Answer ImageE.Staphylococcus aureus 

A 50-year-old man comes to the physician because of a 3-month history of joint pain. The pain
fluctuates and typically occurs in one joint, resolves, and then develops in another joint. The pain
neither worsens nor is relieved with activity. He denies morning stiffness. He has a history of a
rash on his left shoulder that began shortly after one of his hunting trips and resolved 6 months
ago. The patient works as a police officer and hunts twice monthly. He drinks 1 to 2 beers daily.
His temperature is 37.8°C (100ºF), and pulse is 75/min. Which of the following organisms is the
most likely cause of this patient's symptoms?
 Correct Answer ImageA.Borrelia burgdorferi 
 Incorrect Answer ImageB.Borrelia recurrentis 
 Incorrect Answer ImageC.Staphylococcus aureus 
 Incorrect Answer ImageD.Treponema carateum 
 Incorrect Answer ImageE.Treponema pallidum 

A 50-year-old man who has a history of alcohol abuse for the last 20 years is brought to the
hospital because of fever, dizziness, and a productive cough with brownish sputum. He is not
taking any medications and has a sulfa allergy. His temperature is 37.5ºC (99.5ºF), pulse is
91/min, respirations are 14/min, and blood pressure is 100/70 mm Hg. Physical examination
shows crackles in the left upper lobe, and cardiac examination shows a regular rate and rhythm.
Chest radiograph shows an infiltrate in the left upper lobe. Which of the following is appropriate
pharmacologic management?
 Incorrect Answer ImageA.Azithromycin
 Correct Answer ImageB.Cefuroxime and azithromycin
 Incorrect Answer ImageC.Clindamycin
 Incorrect Answer ImageD.Erythromycin and levofloxacin
 Incorrect Answer ImageE.Trimethoprim-sulfamethoxazole

A 33-year-old man comes to the physician because of a 1-year history of a rash over his arms,
face, and legs. He has had no pain or pruritis. He denies fever, myalgias, chills, or recent tick
bites. His medical history is unremarkable, and he takes no medications. He emigrated from
Mexico 2 years ago. He does not smoke or drink alcohol. His temperature is 36.8°C (98.2°F),
pulse is 70/min, and blood pressure is 118/76 mm Hg. Physical examination shows scattered
plaques that are hypopigmented, and surrounded by an erythematous border. Palpable nodules
and cord-like structures are noticed along his distal extremities. The remainder of the
examination shows no abnormalities. A skin biopsy specimen of a lesion shows rare acid-fast
bacilli. Laboratory studies show:
White blood cells  5,700/mm3
Hematocrit  44%
Platelets  235,000/mm3
Which of the following findings is most likely associated with this patient's condition? 
 Correct Answer ImageA.Cell-mediated immune response
 Incorrect Answer ImageB.Cure with monotherapy
 Incorrect Answer ImageC.Negative lepromatous skin test
 Incorrect Answer ImageD.Polyclonal gammopathy on SPEP
 Incorrect Answer ImageE.Positive serologic assay

A 41-year-old gastroenterologist attended an annual national conference in San Francisco. On his


return home, he develops a fever for 5 days with temperatures as high as 39.4ºC (102.9ºF) and a
severe, nonproductive cough associated with shortness of breath at rest. At the hospital, he
describes headache and fatigue, nausea, and frequent loose stools. His wife reports that he has
been confused. On physical examination, his temperature is 39.1ºC (102.4ºF), pulse is 76/min,
respirations are 28/min, and blood pressure is 116/84 mm Hg. On lung examination, there are
scattered loud rhonchi in the right lung fields. Abdominal examination is normal. Which of the
following is the most likely diagnosis?
 Incorrect Answer ImageA.Cytomegalovirus pneumonia
 Correct Answer ImageB.Legionnaires' disease
 Incorrect Answer ImageC.Mycobacterium tuberculosis 
 Incorrect Answer ImageD.Pneumocystis jirovecii pneumonia (PJP)
 Incorrect Answer ImageE.Streptococcal pneumonia

A 21-year-old woman returns to the clinic for further evaluation of bilateral facial weakness and
radicular arm pain. For the past few weeks she has suffered progressive facial palsy on both sides
of her face and hyperacusis and dry eyes. On her previous visit she described a 2-week history of
low-grade fevers, headaches, mild neck stiffness, and photophobia. Review of systems revealed a
recent rash that had developed after a camping trip, and then, after expanding outward from the
site of a small tick-bite, resolved spontaneously. A lumbar puncture performed at that time
revealed an increased amount of immunoglobulin with two oligoclonal bands on analysis.
Cerebrospinal fluid (CSF) PCR analysis for Borrelia burgdorferi and peripheral blood antibody
analysis are positive. Which of the following is the most appropriate treatment for this patient's
current condition? 
 A.IV acyclovir and corticosteroids
 B.IV interferon and corticosteroids
 Correct Answer C.IV penicillin G
 D.Oral amoxicillin
 E.Oral doxycycline

A 24-year-old man comes to the physician for a follow-up visit after completing a 6-month
course of treatment for active tuberculosis that he acquired while traveling. He currently feels
well and denies any specific complaints. His past medical history includes a new and difficult-to-
treat case of seborrheic dermatitis and a recent episode of herpes zoster. Vital signs are within
normal limits. Physical examination is unremarkable, with the exception of skin findings
consistent with seborrheic dermatitis. The patient is concerned about his likelihood of having
another episode of tuberculosis. Which of the following is the most appropriate next step in
management?
 A.Allergy skin testing
 B.Chest radiograph
 Correct Answer C.HIV testing
 D.PPD placement
 E.Sputum culture and smear

A 59-year-old man is seen by a specialist because of paranasal swelling, halitosis, and nose
bleeding with clots and pus. His temperature is 37.0ºC (98.6ºF), pulse is 73/min, respirations are
20/min, and blood pressure is 128/79 mm Hg. The decision is made to proceed with surgical
debridement of the sinuses. Preoperative physical examination shows a glucose level of 300
mg/dL and hemoglobin A1c is 11%. His medical condition rapidly deteriorates, and the surgical
procedure is performed urgently. The sinus contents are sent for pathologic examination, and
fungi with broad, nonseptate, irregularly shaped hyphae are identified. Systemic antifungal
therapy is administered, but the patient further deteriorates and is now stuporous. He dies 3 days
later despite appropriate treatment. Which of the following is the most likely organism
responsible for this patient's condition?
 A.Aspergillus
 B.Blastomyces
 C.Candida
 Correct Answer D.Rhizopus
 E.Sporothrix

A 47-year-old man comes to the emergency department because of fever and a cough that started
3 days ago. He had little relief from over-the-counter cold medications and as his cough
worsened, he observed the production of greenish sputum. He also complains of right-sided chest
pain. He does not smoke or drink and has been in good health. His temperature is 38ºC
(101.1ºF), blood pressure is 110/70 mm Hg, pulse is 90/min, and respirations are 20/min. To
view the examination, click on the "Play Media" button. Which of the following is the most
likely diagnosis? 
 A.Costochondritis
 B.Mycoplasma pneumonia
 C.Pseudomonas pneumonia
 D.Respiratory syncytial virus (RSV)
 Correct Answer E.Streptococcal pneumonia

A 48-year-old man with an 8-year history of HIV infection comes to the physician for a follow-
up examination. He recently completed a course of pentamidine for an infection
with Pneumocystis jirovecii. He has had no recent fever, cough, or shortness of breath. Current
medications include zidovudine, lamivudine, and nelfinavir. He admits that he occasionally skips
taking his medication. He is allergic to trimethoprim-sulfamethoxazole (TMP-SMX). His CD4+
T lymphocyte count is 49 cells/mm3, and viral load is 100,000/mL. Radiograph of the chest
shows resolution of the pneumonia. In addition to his current highly active antiretroviral therapy
(HAART) regimen, which of the following is the most appropriate prophylaxis regimen?
 Incorrect Answer ImageA.Aerosolized pentamidine and azithromycin
 Correct Answer ImageB.Atovaquone and azithromycin
 Incorrect Answer ImageC.Atovaquone and rifabutin
 Incorrect Answer ImageD.Clarithromycin and ethambutol
 Incorrect Answer ImageE.Dapsone and aerosolized pentamidine
 Incorrect Answer ImageF.Dapsone and azithromycin
A 30-year-old man comes to the physician because of a 2-week history of low-grade fever,
painful swelling under the left angle of the jaw, and painful swelling of his left testicle for 5
days. He had infectious mononucleosis 15 years ago. He traveled to South America last year. He
does not smoke cigarettes and drinks 4–5 beers on the weekends. His maternal uncle has type 1
diabetes mellitus. His temperature is 38.1°C (100.6°F), and pulse is 75/min. Which of the
following is the most likely cause of this patient's symptoms?
 Incorrect Answer ImageA.Actinomyces israelii 
 Incorrect Answer ImageB.Adenovirus
 Incorrect Answer ImageC.Chlamydia trachomatis 
 Incorrect Answer ImageD.Coxsackievirus B
 Correct Answer ImageE.Mumps virus

A 38-year-old man with a 2-year history of hypertension comes to the emergency department
because of a seizure. On arrival, the patient appears drowsy. Over the next 2 hours, his mentation
improves to normal. He has never had seizures in the past. He has had no fever, chills, vomiting,
weakness, or incontinence. He emigrated from Thailand 5 years ago. His temperature is 36.8ºC
(98.2ºF), pulse is 76/min, respirations are 16/min, and blood pressure is 120/70 mm Hg. Physical
examination is normal. Laboratory studies show:
Hgb  13.6 g/dL
WBC 8,000/mm 3 
Lymphocytes  40%
Eosinophils  13%
Platelets  270,000/mm3
Serum studies show:
Na+ 134 mEq/L 
Cl– 102 mEq/L 
K +
4.6 mEq/L 
HCO3– 27 mEq/L 
Blood urea nitrogen (BUN)  16 mg/dL 
Creatinine  0.9 mg/dL 
Glucose  80 mg/dL 
Ca+ 9.2 mg/dL 
A CT scan of the head with contrast shows multiple ring-enhancing lesions at the gray-white
junction. Lumbar puncture and cerebrospinal fluid (CSF) examination show normal protein and
glucose levels and 0 to 1 cells/mm3. The CSF cryptococcal antigen and toxoplasma PCR are
negative. Which of the following is the most likely diagnosis? 
 Incorrect Answer ImageA.Bacterial meningitis
 Incorrect Answer ImageB.Hemorrhagic stroke
 Correct Answer ImageC.Neurocysticercosis
 Incorrect Answer ImageD.Thrombotic stroke
 Incorrect Answer ImageE.Toxoplasmosis

A 30-year-old man is brought to the emergency department by his wife with a 6-day history of
fatigue, cough, headache, backache, joint pain, and chest pain. One day ago, he had a sudden
onset of fever and chills, a worsening headache, and a rash that began on his back and chest and
spread to his limbs. He returned from a missionary trip to South America 2 weeks ago. He has
had no exposure to farm animals. His temperature is 38.8°C (101.8°F) and pulse is 125/min.
Examination shows a confused man with an erythematous maculopapular rash over the trunk and
extremities that spares the face, palms, and soles. There is diffuse muscle tenderness to palpation.
Which of the following is the most likely diagnosis? 
 Incorrect Answer ImageA.Endemic typhus
 Correct Answer ImageB.Epidemic typhus
 Incorrect Answer ImageC.Q fever
 Incorrect Answer ImageD.Rickettsial pox
 Incorrect Answer ImageE.Rocky Mountain spotted fever

A 34-year-old man comes to his physician for increasing pedal edema for three months. He has
no history of medical illness but is a former intravenous drug abuser, quitting one year ago. He
denies arthralgia, rash, or hematuria. His temperature is 37°C (98.6°F), and his blood pressure is
130/90 mm Hg. On physical examination, his lungs are clear to auscultation. Cardiac
examination shows an S4 with a laterally displaced PMI. Abdominal examination is significant
for an enlarged, palpable liver. There is pitting edema to mid-calf, and multiple old skin ulcers
are noted on both lower extremities. Laboratory analysis shows:
Potassium 3.7 mEq/L
Creatinine 1.6 mg/dL
Urea nitrogen  20 mg/dL
Albumin 2.1 mg/dL
Urinalysis shows 4+ protein, no heme, and no cells or casts. A spot urine albumin to creatinine
ratio is 8.5. A renal ultrasound shows large kidneys, each measuring approximately 13 cm. An
echocardiogram shows diffuse cardiac enlargement with pericardial thickening. An HIV test is
negative. Which of the following is the most likely diagnosis? 
 Incorrect Answer ImageA.AIDS nephropathy
 Correct Answer ImageB.Amyloidosis
 Incorrect Answer ImageC.Focal segmental glomerulosclerosis
 Incorrect Answer ImageD.Hypertensive nephropathy
 Incorrect Answer ImageE.Postinfectious glomerulonephritis

A 20-year-old man comes to the emergency department because of a 6-day history of fever,
chills, and muscle aches. Two weeks ago, he returned from a hiking trip to the White Mountains
of New Hampshire. His temperature is 38.3°C (100.9°F). There are two erythematous, round
patches on his left thigh, each with central clearing. Which of the following is the most
appropriate next step in management? 
 Incorrect Answer ImageA.Acyclovir
 Correct Answer ImageB.Doxycycline
 Incorrect Answer ImageC.ELISA for antibodies against the organism
 Incorrect Answer ImageD.Immunofluorescence assay of antibodies against the organism
 Incorrect Answer ImageE.Prednisone

A 45-year-old man is brought to the emergency department because of a 2-day history of fever,
headache, and confusion. He has a history of prostate cancer, type 2 diabetes mellitus, and
hypertension. Current medications include lisinopril and metformin. He appears confused. His
temperature is 39.7ºC (103.5ºF), pulse is 102/min, and blood pressure is 120/84 mm Hg. Physical
examination shows neck stiffness. A lumbar puncture is performed. Gram stain of the
cerebrospinal fluid shows lancet-shaped, gram-positive diplococci. Which of the following is the
most appropriate pharmacotherapy? 
 Incorrect Answer ImageA.Ampicillin, ceftriaxone, and vancomycin
 Incorrect Answer ImageB.Ceftriaxone
 Correct Answer ImageC.Ceftriaxone and vancomycin
 Incorrect Answer ImageD.Penicillin
 Incorrect Answer ImageE.Rifampin
 Incorrect Answer ImageF.Vancomycin plus ampicillin

A 25-year-old man comes to the emergency department because of a headache and mild
photophobia. The symptoms have been ongoing for the past 3 hours. He denies nausea, vomiting,
or prior similar symptoms. He denies any head trauma. His past medical history is notable for
hydrocephalus for which he has an indwelling ventriculoperitoneal shunt. This was manipulated
a few days ago during a neurosurgical clinic visit. The patient's shunt series films reveal no
kinking in the shunt tubing. Head CT scan reveals no increased hydrocephalus. Cerebrospinal
fluid obtained by way of a cisternal tap reveals several white cells consistent with an infection.
The patient is started on antibiotics. Which of the following pathogens is most likely responsible
for this patient's presentation?
 Incorrect Answer ImageA.Escherichia coli
 Incorrect Answer ImageB.Haemophilus influenzae
 Incorrect Answer ImageC.Neisseria meningitidis
 Correct Answer ImageD.Staphylococcus epidermidis
 Incorrect Answer ImageE.Streptococcus pneumoniae

An 81-year-old woman with diabetes consults a physician because of a severe and persistent
earache. Otoscopic examination demonstrates foul-smelling purulent otorrhea and a red mass of
the external ear canal. Biopsy of the mass demonstrates granulation tissue rather than tumor.
Which of the following is the most likely causative organism? 
 Incorrect Answer ImageA.Escherichia coli
 Incorrect Answer ImageB.Haemophilus influenzae
 Incorrect Answer ImageC.Proteus vulgaris
 Correct Answer ImageD.Pseudomonas aeruginosa
 Incorrect Answer ImageE.Staphylococcus aureus

A 69-year-old woman comes to the physician because of right-sided back pain for 6 months. She
has felt burning and tingling over her upper right back. Six months ago, she had a low-grade
fever, muscle aches, and a painful rash on the right side of her back that lasted for 10 days and
then resolved. She has a history of hypertension and hypercholesterolemia. Current medications
include hydrochlorothiazide, lisinopril, and pravastatin. Her temperature is 37.1°C (98.8°F),
pulse is 82/min, and blood pressure is 136/88 mm Hg. Examination shows hyperalgesia in the T4
dermatome on the right side. Which of the following is the most appropriate next step in the
management of this patient?
 Incorrect Answer ImageA.Oral acyclovir
 Correct Answer ImageB.Oral desipramine
 Incorrect Answer ImageC.Oral prednisone with taper
 Incorrect Answer ImageD.Physical therapy
 Incorrect Answer ImageE.Rest, heat, and supportive care

A 40-year-old man comes to his physician with dysphagia and odynophagia for the past 5 days.
He also reports bloody diarrhea, intermittent fever, and floaters within his visual field without a
change in vision. On further questioning, he reports unintentional weight loss of 9 kg (20 lb) in
the past 3 months. Past medical history includes HIV infection diagnosed 5 years ago. He refuses
treatment with HIV medications. His temperature is 38.3ºC (100.9ºF), pulse is 98/min,
respirations are 16/min, and blood pressure 108/70 mm Hg. He is cachectic. There is no oral
thrush or ulcers. Funduscopic examination shows yellowish-white granules with perivascular
exudates and hemorrhages. CD4+ T-cell count is 25 cells/mm3. Endoscopy shows one large,
shallow, 5 cm ulcer in the mid-esophagus surrounded by normal-appearing mucosa. Which of
the following is the most appropriate pharmacologic therapy? 
 Incorrect Answer ImageA.Acyclovir
 Incorrect Answer ImageB.Amphotericin B
 Incorrect Answer ImageC.Fluconazole
 Correct Answer ImageD.Ganciclovir
 Incorrect Answer ImageE.Pyrimethamine and sulfadiazine

A 30-year-old previously healthy man is brought to the emergency department because of fever,
severe headache, and seizures of abrupt onset. His temperature is 39.4ºC (103ºF), pulse is
100/min, respirations are 17/min, and blood pressure is 120/80 mm Hg. The patient is not
oriented to place and time. Physical examination shows mild nuchal rigidity. On admission,
laboratory studies and lumbar puncture findings are:
Hematocrit 43%
Hemoglobin 14 g/dL
WBC 10,200/mm3
Neutrophils 6,000/mm3
Lymphocytes 4,200/mm3
Platelets 200,000/mm3
CSF analysis:
Opening pressure 100 mm H2O 
Appearance Clear
Cell count 500/mm3
Dominant cell type Lymphocytes, 100 red blood cells/field
Glucose 60 mg/dL
Protein 60 mg/dL
Electroencephalographic studies demonstrate bitemporal periodic complexes on a slow
background. Neuroimaging shows multifocal hemorrhagic lesions in the temporal lobes. Which
of the following is the most appropriate next step in management? 
 Incorrect Answer ImageA.Polymerase chain reaction for suspected organism on CSF
 Incorrect Answer ImageB.Raise the head of the patient's bed, hyperventilate him, and
arrange urgent neurosurgical consult
 Incorrect Answer ImageC.Repeat a spinal puncture in 3 tubes to determine if the RBC are
indeed from the CSF
 Incorrect Answer ImageD.Supportive treatment, because this is a viral meningitis
 Incorrect Answer ImageE.Treatment with ceftriaxone plus vancomycin
 Correct Answer ImageF.Treatment with IV acyclovir

A 72-year-old man comes to the physician because of severe back pain for the past 3 days. He
was seen yesterday in the emergency department and was sent home with acetaminophen and
advised to avoid any strenuous activity. Despite following this advice, he continues to have back
pain and wishes to get a second opinion. The pain is located in the right lower lumbar region and
is described as constant, sharp, and stabbing. He reports some recent low-grade fevers that have
resolved with acetaminophen and diffuse myalgias in the days preceding his back pain. The
patient is concerned because he has a history of severe osteoporosis, though he has never had any
lumbar pain before. Past medical history includes syphilis and gonorrhea during his twenties for
which he received treatment. His temperature is 37ºC (98.6ºF), pulse is 98/min, respirations are
20/min, and blood pressure is 138/88 mm Hg. Examination of the patient's back shows no focal
tenderness although the patient is in obvious pain. Skin examination shows one small group of
vesicles in the right paraspinal lumbar region. The patient is surprised by the lesions and reports
that they were not present the previous day. Which of the following is the most appropriate next
step in management?
 Incorrect Answer ImageA.Admission for IV antibiotics
 Incorrect Answer ImageB.High-dose ibuprofen around the clock
 Incorrect Answer ImageC.Oral acyclovir and oral prednisone
 Correct Answer ImageD.Oral famciclovir
 Incorrect Answer ImageE.Rest, ice, supportive therapy, and reassurance

A 37-year-old woman with HIV comes to the medical walk-in clinic with cough, fever, and
shortness of breath. She has been well and has not been compliant with her triple-drug therapy.
She reports that 2 weeks ago, she began to experience increasing shortness of breath and
developed a nonproductive cough. Over the past few weeks, symptoms have gradually worsened.
Her last CD4 count was 190/mm3 and last viral RNA copy number was 120,000 copies/mL. She
also reports fevers to 39.4ºC (103.0ºF) over the past few days. On physical examination, her
pulse is 90/min, respirations are 24/min, blood pressure is 130/70 mm Hg, and oxygen saturation
is 80% on room air. Her lungs have diffuse crackles with no egophony or dullness to percussion.
The remainder of her examination is normal. Chest radiograph shows diffuse bilateral interstitial
and alveolar infiltrates bilaterally. Arterial blood gas analysis shows pH 7.32, PaCO2 52 mm Hg,
and PaO269 mm Hg. Which of the following is the most appropriate course of therapy?
 Incorrect Answer ImageA.IV amphotericin B
 Incorrect Answer ImageB.IV azithromycin
 Incorrect Answer ImageC.IV trimethoprim-sulfamethoxazole (TMP-SMX)
 Correct Answer ImageD.IV TMP-SMX and prednisone
 Incorrect Answer ImageE.Oral isoniazid, rifampin, pyrazinamide, and ethambutol

A 70-year-old man comes to the emergency department because of a 3-day history of fever,
chills, malaise, and fatigue. He has a history of hypertension, type 2 diabetes mellitus, and
ischemic heart disease. Two weeks ago, he was transferred to a nursing home after undergoing 3
months of inpatient rehabilitation following a cerebrovascular accident. Current medications
include lisinopril, hydrochlorothiazide, metformin, metoprolol, and simvastatin. His temperature
is 39.2°C (102.6°F), pulse is 85/min, and blood pressure is 115/70 mm Hg. Lungs are clear to
auscultation. Cardiac examination shows a normal S1 and S2 without murmurs. There is no
costovertebral tenderness. Abdominal examination shows no tenderness with normal bowel
sounds. Blood cultures grow methicillin-resistant Staphylococcus aureus. Which of the following
is the most appropriate pharmacotherapy?
 Incorrect Answer ImageA.Ceftriaxone
 Incorrect Answer ImageB.Linezolid
 Incorrect Answer ImageC.Oxacillin
 Incorrect Answer ImageD.Penicillin
 Correct Answer ImageE.Vancomycin

A 26-year-old man arrives in the emergency department because of an acutely painful and
swollen left calf. He is an IV drug user who admits to the use of black-tar heroin on a daily basis
for the past 2 weeks. He has resorted to "skin-popping" because he is no longer able to find veins
to inject. He has been hospitalized several times over the past few years for infective
endocarditis. He is HIV-negative as of 3 months ago. On physical examination, his temperature
is 38.7ºC (101.7ºF), pulse is 120/min, and blood pressure is 106/65 mm Hg. He appears
undernourished and disheveled with diaphoresis and pallor. He has poor dentition without oral
ulcers or abscesses. There is no cervical lymphadenopathy. Lungs are clear to auscultation and
cardiovascular examination shows no murmurs. Abdominal examination shows no
hepatosplenomegaly. Skin examination shows no lesions on his palms or soles. An indurated, 3
cm fluctuant lesion is noted on the left calf. It is acutely painful to the touch, and there is
surrounding crepitus. Laboratory data show WBC 18,000/mm3, hematocrit 38%, and platelets
568,000/mm3. Which of the following is the most likely etiologic agent?
 Correct Answer ImageA.Clostridium spp.
 Incorrect Answer ImageB.Neisseria spp.
 Incorrect Answer ImageC.Pseudomonas spp.
 Incorrect Answer ImageD.Staphylococcus spp.
 Incorrect Answer ImageE.Streptococcus spp.

A 72-year-old woman is recovering from a prolonged hospital stay for complicated community-
acquired pneumonia. While she was in the intensive care unit, a central venous catheter was
inserted into her internal jugular vein for blood sampling and hemodynamic monitoring. She
recovers well, and 1 week later is being considered for transfer to the general medicine ward.
However, after being afebrile for 5 days, she develops a fever. No obvious source of infection is
found, and the line insertion site appears normal without any erythema or induration. Blood
cultures are drawn, both through the line and from her arm. Although final results are not
available, Gram stains from three of the four bottles show gram-positive cocci in clusters. The
line is removed and the tip is sent for culture. Which of the following is the most appropriate
pharmacotherapy?
 Incorrect Answer ImageA.Cefepime, intravenous administration
 Incorrect Answer ImageB.Clindamycin, oral or intravenous administration
 Incorrect Answer ImageC.Gentamicin, intravenous administration
 Incorrect Answer ImageD.Nafcillin, intravenous administration
 Correct Answer ImageE.Vancomycin, intravenous administration

A 24-year-old woman comes to the emergency department after being bitten by a bat earlier in
the morning. She is an avid outdoors enthusiast. While taking a spelunking course, she disturbed
a group of resting bats. One of the animals bit her on the left leg, and she did not notice the
wound at the time. Her medical history is unremarkable. She reports feeling well, yet a bit tired.
The woman received all childhood vaccinations, and had a booster tetanus vaccination 18
months ago. Vital signs are within normal limits. Physical examination shows a healthy woman.
There is a series of mildly tender, small bite wounds on her left leg near the knee. The knee joint
is nontender, has full range of motion, and is without edema. Which of the following is the most
appropriate management? 
 Correct Answer ImageA.Both rabies vaccine and immune globulin
 Incorrect Answer ImageB.Close monitoring only for now; defer treatment
 Incorrect Answer ImageC.Observation of bats for massive death for the next 10 days, and
advise that the patient return immediately if that happens
 Incorrect Answer ImageD.Rabies immune globulin
 Incorrect Answer ImageE.Rabies vaccination
 Incorrect Answer ImageF.Reassurance and advice that the patient call if the wound
worsens
 Incorrect Answer ImageG.Tetanus toxoid

A previously healthy 21-year-old college senior comes to the emergency department because of
a 1-day history of severe headache, malaise, and vomiting. He states that bright lights make his
headache worse. He is alert and oriented. His temperature is 38.4ºC (101.1ºF), blood pressure is
110/76 mm Hg, and pulse 116/min. Examination shows a petechial rash of discrete lesions
approximately 1 to 2 mm in diameter on the patient’s trunk and lower body. Flexion of the
patient's neck causes flexion of the hips and knees. Fundoscopic examination shows no
abnormalities. Laboratory studies show: 
Hemoglobin  13.4 g/dL
Leukocyte count 13,400/mm3
Sodium  138 mEq/L
Segmented neutrophils  85%
Eosinophils 1%
Lymphocytes  10%
Monocytes 4% 4%
Platelet count 136,000/mm3
Which of the following is the most likely causative organism? 
 Incorrect Answer ImageA.Haemophilus influenzae
 Incorrect Answer ImageB.Listeria monocytogenes
 Correct Answer ImageC.Neisseria meningitidis
 Incorrect Answer ImageD.Staphylococcus aureus
 Incorrect Answer ImageE.Streptococcus pneumoniae

A 28-year-old medical student comes to the emergency department 30 minutes after a needle-
stick injury. He was stuck with a needle from a patient who has advanced hepatitis C and HIV.
His medical history is unremarkable and he takes no medications. Which of the following is the
most appropriate next step in management? 
 Correct Answer ImageA.HIV test and prophylaxis with emtricitabine, tenofovir, and
raltegravir
 Incorrect Answer ImageB.Observation without testing
 Incorrect Answer ImageC.HIV test only
 Incorrect Answer ImageD.HIV test and prophylaxis with zidovudine
 Incorrect Answer ImageE.Prophylaxis with lamivudine, zidovudine, and
lopinavir/ritonavir
A 19-year-old woman comes to the physician because of a 2-week history of frequent episodes
of loose stools. She has had severe fecal urgency and awoke with diarrhea several nights the past
month. Over the past week, her stools have become increasingly bloody. Her medical history is
unremarkable. Her temperature is 38.5°C (101.3°F), blood pressure is 115/70 mm Hg, and pulse
is 80/min. Sigmoidoscopy shows continuous, symmetric inflammation from the anal verge to the
proximal sigmoid colon. Which of the following is the most likely cause of this patient's
symptoms?
 Correct Answer ImageA.Campylobacter jejuni 
 Incorrect Answer ImageB.Cryptosporidium 
 Incorrect Answer ImageC.Cytomegalovirus
 Incorrect Answer ImageD.Giardia lamblia 
 Incorrect Answer ImageE.Yersinia enterocolitica 

A 35-year-old wood logger with no significant past medical history comes to the physician
because of a 4-day history of fevers, chills, severe headache, and diffuse muscle aches and pains.
Within 48 hours of these symptoms beginning, he developed a rash on his wrists and ankles.
Over the next 48 hours, the rash spread to involve the arms, legs, and trunk. He denies
photophobia or nuchal rigidity. He is sexually active with his wife and reports having
unprotected sexual intercourse with a prostitute one month ago. He owns a farm and raises cattle.
He has a dog and a cat that live inside the house. Physical examination shows a weak, pale, and
fatigued-appearing man. His temperature is 39.0ºC (102.3ºF), pulse is 113/min, respirations are
15/min, and blood pressure is 110/60 mm Hg. He has a diffuse maculopapular rash on his trunk,
extremities, palms, and soles. The remainder of his physical examination is normal. Laboratory
studies show a normal white count and liver function tests. Which of the following is the most
likely pathogen?
 Incorrect Answer ImageA.Bartonella henselae 
 Incorrect Answer ImageB.Borrelia burgdorferi 
 Incorrect Answer ImageC.Coxiella burnetii 
 Incorrect Answer ImageD.Ehrlichia chaffeensis 
 Correct Answer ImageE.Rickettsia rickettsii 

A 35-year-old wood logger with no significant past medical history comes to the physician
because of a 4-day history of fevers, chills, severe headache, and diffuse muscle aches and pains.
Within 48 hours of these symptoms beginning, he developed a rash on his wrists and ankles.
Over the next 48 hours, the rash spread to involve the arms, legs, and trunk. He denies
photophobia or nuchal rigidity. He is sexually active with his wife and reports having
unprotected sexual intercourse with a prostitute one month ago. He owns a farm and raises cattle.
He has a dog and a cat that live inside the house. Physical examination shows a weak, pale, and
fatigued-appearing man. His temperature is 39.0ºC (102.3ºF), pulse is 113/min, respirations are
15/min, and blood pressure is 110/60 mm Hg. He has a diffuse maculopapular rash on his trunk,
extremities, palms, and soles. The remainder of his physical examination is normal. Laboratory
studies show a normal white count and liver function tests. Which of the following is the most
likely pathogen?
 Incorrect Answer ImageA.Bartonella henselae 
 Incorrect Answer ImageB.Borrelia burgdorferi 
 Incorrect Answer ImageC.Coxiella burnetii 
 Incorrect Answer ImageD.Ehrlichia chaffeensis 
 Correct Answer ImageE.Rickettsia rickettsii 

A 36-year-old woman comes to the physician for evaluation of a purified protein derivative
(PPD) skin test placed 48 hours ago. She has had no cough, fevers, night sweats, chills, or weight
loss. She has no contact with individuals who have tuberculosis. One year ago, her PPD was
negative. She is a physician. Examination shows a 10-mm induration at the site of the skin test.
Radiograph of the chest is shown. Which of the following is the most appropriate next step in
management?
 Correct Answer ImageA.Daily isoniazid and pyridoxine for 9 months
 Incorrect Answer ImageB.Daily isoniazid, rifampin, ethambutol, and pyrazinamide for 2
months, followed by isoniazid and rifampin for 4 more months
 Incorrect Answer ImageC.Daily rifampin for 4 months
 Incorrect Answer ImageD.Repeat PPD in 1 year, no treatment necessary at the moment
 Incorrect Answer ImageE.Sputum induction for culture and sensitivity, and start
appropriate therapy afterward

A 44-year-old homosexual man who was born in the United States was recently diagnosed with
HIV-1 infection based on positive results from both routine antigen/antibody screening and
confirmatory immunoassay testing. While he is devastated about the diagnosis, he commits to
being proactive about managing his disease, joining a support group and enrolling in HIV
education classes. He is seen at a follow-up clinic appointment for discussion of therapeutic
options and appears to be doing well. His vital signs and physical examination are unremarkable.
At this time, his CD4 cell count is 328 cells/mm3 and viral load is 125,000 copies/mL. The
patient wants to know what else he can do to protect himself, as his job as a taxi driver brings
him into frequent contact with other people who could be ill. Which of the following is an
appropriate intervention at this time?
 Incorrect Answer ImageA.Clarithromycin and ethambutol treatment to prevent
Mycobacterium avium complex (MAC) infection
 Incorrect Answer ImageB.Consumption of only bottled or boiled water to reduce the risk
of cryptosporidiosis
 Correct Answer ImageC.PPD placement, followed by isoniazid for 9 months if >5 mm of
induration and chest x-ray is negative
 Incorrect Answer ImageD.Trimethoprim/sulfamethoxazole for Pneumocystis
jirovecii pneumonia prophylaxis
 Incorrect Answer ImageE.Varicella vaccine if the patient was not exposed to varicella as
a child

A 20-year-old college student comes to the campus clinic because of an intense headache and
fever since last night. She reports that her temperature at the dormitory last night was 39.5ºC
(103.1ºF). She attempted to control it with acetaminophen. She is otherwise healthy and does not
take any medications, including oral contraceptive pills. On physical examination, she has pain
and resistance to motion and to flexion of the hips and knees when her neck is flexed to her
sternum. There is also pain and resistance when she lies on her back and her leg is flexed at the
hip and knee. She has a diffuse petechial rash. Her temperature today is 38.9ºC (102ºF).
Neurologic examination is nonfocal; however, she is quite irritable and photophobic. She is
admitted to the hospital for further management. Which of the following is the most appropriate
prophylactic strategy for this patient's roommate? 

 Incorrect Answer ImageA.Chloramphenicol


 Incorrect Answer ImageB.No prophylaxis needed
 Incorrect Answer ImageC.Penicillin
 Incorrect Answer ImageD.Polysaccharide conjugate vaccine
 Correct Answer ImageE.Rifampin
 Incorrect Answer ImageF.Vancomycin plus ceftriaxone

A 62-year-old man with hypertension and hyperlipidemia comes to the emergency department
complaining of a severe headache and high fever. He also reports neck pain, photophobia,
nausea, and vomiting. He has no HIV risk factors. His temperature is 39.0ºC (102.2ºF), blood
pressure is 100/70 mm Hg, pulse is 123/min, and respirations are 16/min. He is in no respiratory
distress. The light bothers his eyes and he reports neck pain on passive lateral movement. There
is no edema of the optic discs. Cranial nerve and mental status examination are normal. Cardiac
examination shows tachycardia with a regular rhythm and no murmurs. His lungs are clear to
auscultation and abdomen is soft and nontender. There is no edema. He has 5/5 strength in all 4
extremities with normal reflexes. Which of the following is the most appropriate sequence in
management?
 Incorrect Answer ImageA.Begin treatment with ceftriaxone and vancomycin; send him
for head CT scan; if CT is normal, perform lumbar puncture
 Incorrect Answer ImageB.Begin treatment with ceftriaxone, vancomycin,
dexamethasone, and ampicillin; send the patient for head CT scan; if CT is normal, perform
lumbar puncture
 Incorrect Answer ImageC.Perform lumbar puncture and begin treatment with ampicillin
and ceftriaxone
 Correct Answer ImageD.Perform lumbar puncture and begin treatment with ceftriaxone,
vancomycin, ampicillin, and dexamethasone
 Incorrect Answer ImageE.Stat CT scan, then start antibiotics and then perform lumbar
puncture

A 52-year-old woman with a history of emphysema spends a 2-week vacation on a cruise ship.
Shortly after returning home, she develops high fevers and becomes lethargic and disoriented.
Her husband describes that she has been coughing and short of breath since returning home. He
also describes that she has vomited several times over the past 48 hours and has had diarrhea. On
physical examination, she appears lethargic, but arousable. She is disoriented to the current date.
She has loud, coarse rhonchi in both lung fields. Her abdominal examination shows mild
tenderness over the liver edge. There is no splenomegaly or ascites present. Her neurologic
examination is nonfocal. Laboratory results are notable for an aspartate aminotransferase (AST)
of 112 U/L and an alanine aminotransferase (ALT) of 157 U/L. Which of the following is the
most appropriate treatment for this patient? 
 Incorrect Answer ImageA.IV ceftazidime
 Incorrect Answer ImageB.IV gentamicin
 Correct Answer ImageC.IV levofloxacin
 Incorrect Answer ImageD.IV nafcillin
 Incorrect Answer ImageE.IV vancomycin

A 40-year-old man with a 1-year history of HIV comes to the emergency department because of
a 4-week history of fever and headaches. Current medications include raltegravir, tenofovir, and
emtricitabine. He has a past history of IV drug abuse, and he has been in remission for 5 years.
His temperature is 37.8ºC (100ºF), pulse is 84/min, respirations are 14/min, and blood pressure is
140/90 mm Hg. On physical examination, the pupils are equal, round, and reactive, and there is
no papilledema. There is no neck rigidity or lymphadenopathy. The lungs are clear to
auscultation. Cardiac examination shows normal S1 and S2 without murmurs. His CD4+ T
lymphocyte count of 20 cells/mm3. A lumbar puncture is performed, and CSF analysis shows a
leukocyte count of 20 cells/mm3. India ink stain is negative. Cryptococcal antigen in the
cerebrospinal fluid is positive. The patient receives a 2-week course of amphotericin B and
flucytosine, and he improves significantly. Which of the following is the most appropriate
pharmacotherapy for this patient? 
 Correct Answer ImageA.Fluconazole until CD4+ count is >100 cells/mm3 for one year,
raltegravir, tenofovir, and emtricitabine
 Incorrect Answer ImageB.Raltegravir, tenofovir, and emtricitabine
 Incorrect Answer ImageC.Raltegravir, tenofovir, emtricitabine, and intrathecal
amphotericin B
 Incorrect Answer ImageD.Raltegravir, tenofovir, emtricitabine, and lifelong amphotericin
B and flucytosine
 Incorrect Answer ImageE.Raltegravir, tenofovir, emtricitabine, and lifelong fluconazole

An adult develops insidious onset of a severe infectious disease. The condition is characterized
initially by high fever, headache, pharyngitis, and arthralgias. The patient then goes on to
develop intestinal symptoms of constipation, anorexia, abdominal pain and tenderness. During
the second week of the illness, he has a rash with discrete pink, blanching lesions on the chest
and abdomen. The rash resolves about three days later. By the third week of the disease, the
patient appears very ill and has developed a florid diarrhea that is positive for occult blood.
During this same period, the man also develops secondary pneumococcal pneumonia. At the
height of the patient's illness, he was stuporous and had short periods of delirium. The spleen
was palpable during this period. Laboratory studies show leukopenia, anemia, liver function
abnormalities, and a mild consumptive coagulopathy. Which of the following is the most likely
diagnosis?
 Incorrect Answer ImageA.Brucellosis
 Incorrect Answer ImageB.Cholera
 Incorrect Answer ImageC.Melioidosis
 Incorrect Answer ImageD.Plague
 Correct Answer ImageE.Typhoid fever
A 36-year-old man comes to the physician because of a 2-day history of fever, headache, retro-
orbital pain, and muscle and joint pains. His past medical history is unremarkable. He returned
from a trip to India 5 days ago. He does not drink or smoke. His temperature is 38.4ºC (101.1ºF),
pulse is 100/min, and blood pressure is 100/70 mm Hg. Physical examination shows a diffuse
macular, confluent rash. There is cervical lymphadenopathy, oropharyngeal injection, injected
conjunctiva, and palatal and extremity petechiae. Laboratory studies show:
Hemoglobin  16 mg/dL 
Leukocyte count  3,000/mm3
Segmented neutrophils  40% 
Lymphocytes  60% 
Platelet count  40,000/mm3
Aspartate aminotransferase (GOT, AST)  77 U/L
Alanine aminotransferase (GLP, ALT)  87 U/L
Serum studies show:
Na+ 133 mEq/L 
Cl -
98 mEq/L 
K+ 3.5 mEq/L 
HCO3 -
21 mEq/L 
Urea nitrogen (BUN)  25 mg/dL 
Creatinine  0.98 mEq/L 
Alkaline phosphatase  98 U/L
Peripheral blood smear  Negative for Plasmodium spp.
Radiograph of the chest shows bilateral pleural effusions. Which of the following is the most
appropriate next step in management? 
 Correct Answer ImageA.Admission to the hospital
 Incorrect Answer ImageB.CT scan of the chest and abdomen
 Incorrect Answer ImageC.Diuretic therapy for acute heart failure
 Incorrect Answer ImageD.NSAIDs to relieve symptoms
 Incorrect Answer ImageE.Reassurance

A 22-year-old man comes to the clinic this morning because he noticed that his "eyelids are
sticking together". The patient also reports that he has had clear, watery discharge and a burning
sensation in both his eyes. He denies photophobia or ocular pain. The patient reports that he had
an upper respiratory infection for the last 3 days. He is an otherwise healthy college student.
Examination shows preauricular adenopathy, moderate clear discharge, and moderate
conjunctival injection in both eyes. His visual acuity is 20/20 bilaterally. Extraocular muscle
function and fundoscopic examination are normal. Which of the following is the most
appropriate management?
 Incorrect Answer ImageA.Antibiotic drops
 Incorrect Answer ImageB.Antihistamine drops
 Incorrect Answer ImageC.Cycloplegics
 Incorrect Answer ImageD.Immediate ophthalmology referral
 Correct Answer ImageE.No specific treatment is necessary
 Incorrect Answer ImageF.Routine ophthalmology referral
 Incorrect Answer ImageG.Topical corticosteroids

A college senior comes to the emergency department because of fever and vomiting for the past
6 hours. He has had a fever and sore throat for 2 days, which his primary care provider treated
with amoxicillin. His temperature is 39.4°C (102.9°F). Examination shows photophobia. A
lumbar puncture is performed and cerebrospinal fluid analysis is shown in the table below. 
Opening Pressure 350 mm H2O
White Blood Cell Count 1,000/mm3
85% segmented neutrophils and 15% lymphocytes
Glucose 20 mg/dL
Protein 250 mg/dL
 Incorrect Answer ImageA.Aseptic meningitis
 Incorrect Answer ImageB.Brain abscess
 Incorrect Answer ImageC.Granulomatous meningitis
 Incorrect Answer ImageD.Neighborhood reaction meningitis
 Correct Answer ImageE.Partially treated bacterial meningitis

A 22-year-old man comes to the emergency department with a 3-day history of fever, chills, a
cough, pleuritic chest pain, and low back pain. He says that the symptoms came on "out of the
blue." He endorses intravenous drug use. His temperature is 39ºC (102.2ºF), pulse is 70/min,
respirations are 16/min, and blood pressure is 120/80 mm Hg. Physical examination shows oval
retinal hemorrhages with a clear, pale center and pinpoint lesions in his nails. He has bruises on
his arms. Heart examination is unremarkable. Blood cultures are drawn. Chest x-ray shows
multiple patchy infiltrates. Laboratory studies show:
Hemoglobin 11 g/dL
Hematocrit 39%
ESR 39 mm/hr
Which of the following is the most likely pathogen? 
 Incorrect Answer ImageA.Candida albicans 
 Incorrect Answer ImageB.Pseudomonas aeruginosa 
 Incorrect Answer ImageC.Serratia marcescens 
 Correct Answer ImageD.Staphylococcus aureus 
 Incorrect Answer ImageE.Streptococcus viridans 

A 66-year-old man is brought to the emergency department with a 3-day history of fever, cough,
and confusion. Review of systems also is positive for the presence of mild abdominal pain and
non-bloody diarrhea for the past 2 days. He was previously healthy and living an active lifestyle.
He does not smoke or consume alcohol. His temperature is 38.9ºC (102.0ºF), pulse is 110/min,
respirations are 22/min, and blood pressure is 135/80 mm Hg. The patient is confused. Lung
examination shows mildly reduced air entry at the right base. Heart sounds are normal, and the
abdomen is soft and nontender with normal bowel sounds. Laboratory studies show:
Blood chemistry:
RBCs 4.5 million/mm3
Hb 15 g/dL
Hct 45%
Leukocytes  15,500/mm3
Differential:
Neutrophils  78%
Bands  12%
Lymphocytes 5%
Monocytes 5%
Chest radiograph shows right lower lobe patchy infiltrates. Sputum examination shows abundant
neutrophils with no organisms. The fecal occult blood test is negative. Which of the following is
the most appropriate pharmacotherapy? 

 Incorrect Answer ImageA.Ceftriaxone


 Correct Answer ImageB.Ceftriaxone and azithromycin
 Incorrect Answer ImageC.Ceftriaxone and gentamicin
 Incorrect Answer ImageD.Doxycycline
 Incorrect Answer ImageE.Erythromycin

A 32-year-old woman is brought to the emergency department by her friends for evaluation of
altered mental status. They report that for the last few days she has been acting strangely,
speaking with normal words but in a random fashion that makes no sense. Today she was found
naked on her couch, unable to explain where she was or what she was doing. Before this current
episode, her friends report that had been in good health except for a minor cold last week, with
fever and headaches. Her blood pressure is 129/82 mm Hg, pulse is 112/min, respirations are
20/min, and temperature is 38.3ºC (101.0ºF). Examination shows a disoriented woman in
moderate distress. Although her pupils and extraocular movements are normal, she shies away
from your penlight. Additionally, she moans in pain as her neck is flexed. CT scan shows mild
cerebral edema of the left temporal lobe. A lumbar puncture shows: 
Opening pressure 90 mm (normal: 70–180 mm)
White blood cells 382 cells/mm3 (80% lymphocytes) 
Red blood cells 470 cells/mm3
Protein 78 mg/dL
Glucose 70 mg/dL
Gram stain reveals no organisms. Cultures are sent, and the patient is treated empirically with
ceftriaxone and vancomycin pending results. In addition to these measures, this patient would
likely benefit from treatment with which of the following? 

 Correct Answer ImageA.Acyclovir


 Incorrect Answer ImageB.Amphotericin B
 Incorrect Answer ImageC.Dexamethasone
 Incorrect Answer ImageD.Foscarnet
 Incorrect Answer ImageE.Ganciclovir

A 24-year-old man comes to the physician because of a 10-day history of fever up to 38.2°C
(100.8°F), muscle aches, loss of appetite, and sore throat. He denies nausea, vomiting, or
diarrhea. He does not smoke cigarettes or drink alcohol. He is not sexually active. His
temperature today is 38.4°C (101.1°F), pulse is 90/min, and blood pressure is 124/78 mm Hg.
Physical examination shows tonsillar erythema with an exudate and bilateral anterior and
posterior cervical adenopathy. There is a fine, diffuse maculopapular rash. A streptococcal
antigen test on a throat swab is negative. Which of the following is the most sensitive and
specific diagnostic study for this patient's condition? 

 Incorrect Answer ImageA.CBC with heterophile test


 Incorrect Answer ImageB.IgG to Epstein-Barr (EB) viral capsid antigen (VCA)
 Incorrect Answer ImageC.IgM to cytomegalovirus (CMV)
 Correct Answer ImageD.IgM to EB VCA
 Incorrect Answer ImageE.Rapid plasma reagin (RPR)

A 67-year-old man with a history of chronic sinusitis comes to the emergency department with a
severe headache. While waiting to be seen, he develops a generalized seizure. The seizure
subsides after 10 minutes. The patient’s temperature is 40ºC (104ºF), pulse is 65/min,
respirations are 12/min, and blood pressure is 150/90 mm Hg. Ophthalmologic examination
shows protrusion of both eyes, and fundoscopic examination shows bilateral papilledema. The
right eye is noted to be pointed inward with the pupil nonreactive to light. Blood cultures are
drawn, and the patient is started on broad-spectrum IV antibiotics. Which of the following is the
most appropriate next step in management?

 Incorrect Answer ImageA.Dexamethasone


 Incorrect Answer ImageB.IV acyclovir
 Incorrect Answer ImageC.Loading dose of phenytoin
 Correct Answer ImageD.Obtain head CT scan
 Incorrect Answer ImageE.Perform lumbar puncture

An 18-year-old college student comes to the emergency department because of headache, fever,
and nausea. For the last 5 days, she has had severe fatigue, myalgias, and worsening fevers. She
reports that prior to the previous week, she was healthy and active. She recently returned from a
camping, hiking, and rock climbing trip through the hills of North Carolina prior to the
development of this current illness. She does not recall any tick bites, but she did have significant
mosquito exposure with bites on her arms and legs. Her temperature is of 39.1ºC (102.4ºF), pulse
is 65/min, respirations are 18/min, and blood pressure is 138/90 mm Hg. Physical examination
shows mild hepatosplenomegaly and faint macules on the hands, wrists, and ankles. Based on
this presentation, which of the following is the most likely pathogen?

 Incorrect Answer ImageA.Babesia microti


 Incorrect Answer ImageB. Borrelia burgdorferi
 Incorrect Answer ImageC.Leptospira interrogans
 Correct Answer ImageD.Rickettsia rickettsii
 Incorrect Answer ImageE.Treponema pallidum

An 18-year-old male college student comes to the clinic 36 hours after a dog bite. He works in an
animal physiology laboratory. While administering experimental medications to a poodle, the
animal bit his arm. The dog's behavior was normal prior to and after the bite. The animal is being
used for drug testing, has received appropriate vaccinations, and is certified by the breeder as
disease-free. The student originally was not going to see a physician, but the area around the bite
has become progressively more painful. His temperature is 37.2ºC (99.0ºF), pulse is 90/min,
respirations are 14/min, and blood pressure is 122/80 mm Hg. Physical examination shows a 2
cm area of erythema, warmth, and swelling surrounding the small, superficial puncture wound on
his right forearm. There is minimal induration but no fluctuation or crepitation. Plain radiograph
shows no bony injury or subcutaneous gas. Which of the following is the most appropriate next
step in management?
 Correct Answer ImageA.Amoxicillin-clavulanate
 Incorrect Answer ImageB.Ciprofloxacin
 Incorrect Answer ImageC.Rabies vaccination
 Incorrect Answer ImageD.Rabies vaccination and immunoglobulin
 Incorrect Answer ImageE.Reassurance

A 60-year-old man with a recent diagnosis of T-cell lymphoma comes to the physician for a
follow-up examination. Current medications include cyclophosphamide, doxorubicin,
vincristine, and prednisone. He has not received any immunizations over the past 10 years.
Which of the following is the most appropriate immunization to administer at this visit? 
 Incorrect Answer ImageA.Hepatitis A vaccine
 Incorrect Answer ImageB.Hepatitis A and hepatitis B vaccines
 Incorrect Answer ImageC.Inactivated influenza vaccine
 Correct Answer ImageD.Inactivated influenza vaccine and pneumococcal polysaccharide
vaccine
 Incorrect Answer ImageE.Inactivated influenza and zoster vaccines
 Incorrect Answer ImageF.Live-attenuated influenza vaccine
 Incorrect Answer ImageG.Live-attenuated influenza vaccine and tetanus toxoid (Td)
 Incorrect Answer ImageH.Measles, mumps, and rubella vaccine (MMR)
 Incorrect Answer ImageI.MMR and zoster vaccines
 Incorrect Answer ImageJ.Pneumococcal polysaccharide vaccine

A 70-year-old previously healthy woman is brought to the physician by her daughter because of
personality changes and forgetfulness for 3 months. She has quit her volunteer commitments and
does not go out with her family and friends. Six months ago, her husband died. She takes no
medications. Her temperature is 36.7°C (98°F). The patient answers questions with one-word
answers and appears apathetic. When asked what she ate for dinner the prior night and where she
went for Thanksgiving, she replies, "I don't know." Neurologic examination is significant for 4/5
strength in all extremities with poor effort throughout formal strength testing. Laboratory studies
show: 
Hemoglobin 13.1 g/dL
Leukocyte count 6,500/mm3
Thyroid-stimulating hormone 2.5 mU/L
Serum:
Na+ 138 mEq/L
Cl− 105 mEq/L
K +
4.3 mEq/L
HCO3− 24 mEq/L
Blood urea nitrogen (BUN) 12 mg/dL
Creatinine 0.8 mg/dL
An MRI scan of the brain is shown. Which of the following is the most likely diagnosis?
 Incorrect Answer ImageA.Alzheimer's dementia
 Incorrect Answer ImageB.Brain tumor
 Correct Answer ImageC.Depression
 Incorrect Answer ImageD.Frontotemporal dementia
 Incorrect Answer ImageE.Normal pressure hydrocephalus

A 65-year-old is brought to the emergency department because of pain in his right leg for 3 days.
He has been unable to walk since yesterday because of the pain. He states that his right leg feels
like “a log of ice” and he is unable to move it even when lying down. He does not recall having
any trauma to the leg. He has a history of atrial fibrillation, gastrointestinal bleeding while on
anticoagulation therapy, and hypertension. He had a four-vessel coronary artery bypass graft
surgery 1 year ago. Current medications include ramipril and metoprolol. He smoked one pack of
cigarettes daily for 40 years, but he quit 1 year ago. His temperature is 37.2ºC (99.0ºF), pulse is
70/min and irregular, respirations are 16/min, and blood pressure is 150/80 mm Hg. The right
lower extremity appears pale below the knee, is cool to the touch, and pulses are absent.
Sensation is also completely absent below the knee. Doppler examination shows a lack of arterial
and venous signals in the right lower extremity. Which of the following is the most appropriate
next step in management?
 Correct Answer ImageA.Amputation of the right leg
 Incorrect Answer ImageB.Arteriography of the right lower extremity
 Incorrect Answer ImageC.CT scan of the right lower extremity
 Incorrect Answer ImageD.Intravenous thrombolytic therapy
 Incorrect Answer ImageE.Percutaneous mechanical thrombectomy

A 24-year-old woman comes to the emergency department because of pain and blurred vision in
the left eye for 2 days. She has had progressive difficulty breathing over the past 2 months. Her
temperature is 37.9°C (100.2°F), blood pressure is 116/78 mm Hg, and pulse is 90/min.
Examination shows tender, erythematous nodules on the shins of both lower extremities. The
lungs are clear to auscultation. Slit-lamp examination of the left eye shows photophobia, corneal
erythema, and white precipitates on the cornea. Intraocular pressure is normal. A radiograph of
the chest shows bilateral hilar fullness. Which of the following characteristics is most likely in
this condition? 
 Incorrect Answer ImageA.Bloody diarrhea and tenesmus
 Incorrect Answer ImageB.Chronic, unremitting course
 Incorrect Answer ImageC.Parotid and lacrimal gland disease
 Incorrect Answer ImageD.Recurrent infection with encapsulated organisms
 Correct Answer ImageE.Symmetric polyarthritis

A 27-year-old office manager with a history of generalized anxiety disorder has had five recent
visits to the physician for a several-month history of fatigue. Her previous workup thus far has
been unremarkable and screening laboratories are within normal limits. Inquiries about specific
symptoms of depression on the last visit yielded the diagnosis of persistent depressive disorder,
and treatment options were discussed with the patient. On this visit the patient expresses
frustration that a medical etiology of her fatigue has not been identified and demands to see the
clinic notes from her last several visits. Which of the following is the most appropriate response
by the physician at this time?
 Incorrect Answer ImageA.Agree to show her the records when her depressive symptoms
improve
 Correct Answer ImageB.Arrange to review her clinic records with her as soon as possible
 Incorrect Answer ImageC.Discuss her case with a psychiatrist prior to showing her the
records
 Incorrect Answer ImageD.Explain to her that physician-patient confidentiality prohibits
her from seeing her medical records
 Incorrect Answer ImageE.Make copies of her medical records for her to review at her
convenience

A 65-year-old man with a history of hypertension and osteoarthritis comes to the physician
because of a 3-year history of progressive weakness. He had difficulty getting up from chairs and
reports mild muscle cramps involving his thighs, weakness of finger and wrist flexor, and
difficulty swallowing. He denies any recent illnesses. Muscle strength is 4/5 bilaterally in the
proximal lower extremities. Patellar reflexes are delayed bilaterally. Sensation is normal.
Laboratory studies show: 
Hemoglobin  12.9 g/dL
Hematocrit  44%
Leukocyte count  8,300/mm3
Platelet count  255,000/mm3
C-reactive protein  1.2 mg/dL 
Creatine kinase activity  300 U/L 
Results from which of the following diagnostic tests are most likely to provide a definitive
diagnosis for this patient? 
 Incorrect Answer ImageA.Antinuclear antibodies
 Incorrect Answer ImageB.Electromyography
 Incorrect Answer ImageC.Liver function tests
 Incorrect Answer ImageD.Magnetic resonance imaging of spine and bilateral lower legs
 Correct Answer ImageE.Muscle biopsy

A 40-year-old woman comes for a routine health maintenance examination prior to starting new
employment as a school teacher. Her medical history is unremarkable and she takes no
medications. She has 3 children, ages 17, 14, and 9 years. During the birth of her first child, she
required several blood transfusions. Physical examination shows no abnormalities. Laboratory
studies show: 
Anti-HAV Negative
HBsAg Negative 
Anti-HBs Positive 
Anti-HCV Positive 
Serum studies show:
Aspartate aminotransferase (AST, GOT)  15 U/L
Alanine aminotransferase (ALT, GPT)  17 U/L
Which of the following is the most appropriate next step in management? 
 Correct Answer ImageA.Direct-Acting Antiviral (DAA) therapy
 Incorrect Answer ImageB.Hepatitis B vaccination
 Incorrect Answer ImageC.Interferon/ribavirin treatment
 Incorrect Answer ImageD.Liver biopsy
 Incorrect Answer ImageE.Right upper abdominal ultrasound

A 48-year-old woman with a history of depression has been maintained on phenelzine for several
years. She comes to the physician with concerns about worsening depression. She would like to
discuss switching to a newer medication, such as a selective serotonin reuptake inhibitor (SSRI).
Additionally, she is tired of sticking to such a rigid diet. After discussion with the patient, the
doctor agrees to switch the patient to an SSRI. Which of the following statements concerning
switching her from phenelzine to an SSRI is correct?
 Incorrect Answer ImageA.A 4-week washout after discontinuing phenelzine is needed
before an SSRI can be started
 Correct Answer ImageB.A 14-day washout period after discontinuing phenelzine is
needed before an SSRI can be started
 Incorrect Answer ImageC.Abrupt switching typically causes hypotension
 Incorrect Answer ImageD.Abruptly stopping phenelzine typically causes a
discontinuation syndrome
 Incorrect Answer ImageE.No washout period after stopping phenelzine is needed before
an SSRI can be started

An 81-year-old woman was admitted 3 days ago after being brought to the hospital by her family
for worsening mental status. The patient was diagnosed with breast cancer 2 years ago, and the
disease has now widely metastasized to her spine and brain. Her family is concerned that she is
not being given adequate sedation and analgesia. She has long-standing bone pain and had been
using a fentanyl patch and oral hydromorphone at home. She is currently on a morphine drip.
The family would like the rate of the drip increased. Evaluation of the patient shows a cachectic
woman. Her blood pressure is 100/60 mm Hg, pulse is 76/min, and respirations are 14/min. She
is lying in bed with eyes closed and appears comfortable. The family demands that she be “made
to stop suffering” and asks that her quantity of morphine be increased. Which of the following is
the most appropriate course of action? 
 Incorrect Answer ImageA.Decrease the morphine drip rate, as the patient appears sedated
 Correct Answer ImageB.Explain to the family that the morphine dose is appropriate, as
the patient appears comfortable
 Incorrect Answer ImageC.Increase the morphine drip rate as the family requests
 Incorrect Answer ImageD.Inform the family that increasing the morphine would be
considered unethical
 Incorrect Answer ImageE.Inform the family that the patient appears comfortable, and ask
them to not make further requests of the medical team

A 44-year-old man complains of vague right upper abdominal discomfort that he has had for
approximately 1 month. He describes no other symptoms and, except for enucleation of one eye
at age 21 "for a tumor," he has been in excellent health all his life. He exercises regularly and
does not smoke or drink. The only findings on physical examination include the artificial eye and
a tender, enlarged, and nodular liver. Viral hepatitis serology is negative. CT scan of the upper
abdomen demonstrates multiple masses within the liver. Biopsy specimens of these masses will
most likely reveal which of the following? 
 Correct Answer ImageA.Metastatic malignant melanoma
 Incorrect Answer ImageB.Metastatic prostate cancer
 Incorrect Answer ImageC.Metastatic retinoblastoma
 Incorrect Answer ImageD.Metastatic sarcoma
 Incorrect Answer ImageE.Primary hepatocellular carcinoma

A 44-year-old-woman with diet-controlled type 2 diabetes presents to the office with a 4 month
history of gradually worsening weakness. She initially noticed weakness in her muscles and
generalized fatigue at the end of the day. This weakness has progressed to affect more of her
daily activities, and she has recently been unable to brush her hair or walk for more than a few
blocks. She admits to poor appetite and has lost approximately 20 lbs in the last 3 months.
Examination shows normal breath sounds in all lung fields. Neurologic examination shows
normal muscle tone. She has 3/5 strength in her proximal extremities, 4/5 strength in her distal
extremities. There is no loss of sensation in her extremities. Chest x-ray shows a 3.5-cm right
central lung mass. Bronchoscopy with biopsy was performed and results are pending. Which of
the following is the most likely cause of her neurologic symptoms?
 Incorrect Answer ImageA.Amyotrophic lateral sclerosis (ALS)
 Correct Answer ImageB.Antibodies to voltage-gated calcium channels (VGCC)
 Incorrect Answer ImageC.Brain metastases
 Incorrect Answer ImageD.Myasthenia gravis
 Incorrect Answer ImageE.Spinal cord metastases

A 54-year-old lawyer comes to the physician because of a 6-month history of fatigue. He reports
that he constantly feels tired and becomes short of breath with minimal physical activity. He has
unexpectedly lost approximately 5 kg (11 lb) over this period. He also reports having diarrhea
almost daily over the past year. He has no history of major medical illness and takes no
medications. He does not smoke tobacco or drink alcohol. His temperature is 37.2ºC (99.0ºF),
pulse is 108/min, respirations are 16/min, and blood pressure is 115/75 mm Hg. His skin is pale.
Lower extremity examination shows a loss of vibration sense, spasticity, and a positive Babinski
sign. Laboratory studies show:
Hematocrit 26%
WBC 3,700/mm3
Platelets 110,000/mm3
MCV 116 µm3
Reticulocyte count 0.5%
Serum lactate dehydrogenase 650 U/L
Serum total bilirubin 2 mg/dL
Which of the following is the most likely diagnosis? 
 Incorrect Answer ImageA.Alcoholic liver disease
 Incorrect Answer ImageB.Folate deficiency
 Incorrect Answer ImageC.Hemolytic anemia
 Incorrect Answer ImageD.Myelodysplastic syndrome
 Correct Answer ImageE.Vitamin B12 deficiency

 44-year-old woman with no significant past medical history presents to the emergency
department with progressively worsening shortness of breath that started approximately 10 days
ago. Upon further evaluation, she is found to have atrial fibrillation with rapid ventricular rate
and congestive heart failure. A transesophageal echocardiogram is done, which shows normal
atrial size, normal ejection fraction and no evidence of a thrombus. Cardioversion is done in the
emergency department and the patient converts to normal sinus rhythm. Routine laboratory
studies done in the emergency department show the following results: 
WBC 8,000/mm3
Hct 35%
Hgb 11.6 g/dL
Platelets 28,000/mm3
Sodium 134 mEq/L
Potassium 4.6 mEq/L
Chloride 101 mEq/L
Bicarbonate 26 mEq/L
BUN 10 mg/dL
Creatinine 0.8 mg/dL
Glucose 80 mg/dL
Calcium 8.2 mg/dL
LDH normal
Peripheral blood smear shows no abnormalities. On detailed history, the patient states that she
has noticed excessive bruising on her legs and episodes of bright red blood in the toilet for the
last few months. Which of the following is the most likely diagnosis? 
 Incorrect Answer ImageA.Aplastic bone disease
 Incorrect Answer ImageB.Disseminated intravascular coagulation
 Incorrect Answer ImageC.Hemolytic uremic syndrome
 Correct Answer ImageD.Idiopathic thrombocytopenic purpura
 Incorrect Answer ImageE.Thrombotic thrombocytopenic purpura

A 48-year-old man with a history of type 2 diabetes mellitus comes to the physician for a follow-
up examination. A review of his records shows that the patient has attempted to control his
diabetes and cardiac risk factors with diet and exercise for the past several years. His blood
pressure is 148/92 mm Hg. His fasting blood glucose level is 82 mg/dL and his lipid levels are
within normal range. Which of the following is the most appropriate next step in management for
this patient's hypertension?
 Incorrect Answer ImageA.Continue diet and exercise
 Incorrect Answer ImageB.Hydrochlorothiazide
 Correct Answer ImageC.Lisinopril
 Incorrect Answer ImageD.Metoprolol
 Incorrect Answer ImageE.Spironolactone
A 49-year-old veteran comes to the physician to discuss smoking cessation. He was recently
diagnosed with high blood pressure, hypercholesterolemia, and coronary artery disease, and
realizes that it is finally time to quit. He has been a heavy smoker for years and finds it
impossible to stop on his own, but his wife has heard about the use of bupropion for the
treatment of nicotine dependence. After discussing the pros and cons of starting bupropion, the
patient agrees to give it a try. Which of the following instructions should be given to the patient? 
 Correct Answer ImageA.Stop smoking completely one week after starting bupropion
 Incorrect Answer ImageB.Stop smoking completely upon reaching a dose of 450 mg/day
 Incorrect Answer ImageC.Stop smoking completely on the third day after starting
bupropion
 Incorrect Answer ImageD.Stop smoking on the same day as starting bupropion
 Incorrect Answer ImageE.Stop smoking after 6 months of treatment with bupropion

A 65-year-old man is hospitalized following an exploratory laparotomy for severe abdominal


pain. He has a history of hypertension, mild obesity, and hyperparathyroidism. On the second
postoperative day, the patient is awakened from sleep with severe pain in his right knee. He has
never had joint pain like this before. His temperature is 37.8°C (100°F) and blood pressure is
130/90 mm Hg. Examination shows an erythematous and edematous right knee that is tender to
palpation. A radiograph of the right knee shows calcification of the intra-articular cartilage and
meniscus. Joint aspiration shows rhomboid-shaped crystals with weak positive birefringence.
Which of the following is the most likely cause of this patient's arthritis? 
 Incorrect Answer ImageA.Calcium hydroxyapatite
 Incorrect Answer ImageB.Calcium oxalate
 Correct Answer ImageC.Calcium pyrophosphate dihydrate
 Incorrect Answer ImageD.Cystine
 Incorrect Answer ImageE.Monosodium urate

A 56-year-old man with a history of alcohol use disorder comes to the physician because of
several years of epigastric pain that is present at all times and is exacerbated by eating. He also
reports that for the past 3 years he has had a diagnosis of diabetes and oily spots on the surface of
his stool. He began to have acute episodes of severe upper abdominal pain in his mid-thirties and
has continued to have intermittent attacks of intense pain at unpredictable intervals, on top of his
constant. He has been smoking one pack of cigarettes and drinking half a bottle of vodka per day
for the past 30 years. He is 178 cm (5 ft 10 in) tall and weighs 57 kg (126 lb), BMI is 18 kg/m2.
His blood pressure is 120/76 mm Hg, pulse is 86/min, and respirations are 18/min. Physical
examination shows upper abdominal tenderness. Which of the following is the most appropriate
initial diagnostic test?
 Incorrect Answer ImageA.Abdominal x-ray
 Incorrect Answer ImageB.Abdominal ultrasound
 Correct Answer ImageC.CT scan of the upper abdomen
 Incorrect Answer ImageD.Secretin stimulation test
 Incorrect Answer ImageE.Upper GI series with barium
A 55-year-old man comes to the emergency department because of shortness of breath,
wheezing, and progressive dyspnea on exertion. He reports a past history of smoking over 3
packs of cigarettes a day for the last 40 years. He has never been intubated but has been admitted
to the hospital in the past for treatment with bronchodilators. Physical examination shows poor
air movement in his lungs. Chest x-ray shows flattening of the diaphragm and a barrel chest.
Recent pulmonary function testing showed severely reduced FEV1. The patient appears
increasingly confused and combative, pulling out all intravenous lines. Arterial blood gas shows:
pH 7.2
PO2 45 mm Hg
PCO2 80 mm Hg
Which of the following is the most appropriate next step in management?
 Incorrect Answer ImageA.Administer bronchodilators
 Incorrect Answer ImageB.Administer lorazepam
 Incorrect Answer ImageC.Administer oxygen through a non-rebreather mask
 Incorrect Answer ImageD.Apply non-invasive positive pressure ventilation
 Correct Answer ImageE.Sedate and emergently intubate

A 20-year-old man comes to the physician because of a 3-month history of occasional chest
tightness with exertion and with cold exposure. He has no medical history except eczema and
mild seasonal allergies. He takes no medications. His symptoms occur 2x/week, and about
1x/month he wakes up at night short of breath. His temperature is 37.0°C (98.6°F), blood
pressure is 120/70 mm Hg, pulse is 73/min, and respirations are 13/min. Physical examination
shows scattered wheezes bilaterally and a prolonged expiratory phase. Which of the following is
the most appropriate next step in management? 
 Correct Answer ImageA.Albuterol
 Incorrect Answer ImageB.Cromolyn sodium
 Incorrect Answer ImageC.Inhaled steroids
 Incorrect Answer ImageD.Long-acting muscarinic agonist

 Incorrect Answer ImageE.Zafirlukast

A 52-year-old woman with a history of psoriatic arthritis presents to her physician with moderate
pain and blurred vision in her left eye. She reports photophobia and left eye redness. She denies
any history of trauma or illness, and she does not wear contact lenses. Examination shows equal
and reactive pupils; however, the patient has difficulty keeping her eyes open even in normal
room lighting. Her visual acuity is 20/20 in the right and 20/60 in the left. There is no discharge
from the eye. There is no evidence of a corneal abrasion or foreign body. She is emergently sent
to her ophthalmologist for further evaluation and management. Which of the following is the
most likely etiology?
 Correct Answer ImageA.Anterior uveitis
 Incorrect Answer ImageB.Bacterial conjunctivitis
 Incorrect Answer ImageC.Decreased drainage of aqueous through the trabecular
meshwork
 Incorrect Answer ImageD.Primary acute angle-closure glaucoma
 Incorrect Answer ImageE.Retinal vasculitis

An 18-year-old man is taken to the emergency room by his family for a severe headache
accompanied by high fever. On physical examination, he is incoherent with nuchal rigidity.
Cerebrospinal fluid (CSF) shows gram-negative diplococci. When talking to the family, the
physician learns that the patient had five episodes of meningococcal meningitis in the past, the
earliest at the age of 6 years. Immunodeficiency related to impaired function of which of the
following should be suspected?
 Incorrect Answer ImageA.B cells
 Correct Answer ImageB.Complement factors
 Incorrect Answer ImageC.Eosinophils
 Incorrect Answer ImageD.Neutrophils
 Incorrect Answer ImageE.T cells

A study is conducted to assess the effectiveness of a new urine test for early detection of prostate
cancer. Two hundred men ages 60 to 75 years tested positive with the new urine test. Of these
men, 177 have an abnormal biopsy specimen result and 23 have normal biopsy specimen results
(p<0.05). Researchers conclude that this urine test should be used widely as a screening test for
prostate cancer in men ages 40 years and older because it offers high sensitivity and specificity
as compared to prostate-specific antigen (PSA). Which of the following is most likely to happen
if these recommendations are followed?
 Incorrect Answer ImageA.Incidence will increase
 Incorrect Answer ImageB.Negative predictive value will decrease
 Incorrect Answer ImageC.No change, as the study yielded a p-value <0.05
 Correct Answer ImageD.Positive predictive value will decrease
 Incorrect Answer ImageE.Positive predictive value will increase
 Incorrect Answer ImageF.Prevalence will increase

A 65-year-old man with a history of dyslipidemia and coronary artery disease comes to the
urologist stating that he has "problems having sex." He reports a gradual decrease in libido over
the last year, and he is now unable to achieve an erection. He states that he shares a healthy,
monogamous relationship with his wife. Current medications include aspirin, atorvastatin,
lisinopril, nitrates, and fish oil. After a thorough history and physical examination, which of the
following is the most appropriate next step in the management of this patient?
 Correct Answer ImageA.Obtain serum testosterone, prolactin, and glucose levels
 Incorrect Answer ImageB.Perform a digital rectal examination
 Incorrect Answer ImageC.Prescribe a vacuum constriction device
 Incorrect Answer ImageD.Prescribe clomipramine orally
 Incorrect Answer ImageE.Prescribe sildenafil orally
A 24-year-old woman comes to the emergency department with a 7-day history of a progressive
sore throat. She reports that she is now having difficulty swallowing solids and liquids because
of the pain. She has had episodes of tonsillitis in the past, and her symptoms initially started this
way. However, her symptoms have not progressed to be this severe before. She denies any
history of smoking or alcohol abuse. On review of symptoms, she admits to some changes in her
voice, which now sounds somewhat muffled. She also says that her left ear hurts, and she is
having trouble opening her mouth all the way. In addition, she has had some low-grade fevers.
She denies respiratory distress or drooling. Her temperature is 38.8ºC (101.8ºF), but otherwise
her vital signs within limits. Her leukocyte count is 16,000/mm3. Physical examination shows
she is in mild distress with a muffled voice but no stridor. Her neck is tender along the upper
cervical lymph node chain, particularly on the left, but there is no palpable mass or fluctuance.
Intraoral examination is difficult due to trismus, but reveals significant erythema, edema, and
palpable tenderness of the left soft palate area with secondary deviation of the uvula to the right.
Bilateral otoscopy is benign. This presentation is most consistent with which of the following
conditions?
 Incorrect Answer ImageA.Acute laryngitis
 Incorrect Answer ImageB.Acute tonsillitis
 Incorrect Answer ImageC.Chronic tonsillitis
 Correct Answer ImageD.Peritonsillar abscess
 Incorrect Answer ImageE.Retropharyngeal abscess

A 66-year-old man comes to the physician because of recurrent diarrhea, episodic cutaneous
flushing and shortness of breath for the past 3 months. He mentions that his flushing episodes
last for 2–3 minutes, resolve spontaneously and mostly involve his face and neck. He has had 10
to 12 foul-smelling, non-bloody stools daily. His shortness of breath is associated with wheezing
and has no triggering factors. He has had no fever, weight changes, headaches, or abdominal
pain. He drinks beer 2–3 times a week but does not smoke or use any recreational drugs. His
temperature is 37.2°C (99°F), pulse is 80/min, respirations are 20/min, and blood pressure is
110/70 mm Hg. On physical examination, a grade 3/6 systolic murmur is best heard over the left
sternal border. The lungs are clear to auscultation. A stool culture grows no organisms, and no
ova or parasites are seen. Which of the following is the most appropriate next step in the
diagnosis of this patient?
 Correct Answer ImageA.24-hour urine for 5-hydroxyindoleacetic acid (5-HIAA)
 Incorrect Answer ImageB.24-hour urine for catecholamine and metanephrine
 Incorrect Answer ImageC.Adrenocorticotropin hormone (ACTH) stimulation test
 Incorrect Answer ImageD.Dexamethasone stimulation test
 Incorrect Answer ImageE.Follicle-stimulating hormone (FSH) and luteinizing hormone
(LH) levels
 Incorrect Answer ImageF.Serum catecholamine levels
A 67-year-old man is brought to the emergency department because of chest discomfort and
nausea that started suddenly while eating 2 hours ago. He has a past medical history of peripheral
vascular disease, gastroesophageal reflux disease, diabetes, hypertension, chronic renal failure,
and an abdominal aortic aneurysm of 6.1 cm that is scheduled for surgical repair next week. He
is a current smoker. His temperature is 37.6ºC (99.7ºF), pulse is 129/min, and blood pressure is
160/94 mm Hg. On physical examination, the patient is sweating excessively. On closer
inspection, the patient has 7 cm JVD. There is no chest tenderness, and breath sounds are clear
bilaterally. There are muffled heart sounds that are regular in rate and rhythm with no murmurs
or rubs. An electrocardiogram is shown. Twenty-four hours after percutaneous coronary
intervention, the patient experiences intense chest pain with no radiation. His temperature is
36.5ºC (98ºF), pulse is 42/min, respirations are 23/min, and blood pressure is 104/62 mm Hg. He
does not improve following the administration of atropine. What is the most appropriate next
step in the management of this patient?
 Incorrect Answer ImageA.Administer a second dose of atropine
 Incorrect Answer ImageB.Coronary angiography
 Incorrect Answer ImageC.IV fluids
 Incorrect Answer ImageD.Percutaneous coronary intervention
 Incorrect Answer ImageE.Pericardiocentesis and pericardial window
 Correct Answer ImageF.Transcutaneous pacemaker
 Incorrect Answer ImageG.Transvenous pacemaker

A 77-year-old man with a history of hypertension, atrial fibrillation, diabetes, and emphysema
comes to the emergency department because of a 1-week history of progressive shortness of
breath. He was in his usual state of health until 1 week ago when he developed a worsening
cough with increased sputum production from his baseline. He uses home oxygen at bedtime but
now is requiring the oxygen 24 hours a day. He denies any chest pain, fever, or chills. He is
compliant with his medications which include diltiazem, insulin, warfarin, albuterol metered-
dose inhaler, ipratropium-bromide metered-dose inhaler, and a steroid metered-dose inhaler. His
temperature is 38.0ºC (100.4ºF), blood pressure is 170/70 mm Hg, pulse is 113/min, and
respirations are 33/min and labored. Physical examination shows a man in significant respiratory
distress. His heart is irregularly irregular and tachycardic. His lungs have poor air movement
with diffuse wheezing. There is no lower extremity edema. Chest x-ray shows no edema or
infiltrates. The heart is small and there is flattening of the diaphragm. During admission to the
medical floor, the patient warns the physician that during past hospitalizations, his blood glucose
significantly increased after being given systemic steroids. Which of the following is the most
appropriate pharmacotherapy?
 Incorrect Answer ImageA.Antibiotics alone
 Incorrect Answer ImageB.Inhaled steroids alone
 Incorrect Answer ImageC.Inhaled steroids and antibiotics
 Incorrect Answer ImageD.Systemic steroids alone
 Correct Answer ImageE.Systemic steroids, inhaled ipratropium and albuterol, and
antibiotics
A 20-year-old woman comes to the emergency department because of a severe, left-sided
headache, nausea, and vomiting that started suddenly 3 hours earlier. This headache is the worst
headache she has ever had. She has a 4-year history of similar left-sided headaches, nausea, and
vomiting that last 4 to 5 hours; she has one to two episodes monthly. During the episodes, she
typically rests in a dark room for relief. She is in mild distress and reports worsening headache to
light. Her temperature is 36.8°C (98.2°F), blood pressure is 110/70 mm Hg, pulse is 98/min, and
respirations are 16/min. There is no neck stiffness on examination. Strength and sensation are
intact without focal neurologic deficit. A CT of the head shows no abnormalities. Which of the
following is the most likely diagnosis?
 Incorrect Answer ImageA.Intracranial tumor
 Incorrect Answer ImageB.Meningitis
 Correct Answer ImageC.Migraine headache
 Incorrect Answer ImageD.Subarachnoid hemorrhage
 Incorrect Answer ImageE.Tension headache

The anthrax attacks in the United States raised concerns about the vulnerability of the military
forces to the use of biologic weapons in war, such as the smallpox virus. Some healthcare
workers and soldiers have been revaccinated. Which of the following conditions is a relative
contraindication to immunization against the smallpox virus? 
 Incorrect Answer ImageA.Chronic active hepatitis B
 Incorrect Answer ImageB.Chronic obstructive pulmonary disease
 Correct Answer ImageC.Coronary artery disease
 Incorrect Answer ImageD.Hypothyroidism
 Incorrect Answer ImageE.Inflammatory bowel disease

A 61-year-old man comes to his physician because of difficulty with erections. The patient states
that over the past 2 years, he has noticed that his erections have been progressively decreasing in
firmness. Initially, the problem was limited to an inability to maintain the erection until the
completion of intercourse; however, it has progressed to the point where his erections are
inadequate for penetration. He reports having a healthy relationship with his wife otherwise and
no reduction in sexual desire. His past medical history is significant for diabetes, chronic
obstructive pulmonary disease (COPD), and mild pulmonary hypertension. Current medications
include metformin, albuterol, an ipratropium inhaler, and a vasodilator for his pulmonary
hypertension that preferentially causes venous vasodilation. His genital examination is
unremarkable. Which of the following therapies is contraindicated in this patient?
 Incorrect Answer ImageA.Injectable alprostadil
 Correct Answer ImageB.Sildenafil citrate
 Incorrect Answer ImageC.Surgical insertion of penile prosthesis
 Incorrect Answer ImageD.Transurethral alprostadil
 Incorrect Answer ImageE.Vacuum erection device

A 48-year-old Caucasian man comes to the physician for a routine health maintenance
examination. He has had no chest pain, shortness of breath, or palpitations. His medical history
and family history are unremarkable, and he takes no medications. He does not smoke cigarettes,
drink alcohol, or use illicit drugs. He is 178 cm (5 ft 10 in) tall and weighs 92 kg (203 lb), BMI is
29.1 kg/m2. His pulse is 92/min and blood pressure is 129/80 mm Hg. Examination shows no
abnormalities. Serum electrolyte levels, complete blood count, and cholesterol levels are within
normal limits. Which of the following is the most appropriate recommendation for this patient?
 Incorrect Answer ImageA.Annual fecal occult blood or fecal immunochemical testing
 Incorrect Answer ImageB.Measurement of erythrocyte sedimentation rate
 Correct Answer ImageC.Measurement of fasting plasma glucose
 Incorrect Answer ImageD.Measurement of serum homocysteine levels
 Incorrect Answer ImageE.Offer behavioral interventions to prevent obesity-related
morbidity/mortality

A 56-year-old man comes to the physician because of a 6-month history of progressive mid-
epigastric pain that radiates to his back and is worse with eating. He has had greasy, foul-
smelling, large-volume stools for 2 months. He has a 1-month history of diabetes mellitus treated
with insulin. He drinks 5 to 6 glasses of vodka daily. He has been hospitalized twice in the past 3
years for acute pancreatitis. Which of the following is the most likely cause of this patient's
steatorrhea? 
 Incorrect Answer ImageA.Acid inactivation of pancreatic enzymes
 Incorrect Answer ImageB.Bacterial overgrowth of the small bowel
 Incorrect Answer ImageC.Diabetic neuropathy involving the colon
 Incorrect Answer ImageD.Excess secretion of glucagon
 Correct Answer ImageE.Pancreatic exocrine insufficiency

A 49-year-old man comes to the emergency department because of increasing nausea, diarrhea,
and diffuse abdominal pain for 5 days. His past medical history is notable for borderline diabetes
mellitus (not treated) and a positive PPD test, for which he was recently started on isoniazid
(INH) therapy one week ago. His blood pressure is 110/60 mmHg, pulse is 90/min, respirations
are 30/min, and temperature is 37.6ºC (99.7ºF). Pulse oximetry shows 98% oxygen saturation on
room air. Physical examination shows mild scleral icterus, dry mucous membranes, and diffuse
mild tenderness in the abdomen, more localized to the right upper quadrant; there is a negative
Murphy‘s sign and no tenderness at McBurney’s point. Laboratory studies show:
Blood
WBC 7,000 mm3
Hemoglobin 13 g/dL
Hematocrit 39%
Platelets 250,000 mm3
Serum
Na+ 135 mEq/L
K +
3.0 mEq/L
Cl- 100 mEq/L
HCO3 -
15 mEq/L
Glucose 196 mg/dL
BUN 30 mg/dL
Creatinine 1.0 mg/dL
The urinalysis is normal. Which of the following best explains the patient’s lab abnormalities?
 Incorrect Answer ImageA.Diarrhea
 Incorrect Answer ImageB.Distal renal tubular acidosis (Type 1)
 Incorrect Answer ImageC.Ketoacidosis
 Correct Answer ImageD.Lactic acidosis

 Incorrect Answer ImageE.Proximal renal tubular acidosis (Type 2)


 Incorrect Answer ImageF.Respiratory alkalosis

A 70-year-old man comes to the emergency department because of nausea, vomiting, and a
severe headache. He is an Alaskan native with no past medical history. He denies any trauma and
has never suffered symptoms like this in the past. His headache is mainly retro-orbital, and much
of his pain originates from his left eye. Examination shows the man shielding his left eye with
his hand. The eye is markedly red and injected and the cornea appears hazy. The pupil is mid-
dilated and minimally reactive to light. The right eye appears normal. Visual acuity tested with a
Snellen chart is 20/40 in the right eye and 20/200 in the left eye. His eye pressure in the left eye
is markedly elevated. Which of the following is the location of pathology for this patient’s most
likely condition?
 Incorrect Answer ImageA.Conjunctiva
 Incorrect Answer ImageB.Cornea
 Correct Answer ImageC.Iridocorneal angle
 Incorrect Answer ImageD.Lens
 Incorrect Answer ImageE.Retina

A 58-year-old African American man is brought to the emergency department because he could
not get out of bed this morning. He reports 2 weeks of worsening pain in his lower back and right
shoulder, and he has had difficulty urinating for the past 10 months. He denies any other
problems. Additionally, he has had no regular medical checkups. His temperature is 37.2ºC
(99ºF), blood pressure is 130/68 mm Hg, pulse is 72/min, and respirations are 16/min. There are
no sensory or motor deficits in the upper extremities. His neck has a full range of motion. Muscle
strength is diminished in both lower extremities. Straight leg raising does not elicit pain, but
temperature and pain sensation are absent in the right foot and nearly absent in the left foot.
Proprioception is spared. Plantar and deep tendon reflexes at the knees and ankles are absent. On
rectal examination, the prostate is large and firm, with non-tender nodularity. Which of the
following is the most likely diagnosis?
 Incorrect Answer ImageA.Amyotrophic lateral sclerosis (ALS)
 Correct Answer ImageB.Metastatic prostate cancer
 Incorrect Answer ImageC.Multiple sclerosis
 Incorrect Answer ImageD.Pott disease
 Incorrect Answer ImageE.Spinal stenosis
A 58-year-old man comes to the clinic because of erectile dysfunction. Over the past 6 months
he has tried sildenafil and testosterone as well as a variety of herbal medications, but has had no
success with these treatments. Review of systems is positive for mild arthralgias and fatigue,
which the patient has attributed to old age. Examination shows some mildly yellow-pigmented
patches on the torso as well as a firm, palpable liver tip, but is otherwise unremarkable.
Laboratory studies show an elevated fasting glucose with a mild elevation of the transaminases.
This patient is at risk for developing which of the following?
 Incorrect Answer ImageA.Hyperthyroidism
 Incorrect Answer ImageB.Hypertrophic cardiomyopathy
 Incorrect Answer ImageC.Iron deficiency anemia
 Incorrect Answer ImageD.Testicular cancer
 Correct Answer ImageE.Vibrio vulnificus  infection

A 69-year-old woman comes for a routine health maintenance examination. She denies fever,
weight loss, or fatigue. She has a history of hypothyroidism. Current medications include
levothyroxine and a multivitamin. Her temperature is 37°C (98.6°F). Physical examination
shows a 1 cm, firm, nontender, pearly nodule on the lower eyelid margin with a central
indentation, mild crusting, ulceration, and small telangiectasias spread outward from the lesion.
The eye shows no abnormalities and there is no localized lymphadenopathy. Which of the
following is the most appropriate next step in management?
 Correct Answer ImageA.Biopsy and resection of nodule with frozen section examination
of surgical margins
 Incorrect Answer ImageB.Hot compresses and topical antibiotic ointment applied to
lesion 3 times daily
 Incorrect Answer ImageC.Incision and drainage of nodule, apply hot compresses, return
to the clinic in 1 week
 Incorrect Answer ImageD.Intralesional injection of long-acting corticosteroid, follow up
in 1 month
 Incorrect Answer ImageE.No further intervention

A 39-year-old woman comes to the physician because of a 2-month history of fatigue, weight
loss, nausea, and muscle pains. She has a history of severe asthma treated with inhaled albuterol.
She took oral prednisone for 4 months but 3 months ago she stopped taking it because it was
making her gain weight. Her temperature is 37.0°C (98.6°F), blood pressure is 95/40 mm Hg,
pulse is 108/min, and respirations are 15/min. Diffuse wheezes are heard on auscultation.
Cardiac examination shows a normal S1 and S2 with no murmurs, rubs, or gallops. Laboratory
studies show: 
Na+ 127 mEq/L
K +
4.6 mEq/L
Cl− 93 mEq/L
HCO3 −
17 mEq/L
Glucose 59 mg/dL
BUN 35 mg/dL
Creatinine 1.0 mg/dL
ACTH Decreased
Which of the following is the most likely cause of this patient's condition? 
 Incorrect Answer ImageA.Autoimmune
 Incorrect Answer ImageB.Hemorrhagic
 Correct Answer ImageC.Iatrogenic
 Incorrect Answer ImageD.Infectious
 Incorrect Answer ImageE.Pituitary

A 45-year-old patient on hemodialysis for 1 week has noted that his blood pressure is more
difficult to control. He reports good compliance with his medications, which include epoetin alfa
(erythropoietin), ferrous sulfate, vancomycin, and vitamin D. His blood pressure is 180/99 mm
Hg. Which of the following is the most likely cause for the worsening control of his blood
pressure?
 Correct Answer ImageA.Epoetin alfa (Erythropoietin)
 Incorrect Answer ImageB.Ferrous sulfate
 Incorrect Answer ImageC.Vancomycin
 Incorrect Answer ImageD.Vitamin D
 Incorrect Answer ImageE.Uremia

A 20-year-old college student comes to the campus clinic because of a one-week history of fever
and muscle aches. Over the past 2 days, he developed a sore throat, headache, and swelling
behind his ear. Before this time, he was healthy without any major medical problems. Review of
systems is unremarkable. He denies the use of drugs or alcohol and states he has never had
sexual intercourse. His temperature is 38.9ºC (102.0ºF), pulse is 98/min, respirations are 20/min,
and blood pressure is 122/75 mm Hg. Head and neck examination shows symmetric, tender,
mobile, posterior cervical lymph nodes. A gray-green tonsillar exudate is present, and the tonsils
appear edematous and symmetrically enlarged without airway occlusion. The remainder of the
physical examination is normal. Laboratory studies show:
Serum Chemistry:
Bicarbonate 24 mEq/L
BUN 12 mg/dL
Chloride 101 mEq/L
Creatinine 0.6 mg/dL
Glucose 108 mg/dL
Potassium 4.3 mEq/L
Sodium 138 mEq/L
Hematology:
Hematocrit 36%
Leukocytes 19,000/mm3
Platelets 105,000/mm3
Differential review of a blood smear shows lymphocytosis (68%) with 10% atypical
lymphocytes. A heterophile test is positive. Which of the following is the most likely potential
fatal complication of this disease? 
 Incorrect Answer ImageA.Airway blockage
 Incorrect Answer ImageB.Disseminated intravascular coagulopathy
 Incorrect Answer ImageC.Myelodysplastic syndrome
 Incorrect Answer ImageD.Nasopharyngeal carcinoma
 Correct Answer ImageE.Splenic rupture

A double-blind, randomized, controlled trial is aimed at finding an association between artificial


tanning and skin cancer. The null hypothesis states: "Artificial tanning is not associated with an
increased risk for developing skin cancer." A sample of 200 patients are randomized to either
undergo artificial tanning twice every year or not, and the sample is followed for 5 years with
dermatology visits every 6 months. Any suspicious lesions are biopsied, and findings are
computed to yield a P value of 0.1. Which of the following would be an appropriate statement at
this point? 
 Incorrect Answer ImageA.Artificial tanning causes skin cancer
 Incorrect Answer ImageB.Artificial tanning has a protective effect against skin cancer
 Incorrect Answer ImageC.No statement can be made, as the study has a confounding bias
 Incorrect Answer ImageD.People who use artificial tanning have an increased risk for
skin cancer
 Incorrect Answer ImageE.There is no associated risk for skin cancer with artificial
tanning
 Correct Answer ImageF.There is not sufficient evidence to conclude that artificial
tanning increases the risk of skin cancer

A 69-year-old man in the cardiothoracic intensive care had a coronary artery bypass graft 6 days
ago. His surgery was uncomplicated and he was extubated immediately after the surgery. He
developed a fever and shortness of breath 4 days later, requiring intubation. One hour ago he
passed a large volume of black stool. Current medications include penicillin, gentamicin,
furosemide, aspirin, and subcutaneous heparin. His temperature is 38.9ºC (102ºF), blood pressure
is 180/94 mmHg, and pulse is 102/min. Loud rhonchi are heard bilaterally. Cardiac examination
shows a normal S1 and S2 with a 2/6 systolic murmur heard best along the left sternal border.
The abdomen is soft with mild epigastric tenderness and without masses. Rectal examination
shows hemoccult-positive stool. Laboratory studies show a leukocyte count of 14,400/mm3,
hemoglobin of 9.8 g/dL, and hematocrit of 29%. Which of the following is the most likely source
of this patient's bleeding?
 Incorrect Answer ImageA.Cecal arteriovenous malformations

 Correct Answer ImageB.Diffuse gastritis


 Incorrect Answer ImageC.Gastric varices
 Incorrect Answer ImageD.Giant gastric ulcer
 Incorrect Answer ImageE.Ischemic colitis

A 49-year-old man with a history of hypertension comes to the emergency department because
of severe chest pain. Laboratory studies and electrocardiogram are consistent with an anterior
wall myocardial infarction. He undergoes percutaneous coronary intervention with placement of
a stent into his left anterior descending artery and has complete resolution of symptoms.
Echocardiography shows a left ventricular ejection fraction of 52%. A pre-discharge stress test
shows no symptoms or electrocardiographic evidence of ischemia with maximal effort. When the
patient is ready for discharge, the physician is considering the best regimen for him. Which of
the following drugs would provide the greatest mortality benefit for this patient?
 Incorrect Answer ImageA.Angiotensin-converting enzyme (ACE) inhibitor
 Correct Answer ImageB.Beta-blocker
 Incorrect Answer ImageC.Loop diuretic
 Incorrect Answer ImageD.Spironolactone
 Incorrect Answer ImageE.Warfarin

A 66-year-old man with diabetes mellitus, hypercholesterolemia, and generalized atherosclerotic


occlusive disease comes to the physician because of erectile dysfunction (ED) over the last
several years. Initially, he was able to obtain erections, but they did not last long enough to
complete intercourse. He has lately noticed a decline in the quality of his erections which has
made penetration impossible. He further mentions that his libido is normal, and he shares a
healthy monogamous relationship with his wife of 30 years. He is very frustrated and is tearful as
he describes his symptoms. His diabetes is poorly controlled, and his last HbA1c was 7.5%.
Current medications include metformin, glimepiride, and atorvastatin. He does not smoke, drink
alcohol, or use any recreational drugs. Physical examination shows a circumcised male with
descended testicles bilaterally. He has a grade 1 left varicocele. On digital rectal examination, the
prostate is smooth, nontender, and weighs approximately 35 grams. Which of the following is
the most appropriate initial step in the management of this patient?
 Incorrect Answer ImageA.Erectile nerve reconstruction
 Incorrect Answer ImageB.Implantable penile prosthesis
 Correct Answer ImageC.Pharmacologic therapy
 Incorrect Answer ImageD.Psychotherapy
 Incorrect Answer ImageE.Pudendal artery revascularization

A 62-year-old Caucasian man presents for evaluation of chronic right-sided otitis media and
difficulty swallowing. He was recently treated with oral ciprofloxacin for 10 days in an effort to
relieve his ear infection, but it is not better. His difficulty swallowing has come on slowly,
subsequent to the development of the otitis. He has a history of obesity, hypertension, and poorly
controlled diabetes mellitus. He drinks 6-10 beers daily and chews tobacco daily. Examination of
his neck shows a non-tender 3 cm posterior cervical lymph node on palpation, and a 1.5 cm ulcer
with heaped up edges in the right posterior oropharynx. Which of the following is the next step
in the management of this patient?
 Incorrect Answer ImageA.Amoxicillin/clavulanic acid for his ear infection and a 6 week
follow-up
 Correct Answer ImageB.Biopsy of the ulcer
 Incorrect Answer ImageC.Lymph node excision
 Incorrect Answer ImageD.Nystatin swish and swallow
 Incorrect Answer ImageE.Tympanostomy (ear tube) insertion
A 74-year-old retired psychiatrist comes to the physician because of difficulty urinating. He
constantly feels the need to urinate and wakes up many times in the middle of the night to use the
bathroom. He states that his urine stream is weak and he always feels like there is "more left"
when he is done. He is otherwise healthy, plays tennis daily, and eats a healthy diet. He is
married and has 2 adult sons; his wife and children also see this physician. The patient often
spends his weekends at his tennis club, teaching his 4 grandchildren how to play his favorite
sport. He takes no medications, drinks a glass of wine with dinner, and does not smoke
cigarettes. His temperature is 36.7ºC (98ºF). Physical examination shows a firm, nodular
prostate. The remainder of the examination is unremarkable. Laboratory studies show a prostate-
specific antigen level of 10 ng/mL. A biopsy specimen of the prostate gland reveals
adenocarcinoma with a Gleason score of 4. When the patient arrives for his follow-up visit he
says that, against his wife's wishes, he has decided that he doesn't want to hear what the biopsy
specimen showed. He says that he would rather continue on with his life the way that it is. When
the physician asks why the patient does not want to hear the results of the biopsy, the patient
replies “I have decided that it is not necessary to worry about things that are out of my control,
and I will not undergo treatment even if I do have cancer.” Which of the following is the most
appropriate next step in management? 
 Incorrect Answer ImageA.Call the patient's wife and tell her the diagnosis
 Incorrect Answer ImageB.Have the adult sons come to the office with the patient to
convince him to hear the diagnosis
 Incorrect Answer ImageC.Have the patient see another physician and have the other
physician advise the patient about his diagnosis and prognosis
 Correct Answer ImageD.Honor the patient's wishes and do not tell him the diagnosis
 Incorrect Answer ImageE.Schedule the patient for treatment and hope that he changes his
mind
 Incorrect Answer ImageF.Tell the patient his diagnosis anyway, as you think he will
benefit from treatment

A 66-year-old man comes to the physician because of a 3-month history of progressive


weakness. He is no longer able to climb stairs. He has had a rash on his upper trunk and face. He
has had no tingling, numbness, headaches, difficulty swallowing, or incontinence. He takes no
medications. His temperature is 37.2ºC (99ºF), blood pressure is 135/80 mm Hg, and pulse is
80/min. Skin examination is shown. There is bilateral proximal muscle weakness. Deep tendon
reflexes are decreased. Laboratory studies show:
Na+ 138 mEq/L
Cl- 102 mEq/L
K +
4.4 mEq/L
HCO3- 25 mEq/L
Creatine kinase 30,000 ng/mL
Antinuclear antibody  Positive
Which of the following is the most appropriate next step in management? 
 Incorrect Answer ImageA.Cyclosporine
 Correct Answer ImageB.Electromyogram (EMG)
 Incorrect Answer ImageC.IV immunoglobulin
 Incorrect Answer ImageD.Muscle biopsy
 Incorrect Answer ImageE.Prednisone

A 50-year-old man comes to the physician because of sudden-onset pain around his ears for 1
week. Three days ago his ears began to swell. He denies any changes to his diet or recent travel.
His medical history is unremarkable. He takes a multivitamin daily. His temperature is 36.8°C
(98.2°F), pulse is 75/min, and blood pressure is 110/80 mm Hg. His oxygen saturation is 94% on
room air. Examination shows bilateral tenderness and swelling over the cartilaginous portions of
his ears. The noncartilaginous areas show no abnormalities. A biopsy specimen of the ear shows
granulation tissue in the cartilage. Pulmonary function testing is most likely to show which of the
following?
 Incorrect Answer ImageA.Decreased diffusion of carbon monoxide
 Correct Answer ImageB.Decreased expiratory flow
 Incorrect Answer ImageC.Increased FEV1
 Incorrect Answer ImageD.Reduced functional residual capacity
 Incorrect Answer ImageE.Reduced vital capacity

An 18-year-old man comes to the emergency department with his mother because of recurrent
episodes of vomiting for the past 3 months. The patient is preparing for a national wrestling
championship. In order to maintain his weight for the competition, he self-induces vomiting after
consuming large meals. However, his condition has now progressed to the point where he
instinctively vomits after each meal. In addition, he has recently been feeling lethargic. His
temperature is 37ºC (98.6ºF), pulse is 80/min, respirations are 8/min, and blood pressure is 90/60
mm Hg. Body mass index is 23 kg/m2. Physical examination shows dry mucous membranes,
mild bilateral parotid gland tenderness, multiple dental erosions, and dorsal hand calluses.
Laboratory studies show :
pH 7.50
PaCO2 50 mm Hg
PaO2 90 mm Hg
Na +
145 mEq/L
K+ 3.0 mEq/L
Cl –
93 mEq/L
HCO3– 30 mEq/L
BUN 20 mg/dL
Creatinine 1.0 mg/dL
Which of the following correctly describes the patient’s acid-base disorder?
 Incorrect Answer ImageA.Anion gap hypochloremic metabolic acidosis
 Incorrect Answer ImageB.Hyperchloremic respiratory alkalosis
 Correct Answer ImageC.Hypochloremic metabolic alkalosis
 Incorrect Answer ImageD.Hypochloremic respiratory acidosis
 Incorrect Answer ImageE.Primary respiratory alkalosis
A 23-year-old factory forklift operator is rushed to the emergency department 35 mins after
accidentally being pinned on the wall by a forklift machine. He has no medical or surgical
history, takes no medications, and uses marijuana on social occasions. Vital signs show a
temperature 36.3ºC (97.2ºF), blood pressure of 97/62 mm Hg, pulse 115/min, and respirations
28/min. His mental status is normal. Chest examination shows normal heart and lungs. There are
no peritoneal signs or bruising of the abdomen but multiple abrasions were present over his
thighs and calves. He is able to move both legs spontaneously but complains of severe pain. The
right calf is extremely tense. Distal pulses on both feet are present but diminished. The
genitourinary exam is normal. On rectal examination, the sphincter tone is normal and stool
heme negative. The prostate is smooth and nontender. A Foley catheter is inserted and a small
amount of dark urine is returned which is heme positive. Serum studies show: 
Potassium 6.1 mEq/L
BUN 41 mg/dL
Creatinine 2.7 mg/dL
Which of the following additional lab abnormalities is most likely? 
 Incorrect Answer ImageA.Bilateral hydronephrosis on renal ultrasound
 Correct Answer ImageB.Elevated serum creatine phosphokinase
 Incorrect Answer ImageC.Over 2 grams of albuminuria/24 hours on urine collection
 Incorrect Answer ImageD.Red blood cell casts on urinalysis
 Incorrect Answer ImageE.Urine eosinophils

A 66-year-old man is scheduled to undergo coronary angiography because of 6 months of


anginal chest pain with an exercise stress test that showed reversible ischemia in the left anterior
descending artery distribution. His past medical history is significant for essential hypertension
and type 2 diabetes, for which he takes glyburide, lisinopril, and hydrochlorothiazide. His pulse
is 80/min and his blood pressure is 135/79 mm Hg. His physical examination is remarkable for a
fourth heart sound and trace pretibial edema. His coronary catheterization is scheduled for 2 days
from now. His serum creatinine, drawn 1 week ago, was 1.8 mg/dL. The urinalysis and renal
ultrasound are normal. What is the most appropriate next step in the management of this patient? 
 Incorrect Answer ImageA.Cancel the angiogram

 Incorrect Answer ImageB.N-acetylcysteine before catheterization

 Correct Answer ImageC.Normal saline before catheterization

 Incorrect Answer ImageD.Renal CT scan before catheterization

 Incorrect Answer ImageE.Sodium bicarbonate before catheterization

A 48-year-old man is brought to the emergency department by his friends because of severe
shortness of breath. He was recently in a motor vehicle collision where he claimed to have
received no injuries except for “bruised ribs,” for which he is taking oxycodone for the pain. He
also reports one bout of vomiting after the car accident. On examination, the patient appears to
be wincing in severe pain every time he takes a deep breath. He is afebrile. His pulse is 110/min,
respirations are 8/min, and blood pressure is 150/80 mm Hg. He is lethargic and oriented only to
person and place. Chest examination shows diminished breath sounds throughout and pain upon
palpation all along the left side of the rib cage. He has a regular heart rhythm with normal S1 and
S2 and no murmurs or rubs. Abdomen is soft and mildly tender to palpation in the epigastrium.
An arterial blood gas shows: 
pH 7.25
PaCO2 60 mm Hg
PaO2 75 mm Hg
HCO3 25 mEq/L
The chest x-ray is still pending. Which of the following best explains the abnormal arterial blood
gas?
 Incorrect Answer ImageA.Lactic acidosis due to sepsis
 Incorrect Answer ImageB.Lactic acidosis due to visceral trauma
 Incorrect Answer ImageC.Mixed metabolic and respiratory acidosis
 Incorrect Answer ImageD.Respiratory acidosis due to hemothorax
 Correct Answer ImageE.Respiratory acidosis due to hypoventilation

A 21-year-old man is brought to the emergency department because of chest pain and dyspnea.
His girlfriend states that they were at a party when the patient began clutching his chest. He
appeared pale, diaphoretic, and was gasping for air. A review of his records shows that he has no
history of any major medical illnesses, takes no medications, and has no known drug allergies.
His father had a myocardial infarction at age 60 years. His mother has hyperlipidemia and needs
to take antibiotics before dental procedures because of a heart condition. His temperature is
37.0°C (98.6°F), pulse is 120/min, respirations are 18/min, blood pressure is 185/105 mm Hg,
and oxygen saturation 99% in room air. Physical examination shows dilated pupils. He is hostile
and mumbling incoherently. ECG is shown. Which of the following is the most likely diagnosis?
 Incorrect Answer ImageA.Cardiac contusion
 Correct Answer ImageB.Drug overdose
 Incorrect Answer ImageC.Pheochromocytoma
 Incorrect Answer ImageD.Plaque rupture
 Incorrect Answer ImageE.Valvular incompetence

A 32-year-old woman is brought to the emergency department by her husband after he found her
minimally responsive upon his arrival home from work. Her temperature is 37ºC (98.6ºF), blood
pressure is 105/65 mm Hg, pulse is 65/min, and respirations are 12/min. She is sedated and able
to utter only a few phrases, with slurred speech. Her past medical history is significant for
asthma since childhood and panic disorder for 4 years, on treatment. Which of the following is
the most appropriate next step?
 Incorrect Answer ImageA.Flumazenil immediately
 Incorrect Answer ImageB.Haloperidol immediately
 Incorrect Answer ImageC.Lorazepam
 Correct Answer ImageD.Monitor respiration and consciousness
 Incorrect Answer ImageE.Naloxone

An 86-year-old woman with a 6-month history of type 2 diabetes mellitus comes to the office
because she has noticed one week of ankle swelling. She otherwise feels well. She had a sore
throat 5 weeks ago, for which she received a 5-day course of azithromycin. Her diabetes is well
controlled, with daily fingerstick fasting glucose readings of 100–110. There was no albumin in
her urine 6 months ago, and a recent hemoglobin A1c was 6.5%. The patient has a past medical
history of type 2 diabetes, osteoarthritis, and hypertension. Her medications are metformin,
ibuprofen four times daily, and lisinopril. She does not smoke or drink. Her vital signs are
temperature 37.1ºC (98.7ºF), pulse 88/min, respirations 12/min, and blood pressure 140/100 mm
Hg. The examination shows normal optic fundi, a normal heart and lung examination, 2+ pitting
bipedal edema, and stiff non-inflamed joints in her hands with several nodes on the MCP joints.
The serum electrolytes, BUN, and creatinine are normal. The random glucose is 120 mg/dL.
Urinalysis shows: 
Specific gravity 1.020
pH 6.5
Blood Negative
Protein 4+
Ketones Negative
Glucose Negative
RBCs 0/HPF
WBCs 0/HPF
Epithelial cells Negative
Casts None
Crystals Negative
The spot urine protein/creatinine ratio is 5.0. Which of the following is the most likely cause of
her proteinuria? 
 Incorrect Answer ImageA.Azithromycin
 Incorrect Answer ImageB.Diabetic nephropathy
 Correct Answer ImageC.Ibuprofen
 Incorrect Answer ImageD.Lisinopril
 Incorrect Answer ImageE.Metformin

 Incorrect Answer ImageF.Postinfectious glomerulonephritis

A 30-year-old East Asian woman comes to the physician because of intermittent nocturnal chest
pain for 2 years. The pain lasts up to 10 minutes, is substernal, and is described as a heavy
pressure that radiates to her throat. The pain is a 6/10 on a pain scale and awakens her from
sleep. It is associated with mild nausea and a clammy sensation. In the past, she has tried
antacids and a proton-pump inhibitor, neither of which seem to help. Occasionally, significant
aerobic exercise induces the pain. Medical history includes Raynaud phenomenon, which is
severe during the winter months, and occasional migraine headaches treated with sumatriptan.
Social history is remarkable for the occasional use of cocaine. Vital signs are within normal
limits and physical examination shows no abnormalities. ECG shows no abnormalities. A Holter
monitor study is ordered. Given this patient's likely diagnosis, which of the following is the most
likely finding on Holter monitor during an episode of chest pain?
 Incorrect Answer ImageA.Diffuse ST-segment elevation and PR segment depression
 Incorrect Answer ImageB.Normal electrocardiographic tracings
 Incorrect Answer ImageC.Prolonged QT interval with increased duration at night
 Incorrect Answer ImageD.Transient ST-segment depressions in the lateral leads
 Correct Answer ImageE.Transient ST-segment elevation in the inferior leads

A 30-year-old man is being evaluated for pancytopenia. He has been complaining of weakness,
fatigue, and lethargy for the last 6 months. He initially thought that it was due to stress at work
and thus did not seek medical help. When he did get evaluated, he was found to be pancytopenic
with low red blood cell, white blood cell, and platelet counts. He denies any bleeding, epistaxis,
or recent infections. On examination, he appears pale. Pulse is 100/min and regular. He is
afebrile. Blood pressure is 120/78 mm Hg. Cardiovascular and lung exams are within normal
limits. There is no splenomegaly. Extremities show no pitting edema. Laboratory workup
shows: 
Hgb 7.2 g/dL
Hct 22%
WBC 2,000/mm3
Platelets 60,000/mm3
Blood chemistry Within normal limits
A bone-marrow biopsy is done which shows a hypocellular marrow. There is a decrease in all
cell elements. There is a predominance of fat cells and marrow stroma. There is no malignant
infiltrate. Which of the following is the most likely etiology of the patient's pancytopenia? 
 Correct Answer ImageA.Aplastic anemia
 Incorrect Answer ImageB.Hemolytic anemia
 Incorrect Answer ImageC.Leukemia
 Incorrect Answer ImageD.Lymphoma
 Incorrect Answer ImageE.Megaloblastic anemia

 52-year-old man with a 10-year history of pemphigus vulgaris comes for a follow-up visit. He
has had multiple hospitalizations, 4 courses of plasmapheresis, and complicated treatment
regimens including long courses of prednisone, mycophenolate mofetil, azathioprine, and
cyclophosphamide. He also has a history of hypertension and type 2 diabetes mellitus. Current
medications include rituximab, hydrochlorothiazide, and metformin. He weighs 90 kg (198 lb)
and is 173 cm (68 in) tall. His blood pressure is 140/85 mm Hg, pulse is 75/min, and respirations
are 16/min. Examination shows thin extremities. There are moderately large fat pads over the
nape of the neck and clavicles. There are multiple striae (stretch marks) on his abdomen and
proximal extremities. Which of the following is the most likely cause of these findings? 
 Incorrect Answer ImageA.Azathioprine
 Incorrect Answer ImageB.Diabetes mellitus
 Incorrect Answer ImageC.Mycophenolate mofetil
 Incorrect Answer ImageD.Pemphigus vulgaris
 Correct Answer ImageE.Prednisone

A 48-year-old man has had sudden, unilateral vision loss in the left eye upon awakening. He
denies any pain, headache, chest pain, or heart palpitations. He denies any past medical history
or current medication use. However, he has not seen a physician in approximately 10 years. At
this time, the ER obtains an immediate ophthalmology consult. The ophthalmologist finds the
visual acuity to be 20/25 in the right eye and 20/80 in the left eye. Fundoscopic examination
shows dilation of retinal veins, microaneurysms, multiple scattered retinal hemorrhages, macular
edema, and hard exudates in the patient's left eye. A small retinal hemorrhage, microaneurysms,
and dilated veins are seen in the right eye but without macular edema. Which of the following is
the most likely diagnosis of the underlying cause?
 Incorrect Answer ImageA.Acquired immunodeficiency syndrome
 Incorrect Answer ImageB.Giant-cell arteritis
 Incorrect Answer ImageC.Multiple sclerosis
 Incorrect Answer ImageD.Severe hypertension
 Correct Answer ImageE.Type 2 diabetes

A 36-year-old man was recently diagnosed with acute promyelocytic leukemia. He is started on
all-trans-retinoic acid and daunorubicin. Two days after receiving his first dose, the patient
presents to the emergency department with weakness and lethargy. He has new-onset epistaxis,
bright red blood per rectum, and hematuria. Physical examination shows conjunctival pallor and
obvious bleeding from Kiesselbach's plexus. His underwear is stained with bright red blood, and
rectal examination shows bright red blood on the examining finger. His pulse is 100/min,
respirations are 16/min and blood pressure is 90/70 mm Hg. He is afebrile. Skin examination
shows diffuse petechiae. Cardiovascular examination shows tachycardia. Lungs are clear to
auscultation. Extremities show no pitting edema. Laboratory workup shows:
Hgb 9.2 g/dL
WBC 28,000/mm3 (previously 29,000/mm3) 
Platelets 12,000/mm3
Sodium 136 mEq/L
Potassium 4.7 mEq/L
Chloride 101 mEq/L
Bicarbonate 27 mEq/L
BUN 40 mg/dL
Creatinine 1.8 mg/dL
Glucose 80 mg/dL
Calcium 9.2 mg/dL
PT 18 sec
INR 2.0
PTT 60 sec
Peripheral smear shows red-cell fragmentation. Which of the following is the most likely
diagnosis? 
 Incorrect Answer ImageA.Aplastic crisis
 Correct Answer ImageB.Disseminated intravascular coagulation
 Incorrect Answer ImageC.Fibrinogenolysis
 Incorrect Answer ImageD.Idiopathic thrombocytopenic purpura
 Incorrect Answer ImageE.Hemolytic uremic syndrome
 Incorrect Answer ImageF.Thrombotic thrombocytopenic purpura

A 67-year-old man presents to the clinic because of worsening urinary symptoms. He has noticed
that he is waking up more often at night to urinate, and when he tries to urinate he has to "push
harder" to start his urinary stream. Previous medical and surgical history is unremarkable.
Physical examination shows an uncircumcised phallus and normal testes. On rectal examination
his prostate is smooth, symmetric, and moderately enlarged. No nodules are felt. He is given a
prescription for tamsulosin and sent home. During a follow-up visit 8 weeks later, the patient
reports only minimal improvement of his symptoms. He still reports urinary frequency and
dribbling. A urinalysis with microscopy is performed and shows the following: 
pH 6.5
RBC 0–1/hpf
WBC 4/hpf
Bacteria scant
Culture 25,000 CFU/mL of E. coli
Which of the following is the best next step in management? 
 Incorrect Answer ImageA.Abdominal and pelvic CT scan
 Incorrect Answer ImageB.Bring to the OR for a transurethral microwave treatment
(TUMT)
 Incorrect Answer ImageC.Ciprofloxacin
 Incorrect Answer ImageD.Doxazosin
 Correct Answer ImageE.Finasteride
 Incorrect Answer ImageF.Prescribe goserelin acetate
 Incorrect Answer ImageG.Repeat urinalysis
 Incorrect Answer ImageH.Ultrasound-guided prostate biopsy

A 20-year-old woman with a 10-year history of type 1 diabetes mellitus comes to the physician
for a routine follow-up visit. She currently feels well, regularly plays soccer, and is doing well in
her studies. She has had no fever, weight loss, visual changes, polydipsia, or polyuria. She has
never had diabetic ketoacidosis (DKA) or hypoglycemic episodes. Current medications include
NPH (20 units twice daily) and insulin lispro (5 units pre-meal three times daily). Her home
blood glucose monitoring chart shows levels of 100–140 mg/dl pre-meals and at bedtime, with
an average of 116 mg/dL. The physical examination is unremarkable. Which of the following is
the most appropriate next step in the management of this patient?
 Incorrect Answer ImageA.Decrease the dosage of insulin lispro
 Incorrect Answer ImageB.Decrease the dosage of NPH
 Incorrect Answer ImageC.Increase the dosage of insulin lispro
 Incorrect Answer ImageD.Increase the dosage of NPH
 Correct Answer ImageE.Obtain a hemoglobin A1c level
An 18-year-old man comes to his physician because of increasing shortness of breath. He has
recently started working in a scientific laboratory doing experiments with mice. He reports
increasing dyspnea, a nonproductive cough, low-grade fever, and fatigue. He has never had these
symptoms before. He denies chills, shakes, vomiting, or diarrhea. He has no other medical
history, medications, or allergies. He denies smoking. His temperature is 37.9ºC (99.1ºF).
Examination shows bilateral crackles in the lungs. Chest x-ray is notable for reticulonodular
opacities at the lung bases and patchy infiltrates in the right middle lung. Which of the following
is the most appropriate next step in the management of this patient? 
 Incorrect Answer ImageA.Bronchoscopy and biopsy
 Incorrect Answer ImageB.CT scan of the chest
 Incorrect Answer ImageC.Ceftriaxone and azithromycin
 Incorrect Answer ImageD.Oral steroids
 Correct Answer ImageE.Stop exposure to the mice

 66-year-old man is brought to the emergency department by his son because of fever, cough,
and confusion. The patient's son reports that his father initially developed a cough and fever 2
days ago and today has become confused. The patient was healthy prior to this. On physical
examination, the patient seems confused and disoriented. His temperature is 38.5ºC (101.3ºF),
pulse is 110/min, respirations are 26/min, blood pressure is 98/57 mm Hg, and oxygen saturation
92% on room air. Cardiovascular examination shows tachycardia and normal heart sounds.
Crackles are heard in the right lower lobe. Abdomen is soft, nontender, and nondistended.
Neurologic examination shows no focal signs. Complete blood count shows leukocytosis. The
chest radiograph shows patchy infiltrates in the right lower lobe. Sputum shows neutrophils with
no organisms. Which of the following is the most appropriate treatment for this patient?
 Correct Answer ImageA.IV ceftriaxone plus azithromycin
 Incorrect Answer ImageB.IV clindamycin
 Incorrect Answer ImageC.IV nafcillin
 Incorrect Answer ImageD.Oral amoxicillin-clavulanate
 Incorrect Answer ImageE.Oral cefuroxime plus azithromycin
 Incorrect Answer ImageF.Oral erythromycin

A 54-year-old man comes to the physician because of inability to have and maintain an erection
for 4 months. He has had tenderness in his pectoral region and his nipples. He reports no recent
stressors. His blood pressure is 115/80 mmHg, and his pulse is 78/min. Examination shows
enlarged breast tissue bilaterally. Genitourinary examination shows a small mass in the left
testicle. Laboratory studies show: 
Estradiol 95 pc/mL (normal,10–50)
Alpha fetoprotein 25 ug/L (normal,<40)
Beta-human chorionic gonadotropin (HCG) Not detected
Ultrasound shows a 1.5-cm, well circumscribed mass in the left testicle. The patient undergoes
left-sided orchiectomy; the testicular mass specimen shows large cells with an eosinophilic
cytoplasm, bland nuclei, and small crystalline cytoplasmic inclusions. Which of the following is
the most likely diagnosis? 
 Incorrect Answer ImageA.Choriocarcinoma
 Correct Answer ImageB.Leydig cell tumor
 Incorrect Answer ImageC.Seminoma
 Incorrect Answer ImageD.Sertoli cell tumor
 Incorrect Answer ImageE.Teratoma
 Incorrect Answer ImageF.Yolk sac tumor

A 56-year-old woman comes to the physician because of severe back pain and mild hip pain for
2 days. The symptoms began when she bent over to pick up her cat. She has had fatigue and
malaise for 3 months. She has had no neurologic dysfunction or history of trauma. Her medical
history is otherwise unremarkable. Her blood pressure is 135/85 mm Hg and pulse is 80/min.
There is extreme tenderness to palpation in the mid-lumbar spine and an inability to flex and
extend her back because of pain. Radiographs of the lumbar spine and pelvis show a
compression fracture of her L3 vertebra and several round areas of radiolucency in her pelvis and
proximal femur. Laboratory studies show the following: 
Hb 11g/dL
MCV  85 fL
WBC  8,000/mm
Platelets  200,000/uL
Total protein  9.8 gm/dL
Albumin  3.0 gm/dL
Ca+ 12.0 mg/dL
Alkaline phosphatase  65 U/L
ESR  35 mm/hr
Which of the following is the most likely diagnosis? 
 Incorrect Answer ImageA.Lumbar disc herniation
 Incorrect Answer ImageB.Metastatic bone cancer
 Correct Answer ImageC.Multiple myeloma
 Incorrect Answer ImageD.Osteoporosis
 Incorrect Answer ImageE.Paget disease of bone

 26-year-old woman comes to the physician because of a 3-month history of joint pain in both of
her hands. The pain is relieved with ibuprofen. She has also recently started developing a
"blotchy, red rash" on her face after playing outdoor sports and reports having pain in her mouth
while brushing her teeth. Her temperature is 37.4ºC (99.3ºF), pulse is 76/min, respirations are
16/min, and blood pressure is 118/78 mm Hg. Examination shows a soft-tissue swelling at the
proximal interphalangeal joint of the left index finger. There are multiple punctate ulcers on her
buccal mucosa as well. The rest of her physical examination is unremarkable. Her laboratory
studies show:
Na+ 141 mmol/L
K+ 4.8 mmol/L
Cl −
97 mmol/L
HCO3− 23 mmol/L
Hemoglobin  12.5 g/dL
Platelet count 20,000/mm3
Leukocyte count 4,700/mm3
Segmented neutrophils 58%
Bands  3%
Eosinophils 1%
Basophils 0.50%
Lymphocytes 30%
Monocytes 3%
Serum ANA Positive
Serum anti-dsDNA Positive
Which of the following is an indication for starting steroid therapy in this patient? 
 Incorrect Answer ImageA.Joint pain
 Correct Answer ImageB.Low platelets
 Incorrect Answer ImageC.Oral ulcers
 Incorrect Answer ImageD.Photosensitivity
 Incorrect Answer ImageE.Positive serum ANA

 60-year-old man comes to the clinic for a physical examination. His company's health-insurance
provider has changed and the new insurance company is requiring all health plan members to
have a full physical examination, including a chest radiograph. The man reports smoking one to
2 packs of cigarettes per day since his mid-20s. He has no symptoms, reports being quite healthy,
and jogs 5 miles a day. Review of systems is negative for chest pain, hemoptysis, chronic cough,
weight loss, and melena. Physical examination is normal. His chest radiograph shows a solitary
pulmonary nodule in the periphery of his left mid lung zone. No old radiographs are available for
comparison. A CT scan of the chest shows a 2.8-cm mass with no calcifications. Which of the
following is the most appropriate next step in management?
 Incorrect Answer ImageA.Bronchoscopy
 Incorrect Answer ImageB.Needle biopsy
 Incorrect Answer ImageC.PET scan
 Incorrect Answer ImageD.Reassurance
 Correct Answer ImageE.Resection

A 32-year-old man is comatose in the intensive care unit. He sustained a massive head injury in a
motor vehicle accident 3 months ago and has been in the ICU since then. His O2 saturation is
93% on the ventilator, and he is receiving nutrition through a gastrostomy tube. On examination,
the patient is unresponsive to touch and painful stimuli. The general slowing of electrical brain
activity seen on the EEG suggests that the patient is in a persistent vegetative state. The patient
who is single, has no advance directive and has never discussed his wishes if he were unable to
make decisions for himself. His father desires care to be withdrawn, because he thinks that his
son would have wanted that, whereas his mother wants to continue maximal supportive care.
Which of the following is the best next step in management? 
 Correct Answer ImageA.Arrange a family meeting to make a decision about whether or
not to withdraw life support
 Incorrect Answer ImageB.Do not initiate terminal wean, as the EEG shows brain activity
 Incorrect Answer ImageC.Discontinue care based on the father's opinion, because he is a
better representative of the patient
 Incorrect Answer ImageD.Obtain a consult from the hospital ethics committee
 Incorrect Answer ImageE.Obtain a court-appointed legal guardian

A 17-year-old boy comes to the physician because of watery diarrhea for 3 weeks. He has had no
fever, nausea, vomiting, or blood in his stools. He has a history of recurrent sinusitis and
pneumonia. He plays soccer and goes camping frequently. He does not smoke, drink alcohol, or
use illicit drugs. His parents and a sister are in excellent health. His temperature is 37°C
(98.6°F). The cervical, axillary, and inguinal lymph nodes are enlarged. Laboratory studies
show: 
Hematocrit  44%
Leukocyte count  9,800/mm3
Neutrophils  55%
Lymphocytes  30%
Monocytes  5%
CD4 T-cell count  1,000 cells/mm3
Proteins, serum  6.2 g/dL
Albumin  5.0 g/dL
Globulin  1.2 g/dL
Serum IgG level is markedly decreased, and there are slightly below normal levels of IgM and
IgA. Examination of a stool sample shows Giardia lamblia. A biopsy specimen of a lymph node
shows hyperplastic follicles without plasma cells. Which of the following is the most likely
diagnosis? 
 Incorrect Answer ImageA.Acquired immunodeficiency syndrome (AIDS)
 Correct Answer ImageB.Common variable immunodeficiency
 Incorrect Answer ImageC.Hodgkin disease
 Incorrect Answer ImageD.Isolated IgA deficiency
 Incorrect Answer ImageE.X-linked agammaglobulinemia of Bruton

A 54-year-old man comes to the physician because of blurry vision and weakness for 1 month.
He has had difficulty raising his arms above his head and getting out of a chair. His eyes have
appeared more "droopy" than before. His medical history is unremarkable. He has smoked a
pack of cigarettes daily for 40 years. His temperature is 37.2°C (99°F) and blood pressure 136/82
mm Hg. Examination shows ptosis that improves with repetitive blinking. The chest is clear to
auscultation. Muscle strength is 4/5 in the proximal shoulders and legs and 5/5 in the rest of the
muscle groups. The weakness improves with repetitive exercise. Which of the following is the
most appropriate next step in management? 
 Correct Answer ImageA.CT scan of the chest
 Incorrect Answer ImageB.No further management is necessary
 Incorrect Answer ImageC.Proximal muscle biopsy
 Incorrect Answer ImageD.Pyridostigmine administration
 Incorrect Answer ImageE.Temporal artery biopsy
A 21-year-old African American man with a history of sickle cell disease comes to the
emergency department because of a 3-day history of severe pain and swelling of his right knee.
His temperature is 38.6°C (101.5°F), pulse is 110/min, respirations are 18/min, and blood
pressure is 115/65 mm Hg. Examination of the right knee shows swelling, tenderness, and
warmth with limited range of motion. An x-ray of the knee shows no abnormalities. Joint
aspiration and synovial fluid analysis shows leukocyte count 96,000/mm3 with 80% segmented
neutrophils; no organisms or crystals are seen on microscopy. Which of the following is the most
likely causal organism? 
 Incorrect Answer ImageA.Eshcerichia coli
 Incorrect Answer ImageB.Haemophilus influenzae
 Incorrect Answer ImageC.Salmonella
 Correct Answer ImageD.Staphylococcus aureus 
 Incorrect Answer ImageE.Streptococcus

A 66-year-old African American woman is brought to the emergency department by her husband
because of a severe headache for the past 2 hours. The pain started when she was watching a
movie at the movie theatre. She describes the pain as the “worst headache of her life,” unilateral,
and throbbing in character. It is associated with severe right-sided eye pain, blurred vision,
excessive lacrimation, and photophobia. She has also vomited several times since the onset of the
headache. She does not report any trauma or foreign-body sensation. Her past medical history is
remarkable for diabetes mellitus, for which she takes metformin. On physical examination, her
temperature is 37 ºC (98.3ºF), pulse is 97/min, respirations are 22/min, and blood pressure is
150/82 mm Hg. The patient is in distress and is covering her right eye with her hand. The right
eye has conjunctival injection, but there is no eyelid swelling. The patient refuses further eye
examination. The rest of her physical examination is unremarkable. Which of the following is the
best test to confirm the diagnosis?
 Incorrect Answer ImageA.Computed tomography of the head
 Incorrect Answer ImageB.Funduscopic examination
 Incorrect Answer ImageC.Lumbar puncture
 Incorrect Answer ImageD.Temporal artery biopsy
 Correct Answer ImageE.Tonometry
 Incorrect Answer ImageF.Topical fluorescein administration

A 72-year-old man develops a pruritic, red rash 1 hour after undergoing an angiogram. He has
had no lip swelling, shortness of breath, diarrhea, or abdominal pain. He has a history of type 2
diabetes mellitus, and he underwent coronary artery bypass grafting 5 years ago. Current
medications include metformin, aspirin, and omeprazole. He reports an allergy to penicillin. He
is in no acute distress. His temperature is 36.7ºC (98.0ºF), pulse is 70/min, respirations are
18/min, and blood pressure is 130/80 mm Hg. Physical examination of the skin shows diffuse
erythema and edema of the eyelids. The neck, abdomen, back, and proximal extremities have
discrete and confluent erythematous macules and patches that blanch with pressure.
Dermographism is present. The remainder of the examination is unremarkable. Which of the
following is the most likely underlying mechanism for this patient's skin lesions?
 Incorrect Answer ImageA.Cell-mediated hypersensitivity
 Incorrect Answer ImageB.Cytotoxic hypersensitivity
 Correct Answer ImageC.Direct mast-cell degranulation
 Incorrect Answer ImageD.IgE-mediated hypersensitivity
 Incorrect Answer ImageE.Immune complex-mediated hypersensitivity

A 50-year-old man comes to the physician because of progressive thickening and swelling of the
skin on his legs and arms for 2 weeks. He has had no fevers, weight loss, chest pain, heartburn,
or weakness. His past medical history is unremarkable. His blood pressure is 126/82 mm Hg.
Physical examination shows patches of thickened erythematous skin over the anterior surfaces of
the upper and lower extremities sparing the fingers with an orange-peel appearance. There are no
telangiectasias or calcinosis on the skin. Which of the following is the most likely diagnosis?
 Incorrect Answer ImageA.CREST syndrome
 Incorrect Answer ImageB.Dermatomyositis
 Correct Answer ImageC.Eosinophilic fasciitis
 Incorrect Answer ImageD.Polymyalgia rheumatica
 Incorrect Answer ImageE.Polymyositis

A 63-year-old woman comes to the emergency department because of a 45-minute history of


palpitations, nausea, sweating, and feeling a sense of impending doom. She has had anxiety,
restlessness, and tremors. She also reports feeling hot. Her blood pressure is 148/90 mm Hg and
pulse is 103/min. Laboratory studies show a thyroid-stimulating hormone (TSH) level of 0.1
µU/mL. Serum creatinine kinase and troponin enzymes are negative. Which of the following is
the most appropriate next step in management? 
 Incorrect Answer ImageA.Alprazolam and paroxetine

 Incorrect Answer ImageB.Fine-needle aspiration of the thyroid

 Incorrect Answer ImageC.Methimazole

 Correct Answer ImageD.Propranolol

 Incorrect Answer ImageE.Radioactive iodine ablation

A 70-year-old woman with a history of diabetes mellitus is brought to the emergency department
with sudden onset of weakness on the right side of her body. She has a 20-pack-year history of
smoking and is currently taking metformin and hormone replacement therapy. The patient is
oriented to time, place, and person, and seems to answer all the questions very well. She was
able to dress herself before being brought to the ER. Her blood pressure is 140/90 mm Hg and
pulse is 85/min. Physical examination shows loss of strength in the right upper and lower
extremities, right-sided facial weakness, and (click media file). The visual fields are intact. A CT
scan of the head is ordered. Occlusion of which of the following vessels is the most likely cause
of this patient's symptoms?
 Incorrect Answer ImageA.Anterior cerebral artery
 Incorrect Answer ImageB.Middle cerebral artery
 Incorrect Answer ImageC.Penetrating arterioles of the anterior thalamus
 Correct Answer ImageD.Penetrating arterioles of the posterior limb of the internal
capsule
 Incorrect Answer ImageE.Posterior cerebral artery

A 56-year-old woman comes to the physician because of a 2-year history of a burning sensation
in her legs. She has a “pins-and-needles” feeling from her soles to her ankles, as well as
associated numbness in the same distribution. This feeling is worse at night. Hydrocodone and
acetaminophen provide minimal relief. She has had no weakness, headaches, confusion, or
memory loss. She has a 22-year history of hypertension, chronic kidney disease, and type 2
diabetes mellitus with retinopathy and diabetic gastroparesis. Current medications include
insulin, metformin, lisinopril, and hydrochlorothiazide. Serum creatinine is 2.8 mg/dL and
hemoglobin A1c level is 11.6%. Which of the following is the most appropriate
pharmacotherapy for this patient's leg pain?
 Incorrect Answer ImageA.Acetaminophen
 Incorrect Answer ImageB.Ketorolac
 Incorrect Answer ImageC.Lorazepam
 Incorrect Answer ImageD.Pioglitazone
 Correct Answer ImageE.Pregabalin

A 28-year-old woman with a history of peptic ulcer disease comes to the physician because of a
5-month history of watery diarrhea. She has severe burning in her chest and abdomen following
meals. She underwent therapy with amoxicillin, clarithromycin, and omeprazole for Helicobacter
pylori 2 months ago without improvement in her symptoms. Her temperature is 37ºC (98.6ºF),
pulse is 86/min, respirations are 12/min, and blood pressure is 110/67 mm Hg. Abdominal
examination shows no distention, tenderness, or shifting dullness. Upper endoscopy shows three
ulcers in the first and second portions of the duodenum. Which of the following is the most
appropriate next step in management?
 Incorrect Answer ImageA.Biopsy of the duodenal ulcers
 Incorrect Answer ImageB.Culture of the watery stools
 Correct Answer ImageC.Measurement of serum gastrin
 Incorrect Answer ImageD.Repetition of the H. pylori eradication every 2 months
 Incorrect Answer ImageE.Replenishment of the normal gut flora

A 34-year-old woman comes to the physician because of a 1-week history of a constant, dull,
throbbing headache in the front of her head and face. She reports that the pain is worse when she
leans forward. She has had a recurrent cough that is worse at night, nasal congestion, and a
yellow-green nasal discharge. Examination shows tenderness to palpation of the maxillary
sinuses. Which of the following is the most appropriate next step in diagnosis?
 Incorrect Answer ImageA.Biopsy
 Incorrect Answer ImageB.CT scan of the head
 Incorrect Answer ImageC.MRI scan of the head
 Correct Answer ImageD.No further testing needed
 Incorrect Answer ImageE.Sinus radiography
 Incorrect Answer ImageF.Ultrasonography

A 23-year-old woman comes to the emergency department because of abdominal pain. The pain
began 12 hours ago and was vague. Two hours later, she began feeling nauseated and vomited
twice. Over the past 4 hours, the abdominal pain has become more severe and localized to the
right lower quadrant (RLQ). Her last menstrual period was 10 days ago. She is sexually active
with a single partner and uses condoms intermittently. Physical examination shows right lower
quadrant tenderness. Deep palpation in the left lower quadrant causes pain in the right lower
quadrant. Pelvic examination shows no abnormalities. Her laboratory values are normal except
for a leucocytosis of 12,000/mm2. Which of the following is the most appropriate next step? 
 Correct Answer ImageA.Bring her to the operating room
 Incorrect Answer ImageB.Check a urinalysis
 Incorrect Answer ImageC.Colonoscopy
 Incorrect Answer ImageD.Discharge to follow-up with her primary care physician
 Incorrect Answer ImageE.Treat with antibiotics for pelvic inflammatory disease

A 65-year-old man comes to the physician because of progressive difficulty in maintaining an


erection long enough for satisfying intercourse for 10 years. He also complains of decreased
ejaculate volume. Sometimes he is not able to ejaculate at all. He has a history of type 2 diabetes
and hypertension controlled with lisinopril and glyburide. His blood pressure is 122/74 mm Hg
and pulse is 70/min. Genital examination shows a normal-appearing penis and testes. The
prostate is mildly enlarged without palpable masses. There are decreased pulses in the lower
extremities bilaterally. Sensation is normal. Laboratory studies show:
Serum total 28 ng/mL (normal, 10–35 ng/mL)
testosterone
Serum luteinizing 15 IU/L (normal, 6–20 IU/L for a healthy
hormone 65-year-old man)
Which of the following is the most appropriate next step in management? 
 Incorrect Answer ImageA.Change his oral hypoglycemic agent to metformin
 Incorrect Answer ImageB.Prescribe intracavernous prostaglandin injections
 Correct Answer ImageC.Prescribe oral sildenafil
 Incorrect Answer ImageD.Prescribe testosterone supplementation
 Incorrect Answer ImageE.Recommend intensive couples psychotherapy

A 44-year-old woman comes to the emergency department because of acute epigastric pain
radiating to her back. These symptoms occurred abruptly, and the pain has been increasing in
severity ever since its onset a few hours ago. She has never had this pain before. She has an
unremarkable past medical history and does not take any medications. Her blood pressure is
125/75 mm Hg and pulse is 92/min. On physical examination, she is obese. Her epigastrium is
tender to palpation. Amylase and lipase serum levels are elevated and abdominal CT scan shows
pancreatitis. She is hospitalized and treated supportively. Five weeks later she notes that her eyes
are yellow. Her total bilirubin is 3.5 mg/dL, with a conjugated fraction of 3.0 mg/dL. Which of
the following is the most likely etiology for her jaundice?
 Incorrect Answer ImageA.Cholangiocarcinoma
 Incorrect Answer ImageB.Gilbert syndrome
 Incorrect Answer ImageC.Hemolysis
 Incorrect Answer ImageD.Pancreatic head tumor
 Correct Answer ImageE.Pseudocyst formation

A 58-year-old woman is hospitalized because of fatigue, night sweats, low-grade fever, and
splenomegaly. Laboratory studies showed acute myelocytic leukemia with Auer rods. The
physician who is caring for this patient stops by her room on the medical floor to give her the
diagnosis. As he enters the room, the patient is in bed, eating, with her relatives, with whom she
is very close. She is quite anxious and inquires about her laboratory results. Which of the
following is the most appropriate response from the physician? 
 Correct Answer ImageA.Ask the family to leave the room, then ask whether she would
like anyone else to be present while discussing the results
 Incorrect Answer ImageB.Ask the patient if she wants you to speak about her results
while her relatives are in the room
 Incorrect Answer ImageC.Proceed to tell her she has acute myelocytic leukemia with a
poor prognosis
 Incorrect Answer ImageD.Proceed to tell her she has acute myelocytic leukemia but she
should be optimistic
 Incorrect Answer ImageE.Tell her that she has acute myelocytic leukemia and will
require aggressive chemotherapy

A 74-year-old diabetic woman is brought to the emergency department because of chest pain.
Twelve hours earlier she began to experience severe, retrosternal chest pain that radiated to her
left arm. She has a history of gastroesophageal reflux disease and initially thought that this
current episode was an episode of "heartburn." On physical examination, the patient appears in
great discomfort and is currently breathing through a face mask set at 100% O2. She is pale and
sweating profusely. Her chest pain resolves after treatment with aspirin, clopidogrel, IV heparin,
beta-blockers, and IV nitrates is initiated. Two days after admission to the coronary care unit, she
re-experiences severe chest pain and her blood pressure drops to 105/67 mm Hg. She has a pulse
rate of 126/min. Scattered rales are heard on auscultation and a systolic thrill is felt on chest
palpation. Cardiovascular examination is significant for a pansystolic murmur heard greatest at
the apex and radiating to the axilla. Which of the following is the most likely diagnosis?
 Incorrect Answer ImageA.Cardiac tamponade
 Incorrect Answer ImageB.Congestive heart failure
 Incorrect Answer ImageC.Free wall rupture
 Correct Answer ImageD.Papillary muscle rupture
 Incorrect Answer ImageE.Right ventricular infarct
A 25-year-old man comes to the physician because of new-onset lesions on his genitals. He has
been sexually active since age 18 and has not had any discharge or problems urinating in the
past. The patient has been in a steady, sexually monogamous relationship for the past year. His
medical history is remarkable only for childhood asthma. Current medication consists of a daily
multivitamin. Vital signs are within normal limits. Examination of the genital area shows several
flat, pink to tan, sessile papules with a slightly keratotic surface on the coronary sulcus and shaft
of the penis. There are no palpable inguinal lymph nodes. The urethral meatus is unaffected and
there is no spontaneous discharge. After discussing the diagnosis and available treatment options,
the physician and patient agree that conservative medical treatment is best suited for him. He
requests a medication that he can apply at home at his convenience so that he does not have to
make repeated office visits in the near future. Which of the following treatments would be most
appropriate to prescribe?
 Incorrect Answer ImageA.Cantharidin solution
 Correct Answer ImageB.Imiquimod cream
 Incorrect Answer ImageC.Liquid nitrogen
 Incorrect Answer ImageD.Mupirocin cream
 Incorrect Answer ImageE.Podophyllin solution

A 39-year-old man comes to his physician because of a 1-year history of personality changes,
abnormal involuntary movements, and memory dysfunction. His father and grandfather died in
their 50s because of progressive mental deterioration accompanied by abnormal movements. The
patient is married but has no children. Neurologic examination and psychometric testing reveal
difficulty in concentration, mild depression, and marked restlessness. During the examination,
grimacing of the face and intermittent shrugging of the shoulders are noted. Which of the
following disorders is the most likely in this patient? 
 Incorrect Answer ImageA.Creutzfeldt-Jakob disease
 Incorrect Answer ImageB.Gilles de la Tourette syndrome
 Correct Answer ImageC.Huntington disease
 Incorrect Answer ImageD.Sydenham chorea
 Incorrect Answer ImageE.Tardive dyskinesia

A 25-year-old man comes to his physician because of syncope associated with exercise. He
states that this symptom began approximately a year ago, while he was playing basketball. He
frequently feels lightheaded when he exerts himself. Recently, however, he has transiently lost
consciousness. He denies chest pain and palpitations and reports no leg swelling. He has no
history of any major medical illnesses and takes no medications. A review of his records shows
that several of his family members have died suddenly at young ages during exertion. His pulse
is 78/min and his blood pressure is 125/72 mm Hg. A grade 2/6 systolic crescendo-decrescendo
murmur is heard along the left sternal border. The carotid pulses are brisk without delay or
weakness. There is no clubbing, cyanosis, or edema. Which of the following is the most
appropriate next step in patient care?
 Incorrect Answer ImageA.Atenolol
 Incorrect Answer ImageB.Diltiazem
 Incorrect Answer ImageC.Disopyramide
 Correct Answer ImageD.Implantable cardiac defibrillator
 Incorrect Answer ImageE.Myectomy
 Incorrect Answer ImageF.Stress test

A 47-year-old woman comes to the physician's office with bone pain, myalgias, constipation,
and a depressed mood. She has also been hospitalized twice in the last year for urolithiasis.
Laboratory studies show hypercalcemia and elevated levels of parathyroid hormone. The
patient's renal function is normal, and neck ultrasound shows a parathyroid mass. Which of the
following laboratory abnormalities would also be expected in this patient? 
 Incorrect Answer ImageA.Hyperphosphatemia
 Incorrect Answer ImageB.Hyponatremia
 Correct Answer ImageC.Hypophosphatemia
 Incorrect Answer ImageD.Low alkaline phosphatase
 Incorrect Answer ImageE.Low dihydroxy-vitamin D

A 38-year-old woman comes to the physician because of a progressive, itchy rash on her elbows,
knees, and buttocks for 3 months. Her medical history is unremarkable and she takes no
medications other than vitamins. Physical examination shows grouped vesicles on an
erythematous base with excoriations on the elbows, knees, and buttocks. Serum anti-tissue
transglutaminase antibody assay is positive. A biopsy specimen with direct immunofluorescence
of the lesion shows IgA positivity. This patient is at greatest risk for which of the following
conditions? 
 Correct Answer ImageA.Celiac disease
 Incorrect Answer ImageB.Crohn disease
 Incorrect Answer ImageC.Menetrier disease
 Incorrect Answer ImageD.Ulcerative colitis
 Incorrect Answer ImageE.Varicella-zoster zoster

A 32-year-old man is admitted to the hospital with acute renal failure. He had been well until 3
weeks earlier when he began to have a productive cough with blood-tinged sputum. He has no
past medical history, doesn't use illicit drugs, and does not have a history of cigarette smoking.
His temperature is 37.2ºC (99.0ºF), pulse is 118/min, respirations are 24/min, and blood pressure
is 142/88 mm Hg. Pulmonary examination shows crackles in both lung bases. A chest x-ray
shows bilateral infiltrates. Laboratory studies show:
Hematocrit 28%
Leukocyte count 9,600/mm3
Sodium 145 mEq/L
Chloride 107 mEq/L
Potassium 4.0 mEq/L
Bicarbonate 22 mEq/L
Total protein 7.2 g/dL
Albumin 3.2 mg/dL
Blood urea nitrogen 23 mg/dL
Creatinine 4.5 mg/dL
Phosphorus 4.2 mg/dL
Urinalysis shows many red blood cells, RBC casts, and +3 proteinuria. C3 and C4 levels are
normal. ANA, antistreptolysin O, c-ANCA, and p-ANCA are negative. Which of the following is
the most likely diagnosis? 
 Incorrect Answer ImageA.Acute poststreptococcal glomerulonephritis
 Correct Answer ImageB.Anti-glomerular basement membrane (anti-GBM) disease
(Goodpasture syndrome)
 Incorrect Answer ImageC.Classic polyarteritis nodosa
 Incorrect Answer ImageD.Granulomatosis with polyangiitis
 Incorrect Answer ImageE.Systemic lupus erythematosus

A 58-year-old man is brought to the emergency department by his friend because of confusion
and "seeing double" for the past week. The friend reports that the patient had a seizure 3 days
ago. He is sexually active with men and uses condoms inconsistently. The patient lives in a
homeless shelter. His past medical history is significant for intravenous substance use disorders.
His temperature is 36.6°C (97.9°F), pulse is 80/min, respirations are 22/min, and blood pressure
is 100/60 mm Hg. On physical examination, he is only oriented to person and is uncooperative.
There is no nuchal rigidity or papilledema. There are white plaques on the tongue. Neurologic
examination shows an ataxic gait, and muscle strength is 4/5 in all extremities. CT scan of the
head shows patchy hypodense lesions in the white matter with no edema. His HIV antibody test
is positive. Cerebrospinal fluid (CSF) is positive for JC virus via PCR. Which of the following is
the most appropriate next step in management?
 Incorrect Answer ImageA.Albendazole and methylprednisolone
 Incorrect Answer ImageB.Brain biopsy
 Correct Answer ImageC.Highly-active antiretroviral therapy (HAART)
 Incorrect Answer ImageD.Intravenous methylprednisolone
 Incorrect Answer ImageE.Sulfadiazine and pyrimethamine

A 62-year-old man is brought to the emergency department by his wife after he had some facial
twitching that progressed to generalized, uncontrollable shaking that lasted almost 4 minutes. He
has no history of a seizure disorder. He was recently diagnosed with squamous cell carcinoma of
the lung but has not yet begun treatment. His first chemotherapy session is scheduled to begin in
a few days. He has a history of hypertension treated with hydrochlorothiazide and lisinopril. He
has smoked two packs of cigarettes daily for 45 years. Upon arrival at the emergency
department, the patient is drowsy. He does not follow verbal commands. A CT scan of the head
shows multiple brain metastases with cerebral edema. The patient is started on intravenous
steroids and phenytoin. Which of the following additional treatments is most appropriate for this
patient?
 Incorrect Answer ImageA.Chemotherapy as planned
 Correct Answer ImageB.Cranial irradiation
 Incorrect Answer ImageC.Intravenous carbamazepine
 Incorrect Answer ImageD.Offer no further treatment
 Incorrect Answer ImageE.Surgical removal of the metastatic lesions

A 71-year-old man comes to the emergency department with a 3-day history of malaise,
anorexia, and decreased urinary output. He denies any prior urinary complaints. He denies fever,
nausea, or vomiting. Past medical history is significant for osteoarthritis that is well controlled
with ibuprofen. Recent illnesses include sinusitis for which he was prescribed amoxicillin. He
has been taking the amoxicillin for 3 days and has not noticed any rash or other ill effects from
the medication. His temperature is 36.7ºC (98.0ºF), pulse is 98/min, respirations are 18/min, and
blood pressure is 158/84 mm Hg. He appears fatigued, but not in distress. His lungs are clear and
his heart has a regular rhythm without murmurs. His abdomen is soft, nontender, and without
masses. He has a smooth, small, nontender prostate. Laboratory studies show a BUN of 54
mg/dL, creatinine of 3.5 mg/dL, and potassium of 5.6 mEq/L. Serum liver function studies are
normal. Urinalysis shows 1+ proteinuria, 11-50 white blood cells/hpf with WBC casts, and 6-10
erythrocytes/hpf. There are no bacteria. Urine osmolality is 250 mOsm/L. Which of the
following is the most likely diagnosis?
 Incorrect Answer ImageA.Acute tubular necrosis
 Correct Answer ImageB.Acute tubulointerstitial nephritis
 Incorrect Answer ImageC.Postrenal acute renal failure
 Incorrect Answer ImageD.Prerenal acute renal failure
 Incorrect Answer ImageE.Right nephrolithiasis

A 21-year-old African American woman is brought to the physician because she is "more pale
than usual," lethargic, and having some respiratory problems. Approximately 5 days before this
visit she developed a low-grade fever of 38.2ºC (100.8ºF) along with generalized body aches,
loss of appetite, and lethargy. Her mother also noticed development of a rash that has since
disappeared. The patient denies any substance use and has not been sexually active. Current
medications include folic acid and hydroxyurea. Her temperature is 38.2ºC (100.8ºF), pulse is
100/min, respirations are 22/min, blood pressure is 110/50 mm Hg, and oxygen saturation is 96%
on room air. Physical examination shows a woman who has mildly labored breathing, pale
mucous membranes, cool extremities, and delayed capillary refill. There is no evidence of any
localized tenderness. The rest of her physical examination is unremarkable. Laboratory studies
show: 
Leukocyte count  8,000 cells/mm3
Hemoglobin  5 g/dL
Hematocrit  15%
Platelets 180,000/mm3
Reticulocyte count  0.1%
A peripheral blood smear shows no evidence of any erythroid precursors. Which of the following
is the most likely cause of this woman's symptoms? 
 Incorrect Answer ImageA.Adenovirus
 Incorrect Answer ImageB.Epstein-Barr virus
 Incorrect Answer ImageC.Folate deficiency
 Correct Answer ImageD.Parvovirus
 Incorrect Answer ImageE.Salmonella 
 Incorrect Answer ImageF.Streptococcus pneumoniae 

A 43-year-old woman comes to the physician because of a 4-day history of pain and swelling of
her right knee. She has had no recent trauma. The use of over-the-counter pain medication has
provided minimal relief. Her temperature is 36.7ºC (98ºF). Examination shows a boggy effusion
in the right knee with decreased range of motion. The knee is stable to varus and valgus stress
test. There is no joint line tenderness. The index finger on her right hand is swollen with an
overlying lesion, as shown. Which of the following is the most appropriate next step in
management?
 Correct Answer ImageA.Assess other joints for swelling and tenderness
 Incorrect Answer ImageB.Increase the patient's anti-inflammatory medication dose and
have her return to the clinic in 6 months
 Incorrect Answer ImageC.Order a complete blood count, rheumatoid factor, and
erythrocyte sedimentation rate
 Incorrect Answer ImageD.Order a radiograph of the right knee
 Incorrect Answer ImageE.Perform arthrocentesis for synovial fluid culture and
microscopic analysis

A 65-year-old woman is brought to the emergency department by ambulance because of


shortness of breath. For the past 8 hours, she has experienced dyspnea at rest and has been
coughing up sputum. She denies chest pain. Her medical history is significant for untreated
hyperthyroidism. The patient is in distress and is breathing 60% oxygen through a facemask. She
is unable to complete full sentences. While examining the patient, she coughs up blood tinged
sputum. There is evidence of jugular venous distention and pedal edema. There is no chest wall
tenderness; however, auscultation shows fine crackles throughout the chest. There are no
murmurs or rubs. An electrocardiogram is normal. Which of the following is the most
appropriate next step in management?
 Incorrect Answer ImageA.Dobutamine
 Incorrect Answer ImageB.Dopamine
 Incorrect Answer ImageC.Echocardiography
 Incorrect Answer ImageD.Hydralazine
 Correct Answer ImageE.Intravenous loop diuretic, nitrates, and morphine
 Incorrect Answer ImageF.MUGA scan
 Incorrect Answer ImageG.Synchronized cardioversion
 Incorrect Answer ImageH.Verapamil

A patient comes to his psychiatrist to talk about conflicts he has with his partner. They argue
because the partner complains that the patient is inefficient and procrastinates doing things that
are his responsibility. He never completes tasks in the manner in which he was asked, but rather
in his own way and usually much later. Which of the following defense mechanisms is this
patient exhibiting?
 Incorrect Answer ImageA.Acting out
 Incorrect Answer ImageB.Blocking
 Correct Answer ImageC.Passive-aggressiveness
 Incorrect Answer ImageD.Regression
 Incorrect Answer ImageE.Splitting

A 24-year-old man presents to the office for evaluation of hematuria. He was found to have
microscopic hematuria during a routine insurance physical examination. The patient states that
he has been generally healthy until approximately 1 year ago when he started noticing
diminished vision and decreased hearing. He was diagnosed as having anterior lenticonus and
sensorineural deafness. The patient has a family history of end-stage renal disease. Two of his
maternal uncles have kidney failure and deafness. Physical examination shows blood pressure of
160/100 mm Hg, pulse of 80/min, and respirations of 12/min. He is afebrile. Skin examination
shows no rashes. Lungs are clear to auscultation. Cardiac examination shows normal S1 and S2
sounds with no rubs or gallops. Abdomen is soft, nontender, and nondistended. Extremities show
no pitting edema. 
Urinalysis shows:
Specific gravity 1.010
pH 6.0
Blood 1+
Protein 1+
Ketones Negative
Glucose Negative
RBCs 12–15/HPF
WBCs 0/HPF
Epithelial cells None
Casts None
Crystals None
A few dysmorphic RBCs are seen
Which of the following is the most appropriate initial confirmatory test? 
 Incorrect Answer ImageA.Audiologic assessment
 Incorrect Answer ImageB.CT scan of the head
 Incorrect Answer ImageC.Kidney biopsy
 Incorrect Answer ImageD.Repeat eye examination
 Correct Answer ImageE.Skin biopsy
 Incorrect Answer ImageF.Upper gastrointestinal endoscopy

An 85-year-old man comes to the physician because of sores in his mouth and on his skin that
developed over the last month. The patient states that the skin lesions started as blisters that
ruptured easily. The involved areas are painful, but not pruritic. Physical examination shows
multiple erosions on the oral mucosa and tongue. There are lesions with crusting over the face
and trunk. Several flaccid bullae are noted. Rubbing the skin near an affected area causes
epidermal sloughing from the underlying skin. There are no targetoid lesions. Which of the
following is the most likely diagnosis?
 Incorrect Answer ImageA.Bullous pemphigoid
 Incorrect Answer ImageB.Dermatitis herpetiformis
 Correct Answer ImageC.Pemphigus vulgaris
 Incorrect Answer ImageD.Stevens-Johnson syndrome
 Incorrect Answer ImageE.Toxic epidermal necrolysis

A 77-year-old woman comes to the emergency department because of a 12-hour history of


passing large amounts of bright red blood from her rectum without pain. She has become dizzy
upon standing. She denies any associated nausea, vomiting, diarrhea, constipation, or abdominal
pain. She appears pale. Her temperature is 36.6°C (97.9°F), blood pressure is 112/60 mmHg
supine and 90/56 mmHg sitting upright, and pulse is 110/min. Abdominal examination shows no
abnormalities and elicits no pain. Which of the following is the most likely cause of her
bleeding?
 Incorrect Answer ImageA.Acute bleeding from primary colonic carcinoma
 Correct Answer ImageB.Communication between an arteriole and venule in the cecum
 Incorrect Answer ImageC.Laceration at the anal canal
 Incorrect Answer ImageD.Perforation of a sigmoid diverticulum
 Incorrect Answer ImageE.Rectal polyp

A 67-year-old man comes to the emergency department with high-grade fever and abdominal
and back pain with an inability to urinate for 12 hours. He also reports urinary urgency and
urinary frequency. Over the last 2 years, he has had progressive urinary hesitancy and increased
urine "dribbling." Also, his urine stream has been becoming progressively weaker. He voids
approximately 10 times/day and 3 times/night. His temperature is 39.4ºC (103.0ºF) and pulse is
105/min. His mucous membranes are moist. His lungs are clear. His abdomen is soft and
diffusely tender. His bladder is dull to percussion just below the umbilicus. There is no edema in
his lower extremities. Rectal examination shows a boggy, large, and tender prostate. There is
trace blood in his stool. Laboratory studies show:
Sodium 146 mEq/L
Potassium 5.2 mEq/L
Chloride 100 mEq/L
Bicarbonate 23 mEq/L
Glucose 106 mg/dL
BUN 77 mg/dL
Creatinine 3.3 mg/dL
Which of the following is the most appropriate next step in management? 
 Incorrect Answer ImageA.Emergent transurethral resection of the prostate gland
 Incorrect Answer ImageB.Fluid challenge
 Incorrect Answer ImageC.Furosemide challenge
 Incorrect Answer ImageD.Renal ultrasound
 Correct Answer ImageE.Suprapubic catheterization
 Incorrect Answer ImageF.Urethral catheterization
A 55-year-old man is brought to the emergency department 30 minutes after having a seizure. He
has had a fever and a headache for one day. He has had no known sick contacts. He appears
confused and lethargic. His temperature is 38.5°C (101.3°F), pulse is 115/min, and blood
pressure is 100/70 mm Hg. Examination shows nuchal rigidity. There is no papilledema. Culture
of cerebrospinal fluid grows gram-negative diplococci. In which of the following groups is
vaccination for this patient's condition recommended?
 Incorrect Answer ImageA.Children between the ages of 2 and 10 years
 Incorrect Answer ImageB.Children younger than 2 years
 Incorrect Answer ImageC.Entire community
 Correct Answer ImageD.Military recruits
 Incorrect Answer ImageE.Pregnant women

A 20-year-old woman comes to her physician because of intermittent episodes of sweating and
palpitations for the past 1 month. She reports an unintentional 3 kg (6.6 lbs) weight loss during
this period. She denies mood changes, heat intolerance, and gastrointestinal disturbances. Her
temperature is 38ºC (100.4ºF), pulse is 105/min, respirations are 18/min, and blood pressure is
155/92 mm Hg. On physical examination, she is in mild distress. Physical examination shows
thick, scaly skin on her shins and a marked stare with lid lag. There is a tremor in her upper
extremities. Deep tendon reflexes are brisk. Muscle strength is 4/5 in the proximal upper and
lower extremities. Cardiac examination shows normal heart sounds and an irregular rhythm.
Laboratory studies show:
Thyroid-stimulating hormone < 0.1 mU/L
Anti-thyrotropin receptor antibody Positive
Which of the following physical findings is most specific to the etiology of this patient's
condition? 
 Incorrect Answer ImageA.Hair changes
 Incorrect Answer ImageB.Hyperreflexia
 Incorrect Answer ImageC.Lid lag and stare
 Correct Answer ImageD.Skin changes
 Incorrect Answer ImageE.Tremor and weakness

A 25-year-old patient with asthma is brought to the emergency department because of extreme
shortness of breath. Over the past 6 hours, his wheezing and shortness of breath have become
increasingly severe. He has been compliant with his medications, which include albuterol and
inhaled betamethasone. His blood pressure is 136/84 mm Hg and with deep inspiration falls to
116/56 mm Hg. His pulse is 126/min and respirations are 32/min and labored. There is retraction
of the sternocleidomastoid muscles with inspiration. There are soft wheezes and poor air
movement diffusely throughout both lung fields. His peak expiratory flow rate is 60 L/min, and
he is immediately started on an albuterol nebulizer and given IV methylprednisolone. His arterial
blood gas shows: 
pH 7.55
pCO2 21 mm Hg
pO2 60 mm Hg
Twenty minutes later, a repeat peak flow rate is still 60 L/min. A repeat arterial blood gas
reveals: 
pH 7.46
pCO2 34 mm Hg
pO2 64 mm Hg
Which of the following is the most appropriate next step in management? 
 Incorrect Answer ImageA.Administer 100% oxygen via a tight-fitting face mask
 Incorrect Answer ImageB.Administer IV epinephrine
 Incorrect Answer ImageC.Administer subcutaneous epinephrine
 Incorrect Answer ImageD.Continue albuterol nebulizers until the pH reaches a normal
level
 Correct Answer ImageE.Prepare for intubation

A 70-year-old Caucasian woman is brought to the clinic because of a chronic cough with
associated chest pain. She states that when she coughs or exerts herself she experiences pain with
breathing localized to the left lateral chest, along with a sensation of difficulty catching her
breath. She has also lost 12 pounds in the last 4 months. She has been a housewife since she
married her husband 47 years ago. She and her husband have never smoked, and she endorses
only occasional alcohol consumption. She denies illicit drug use. Her husband worked as a
salesman for a local insurance company. She has not traveled internationally in years. On
examination the patient is a thin and frail elderly woman. Her heart sounds are normal.
Pulmonary auscultation shows decreased breath sounds over the left lower lobe. Dullness to
percussion is also present over the left lower lobe. Chest radiograph shows a 3-cm peripheral
mass at the periphery of the left lower lobe. Which of the following lung malignancies does this
patient most likely have? 
 Correct Answer ImageA.Adenocarcinoma
 Incorrect Answer ImageB.Bronchoalveolar carcinoma
 Incorrect Answer ImageC.Large-cell carcinoma
 Incorrect Answer ImageD.Mesothelioma
 Incorrect Answer ImageE.Small-cell carcinoma
 Incorrect Answer ImageF.Squamous cell carcinoma

A 45-year-old man comes to the physician because of increasing pain in his calves while
walking. The pain is relieved when he rests for a while but recurs a few minutes after he starts
walking again. He does not report having any chest pain or shortness of breath. His past medical
history is significant for diabetes mellitus. Current medications include insulin. His temperature
is 37.2ºC (99.0ºF), pulse is 80/min, respirations are 16/min, and blood pressure is 144/92 mm
Hg. His low-density lipoprotein (LDL) cholesterol level is 138 mg/dL (normal = <160 mg/dL).
The ankle-brachial index (ABI) is 0.5 on the left and 0.6 on the right. Which of the following is
the most appropriate study to help guide the management of this patient’s condition?
 Incorrect Answer ImageA.Angiography of lower extremities
 Incorrect Answer ImageB.C-reactive protein levels
 Incorrect Answer ImageC.Exercise stress test
 Correct Answer ImageD.Hemoglobin A1c levels
 Incorrect Answer ImageE.Lipoprotein (a) levels

A study is designed to understand the association between occupational exposure in radiology


laboratories and a rare skin disease. The employment records from selected laboratories,
including healthcare records, are reviewed. On the basis of specific tasks performed by various
individuals within these laboratories, the employees are divided into those with a high level of
exposure and those with a relatively low level of exposure. The incidence of the disease in these
two groups is then compared. This study presents which one of the following models? 
 Incorrect Answer ImageA.Cross-sectional study
 Incorrect Answer ImageB.Prospective case-control study
 Incorrect Answer ImageC.Prospective cohort study
 Incorrect Answer ImageD.Retrospective case-control study
 Correct Answer ImageE.Retrospective cohort study

A 73-year-old woman comes to the emergency department because of sudden onset of severe
pain in her right eye. The pain started when she was preparing her dinner 2 hours ago. She also
reports blurred vision, nausea, and one episode of vomiting. She does not report any trauma,
foreign body sensation, or ocular discharge. Her past medical history is significant for type 2
diabetes mellitus and seasonal allergies. Current medications include metformin and occasional
loratadine. She does not smoke tobacco, drink alcohol, or use any recreational drugs. Vital signs
are normal. On physical examination, she is distressed. Her right eye is extremely tender, the
cornea is hazy and the pupil is fixed and dilated. The left eye is normal. Which of the following
is the most likely diagnosis?
 Incorrect Answer ImageA.Central retinal artery occlusion (CRAO)
 Incorrect Answer ImageB.Central retinal vein occlusion (CRVO)
 Correct Answer ImageC.Closed-angle glaucoma
 Incorrect Answer ImageD.Keratitis
 Incorrect Answer ImageE.Open-angle glaucoma
 Incorrect Answer ImageF.Optic neuritis
 Incorrect Answer ImageG.Retinal detachment

A 19-year-old man comes to the physician because of a neck lump that he has noticed for 4
weeks. The patient has no systemic symptoms at that time. On examination he has a mobile, firm
2-cm mass in the posterior triangle of the neck. No other masses are palpable, and the remaining
examination is normal. The mass is resected and proves to be a lymph node showing replacement
of normal follicles by sheets of a mixed population of cells, including histiocytes, lymphocytes,
monocytes, plasma cells, and eosinophils. Scattered large binucleated cells are also seen. These
cells are positive for CD15 and CD30 on immunohistochemical staining. Which of the following
is the most likely diagnosis? 
 Incorrect Answer ImageA.Burkitt lymphoma
 Incorrect Answer ImageB.Hairy cell leukemia
 Correct Answer ImageC.Hodgkin disease
 Incorrect Answer ImageD.Mycosis fungoides
 Incorrect Answer ImageE.Reactive lymphadenopathy

A 36-year-old man is brought to the emergency department by a friend because he has had
multiple seizures over the last hour. The friend says he has had 3 episodes in which he abruptly
lost consciousness and all of the muscles of the arms and legs as well as the chest and back
became stiff. Soon afterward he appeared to be in a deep sleep and then gradually woke up.
Three years ago, he was diagnosed with a nonspecific seizure disorder for which he has been
taking phenytoin. Because of the recent loss of a job, he has been unable to pay for his
phenytoin. The patient stops seizing long enough for an IV to be established and a set of vital
signs to be obtained, though he does not return to his baseline mental status. His temperature is
37.0ºC (98.6ºF), blood pressure is 120/80 mm Hg, pulse is 110/min, and respirations are 16/min.
During the physical examination the patient begins to have another episode in which his body
stiffens and begins to jerk and twitch and he loses consciousness.
Which of the following is the most appropriate next step in management? 
 Correct Answer ImageA.Administer lorazepam
 Incorrect Answer ImageB.Administer phenytoin
 Incorrect Answer ImageC.Administer succinylcholine
 Incorrect Answer ImageD.Attempt to place an oropharnygeal airway
 Incorrect Answer ImageE.Endotracheal intubation

A 36-year-old man is brought to the emergency department by a friend because he has had
multiple seizures over the last hour. The friend says he has had 3 episodes in which he abruptly
lost consciousness and all of the muscles of the arms and legs as well as the chest and back
became stiff. Soon afterward he appeared to be in a deep sleep and then gradually woke up.
Three years ago, he was diagnosed with a nonspecific seizure disorder for which he has been
taking phenytoin. Because of the recent loss of a job, he has been unable to pay for his
phenytoin. The patient stops seizing long enough for an IV to be established and a set of vital
signs to be obtained, though he does not return to his baseline mental status. His temperature is
37.0ºC (98.6ºF), blood pressure is 120/80 mm Hg, pulse is 110/min, and respirations are 16/min.
During the physical examination the patient begins to have another episode in which his body
stiffens and begins to jerk and twitch and he loses consciousness.
After repeated doses of intravenous lorazepam, the patient stops seizing. Laboratory studies
show: 
Leukocyte count 12,000 mm3
Hemoglobin  14 g/dL 
Hematocrit  40% 
Platelets  400,000 mm3
pH  7.30 
PaCO2 30 mm Hg 
PaO2 90 mm Hg 
Sodium (Na ) +
140 mEq/L 
Potassium (K+) 3.5 mEq/L 
Chloride (Cl )-
95 mEq/L 
Bicarbonate (HCO3-) 18 mEq/L 
Urea nitrogen  20 mg/dL 
Creatinine  2.0 mg/dL 
Glucose  80 mg/dL 
Lactate  8.6 mmol/L (normal, 0.5–1.0 mmol/L)
Which of the following is the most appropriate next step in the management? 
 Correct Answer ImageA.Aggressive fluid hydration
 Incorrect Answer ImageB.Bicarbonate IV
 Incorrect Answer ImageC.Blood cultures
 Incorrect Answer ImageD.Broad-spectrum antibiotic therapy
 Incorrect Answer ImageE.Dialysis

A 42-year-old woman comes to the physician for a routine health maintenance examination. She
lives at home with her partner, daughter, and cat. She has smoked 8 to 10 cigarettes daily for 10
years. She takes no medications and has no significant past medical history. After discussing her
options, she states that she is willing to attempt smoking cessation with nicotine replacement
therapy. Which of the following is the most appropriate next step in management?
 Incorrect Answer ImageA.Smoking cessation counseling
 Incorrect Answer ImageB.Bupropion
 Incorrect Answer ImageC.Nicotine patch
 Correct Answer ImageD.Nicotine patch and nicotine gum as needed
 Incorrect Answer ImageE.Varenicline

A 31-year-old woman with AIDS comes to the physician because of a 3-month history of watery
diarrhea, low-grade fever, and severe weakness. She has had a 10 kg (22 lb) weight loss over the
past 3 months. Her temperature is 37.8°C (100°F). Abdominal examination shows no
abnormalities. Laboratory studies including stool bacteria, ova, and parasites are repeatedly
negative. Which of the following is the most likely cause of her diarrhea? 
 Correct Answer ImageA.Cryptosporidiosis
 Incorrect Answer ImageB.Cytomegalovirus infection
 Incorrect Answer ImageC.Entamoeba histolytica 
 Incorrect Answer ImageD.Enterotoxigenic Escherichia coli
 Incorrect Answer ImageE.Shigella dysenteriae

A 58-year-old woman comes to the physician for a follow-up examination 2 weeks after
undergoing a barium enema. She denies fever, weight loss, rectal bleeding, or fatigue. She has a
4-year history of hypertension controlled with hydrochlorothiazide. Examination shows no
abnormalities. The barium enema shows four to five polyps scattered throughout her colon,
ranging in size from 4 mm to 1.1 cm. Which of the following is the most appropriate next step in
management? 
 Correct Answer ImageA.Colonoscopy
 Incorrect Answer ImageB.Fecal DNA testing
 Incorrect Answer ImageC.Repeat barium enema in 6 months
 Incorrect Answer ImageD.Screening colonoscopy for her 25-year-old son
 Incorrect Answer ImageE.Upper gastrointestinal barium study

A 27-year-old woman comes to the hospital because of 4 days of joint pain. Initially, she had
diffuse joint pain in her wrists, elbows, and knees. Over the last 24 hours, however, she has had
intense pain and swelling localized to the right knee. She denies any trauma to the area. She
denies any fever or chills but admits to mild dysuria that she attributed to a urinary tract
infection. Her temperature is 38.1ºC (100.6ºF), blood pressure is 120/70 mm Hg, pulse is 98/min,
and respirations are 18/min. Physical examination shows effusion and tenderness of the right
knee. She has scattered pustules on her palms and soles. Arthrocentesis is performed and a Gram
stain of the synovial fluid is shown. Which of the following is the most appropriate next step in
management?
 Incorrect Answer ImageA.IV azithromycin
 Correct Answer ImageB.IV ceftriaxone
 Incorrect Answer ImageC.IV nafcillin
 Incorrect Answer ImageD.IV penicillin
 Incorrect Answer ImageE.IV vancomycin

A 58-year-old man with a history of hypertension and diabetes mellitus is brought to the
emergency department by his friend because of a 1-month history of inability to walk straight,
confusion, and difficulty with memory. The patient is unkempt, which the friend says is very
unlike him. He has smoked two packs of cigarettes daily for 28 years and drinks 15 to 18 beers
daily. He has not had alcohol for the last 2 days. He can give his name, but is unable to name the
date or where he is. His temperature is 36.8°C (98.2 F°), blood pressure is 140/96 mm Hg, pulse
is 78/min, and respirations are 12/min. Eye examination is shown (see media file). There is no
tremor or asterixis. A CT scan of the head shows cerebral atrophy but no bleed or mass. Which
of the following is the most appropriate next step in management? 
 Incorrect Answer ImageA.Administer 50% dextrose
 Incorrect Answer ImageB.Administer intravenous alcohol
 Incorrect Answer ImageC.Administer lorazepam and haloperidol
 Correct Answer ImageD.Administer thiamine and magnesium followed by glucose
infusion
 Incorrect Answer ImageE.Psychiatric consultation

A 48-year-old woman comes to the emergency department after slipping and falling on her
kitchen floor. She has severe right hip pain. She reports a 2-year history of bone pain. Her
medical history is unremarkable and she takes no medications. Palpation of her bones elicits a
dull pain. An x-ray film of the hip shows a fracture of the right femoral neck. Laboratory studies
show: 
White blood cells  8,000/mm3
Hematocrit  37%
Platelets  220,000/mm3
Na+ 140 mEq/L
K+ 2.9 mEq/L
Cl −
117 mEq/L
HCO3− 13 mEq/L
Urea nitrogen  15 mg/dL
Creatinine  0.9 mg/dL
Phosphate  2.8 mg/dL
Calcium  9 mg/dL
Urinary pH 5.3
Fractional excretion of bicarbonate >15%
Which of the following is the most likely cause of this patient's fracture? 
 Incorrect Answer ImageA.Decreased bone density
 Correct Answer ImageB.Decreased bone mineralization
 Incorrect Answer ImageC.Diffuse metastatic disease to bones
 Incorrect Answer ImageD.Gastrointestinal phosphate loss
 Incorrect Answer ImageE.Primary hyperparathyroidism

A previously healthy 37-year-old woman comes to the physician because of recurrent episodes
of double vision and ptosis for the last month. These episodes are unpredictable, though they are
more likely to occur at the end of the day. She also reports occasional hoarseness and difficulty
in swallowing, which also come and go. Vital signs and physical examination are within normal
limits. Which of the following tests is most likely to confirm the diagnosis?
 Incorrect Answer ImageA.Blood, urine, and CSF analyses
 Incorrect Answer ImageB.CT of the chest
 Incorrect Answer ImageC.Edrophonium challenge test
 Correct Answer ImageD.Electrodiagnostic study with repetitive stimulation
 Incorrect Answer ImageE.Muscle biopsy

A 19-year-old college student comes to the physician because of yellowish discharge with
crusting in his eyes since this morning. The patient is very concerned because he has to take an
important exam later today. His personal and medical histories are unremarkable. He does not
smoke tobacco, drink alcohol, or use any recreational drugs. He is sexually active with his
girlfriend and uses condoms. Physical examination shows a moderate amount of mucopurulent
discharge and conjunctival injection bilaterally. His visual acuity is 20/20. Extraocular muscle
function and funduscopic examination are normal. Which of the following is the most
appropriate next step in the management?
 Incorrect Answer ImageA.Artificial tears
 Incorrect Answer ImageB.Dexamethasone drops
 Incorrect Answer ImageC.Emergency ophthalmology referral
 Incorrect Answer ImageD.Erythromycin ophthalmic ointment
 Incorrect Answer ImageE.Naphazoline ophthalmic drops
 Correct Answer ImageF.Sulfa ophthalmic drops
 Incorrect Answer ImageG.Warm compresses
A 42-year-old man comes to the emergency department because of intermittent episodes of
severe crushing chest pain that radiates to the back and the jaw lasting anywhere from a few
seconds to several minutes. The pain is accompanied by dysphagia and is triggered by the
ingestion of certain foods. There is no history of regurgitation of food. Although the problem has
been present for many years, his symptoms have not progressed. Repeated electrocardiograms
and cardiac enzymes have been negative. Barium swallow shows disorganized esophageal
contractions. Manometry shows repetitive simultaneous contractions of the esophageal body and
lower esophageal sphincter with normal pressure and relaxation. Which of the following is the
most likely diagnosis?
 Incorrect Answer ImageA.Achalasia of the esophagus
 Incorrect Answer ImageB.Cancer of the lower esophagus
 Correct Answer ImageC.Diffuse esophageal spasm
 Incorrect Answer ImageD.Nutcracker esophagus
 Incorrect Answer ImageE.Zenker diverticulum

A 45-year-old woman comes to the physician for a follow-up examination. She was bitten on the
left foot by her cat 2 days ago. The cat recently gave birth to a litter and attacked her when she
tried to pick up one of the kittens. She has had progressive swelling and redness of the foot
despite treatment with oral amoxicillin and clavulanic acid. She has had no fever. The patient
reports that the cat is not immunized against rabies. The cat has had no change in her behavior
recently. Her temperature is 36.7ºC (98ºF), blood pressure is 114/82 mm Hg, and pulse is
82/min. Examination of the left foot shows a 3 cm area of erythema and edema surrounding 4
clean bite marks. There is no fluctuance. The wound is cleaned and irrigated, and IV antibiotics
are started. The patient's last tetanus vaccine was 7 years ago. Which of the following is the most
appropriate next step in management?
 Incorrect Answer ImageA.Rabies human diploid cell vaccine
 Incorrect Answer ImageB.Rabies immune globulin
 Incorrect Answer ImageC.Rabies immune globulin and vaccine
 Correct Answer ImageD.Tetanus toxoid and observation of the cat for 10 days
 Incorrect Answer ImageE.Observation of the patient in the hospital

A 45-year-old woman is admitted to the hospital because of severe abdominal pain, nausea, and
vomiting for the past 5 days. She has had a decreased appetite, fever, and chills. She has a history
of acute pancreatitis that resolved 5 weeks ago. She takes no medications. She drinks 10 to 15
beers daily. Her temperature is 39.3ºC (102.7ºF), pulse is 135/min, respirations are 20/min, and
blood pressure 125/70 mm Hg. Abdominal examination shows epigastric tenderness, with no
rebound or guarding. Laboratory studies show a leukocyte count of 13,500/mm3 and an amylase
level of 190 U/L. Tests of liver function are normal. CT scan of the abdomen is shown. Which of
the following is the most appropriate management?
 Incorrect Answer ImageA.Endoscopic retrograde cholangiopancreatography (ERCP)
 Incorrect Answer ImageB.Fluids and antibiotics
 Correct Answer ImageC.Percutaneous drainage
 Incorrect Answer ImageD.Supportive care only
 Incorrect Answer ImageE.Surgical excision
A 36-year-old woman comes to the physician because of a 3-week history of fatigue, malaise,
and low-grade fever and chills. She also reports increasing anorexia and loss of energy. At times,
she is unable to rise out of bed because of generalized muscle fatigue and arthralgias. She was in
her usual state of health before the development of these symptoms. She recently had four
wisdom teeth extracted secondary to impaction. She has not seen a doctor since she was a
teenager, but was told then she had a heart murmur. She does not take any medications and
denies any substance use. Her temperature is 38.9ºC (102ºF), pulse is 125/min, blood pressure is
101/45 mm Hg, and respirations are 22/min. Physical examination shows a thin, fatigued, pale
woman with dry mucous membranes and petechial spots on her mouth and conjunctiva.
Cardiovascular examination shows a grade 3/6 harsh, pansystolic, crescendo-decrescendo
murmur at the left fifth intercostal space in the midclavicular line. There are small linear spots on
her fingernails and painless dark nodules on her palms and soles. There are painful nodules on
her fingers and toes. Funduscopic examination shows retinal hemorrhages with central pallor.
Which of the following is the most appropriate next step in management?
 Incorrect Answer ImageA.Administer intravenous antibiotics with corticosteroids
 Incorrect Answer ImageB.Call for a cardiology consultation
 Incorrect Answer ImageC.Obtain a CT scan of chest, abdomen, and pelvis
 Correct Answer ImageD.Obtain blood cultures and administer intravenous antibiotics
 Incorrect Answer ImageE.Transthoracic echocardiography

A 35-year-old man with end-stage renal disease comes to the emergency department after an
episode of syncope in a grocery store. He reports that he skipped his last hemodialysis session,
and has been consuming considerable amounts of citrus fruits, for which he was shopping when
he collapsed. He denies chest pain or dyspnea. His blood pressure is 160/95 mm Hg.
Examination shows a left forearm AV fistula. Laboratory studies show:
Potassium 8.8 mEq/L
Blood urea nitrogen 88 mg/dL
Creatinine 14 mg/dL
Bicarbonate 16 mmol/L
An ECG shows sinus bradycardia with a sine wave pattern. Which of the following is the most
appropriate next step in management?
 Correct Answer ImageA.IV calcium gluconate
 Incorrect Answer ImageB.IV dextrose and insulin
 Incorrect Answer ImageC.IV sodium bicarbonate
 Incorrect Answer ImageD.Nebulized inhaled albuterol
 Incorrect Answer ImageE.Oral potassium binder (patiromer or sodium zirconium
cyclosilicate)

A 24-year-old woman comes to the clinic complaining of a rash. For the past 2 weeks, she has
had small red, non-blanching, non-pruritic spots on her arms and legs. Additionally, she has
suffered from easy bruising and occasional nose bleeds. She has never had such symptoms
before and has been healthy all her life. She takes no medications aside from an oral
contraceptive pill and denies the use of any over-the-counter supplements. She does not drink
alcohol nor smoke cigarettes, and her family history is unremarkable. A review of systems is
within normal limits. Physical examination reveals a young, generally healthy-appearing woman
in no apparent distress. Examination of the woman's skin is remarkable for numerous bruises and
petechiae on her arms and legs and a small amount of dried blood on her gums. Laboratory
studies show:
Hgb  12.5 g/dL
Leukocyte count  6,000/mm3 
Platelets  20,000/mm3 
aPTT  28 seconds
PT  13 seconds
Peripheral blood smear, basic chemistries and electrolytes, and tests of liver function are
unremarkable. What findings would be expected on a bone marrow biopsy? 
 Incorrect Answer ImageA.Hypercellularity and fibrosis
 Correct Answer ImageB.Increased platelet production (megakaryocytes)
 Incorrect Answer ImageC.Myeloid cell hypercellularity
 Incorrect Answer ImageD.Normal findings
 Incorrect Answer ImageE.Proliferation of lymphocytic precursors

A 41-year-old man is brought to the emergency department by paramedics after being found
conscious at the scene of a motor vehicle accident. He had been the passenger in a high-speed
collision and was thrown a distance of 20 feet through the front windshield. The evaluation
shows fractures in both femurs, the pelvis, left tibia, and left humerus. The patient undergoes
surgical repair of the fractures. He is admitted to the intensive care unit post-surgery. Twenty-
four hours later he becomes confused and markedly dyspneic. He is administered 100% oxygen
via face mask. Arterial blood gas shows pH 7.45, pCO2 30 mm Hg, and PO2 54 mm Hg.
Numerous petechiae are noted on the axilla, chest wall, head and neck. No clubbing, cyanosis, or
edema is present. Which of the following is the most likely cause of this man's clinical
condition? 
 Incorrect Answer ImageA.Aspiration pneumonia
 Incorrect Answer ImageB.Cardiogenic pulmonary edema
 Correct Answer ImageC.Fat embolism
 Incorrect Answer ImageD.Pulmonary embolism
 Incorrect Answer ImageE.Tension pneumothorax

A 69-year-old woman comes to the physician because of nausea, dizziness, shortness of breath,
somnolence, and fatigue for 4 days. She denies any emesis or fevers. A week before her
presentation, she was diagnosed with a non-ST elevation myocardial infarction and received a
coronary angiogram, but no intervention was done. She has hypertension, congestive heart
failure, coronary artery disease, atrial fibrillation, and emphysema. Her medications include
atorvastatin, digoxin, lisinopril, carvedilol, and aspirin. She used to smoke one pack of cigarettes
per day, but quit 10 years ago. Her blood pressure is 88/46 mm Hg, pulse is 48/min, and
respirations are 18/min. She appears uncomfortable and confused. On vision examination, she
confuses green for yellow. Her lung examination shows a prolonged expiratory phase with mild
crackles at the bases, and both lower extremities show 1+ pitting edema. Chest x-ray shows an
enlarged cardiac silhouette, flattened diaphragms, and mixed opacities. An ECG shows
bradycardia and scooped ST segments. Laboratory studies show:
Leukocyte count 6,300/mm3
Hematocrit 45%
Platelets 225,000/mm3
Sodium 137 mEq/L
Potassium 4.3 mEq/L
Chloride 103 mEq/L
Bicarbonate 21 mEq/L
Glucose 122 mg/dL
Urea nitrogen 30 mg/dL
Creatinine:  2.2 mg/dL
Which of the following is the most appropriate next step in management?
 Incorrect Answer ImageA.Correct potassium levels
 Correct Answer ImageB.Digoxin-specific antibody fragments (digoxin immune Fab)
 Incorrect Answer ImageC.Glucagon
 Incorrect Answer ImageD.Synchronized cardioversion
 Incorrect Answer ImageE.Transcutaneous pacemaker

A 63-year-old man comes to the emergency department because of pain in his left leg after
slipping on his kitchen floor. He heard a "snap" when he hit the ground. He denies recent weight
loss. He has a history of diabetes and hypertension. Current medications include metformin and
lisinopril. His temperature is 36.9°C (98.4°F) and blood pressure is 130/80 mm Hg. Examination
shows decreased hearing in both ears. A 2/6 mid-systolic crescendo-decrescendo murmur at the
second right intercostal space is heard on auscultation. There is extreme tenderness to palpation
of the left thigh and there is mild tenderness to palpation of the contralateral thigh. A radiograph
of the left femur shows a mid-shaft fracture and an expanded, deformed cortical contour. A
radiograph of the right femur shows a bowed femur. A bone scan shows focal areas of intense
uptake in both femurs. Laboratory studies show:
Na+ 140
mEq/L
K+ 4.8
mEq /
L
Cl −
105
mEq/L
HCO3− 24
mEq/L
Urea Nitrogen  13
mg/dL
Creatinine  1.0
mg/dL
Phosphate 2.9
mg/dL
Ca2+ 9.9
mg/dL
AST  32
IU/L
ALT 25
IU/L
GGT  15
IU/L
(norm
al, 9-
50
IU/L)
ESR  18
mm/h
In addition to a referral to orthopedic surgery, which of the following is the most appropriate
pharmacotherapy?
 Correct Answer ImageA.Alendronate
 Incorrect Answer ImageB.Autologous bone marrow transplantation
 Incorrect Answer ImageC.Calcium supplementation
 Incorrect Answer ImageD.Doxorubicin
 Incorrect Answer ImageE.High-dose corticosteroids

A 50-year-old man comes to the physician for a routine health maintenance examination. He has
a history of hypertension. He exercises 4 times weekly. He smoked one pack of cigarettes daily
for 10 years but quit 20 years ago. His mother has type 2 diabetes mellitus and his father died of
a myocardial infarction at age 72 years. He weighs 72.5 kg (160 lb) and is 178 cm (70 in) tall.
His temperature is 37ºC (98.6ºF), pulse is 68/min, and blood pressure is 138/88 mm Hg. Physical
examination shows no abnormalities. Laboratory studies show: 
Alanine aminotransferase (ALT) 19 U/L
Alkaline phosphatase 450 U/L
Aspartate aminotransferase (AST) 20 U/L
Bilirubin, total 1 mg/dL
Which of the following is the most appropriate next step in management? 
 Incorrect Answer ImageA.Fractionated bilirubin level (direct and indirect)
 Correct Answer ImageB.Gamma-glutamyl transpeptidase level
 Incorrect Answer ImageC.Percutaneous liver biopsy
 Incorrect Answer ImageD.Repeat liver panel in 6 months
 Incorrect Answer ImageE.Right upper quadrant ultrasound

A 12-year-old boy is brought to the clinic for evaluation of headaches. For the last month, he has
had daily, throbbing pain in the frontal region that is worse in the morning. Occasionally, the
pain is associated with nausea and vomiting. His mother states that he has been getting bad
grades in school and he attributes it to diminished vision. Past medical and birth histories are
unremarkable. He does not take any medications. Vital signs are normal. Physical examination is
remarkable for decreased visual acuity, peripheral visual field defects, and a left-sided sixth-
nerve palsy. Fundus examination shows papilledema bilaterally. There are no other neurologic
deficits. The remainder of the physical examination is normal. MRI of the head is normal. Which
of the following would be the most appropriate next step in the management of this patient's
condition?
 Incorrect Answer ImageA.Acetazolamide
 Incorrect Answer ImageB.Corticosteroids
 Correct Answer ImageC.Lumbar puncture
 Incorrect Answer ImageD.No treatment
 Incorrect Answer ImageE.Sumatriptan

A 61-year-old white man comes to the clinic for a physical examination that is required before he
can purchase life insurance. He is generally healthy and has no complaints. He has no chronic
medical conditions that he is aware of. He exercises sporadically, does not smoke cigarettes, and
drinks four to six glasses of wine per week. He is married and has a 9-year-old son who was
recently hospitalized with splenomegaly and a parvovirus infection. His father suffered from
chronic anemia and died at age 25 years during a "surgery to remove an abdominal organ." His
mother is alive and well. His pulse is 70/min and his blood pressure is 125/90 mm Hg. Physical
examination is unremarkable. Laboratory studies show a hemoglobin of 12 g/dL, a hematocrit of
32%, a mean corpuscular hemoglobin concentration of 39% Hb/cell, and a mean corpuscular
volume of 88 fl. Reticulocyte count is 4.2% of red blood cells, and a Coombs test is negative.
The red blood cells have increased sensitivity to lysis in a hypotonic solution. At this time, which
of the following is the most appropriate management? 
 Correct Answer ImageA.Folic acid
 Incorrect Answer ImageB.Hydroxyurea
 Incorrect Answer ImageC.Iron
 Incorrect Answer ImageD.Prednisone
 Incorrect Answer ImageE.Splenectomy

A 59-year-old woman had a diverticulectomy. During the postoperative observation period, the
resident notices that the patient is nauseated, restless, sweating, and she seems confused,
disoriented, and delirious. On review of her chart, the intern notices that the woman has a past
history of hypertension, diabetes mellitus, anxiety, gout, and pelvic inflammatory disease. Her
medications include hydrochlorothiazide, metformin, alprazolam, and allopurinol. Her father has
bipolar disorder and is on lithium. She is hepatitis B positive. She is a smoker, and drinks only
socially. Which of the following would be the most appropriate next step in management?
 Incorrect Answer ImageA.Dialysis to wash out lithium
 Incorrect Answer ImageB.Start dantrolene
 Correct Answer ImageC.Start lorazepam
 Incorrect Answer ImageD.Start naloxone
 Incorrect Answer ImageE.Start promethazine
A 58-year-old man comes to the emergency department via ambulance because of jaw pain. Two
hours previously, he had been with his family when he suddenly felt a dull pain in his jaw. He
assumed he had a toothache and took two ibuprofen tablets. His son became very concerned
about his gray and sweaty complexion and asked him to drink some milk because he thought his
father was having a hypoglycemic episode, which occurs frequently. When the patient started
becoming increasingly confused, the son called 911. His medical history is significant for poorly
controlled type 1 diabetes mellitus, previous myocardial infarction, and left bundle branch block.
His temperature is 37.5ºC (100ºF), pulse is 108/min, respirations are 23/min and blood pressure
80/60 mm Hg. Oxygen saturation is 91% in room air and he is placed on 100% oxygen via a
facemask. The patient appears pale, anxious, and sweaty and he is not oriented to time, person,
or place. There is jugular venous distension. Cardiovascular examination shows muffled heart
sounds with no murmurs or rubs. Peripheral pulse is thready. Auscultation of the chest shows
end-inspiratory crackles and wheezes. Pitting edema of the lower extremities is present with
cold, clammy skin. The patient's blood glucose level is 130 mg/dL. Which of the following is the
next best step in management?
 Incorrect Answer ImageA.Coronary angiography
 Incorrect Answer ImageB.Coronary artery bypass grafting (CABG)
 Correct Answer ImageC.Echocardiogram
 Incorrect Answer ImageD.Electrocardiogram
 Incorrect Answer ImageE.Give furosemide
 Incorrect Answer ImageF.Give insulin
 Incorrect Answer ImageG.Percutaneous coronary intervention (PCI)

A 42-year-old man comes to the emergency department because of a painful lesion at the back of
his neck for the past week. The lesion has been draining pus for the past day. His past medical
history is unremarkable. He does not smoke but drinks a bottle of wine daily. He works as a
mechanic and is frequently exposed to dirt and machine oils. Physical examination shows an
erythematous, edematous, 8 cm, indurated plaque at the nape of his neck with multiple necrotic,
suppurating follicular ostia within it. The plaque is ill-defined and blends into the surrounding
skin. There is induration palpable beyond the erythematous edge. Which of the following is the
most likely diagnosis?
 Correct Answer ImageA.Carbuncle
 Incorrect Answer ImageB.Epidermal cyst
 Incorrect Answer ImageC.Folliculitis
 Incorrect Answer ImageD.Herpes simplex
 Incorrect Answer ImageE.Herpes zoster

A 64-year-old man comes to the emergency department because of an inability to void for 16
hours. He has severe suprapubic pain and discomfort. Initial assessment shows a temperature of
37.6°C (99.6°F), blood pressure of 160/90 mm Hg, pulse of 105/min, and respirations of 28/min.
On physical examination, the physician palpates a soft mass in the suprapubic region. A 16-
French urinary catheter is placed with some difficulty and 1,000 mL of clear urine is drained.
The suprapubic mass resolves and his vital signs improve. The patient states that he is currently
being treated for benign prostatic hyperplasia (BPH), hypertension, and gout. He denies allergies
and previous surgeries. His current medications include tamsulosin, hydrochlorothiazide,
atenolol, and allopurinol. Social and family histories are otherwise negative, as is his review of
systems. His creatinine is 1.9 mg/dL, and a renal ultrasound shows mild hydronephrosis
bilaterally. Which of the following is the best option for him at this time?
 Incorrect Answer ImageA.Admit to the hospital and renal consultation
 Incorrect Answer ImageB.CT urogram
 Correct Answer ImageC.Keep the urinary catheter in place and treat with a 5α-reductase
inhibitor

 Incorrect Answer ImageD.Remove the urinary catheter and increase tamsulosin dose
 Incorrect Answer ImageE.Transurethral microwave therapy (TUMT)

An 18-year-old girl is brought to the clinic by her mother because she periodically pulls her hair
and puts it in her mouth. She has several bald patches on both sides of her head. Initially the
mother thought the girl had some sort of infection but states the dermatologist "did not find
anything." The patient describes a sense of relief when she pulls out her hair and states that she
mainly does it when upset, anxious, or overwhelmed. She admits that she has been doing it on
and off for at least 2 years and thinks that it started after her parents got divorced. Which of the
following is the most appropriate initial step in management?
 Correct Answer ImageA.Identify the presence of comorbid conditions
 Incorrect Answer ImageB.Start interpersonal psychotherapy
 Incorrect Answer ImageC.Start treatment with a benzodiazepine
 Incorrect Answer ImageD.Start treatment with carbamazepine
 Incorrect Answer ImageE.Start treatment with a tricyclic antidepressant

An HIV-positive patient comes to the office because of painless leg weakness, stiffness, and
imbalance and incontinence for the past few months. He was diagnosed with AIDS 6 years ago.
Medications include an antiretroviral regimen and oral acetaminophen for tension headaches. His
vital signs are normal. Physical examination reveals reduced strength in the lower extremities
with accompanying mild spasticity and hyperreflexia. There is diminished sensation in the feet
and legs bilaterally. Lumbar puncture shows:
Opening pressure 100 mm H20 
Cell count 5 lymphocytes/mm3
Glucose 48 mg/dL
Proteins (total) 33 mg/dL
Gamma globulin 8% total protein
Additional laboratory investigations show a CD4 cell count of 50 cells/mm3, Vitamin B12 level is
normal, and the serologic test for syphilis is negative. MRI of the head does not reveal any focal
abnormality. Which of the following is the most likely diagnosis?
 Incorrect Answer ImageA.AIDS dementia complex
 Incorrect Answer ImageB.CMV polyradiculopathy
 Incorrect Answer ImageC.Cryptococcal meningoencephalitis
 Correct Answer ImageD.Vacuolar (HIV) myelopathy
 Incorrect Answer ImageE.Zidovudine-related toxicity
A 70-year-old pediatrician develops a sudden onset headache, slurring of speech, and confusion
while eating breakfast. He is brought to the emergency department, where he arrives with a
Glasgow coma score of 8. The patient's history is negative for hypertension or vascular risk
factors. His blood pressure is 140/85 mm Hg. An emergency CT scan of the head shows well-
demarcated lobar bleeding involving the left frontal lobe, centered in the cortex and extending
into the underlying white matter. MRI shows multiple small foci of prior hemorrhage throughout
other cortical portions of the cerebrum and cerebellum. Which of the following is the most
probable underlying cause of the patient's bleeding?
 Incorrect Answer ImageA.Berry aneurysm rupture
 Incorrect Answer ImageB.Bridging vein tears
 Correct Answer ImageC.Cerebral amyloid angiopathy
 Incorrect Answer ImageD.Middle meningeal artery laceration
 Incorrect Answer ImageE.Undiagnosed hypertension

A 24-year-old man is brought to the emergency department because of a painful rash that started
this morning. It began on his face initially, and since coming to the ED, has spread down to his
chest and arms. He had a furuncle on his right leg 1 week ago treated with trimethoprim-
sulfamethoxazole. His temperature is 38.9°C (102°F), blood pressure is 124/90 mm Hg, and
respirations are 13/min. There are hemorrhagic erosions with crusting on the lips, tongue, and
buccal mucosa. The conjunctivae show erythematous injection. Skin examination shows bright
erythematous bullae and sloughing with mild traction on the upper chest and proximal
extremities, covering approximately 8% of his total body surface area. Which of the following is
the most likely diagnosis?
 Incorrect Answer ImageA.Drug rash with eosinophilia and systemic symptoms (DRESS)
 Incorrect Answer ImageB.Erythema multiforme minor
 Incorrect Answer ImageC.Serum sickness-like reaction
 Correct Answer ImageD.Stevens-Johnson syndrome
 Incorrect Answer ImageE.Toxic epidermal necrolysis

A 36-year-old woman comes to the physician because of a painful neck mass and fever for 4
days. She has had irritability and muscle pain in the upper extremities. She had an upper
respiratory tract infection 1 week ago. Her medical history is unremarkable, and current
medications include oral contraceptive pills. Her temperature is 39ºC (101.4ºF), blood pressure
115/78 mm Hg, and pulse 116/min. The mass is in the midline and tender. Ocular examination
shows no abnormalities. Which of the following is the most likely diagnosis? 
 Incorrect Answer ImageA.Branchial cleft cyst
 Incorrect Answer ImageB.Hashimoto thyroiditis
 Incorrect Answer ImageC.Peritonsillar abscess
 Correct Answer ImageD.Subacute thyroiditis
 Incorrect Answer ImageE.Thyroglossal duct cyst infection
A 62-year-old woman comes to her physician for scheduled chemotherapy administration for
breast cancer metastatic to brain and bones. She complains of fatigue and mild headaches of
several months duration. She does not take diuretics. Her vital signs and physical examination
are normal except for well healed biopsy scars on the right breast and axilla. In particular, there
are no signs of hypovolemia (orthostatic hypotension or tachycardia); hypervolemia (edema,
ascites); heart failure (raised jugular venous pressure, lung crackles, S3 gallop); or cirrhosis
(hepatosplenomegaly, spider angiomas, palmar erythema). Serum studies show:
Sodium 122 mEq/L
Potassium 4.0 mEq/L
Chloride 88 mEq/L
Bicarbonate 23 mEq/L
Glucose 126 mg/dL
BUN 16 mg/dL
Creatinine 1.1 mg/dL
Osmolality 256 mOsm/kg
Uric acid 2.2 mg/dL
Serum values of thyroid stimulating hormone, cortisol (at 8:00 AM), albumin, and liver tests are
normal. Urinalysis is negative for protein. Additional urine studies show: osmolality 556
mOsm/kg; sodium 50 meq/L, potassium 50 meq/L. Which of the following is the best next step
in management of this patient?
 Incorrect Answer ImageA.3% saline
 Correct Answer ImageB.Dietary water restriction
 Incorrect Answer ImageC.Furosemide
 Incorrect Answer ImageD.IV normal saline
 Incorrect Answer ImageE.Tolvaptan

A 48-year-old man with cirrhosis complicated by esophageal varices and ascites is brought to the
emergency department by his spouse because of fever, dull diffuse abdominal pain, diarrhea, and
malaise. He seems mildly disoriented and is easily confused. His current symptoms began
approximately 2 weeks ago. Review of systems is normal. His spouse states that he is normally
more alert and aware. His temperature is 38.5ºC (101.3ºF), blood pressure is 148/88 mm Hg, and
pulse is 89/min. Physical examination shows a large, tense abdomen with shifting dullness.
There is mild, diffuse tenderness to palpation without guarding or rebound tenderness. The
remainder of the physical examination is normal. Complete blood count shows leukocytosis.
Blood cultures and urine cultures are sent to the laboratory and results are pending. Which of the
following is the most appropriate next step in management? 
 Correct Answer ImageA.Abdominal paracentesis

 Incorrect Answer ImageB.CT scan of abdomen

 Incorrect Answer ImageC.Endoscopy


 Incorrect Answer ImageD.Lactulose
 Incorrect Answer ImageE.Upright x-ray of the abdomen
A 36-year-old Caucasian woman comes to the emergency department becaue of a 3-day history
of acute onset of blurry vision and left-eye pain. The pain is worse with movement of the eye.
She has had no fever, history of eye trauma, or excessive lacrimation. Her medical history is
unremarkable, and she takes no medications. Her temperature is 37.1°C (98.8°F). The
examination shows her visual acuity to be 20/20 in the right eye and 20/80 in the left eye. The
left eye shows a central scotoma on visual field testing and a loss of color vision. The right eye
visual field and color vision are within normal limits. The pupils are equal in size, but there is a
slow and diminished response to light in the left eye. On fundus examination, the optic disc is
swollen, with an adjacent flame-shaped hemorrhage. Which of the following is the most
appropriate first diagnostic test?
 Incorrect Answer ImageA.Anti-Smith antibody
 Incorrect Answer ImageB.Cerebral angiography
 Incorrect Answer ImageC.Computerized tomography of the orbit
 Incorrect Answer ImageD.Computerized tomography of the sinuses
 Correct Answer ImageE.Magnetic resonance imaging of the brain

A 25-year-old man of Eastern European Jewish descent is being evaluated for a right hip fracture
and splenomegaly. He reports recurrent acute episodes of bone pain for the past 5 years. He
works as an engineer, is married, and does not smoke or drink alcohol. 
Blood chemistry:
Hemoglobin 9.0 g/dL
Leukocyte count 3,300/mm3
Platelet count 60,000/mm3
Radiographs demonstrate multiple osteolytic lesions in the vertebral column and femurs. A bone
marrow aspirate reveals clusters of histiocytes showing fibrillar cytoplasm with a finely and
uniformly vacuolated appearance. The fibrillar material in the cytoplasm is PAS-positive. Which
of the following is the most effective treatment available for this condition? 
 Incorrect Answer ImageA.Administration of hematin
 Incorrect Answer ImageB.Administration of pyridoxine and folate
 Incorrect Answer ImageC.Allogeneic bone marrow transplantation
 Incorrect Answer ImageD.Cytotoxic drugs such as hydroxyurea
 Incorrect Answer ImageE.High-carbohydrate diet
 Correct Answer ImageF.Replacement therapy with imiglucerase

A 35-year-old man comes to the physician because of a 5-day history of cough, fever, malaise,
and muscle pains. He has had no sore throat, rhinorrhea, or sneezing. His medical history is
unremarkable and he takes no medications. He does not smoke or use illicit drugs. He returned
from a trip exploring caves 2 weeks ago. His temperature is 38.5ºC (101.3ºF), blood pressure is
124/62 mm Hg, pulse is 110/min, and respirations are 24/min. Decreased breath sounds
bilaterally and occasional crackles are heard on auscultation. Cardiac examination shows normal
S1 and S2; no murmurs are heard. The remainder of the physical examination shows no
abnormalities. A radiograph of the chest is shown. Which of the following is the most likely
diagnosis?
 Incorrect Answer ImageA.Aspergillosis
 Incorrect Answer ImageB.Blastomycosis
 Incorrect Answer ImageC.Coccidioidomycosis
 Correct Answer ImageD.Histoplasmosis
 Incorrect Answer ImageE.Sarcoidosis

A 56-year-old man is admitted to the hospital with right-sided scrotal pain and swelling for the
last 3 days. He had a low-grade fever last night, and today his pain became worse. He does not
report any trauma, dysuria, hematuria or urinary incontinence. He denies any similar prior
episodes of scrotal pain. His medical history is significant for insulin-dependent diabetes and
hypertension. He does not smoke or drink. At the time of admission, ultrasound showed right
testicular hyperemia and enlargement as compared to the left testicle. No fluid collection is
appreciated in the testicle. Leukocyte count on admission is 22,000/mm3. He is started on IV
ciprofloxacin, scrotal elevation, and ice packs. After 72 hours in the hospital, he continues to
have fever and a leukocyte count of 18,500/mm3, and his examination is essentially unchanged.
There is persistent right hemiscrotum edema with scrotal wall erythema. The testicle remains
very firm and tender to palpation. Repeat ultrasound shows increased regional blood flow to the
right epididymis, thickened edematous scrotal wall, and a new complex, loculated fluid
collection within the right hemiscrotum. Which of the following is the most appropriate
management? 
 Incorrect Answer ImageA.Aspirate fluid collection
 Incorrect Answer ImageB.Change antibiotics for broader coverage
 Incorrect Answer ImageC.Continue with current management
 Incorrect Answer ImageD.Incision and debridement of scrotal wall necrotic tissue
 Correct Answer ImageE.Surgical drainage

A 44-year-old man with a 2-year history of a rash on his scalp, face, and chest comes to the
physician for a follow-up examination. The rash has persisted despite treatment with topical
ketoconazole, topical corticosteroids, and crude coal-tar shampoo. The rash is not painful but has
been itchy on occasion. Current medications include topical triamcinolone cream. He is a veteran
who fought in the Gulf War. He has smoked one pack of cigarettes daily for 25 years. Physical
examination shows erythema with a greasy scale on the scalp, ears, midface, and central chest.
Which of the following complicating factors is most likely to explain the treatment resistance of
this patient's disease?
 Incorrect Answer ImageA.Agent Orange exposure
 Incorrect Answer ImageB.Coal-tar carcinogenesis
 Correct Answer ImageC.HIV seropositivity
 Incorrect Answer ImageD.Pityrosporum colonization
 Incorrect Answer ImageE.Staphylococcal superinfection

A 25-year-old accountant with a history of asthma is brought to the emergency department from
her office in severe respiratory distress. A coworker reports that the patient had a cold that day
and inadvertently inhaled chemical fumes from a nearby construction area on their floor after
which time she was unable to breathe. Her respiratory rate is 44/min, pulse is 78/min, and
oxygen saturation is 97% on room air. In the emergency department, she is unable to speak in
full sentences and is markedly diaphoretic. She is using accessory muscles of respiration and
there are suprasternal retractions. Pulmonary examination shows bilateral wheezes. She is given
two albuterol nebulizer treatments and is started on IV corticosteroids. She has never been
intubated in the past. She has had asthma symptoms for about 1 week and has never had
nocturnal symptoms. Arterial blood gas after two treatments shows:
pH 7.39 mm Hg
pCO2 42 mm Hg
pO2 78 mm Hg
After treatment, she is now able to speak a full sentence. Her respiratory rate is 30/min, pulse is
97/min, and oxygen saturation is 98%. Which of the following is the most appropriate next step
in management? 
 Correct Answer ImageA.Albuterol
 Incorrect Answer ImageB.Elective intubation
 Incorrect Answer ImageC.Epinephrine
 Incorrect Answer ImageD.Inhaled fluticasone and salmeterol
 Incorrect Answer ImageE.Salmeterol

A 45-year-old woman comes to her physician because of a 3-week history of generalized


weakness and a “pins and needles” sensation in her lower extremities. She reports feeling
exhausted and having low energy throughout the day. She denies mood changes, back pain, and
urinary and bowel incontinence. She has been a strict vegan since adolescence. She exercises
daily and drinks alcohol occasionally. Her temperature is 37ºC (98.6ºF), pulse is 60/min,
respirations are 18/min, and blood pressure is 110/70 mm Hg. Physical examination shows an
ataxic gait. Lower extremity examination shows spastic weakness of the proximal and distal
muscles of the lower extremities. Deep tendon reflexes are brisk. Which of the following is the
most likely diagnosis?
 Incorrect Answer ImageA.Guillain-Barré syndrome
 Incorrect Answer ImageB.Lambert-Eaton syndrome
 Incorrect Answer ImageC.Myasthenia gravis
 Incorrect Answer ImageD.Polymyositis
 Correct Answer ImageE.Subacute combined degeneration

A 16-year-old boy comes to the emergency department because of sudden onset right lower
quadrant abdominal pain. He has tenderness at McBurney’s point and is diagnosed with acute
appendicitis. He undergoes an appendectomy under general anesthesia. Postoperatively he
develops a large abdominal wall bruise. His family members are very concerned about the bruise
and request further evaluation. His parents report that he has had a history of easy bruising all of
his life. At age 6 he had hemarthrosis in his left knee and blood was aspirated from the joint at
that time. The patient's maternal uncle died when only a few months old due to an intracranial
bleed. Laboratory work shows a normal platelet count, normal PT, and prolonged PTT. Which of
the following is the most appropriate next step in diagnosis? 
 Correct Answer ImageA.1:1 Mixing studies
 Incorrect Answer ImageB.Check blood warfarin level

 Incorrect Answer ImageC.Check factor V Leiden

 Incorrect Answer ImageD.Check protein C level

 Incorrect Answer ImageE.Check protein S level

 Incorrect Answer ImageF.Repeat platelet count

A 40-year-old man with symptomatic bradycardia is admitted to the hospital for scheduled
surgical placement of a pacemaker. His medical history is otherwise unremarkable and he takes
no medications. Ten hours after the procedure, he develops shortness of breath. His blood
pressure is 65/40 mm Hg, pulse is 60/min, and respirations are 38/min. There is distension of his
jugular veins, and his trachea is deviated to the right. Breath sounds are decreased on the left.
Cardiac examination shows a normal S1 and S2; no murmurs are heard. ECG shows a paced
rhythm. Which of the following is the most appropriate next step in management?
 Incorrect Answer ImageA.Diuresis with IV furosemide
 Incorrect Answer ImageB.Intubate the patient
 Correct Answer ImageC.Place a chest tube on the left side
 Incorrect Answer ImageD.Place a chest tube on the right side
 Incorrect Answer ImageE.Portable chest x-ray (CXR)

A 75-year-old man comes to the physician for a routine health maintenance examination. He has
fallen in his apartment three times in the past 4 weeks. He has had no headache, loss of
consciousness, or seizures. He denies fever, chills, palpitations, dyspnea on exertion, or shortness
of breath. He has a history of coronary artery disease, hypertension, hyperlipidemia,
osteoarthritis, benign prostatic hyperplasia, and depression. His medication regimen includes
isosorbide dinitrate, atenolol, lisinopril, simvastatin, ibuprofen, terazosin, and sertraline. His
temperature is 36.7ºC (98.0ºF), blood pressure is 128/68 mm Hg sitting and 122/64 mm Hg
standing, pulse is 82/min sitting and 88/min standing, and respirations are 16/min. Decreased
breath sounds at the bases are heard on auscultation of the lungs. Which of the following is the
most appropriate next step in evaluation?
 Incorrect Answer ImageA.Basic metabolic panel
 Incorrect Answer ImageB.Complete blood count
 Incorrect Answer ImageC.CT scan of the head
 Incorrect Answer ImageD.Echocardiogram
 Incorrect Answer ImageE.Electrocardiogram
 Correct Answer ImageF."Get up and go" test
 Incorrect Answer ImageG.Holter monitoring

A 42-year-old man presents to his doctor’s office with dyspnea on exertion, cough, and sputum
production. He has noticed these symptoms progress over the past few months. Initially he
believed he had contracted an upper respiratory infection, but his symptoms have lingered. He
denies any previous medical history and does not take any medications. He denies smoking, and
he drinks a glass of wine occasionally with meals. On physical examination, temperature is
36.7°C (98°F), pulse is 78/min, respirations are 16/min, and blood pressure is 130/82 mm Hg.
Head and neck examination shows no jugular venous distension. Heart exam shows a regular
rate and rhythm with a loud P2. Lung examination demonstrates diffusely decreased breath
sounds. Abdomen is soft and nontender to palpation. Peripheral pulses are normal. Pulmonary
function tests show an obstructive pattern. Laboratory data are:
Na 137 mEq/L
K 4.1 mEq/L
Cl 105 mEq/L
HCO3 27 mEq/L
ALT 115 IU/L
AST 130 IU/L
Alkaline phosphatase 113 IU/L
Which of the following is the most likely explanation for this man’s elevated transaminases? 
 Correct Answer ImageA.Accumulation of alpha-1-antitrypsin in hepatocytes
 Incorrect Answer ImageB.Alcoholism
 Incorrect Answer ImageC.Alpha-1 antitrypsin overactivity in the liver
 Incorrect Answer ImageD.Congestive hepatopathy
 Incorrect Answer ImageE.Unchecked hepatic elastase activity

A 67-year-old woman comes to the clinic for an annual visit. She says she has been in good
health other than experiencing back pain that started 2 months ago right after she moved into her
new house. The pain is constant but tolerable, and not associated with bladder or bowel
incontinence. She has a past medical history of hypertension, hypercholesterolemia, and
coronary heart disease. She underwent an angiogram with a stent placement 2 years ago.
Physical examination shows a temperature of 36.9ºC (98ºF), pulse of 80/min, respirations of
16/min, and blood pressure of 130/90 mm Hg. HEENT examination is normal. Cardiovascular
examination shows normal S1 and S2 with no rubs or gallops. Lungs are clear to auscultation.
Examination of the spine shows focal areas of tenderness on palpation of the lower spine.
Straight leg raise test is negative. Neurologic examination is normal. Laboratory workup shows:
Hgb 10.2 g/dL
WBC 8,000/mm3 
Platelets 110,000/mm3 
Sodium 132 mEq/L
Potassium 4.7 mEq/L
Chloride 101 mEq/L
Bicarbonate 27 mEq/L
BUN 46 mg/dL
Creatinine 1.9 mg/dL
Glucose 80 mg/dL
Calcium 11.2 mg/dL
Total protein 8.2 g/dL
Albumin 2.0 g/dL
Tests of liver function and INR are normal. Which of the following is most likely to confirm the
suspected diagnosis? 
 Correct Answer ImageA.Bone marrow biopsy
 Incorrect Answer ImageB.MRI of the spine
 Incorrect Answer ImageC.Serum protein electrophoresis
 Incorrect Answer ImageD.Urine protein electrophoresis
 Incorrect Answer ImageE.X-ray of the spine

A 64-year-old woman comes to the physician because of photophobia and a dry, gritty sensation
in both her eyes. Her symptoms are aggravated by prolonged activities such as reading and
watching television. Medical history is significant for hypertension, coronary artery disease, and
rheumatoid arthritis. Her medications include hydrochlorothiazide, atenolol, isosorbide dinitrate,
and aspirin. Temperature is 36.8ºC (98.3ºF), pulse is 76/min, blood pressure is 130/75 mm Hg,
and respirations are 16/min. Examination shows conjunctival injection and corneal dryness.
Visual acuity is 20/20 in both eyes. Extraocular muscle function and fundus examination are
normal. Which of the following is the most appropriate next step in management?
 Incorrect Answer ImageA.Discontinue hydrochlorothiazide
 Incorrect Answer ImageB.Immediate ophthalmology referral
 Incorrect Answer ImageC.Perform Schirmer's test
 Correct Answer ImageD.Prescribe artificial tears
 Incorrect Answer ImageE.Prescribe erythromycin ointment
 Incorrect Answer ImageF.Prescribe topical corticosteroids
 Incorrect Answer ImageG.Quantify anti-Ro (SSA) antibodies

A 61-year-old man with a 2-month history of hypercholesterolemia comes for a follow-up visit.
He denies abdominal pain, nausea, vomiting, diarrhea, or constipation. He reports mild, chronic
bilateral ringing in his ears. His medical history is unremarkable. He has had one surgery to
repair a femoral fracture 18 years ago. He exercises 4 times a week. Current medications include
pravastatin. Examination shows bilateral mild hearing loss. Serum studies show: 
Aspartate aminotransferase (AST, GOT) 20 U/L
Alanine aminotransferase (ALT, GPT) 12 U/L
Alkaline phosphatase 512 U/L
Bilirubin, total 0.8 mg/dL
Serum gamma-glutamyl transpeptidase level and calcium level are within normal limits. Which
of the following is the most likely diagnosis? 
 Correct Answer ImageA.Osteitis deformans
 Incorrect Answer ImageB.Osteitis fibrosa cystica
 Incorrect Answer ImageC.Osteogenesis imperfecta
 Incorrect Answer ImageD.Osteomalacia
 Incorrect Answer ImageE.Osteopenia

A 52-year-old man with a 2-week history of lower extremity cellulitis is brought to the
emergency department because of fever, confusion, and severe shortness of breath for 8 hours.
He is emergently intubated and admitted to the intensive care unit (ICU). His temperature is
39.4ºC (102.9ºF), pulse is 114/min, blood pressure is 85/45 mm Hg, and oxygen saturation is
90% on 100% oxygen by mechanical ventilation. Examination shows no jugular venous
distention. Decreased breath sounds and bilateral crackles are heard on auscultation. Cardiac
examination shows a normal S1 and S2 with no murmurs. There is no pedal edema. Laboratory
studies show: 
Hemoglobin  12.8 g/dL 
Leukocyte count  18,300/mm3
Platelet count  225,000/mm3
An ECG shows no abnormalities. A chest x-ray is shown. Which of the following findings is
most likely in this patient?
 Incorrect Answer ImageA.Bacterial vegetations on the mitral valve
 Incorrect Answer ImageB.Central venous pressure >12 cm H2O
 Incorrect Answer ImageC.Pulmonary capillary wedge pressure >18 mm Hg
 Correct Answer ImageD.Ratio of PaO2/FiO2 ≤200 mm Hg
 Incorrect Answer ImageE.Segmental perfusion defects with normal ventilation

A 60-year-old man with a history of type 2 diabetes mellitus variably controlled with metformin
comes to the emergency department because of a sudden onset of shaking chills, severe pain in
his right flank, and pain on urination for 1 day. His temperature is 38.7ºC (101.6ºF), pulse is
90/min, and blood pressure is 130/72 mm Hg. Examination shows moderate right flank
tenderness. A fingerstick glucose is 250 mg/dL, and the hemoglobin A1c level is 10.7%. A Gram
stain of the urine is shown. Which of the following is the most appropriate pharmacotherapy?
 Incorrect Answer ImageA.Ampicillin and gentamicin
 Incorrect Answer ImageB.Ceftriaxone and tetracycline
 Correct Answer ImageC.Ciprofloxacin
 Incorrect Answer ImageD.Nafcillin
 Incorrect Answer ImageE.Trimethoprim-sulfa
 Incorrect Answer ImageF.Nitrofurantoin

A 47-year-old woman undergoes a minor elective operation and develops a cascade of


complications that the medical team has no explanation for. Her temperature is 35.6°C (96.1°F),
pulse is 55/min, respirations are 10/min, and blood pressure is 90/60 mm Hg. When the team
reports the complications to the family, the patient's older sister offers the observation that the
patient was normal when they last saw each other before the sister moved to a distant city, but
that when they met again several years later the patient seemed to be "in slow motion" all the
time: she spoke slowly and with a "croaky" voice, had gained weight, and seemed to be mentally
limited, and all she talked about was how cold she was and how ineffective all the laxatives that
she took were. The team runs a stat T4 test which shows an extremely low value. Which of the
following therapies does this patient most urgently need?
 Incorrect Answer ImageA.ACTH
 Incorrect Answer ImageB.Iodine
 Correct Answer ImageC.Steroids
 Incorrect Answer ImageD.T3
 Incorrect Answer ImageE.T4

A 71-year-old man comes to his physician because of fatigue and shortness of breath for the past
6 months. He reports progressive dyspnea with exertion. He denies chest pain, chronic cough,
diarrhea, cramping, hematochezia, melena, orthopnea, paroxysmal nocturnal dyspnea, edema,
weight loss, or fever. He has no history of significant medical illnesses. He does not take any
medications. He occasionally drinks alcohol, but he does not smoke. His temperature is 37.1°C
(98.8°F), pulse is 98/min, respirations are 12/min, and blood pressure is 131/72 mm Hg. Oxygen
saturation is 96% on room air. On physical examination, there is pallor and bounding pulses. The
remainder of the physical examination is unremarkable. Laboratory studies show:
White blood cells 5,600/mm3
Hematocrit 29%
Platelets 225,000/mm3
Mean corpuscular volume 118 μm3
Iron 50 μg/dL
Ferritin 150 ng/mL
Serum folate 8 ng/mL
Serum B12 35 pg/mL
LDH 113 U/L
Peripheral blood smear hypersegmented neutrophils, macrocytosis
Antiparietal cell antibodies and anti-intrinsic factor antibodies are negative. A Schilling test is
performed, and a urine assay for radiolabeled oral B12 is negative. The Schilling test is repeated
with intrinsic factor supplementation four weeks later. Urine assay for oral B12 is still
negative. What is the most likely diagnosis in this patient?
 Incorrect Answer ImageA.Dietary deficiencyIncorrect Answer Image
 Correct Answer ImageB.Diphyllobothrium latum infection
 Incorrect Answer ImageC.Metastatic disease
 Incorrect Answer ImageD.Pernicious anemia
 Incorrect Answer ImageE.Ulcerative colitis

A 26-year-old woman comes to her physician because of a 1-year history of amenorrhea. She
also reports occasional discharge from both breasts. She has no history of significant medical
illnesses, and she takes no medications. She has a 5-year-old child that she breast-fed until 9
months of age. Her temperature is 37.1°C (98.8°F), pulse is 80/min, respirations are 12/min, and
blood pressure is 130/80 mm Hg. Physical examination shows diffuse enlargement of the thyroid
gland. Breast examination shows the expression of milk from both breasts. Laboratory studies
show:
Serum prolactin  160 ng/mL 
(normal, 4–30 ng/mL)
Thyroid-stimulating hormone 36 mU/L
(normal, 0.4–5 mU/L) 
Thyroxine (T4) 0.6 ng/dL
(normal, 0.8–1.8 ng/dL) 
Urine β-hCG Negative
MRI of the brain shows pituitary enlargement. Which of the following is the most appropriate
next step in management?
 Incorrect Answer ImageA.Bromocriptine
 Incorrect Answer ImageB.Cabergoline
 Incorrect Answer ImageC.Radiotherapy
 Correct Answer ImageD.Thyroxine replacement
 Incorrect Answer ImageE.Transsphenoidal surgery

A 70-year-old man comes to the physician because of increased fatigue, dyspnea, and weight
loss over the past 6 months. He denies chest pain or anorexia. He has previously been healthy but
has not seen a physician in 15 years. Review of systems is positive for a change in his stool
pattern. He has had 3 bowel movements a day instead of one, and they are looser. Family
members have commented on his pale appearance. He is currently not taking any medications.
He denies allergies. He has no history of alcohol or drug abuse. On examination, he is ill-
appearing and pale. Pulse is 120/min and blood pressure is 100/70 mm Hg. His abdomen is
nontender to palpation. Rectal examination shows stool that is positive for occult blood. His
electrocardiogram shows sinus tachycardia. His hematocrit is 20%. Which of the following is the
most appropriate next step in management?
 Incorrect Answer ImageA.Abdominal CT scan
 Correct Answer ImageB.Colonoscopy
 Incorrect Answer ImageC.Esophagogastroduodenoscopy
 Incorrect Answer ImageD.Increased fiber diet
 Incorrect Answer ImageE.Stool culture

A 24-year-old woman comes to the emergency department because of an acute onset of


abdominal cramping and hives for 12 hours. She states that 2 days ago, she had a sore throat and
started taking an over-the-counter cold medicine containing dextromethorphan,
diphenhydramine, pseudoephedrine, and ibuprofen. She also has had a generalized itch. She is in
mild distress because of her symptoms but denies shortness of breath or difficulty swallowing.
Of note, she has a history of asthma and takes doxycycline for acne. Her temperature is 36ºC
(96.8ºF), pulse is 90/min, respirations are 24/min, and blood pressure is 105/65 mm Hg. Physical
examination shows multiple 1 to 10 cm, pink-and-white urticarial papules and plaques on the
trunk and proximal extremities. Which of the following is the most likely cause of this patient's
condition?
 Incorrect Answer ImageA.Dextromethorphan
 Incorrect Answer ImageB.Diphenhydramine
 Correct Answer ImageC.Ibuprofen
 Incorrect Answer ImageD.Pseudoephedrine
 Incorrect Answer ImageE.Doxycycline
A 32-year-old man comes to his physician because of worsening nausea, polyuria, abdominal
pain, constipation, diffuse bone pain, and intermittent headaches for the past year. The patient
has a 10 pack-year history of smoking, but does not use alcohol or recreational drugs. He has no
other past medical history and takes no medications. Family history is positive for kidney stones
in his mother. Vital signs are within normal limits. Physical examination shows dry mucous
membranes. There are no palpable neck masses. The chest is clear to auscultation. There is mild
left lower quadrant abdominal pain on deep palpation. Laboratory studies show:
Complete blood count
Hemoglobin 14 g/dL
Hematocrit 50%
Leukocyte count 7,000/mm3
Platelets 250,000 mm3
Serum Chemistry
Na+ 145 mEq/L
K+ 5.0 mEq/L
Ca 2+
12.1 mg/dL
Cl- 100 mEq/L
HCO3 -
24 mEq/L
BUN 30 mg/dL
Creatinine  1.0 mg/dL
Glucose  100 mg/dL
Albumin  4 g/dL
Phosphorus  3.0 mg/dL
Parathyroid hormone  94 pg/mL (N = 10–65 pg/mL)
The chest x-ray is normal. Which of the following is the most appropriate next step in the
management of this patient? 

 Incorrect Answer ImageA.Measure serum parathyroid hormone-related peptide (PTHrP)


 Incorrect Answer ImageB.Measure serum vitamin D
 Correct Answer ImageC.Measure urinary calcium
 Incorrect Answer ImageD.Order MRI of the thyroid and parathyroid glands
 Incorrect Answer ImageE.Order Sestamibi parathyroid scintigraphy

A previously healthy 31-year-old man comes to the emergency department writhing in pain. He
awoke 1 hour ago with excruciating pain in his right lower quadrant. The pain is sharp,
intermittent, and non-radiating. He can eat and drink normally. He has no past history of
abdominal pain. He has mild seasonal rhinitis, for which he takes fluticasone as needed. His
temperature is 36.7ºC (98.0ºF), pulse is 112/min, respirations are 24/min, and blood pressure is
137/84 mm Hg. His abdomen is soft and nondistended, with normal bowel sounds. He has mild
right-sided tenderness to deep palpation, with lower quadrant greater than upper quadrant. He
has a circumcised phallus with no urethral discharge. Both testes are within the scrotum and
nontender. Rectal examination is nontender and reveals a firm, small prostate. His stool is
negative for occult blood. Laboratory studies show:
WBC 8,400/mm3
BUN 11 mg/dL
Creatinine 1.1 mg/dL
Urinalysis findings
Urine specific gravity 1.030
pH 5.0
RBC 25–30/hpf
WBC 3–4/hpf
CT scan of the abdomen and pelvis shows a 3 mm calcification at the level of the right
ureterovesical junction. Which of the following is the most appropriate management?
 Incorrect Answer ImageA.Admit to the hospital for serial abdominal examinations
 Correct Answer ImageB.Discharge home with a prescription for a nonsteroidal anti-
inflammatory agent (NSAID)
 Incorrect Answer ImageC.Open surgical removal of stone
 Incorrect Answer ImageD.Placement of a ureteral stent
 Incorrect Answer ImageE.Placement of a percutaneous nephrostomy tube

A 72-year-old man comes to his physician because he has had decreased energy and severe pain
in his back and leg, such that he has a distinct limp when walking. He denies any trauma. On
examination, he is pale and has tenderness to palpation along the vertebrae as well as pain upon
flexion and extension of his left hip. He has no other positive physical findings. A complete
blood count shows a hematocrit of 27% with marked rouleaux formation. His white blood cells
and platelets are normal in number and appearance. An x-ray of his left hip and upper leg shows
a fracture of the femoral neck. Laboratory studies show a serum calcium of 12 mg/dL; ESR of 40
mm/h; and urine dipstick positive for 3+ protein. Which of the following describes the hallmark
of the diagnosis?
 Incorrect Answer ImageA.A bone-marrow aspiration showing peritrabecular lymphoid
aggregates
 Incorrect Answer ImageB.A bone-marrow aspiration showing the Philadelphia
chromosome
 Incorrect Answer ImageC.A decrease in serum viscosity

 Correct Answer ImageD.A monoclonal spike in the beta- or gamma-globulin region on


serum protein electrophoresis
 Incorrect Answer ImageE.Hypogammaglobulinemia

A 42-year-old man comes to the physician because of a 3-day history of right elbow pain. The
pain is worse with right arm movement. His pain is not relieved by rest or ice application. He
reports having a similar pain in the past, relieved with rest and ice. He denies having either a
fever or fall. He plays tennis for 4 hours per day and gives tennis lessons. Physical examination
shows tenderness over the lateral epicondyle. Extension of the hand at the wrist against
resistance produces pain. The tendon of which of the following muscles is most likely injured in
this patient? 
 Incorrect Answer ImageA.Brachioradialis
 Correct Answer ImageB.Extensor carpi radialis brevis
 Incorrect Answer ImageC.Extensor carpi radialis longus
 Incorrect Answer ImageD.Flexor carpi radialis
 Incorrect Answer ImageE.Pronator teres

A 45-year-old man who had been diagnosed with type 1 diabetes mellitus at the age of 15 comes
to the physician for a follow-up visit. The patient has no complaints at this time and is compliant
with his medications, which include NPH insulin, regular insulin, and lisinopril. His temperature
is 37ºC (98.6ºF), blood pressure is 130/85 mm Hg, pulse is 86/min, and respirations are 12/min.
His last hemoglobin A1C was 7.2%. Physical examination shows deformity of both feet with loss
of the normal arches and abnormal orientation of the toes. No inflammation is noted.
Radiographs show deformed bones with new bone formation adjacent to the bony cortex. Several
large, bizarrely shaped osteophytes are seen at the joint margins. Which of the following is the
most likely diagnosis?
 Incorrect Answer ImageA.Avascular necrosis
 Incorrect Answer ImageB.Gout
 Incorrect Answer ImageC.Infectious arthritis
 Correct Answer ImageD.Neurogenic arthropathy
 Incorrect Answer ImageE.Osteoarthritis

 A 64-year-old man comes to the physician for follow-up of his congestive heart failure (CHF).
He has had CHF for 2 years following a myocardial infarction. He was hospitalized 4 weeks ago
because of a CHF exacerbation caused by excessive salt intake. He currently takes metoprolol,
simvastatin, and a loop diuretic. His blood pressure is 150/72 mm Hg and pulse is 52/min.
Physical examination shows a laterally displaced point of maximal impulse. There is no jugular
venous distension or dependent edema. Which of the following changes in management will
provide the greatest survival benefit for this patient?
 Incorrect Answer ImageA.Begin a calcium channel blocker
 Correct Answer ImageB.Begin an ACE inhibitor
 Incorrect Answer ImageC.Begin digoxin therapy
 Incorrect Answer ImageD.Change metoprolol to propranolol
 Incorrect Answer ImageE.Increase diuretic dose

A 52-year-old man with a 3-month history of hypertension comes to the physician because of
swelling in his hands and feet for 2 years. He has had no change in vision, chest pain, or changes
in weight. His blood pressure is 152/90 mm Hg. Examination shows macroglossia,
macrognathia, and soft-tissue swelling in the extremities. Cardiac examination shows a left
displaced point of maximal impulse. A grade 2/6 systolic murmur is heard over the left sternal
border. Serum growth hormone level is 4 ng/mL 1 hour after ingestion of 75 g of glucose (n, <1
ng/mL). MRI scan of the head shows a 1.5 cm pituitary macroadenoma. The patient does not
wish to undergo surgery. Which of the following is the most appropriate next step in
management?
 Incorrect Answer ImageA.Chlorambucil
 Incorrect Answer ImageB.Cyclophosphamide
 Incorrect Answer ImageC.Cyclosporine
 Incorrect Answer ImageD.External beam radiation
 Incorrect Answer ImageE.No further treatment
 Correct Answer ImageF.Octreotide

A 66-year-old diabetic man is brought to the emergency department via ambulance because of
shortness of breath. For the past 2 hours, he has experienced dyspnea at rest and has been
coughing up sputum. He denies chest pain. His temperature is 37.2ºC (98.9ºF), pulse is 124/min,
respirations are 22/min, and blood pressure is 156/94 mm Hg. On physical examination, the
patient is lying down and in distress. He is unable to complete full sentences. An
electrocardiogram is normal. What is the next best step in management? 
 Correct Answer ImageA.Assist the patient to a sitting position

 Incorrect Answer ImageB.Dobutamine


 Incorrect Answer ImageC.Dopamine
 Incorrect Answer ImageD.Echocardiography
 Incorrect Answer ImageE.Hydralazine
 Incorrect Answer ImageF.MUGA scan
 Incorrect Answer ImageG.Synchronized cardioversion
 Incorrect Answer ImageH.Verapamil

A 27-year-old woman comes to the emergency department because of shortness of breath. The
current symptoms began approximately 5 hours ago and have become gradually more severe.
She reports a history of mild episodic asthma but has not had an attack in more than 3 years. She
is on no medications. Her respiratory rate is 25/min. Lung examination is notable for loud
expiratory wheezing bilaterally. The expiratory phase is prolonged, and air movement is fair, but
decreased. A peak expiratory flow rate is 110 L/min. Her oxygen saturation on room air is 92%.
Which of the following is the most appropriate next step in management?
 Correct Answer ImageA.Albuterol nebulizer
 Incorrect Answer ImageB.Inhaled corticosteroids
 Incorrect Answer ImageC.Intravenous methylprednisone
 Incorrect Answer ImageD.Oral prednisone
 Incorrect Answer ImageE.Subcutaneous epinephrine
A 45-year-old woman comes to her physician for increasing edema for 1 year. The edema was
initially in both feet, but 3 weeks ago she noticed severe diffuse swelling and blood in her urine.
Her shoes no longer fit. Her pulse is 90/min and her blood pressure is 175/100 mm Hg. She has
diffuse 4+ edema to mid-thigh. Laboratory studies show: 
Sodium 136 mEq/L
Potassium 5.7 mEq/L
Chloride 100 mEq/L
Bicarbonate 19 mEq/L
Creatinine 9.6 mg/dL
Urea nitrogen 100 mg/dL
Albumin 3.1 mg/dL
Calcium 8.1 mg/dL
Phosphorus 7.5 mg/dL
Urinalysis shows 4+ protein, 2+ blood, and 5–10 RBC/HPF. ECG is normal. Renal ultrasound
shows small, echogenic kidneys. The patient is admitted to the hospital, has a dialysis catheter
placed, and hemodialysis is started on hospital day 2. Which of her problems will be least
improved by the hemodialysis?
 Incorrect Answer ImageA.Acidemia
 Incorrect Answer ImageB.Edema
 Incorrect Answer ImageC.Hyperkalemia
 Correct Answer ImageD.Hyperphosphatemia
 Incorrect Answer ImageE.Hypertension

A 61-year-old woman comes to the physician because of shortness of breath. She states that she
has had a 6-month history of progressively worsening dyspnea while climbing the stairs in her
house. She denies both chest pain and dyspnea at rest. She appears to be comfortable at rest and
has a barrel-shaped chest. Her medical history is remarkable for hypertension and chronic
obstructive pulmonary disease. Physical examination shows a prolonged expiratory phase and
wheezes on forced exhalation. Heart sounds are regular in rate and rhythm with no murmurs,
rubs, or gallops. A transthoracic echocardiogram shows diastolic left ventricular dysfunction.
Which of the following is the most appropriate next step in the management of this patient? 
 Incorrect Answer ImageA.Digoxin
 Incorrect Answer ImageB.Dobutamine
 Incorrect Answer ImageC.Propranolol
 Incorrect Answer ImageD.Transesophageal echocardiography
 Correct Answer ImageE.Verapamil

A 57-year-old man comes to the physician because of shortness of breath and progressive cough
over the past month. He denies fever or weight loss. He has a 5-year history of chronic
obstructive pulmonary disease treated with albuterol and ipratropium bromide. He works as a
construction worker and has smoked 2 packs of cigarettes daily for the past 39 years. His
temperature is 36.8ºC (98.2ºF), blood pressure is 130/70 mm Hg, pulse is 110/min, and
respirations are 25/min. Physical examination shows diffuse wheezing bilaterally. The remainder
of the examination is normal. Chest x-ray shows an opacity in the right lung. CT scan of the
chest without contrast shows a right lung mass and an enlarged paratracheal lymph node. Which
of the following is the most appropriate next step?
 Incorrect Answer ImageA.Bronchoscopy with a biopsy
 Incorrect Answer ImageB.Helical CT of the chest with contrast
 Incorrect Answer ImageC.MRI of the chest
 Correct Answer ImageD.Positron emission tomography (PET) CT of the chest with
fluorodeoxyglucose
 Incorrect Answer ImageE.Pulmonary function testing

A previously healthy 28-year-old woman comes to the emergency department because of fevers,
headache, photophobia, and mild slurred speech for 48 hours. She recently returned from a trip to
upstate New York with her friends. A day later, she noted a rash with a central area of clearing
on her forearm. The rash resolved over the next three weeks and she was feeling fine until
yesterday. Currently, her temperature is 38.0ºC (100.4ºF), blood pressure is 120/70 mm Hg,
pulse is 73/min, and respirations are 13/min. Physical examination shows mild photophobia and
nuchal rigidity. She has a mild left-sided facial droop. There is no rash present. The remainder of
her physical examination is normal. Which of the following is the most appropriate next step in
management? 
 Incorrect Answer ImageA.Administration of intravenous vancomycin
 Incorrect Answer ImageB.Administration of oral doxycycline
 Incorrect Answer ImageC.Determination of serum Lyme antibody titers
 Correct Answer ImageD.Lumbar puncture
 Incorrect Answer ImageE.Lyme polymerase chain reaction of serum

A 33-year-old man comes to the physician for a routine health maintenance examination. He
denies fever, chills, weight loss, loss of appetite, abdominal pain, diarrhea, chest pain, or cough.
He has a history of childhood asthma that resolved at the age of 6 years. He underwent an
appendectomy at age 11 years. He exercises regularly and takes no medications. His pulse is
75/min, and blood pressure is 115/80 mm Hg. His lungs are clear to auscultation. Cardiac
examination shows a normal S1 and S2 without murmurs. Laboratory studies are normal. A
tuberculin skin test with purified protein derivative (PPD) shows an induration of 11 mm in
diameter after 48 hours. Which of the following is the most appropriate interpretation of this
patient's PPD test result?
 Incorrect Answer ImageA.A tuberculin test cannot be interpreted in asymptomatic
patients
 Incorrect Answer ImageB.A tuberculin test is always equivocal if the patient does not
have a cough
 Incorrect Answer ImageC.A tuberculin test is always positive if the chest radiograph is
normal
 Incorrect Answer ImageD.The tuberculin test result is equivocal
 Correct Answer ImageE.The tuberculin test result is negative

A 19-year-old woman comes to the emergency department after a syncopal event 1 hour ago.
While working in the garden, she accidentally disturbed a wasp nest and was stung on her right
hand. She has never been stung before. The woman tried to rest at home but felt extremely ill and
called for an ambulance. She says she is dizzy and is concerned about some swelling in both of
her hands and feet. Her temperature is 37.8ºC (100.0ºF), pulse is 130/min, respirations are
22/min, and blood pressure is 83/40 mm Hg. Oxygen saturation is 97% on room air. Pulmonary
examination shows mild diffuse wheezing in all lung fields. Which of the following is the most
appropriate next step in management?
 Correct Answer ImageA.Administer epinephrine
 Incorrect Answer ImageB.Bolus patient with 500 mL of half-normal saline
 Incorrect Answer ImageC.Perform endotracheal intubation
 Incorrect Answer ImageD.Prescribe diphenhydramine and cimetidine
 Incorrect Answer ImageE.Start IV corticosteroids

A 74-year-old woman comes to the physician because of a 3-month history of bone pain. The
pain is not relieved by ibuprofen. She has a history of hypertension and coronary artery disease.
Current medications include lisinopril, metoprolol, and aspirin. She has smoked 2 packs of
cigarettes daily for 30 years. She does not drink alcohol. Her temperature is 36.8°C (98.2°F),
blood pressure is 130/85 mm Hg, and pulse is 65/min. A photograph of her hands is shown. A
prolonged expiratory phase and an S4 gallop are heard on auscultation. There is tenderness to
palpation of her distal long bones. Radiographs of the forearms and femurs show bilateral
periostosis. Which of the following is the most appropriate next step in management?
 Incorrect Answer ImageA.Bone marrow biopsy
 Correct Answer ImageB.Chest radiography
 Incorrect Answer ImageC.Increase her ibuprofen frequency and dosage
 Incorrect Answer ImageD.Joint aspiration
 Incorrect Answer ImageE.No further management is indicated

A 25-year-old man comes to the physician because of chest pain. He says that the pain is
spontaneous and intermittent. He also reports two prior episodes of fainting during exercise but
has otherwise been healthy. He reports a similar history in other family members and notes that
his father collapsed and died at age 50 while playing tennis. The patient denies use of cocaine or
other recreational drugs and takes no medications. Cardiac examination shows a systolic ejection
murmur that is loudest along the left sternal border. The remainder of the physical examination
shows no abnormalities. Echocardiography shows asymmetric septal hypertrophy without
obstruction. Which of the following interventions would most likely decrease this patient's
systolic murmur?
 Incorrect Answer ImageA.Amyl nitrite inhalation
 Incorrect Answer ImageB.Digoxin
 Correct Answer ImageC.Lying down
 Incorrect Answer ImageD.Standing up
 Incorrect Answer ImageE.Valsalva maneuver

A 60-year-old man initially develops an inflamed papule on his thumb followed by the lesion
shown below one day after hunting and skinning rabbits. Several days later, he develops a severe
illness with cough and confusion. Physical examination performed in the emergency department
shows left axillary lymphadenopathy, reduced breath sounds, rales bilaterally over the lung
bases, and splenomegaly. Blood studies show a mild leukocytosis. Which of the following is the
most likely diagnosis?
 Incorrect Answer ImageA.Actinomycosis
 Incorrect Answer ImageB.Brucellosis
 Incorrect Answer ImageC.Melioidosis
 Incorrect Answer ImageD.Plague
 Correct Answer ImageE.Tularemia

An 18-year-old man comes to the physician for a physical examination before leaving for
college. Although he has no complaints and states that he is in good health, he has noticed a
small mass in the left supraclavicular region. The mass has been present for at least the last
month and he believes it is getting larger. He denies having any pets or any recent travel. His
past medical history is significant for a tonsillectomy as a child. He is a nonsmoker. His
temperature is 37.0ºC (98.6ºF), blood pressure is 128/72 mm Hg, pulse is 60/min, and
respirations are 20/min. Examination shows a hard, 3 cm supraclavicular lymph node that is
nontender and immobile. The remainder of the examination is normal. Chest radiograph and
complete blood count are ordered. Which of the following is most likely to confirm the
diagnosis?
 Incorrect Answer ImageA.Chest CT with contrast
 Incorrect Answer ImageB.Close observation with serial measurement
 Incorrect Answer ImageC.Fine-needle aspiration of the mass
 Correct Answer ImageD.Surgical consult for excisional biopsy
 Incorrect Answer ImageE.Ultrasound of the mass

A 47-year-old man is brought to the emergency room after vomiting blood. He reports
abdominal pain and severe nausea. He has a history of drinking alcohol for the past 25 years and
has drunk non-stop for the last six days. He reports multiple episodes of severe abdominal pain
in the previous three weeks. His medical records show a history of pancreatitis and
ultrasonographic changes of liver cirrhosis. His temperature is 37.6ºC (99.6ºF), blood pressure is
98/55 mm Hg, pulse is 105/min, and respirations are 18/min. He is given oxygen and 2 large
bore IV with normal saline are started. Laboratory tests are taken and show the following: 
WBC 13,200/mm3
Neutrophils 68%
Hgb 8.6 g/dL
Hct 26%
Sodium 133 mEq/L
Potassium 3.9 mEq/L
Chloride 97 mEq/L
Amylase 625 U/L
Lipase 937 U/L
ALT 103 U/L
AST 198 U/L
Alkaline phosphatase 104 IU/L
Abdominal examination shows a mildly distended abdomen, tympanic to percussion, and painful
to palpation with no voluntary guarding or rebound. No masses are palpated, although there is
slight liver and splenic enlargement. An abdominal ultrasound shows changes consistent with
portal hypertension, and a thrombosis of the splenic vein not seen on a previous study. An upper
endoscopy is planned. Which of the following is the most likely source of his bleeding? 
 Incorrect Answer ImageA.Alcoholic gastritis
 Incorrect Answer ImageB.Esophageal varices
 Correct Answer ImageC.Gastric varices
 Incorrect Answer ImageD.Peptic ulcer disease
 Incorrect Answer ImageE.Squamous cell cancer of the oropharynx

A 21-year-old woman goes to the doctor because of itchy, watery eyes and clear nasal discharge.
Further questioning indicates that she typically has problems all year long but that these two
complaints are more pronounced during the spring and fall seasons. She denies episodes of fever,
thick nasal drainage, or facial pain. Physical examination shows bilateral conjunctival injection
and grayish discoloration of her lower eyelids. She has pale, boggy nasal mucosa with swollen
inferior turbinates. The nasal septum is midline. She is diagnosed with allergic rhinitis and given
a prescription for an intranasal glucocorticoid, oral loratadine and nasal phenylephrine spray as
needed for symptoms. She returns to the clinic two weeks later with no improvement. She is
unable to work or sleep. Which of the following is the best next step in the management of all of
her complaints? 
 Correct Answer ImageA.A 10-day course of an oral steroid
 Incorrect Answer ImageB.Continue the current regimen
 Incorrect Answer ImageC.Oral amoxicillin for 10 days
 Incorrect Answer ImageD.Oral amoxicillin-clavulanate for 10 days
 Incorrect Answer ImageE.Oxymetazoline
 Incorrect Answer ImageF.X-ray of the sinuses
A 70-year-old man with a history of chronic arthritis comes to the physician with two days of
acute right-sided abdominal pain that is cramping in nature. Physical examination is abnormal
for right knee redness and swelling. Urinalysis shows hematuria. A CT scan shows multiple
kidney stones in the right renal pelvis. He passes a kidney stone and analysis indicates it is a uric
acid stone. His serum urate level is 12 mg/dL and creatinine is 1.9 mg/dL. Aspirated fluid from
his right knee shows urate crystals. Which of the following is the most appropriate long-term
treatment for this patient?
 Correct Answer ImageA.Allopurinol
 Incorrect Answer ImageB.Corticosteroids
 Incorrect Answer ImageC.Lithotripsy
 Incorrect Answer ImageD.Probenecid
 Incorrect Answer ImageE.Ureteral stent placement

A 66-year-old man comes to the physician because of unrelenting diarrhea, fatigue, and weight
loss. The symptoms have been insidiously progressing for the past 2 months. He has also noticed
that his stools are pale, oily, bulky, and difficult to flush. He voids approximately 7 times per
day. He also complains of increasing arthralgias. He denies recent travel or recent camping trips.
His past medical history is significant for hypertension treated with hydrochlorothiazide. On
physical examination, he appears frail and in no acute distress. His blood pressure is 110/75 mm
Hg and pulse is 88/min. Head and neck examination shows mild glossitis and lymphadenopathy.
On cardiac examination, a systolic murmur is present. His lungs are clear to auscultation. He has
increased bowel sounds overall, without hepatosplenomegaly or tenderness. Peripheral
examination shows no cyanosis, clubbing, or edema. Neurologic examination shows profound
ataxia and difficulty with rapid alternating movements. Steatorrhea is confirmed on a 24-hour
stool collection. Biopsy of his small bowel demonstrates periodic acid-Schiff-stained
macrophages in the lamina propria. Stains for fungi and acid-fast bacteria are negative. He is also
found to be HIV-negative. Which of the following is the most appropriate next step in
management?
 Correct Answer ImageA.Antibiotics
 Incorrect Answer ImageB.Corticosteroids
 Incorrect Answer ImageC.Gluten avoidance
 Incorrect Answer ImageD.Lactose avoidance
 Incorrect Answer ImageE.Loperamide

A 43-year-old woman comes to the emergency department because of a productive cough of 3


days' duration and spiking fevers. She has a 10-year history of rheumatoid arthritis and takes
NSAIDs to control her disease. Her temperature is 38.6ºC (101.5ºF), pulse is 98/min, respirations
are 22/min, and blood pressure is 123/75 mm Hg. Her oxygen saturation is 94% on room air.
Physical examination shows rhonchi in the right middle and lower lobes and normal heart
sounds. Her abdomen is nontender and nondistended. The spleen is palpable 2 cm below the
costal margin. She has rheumatoid nodules on the extensor surfaces of her fingers and lower legs
bilaterally. A chest radiograph shows a right lower lobe consolidation with an effusion. Which of
the following additional findings is most likely to be seen in this patient? 
 Correct Answer ImageA.Absolute neutrophil count <1,500/mm3
 Incorrect Answer ImageB.Decreased serum immunoglobulins
 Incorrect Answer ImageC.Hypocellular bone marrow
 Incorrect Answer ImageD.Low-titer rheumatoid factor
 Incorrect Answer ImageE.Small granular lymphocytes on peripheral blood smear

A 34-year-old man comes to his physician for a routine health maintenance examination. He
reports feeling well. He has no history of significant medical illnesses. He takes no medications.
His family history is significant for polycystic kidney disease in his mother and brother. His vital
temperature is 37.1°C (98.8°F), pulse is 80/min, respirations are 12/min, and blood pressure is
145/85 mm Hg. Laboratory data show a serum creatinine of 1.2 mg/dL. CT scan of the abdomen
shows normal-sized kidneys with no cysts. However, there is a 1.7 cm hypodense nodule with
smooth rounded contours on the left adrenal gland. Which of the following is the most
appropriate next step in the work-up for this patient?
 Correct Answer ImageA.24-hour urine catecholamine and metanephrine, low-dose
dexamethasone suppression test, and serum aldosterone-to-renin ratio
 Incorrect Answer ImageB.Cortisol stimulation test
 Incorrect Answer ImageC.CT abdomen adrenal protocol (multi-phase) to better evaluate
the lesion
 Incorrect Answer ImageD.CT chest, abdomen, and pelvis to assess for metastases
 Incorrect Answer ImageE.MRI abdomen adrenal protocol (multi-phase) to better evaluate
the lesion

A 60-year-old alcoholic woman with cirrhosis comes to the physician because of a 3-month
history of abdominal pain, weakness, fatigue, and 15-lb weight loss. The pain is vague and
located over the right upper quadrant. It is not associated with meals, movement, or positional
changes. There is occasional vomiting after meals. Her last colonoscopy a year ago was normal.
On examination, she is fatigued and moves slowly. Her abdomen is non-tender, distended, with
mild shifting dullness and a palpable mass in the right upper quadrant. The patient is most likely
to have which of the following laboratory values?
 Correct Answer ImageA.Increased alpha-fetoprotein
 Incorrect Answer ImageB.Increased beta-human chorionic gonadotropin
 Incorrect Answer ImageC.Increased carcinoembryonic antigen
 Incorrect Answer ImageD.Leukocytosis
 Incorrect Answer ImageE.Serum albumin concentration of 5.6 g/dL

A 34-year-old man with a history of epilepsy comes to the physician because he is planning a
trip to Africa. He has plans to visit rural areas in addition to major cities. He has had no loss of
consciousness or migraines. His last seizure was 2 years ago. Current medications include
phenytoin. He has no allergies. He has been vaccinated against hepatitis B. In addition to
mosquito-avoidance measures, which of the following is recommended for malaria prophylaxis? 
 Incorrect Answer ImageA.Chloroquine
 Incorrect Answer ImageB.Ciprofloxacin
 Correct Answer ImageC.Doxycycline
 Incorrect Answer ImageD.No prophylaxis if he is using mosquito-avoidance measures
 Incorrect Answer ImageE.Pyrimethamine-sulfadoxine

A 60-year-old woman comes to the physician for a follow up examination 2 months after
undergoing partial parathyroidectomy for a parathyroid adenoma that caused
hyperparathyroidism. She now has had 2 weeks of increasing muscle twitching as well as a
tingling sensation around her mouth, hands and feet. Her blood pressure is 126/82 mm Hg and
pulse is 76/min. Physical examination shows a well-healed surgical neck scar. Tapping the facial
nerve anterior to the ear causes a muscle spasm of the ipsilateral facial muscles. Her serum
calcium level is 6.6 mg/dL, with phosphate 2.0 mg/dL. Which of the following is the most likely
cause of this patient's hypocalcemia?
 Correct Answer ImageA.Hungry bone syndrome
 Incorrect Answer ImageB.Pancreatitis from hyperparathyroidism
 Incorrect Answer ImageC.Pseudohypoparathyroidism
 Incorrect Answer ImageD.Removal of the wrong parathyroid gland
 Incorrect Answer ImageE.Surgical damage to the remaining parathyroid glands

A 32-year-old woman comes to the office for evaluation of hypertension detected on a public
health screen in the local park where her blood pressure was 178/94 mm Hg. She has not had any
major health problems in the past and is currently asymptomatic. She denies gross hematuria,
headaches, visual changes, chest pain, cough, hemoptysis, dyspnea, numbness, weakness, skin
rash, or arthralgias. She does not take any medications. She does not smoke, drink alcohol, or use
illicit drugs. She has never been pregnant. Her mother has diabetes and her father has
hypertension. At this visit, her blood pressure is 180/92 mm Hg in the left arm and 178/90 mm
Hg in the right arm, and pulse is 88/min. The optic fundi are normal. The neck is supple with no
JVD or lymphadenopathy. Cardiopulmonary examination shows no abnormalities. Abdominal
examination is normal. Extremities show no edema. The serum BUN is 21 mg/dL, creatinine 2.0
mg/dL. Prior BUN and creatinine measurements are not available. Complete blood count and
serum electrolytes, blood sugar, and liver function studies are normal. Urinalysis shows:
Specific gravity 1.015
pH 5.5 
Blood 1+
Protein 2+
Ketones Negative
Glucose Negative
RBCs 15–20/HPF
WBCs 0/HPF
Epithelial cells Negative
Casts 1–2 RBC casts/LPF
Crystals Negative
Anti-antistreptolysin O, ANA, hepatitis B/C, HIV titers are negative, and C3 and C4 complement
levels are normal. A 24-hour urine test collection shows 2.2 g of protein in 24 hours. Which of
the following is the most appropriate next step in management?
 Incorrect Answer ImageA.CT scan of the abdomen
 Incorrect Answer ImageB.Cystoscopy
 Correct Answer ImageC.Kidney biopsy
 Incorrect Answer ImageD.No further tests are required
 Incorrect Answer ImageE.Skin biopsy
 Incorrect Answer ImageF.Urine culture

A 30-year-old man is admitted to the intensive care unit after a motor-vehicle accident in which
he sustained a right femur fracture. On admission, his leukocyte count is 30,000/mm3. Physical
examination shows rales in the right lung base. There is no hepatosplenomegaly. Laboratory
values show hemoglobin 9 g/dL, leukocyte count 55,000/mm3 with 95% neutrophils, and
platelets 160,000/mm3. No myelocytes or metamyelocytes are noted. Leukocyte alkaline
phosphatase level is high. Which of the following is the most likely cause of the high leukocyte
count? 
 Incorrect Answer ImageA.Agnogenic myeloid metaplasia
 Incorrect Answer ImageB.Chronic lymphocytic leukemia
 Incorrect Answer ImageC.Chronic myelogenous leukemia
 Correct Answer ImageD.Leukemoid reaction
 Incorrect Answer ImageE.Polycythemia vera

A 68-year-old man comes to the emergency department because of a 3-hour history of pain in his
left foot. He has a 20-year history of hypertension, chronic renal insufficiency, and peptic ulcer
disease. Current medications include hydrochlorothiazide, nifedipine, and omeprazole. His
temperature is 37.8ºC (100ºF), blood pressure is 140/85 mm Hg, and pulse is 85/min. A
photograph of his foot is shown. Arthrocentesis shows needle-shaped, negatively birefringent
crystals. Serum creatinine level is 2.4 mg/dL. Which of the following is the most appropriate
pharmacotherapy for this patient?
 Incorrect Answer ImageA.Allopurinol
 Correct Answer ImageB.Corticosteroids
 Incorrect Answer ImageC.High-dose colchicine
 Incorrect Answer ImageD.Indomethacin
 Incorrect Answer ImageE.Probenecid

A 60-year-old woman with a 1-year history of osteoarthritis comes to the physician because of
pain in her left knee not relieved by 2,200 mg of ibuprofen per day. She denies abdominal pain
or blood in her stool. She had a 4.5-kg (10-lb) weight gain over the past year and currently
weighs 90.7 kg (200 lbs) and is 160 cm (63 in). Her temperature is 37°C (98.6°F), blood pressure
is 125/75 mm Hg, and pulse is 70/min. The left knee shows moderate joint effusion that is
diffusely tender to palpation with crepitus, and mild medial and lateral joint instability. Which of
the following is the most appropriate next step in management?
 Incorrect Answer ImageA.Add acetaminophen to her regimen
 Incorrect Answer ImageB.Intra-articular injection of steroids every 2 weeks for 6 months
 Incorrect Answer ImageC.Knee replacement
 Incorrect Answer ImageD.Switch to a different non-steroidal anti-inflammatory drug
(NSAID)
 Correct Answer ImageE.Weight loss plus resistance training

A 70-year-old woman with a history of hypertension and hypercholesterolemia comes to the


physician because of worsening stable angina. She used to smoke one pack of cigarettes daily for
35 years but quit 5 years ago. Physical examination shows a thrill at the sternal notch on
palpation, an S4 on auscultation, and a crescendo-decrescendo systolic murmur in the
precordium. The murmur decreases with a Valsalva maneuver and upon standing, and increases
while squatting. Which of the following is the most likely diagnosis?
 Incorrect Answer ImageA.Aortic regurgitation
 Correct Answer ImageB.Aortic stenosis
 Incorrect Answer ImageC.Mitral regurgitation
 Incorrect Answer ImageD.Mitral stenosis
 Incorrect Answer ImageE.Pulmonary regurgitation
 Incorrect Answer ImageF.Pulmonary stenosis

A 35-year-old man comes to the physician because of headaches and difficulty with his vision
for 6 months. He often hits the curb while driving. He has had no fever, neck stiffness, loss of
consciousness, or seizures. He takes no medications. His temperature is 37ºC (98.6°F), blood
pressure is 120/70 mm Hg, pulse is 83/min, and respirations are 14/min. Neurologic examination
shows decreased vision bilaterally in the temporal fields. Muscle strength is 5/5 in all
extremities. Reflexes and sensation are normal. MRI of the brain shows a pituitary adenoma.
Which of the following tests is most likely to be abnormal in this patient? 
 Incorrect Answer ImageA.Adrenocorticotropic hormone (ACTH)
 Incorrect Answer ImageB.Growth hormone (GH)
 Correct Answer ImageC.Prolactin
 Incorrect Answer ImageD.Somatomedin C
 Incorrect Answer ImageE.Thyroid-stimulating hormone (TSH)

A 92-year-old man with a history of pulmonary emphysema is intubated in the intensive care
unit because of progressive pneumonia that has failed to improve after 72 hours of antibiotic
therapy. Although the inspired fraction of oxygen is 100%, the patient's pO2 remains at 57 mm
Hg. Positive end-expiratory pressure (PEEP) is added and twelve hours later the patient suddenly
becomes hypotensive with oxygen saturation dropping from 92% to 61%. His blood pressure is
80/50 mm Hg and pulse is 124/min. He has distended neck veins and distant heart sounds. Which
of the following would also most likely be seen on this patient's physical examination?
 Correct Answer ImageA.Absence of breath sounds on the affected side
 Incorrect Answer ImageB.High-amplitude carotid artery upstroke
 Incorrect Answer ImageC.Pleural friction rub
 Incorrect Answer ImageD.Pulsus alternans
 Incorrect Answer ImageE.Splenomegaly

A 19-year-old man comes to the physician for a follow-up visit 48 hours after the placement of a
purified protein derivative (PPD) tuberculin skin test. His PPD test showed 7 mm of induration
when he was tested 3 weeks ago. He has had no cough, fever, night sweats, weight loss, or sick
contacts. He emigrated from India 3 years ago and has not returned to India since then. He works
as a bartender. His medical history is unremarkable, and he takes no medications. He does not
smoke cigarettes and drinks alcohol socially. His temperature is 37.1°C (98.8°F), pulse is
78/min, respirations are 16/min, and blood pressure is 118/79 mm Hg. The lungs are clear to
auscultation. Cardiac examination shows a normal S1 and S2; no murmurs are heard. The PPD
that was placed 48 hours ago shows 13 mm of induration. Which of the following is the most
likely explanation for the increase in induration as compared to the previous test?
 Incorrect Answer ImageA.Active pulmonary tuberculosis
 Correct Answer ImageB.Booster phenomenon
 Incorrect Answer ImageC.Disseminated tuberculosis
 Incorrect Answer ImageD.False positive
 Incorrect Answer ImageE.Interval development of tuberculosis since his prior test

A 28-year-old woman with a history of asthma and seasonal allergies comes to the physician
because of a 7-kg (15-lb) weight gain. She has had no change in her diet and she walks 2 miles
daily. She has been on the same medications for several months which include cetirizine,
prednisone, and inhaled fluticasone-salmeterol. The combined inhaler was added 2 months ago
due to frequent albuterol use. She has had no cough, wheeze or shortness of breath, and has not
used her PRN albuterol for 4 weeks. She is 160 cm (63 in) tall, and she weighs 86 kg (190 lb).
Her temperature is 37.1ºC (98.8ºF), blood pressure is 150/75 mm Hg, pulse is 90 /min, and
respirations are 16/min. The lungs are clear to auscultation. The remainder of the examination
shows no abnormalities. Which of the following is the most appropriate next step in
management? 
 Incorrect Answer ImageA.Discontinue inhaled budesonide and re-evaluate in 1 month
 Incorrect Answer ImageB.Discontinue prednisone and re-evaluate in 1 month
 Incorrect Answer ImageC.High-dose dexamethasone suppression test
 Incorrect Answer ImageD.Low-dose dexamethasone suppression test
 Incorrect Answer ImageE.Measure 24-hr urinary excretion of cortisol
 Incorrect Answer ImageF.Measure morning serum cortisol
 Correct Answer ImageG.Taper the dose of prednisone and re-evaluate in 1 month
A 32-year-old man comes to the physician because of a 2-week history of fecal urgency, a
sensation of incomplete rectal evacuation, and bloody diarrhea that occurs up to 12 times daily.
He has had abdominal cramps in his left lower abdomen. He has not traveled outside the country.
He has a history of Pneumocystis jiroveci pneumonia 8 months ago. He does not drink alcohol.
He has smoked a pack of cigarettes daily for 14 years. He used heroin daily for 8 years, but quit
5 years ago. Which of the following is the most likely causal organism?
 Incorrect Answer ImageA.Clostridium difficile 
 Incorrect Answer ImageB.Cryptosporidium parvum 
 Correct Answer ImageC.Cytomegalovirus
 Incorrect Answer ImageD.Entamoeba histolytica 
 Incorrect Answer ImageE.Enterotoxigenic Escherichia coli

A 31-year-old woman with HIV is brought to the emergency department by ambulance


accompanied by her husband after a seizure that lasted 2 minutes. She has had headaches and
fevers for the past 2 months. Her husband also reports that she "has not been herself lately."
Current medications include emtricitabine, tenofovir, and efavirenz. On arrival, she is alert and
oriented. Her temperature is 38.5ºC (101.3ºF). Physical examination shows no abnormalities. Her
CD4+ T lymphocyte count is 68/µL. A CT scan of the head is shown. The lesions and symptoms
respond to a 6-week course of pyrimethamine and sulfadiazine. Which of the following is the
most appropriate next step in management?
 Incorrect Answer ImageA.Azithromycin
 Incorrect Answer ImageB.Biopsy of one of the lesions
 Incorrect Answer ImageC.Change antiviral medications to emtricitabine, tenofovir, and
raltegravir
 Correct Answer ImageD.Lower the doses of pyrimethamine and sulfadiazine
 Incorrect Answer ImageE.Radiation and corticosteroids

 30-year-old woman with a history of asthma comes to the physician because of a low-grade
fever and cough productive of whitish sputum with brown-colored cords in the phlegm for 2
weeks. She has completed a 5-day course of azithromycin with no improvement. She has had no
hemoptysis, weight loss, chest pain, or leg swelling. Current medications include inhaled
salmeterol and fluticasone. She has no allergies. She has not had any recent travel. She smokes
marijuana daily and does not use any other illicit drugs. Her temperature is 37.3ºC (99.1ºF). The
lungs are clear to auscultation. Laboratory studies show: 
Leukocyte count 13,000/mm3
Segmented neutrophils 44%
Eosinophils 20%
Lymphocytes 30%
Monocytes 6% 
A radiograph of the chest is shown. Which of the following is the most likely diagnosis?
 Incorrect Answer ImageA.Acute eosinophilic pneumonia
 Correct Answer ImageB.Allergic bronchopulmonary aspergillosis
 Incorrect Answer ImageC.Churg-Strauss syndrome
 Incorrect Answer ImageD.Löffler syndrome
 Incorrect Answer ImageE.Mycoplasma pneumonia
 Incorrect Answer ImageF.Psittacosis

Four friends go on a camping trip in the desert but one of them gets separated from the group. He
is found and rescued 3 days later. When brought to the emergency department he is awake and
alert, and reports muscle weakness and cramps. His vital signs include blood pressure 100/70
mm Hg, pulse 110/min, and respirations 19/min. Physical examination shows decreased skin
turgor and dry mucous membranes. Laboratory results show Na+ 152 mEq/L, K+ 5.2 mEq/L,
and Cl− 107 mEq/L. After providing a bolus of 1 liter of normal saline, which of the following
would be the best IV maintenance fluid to rehydrate this patient?
 Correct Answer ImageA.0.45% sodium chloride
 Incorrect Answer ImageB.0.9% sodium chloride
 Incorrect Answer ImageC.3% sodium chloride
 Incorrect Answer ImageD.5% dextrose
 Incorrect Answer ImageE.Ringer's lactate

 50-year-old man comes to the emergency department because of a 2-week history of dull
abdominal pain. He reports a gnawing periumbilical pain that has no association with food
intake. He denies fever, nausea, vomiting, changes in stool color, diarrhea, and constipation. He
has no history of significant medical illnesses, and he does not take any medications. Vital signs
are normal, and physical examination shows mild tenderness to palpation over the periumbilical
region. Laboratory studies are unremarkable. A CT scan of the abdomen shows a 1.5 cm
hypodense, homogenous, right adrenal nodule with smooth, rounded contours. Which of the
following is the most appropriate next step in management of the adrenal nodule?
 Correct Answer ImageA.Assure the patient that the adrenal nodule does not appear to be
malignant
 Incorrect Answer ImageB.Fine-needle aspiration biopsy
 Incorrect Answer ImageC.Open biopsy
 Incorrect Answer ImageD.Serial CT scan of the abdomen every 6 months for the next 3
years
 Incorrect Answer ImageE.Surgery for removal of the right adrenal gland
A heterosexual 52-year-old man has had mouth dryness and an unpleasant taste for 3 weeks. He
has no other complaints. He denies pain on swallowing. He reports that he has no chronic
medical conditions, takes no medications, and that he is "surprisingly healthy" for his age
compared with his friends. He plays racquetball on the weekends and lifts weights during the
week. He drinks a glass of wine with dinner and does not smoke cigarettes. He works as a
salesman for a large computer company and often attends business conferences in Asia and
Africa. He was married for 25 years, but his wife died from breast cancer 3 years ago. He has 2
grown children who now have children of their own. He enjoys spending time with his
grandchildren and says that they are the reason he needs to stay healthy. His mother and father
both died from complications of diabetes mellitus. His temperature is 37.7ºC (99.9ºF). An image
of his mouth is shown. The lesions are easily rubbed off with a tongue depressor. The remainder
of the examination is unremarkable. Nonfasting blood glucose level is 103 mg/dL. Which of the
following is the most appropriate management?
 Incorrect Answer ImageA.Prescribe oral acyclovir and recommend HIV testing
 Incorrect Answer ImageB.Prescribe oral acyclovir and reevaluate in 2 to 4 weeks
 Incorrect Answer ImageC.Prescribe clotrimazole troches and determine fasting blood
glucose level
 Correct Answer ImageD.Prescribe clotrimazole troches and recommend HIV testing
 Incorrect Answer ImageE.Prescribe clotrimazole troches and reevaluate in 2 to 4 weeks
for HIV testing

A 60-year-old man comes to the emergency department because of a 2-day history of severe
abdominal pain. He denies nausea, vomiting, or diarrhea. He does not drink alcohol or smoke
cigarettes. He has not traveled outside of the country and has no sick household contacts.
Examination shows mild jaundice. The liver has a span of 14 cm and is tender and smooth on
palpation. Laboratory studies show: 
Hemoglobin 20.1 g/dL
Hematocrit 63%
Leukocyte count 11,800/mm3
Platelet count 476,000/mm3
Serum studies show:
Aspartate aminotransferase (AST, GOT)  412 U/L
Alanine aminotransferase (ALT, GPT)  622 U/L
Peripheral smear shows normal-appearing morphology of the cells. Which of the following is the
most likely diagnosis? 
 Incorrect Answer ImageA.Alcoholic cirrhosis
 Correct Answer ImageB.Budd-Chiari syndrome
 Incorrect Answer ImageC.Cholelithiasis
 Incorrect Answer ImageD.Chronic hepatitis B
 Incorrect Answer ImageE.Fulminant hepatitis A
A 71-year-old man comes to the physician because of a 2-month history of episodes of severe
pain over the right jaw that lasts for 60 seconds. The pain is sudden in onset, "shock-like" and
comes on after brushing his teeth or chewing tough food. He has a history of hypertension, and
he had a myocardial infarction 4 years ago. Current medications include aspirin, metoprolol,
lisinopril, and atorvastatin. His blood pressure is 132/84 mm Hg and pulse is 64/min.
Examination shows no abnormalities. Laboratory studies show:
 Hemoglobin  14.2 mg/dL
 Leukocyte count  7,800/mm3
 Platelet count  308,000/mm3
 Erythrocyte sedimentation rate  9 mm/hr
 Which of the following is the most appropriate pharmacotherapy? 
 Incorrect Answer ImageA.Baclofen
 Correct Answer ImageB.Carbamazepine
 Incorrect Answer ImageC.Indomethacin
 Incorrect Answer ImageD.Prednisone
 Incorrect Answer ImageE.Sumatriptan

A 72-year-old man was admitted to the hospital 10 days ago because of a cerebrovascular
accident. His past medical history includes hypertension, coronary artery disease, and non-
insulin-dependent diabetes mellitus. The patient's neurologic condition slowly improved, but on
hospital day 8, he developed cough, shortness of breath, and fever. Chest x-ray at that time
showed an opacity in the right middle lobe. The patient was diagnosed with hospital-acquired
pneumonia, and sputum samples are obtained to assess the etiology. Gram stain showed multiple
leukocytes and gram-negative coccobacilli. He was started on levofloxacin. Despite three days of
treatment with levofloxacin, his condition has failed to improve. Culture results
show Acinetobacter baumannii. Drug sensitivity test results are pending and will be available
within the next 24 hours. Which of the following is the most appropriate next step in
management? 
 Incorrect Answer ImageA.Add cefepime to current treatment until antibiogram is
available and modify therapy accordingly
 Incorrect Answer ImageB.Continue levofloxacin until antibiogram is available and
modify therapy accordingly
 Incorrect Answer ImageC.Switch to ceftriaxone
 Incorrect Answer ImageD.Switch to ciprofloxacin
 Correct Answer ImageE.Switch to imipenem

A 20-year-old man with a history of nephrotic syndrome undergoes a renal biopsy. He came to
his physician 1 week ago with generalized edema and foamy urine without hematuria. His blood
pressure is 100/60 mm Hg. Examination shows periorbital edema, with clear lungs, a normal
cardiac examination, and 4+ edema and anasarca. His 24-hour urine contains 8 grams of protein
and his serum creatinine is 0.8 mg/dL. His urinalysis shows 4+ protein but no red cells or casts.
Electron microscopic evaluation of the renal biopsy shows that the basement membrane of the
glomeruli is morphologically normal, with no electron-dense material deposition. In the visceral
epithelial cells (podocytes), there is a uniform and diffuse effacement of foot processes. Which
of the following is the most appropriate pharmacotherapy?
 Incorrect Answer ImageA.Cyclophosphamide
 Incorrect Answer ImageB.Mycophenolate mofetil
 Incorrect Answer ImageC.Observation
 Correct Answer ImageD.Prednisone
 Incorrect Answer ImageE.Rituximab

A 50-year-old man comes to the clinic for evaluation of an elevated fasting blood glucose level,
which was discovered as part of a preoperative evaluation for bilateral carpal tunnel surgery. He
also reports a history of hypertension and nonspecific arthralgias of the knees, ankles, and hips.
A review of systems demonstrates that over the past year, he has experienced significant
headaches, excessive diaphoresis, and impotence. He states that his appearance has changed
remarkably, which he noticed while watching his friend's wedding video. He says he now looks
much older. He also noticed recent changes in his facial appearance and says he wears a larger
hat size. Current medications include hydrochlorothiazide and a daily aspirin. His pulse is
62/min and blood pressure is 155/75 mm Hg. Physical examination shows coarse skin, skin tags,
and large, fleshy hands and feet. Laboratory tests are ordered to confirm the likely diagnosis.
Which of the following is most likely to be elevated?
 Incorrect Answer ImageA.Alkaline phosphatase
 Incorrect Answer ImageB.Cortisol
 Correct Answer ImageC.IGF-1
 Incorrect Answer ImageD.Prolactin
 Incorrect Answer ImageE.Somatostatin
 Incorrect Answer ImageF.Thyroid-stimulating hormone (TSH)

A 52-year-old woman comes to the urgent care clinic with difficulty breathing. She has a history
of bronchial asthma and has suffered with this disease since an early age. For the past years she
has had to use her short-acting bronchodilator inhaler approximately 3 or 4 times per month.
Over the past 2 months, however, she has had to use the inhaler 3 times per week. She has no
other past medical history and takes no medications besides her inhaler. She denies smoking or
alcohol use. On physical examination, you note her appearance (click media file) and
auscultation shows diffuse wheezes. No abnormal heart sounds are heard on auscultation. The
remainder of her physical examination is normal. She is given a nebulizer treatment, which
improves her acute symptoms. Which of the following is the next step in management?
 Correct Answer ImageA.Add an inhaled corticosteroid to her regimen
 Incorrect Answer ImageB.Add theophylline to her regimen
 Incorrect Answer ImageC.Administer subcutaneous epinephrine
 Incorrect Answer ImageD.Change her short-acting beta agonist to a long-acting one
 Incorrect Answer ImageE.Encourage more frequent inhaler use

A 45-year-old woman comes to the physician because of progressive weakness of 4 months


duration. She has had difficulty climbing stairs and lifting objects over her head. She had a fever
up to 38.5ºC (101.3ºF). Her medical history is otherwise unremarkable. She takes no
medications. She drinks alcohol on social occasions and does not smoke cigarettes. Physical
examination shows palpable proximal muscle tenderness in the upper arm and lower extremities
without atrophy. There is periorbital edema and an erythematous rash on the eyelids. An
photograph of her hands is shown. Laboratory studies show:
Hemoglobin 13.2 g/dL
Leukocyte count  9,300/mm3
Creatine kinase 2,550 U/L
Erythrocyte sedimentation rate (ESR)  22 mm/hr
Antinuclear antibody (ANA) titer  Positive at 1:1280
Rheumatoid factor  Positive at 1:512
Which of the following is the most likely diagnosis?
 Correct Answer ImageA.Dermatomyositis
 Incorrect Answer ImageB.Mixed connective tissue disease
 Incorrect Answer ImageC.Polymyalgia rheumatica
 Incorrect Answer ImageD.Psoriatic arthritis
 Incorrect Answer ImageE.Scleroderma

A 47-year-old man comes to the physician because of pain when he urinates. He has been
urinating more often than usual and has now developed burning on urination. The pain started
vaguely in the pelvis. He denies any experiences of similar episodes in the past. His temperature
is 38.5°C (101.3°F). Blood pressure and pulse are within normal limits. His abdomen is non-
distended and is soft to palpation with no tenderness or palpable masses. Bowel sounds are
normoactive. On genital examination, there is no urethral discharge and testes are non-tender.
Digital rectal examination causes a significant amount of pain and is unable to be adequately
completed. Which of the following is the most likely diagnosis?
 Incorrect Answer ImageA.Anal fissure
 Incorrect Answer ImageB.Cystitis
 Incorrect Answer ImageC.Epididymitis
 Correct Answer ImageD.Prostatitis
 Incorrect Answer ImageE.Urethriti

A 53-year-old man with a longstanding history of alcohol abuse sees his physician for a 2 to 3-
month history of progressive malaise, increased abdominal girth, weight loss, decreased appetite,
and yellow eyes. He has not seen a physician in several years. He smokes one pack of cigarettes
per day and consumes approximately 5 to 6 beers per day. He used intravenous drugs
approximately 20 years ago but denies current use. He takes no medications. His pulse is 95/min
and his blood pressure is 101/67 mm Hg. On physical examination, he has temporal wasting,
scleral icterus, and jaundice. He has scattered spider angiomata, gynecomastia, palmar erythema,
abdominal fluid wave, a nodular liver edge extending 3 cm below the right costal margin and
splenomegaly. An abdominal ultrasound shows a small liver and ascitic fluid. Which of the
following is most likely to be found in the ascites fluid?
 Incorrect Answer ImageA.Elevated amylase
 Incorrect Answer ImageB.Elevated triglycerides
 Incorrect Answer ImageC.Lymphocytosis
 Incorrect Answer ImageD.Neutrophilia

 Correct Answer ImageE.Serum-ascites albumin gradient >1.1

A 20-year-old pregnant woman is experiencing rapidly progressive hearing loss. Several


relatives in her family developed hearing loss prior to age 25. Otoscopic examination shows
normal tympanic membranes bilaterally. Hearing testing shows a conductive hearing loss, with
one ear being affected more severely than the other. Which of the following is the most likely
diagnosis?
 Incorrect Answer ImageA.Acoustic neuroma
 Incorrect Answer ImageB.Chronic otitis media
 Incorrect Answer ImageC.Meniere disease
 Correct Answer ImageD.Otosclerosis
 Incorrect Answer ImageE.Presbycusis

A 46-year-old man comes to the physician because of an itchy rash in his groin for 3 weeks. He
experienced no relief after using an over-the-counter antifungal cream. Past medical history is
significant for type 2 diabetes mellitus controlled with diet and essential hypertension treated
with losartan. He has no known allergies to medications and does not drink or smoke. Physical
examination shows sharply demarcated, dry, brown, slightly scaly patches bilaterally in both
genitocrural folds. There are similar but smaller patches in the axillae and in the fourth
interdigital space on the feet. A Woods light examination shows coral-red fluorescence in all
three locations. Which of the following is the most likely diagnosis?
 Correct Answer ImageA.Erythrasma
 Incorrect Answer ImageB.Intertrigo
 Incorrect Answer ImageC.Inverse psoriasis
 Incorrect Answer ImageD.Seborrheic dermatitis
 Incorrect Answer ImageE.Tinea cruris

A 55-year-old man comes to the physician because he "bleeds too much" every time he brushes
his teeth. For the past three days, he has not been feeling well and has been also complaining of
headaches and chest pain. He has been trying to workout lately but as soon as he starts walking,
he feels very fatigued and has to stop. His temperature is 39ºC (102ºF), pulse is 110/min,
respirations are 20/min, and blood pressure is 130/80 mm Hg. There is a 2/6 systolic ejection
murmur heard on auscultation. Lung examination shows bilateral rhonchi. There are scattered
petechiae over the thorax and extremities. The patient is oriented to person, time, and place, but
has 4/5 strength throughout with normal reflexes. Laboratory studies show: 
Hemoglobin  7.1 gm/dL
Platelets  30,000/mm3
White blood cells  11,800/mm3
Peripheral blood numerous blasts
Bone marrow biopsy hypercellular with 35% blasts
Blast cells presence of rod-shaped structures in the cytosol 
Myeloperoxidase positive
Chest X-ray scattered infiltrations on the base, bilaterally 
Which of the following is the most likely diagnosis?
 Incorrect Answer ImageA.Acute lymphocytic leukemia
 Correct Answer ImageB.Acute myelogenous leukemia
 Incorrect Answer ImageC.Aplastic anemia
 Incorrect Answer ImageD.Chronic myelogenous leukemia
 Incorrect Answer ImageE.Myelofibrosis

A 15-year-old boy comes to the emergency department because of nasal bleeding from the left
nostril for 20 minutes. He has had no history of trauma to his nose or face. He has had episodes
of epistaxis for the past 4 months that are increasing in severity and frequency. He also has had
decreased hearing, nasal congestion, and rhinorrhea with occasional blood-tinged nasal discharge
from his left nostril. His temperature is 37°C (98.6°F), blood pressure is 110/75 mm Hg, and
pulse is 68/min. Examination with bedside nasopharyngoscopy shows a firm, grayish-red mass in
the left nasopharynx. There are no skin lesions. Which of the following is the most likely
diagnosis?
 Incorrect Answer ImageA.Inverted papilloma
 Correct Answer ImageB.Juvenile nasopharyngeal angiofibroma
 Incorrect Answer ImageC.Nasopharyngeal carcinoma
 Incorrect Answer ImageD.Pyogenic granuloma
 Incorrect Answer ImageE.Rhabdomyosarcoma

A 37-year-old man with a history of type 1 diabetes mellitus and hypertension comes to the
physician because of erectile dysfunction. He reports that two weeks ago, after heavy eating and
heavy drinking, he was unable to get an erection when attempting intercourse with his wife. He
has remained impotent since then. He does report having an erection in the morning when he
wakes up. There is no history of perineal trauma or pelvic surgery. His diabetes is well controlled
with insulin. He was recently started on gemfibrozil and metoprolol for hypertriglyceridemia and
hypertension, respectively. His testosterone level is 557 ng/dL (normal, 437–707 ng/dL). Which
of the following is the most likely underlying cause of this patient's current complaint? 
 Incorrect Answer ImageA.Atherosclerosis
 Incorrect Answer ImageB.Diabetes mellitus
 Incorrect Answer ImageC.Gemfibrozil
 Incorrect Answer ImageD.Metoprolol
 Correct Answer ImageE.Psychogenic causes

One week after an upper respiratory infection, an adult develops conjunctival hyperemia, watery
discharge, and ocular irritation. While both eyes are involved by the time a physician is
consulted, the symptoms began in one eye. On the morning of the doctor's visit, the patient had
difficulty opening his eyelids on awakening as they were "glued shut". Physical examination
demonstrates hyperemic bulbar and tarsal conjunctiva. There is a watery, non purulent discharge
noted. The right preauricular lymph node is enlarged on palpation. Which of the following
pathogens would most likely cause these symptoms?
 Correct Answer ImageA.Adenovirus
 Incorrect Answer ImageB.Herpes simplex I
 Incorrect Answer ImageC.Herpes simplex II
 Incorrect Answer ImageD.Neisseria gonorrhoeae
 Incorrect Answer ImageE.Staphylococcus aureus

A 73-year-old woman comes to the physician because of a 6-month history of an enlarging dark
skin lesion on her right temple. She had a basal cell carcinoma removed from her back 2 years
ago. Her past medical history is also significant for type 2 diabetes mellitus and essential
hypertension. Current medications include metformin, pentoxifylline, and nifedipine. Physical
examination shows a 2 x 3 cm, dark brown, waxy, keratotic plaque with a verrucous surface
punctuated by multiple pseudocysts on the right forehead. It appears to be “pasted” on her skin.
Gentle scraping of the surface causes keratotic debris to dislodge. Which of the following is the
most likely diagnosis?
 Incorrect Answer ImageA.Actinic keratosis
 Incorrect Answer ImageB.Basal cell carcinoma
 Incorrect Answer ImageC.Melanoma
 Correct Answer ImageD.Seborrheic keratosis
 Incorrect Answer ImageE.Solar lentigo

A 40-year-old woman comes to the physician for a routine health maintenance examination. Her
medical history is unremarkable and she takes no medications. She does not smoke cigarettes or
drink alcohol. She works as a librarian. She is not sexually active. Her sister had breast cancer at
age 44, and her father died of colon cancer at age 63. Her temperature is 37°C (98.6°F), pulse is
78/min, respirations are 18/min, and blood pressure is 120/68 mm Hg. Physical examination
shows no abnormalities. Which of the following screening tests is most appropriate in this
patient? 
 Incorrect Answer ImageA.CA-125 level
 Incorrect Answer ImageB.Fecal occult blood testing
 Incorrect Answer ImageC.Lipid panel
 Correct Answer ImageD.Mammography
 Incorrect Answer ImageE.Ovarian ultrasound

A 34-year-old woman with no significant medical history who is healthy without underlying
medical problems comes to the clinic with complaints of fever with a temperature up to 38.3ºC
(101.0ºF) and cough with greenish sputum production for 2 days without dyspnea. Her blood
pressure is 120/80 mm Hg, pulse is 88/min, and respirations are 18/min. There is no accessory
muscle use, conversational dyspnea, wheezes, bronchial breath sounds, rales, or egophony over
the right lower lung fields. Chest radiograph shows a right lower lobe consolidation. Leukocyte
count is 13,000/mm3. Which of the following is the most appropriate pharmacotherapy?
 Correct Answer ImageA.Amoxicillin plus azithromycin
 Incorrect Answer ImageB.Ampicillin-sulbactam plus doxycycline
 Incorrect Answer ImageC.Ceftriaxone plus azithromycin
 Incorrect Answer ImageD.Ciprofloxacin plus piperacillin-tazobactam
 Incorrect Answer ImageE.Clarithromycin

A 37-year-old woman comes to the emergency department because of difficulty with her vision.
She has noticed symptoms of "clear things floating in front of [her] eyes" and "flashing lights"
over the past 2 days. She states that she was diagnosed with HIV several years ago, but has not
seen a physician since the diagnosis. Her appetite has been poor recently, and she has noticed
significant weight loss. She is afebrile, pulse is 90/min, blood pressure is 105/62 mm Hg, and
oxygen saturation is 96% on room air. Head and neck examination shows cervical
lymphadenopathy. Her chest is clear to auscultation, and heart sounds are normal. Peripheral
examination shows poor skin turgor without cyanosis, clubbing, or edema. Which of the
following is the most appropriate next step in management?
 Correct Answer ImageA.Dilated funduscopic examination
 Incorrect Answer ImageB.Intravenous foscarnet
 Incorrect Answer ImageC.Intravenous ganciclovir
 Incorrect Answer ImageD.Magnetic resonance imaging of the brain
 Incorrect Answer ImageE.Ocular ganciclovir implantation

A 24-year-old HIV-positive man who works at a homeless shelter comes to the emergency
department because of a cough and weight loss. He is given a provisional diagnosis
of pneumocystis jiroveci pneumonia (PJP) due to a low CD4 count and lack of antimicrobial
prophylaxis. He is treated with IV trimethoprim/sulfamethoxazole and prednisone. Despite
treatment, his condition progresses. The patient is transferred to a negative-pressure room and
sputum samples are collected for acid-fast bacillus (AFB) smear analysis and culture. He is
found to have active pulmonary tuberculosis and is started on a 4-drug regimen of isoniazid,
pyrazinamide, rifampin, and ethambutol, as well as pyridoxine. After appropriate inpatient
treatment he is discharged with close follow-up planned in the outpatient clinic. The patient
returns to the clinic 2 weeks later and is prescribed highly active antiretroviral therapy (HAART)
for the treatment of his human immunodeficiency (HIV). At that time he also reports sudden
blurred or “foggy” vision and pain that is worse with movement of the right eye. Visual testing
shows a visual acuity of 20/25 OS and 20/200 OD, with intact peripheral vision. Which of the
following is the most likely cause of this patient's symptoms?
 Correct Answer ImageA.Ethambutol
 Incorrect Answer ImageB.Isoniazid
 Incorrect Answer ImageC.Pyrazinamide
 Incorrect Answer ImageD.Pyridoxine
 Incorrect Answer ImageE.Rifampin

A 70-year-old woman with aortic stenosis is admitted to the hospital with chest pain. A
myocardial infarction is ruled out by cardiac enzymes, but the patient has recurrent symptoms
while being weaned off heparin. On hospitalization day 5, she has right arm pain, absent brachial
pulse on the right, and a cold distal right arm. Her hematocrit is 34% and platelets are
30,000/mm3. Her partial thromboplastin time is 64 seconds. Which of the following is the most
likely cause of this patient's absent brachial pulse? 
 Incorrect Answer ImageA.Embolization from aortic stenosis
 Correct Answer ImageB.Heparin-induced thrombocytopenia
 Incorrect Answer ImageC.Hypercoagulable state from immobilization
 Incorrect Answer ImageD.Paradoxical embolus
 Incorrect Answer ImageE.Vasospasm of the brachial artery

A 17-year-old boy is brought to the emergency department by his parents after they found him
wandering aimlessly in the street. His parents say that 1 day ago his friends dared him to drink a
bottle of windshield wiper fluid. The patient appears ill, is unable to speak coherently, and
complains of blurry vision. His temperature is 36.6ºC (98ºF), blood pressure is 110/70 mm Hg,
pulse is 100/min, and respirations are 24/min. Physical examination shows a poorly groomed boy
who is disheveled. Breath sounds are clear bilaterally. Cardiac examination shows a normal S1
and S2 with no murmurs. The abdomen is diffusely tender to palpation with hypoactive bowel
sounds. Neurologic examination shows dilated pupils. Laboratory studies show:
Na+ 140 mEq/L
K+ 3.0 mEq/L
Cl -
100 mEq/L
HCO3- 10 mEq/L
BUN  15 mg/dL
Creatinine  1.0 mg/dL
Arterial blood gas analysis is most likely to show which of the following? 

 Incorrect Answer ImageA.


 Correct Answer ImageB.
 Incorrect Answer ImageC.
 Incorrect Answer ImageD.
 Incorrect Answer ImageE.

A 35-year-old woman comes to the physician because of a 3-day history of fever and confusion.
She has no past medical history and does not smoke or use alcohol. She takes no medications.
Her temperature is 38.5ºC (101.3ºF), pulse is 98/min, and blood pressure is 110/75 mm Hg. She
is alert and oriented to person only. It is difficult to engage her in a focused conversation. There
are scattered petechiae and ecchymosis over the chest and both lower extremities. There is no
neck stiffness and no localizing neurologic signs. Laboratory studies show:
Hematocrit 21%
Hemoglobin 7.2 gm/dL
WBC 9,000/mm3
Platelets 13,000/mm3
MCV 89 µm3
Reticulocyte count 5%
Creatinine 1.8 mg/dL
AST 24 U/L
ALT 25 U/L
LDH 648 U/L
Total bilirubin 2.2 mg/dL
Prothrombin time 12 sec
Partial thromboplastin time 30 sec
A peripheral smear shows schistocytes and decreased platelets. Which of the following is the
most likely diagnosis? 
 Incorrect Answer ImageA.Disseminated intravascular hemolysis
 Incorrect Answer ImageB.HELLP syndrome
 Incorrect Answer ImageC.Hemolytic uremic syndrome
 Incorrect Answer ImageD.Idiopathic thrombocytopenic purpura
 Correct Answer ImageE.Thrombotic thrombocytopenic purpura

A 43-year-old man is admitted to the hospital with significant shortness of breath. The patient
was previously diagnosed with interstitial pulmonary disease, based on imaging studies and
pulmonary function testing. Despite multiple tests, the exact etiology of the patient's condition is
not established. Due to his worsening condition, bronchoscopy with bronchoalveolar lavage
(BAL) is performed. BAL shows the presence of big cells with pentalaminar in-foldings of the
cell membrane (Birbeck granules). Which of the following is the most likely diagnosis in this
patient? 
 Incorrect Answer ImageA.Alveolar proteinosis
 Correct Answer ImageB.Pulmonary Langerhans cell histiocytosis
 Incorrect Answer ImageC.Sarcoidosis
 Incorrect Answer ImageD.Tuberculosis
 Incorrect Answer ImageE.Granulomatosis with polyangiitis

A 37-year-old homeless man with schizophrenia comes to the hospital because of a dry cough,
hemoptysis, weakness, fever, sweating, and weight loss. He came voluntarily, wanting to get
"something for the cough." However, after necessary studies are performed, he is admitted
because of the high possibility of having active tuberculosis. On the unit, he is started on anti-
tuberculosis medication and an antipsychotic. Initially, he is cooperative. After 2 days, however,
he shouts at the nurses and demands to be discharged immediately. Conversations with
physicians and staff to persuade him that he needs treatment are not helpful and he threatens to
sign himself out of the hospital against medical advice. Which of the following facts should most
likely lead the physician to justify involuntary commitment?
 Incorrect Answer ImageA.Patient has a potentially life-threatening medical condition
 Correct Answer ImageB.Patient has tuberculosis
 Incorrect Answer ImageC.Patient is agitated
 Incorrect Answer ImageD.Patient is homeless and unable to keep follow-up appointments
 Incorrect Answer ImageE.Patient is schizophrenic

A 39-year-old African-American man has had a 16-pound weight gain along with progressively
worsening massive lower extremity edema and fatigue over the last 2 weeks. His urine is normal
colored but foamy. He denies any recent skin or throat infections, diabetes, or carpal tunnel
syndrome. He takes no medications and does not use illicit drugs. There is no personal or family
history of cancer and no history of urinary infection. His temperature is 37.0ºC (98.6ºF), pulse is
93/min, respirations are 18/min, and blood pressure is 170/70 mm Hg. Prior to the onset of the
present illness, his BMI was 22 kg/m2. Physical examination shows massive upper and lower
extremity edema together with sacral edema. His lungs have decreased breath sounds at the bases
bilaterally. Cardiac and abdominal examinations are within normal limits. He has no retinopathy.
Fasting laboratory studies show:
Sodium 130 mEq/L
Potassium 4.5 mEq/L
Chloride 100 mEq/L
Bicarbonate 24 mEq/L
Glucose 96 mg/dL
BUN 27 mg/dL
Creatinine 1.7 mg/dL
Albumin 1.7 g/dL
LDL cholesterol 450 mg/dL
Hemoglobin 12 mg/dL
HCT 31%
WBC 7,700/mm3
Platelets 270,000/mm3
Urinalysis:
Leukocyte esterase Negative
Nitrite Negative
Protein ++++
Blood Negative
24-h protein collection 8.3 g/24 h
Tests for HIV antibody and sickle hemoglobin are negative. Biopsy of the kidney would most
likely reveal which of the following findings on light microscopy? 
 Incorrect Answer ImageA.Diffuse interstitial infiltration with inflammatory cells
(tubulointerstitial nephritis)
 Correct Answer ImageB.Glomerulosclerosis involving segments of some glomeruli with
other glomeruli appearing normal (focal segmental glomerulosclerosis)
 Incorrect Answer ImageC.Kimmelstiel-Wilson lesions
 Incorrect Answer ImageD.Normal appearance
 Incorrect Answer ImageE.Normal glomerular cellularity, thickened glomerular capillary
walls (membranous nephropathy)

A 70-year-old woman with a history of congestive heart failure, atrial fibrillation, and dementia
comes to the physician for a routine health maintenance examination. Current medications
include digoxin, warfarin, fosinopril, aspirin, and furosemide. Her blood pressure is 124/82 mm
Hg, and her pulse is 76/min and irregular. The lungs are clear to auscultation. There is 1+ pedal
edema. Her international normalized ratio (INR) is 2.3. Her husband is taking herbal medications
and she inquires about herbal supplements. Which of the following herbal supplements is most
likely to increase this patient's risk for bleeding? 
 Correct Answer ImageA.Dong quai
 Incorrect Answer ImageB.Echinacea
 Incorrect Answer ImageC.Kava kava
 Incorrect Answer ImageD.Psyllium
 Incorrect Answer ImageE.St. John's wort

A 41-year-old non-insulin dependent patient with diabetes comes to the office because of a
strange taste and pain in her mouth. She has noticed these symptoms over the past week and they
seem to be getting worse. Aside from diabetes, her past medical history includes asthma since
childhood that she is able to control with an albuterol rescue inhaler and a twice daily
maintenance steroid inhaler. Review of systems and family history are unremarkable. Her
temperature is 37.0ºC (98.6ºF), pulse 73/min, respirations 20/min, and blood pressure 128/79
mmHg. Oral examination shows white patches on her tongue and a generally erythematous
mucosa. With a tongue depressor, some of the material is easily removed, leaving a raw,
bleeding surface. Physical examination is otherwise unremarkable. Which of the following is the
most appropriate initial step in management?
 Correct Answer ImageA.Clotrimazole troche
 Incorrect Answer ImageB.Discontinuation of steroid inhaler
 Incorrect Answer ImageC.Hemoglobin A1c
 Incorrect Answer ImageD.Oral fluconazole
 Incorrect Answer ImageE.Rinse the mouth with water after using the inhaler

A 26-year-old woman has a history of type 2 diabetes controlled with sitagliptin. Her
hemoglobin A1c level is 6.3%. The woman's physician gets a call from another physician
identifying herself as the physician of the patient's brother. Apparently, the patient's brother also
has a history of type 2 diabetes that is poorly controlled. The brother's physician asks if the
sister's physician would discuss the details of the sister’s treatment so she can recommend it to
the brother. Which of the following is the most appropriate response by the sister's physician?
 Incorrect Answer ImageA.Offer to meet with the brother's physician to discuss the sister's
treatment regimen
 Incorrect Answer ImageB.Offer to send a letter to the brother's physician, detailing the
treatment plan for the sister
 Incorrect Answer ImageC.Offer to send the brother's physician literature about the
benefits and risks of the sister's drug regimen
 Incorrect Answer ImageD.Provide the details of the sister's treatment over the phone to
the brother's physician
 Correct Answer ImageE.Refuse to provide any information to the brother's physician
without the sister's permission

A 67-year-old woman comes to the physician with fever and back pain. She states that five days
ago she developed burning pain with urination, followed by fever and chills. She has a history of
hypertension well controlled with enalapril. There is no history of diabetes mellitus. Her
temperature is 38.9ºC (102ºF), pulse is 102/min, respirations are 16/min, and blood pressure is
90/70 mm Hg. The examination shows mild distress with rigors and right flank tenderness. The
rest of the exam is normal. The blood leukocyte count is 16,000/mm3. Urinalysis shows 50
leukocytes/high-power field and many bacteria. She is admitted to the hospital and given
intravenous saline and ciprofloxacin. Which of the following clinical findings indicates the need
for an abdominal CT or ultrasound study in this patient? 
 Incorrect Answer ImageA.Abdominal pain
 Incorrect Answer ImageB.Costovertebral tenderness
 Incorrect Answer ImageC.Fever
 Correct Answer ImageD.Hypotension
 Incorrect Answer ImageE.Urine white blood cell cast

A 42-year-old woman comes to the emergency department because of double vision and
difficulty keeping her right eye open for 1 day. She had a severe migraine 2 days ago. She has
had no recent fever or seizure. She has a history of type 2 diabetes mellitus and migraine
headaches. Her mother died of a myocardial infarction at age 54, and her brother has renal
failure. Examination shows a droopy right eyelid and an inability to elevate, adduct, and depress
the right eye. The right pupil also appears dilated. Examination of the left eye shows no
abnormalities. Visual acuity is normal bilaterally. Hemoglobin A1c level is 9.4%. Which of the
following is the most likely cause of her visual complaints? 
 Incorrect Answer ImageA.Cavernous sinus thrombosis
 Incorrect Answer ImageB.Diabetic microvascular infarct
 Incorrect Answer ImageC.Hyperglycemia
 Correct Answer ImageD.Intracranial aneurysm
 Incorrect Answer ImageE.Ophthalmic migraine
 

A 45-year-old woman with a longstanding history of uncontrolled hypertension comes to the


physician because of a 2-month history of worsening shortness of breath with minimal exertion.
She denies chest pain. She takes no medications and has no allergies. She has smoked one pack
of cigarettes daily for the past 15 years. Her temperature is 37.7°C (99.9°F), pulse is 75/min,
respirations are 16/min, and blood pressure is 180/65 mm Hg. Physical examination shows no
abnormalities except a prominent precordial impulse. A chest x-ray shows a prominent left
ventricle (LV). ECG shows left ventricular hypertrophy. An echocardiogram shows concentric
LV hypertrophy, without areas of hypokinesis, ejection fraction of 58% and a LV wall thickness
of 1.4 cm. Which of the following is the most likely diagnosis?
 Incorrect Answer ImageA.Acute myocardial infarction
 Incorrect Answer ImageB.Chronic obstructive pulmonary disease
 Correct Answer ImageC.Diastolic dysfunction
 Incorrect Answer ImageD.Interstitial lung disease
 Incorrect Answer ImageE.Pulmonary embolism

Infectologia

A 45-year-old man presents to the emergency department with worsening blurry vision,
difficulty speaking, and difficulty swallowing. He reports that 2 colleagues are also sick with
similar symptoms. He also has nausea, constipation, and xerostomia. He has no history of travel
and takes no medications. On physical examination he is alert and oriented. Temperature is
normal. There is bilateral ptosis and pupillary dilatation. Muscle power is reduced in the shoulder
and arm and normal in the lower extremities. Sensory examination is unremarkable. Which of
the following is the most likely organism resulting in this patient's presentation? 
 A.Campylobacter jejuni
 Correct Answer B.Clostridium botulinum
 C.Cytomegalovirus
 D.Epstein-Barr virus
 E.Mycoplasma pneumoniae

A 62-year-old man is brought to the emergency department by his son because of a 3-week
history of fever, night sweats, and a cough productive of blood-tinged sputum for 3 weeks. He
has had a 5 kg (11 lb) weight loss. Two weeks ago, he was treated for an upper respiratory
infection with amoxicillin he took for only six days. He had a positive purified protein derivative
(PPD) test 4 years ago. He drinks 8-10 cans of beer daily and has smoked a pack of cigarettes
daily for 40 years. On physical examination, his temperature is 39ºC (102.2ºF), pulse is 96/min,
respirations are 20/min, and blood pressure is 140/90 mm Hg. Physical examination shows
halitosis with egophony and vocal and tactile fremitus in the right upper lobe. Cardiac
examination shows a normal S1 and S2 without murmurs. Radiograph of the chest shows a thick-
walled cavitary lesion in the right upper lobe. Which of the following is the most appropriate
next step in management?
 A.CT scan of the chest
 B.Fiberoptic bronchoscopy to aspirate fluids
 C.Gram stain and culture of sputum
 Correct Answer D.IV clindamycin
 E.IV vancomycin
A 48-year-old woman comes to the physician because of a 2-month history of fatigue and
malaise. She takes no medications. She has smoked 1 pack of cigarettes daily for 30 years. She
underwent emergency splenectomy following a motor vehicle accident 22 years ago. During that
operation, she was transfused with multiple units of red blood cells. Laboratory studies show: 
HAV IgG Positive
Anti-HBs Positive
Anti-HBc Negative
Anti-HCV Positive
Liver biopsy specimen shows chronic inflammation with moderate fibrosis. Which of the
following is the most appropriate pharmacotherapy for this patient? 
 Incorrect Answer ImageA.Azathioprine
 Incorrect Answer ImageB.Interferon-alpha (IFN-alpha)
 Incorrect Answer ImageC.Lamivudine and peginterferon
 Correct Answer ImageD.Ledipasvir/sofosbuvir
 Incorrect Answer ImageE.Ribavirin

A 67-year-old man comes to his physician because of rectal bleeding that started 2 weeks ago.
He has no symptoms of rectal pain or change in stool consistency but has noted blood in the
toilet water and on the toilet paper. Approximately 10 years ago, he had a benign colon polyp
that was removed endoscopically. Additional medical history is positive for hyperlipidemia and
coronary artery disease. Three years ago, he underwent placement of a permanent pacemaker.
Two years ago, following a myocardial infarction, he underwent coronary angiography with
angioplasty and placement of a coronary artery stent. He has been asymptomatic since that time,
and his exercise tolerance is good. There is no evidence of congestive heart failure, and
echocardiography shows a systolic ejection fraction of 50%. He has no drug allergies, and his
medications include metoprolol, aspirin, and atorvastatin. You refer the patient to a
gastroenterologist for colonoscopy to evaluate the rectal bleeding, but before you do so, the
patient asks about antibiotic endocarditis prophylaxis. What would be the best approach to
endocarditis prophylaxis for this patient?
 Incorrect Answer ImageA.Prescribe oral amoxicillin to be taken before the procedure
 Incorrect Answer ImageB.Tell the patient that he should be given parenteral ampicillin
and gentamicin before the procedure and again 6 hours later
 Incorrect Answer ImageC.Tell the patient that he should be given parenteral ampicillin
before the procedure
 Incorrect Answer ImageD.Tell the patient that he should be given parenteral vancomycin
before the procedure
 Correct Answer ImageE.Tell the patient that he should not receive endocarditis
prophylaxis as he does not require it

A 62-year-old truck driver comes to the emergency department with significant shortness of
breath. He denies any significant medical problems. He has smoked two packs of cigarettes per
day for the last 40 years. His family history is significant for hypertension and coronary artery
disease. His temperature is 38.4ºC (101.1ºF), pulse is 112/min, respirations are 24/min, and
blood pressure 128/64 mm Hg. His oxygen saturation is 87% on room air. The physical
examination is significant for decreased breath sounds with crackles at the bases bilaterally.
Arterial blood gas (ABG) analysis and complete blood count (CBC) show:
pH 7.4
PCO2 32 mm Hg
PO2 54 mm Hg
HCO3– 20 mEq/l
Hemoglobin  14.8 g/dL
Hematocrit  43%
WBC  4,500/mm3
Neutrophils  94%
Lymphocytes  2%
Monocytes  2%
Eosinophils  1%
Basophils  1%
Platelets  256,000/mm3
Chest x-ray is shown. What is the most definitive diagnostic study that should be obtained next?
 Incorrect Answer ImageA.Basic metabolic panel
 Incorrect Answer ImageB.Bronchoscopy with bronchoalveolar lavage
 Incorrect Answer ImageC.CT scan of the chest
 Correct Answer ImageD.Immunofluorescence of induced sputum sample
 Incorrect Answer ImageE.Liver function tests

A 29-year-old scuba diver visits his physician for ear pain and whitish discharge from his right
ear for the last 3 days. The patient is otherwise healthy. He does not smoke and does not drink
alcohol. His vitals are temperature 36.9ºC (98.2ºF), blood pressure 120/68 mm Hg, pulse 76/min,
and respirations 14/min, and oxygen saturation of 100% on room air. Physical examination
shows pain with gentle traction of the external ear. Speculum examination of the auditory canal
shows erythema and swelling of the external auditory canal. There is also white discharge and
moist debris in the canal. The tympanic membrane is difficult to visualize. There is no pain on
palpation and percussion of the mastoid bone behind the ear. The remainder of the physical
examination is unremarkable. What is the most likely causative organism? 
 Incorrect Answer ImageA.Aspergillus
 Incorrect Answer ImageB.Candida
 Incorrect Answer ImageC.Haemophilus influenzae
 Incorrect Answer ImageD.Moraxella catarrhalis
 Correct Answer ImageE.Pseudomonas aeruginosa 

A 78-year-old man with type 2 diabetes mellitus comes to the emergency department because of
a 2-day history of fever, chills, and a headache. His temperature is 39.7ºC (103.5ºF).
Examination shows nuchal rigidity and photophobia. His hemoglobin A1c level is 11.8%. Lumbar
puncture is performed and cerebrospinal fluid analysis is shown below.
CSF Analysis:
Opening Pressure 300 mm H2O
Appearance Turbid
White Blood Cell 1,500/microL, 85% neutrophils, and 15%
Count lymphocytes
Glucose 25 mg/dL
Protein 250 mg/dL
Gram stain is negative and cultures are pending. Which of the following is the most appropriate
initial pharmacotherapy?
 Incorrect Answer ImageA.Ceftriaxone
 Incorrect Answer ImageB.Ceftriaxone and vancomycin
 Correct Answer ImageC.Ceftriaxone, vancomycin, and ampicillin
 Incorrect Answer ImageD.Penicillin G
 Incorrect Answer ImageE.Vancomycin

A 33-year-old man comes to the emergency department with fevers, chills, and a productive
cough. He has a prior history of cellulitis and tricuspid valve endocarditis due to intravenous
drug use. A recent HIV test at his methadone clinic is pending. On physical examination, his
temperature is 39.1ºC (102.4ºF), pulse is 98/min, respirations are 28/min, and blood pressure is
112/62 mm Hg. Lung examination shows rhonchi in the right mid posterior lung field. There is a
soft, holosystolic murmur heard at the left lower sternal border that increases with inspiration.
Chest radiograph is shown below. Which of the following is the best pharmacotherapy for this
patient?
 Incorrect Answer ImageA.Azithromycin, IV
 Incorrect Answer ImageB.Ceftriaxone, intramuscular
 Incorrect Answer ImageC.Daptomycin, IV
 Correct Answer ImageD.Linezolid, IV
 Incorrect Answer ImageE.Trimethoprim/sulfamethoxazole, IV

A 22-year-old Spanish-speaking man comes to the emergency department because of a fever and
abdominal pain. The patient recently emigrated from Mexico and has been working at a
construction site. For the past 48 hours he has been having temperatures to 38.7ºC (101.6ºF) and
this morning developed chills and has been unable to work. On physical examination, his
abdomen is nondistended and soft. There is tenderness over the liver. No organomegaly is noted.
Ultrasound shows an abscess in the right lobe of the liver. Alkaline phosphatase and
aminotransferase levels are elevated. Which of the following is the most appropriate next step in
management? 
 Incorrect Answer ImageA.Begin therapy with levofloxacin
 Correct Answer ImageB.Begin therapy with metronidazole
 Incorrect Answer ImageC.Begin therapy with praziquantel
 Incorrect Answer ImageD.Percutaneous drainage of abscess
 Incorrect Answer ImageE.Surgical drainage and injection of ethyl alcohol

A 48-year-old Vietnamese woman is discharged from the hospital after a 2-week stay for the
treatment of pulmonary tuberculosis. She is started on a 4-drug treatment regimen pending the
sensitivity results of her sputum cultures. Her regimen consists of isoniazid, rifampin,
pyrazinamide, and ethambutol. She initially tolerates the therapy well and takes all of her
medications as prescribed. When she returns to clinic 3 weeks later for culture results, she
reports the presence of red-orange colored urine. She denies dysuria, polyuria, urgency, or
myalgias. On physical examination, she is anicteric. The abdomen is nontender and the liver is
nonpalpable. The remainder of the physical examination is normal. Which of the following drugs
is most likely to cause this side effect? 
 Incorrect Answer ImageA.Ethambutol
 Incorrect Answer ImageB.Isoniazid
 Incorrect Answer ImageC.Pyrazinamide
 Incorrect Answer ImageD.Pyridoxine
 Correct Answer ImageE.Rifampin

A previously healthy 26-year-old man comes to the physician because of a 2-day history of sore
throat, malaise, and fever. He has had no nausea, vomiting, myalgias, shortness of breath, or skin
rash. He denies recent travel. He is sexually active with one person, and he uses condoms
consistently. His temperature is 38.5ºC (101.3ºF), pulse is 100/min, respirations are 18/min, and
blood pressure is 120/80 mm Hg. Physical examination shows tonsillar swelling and exudates,
tender cervical adenopathy, and mild splenomegaly. Laboratory studies show leukocyte count
12,200/mm3 with 11% atypical lymphocytes. Which of the following is the most appropriate next
step in management?
 Correct Answer ImageA.Acetaminophen
 Incorrect Answer ImageB.Acyclovir
 Incorrect Answer ImageC.Amoxicillin
 Incorrect Answer ImageD.No treatment needed
 Incorrect Answer ImageE.Prednisone

A 74-year-old woman who lives in a nursing home is brought to the emergency department
because of progressive drowsiness for the past 6 hours. She also has had a headache for the past
day. She has a history of hypertension, type 2 diabetes mellitus, coronary artery disease, and
hypothyroidism. Examination shows the patient is somnolent, unable to follow commands, and
has neck stiffness. Her temperature is 38.7ºC (101.8ºF), blood pressure is 110/78 mm Hg, and
pulse is 102/min. The lungs are clear to auscultation. Cardiac examination shows normal S1 and
S2; no murmurs are heard. There is pain with extension of the knee while the hip is flexed. CT
scan of the head shows no mass lesions. CSF culture is pending. A lumbar puncture is
performed, and CSF analysis is shown: 
Leukocyte count  2,000/mm3
Segmented neutrophils  90% 
Total protein 250 mg/dL 
Glucose  30 mg/dL 
Gram stain  No organisms 
Which of the following is the most appropriate next step in management? 
 Incorrect Answer ImageA.Acyclovir alone
 Incorrect Answer ImageB.Amphotericin B plus flucytosine
 Incorrect Answer ImageC.Ampicillin alone
 Incorrect Answer ImageD.Ceftriaxone alone
 Incorrect Answer ImageE.Ceftriaxone and ampicillin
 Incorrect Answer ImageF.Ceftriaxone and vancomycin
 Correct Answer ImageG.Ceftriaxone, vancomycin, and ampicillin
 Incorrect Answer ImageH.Supportive care only
 Incorrect Answer ImageI.Vancomycin alone

A 36-year-old man with a history of HIV, hypertension, and diabetes mellitus comes to the
physician because of progressive weakness of 3 months' duration. He has difficulty arising from
a seated position and mild persistent headache. He denies difficulty swallowing, incontinence,
tingling, or numbness. Current medications include zidovudine, lamivudine, efavirenz,
trimethoprim-sulfamethoxazole, insulin, and lisinopril. His vital signs are normal. Muscle
strength is 4/5 in all extremities. Deep tendon reflexes are decreased with normal sensation.
Which of the following is the most likely cause of this patient's weakness?
 Incorrect Answer ImageA.Hemorrhagic stroke
 Incorrect Answer ImageB.Insulin
 Incorrect Answer ImageC.Lisinopril-associated myopathy
 Incorrect Answer ImageD.Spinal stenosis
 Incorrect Answer ImageE.Thrombotic stroke
 Correct Answer ImageF.Zidovudine-associated myopathy

A woman is visiting her neighbor who adopts stray cats and then finds them homes; she has just
taken in a new cat. The visiting woman attempts to pick up the cat, but it bites her on the hand.
She does nothing about the bite, but 3 days later requires attention at the local urgent care center.
At this time the wound appears infected. The clinician suspects infection with Pasteurella
multocida. Cultures and sensitivities of the wound are sent to the lab and results are pending.
Past medical history is positive for a "rash" during treatment for an ear infection 3 months ago,
but the patient can't remember the name of the drug. Which of the following is the best initial
treatment? 
 Incorrect Answer ImageA.Ceftazidime
 Correct Answer ImageB.Clindamycin and cefuroxime
 Incorrect Answer ImageC.Oral clindamycin
 Incorrect Answer ImageD.Oral doxycycline
 Incorrect Answer ImageE.Penicillin G

A 35-year-old man with a 10-year history of HIV infection comes to the emergency department
because of a fever, headache, and confusion for 4 days. He has been irritable for the past 4
weeks. He has had no seizures or loss of consciousness. He has a 2-year history of known
positive anti toxoplasma IgG antibody. He appears lethargic. His temperature is 38.3°C (101°F),
pulse is 92/min, and blood pressure is 115/75 mm Hg. Examination of the skin shows no
abnormalities. The neck is supple. His CD4 T lymphocyte count is 10 cells/mm3, and HIV viral
load is 450,000/mL. CT scan of the brain shows 1 small ring-enhancing mass lesion in the basal
ganglia. Which of the following is the most appropriate next step in management? 
 Incorrect Answer ImageA.Anti-toxoplasma IgM test
 Incorrect Answer ImageB.Lumbar puncture and cerebrospinal fluid studies
 Incorrect Answer ImageC.MRI scan of the brain
 Correct Answer ImageD.Pyrimethamine and sulfadiazine
 Incorrect Answer ImageE.Stereotactic brain biopsy

A 37-year-old woman has the lesion shown for 2 days. She went hiking in upstate New York
with her daughter's seventh-grade class 15 days ago and found a tick on her arm 36 hours after
arriving home. She removed the tick with a tweezer and thought that she was going to be fine.
She is now upset because she has a friend who had a similar experience and is now suffering
from chronic synovitis and memory loss. The patient is anxious and says that she is worried
because she needs to participate in a blood drive at her daughter's school. She then adds, "I have
to be around for my daughter to go on class trips, see her off to college, and be there for her
wedding. This tick can't kill me!" Her temperature is 37.0ºC (98.6ºF). Physical examination
shows no other abnormalities. At this time, which of the following is the most appropriate
statement on the part of the physician? 
 Incorrect Answer ImageA."A vaccine is widely available to individuals who commonly
visit geographic areas of high risk during peak transmission season. Maybe you should look into
this for other members of your family."
 Correct Answer ImageB."I understand that you are very concerned. We'll treat this with
an antibiotic that most patients respond to with a prompt resolution of symptoms within 4 weeks.
This disease is rarely, if ever, fatal."
 Incorrect Answer ImageC."I understand that you are very concerned. We'll treat this with
an antibiotic that most patients respond to with a prompt resolution of symptoms within 4 weeks.
When you're finished here today, you can go donate blood."
 Incorrect Answer ImageD."The tick that caused this infection was most likely the size of
common dog and cattle ticks."
 Incorrect Answer ImageE."This could have been prevented if you had removed the tick
with petroleum jelly and a hot match. Tweezers may leave the mouthparts of the tick in the skin."

A 40-year-old man with HIV is admitted to the hospital for pneumocystis jirovecipneumonia


(PJP). He appears lethargic. His pulse is 125/min, and blood pressure is 80/58 mm Hg. He is
started on IV fluids, IV trimethoprim-sulfamethoxazole, and IV dopamine drip. Laboratory
studies show:
Na+ 133 mEq/L
K +
5.9 mEq/L
Cl− 104 mEq/L
HCO3− 25 mEq/L
BUN 22 mg/dL
Creatinine 0.9 mg/dL
Glucose 82 mg/dL
CD4 count 30/mm3 
Cortisol (AM) 2 µg/dL
Which of the following is the most likely underlying cause of this patient's electrolyte
abnormalities? 
 Incorrect Answer ImageA.Dopamine drug effect
 Incorrect Answer ImageB.Deficiency of ACTH
 Incorrect Answer ImageC.Intravenous fluids
 Correct Answer ImageD.Mycobacterium avium-intracellulare
 Incorrect Answer ImageE.PJP pneumonia

A 19-year-old college student has new-onset bloody diarrhea and abdominal pain that started
approximately 4 hours before this office visit. He has no significant past medical history and
takes no medications. He recently returned from a camping trip where he was drinking water
from fresh mountain streams. In addition to drinking the water, he often would barbecue shrimp
and hamburgers and eat potato salad that was prepared 2 days before the trip. Last night, after
arriving home, he ate his roommate's cold, leftover fried rice from a Chinese takeout restaurant.
His temperature is 39.0ºC (102.2ºF), blood pressure is 110/70 mmHg, pulse is 93/min, and
respirations are 16/min. His abdomen is soft and nontender with hyperactive bowel sounds. Stool
is positive for occult blood. Which of the following organisms is most likely responsible for this
patient's diarrhea? 
 Incorrect Answer ImageA.Bacillus cereus
 Correct Answer ImageB.Campylobacter jejuni
 Incorrect Answer ImageC.Giardia lamblia
 Incorrect Answer ImageD.Staphylococcus aureus
 Incorrect Answer ImageE.Vibrio parahaemolyticus

A 43-year-old homosexual man comes to the physician because of a small plaque in his mouth.
He denies any symptoms. His past medical history is significant for HIV infection of 3 years'
duration and borderline hypertension. He is not currently on any medication. Family history is
unremarkable. His blood pressure is 140/85 mmHg, pulse 65/min, and respirations 16/min. The
patient is in no acute distress. Oral examination shows a violaceous 1x1.3 cm infiltrated plaque
on the soft palate. Inspection of the skin is remarkable for multiple violaceous 0.5 to 2 cm
macules and patches on the lower back and legs. Palpation of the neck, axillae, inguinal, and
popliteal areas do not show any enlarged lymph nodes. Complete blood count and chest x-ray are
normal. A punch biopsy of one of the skin lesions is performed and histopathologic evaluation
confirms the vascular nature of the process. Which of the following is the most likely causative
agent of this patient's skin disease?
 Incorrect Answer ImageA.Cytomegalovirus
 Incorrect Answer ImageB.Epstein-Barr virus
 Incorrect Answer ImageC.Human herpesvirus type 6
 Correct Answer ImageD.Human herpesvirus type 8
 Incorrect Answer ImageE.Human papillomavirus type 6

A 59-year-old male truck driver comes to the hospital because of cough and conjunctivitis. The
patient describes a cough that has progressively worsened over the past 3 days, becoming more
productive of yellowish-white sputum. He had one episode of shaking chills 2 days ago with a
fever up to 38.8ºC (101.8ºF). He has smoked one pack of cigarettes per day for the past 20 years.
His past medical history is significant for rheumatoid arthritis, for which he takes a nonsteroidal
agent for pain control. On physical examination, his temperature is 37.5ºC (99.5ºF), pulse is
100/min, respirations are 13/min, and blood pressure is 140/90 mm Hg. He has coarse breath
sounds with crackles over his right base (click media file) and a normal cardiac examination.
Examination of the oral cavity shows tonsillar erythema and mild enlargement. There is
conjunctivitis bilaterally, and no adenopathy is appreciated. Examination of the extremities
shows no clubbing, cyanosis, or edema. Which of the following is the most appropriate
diagnostic study for this patient?
 Correct Answer ImageA.Chest radiograph
 Incorrect Answer ImageB.Heterophil antibody test
 Incorrect Answer ImageC.No further testing
 Incorrect Answer ImageD.Rapid strep test
 Incorrect Answer ImageE.Ventilation-perfusion scan

A 45-year-old man comes to the physician because of a new onset of progressive rash that has
been worsening daily for the last week and is accompanied by malaise, headache, and low-grade
fever. He has otherwise been in good health. He reluctantly discloses having unprotected sexual
intercourse with a "friend of a friend" approximately 5 weeks ago during a business trip.
Otherwise, he has had a single steady sexual partner for the past 10 months. Past medical history
is unremarkable and he is not taking any medications. He reports being in mild distress. His
temperature is 37.8°C (100.0°F), blood pressure is 120/85 mm Hg, and pulse is 95/min. There is
a diffuse skin eruption on the trunk, proximal extremities, and palms and soles (see photograph).
The axillary and inguinal lymph nodes are enlarged but nontender and freely movable. The
remainder of the physical examination is normal. Which of the following is the most appropriate
next step in management?
 Incorrect Answer ImageA.Dark-field microscopy of a lesional swab
 Incorrect Answer ImageB.Fluorescent treponemal antibody-absorption
 Correct Answer ImageC.Rapid plasma reagin test
 Incorrect Answer ImageD.Skin biopsy
 Incorrect Answer ImageE.Urethral swab

A 34-year-old man with a 4-year history of HIV comes to the physician because of 2 weeks of
diarrhea that has become bloody over the past week. He has 6 bowel movements daily, with
severe fecal urgency. He has had 2 episodes of fecal incontinence during the past week. He has a
history of a Pneumocystis jiroveciinfection 3 years ago. He is sexually active with one male
partner, and he and his partner use condoms consistently. His temperature is 38.2°C (100.8°F)
and blood pressure is 112/75 mm Hg. The lungs are clear to auscultation. Cardiac examination
shows normal S1 and S2 and a grade 2/6 systolic murmur is heard along the left sternal border.
Abdominal examination shows mild tenderness without masses. CD4+ T lymphocyte count is
32/mm3. Flexible sigmoidoscopy shows inflammation of the sigmoid colon and rectum with deep
ulcers and inclusion bodies. Which of the following is the most likely diagnosis?
 Incorrect Answer ImageA.Crohn's disease
 Correct Answer ImageB.Cytomegalovirus colitis
 Incorrect Answer ImageC.Entamoeba histolytica infection
 Incorrect Answer ImageD.Salmonella enteritidis infection
 Incorrect Answer ImageE.Ulcerative colitis

A concerned family comes to the physician after the grandmother, who had been living with
them, was diagnosed with active TB. She was started on appropriate treatment. The family
consists of a 45-year-old father, a 43-year-old mother, and an 8-year-old daughter. All of the
family members presently are asymptomatic. Which of the following is the most appropriate next
step in management? 
 Incorrect Answer ImageA.Obtain chest radiographs on all family members
 Correct Answer ImageB.Place protein purified derivative (PPD) test on all members and
make further studies if they are 5 mm or more
 Incorrect Answer ImageC.Place protein purified derivative (PPD) test on all members
and make further studies if they are 10 mm or more
 Incorrect Answer ImageD.Place protein purified derivative (PPD) test on all members
and make further studies if they are 15 mm or more
 Incorrect Answer ImageE.Start 4-drug treatment for all family members
 Incorrect Answer ImageF.Start prophylaxis with isoniazid for all family members
A 24-year-old man comes to the physician because of a 1-week history of fever, cough,
hemoptysis, wheezing, and shortness of breath. He developed an itchy rash on his chest and
abdomen 1 day ago. He is a farmer in the rural southeastern United States. His respirations are
30/min. Physical examination shows scattered bilateral wheezes on auscultation, urticarial rash
on the trunk and abdomen, mild abdominal distension, and diffuse abdominal tenderness to
palpation. Radiograph of the chest showed several bilateral patchy infiltrates worse in the right
lung. Laboratory studies show:
Hemoglobin  14.3 g/dL 
Leukocyte count  5,400/mm3
Segmented neutrophils  35%
Eosinophils  30% 
Lymphocytes  30% 
Monocytes 5%
Which of the following is the most appropriate pharmacotherapy? 
 Incorrect Answer ImageA.Ivermectin
 Incorrect Answer ImageB.Levamisole
 Correct Answer ImageC.Mebendazole
 Incorrect Answer ImageD.Pyrantel pamoate
 Incorrect Answer ImageE.Thiabendazole

A 30-year-old man comes to the clinic for evaluation of joint pain. For the last month, he has had
diffuse arthralgias, myalgias, low-grade fever, occasional palpitations, and fatigue. He has been
taking acetaminophen alternating with naproxen, only achieving minimal pain relief. He is
normally physically active outdoors and reported spending upward of 2 weeks at a time hiking
throughout the Northeastern United States and Western Europe. He is sexually active in a
monogamous relationship with his sexual partner over the past year. The man cannot recall any
rash or genital lesions, but he does report that he had ticks that he removed from his legs. Past
medical history is significant for a 3-week episode of a left facial droop. His temperature is
37.0ºC (98.6ºF), pulse is 50/min, respirations are 20/min, and blood pressure is 122/70 mm Hg.
Physical examination, aside from some ectopy on cardiac auscultation, is unremarkable. Which
of the following is the most likely cardiac abnormality?
 Incorrect Answer ImageA.Dilated aortic root on transthoracic echocardiogram
 Incorrect Answer ImageB.Dilated cardiomyopathy on transthoracic echocardiogram
 Correct Answer ImageC.Electrocardiogram showing atrioventricular block
 Incorrect Answer ImageD.Electrocardiogram with diffuse ST-segment elevations
 Incorrect Answer ImageE.Electrocardiogram with low-amplitude QRS complexes

A 39-year-old alcoholic man is brought to the emergency department by his wife immediately
after he had an episode of loss of consciousness and violent shaking that lasted 3 minutes. He
had a sinus infection 1 month ago. He has had a severe headache over the past 3 weeks not
relieved by over-the-counter analgesics. His temperature is 38.6ºC (101.5ºF), pulse is 116/min,
and blood pressure is 102/64 mm Hg. He is disoriented to time, person, and place. Muscle
strength is 3/5 in the left upper extremity and normal on the right side. CT scan of the head with
contrast is shown. A brain biopsy specimen and culture grows Bacteroides fragilis only. Which
of the following is the most appropriate next step in management?
 Incorrect Answer ImageA.Ceftazidime
 Incorrect Answer ImageB.Clindamycin
 Incorrect Answer ImageC.Fluconazole
 Correct Answer ImageD.Metronidazole
 Incorrect Answer ImageE.Penicillin
 Incorrect Answer ImageF.Pyrimethamine and sulfadiazine
 Incorrect Answer ImageG.Surgical excision

A 22-year-old medical student comes to the clinic complaining of severe lethargy, sore throat,
fever of 38.9ºC (102.0ºF) for the last 4 days, and a pink, generalized rash for the last 2 days. He
had been in good health before this episode. After obtaining a history and physical, it is noted
that the patient has not had any sick contacts, has been involved in a monogamous heterosexual
relationship for the last year, and has not traveled over the past year. He denies any illicit drug
use. He admits to self-treating with amoxicillin. He appears pale and exhausted. Physical
examination is significant for diffuse lymphadenopathy and inflamed erythematous tonsils. He
has a palpable and tender liver and spleen. A complete blood count shows:
Hematocrit 42%
Hemoglobin 14 g/dL
WBC 11,500/mm3
Neutrophils 35%
Lymphocytes 60%
Eosinophils 2%
Monocytes 3%
Platelets 140,000/mm3
A peripheral blood smear is significant for >10% of large lymphocytes with abundant cytoplasm.
Which of the following is the most appropriate next step? 
 Correct Answer ImageA.Heterophil antibody test
 Incorrect Answer ImageB.IgM toxoplasmosis
 Incorrect Answer ImageC.Lymph node biopsy
 Incorrect Answer ImageD.p24 antigen testing
 Incorrect Answer ImageE.Rapid streptococcal antigen testing

A 35-year-old man is taken to an emergency room by a friend after the man starts having severe
tonic spasms that cause his arms and legs to contract. The friend thinks that he is having seizures,
but there is no loss of consciousness and the man is able to describe these episodes. The patient's
difficulties began a day and a half earlier when he noticed jaw stiffness. This has since
progressed to difficulty swallowing, sore throat, stiff neck, fever, chills, and tonic spasms of his
arms and legs. The patient's speech is difficult to understand because he has trouble "making his
jaw work right." His mental status is intact. Which of the following is the most likely diagnosis?
 Incorrect Answer ImageA.Botulism
 Incorrect Answer ImageB.Diphtheria
 Incorrect Answer ImageC.Lyme disease
 Incorrect Answer ImageD.Seizure disorder
 Correct Answer ImageE.Tetanus

An 84-year-old woman is brought from home after being found by her daughter to have an
altered mental status. For the last 2 days, the woman reports a productive cough with green-
tinged sputum, fevers, shaking chills, and anorexia. Her past medical history includes diabetes
mellitus type 2, hypertension, and a myocardial infarction. Her temperature is 37.9ºC (100.2ºF),
pulse is 118/min, respirations are 26/min, and blood pressure is 145/95 is mm Hg. Oxygen
saturation is 91% while the patient is breathing room air. Examination of the chest shows
dullness to percussion and coarse inspiratory crackles at the left lung base. The remainder of her
physical examination is normal. Chest radiograph shows opacification of the left lower lobe with
air bronchograms without effusion or interstitial edema. The patient is admitted to the hospital.
Which of the following is the most appropriate single empiric antibiotic for this patient?
 Incorrect Answer ImageA.Azithromycin
 Incorrect Answer ImageB.Ceftriaxone
 Correct Answer ImageC.Moxifloxacin
 Incorrect Answer ImageD.Piperacillin/tazobactam
 Incorrect Answer ImageE.Vancomycin

A 6-year-old girl is brought to the emergency department by her parents immediately after she
was bitten on the hand by the neighbor's domestic dog. The father reports that the girl tried to pet
the dog while it was eating. The dog has been vaccinated regularly. Examination shows three
bite marks on the dorsum of the left hand with broken skin and dried blood. The wound is
cleaned and bandaged. Which of the following is the most appropriate next step for rabies
prophylaxis in this patient? 
 Incorrect Answer ImageA.Rabies immunoglobulin
 Incorrect Answer ImageB.Rabies immunoglobulin and vaccine
 Incorrect Answer ImageC.Rabies vaccine
 Incorrect Answer ImageD.The dog should be killed and the brain examined for signs of
rabies
 Correct Answer ImageE.The dog should be observed for behavioral changes suggestive
of rabies

A 43-year-old man comes to your office for evaluation of a spreading skin rash that appeared 5
days earlier. He noticed an initial red "bump" after returning from a camping trip to Vermont and
thought it was an insect bite. He also reports a headache and joint pain. His past medical and
family history is unremarkable, and he does not take medications. His temperature is 37.8°C
(100.0°F), pulse is 90/min, respirations are 18/min, and blood pressure is 110/70 mm Hg.
Physical examination of the skin shows an erythematous annular plaque 15 cm in diameter on his
right flank. It is centered by a resolving petechial macule. There is diffuse joint tenderness but no
impaired range of motion or localized inflammation of the small or large joints. An
ophthalmologic examination is normal. Which of the following vectors is most likely responsible
for this patient's disease?
 Incorrect Answer ImageA.Amblyomma americanum
 Incorrect Answer ImageB.Dermacentor andersoni
 Correct Answer ImageC.Ixodes scapularis
 Incorrect Answer ImageD.Loxosceles reclusa
 Incorrect Answer ImageE.Pediculus humanus corporis

A 24-year-old graduate student comes to the clinic complaining of pain on urination. The
discomfort began approximately 2 weeks ago and has not improved. He states that there is
purulent drainage from the tip of the penis, frequent urination, and the constant sensation to
urinate. There is no change in the caliber of his stream and no urinary incontinence. Besides an
episode of hypotension and tachycardia after a penicillin injection during childhood, he has been
healthy and denies any similar episode in the past. He recently returned from a spring-break trip
with his college friends. On physical examination, he is anxious but in no distress. His
temperature is 37.0ºC (98.6ºF), pulse is 90/min, and blood pressure is 126/81 mm Hg. Heart,
lung, and abdominal examinations are normal. His penis is circumcised, and there is a purulent
discharge at the meatus. The testes are descended, normal in contour, and without tenderness.
Prostate is smooth, firm, and without fluctuance or tenderness. Gram stain shows intracellular
gram-negative diplococci. A DNA probe for chlamydia is negative. Which of the following is the
most appropriate next step in the management of this patient?
 Incorrect Answer ImageA.Culture and sensitivity of the discharge to establish appropriate
therapy
 Correct Answer ImageB.Treat with azithromycin and gemifloxacin
 Incorrect Answer ImageC.Treat with azithromycin
 Incorrect Answer ImageD.Treat with ceftriaxone
 Incorrect Answer ImageE.Treat with ciprofloxacin
A 35-year-old man comes to the emergency department because of a severe headache, vomiting,
watery diarrhea, sore throat, muscle aches, rash, and fever to 39.9°C (103.8°F) for 2 days. Four
days ago, he sustained a deep laceration on his left arm after tripping and sliding down a hill. He
was able to stop the bleeding and cleaned the wound subsequently. He had a tetanus vaccination
four years ago. His temperature is 39.4°C (102.9°F), pulse is 110/min, and blood pressure is
86/60 mm Hg. Physical examination shows a diffuse macular erythematous rash and bilateral
conjunctival inflammation. Which of the following is the most likely causal organism?
 Incorrect Answer ImageA.Borrelia burgdorferi 
 Incorrect Answer ImageB.Clostridium perfringens 
 Incorrect Answer ImageC.Clostridium tetani 
 Incorrect Answer ImageD.Pseudomonas aeruginosa 
 Correct Answer ImageE.Staphylococcus aureus 

A 50-year-old man comes to the physician because of a 3-month history of joint pain. The pain
fluctuates and typically occurs in one joint, resolves, and then develops in another joint. The pain
neither worsens nor is relieved with activity. He denies morning stiffness. He has a history of a
rash on his left shoulder that began shortly after one of his hunting trips and resolved 6 months
ago. The patient works as a police officer and hunts twice monthly. He drinks 1 to 2 beers daily.
His temperature is 37.8°C (100ºF), and pulse is 75/min. Which of the following organisms is the
most likely cause of this patient's symptoms?
 Correct Answer ImageA.Borrelia burgdorferi 
 Incorrect Answer ImageB.Borrelia recurrentis 
 Incorrect Answer ImageC.Staphylococcus aureus 
 Incorrect Answer ImageD.Treponema carateum 
 Incorrect Answer ImageE.Treponema pallidum 

A 50-year-old man who has a history of alcohol abuse for the last 20 years is brought to the
hospital because of fever, dizziness, and a productive cough with brownish sputum. He is not
taking any medications and has a sulfa allergy. His temperature is 37.5ºC (99.5ºF), pulse is
91/min, respirations are 14/min, and blood pressure is 100/70 mm Hg. Physical examination
shows crackles in the left upper lobe, and cardiac examination shows a regular rate and rhythm.
Chest radiograph shows an infiltrate in the left upper lobe. Which of the following is appropriate
pharmacologic management?
 Incorrect Answer ImageA.Azithromycin
 Correct Answer ImageB.Cefuroxime and azithromycin
 Incorrect Answer ImageC.Clindamycin
 Incorrect Answer ImageD.Erythromycin and levofloxacin
 Incorrect Answer ImageE.Trimethoprim-sulfamethoxazole

A 33-year-old man comes to the physician because of a 1-year history of a rash over his arms,
face, and legs. He has had no pain or pruritis. He denies fever, myalgias, chills, or recent tick
bites. His medical history is unremarkable, and he takes no medications. He emigrated from
Mexico 2 years ago. He does not smoke or drink alcohol. His temperature is 36.8°C (98.2°F),
pulse is 70/min, and blood pressure is 118/76 mm Hg. Physical examination shows scattered
plaques that are hypopigmented, and surrounded by an erythematous border. Palpable nodules
and cord-like structures are noticed along his distal extremities. The remainder of the
examination shows no abnormalities. A skin biopsy specimen of a lesion shows rare acid-fast
bacilli. Laboratory studies show:
White blood cells  5,700/mm3
Hematocrit  44%
Platelets  235,000/mm3
Which of the following findings is most likely associated with this patient's condition? 
 Correct Answer ImageA.Cell-mediated immune response
 Incorrect Answer ImageB.Cure with monotherapy
 Incorrect Answer ImageC.Negative lepromatous skin test
 Incorrect Answer ImageD.Polyclonal gammopathy on SPEP
 Incorrect Answer ImageE.Positive serologic assay

A 41-year-old gastroenterologist attended an annual national conference in San Francisco. On his


return home, he develops a fever for 5 days with temperatures as high as 39.4ºC (102.9ºF) and a
severe, nonproductive cough associated with shortness of breath at rest. At the hospital, he
describes headache and fatigue, nausea, and frequent loose stools. His wife reports that he has
been confused. On physical examination, his temperature is 39.1ºC (102.4ºF), pulse is 76/min,
respirations are 28/min, and blood pressure is 116/84 mm Hg. On lung examination, there are
scattered loud rhonchi in the right lung fields. Abdominal examination is normal. Which of the
following is the most likely diagnosis?
 Incorrect Answer ImageA.Cytomegalovirus pneumonia
 Correct Answer ImageB.Legionnaires' disease
 Incorrect Answer ImageC.Mycobacterium tuberculosis 
 Incorrect Answer ImageD.Pneumocystis jirovecii pneumonia (PJP)
 Incorrect Answer ImageE.Streptococcal pneumonia

A 21-year-old woman returns to the clinic for further evaluation of bilateral facial weakness and
radicular arm pain. For the past few weeks she has suffered progressive facial palsy on both sides
of her face and hyperacusis and dry eyes. On her previous visit she described a 2-week history of
low-grade fevers, headaches, mild neck stiffness, and photophobia. Review of systems revealed a
recent rash that had developed after a camping trip, and then, after expanding outward from the
site of a small tick-bite, resolved spontaneously. A lumbar puncture performed at that time
revealed an increased amount of immunoglobulin with two oligoclonal bands on analysis.
Cerebrospinal fluid (CSF) PCR analysis for Borrelia burgdorferi and peripheral blood antibody
analysis are positive. Which of the following is the most appropriate treatment for this patient's
current condition? 
 A.IV acyclovir and corticosteroids
 B.IV interferon and corticosteroids
 Correct Answer C.IV penicillin G
 D.Oral amoxicillin
 E.Oral doxycycline

A 24-year-old man comes to the physician for a follow-up visit after completing a 6-month
course of treatment for active tuberculosis that he acquired while traveling. He currently feels
well and denies any specific complaints. His past medical history includes a new and difficult-to-
treat case of seborrheic dermatitis and a recent episode of herpes zoster. Vital signs are within
normal limits. Physical examination is unremarkable, with the exception of skin findings
consistent with seborrheic dermatitis. The patient is concerned about his likelihood of having
another episode of tuberculosis. Which of the following is the most appropriate next step in
management?
 A.Allergy skin testing
 B.Chest radiograph
 Correct Answer C.HIV testing
 D.PPD placement
 E.Sputum culture and smear

A 59-year-old man is seen by a specialist because of paranasal swelling, halitosis, and nose
bleeding with clots and pus. His temperature is 37.0ºC (98.6ºF), pulse is 73/min, respirations are
20/min, and blood pressure is 128/79 mm Hg. The decision is made to proceed with surgical
debridement of the sinuses. Preoperative physical examination shows a glucose level of 300
mg/dL and hemoglobin A1c is 11%. His medical condition rapidly deteriorates, and the surgical
procedure is performed urgently. The sinus contents are sent for pathologic examination, and
fungi with broad, nonseptate, irregularly shaped hyphae are identified. Systemic antifungal
therapy is administered, but the patient further deteriorates and is now stuporous. He dies 3 days
later despite appropriate treatment. Which of the following is the most likely organism
responsible for this patient's condition?
 A.Aspergillus
 B.Blastomyces
 C.Candida
 Correct Answer D.Rhizopus
 E.Sporothrix

A 47-year-old man comes to the emergency department because of fever and a cough that started
3 days ago. He had little relief from over-the-counter cold medications and as his cough
worsened, he observed the production of greenish sputum. He also complains of right-sided chest
pain. He does not smoke or drink and has been in good health. His temperature is 38ºC
(101.1ºF), blood pressure is 110/70 mm Hg, pulse is 90/min, and respirations are 20/min. To
view the examination, click on the "Play Media" button. Which of the following is the most
likely diagnosis? 
 A.Costochondritis
 B.Mycoplasma pneumonia
 C.Pseudomonas pneumonia
 D.Respiratory syncytial virus (RSV)
 Correct Answer E.Streptococcal pneumonia
A 48-year-old man with an 8-year history of HIV infection comes to the physician for a follow-
up examination. He recently completed a course of pentamidine for an infection
with Pneumocystis jirovecii. He has had no recent fever, cough, or shortness of breath. Current
medications include zidovudine, lamivudine, and nelfinavir. He admits that he occasionally skips
taking his medication. He is allergic to trimethoprim-sulfamethoxazole (TMP-SMX). His CD4+
T lymphocyte count is 49 cells/mm3, and viral load is 100,000/mL. Radiograph of the chest
shows resolution of the pneumonia. In addition to his current highly active antiretroviral therapy
(HAART) regimen, which of the following is the most appropriate prophylaxis regimen?
 Incorrect Answer ImageA.Aerosolized pentamidine and azithromycin
 Correct Answer ImageB.Atovaquone and azithromycin
 Incorrect Answer ImageC.Atovaquone and rifabutin
 Incorrect Answer ImageD.Clarithromycin and ethambutol
 Incorrect Answer ImageE.Dapsone and aerosolized pentamidine
 Incorrect Answer ImageF.Dapsone and azithromycin
A 30-year-old man comes to the physician because of a 2-week history of low-grade fever,
painful swelling under the left angle of the jaw, and painful swelling of his left testicle for 5
days. He had infectious mononucleosis 15 years ago. He traveled to South America last year. He
does not smoke cigarettes and drinks 4–5 beers on the weekends. His maternal uncle has type 1
diabetes mellitus. His temperature is 38.1°C (100.6°F), and pulse is 75/min. Which of the
following is the most likely cause of this patient's symptoms?
 Incorrect Answer ImageA.Actinomyces israelii 
 Incorrect Answer ImageB.Adenovirus
 Incorrect Answer ImageC.Chlamydia trachomatis 
 Incorrect Answer ImageD.Coxsackievirus B
 Correct Answer ImageE.Mumps virus

A 38-year-old man with a 2-year history of hypertension comes to the emergency department
because of a seizure. On arrival, the patient appears drowsy. Over the next 2 hours, his mentation
improves to normal. He has never had seizures in the past. He has had no fever, chills, vomiting,
weakness, or incontinence. He emigrated from Thailand 5 years ago. His temperature is 36.8ºC
(98.2ºF), pulse is 76/min, respirations are 16/min, and blood pressure is 120/70 mm Hg. Physical
examination is normal. Laboratory studies show:
Hgb  13.6 g/dL
WBC 8,000/mm 3 
Lymphocytes  40%
Eosinophils  13%
Platelets  270,000/mm3
Serum studies show:
Na+ 134 mEq/L 
Cl –
102 mEq/L 
K+ 4.6 mEq/L 
HCO3 –
27 mEq/L 
Blood urea nitrogen (BUN)  16 mg/dL 
Creatinine  0.9 mg/dL 
Glucose  80 mg/dL 
Ca +
9.2 mg/dL 
A CT scan of the head with contrast shows multiple ring-enhancing lesions at the gray-white
junction. Lumbar puncture and cerebrospinal fluid (CSF) examination show normal protein and
glucose levels and 0 to 1 cells/mm3. The CSF cryptococcal antigen and toxoplasma PCR are
negative. Which of the following is the most likely diagnosis? 
 Incorrect Answer ImageA.Bacterial meningitis
 Incorrect Answer ImageB.Hemorrhagic stroke
 Correct Answer ImageC.Neurocysticercosis
 Incorrect Answer ImageD.Thrombotic stroke
 Incorrect Answer ImageE.Toxoplasmosis

A 30-year-old man is brought to the emergency department by his wife with a 6-day history of
fatigue, cough, headache, backache, joint pain, and chest pain. One day ago, he had a sudden
onset of fever and chills, a worsening headache, and a rash that began on his back and chest and
spread to his limbs. He returned from a missionary trip to South America 2 weeks ago. He has
had no exposure to farm animals. His temperature is 38.8°C (101.8°F) and pulse is 125/min.
Examination shows a confused man with an erythematous maculopapular rash over the trunk and
extremities that spares the face, palms, and soles. There is diffuse muscle tenderness to palpation.
Which of the following is the most likely diagnosis? 
 Incorrect Answer ImageA.Endemic typhus
 Correct Answer ImageB.Epidemic typhus
 Incorrect Answer ImageC.Q fever
 Incorrect Answer ImageD.Rickettsial pox
 Incorrect Answer ImageE.Rocky Mountain spotted fever

A 34-year-old man comes to his physician for increasing pedal edema for three months. He has
no history of medical illness but is a former intravenous drug abuser, quitting one year ago. He
denies arthralgia, rash, or hematuria. His temperature is 37°C (98.6°F), and his blood pressure is
130/90 mm Hg. On physical examination, his lungs are clear to auscultation. Cardiac
examination shows an S4 with a laterally displaced PMI. Abdominal examination is significant
for an enlarged, palpable liver. There is pitting edema to mid-calf, and multiple old skin ulcers
are noted on both lower extremities. Laboratory analysis shows:
Potassium 3.7 mEq/L
Creatinine 1.6 mg/dL
Urea nitrogen  20 mg/dL
Albumin 2.1 mg/dL
Urinalysis shows 4+ protein, no heme, and no cells or casts. A spot urine albumin to creatinine
ratio is 8.5. A renal ultrasound shows large kidneys, each measuring approximately 13 cm. An
echocardiogram shows diffuse cardiac enlargement with pericardial thickening. An HIV test is
negative. Which of the following is the most likely diagnosis? 
 Incorrect Answer ImageA.AIDS nephropathy
 Correct Answer ImageB.Amyloidosis
 Incorrect Answer ImageC.Focal segmental glomerulosclerosis
 Incorrect Answer ImageD.Hypertensive nephropathy
 Incorrect Answer ImageE.Postinfectious glomerulonephritis
A 20-year-old man comes to the emergency department because of a 6-day history of fever,
chills, and muscle aches. Two weeks ago, he returned from a hiking trip to the White Mountains
of New Hampshire. His temperature is 38.3°C (100.9°F). There are two erythematous, round
patches on his left thigh, each with central clearing. Which of the following is the most
appropriate next step in management? 
 Incorrect Answer ImageA.Acyclovir
 Correct Answer ImageB.Doxycycline
 Incorrect Answer ImageC.ELISA for antibodies against the organism
 Incorrect Answer ImageD.Immunofluorescence assay of antibodies against the organism
 Incorrect Answer ImageE.Prednisone

A 45-year-old man is brought to the emergency department because of a 2-day history of fever,
headache, and confusion. He has a history of prostate cancer, type 2 diabetes mellitus, and
hypertension. Current medications include lisinopril and metformin. He appears confused. His
temperature is 39.7ºC (103.5ºF), pulse is 102/min, and blood pressure is 120/84 mm Hg. Physical
examination shows neck stiffness. A lumbar puncture is performed. Gram stain of the
cerebrospinal fluid shows lancet-shaped, gram-positive diplococci. Which of the following is the
most appropriate pharmacotherapy? 
 Incorrect Answer ImageA.Ampicillin, ceftriaxone, and vancomycin
 Incorrect Answer ImageB.Ceftriaxone
 Correct Answer ImageC.Ceftriaxone and vancomycin
 Incorrect Answer ImageD.Penicillin
 Incorrect Answer ImageE.Rifampin
 Incorrect Answer ImageF.Vancomycin plus ampicillin

A 25-year-old man comes to the emergency department because of a headache and mild
photophobia. The symptoms have been ongoing for the past 3 hours. He denies nausea, vomiting,
or prior similar symptoms. He denies any head trauma. His past medical history is notable for
hydrocephalus for which he has an indwelling ventriculoperitoneal shunt. This was manipulated
a few days ago during a neurosurgical clinic visit. The patient's shunt series films reveal no
kinking in the shunt tubing. Head CT scan reveals no increased hydrocephalus. Cerebrospinal
fluid obtained by way of a cisternal tap reveals several white cells consistent with an infection.
The patient is started on antibiotics. Which of the following pathogens is most likely responsible
for this patient's presentation?
 Incorrect Answer ImageA.Escherichia coli
 Incorrect Answer ImageB.Haemophilus influenzae
 Incorrect Answer ImageC.Neisseria meningitidis
 Correct Answer ImageD.Staphylococcus epidermidis
 Incorrect Answer ImageE.Streptococcus pneumoniae

An 81-year-old woman with diabetes consults a physician because of a severe and persistent
earache. Otoscopic examination demonstrates foul-smelling purulent otorrhea and a red mass of
the external ear canal. Biopsy of the mass demonstrates granulation tissue rather than tumor.
Which of the following is the most likely causative organism? 
 Incorrect Answer ImageA.Escherichia coli
 Incorrect Answer ImageB.Haemophilus influenzae
 Incorrect Answer ImageC.Proteus vulgaris
 Correct Answer ImageD.Pseudomonas aeruginosa
 Incorrect Answer ImageE.Staphylococcus aureus

A 69-year-old woman comes to the physician because of right-sided back pain for 6 months. She
has felt burning and tingling over her upper right back. Six months ago, she had a low-grade
fever, muscle aches, and a painful rash on the right side of her back that lasted for 10 days and
then resolved. She has a history of hypertension and hypercholesterolemia. Current medications
include hydrochlorothiazide, lisinopril, and pravastatin. Her temperature is 37.1°C (98.8°F),
pulse is 82/min, and blood pressure is 136/88 mm Hg. Examination shows hyperalgesia in the T4
dermatome on the right side. Which of the following is the most appropriate next step in the
management of this patient?
 Incorrect Answer ImageA.Oral acyclovir
 Correct Answer ImageB.Oral desipramine
 Incorrect Answer ImageC.Oral prednisone with taper
 Incorrect Answer ImageD.Physical therapy
 Incorrect Answer ImageE.Rest, heat, and supportive care

A 40-year-old man comes to his physician with dysphagia and odynophagia for the past 5 days.
He also reports bloody diarrhea, intermittent fever, and floaters within his visual field without a
change in vision. On further questioning, he reports unintentional weight loss of 9 kg (20 lb) in
the past 3 months. Past medical history includes HIV infection diagnosed 5 years ago. He refuses
treatment with HIV medications. His temperature is 38.3ºC (100.9ºF), pulse is 98/min,
respirations are 16/min, and blood pressure 108/70 mm Hg. He is cachectic. There is no oral
thrush or ulcers. Funduscopic examination shows yellowish-white granules with perivascular
exudates and hemorrhages. CD4+ T-cell count is 25 cells/mm3. Endoscopy shows one large,
shallow, 5 cm ulcer in the mid-esophagus surrounded by normal-appearing mucosa. Which of
the following is the most appropriate pharmacologic therapy? 
 Incorrect Answer ImageA.Acyclovir
 Incorrect Answer ImageB.Amphotericin B
 Incorrect Answer ImageC.Fluconazole
 Correct Answer ImageD.Ganciclovir
 Incorrect Answer ImageE.Pyrimethamine and sulfadiazine

A 30-year-old previously healthy man is brought to the emergency department because of fever,
severe headache, and seizures of abrupt onset. His temperature is 39.4ºC (103ºF), pulse is
100/min, respirations are 17/min, and blood pressure is 120/80 mm Hg. The patient is not
oriented to place and time. Physical examination shows mild nuchal rigidity. On admission,
laboratory studies and lumbar puncture findings are:
Hematocrit 43%
Hemoglobin 14 g/dL
WBC 10,200/mm3
Neutrophils 6,000/mm3
Lymphocytes 4,200/mm3
Platelets 200,000/mm3
CSF analysis:
Opening pressure 100 mm H2O 
Appearance Clear
Cell count 500/mm3
Dominant cell type Lymphocytes, 100 red blood cells/field
Glucose 60 mg/dL
Protein 60 mg/dL
Electroencephalographic studies demonstrate bitemporal periodic complexes on a slow
background. Neuroimaging shows multifocal hemorrhagic lesions in the temporal lobes. Which
of the following is the most appropriate next step in management? 
 Incorrect Answer ImageA.Polymerase chain reaction for suspected organism on CSF
 Incorrect Answer ImageB.Raise the head of the patient's bed, hyperventilate him, and
arrange urgent neurosurgical consult
 Incorrect Answer ImageC.Repeat a spinal puncture in 3 tubes to determine if the RBC are
indeed from the CSF
 Incorrect Answer ImageD.Supportive treatment, because this is a viral meningitis
 Incorrect Answer ImageE.Treatment with ceftriaxone plus vancomycin
 Correct Answer ImageF.Treatment with IV acyclovir

A 72-year-old man comes to the physician because of severe back pain for the past 3 days. He
was seen yesterday in the emergency department and was sent home with acetaminophen and
advised to avoid any strenuous activity. Despite following this advice, he continues to have back
pain and wishes to get a second opinion. The pain is located in the right lower lumbar region and
is described as constant, sharp, and stabbing. He reports some recent low-grade fevers that have
resolved with acetaminophen and diffuse myalgias in the days preceding his back pain. The
patient is concerned because he has a history of severe osteoporosis, though he has never had any
lumbar pain before. Past medical history includes syphilis and gonorrhea during his twenties for
which he received treatment. His temperature is 37ºC (98.6ºF), pulse is 98/min, respirations are
20/min, and blood pressure is 138/88 mm Hg. Examination of the patient's back shows no focal
tenderness although the patient is in obvious pain. Skin examination shows one small group of
vesicles in the right paraspinal lumbar region. The patient is surprised by the lesions and reports
that they were not present the previous day. Which of the following is the most appropriate next
step in management?
 Incorrect Answer ImageA.Admission for IV antibiotics
 Incorrect Answer ImageB.High-dose ibuprofen around the clock
 Incorrect Answer ImageC.Oral acyclovir and oral prednisone
 Correct Answer ImageD.Oral famciclovir
 Incorrect Answer ImageE.Rest, ice, supportive therapy, and reassurance

A 37-year-old woman with HIV comes to the medical walk-in clinic with cough, fever, and
shortness of breath. She has been well and has not been compliant with her triple-drug therapy.
She reports that 2 weeks ago, she began to experience increasing shortness of breath and
developed a nonproductive cough. Over the past few weeks, symptoms have gradually worsened.
Her last CD4 count was 190/mm3 and last viral RNA copy number was 120,000 copies/mL. She
also reports fevers to 39.4ºC (103.0ºF) over the past few days. On physical examination, her
pulse is 90/min, respirations are 24/min, blood pressure is 130/70 mm Hg, and oxygen saturation
is 80% on room air. Her lungs have diffuse crackles with no egophony or dullness to percussion.
The remainder of her examination is normal. Chest radiograph shows diffuse bilateral interstitial
and alveolar infiltrates bilaterally. Arterial blood gas analysis shows pH 7.32, PaCO2 52 mm Hg,
and PaO269 mm Hg. Which of the following is the most appropriate course of therapy?
 Incorrect Answer ImageA.IV amphotericin B
 Incorrect Answer ImageB.IV azithromycin
 Incorrect Answer ImageC.IV trimethoprim-sulfamethoxazole (TMP-SMX)
 Correct Answer ImageD.IV TMP-SMX and prednisone
 Incorrect Answer ImageE.Oral isoniazid, rifampin, pyrazinamide, and ethambutol

A 70-year-old man comes to the emergency department because of a 3-day history of fever,
chills, malaise, and fatigue. He has a history of hypertension, type 2 diabetes mellitus, and
ischemic heart disease. Two weeks ago, he was transferred to a nursing home after undergoing 3
months of inpatient rehabilitation following a cerebrovascular accident. Current medications
include lisinopril, hydrochlorothiazide, metformin, metoprolol, and simvastatin. His temperature
is 39.2°C (102.6°F), pulse is 85/min, and blood pressure is 115/70 mm Hg. Lungs are clear to
auscultation. Cardiac examination shows a normal S1 and S2 without murmurs. There is no
costovertebral tenderness. Abdominal examination shows no tenderness with normal bowel
sounds. Blood cultures grow methicillin-resistant Staphylococcus aureus. Which of the following
is the most appropriate pharmacotherapy?
 Incorrect Answer ImageA.Ceftriaxone
 Incorrect Answer ImageB.Linezolid
 Incorrect Answer ImageC.Oxacillin
 Incorrect Answer ImageD.Penicillin
 Correct Answer ImageE.Vancomycin

A 26-year-old man arrives in the emergency department because of an acutely painful and
swollen left calf. He is an IV drug user who admits to the use of black-tar heroin on a daily basis
for the past 2 weeks. He has resorted to "skin-popping" because he is no longer able to find veins
to inject. He has been hospitalized several times over the past few years for infective
endocarditis. He is HIV-negative as of 3 months ago. On physical examination, his temperature
is 38.7ºC (101.7ºF), pulse is 120/min, and blood pressure is 106/65 mm Hg. He appears
undernourished and disheveled with diaphoresis and pallor. He has poor dentition without oral
ulcers or abscesses. There is no cervical lymphadenopathy. Lungs are clear to auscultation and
cardiovascular examination shows no murmurs. Abdominal examination shows no
hepatosplenomegaly. Skin examination shows no lesions on his palms or soles. An indurated, 3
cm fluctuant lesion is noted on the left calf. It is acutely painful to the touch, and there is
surrounding crepitus. Laboratory data show WBC 18,000/mm3, hematocrit 38%, and platelets
568,000/mm3. Which of the following is the most likely etiologic agent?
 Correct Answer ImageA.Clostridium spp.
 Incorrect Answer ImageB.Neisseria spp.
 Incorrect Answer ImageC.Pseudomonas spp.
 Incorrect Answer ImageD.Staphylococcus spp.
 Incorrect Answer ImageE.Streptococcus spp.
A 72-year-old woman is recovering from a prolonged hospital stay for complicated community-
acquired pneumonia. While she was in the intensive care unit, a central venous catheter was
inserted into her internal jugular vein for blood sampling and hemodynamic monitoring. She
recovers well, and 1 week later is being considered for transfer to the general medicine ward.
However, after being afebrile for 5 days, she develops a fever. No obvious source of infection is
found, and the line insertion site appears normal without any erythema or induration. Blood
cultures are drawn, both through the line and from her arm. Although final results are not
available, Gram stains from three of the four bottles show gram-positive cocci in clusters. The
line is removed and the tip is sent for culture. Which of the following is the most appropriate
pharmacotherapy?
 Incorrect Answer ImageA.Cefepime, intravenous administration
 Incorrect Answer ImageB.Clindamycin, oral or intravenous administration
 Incorrect Answer ImageC.Gentamicin, intravenous administration
 Incorrect Answer ImageD.Nafcillin, intravenous administration
 Correct Answer ImageE.Vancomycin, intravenous administration

A 24-year-old woman comes to the emergency department after being bitten by a bat earlier in
the morning. She is an avid outdoors enthusiast. While taking a spelunking course, she disturbed
a group of resting bats. One of the animals bit her on the left leg, and she did not notice the
wound at the time. Her medical history is unremarkable. She reports feeling well, yet a bit tired.
The woman received all childhood vaccinations, and had a booster tetanus vaccination 18
months ago. Vital signs are within normal limits. Physical examination shows a healthy woman.
There is a series of mildly tender, small bite wounds on her left leg near the knee. The knee joint
is nontender, has full range of motion, and is without edema. Which of the following is the most
appropriate management? 
 Correct Answer ImageA.Both rabies vaccine and immune globulin
 Incorrect Answer ImageB.Close monitoring only for now; defer treatment
 Incorrect Answer ImageC.Observation of bats for massive death for the next 10 days, and
advise that the patient return immediately if that happens
 Incorrect Answer ImageD.Rabies immune globulin
 Incorrect Answer ImageE.Rabies vaccination
 Incorrect Answer ImageF.Reassurance and advice that the patient call if the wound
worsens
 Incorrect Answer ImageG.Tetanus toxoid

A previously healthy 21-year-old college senior comes to the emergency department because of
a 1-day history of severe headache, malaise, and vomiting. He states that bright lights make his
headache worse. He is alert and oriented. His temperature is 38.4ºC (101.1ºF), blood pressure is
110/76 mm Hg, and pulse 116/min. Examination shows a petechial rash of discrete lesions
approximately 1 to 2 mm in diameter on the patient’s trunk and lower body. Flexion of the
patient's neck causes flexion of the hips and knees. Fundoscopic examination shows no
abnormalities. Laboratory studies show: 
Hemoglobin  13.4 g/dL
Leukocyte count 13,400/mm3
Sodium  138 mEq/L
Segmented neutrophils  85%
Eosinophils 1%
Lymphocytes  10%
Monocytes 4% 4%
Platelet count 136,000/mm3
Which of the following is the most likely causative organism? 
 Incorrect Answer ImageA.Haemophilus influenzae
 Incorrect Answer ImageB.Listeria monocytogenes
 Correct Answer ImageC.Neisseria meningitidis
 Incorrect Answer ImageD.Staphylococcus aureus
 Incorrect Answer ImageE.Streptococcus pneumoniae

A 28-year-old medical student comes to the emergency department 30 minutes after a needle-
stick injury. He was stuck with a needle from a patient who has advanced hepatitis C and HIV.
His medical history is unremarkable and he takes no medications. Which of the following is the
most appropriate next step in management? 
 Correct Answer ImageA.HIV test and prophylaxis with emtricitabine, tenofovir, and
raltegravir
 Incorrect Answer ImageB.Observation without testing
 Incorrect Answer ImageC.HIV test only
 Incorrect Answer ImageD.HIV test and prophylaxis with zidovudine
 Incorrect Answer ImageE.Prophylaxis with lamivudine, zidovudine, and
lopinavir/ritonavir

A 19-year-old woman comes to the physician because of a 2-week history of frequent episodes
of loose stools. She has had severe fecal urgency and awoke with diarrhea several nights the past
month. Over the past week, her stools have become increasingly bloody. Her medical history is
unremarkable. Her temperature is 38.5°C (101.3°F), blood pressure is 115/70 mm Hg, and pulse
is 80/min. Sigmoidoscopy shows continuous, symmetric inflammation from the anal verge to the
proximal sigmoid colon. Which of the following is the most likely cause of this patient's
symptoms?
 Correct Answer ImageA.Campylobacter jejuni 
 Incorrect Answer ImageB.Cryptosporidium 
 Incorrect Answer ImageC.Cytomegalovirus
 Incorrect Answer ImageD.Giardia lamblia 
 Incorrect Answer ImageE.Yersinia enterocolitica 

A 35-year-old wood logger with no significant past medical history comes to the physician
because of a 4-day history of fevers, chills, severe headache, and diffuse muscle aches and pains.
Within 48 hours of these symptoms beginning, he developed a rash on his wrists and ankles.
Over the next 48 hours, the rash spread to involve the arms, legs, and trunk. He denies
photophobia or nuchal rigidity. He is sexually active with his wife and reports having
unprotected sexual intercourse with a prostitute one month ago. He owns a farm and raises cattle.
He has a dog and a cat that live inside the house. Physical examination shows a weak, pale, and
fatigued-appearing man. His temperature is 39.0ºC (102.3ºF), pulse is 113/min, respirations are
15/min, and blood pressure is 110/60 mm Hg. He has a diffuse maculopapular rash on his trunk,
extremities, palms, and soles. The remainder of his physical examination is normal. Laboratory
studies show a normal white count and liver function tests. Which of the following is the most
likely pathogen?
 Incorrect Answer ImageA.Bartonella henselae 
 Incorrect Answer ImageB.Borrelia burgdorferi 
 Incorrect Answer ImageC.Coxiella burnetii 
 Incorrect Answer ImageD.Ehrlichia chaffeensis 
 Correct Answer ImageE.Rickettsia rickettsii 

A 35-year-old wood logger with no significant past medical history comes to the physician
because of a 4-day history of fevers, chills, severe headache, and diffuse muscle aches and pains.
Within 48 hours of these symptoms beginning, he developed a rash on his wrists and ankles.
Over the next 48 hours, the rash spread to involve the arms, legs, and trunk. He denies
photophobia or nuchal rigidity. He is sexually active with his wife and reports having
unprotected sexual intercourse with a prostitute one month ago. He owns a farm and raises cattle.
He has a dog and a cat that live inside the house. Physical examination shows a weak, pale, and
fatigued-appearing man. His temperature is 39.0ºC (102.3ºF), pulse is 113/min, respirations are
15/min, and blood pressure is 110/60 mm Hg. He has a diffuse maculopapular rash on his trunk,
extremities, palms, and soles. The remainder of his physical examination is normal. Laboratory
studies show a normal white count and liver function tests. Which of the following is the most
likely pathogen?
 Incorrect Answer ImageA.Bartonella henselae 
 Incorrect Answer ImageB.Borrelia burgdorferi 
 Incorrect Answer ImageC.Coxiella burnetii 
 Incorrect Answer ImageD.Ehrlichia chaffeensis 
 Correct Answer ImageE.Rickettsia rickettsii 

A 36-year-old woman comes to the physician for evaluation of a purified protein derivative
(PPD) skin test placed 48 hours ago. She has had no cough, fevers, night sweats, chills, or weight
loss. She has no contact with individuals who have tuberculosis. One year ago, her PPD was
negative. She is a physician. Examination shows a 10-mm induration at the site of the skin test.
Radiograph of the chest is shown. Which of the following is the most appropriate next step in
management?
 Correct Answer ImageA.Daily isoniazid and pyridoxine for 9 months
 Incorrect Answer ImageB.Daily isoniazid, rifampin, ethambutol, and pyrazinamide for 2
months, followed by isoniazid and rifampin for 4 more months
 Incorrect Answer ImageC.Daily rifampin for 4 months
 Incorrect Answer ImageD.Repeat PPD in 1 year, no treatment necessary at the moment
 Incorrect Answer ImageE.Sputum induction for culture and sensitivity, and start
appropriate therapy afterward

A 44-year-old homosexual man who was born in the United States was recently diagnosed with
HIV-1 infection based on positive results from both routine antigen/antibody screening and
confirmatory immunoassay testing. While he is devastated about the diagnosis, he commits to
being proactive about managing his disease, joining a support group and enrolling in HIV
education classes. He is seen at a follow-up clinic appointment for discussion of therapeutic
options and appears to be doing well. His vital signs and physical examination are unremarkable.
At this time, his CD4 cell count is 328 cells/mm3 and viral load is 125,000 copies/mL. The
patient wants to know what else he can do to protect himself, as his job as a taxi driver brings
him into frequent contact with other people who could be ill. Which of the following is an
appropriate intervention at this time?
 Incorrect Answer ImageA.Clarithromycin and ethambutol treatment to prevent
Mycobacterium avium complex (MAC) infection
 Incorrect Answer ImageB.Consumption of only bottled or boiled water to reduce the risk
of cryptosporidiosis
 Correct Answer ImageC.PPD placement, followed by isoniazid for 9 months if >5 mm of
induration and chest x-ray is negative
 Incorrect Answer ImageD.Trimethoprim/sulfamethoxazole for Pneumocystis
jirovecii pneumonia prophylaxis
 Incorrect Answer ImageE.Varicella vaccine if the patient was not exposed to varicella as
a child

A 20-year-old college student comes to the campus clinic because of an intense headache and
fever since last night. She reports that her temperature at the dormitory last night was 39.5ºC
(103.1ºF). She attempted to control it with acetaminophen. She is otherwise healthy and does not
take any medications, including oral contraceptive pills. On physical examination, she has pain
and resistance to motion and to flexion of the hips and knees when her neck is flexed to her
sternum. There is also pain and resistance when she lies on her back and her leg is flexed at the
hip and knee. She has a diffuse petechial rash. Her temperature today is 38.9ºC (102ºF).
Neurologic examination is nonfocal; however, she is quite irritable and photophobic. She is
admitted to the hospital for further management. Which of the following is the most appropriate
prophylactic strategy for this patient's roommate? 

 Incorrect Answer ImageA.Chloramphenicol


 Incorrect Answer ImageB.No prophylaxis needed
 Incorrect Answer ImageC.Penicillin
 Incorrect Answer ImageD.Polysaccharide conjugate vaccine
 Correct Answer ImageE.Rifampin
 Incorrect Answer ImageF.Vancomycin plus ceftriaxone

A 62-year-old man with hypertension and hyperlipidemia comes to the emergency department
complaining of a severe headache and high fever. He also reports neck pain, photophobia,
nausea, and vomiting. He has no HIV risk factors. His temperature is 39.0ºC (102.2ºF), blood
pressure is 100/70 mm Hg, pulse is 123/min, and respirations are 16/min. He is in no respiratory
distress. The light bothers his eyes and he reports neck pain on passive lateral movement. There
is no edema of the optic discs. Cranial nerve and mental status examination are normal. Cardiac
examination shows tachycardia with a regular rhythm and no murmurs. His lungs are clear to
auscultation and abdomen is soft and nontender. There is no edema. He has 5/5 strength in all 4
extremities with normal reflexes. Which of the following is the most appropriate sequence in
management?
 Incorrect Answer ImageA.Begin treatment with ceftriaxone and vancomycin; send him
for head CT scan; if CT is normal, perform lumbar puncture
 Incorrect Answer ImageB.Begin treatment with ceftriaxone, vancomycin,
dexamethasone, and ampicillin; send the patient for head CT scan; if CT is normal, perform
lumbar puncture
 Incorrect Answer ImageC.Perform lumbar puncture and begin treatment with ampicillin
and ceftriaxone
 Correct Answer ImageD.Perform lumbar puncture and begin treatment with ceftriaxone,
vancomycin, ampicillin, and dexamethasone
 Incorrect Answer ImageE.Stat CT scan, then start antibiotics and then perform lumbar
puncture

A 52-year-old woman with a history of emphysema spends a 2-week vacation on a cruise ship.
Shortly after returning home, she develops high fevers and becomes lethargic and disoriented.
Her husband describes that she has been coughing and short of breath since returning home. He
also describes that she has vomited several times over the past 48 hours and has had diarrhea. On
physical examination, she appears lethargic, but arousable. She is disoriented to the current date.
She has loud, coarse rhonchi in both lung fields. Her abdominal examination shows mild
tenderness over the liver edge. There is no splenomegaly or ascites present. Her neurologic
examination is nonfocal. Laboratory results are notable for an aspartate aminotransferase (AST)
of 112 U/L and an alanine aminotransferase (ALT) of 157 U/L. Which of the following is the
most appropriate treatment for this patient? 
 Incorrect Answer ImageA.IV ceftazidime
 Incorrect Answer ImageB.IV gentamicin
 Correct Answer ImageC.IV levofloxacin
 Incorrect Answer ImageD.IV nafcillin
 Incorrect Answer ImageE.IV vancomycin

A 40-year-old man with a 1-year history of HIV comes to the emergency department because of
a 4-week history of fever and headaches. Current medications include raltegravir, tenofovir, and
emtricitabine. He has a past history of IV drug abuse, and he has been in remission for 5 years.
His temperature is 37.8ºC (100ºF), pulse is 84/min, respirations are 14/min, and blood pressure is
140/90 mm Hg. On physical examination, the pupils are equal, round, and reactive, and there is
no papilledema. There is no neck rigidity or lymphadenopathy. The lungs are clear to
auscultation. Cardiac examination shows normal S1 and S2 without murmurs. His CD4+ T
lymphocyte count of 20 cells/mm3. A lumbar puncture is performed, and CSF analysis shows a
leukocyte count of 20 cells/mm3. India ink stain is negative. Cryptococcal antigen in the
cerebrospinal fluid is positive. The patient receives a 2-week course of amphotericin B and
flucytosine, and he improves significantly. Which of the following is the most appropriate
pharmacotherapy for this patient? 
 Correct Answer ImageA.Fluconazole until CD4+ count is >100 cells/mm3 for one year,
raltegravir, tenofovir, and emtricitabine
 Incorrect Answer ImageB.Raltegravir, tenofovir, and emtricitabine
 Incorrect Answer ImageC.Raltegravir, tenofovir, emtricitabine, and intrathecal
amphotericin B
 Incorrect Answer ImageD.Raltegravir, tenofovir, emtricitabine, and lifelong amphotericin
B and flucytosine
 Incorrect Answer ImageE.Raltegravir, tenofovir, emtricitabine, and lifelong fluconazole
An adult develops insidious onset of a severe infectious disease. The condition is characterized
initially by high fever, headache, pharyngitis, and arthralgias. The patient then goes on to
develop intestinal symptoms of constipation, anorexia, abdominal pain and tenderness. During
the second week of the illness, he has a rash with discrete pink, blanching lesions on the chest
and abdomen. The rash resolves about three days later. By the third week of the disease, the
patient appears very ill and has developed a florid diarrhea that is positive for occult blood.
During this same period, the man also develops secondary pneumococcal pneumonia. At the
height of the patient's illness, he was stuporous and had short periods of delirium. The spleen
was palpable during this period. Laboratory studies show leukopenia, anemia, liver function
abnormalities, and a mild consumptive coagulopathy. Which of the following is the most likely
diagnosis?
 Incorrect Answer ImageA.Brucellosis
 Incorrect Answer ImageB.Cholera
 Incorrect Answer ImageC.Melioidosis
 Incorrect Answer ImageD.Plague
 Correct Answer ImageE.Typhoid fever
A 36-year-old man comes to the physician because of a 2-day history of fever, headache, retro-
orbital pain, and muscle and joint pains. His past medical history is unremarkable. He returned
from a trip to India 5 days ago. He does not drink or smoke. His temperature is 38.4ºC (101.1ºF),
pulse is 100/min, and blood pressure is 100/70 mm Hg. Physical examination shows a diffuse
macular, confluent rash. There is cervical lymphadenopathy, oropharyngeal injection, injected
conjunctiva, and palatal and extremity petechiae. Laboratory studies show:
Hemoglobin  16 mg/dL 
Leukocyte count  3,000/mm3
Segmented neutrophils  40% 
Lymphocytes  60% 
Platelet count  40,000/mm3
Aspartate aminotransferase (GOT, AST)  77 U/L
Alanine aminotransferase (GLP, ALT)  87 U/L
Serum studies show:
Na+ 133 mEq/L 
Cl- 98 mEq/L 
K +
3.5 mEq/L 
HCO3- 21 mEq/L 
Urea nitrogen (BUN)  25 mg/dL 
Creatinine  0.98 mEq/L 
Alkaline phosphatase  98 U/L
Peripheral blood smear  Negative for Plasmodium spp.
Radiograph of the chest shows bilateral pleural effusions. Which of the following is the most
appropriate next step in management? 
 Correct Answer ImageA.Admission to the hospital
 Incorrect Answer ImageB.CT scan of the chest and abdomen
 Incorrect Answer ImageC.Diuretic therapy for acute heart failure
 Incorrect Answer ImageD.NSAIDs to relieve symptoms
 Incorrect Answer ImageE.Reassurance
A 22-year-old man comes to the clinic this morning because he noticed that his "eyelids are
sticking together". The patient also reports that he has had clear, watery discharge and a burning
sensation in both his eyes. He denies photophobia or ocular pain. The patient reports that he had
an upper respiratory infection for the last 3 days. He is an otherwise healthy college student.
Examination shows preauricular adenopathy, moderate clear discharge, and moderate
conjunctival injection in both eyes. His visual acuity is 20/20 bilaterally. Extraocular muscle
function and fundoscopic examination are normal. Which of the following is the most
appropriate management?
 Incorrect Answer ImageA.Antibiotic drops
 Incorrect Answer ImageB.Antihistamine drops
 Incorrect Answer ImageC.Cycloplegics
 Incorrect Answer ImageD.Immediate ophthalmology referral
 Correct Answer ImageE.No specific treatment is necessary
 Incorrect Answer ImageF.Routine ophthalmology referral
 Incorrect Answer ImageG.Topical corticosteroids

A college senior comes to the emergency department because of fever and vomiting for the past
6 hours. He has had a fever and sore throat for 2 days, which his primary care provider treated
with amoxicillin. His temperature is 39.4°C (102.9°F). Examination shows photophobia. A
lumbar puncture is performed and cerebrospinal fluid analysis is shown in the table below. 
Opening Pressure 350 mm H2O
White Blood Cell Count 1,000/mm3
85% segmented neutrophils and 15% lymphocytes
Glucose 20 mg/dL
Protein 250 mg/dL
 Incorrect Answer ImageA.Aseptic meningitis
 Incorrect Answer ImageB.Brain abscess
 Incorrect Answer ImageC.Granulomatous meningitis
 Incorrect Answer ImageD.Neighborhood reaction meningitis
 Correct Answer ImageE.Partially treated bacterial meningitis

A 22-year-old man comes to the emergency department with a 3-day history of fever, chills, a
cough, pleuritic chest pain, and low back pain. He says that the symptoms came on "out of the
blue." He endorses intravenous drug use. His temperature is 39ºC (102.2ºF), pulse is 70/min,
respirations are 16/min, and blood pressure is 120/80 mm Hg. Physical examination shows oval
retinal hemorrhages with a clear, pale center and pinpoint lesions in his nails. He has bruises on
his arms. Heart examination is unremarkable. Blood cultures are drawn. Chest x-ray shows
multiple patchy infiltrates. Laboratory studies show:
Hemoglobin 11 g/dL
Hematocrit 39%
ESR 39 mm/hr
Which of the following is the most likely pathogen? 
 Incorrect Answer ImageA.Candida albicans 
 Incorrect Answer ImageB.Pseudomonas aeruginosa 
 Incorrect Answer ImageC.Serratia marcescens 
 Correct Answer ImageD.Staphylococcus aureus 
 Incorrect Answer ImageE.Streptococcus viridans 

A 66-year-old man is brought to the emergency department with a 3-day history of fever, cough,
and confusion. Review of systems also is positive for the presence of mild abdominal pain and
non-bloody diarrhea for the past 2 days. He was previously healthy and living an active lifestyle.
He does not smoke or consume alcohol. His temperature is 38.9ºC (102.0ºF), pulse is 110/min,
respirations are 22/min, and blood pressure is 135/80 mm Hg. The patient is confused. Lung
examination shows mildly reduced air entry at the right base. Heart sounds are normal, and the
abdomen is soft and nontender with normal bowel sounds. Laboratory studies show:
Blood chemistry:
RBCs 4.5 million/mm3
Hb 15 g/dL
Hct 45%
Leukocytes  15,500/mm3
Differential:
Neutrophils  78%
Bands  12%
Lymphocytes 5%
Monocytes 5%
Chest radiograph shows right lower lobe patchy infiltrates. Sputum examination shows abundant
neutrophils with no organisms. The fecal occult blood test is negative. Which of the following is
the most appropriate pharmacotherapy? 

 Incorrect Answer ImageA.Ceftriaxone


 Correct Answer ImageB.Ceftriaxone and azithromycin
 Incorrect Answer ImageC.Ceftriaxone and gentamicin
 Incorrect Answer ImageD.Doxycycline
 Incorrect Answer ImageE.Erythromycin

A 32-year-old woman is brought to the emergency department by her friends for evaluation of
altered mental status. They report that for the last few days she has been acting strangely,
speaking with normal words but in a random fashion that makes no sense. Today she was found
naked on her couch, unable to explain where she was or what she was doing. Before this current
episode, her friends report that had been in good health except for a minor cold last week, with
fever and headaches. Her blood pressure is 129/82 mm Hg, pulse is 112/min, respirations are
20/min, and temperature is 38.3ºC (101.0ºF). Examination shows a disoriented woman in
moderate distress. Although her pupils and extraocular movements are normal, she shies away
from your penlight. Additionally, she moans in pain as her neck is flexed. CT scan shows mild
cerebral edema of the left temporal lobe. A lumbar puncture shows: 
Opening pressure 90 mm (normal: 70–180 mm)
White blood cells 382 cells/mm3 (80% lymphocytes) 
Red blood cells 470 cells/mm3
Protein 78 mg/dL
Glucose 70 mg/dL
Gram stain reveals no organisms. Cultures are sent, and the patient is treated empirically with
ceftriaxone and vancomycin pending results. In addition to these measures, this patient would
likely benefit from treatment with which of the following? 

 Correct Answer ImageA.Acyclovir


 Incorrect Answer ImageB.Amphotericin B
 Incorrect Answer ImageC.Dexamethasone
 Incorrect Answer ImageD.Foscarnet
 Incorrect Answer ImageE.Ganciclovir

A 24-year-old man comes to the physician because of a 10-day history of fever up to 38.2°C
(100.8°F), muscle aches, loss of appetite, and sore throat. He denies nausea, vomiting, or
diarrhea. He does not smoke cigarettes or drink alcohol. He is not sexually active. His
temperature today is 38.4°C (101.1°F), pulse is 90/min, and blood pressure is 124/78 mm Hg.
Physical examination shows tonsillar erythema with an exudate and bilateral anterior and
posterior cervical adenopathy. There is a fine, diffuse maculopapular rash. A streptococcal
antigen test on a throat swab is negative. Which of the following is the most sensitive and
specific diagnostic study for this patient's condition? 

 Incorrect Answer ImageA.CBC with heterophile test


 Incorrect Answer ImageB.IgG to Epstein-Barr (EB) viral capsid antigen (VCA)
 Incorrect Answer ImageC.IgM to cytomegalovirus (CMV)
 Correct Answer ImageD.IgM to EB VCA
 Incorrect Answer ImageE.Rapid plasma reagin (RPR)

A 67-year-old man with a history of chronic sinusitis comes to the emergency department with a
severe headache. While waiting to be seen, he develops a generalized seizure. The seizure
subsides after 10 minutes. The patient’s temperature is 40ºC (104ºF), pulse is 65/min,
respirations are 12/min, and blood pressure is 150/90 mm Hg. Ophthalmologic examination
shows protrusion of both eyes, and fundoscopic examination shows bilateral papilledema. The
right eye is noted to be pointed inward with the pupil nonreactive to light. Blood cultures are
drawn, and the patient is started on broad-spectrum IV antibiotics. Which of the following is the
most appropriate next step in management?

 Incorrect Answer ImageA.Dexamethasone


 Incorrect Answer ImageB.IV acyclovir
 Incorrect Answer ImageC.Loading dose of phenytoin
 Correct Answer ImageD.Obtain head CT scan
 Incorrect Answer ImageE.Perform lumbar puncture

An 18-year-old college student comes to the emergency department because of headache, fever,
and nausea. For the last 5 days, she has had severe fatigue, myalgias, and worsening fevers. She
reports that prior to the previous week, she was healthy and active. She recently returned from a
camping, hiking, and rock climbing trip through the hills of North Carolina prior to the
development of this current illness. She does not recall any tick bites, but she did have significant
mosquito exposure with bites on her arms and legs. Her temperature is of 39.1ºC (102.4ºF), pulse
is 65/min, respirations are 18/min, and blood pressure is 138/90 mm Hg. Physical examination
shows mild hepatosplenomegaly and faint macules on the hands, wrists, and ankles. Based on
this presentation, which of the following is the most likely pathogen?

 Incorrect Answer ImageA.Babesia microti


 Incorrect Answer ImageB. Borrelia burgdorferi
 Incorrect Answer ImageC.Leptospira interrogans
 Correct Answer ImageD.Rickettsia rickettsii
 Incorrect Answer ImageE.Treponema pallidum

An 18-year-old male college student comes to the clinic 36 hours after a dog bite. He works in an
animal physiology laboratory. While administering experimental medications to a poodle, the
animal bit his arm. The dog's behavior was normal prior to and after the bite. The animal is being
used for drug testing, has received appropriate vaccinations, and is certified by the breeder as
disease-free. The student originally was not going to see a physician, but the area around the bite
has become progressively more painful. His temperature is 37.2ºC (99.0ºF), pulse is 90/min,
respirations are 14/min, and blood pressure is 122/80 mm Hg. Physical examination shows a 2
cm area of erythema, warmth, and swelling surrounding the small, superficial puncture wound on
his right forearm. There is minimal induration but no fluctuation or crepitation. Plain radiograph
shows no bony injury or subcutaneous gas. Which of the following is the most appropriate next
step in management?

 Correct Answer ImageA.Amoxicillin-clavulanate


 Incorrect Answer ImageB.Ciprofloxacin
 Incorrect Answer ImageC.Rabies vaccination
 Incorrect Answer ImageD.Rabies vaccination and immunoglobulin
 Incorrect Answer ImageE.Reassurance

A 23-year-old man comes to the student health clinic because of burning, itchy red eyes.
Approximately 3 days ago, he noticed that one of his eyes appeared pink. Since then, his other
eye also has become involved. He has noticed morning crusting of his eyelids and a watery
discharge throughout the day. He also reports low-grade fever, malaise, diffuse myalgias, and a
sore throat, lasting approximately a day. Examination of his eyes show bilaterally injected
conjunctivae and a scant amount of thin, mucoid discharge in the corner of his eyes, though there
is profuse tearing. Examination of the tarsal conjunctivae shows a “bumpy” appearing epithelium
by gross examination. There is bilateral, tender preauricular adenopathy. Examination of the
oropharynx shows mild erythema. The rest of the physical examination is unremarkable. He is
concerned, as he had “pink eye” as a child, and is worried about spreading an eye infection. He is
hoping you will prescribe an appropriate antibiotic, as he does not want to infect friends and
family. Which of the following is the most appropriate next step in the management? 
Incorrect Answer ImageA.Admit for further workup and intravenous antibiotics
Incorrect Answer ImageB.Give prescription for topical antibiotic ointment
Incorrect Answer ImageC.Perform slit-lamp examination with fluorescein dye
Correct Answer ImageD.Recommend hand-washing and supportive care
Incorrect Answer ImageE.Suggest over-the-counter antihistamines and decongestants
A 27-year-old ski instructor comes to the physician for a nonproductive cough for 10 days. He
also has a low-grade fever and diffuse muscle aches. Two weeks ago he went to an instructors'
convention and he thinks that the air conditioner in his room was the cause of his symptoms. He
has been able to work 6 hours per day, but has become increasingly dyspneic on long ski runs.
He has no prior medical history and does not smoke or drink. He has a temperature of 38.3ºC
(100.9ºF) and has scattered bilateral crackles. He has a regular heart rhythm. The remainder of
his physical examination is normal. A chest radiograph shows faint bilateral interstitial
infiltrates. Which of the following is the most likely diagnosis?
Incorrect Answer ImageA.Legionella pneumonia
Incorrect Answer ImageB.Mycobacterium tuberculosis
Correct Answer ImageC.Mycoplasma pneumonia
Incorrect Answer ImageD.Pneumocystis carinii pneumonia
Incorrect Answer ImageE.Streptococcal pneumonia

A 21-year-old woman comes to the physician because of an itchy rash on her hands for 2 weeks.
She has had difficulty sleeping at night because of irritation and scratching. She has been
applying over-the-counter hydrocortisone cream with minimal improvement. She has had no
fever, fatigue, shortness of breath, wheezing, or cough. Past medical history is significant for
allergic rhinitis, childhood asthma, and a chronic rash as a child. Current medications include
fexofenadine and an oral contraceptive pill she has taken for the past 5 years. She has no
allergies to medications. Her brother had a similar rash on his face. Her temperature is 36.7ºC
(98ºF). Physical examination shows palmar erythema and weeping, crusting plaques on the
hands and fingers. There are scattered vesicles and edematous papules with excoriations. Which
of the following is the most appropriate recommendation for this patient's rash? 
Incorrect Answer ImageA.Chronic use of antihistamines to minimize atopic response
Incorrect Answer ImageB.Frequent use of soap
Incorrect Answer ImageC.Low-humidity environments
Correct Answer ImageD.Regular use of over-the-counter bland emollients
Incorrect Answer ImageE.Shower frequently with warm water to remove bacterial debris

A 58-year-old man comes to the emergency department because of sudden-onset chest pain for 6
hours. He describes the pain as sharp and reports a severity of 4/10 on the pain scale. He also
says that the pain becomes worse when he is in a supine position or takes deep breaths. It is
improved by sitting up or leaning forward. He underwent coronary artery bypass graft (CABG)
three weeks ago. His postoperative course was uncomplicated and he was discharged as planned.
He also has a 20-year history of well-controlled hypertension. His temperature is 38.1ºC
(100.6ºF), pulse is 104/min, blood pressure is 122/72 mm Hg, and oxygen saturation is 96% in
room air. Physical examination shows bibasilar crackles and a pericardial friction rub heard over
the sternum. Abdominal and peripheral vascular examinations are unremarkable. An ECG shows
diffuse ST-segment elevation along with PR depression; these findings were not present on the
ECG performed when he was discharged two weeks ago. Chest x-ray shows no new changes
compared to the one performed after his CABG surgery. Which of the following is the most
likely cause of this patient's chest pain?
Incorrect Answer ImageA.Aortic dissection
Incorrect Answer ImageB.Costochondritis
Correct Answer ImageC.Dressler syndrome
Incorrect Answer ImageD.Gastroesophageal reflux disease
Incorrect Answer ImageE.Myocardial infarction

A 33-year-old woman presents to the emergency room with acute onset blurred vision, nausea,
dizziness, diarrhea, and severe abdominal cramps. She has no significant past medical history.
The patient is an avid traveler and tries to live “off-the-grid” mostly at her grandparent’s farm.
She cans vegetables and sells them to the local markets. On examination she is sweating,
anxious, and restless. Her temperature is 37.3ºC (99.0ºF), blood pressure 86/54 mm Hg, pulse
52/min, and respirations 26/min. Physical examination shows increased oral secretions, watery
eyes with symmetric 2 mm pupils, and scattered fasciculations. Which of the following is the
most likely diagnosis?
Incorrect Answer ImageA.Amyotrophic lateral sclerosis
Incorrect Answer ImageB.Anticholinergic overdose
Incorrect Answer ImageC.Botulism
Incorrect Answer ImageD.Multiple sclerosis
Correct Answer ImageE.Organophosphate poisoning

A 69-year-old man comes to the emergency department because of dizziness, blurry vision, and
slurred speech for 30 minutes. Earlier in the day, he suddenly lost his balance, felt like the room
was spinning, and had difficulty with his vision and speech. The symptoms resolved 30 minutes
later. He denies headache, fever, or loss of consciousness. He has a history of hypertension and
hyperlipidemia. Current medications include hydrochlorothiazide. He smoked one pack of
cigarettes daily for 30 years, but he quit 12 years ago. His blood pressure is 130/85 mm Hg and
pulse is 75/min. Examination shows no abnormalities. ECG shows sinus rhythm, and carotid
ultrasound shows no abnormalities. An abnormality in which of the following arteries is the most
likely cause of this patient's symptoms? 
Incorrect Answer ImageA.Internal carotid artery
Incorrect Answer ImageB.Middle cerebral artery
Incorrect Answer ImageC.Penetrating thalamic branches
Incorrect Answer ImageD.Posterior inferior cerebellar artery
Correct Answer ImageE.Vertebrobasilar artery

A 68-year-old woman comes to the physician because of sadness, fatigue, difficulty


concentrating, and constipation for 4 months. She has had no fever, vomiting, diarrhea, or blood
in her stools. She has had no feelings of guilt, worthlessness, or desire to commit suicide. She
has had no recent stressors. She has had a 7 kg (15 lb) weight loss. She retired 1 year ago from
her job as a lawyer. Her blood pressure is 134/70 mm Hg and pulse is 78/min. Examination
shows no abnormalities. Which of the following is the most likely diagnosis? 
Incorrect Answer ImageA.Hyperthyroidism
Incorrect Answer ImageB.Islet cell tumors
Correct Answer ImageC.Multiple myeloma
Incorrect Answer ImageD.Pheochromocytoma
Incorrect Answer ImageE.Vitamin D deficiency
A 45-year-old man comes to the physician because of a 4-week history of low back pain that
radiates to the scrotum, dysuria, and pain on defecation. His physical examination is consistent
with prostatitis, and he is treated with trimethoprim-sulfamethoxazole for 3 weeks. His
symptoms recur 1 week after antibiotic therapy is stopped, and he is given ciprofloxacin for 4
weeks. His symptoms again recur after cessation of the antibiotic. Today, the patient appears ill.
He has shaking chills and sweats. Rectal examination shows an enlarged prostate with areas of
tenderness and fluctuance. Which of the following is the most likely diagnosis? 
Incorrect Answer ImageA.Acute multidrug resistant bacterial cystitis
Incorrect Answer ImageB.Chronic nonbacterial prostatitis
Incorrect Answer ImageC.Colovesical fistula
Incorrect Answer ImageD.Ischiorectal abscess
Correct Answer ImageE.Prostatic abscess

A 52-year-old man is brought to the emergency department after being found unconscious at the
bus stop near the hospital. The patient's medical and social history is unknown. The patient
appears malnourished and poorly dressed. His temperature is 35.6ºC (96.1ºF), pulse is 120/min,
respirations are 24/min, blood pressure is 96/65 mm Hg, and oxygen saturation is 94% on 2 L
oxygen via nasal cannula. The patient responds only to pain. Head is atraumatic. The pupils are
equal and slightly reactive to light. Skin is dry with decreased turgor. Physical examination
shows decreased breath sounds bilaterally with no crackles or wheezes. Heart sounds are normal
with no murmurs. Abdomen is soft and nondistended, with normoactive bowel sounds. There is a
3x4-cm wound on the left foot with purulent discharge. Neurologic examination is normal except
for the altered mental state. Serum studies show:
Na+ 141 mEq/L
K+ 4.5 mEq/L
Cl -
111 mEq/L
HCO3- 13 mEq/L
Urea nitrogen (BUN) 15 mg/dL
Creatinine 0.9 mg/dL
Calcium 8.7 mg/dL
Glucose 310 mg/dL
A Foley catheter is placed. The urinalysis shows 3+ glucose with no protein, blood, ketones, or
cells. 
The patient remains hypotensive and confused after intravenous administration of 1 liter of
normal saline. Which of the following tests would best diagnose the cause of the patient’s lab
abnormalities? 
Incorrect Answer ImageA.Blood alcohol level
Incorrect Answer ImageB.Blood pH
Incorrect Answer ImageC.Serum beta-hydroxybutyrate level
Correct Answer ImageD.Serum lactic acid level
Incorrect Answer ImageE.Plasma osmolality

An 18-year-old man is evaluated for possible immunodeficiency disease because of a life-long


history of chronic lung infections, recurrent otitis media, and multiple episodes of bacterial
meningitis. Although total IgG is normal, the patient is found to have a selective deficiency of
IgG2. An associated deficiency of which of the following other substances may produce
anaphylaxis when blood products are administered? 
Incorrect Answer ImageA.C3
Incorrect Answer ImageB.C4
Correct Answer ImageC.IgA
Incorrect Answer ImageD.IgE
Incorrect Answer ImageE.IgM

A 21-year-old man who was recently diagnosed with asthma comes to see his primary care
physician. The patient says that he has about 2 episodes of shortness of breath per week with no
nocturnal symptoms. Temperature is 36.9°C (98.4°F), pulse is 78/min, blood pressure is 118/66
mm Hg, respirations are 16/min, and oxygen saturation is 96% on room air. Physical
examination shows a well-developed young man in no distress. Breath sounds are clear
bilaterally. Heart sounds are normal with no murmurs or gallops. Abdomen is soft and
nontender, with normoactive bowel sounds. Pulmonary function testing shows FEV1/FVC that is
86% of predicted. Which of the following is the most appropriate next step in management?
Correct Answer ImageA.Albuterol
Incorrect Answer ImageB.Fluticasone
Incorrect Answer ImageC.Formoterol
Incorrect Answer ImageD.Prednisone
Incorrect Answer ImageE.Theophylline

A 17-year-old female is seen in the emergency department for a broken ankle. She describes a
fall down the stairs at her home earlier that day. She denies any alcohol or drug use at the time.
She has no significant past medical history, but she has been admitted to the hospital many times
for a previous fracture, and she has been seen in the emergency department on multiple
occasions for various lacerations and minor injuries. The patient explains that all these visits are
because of her "active lifestyle" and her frequent home-repair projects. The patient states that she
has been happily married for 2 years. She is employed as a waitress at a local restaurant; her
husband is unemployed and on disability pay for "back problems". The patient is admitted to the
hospital and undergoes successful open reduction and internal fixation for her ankle. After the
surgery, the patient gives a more detailed history about her social situation. She reports having
arguments with her husband but strongly denies the possibility that her husband may have
anything to do with her injuries. You suspect that the injuries are a result of domestic abuse. As
her physician, which of the following is your most appropriate statement at this time? 
Incorrect Answer ImageA."I am obligated to contact the police, as I am concerned about your
safety."
Incorrect Answer ImageB."If you don't feel comfortable telling me about the reason for your
injuries, I can have you speak to someone else at my office."
Incorrect Answer ImageC."I recommend that you report domestic abuse, but if you don't, I will
respect that decision."
Incorrect Answer ImageD."I recommend that you speak to a domestic-abuse hotline and tell them
your concerns."
Correct Answer ImageE.“I want to see you again in a week, but if you feel like talking to me about
anything, I will give you my contact information and feel free to contact me sooner.”
A 37-year-old man comes to the emergency department with a 2-hour history of chest pain and
dyspnea. He suddenly felt light-headed and had to stop while driving home to call 911. Shortly
thereafter, he began to experience chest pain and shortness of breath. He attributes these
symptoms to stress associated with his job and admits to drinking 8 cups of coffee a day. Further
history shows occasional cocaine use "on the weekends." He is currently not on any medications.
He is a 25 pack-year smoker but is determined to quit following the death of his father from lung
cancer. On physical examination, the patient is pale and diaphoretic and appears disoriented to
time, place, and person. His temperature is 37.0ºC (98.6ºF), blood pressure is 80/60 mm Hg, and
pulse is 155/min. Electrocardiogram shows a shortened P-R interval (<0.12 seconds), a slurred
upslope of the QRS complex, and a broad QRS interval (>0.12 seconds). Which of the following
is the most appropriate next step in management?
Incorrect Answer ImageA.Administer amiodarone
Incorrect Answer ImageB.Administer digoxin
Incorrect Answer ImageC.Administer diltiazem
Incorrect Answer ImageD.Administer IV fluids
Incorrect Answer ImageE.Administer procainamide
Incorrect Answer ImageF.Check a urine drug screen
Incorrect Answer ImageG.Radiofrequency ablation
Correct Answer ImageH.Synchronized cardioversion
Incorrect Answer ImageI.Transthoracic ultrasound

An 18-year-old woman with a history of type 1 diabetes comes to the emergency department
because of a 2-week history of nausea, vomiting, diarrhea, and abdominal cramping. She has had
no fever or chills. She drinks 2 to 3 glasses of wine on weekends. She denies smoking or illicit
drug use. Her temperature is 36.9ºC (98ºF), blood pressure 90/60 mm Hg, pulse 110/min, and
respirations 16/min. Abdominal examination shows mild tenderness to palpation in the lower
quadrants. The remainder of the examination shows no abnormalities. Serum studies show: 
Na+ 136 mEq/L
Cl-
116 mEq/L
K+ 3.0 mEq/L
HCO3 -
10 mEq/L
Urea nitrogen (BUN)  16 mg/dL 
Creatinine  0.9 mg/dL 
Glucose  200 mg/dL 
Urine analysis shows:
Na+ 12 mEq/L
K +
10 mEq/L
Cl- 110 mEq/L
Which of the following is the most likely explanation for these laboratory findings? 
Correct Answer ImageA.Diarrhea
Incorrect Answer ImageB.Distal renal tubular acidosis (RTA), type 1
Incorrect Answer ImageC.Ketoacidosis due to diabetes (DKA)
Incorrect Answer ImageD.Lactic acidosis
Incorrect Answer ImageE.Vomiting
Non-anion gap metabolic acidosis can be caused by diarrhea with loss of bicarbonate in stools,
characterized by high urinary H+ concentration. 
Renal tubular acidosis (RTA) with failure to excrete H+ ions in the urine can also cause a non-anion
gap metabolic acidosis, characterized by low urinary H+ concentration

A 62-year-old man with a 55-pack-year smoking history comes to the urgent care clinic with
worsening of his chronic shortness of breath. He has noticed a change in his chronic daily cough.
He has also had several episodes of blood-tinged sputum over the past few weeks. His wife states
that over the past 2 weeks he has seemed more hoarse than usual. Physical examination shows
dense rhonchi in the right posterior mid-lung field. There are also soft, scattered rhonchi in both
lung fields with a prolonged expiratory phase. A chest x-ray shows a 7-cm, irregularly-shaped
mass in the right middle lobe with associated lobar consolidation. Which of the following most
suggests that the patient has surgically inoperable lung cancer? 
Incorrect Answer ImageA.A postobstructive pneumonia
Incorrect Answer ImageB.Hemoptysis
Correct Answer ImageC.Increasing hoarseness
Incorrect Answer ImageD.The change in his pattern of cough
Incorrect Answer ImageE.Worsening dyspnea

A 46-year-old man is admitted to the hospital because of a 3-week history of fever, back pain,
leg weakness, fatigue and malaise. He has a history of type 1 diabetes mellitus and had a renal
transplant 4 years ago. Current medications include insulin, cyclosporine, and prednisone. His
temperature is 38.1°C (100.6°F) and pulse is 90/min. A grade 3/6 holosystolic murmur is heard
at the apex. Muscle strength is 4/5 bilaterally in the lower extremities. Blood cultures
grew Streptococcus viridans. Echocardiography shows vegetations on the mitral valve. Which of
the following is the most likely explanation for this patient's back pain and leg weakness? 
Incorrect Answer ImageA.Cyclosporine-induced neuropathy
Incorrect Answer ImageB.Diabetic neuropathy
Incorrect Answer ImageC.Prednisone-induced osteoporosis
Incorrect Answer ImageD.Spinal stenosis
Correct Answer ImageE.Vertebral osteomyelitis

A 65-year-old man comes to the emergency department because of a 3-month history of


progressive shortness of breath. He has had no chest pain or fever. Five years ago, he had
multiple myeloma, for which he was treated. He takes no medications and has no allergies. He
drinks 1 beer with dinner on a daily basis. His temperature is 37.2ºC (99.0ºF), blood pressure is
110/70 mm Hg, pulse is 105/min, and respirations are 22/min. There is jugular venous distention
to the angle of the jaw. The lungs are clear to auscultation. Cardiac examination shows distant
heart sounds. An S3 and S4 are heard. There is hepatomegaly and 2+ pedal edema. Laboratory
studies show serum iron of 100 μg/dL, a total iron-binding capacity of 350 μg/dL (normal value
250–450 μg/dL), and ferritin of 100 ng/mL. ECG shows diminished amplitude of the QRS
complexes. X-ray of the chest shows no abnormalities. An echocardiogram shows a speckled-
appearing myocardium. Which of the following is the most likely diagnosis?
Incorrect Answer ImageA.Alcoholic cardiomyopathy
Correct Answer ImageB.Amyloidosis
Incorrect Answer ImageC.Hemochromatosis
Incorrect Answer ImageD.Ischemic cardiomyopathy
Incorrect Answer ImageE.Sarcoidosis

A 65-year-old woman comes to the physician because of a lesion that has been slowly growing
on her cheek for 3 years. She has had no fevers, fatigue, or weight loss. Her medical history is
unremarkable. A photograph of the lesion is shown. The remainder of the physical examination
is normal. Which of the following is the most likely diagnosis?
Correct Answer ImageA.Basal cell carcinoma
Incorrect Answer ImageB.Invasive melanoma
Incorrect Answer ImageC.Keratoacanthoma
Incorrect Answer ImageD.Pyogenic granuloma
Incorrect Answer ImageE.Squamous cell carcinoma

A 34-year-old Caucasian man comes to the physician because of blood in his stool for the last 2
weeks. He is concerned because his father died at a young age of colon cancer. Physical
examination shows multiple epidermal inclusion cysts and a fecal occult blood test is positive.
Colonoscopy shows innumerable colonic polyps. A biopsy of one friable polyp confirms the
diagnosis of adenocarcinoma of the colon. Genetic studies reveal a mutation in the APC gene on
chromosome 5 and a diagnosis of Gardner syndrome is suspected. Which of the following
clinical findings would help confirm this diagnosis? 
Incorrect Answer ImageA.Acral and periorificial pigmented macules
Incorrect Answer ImageB.Cancer most commonly occurs in the proximal colon
Correct Answer ImageC.Congenital hypertrophy of the retinal pigment epithelium
Incorrect Answer ImageD.Medulloblastoma
Incorrect Answer ImageE.Oral papillomas and palmar pits

ReKap

Familial adenomatous polyposis is caused by mutations in the adenomatous polyposis coli gene.
Gardner syndrome findings include: colonic polyps, colon cancer before age 50, osteomas,
epidermal inclusion cysts, congenital hypertrophy of the retinal pigment epithelium. 
Turcot syndrome findings include colonic polyps, colon cancer before age 50 and tumors of the
central nervous system.

A 45-year-old man undergoes a routine examination. Although the history and physical
examination is unrevealing, a hematocrit performed in the physician's office gives a value of
25%. Review of the peripheral smear reveals smaller than normal erythrocytes. The cells vary in
size, and some have abnormal shapes. The cells do not appear paler than normal. Reticulocytes
are decreased. Assuming that there is only a single cause for this patient's anemia, which of the
following is most likely to be seen on further evaluation? 
Correct Answer ImageA.Low iron
Incorrect Answer ImageB.Low iron-binding capacity
Incorrect Answer ImageC.Low folate
Incorrect Answer ImageD.Low globin production
Incorrect Answer ImageE.Low vitamin B12
A 67-year-old man with a history of hypothyroidism, hypertension, and hypercholesterolemia
comes for a follow-up examination. He started receiving levothyroxine 6 months ago. He has had
no lethargy, constipation, or cold intolerance. He has had a 3-kg (7-lb) weight loss. Laboratory
studies show: 
Serum TSH  11.2 μU/mL
Serum thyroxine  2.2 mcg/dL
Serum total cholesterol  243 mg/dL
Serum triglycerides 287 mg/dL
Serum LDL  151 mg/dL
Which of the following are the most likely changes to this patient's blood pressure and lipid
profile following treatment of his hypothyroidism? 
Correct Answer ImageA.Decreased blood pressure, decreased LDL, and decreased triglycerides
Incorrect Answer ImageB.Decreased blood pressure, increased LDL, and increased triglycerides
Incorrect Answer ImageC.Increased blood pressure, decreased LDL, and decreased triglycerides
Incorrect Answer ImageD.Increased blood pressure, increased LDL, and increased triglycerides
Incorrect Answer ImageE.No change in blood pressure or lipid profile

A 54-year-old woman comes to the physician for a routine health maintenance examination. She
has had no weight loss, fever, cough, decreased appetite, chest pain, lower extremity swelling, or
blood in the stool or urine. She has a history of osteoarthritis in her right knee treated with
ibuprofen. Her temperature is 36.5ºC (97.7ºF), blood pressure is 128/72 mm Hg, and pulse is
75/min. Neck examination shows a solitary nodule in the thyroid gland that is readily palpable.
The remainder of the examination shows no abnormalities. Laboratory studies show TSH 0.4
µU/mL and free thyroxine (FT4) 10 ng/L. Ultrasound of the thyroid shows a 1.5-cm hyperechoic
nodule in the right lower lobe. Which of the following is the most appropriate next step in
management? 
Incorrect Answer ImageA.Fine-needle aspiration (FNA)
Incorrect Answer ImageB.No further management is necessary
Correct Answer ImageC.Radioactive iodine uptake (RAIU)
Incorrect Answer ImageD.Surgical resection
Incorrect Answer ImageE.Treatment of subclinical hyperthyroidism

A 58-year-old man comes to the physician for a follow-up appointment because of a non-ST-
segment elevation myocardial infarction he sustained 2 months ago. Laboratory studies show
elevated cholesterol, triglycerides, LDL cholesterol levels, and a low HDL cholesterol level.
After his myocardial infarction, treatment with a statin, beta-blocker, aspirin, and clopidogrel
was started. Which of the following puts this patient at greatest risk for future coronary events?
Incorrect Answer ImageA.Elevated total cholesterol levels
Incorrect Answer ImageB.High triglyceride levels
Correct Answer ImageC.Increased LDL cholesterol levels
Incorrect Answer ImageD.Low HDL cholesterol levels
Incorrect Answer ImageE.Ratio of HDL to LDL cholesterol

A 70-year-old woman has been in long-standing poor health, with severe diabetes mellitus and
rheumatoid arthritis. Her physician notes that she appears pale and orders a hematocrit, which is
35%. Examination of the blood smear reveals microcytic anemia. Which of the following
laboratory tests would help distinguish iron deficiency anemia from anemia of chronic disease? 
Incorrect Answer ImageA.Erythrocyte:granulocyte ratio in bone marrow
Incorrect Answer ImageB.Presence or absence of polychromatophilic target cells
Incorrect Answer ImageC.Presence or absence of stippled erythrocytes
Correct Answer ImageD.Serum ferritin
Incorrect Answer ImageE.Serum iron

A 35-year-old HIV-positive man is brought to the clinic by his partner because of a 6-month
history of progressive memory loss and incontinence. He is taking zidovudine and a protease
inhibitor. He first noticed difficulties with handwriting. Neurologic examination demonstrates
deficits in cognitive and fine motor control functions. Laboratory investigations show a CD4 cell
count of 25/mm3. MRI studies show moderate brain atrophy but no focal lesions. A lumbar
puncture shows no cerebrospinal fluid abnormalities. Which of the following is the most likely
diagnosis? 
Incorrect Answer ImageA.CMV encephalitis
Incorrect Answer ImageB.Cryptococcal meningoencephalitis
Correct Answer ImageC.HIV associated neurocognitive disorder
Incorrect Answer ImageD.HIV myelopathy
Incorrect Answer ImageE.Primary brain lymphoma
Incorrect Answer ImageF.Progressive multifocal leukoencephalopathy

A 60-year-old man comes to his physician with increasing nausea, vomiting and malaise. He has
been taking ibuprofen for osteoarthritis for the past 18 months. For the past 3 months, he had
increased inflammation in his finger joints, so he increased his dosage. His temperature is 37.2°C
(98.06°F) and blood pressure is 130/94 mm Hg. The exam is normal except for joint swelling
and limited range of motion in his hands. Laboratory studies show:
RBC count 4.4 million/mm3
HCT 39%
Hemoglobin 12 g/dL
WBC count 4,500/mm3
Neutrophils 60%
Lymphocytes 30%
Monocytes 6%
Eosinophils 3.5%
Sodium 139 mEq/L
Potassium 3.4 mEq/L
Chloride 106 mEq/L
Bicarbonate 33 mEq/L
BUN 26 mg/dL
Creatinine 2.0 mg/dL
Three months ago, the serum creatinine was 1.0 mg/dL. Urinalysis: 2+ blood, 3+ protein, nitrite
negative; 10–20 RBCs, 10–20 WBCs, +white cell casts. A renal ultrasound is normal. What is
the best first step in management?
Correct Answer ImageA.Discontinue the ibuprofen
Incorrect Answer ImageB.Perform a renal biopsy
Incorrect Answer ImageC.Treat with cyclophosphamide
Incorrect Answer ImageD.Treat with intravenous ampicillin and gentamicin
Incorrect Answer ImageE.Treat with prednisone

A 57-year-old man comes to the physician because of erectile problems. He says that he has not
been able to have an erection in months. He says he does wake up with an erection in the
morning. He has no headaches or visual defects. He is wondering about medical treatment for
this issue. He is married, has 3 children, works as a trader on the stock exchange, and is an avid
cyclist. He takes no medications, drinks no alcohol, and has no serious medical conditions. There
is no history of perineal trauma or pelvic surgery. Physical examination of the heart, lungs, and
abdomen is unremarkable. The testicles are descended bilaterally. There is a right epididymal
cyst measuring 5 mm. The prostate is small and has no palpable nodules. His serum PSA is 1.0
ng/mL. Urinalysis is normal. Which of the following is the most likely cause of this condition? 
Incorrect Answer ImageA.Epididymal cyst
Incorrect Answer ImageB.Hyperprolactinemia
Incorrect Answer ImageC.Low testosterone
Incorrect Answer ImageD.Perineal trauma
Correct Answer ImageE.Psychogenic impotence

A 35-year-old man comes to the emergency department with severe hypertension, palpitations,
and a headache. Blood tests show hypercalcemia and elevated chromogranin A. Urine
metanephrines are elevated. The patient is admitted for control of his blood pressure. A
subsequent gene study reveals a mutation in the RET proto-oncogene. Which of the following
other diseases is this patient at risk for developing?
Correct Answer ImageA.Medullary thyroid carcinoma
Incorrect Answer ImageB.Mucosal carcinoma
Incorrect Answer ImageC.Pancreatic adenoma
Incorrect Answer ImageD.Pituitary adenoma
Incorrect Answer ImageE.Renal cysts

A 56-year-old previously healthy woman is admitted to the hospital because of a 1-week history
of severe mid-abdominal pain that radiates to the back. One week ago, she was in a motor
vehicle accident, and her abdomen was struck by the steering wheel. Her temperature is 37.0ºC
(98.6ºF), pulse is 96/min, respirations are 20/min, and blood pressure is 130/75 mm Hg. The
lungs are clear to auscultation bilaterally. Cardiac examination shows a normal S1 and S2; no
murmurs are heard. The abdomen is diffusely tender in the epigastric region without masses,
rebound tenderness, or guarding. CT scan of the abdomen is shown. Which of the following is
the most likely location of the patient's injury?
Incorrect Answer ImageA.Aorta
Incorrect Answer ImageB.Kidney
Incorrect Answer ImageC.Liver
Correct Answer ImageD.Pancreas
Incorrect Answer ImageE.Spleen
Incorrect Answer ImageF.Stomach
A 61-year-old woman comes to her doctor's office for steadily increasing abdominal girth and
fatigue with mild exertion. She has noticed this symptom for the past few months. She reports a
5 kg (11 lb) increase in her weight without making any change in her regular diet. Her past
medical history is unremarkable, although she has not seen a physician for many years. She
denies smoking, but admits drinking a glass of wine with meals on weekends. On physical
examination, she is afebrile and normotensive. Examination of her abdomen shows significant
distention with shifting dullness and a fluid wave. A bedside ultrasound is performed, which
demonstrates a large amount of ascitic fluid. A paracentesis is performed and fluid analysis
shows a serum albumin to ascites gradient of 0.9 g/dL. Which of the following conditions is the
most likely cause of this patient's current condition? 
Incorrect Answer ImageA.Alcoholic cirrhosis
Incorrect Answer ImageB.Budd-Chiari syndrome
Correct Answer ImageC.Peritoneal carcinomatosis
Incorrect Answer ImageD.Portal vein thrombosis
Incorrect Answer ImageE.Right heart failure

A 10-year-old girl is brought to the clinic by her parents because of shortness of breath, chest
pain, and cough for the past 3 months. Chest x-ray shows a mass in the right hemithorax, and
needle biopsy of the tumor showed that the predominant cells are CD3, CD4, and CD8 positive.
Her white blood cell count is 11,000/mm3 with 68% eosinophils, 30% neutrophils, 6%
lymphocytes, and 6% monocytes. Which of the following is the cytokine that is most likely
being produced by these tumor cells?
Incorrect Answer ImageA.IL-1
Incorrect Answer ImageB.IL-2
Incorrect Answer ImageC.IL-3
Incorrect Answer ImageD.IL-4
Correct Answer ImageE.IL-5

ReKap

IL-5 production in the bone marrow induces eosinophil production.


IL-5 production in the submucosa causes isotype switching to IgA synthesis.

A 35-year-old woman comes to the physician because of tingling in her hands and feet for 4
weeks. She tells the doctor that she recently had a baby and is enjoying her life as a stay-at-home
mother. She takes no medications, does not smoke cigarettes or use drugs, and drinks no alcohol.
She had an episode of optic neuritis 6 months ago. Physical examination shows weakness in the
extremities, increased muscle tone, and hyperreflexia. An MRI and cerebrospinal fluid (CF)
analysis are scheduled. MRI scan of the brain and spinal cord shows multiple plaques. CF
analysis shows increased levels of intrathecally synthesized IgG, oligoclonal banding, and
mononuclear cell pleocytosis. An extensive battery of additional laboratory studies shows no
abnormalities. After one week, the patient returns to the physician to receive the results of the
studies. After the physician greets the patient and determines that she is comfortable having this
conversation alone without family members, which of the following is the most appropriate
initial statement by the doctor?
Incorrect Answer ImageA."A great team of doctors, nurses, and social workers are here to work with
you and help you to deal with this condition."
Incorrect Answer ImageB."Do you have an idea what your diagnosis may be?"
Incorrect Answer ImageC."Multiple sclerosis is serious, but manageable in most patients."
Incorrect Answer ImageD."What do you know about demyelinating diseases?"
Correct Answer ImageE."Will you please tell me what you know about your condition so far?"
Incorrect Answer ImageF."Would you like me to tell you the full details of the diagnosis?"
Incorrect Answer ImageG."Your condition is serious; how much would you like to know about it?"

ReKap

Remember the SPIKES protocol for delivering bad news.


Before breaking bad news to a patient about a diagnosis, adequate measures should be taken to make
sure the patient is comfortable and to assess how much the patient already knows; then the doctor
can assess how much the patient wants to know.
Providing reassurance and support follow giving the patient the news about the diagnosis.

A 70-year-old man comes to the physician because of recurrent episodes of pain in his left knee.
He has a history of type 2 diabetes mellitus and osteoarthritis. Current medications include
metformin and naproxen. His temperature is 37°C (98.6°F). Examination of the left knee shows
tenderness, swelling, and erythema. Radiographs of the left knee show punctate and linear
radiopaque deposits in the menisci and articular hyaline cartilage. Joint aspiration shows
rhomboid crystals with weak positive birefringence. Which of the following is the most
appropriate pharmacotherapy to prevent further painful episodes in this patient? 
Incorrect Answer ImageA.Alendronate
Incorrect Answer ImageB.Allopurinol
Incorrect Answer ImageC.Calcitonin
Correct Answer ImageD.Colchicine
Incorrect Answer ImageE.Probenecid

Paramedics bring a 38-year-old woman to the emergency department for evaluation and
treatment of massive hematemesis. She is found to have a posterior duodenal ulcer from which
bright red blood is gushing. The bleeding is unable to be stopped endoscopically, so she
undergoes a Billroth II procedure (antrectomy and gastro-jejunal anastomosis). After resuming a
normal diet, she reports sweating, dizziness, abdominal pain, diarrhea, palpitations, and facial
flushing 15–20 minutes after eating. Which of the following is the most likely cause of her
symptoms? 
Incorrect Answer ImageA.Afferent loop syndrome
Incorrect Answer ImageB.Anastomotic leak
Correct Answer ImageC.Dumping syndrome
Incorrect Answer ImageD.Gastroparesis
Incorrect Answer ImageE.Lactose intolerance
Incorrect Answer ImageF.Postvagotomy diarrhea
Incorrect Answer ImageG.Short-bowel syndrome
A couple comes to the physician because they have been unable to conceive for the past 15
months. They have been having regular, unprotected intercourse 3–4 times a week. The woman
is 24 years old. Her menstrual cycle is regular. Her past medical and surgical history is
unremarkable. Her physical examination is normal. The man is 27 years old. His history is not
significant for testicular trauma, sexually transmitted infections, or any previous surgeries. He
works as a banker and is not exposed to any hazardous environmental agents. He does not
smoke, drink alcohol, or use any recreational drugs. His physical examination shows normal-
appearing genitalia. A semen analysis is performed which shows a decreased sperm count,
abnormal morphology, and a normal ejaculate volume. Which of the following is the most
appropriate next step in management?
Incorrect Answer ImageA.Check serum FSH, LH, and testosterone levels of the male partner
Incorrect Answer ImageB.Perform hysterosalpingogram on the female partner
Correct Answer ImageC.Repeat the semen analysis in 3 months
Incorrect Answer ImageD.Tell the couple they will require a sperm donor to achieve pregnancy
Incorrect Answer ImageE.Tell the couple they will require in vitro fertilization (IVF) to achieve
pregnancy

A 58-year-old man presents for a routine medical check-up. He denies any symptoms and is not
currently on any medications. Fundoscopic examination shows generalized retinal arteriolar
constriction, and the light reflex is broad and dull. There is noted arteriovenous nicking, and
multiple “cotton wool spots” are also seen. Visual acuity is 20/25 in both eyes. These findings
are most suggestive of which of the following?
Incorrect Answer ImageA.Central retinal artery occlusion
Incorrect Answer ImageB.Central retinal vein occlusion
Correct Answer ImageC.Hypertensive retinopathy
Incorrect Answer ImageD.Non-proliferative diabetic retinopathy
Incorrect Answer ImageE.Proliferative diabetic retinopathy

The wife of an elderly patient with chronic obstructive pulmonary disease (COPD) calls the
physician because her husband is extremely short of breath and appears "blue." The patient has a
long history of COPD and has always been compliant with his medications: albuterol inhalers,
rotating antibiotics, and theophylline. According to his wife, he has developed an increasingly
productive cough and shortness of breath over the past few days. The patient is brought to the
hospital by ambulance, where he is found to be barely responsive and cyanotic. It is determined
that on arrival at his house, Emergency Medical Service (EMS) placed him on a 100% non-
rebreather mask. Which of the following mechanisms is responsible for the clinical deterioration
secondary to his treatment?
Incorrect Answer ImageA.Decreased PCO2
Incorrect Answer ImageB.Decreased PO2
Correct Answer ImageC.Increased PO2
Incorrect Answer ImageD.Metabolic acidosis
Incorrect Answer ImageE.Metabolic alkalosis

A 12-year-old boy is brought to the physician by his parents because of a "changing spot on his
scalp." The patient was born with a 1x1 cm bald spot near the crown of his scalp that has
gradually enlarged over time. The lesion always had a yellowish color without hair. Over the
past few months, the parents noted that it had become warty and more intensely yellow in color
and are concerned about these changes. The patient is otherwise healthy and has no significant
medical problems. He has not received any medication in the previous 6 months and has no
known drug allergies. Examination of the scalp shows a well-demarcated, 2x3 cm yellow,
verrucous plaque with no hair. The surrounding scalp hair appears to be of normal quality and
density. Which of the following is the most likely diagnosis?
Incorrect Answer ImageA.Alopecia areata
Incorrect Answer ImageB.Aplasia cutis congenita
Incorrect Answer ImageC.Inflammatory linear verrucous epidermal nevus
Correct Answer ImageD.Nevus sebaceous
Incorrect Answer ImageE.Sebaceous hyperplasia

A 50-year-old woman comes to the physician because she is "feeling poorly." The woman has a
known history of hypothyroidism and diabetes mellitus diagnosed 5 years ago. She was recently
treated with a 7-day course of trimethoprim-sulfamethoxazole for recurrent urinary tract
infection. Current medications include levothyroxine, captopril, and metformin. Her temperature
is 37.5ºC (99.5ºF), pulse is 89/min, respirations are 14/min, and blood pressure is 130/85 mm
Hg. Physical examination shows pallor of the skin and mucosal membranes. She has a
hematocrit of 30% and blood smear shows enlarged erythrocytes that have enlarged central pale
areas. Neutrophils and other white blood cells are present in normal numbers. Some of the
neutrophils have multiple nuclei. Stool guaiac is negative. Which of the following is most likely
responsible for this patient's anemia?
Correct Answer ImageA.Autoimmune disease
Incorrect Answer ImageB.Celiac disease
Incorrect Answer ImageC.Diphyllobothrium latum infection
Incorrect Answer ImageD.Poor diet
Incorrect Answer ImageE.Recent antibiotic use

A 68-year-old woman comes to her physician because of a nodule on her right eyelid for the past
5 years. She does not report pain in the lesion, but it has been slowly growing. She has a history
of recurrent chalazia in the right upper eyelid which resolves within weeks with treatment. She
has no other history of significant medical illnesses. Physical examination of the right upper
eyelid shows a firm, painless, indurated, nodular lesion with minimal conjunctival injection. On
palpation, a small amount of pus can be expressed. The upper eyelid has a yellowish appearance
with a loss of eyelashes around the area of induration. The rest of the physical examination,
including visual acuity and pupillary examination, is normal. Which of the following is the most
appropriate management?
Correct Answer ImageA.Biopsy of the nodule
Incorrect Answer ImageB.Hot compresses
Incorrect Answer ImageC.Hot compresses and topical antibiotics
Incorrect Answer ImageD.Incision and drainage of the nodule then topical antibiotics
Incorrect Answer ImageE.Topical corticosteroids

An 87-year-old man with Alzheimer disease is brought to the emergency department because of
a 2-day history of diarrhea and vomiting. Over the last 24 hours, the patient has not been eating
or drinking and has been more lethargic and confused than his baseline. His temperature is
37.0ºC (98.6ºF), pulse is 133/min, respirations are 18/min, and blood pressure is 90/70 mm Hg.
Examination shows dry mucous membranes. Laboratory studies show:
Na+ 148 mEq/L
K+ 5.5 mEq/L
Cl −
90 mEq/L
HCO3− 26 mEq/L
Glucose 116 mg/dL
BUN 47 mg/dL
Creatinine 2.3 mg/dL
His baseline serum creatinine level is 1.4 mg/dL. Which of the following findings is most likely
in this patient? 
Incorrect Answer ImageA.Fractional excretion of sodium of 3%
Incorrect Answer ImageB.Granular casts
Incorrect Answer ImageC.Urine osmolality of 220 mOsm/L
Correct Answer ImageD.Urine sodium of 16 mEq/L
Incorrect Answer ImageE.White blood cell casts

A 32-year-old woman with hypertension presents to the office with difficulty swallowing. The
patient reports having difficulty swallowing both solids and liquids, relieved by drinking water,
for the past 4 months. She has lost approximately 10 lb during this same period of time. On
physical examination, she is a moderately obese woman who has no apparent discomfort. Her
temperature is 98.6ºF (37.0ºC), pulse is 72/min, and blood pressure is 137/84 mm Hg. The rest of
the examination shows no abnormality. Esophagram shows smooth, point-like tapering of the
distal esophagus and dilatation of the proximal esophagus. Which of the following is the most
likely explanation for her symptoms?
Correct Answer ImageA.Achalasia
Incorrect Answer ImageB.Diffuse esophageal spasm
Incorrect Answer ImageC.Esophageal cancer
Incorrect Answer ImageD.Schatzki ring
Incorrect Answer ImageE.Scleroderma

A 32-year-old woman comes to the physician because of a red rash on her cheeks, nose, and
forehead for 1 year. She has had facial flushing and redness and occasionally small pimples. She
states that the rash is worsened by sun exposure and drinking hot beverages. She has had dryness
and stinging in her eyes for the past 3 months. Her past medical history is unremarkable, and she
takes no medications. Her temperature is 36.8°C (98.2°F) and blood pressure is 115/70 mm Hg.
Physical examination shows mild erythema and telangiectasias over the malar cheeks, nasal
sidewalls, and forehead; there are few pustules. The eyelid margins are erythematous with mild
crusting. Which of the following is the most likely diagnosis? 
Incorrect Answer ImageA.Acne vulgaris
Incorrect Answer ImageB.Dermatomyositis
Correct Answer ImageC.Rosacea
Incorrect Answer ImageD.Sjögren syndrome
Incorrect Answer ImageE.Systemic lupus erythematosus
A 43-year-old woman comes to the physician because of a 3-month history of dyspnea on
exertion. She denies chest pain. She has a history of rheumatoid arthritis, but she has no family
history of coronary artery disease. A lateral chest x-ray film shows calcification of the heart
border. A diagnosis of constrictive pericarditis is made. Which of the following physical
examination findings is most likely to be seen in this patient? 
Incorrect Answer ImageA.Central cyanosis
Incorrect Answer ImageB.Opening snap and diastolic rumble
Correct Answer ImageC.Pericardial knock
Incorrect Answer ImageD.Systolic murmur
Incorrect Answer ImageE.Widely split S2

A 67-year-old woman comes to the emergency department because of constant, watery nasal
discharge and a headache for 5 days. She has had no cough, fever, chills, or sneezing. She took
loratadine for the past 4 days, which did not improve her condition. She has a history of
hypertension, osteoporosis, and depression. Current medications include hydrochlorothiazide,
calcium supplements, and venlafaxine. Her temperature is 37ºC (98.6ºF), blood pressure is
138/76 mm Hg, pulse is 88/min, and respirations are 18/min. Examination shows a clear, thin
nasal discharge. Beta-2-transferrin test of the nasal discharge is positive. Which of the following
is the most likely complication of this patient's condition? 
Incorrect Answer ImageA.Haemophilus influenzae meningitis
Incorrect Answer ImageB.Neisseria meningitidis meningitis
Incorrect Answer ImageC.Staphylococcus aureus meningitis
Incorrect Answer ImageD.Staphylococcus epidermidis meningitis
Correct Answer ImageE.Streptococcus pneumoniae  meningitis

A 38-year old man is admitted to the trauma service with fractures of his pelvis and right femur
secondary to a motor vehicle accident. His past medical history is significant for gout, and his
daily medications include allopurinol, folate, and thiamine. On day 3 of hospitalization, the
patient starts randomly calling the nurse "Auntie." He states that he is in a shopping mall and
becomes agitated when the nurse tells him he is in a hospital. He tries to stand up to leave, but
has an unsteady gait and falls back into bed. His blood pressure is 160/100 mm Hg, pulse is
102/min, and respirations are 16/min. Which of the following is the most appropriate initial step
in the management of this patient's altered mental status?
Incorrect Answer ImageA.Clonidine
Incorrect Answer ImageB.Haloperidol
Correct Answer ImageC.Lorazepam
Incorrect Answer ImageD.Naloxone
Incorrect Answer ImageE.Thiamine

A 32-year-old man comes to the physician because of a 4-day history of back pain that came on
suddenly during a workout. He says that the pain is located to the right of the spine. He recently
began lifting weights. He has had no fever, morning stiffness, numbness or tingling of the
extremities, weakness, or incontinence. He drinks wine occasionally. He does not use illicit
drugs. His temperature is 36.8°C (98.2°F) and pulse is 72/min. Physical examination shows no
deformities of the spine. Range of motion is limited by back pain. There is tenderness on
palpation over the paraspinal region. Muscle strength is 4/5 bilaterally in the upper and lower
extremities with normal reflexes. There is no tenderness to palpation over the muscles of the
legs. Which of the following is the most likely diagnosis? 
Incorrect Answer ImageA.Ankylosing spondylitis
Incorrect Answer ImageB.Intervertebral disk herniation
Incorrect Answer ImageC.Lumbar interspinous bursitis
Correct Answer ImageD.Lumbosacral muscle strain
Incorrect Answer ImageE.Sacroiliac joint injury

A 56-year-old man arrives in the emergency department after vomiting a "considerable" amount
of bright red blood 1 hour earlier. He has never had this happen before. His past medical history
is notable for coronary artery disease, hypertension, and emphysema. He has a significant
smoking history, but he quit 5 years ago. He does not drink a significant amount of alcohol. On
physical examination, his temperature is 36.3ºC (97.6ºF), pulse is 100/min, and blood pressure is
95/55 mm Hg. Head and neck examination shows supple neck and some facial edema. He has
diffusely distant breath sounds. Cardiovascular examination is normal. Abdominal examination
is unremarkable. There is no peripheral clubbing, cyanosis, or edema of extremities. Emergent
upper endoscopy is performed, which shows numerous bleeding varices in the mid-esophagus,
with a normal distal esophagus and stomach. Which of the following is the most likely
explanation for the patient's presentation? 
Correct Answer ImageA.Compression of superior vena cava
Incorrect Answer ImageB.Fibrosis of hepatic parenchyma
Incorrect Answer ImageC.Thrombosis of hepatic vein
Incorrect Answer ImageD.Thrombosis of portal vein
Incorrect Answer ImageE.Thrombosis of splenic vein

A 55-year-old man with a history of prostate cancer status post radiation and chemotherapy 6
months ago comes to the emergency department because of progressive weakness in his right
arm and leg since yesterday. He is unable to walk. He denies headaches, seizures, visual
disturbances, or trauma. He takes no medications. He does not smoke, drink alcohol or use any
recreational drugs. His temperature is 36.8°C (98.2°F), pulse is 80/min, respirations are 22/min,
and blood pressure is 140/70 mm Hg. On physical examination, muscle strength is 0/5 in the
right upper and lower extremities. There is decreased sensation to proprioception and vibration
of the right upper and lower extremities; in the left extremities, there is decreased sensation to
pinprick and temperature. A Babinski sign is present on the right side. All cranial nerves are
grossly intact. Which of the following is the most appropriate initial step in management?
Incorrect Answer ImageA.CT myelography of the spinal column
Correct Answer ImageB.Intravenous dexamethasone
Incorrect Answer ImageC.Lumbar puncture
Incorrect Answer ImageD.MRI of the spine
Incorrect Answer ImageE.Urgent surgical intervention

A 54-year-old man comes to the emergency room because of sudden onset of flank pain and
blood in his urine eight hours ago. His past medical history is significant for hypertension,
variably controlled with hydrochlorothiazide. He does not smoke, drink alcohol, or use
recreational drugs. His elder brother and his maternal aunt both had “some sort of kidney
disease.” On physical examination, he appears to be in moderate pain. His pulse is 90/min and
blood pressure is 140/79 mm Hg. The examinations of the head and neck, chest, and heart are
normal. Left greater than right flank tenderness is elicited with palpation. Extremities are normal.
Urinalysis shows 15–20 WBCs/HPF and 20–25 RBCs/HPF. Renal ultrasound shows that both
kidneys are enlarged, with multiple large cysts within each. Which of the following would be
most likely to develop in this patient?
Incorrect Answer ImageA.Bony metastases
Correct Answer ImageB.Dialysis-requiring chronic kidney disease
Incorrect Answer ImageC.Nephrotic syndrome
Incorrect Answer ImageD.Obstructive uropathy
Incorrect Answer ImageE.Periportal fibrosis and dilation of intrahepatic bile ducts

A 45-year-old man with a history of hypercholesterolemia comes to his physician after noticing a
pink color to his urine, joint pains in both knees, and a rash on his legs for 2–3 weeks. The rash is
tender and did not go away with topical corticosteroids. He was previously well except for
hypercholesterolemia now treated with simvastatin and an upper respiratory tract infection 4
weeks ago. His temperature is 36.9ºC (98.4ºF), blood pressure is 150/90 mm Hg, pulse is 84/min,
and respirations are 16/min. There is a raised, tender, purplish rash on his buttocks and legs.
Head examination is normal. His neck is supple with no JVD. Lungs are clear to auscultation
with no wheezes or crackles. Cardiac examination shows normal S1 and S2 with no rub or
gallop. Abdomen is soft, nontender, and non-distended. Bowel sounds are present and active.
Extremities show mild swelling in both knees with limited range of motion; there is no ankle or
foot edema.
Urinalysis shows:
Specific gravity 1.005
pH 6.0
Blood 1+
Protein 2+
Ketones Negative 
Glucose Negative
RBCs 10–12/HPF 
WBCs 0/HPF
Epithelial cells Negative
Casts Negative
Crystals Negative
BUN 20 mg/dL
Creatinine 0.7 mg/dL
A 24-h urine collection shows 2.1 g of protein in 24 h. Complement levels are normal. ANCA
(anti-neutrophil cytoplasmic antibodies) are negative. Serum IgA is elevated. Which of the
following is the most likely diagnosis?
Incorrect Answer ImageA.Anti-glomerular basement membrane disease
Incorrect Answer ImageB.Granulomatosis with polyangiitis
Correct Answer ImageC.Henoch–Schönlein purpura
Incorrect Answer ImageD.Membranoproliferative glomerulonephritis
Incorrect Answer ImageE.Polyarteritis nodosa
A 62-year-old man with history of hypertension, hyperlipidemia, claudication, chronic renal
failure, and type 2 diabetes mellitus comes to the emergency department because of substernal
chest pressure for the past 1 hour. He states that the pain comes in waves and is associated with
shortness of breath and diaphoresis. He says he has had anginal-type chest pain in the past, but
this pain is different in nature and more severe. ECG shows T-wave flattening and inversions in
the lateral and inferior leads. Lab studies show elevated troponin level. Treatment with
sublingual nitroglycerin, aspirin, metoprolol, enoxaparin, morphine, and a glycoprotein IIb/IIIa
inhibitor is begun, but the patient continues to have chest pressure. His blood pressure is 100/55
mm Hg. Which of the following is the most appropriate next step in patient care? 
Incorrect Answer ImageA.Admission to a cardiac care unit for monitoring and medical treatment
Incorrect Answer ImageB.Cardiothoracic surgery evaluation for coronary artery bypass grafting
Incorrect Answer ImageC.Continued management in the emergency department until 2 more sets of
enzymes are available
Incorrect Answer ImageD.Thrombolytic therapy administered immediately in the emergency
department
Correct Answer ImageE.Urgent transfer to coronary catheterization laboratory for angiography

A 26-year-old man is brought to the emergency department after a syncopal episode that
occurred during exercise. He immediately regained consciousness, and there was no seizure
activity noted. His blood pressure is 130/70 mm Hg. On examination, there is a brisk upstroke in
the carotid pulse. There is a systolic ejection murmur with a systolic thrill at the left lower sternal
border. The intensity of the murmur is increased with standing. Which of the following is the
most appropriate next step in the evaluation of this patient? 
Incorrect Answer ImageA.Chest radiograph
Correct Answer ImageB.Echocardiography
Incorrect Answer ImageC.Pulmonary function tests
Incorrect Answer ImageD.Stress test
Incorrect Answer ImageE.Ultrasound of the abdominal aorta

A 54-year-old man comes to the physician because of epigastric pain that radiates to the back for
12 weeks. The pain has become progressively worse. He also has noticed yellowing of the eyes
and skin. His stools have appeared white and his urine is tea-colored. He also reports weight loss,
persistent nausea, and anorexia. He denies fevers, night sweats, hematemesis, melena, or
hematochezia. He does not use alcohol, tobacco, or illicit drugs. He has lost 4.9 kg (11 lb) since
his last visit. Physical examination shows scleral icterus. Lung and heart examinations are
normal. There is pain on palpation of the epigastrium without hepatosplenomegaly. His
neurologic examination is nonfocal. Liver function tests show:
Aspartate aminotransferase (AST) 56 U/L
Alanine aminotransferase (ALT) 45 U/L
Alkaline phosphatase 569 U/L
Bilirubin, total 8.2 mg/dL
Bilirubin, conjugated 7.5 mg/dL
Which of the following is the most appropriate initial step in management? 
Correct Answer ImageA.Abdominal CT scan
Incorrect Answer ImageB.Abdominal ultrasound
Incorrect Answer ImageC.Endoscopic retrograde cholangiopancreatography (ERCP)
Incorrect Answer ImageD.Endoscopic ultrasound with fine-needle aspiration (FNA)
Incorrect Answer ImageE.Percutaneous liver biopsy

A 20-year-old man presents to the emergency department by the paramedic staff in an


unconscious state. He was knocked unconscious during a college football practice. He was in this
state for approximately 2 minutes. He cannot remember events preceding to the accident or right
after it. He reports a severe headache and tingling in his left hand. He is in mild distress. His vital
signs are as follows: temperature is 37.0ºC (98.6ºF), pulse is 73/min, respirations are 18/min and
blood pressure is 110/60 mm Hg. On physical examination, he is in mild distress. There is a
contusion above his right eye. Neurologic examination shows a decreased pupillary response to
light in the left eye. Which of the following is the most appropriate next step in management?
Correct Answer ImageA.CT scan of the head without contrast
Incorrect Answer ImageB.Intubation, hyperventilation, and mannitol
Incorrect Answer ImageC.Reassurance, acetaminophen, and neurologic checks every 4 hours at
home
Incorrect Answer ImageD.Reassurance, acetaminophen, and observation in hospital overnight
Incorrect Answer ImageE.Skull x-rays

A 60-year-old man who has a history of seizure disorder and a recent seizure 3 days ago is
brought to the emergency department with a fever and a productive cough. His temperature is
38.7ºC (101.7ºF), pulse is 118/min, respirations are 24/min, and blood pressure is 126/72 mm
Hg. On physical examination, the patient is noted to have poor oral hygiene and decaying teeth.
Auscultation of the lungs demonstrates decreased breath sounds and rales. The remainder of the
physical examination is normal. Chest radiograph result is pending. Which of the following is the
most likely causative microorganism in this patient?
Correct Answer ImageA.Bacteroides melaninogenicus
Incorrect Answer ImageB.Moraxella catarrhalis
Incorrect Answer ImageC.Mycoplasma pneumoniae
Incorrect Answer ImageD.Pseudomonas aeruginosa
Incorrect Answer ImageE.Toxoplasma gondii

A 69-year-old woman is brought by ambulance to the emergency room complaining of inability


to walk and paresthesias in both legs. She has a history of end-stage renal disease due to
hypertension and diabetes. She has missed her last two hemodialysis appointments. Physical
examination reveals a flaccid paraparesis. The serum creatinine is 9.4 mg/dL. A stat
electrocardiogram is ordered to gauge the severity of her electrolyte abnormality. Chemistries are
pending. Which of the following is the earliest change likely to be seen on the electrocardiogram
in this patient's most probable electrolyte abnormality?
Incorrect Answer ImageA.Diffuse P wave flattening
Incorrect Answer ImageB.Prolonged PR interval
Incorrect Answer ImageC.Prominent U waves
Correct Answer ImageD.Tall, peaked T waves
Incorrect Answer ImageE.Widened QRS

A 40-year-old woman comes to the physician because of a 3-year history of lesions on her neck
that she finds cosmetically unattractive. She has had no fevers, chills, night sweats, or weight
loss. Her past medical history is unremarkable. She takes no medications, and she has no known
allergies to medications. Physical examination shows multiple small, light brown, pedunculated
lesions on the neck. The largest lesion is 4 mm in diameter. Which of the following is the most
likely diagnosis?
Correct Answer ImageA.Acrochordons
Incorrect Answer ImageB.Lentigines
Incorrect Answer ImageC.Lipomas
Incorrect Answer ImageD.Seborrheic keratoses
Incorrect Answer ImageE.Spider angiomas

A 59-year-old woman comes to the physician because of a painful facial rash for 3 days. She
underwent surgical excision of a basal cell carcinoma on her forehead 1 week ago. She has never
had an eruption like this before. Her daughter is an ICU nurse and so has been changing the
dressings on her forehead. The patient has been caring for her grandson who has chickenpox.
The patient has a history of rheumatoid arthritis treated with NSAIDs. Her temperature is 36.8°C
(98.2°F). Physical examination shows a vesicular and crusted eruption on the right side of the
forehead, scalp, nose, and nasal tip. The eruption does not cross the midline. Which of the
following put the patient at greatest risk for developing this facial rash?
Incorrect Answer ImageA.Contact with grandchild with active chickenpox
Incorrect Answer ImageB.Contact with pathogens from daughter's hands
Incorrect Answer ImageC.Immunosuppression due to rheumatoid arthritis
Correct Answer ImageD.Recent surgery for carcinoma on the forehead
Incorrect Answer ImageE.Older age

A study is conducted to assess a new screening test for lymphoma in 100 volunteers. The test
involves taking blood from screening candidates, attaching antibodies to a unique antigen
(CD33-test), and analyzing the blood through flow cytometry. Those showing positivity on the
test with the presence of the antigen CD33-test are believed to have lymphoma. All subjects
undergo bone marrow biopsy for confirmation of the presence or absence of lymphoma. The
results are shown: 
Biopsy Results

CD33-Test Results Positive Negative

Positive 25 15

Negative 40 20
Which of the following ratios most likely represents this new test's sensitivity for lymphoma? 
Incorrect Answer ImageA.20/35
Incorrect Answer ImageB.20/60
Incorrect Answer ImageC.25/40
Correct Answer ImageD.25/65
Incorrect Answer ImageE.25/100

A 65-year-old man comes to the clinic for a follow-up visit. He has a long history of diabetes
mellitus and recently had his diabetic screening exams, including urine microalbumin check, foot
examination, lipid levels, and hemoglobin A1c. Having failed oral therapy, the patient's diabetes is
now well controlled with a combination of short- and long-acting insulin. His foot exam is
unremarkable. All laboratory values are within established normal ranges. However, during the
man's annual eye exam 2 months ago, the ophthalmologist's funduscopic examination showed
diffuse, opaque white spots throughout the fundus; a few scattered hard exudates; and multiple
small microaneurysms. Visual examination today is 20/25 OD (right eye) and 20/20 OS (left
eye). The man denies any floaters, diplopia, or blurred vision. Which of the following is most
likely the cause of these findings?
Incorrect Answer ImageA.Age-related ocular changes
Incorrect Answer ImageB.Diabetic cataract
Incorrect Answer ImageC.Macular edema
Correct Answer ImageD.Nonproliferative diabetic retinopathy
Incorrect Answer ImageE.Proliferative diabetic retinopathy

A previously healthy 70-year-old man comes to the physician for a routine health maintenance
examination. His blood pressure is 155/95 mm Hg and pulse 75/min. Laboratory studies show: 
Na+ 138 mEq/L 
K+ 3.3 mEq/L 
Cl− 95 mEq/L 
HCO3 −
32 mEq/L 
Urea nitrogen (BUN)  14 mg/dL 
Creatinine  0.9 mg/dL
Aldosterone-to-renin ratio  28 
CT scan of the adrenal glands shows bilateral adrenal hyperplasia. Which of the following is the
most appropriate pharmacotherapy? 
Incorrect Answer ImageA.Amiloride
Incorrect Answer ImageB.Amlodipine
Incorrect Answer ImageC.Clonidine
Incorrect Answer ImageD.Hydrochlorothiazide
Correct Answer ImageE.Spironolactone

A 38-year-old man comes to the physician because of a painful and red right eye. He works
grinding industrial metal parts and is worried that he may have some metal in his right eye.
Yesterday afternoon at work the patient began to suffer from pain in his right eye. The pain
progressed throughout the afternoon and evening and was severe enough to cause the patient to
have difficulty sleeping. The pain is described currently as “excruciating” and he is now too
uncomfortable to work, drive, or read. He has washed his eye numerous times but has difficulty
keeping the affected eye open. Examination shows an injected red right eye with mild ciliary
flush, but without any discharge or corneal opacity. The red reflex appears normal and the
chamber is well-formed. The pupil is round, small, and minimally reactive on the right, making
proper fundoscopic examination difficult. He has photophobia in the affected eye. There are no
lid abnormalities or obvious foreign bodies present. Which of the following is the most
appropriate next step in the management of this patient?
Incorrect Answer ImageA.Administer topical ophthalmic dexamethasone
Correct Answer ImageB.Administer topical ophthalmic tetracaine
Incorrect Answer ImageC.Administer topical ophthalmic tobramycin
Incorrect Answer ImageD.Fluorescein slit-lamp examination
Incorrect Answer ImageE.Patch the eye and follow up in 24 hours

A 25-year-old medical student comes to the student health clinic 1 month prior to traveling to the
Dominican Republic for a 2-month clinical rotation. He plans to work in a community clinic in a
rural area. His medical history is unremarkable, and he takes no medications. He has no allergies.
His blood pressure is 108/74 mm Hg and pulse is 64/min. Examination shows no abnormalities.
Which of the following is the most appropriate malarial prophylaxis in this patient? 
Incorrect Answer ImageA.Atovaquone
Correct Answer ImageB.Chloroquine
Incorrect Answer ImageC.Ciprofloxacin
Incorrect Answer ImageD.Mefloquine
Incorrect Answer ImageE.Trimethoprim/sulfamethoxazole

A 60-year-old man is brought to the physician by his daughter because of the frequent urge to
urinate with some episodes of incontinence for the last 3 weeks. The daughter reports that he also
has had difficulty remembering names and recognizing his family members. Three days ago, he
went to the local grocery store by himself and got lost on the way home. He recently retired from
his job as a teacher. Before his current symptoms began, he went to the gym three times weekly
and was involved in his community. He appears confused and is in no distress. Muscle strength
is 5/5 in all extremities. Sensation is normal. He has a shuffling gait with small steps and postural
instability. Which of the following is most likely to be seen in this patient? 
Correct Answer ImageA.Cognitive impairment improves with surgery
Incorrect Answer ImageB.Cytoplasmic eosinophilic inclusions in the substantia nigra
Incorrect Answer ImageC.Increased amyloid proteins on CSF analysis
Incorrect Answer ImageD.Multiple subcortical infarcts visualized on CT scan
Incorrect Answer ImageE.Opening CSF pressure of 260 cm H2O

A 40-year-old woman comes to the physician because of a 1-month history of a progressive,


light-colored rash on her chest that has been rapidly spreading. Her past medical history is
unremarkable, and she takes no medications. She recently started working as a cook in a local
fast-food restaurant. Her husband and 2 school-aged children are in good health. Her vital signs
are normal. Skin examination shows well-demarcated, slightly scaly, hypopigmented patches.
The remainder of the physical examination is normal. Which of the following is the most
appropriate next step in management?
Incorrect Answer ImageA.Monospot test
Correct Answer ImageB.Potassium hydroxide preparation
Incorrect Answer ImageC.Skin biopsy
Incorrect Answer ImageD.Skin culture
Incorrect Answer ImageE.Tzanck smear

A 62-year-old man comes to the physician because of erectile dysfunction. He reports that over
the last few years, he has lost interest in sex, although he still loves his wife and does wish to
have sex with her. He has lost the desire to masturbate as well and does not remember having
any nocturnal erections recently. He has no prior history of sexual dysfunction. His past medical
history is significant for osteoporosis (diagnosed after a recent radial fracture) and hypertension
that is well-controlled with valsartan. He denies having any symptoms of depression. Physical
examination is unremarkable except for a small, reducible right inguinal hernia. The testes are
descended bilaterally and measure 4 cm in greatest dimension. Which of the following is the
most appropriate next step in management?
Correct Answer ImageA.Determine serum free testosterone level
Incorrect Answer ImageB.Discontinue valsartan
Incorrect Answer ImageC.Refer the patient for psychosocial evaluation
Incorrect Answer ImageD.Schedule an appointment with the patient's wife
Incorrect Answer ImageE.Start a trial of a phosphodiesterase-5 inhibitor

A 34-year-old man comes to the physician because of a 2-year history of increasing pain and
stiffness in his back. His symptoms improve with activity. Recently he has awakened at night
because of the pain. He has had a 2.3 kg (5 lb) weight loss over the last 5 months. He works in
construction. Which of the following is the most appropriate next step in diagnosis? 
Correct Answer ImageA.Assess spinal mobility using the Schober test
Incorrect Answer ImageB.Check rheumatoid factor and anti-cyclic citrullinated peptide antibody
Incorrect Answer ImageC.Measure C-reactive protein levels and determine presence or absence of
HLA-B27
Incorrect Answer ImageD.Obtain MRI scan of his spine
Incorrect Answer ImageE.Obtain plain radiographs of the pelvis

A 28-year-old patient who has end-stage renal disease (ESRD) on continuous ambulatory
peritoneal dialysis (CAPD) for 2 months is brought to the emergency department because of
fever, abdominal pain, and cloudy dialysis fluid. There is no diarrhea or vomiting and the pain
has been present for approximately 12 hours. The patient has ESRD secondary to chronic
glomerulonephritis; there is no history of diabetes, urinary infections, or antibiotic use. Her
temperature is 38.9ºC (102.0ºF) and blood pressure is 110/70 mm Hg. Her throat is clear, and
lungs are clear to auscultation. Cardiac examination shows a grade 2/6 systolic murmur.
Abdominal examination shows decreased bowel sounds with diffuse tenderness. There is mild
rebound tenderness. There is no erythema over the dialysis catheter subcutaneous tunnel or at its
exit site. There is no dependent edema or skin rash. A complete blood count shows a leukocyte
count of 14,200/mm3; hemoglobin is 12.5 g/dL. Peritoneal fluid is cloudy with 1,000 white blood
cells/mm3, 85% of which are polymorphonuclear leukocytes. Gram stain of the peritoneal fluid is
negative. Cultures of blood and peritoneal dialysis fluid are taken. Which of the following is the
most appropriate initial step in management?
Incorrect Answer ImageA.Fluconazole
Incorrect Answer ImageB.Immediate removal of dialysis catheter
Correct Answer ImageC.Intraperitoneal administration of cefazolin, together with ceftazidime
Incorrect Answer ImageD.Intravenous gentamicin
Incorrect Answer ImageE.Oral ciprofloxacin

An 82-year-old woman who has advanced Alzheimer disease is brought to the emergency
department because of fever and change from her baseline mental status. She has taken very little
food or drink for the last 4 days. Vital signs on arrival show temperature 38.9ºC (102ºF), blood
pressure 110/60 mm Hg, pulse 89/min, respirations 30/min, and oxygen saturation 92% on room
air. Initial physical examination shows crackles in the right lung base and dullness to percussion,
dry mucous membranes and decreased capillary refill. Neurologic examination is nonfocal and
not significantly changed from her previous admission 2 weeks ago. Guttural sounds and
inattention, however, have now replaced her short, one-sentence conversations. Chest radiograph
shows a right lower lobe infiltrate. Laboratory examination is significant for a serum sodium
level of 115 mEq/L. A CT scan of the head is unremarkable. The house officer on call, after
receiving all the pending laboratory results, admits the patient and initiates treatment for her
electrolyte abnormalities. He also writes orders to administer 500 mL/hour of 3% saline slowly
for 5 hours. This patient is still at greatest risk for developing which of the following conditions?
Incorrect Answer ImageA.Acute cerebrovascular accident
Incorrect Answer ImageB.Acute uncal herniation
Correct Answer ImageC.Central pontine myelinolysis
Incorrect Answer ImageD.Cerebral edema
Incorrect Answer ImageE.Internuclear ophthalmoplegia
Incorrect Answer ImageF.Spasmodic torticolli

A 29-year-old man comes to the clinic because of generalized fatigue. The patient is new to the
clinic and reports that over the past few weeks, he has been feeling much more tired than usual.
He also reports that this happens to him about one to two times a year and that other physicians
have told him that he is "overworked." His review of symptoms is notable for frequent sneezing,
postnasal drip, eye-watering, and an itch of his posterior pharynx. These symptoms tend to be
worse in the spring and summer. His past medical history is remarkable only for mild asthma
induced by being outdoors. He takes no regular medications but does take diphenhydramine on
occasion. He denies using tobacco, ethanol, or illicit substances. Which of the following is the
most appropriate diagnostic test at this time?
Incorrect Answer ImageA.Blood radioallergosorbent test
Correct Answer ImageB.No further testing is indicated
Incorrect Answer ImageC.Serum protein electrophoresis
Incorrect Answer ImageD.Sinus x-rays
Incorrect Answer ImageE.Skin-prick testing

A 43-year-old man comes to the physician because of a 4-year history of joint pain. He has pain
in the proximal and distal small joints involving the right hand, the left knee, the right ankle, and
the left elbow. He has had morning stiffness that lasts for 2 hours. Past medical history is
significant only for scaly patches on his elbows for which he never saw a doctor. His temperature
is 37.0ºC (98.6ºF), blood pressure is 120/70 mmHg, pulse is 76/min, and respirations are 13/min.
Examination is shown. There is pitting of the nails of both hands, and the distal third
interphalangeal joint is partially subluxed. Laboratory studies show:
Hemoglobin  14.0 mg/dL
Hematocrit  42%
Leukocyte count  5,000/mm3
Platelet count  200,000/mm3
Rheumatoid factor  Negative
Erythrocyte sedimentation rate (ESR)  46 mm/hr
Which of the following is the most likely diagnosis? 
Incorrect Answer ImageA.Onychomycosis
Incorrect Answer ImageB.Osteoarthritis
Incorrect Answer ImageC.Pseudogout
Correct Answer ImageD.Psoriatic arthritis
Incorrect Answer ImageE.Reactive arthritis
Incorrect Answer ImageF.Rheumatoid arthritis

A 56-year-old man comes to the physician because of muscle weakness, dry skin, and lethargy
for 2 months. He has also felt cold when other individuals were comfortable. He has a history of
asthma and atrial fibrillation. He underwent DC cardioversion 2 years ago. Current medications
include amiodarone, albuterol, and prednisone. His blood pressure is 110/70 mm Hg, pulse is
64/min and regular, and respirations are 12/min. Examination shows dry, coarse skin. There is 1+
pedal edema. Laboratory studies show a TSH level of 8 µU/mL. Which of the following is the
most appropriate next step in management? 
Incorrect Answer ImageA.Advise iodine supplementation
Incorrect Answer ImageB.Discontinue amiodarone immediately
Incorrect Answer ImageC.Order a 12-lead ECG
Correct Answer ImageD.Start levothyroxine
Incorrect Answer ImageE.Stop anti-asthmatic medications

A 78-year-old woman comes to the physician because of pain and stiffness in her hands for 6
months. She has stiffness in the morning that improves after 20 minutes and pain that often
worsens with activity. She occasionally takes ibuprofen when the pain is unbearable but is
generally reluctant to take medication. She has a history of psoriasis and hypertension. Current
medications include a topical steroid and hydrochlorothiazide. Her blood pressure is 123/74 mm
Hg. Examination is shown. No swelling or redness is noted. She has scattered scaly patches on
her arms and back. Which of the following is the most likely diagnosis?
Incorrect Answer ImageA.Gout
Correct Answer ImageB.Osteoarthritis
Incorrect Answer ImageC.Psoriatic arthritis
Incorrect Answer ImageD.Reactive arthritis
Incorrect Answer ImageE.Rheumatoid arthritis

A 56-year-old man is brought to the emergency department because of worsening shortness of


breath. His past medical history is significant for coronary artery disease, hypertension, and
episodes of ventricular tachycardia for which he was placed on amiodarone several months ago.
There are decreased breath sounds bilaterally. Echocardiogram shows a normal ejection fraction,
and pulmonary function testing results are consistent with restrictive lung disease. Amiodarone-
induced pulmonary toxicity is suspected and bronchoscopy with bronchoalveolar lavage is
performed. Which of the following findings is most likely consistent with amiodarone-induced
lung disease?
Correct Answer ImageA."Foamy" macrophages
Incorrect Answer ImageB.Increased eosinophils
Incorrect Answer ImageC.Increased lymphocytes
Incorrect Answer ImageD.Increased neutrophils
Incorrect Answer ImageE.Langerhans cells
Incorrect Answer ImageF.Turbid, PAS-positive material
A 24-year-old woman with a history of epilepsy that has been difficult to control is brought to
the emergency department with fever and a productive cough. According to her mother, the
patient developed symptoms 2–3 days ago. She is worried that, because of her daughter's
symptoms, she is not able to take her to her dental appointment. Her medical history is
significant for a seizure disorder, and her last seizure was 1 week ago. Her temperature is 38.9ºC
(102.0ºF), pulse is 122/min, respirations are 26/min, and blood pressure is 120/70 mm Hg.
Auscultation of the lungs shows crackles in the right upper and lower lobe. Pulse is tachycardic
with normal S1 and S2. Neurologic examination does not show any focal signs. Which of the
following is the most appropriate pharmacotherapy for this patient?
Incorrect Answer ImageA.Azithromycin
Correct Answer ImageB.Clindamycin
Incorrect Answer ImageC.Cefoxitin
Incorrect Answer ImageD.Doxycycline
Incorrect Answer ImageE.Imipenem
Incorrect Answer ImageF.Vancomycin

A 22-year-old woman is brought to the emergency department by her college roommate because
“she is freaking out.” The two of them have been awake all night long studying for their final
exams and have been drinking many cups of coffee to stay awake. The friend reports that 30
minutes after the exam started, the patient became sweaty and fidgety and had to be removed
from the exam. She now looks so frightened that she is almost unable to speak. Her past medical
history is significant for mitral valve prolapse for which she is taking atenolol. The patient
appears very nervous and agitated. Her pulse is 120/min, respirations are 30/min, and blood
pressure is 110/80 mm Hg. There is some perioral numbness. Chest examination shows clear
lungs and a midsystolic click at the apex. Neurologic examination shows hyperreflexia. Which of
the following sets of arterial blood gas values would be most likely in this patient? 
pH PaCO2 PaO2 HCO3– Ionized Ca2+*

(mm Hg) (mm Hg) (mEq/L) (mg/dL)

A. 7.30 52 82 29 3.9

B. 7.32 34 96 18 5.1

C. 7.48 48 94 32 4.5

D. 7.48 32 96 22 4.6

E. 7.52 28 97 24 3.8
* normal value of ionized calcium = 4.4 — 5.3 mg/dL

Incorrect Answer ImageA.A


Incorrect Answer ImageB.B
Incorrect Answer ImageC.C
Incorrect Answer ImageD.D
Correct Answer ImageE.E
A 67-year-old hypertensive man is brought to the emergency department by ambulance because
of abdominal pain. Six hours earlier he was sitting down to eat when he suddenly felt severe
epigastric pain. There was associated nausea. He has a past medical history of duodenal ulcers
treated with clarithromycin, amoxicillin, and omeprazole. His temperature is 37.6°C (99.7°F),
pulse is 126/min, and blood pressure is 146/94 mm Hg. The patient is sweating excessively.
There is no chest tenderness, and breath sounds are clear bilaterally. Cardiac examination shows
muffled heart sounds which are regular in rate and rhythm, with no murmurs, rubs, or gallops.
Electrocardiogram reveals ST-segment elevation in leads II, III, and aVF. Twenty-four hours
post-percutaneous coronary intervention, the patient's blood pressure drops to 104/62 mm Hg,
with pulse 40/min. On examining the patient while sitting up in bed, the jugular venous pulse is
noted to have quite vigorous irregular outward pulsations. After performing hepatojugular reflux,
a persistent increase of the jugular venous pulse is seen. Which of the following is the most
appropriate initial treatment?
Incorrect Answer ImageA.Adenosine
Correct Answer ImageB.Atropine
Incorrect Answer ImageC.IV fluids
Incorrect Answer ImageD.Operate
Incorrect Answer ImageE.Pericardiocentesis with pericardial window
Incorrect Answer ImageF.Transcutaneous pacemaker
Incorrect Answer ImageG.Transvenous pacemaker

A 51-year-old man comes to the emergency department because of a 2-day history of fever and a
productive cough with bloody sputum. He drinks eight to nine glasses of wine daily. His
temperature is 38.8ºC (101.8ºF), pulse is 120/min, respirations are 24/min, and blood pressure is
130/70 mm Hg. There are decreased breath sounds and crackles in the right upper lung field on
auscultation. Cardiac examination shows a normal S1 and S2; no murmurs are heard. A
radiograph of the chest is shown. A Gram stain of the sputum shows gram-negative rods. Which
of the following organisms is the most likely cause of this radiograph finding?
Incorrect Answer ImageA.Haemophilus influenzae
Correct Answer ImageB.Klebsiella pneumoniae
Incorrect Answer ImageC.Moraxella catarrhalis
Incorrect Answer ImageD.Mycoplasma pneumoniae
Incorrect Answer ImageE.Pseudomonas aeruginosa
Incorrect Answer ImageF.Staphylococcus aureus
Incorrect Answer ImageG.Streptococcus pneumoniae

A 50-year-old man with a history of hypertension comes to the emergency department because
of dyspnea on exertion and fatigue. His only medication is hydrochlorothiazide. He denies ever
using alcohol. His vital signs are normal. Examination of the lungs is normal. Cardiac
examination shows a grade 2/6 flow murmur over the aortic area. Chest x-ray is normal.
Laboratory studies show: 
White blood cells 6,200/mL
MCV 65 fl
Erythrocyte count 6.1 million/mm3
RDW normal
Platelets 253,000/mm3
White blood cells 6,200/mL
Reticulocyte count 2%
Hematocrit 34%
Which one of the following is most likely to confirm the diagnosis?
Incorrect Answer ImageA.Bone marrow biopsy
Incorrect Answer ImageB.Esophagogastroduodenoscopy and colonoscopy
Correct Answer ImageC.Hemoglobin electrophoresis
Incorrect Answer ImageD.Serum ferritin
Incorrect Answer ImageE.Serum iron and total iron binding capacity

A 47-year-old woman with type 1 diabetes mellitus comes for a routine health maintenance
examination. She has had no increase in urination, weight gain or loss, or visual changes. She
last went to the podiatrist and ophthalmologist 6 months ago. Her daily glucose log and current
insulin regimen are shown: 
23 U NPH and 9 U regular insulin 30 minutes before
breakfast
15 U regular insulin 30 minutes before dinner
18 U NPH insulin before bedtime
Sund Mon Tuesd Wednes Thurs Frid Satur
ay day ay day day ay day

11 189 170 190 211 169 202 150


A.
M.

4 134 100 112 131 145 87 98


P.
M.

9 278 100 103 132 111 108 100


P.
M.

8 101 122 145 99 123 100 90


A.
M.
*
Numbers reflect glucose in mg/dL.
*8 A.M. sugars are 
fasting
Which of the following is the most appropriate next step in management? 
Incorrect Answer ImageA.Begin oral hypoglycemic medications
Incorrect Answer ImageB.Increase pre-breakfast NPH insulin
Correct Answer ImageC.Increase pre-breakfast regular insulin
Incorrect Answer ImageD.Increase pre-dinner regular insulin
Incorrect Answer ImageE.No changes needed in her current treatment
A hospitalist on duty for the weekend is called to consult on a case involving a 53-year-old
woman in the medical intensive care unit. The patient has been hospitalized for 5 weeks. Her
initial injury was a massive subarachnoid bleed complicated by an ischemic infarct of her brain 3
days later. Since that time, she has been persistently vegetative. She requires mechanical
ventilation and external feeding to maintain her vital functions. It is discovered that, during a
previous admission to the hospital, the patient clearly stated that she would want to be
maintained on life support only if she were likely to regain a meaningful quality of life. The
medical team believes that she does not have a significant chance of regaining an acceptable
level of function. Her adult children maintain that the situation is reversible and want to continue
care. Her husband wants to withdraw care in fulfillment of his wife's wishes. Which of the
following is the most appropriate next step? 
Correct Answer ImageA.Arrange a family meeting, hoping to resolve the patient's previously
expressed wishes with those of the children and husband
Incorrect Answer ImageB.Begin the withdrawal of care, despite the reservations of the children
Incorrect Answer ImageC.Consult a psychiatrist to speak to the children
Incorrect Answer ImageD.Petition the court to obtain a legal guardian to make the decision
Incorrect Answer ImageE.Refer the case to the ethics committee for review

A 55-year-old man comes to the physician because of dyspnea, especially when sitting or
standing upright. He noticed these symptoms have progressed over the past few months. He
generally feels fatigued and has limited activity. He denies orthopnea or paroxysmal nocturnal
dyspnea. He smoked for 16 years and has a history of alcoholism for over 25 years. His blood
pressure is 105/65 mm Hg and pulse is 90 beats/min. Oxygen saturation is 91% on room air.
Physical examination shows spider angiomata, palmar erythema, mild gynecomastia, and mild
ascites. Slightly decreased breath sounds are present at the right lung base. There is a grade 2/6
early systolic ejection murmur at the second right intercostal space and mild pretibial edema.
When he stands, his oxygen saturation drops to 85% and returns to 94% while lying flat. Chest
x-ray shows a small right-sided pleural effusion with right basilar subsegmental atelectasis.
Laboratory studies show:
Leukocyte count 8,300/mm3
Hemoglobin 13 g/dl
Hematocrit 37%
Creatinine 1.7 mg/dl
Aspartate aminotransferase (AST) 23 U/L
Alanine aminotransferase (ALT) 16 U/L
Albumin 2.4 gm/dl
Prothrombin time 18 seconds
Which of the following is the most likely etiology for the patient's hypoxia? 
Incorrect Answer ImageA.Atelectasis
Incorrect Answer ImageB.Emphysema
Correct Answer ImageC.Hepatopulmonary syndrome
Incorrect Answer ImageD.Intracardiac shunt
Incorrect Answer ImageE.Pulmonary embolism

A 30-year-old man is brought to the physician by his wife because of persistent yellowing of his
eyes. He reports that his urine color has been dark lately. He denies fever, pruritus, or changes in
his stool color. His past medical history is significant for recurrent bacterial pneumonias and
middle ear infections. He denies alcohol and illicit drug use. His temperature is 37.1°C (98.6°F),
pulse is 110/min, respirations are 12/min, and blood pressure is 130/80 mm Hg. Physical
examination shows scleral icterus. The rest of the physical examination, including cardiovascular
and abdominal examinations, is unremarkable. Laboratory studies show:
AST 100 U/L
ALT 125 U/L
Bilirubin, serum 2.0 mg/dL
Hepatitis A IgM Negative
Hepatitis A IgG Negative
HBsAg Negative
HIV antibody Negative
IgG total  100 mg/dL (normal: 723–1,685 mg/dL)
IgA  15 mg/dL (normal: 81–463 mg/dL)
IgM  450 mg/dL (normal: 48–271 mg/dL)
He undergoes a liver biopsy, which shows hepatic fibrosis. Which of the following is the most
likely diagnosis?
Incorrect Answer ImageA.Adenosine deaminase deficiency
Incorrect Answer ImageB.Bruton agammaglobulinemia
Correct Answer ImageC.Hyper IgM immunodeficiency
Incorrect Answer ImageD.IgG subclass deficiency
Incorrect Answer ImageE.Selective IgA deficiency

A 67-year-old obese man diagnosed with type 2 diabetes mellitus found during a screening
evaluation conducted at a local shopping center is referred to a physician. In the comprehensive
evaluation that ensues, several abnormalities, in addition to diabetes, are found. Alkaline
phosphatase level is 4 times the upper limit of normal, total bilirubin is 2.7 mg/dL, and
hemoglobin is 9 mg/dL with hemoccult positive stools. An ultrasound of the abdomen shows
dilated intrahepatic and extrahepatic ducts. Which of the following is the best diagnostic test?
Incorrect Answer ImageA.Colonoscopy
Incorrect Answer ImageB.CT scan of the abdomen
Correct Answer ImageC.Endoscopic examination of the duodenum
Incorrect Answer ImageD.Percutaneous transhepatic cholangiogram
Incorrect Answer ImageE.Upper GI series with barium

An 18-year-old man comes to the physician for a routine health maintenance examination. He
has had numbness and tingling in his left shin and foot for 2 months. He does not smoke
cigarettes or drink alcohol. His father and maternal aunt both had Tangier disease. Laboratory
studies show: 
Hemoglobin 14.8 g/dL
Leukocyte count 8,200/mm3
Platelet count 320,000/mm3
Low-density lipoprotein (LDL) cholesterol 80 mg/dL
High-density lipoprotein (HDL) cholesterol 22 mg/dL
Triglycerides 110 mg/dL
Which of the following is the most likely physical finding on examination? 
Incorrect Answer ImageA.Angiokeratomas
Incorrect Answer ImageB.Grey-brown pigmentation of the forehead, hands, and pretibial region
Incorrect Answer ImageC.Irregular black deposits of clumped pigment in the peripheral retina
Correct Answer ImageD.Orange-yellow tonsillar hyperplasia
Incorrect Answer ImageE.Pingueculae

A 42-year-old woman comes to her physician because of a lower extremity ulcer first noticed 5
days ago. She has chronically worsening edema, pruritis, and dry skin that are not relieved with
topical moisturizers. Her history is significant for protein C deficiency and deep vein thrombosis
(DVT) in both femoral veins for more than 20 years, treated with warfarin, and her INR has been
in the therapeutic range. She has not had any complications since, and sees a hematologist
regularly. On physical examination she is in no apparent distress and is afebrile with normal vital
signs. Her lower extremity examination is notable for 1+ non-pitting edema and hyperpigmented
skin bilaterally. She has a well-circumscribed 2-cm ulcer on her right anterior shin with some
exudate at the base. There is no surrounding erythema. Her distal pulses are palpable. What is the
most appropriate treatment for this patient? 
Incorrect Answer ImageA.Arterial bypass of the right leg
Incorrect Answer ImageB.Biopsy of the lesion
Incorrect Answer ImageC.Complete excision of the lesion
Incorrect Answer ImageD.Increase the warfarin dose
Correct Answer ImageE.Leg compression wraps
Incorrect Answer ImageF.Topical hydrocortisone to both legs

A 62-year-old man comes to the physician because of difficulty walking for 2 months. He denies
tingling or numbness in his legs. Four months ago, he had a shock-like pain and tingling in digits
1 through 3 of his left hand. He has a history of hypertension, diabetes mellitus, and hepatitis C.
Current medications include aspirin, metoprolol, lisinopril, glyburide, and multivitamins. His
temperature is 37.1°C (98.8°F), blood pressure 145/86 mm Hg, pulse 72/min, and respirations
12/min. Examination is shown (see media file). Neurologic examination shows 3/5 muscle
strength with dorsiflexion of the left ankle. There is palpable purpura over the abdomen. Serum
creatinine level is 3.2 mg/dL and complement level is 22 mg/dL (normal 75–135 mg/dL). Which
of the following is most likely causing the patient's symptoms? 
Incorrect Answer ImageA.Guillain-Barré syndrome
Incorrect Answer ImageB.Hemorrhagic stroke
Correct Answer ImageC.Mononeuritis multiplex
Incorrect Answer ImageD.Neuropathy caused by diabetes mellitus
Incorrect Answer ImageE.Thrombotic stroke

A 72-year-old man comes to the physician because of a worsening cough productive of blood-
tinged sputum and progressively darkening skin. He has a history of type 2 diabetes mellitus,
hypertension, and obesity. Current medications include metformin, glyburide, lisinopril,
hydrochlorothiazide, and metoprolol. He has smoked two packs of cigarettes daily for 50 years.
His blood pressure is 164/94 mm Hg. Examination shows truncal obesity with thin extremities.
There are violaceous striae on the abdomen. Laboratory studies show: 
Serum glucose 265 mg/dL
Adrenocorticotropic hormone (ACTH) 260 pg/mL (n, 20–80
level pg/mL) 
Which of the following is the most likely cause of this patient's condition? 
Incorrect Answer ImageA.Anterior pituitary adenoma
Incorrect Answer ImageB.Congenital adrenal hyperplasia
Incorrect Answer ImageC.Prostate cancer
Correct Answer ImageD.Small-cell carcinoma
Incorrect Answer ImageE.Squamous cell carcinoma of the lung

A 28-year-old woman who is an English professor comes to the physician because of chest pain
with exertion. She states that the symptoms began approximately 4 months ago and have been
accompanied by dyspnea, and on several occasions, she became lightheaded and felt faint while
exercising. She has no history of any major medical illnesses and takes no medications. She does
not smoke or drink. Her temperature is 37.0ºC (98.6ºF), blood pressure is 154/92 mm Hg, pulse
is 82/min, and respirations are 16/min. Physical examination shows delayed carotid upstrokes
that are diminished in amplitude. There is a forceful apical impulse and a soft S2. There is a
harsh, late-peaking, crescendo-decrescendo systolic murmur that is heard best at the right second
intercostal space and radiates to the carotids. The remainder of the physical examination is
unremarkable. An ECG shows normal sinus rhythm and findings suggestive of left ventricular
hypertrophy. Which of the following is the most likely cause of this patient's condition?
Incorrect Answer ImageA.Aortic insufficiency
Correct Answer ImageB.Bicuspid valve
Incorrect Answer ImageC.Calcification of normal aortic valve
Incorrect Answer ImageD.Rheumatic valvular disease
Incorrect Answer ImageE.Subaortic membrane

A 43-year-old woman comes to the emergency department because of pain and redness in her
left eye. She has also noticed some blurry vision in the affected eye. The pain and redness began
approximately 2 days ago and have progressively worsened since then. She denies any other
symptoms, including fever. Her past medical history is unremarkable, and she does not take any
medications. Her temperature is 36.7°C (98°F), pulse is 90/min, respirations are 16/min, and
blood pressure is 118/78 mm Hg. Examination of her left eye shows injection, mild proptosis,
and pain with extraocular movement. No corneal abrasion is appreciated, and the anterior
chamber is unremarkable. Her fundoscopic examination is normal. The rest of the head and neck
examination, chest, abdominal, and peripheral vascular examinations are unremarkable.
Computed tomography of the left orbit reveals diffuse infiltration in the orbital fat surrounding
the globe and optic nerve sheath complex, which is consistent with orbital inflammation.
Enlargement of the lacrimal gland and extraocular muscles is also noted. Which of the following
is the most likely diagnosis?
Incorrect Answer ImageA.Infectious retinitis
Incorrect Answer ImageB.Optic neuritis
Correct Answer ImageC.Orbital pseudotumor
Incorrect Answer ImageD.Uveitis
Incorrect Answer ImageE.Viral conjunctivitis

A 38-year-old woman comes to the physician because of a rash on her legs for 1 week. She
denies fever or pruritis. Her medical history is unremarkable, and she takes no medications. Her
temperature is 36.9°C (98.4°F), blood pressure is 125/85 mm Hg, and pulse is 75/min. Her
oxygen saturation is 98% on room air. Examination shows raised, erythematous papules that are
tender over both shins. The spleen tip is palpated 3 cm below the left costal margin. An x-ray
film of the chest is shown. A biopsy specimen of the right hilar lymph node shows non-caseating
granulomas. Which of the following findings is most likely associated with the best prognosis in
this patient?
Correct Answer ImageA.Erythema nodosum
Incorrect Answer ImageB.Heart block
Incorrect Answer ImageC.Lung consolidation with hilar lymph nodes
Incorrect Answer ImageD.Lupus pernio
Incorrect Answer ImageE.Ocular involvement

A 42-year-old man is admitted to the hospital for the treatment of multidrug-resistant pulmonary
tuberculosis (TB). He has been on an intense antitubercular drug regimen for the last 2 months in
respiratory isolation and his sputum still shows the presence of acid-fast bacilli. Over the past
week, he has developed fatigue. He has a normal physical examination except for conjunctival
pallor. He has been on a regular diet and is tolerating oral intake well. He still has on-and-off
spiking fever, but his condition has slowly improved apart from the new-onset fatigue.
Laboratory studies show the following: 
Hematocrit 30%
Hemoglobin 10 g/dL
WBC 9,000/mm3
Platelets 200,000/mm3
MCV 76 µm3
Peripheral blood smear normocytic and hypochromic red blood
cells
Subsequently, three tests of stool for blood are negative. A bone marrow examination shows iron
deposits encircling the red-cell nuclei when stained with Prussian blue. Which of the following is
the most likely cause of this patient's anemia? 
Incorrect Answer ImageA.Chloramphenicol
Incorrect Answer ImageB.Ethambutol
Correct Answer ImageC.Isoniazid
Incorrect Answer ImageD.Rifampin
Incorrect Answer ImageE.Streptomycin

A 54-year-old woman with a long history of emphysema comes to the emergency department
because of an exacerbation of her cough and dyspnea. Her pulse is 82/min, respirations are
24/min, and blood pressure is 126/64 mm Hg. Physical examination shows loud expiratory
wheezes and rhonchi. Cardiac examination is normal. An arterial blood gas is performed. Which
of the following results would most likely be expected? 
Incorrect Answer ImageA.pH of 7.20, pCO2 of 60, and pO2 of 46
Incorrect Answer ImageB.pH of 7.30, pCO2 of 50, and pO2 of 94
Correct Answer ImageC.pH of 7.35, pCO2 of 55, and pO2 of 60
Incorrect Answer ImageD.pH of 7.46, pCO2 of 25, and pO2 of 76
Incorrect Answer ImageE.pH of 7.52, pCO2 of 30, and pO2 of 82
A 30-year-old woman comes to the physician because of a 1-year history of headaches. She
describes throbbing headaches located above her left eye and nausea that lasts 4 hours. She has
noticed that they are more likely to occur with her menses. NSAIDs have been ineffective for her
headaches. She denies fever, neck stiffness, or recent head trauma. Her medical history is
otherwise unremarkable. She has no drug allergies. She drinks wine on social occasions. Her
temperature is 36.8°C (98.2°F), blood pressure is 110/75 mm Hg, and pulse is 88/min.
Neurologic examination shows no abnormalities. Which of the following is the most appropriate
next step in management?
Incorrect Answer ImageA.Acetaminophen
Incorrect Answer ImageB.Metoclopramide
Incorrect Answer ImageC.Morphine
Incorrect Answer ImageD.Oxycodone
Correct Answer ImageE.Sumatriptan

A 58-year-old man comes to the clinic complaining of pain when he urinates. He has not had
these symptoms before, though he does suffer occasionally from urinary hesitancy and nocturia
caused by benign prostatic hyperplasia (BPH). The present illness began yesterday and is not
associated with fever, chills, or nausea. He has had no worsening of his chronic urinary
obstructive symptoms. He denies any new sexual partners and has never had a sexually
transmitted disease. Aside from prostate enlargement and stage 1 hypertension, he has no major
medical problems. His temperature is 37.0ºC (98.6ºF), pulse 83/min, respirations 18/min, and
blood pressure 140/90 mm Hg. Physical examination is remarkable for mild suprapubic
tenderness. There is no costovertebral angle tenderness and no penile discharge or lesions. The
prostate is diffusely enlarged and nontender. A dipstick urinalysis reveals:
Specific gravity 1.010
pH 4.5
Glucose negative
Protein negative
Bilirubin negative
Ketones +
Urobilinogen negative
Nitrites +
Leukocytes +++
Leukocyte esterase +
RBC +
Hemoglobin +
On microscopic examination, many white blood cells and bacteria are seen. A urine sample for
culture and sensitivity is obtained, and results are pending. Which of the following is the most
appropriate first step in management?
Correct Answer ImageA.Begin a 7-day course of empiric antibiotics
Incorrect Answer ImageB.Begin a 30 day course of antibiotics
Incorrect Answer ImageC.Begin an alpha-adrenergic blocker
Incorrect Answer ImageD.Order a renal ultrasound or CT scan
Incorrect Answer ImageE.No further treatment; provide reassurance
A 44-year-old woman comes to the emergency department because of fatigue and generalized
weakness for 2 months. She has had dizziness on standing, episodes of coughing with blood
tinged sputum, night sweats, nausea, occasional vomiting, and abdominal pain. She has also had
a 9-kg (20-lb) weight loss in 2 months. She recently emigrated from Mexico. Her temperature is
37.6ºC (99.7ºF), blood pressure 94/65 mm Hg, pulse 95/min, and respirations 12/min.
Examination shows decreased breath sounds bilaterally. A grade 3/6 diastolic murmur is heard at
the left sternal border. There is diffuse abdominal tenderness without any rebound or guarding.
Laboratory studies show: 
Hemoglobin  11.5 g/dL
Leukocyte count 10,800/mm3
Serum:
Na+ 130 mEq/L 
Cl −
110 mEq/L 
K+ 5.8 mEq/L 
HCO3 −
22 mmol/L 
Glucose  100 mg/dL 
Urea nitrogen (BUN)  16 mg/dL 
Creatinine  1.9 mg/dL 
Which of the following is the most appropriate next step in management? 
Incorrect Answer ImageA.Broad-spectrum antibiotics
Incorrect Answer ImageB.Dexamethasone suppression test
Correct Answer ImageC.Chest radiograph
Incorrect Answer ImageD.CT scan of the abdomen
Incorrect Answer ImageE.Transthoracic echocardiogram

A 68-year-old man comes for a routine maintenance examination. He denies chest pain, dyspnea,
cough, or shortness of breath. He has a 5-year history of hypercholesterolemia controlled with
atorvastatin. He recently retired from his job as a mail carrier. He has smoked one pack of
cigarettes daily for 45 years. He drinks three to five beers on the weekends. His mother had a
myocardial infarction at age 52 years, and his brother had a myocardial infarction at age 48
years. Complete blood count, metabolic panel, and lipid panel were within normal limits 8
months ago. He had a colonoscopy 3 years ago that showed no abnormalities. Which of the
following is the most appropriate screening test for this patient? 
Incorrect Answer ImageA.12-lead electrocardiography
Correct Answer ImageB.Abdominal ultrasound
Incorrect Answer ImageC.Chest radiograph
Incorrect Answer ImageD.Lipid panel
Incorrect Answer ImageE.Prostate-specific antigen

A previously healthy 60-year-old man is referred for neurologic consultation because of rapidly
progressive memory loss and jerking movements of the upper and lower extremities. Neurologic
examination confirms the presence of severe cognitive deficits and memory dysfunction. An
electroencephalogram shows generalized periodic sharp waves. Neuroimaging studies show
moderately advanced cerebral atrophy. Cerebrospinal fluid analysis is unremarkable. Which of
the following is the most likely cause of this patient's symptoms? 
Incorrect Answer ImageA.Cadaveric transplant
Incorrect Answer ImageB.Consumption of contaminated meat
Incorrect Answer ImageC.Familial inheritance
Incorrect Answer ImageD.Intranasal growth hormone
Correct Answer ImageE.Sporadic mutation

A patient comes to his physician because of chronic pain and tingling of the buttocks. The pain is
exacerbated when the buttocks are compressed by sitting on a toilet seat or chair for long
periods. No lumbar pain is noted. Pain is elicited when the physician performs a Freiberg
maneuver, in which there is a forceful internal rotation of the extended thigh. Which of the
following is the most likely diagnosis? 
Incorrect Answer ImageA.Disk compression of the sciatic nerve
Incorrect Answer ImageB.Fibromyalgia
Correct Answer ImageC.Piriformis syndrome
Incorrect Answer ImageD.Popliteus tendinitis
Incorrect Answer ImageE.Posterior femoral muscle strain

A 62-year-old woman comes to the emergency department because of chest pain for the past 24
hours. She states that the pain is located on her left side and decreases when she sits up and leans
forward. It is worse with deep inspiration. She has a history of hypertension that is controlled
with a DASH diet and exercise. Physical examination shows no abnormalities except for a
scratchy, leathery sound that is heard during both systole and diastole. An ECG is shown. Which
of the following drugs is most appropriate for this patient?
Incorrect Answer ImageA.Acetaminophen
Incorrect Answer ImageB.Atenolol
Correct Answer ImageC.Ibuprofen
Incorrect Answer ImageD.Morphine
Incorrect Answer ImageE.Nitroglycerin
Incorrect Answer ImageF.Prednisone
Incorrect Answer ImageG.Streptokinase

A 54-year-old man comes to his physician because he feels that his right eye has been “sticking
too far out." He has noticed this symptom for the past six months. He also reports constant
headaches, which are localized to the right periorbital area and have progressively increased in
severity over these six months. He reports decreased vision as well in his right eye over the past
few months. His past medical history is unremarkable, and he does not take any medications. He
does not smoke or drink alcohol. On physical examination, his temperature is 36.7°C (98.0°F),
pulse is 80/min, respirations are 16/min, and blood pressure is 118/78 mmHg. Head and neck
examination reveals right-eye proptosis with inferior and nasal deviation of the globe. Visual
acuity is 6/12 (20/40) in the right eye and 6/6 (20/20) in the left eye. Lateral movement of the
right eye is restricted but not painful. No erythema or tenderness is noted in the right periorbital
region, nor is there regional lymphadenopathy. Left-eye examination is unremarkable as are
pulmonary, cardiovascular and abdominal examinations. Computed tomography reveals a mass
with irregular margins arising from the right lacrimal gland. It is compressing the lateral rectus
muscle and eroding the lateral orbital wall. Which of the following is the most likely diagnosis?
Correct Answer ImageA.Adenoid cystic carcinoma
Incorrect Answer ImageB.Intralacrimal abscess
Incorrect Answer ImageC.Orbital cellulitis
Incorrect Answer ImageD.Pleomorphic adenoma
Incorrect Answer ImageE.Sarcoidosis

The 18-year-old son of a wealthy Middle Eastern businessman is flown to a hospital in the
United States after his local doctor discovers an abdominal mass. The young man has a 2-year
history of weight loss accompanied by production of malodorous stool with excessive flatulence.
Duodenal biopsy specimen with histopathologic examination demonstrates villous atrophy and
massive infiltration of the lamina propria by lymphocytes, plasma cells, and immunoblasts that
do not morphologically appear overtly malignant. A biopsy specimen of a mesenteric lymph
node shows node replacement by a similar infiltrate. Which of the following is the most likely
diagnosis? 
Correct Answer ImageA.Heavy chain disease
Incorrect Answer ImageB.Macroglobulinemia
Incorrect Answer ImageC.Monoclonal gammopathy of undetermined significance
Incorrect Answer ImageD.Multiple myeloma
Incorrect Answer ImageE.Myelodysplastic syndrome

A 62-year-old woman with hypertension and osteoporosis comes to the physician for her 6-
month follow-up examination. She reports that she has been nauseated and has had some vague
chest discomfort for the last week; she has also noticed that she has been more fatigued than
usual. In addition, she states that she has been having generalized muscle soreness. She is taking
furosemide 40 mg twice a day, as well as occasional calcium supplements. Her vital signs and
physical examination are unremarkable. An electrocardiogram shows presence of a U wave and
of diffuse but <1-mm ST segment depressions. Supplementation with which of the following
may have prevented this condition? 
Incorrect Answer ImageA.Calcium
Incorrect Answer ImageB.Iron
Incorrect Answer ImageC.Magnesium
Incorrect Answer ImageD.Manganese
Correct Answer ImageE.Potassium

A 66-year-old woman with a recent diagnosis of type 2 diabetes mellitus comes to the physician
for a follow-up examination. Her blood pressure 3 months ago was 140/95 mm Hg, and on a
previous visit 6 months ago it was 142/93 mm Hg. She was advised to exercise and eat a low-salt
diet. She has gained 2 kg (4.4 lb) since her last visit. Today her blood pressure is 144/95 mm Hg.
Laboratory studies show:
Na+ 140 mEq/L
K+ 3.9 mEq/L
Cl −
109 mEq/L
HCO3− 22 mEq/L
Glucose 176 mg/dl
BUN 23 mg/dl
Creatinine 1.2 mg/dl
Urinalysis shows:
Color Clear
Leukocyte esterase Negative
Nitrite Negative
Blood Negative
Microscopic No cells, no casts Protein 1+
Which of the following is the most appropriate next step in management? 
Incorrect Answer ImageA.Begin therapy with atenolol
Incorrect Answer ImageB.Begin therapy with labetalol
Correct Answer ImageC.Begin therapy with lisinopril
Incorrect Answer ImageD.Begin therapy with verapamil
Incorrect Answer ImageE.Continue lifestyle modification

A 27-year-old woman comes to the physician because of fever and back pain. She states that 5
days ago she developed burning with urination, progressing to fever with chills and pain in the
right flank. She has a history of kidney stones, last passing one 10 years ago. She takes no
medications. Her temperature is 38.9ºC (102ºF), blood pressure is 110/70 mm Hg, pulse is
102/min, and respirations are 16/min. Examination shows a patient in mild distress with shaking
chills and right costovertebral angle tenderness. Leukocyte count is 18,000/mm3. The serum
creatinine is 2.0 mg/dL. Urinalysis shows 100 leukocytes/high-power field. A CT scan shows
mild right hydronephrosis with a 1.5 cm stone in the right renal pelvis. She is admitted to the
hospital and given intravenous saline and ciprofloxacin. Which of the following is the most
appropriate next step in management?
Correct Answer ImageA.Nephrostomy tube placement
Incorrect Answer ImageB.Observation with continued antibiotics
Incorrect Answer ImageC.Open surgical stone removal
Incorrect Answer ImageD.Percutaneous lithotripsy
Incorrect Answer ImageE.Ureteroscopic retrieval of the stone

A 52-year-old man comes to the clinic because of aching in his hands for the past 6 months. The
pain is partially relieved with acetaminophen and ibuprofen, but the patient is concerned because
the pain in his joints seems to be getting worse. He also reports fatigue and erectile dysfunction.
While vital signs are within normal limits, physical examination shows a variety of unexpected
findings. Cardiac auscultation shows an S4, a firm and enlarged liver is present on abdominal
examination and large, hyperpigmented patches are present on the patient's skin. A review of
recent laboratory work shows mildly elevated liver function tests, a borderline elevated fasting
glucose level, and transferrin saturation level 55%. Which of the following is the most
appropriate next step in management of this patient? 
Incorrect Answer ImageA.Chelation therapy with deferoxamine
Correct Answer ImageB.Genetic testing for HFEmutation

Incorrect Answer ImageC.Liver biopsy


Incorrect Answer ImageD.Liver transplantation
Incorrect Answer ImageE.Phlebotomy

A 28-year-old woman comes to the physician for evaluation of infertility. She and her husband
have been having regular, unprotected intercourse for the past 3 years. Her menstrual periods
occur every 10 to 12 weeks; her last menstrual period was 3 weeks ago. Her medical history is
unremarkable and she takes no medications. Her temperature is 37°C (98.6°F), pulse is 68/min,
respirations are 22/min, and blood pressure is 110/74 mm Hg. Physical examination shows acne
and hirsutism. Pelvic examination shows mild clitoromegaly. The remainder of the examination
is normal. Laboratory studies show:
17-hydroxyprogesterone 348 ng/dL (normal, <200)
Dehydroepiandrosterone (DHEA) 125 ug/dL (normal, 65–380)
After administration of high-dose ACTH, serum 17-hydroxyprogesterone is 5,400 ng/dL; there is
no change in aldosterone levels. The patient is diagnosed with a defect in CYP21A2. Which of
the following is the most likely cause for the late onset of her disease? 
Incorrect Answer ImageA.Complete 3-β-hydroxysteroid dehydrogenase defect
Incorrect Answer ImageB.CYP21A2 function is carried out partially by 11-β-hydroxylase
Incorrect Answer ImageC.CYP21A2 function is carried out partially by 17-α-hydroxylase
Correct Answer ImageD.Incomplete 21-hydroxylase defect
Incorrect Answer ImageE.Incomplete cholesterol desmolase defect

A 59-year-old man comes to the physician because of severe perianal pain and swelling for the
past 4 days. It began suddenly while he was exercising. He has no significant past medical
history. His regular diet consists mainly of fruits, vegetables, and fish. He has had no changes in
his bowel habits. His temperature is 37.0ºC (98.6ºF). Physical examination shows a 3 cm tender,
purple, lump on the perianal skin. There is no ulceration or drainage. He has been trying sitz
baths at home, but gets only temporary relief. He needs to go on a business trip in 5 days and
wants to make sure that he is comfortable by then. Which of the following is the best course of
action at this time?
Correct Answer ImageA.Enucleation of the clot
Incorrect Answer ImageB.Excisional biopsy to exclude carcinoma
Incorrect Answer ImageC.Incision and drainage of the abscess
Incorrect Answer ImageD.Infrared coagulation
Incorrect Answer ImageE.Rubber band ligation

A 26-year-old African American woman comes to the emergency department complaining of


severe shortness of breath that began while she was jogging in the park earlier today. Her
symptoms did not improve despite the repeated use of her inhaler. Her past medical history is
significant for asthma and seasonal allergies. Her medications include oral contraceptive pills
and 3 respiratory medications, 2 of which are inhaled and, she says, are closely managed by her
allergist. Her respiratory rate is 44/min and she is exhibiting supraclavicular retractions. She has
difficulty completing her sentences while speaking. On auscultation her lungs have expiratory
and inspiratory wheezes. Her nail beds and oral mucosa are pink. In addition to instituting
prompt therapy with nebulized beta-agonists, IV steroids, and oxygen 15 L through a mask with
a reservoir bag, chest x-ray, and arterial blood gas (ABG) are ordered. ABG shows pH 7.46,
pCO2 32 mm Hg, and pO2 86 mmHg. Thirty minutes later her respiratory rate is 25/min. Her
wheezing is now less audible on auscultation and subcostal retractions have diminished. Her lips
appear blue. A second stat ABG shows pH 7.38, pCO2 41 mm Hg, and pO2 84 mm Hg. Which of
the following is the most appropriate next step in management?
Incorrect Answer ImageA.Check the chest x-ray and discharge the patient if it is normal
Incorrect Answer ImageB.Continue treatment and check on the patient in 30 minutes
Correct Answer ImageC.Explain the meaning of respiratory failure to the patient, then intubate her
Incorrect Answer ImageD.Give another dose of beta-agonists and add racemic epinephrine to the
regimen
Incorrect Answer ImageE.Obtain a prompt pulmonary/critical care consultation and await
recommendations

A 38-year-old woman comes to the physician because of progressive difficulty swallowing for 3
months. She has heartburn and the sensation that food "gets stuck" in the lower portion of the
esophagus. She denies nausea or vomiting. She has pain, tingling, and blanching of her hands in
the cold. Current medications include nifedipine and topical nitroglycerin. Skin findings are
shown. There is a 1 cm hard nodule near the elbow. A biopsy specimen of the nodule shows
calcium deposits in the dermis. Serum testing is most likely to show which of the following
autoantibodies?
Correct Answer ImageA.Anti-centromere
Incorrect Answer ImageB.Anti-cyclic citrullinated peptide
Incorrect Answer ImageC.Anti-double stranded DNA
Incorrect Answer ImageD.Anti-histone
Incorrect Answer ImageE.Anti-Ro (SS-A)

A 65-year-old retired accountant with a 60-pack-year smoking history comes to the physician for
her annual physical examination. She was last seen 3 years ago when she presented with an
episode of acute bronchitis. She reports that over the past several months she has a cough that is
productive of green sputum. She denies dyspnea or limitation in her daily activities. Her blood
pressure is 126/62 mm Hg, pulse is 80/min, and respirations are 24/min. She has diffuse bilateral
expiratory rhonchi with a markedly prolonged expiratory phase. Cardiac examination shows a
second heart sound that increases in intensity with inspiration. The liver edge is 14 cm into the
midclavicular line and mildly tender to palpation. There is bilateral lower extremity edema to the
knees. Chest radiograph shows hyperinflation of both lung fields, and pulmonary function tests
show a decreased ratio of forced expiratory volume in 1 second (FEV1) to forced vital capacity
(FVC) and low FVC. A complete blood cell count is sent to the laboratory. Which of the
following would most likely be expected?
Incorrect Answer ImageA.Anemia
Correct Answer ImageB.Erythrocytosis
Incorrect Answer ImageC.Lymphopenia
Incorrect Answer ImageD.Thrombocytopenia
Incorrect Answer ImageE.Thrombocytosis

An 85-year-old woman is brought to the emergency department by her daughter because her face
and neck have been stuck in one position for 8 hours. The daughter reports that the patient has
had strange facial expressions and involuntary head movements for 4 days. She has had no loss
of consciousness, convulsions, or episodes of incontinence. She has a history of Parkinson’s
disease. Examination shows a sustained muscle contraction and spasm of the left
sternocleidomastoid muscle with limited range of cervical motion. There is pooling of saliva in
the oropharynx. Which of the following is the most appropriate next step in management?
Incorrect Answer ImageA.Cervical radiograph
Incorrect Answer ImageB.CT scan of the head
Correct Answer ImageC.Intramuscular injection of botulinum toxin
Incorrect Answer ImageD.Intravenous pancuronium
Incorrect Answer ImageE.Lumbar puncture

A 73-year-old man comes to the physician because of a 1-year history of a painless, enlarging,
white chalky nodule on the third metacarpophalangeal joint of the right hand. He has a history of
gout (recurrent arthritis in his left great toe), stable angina pectoris, hypercholesterolemia, and
hypertension. Medications include enalapril, sublingual nitroglycerin, simvastatin, and
metoprolol. He smoked one pack of cigarettes daily for 40 years, but quit 5 years ago. Physical
examination shows normal vital signs, and is normal except for a 5x3x3 cm firm, nodular mass
overlying the third right metacarpophalangeal joint. It does not transilluminate when a light is
held against it. A radiograph of the hand shows periarticular erosions and soft-tissue calcification
overlying the third metacarpophalangeal joint. Which of the following is the most likely
diagnosis? 
Incorrect Answer ImageA.Epidermal inclusion cyst
Incorrect Answer ImageB.Ganglion cyst
Incorrect Answer ImageC.Lipoma
Incorrect Answer ImageD.Rheumatoid nodule
Incorrect Answer ImageE.Soft-tissue sarcoma
Correct Answer ImageF.Tophaceous gout

A 50-year-old African American man comes to the physician for a routine health maintenance
examination. His medical history is unremarkable and he takes no medications. He smoked 2
packs of cigarettes daily for 2 years but quit 27 years ago. His brother died of squamous cell lung
cancer at the age of 56 years. His pulse is 74/min, respirations are 12/min, and his blood pressure
is 122/84 mm Hg. Physical examination of the heart shows an S1 and S2 heart sound. There are
no murmurs, rubs or gallops. The lungs are clear other than some decreased breath sounds at
both lung bases. The abdomen is soft. He has a right inguinal hernia scar. Because of his
brother's recent death from cancer, he is very anxious about his risk for cancer. This patient is
most likely at greatest risk for which of the following malignancies?
Incorrect Answer ImageA.Colorectal carcinoma
Incorrect Answer ImageB.Hepatocellular carcinoma
Incorrect Answer ImageC.Lung carcinoma
Incorrect Answer ImageD.Pancreatic carcinoma
Correct Answer ImageE.Prostatic carcinoma

A 21-year-old woman has a 1-year history of intermittent abdominal cramping with alternating
diarrhea and constipation. She comes to the physician because the symptoms have worsened over
the last 2 weeks making it hard for her to study for her final examinations. She has not noticed
any blood in her stools. The pain is relieved temporarily with bowel movements but typically
returns hours later. She is not awakened at night because of the pain. She denies fever, chills, or a
change in weight. Her temperature is 37.0ºC (98.6ºF), blood pressure is 120/70 mm Hg, pulse is
73/min, and respirations are 13/min. Physical examination shows mild abdominal distention
without guarding or rebound tenderness. Bowel sounds are present. Which of the following is the
most likely colonoscopic finding?
Incorrect Answer ImageA.Angiodysplasia
Incorrect Answer ImageB.Cobblestone appearance
Incorrect Answer ImageC.Diverticulosis
Incorrect Answer ImageD.Friable mucosa
Correct Answer ImageE.Normal mucosa

A 48-year-old man comes to the physician because of recurrent chest tightness that is associated
with exertion. He states that if he rests, the pain subsides within a few minutes. His blood
pressure in the left arm while sitting is 150/90 mm Hg. Physical examination shows no
abnormalities. Laboratory studies including complete blood count, serum chemistries, fasting
blood glucose, and a serum lipid panel are ordered. The patient currently denies chest pain.A
resting ECG shows a left bundle branch block. Which of the following is the most appropriate
next step in management? 
Incorrect Answer ImageA.Ambulatory electrocardiographic monitoring
Incorrect Answer ImageB.Coronary angiography
Incorrect Answer ImageC.Exercise treadmill stress test
Incorrect Answer ImageD.Reassurance
Correct Answer ImageE.Stress echocardiography

A 44-year-old woman has a 10-year history of progressive dysphagia for both solids and liquids.
She denies pain and reports that food sticks in her lower chest and is improved by drinking large
amounts of water and sitting up straight. She also describes frequent episodes of regurgitation of
foul-smelling, undigested food, either when she leans forward or when she is asleep at night.
There has beens weight loss as the disease has progressed. A barium swallow shows a massively
dilated proximal esophagus with a narrow, tapered appearance at the lower sphincter area. Which
of the following is the most likely pathophysiologic process that explains this patient’s disease?
Correct Answer ImageA.Loss of inhibitory neurons in the lower esophageal sphincter
Incorrect Answer ImageB.Outpocketing of posterior pharyngeal constrictor muscles
Incorrect Answer ImageC.Reflux of acid gastric juice into the lower esophagus
Incorrect Answer ImageD.Replacement of smooth muscle by fibrous tissue
Incorrect Answer ImageE.Weak or nonexistent peristaltic activity in the upper esophagus

A 54-year-old woman comes to the physician because of a mole that has changed in size and
color over the past 2 months. She has had no fever, fatigue, or weight loss. She has a history of
hypertension treated with lisinopril. Her mother and sister have been diagnosed with melanoma,
her maternal grandmother died of melanoma at age 43 years, and her brother died of pancreatic
cancer at age 56 years. Physical examination shows a darkly pigmented nodule on the left
anterior thigh with irregular borders. Excisional biopsy specimen of the lesion shows nodular
melanoma with a depth of invasion of 3.3 mm. This patient most likely has a germline mutation
in which of the following genes?
Correct Answer ImageA.CDKN2A
Incorrect Answer ImageB.p53
Incorrect Answer ImageC.PTCH
Incorrect Answer ImageD.RET
Incorrect Answer ImageE.TSC1

A 64-year-old man has a 6-month history of progressively worsening urinary symptoms,


including hesitancy, intermittency, and sensation of incomplete bladder emptying. Over the past
3 days, his symptoms have become markedly worse. Yesterday he could barely generate a urine
stream and today he has dribbled only a little bit of urine. He also has developed lower
abdominal pain radiating to the back, lethargy, fatigue, nausea, and vomiting. Before this he was
otherwise healthy, with no significant medical or surgical history. His only medication is an
aspirin 1x/day. His temperature is 36.7ºC (98.0ºF), blood pressure is 125/80 mm Hg, pulse is
65/min, and respirations are 14/min. Physical examination shows his abdomen is soft with
moderate infraumbilical tenderness and suprapubic distention to the umbilicus. The prostate is
extremely enlarged but without nodules or tenderness. Laboratory studies show: 
Potassium 5.4 mEq/L
Bicarbonate 21 mEq/L
BUN 71 mg/dL
Creatinine 5.6 mg/dL
Abdominal and pelvic ultrasound shows a distended bladder with moderate bilateral
hydronephrosis. Which of the following the next best step in management? 
Incorrect Answer ImageA.Emergent placement of bilateral percutaneous nephrostomy tubes
Incorrect Answer ImageB.Immediate hemodialysis
Incorrect Answer ImageC.Institute medical therapy with prazosin and finasteride
Incorrect Answer ImageD.Move to the OR for immediate transurethral resection of the prostate
(TURP)
Correct Answer ImageE.Placement of a Foley catheter and admit him to the hospital

A 57-year-old painter comes to his physician for evaluation of difficulty voiding for the past few
years. He describes a delay when he "attempts to go" and that he often wets his underwear with
additional urine after he has finished. His past medical history is remarkable for essential
hypertension that he claims is diet controlled. He is married and has one teenage child. His blood
pressure is 140/92 mm Hg, and pulse is 82/min. His physical examination is remarkable for a
prominent and laterally displaced apical impulse. His prostate is non-tender and appears large
with no palpable nodules or irregularities. Which of the following is the most appropriate
antihypertensive therapy for this patient?
Incorrect Answer ImageA.Lisinopril
Incorrect Answer ImageB.Metoprolol
Incorrect Answer ImageC.Nifedipine
Correct Answer ImageD.Terazosin
Incorrect Answer ImageE.Verapamil

A 36-year-old woman with a history of mitral valve prolapse calls your clinic and leaves a
message requesting a prescription for antibiotics. She has a root canal and likely tooth extraction
planned for the upcoming week and is requesting you to call a prescription in to the pharmacy
for prophylaxis prior to the procedure. She has been your patient for 10 years, and you just saw
her in the clinic last month. She is compliant with your medical advice and is knowledgeable
about her medical condition. You recall that she has no antibiotic allergies and has mitral valve
prolapse as documented by echocardiography. Which of the following is the most appropriate
course of action at this time?
Incorrect Answer ImageA.Ask the patient to come in for physical examination before any antibiotic
treatment
Incorrect Answer ImageB.Call in a prescription for amoxicillin, taken 1 hour before the procedure
Incorrect Answer ImageC.Discuss alternative dental surgery options with the patient's dentist
Incorrect Answer ImageD.Phone the pharmacy with a 1-week prescription for clindamycin
Correct Answer ImageE.Return the patient's call and advise her that the situation does not warrant
antibiotic therapy

A 38-year-old woman with HIV comes to the physician because of a 1-week history of a
progressive rash on her face, arms, and legs. She denies fever, pruritus, night sweats, or weight
loss. She takes no medications. Her temperature is 36.8°C (98.2°F), pulse is 82/min, and blood
pressure is 135/80 mm Hg. There are numerous papules with central umbilication over the face,
upper and lower extremities, fingers, and the palms of the hands. There are no lesions on the feet.
The remainder of the physical examination is normal. CD4+ T lymphocyte count is 100
cells/mm3. Which of the following is the most likely diagnosis?
Incorrect Answer ImageA.Disseminated varicella
Incorrect Answer ImageB.Erythema multiforme
Incorrect Answer ImageC.Erythema nodosum
Incorrect Answer ImageD.Kaposi sarcoma
Correct Answer ImageE.Molluscum contagiosum
Incorrect Answer ImageF.Pityriasis rosea
Incorrect Answer ImageG.Scabies

A 38-year-old previously healthy woman comes to the physician because of menstrual


irregularities and breast discharge for 4 months. Her last menstrual period was 7 weeks ago.
Prior to 4 months ago, she has had regular menses since age 12 years. She has had no fatigue,
voice changes, facial hair growth, or any recent changes in weight. She takes ibuprofen
occasionally for headaches. She takes no other medications. Laboratory studies show: 
Thyroid-stimulating hormone (TSH)  2.5 μU/L 
Luteinizing hormone  3.8 mU/mL 
Follicle-stimulating hormone  3.0 mU/mL 
Which of the following is the most likely cause of her menstrual irregularities? 
Correct Answer ImageA.Hyperprolactinemia
Incorrect Answer ImageB.Hyperthyroidism
Incorrect Answer ImageC.Hypogonadism
Incorrect Answer ImageD.Hypothyroidism
Incorrect Answer ImageE.Idiopathic galactorrhea
Incorrect Answer ImageF.Menopause

A 71-year-old man comes to his physician because of a 4-week history of a non-healing ulcer at
the heel of his right foot. He denies fever, pain, or a history of trauma. His past medical history is
significant for type 2 diabetes mellitus, hypercholesterolemia, and varicose veins. Current
medications include metformin, glyburide, and lovastatin. His temperature is 37.1°C (98.8°F),
pulse is 80/min, respirations are 12/min, and blood pressure is 130/80 mm Hg. Physical
examination shows a 3.5 cm ulcer with granulation tissue and some dirt and debris in the base.
The skin around the ulcer is normal-appearing. There is absent sensation to pinprick over the
right lower extremity and foot. Peripheral pulses are weak. Laboratory data shows a hemoglobin
A1c level is 9.8%. Which of the following is the most important factor in the poor healing of this
patient's ulcer?
Incorrect Answer ImageA.Chronic trauma
Incorrect Answer ImageB.Elevated blood glucose level
Correct Answer ImageC.Hypoxia
Incorrect Answer ImageD.Infection
Incorrect Answer ImageE.Neuropathy

A 19-year-old African woman with a history of Burkitt lymphoma is currently hospitalized and
undergoing her first round of chemotherapy. The nurse is worried because the patient's urine
output has dropped precipitously. The nurse has changed and flushed the Foley catheter, but over
the last 4 hours the patient has produced only 15 mL of urine. The patient states that aside from
the fever and chills related to her cancer, she has no pain. She was recently started on
combination treatment for her Burkitt lymphoma, which includes prednisone, three
chemotherapeutic medications, acetaminophen, diphenhydramine, a sleep aid, and normal saline
at 250 mL/hour. Her temperature is 37.0ºC (98.6ºF), pulse is 87/min, respirations are 22/min, and
blood pressure 128/88 mm Hg supine and 122/82 mm Hg standing. Examination shows an ill-
appearing, cachectic woman. Diffuse cervical and inguinal lymphadenopathy is present. The rest
of the examination is unremarkable. Laboratory studies show: 
Bicarbonate 20 mEq/L
BUN 21 mg/dL
Calcium 7.6 mg/dL
Chloride 102 mEq/L
Creatinine 4.3 mg/dL
Glucose 108 mg/dL
Potassium 5.5 mEq/L
Phosphate 9 mg/dL
Sodium 138 mEq/L
Uric acid 15 mg/dL
Urinalysis shows acidic urine with numerous uric acid crystals. Which of the following therapies
could have prevented this patient’s renal failure? 
Incorrect Answer ImageA.Calcium supplementation
Incorrect Answer ImageB.Chemotherapy without steroids
Incorrect Answer ImageC.Potassium binder
Correct Answer ImageD.Rasburicase
Incorrect Answer ImageE.Thiazide diuretic

A 39-year-old woman with a past medical history of recurrent acute bacterial sinusitis comes to
the physician because of a sudden onset of vision difficulties that started 2 hours ago. She reports
severe pain in her left eye, with and without movement of the eye, and trouble looking down
while walking down the stairs. Her temperature is 39.5ºC (103.1ºF), pulse is 110/min,
respirations are 12/min, and blood pressure is 130/80 mm Hg. Oxygen saturation is 98% on room
air. Physical examination shows erythema and edema of her left conjunctiva and eyelids with
proptosis. The left pupil is fixed and dilated. Extraocular muscle examination shows an inability
to adduct and depress the left eye. Abduction of the left eye is normal. Which of the following is
the most likely diagnosis?
Incorrect Answer ImageA.Bacterial conjunctivitis
Incorrect Answer ImageB.Blepharitis
Incorrect Answer ImageC.Cavernous sinus thrombosis
Incorrect Answer ImageD.Dacryocystitis
Correct Answer ImageE.Orbital cellulitis

A 32-year-old man with a recent diagnosis of HIV comes to the physician for a follow-up
examination; he had a purified protein derivative (PPD) test placed 48 hours earlier. His
temperature is 36.5ºC (97.7ºF), pulse is 65/min, and blood pressure is 116/72 mm Hg. Physical
examination of his forearm where the PPD was placed shows a small, indurated area of 4 mm
surrounded by an 8 mm area of erythema. His CD4+ T lymphocyte count is 755/mm3. Which of
the following is the most appropriate next step in management for this patient's PPD result?
Incorrect Answer ImageA.Chest radiography
Incorrect Answer ImageB.Immediate isolation
Correct Answer ImageC.Observation
Incorrect Answer ImageD.Sputum induction and microscopy
Incorrect Answer ImageE.Treatment with isoniazid

A 35-year-old woman comes to the physician because of a mass in her right axilla for 5 weeks.
During the past week, she has seen 2 small masses on the left side of her neck, and she has had 2
episodes of night sweats. She has had no fevers or weight loss. Her maternal aunt died of breast
cancer at age 56 years. Examination shows 2 firm, non-tender, mobile lymph nodes, 2- to 3-cm,
in the left anterior cervical chain and right axilla. The lungs are clear to auscultation. Cardiac
examination shows a normal S1 and S2 with no murmurs. There is no abdominal tenderness or
hepatosplenomegaly. A biopsy specimen of the axillary node is shown. Which of the following is
the most likely associated finding on further workup?
Incorrect Answer ImageA.Acid-fast bacilli
Incorrect Answer ImageB.Atypical lymphocytes
Incorrect Answer ImageC.Bilateral hilar adenopathy
Correct Answer ImageD.Mediastinal adenopathy
Incorrect Answer ImageE.Upper lobe cavitation

A 45-year-old man who has been admitted to the hospital for a hip fracture undergoes a total
replacement of the left hip. Forty-eight hours later, he reports increased shortness of breath and
palpitations. He denies a cough, fever, chills, shakes, and hemoptysis. He has a history of
hypertension, for which he takes amlodipine 5 mg daily. He has no medical allergies. He does
not smoke. Blood pressure is 100/70 mm Hg and pulse is 120/min. Oxygen saturation is 87% on
3 L oxygen. Physical examination is notable for tachycardia. Lungs are otherwise clear. CT
pulmonary angiogram of the chest is positive for multiple pulmonary emboli in the right lung.
Which of the following is the most appropriate next step in management? 
Incorrect Answer ImageA.Administer tissue plasminogen activator (tPA)
Incorrect Answer ImageB.Perform pulmonary angiogram
Incorrect Answer ImageC.Perform ventilation-perfusion scan
Incorrect Answer ImageD.Start aspirin and clopidogrel
Correct Answer ImageE.Subcutaneous low-molecular-weight heparin

A 64-year-old man comes to the office because of blood in his urine. He states that he saw the
blood on 2 separate occasions, recently and 1 month ago. He denies dysuria, urgency, frequency,
fever, or chills. He did not take any medications or seek other medical help at that time. His
medical history is significant for cataracts, migraine, and gastroesophageal reflux disease.
Medications include a proton-pump inhibitor daily and ibuprofen as needed for headaches. He
has never had surgery. He has a 45-pack-year history of cigarette smoking. He works as a house
painter. Physical examination of the heart and lungs are within normal limits. The abdomen is
soft and nontender. There is no evidence of guarding or rebound tenderness. The testes are
descended bilaterally. Digital rectal examination reveals a moderately sized prostate without
evidence of masses. Serum creatinine electrolyte levels and complete blood count are all within
normal limits. Urinalysis shows: 
pH 4.7
Specific gravity 1.022
RBC 55/hpf
WBC 3/hpf
Protein negative
Ketones negative
Nitrites negative
Bacteria scant
The patient returns to the office to discuss the results of the studies. He asks what the physician
thinks caused the hematuria. At this time, which of the following is the most appropriate
physician response?
Incorrect Answer ImageA."Blood in the urine is a common finding, likely caused by local irritation."
Correct Answer ImageB."I would like to do some more tests."
Incorrect Answer ImageC."Let's repeat the urinalysis in 2 weeks."
Incorrect Answer ImageD."You have a urinary tract infection. I'll prescribe ciprofloxacin."
Incorrect Answer ImageE."You have bladder cancer and need a radical cystectomy."

A 68-year-old woman is admitted to the hospital after sustaining a right hip fracture. An x-ray
film of the right hip shows a femoral head fracture. She has no history of any major medical
illnesses and takes no medications. Her blood pressure is 120/80 mm Hg and her pulse is 80/min.
Physical examination shows a 2+ carotid upstroke without bruits. A grade 3/6 nonradiating
systolic ejection murmur is heard on auscultation of the chest. There is a normal S1 and a
physiologically split S2. A transthoracic echocardiogram shows aortic valve diameter of 1.4 cm
compatible with moderate aortic stenosis, with an ejection fraction of 55%. Which of the
following is the most appropriate treatment for this patient? 
Incorrect Answer ImageA.Balloon valvotomy
Correct Answer ImageB.Clear the patient for hip surgery
Incorrect Answer ImageC.External traction
Incorrect Answer ImageD.Schedule a thallium stress test
Incorrect Answer ImageE.Transesophageal echocardiogram

A 25-year-old man comes to the physician because of a 2-day history of dyspnea on exertion and
yellow eyes. He has also noticed that his urine is dark-colored. He denies chest pain, orthopnea,
paroxysmal nocturnal dyspnea, night sweats, fevers or weight loss. Further, he denies any
intravenous drug use or recent sexual contacts and does not know anyone with hepatitis. He was
recently diagnosed with pneumonia and completed a course of levofloxacin. He has an
unremarkable past medical history and does not take any regular medications. The family history
is unremarkable. On physical examination, his blood pressure is 110/66 mm Hg and pulse is
94/min. Pulse oximetry shows oxygen saturation of 98% on room air. He has scleral icterus and
pale mucous membranes. The lungs are clear to auscultation. No jugular venous distension is
present. There is a 2/6 early systolic murmur. No hepatosplenomegaly or ascites is observed. He
has a rapid bounding pulse. Extremities show absence of clubbing, cyanosis, and edema.
Laboratory studies show: 
White blood cells 8,900/mm3
Hematocrit 27%
Platelets 198,000/mm3
AST 32 U/L
ALT 24 U/L
Alkaline phosphatase 98 U/L
Total bilirubin 3.8 mg/dL
Conjugated bilirubin 0.7 mg/dL
Spherocytes are seen on peripheral blood smear. Which of the following is the most likely cause
of the patient's anemia? 
Correct Answer ImageA.Autoimmune hemolysis
Incorrect Answer ImageB.Drug-mediated hemolysis
Incorrect Answer ImageC.Hemolytic uremic syndrome
Incorrect Answer ImageD.Hereditary spherocytosis
Incorrect Answer ImageE.Paroxysmal nocturnal hemoglobinuria

A 54-year-old man comes to the physician because of anorexia, weight loss, malaise, vomiting,
headache, and arm and leg pain for the past 4 months. He has a long-standing history of
hypertension and diabetes mellitus and has a GFR of 15 mL/min. His blood pressure is 150/86
mm Hg and respirations are 22/min. On examination, he appears pale. The rest of the
examination appears normal. Laboratory studies show:
RBC count 3.9 million/mm3
HCT 35%
Hemoglobin 10 g/dL
WBC count 4,500/mm3
Neutrophils 60%
Lymphocytes 30%
Monocytes 5%
Eosinophils 2%
BUN 80 mg/dL
Creatinine 7.2 mg/dL
Sodium 136 mEq/L
Potassium 5 mEq/L
Chloride 105 mEq/L
Bicarbonate 18 mEq/L
Calcium 7.9 mg/dL
Albumin 3.7 g/dL
Phosphate 8.2 mg/dL
Parathyroid hormone: 680 pg/mL
Which of the following is the most appropriate initial step in management? 
Incorrect Answer ImageA.Alimentary phosphate binder
Incorrect Answer ImageB.Calcimimetic therapy
Correct Answer ImageC.Initiate chronic dialysis
Incorrect Answer ImageD.Subtotal parathyroidectomy
Incorrect Answer ImageE.Vitamin D

A 26-year-old African American woman comes to the local emergency department with her first
experience of a severe headache. She is fully conscious and ranks this episode as 8/10 in terms of
pain. Additionally, she has noticed both her upper arms being weak. She denies any neck
stiffness. Her past medical history is notable for a rough course of lupus, for which she takes
prednisone. She is also on oral contraceptives for birth control. She denies alcohol or tobacco
use, and she has not used any illicit substances. On physical examination, she is afebrile and
normotensive. Her pulse is 93/min and oxygen saturation 97% on room air. Head and neck
examination is unremarkable. The chest is clear to auscultation and she has no abnormal heart
sounds. Abdominal examination reveals no focal tenderness. Extremities demonstrate no
cyanosis, clubbing, or edema. She has 3/5 strength in both upper extremities, a decrease in
strength from her previously documented physical examination, and her left facial muscles are
mildly weak. Complete blood count shows:
White blood cells 7,800/mm3
Hematocrit 43%
Platelets 322,000/mm3
Computed tomography of the brain demonstrates bilateral infarcts along the anterior and
posterior frontal lobes and parietal lobes, extending into the white matter. Which of the following
is most likely to reveal the diagnosis? 
Incorrect Answer ImageA.Cerebral arteriogram
Correct Answer ImageB.Cerebral venogram
Incorrect Answer ImageC.Echocardiogram
Incorrect Answer ImageD.Funduscopic examination
Incorrect Answer ImageE.Lumbar puncture

A 64-year-old man comes to the emergency room with bloody stools for 2 days. While waiting
to be seen, he had another large evacuation of dark red blood. His pulse is 110/min and blood
pressure is 90/70 mm Hg. Hemoglobin level is 9 mg/dL. A nasogastric tube is inserted and
aspiration produces clear, green fluid without blood. Digital rectal examination and anoscopy
show only blood in the rectal vault but do not identify a source. Which of the following is the
most appropriate diagnostic study at this time?
Correct Answer ImageA.Angiogram
Incorrect Answer ImageB.Barium enema
Incorrect Answer ImageC.Capsular endoscopy
Incorrect Answer ImageD.Colonoscopy
Incorrect Answer ImageE.Esophagogastroduodenoscopy (EGD)

A 31-year-old man comes to the physician because of a 7 year history of progressive back pain.
He has pain involving his entire back and hips. The back pain is worse at night and occasionally
wakes him from sleep. He has morning stiffness lasting 1 hour. He takes ibuprofen which
provides only partial relief from his pain. Examination shows thoracic kyphosis. The patient is
unable to stand with his back flat against a wall. There is tenderness on the pelvis at the
sacroiliac joint. Erythrocyte sedimentation rate is 70 mm/hr. A radiograph of the spine is
shown. Which of the following is the most appropriate pharmacotherapy for this patient?
Incorrect Answer ImageA.Abl tyrosine kinase inhibitor
Incorrect Answer ImageB.Anti-CD11a agent
Incorrect Answer ImageC.Anti-CD20 agent
Correct Answer ImageD.Anti-TNF-α agent
Incorrect Answer ImageE.EGFR inhibitor

A 68-year-old woman is brought to the hospital because of difficult to control hypertension. She
has a 3-year history of hypertension and has a documented intolerance to ACE inhibitors
manifested by a rapid decline in her renal function. She also has had two episodes of acute
pulmonary edema in the past. She quit smoking 2 years ago and is compliant with her
medications. Two weeks ago, her creatinine was 1.3 mg/dL and urinalysis showed microscopic
hematuria. Her blood pressure is 180/100 mm Hg. Physical examination shows a prominent
apical impulse. Her pulses are 2+ bilaterally in the upper and lower extremities. Laboratory
studies show:
Hematocrit 38%
Leukocyte count 5,800/mm3
Na+ 144 mEq/L
K+ 3.6 mEq/L
HCO3 −
28 mEq/L
Urea nitrogen (BUN) 22 mg/dL
Creatinine 1.3 mg/dL
Which of the following is the most likely cause of this patient's hypertension? 
Incorrect Answer ImageA.Aldosterone secreting tumor
Incorrect Answer ImageB.Coarctation of the aorta
Incorrect Answer ImageC.Essential hypertension
Incorrect Answer ImageD.Pheochromocytoma
Correct Answer ImageE.Renal artery stenosis

A 24-year-old man comes to the physician because of 3 days of progressive left testicular pain
and swelling. The pain began gradually, radiates toward the left flank region and is not
associated with nausea or vomiting. He does not report any trauma. He is sexually active with
multiple partners and inconsistently uses condoms. His temperature is 38.2°C (100.8°F), pulse is
97/min, respirations are 22/min, and his blood pressure is 120/70 mm Hg. On physical
examination, the left testicle is lower in the scrotum than the right testicle. There is a distinct,
tender mass on the superior side of the left testis. The left testicular pain improves with testicular
elevation. The right testis is normal. The prostate is mildly tender on digital rectal examination.
A small amount of cloudy urethral discharge is present. Which of the following is the next best
step in the management of this patient?
Incorrect Answer ImageA.Analgesics
Correct Answer ImageB.Antibiotics
Incorrect Answer ImageC.Percutaneous drainage of the mass
Incorrect Answer ImageD.Testicular elevation
Incorrect Answer ImageE.Urgent surgery
A 65-year-old woman consults a physician because of a 3-month history of weight loss, burning
sensation of the tongue, fatigue, anorexia, and poorly localized abdominal pain. The woman
appears pale to the physician and she does not feel a tuning fork that the physician places on the
big toe bilaterally. In-office hematocrit is 35%. An upper endoscopy is performed that shows
inconspicuous rugae on gross inspection. Multiple stomach and esophageal biopsy specimens are
sent and pathology is pending. Which of the following is most likely to be found on further
laboratory testing? 
Incorrect Answer ImageA.Abnormal secretin stimulation test
Incorrect Answer ImageB.Hemoccult-positive stool
Incorrect Answer ImageC.High-grade dysplasia on endoscopy biopsies
Correct Answer ImageD.Markedly elevated serum gastrin levels
Incorrect Answer ImageE.Serum folate of 1.4 ng/mL (normal, >1.9 ng/mL)

A 33-year-old woman comes to the physician because of palpitations, restlessness, sweating,


weight loss, and a tremor for 3 weeks. She does not drink caffeinated beverages or alcohol, and
she does not smoke cigarettes. Her temperature is 37ºC (98.6ºF), blood pressure 130/80 mm Hg,
and pulse 90/min. Examination shows a fine tremor and pretibial myxedema. The thyroid gland
is diffusely enlarged, asymmetric, and lobular. A bruit is present over the gland. Her thyroid-
stimulating hormone level is 0.1 mU/L and T4 level is 25 mcg/dL. There is an increased
radioactive iodine uptake. The patient undergoes radioactive iodine therapy. This patient is at
greatest risk for which of the following conditions? 
Incorrect Answer ImageA.Granulocytopenia
Incorrect Answer ImageB.Graves ophthalmopathy
Correct Answer ImageC.Hypothyroidism
Incorrect Answer ImageD.Radiation thyroiditis
Incorrect Answer ImageE.Thyroid carcinoma

A 24-year-old woman comes to your office for evaluation 2 days after a visit to the emergency
department. She says that she went to the emergency department because of an "allergic
reaction" that consisted of itching in the throat after eating ice cream, followed by nausea,
vomiting, a sensation of flushing, and hives. She experienced the same symptoms last year after
eating chocolate candy and once before in a Thai restaurant. Her friend suggested that she might
have a peanut allergy, but the patient does not recall eating peanuts when she had these reactions.
When she is in your office, she does not have any symptoms. The patient seems healthy and has
no significant past medical history. Her only medication is an oral contraceptive. Physical
examination in your office is unremarkable. Which of the following is an appropriate next step in
evaluating and managing this patient?
Incorrect Answer ImageA.Begin desensitization (allergy shots) with peanut extract
Incorrect Answer ImageB.Have the patient keep a food diary to identify food consumed before the
next allergic episode
Incorrect Answer ImageC.Initiate a food challenge with peanuts
Correct Answer ImageD.Order a skin-prick test or radioallergosorbent test for peanut allergy
Incorrect Answer ImageE.Start the patient on preventive therapy with H2 blockers and
diphenhydramine
A 40-year-old woman has an 8-month history of diffuse joint pain and swelling that involves
both hands and knees. She states that she cannot get any work done in the morning because of
the pain, but it usually subsides as the day progresses. She also complains of fatigue, weight loss,
and generalized weakness. Her temperature is 37.8ºC (100ºF), blood pressure is 110/70 mm Hg,
pulse is 60/min, and respirations are 18/min. Physical examination shows tenderness and
swelling of proximal interphalangeal and metacarpophalangeal joints in the hands and knees.
Laboratory studies show hematocrit 34% and hemoglobin 10 g/dL. A radiograph of this patient's
knee is most likely to show which of the following? 
Correct Answer ImageA.Bone erosions
Incorrect Answer ImageB.Osteophyte formation
Incorrect Answer ImageC.Subchondral cyst formation
Incorrect Answer ImageD.Subchondral sclerosis
Incorrect Answer ImageE.Subchondral tophi

A 67-year-old man comes to the physician for a follow-up visit after being diagnosed with 10
squamous cell carcinomas in the last 18 months. Prior to this, he had 3 to 4 actinic keratoses. He
has a history of hypertension, dyslipidemia, type 2 diabetes mellitus, and diabetic nephropathy
leading to renal failure; he received a renal transplant 6 years ago. Current medications include
hydrochlorothiazide, atenolol, lovastatin, cyclosporine, prednisone, and mycophenolate mofetil.
He has smoked 2 1/2 packs of cigarettes daily for 48 years. He retired 2 years ago from his job as
a teacher. Which of the following is the most significant risk factor for this patient's cutaneous
squamous cell carcinomas?
Correct Answer ImageA.Immunosuppression
Incorrect Answer ImageB.Past history of actinic keratoses
Incorrect Answer ImageC.Past history of diabetes mellitus
Incorrect Answer ImageD.Smoking history
Incorrect Answer ImageE.Sun exposure

A 47-year-old man comes to the physician for a routine health maintenance examination. His
medical history is unremarkable and he takes no medications. His temperature is 36.7ºC (98ºF),
pulse is 88/min, and blood pressure is 125/70 mm Hg. Physical examination shows no
abnormalities. Laboratory studies show: 
Hematocrit  42%
Leukocytes  6,000/mm3
Na+ 141 mEq/L
K+ 4.1 mEq/L
Cl −
102 mEq/L
HCO3− 24 mEq/L
Protein, total  7.0 g/dL
Albumin  4.1 g/dL 
Urea nitrogen (BUN)  10 mg/dL
Creatinine  0.8 mg/dL
Phosphate 1.8 mg/dL
Calcium 11.4 mg/dL
Alkaline phosphatase  45 U/L
Uric acid  7.3 mg/dL
A chest x-ray shows no abnormalities. Which of the following is the most likely cause of this
patient's hypercalcemia? 
Incorrect Answer ImageA.Lymphoma
Incorrect Answer ImageB.Milk-alkali syndrome
Incorrect Answer ImageC.Multiple myeloma
Correct Answer ImageD.Primary hyperparathyroidism
Incorrect Answer ImageE.Sarcoidosis
Incorrect Answer ImageF.Secondary hyperparathyroidism
Incorrect Answer ImageG.Squamous cell lung cancer
Incorrect Answer ImageH.Vitamin D excess

A 20-year-old man comes to the physician because of a 3-day history of slight fever, runny nose,
and productive cough. He has had sputum streaked with small amounts of blood for the past day.
He says that his roommate was ill recently with an upper respiratory infection. He has had no
fever, chills, shakes, nausea, vomiting, diarrhea, weight loss, chest pain, leg swelling, dysuria,
hematuria, or any skin rash. His medical history is unremarkable. He takes no medications and
he has no allergies to medications. He denies tobacco and drug use. His temperature is 37.8ºC
(100ºF), blood pressure is 118/76 mm Hg, pulse is 90/min, and respirations are 16/min. The
lungs are clear to auscultation bilaterally. Laboratory studies show:
Hemoglobin 14.1 g/dL
Leukocyte count 6,600/mm3
INR 1.1 
Which of the following is the most appropriate next step in management? 
Incorrect Answer ImageA.Check antinuclear cytoplasmic antibody
Incorrect Answer ImageB.Check urinalysis
Incorrect Answer ImageC.Obtain a chest CT scan
Correct Answer ImageD.Obtain a chest radiograph
Incorrect Answer ImageE.Supportive care only

A 50-year-old woman comes to the physician because of a tingling sensation and loss of feeling
in her legs. She also states that she has been having problems with her balance while standing.
These problems have been occurring over the past several months. Her medical history is
remarkable for Crohn’s disease. The rest of her neurologic examination is normal. Her resting
pulse is 100/min and a left-arm sitting blood pressure is 100/58 mm Hg. The hematocrit is found
to be 26% with an MCV of 110 fL. The peripheral blood smear shows anisocytosis and
poikilocytosis with macrocytes and six-lobed neutrophils. The white-cell count is 4,000/L and
the platelet count is 140,000/µL. Which of the following is the most likely diagnosis? 
Incorrect Answer ImageA.Aplastic anemia
Correct Answer ImageB.Cobalamin deficiency
Incorrect Answer ImageC.Folic acid deficiency
Incorrect Answer ImageD.Improper diet
Incorrect Answer ImageE.Myelodysplasia

A 64-year-old woman comes to the physician for a routine physical examination. She has no
history of any major medical illness and takes no medications. Her pulse is 90/min, respirations
are 16/min, and blood pressure is 145/95 mm Hg. Cardiac examination shows an apical impulse
that is displaced to the sixth intercostal space, midclavicular line. There is a crescendo-
decrescendo systolic murmur heard best at the second right intercostal space and radiating to the
carotids. ECG shows no abnormalities. Echocardiogram shows mild aortic stenosis with normal
EF and aortic root diameter. A repeat blood pressure one week later is 148/90 mm Hg. Which of
the following is the most appropriate next step in patient care?
Incorrect Answer ImageA.Balloon valvuloplasty
Correct Answer ImageB.Enalapril
Incorrect Answer ImageC.Hydrochlorothiazide
Incorrect Answer ImageD.Metoprolol
Incorrect Answer ImageE.Valve replacement

A 48-year-old woman comes to her physician for pain in the upper abdomen and intermittent
diarrhea that started about 2 months ago. She has tried taking antacids and loperamide with
limited relief. She has no history of recent travel. On physical examination, her temperature is
36.7ºC (98.6ºF) and blood pressure is 125/80 mm Hg. Head, neck, cardiovascular, and chest
examinations are unremarkable. There is tenderness to palpation in the epigastrium, but her
abdomen is soft. Peripheral examination shows no clubbing, cyanosis, or edema. Rectal
examination does not show any blood or mucus, the test for ova and parasites is negative. An
upper gastrointestinal endoscopy is performed, which shows a solitary ulcer in the third portion
of the duodenum. A biopsy is unremarkable. The rapid urease CLO test is negative. She is
prescribed omeprazole but returns several weeks later saying that the symptoms persist. Which
of the following is the next best step in the management of this patient?
Incorrect Answer ImageA.Add amoxicillin with clarithromycin
Incorrect Answer ImageB.Increase the dose of omeprazole
Incorrect Answer ImageC.Secretin stimulation test
Incorrect Answer ImageD.Serum gastrin level
Incorrect Answer ImageE.Stop omeprazole and order secretin stimulation test
Correct Answer ImageF.Stop omeprazole and order serum gastrin level

A 24-year-old woman comes to the physician because of palpitations for 1 month. She has had a
6-kg (13-lb) weight loss over the past month. Her medical history is unremarkable and she takes
no medications. She does not smoke cigarettes, drink alcohol, or use illicit substances. She
weighs 48 kg (106 lb) and is 168 cm (66 in) tall. Her temperature is 37.7ºC (99.9ºF), blood
pressure is 140/80 mm Hg, pulse is 105/min, and respirations are 20/min. The skin is warm and
moist. Laboratory studies show: 
Thyroid stimulating hormone 0.4 uU/mL (normal, 0.5–5.0
(TSH)  uU/mL) 
Free T4  25 ug/mL (normal, 5–12
ug/mL)
Which of the following is the most appropriate next step in diagnosis? 
Incorrect Answer ImageA.Exploratory neck surgery
Incorrect Answer ImageB.MRI of the pituitary gland
Incorrect Answer ImageC.Needle core biopsy of the thyroid mass
Correct Answer ImageD.Radionuclide thyroid scan
Incorrect Answer ImageE.Serum levels of T3
A 23-year-old man known to have type 1 neurofibromatosis comes to the physician with a left
lower quadrant abdominal mass and signs of neurologic deficits in his left leg. In the following
workup, it is determined that he has higher than normal levels of metabolites of epinephrine and
norepinephrine in a 24-hour urinary collection. He is currently normotensive. Before invasive
steps are taken to biopsy and eventually remove his left lower quadrant abdominal mass, which
of the following is the most appropriate next step in management? 
Incorrect Answer ImageA.CT scan of the head, looking for meningiomas
Correct Answer ImageB.MRI scan of his adrenal glands and pelvis
Incorrect Answer ImageC.MRI scan of the acoustic nerves
Incorrect Answer ImageD.Radiation therapy to the left lower quadrant abdominal mass
Incorrect Answer ImageE.Radionuclide scans from the neck to the pelvis, looking for extra-adrenal
pheochromocytomas

A 45-year-old woman comes to the physician because of progressive fatigue for 3 months and
ulcers on her fingers. She has had stiff joints, weakness, insomnia, and mild shortness of breath.
She has had a 2.5 kg (5.5 lb) weight loss over the last 2 months. She has a history of
hypertension and gastroesophageal reflux, which has been difficult to treat. Her blood pressure is
150/92 mm Hg and respirations are 18/min. Perioral examination and left arm are shown. There
are digital ulcers and pitting of the fingertips. Because of her condition, this patient is at greatest
risk for death from which of the following?
Incorrect Answer ImageA.Intestinal pseudo-obstruction
Incorrect Answer ImageB.Myocardial fibrosis and heart failure
Incorrect Answer ImageC.Paralysis of the diaphragm
Correct Answer ImageD.Pulmonary fibrosis and hypertension
Incorrect Answer ImageE.Ruptured abdominal aortic aneurysm (AAA)

At a health-fair screening, an obese 68-year-old woman is found to have a random blood sugar of
360 mg/dL. Subsequent tests show fasting levels of 240 mg/dL and a glycosylated hemoglobin
A1c level of 12%. Ophthalmologic evaluation shows "cotton-wool" spots, microaneurysms, and
neovascularization of the optic nerve and the retinal periphery. However, no neovascularization
of the iris or macular edema was noted. Which of the following is the most appropriate
management to prevent vision loss in this patient? 
Incorrect Answer ImageA.Cryotherapy
Incorrect Answer ImageB.Intravitreal steroid Injection
Correct Answer ImageC.Laser photocoagulation
Incorrect Answer ImageD.Tight glycemic control
Incorrect Answer ImageE.Vitrectomy

A 33-year-old woman with no significant past medical history is brought to the emergency
department with acute onset of severe shortness of breath. She reports that right-sided calf pain
developed after a long plane trip home from Australia and that several hours after the onset of
this calf pain she became acutely short of breath. History is difficult to obtain because of the
patient's extreme shortness of breath, but her companion reports that she has had no recent
surgeries or history of bleeding. Her temperature is 37.8°C (100.0°F), pulse is 140/min,
respirations are 34/min, and systolic blood pressure is 70 mm Hg. Physical examination shows a
woman who is in severe respiratory distress and using accessory respiratory muscles. Her heart is
rapid and regular, lungs are clear, and abdomen is benign. There is jugular venous distension.
Her right calf is swollen and tender. The ventilation-perfusion scan shows a high probability of
pulmonary embolism. Arterial blood gases show pH 7.48, pCO2 20 mm Hg, and PaO2 48 mm Hg
on 6 L oxygen. Which of the following is the most appropriate management?
Incorrect Answer ImageA.Embolectomy
Incorrect Answer ImageB.Low molecular-weight heparin
Incorrect Answer ImageC.Spiral CT Angiogram of the chest
Correct Answer ImageD.Thrombolysis
Incorrect Answer ImageE.Unfractionated heparin
Incorrect Answer ImageF.Warfarin alone

A 33-year-old woman arrives in her physician's office because of diarrhea, abdominal pain,
fatigue, and weight loss. She has had these symptoms for the past several months, and they are
especially severe at present. She has noticed her stools "floating." She has also had occasional
fevers. Her past medical history is unremarkable, and she takes no medications. Her family
history is positive for inflammatory bowel disease. On physical examination, she is thin-
appearing but in no acute distress. She appears dehydrated. Head, neck, oral, cardiovascular, and
chest examinations are normal. She has generalized pain to palpation of her hypogastrium. She is
hemoccult-positive on rectal examination. Her peripheral examination demonstrates no clubbing,
cyanosis, or edema. A small bowel follow-through demonstrates narrowing of the last 60 cm of
her ileum, and colonoscopy with ileoscopy demonstrates deep fissures and inflammation of the
terminal ileum, with stricturing. Given her presentation, which of the following may also be
seen? 
Incorrect Answer ImageA.Hyperactivity of the clotting cascade
Incorrect Answer ImageB.Hypercalcemia
Incorrect Answer ImageC.Hypervitaminosis A
Correct Answer ImageD.Macrocytic anemia
Incorrect Answer ImageE.Struvite nephrolithiasis

A 28-year-old woman comes to the physician because of an 8-month history of headaches that
are not relieved with high doses of acetaminophen or ibuprofen. She says that the headaches
usually occur in the morning and are worsened by coughing and sneezing. She has not had
nausea or vomiting associated with the headaches but has occasional blurry vision with them.
Current medications include oral contraceptive pills. Her temperature is 37.0°C (98.6°F), pulse is
68/min, respirations are 16/min, and blood pressure is 132/82 mm Hg. Physical examination
shows bilateral papilledema. There is no nuchal rigidity and Kernig and Brudzinski signs are
negative. The remainder of the physical examination shows no abnormalities. Which of the
following is the most appropriate next step in management? 
Incorrect Answer ImageA.Acetazolamide
Incorrect Answer ImageB.Intramuscular injection of sumatriptan
Incorrect Answer ImageC.Lumbar puncture
Incorrect Answer ImageD.Lumbar puncture plus ceftriaxone and ampicillin
Correct Answer ImageE.MRI of the brain

A 48-year-old man comes to the physician for a routine health maintenance examination. Current
medications include aspirin. His mother died of a stroke at age 72 years and his father died of a
myocardial infarction at age 68 years. His brother developed sigmoid colon cancer at age 56
years. Physical examination shows no abnormalities. Which of the following is the most
appropriate next step in management?
Incorrect Answer ImageA.Annual digital rectal examination and fecal occult blood testing
Correct Answer ImageB.Colonoscopy
Incorrect Answer ImageC.Flexible sigmoidoscopy
Incorrect Answer ImageD.Flexible sigmoidoscopy and barium enema
Incorrect Answer ImageE.Genetic testing for the p53 gene

A 33-year-old woman comes to the emergency department because of lower extremity weakness.
She recovered from a gastrointestinal infection 5 days ago. Yesterday, she initially developed
paresthesias in her hands and feet followed over the next few hours with weakness in her legs
and arms. She has difficulty standing and lifting objects. She denies any vision loss, hemiparesis,
facial droop, and dizziness. Her past medical history is unremarkable, and she does not take any
medications. She does not use tobacco, alcohol or illicit substances. She works as a lawyer in a
local firm. Her blood pressure is 131/72 mm Hg, pulse is 79/min, and oxygen saturation is 96%
on room air. Neurologic examination shows bilaterally decreased reflexes, normal plantar
response, and 2/5 muscle strength in her arm and leg flexors and extensors. Her sensation is
intact. Pulmonary, cardiovascular, and abdominal examinations are normal. Computed
tomography of the brain is unremarkable. Over the next few hours, she becomes progressively
obtunded, and her oxygen saturation falls to 75% on room air. She is intubated and sent to the
intensive care unit. Which of the following is the most likely explanation for her hypoxemia?
Incorrect Answer ImageA.Decreased alveolar to capillary diffusion
Incorrect Answer ImageB.Decreased fraction of inspired oxygen
Correct Answer ImageC.Hypoventilation
Incorrect Answer ImageD.Pulmonary shunt
Incorrect Answer ImageE.Ventilation-perfusion mismatch

A 69-year-old farmer of Irish descent has a non-healing, indolent, 0.5 cm ulcer on his lower lip,
arising from the vermilion border. The ulcer has been present and growing over the past 8
months. He does not drink but smokes 1 pack of cigarettes per day. Physical examination shows
no other ulcers or enlarged lymph nodes. Which of the following are the most likely diagnosis
and best method to confirm the diagnosis?
Incorrect Answer ImageA.Adenocarcinoma; biopsy the center of the ulcer
Incorrect Answer ImageB.Basal cell carcinoma; fine-needle aspiration
Incorrect Answer ImageC.Benign fungal ulceration; culture scrapings
Incorrect Answer ImageD.Melanoma; full thickness biopsy at the edge of the lesion
Correct Answer ImageE.Squamous cell carcinoma; full-thickness biopsy at the edge of the lesion

A 28-year-old man comes to the physician because of a skin rash his girlfriend noticed on his
chest and back 1 week ago. He has had occasional pruritis after exercising but denies fevers or
weight loss. His past medical history is unremarkable, and he has no allergies to medications. He
works in a demanding job and exercises in a gym 5 nights a week. His mother died of breast
cancer at age 62 years. He is in no acute distress. Physical examination shows multiple
hypopigmented patches with a fine scale accentuated by rubbing over the neck, anterior chest,
shoulders, and upper back. The remainder of the examination shows no abnormalities. Which of
the following is the most likely underlying mechanism for this pigmentary change?
Incorrect Answer ImageA.Abnormal transfer of melanosomes
Incorrect Answer ImageB.Autoimmune destruction of melanocytes
Correct Answer ImageC.Competitive inhibition of tyrosinase
Incorrect Answer ImageD.Lysosomal defect
Incorrect Answer ImageE.Self-inflicted physical trauma

A 52-year-old African American man presents to his physician for a routine health checkup. He
has been relatively well and has no new complaints. He denies fatigue, weight loss, polyuria,
polydipsia, fevers, or night sweats. His only complaint is that he has to hold the newspaper
farther away than usual to see properly. He denies any difficulty seeing objects at a distance. He
has not had any eye pain or red eye. His past medical history is unremarkable and he does not
take any medications. On physical examination, he is afebrile and normotensive. His
funduscopic examination is unremarkable. Head and neck, chest, cardiovascular, and abdominal
examinations are normal. He has no clubbing, cyanosis, or edema. He is referred to an
ophthalmologist for visual acuity and glaucoma screening. How often should he be tested for
glaucoma? 
Incorrect Answer ImageA.Every 3-5 years
Incorrect Answer ImageB.Every 3-5 years after the age of 60 years
Incorrect Answer ImageC.Every 10 years
Correct Answer ImageD.Every year
Incorrect Answer ImageE.Once only

A 24-year-old man is being evaluated for a declining serum sodium concentration. Four days ago
he suffered a subarachnoid hemorrhage from a ruptured anterior communicating artery
aneurysm. The hemorrhage was evacuated and the aneurysm was clipped. He is now on the
neurosurgical ward. He has mild incisional head pain, but no nausea, vomiting, or cognitive
disturbances. His serum sodium has progressively decreased from 138 mEq/L to 128 mEq/L. On
physical examination, the temperature is 98°F (36.7°C), pulse is 92/min supine and 104/min
sitting, respirations are 18/min, and blood pressure is 130/70 mm Hg supine and 124/68 mm Hg
sitting. He is alert and oriented without neurologic deficits. The glucose, BUN, creatinine, 8:00
AM plasma cortisol, and TSH concentrations are within normal limits. The serum urate is 2.1
mg/dL. The plasma osmolality is 268 mOsm/kg H2O. In a spot urine sample, the osmolality is
460 mOsm/kg H2O, the sodium concentration is 70 meq/L, and the potassium concentration is 50
meq/L. Which of the following is an appropriate next test strategy? 
Incorrect Answer ImageA.5% dextrose in water bolus with measurement of plasma osmolality
Correct Answer ImageB.Change in urine osmolality after normal (0.9%) saline infusion
Incorrect Answer ImageC.Desmopressin administration with measurement of plasma osmolality
Incorrect Answer ImageD.Restrict water intake to 1 L/day with measurement of plasma osmolality
Incorrect Answer ImageE.Tolvaptan administration with measurement of plasma osmolality

A 20-year-old man is brought to the emergency department after a witnessed seizure. He reports
that he has had a persistent headache and a fever for the past week accompanied by greenish
nasal discharge. He has no prior medical history and is not taking any medications. He denies
being sexually active. His family history is significant for an uncle with type 1 diabetes mellitus.
His temperature is 39.7ºC (103.5ºF), pulse is 100/min, respirations are 19/min, and blood
pressure is 120/85 mm Hg. Physical examination shows left-sided weakness and a rooting reflex.
He appears disinhibited in his behavior. A CT scan of the head shows a ring-enhancing lesion in
the right frontal lobe and an air-fluid level in the right frontal sinus. Which of the following will
likely be seen on aspiration of the lesion?
Correct Answer ImageA.Alpha-hemolytic Streptococcus and mixed anaerobes
Incorrect Answer ImageB.Bacteroides fragilis 
Incorrect Answer ImageC.Budding yeast organisms with hyphae
Incorrect Answer ImageD.Immature B cells suggestive of non-Hodgkin lymphoma
Incorrect Answer ImageE.Toxoplasma gondii cysts

A 55-year-old woman comes to the office because of burning and tingling sensations in the left
hand for several months. She states that she has been frequently awakened at night by aching
pain in the same hand. Past medical history is significant for hypertension for 3 years and
hypercholesterolemia for 2 years. She is on hydrochlorothiazide and atorvastatin. Pain is elicited
by extreme palmar flexion of the wrist. The patient is unable to correctly identify different types
of cloth by rubbing it between the left thumb and index finger. Which of the following is the
most likely diagnosis? 
Incorrect Answer ImageA.Angina pectoris
Correct Answer ImageB.Carpal tunnel syndrome
Incorrect Answer ImageC.Complex regional pain syndrome
Incorrect Answer ImageD.Dupuytren contracture
Incorrect Answer ImageE.Fibrositis

A 25-year-old African American woman comes to the emergency department because of a


severe sore throat, fever, and shaking chills. Her symptoms began 2 days ago and have
progressed to the point that she is weak, confused, and barely able to get out of bed. She reports
diffuse myalgias and arthralgias for 1 week, accompanied by mild nonproductive cough,
headache, and mild abdominal pain. Before this she has been generally healthy, although she
states that she takes medication for her thyroid condition. In addition to this, she takes an oral
contraceptive pill daily and over-the-counter pain medication as needed. Temperature is 39.4ºC
(103.0ºF), pulse is 120/min (regular), respirations are 30/min, and blood pressure is 80/40 mm
Hg. Examination shows an ill-appearing young woman without any localizing findings. A
complete blood count shows: 
Hematocrit  36%
Hemoglobin  12 g/dL
Leukocyte count  200/mm3
Platelet count  130,000/mm3
Which of the following is the most likely cause of this patient's current condition? 
Incorrect Answer ImageA.Fanconi anemia
Incorrect Answer ImageB.Hepatitis B infection
Incorrect Answer ImageC.Methotrexate
Incorrect Answer ImageD.Parvovirus B19
Correct Answer ImageE.Propylthiouracil
A 32-year-old man with a history of IV drug use comes to the physician because of progressively
worsening edema for one month. His blood pressure is 170/95 mm Hg and the physical exam
shows 3+ bilateral pedal edema. The serum creatinine is 2.5 mg/dL, and urinalysis shows 10–20
RBCs/hpf and 4+ proteinuria with oval fat bodies. A renal biopsy shows segmental sclerosis of
50–60% of glomeruli. Which of the following laboratory studies is the most appropriate next
step in diagnosis?
Incorrect Answer ImageA.Anti-glomerular basement membrane antibodies
Incorrect Answer ImageB.Antistreptolysin O titer
Incorrect Answer ImageC.Blood cultures
Correct Answer ImageD.HIV test
Incorrect Answer ImageE.Serum IgA level

A 69-year-old woman with a history of gastroesophageal reflux disease and no prior myocardial
infarction is brought to the emergency department because of chest pain. Ten hours earlier, she
had been walking the dog when she felt a crushing pain in her upper abdomen and chest. During
this time she also felt nauseous, lightheaded, and began sweating. She promptly returned home
and took two ibuprofen. However, her pain did not respond and continued to worsen. She is a 35
pack-year smoker and drinks 10 beers a week. Her temperature is 37.7°C (99.9°F), pulse is
126/min, respirations are 20/min, and blood pressure is 100/59 mm Hg, which does not change
on inspiration. Oxygen saturation on room air is 91%. Upon arrival in the emergency
department, she is placed on 100% oxygen via facemask. On physical examination, the patient is
in obvious distress. She appears sweaty and pale. She is disorientated to time and place, but is
oriented to person. There is jugular venous distension. Cardiovascular examination shows
muffled heart sounds without murmurs, rubs, or gallops. Lungs are clear to auscultation and
there is no evidence of peripheral edema. ECG shows no evidence of ST-segment elevation.
There are tall R waves in leads V1 and V2, with ST-segment depression in leads V1 and V3. Which
of the following is the most appropriate next step in management?
Correct Answer ImageA.Aspirin
Incorrect Answer ImageB.Esophageal ventricular electrocardiogram
Incorrect Answer ImageC.Pericardiocentesis
Incorrect Answer ImageD.Streptokinase
Incorrect Answer ImageE.Stress echocardiogram
Incorrect Answer ImageF.Tissue-plasminogen activator

A 65-year-old male comes to his physician for evaluation of recurrent small cell lung carcinoma
after undergoing initial systemic chemotherapy and radiation one year ago. Cells are harvested
from the tumor and are treated with various chemotherapeutic agents in vitro. None of the agents
are able to cause cell cycle arrest or cell death. Which of the following proteins is likely to be
expressed in increased quantities in these cells and accounts for the observed response?
Correct Answer ImageA.MDR1
Incorrect Answer ImageB.p53
Incorrect Answer ImageC.RAS
Incorrect Answer ImageD.Rb
Incorrect Answer ImageE.Telomerase
A 40-year-old man comes to the emergency department with right-sided flank pain and blood in
his urine for the last 12 hours. He denies any fever, chills, or difficulty in urination.
Approximately 4 months ago he came to his primary care physician with new hypertension,
swelling of his feet and frothy urine. Workup at that time revealed 20 g of protein in a 24-hour
urine specimen. A kidney biopsy showed membranous nephropathy. Tests for hepatitis B antigen
and antinuclear antibodies were negative. Therapy with rituximab was started. However, the
patient continues to have significant proteinuria. His last 24-hour urine specimen a week ago
showed 10 g of protein in 24 hours. Physical examination shows a man in a moderate amount of
distress caused by pain. His temperature is 36.0ºC (97.0ºF), pulse is 110/min, respirations are
12/min, and blood pressure is 150/90 mm Hg. Lungs are clear to auscultation. Cardiovascular
examination shows no rub or gallop. Abdomen is soft, nontender, and nondistended with right
flank tenderness. Urinalysis shows:
Specific gravity  1.012
Blood 4+
Protein  3+
Leukocyte esterase  Negative
RBCs  10–20/hpf
WBCs  None
Epithelial cells  None
Serum creatinine 1.0 mg/dL
A renal ultrasound shows a diffusely enlarged right kidney without cysts or hydronephrosis.
Which of the following is the best next step in establishing a diagnosis? 
Incorrect Answer ImageA.24-h urine for protein
Incorrect Answer ImageB.Blood culture
Incorrect Answer ImageC.Creatinine clearance
Correct Answer ImageD.CT angiogram scan of the renal arteries and veins
Incorrect Answer ImageE.Renal biopsy
Incorrect Answer ImageF.Urine immunofixation

A 38-year-old man comes to the physician because of restlessness and involuntary jerking
movements of his arms for 8 months. He has also had difficulty remembering things, particularly
recent events. He has a history of hypertension and type 1 diabetes mellitus. Current medications
include lisinopril, hydrochlorothiazide, and insulin. His brother has similar jerking movements of
his arm. Blood pressure is 130/84 mm Hg and pulse is 76/min. Examination shows choreiform
movements of both upper extremities. He is unable to remember what he ate for breakfast. His
hemoglobin A1c is 7.2%. Which of the following is the most appropriate pharmacotherapy for
this patient's involuntary movements? 
Incorrect Answer ImageA.Bromocriptine
Incorrect Answer ImageB.Donepezil
Correct Answer ImageC.Haloperidol
Incorrect Answer ImageD.Levodopa-carbidopa
Incorrect Answer ImageE.Propranolol

A 22-year-old woman comes to the emergency department because of weight loss, irritability,
and insomnia for the past 2 months. She reports intermittent palpitations, diarrhea, and heat
intolerance. She has no history of significant medical illnesses. She has recently started taking
her mother’s medication for weight loss. Her family history is significant for hypothyroidism in
her mother and stroke in her father. She denies alcohol and illicit drug use. Her temperature is
38ºC (100.4ºF), pulse is 132/min, respirations are 25/min, and blood pressure is 152/62 mm Hg.
On physical examination, she is anxious and agitated. There is lid lag and visible sclera above
the limbus of both eyes. Thyroid examination is normal. Cardiovascular examination shows
normal heart sounds with a regular rhythm. Chest auscultation shows normal breath sounds.
Deep tendon reflexes are brisk. An ECG at the time of admission shows sinus tachycardia.
Which of the following laboratory results is most likely to be seen in this patient?
Thyroid- Thyroglobuli Radioactive Iodine
stimulating n Uptake (RAIU)
hormone (TSH)

A. ↑ ↑ ↑

B. ↓ ↑ ↑

C. ↓ ↑ ↓

D. ↓ ↓ ↑

E. ↓ ↓ ↓
Incorrect Answer ImageA.

Incorrect Answer ImageB.

Incorrect Answer ImageC.

Incorrect Answer ImageD.

Correct Answer ImageE.

A 35-year-old woman comes to the physician for a routine health maintenance examination. She
has a history of asthma and seasonal allergies. Current medications include an albuterol inhaler
and loratadine. She has no allergies to medications and has smoked a half-pack of cigarettes
daily for 17 years. Her mother had breast cancer at the age of 64 years, and her father has
hypertension. Physical examination shows a flat, 2 cm diameter patch on the lateral neck with
varying colors from red to brown to black and a poorly defined edge. There are small nodules
within the patch. There is no lymphadenopathy. A biopsy specimen of the lesion shows a
malignant tumor of melanocytes. Which of the following is the most likely diagnosis?
Incorrect Answer ImageA.Acral lentiginous melanoma
Incorrect Answer ImageB.Lentigo maligna melanoma
Incorrect Answer ImageC.Nodular melanoma
Incorrect Answer ImageD.Subungual melanoma
Correct Answer ImageE.Superficial spreading melanoma
A 72-year-old man with a known history of chronic obstructive pulmonary disease (COPD)
comes to the physician because of a worsening cough. He states that he frequently has bronchitis
and believes he is experiencing another episode of the same. His cough is productive with
copious amounts of green-tinged sputum and is associated with mild chest pain. Prior to this
episode, he has been in his usual state of health. He denies fever or chills. In addition to his
COPD, his past medical history is significant for rate-controlled atrial fibrillation, hypertension,
gout, and depression. His current medications include warfarin, atenolol, hydrochlorothiazide
(HCTZ), colchicine, and fluoxetine. His temperature 37.0ºC (98.6ºF), blood pressure 142/98 mm
Hg, pulse is 74/min, and respirations are 22/min. Physical examination shows diffusely
diminished breath sounds, scattered end-expiratory wheezes, and rare rhonchi. Chest radiograph
shows flattened diaphragms, no consolidation, effusion or mass. He is given a prescription for
erythromycin to treat the COPD exacerbation. Which of the following is an appropriate course of
action to minimize harmful drug interactions?
Incorrect Answer ImageA.Check electrolytes, adjust HCTZ appropriately
Incorrect Answer ImageB.Decrease atenolol, monitor pulse, and blood pressure
Correct Answer ImageC.Decrease warfarin, monitor INR
Incorrect Answer ImageD.Hold fluoxetine until after antibiotics are completed
Incorrect Answer ImageE.Replace colchicine with allopurinol

A 49-year-old woman comes to the physician because of difficulty eating for 6 months. She has
had mouth dryness for 1 year. She has stopped wearing her contact lenses because of dryness and
irritation in her eyes. She was diagnosed with 3 dental caries over the past year. Examination
shows bilaterally enlarged cheeks. The oral mucosa is dry, and the tongue is atrophic. There is
diffuse dry skin. A biopsy specimen of the labial salivary gland shows lymphocytic
inflammation. Which of the following autoantibody profiles is most specific for this patient's
condition? 
Incorrect Answer ImageA.Anti-cyclic citrullinated protein (CCP) and anti-IgG
Incorrect Answer ImageB.Anti-double-stranded DNA and anti-Ro
Incorrect Answer ImageC.Anti-IgG and anti-U1-RNP
Correct Answer ImageD.Anti-La and anti-Ro
Incorrect Answer ImageE.Anti-Smith and anti-SSA

A 53-year-old man with a history of cirrhosis is admitted to the hospital with a 5-day history of
fever and abdominal pain. He has a history of hepatitis C infection. Current medications include
spironolactone and propranolol. He does not drink alcohol or smoke cigarettes. His temperature
is 38.3ºC (101ºF), pulse is 110/min, and blood pressure is 100/50 mm Hg. Physical examination
is shown (see media clip). Lungs are clear to auscultation. There are spider angiomata on the
chest and back. Laboratory studies show:
Leukocytes  13,200/mm3
Hematocrit  33%
Prothrombin time 15.2 seconds
Albumin  0.1 g/dL
Sodium  135 mEq/L
Potassium  4.7 mEq/L
Which of the following is the most appropriate next step in diagnosis? 
Incorrect Answer ImageA.Abdominal CT scan
Correct Answer ImageB.Abdominal paracentesis
Incorrect Answer ImageC.Abdominal ultrasound
Incorrect Answer ImageD.Electrocardiogram
Incorrect Answer ImageE.Lumbar puncture

A 57-year-old woman comes to the physician because of a 3 month history of left shoulder pain.
The pain began shortly after casting the wrist and elbow and use of an arm sling for a fractured
radius. The pain is worse at night and she has difficulty sleeping because of the pain. She reports
stiffness and difficulty moving the left shoulder. She takes no medications. Her temperature is
37°C (98.6°F), blood pressure is 134/88 mm Hg, and pulse is 64/min. Physical examination
shows limited active and passive range of motion of the left shoulder. Arthrography shows
decreased capacity of the joint space. Which of the following is the most likely diagnosis? 
Correct Answer ImageA.Adhesive capsulitis
Incorrect Answer ImageB.Rotator cuff tear
Incorrect Answer ImageC.Rheumatoid arthritis
Incorrect Answer ImageD.Septic arthritis
Incorrect Answer ImageE.Subacromial bursitis

A 45-year-old man has a chest radiograph performed in the emergency department following an
automobile accident to evaluate for rib fractures. The radiograph incidentally found a solitary 2-
cm nodule in the right middle lobe. Which of the following features is most consistent with a
malignant lesion?
Incorrect Answer ImageA.Failure to accumulate fluorodeoxyglucose (FDG)
Incorrect Answer ImageB.Popcorn-shaped calcification
Incorrect Answer ImageC.Size doubling time less than 30 days
Correct Answer ImageD.Stippled calcification
Incorrect Answer ImageE.Young age at presentation

A 21-year-old male patient is brought to the emergency department because of seizures. He has
had three convulsive episodes since yesterday. His relatives initially thought it was related with
his excessive ingestion of alcohol during the past weekend, but as the episodes persisted they
decided to seek medical attention. Seven months ago, during a routine medical evaluation, he
was found to have a PPD of 16 mm diameter. He was started on isoniazid at that time. His
temperature is 37.1ºC (98.7ºF), pulse is 94/min, respirations are 16/min, and blood pressure is
130/80 mmHg. On physical examination, he appears confused. There are inflammatory cracks on
the corners of the mouth, while his tongue is smooth and inflamed. Greasy scaling lesions over
red, inflamed skin are observed over the forehead, eyebrows, and pre-sternal region.
Neurological examination demonstrates impaired vibration and proprioception in the lower
extremities bilaterally. Laboratory reports include: 
Hemoglobin 9 g/dL
Platelets 200,000/mm3
White blood cells 6,000/mm3
Segmented 56%
neutrophils
Bands 5%
Eosinophils 2%
Basophils 0%
Lymphocytes 30%
Monocytes 1%
MCV 76 µm3
Serum ferritin elevated
Serum iron elevated
TIBC low
Peripheral smear dimorphic red cell population, microcytic
red cells
The hematological condition of this patient is associated with which of the following diseases? 
Incorrect Answer ImageA.Alcohol-related anemia
Incorrect Answer ImageB.Anemia of chronic disease
Incorrect Answer ImageC.Chronic Lymphocytic Leukemia
Incorrect Answer ImageD.Hemolytic anemia
Correct Answer ImageE.Myelodysplastic syndrome

A 24-year-old man comes to the emergency department complaining of 5 hours of severe


wheezing and shortness of breath. He has used his bronchodilator inhaler 6 times during the past
5 hours with only minimal symptomatic relief. On physical examination he appears dyspneic.
Temperature is 37.4°C (99.3°F), blood pressure is 118/64 mm Hg, pulse is 106/min, and
respirations are 32/min. There is use of the sternocleidomastoid muscles with each inspiration. A
lung examination shows bilateral diffuse inspiratory and expiratory wheezing with poor air
movement and a prolonged expiratory phase. Which of the following will most likely be found
on chest x-ray?
Incorrect Answer ImageA.Bilateral interstitial infiltrate
Incorrect Answer ImageB.Lobar consolidation
Correct Answer ImageC.Normal findings
Incorrect Answer ImageD.Pleural effusion
Incorrect Answer ImageE.Pneumothorax

A 30-year-old man is being evaluated in the emergency department for gross hematuria. He has a
long history of fever, diarrhea, abdominal pain, weight loss, and fatigue. He is scheduled to see
the gastroenterologist for evaluation of these symptoms. When asked about other signs and
symptoms, he states he has noted multiple bruises all over his body, as well as nasal and gum
bleeding. On physical examination, he has multiple ecchymoses on the abdomen and lower
extremities. Anal examination discloses a perianal fistula. Ten minutes after blood is drawn,
oozing is still evident. Laboratory report is as follows:
Hemoglobin  12 g/dL
Platelets  180,000/mm3
White blood cells  6,500/mm3
PT  30 seconds
PTT  45 seconds
Which of the following is the most likely cause of this patient's bleeding? 
Incorrect Answer ImageA.Colovesical fistula
Incorrect Answer ImageB.Hemophilia A
Incorrect Answer ImageC.Immune thrombocytopenic purpura (ITP)
Incorrect Answer ImageD.Paroxysmal nocturnal hemoglobinuria
Incorrect Answer ImageE.Vitamin C deficiency
Correct Answer ImageF.Vitamin K deficiency
Incorrect Answer ImageG.Von Willebrand disease

A 66-year-old man with a history of diabetes and tobacco use comes to his physician for a
routine yearly ophthalmologic examination. He has been treated with an oral hypoglycemic
agent for several years. The doctor dilates the pupils and in the left eye sees the beginning of an
opacity that is cutting off visualization of part of the retina. The patient's visual acuity has not
changed at this time. What is the most appropriate step in management? 
Correct Answer ImageA.Follow his exam more frequently
Incorrect Answer ImageB.Perform laser therapy on the affected eye
Incorrect Answer ImageC.Perform ultrasonic fragmentation of the lens nucleus
Incorrect Answer ImageD.Prescribe heavy glasses
Incorrect Answer ImageE.Treat the affected eye with topical steroids

A 65-year-old woman comes to the emergency department because of generalized weakness,


dizziness, and palpitations for 1 day. She has had no chest pain or shortness of breath. She has a
history of hypertension and coronary artery disease. Current medications include
hydrochlorothiazide, aspirin, and nitroglycerin. She is a retired office manager. She does not
drink alcohol or smoke cigarettes. Her temperature is 37.0ºC (98.6ºF), pulse is 130/min,
respirations are 20/min, blood pressure is 106/65 mm Hg, and oxygen saturation is 96% on room
air. The lungs are clear to auscultation. Cardiac examination shows irregular rhythm; no
murmurs are heard. Complete blood count and basic metabolic panel are within normal limits.
An ECG is shown. Which of the following is the most appropriate next step in management?
Incorrect Answer ImageA.Defibrillation
Incorrect Answer ImageB.IV amiodarone
Incorrect Answer ImageC.IV diltiazem
Correct Answer ImageD.IV metoprolol
Incorrect Answer ImageE.Transcutaneous pacing
Incorrect Answer ImageF.Warfarin

A 70-year-old woman comes to her physician because of intermittent head and neck pain for the
past 6 months. She reports episodes of dizziness that are brought on by turning her head to the
right. She denies chest pain, diaphoresis, nausea, and shortness of breath. Her past medical
history is significant for endometriosis, for which she underwent a hysterectomy with bilateral
oophorectomy 30 years ago. She takes no medications. Her temperature is 36.5°C (97.7°F), pulse
is 80/min, respirations are 12/min, and blood pressure is 132/70 mm Hg. Physical examination
shows decreased range of motion of the neck. Flexion of the neck to the right side produces pain,
which radiates to the right upper extremity. Deep tendon reflexes are normal. Muscle strength is
5/5 in all extremities. Which of the following is the most appropriate next step in diagnosis?
Incorrect Answer ImageA.Angiographic studies of cerebral vessels
Incorrect Answer ImageB.Electrocardiogram
Incorrect Answer ImageC.Electromyography (EMG)/ Nerve conduction studies (NCS)
Incorrect Answer ImageD.MRI of the spinal column
Correct Answer ImageE.X-rays of the head and neck
A previously healthy 27-year-old woman comes to her physician with ankle swelling that started
a few days ago. She has been well and has no other symptoms. She takes no medications, does
not drink alcohol or smoke, and takes no recreational drugs. Her vital signs were normal during a
routine evaluation last year. Vital signs today show: temperature 36.8ºC (98.2ºF), pulse 90/min,
and blood pressure 170/103 mm Hg. The optic fundi are normal. The neck, heart, and lung
exams are normal. The abdomen is normal without bruits. Extremities show 1+ bilateral edema. 
Serum studies show:
Normal electrolytes and glucose
BUN 13 mg/dL
Creatinine 1.2 mg/dL
Albumin 2.5 mg/dL
Cholesterol 260 mg/dL
Urinalysis shows 4+ protein, otherwise normal (no cells or casts)
ECG is normal
In addition to restricting sodium in her diet and rechecking the blood pressure in 1 week, which
of the following is the best next step in managing her hypertension? 
Correct Answer ImageA.Enalapril
Incorrect Answer ImageB.Hydrochlorothiazide
Incorrect Answer ImageC.Hydralazine
Incorrect Answer ImageD.Metoprolol
Incorrect Answer ImageE.No added drugs needed

A 39-year-old man comes to the physician because of occasional blood appearing in his stool. He
has noticed this occurring approximately 1 to 2 times per month for the past several months.
There is no history of abdominal pain or difficulties with urination. He denies diarrhea,
constipation, rectal pain, or any other abnormal bowel habits. His temperature is 37ºC (98.6ºF),
blood pressure is 115/75 mm Hg, pulse is 78/min, and respirations are 13/min. Head,
cardiovascular, pulmonary, abdominal, and peripheral examinations are normal. On digital rectal
examination, there is normal sphincter tone, without internal or external hemorrhoids. Which of
the following is the most appropriate next step in management?
Incorrect Answer ImageA.Anoscopy
Correct Answer ImageB.Anoscopy and flexible sigmoidoscopy
Incorrect Answer ImageC.Barium enema
Incorrect Answer ImageD.Colonoscopy
Incorrect Answer ImageE.No further workup is necessary

A 65-year-old man comes to the emergency department because of nasal discharge, ringing in
his ears, and a headache for 4 days. The headache is worse when the patient sits up or stands up.
The fluid has been "oozing" out of his nose. He has had no fever, chills, cough, or sneezing. He
has a history of hypertension and gastroesophageal reflux disease. Current medications include
hydrochlorothiazide and omeprazole. His father had asthma and allergic rhinitis and his brother
has asthma and eczema. He is in no distress. His temperature is 36.8ºC (98.3ºF), blood pressure
is 138/76 mm Hg, pulse is 78/min, and respirations are 15/min. Examination shows a clear, thin
discharge draining from both nares. Neurologic examination shows no abnormalities. Which of
the following is the most appropriate next step in management? 
Correct Answer ImageA.Beta-2-transferrin in nasal fluid
Incorrect Answer ImageB.High-resolution facial CT scan
Incorrect Answer ImageC.Positive contrast myelographic CT cisternography
Incorrect Answer ImageD.Radionucleotide cisternography
Incorrect Answer ImageE.Start loratidine

A 56-year-old man is brought to the emergency department 10 minutes after losing


consciousness in a grocery store. He responds only to pain. His temperature is 36.9ºC (98.4ºF),
blood pressure is 110/70 mm Hg, pulse is 116/min, respirations are 25/min, and oxygen
saturation is 86% on 100% oxygen via venturi mask. The pupils are equal and reactive to light.
Cardiac examination shows normal S1 and S2 without murmurs. The abdomen is soft and
nontender. Arterial blood gases shows pH 7.58, pCO2 22 mm Hg, and pO2 48 mm Hg. Which of
the following is the most likely diagnosis? 
Incorrect Answer ImageA.Acute hyperventilation
Incorrect Answer ImageB.Aspirin overdose
Incorrect Answer ImageC.Chronic obstructive lung disease exacerbation due to bronchitis
Correct Answer ImageD.Pulmonary embolism
Incorrect Answer ImageE.Renal failure

A 42-year-old man comes to the physician because of a 2-week history of severe, intermittent
headache located behind the left eye and radiating to the neck. The pain is always on the left. The
headaches usually occur in the evening, have woken him from sleep, and last between 20
minutes and 2 hours. His headaches are accompanied by restlessness, rhinorrhea, and ptosis. He
had the "worst headache of his life" 4 days ago. His medical history is unremarkable. He works
as a banker and he has a lot of stress at work. His temperature is 37.3ºC (99.2ºF), blood pressure
is 145/90 mm Hg, pulse is 85/min, and respirations are 14/min. Physical examination, including
neurologic examination, shows no abnormalities. Which of the following is the most likely
diagnosis?
Incorrect Answer ImageA.Brain tumor
Correct Answer ImageB.Cluster headache
Incorrect Answer ImageC.Migraine
Incorrect Answer ImageD.Subarachnoid hemorrhage
Incorrect Answer ImageE.Tension-type headache

A 37-year-old woman comes to the physician because of a severely itchy rash on her legs for 2
days. Four days ago, she did yard work in her backyard. She has a history of hypertension
controlled with diet and exercise, and she has asthma. Current medications include an albuterol
inhaler. Her temperature is 37.2°C (99°F) and blood pressure is 138/80 mm Hg. Examination
shows multiple discrete, linear, erythematous plaques with overlying vesicles on her left leg.
Which of the following is the most likely cause of this patient's condition?
Incorrect Answer ImageA.Antibody-dependent cell-mediated cytotoxicity
Incorrect Answer ImageB.Antibody-dependent complement activation
Correct Answer ImageC.Cell-mediated delayed hypersensitivity
Incorrect Answer ImageD.Crosslinking of cell surface IgE by antigen
Incorrect Answer ImageE.Fixation of complement by immune complexes
A 25-year-old man is brought to the emergency department 20 minutes after collapsing while
playing tennis on a hot summer day. He has had intermittent weakness and dizziness when
arising from supine position for approximately 1 year. On physical examination, he is alert,
oriented, and restless. His temperature is 36.6ºC (97.8ºF); blood pressure is 90/60 mm Hg when
supine and 65/40 mm Hg when sitting; and pulse is 100/min when supine and 140/min when
sitting. Oxygen saturation is 97% on room air. His skin appears diffusely suntanned and the
mucous membranes are dry. The heart and lungs are normal. Abdomen shows diffuse tenderness
to palpation; there is no rebound, guarding, or hepatosplenomegaly. There are weak pulses. An
ECG shows only sinus tachycardia. Blood and serum studies show: 
Leukocyte count 5,400/mm3
Hematocrit 47%
Platelet count 298,000/mm3
Sodium 125 mEq/L
Potassium 5.9 mEq/L
Chloride 96 mEq/L
Bicarbonate 18 mEq/L
Urea nitrogen (BUN) 22 mg/dL
Creatinine 0.8 mg/dL
Glucose 66 mg/dL
In addition to administration of intravenous normal saline, which of the following is the most
appropriate therapy? 
Incorrect Answer ImageA.Fludrocortisone
Incorrect Answer ImageB.Glucagon
Correct Answer ImageC.Hydrocortisone, prednisone, and fludrocortisone
Incorrect Answer ImageD.Ice packs under the axillae, groin, and neck
Incorrect Answer ImageE.Insulin plus glucose
Incorrect Answer ImageF.Prednisone

A 70-year-old man is brought to the emergency department because of worsening dyspnea, leg
swelling, and weight gain. He has a history of myocardial infarction 2 years ago. He is currently
taking lisinopril, furosemide, warfarin, and digoxin. Physical examination shows jugular venous
pressure of 14 cm, bibasilar rales, diffuse pulmonary inspiratory crackles, displaced apical
impulse, and a third heart sound. An ECG shows an old left bundle branch block and atrial
fibrillation. Treatment with oxygen, furosemide, and nitrate is initiated. Which of the following
is the most appropriate next step after stabilizing the patient?
Incorrect Answer ImageA.Brain natriuretic peptide (BNP)
Correct Answer ImageB.Echocardiogram
Incorrect Answer ImageC.High-resolution CT scan of the chest
Incorrect Answer ImageD.Pulmonary artery catheter placement
Incorrect Answer ImageE.Ventilation-perfusion scan

A college student visits the campus clinic in April because of bilateral itching, watery, red eyes
for 2 days. The student states that she has a history of intermittent asthma, and has recently
needed to use her albuterol metered-dose inhaler for a mild flare-up. She also reports having
similar symptoms twice in the past month, which resolved in a couple of days after she used
some over-the-counter eye drops. Examination shows bilateral chemosis. On eversion of the
upper lid, the tarsal conjunctiva shows large cobblestone papillae. She has mild photophobia but
visual acuity is 20/20 in both eyes. Which of the following is the most appropriate initial
therapy?
Incorrect Answer ImageA.Cromolyn sodium 4% topically
Incorrect Answer ImageB.Erythromycin topically
Incorrect Answer ImageC.Fexofenadine orally
Correct Answer ImageD.Ketotifen 0.025% topically
Incorrect Answer ImageE.Prednisone orally
Incorrect Answer ImageF.Reassurance only

A 55-year-old man comes to the clinic for a routine physical examination. He has not seen a
doctor in years and states that had it not been for a case of "tennis elbow" some years back, he
probably would not have seen one at all. Aside from a baby aspirin, he does not take any
medications. He has never smoked or used illicit drugs, has at most a glass of wine a day, and
exercises regularly. A review of systems is unremarkable for any cardiac symptoms. His family
history includes a mother with a stroke, a brother with a myocardial infarction at age 50, and a
father who died of a myocardial infarction at age 58. Vital signs are temperature 37.0ºC (98.6ºF),
pulse 88/min, respirations 12/min, and blood pressure 142/78 mm Hg. Physical examination is
unremarkable. Which of the following is the most appropriate screening test for this patient?
Incorrect Answer ImageA.Carotid-artery duplex study
Incorrect Answer ImageB.Electrocardiogram (ECG)
Correct Answer ImageC.Serum lipid studies
Incorrect Answer ImageD.Stress echocardiogram
Incorrect Answer ImageE.Stress electrocardiogram

A 45-year-old man comes to the office complaining of vague abdominal pain and fatigue for the
past two months. He had been previously well until he started feeling fatigued most of the time,
even when he was not exercising. His appetite has decreased and he thinks he has lost about 13
pounds during this period. A review of systems is positive for intermittent arthralgias. His
temperature is 38.3ºC (100.9ºF), pulse is 92/min, respirations are 16/min, and blood pressure is
120/80 mm Hg. Heart examination shows a regular rate and rhythm with no murmurs. Lungs are
clear to auscultation bilaterally. The abdomen is mildly tender and the spleen is palpable 3 cm
below the left costal margin. Neurological examination is unremarkable. Laboratory data show:
Hemoglobin  17 g/dL
Platelets  450,000/mm3
White blood cells  130,000/mm3
Segmented neutrophils 76%
Bands 10%
Eosinophils 4%
Basophils 5%
Lymphocytes 18%
Monocytes 2%
Peripheral blood blasts 3%
Leukocyte alkaline low
phosphatase
Chromosomal analysis translocation of long arm between
chromosomes 22 and 9 
Which of the following is the most likely diagnosis?
Incorrect Answer ImageA.Acute myelogenous leukemia
Incorrect Answer ImageB.Chronic lymphocytic leukemia
Correct Answer ImageC.Chronic myelogenous leukemia
Incorrect Answer ImageD.Leukemoid reaction
Incorrect Answer ImageE.Myelodysplastic syndrome

A 40-year-old woman with a 30-year history of asthma comes to the physician for a routine
health maintenance examination in December. She has no shortness of breath, wheeze, or cough.
She has no sick contacts. Current medications include daily inhaled fluticasone. She has used her
albuterol inhaler on 2 occasions within the past 3 months. Her current peak expiratory flow meter
readings are 85% of her personal best. Her temperature is 36.9°C (98.4°F). Which of the
following is the most appropriate immunization to administer at this visit?
Incorrect Answer ImageA.Inactivated influenza vaccine
Correct Answer ImageB.Inactivated influenza vaccine and pneumococcal polysaccharide vaccine
Incorrect Answer ImageC.Live-attenuated influenza vaccine
Incorrect Answer ImageD.Pneumococcal polysaccharide vaccine
Incorrect Answer ImageE.Zoster vaccine

A 56-year-old man comes to the clinic because of a worsening cough. Over the last 3 months he
has developed a hacking cough and occasionally brings up brown- or red-tinged sputum. In
addition to his cough, he reports progressive fatigue, an unexpected 20-lb weight loss, and
occasional night sweats. When asked specifically, the man admits to a 90-pack-year tobacco
history. Physical examination shows a thin, ill-appearing man. He appears frankly cachectic, and
has crackles and dullness to percussion over the right lower lung zone. His fingernails are
enlarged and rounded. A chest x-ray shows a large mass with ill-defined margins confined to the
right lower lobe, without evidence of effusion or other abnormalities. The likelihood of cancer is
explained to the patient, together with his options for further diagnostic workup and treatment.
The man refuses surgery and prefers to have chemotherapy. For which of the following types of
lung cancer would the patient’s desired therapy be most appropriate? 
Incorrect Answer ImageA.Adenocarcinoma
Incorrect Answer ImageB.Large-cell lung cancer
Incorrect Answer ImageC.Neuroendocrine lung tumor
Correct Answer ImageD.Small-cell lung cancer
Incorrect Answer ImageE.Squamous cell lung cancer

A 54-year-old woman comes to the physician for a routine health maintenance examination. She
has a history of hypertension and hypercholesterolemia. Her family history is positive for type 2
diabetes mellitus. She denies having increased urination, increased thirst, changes in appetite, or
any change in vision. She experienced menopause 1 year ago. She smoked 1 pack of cigarettes
daily for 32 years but quit 2 months ago. She drinks 2 glasses of wine on the weekends. Current
medications include aspirin, simvastatin, and lisinopril. She weighs 83.9 kg (185 lbs) and is 160
cm (63 in) tall. Her blood pressure is 122/82 mm Hg. At her last routine visit 1 year ago, her
fasting serum glucose levels were found to be 112 and 114 mg/dL, respectively. She was advised
to make dietary changes which she has implemented stringently. She also exercises regularly by
walking briskly for 30 minutes 4 to 5 times per week. Two recent fasting serum glucose levels
are 115 and 121 mg/dL, respectively. Her hemoglobin A1c level is 6.2%. Which of the following
is the most appropriate next step in management?
Incorrect Answer ImageA.Add hydrochlorothiazide
Incorrect Answer ImageB.No further treatment; follow clinically
Incorrect Answer ImageC.Start hormone replacement therapy
Incorrect Answer ImageD.Start insulin therapy
Correct Answer ImageE.Start metformin

A 40-year-old man is brought to the emergency department unresponsive. He is a known


alcoholic and was found lying in his apartment. On examination, he is unresponsive and
dehydrated. Laboratory studies show:
RBC count 4.5 million/mm3
Hemoglobin 12 mg/dL
Hematocrit 36%
WBC count 4,500/mm3
Potassium 5.8 mg/dL
Phosphate 2.2 mg/dL
Calcium 7.6 mg/dL
Albumin 5.0 g/dL
BUN 50 mg/dL
Creatinine 3.5 mg/dL
Creatine kinase 15,000 U/L
Lactate dehydrogenase (LDH) 489 U/L
Urinalysis (dipstick)
Specific gravity 1.020
Blood +++
Protein +
Urinalysis (microscopic)
RBC 0–2/hpf
WBC 0–5/hpf
Casts Pigmented casts present
Which of the following is the most likely diagnosis? 
Incorrect Answer ImageA.Acute glomerulonephritis
Incorrect Answer ImageB.Allergic interstitial nephritis
Incorrect Answer ImageC.Intravascular hemolysis
Incorrect Answer ImageD.Prerenal acute kidney injury
Correct Answer ImageE.Rhabdomyolysis

A 45-year-old man comes to the physician with his partner because of shooting pains and
tingling in his thumbs, pointer finger, and middle finger that have been worse at night for
approximately 4 years. He has a history of headaches. His partner reports that his face has
changed. He has had a 9-kg (20-lb) weight gain. He has a history of hypertension controlled with
diet. His temperature is 37ºC (98.6ºF), blood pressure is 165/90 mm Hg, pulse is 73/min, and
respirations are 13/min. Examination is shown. There is bilateral weakness on abduction of the
thumbs. The size of the hands and feet appear out of proportion with the patient's body size.
Which of the following is the most important diagnostic test in this patient?
Incorrect Answer ImageA.Cortisol level
Incorrect Answer ImageB.CT scan of brain
Incorrect Answer ImageC.Fasting glucose level
Incorrect Answer ImageD.Growth hormone levels
Correct Answer ImageE.Insulin-like growth factor

A 32-year-old man comes to the physician because of an itchy rash on his thighs for 4 weeks. He
reports that the itchiness subsides overnight. His medical history is unremarkable, and he takes
no medications. He has no known allergies to medications. He reports that he cannot wear a
metal watch because he develops a rash on his wrist. Skin examination shows poorly
circumscribed erythematous, scaly, and crusted plaques with multiple excoriations on both upper
thighs. There are tiny, translucent vesicles within the plaques. As the patient finishes dressing, he
places his car keys into one trouser pocket and some loose change into the other. Which of the
following is the most likely underlying mechanism for this patient's condition? 
Incorrect Answer ImageA.Cytotoxic hypersensitivity
Correct Answer ImageB.Delayed hypersensitivity
Incorrect Answer ImageC.Immediate hypersensitivity
Incorrect Answer ImageD.Immune complex-mediated hypersensitivity
Incorrect Answer ImageE.Trauma-induced

A 19-year-old college student comes to the physician because of a 2-week history of jaundice
that began during an upper respiratory tract infection. He denies abdominal pain, fever, chills,
diarrhea, constipation, or pruritus. He has no known sick contacts. He consumes 3 to 4 beers on
the weekends and denies using illicit drugs. He has had episodes of jaundice associated with
infections in the past. His temperature is 36.7°C (98°F). Examination shows scleral icterus. The
remainder of the physical examination shows no abnormalities. Laboratory studies show:
Protein 6.5 mg/dL
Albumin 3.9 mg/dL
Bilirubin total 3.6 mg/dL
Bilirubin direct 2.9 mg/dL
Alkaline phosphatase 30 U/L
Serum studies show:
Aspartate aminotransferase (AST, GOT) 10 U/L
Alanine aminotransferase (ALT, GPT) 23 U/L
Which of the following is the most likely diagnosis? 
Incorrect Answer ImageA.Crigler-Najjar syndrome
Correct Answer ImageB.Dubin-Johnson syndrome
Incorrect Answer ImageC.Gilbert disease
Incorrect Answer ImageD.Primary biliary cirrhosis
Incorrect Answer ImageE.Viral hepatitis

A 28-year-old man who has chronic glomerulonephritis was seen in the clinic for a checkup 1
month ago. At that time he had a creatinine level of 6.7 mg/dL, urea nitrogen of 70 mg/dL, and
hemoglobin of 8.4 g/dL. His platelet count was 200,000/mm3. After negative testing for iron
deficiency or other causes of anemia, he was diagnosed with anemia of chronic renal disease. He
was started on epoetin alfa (recombinant human erythropoietin) injections and his hemoglobin
increased to 11.4 g/dL after 3 weeks of therapy. Shortly thereafter, the hemoglobin levels started
to decline. At present, he feels well except for some mild fatigue with exercise. His temperature
is 38.0ºC (100.4ºF) and blood pressure is 140/80 mm Hg. His lungs are clear to auscultation. His
heart is regular in rhythm with a soft systolic murmur. His abdomen is soft. His stool is guaiac-
negative. His current hemoglobin level is 8 g/dL, serum ferritin is 200 ng/mL, and iron saturation
is 25%. His mean corpuscular volume is 85 µm3. Which of the following would be the most
appropriate treatment for his anemia at this point?
Correct Answer ImageA.Assess and treat the source of fever
Incorrect Answer ImageB.Blood transfusion
Incorrect Answer ImageC.Folic acid
Incorrect Answer ImageD.Intravenous iron
Incorrect Answer ImageE.Vitamin C

A 29-year-old man comes to the physician because of a rash that developed 2 weeks earlier in
the form of numerous red, scaly lesions on his stomach, chest, palms, and soles. He works on an
ocean freighter and spends 6 months at a time at sea. He thinks that the close contact with
seawater could have caused this problem. When asked about his sexual activity, he endorses
unprotected sex about 2 months ago. There are no other medical problems in his history, and he
only takes an over the counter medication for allergies in the spring. On physical examination, he
is a well-developed and fit man in no acute distress. Vital signs are normal. There are
symmetrical, diffuse, discrete, 2 to 3 mm, erythematous, scaly papules and firm, deeply seated,
salmon-colored, tender papules present on the palms and soles. Which of the following is the
most appropriate pharmacotherapy? 
Incorrect Answer ImageA.Acyclovir orally for 7–10 days
Incorrect Answer ImageB.Azithromycin orally in a single dose
Incorrect Answer ImageC.Benzathine penicillin G intramuscularly in 3 doses with a 1-week interval
Correct Answer ImageD.Benzathine penicillin G, single dose
Incorrect Answer ImageE.Ceftriaxone intramuscularly, single dose
Incorrect Answer ImageF.Penicillin G IV for 10–14 days
Incorrect Answer ImageG.Prednisone orally for 2–3 weeks

A 60-year-old woman with a 30-year history of type 2 diabetes mellitus comes to the physician
because of a swollen foot. She has had swelling, redness, and mild pain in her right foot for 3
months which has made it difficult for her to walk. She has a 12-year history of hypertension.
Current medications include metformin, insulin, lisinopril, and hydrochlorothiazide. She does not
smoke, drink alcohol, or use recreational drugs. Her temperature is 36.8ºC (98.3ºF), pulse is
86/min, and oxygen saturation is 94% on room air. Bilateral basilar crackles and an S4 are heard
on auscultation. There are bilateral carotid bruits. The right foot is edematous, indurated, and
warm to palpation. There is mild pain with manipulation of the foot and decreased sensation to
pinprick and vibratory stimuli bilaterally below the level of the knee. Hemoglobin A1c level is
9.1%. Her leukocyte count is 5,500/mm3. Which of the following is the most likely diagnosis?
Incorrect Answer ImageA.Calcium pyrophosphate deposition disease
Incorrect Answer ImageB.Cellulitis
Correct Answer ImageC.Charcot joint
Incorrect Answer ImageD.Gout
Incorrect Answer ImageE.Septic arthritis

A 70-year-old man who has not seen a physician in the past 30 years comes to the emergency
room with a one month history of fatigue and muscle weakness. He was was told he had
hypertension >20 years ago, but has never taken medication for it. His blood pressure is 188/106
mm Hg and pulse is 80/minute. The physical examination shows AV nicking on fundoscopy, but
is otherwise normal. Laboratory studies show: 
Sodium 140 mEq/L
Potassium 5.8 mEq/L
Chloride 110 mEq/L
HCO3 20 mEq/L
Creatinine 3.0 mg/dL
Glucose 397 mg/dL
The urinalysis shows 2+ protein and is otherwise normal. The electrocardiogram is normal.
Which of the following is the most appropriate initial step in management? 
Incorrect Answer ImageA.Albuterol (nebulized)
Correct Answer ImageB.Insulin
Incorrect Answer ImageC.Insulin with dextrose
Incorrect Answer ImageD.IV calcium chloride
Incorrect Answer ImageE.Kayexalate
Incorrect Answer ImageF.Sodium bicarbonate

A 52-year-old man comes to the physician because of pruritic skin lesions on his arms and legs
for 6 months. He has a history of hypothyroidism, type 1 diabetes, severe hyperlipidemia,
coronary artery disease, and chronic renal insufficiency. He had been on renal dialysis for the
past 2 years before undergoing a kidney transplant 3 weeks ago. Current medications include
cyclosporine, prednisone, metformin, aspirin, and diltiazem. Physical examination of the skin
shows numerous dome-shaped papules on his legs and arms. Some lesions have a central
keratotic scale and others are excoriated with a central pustule. Which of the following
conditions is most likely associated with the appearance of these skin lesions?
Incorrect Answer ImageA.Coronary artery disease
Incorrect Answer ImageB.Cyclosporine use
Correct Answer ImageC.End-stage renal disease
Incorrect Answer ImageD.Hyperlipidemia
Incorrect Answer ImageE.Hypothyroidism

A 72-year-old woman is brought to the emergency department by her daughter, who is


concerned that her mother had a "stroke." According to the daughter, the woman woke up that
morning not knowing where she was. She was confused and did not remember what had
happened the day before. Her history is significant for hypertension, type 2 diabetes mellitus, and
an anxiety disorder. Her medications include atenolol, hydrochlorothiazide, metformin, and
alprazolam, and her daughter reports that the patient is compliant with her medications. On
physical examination, her temperature is 37.1ºC (98.8ºF), pulse is 86/min, respirations are
14/min, and blood pressure is 128/76 mm Hg. Neurologic examination is unremarkable. ECG
and arterial blood gases are normal. Which of the following is the most appropriate next step in
management?
Incorrect Answer ImageA.Carotid duplex scanning
Incorrect Answer ImageB.Discontinue atenolol
Incorrect Answer ImageC.Echocardiogram
Incorrect Answer ImageD.Order troponins
Correct Answer ImageE.Reduce alprazolam

A 38-year-old man is brought to the emergency department because of bizarre behavior. He is


combative and refuses to answer questions. His temperature is 37.0ºC (98.6ºF), pulse is 123/min,
respirations are 28/min, and blood pressure is 100/60 mm Hg. His breath smells of alcohol.
Serum studies show: 
Blood chemistry:
pH 7.3
PCO2 30 mm Hg
PO2 95 mm Hg
Na +
138 mEq/L
K+ 4 mEq/L
Cl− 103 mEq/L
HCO3 −
14 mEq/L
Creatinine  1.5 mg/dL
Blood urea nitrogen 25 mg/dL
Urinalysis shows envelope-shaped crystals. Ingestion of which of the following is the most likely
explanation for these findings? 
Incorrect Answer ImageA.Ethanol
Correct Answer ImageB.Ethylene glycol
Incorrect Answer ImageC.Fomepizole
Incorrect Answer ImageD.Isopropyl alcohol
Incorrect Answer ImageE.Methanol

A 19-year-old African American man comes to the clinic because of abdominal distention,
flatulence, and diarrhea over the last 3 hours. He states that he has been having more of these
episodes over the last 6 months, but this particular instance was so bothersome that he decided to
seek medical evaluation. He reports that his diarrhea is watery with no evidence of blood and is
associated with increased bowel sounds during the past few months after eating certain foods. He
denies exposure to any sick contacts. Today, he consumed spaghetti with Alfredo sauce, a large
ice-cream sundae, and three glasses of milk. His vital signs are normal. His physical examination
is significant only for borborygmi during auscultation of the abdomen. Which of the following
would be the most appropriate pharmacotherapy to prevent future discomfort?
Incorrect Answer ImageA.Antibiotics
Incorrect Answer ImageB.Antimotility agents (e.g., loperamide)
Incorrect Answer ImageC.Antisecretory agents (e.g., bismuth subsalicylate)
Correct Answer ImageD.Enzyme replacement
Incorrect Answer ImageE.Opiate derivatives
A 63-year-old man with a history of non-insulin-dependent diabetes and hypertension comes to
the physician after being awakened from sleep by severe pain in his right first toe. He reports the
sudden onset of acute pain in the toe, rapidly followed by reddening of the skin, swelling,
tenderness, and warmth. His temperature is 37.0ºC (98.6ºF), blood pressure is 170/60 mm Hg,
pulse is 97/min, and respirations are 19/min. Physical examination is normal except for swelling
and severe tenderness over his metatarsophalangeal joint on the right foot. Which of the
following is the most appropriate first-line treatment?
Incorrect Answer ImageA.Allopurinol
Incorrect Answer ImageB.Dietary modifications
Correct Answer ImageC.Indomethacin
Incorrect Answer ImageD.Prednisone
Incorrect Answer ImageE.Probenecid

A 42-year-old, right-handed man comes to the emergency department with a history of


progressive speech difficulties and right hemiparesis for the past 5 months. He also reports
progressively severe headaches for the past 2 months which are worse in the mornings. At the
time of admission, he appears confused and vomits twice. On physical examination, he has
blurred vision, papilledema, and diplopia. Shortly thereafter, his blood pressure increases to
190/110 mm Hg, respirations decrease to 8/min, and he develops bradycardia. Which of the
following best explains the cause of his sudden hypertension, bradypnea, and bradycardia?
Incorrect Answer ImageA.Aortic dissection
Incorrect Answer ImageB.Chronic subdural hematoma
Incorrect Answer ImageC.Hypothalamic compression
Correct Answer ImageD.Increased intracranial pressure
Incorrect Answer ImageE.Transtentorial herniation

A 68-year-old man comes to the physician because of a slowly growing lesion on the side of his
forehead that he has had for approximately 10 years. It is not painful, it does not itch, and it does
not spontaneously bleed. He has had no fevers, night sweats, or weight loss. His medical history
is unremarkable. He has played golf every day since retiring 3 years ago. Examination is shown.
There is an adherent greasy scale. The lesion is nontender. There is no lymphadenopathy. Which
of the following is the most likely diagnosis?
Incorrect Answer ImageA.Actinic keratosis
Incorrect Answer ImageB.Basal cell carcinoma
Incorrect Answer ImageC.Dermatofibroma
Incorrect Answer ImageD.Melanoma
Correct Answer ImageE.Seborrheic keratosis
Incorrect Answer ImageF.Squamous cell carcinoma

A 38-year-old woman is an usher at the local movie theater, has a BMI of 34 kg/m2, and has been
on orlistat for the past 6 months. She comes to the office because of difficulty driving in low
lights or foggy conditions. She reports multiple small accidents while working during the movie
function. Review of systems reveals dryness of the eyes and multiple ecchymoses. Past medical
history is positive for impaired glucose tolerance and hyperlipidemia. She states that she smokes
marijuana at least once a week. Vitals signs are unremarkable. Laboratory studies show:
Hb 10.5 g/dL
Hct 34%
Platelets 220,000/mm3
WBC 5,000/mm3
Neutrophils 58%
Eosinophils 1%
Lymphocytes 30%
PT 16 sec
PTT 37 sec
Visual examination is significant only for her difficulty visualizing letters or structures when the
lights are dimmed. Examination of the eye shows conjunctival dryness and the appearance of
small, white patches on the sclera. Corneal and retinal examination is without abnormalities.
Which of the following is the most likely cause of this patient’s symptoms? 
Correct Answer ImageA.Adverse drug effect
Incorrect Answer ImageB.Cataracts
Incorrect Answer ImageC.Diabetic retinopathy
Incorrect Answer ImageD.Retinitis pigmentosa
Incorrect Answer ImageE.Vitamin B12 deficiency
Incorrect Answer ImageF.Vitamin C deficiency

A 40-year-old woman comes to the physician because of multiple pigmented skin lesions.
Examination of the skin and scalp shows >100 round pigmented macules from 5–10 mm in
diameter. The lesions are most dense on sun-exposed skin. Several macules have notched
borders and are slightly asymmetric. These pigmented macules are unchanged from the previous
examination that was photographed 1 year ago. Which of the following is the most likely
diagnosis? 
Correct Answer ImageA.Atypical nevi
Incorrect Answer ImageB.Compound nevi
Incorrect Answer ImageC.Halo nevi
Incorrect Answer ImageD.Lentigines
Incorrect Answer ImageE.Melanomas

A 65-year-old man is admitted to the hospital for worsening headache, lightheadedness, blurry
vision, and fatigue over the last 3 months. He also reports that he is getting very weak and has
generalized itching, which usually occurs in hot weather. He does not appear distressed. His
temperature is 37°C (98.6°F), pulse is 98/min, and blood pressure is 137/80 mm Hg. He has
facial plethora. Abdominal examination shows the spleen edge 4 cm below the left costal margin.
Laboratory studies show:
Leukocyte count 18,000/mm3
Hemoglobin 20 mg/dL
Hematocrit 62%
Platelet count 550,000/mm3
Erythropoietin level Low
Leukocyte alkaline phosphatase Increased
Which of the following is the most likely diagnosis?
Incorrect Answer ImageA.Chronic myeloid leukemia
Incorrect Answer ImageB.Myelodysplasic syndromes
Incorrect Answer ImageC.Myelofibrosis
Correct Answer ImageD.Polycythemia vera
Incorrect Answer ImageE.Secondary erythrocytosis

A 53-year-old man who works as a financial analyst comes to the clinic because of shortness of
breath and cough for the past 2 weeks. He recalls previous episodes occurring whenever he goes
to places that have cold weather. He was seen by a physician for the problem years ago and was
prescribed an unrecalled medication for the episodes. The last episode was 30 years ago. He has
just transferred to this bank in Chicago from his hometown in Arizona. Each evening, he
develops a dry, nonproductive cough that typically lasts for several hours before he falls asleep.
He is awakened from sleep with shortness of breath, coughing, and wheezing. He denies
cigarette smoking, but drinks several beers on weekends and occasionally smokes marijuana. On
physical examination the lungs show (click media file) bilaterally. Which of the following is the
most likely mechanism for these symptoms? 
Correct Answer ImageA.Bronchospasm
Incorrect Answer ImageB.Laryngeal spasm
Incorrect Answer ImageC.Pleural effusion
Incorrect Answer ImageD.Pulmonary congestion
Incorrect Answer ImageE.Ventilation-perfusion mismatch

A 44-year-old man comes to the physician because of 2 days of worsening right-sided flank pain
and hematuria. He denies any fever, chills, or rigors. His past medical history is significant for
gouty arthritis. Vital signs are normal. The physical examination is significant for right flank
tenderness. Urinalysis shows 5 RBCs per high-power field and is otherwise unremarkable. A CT
scan of the abdomen and pelvis shows a 0.8-cm stone in the right mid-ureter with mild right
hydronephrosis. Laboratory studies show normal serum creatinine and elevated urate
concentrations. The urine pH is 5.2. After 2 days of hydration and analgesia, and administration
of allopurinol, potassium bicarbonate, and potassium citrate, the stone has not passed, and his
severe pain continues. Which of the following is the most appropriate next step in management?
Incorrect Answer ImageA.Addition of colchicine
Incorrect Answer ImageB.Addition of hydrochlorothiazide
Incorrect Answer ImageC.Addition of probenecid
Correct Answer ImageD.Lithotripsy
Incorrect Answer ImageE.Open surgical removal of the stone

A 35-year-old woman comes to the physician because of increasing weakness, loss of energy,
sadness, and shortness of breath for the last 2 months and is finding it increasingly difficult to
keep up with her children. She explains that she is just "tired of it all." She endorsees difficulty
sleeping, decreased appetite, and loss of interest in her hobbies. Her past medical history is
otherwise unremarkable. She takes oral contraceptive pills and ibuprofen for tension headaches.
Temperature is 37ºC (98.6ºF), pulse is 89/min, respirations are 12/min, and blood pressure is
120/72 mm Hg. Physical examination shows an inability to distinguish when the examiner flexes
her toes bilaterally. Chest is clear to auscultation and heart sounds are normal. Abdominal
examination is benign. Conjunctivae are pale. Laboratory studies show the following: 
Hemoglobin  9 g/dL
Hematocrit  27%
WBC  7,000/mm3
Platelets  200,000/mm3
TSH  10 µU/mL
Which of the following is the most likely diagnosis? 
Correct Answer ImageA.Autoimmune gastritis
Incorrect Answer ImageB.Dietary vitamin B12 deficiency
Incorrect Answer ImageC.Iron deficiency anemia
Incorrect Answer ImageD.Major depression
Incorrect Answer ImageE.Multiple sclerosis

A 22-year-old woman comes to the physician because of a 2-year history of spots on her face
and fingers that began as small, white-colored areas. They slowly enlarged and have not returned
to her normal skin color. Her medical history is unremarkable. She works as a civil engineer. Her
mother has hypothyroidism and her father has hyperlipidemia. Her temperature is 37°C (98.6°F).
Her physical examination is shown. There are also several depigmented patches surrounding the
patient's eyes and mouth. Which of the following is the most likely cause of this patient's
condition?
Correct Answer ImageA.Autoimmune destruction of melanocytes
Incorrect Answer ImageB.Damage to melanocytes by dicarboxylic acid
Incorrect Answer ImageC.Defective production of melanin from tyrosine
Incorrect Answer ImageD.Exposure to phenols
Incorrect Answer ImageE.Infection with acid-fast bacillus

A 56-year-old woman comes to the physician because of a two-week history of abdominal


distension and vague abdominal pain. She denies dyspnea, lower extremity swelling, weight loss,
fevers, or night sweats. Her past medical history is unremarkable. There is no history of illicit
drug use or smoking. She consumes alcohol approximately once or twice monthly. She is
afebrile, normotensive and has normal oxygen saturation on room air. There is no cervical
lymphadenopathy or jugular venous distension. Pulmonary examination shows decreased sounds
at the lung bases and cardiac examination shows no abnormalities. Her abdomen shows a
significant amount of ascites as demonstrated by a positive fluid wave and shifting dullness.
There was no clubbing, cyanosis, or edema. Laboratory studies, including paracentesis results,
show:
Ascitic fluid
Albumin 2.8 g/dL
White blood cells 100/mm3, 50% neutrophils 
Serum
White blood cells 5,700/mm3
Hematocrit 37%
Platelets 287,000/mm3
AST 29 U/L
ALT 31 U/L
Albumin 3.7 g/d
Which of the following is the most appropriate next step in management? 
Incorrect Answer ImageA.Administration of ceftriaxone for 5 days
Incorrect Answer ImageB.Echocardiography
Incorrect Answer ImageC.Hepatic venography
Incorrect Answer ImageD.Liver biopsy
Correct Answer ImageE.Pelvic ultrasonography

A 57-year-old man comes to the emergency department because of a 2-day history of blurry
vision, diplopia, and severe headache. He has a history of osteoarthritis treated with ibuprofen.
His temperature is 36.8°C (98.2°F), blood pressure is 130/84 mm Hg, and pulse is 78/min. There
is ptosis of the right eye, and the right eye is divergent and slightly depressed. Extraocular
movements are limited superiorly, medially, and inferiorly. There is inward rotation on
depression of the right eye. The pupil is dilated and does not react to accommodation or light
(anisocoria). Which of the following is the most likely diagnosis?
Incorrect Answer ImageA.Fourth-nerve paralysis from a congenital lesion
Incorrect Answer ImageB.Sixth-nerve palsy from an intraorbital mass
Incorrect Answer ImageC.Third- and fourth-nerve palsies from a compressive lesion
Correct Answer ImageD.Third-nerve palsy from a posterior communicating artery aneurysm
Incorrect Answer ImageE.Third-nerve palsy from ischemia

A 67-year-old man is admitted to the hospital because of a 3-week history of fevers, night
sweats, shortness of breath, and cough productive of yellow-green sputum. He drinks six glasses
of whisky daily. He has smoked 2 packs of cigarettes daily for 32 years. His temperature is
37.4ºC (99.3ºF), pulse is 104/min, respirations are 22/min, blood pressure is 135/75 mm Hg, and
oxygen saturation is 85% on room air. Decreased breath sounds bilaterally and crackles in the
left upper lobe are heard on auscultation. Laboratory studies show:
Hemoglobin  14.8 g/dL 
Leukocyte count  14,300/mm3
Platelet count  256,000/mm3
Arterial blood:
pH  7.36 
PCO2 49 mm Hg 
PO2 51 mm Hg 
HCO3 -
24 mm Hg 
An ECG shows no abnormalities. A radiograph and CT scan of the chest are shown. Which of
the following is the most likely diagnosis?
Incorrect Answer ImageA.Acute respiratory distress syndrome (ARDS)
Incorrect Answer ImageB.Chronic obstructive pulmonary disease (COPD)
Incorrect Answer ImageC.Lobar pneumonia
Incorrect Answer ImageD.Pneumocystis jiroveci pneumonia
Incorrect Answer ImageE.Pulmonary edema
Correct Answer ImageF.Tuberculosis

A 50-year-old man comes to his primary care physician because he has been having transient
periods of rapid heartbeat accompanied by sweating, flushing, and a sense of impending doom.
Physical examination shows no abnormalities. There is no evidence of arrhythmia at the time of
the exam. The man's wife is a nurse so the physician asks her to take her husband's blood
pressure and pulse the next time an episode occurs. During an episode his blood pressure was
found to be 195/140 mm Hg with a pulse of 160/min. She promptly takes her husband to the
emergency room, but the spell is over by the time he is seen. Which of the following is the most
sensitive test for diagnosing this man's condition?
Incorrect Answer ImageA.Abdominal computerized tomography scan
Correct Answer ImageB.Plasma fractionated free metanephrines
Incorrect Answer ImageC.Serum free 5-hydroxyindoleacetic acid
Incorrect Answer ImageD.Urinary free metanephrines
Incorrect Answer ImageE.Urinary vanillylmandelic acid

A 56-year-old woman who recently immigrated from Mexico comes to a community outreach
health center complaining of a hacking cough associated with fever and night sweats that has
persisted over the last 3 months. There is heightened concern that the patient has active drug-
resistant pulmonary tuberculosis (TB) because there has been a recent outbreak of isoniazid-
resistant TB in her community. She is admitted to the hospital and placed in isolation. Chest
radiograph shows a 3 cm, right upper lobe cavitating lesion. Over the next few days, 3 sputum
smears show acid-fast staining organisms consistent with Mycobacterium species. Drug
sensitivity test results are pending. Which of the following is the most appropriate management?
Incorrect Answer ImageA.Delay starting therapy until culture and sensitivity data are available
Incorrect Answer ImageB.Place purified protein derivative (PPD)
Incorrect Answer ImageC.Report the case to the Public Health Department to arrange for isolation
and deportation
Incorrect Answer ImageD.Start isoniazid, rifampin, pyrazinamide, and ethambutol until sensitivities
are available
Correct Answer ImageE.Start pyrazinamide, kanamycin, levofloxacin, ethionamide, cycloserine, and
pyridoxine

A 24-year-old graduate student comes to the physician 3 months prior to a 1-year trip to Brazil.
His medical history is unremarkable. He takes no medications and has no known medication
allergies. He drinks 2 to 4 beers on the weekends. He is sexually active and uses condoms
consistently. Examination shows no abnormalities. Which of the following is the most
appropriate malarial prophylaxis?
Correct Answer ImageA.Atovaquone-proguanil
Incorrect Answer ImageB.Chloroquine
Incorrect Answer ImageC.Ciprofloxacin
Incorrect Answer ImageD.Isoniazid
Incorrect Answer ImageE.Trimethoprim-sulfamethoxazole

A 62-year-old man who has a history of hypertension, type 2 diabetes mellitus, and
hypercholesterolemia comes in for his annual physical examination. He reports fatigue for the
last few months but blames it on "old age." He also has been having back pain for the last year or
so. The pain is mainly in his lower back and has been increasing in intensity for the last few
weeks. He says that his blood pressure and blood sugar have both been under good control. He is
on lisinopril, metformin, and atorvastatin. Physical examination shows a temperature of 36.6ºC
(98.0ºF), blood pressure 140/78 mm Hg, pulse 80/min, and respirations 12/min. HEENT
examination shows pallor and no icterus. Lungs are clear to auscultation. Cardiovascular
examination shows normal S1 and S2, no S3 or S4, and no rubs or gallops. Neurologic
examination is also within normal limits. Laboratory workup shows: 
Hb 10.9 g/dL
WBC 6,000/mm3
Platelets 90,000/mm3
MCV, MCH, and MCHC All within normal limits
Sodium 132 mEq/L
Potassium 4.7 mEq/L
Chloride 101 mEq/L
Bicarbonate 27 mEq/L
BUN 80 mg/dL
Creatinine 2.5 mg/dL
Glucose 80 mg/dL
Calcium 13.2 mg/dL
Albumin 2.4 g/dL
Globulin 4.0 g/dL
LFTs Within normal limits
Which of the following will establish the suspected diagnosis? 
Incorrect Answer ImageA.Biopsy of a peripheral nerve
Correct Answer ImageB.Bone-marrow biopsy
Incorrect Answer ImageC.CSF fluid examination
Incorrect Answer ImageD.CT scan of the abdomen
Incorrect Answer ImageE.CT scan of the head
Incorrect Answer ImageF.Liver biopsy

A 51-year-old woman comes to the physician because of progressive weakness and fatigue for 4
months. She has had difficulty rising from her armchair but denies fever or blood in her stools.
She had a screening colonoscopy 1 year ago that showed no abnormalities. Her medical history
is unremarkable and she takes no medications. Her temperature is 36.8°C (98.2°F), blood
pressure is 128/78 mm Hg, and pulse is 82/min. Muscle strength is 3/5 in the shoulder and pelvic
girdles with normal reflexes. Skin examination shows no abnormalities. Serum creatine kinase
activity is 18,600 mU/mL. Anti-histidyl-tRNA synthetase (Jo-1) antibody assay is positive. A
biopsy specimen of skeletal muscle shows inflammation with CD8+ T lymphocytes without
amyloid deposition. Which of the following is the most likely diagnosis? 
Incorrect Answer ImageA.Dermatomyositis
Incorrect Answer ImageB.Dermatomyositis sine myositis
Incorrect Answer ImageC.Inclusion body myositis
Incorrect Answer ImageD.Mitochondrial myopathy
Correct Answer ImageE.Polymyositis

A 31-year-old African-American man comes to the physician because of generalized weakness


and several episodes of headache over the past 8 months. He has had occasional mild chest pain
and shortness of breath with exercise. His temperature is 37.0ºC (98.6ºF), pulse is 90/min,
respirations are 18/min, and blood pressure is 150/90 mm Hg. Physical examination shows a
midsystolic crescendo-decrescendo murmur best heard at the lower left sternal border and apex.
The remainder of the physical examination is normal. Laboratory studies show no abnormalities.
An echocardiogram shows asymmetric thickness of the ventricular septum without left
ventricular dilation. Which of the following drugs is most appropriate for this patient's
hypertension? 
Correct Answer ImageA.Atenolol
Incorrect Answer ImageB.Captopril
Incorrect Answer ImageC.Furosemide
Incorrect Answer ImageD.Hydrochlorothiazide
Incorrect Answer ImageE.Nitroglycerin
Incorrect Answer ImageF.Sotalol plus disopyramide

A 35-year-old woman comes to her physician after being found to have blood in her urine during
an insurance physical examination 2 days ago. She denies any current urinary symptoms, and a
complete review of systems is normal. She uses ibuprofen for chronic low back pain. She has a
20 pack-year smoking history. She is sexually active, uses barrier contraception, and had her last
menstrual period approximately 12 days ago. Vital signs and physical examination are normal.
The urinalysis shows 1+ protein, 2+ blood, and 5–10 erythrocytes/HPF. Which of the following
is the best next step in management? 
Incorrect Answer ImageA.Bacterial culture of urine
Incorrect Answer ImageB.CT scan of the kidneys
Incorrect Answer ImageC.Cystoscopy
Correct Answer ImageD.Serum creatinine concentration
Incorrect Answer ImageE.Urine cytology

A 72-year-old woman comes to the physician because of worsening generalized weakness and
shortness of breath. Her past medical history is significant for diabetes mellitus, hypertension,
hyperlipidemia, arthritis, and depression. Medications include lisinopril, metformin,
nitroglycerin, isosorbide dinitrate, metoprolol, simvastatin, ibuprofen, and venlafaxine. Her
temperature is 36.8°C (98.2°F), pulse is 76/min, respirations are 24/min, blood pressure is
130/72 mm Hg, and oxygen saturation is 97% on room air. Physical examination reveals an
elderly woman in no distress. Breath sounds are decreased at the bases. Heart sounds are normal
and without murmurs. Her abdomen is soft, nontender, and non-distended. The rest of the
physical examination is unremarkable. Arterial blood gases show pH 7.32, pCO2 33 mm Hg,
pO2 94 mm Hg, and HCO3 17 mmol/L. Serum (fasting) studies show: 
Na+ 142 mEq/L
K+ 5.6 mEq/L
Cl–
112 mEq/L
HCO3– 18 mEq/L
Creatinine 1.2 mg/dL
BUN 20 mg/dL
Glucose 136 mg/dL
Ca 2+
8.7 mg/dL
HgA1c 8%
Urinalysis shows pH 4.7 and specific gravity 1.022 and is negative for glucose, ketones, RBCs,
and WBCs. Chest x-ray and ECG are normal. Which of the following is the most likely
underlying mechanism for this patient's acidosis? 
Incorrect Answer ImageA.Decreased renal bicarbonate reabsorption
Incorrect Answer ImageB.Impaired distal tubule proton secretion
Incorrect Answer ImageC.Increased lactate production
Incorrect Answer ImageD.Increased ketoacid production
Correct Answer ImageE.Suppression of renal ammoniagenesis by hyperkalemia

A 44-year-old man undergoes transsphenoidal resection of a pituitary adenoma. One day later the
patient is thirsty and asks for water numerous times. Current medications include Ringer's lactate
at 100 mL/h, IV pantoprazole, IV cefazolin, and IV morphine as needed for pain. The patient has
a history of depression and hypercholesterolemia. His temperature is 37.1ºC (98.8ºF), blood
pressure is 140/88 mm Hg, and pulse is 85/min. Urine output is 500–700 mL/h. 
Laboratory studies show: 
Na+ 149 mEq/dL 
Cl-  102 mEq/dL 
K +
4.4 mEq/dL 
HCO3-  27 mEq/dL 
Urea nitrogen (BUN)  10 mg/dL 
Creatinine  0.7 mg/dL 
Glucose  121 mg/dL 
Calcium  9.4 mg/dL 
Urine osmolality  88 mOsm/L 
After water deprivation, the plasma osmolality is 300 mOsm/L and urine osmolality is 91
mOsm/L. Which of the following is the most likely diagnosis? 
Incorrect Answer ImageA.Acute renal failure
Correct Answer ImageB.Diabetes insipidus
Incorrect Answer ImageC.Osmotic diuresis caused by hyperglycemia
Incorrect Answer ImageD.Psychogenic polydipsia
Incorrect Answer ImageE.Stroke

A 53-year-old man comes to the hospital with shortness of breath and right-sided chest pain. He
describes the pain as sharp and worse with inspiration. He has noticed the shortness of breath
progressing over the past week. His past medical history is notable for liver cirrhosis, which was
diagnosed approximately 3 years ago. The cirrhosis is secondary to hepatitis C infection. His
pulse is 95/min and blood pressure is 105/65 mm Hg. Pulse oximetry on room air shows an
oxygen saturation of 91%. He appears uncomfortable. Examination of the head and neck is
unremarkable. He has scattered spider angiomata. Markedly decreased breath sounds are noted
on the right. He has no murmurs. There is no appreciable abdominal fluid wave. Peripheral
examination shows 2+ pretibial edema. Chest radiograph shows a large, right-sided pleural
effusion. Which of the following is the next step in management? 
Incorrect Answer ImageA.Abdominal ultrasound
Incorrect Answer ImageB.Electrocardiogram
Incorrect Answer ImageC.Insertion of a chest tube
Incorrect Answer ImageD.Liver function tests
Incorrect Answer ImageE.Propranolol
Incorrect Answer ImageF.Sodium restriction and hydrochlorothiazide
Correct Answer ImageG.Thoracentesis
A 49-year-old woman comes to the physician because of progressive fatigue, abdominal
bloating, and irritability for 6 months. She has had tingling sensations in both of her legs. Her
temperature is 37.2ºC (99ºF), blood pressure is 112/68 mm Hg, and pulse is 88/min. Neurologic
examination shows decreased position and vibratory sensation bilaterally in the lower
extremities. Cardiac examination shows a normal S1 and S2 and a 1/6 systolic murmur is heard
along the left sternal border. Hemoglobin is 8.3 g/dL with mean corpuscular volume 118 µm3. A
Schilling test shows absorption of radiolabeled vitamin B12 occurs after intrinsic factor is
administered. Which of the following is the most likely diagnosis? 
Correct Answer ImageA.Atrophic gastritis
Incorrect Answer ImageB.Bacterial overgrowth
Incorrect Answer ImageC.Chronic pancreatitis
Incorrect Answer ImageD.Crohn's disease
Incorrect Answer ImageE.Gastric ulcer

A 46-year-old woman presents to the emergency department with shortness of breath, nausea,
vomiting, diarrhea, mild anorexia, and abdominal pain for 3 days. Her past medical history is
significant for hypertension, type 2 diabetes mellitus, asthma, rheumatoid arthritis, and Sjogren
syndrome. Her medications include aspirin, atenolol, albuterol inhaler, lisinopril, metformin,
methotrexate, and artificial tears. On examination her temperature is 37.2°C (98.9°F), blood
pressure is 105/70 mm Hg, pulse is 110/min, and respirations are 24/min. She has dry oral
mucosa. Lab results are: 
Na+ 145 mEq/L
K+ 5.0 mEq/L
Cl -
105 mEq/L
HCO3- 17 mEq/L
BUN 48 mg/dL
Cr 2.1 mg/dL
pH 7.30
PaCO2 30 mm Hg
Glucose 240 mg/dL
Urinalysis shows 4+ glucose on urine dipstick. Which of the following is the most likely
diagnosis? 
Incorrect Answer ImageA.Adrenal insufficiency
Incorrect Answer ImageB.Diabetic ketoacidosis
Correct Answer ImageC.Lactic acidosis
Incorrect Answer ImageD.Renal tubular acidosis
Incorrect Answer ImageE.Salicylate poisoning

A 39-year-old man comes to the physician because of a painful left first toe that began one day
ago. The pain woke him up from sleep. He has taken ibuprofen with minimal relief. He attended
a wedding two days ago where he drank 14 to 16 beers. Over the past year, he has had three
similar episodes that resolved with indomethacin. His mother and two additional maternal
relatives recently had similar symptoms. On examination, he is in mild distress. He weighs 127
kg (280 lb) and is 180 cm (71 in) tall. His temperature is 37.8ºC (100ºF), blood pressure is
130/85 mm Hg, and pulse is 98/min. The metatarsophalangeal joint of the left great toe is
swollen, erythematous, and tender. Analysis of joint fluid aspiration shows negatively
birefringent crystals. He is administered indomethacin and he recovers over the next few days.
Which of the following is the most appropriate next step to prevent future episodes?
Correct Answer ImageA.Allopurinol
Incorrect Answer ImageB.Corticosteroids
Incorrect Answer ImageC.NSAIDs
Incorrect Answer ImageD.Obtain serum uric acid
Incorrect Answer ImageE.Probenecid

A 61-year-old woman comes to the emergency department because of shortness of breath. Over
the previous 3 months, the patient has felt progressively dyspneic. She used to be able to climb
the stairs of her house with little problem but now finds it increasingly difficult and occasionally
has to stop halfway up the stairs to catch her breath. She complains of extreme fatigue following
5 minutes of interacting with her grandchildren. She also describes waking up in the middle of
the night with shortness of breath. She finds these episodes distressing and has to rush to a
nearby open window to catch her breath. She denies chest pain. Her temperature is 37.1°C
(98.8°F), pulse is 74/min, and blood pressure is 150/94 mm Hg. The patient becomes mildly
dyspneic on transferring from the chair to the examination table. There is evidence of jugular
venous distention and pedal edema. There is no chest wall tenderness, although there are
scattered, fine crackles throughout the chest. Cardiac examination shows no abnormalities.
Which of the following is the most accurate test to confirm the diagnosis?
Incorrect Answer ImageA.Coronary angiography
Incorrect Answer ImageB.Echocardiography
Incorrect Answer ImageC.Electrocardiogram
Correct Answer ImageD.MUGA scan
Incorrect Answer ImageE.Stress test

A 62-year-old man with chronic gastroesophageal reflux disease (GERD) comes to the physician
for follow-up after endoscopy. He has a history of abdominal discomfort and chest pain,
sensation of stomach contents traveling from the stomach to the chest, and nocturnal coughs for
the past month. He has reduced his alcohol intake, avoided acidic foods, and raised the head of
his bed without relief of symptoms. Current medications include omeprazole. A biopsy specimen
of the esophagus shows Barrett’s esophagus without dysplasia. After confirmation of the
diagnosis, which of the following is the most appropriate next step in management?
Correct Answer ImageA.Endoscopy and biopsy every 3 years
Incorrect Answer ImageB.Fundoplication
Incorrect Answer ImageC.Increased dose of proton-pump inhibitor (PPI)
Incorrect Answer ImageD.Repeat endoscopy and biopsy within 6 months
Incorrect Answer ImageE.Surgical resection

A 23-year-old man comes to the physician because of lightheadedness, fatigue, and confusion.
He has a 1-year history of epilepsy treated with phenytoin. His last seizure was 6 months ago. He
drinks 2 to 3 beers on the weekends but does not smoke. His temperature is 37.5ºC (99.5ºF),
blood pressure 130/85 mmHg, and pulse 90/min. Examination shows no abnormalities.
Laboratory studies show: 
Hemoglobin 9.2 g/dL
Platelets  200,000/mm3
White blood cells  4,000/mm3
Differential:
Granulocytes  58%
Lymphocytes  30%
Monocytes  5%
MCV 105 µm3
MCHC  35%
MCH  33.6 pg/cell
Which of the following is the most appropriate next step in management? 
Incorrect Answer ImageA.Administer folic acid replacement
Incorrect Answer ImageB.Discontinue phenytoin
Correct Answer ImageC.Measure serum folate and vitamin B12 levels
Incorrect Answer ImageD.Measure iron indices
Incorrect Answer ImageE.Measure MMA and homocysteine levels

A 45-year-old woman is brought to the emergency department by her sister 20 minutes after a
generalized seizure. The sister states that the patient has had intermittent episodes of shaking,
palpitations, sweating, and anxiety for 6 months. She takes no medications. She works as a
pharmacist. She is alert and diaphoretic. Her temperature is 36.7ºC (98ºF), pulse is 110/min, and
blood pressure is 130/85 mm Hg. Insulin antibody is negative. Laboratory studies show: 
Blood glucose 45 mg/dL 
Insulin 7 µU/mL (normal, <3 µU/mL) 
C-peptide 1.5 nmol/L (normal, <0.2 nmol/L) 
Proinsulin 9 pmol/L (normal, <5 pmol/L) 
Which of the following is the most likely diagnosis? 
Incorrect Answer ImageA.Autoimmune disease
Incorrect Answer ImageB.Factitious insulin administration
Incorrect Answer ImageC.Glucagonoma
Correct Answer ImageD.Insulinoma
Incorrect Answer ImageE.Pheochromocytoma
Incorrect Answer ImageF.Zollinger-Ellison syndrome

A 23-year-old woman is brought to the emergency department immediately after feeling


lightheaded and losing consciousness for approximately 2 minutes. Prior to passing out, she felt
dizzy, sweaty, and had palpitations. She has had prior episodes of anxiety, palpitations,
diaphoresis, and hunger which have resolved with eating and rest. She denies headaches, fatigue,
chest pain, fever, nausea/vomiting, and weight changes. Her medical history is negative for
hypertension, and is otherwise unremarkable. She takes no prescribed medications. She works as
a pharmacy technician. Her temperature is 37.0°C (98.6°F), pulse is 90/min, respirations are
18/min, and blood pressure is 110/70 mm Hg. There are no orthostatic changes in blood pressure.
Physical examination shows no abnormalities. Serum glucose level is 32 mg/dL. Electrolytes are
all normal. Plasma insulin and C-peptide levels are increased. ECG shows no abnormalities.
Which of the following is the most appropriate next step in management? 
Incorrect Answer ImageA.24-hour urine catecholamines
Incorrect Answer ImageB.Cosyntropin stimulation test
Incorrect Answer ImageC.CT scan of the abdomen
Incorrect Answer ImageD.Endoscopic ultrasonography of the pancreas
Incorrect Answer ImageE.Glucagon levels
Incorrect Answer ImageF.Prolonged (72-hr) fast
Incorrect Answer ImageG.Serum alcohol levels
Correct Answer ImageH.Serum sulfonylurea levels

A 77-year-old man is brought to the clinic by his daughter because he has become "senile" over a
period of 3 to 4 weeks. According to her, he used to be physically active, went horseback riding
daily, and managed all of his own financial affairs. Now, he stares at the wall, barely talks, and
sleeps most of the day. His daughter recalls that he fell from a horse approximately 2 weeks
before the mental changes began. Physical examination shows a confused elderly male but is
otherwise unremarkable. Which of the following will be the most likely finding on a CT scan of
the head of this patient?
Incorrect Answer ImageA.Chronic epidural hematoma
Correct Answer ImageB.Chronic subdural hematoma
Incorrect Answer ImageC.Diffuse intracerebral bleeding
Incorrect Answer ImageD.Frontal lobe infarction
Incorrect Answer ImageE.Generalized, severe brain atrophy

A 49-year-old previously healthy man comes to his physician for follow-up examination because
of a fasting total serum cholesterol level of 299 mg/dL. He says he exercises three to four times a
week and is on a low-fat, low-cholesterol diet. He has a family history of ischemic heart disease
on his father's side, and he has smoked one pack of cigarettes daily for the past 25 years. He says
he leads a low-stress lifestyle and has no complaints. His blood pressure is 135/85 mm Hg.
Laboratory studies from 2 weeks ago show: 
Total cholesterol 299 mg/dL
HDL 65 mg/dL
LDL 170 mg/dL
His current 10-year atherosclerotic cardiovascular disease risk (ASCVD) is 10%. Which of the
following is the best next step in patient care? 
Incorrect Answer ImageA.Educate the patient about diet and exercise and repeat the tests within a
year
Incorrect Answer ImageB.Educate the patient about diet and exercise and repeat the tests within 4
weeks
Incorrect Answer ImageC.Initiate a mandatory low-fat diet for the patient
Correct Answer ImageD.Initiate drug therapy for control of his hyperlipidemia
Incorrect Answer ImageE.No intervention is indicated

A previously healthy 28-year-old man is brought to the emergency department immediately


following a motor vehicle accident. On arrival, he does not respond to verbal or painful stimuli.
His blood pressure is 160/95 mm Hg, pulse is 52/min, and respirations are 8/min. Examination
shows papilledema. A CT of the head without contrast is performed and a single axial image
slice from the study is shown below. The posterior fossa and frontoparietal convexities appeared
unremarkable on additional image slices. This patient is at greatest risk for which of the
following?
Incorrect Answer ImageA.Cerebellar tonsillar herniation
Incorrect Answer ImageB.Reverse cerebellar herniation
Incorrect Answer ImageC.Subfalcine herniation
Incorrect Answer ImageD.Transcalvarial herniation
Correct Answer ImageE.Transtentorial herniation

A 35-year-old woman comes to the physician because of a red lesion on her index finger that
developed after a minor injury 2 weeks ago. The woman reports that the lesion is painless,
although its cosmetic appearance is concerning. Physical examination shows a 1 cm red,
vascular, raised papule with a thin collarette of scale. The lesion bleeds easily and does not
blanch with pressure. There are no changes in the surrounding skin. Which of the following is
the most likely diagnosis?
Incorrect Answer ImageA.Dermatofibroma
Incorrect Answer ImageB.Keloid
Incorrect Answer ImageC.Keratoacanthoma
Correct Answer ImageD.Pyogenic granuloma
Incorrect Answer ImageE.Wart

A 19-year-old man visits his college urgent care because of severe right eye pain and redness,
photophobia, tearing, and decreased vision. He is immediately referred to an ophthalmologist.
The student tells the doctor that he wears contact lenses and has not taken them out for 10 days
because he has been up late every night studying for final exams. The patient’s visual acuity is
20/100 in the right eye and slit lamp examination shows a hazy, edematous cornea, a central
corneal ulcer measuring 4 x 4 millimeters in size, and an adjacent stromal abscess. Which of the
following is the most appropriate next step in management?
Incorrect Answer ImageA.Perform a corneal biopsy
Correct Answer ImageB.Scrape the ulcer for Gram stain and culture
Incorrect Answer ImageC.Start ciprofloxacin ophthalmic drops 0.3% topically
Incorrect Answer ImageD.Start moxifloxacin ophthalmic drops 0.5% topically
Incorrect Answer ImageE.Treat with topical prednisolone acetate 0.12% suspension

A 14-year-old girl is brought to the dermatologist by her mother because of acne. The girl reports
worsening of the lesions despite using an over-the-counter topical acne treatment. Skin
examination shows open and closed comedones on the forehead and some comedones, many
papules, and large pustules on the chin, nasolabial folds, and cheeks. There are a few lesions on
the upper back and none on the chest or deltoid regions. Which of the following is the most
appropriate pharmacotherapy?
Incorrect Answer ImageA.Isotretinoin
Incorrect Answer ImageB.Oral doxycycline plus topical retinoid
Incorrect Answer ImageC.Topical adapalene
Incorrect Answer ImageD.Topical azelaic acid
Correct Answer ImageE.Topical benzoyl peroxide plus clindamycin gel

A 71-year-old man is brought to the emergency department by his relatives with a 1-day history
of lethargy, confusion, slurred speech, and left-sided weakness. According to the patient's
relatives, he has a history of recurrent deep venous thrombosis and has been on warfarin for
several years. He has always been compliant with his medication. He does not have any history
of hypertension, diabetes, hypercholesterolemia, or heart disease. On examination, his
temperature 37ºC (98.6ºF), pulse is 60/min, respirations are 14/min, and blood pressure is 170/60
mm Hg. He is confused and not oriented to person, time, or place. His pupils are sluggishly
reactive to light. Neurologic examination shows 3/5 strength in the left upper and lower
extremities. Deep tendon reflexes are brisk on the left side. A CT scan of the head shows an
intraparenchymal hemorrhage in the right temporoparietal region. Laboratory studies show:
Hgb 14.6 g/dL
WBC 6000/mm3
Platelets  110,000/mm3
Bicarbonate 29 mEq/L
PT  53 seconds
INR  7.0
PTT  25 seconds
Which of the following is the most appropriate next step in management? 
Incorrect Answer ImageA.Admission to the ICU with close neurologic monitoring
Incorrect Answer ImageB.Oral vitamin K
Incorrect Answer ImageC.Subcutaneous vitamin K
Correct Answer ImageD.Transfuse fresh frozen plasma
Incorrect Answer ImageE.Transfuse platelets

A 35-year-old woman comes to the physician because of difficulty breathing, non-productive


cough, and a rash for 1 year. Over the past 6 months, her shortness of breath has worsened and
her coughing has increased. She does not smoke. Bilateral wheezes are heard on auscultation.
Her serum calcium level is 12.1 mg/dL. An intradermal injection of Candida antigen elicits no
response after 72 hours. A chest x-ray shows bilateral hilar fullness. Which of the following is
the most likely cutaneous finding in this patient? 
Incorrect Answer ImageA.Erythema marginatum
Incorrect Answer ImageB.Erythema migrans
Correct Answer ImageC.Erythema nodosum
Incorrect Answer ImageD.Pyoderma gangrenosum
Incorrect Answer ImageE.Pyogenic granuloma

A 38-year-old woman is brought to the emergency department because of acute-onset confusion,


lethargy, and mental status changes. She was recently prescribed quinine for leg cramps. She
works as an office clerk and has two children. Her temperature is 102.1ºF (38.9ºC), blood
pressure is 125/66 mm Hg, and pulse is 110/min. Physical examination shows she is
intermittently arousable and follows commands poorly. Cardiovascular examination shows
tachycardia. Pulmonary and abdominal examination is unremarkable. She has scattered petechiae
on her legs. Her neurologic examination shows no focal findings. A chest radiograph is normal.
Laboratory values show:
Leukocytes 4,000/mm3
Hematocrit 29%
Platelet count 19,000/mm3
Sodium 141 mEq/L
Potassium 3.3 mEq/L
Chloride 105 mEq/L
Bicarbonate 20 mEq/L
Blood urea nitrogen 31 mg/dL
Creatinine 2.1 mg/dL
Aspartate aminotransferase (AST, GOT) 31 IU/L
Alanine aminotransferase (ALT, GPT) 29 IU/L
Alkaline phosphatase 85 IU/L
Total bilirubin 3.3 mg/dL
Conjugated bilirubin 0.6 mg/dL
Which of the following is most likely to confirm the diagnosis?
Correct Answer ImageA.Blood smear
Incorrect Answer ImageB.CT of the head
Incorrect Answer ImageC.Fibrinogen level
Incorrect Answer ImageD.Lactate dehydrogenase (LDH) level
Incorrect Answer ImageE.Ultrasound of the liver

A 72-year-old man comes to the physician because of rough patches of skin on his scalp, face,
and hands for 10 years. He has had no fever or weight loss. He has a history of essential
hypertension, type 2 diabetes mellitus, post-traumatic stress disorder, and tobacco addiction. He
is a veteran of the Vietnam War. Physical examination shows multiple scaly, keratotic,
erythematous macules and papules on his anterior scalp, forehead, temples, nose, forearms, and
dorsal hands. The sizes of the macules range from 2 to 10 mm in diameter. Which of the
following is the most significant risk factor for the development of these skin lesions?
Incorrect Answer ImageA.Agent Orange exposure
Incorrect Answer ImageB.Diabetes mellitus
Correct Answer ImageC.Excessive sun exposure
Incorrect Answer ImageD.Post-traumatic stress disorder
Incorrect Answer ImageE.Tobacco addiction

A 27-year-old woman comes to the physician for a routine health maintenance examination.
Over the past 3 months, she has awoken from sleep on several occasions with cough, wheezing,
and shortness of breath that lasts for 1 to 2 hours. She denies shortness of breath during the day
or while exercising. She has had nausea and burning chest pain after large meals, treated with
famotidine as needed. She weighs 61 kg (135 lb) and is 165 cm (65 in) tall. Her temperature is
36.8ºC (98.2ºF). The lungs are clear to auscultation. The abdomen is soft and nontender with
normal bowel sounds. Which of the following is the most likely cause of this patient's
symptoms? 
Correct Answer ImageA.Gastrointestinal reflux disease
Incorrect Answer ImageB.Laryngospasm
Incorrect Answer ImageC.Mast cell release
Incorrect Answer ImageD.Post-nasal drip
Incorrect Answer ImageE.Upper airway obstruction

A 23-year-old woman comes to the physician because of right-sided pleuritic chest pain. She has
had a pruritic rash on her face and painful swelling of her fingers. She takes no medications. Her
temperature is 37ºC (98.6ºF), blood pressure is 160/90 mm Hg, pulse is 98/min, and respirations
are 15/min. Examination of her face is shown. There is swelling of proximal interphalangeal and
metacarpophalangeal joints bilaterally. There are decreased breath sounds at the right lung base.
Her hemoglobin level is 8 g/dL. Which of the following diagnostic tests is most likely to be
abnormal?
Incorrect Answer ImageA.Anticentromere antibody
Incorrect Answer ImageB.Anti-dsDNA (double-stranded DNA)
Incorrect Answer ImageC.Antihistone antibody
Incorrect Answer ImageD.Anti-Jo-1 antibody
Correct Answer ImageE.Antinuclear antibody
Incorrect Answer ImageF.Anti-Ro antibody
Incorrect Answer ImageG.Rheumatoid factor

A 25-year-old woman comes to the physician because of a 4-week history of progressive


irritability, anxiety, and heat intolerance. She underwent an uncomplicated vaginal delivery of a
healthy male infant 4 weeks ago. Her medical history is unremarkable and she takes no
medications. Her temperature is 37.5ºC (99.5ºF), pulse is 110/min, respirations are 14/min, and
blood pressure is 150/65 mm Hg. Examination shows no thyroid enlargement or tenderness. Her
skin is warm and moist, and there is a fine hand tremor. Which of the following is the most likely
diagnosis?
Incorrect Answer ImageA.Graves' disease
Incorrect Answer ImageB.Hashimoto thyroiditis
Correct Answer ImageC.Postpartum thyroiditis
Incorrect Answer ImageD.Riedel thyroiditis
Incorrect Answer ImageE.Sheehan syndrome
Incorrect Answer ImageF.Subacute (de Quervain) thyroiditis

A 21-year-old man is brought to the emergency department after an apparent fight outside a bar
yesterday. He passed out and was found by police today. When he arrives in the emergency
department he is lethargic, weak, and confused. His temperature is 38.4ºC (101.1ºF), pulse is
143/min, respirations are 28/min, and blood pressure is 80/40 mm Hg. There are multiple bruises
and excoriations, two penetration bullet wounds on his left leg and purulent drainage from
lacerations on his chest and arms. There are hematomas over both flanks and on his abdomen.
An x-ray series reveals a pelvic fracture and fractures of three ribs. A head CT reveals no
traumatic injury. Emergent serum/blood studies show: 
Sodium 145 mEq/L
Potassium 4.5 mEq/L
Chloride 110 mEq/L
Bicarbonate 26 mEq/L
Glucose 116 mg/dL
BUN 77 mg/dL
Creatinine 1.7 mg/dL
Hemoglobin 7 g/dL
WBC 9,700/mm3
Platelets 270,000/mm3
HCT 21%
He is given several liters of normal saline, 4 units of packed red blood cells, vancomycin,
cefepime, and gentamicin. His blood pressure increases to 100/74 mm Hg and his mental state
improves. He is transferred to the intensive care unit for observation. 
24 hours later his temperature is 37.0ºC (98.6ºF), blood pressure is 110/70 mmHg, pulse is
83/min and respirations are 18/min. Urine output has dropped from 100 cc/hour to 20
cc/hour. Serum/blood studies now show:
BUN 43 mg/dL
Creatinine 2.6 mg/dL
CPK 1,400 U/L
WBC 5,700/mm3
Platelets 270,000/mm3
HCT 29%
Urine studies show:
Specific gravity 1.010
Osmolality 300 mOsm/kg H2O
Leukocyte esterase Negative
Nitrite Negative 
Protein Trace
Blood Negative
Microscopy Muddy brown granular casts
Eosinophils None
Sodium 50 mEq/L
Creatinine 15 mg/dL
The fractional excretion of sodium is 6%. Which of the following is the most likely etiology of
his acute kidney injury?
Incorrect Answer ImageA.Acute interstitial nephritis from cephalosporins
Incorrect Answer ImageB.Gentamicin
Incorrect Answer ImageC.Renal contusion
Incorrect Answer ImageD.Rhabdomyolysis
Correct Answer ImageE.Sustained hypotension

A 68-year-old man who has a long history of tobacco use and a diagnosis of chronic obstructive
pulmonary disease (COPD) comes to the clinic because of a cold. He reports a 1-week history of
cough, low-grade fevers and chills, and pleuritic chest pain. Additionally, he is requiring more
home oxygen than he has previously needed and is concerned that things may get worse. In
addition to his COPD, he has a history of paroxysmal atrial fibrillation, for which he receives
anticoagulation with warfarin and rate control with a long-acting calcium channel blocker. Chest
radiograph shows diffuse interstitial changes consistent with bronchiectasis and a small amount
of parenchymal infiltrate in the left lower lobe. A sputum sample is obtained for Gram stain and
culture. Gram stain shows numerous gram-negative diplococci in pairs and gram-positive cocci
in chains. Which of the following is the most appropriate therapy?
Correct Answer ImageA.Amoxicillin/clavulanic acid
Incorrect Answer ImageB.Cefepime plus vancomycin
Incorrect Answer ImageC.Ciprofloxacin
Incorrect Answer ImageD.Fluids, acetaminophen, and rest
Incorrect Answer ImageE.Trimethoprim/sulfamethoxazole
A 38-year-old woman comes to the office complaining of a new-onset rash on her face, along
with hair loss, malaise, and fatigue worsening over the past month. She also noticed that her rash
flares when exposed to the sun. Her medical history is significant for food allergies and acute
thyroiditis as a teenager. Her only medication is levothyroxine taken daily. On physical
examination, there is ill-defined erythema and edema on her nose and cheeks and diffuse
thinning of the scalp hair with shorter hairs on the anterior hairline. The fingertips showed small,
tender violaceous papules. Urinalysis shows 4+ protein. Laboratory studies show:
RBCs  2.9 x 1012/L 
WBCs  2.0 x 109/L 
Platelets  220 x 109/L 
ESR  57 mm/h
ANA  1:160
Which of the following antibodies is the most specific serologic diagnostic study for this
disease? 
Incorrect Answer ImageA.Anticentromere antibodies
Incorrect Answer ImageB.Antihistone antibodies
Incorrect Answer ImageC.Anti-Jo-1 antibodies
Incorrect Answer ImageD.Anti-Ro antibodies
Correct Answer ImageE.Anti-Smith antibodies

A 40-year-old woman comes to the emergency department because of left leg swelling for 1 day.
She has had no fever or trauma to the leg. She has a 2-year history of metastatic breast
adenocarcinoma treated with paclitaxel and radiation therapy. Her temperature is 37.2°C (99°F),
pulse is 80/min, respirations are 14/min, and blood pressure is 118/74 mmHg. Her oxygen
saturation is 99% in room air. Examination shows a warm, erythematous, and tender left thigh
and lower leg. The lungs are clear to auscultation. Ultrasonography of the left leg shows a
thrombus in the superficial femoral vein. A CT angiogram of the chest shows no abnormalities.
Which of the following is the most appropriate next step in management?
Incorrect Answer ImageA.Apixaban
Incorrect Answer ImageB.Aspirin and warfarin with an INR goal of 2–3
Incorrect Answer ImageC.Dalteparin and enoxaparin together
Correct Answer ImageD.Enoxaparin followed by apixaban
Incorrect Answer ImageE.Warfarin with an INR goal of 2–3

A 22-year-old woman with a 4-year history of type 1 diabetes mellitus comes to her physician
because of elevated morning blood glucose levels for the past 2 weeks. She reports that her pre-
breakfast blood glucose levels are in the range of 150 to 180 mg/dL. Current medications include
regular insulin and NPH insulin twice daily. Her vital signs are as follows: temperature is 37.1ºC
(98.8ºF), pulse is 80/min, respirations are 12/min, and blood pressure is 124/80 mm Hg. Physical
examination shows no abnormalities. Laboratory data shows a hemoglobin A1c of 7.1%. Which
of the following is the most appropriate next step in management?
Correct Answer ImageA.Check blood glucose level at 3:00 AM
Incorrect Answer ImageB.Check blood glucose level at 11:00 PM
Incorrect Answer ImageC.Decrease dose of NPH insulin in the evening
Incorrect Answer ImageD.Increase dose of NPH insulin in the evening
Incorrect Answer ImageE.Increase dose of regular insulin in the morning

A 41-year-old man comes to the physician for a routine health maintenance examination. He
denies having any chest pain, shortness of breath, or leg swelling. He has no history of major
medical illness and takes no medications. He runs for 40 minutes, 4 to 5 times a week, and his
diet is low in saturated fat and cholesterol. He has smoked one pack of cigarettes weekly for 20
years. His father had a myocardial infarction at the age of 51 years, and his brother had a
myocardial infarction at the age of 49 years. His temperature is 37.2ºC (99.0ºF), pulse is 78/min,
respirations are 16/min, and blood pressure is 124/82 mm Hg. Examination shows no
abnormalities. His body mass index (BMI) is 22 kg/m2. Which of the following is the most
appropriate screening test indicated at this time for this patient?
Incorrect Answer ImageA.Electrocardiogram
Incorrect Answer ImageB.Fasting blood glucose
Correct Answer ImageC.Fasting lipid profile
Incorrect Answer ImageD.Oral glucose tolerance test
Incorrect Answer ImageE.Prostate-specific antigen
Incorrect Answer ImageF.Sigmoidoscopy

A 55-year-old woman comes to the physician because of progressive fatigue. She has been
feeling weak for the past several months and has recently been short of breath when she performs
moderate physical activity. She is also worried because she has had several episodes of
palpitations. Past medical history is relevant for hysterectomy because of endometrial
hyperplasia with atypia five years ago. She is not taking any medication and does not smoke or
drink alcohol. She is not in acute distress and her oral mucosa and conjunctiva look pale.
Laboratory studies show: 
Hemoglobin  7 g/dL
Platelets  300,000/mm3
White blood cells  8,000/mm3
Segmented neutrophils  56%
Bands  4%
Eosinophils  2%
Lymphocytes  30%
Monocytes  1%
MCV  86 um3
Reticulocytes  Absent
Peripheral smear  Normal RBC morphology
Bone marrow biopsy  Absence of erythroid precursors
This patient should be evaluated for which of the following conditions? 
Incorrect Answer ImageA.Gastric adenocarcinoma
Incorrect Answer ImageB.Medullary thyroid cancer
Incorrect Answer ImageC.Pancreatic adenocarcinoma
Incorrect Answer ImageD.Small cell carcinoma of the lung
Correct Answer ImageE.Thymoma

A 23-year-old man comes to the emergency department because of acutely worsening shortness
of breath, nausea with vomiting, and diarrhea. The patient has a long history of severe asthma
with multiple hospitalizations and one intubation 3 years ago. Two days ago, he cleaned out his
basement and since that time has had progressively worsening shortness of breath. He tried home
albuterol and ipratropium nebulizers, as well as his standard cromolyn therapy, but none of these
interventions relieved his symptoms. In the hospital, the patient's peak flow rates are decreased
50% from his baseline. Which of the following agents should most likely be added to this
patient's therapy to alleviate his current symptoms?
Incorrect Answer ImageA.Disodium cromoglycate
Incorrect Answer ImageB.Inhaled beclomethasone
Correct Answer ImageC.IV methylprednisolone
Incorrect Answer ImageD.Oral prednisone
Incorrect Answer ImageE.Theophylline

A 68-year-old man comes to see you for a hacking cough that is productive of putrid sputum and
seems to have gotten worse over the last few months. He reports having a chronic cough that he
attributes to his nearly 80-pack-year history of smoking, but has never had anything this bad.
Review of systems reveals a 15-lb weight loss over the last 2 months. Examination shows
rhonchi and wheezing over the right upper lobe. A chest radiograph confirms the abnormality,
showing a large, circular opacity with irregular margins. Fiberoptic bronchoscopy with biopsy
reveals cellular atypia suspicious for a primary malignancy, but the final pathology report is still
pending. The patient is fearful of surgery and wants to know if chemotherapy is an option. He
wants to be prepared even though his results are still pending. Which of the following
malignancies is most sensitive to chemotherapy? 
Incorrect Answer ImageA.Bronchoalveolar carcinoma
Incorrect Answer ImageB.Large-cell carcinoma
Incorrect Answer ImageC.Mucinous adenocarcinoma
Correct Answer ImageD.Small-cell carcinoma
Incorrect Answer ImageE.Squamous cell carcinoma

A 73-year-old Caucasian man presents to the clinic because of headaches that wake him from
sleep, nausea, and vomiting. He describes the headaches as rated 8/10 in intensity and involving
his whole head. He states that he also experiences headaches in the morning after waking from
sleep and when he takes naps in the middle of the day. The patient also reports a constant cough
for the past two months with associated left-sided chest pain. As an aside, he also notes that he
has been losing weight for the last 3 months though has not exercised aside from occasional
walks. On examination, the patient is a thin man who has atrophy of the subcutaneous tissues
over his temples. MRI shows multiple enhancing lesions at the grey-white junction of the right
parietal lobe. Which of the following tumors does the patient most likely have?
Incorrect Answer ImageA.Adenocarcinoma of the colon
Correct Answer ImageB.Adenocarcinoma of the lung
Incorrect Answer ImageC.Astrocytoma
Incorrect Answer ImageD.Glioblastoma multiforme
Incorrect Answer ImageE.Prostate cancer

A 30-year-old man comes to see his internist because of unilateral right-eye pain, photophobia,
and decreased vision. The doctor has been following this patient for a number of years for
Crohn's disease. Slit-lamp examination shows conjunctival hyperemia and inflammatory cells in
the aqueous humor. Visual acuity is 20/80 in the right eye and 20/20 in the left eye. The doctor
refers the man to an ophthalmologist. Further examination shows an anterior chamber hypopyon
and fibrin but no large mutton-fat keratic precipitates. Which of the following is the most likely
diagnosis?
Incorrect Answer ImageA.Anterior granulomatous uveitis
Correct Answer ImageB.Anterior nongranulomatous uveitis
Incorrect Answer ImageC.Bacterial keratitis
Incorrect Answer ImageD.Chronic glaucoma
Incorrect Answer ImageE.Posterior uveitis

A 38-year-old man comes to the emergency department because of a 1-day history of knee pain.
He has a 6-month history of intermittent swollen joints, primarily involving his hip and feet, as
well as back stiffness relieved by walking. He had syphilis 20 years ago and currently abuses
alcohol and IV drugs. He has been homeless for 9 years. He weighs 68 kg (150 lb) and is 180 cm
(71 in) tall. His temperature is 38.5ºC (101.3ºF) and his pulse is 85/min. A grade 2/6 systolic
crescendo-decrescendo murmur and diastolic decrescendo murmur is heard along the upper left
sternal border. The left knee is edematous, erythematous, and warm to the touch. The knee is
painful on passive and active motion with a palpable effusion. Which of the following is the
most appropriate next step in management?
Correct Answer ImageA.Arthrocentesis with white blood cell count, crystal identification, Gram
stain, and culture
Incorrect Answer ImageB.Empiric broad-spectrum antibiotics, started immediately
Incorrect Answer ImageC.Spine radiographs and HLA-B27 blood test
Incorrect Answer ImageD.Steroid and analgesic injection of left knee synovium
Incorrect Answer ImageE.Uric acid serum and urine levels, NSAIDs for acute pain

A 55-year-old man comes to the physician because of mid-chest pain and difficulty swallowing
solid foods for 2 months. He has also noticed a 10-lb weight loss over the past 3 months. He has
not seen a physician for many years and is vague when discussing his medical history. He does
not take any medications. He denies the ingestion of excess alcohol. He does not have a
significant smoking history. On physical examination, he is thin-appearing and in no acute
distress. His temperature is 36.8ºC (98.6ºF) and blood pressure is 125/80 mm Hg. Head, neck,
and oral examinations are normal. Cardiovascular and chest examinations are also normal. His
abdomen is soft and nontender, and his peripheral examination is also normal.
Electrocardiography is normal. He is scheduled for an esophagram, which demonstrates an
irregular lesion in his upper to the mid esophagus with hanging edges and luminal narrowing that
fails to open with peristaltic waves. CT scan of the chest demonstrates a circumferential mass
severely narrowing the esophagus. Which of the following, if present in this patient’s medical
history, could explain why he developed this lesion?
Incorrect Answer ImageA.Fibrosing mediastinitis
Incorrect Answer ImageB.HIV-associated esophageal candidiasis
Correct Answer ImageC.Lye ingestion
Incorrect Answer ImageD.Pill esophagitis
Incorrect Answer ImageE.Reflux esophagitis
A 24-year-old man is admitted to the hospital for induction chemotherapy prior to undergoing a
bone marrow transplant. He has a 5-month history of acute myelogenous leukemia. He receives
high-dose busulfan and cyclophosphamide. One day later, he develops severe nausea and
vomiting. He is unable to eat, drink, or take oral medications. His temperature is 37.1°C
(98.8°F). Which of the following is the most appropriate pharmacotherapy? 
Incorrect Answer ImageA.Dopamine
Incorrect Answer ImageB.Erythromycin
Incorrect Answer ImageC.Hydromorphone
Incorrect Answer ImageD.Metoclopramide
Incorrect Answer ImageE.Omeprazole
Correct Answer ImageF.Ondansetron

A 58-year-old man with a history of type 1 diabetes mellitus and alcoholism is brought to the
emergency department because of shortness of breath and a productive cough. He denies chest
pain. He has a history of congestive heart failure with an ejection fraction of 30%. He has
smoked 2 packs of cigarettes daily for the past 30 years and he drinks 40 units of alcohol a week.
Physical examination shows no chest wall tenderness, and rales are heard on auscultation of the
chest. There are no murmurs or rubs. Jugular venous pulsation is elevated to the level of the
mandible. An ECG shows sinus rhythm with no ST-segment depression or elevation. Chest x-ray
shows pulmonary vascular congestion. Treatment with IV furosemide, sublingual nitroglycerin,
and morphine is begun, but he does not improve. His blood pressure is 156/79 mm Hg. Which of
the following is the most appropriate next step in management? 
Incorrect Answer ImageA.Dopamine
Incorrect Answer ImageB.Echocardiography
Correct Answer ImageC.Hydralazine and isosorbide dinitrate
Incorrect Answer ImageD.MUGA scan
Incorrect Answer ImageE.Synchronized cardioversion
Incorrect Answer ImageF.Verapamil

A 50-year-old woman comes to the physician because of a 2-week history of rash above her
ankles and lower extremity swelling for the past 3 months. Physical examination shows pedal
edema with erythema, mild scaling, and brown discoloration of the skin of the distal legs
proximal to the medial malleoli. Varicose veins are present in the lower extremities. Which of
the following is the most likely diagnosis?
Incorrect Answer ImageA.Atopic dermatitis
Incorrect Answer ImageB.Cellulitis
Incorrect Answer ImageC.Lichen simplex chronicus
Incorrect Answer ImageD.Nummular dermatitis
Correct Answer ImageE.Stasis dermatitis

A 35-year-old woman comes to the physician because of a pigmented lesion on the palm of her
right hand. Her medical history is unremarkable. Physical examination shows a 2 mm light
brown macule with well-demarcated edges. A skin biopsy shows clustering of pigmented
melanocytes at the dermoepidermal junction. There are no clusters of melanocytes in the dermis.
Which of the following is the most likely diagnosis?
Incorrect Answer ImageA.Compound nevus
Incorrect Answer ImageB.Halo nevus
Incorrect Answer ImageC.Intradermal nevus
Correct Answer ImageD.Junctional nevus
Incorrect Answer ImageE.Lentigo

A 50-year-old man comes to the clinic because of recurrent attacks of headache, which manifest
with a peculiar rhythmic pattern for the past 3 years. Each episode begins on awakening,
approximately at the same time. He reports severe unilateral, periorbital pain with increased
lacrimation, rhinorrhea and sweating on the same side. The episode resolves spontaneously after
2 hours. Ibuprofen and acetaminophen do not relieve the acute pain. He does not smoke, drink
alcohol or use any recreational drugs. Family history is unremarkable. His temperature is 37°C
(98.6°F), pulse is 80/min, respirations are 20/min and blood pressure is 130/80 mm Hg. On
examination, pupils are equal, round and reactive to light. Funduscopic examination is
unremarkable. Which of the following is the most likely diagnosis?
Incorrect Answer ImageA.Acute glaucoma
Correct Answer ImageB.Cluster headache
Incorrect Answer ImageC.Giant cell arteritis (GCA)
Incorrect Answer ImageD.Glossopharyngeal neuralgia
Incorrect Answer ImageE.Migraine
Incorrect Answer ImageF.Trigeminal neuralgia

A 45-year-old man comes to the physician because of episodes of headaches, sweating, chest
pain, and palpitations for 3 months. The episodes are not related to exercise or exertion. He has a
history of uncontrolled hypertension treated with hydrochlorothiazide, lisinopril, and nifedipine.
He is married and lives at home with his wife and 3 children. He does not smoke cigarettes, and
he does not drink alcohol or coffee. He does not use illicit drugs. His blood pressure is 135/90
mm Hg and pulse is 85/min. The remainder of the examination shows no abnormalities. An ECG
shows no abnormalities. Which of the following is the most appropriate next diagnostic test? 
Incorrect Answer ImageA.24-hour urine 5-hydroxyindoleacetic acid (5-HIAA)
Correct Answer ImageB.24-hour urine fractionated metanephrines and catecholamines
Incorrect Answer ImageC.24-hour urine free cortisol
Incorrect Answer ImageD.CT scan of adrenal glands
Incorrect Answer ImageE.Serum catecholamines

A 60-year-old farmer comes to the physician because of difficulty seeing out of his right eye. He
reports progressive deterioration in his vision over the last few years. He believes this is caused
by “excess skin” growth in his right eye. He denies retro-orbital pain or a history of floaters. He
has no history of significant medical illnesses. Physical examination of the eyes shows a wedge-
shaped growth of superficial tissue on the right eye. The tissue is semi-opaque and vascularized.
It extends from the nasal side of the eye towards the pupil, where it crosses the limbus and
partially blocks the pupil. Visual acuity is 20/80 on the right and 20/25 on the left. Which of the
following is the most appropriate management?
Incorrect Answer ImageA.Corticosteroid eye drops
Incorrect Answer ImageB.Laser photocoagulation
Incorrect Answer ImageC.Reassurance without intervention
Correct Answer ImageD.Surgical resection of the tissue
Incorrect Answer ImageE.Topical antibiotic ointment

A 28-year-old man with a 6-year history of HIV comes to the emergency department because of
a 2-week history of fever and malaise. He has had an intermittent, dull, aching, posterior
headache for the past 4 days. He denies chills, rigors, nausea, vomiting, photophobia, or neck
stiffness and takes no medication. His temperature is 37.4ºC (99.3ºF), pulse is 78/min,
respirations are 16/min, and blood pressure is 100/70 mm Hg. Neurologic examination shows no
abnormalities. CD4+ cell count is 45 cells/mm3. Lumbar puncture found an elevated opening
pressure. CSF fluid analysis showed a leukocyte count of 28 cells/mm3 with 90% lymphocytes,
no erythrocytes, protein of 100 mg/dL, and glucose of 35 mg/dL. India ink stain of the CSF is
negative. Toxoplasma PCR of the CSF is negative. CT scan of the head shows no abnormalities.
Which of the following is the most appropriate next diagnostic test?
Correct Answer ImageA.Cryptococcal antigen in CSF
Incorrect Answer ImageB.Cryptococcal antigen in serum
Incorrect Answer ImageC.HIV antibodies
Incorrect Answer ImageD.MRI scan of the head
Incorrect Answer ImageE.Repeat CD4+ T lymphocyte count

A 48-year-old woman comes to the physician because of a 6-week history of a persistent dry
cough. She states that the cough is worse at night and is not associated with fever or chills. She
has a recent diagnosis of type 2 diabetes mellitus and hypertension currently treated with
metformin and fosinopril, respectively. She started these medications three months ago. Her
temperature is 37.0°C (98.6°F), blood pressure is 128/80 mm Hg, pulse is 76/min, and
respirations are 14/min. Physical examination shows a regular rate and rhythm with a
nondisplaced point of maximal impulse and a jugular venous pressure of 7 cm. Kidney function
is normal. Which of the following is the most appropriate next step in management?
Incorrect Answer ImageA.Discontinue metformin and start subcutaneous insulin
Incorrect Answer ImageB.Prescribe a daily diuretic to help reduce pulmonary edema
Incorrect Answer ImageC.Prescribe antibiotics to treat early community-acquired pneumonia
Incorrect Answer ImageD.Stop fosinopril and replace with lisinopril
Correct Answer ImageE.Stop fosinopril and replace with losartan

A 74-year-old man comes to the emergency department because of two episodes of passing large
amounts of bright red blood from the rectum over the past 4 hours. He has had no fevers, fatigue,
weight loss, abdominal pain, or change in bowel habits. He has a long history of chronic
constipation. His temperature is 37.5°C (99.5°F), blood pressure is 110/70 mm Hg, and pulse is
88/min. Abdominal examination shows no masses and no tenderness. Hemoglobin level is 10.4
g/dL and white blood cell count is 7,500/mm3. Sigmoidoscopy shows multiple large diverticula
in the sigmoid colon with fresh blood. Which of the following is the most likely explanation for
this patient's bleeding?
Incorrect Answer ImageA.Angiodysplasia
Correct Answer ImageB.Hemorrhage from a single diverticular arteriole
Incorrect Answer ImageC.Hemorrhage from an inferior mesenteric vein
Incorrect Answer ImageD.Internal hemorrhoids
Incorrect Answer ImageE.Ischemic colitis
A 49-year-old diabetic man with end-stage renal disease comes to the clinic because of bilateral
leg and arm pain. He describes the pain as diffuse, crampy, and constant in nature. He has also
been extremely weak and states he has trouble moving his lower extremities. The patient reports
having skipped his last two dialysis visits as he has been depressed about "needing to be hooked
up to a machine." His temperature is 36.7°C (98°F), pulse is 98/min, respirations are 20/min, and
blood pressure is 168/108 mm Hg. Physical examination shows periorbital edema and 3+ pitting
edema of the lower extremities. Neurologic examination shows diffuse muscle weakness with
4/5 upper-extremity strength and 3/5 lower-extremity strength. Which of the following
electrocardiogram findings is consistent with this patient's most likely electrolyte disturbance?
Incorrect Answer ImageA.Diffuse ST elevation, diffuse PR segment depressions
Incorrect Answer ImageB.Flattened T waves, prominent U waves
Incorrect Answer ImageC.Large S wave in lead I, Q wave in lead III, and inverted T wave in lead III
Correct Answer ImageD.Peaked T waves, wide QRS complexes, and loss of the P wave
Incorrect Answer ImageE.Short QT interval

A 16-year-old girl comes to the physician because of a 2-year history of "pimples" on her face,
back, and chest that do not improve with over-the-counter benzoyl peroxide. She states that the
number of red, tender nodules on the cheeks and chin increases during menses. She underwent
menarche at age 14, but her menses are irregular, with 9 periods per year. She takes no other
medications. She weighs 106 kg (234 lb) and is 170 cm (67 in) tall. Physical examination shows
deep, inflamed nodules, pustules, and comedones on the face, chest, and back. There are sparse,
coarse, pigmented hairs on the upper lip and chin and thinning of the hair at the temples. Urine
pregnancy test is negative. Which of the following is the most appropriate next step in
diagnosis? 
Incorrect Answer ImageA.CT scan of the head
Incorrect Answer ImageB.Luteinizing hormone (LH) level
Incorrect Answer ImageC.Quantitative human chorionic gonadotropin (hCG) level
Incorrect Answer ImageD.Trial of oral contraceptives
Correct Answer ImageE.Ultrasound of the ovaries

A 40-year-old woman with a history of metastatic breast cancer is brought to the emergency
department by her partner because of confusion for 2 days. She has not been undergoing any
treatment. Her partner reports that she has had crampy abdominal pain and constipation. She has
recently seemed depressed and has been missing her medical appointments. She appears
somnolent, disoriented, and in moderate distress. Her temperature is 37°C (98.6°F), pulse is
122/min, respirations are 20/min, blood pressure is 108/48 mm Hg while supine and 88/40 mm
Hg while standing. Examination shows dry mucous membranes. The abdomen is mildly tender
to palpation in all 4 quadrants but with no peritoneal signs. ECG shows sinus tachycardia and a
markedly short QT interval. Which of the following is the most appropriate next step in
management?
Incorrect Answer ImageA.Dexamethasone
Incorrect Answer ImageB.Furosemide
Incorrect Answer ImageC.Mithramycin
Correct Answer ImageD.Normal saline
Incorrect Answer ImageE.Pamidronate
A 52-year-old woman comes to the emergency department with increasing fatigue for several
weeks. She has a history of hypertension well controlled with lisinopril and hydrochlorothiazide,
ulcerative colitis with intermittent severe diarrhea, and stable chronic kidney disease of unknown
cause (baseline serum creatinine is 1.8–2.0 mg/dL). She takes aspirin as needed for intermittent
mild headaches. Her temperature is 36.0°C (96.8°F), pulse is 120/min, respirations are 24/min,
blood pressure is 148/95 mm Hg, and oxygen saturation is 99% on room air. Heart and lung
examinations are normal. Abdominal examination shows mild diffuse tenderness. There is 1+
edema in both ankles.
Arterial blood gases are pH, 7.36; pCO2, 32 mm Hg; and pO2, 94 mm Hg. Serum chemistries
show: 
Na+ 140 mEq/L
K+ 2.6 mEq/L
Cl –
114 mEq/L
HCO3– 18 mEq/L
Creatinine 1.4 mg/dL
eGFR 43 mL/min/1.73 m2
BUN 20 mg/dL
Glucose 92 mg/dL
Ca2+ 8.7 mg/dL
The serum phosphorus and urate levels are normal. Urinalysis shows pH 5.0, specific gravity
1.018, negative for glucose and ketones. No cells or casts are present. Which of the following
best explains her acidemia? 
Incorrect Answer ImageA.Decreased glomerular filtration rate
Incorrect Answer ImageB.Decreased renal ammoniagenesis
Correct Answer ImageC.GI bicarbonate losses
Incorrect Answer ImageD.Impaired distal tubule proton secretion
Incorrect Answer ImageE.Lactic acidosis
Incorrect Answer ImageF.Salicylate toxicity

A 71-year-old man comes to the physician because of increased cough, decreased vision in his
right eye, and loss of appetite. The physician has been seeing this man for more than 10 years for
management of his COPD. The patient has a 70-pack-year smoking history and quit 5 years ago.
He drinks a glass of wine every day with meals and denies drug use. His review of systems is
remarkable for fatigue and a 9-lb weight loss in the last 6 months. His temperature is 37.8ºC
(100.4ºF), pulse is 87/min, respirations are 18/min, and blood pressure is 150/90 mm Hg.
Physical examination shows bilateral scattered rales at the bases and normal heart sounds. The
anterior-posterior diameter of the chest is increased. The extremities have no evidence of
clubbing, cyanosis, or edema. Ocular examination shows ptosis of the right eye with a
constricted right pupil. The remainder of the ophthalmologic exam is normal. Cranial nerve
function is otherwise intact. Which of the following would most likely be expected on a chest x-
ray film? 
Correct Answer ImageA.Mass in the superior sulcus of the right lung
Incorrect Answer ImageB.Normal chest x-ray film
Incorrect Answer ImageC.Right lower lobe pneumonia
Incorrect Answer ImageD.Right-sided cavitary lesion in the apex
Incorrect Answer ImageE.Tumor invading the lower thoracic vertebrae
A 43-year-old woman presents to her physician with intractable diarrhea. She voids at least 5
times a day. She has experienced a 10-lb weight loss over the past several months. On further
questioning, she complains of severe heartburn also. On review of systems, she reports her hands
turn blue and then red in cold weather. Her past medical history is unremarkable, and she takes
no medications. On physical examination her temperature is 36.8ºC (98.3ºF) and blood pressure
is 130/80 mm Hg. Head and neck examination demonstrates retraction of the soft tissues around
the mouth, with exposure of the upper teeth. There are fine crackles at her lung bases. Cardiac
examination is unremarkable. Abdominal examination demonstrates no tenderness or
hepatosplenomegaly. She is hemoccult-negative. Peripheral examination demonstrates no
clubbing, cyanosis, or edema. The skin over the distal fingers, however, is retracted and smooth.
Which of the following would most likely improve this patient's diarrhea? 
Correct Answer ImageA.Antibiotics
Incorrect Answer ImageB.Avoidance of gluten products
Incorrect Answer ImageC.Avoidance of antibiotics
Incorrect Answer ImageD.Oral loperamide
Incorrect Answer ImageE.Pancreatic enzyme replacement

A 28-year-old woman comes to her physician because of a two month history of increased thirst
and increased urinary frequency without dysuria. She has had to wake up 3–5 times nightly to
urinate and urinates 6–7 times in the daylight hours. She has had no menstrual irregularities and
her last menstrual period was 10 days ago. Her medical history is otherwise unremarkable.
Current medications include oral contraceptive pills. Her temperature is 37°C (98.6°F), blood
pressure is 105/68 mm Hg, pulse is 86/min, and respirations are 12/min. Physical examination is
normal. Laboratory studies show: 
Na+ 144 mEq/L
K+ 4.5 mEq/L
Cl-  111 mEq/L
HCO3 - 
23 mEq/L
Urea nitrogen (BUN)  11 mg/dL
Creatinine 0.9 mg/dL
Calcium 8.7 mg/dL
Glucose 110 mg/dL
Osmolality  287 mOsm/kg
Urine studies show:
Volume/24 hours 7 liters
Specific gravity  1.004
Urine osmolality 160 mOsm/kg
After 72 hours of water deprivation, there is no change in urine production or osmolality.
Administration of exogenous vasopressin also causes no significant changes in these parameters.
Which of the following is the most likely cause of her urinary complaints? 
Incorrect Answer ImageA.Central diabetes insipidus
Incorrect Answer ImageB.Decreased glomerular filtration rate
Incorrect Answer ImageC.Diabetes mellitus
Correct Answer ImageD.Nephrogenic diabetes insipidus
Incorrect Answer ImageE.Psychogenic polydipsia
A 66-year-old man comes to the clinic because of progressively worsening shortness of breath
and nonproductive cough over the past 2 years. He has also lost 15 lbs over the past 3 months.
He retired 1 year ago after working as a rock miner for more than 30 years. He has no other
significant past medical history. On physical examination he is a thin man who appears
tachypneic at rest. His lungs have reduced chest expansion and dry inspiratory rales in the upper
lobes bilaterally. The remainder of his examination is normal. Chest x-ray shows multiple round
nodular opacities in the upper lobes accompanied by hilar lymphadenopathy with lymph node
calcification. Which of the following is a serious complication of the condition with which the
patient presents?
Incorrect Answer ImageA.Asbestosis
Incorrect Answer ImageB.Aspergillosis
Incorrect Answer ImageC.Atypical pneumonia
Incorrect Answer ImageD.Silicosis
Correct Answer ImageE.Tuberculosis

A 72-year-old man with a history of obesity and hypertension treated with nifedipine comes to
the physician because of worsening right-sided knee pain. He has suffered from this knee pain
for 10 years and is no longer receiving relief with nonsteroidal anti-inflammatory drugs. The
knee pain is progressively worse as the day goes on and the pain improves with rest. He also
reports pain in his left wrist. His temperature is 37ºC (98.6ºF), blood pressure is 160/90 mm Hg,
pulse is 68/min, and respirations are 15/min. Physical examination shows an obese man in no
acute distress. There is no warmth or swelling of his right knee but crepitus is felt during passive
range of motion. His left knee and right wrist are normal. Which of the following is consistent
with this patient's history?
Incorrect Answer ImageA.Antinuclear antibody positive
Incorrect Answer ImageB.Elevated erythrocyte sedimentation rate
Incorrect Answer ImageC.Joint erosions on radiograph
Incorrect Answer ImageD.Rheumatoid factor positive
Correct Answer ImageE.Unequal joint space narrowing on radiograph

A 64-year-old man comes to the physician because of a 4-month history of fatigue and shortness
of breath during exercise. He has had no palpitations or cough. His medical history is
unremarkable and he takes no medications. He does not smoke cigarettes or drink alcohol. His
temperature is 36.7°C (98°F), pulse is 93/min, and blood pressure is 135/75 mm Hg.
Examination shows pallor of the mucous membranes. The lungs are clear to auscultation. A
grade 3/6 systolic ejection murmur is heard at the left sternal border. The abdomen is soft and
nontender. Laboratory studies show: 
Hct 27%
Platelets  196,000/mm3
White blood cells  5,400/mm3
Total iron  10 ug/dL
Total iron binding capacity  550 ug/dL (normal 300 to 360 ug/dL)
Peripheral blood smear shows hypochromic, microcytic red blood cells. Which of the following
is the most appropriate next step in management? 
Correct Answer ImageA.Colonoscopy
Incorrect Answer ImageB.Schilling test
Incorrect Answer ImageC.Serum haptoglobin assay
Incorrect Answer ImageD.Serum hemoglobin electrophoresis

A 25-year-old man is brought to the emergency department immediately following a cycling


accident. His medical history is unremarkable and he takes no medications. He immigrated to the
United States 4 years ago and his immunization history is unknown. Examination shows deep
abrasions over the left thigh, shoulder, and forearm, with visible dirt in the wounds. All wounds
are debrided, cleaned, and dressed. Which of the following is the most appropriate next step in
management? 
Incorrect Answer ImageA.Tetanus-diphtheria-acellular pertussis (Tdap) alone
Correct Answer ImageB.Tetanus-diphtheria-acellular pertussis (Tdap) and tetanus immunoglobulin
(TIG)
Incorrect Answer ImageC.Tetanus-diphtheria booster (Td) alone
Incorrect Answer ImageD.Tetanus-diphtheria booster (Td) and tetanus immune globulin (TIG)
Incorrect Answer ImageE.Tetanus immune globulin (TIG) alone

A 55-year-old man with non-Hodgkin lymphoma is admitted to the hospital for administration of
chemotherapy. He is administered cyclophosphamide, vincristine, and prednisone. One day later,
he develops paresthesias, muscle weakness, and fatigue. Two days after administration of
chemotherapy, ECG shows peaked T waves, widened QRS, and prolonged QT. Which of the
following findings is most likely in this patient? 
Incorrect Answer ImageA.Calcium level of 11 mg/dL
Incorrect Answer ImageB.Creatinine level of 1.0 mg/dL
Correct Answer ImageC.Phosphate level of 5 mg/dL
Incorrect Answer ImageD.Potassium level of 2.5 mg/dL
Incorrect Answer ImageE.Uric acid level of 6.5 mg/dL

A 29-year-old man comes to the clinic concerned about worsening shortness of breath. He states
he suffers chest tightness, a mild wheeze, and occasional cough. These symptoms have persisted
for the last year and have become progressively more debilitating. The symptoms are worse
during the day, seem to be worse as the week progresses, diminish over the weekend, and are not
related to exertion. He reports no previous medical illnesses and smoked only occasionally in
college. He denies any history of allergies. Until recently, he has not suffered from respiratory
symptoms. His blood pressure is 121/73 mm Hg, pulse is 72/min, respirations are 20/min, and
temperature is 37.3°C (99.1°F). Pulse oximetry shows a saturation of 99% on room air. Physical
examination is normal. Pulmonary function tests are shown below:
Measurement Result % of
Predicted

Forced vital capacity (FVC) 4.52 L 100

Forced expiratory volume in 1 sec (FEV1)  2.82 L 75

Ratio of FEV1 to FVC  0.62 80


Measurement Result % of
Predicted

Carbon monoxide diffusing capacity 36.4 mL 109


(DLCO)

FEV1 after bronchodilator use  3.79 L 107


The cause of this patient's respiratory condition can best be determined by further history on
which of the following factors? 
Incorrect Answer ImageA.Drug and alcohol use
Incorrect Answer ImageB.Emotional stress at the workplace
Incorrect Answer ImageC.Exercise
Correct Answer ImageD.Occupational exposures
Incorrect Answer ImageE.Weekend activity, including hobbies

A 28-year-old man is brought to the emergency department after hitting a pole with his
motorcycle. On impact, he lurches forward and strikes his right upper abdomen against the
handlebars. He remains conscious after the accident and calls 911 from his cell phone. When
paramedics arrive, his blood pressure is 143/72 mm Hg and pulse is 105/min. There are
contusions and tenderness over his right upper quadrant as well as pain with inspiration. On
arrival at the emergency department his vital signs are unchanged. Head and neck examination is
unremarkable and he actively moves all his extremities. Abdominal examination shows
ecchymoses over the right upper quadrant and tenderness to palpation. Extremity examination is
unremarkable. CT scan of the abdomen shows hematoma formation around the liver and active
extravasation of contrast from a hepatic artery branch. The patient is taken to the fluoroscopy
suite, where the hepatic artery proper is embolized. Which of the following is the most likely
consequence of the proper hepatic artery embolization?
Correct Answer ImageA.Gallbladder ischemia
Incorrect Answer ImageB.Gastric varices
Incorrect Answer ImageC.Hepatic infarct
Incorrect Answer ImageD.Portal hypertension
Incorrect Answer ImageE.Splenic infarct

A 69-year-old woman with a history of diabetes type 2, chronic renal insufficiency (baseline
creatinine 2.3 mg/dL), and hypertension has shortness of breath and right-sided pleuritic chest
pain. She denies fevers, chills, or cough. Her temperature is 38.1°C (100.6°F), pulse is 123/min,
respirations are 29/min, and blood pressure is 130/70 mm Hg. Physical examination shows a
swollen and tender right calf. Pulmonary examination is normal. A chest x-ray shows no active
disease. Which of the following is the most appropriate diagnostic test?
Incorrect Answer ImageA.D-dimer studies
Incorrect Answer ImageB.Lower-extremity venogram
Incorrect Answer ImageC.MR angiogram
Incorrect Answer ImageD.Spiral CT angiogram of the chest
Correct Answer ImageE.Ventilation-perfusion scan
A 55-year-old man comes to the physician for a routine health maintenance examination. This is
his first physical in several years. He reports a 15-lb (6.8-kg) weight gain over the past several
years. His past medical history is unremarkable. He is currently taking over-the-counter vitamin
supplementation and orlistat for weight loss which he has not taken on a regular basis. He denies
drinking alcohol or smoking. On physical examination, the patient is in no apparent distress. His
temperature is 36.5°C (97.7°F), blood pressure is 135/90 mm Hg, pulse is 78/min, and
respirations are 14/min. His height is 5 ft 5 in (165 cm), and weight is 190 lb (90.1 kg). Head,
neck, cardiovascular, and abdominal examinations are unremarkable. There is no peripheral
clubbing, cyanosis, or edema. Laboratory studies show:
Sodium 139 mEq/L
Potassium 4.6 mEq/L
Chloride 104 mEq/L
Bicarbonate 23 mEq/L
Creatinine 1.0 mg/dL
Glucose 135 mg/dL
ALT 150 IU/L
AST 144 IU/L
Alkaline phosphatase 133 IU/L
Total bilirubin 1.0 mg/dL
TSH 4.9 mU/L
Viral hepatitis studies are pending. Ultrasound of the right upper quadrant shows heterogeneous
echotexture of the liver and no gallstones. Which of the following is most likely contributing to
this patient’s condition?
Incorrect Answer ImageA.Acute hepatitis B
Incorrect Answer ImageB.Hypothyroidism
Correct Answer ImageC.Impaired glucose tolerance test
Incorrect Answer ImageD.Orlistat treatment
Incorrect Answer ImageE.Vitamin E supplementation

A 57-year-old man comes to the physician because of progressively increasing dyspnea and
cough over the past 3 months. He used to walk up to half a mile per day until a few months ago
but is unable to do so now because of excessive fatigue and shortness of breath. He now has
dyspnea at rest as well. He has a history of hypertension and type 2 diabetes mellitus controlled
with diet. His home blood glucose measurement log shows a range of fasting values from 160 to
180 mg/dL. Current medications include lisinopril and hydrochlorothiazide. He has no known
drug allergies. He has smoked 2 packs of cigarettes daily for 40 years, and he drinks one shot of
whiskey every night. He weighs 102 kg (225 lb) and is 178 cm (70 in) tall. His temperature is
37.2ºC (99.0ºF), pulse is 82/min, respirations are 16/min, and blood pressure is 152/88 mm Hg.
Pulse oximetry on room air shows an oxygen saturation of 94%. The lungs are hyperresonant to
percussion, and bilateral basilar wheezes are heard on auscultation. A grade 2/6 systolic ejection
murmur is heard along the upper right sternal border that radiates to the carotids bilaterally. An
S4 gallop is audible as well. There is no pedal edema. Pulmonary function testing shows that the
FEV1 is decreased, and the FEV1/FVC ratio is 60% of the predicted value. Which of the
following is the most appropriate preventive measure recommended for this patient at this time?
Incorrect Answer ImageA.Add insulin to his current drug regimen
Incorrect Answer ImageB.Coronary artery angiography with stenting
Incorrect Answer ImageC.Encourage rapid cessation of alcohol use
Incorrect Answer ImageD.Encourage standard dietary modifications
Correct Answer ImageE.Pneumococcal polysaccharide vaccine

A 71-year-old man comes to the emergency department with fever and cough. He has known
hypercholesterolemia and had a right hemicolectomy for colon cancer 2 weeks ago. The patient
states that he has had 3 days of fever to 38.9ºC (102.0ºF), cough productive of green sputum, and
general malaise and weakness. Physical examination shows decreased breath sounds at the left
base, left basilar egophony, and dullness to percussion. Complete blood count shows leukocyte
count 15,000/mm3. Chest radiograph shows a left lower lobe infiltrate. He has not received any
immunizations in the past 10 years. Which of the following is the most important part of the
history to ascertain before initiating therapy? 
Incorrect Answer ImageA.Immunization status
Incorrect Answer ImageB.Invasive GI procedure
Incorrect Answer ImageC.Plasma lipid profile
Correct Answer ImageD.Recent hospitalization
Incorrect Answer ImageE.Social support structure for home therapy
Incorrect Answer ImageF.Stage of the colon cancer

A 65-year-old man is admitted to the hospital with severe lower back pain. He is found to have
lytic bone lesions and a compression fracture of the L4 vertebra. Further diagnostic studies show
monoclonal IgG in the serum and bone marrow plasmacytosis. A diagnosis of multiple myeloma
is made. He also has a history of coronary artery disease, hypertension, hyperlipidemia, and
arthritis. Today he complains of generalized weakness and some shortness of breath. His
temperature is 36.9°C (98.4°F), pulse is 94/min, respirations are 22/min, and blood pressure is
140/78 mm Hg; oxygen saturation is 97% on room air. Physical examination shows an elderly
man in no distress. Heart, lung, and abdominal exams are normal. Back examination shows point
tenderness of the lumbosacral vertebrae. Laboratory studies show:
arterial pH 7.35
PaCO2 29 mm Hg
PaO2 89 mm Hg
HCO3– 17 mEq/L
Na +
139 mEq/L
K+ 3.8 mEq/L
Cl –
113 mEq/L
Creatinine 1.0 mg/dL
BUN 18 mg/dL
Glucose 96 mg/dL
Ca2+ 10.5 mg/dL
Urine dipstick shows pH 5.1, specific gravity 1.018, 2+ glucose, and 1+ protein; urine is negative
for ketones, RBCs, and WBCs. Which of the following is the most likely underlying mechanism
of acidemia in this patient? 
Incorrect Answer ImageA.Addition of an exogenous acid to the blood

Incorrect Answer ImageB.Aldosterone deficiency


Incorrect Answer ImageC.Decreased proton pump (H+-ATPase) activity
Incorrect Answer ImageD.Increased membrane permeability with backleak of hydrogen ions
Correct Answer ImageE.Reduced proximal tubular bicarbonate reabsorption
Incorrect Answer ImageF.Tubular resistance to the action of aldosterone

A 30-year-old woman comes to the physician because of low back pain and a swollen and
painful right ankle and left knee for 1 week. She had diarrhea, abdominal cramps, and fever that
lasted for a few days 2 weeks ago. Her medical history is unremarkable and she takes no
medications. She is sexually active and uses condoms. Physical examination shows red
conjunctiva. The left knee and right ankle are tender, swollen, and erythematous. There is no
hepatomegaly, splenomegaly, or lymphadenopathy. Erythrocyte sedimentation rate is 60 mm/h.
Which of the following is the most likely diagnosis? 
Incorrect Answer ImageA.Lyme disease
Incorrect Answer ImageB.Psoriatic arthritis
Correct Answer ImageC.Reactive arthritis
Incorrect Answer ImageD.Rheumatoid arthritis
Incorrect Answer ImageE.Syphilitic infection

A 26-year-old woman comes to the emergency department because of sudden onset of severe
pain in her left lower extremity for the past hour. She also reports that her left toes appear black.
Her past medical history is significant for 3 spontaneous second-trimester abortions in the last
three years. She has no plans to become pregnant again at this time. She takes no medications.
Physical examination of the left limb shows a pale, cool extremity with a delayed capillary refill.
An urgent doppler ultrasound shows absent distal pulses in the left lower extremity. She
undergoes an angiogram which confirms arterial thrombosis, and she is scheduled for an
emergency surgical thrombectomy. Laboratory studies show positive anticardiolipin antibodies
and lupus anticoagulant. PTT is prolonged. A diagnosis of antiphospholipid antibody syndrome
is made. Which of the following is the most appropriate long-term therapy for this patient?
Incorrect Answer ImageA.Daily lifelong aspirin
Incorrect Answer ImageB.Daily lifelong clopidogrel
Correct Answer ImageC.Lifelong warfarin
Incorrect Answer ImageD.Low molecular-weight heparin until she delivers a baby
Incorrect Answer ImageE.Warfarin for 1 year

A 56-year-old man comes to the emergency department because of burning with urination and an
inability to empty his bladder completely for 1 week. He was recently diagnosed with non-
Hodgkin lymphoma and is being treated with rituximab, cyclophosphamide, doxorubicin,
vincristine, and prednisone (R-CHOP). He is currently on his second of four cycles of
chemotherapy. He noticed last week that he had some burning during micturition and dark-
colored urine. His temperature is 37.3ºC (99.1ºF), blood pressure is 175/93 mm Hg, and pulse is
115/min. He appears to be in moderate distress. There is palpable cervical adenopathy. His lungs
are clear to auscultation and there are no heart murmurs. The abdomen is soft in the upper
quadrants with an 8 x 8 cm mass in the suprapubic region that is slightly tender to palpation.
Genital examination shows blood clots at the urethral meatus with normal-sized testis and
without a palpable testicular mass. Rectal examination shows normal anal tone with a mildly
enlarged smooth prostate. Hematocrit is 31% and creatinine is 0.8 mg/dL. Urinalysis shows 3-4
white blood cells/hpf, packed red blood cells and a negative leukocyte esterase negative. Which
of the following is the most likely explanation for this patient’s hematuria?
Incorrect Answer ImageA.Benign prostatic hyperplasia (BPH)
Incorrect Answer ImageB.Bladder cancer
Correct Answer ImageC.Cyclophosphamide use
Incorrect Answer ImageD.Doxorubicin use
Incorrect Answer ImageE.Vincristine use

A 79-year-old man comes to the physician because of stable angina that is not accompanied by
syncope or dyspnea. Physical examination shows a displaced apical pulse to the sixth intercostal
space, midaxillary line. There is a delayed carotid upstroke. There is also the presence of a
systolic thrill. There is a harsh crescendo-decrescendo systolic murmur heard best at the second
right intercostal space and radiating to the carotids. The murmur decreases in intensity when the
patient is asked to perform a Valsalva maneuver and increases after he is asked to squat. An ECG
shows left ventricular hypertrophy. Which of the following is the most likely diagnosis?
Incorrect Answer ImageA.Autosomal-dominant inheritance
Correct Answer ImageB.Calcification of the aortic valve
Incorrect Answer ImageC.Calcification of the bicuspid aortic valve
Incorrect Answer ImageD.Ischemic heart disease
Incorrect Answer ImageE.Rheumatic fever

A 55-year-old man comes to the physician because of vertigo and disequilibrium while
exercising. He states that his symptoms are precipitated by performing exercises with his arms or
by raising his arms above his head. He has a history of coronary artery disease and hypertension,
and he is currently receiving metoprolol, aspirin, and lisinopril. His blood pressure in his left arm
is 110/60 mm Hg and his pulse is 65/min. His oxygen saturation is 96% on room air. Physical
examination shows bilateral carotid bruits. No jugular venous distension is appreciated. The
chest is clear to auscultation, and there are no abnormal heart sounds. The abdomen is nontender,
and peripheral examination shows no cyanosis, clubbing, or edema. Which of the following is
the most appropriate next step in patient care? 
Incorrect Answer ImageA.Aortic angiography
Incorrect Answer ImageB.Chest x-ray
Incorrect Answer ImageC.Duplex ultrasonography of the carotids
Correct Answer ImageD.Measure blood pressure in the right arm
Incorrect Answer ImageE.MRI angiography of the head
Incorrect Answer ImageF.Spiral CT angiography of the head and neck

A 22-year-old woman comes to the physician because of a severe headache that began 8 hours
ago. The pain is located on the right side of her head and is made worse by head motion or
sneezing. She has had ringing in her left ear, tenderness over her face and scalp on the right side
of her head, and severe pain with brushing her hair. She has had some relief by lying down in a
dark, quiet room. She has had no change in vision. Her temperature is 37.1°C (98.8°F), blood
pressure is 110/70 mm Hg, and pulse is 95/min. Examination shows photophobia. The remainder
of the examination shows no abnormalities. Modulation of which of the following
neurotransmitters would be most effective in treating this patient's disease in an acute setting?
Incorrect Answer ImageA.Dopamine
Incorrect Answer ImageB.Gamma-aminobutyric acid (GABA)
Incorrect Answer ImageC.Histamine
Incorrect Answer ImageD.Norepinephrine
Correct Answer ImageE.Serotonin

A 22-year-old man comes to the physician because of non-bloody diarrhea and increased
flatulence for 5 days. He returned from a camping trip in the Rocky Mountains 2 weeks ago. His
temperature is 36.9°C (98.4°F), pulse is 78/min, and blood pressure is 115/75 mm Hg.
Abdominal examination shows mild tenderness in the midepigastric area. Rectal examination
shows soft stool that is guaiac-negative. Which of the following is the most likely cause of his
symptoms? 
Incorrect Answer ImageA.Entamoeba histolytica
Incorrect Answer ImageB.Enteroinvasive Escherichia coli
Incorrect Answer ImageC.Enterotoxigenic E. coli
Correct Answer ImageD.Giardia lamblia
Incorrect Answer ImageE.Shigella dysenteriae

A 50-year-old man with a history of hypertension presents for a regular health maintenance visit.
His only medication has been hydrochlorothiazide. He denies ever using alcohol. On physical
examination, the vital signs are within normal limits. Examination of the lungs is normal.
Cardiac examination shows a grade 2/6 flow murmur over the aortic area. Chest x-ray is normal.
Laboratory studies show:
White blood cells 6,200/mL
Hematocrit 30%
MCV 65 fl
RBC count Normal
RDW Normal
Platelets 253,000/mm3
Which of the following is the most likely cause of this patient's anemia?
Incorrect Answer ImageA.Anemia of chronic disease
Incorrect Answer ImageB.Iron deficiency
Incorrect Answer ImageC.Colon cancer
Incorrect Answer ImageD.Sideroblastic anemia
Correct Answer ImageE.Thalassemia

A 59-year-old man comes to his primary care physician with 5 days of fever and chills associated
with a productive cough. He has not been hospitalized recently, lives at home with his wife, and
has no sick contacts. His past medical history is significant for osteoarthritis for many years. He
routinely takes only a nonsteroidal anti-inflammatory agent for pain. He has a long smoking
history of >100 pack-years. On physical examination he is comfortable, his temperature is
38.4ºC (101.1ºF), pulse is 80/min, and respirations are 16/min. He has bibasilar crackles heard
best at the left base. Which of the following is the most appropriate next step in diagnosis? 
Incorrect Answer ImageA.Arterial blood gas test
Correct Answer ImageB.Chest radiograph
Incorrect Answer ImageC.Complete blood count
Incorrect Answer ImageD.CT scan of the chest
Incorrect Answer ImageE.Sputum Gram stain

A 56-year-old man is brought to the emergency department by his son immediately after losing
his balance and falling. His son says that the patient's gait has been steadily deteriorating and he
has had multiple falls. In the past two weeks, the patient has experienced several episodes of
urinary incontinence. He has had no fever, weight loss, headache, or loss of consciousness. He
has a history of type 2 diabetes mellitus, hypertension, and one episode of pneumonia. Current
medications include insulin, lisinopril, hydrochlorothiazide, and aspirin. His temperature is 37°C
(98.6°F), blood pressure is 134/84 mm Hg, and pulse is 85/min. Neurologic examination shows a
wide-based, unsteady gait. Muscle strength is 5/5 in all extremities; reflexes are normal. Mini-
mental status examination shows a score of 26/30. A CT scan of the head shows enlarged
ventricles. Which of the following is the most appropriate next step in management? 
Correct Answer ImageA.Large-volume lumbar puncture
Incorrect Answer ImageB.Neuropsychiatric testing
Incorrect Answer ImageC.Referral for physical therapy
Incorrect Answer ImageD.Referral to a nursing home
Incorrect Answer ImageE.Urinalysis

A 42-year-old man comes to the physician because of bloody nasal discharge and cough
productive of blood-streaked sputum. His medical history is unremarkable and he takes no
medications. His temperature is 37.7ºC (99.9ºF) and pulse is 80/min. Examination shows nasal
mucosa swelling and an ulcer located on the nasal septum. Bilateral crackles are heard on
auscultation. Leukocyte count is 12,500/mm3. Serum studies are positive for antineutrophil
cytoplasmic antibodies. Urinalysis shows microscopic blood, red blood cell casts, and 2+ protein.
Which of the following is the most appropriate pharmacotherapy?
Incorrect Answer ImageA.Azathioprine
Correct Answer ImageB.Cyclophosphamide and prednisone
Incorrect Answer ImageC.Methotrexate
Incorrect Answer ImageD.Plasmapheresis
Incorrect Answer ImageE.Prednisone

A 45-year-old man comes to the physician because of weakness, fatigue, nausea, abdominal
pain, and feeling faint when he stands up quickly for the past 2 weeks. His blood pressure is
90/65 mm Hg. Laboratory studies show:
Na+ 128 mEq/L 
K+ 5.2 mEq/L 
Cl −
100 mEq/L
HCO3− 17 mEq/L
Urea nitrogen (BUN) 45 mg/dL
Creatinine 0.9 mg/dL 
Which of the following physical findings is most likely to be found in this patient? 
Correct Answer ImageA.Black freckles on the shoulders
Incorrect Answer ImageB.Large, furrowed tongue
Incorrect Answer ImageC.Protruding eyeballs
Incorrect Answer ImageD.Small glistening bumps on the lips
Incorrect Answer ImageE.Spider angiomas
A 65-year-old man comes to the physician because of a 2-month history of low back pain that
starts while walking. Ibuprofen does not relieve his symptoms. He has a history of
hyperlipidemia, type 2 diabetes mellitus, and hypertension. Current medications include
simvastatin, metformin, and lisinopril. He works as a computer engineer. He has smoked one
pack of cigarettes daily for 33 years. An MRI scan of the spine shows osteophytes and a
decreased spinal canal area. Which of the following is most likely to be seen in this patient? 
Incorrect Answer ImageA.Enthesopathy
Incorrect Answer ImageB.Improvement with activity
Incorrect Answer ImageC.Lhermitte sign
Incorrect Answer ImageD.Positive crossed-straight leg raise
Correct Answer ImageE.Pseudoclaudication

A 37-year-old man comes to the physician because of a 4-month history of multiple papules and
plaques on his body. He has also had blood in his stool for 3 weeks and an 11-kg (25-lb) weight
loss during the past year. He takes no medications. Examination is shown. There are similar
lesions over the trunk, lower extremities, and upper extremities. There is a 1-cm purple lesion on
the hard palate. This patient's skin lesions are most likely related to which of the following
conditions?
Correct Answer ImageA.Acquired immune deficiency syndrome (AIDS)
Incorrect Answer ImageB.Contact allergy
Incorrect Answer ImageC.Hemophilia A
Incorrect Answer ImageD.Melanoma
Incorrect Answer ImageE.Streptococcal infection

A 59-year-old fisherman comes to the emergency department because of a one-day history of


progressive pain and erythematous swelling of the left hand and blisters on his left arm. While
shucking oysters the prior day, he reports that his left hand slipped, causing a small puncture
wound on his hand. Since then, the site has become progressively more inflamed and swollen.
He has a history of alcohol-related cirrhosis and currently drinks approximately 10 beers per
day.His temperature is 38.3ºC (100.9ºF), pulse is 105/min, respirations are 14/min, and blood
pressure is 105/70 mm Hg. Physical examination of his left hand shows multiple erythematous,
violaceous, bullous lesions with a central abrasion and serous drainage. There is no crepitation,
and the wound is only mildly tender to palpation. Blood cultures are drawn, and he is started on
IV antibiotics. Despite prompt treatment over the next 12 hours, he becomes severely
hypotensive and requires intubation and vasopressor support in the intensive care unit. One day
after admission, the blood cultures show a species of motile, gram-negative bacteria that appear
to be shaped almost like small commas. Which of the following organisms is the most likely
pathogen causing this patient's septicemia?
Incorrect Answer ImageA.Vibrio alginolyticus
Incorrect Answer ImageB.Vibrio cholerae
Incorrect Answer ImageC.Vibrio damsela
Incorrect Answer ImageD.Vibrio parahaemolyticus
Correct Answer ImageE.Vibrio vulnificus
A 33-year-old woman is brought to the emergency department after losing consciousness at
work. She was with her office colleagues celebrating and toasting her recent promotion when she
suddenly dropped to the floor. Witnesses report she was out for less than a minute and felt fine
afterward. They deny vigorous tonic-clonic movements or incontinence. The patient reports
lightheadedness and diaphoresis with nausea just before the event. This is the first time this has
happened. Her temperature is 36.7ºC (98.0ºF), pulse is 90/min, respirations are 15/min, and
blood pressure is 113/70 mm Hg. Examination shows flat jugular veins. There is moderate pain
in the left temporal area and shoulder and normal heart sounds with no rubs or gallops. Lungs are
clear and neurologic examination is unremarkable. Which of the following is the most likely
diagnosis?
Incorrect Answer ImageA.Cardiogenic syncope
Incorrect Answer ImageB.Jarisch-Bezold reflex
Correct Answer ImageC.Neurocardiogenic syncope
Incorrect Answer ImageD.Neurogenic syncope
Incorrect Answer ImageE.Orthostatic hypotension

A 52-year-old man comes to the physician because of widespread pruritic coin-shaped lesions on
his buttock and extremities for the past 2 months. The lesions began as pruritic patches of
vesicles and papules and later started oozing and crusting. He states that some of the lesions
healed and reappeared at the same sites. The patient has a history of chronically dry skin. Skin
examination shows crusted erythematous patches and plaques on the extensor surfaces of the
extremities and on the buttocks. Which of the following is the most likely diagnosis?
Correct Answer ImageA.Nummular dermatitis
Incorrect Answer ImageB.Pompholyx
Incorrect Answer ImageC.Psoriasis
Incorrect Answer ImageD.Seborrheic dermatitis
Incorrect Answer ImageE.Stasis dermatitis

A 58-year-old man comes to the emergency department after vomiting a "cupful" of blood. The
patient reports that he was watching television when this happened. This is the first time he has
experienced such an event. His past medical history is remarkable for gallstone pancreatitis,
which was diagnosed and treated approximately 6 months ago. He underwent an elective
cholecystectomy 2 months later. He denies alcohol consumption and reports smoking two packs
per day for the past 20 years. His temperature is 36.7°C (98.0°F), blood pressure is 110/70 mm
Hg, pulse is 88/min, and respirations are 13/min. Head and neck examination is normal without
scleral icterus. Skin examination is normal without evidence of spider angiomata or palmar
erythema. Cardiopulmonary examination shows no abnormalities. There is mild epigastric
tenderness to palpation. Nasogastric lavage shows the presence of blood clots. Ultrasonography
of the liver shows normal echotexture and a normal-sized portal vein with forward flow. The
patient has an upper endoscopy, which shows several gastric varices treated with local sclerosis.
Which of the following underlying conditions is most likely responsible for this man's
condition? 
Incorrect Answer ImageA.Budd-Chiari syndrome
Incorrect Answer ImageB.Cirrhosis
Incorrect Answer ImageC.Gastritis
Incorrect Answer ImageD.Portal vein thrombosis
Correct Answer ImageE.Splenic vein thrombosis

A 64-year-old patient with a history of chronic obstructive pulmonary disease (COPD) comes to
the emergency department because of productive cough and shortness of breath. His temperature
is 39.6ºC (103.0ºF), pulse is 99/min, respirations are a 45/min, and blood pressure is 140/86 mm
Hg. On physical examination, the patient is in severe distress. There are sternocleidomastoid and
intercostal muscle retractions, and no jugular venous distention. Auscultation shows a loud P2,
diffuse wheezing, and loud crackles. The physician orders an arterial blood gas and a chest x-ray.
While waiting for the results of the tests, the patient starts to become severely confused and the
decision is made to intubate him. Arterial blood gases show a pH of 7.10, PaCO2 of 60, and
PaO2 of 55 on 100% oxygen. Chest x-ray shows diffuse alveolar edema and a normal-sized heart.
Which of the following is the most appropriate next step in management?
Incorrect Answer ImageA.Administration of corticosteroids
Incorrect Answer ImageB.High-oxygen ventilation
Incorrect Answer ImageC.Keeping PCO2 in the normal range
Incorrect Answer ImageD.Low PEEP
Correct Answer ImageE.Low tidal volume ventilation

ReKap

Low tidal volume ventilation at 6 cc/kg of ideal body weight reduces mortality in ARDS. 
Permissive hypercapnia and high-PEEP also are indicated in ARDS management.

A 48-year-old woman with diet-controlled type 2 diabetes mellitus comes to the physician
because of difficulty swallowing both solids and liquids for the last year. She also reports a 20 lb
weight loss over the past year. She has no history of heartburn and takes no medications.
Abdominal examination is normal. Chest x-ray shows an air-fluid level in the mediastinum.
Barium swallow fluoroscopy shows a dilated esophagus with pointlike tapering. Manometry
shows incomplete relaxation of the lower esophageal sphincter and no peristalsis in the proximal
part of the esophagus. Which of the following is the most appropriate next step in management?
Incorrect Answer ImageA.Botulinum toxin
Incorrect Answer ImageB.Calcium channel blockers and nitrates
Correct Answer ImageC.Endoscopy
Incorrect Answer ImageD.Pneumatic dilation
Incorrect Answer ImageE.Surgical myotomy

An 18-year-old college student comes to the student health clinic because she is concerned that
she may have "caught something" from her roommate. Her roommate recently returned from a
trip to Mexico; the roommate has had high fevers, nausea, severe abdominal pain, and jaundice.
The patient did not accompany her roommate to Mexico and denies any symptoms. She plans to
travel to South America in 4 months. The infected roommate's records show that she has acute
hepatitis A infection. Which of the following is the most appropriate next step in management
for this patient? 
Incorrect Answer ImageA.Antiviral therapy
Incorrect Answer ImageB.Hepatitis A serology, treat if positive
Correct Answer ImageC.Hepatitis A vaccination
Incorrect Answer ImageD.Prednisone for a 7-day tapering course
Incorrect Answer ImageE.Reassurance and treatment if she develops symptoms

A 56-year-old man with a long history of smoking and a diagnosis of chronic obstructive
pulmonary disease comes to the office with increasing shortness of breath with exertion. He
denies any fevers or increased sputum production. He denies recent travel. His temperature is
37°C (98.6°F), blood pressure is 142/88 mm Hg, pulse is 97/min, and respirations are 19/min.
Neck examination shows a distended internal jugular vein. Cardiac examination shows a right
ventricular heave and a prominent P2. A 2/6 holosystolic murmur is heard at the left lower
sternal border. Pulmonary examination shows a prolonged expiratory phase. Abdomen is
distended with a fluid wave and his peripheral examination is notable for 2+ pitting edema in the
shins. Chest radiograph shows hyperexpansion and enlarged pulmonary arteries with no areas of
consolidation or edema. Complete blood cell count shows WBC 5,400/mm3, hematocrit 48%,
and platelets 254,000/mm3. Which of the following is the most likely pathophysiology
underlying this patient's constellation of symptoms?
Incorrect Answer ImageA.Chronic pulmonary thromboembolism
Incorrect Answer ImageB.Elevated left heart pressures
Correct Answer ImageC.Hypoxic vasoconstriction
Incorrect Answer ImageD.Increased pulmonary blood flow
Incorrect Answer ImageE.Left-sided valvular disease

A 23-year-old African American man with sickle cell anemia (hemoglobin SS) comes to the
emergency department with two days of painless, gross hematuria, but without dysuria, fever, or
chills. He has had polyuria and nocturia for several years. There is no history of diabetes
mellitus. His temperature is 37.0°C (98.6°F), pulse is 94/min, respirations are 15/min, and blood
pressure is 105/40 mm Hg. He has a “tower” skull and mild scleral icterus. There is no
costovertebral angle tenderness, and no dependent edema. Laboratory studies show:
Sodium 145 mEq/L
Potassium 5.4 mEq/L
Chloride 100 mEq/L
Bicarbonate 19 mEq/L
Glucose 106 mg/dL
BUN 25 mg/dL
Creatinine 1.5 mg/dL
Urinalysis
Color Red 
Specific gravity 1.010
pH 7.35
Protein: 1+
Blood 4+
Glucose Negative
Bilirubin Negative
Leukocyte esterase Negative
Nitrite Negative
Casts None
WBC None
RBC Too numerous to count
Bacteria None
In a spot urine sample, the protein/creatinine ratio is 1.0 gm/gm. A low-dose CT scan is negative
for urolithiasis. Pathologic examination of the kidney from this patient would most likely show
which of the following findings?
Incorrect Answer ImageA.Diffuse red material in the glomeruli and surrounding structures when
stained with Congo red
Incorrect Answer ImageB.Increased thickness of the basement membrane with Kimmelstiel-Wilson
nodules
Correct Answer ImageC.Necrosis of the papilla of the kidney with preservation of the outline of the
tubules and collecting ducts
Incorrect Answer ImageD.Widespread fusion of epithelial foot processes seen with electron
microscopy with a normal appearance using light microscopy
Incorrect Answer ImageE.Widespread glomerular basement membrane thickening with proliferation
of the mesangial and endothelial cells

A 72-year-old woman comes to the physician because of a 3 day history of mid-back pain. She
denies any trauma to the back. Examination shows point tenderness over the T8 vertebra. 
Laboratory studies show:
Hemoglobin 13 g/dL
Na+ 137 mEq/L
Cl –
101 mEq/L
K+ 4.3 mEq/L
HCO3 –
25 mEq/L
Urea nitrogen (BUN) 21 mg/dL
Creatinine 0.71 mg/dL
Albumin  3.6 g/dL
Ca+ 9.0 mg/dL 
Alkaline phosphatase  50 U/L
X-ray of the spine shows a compression fracture of the T8 vertebral body. Dual energy x-ray
absorptiometry (DEXA) shows a bone mineral density T-score of -2.7. Which of the following is
the most likely diagnosis? 
Incorrect Answer ImageA.Metastatic disease
Incorrect Answer ImageB.Multiple myeloma
Incorrect Answer ImageC.Osteomalacia
Incorrect Answer ImageD.Osteopenia
Correct Answer ImageE.Osteoporosis

A 60-year-old woman consults a physician because of weakness, headaches, dizziness, and


tingling in her hands and feet. Physical examination shows multiple areas of bruising on the back
of her forearms and shins. On specific questioning, she reports having five episodes of epistaxis
over the past 2 months, which she attributed to “dry air.” Complete blood cell count shows a
platelet count of 1,200,000/µL, RBC count 5,100,000/µL, and WBC count 10,500/µL with a
normal differential cell count. Review of the peripheral smear demonstrates many abnormally
large platelets, platelet aggregates, and megakaryocyte fragments. No abnormal red or white
blood cells are seen. Philadelphia chromosome studies are negative. Which of the following is
the most likely diagnosis? 
Incorrect Answer ImageA.Chronic myelogenous leukemia
Incorrect Answer ImageB.Myelofibrosis
Incorrect Answer ImageC.Polycythemia vera
Correct Answer ImageD.Primary thrombocythemia
Incorrect Answer ImageE.Secondary thrombocythemia

A 12-year-old boy presents to the office with a swollen left knee. He states that he fell a couple
of days ago. Initially he had just pain in his left knee, but in the last 24 hours he has noticed
swelling and increased pain. According to the patient's mother, he has had similar episodes in the
past involving his other knee as well as his left elbow. He has also noticed that it takes a long
time for his bleeding to stop if he has a cut or a wound. The patient has a sister who does not
seem to have any health problems. There is no family history of any bleeding disorders.
Aspiration of the knee joint shows blood and blood clots. Laboratory workup shows: 
Hemoglobin 12 g/dL
Sodium 132 mEq/L
Potassium 4 mEq/L
Chloride 101 mEq/L
Bicarbonate 26 mEq/L
BUN 20 mg/dL
Creatinine 0.9 mg/dL
Glucose 82 mg/dL
PT 11.0 seconds (Normal 11 to 15 seconds)
INR 1.0
PTT 65 seconds (Normal 25 to 40 seconds)
Mixing studies with 1:1 ratio reveal normalization of PTT. Which of the following is the most
appropriate next step in diagnosis? 
Incorrect Answer ImageA.ANA
Incorrect Answer ImageB.Antibodies to factor VIII
Incorrect Answer ImageC.Anticardiolipin antibodies
Incorrect Answer ImageD.Factor V Leiden
Correct Answer ImageE.Factor VIII levels
Incorrect Answer ImageF.Protein C deficiency

A 72-year-old woman with a history of hypertension, diabetes mellitus, and breast cancer is
brought to the emergency department with worsening shortness of breath. She was diagnosed
with breast cancer 3 years ago and is currently on hormone therapy. Her pulse is 126/min,
respirations are 20/min, and blood pressure is 90/60 mm Hg. Systolic blood pressure drops to 60
mm Hg on deep inspiration. Physical examination shows jugular venous distension. Heart sounds
are distant. An electrocardiogram is shown below. An echocardiogram is pending. Which of the
following is the most appropriate next step in management?
Incorrect Answer ImageA.Inotropic infusion
Incorrect Answer ImageB.Intra-aortic balloon pump
Incorrect Answer ImageC.IV diuretics
Incorrect Answer ImageD.IV fluid resuscitation
Correct Answer ImageE.Pericardiocentesis

A 42-year-old healthy man comes to the physician for a routine physical examination. He has a
history of cholecystectomy 2 years ago and rheumatic fever at age 15. He takes no medications
and quit smoking two years ago after smoking 1/2 pack of cigarettes daily for 23 years. His pulse
is 68/min, respirations are 14/min, and blood pressure is 154/56 mm Hg. Physical examination
shows a 2/6 blowing decrescendo diastolic murmur heard at the aortic area. Laboratory studies
show no abnormalities. Which of the following is the most appropriate advice regarding future
preventive health maintenance? 
Incorrect Answer ImageA.Annual echocardiogram
Incorrect Answer ImageB.Annual flexible sigmoidoscopy
Incorrect Answer ImageC.Annual prostate-specific antigen testing
Incorrect Answer ImageD.Antibiotic prophylaxis before dental work
Correct Answer ImageE.No further preventive measures are indicated at this time

A 56-year-old woman with metastatic breast cancer is admitted to the hospital because of back
pain and urinary incontinence. This patient was diagnosed originally with lymph node-positive,
estrogen receptor-negative, progesterone receptor-negative, Her2-Neu-negative breast cancer 3
years ago. She underwent total right mastectomy and completed adjuvant chemotherapy with
doxorubicin, cyclophosphamide, and paclitaxel. Bone scan showed positive radioisotope uptake
in T10, L3, L5, and the right 5th rib. Pleurocentesis revealed exudative effusion with cytology
positive for adenocarcinoma cells consistent with the patient's known primary diagnosis.
Approximately 2 months ago she developed progressive lower back pain. Since this morning,
she has been incontinent of urine. On physical examination, she is in mild distress from back
pain and has an indwelling urinary catheter in place. Upper extremity strength is 5/5 bilaterally,
and lower extremity strength is 4/5 bilaterally. In addition to initiating steroid therapy, which of
the following is the most appropriate initial management?
Incorrect Answer ImageA.CT scan of the thoracic and lumbar spine
Incorrect Answer ImageB.Hospice referral
Correct Answer ImageC.MRI of the thoracic and lumbar spine
Incorrect Answer ImageD.Neurosurgical consultation
Incorrect Answer ImageE.Plain x-rays of the entire spine

A 55-year-old man comes to the emergency department complaining of abdominal pain. He is


well known to the emergency department staff as he has chronic alcoholic pancreatitis and
frequently seeks pain control after alcohol consumption. Today, however, he states that the pain
is different. He reports that for the last several months he had a new chronic, dull pain located in
the left upper quadrant. Previously, the pain was located in the periumbilical region. Temperature
is 37.2ºC (99.0ºF), pulse is 98/min, respirations are 16/min, and blood pressure is 126/84 mm
Hg. Physical examination of the skin shows no jaundice or spider angiomata. The abdomen is
nontender, nondistended, and without caput medusae. The spleen tip is palpable. Laboratory
studies show:
Hematocrit 32%
Leukocytes 7,000/mm3
Mean corpuscular volume (MVC) 70 µm3
Platelets 490,000/mm3
A chemistry panel and tests of liver function are unchanged from the patient's baseline. On the
basis of these findings, he undergoes an upper endoscopy that shows large gastric varices but no
evidence of esophageal varices. Which of the following is the most likely diagnosis?
Incorrect Answer ImageA.Bile duct obstruction
Incorrect Answer ImageB.Cirrhosis
Incorrect Answer ImageC.Hepatic vein thrombosis
Incorrect Answer ImageD.Pancreatic pseudocyst
Correct Answer ImageE.Splenic vein thrombosis

A 60-year-old man has had weight loss and anorexia for the past several months. He also
describes vague epigastric discomfort that is not relieved by antacids. Endoscopy shows 2 2-cm
ulcerations with heaped-up edges on opposite walls of the antrum (“kissing ulcers”), and
multiple biopsy specimens return a diagnosis of low-grade lymphoma of the stomach (MALT-
oma). After a staging workup, he is revealed to have no metastatic or locally advanced disease
and is H. Pylori positive. What is the best treatment option for him at this point?
Correct Answer ImageA.Antibiotic therapy with amoxicillin and clarithromycin and a proton pump
inhibitor (PPI)
Incorrect Answer ImageB.Chemotherapy with 5-FU and oxaliplatin (FOLFOX)
Incorrect Answer ImageC.Cyclophosphamide based combination chemotherapy (CHOP)
Incorrect Answer ImageD.Gastrectomy
Incorrect Answer ImageE.Radiation therapy

A 32-year-old woman with hypothyroidism and depression comes to the physician because of a
3-month history of episodes of dizziness and nausea that lasts 20 to 30 seconds. During these
episodes, she feels like she is physically moving, even though she is not. The symptoms occur
when she turns her head up or to the left. She denies hearing loss, ringing in her ears, fever,
cough, runny nose, or sneezing. Current medications include levothyroxine and paroxetine.
Blood pressures are 120/70 mm Hg sitting and 122/68 mm Hg standing, pulse is 82/min, and
respirations are 16/min. There is lateral nystagmus when the patient's head is turned to the side
and the patient is placed in the supine position. Which of the following is the most likely
diagnosis? 
Incorrect Answer ImageA.Acute labyrinthitis
Correct Answer ImageB.Benign paroxysmal positional vertigo
Incorrect Answer ImageC.Ménière disease
Incorrect Answer ImageD.Orthostatic hypotension
Incorrect Answer ImageE.Stroke

A 28-year-old African American woman comes to the physician because of chest pain for 2 days.
She has otherwise been in good health. She does not drink alcohol, smoke, or take medications.
She denies any recent trauma to the chest. There is no family history of cardiovascular disease.
Her pain is constant and is exacerbated by deep inspiration. Her temperature is 36.7ºC (98ºF),
blood pressure is 120/70 mm Hg, pulse is 65/min, and respirations are 14/min. Physical
examination is normal except for tenderness on palpation of the third left costochondral junction.
Which of the following is the most appropriate next step in management?
Incorrect Answer ImageA.Chest radiograph
Incorrect Answer ImageB.Complete blood count with peripheral blood smear
Incorrect Answer ImageC.ECG
Incorrect Answer ImageD.Fluids and hydroxyurea
Correct Answer ImageE.Nonsteroidal anti-inflammatory drugs and rest

A 39-year-old woman comes to the physician because of a 3-month history of shakiness, sweaty
palms, and heart palpitations. She does not smoke cigarettes, drink caffeine, or use illicit drugs.
She drinks 1–2 glasses of wine on weekends. Her medical history is unremarkable and she takes
no medications. Her temperature is 38ºC (100.4ºF) and pulse is 100/min. Examination shows a
diffusely enlarged, nontender thyroid gland without nodules. The heart and lungs are normal.
Peripheral pulses are rapid and bounding, and her palms are moist. Laboratory studies show TSH
0.05 µU/mL and free thyroxine (FT4) 20 µg/dL. Which of the following is the most appropriate
next step in management?
Correct Answer ImageA.24-hour radioactive iodine uptake (RAIU) and scan
Incorrect Answer ImageB.Assay for thyroid-stimulating antibodies
Incorrect Answer ImageC.Fine-needle aspiration of the thyroid gland
Incorrect Answer ImageD.MRI scan of the pituitary area
Incorrect Answer ImageE.Total triiodothyronine (T3) measurement

A 20-year-old college student comes to student health services because of an itchy rash that
began as a large red mark that spread to her back, upper arms, and thighs over the past 6 days.
She had a sore throat 2 weeks ago that lasted 4 days. She denies fever, vaginal discharge,
abdominal pain, back pain, or joint pain. Current medications include oral contraceptives. She is
sexually active, and she and her partner use condoms inconsistently. Skin examination is
shown. There are multiple pink oval patches on the back, proximal upper extremities, and
proximal lower extremities. There are no lesions on the palms or soles, and there is no
lymphadenopathy. Which of the following is the most likely diagnosis?
Incorrect Answer ImageA.Acute HIV reaction
Incorrect Answer ImageB.Guttate psoriasis
Incorrect Answer ImageC.Herpes zoster
Incorrect Answer ImageD.Lyme disease
Correct Answer ImageE.Pityriasis rosea
Incorrect Answer ImageF.Pityriasis versicolor
Incorrect Answer ImageG.Secondary syphilis

A 21-year-old man who was recently diagnosed with asthma comes to see his primary care
physician because of worsening symptoms. He has been maintained on inhaled albuterol. The
patient describes episodes of shortness of breath occurring nearly daily. He also reports
symptoms of asthma at night occurring typically one time per week. His temperature is 36.6°C
(97.8°F), pulse is 103/min, blood pressure is 120/75 mm Hg, respirations are 22/min, and oxygen
saturation is 90% on room air. Physical examination shows a well-developed young man in no
distress. Auscultation of the chest shows bilateral wheezing. Heart sounds are normal with no
murmurs or gallops. His abdomen is soft and nontender, with normal bowel sounds. Pulmonary
function testing shows an FEV1/FVC that is 72% of predicted. Which of the following is the
most appropriate long-term management?
Incorrect Answer ImageA.Albuterol and salmeterol
Correct Answer ImageB.Fluticasone and salmeterol
Incorrect Answer ImageC.Prednisone and albuterol
Incorrect Answer ImageD.Salmeterol and prednisone
Incorrect Answer ImageE.Theophylline and salmeterol

An 82-year-old woman is brought to the emergency department by ambulance with chest pain.
She awoke early this morning with a dull, centralized chest pain and immediately dialed 911, as
she thought she was "having a heart attack." She states that she has had similar episodes before,
but says that none have been as severe. She began having chest pain 2 weeks prior, especially
when walking around the shopping mall. She is an ex-smoker of 45 pack-years, having quit 10
years ago after the death of her husband from an acute myocardial infarction. Her temperature is
37.2ºC (98.9ºF), pulse is 115/min, and blood pressure is 145/96 mm Hg. The patient appears to
be in pain and is sweating profusely. She is currently breathing 100% oxygen through a face
mask. There is no jugular venous distension. Her heart sounds are distant, but are regular in rate
and rhythm. An electrocardiogram shows no ST-segment elevation. She is started on aspirin,
atorvastatin, enoxaparin, atenolol, nitrates, and morphine. Which of the following is the most
appropriate next step in management?
Incorrect Answer ImageA.Add amlodipine
Incorrect Answer ImageB.Add clopidogrel
Incorrect Answer ImageC.Add enalapril
Incorrect Answer ImageD.Add IV diazepam
Incorrect Answer ImageE.Coronary angiography
Correct Answer ImageF.Measure CK-MB and troponins
Incorrect Answer ImageG.Percutaneous coronary intervention

A 70-year-old man comes to the emergency department because of a 3-day history of crampy
abdominal pain in the left lower abdomen and a low-grade fever. He has a history of
hypertension, hyperlipidemia, gout, and chronic constipation. He had a colonoscopy 5 years ago
and was told to follow-up in 10 years. Current medications include atenolol and simvastatin. His
temperature is 38.0ºC (100.4ºF), blood pressure 140/60 mm Hg, pulse is 100/min, and
respirations are 20/min. The lungs are clear to auscultation. Cardiac examination shows normal
S1 and S2 with no murmurs. Abdominal examination shows tenderness to palpation at the left
lower quadrant, without rebound or guarding. Rectal examination shows no occult blood. Which
of the following is the most likely diagnosis?
Incorrect Answer ImageA.Appendicitis
Correct Answer ImageB.Diverticulitis
Incorrect Answer ImageC.Diverticulosis
Incorrect Answer ImageD.Ischemic colitis
Incorrect Answer ImageE.Sigmoid volvulus

A 72-year-old man is brought to the physician by his wife because she has noticed that his
behavior has been "out of control" for 6 weeks. He has started cursing and passing flatus loudly
in front of others. She states that he has sudden changes in mood, and he bursts into laughter or
tears with no apparent cause. He has had an increase in appetite. He has a history of osteoarthritis
controlled with ibuprofen. His blood pressure is 128/78 mm Hg. The patient is slow to arise from
the chair. There is no cogwheel rigidity. MRI scan of the brain shows frontotemporal atrophy.
Which of the following is the most likely diagnosis? 
Incorrect Answer ImageA.Alzheimer disease
Incorrect Answer ImageB.Dementia pugilistica
Incorrect Answer ImageC.Herpes encephalitis
Incorrect Answer ImageD.Lewy body dementia
Correct Answer ImageE.Pick dementia

A 32-year-old woman with HIV comes to the emergency department because of a 1-day history
of malaise, headache, and vomiting. She states that her headache is worse under the lights of the
examination room. She is oriented to person, place and time. Her temperature is 38.2ºC
(100.7ºF), pulse is 116/min, and blood pressure is 110/76 mm Hg. Passive flexion of the patient's
neck causes involuntary flexion of the hips and knees. She also recoils during fundoscopy. There
is no papilledema. Laboratory studies show CD4+ T lymphocyte count of 455 cells/mm3 and
viral load is 50,000/mL. Which of the following is the most likely cause of this patient's
condition?
Incorrect Answer ImageA.Borrelia burgdorferi
Incorrect Answer ImageB.Cryptococcus neoformans
Incorrect Answer ImageC.Escherichia coli
Incorrect Answer ImageD.Group B Streptococcus
Incorrect Answer ImageE.Listeria monocytogenes
Incorrect Answer ImageF.Mycobacterium tuberculosis
Incorrect Answer ImageG.Neisseria meningitidis
Correct Answer ImageH.Streptococcus pneumoniae
Incorrect Answer ImageI.Treponema pallidum

A 69-year-old woman comes to the physician because of a 1-week history of cramping lower
abdominal pain and bloody diarrhea. She denies any recent travel. She has a history of stable
exertional angina, hypertension, and a myocardial infarction 3 years ago. She retired from her
job as a librarian 4 years ago. Her temperature is 37.6ºC (99.7ºF), blood pressure is 120/84 mm
Hg, and pulse is 96/min. Examination shows moderate tenderness to palpation of the left lower
quadrant. Rectal examination shows bloody stool and no masses. Which of the following is the
most likely diagnosis?
Incorrect Answer ImageA.Arteriovenous malformation
Incorrect Answer ImageB.Diverticulitis
Incorrect Answer ImageC.Diverticulosis
Correct Answer ImageD.Ischemic colitis
Incorrect Answer ImageE.Ulcerative colitis

A 59-year-old man comes to the physician for a routine physical examination. He says he tries to
exercise three times a week, but does not succeed. He also smokes occasionally and admits to
eating an unhealthy diet. He has a history of type 2 diabetes mellitus and hypertension. Current
medications include hydrochlorothiazide and metformin. BMI is 30.4 kg/m2. His blood pressure
is 148/96 mm Hg and pulse is 75/min. Physical examination shows no abnormalities. Which of
the following is the next best step in management?
Correct Answer ImageA.Add lisinopril to his regimen
Incorrect Answer ImageB.Add metoprolol to his regimen
Incorrect Answer ImageC.Recommend smoking cessation and weight loss
Incorrect Answer ImageD.Recommend the DASH diet
Incorrect Answer ImageE.Replace hydrochlorothiazide with hydralazine

A 34-year-old woman presents to her physician with blood in her urine. She has also felt malaise
and had a fever for the past day. She has inconsistent answers to further questioning. Her past
medical history is unremarkable. She does not take any medications and does not drink alcohol
or smoke. Pulse is 89/min and blood pressure is 122/71 mm Hg. Her oxygen saturation is 97%
on room air. She is slightly somnolent but easily arousable. Head and neck examination shows
mild scleral icterus. There is no jugular venous distension. The lungs are clear to auscultation,
and there are no murmurs, rubs, or gallops. Abdominal examination shows no ascites or
hepatosplenomegaly. Her peripheral examination shows a petechial rash but no clubbing,
cyanosis, or edema. Laboratory values are: 
WBC 11,800/mm3
Hct 31%
Platelets 75,000/mm3
Na +
139 mEq/L
K +
3.7 mEq/L
Cl– 105 mEq/L
Bicarbonate 21 mEq/L
BUN 28 mg/dL
Creatinine 1.5 mg/dL
Glucose 101 mg/dL
PTT 23 sec
PT 15 sec
ALT 24 U/L
AST 22 U/L
Alkaline phosphatase 92 U/L
Total bilirubin 3.3 mg/dL
Conjugated bilirubin 0.4 mg/dL
On peripheral blood smear, schistocytes are noted. Which of the following is the most likely
etiology for the patient's symptoms? 
Incorrect Answer ImageA.Disseminated intravascular coagulation (DIC)
Incorrect Answer ImageB.Evan syndrome
Incorrect Answer ImageC.Hemolytic uremic syndrome
Incorrect Answer ImageD.Immune thrombocytopenic purpura
Incorrect Answer ImageE.Liver disease
Correct Answer ImageF.Thrombotic thrombocytopenic purpura

A 54-year-old man comes to the physician because of ringing in his ears, a feeling of spinning,
and a progressive loss of hearing in his left ear. He says that this all began a while ago with a
slight feeling of unsteadiness and ringing. He never came to the doctor because he thought he
was going crazy, but now his wife is getting worried that the television needs to be much louder.
He has no chronic medical conditions, does not take any medications, and does not drink alcohol.
Examination shows nystagmus but no other abnormalities. Which of the following is the most
likely diagnosis? 
Incorrect Answer ImageA.Benign positional vertigo
Incorrect Answer ImageB.Benign recurrent vertigo
Correct Answer ImageC.Ménière disease
Incorrect Answer ImageD.Toxic labyrinthitis
Incorrect Answer ImageE.Vestibular neuronitis

A 46-year-old woman with no significant past medical history presents to the emergency room
with the sudden onset of excruciating back and leg pain. The pain began 2 hours ago when she
attempted to lift a heavy object. She says that she felt a “bolt of lightning" running down the
back of her leg with the pain preventing her from walking or moving. The pain is exacerbated by
coughing, sneezing, or straining. She keeps the affected leg flexed, with straight-leg raises
reproducing her pain. She has a normal sphincter tone and intact perineal sensation. Once the
diagnosis is confirmed, which of the following is the most appropriate treatment? 
Incorrect Answer ImageA.Administer antibiotics
Correct Answer ImageB.Pain control and wait for spontaneous resolution
Incorrect Answer ImageC.Radiotherapy to the affected area
Incorrect Answer ImageD.Steroid injection
Incorrect Answer ImageE.Urgent surgical decompression

A 45-year-old woman comes to the physician because of intermittent difficulty swallowing solid
food for 3 months. Two days ago, she had a choking sensation when eating a piece of bread. She
has had shortness of breath with exertion, and palpitations when climbing stairs. She has a
history of menorrhagia caused by leiomyomas. Her temperature is 37.0ºC (98.6ºF), blood
pressure is 110/80 mm Hg, pulse is 110/min, and respirations are 16/min. Examination shows
pale conjunctiva, erythematous and cracked skin bilaterally on the corners of the mouth, and a
smooth, enlarged tongue. The lungs are clear to auscultation. A 2/6 systolic ejection murmur is
heard at the left sternal border. The abdomen is nontender. Laboratory studies are most likely to
show which of the following? 
Correct Answer ImageA.Mean corpuscular volume of 70 µm3; decreased ferritin; increased total
iron-binding capacity (TIBC)
Incorrect Answer ImageB.Mean corpuscular volume of 70 µm3; increased ferritin; decreased TIBC
Incorrect Answer ImageC.Mean corpuscular volume of 70 µm3; increased ferritin; increased TIBC
Incorrect Answer ImageD.Mean corpuscular volume of 110 µm3; decreased folic acid
Incorrect Answer ImageE.Mean corpuscular volume of 110 µm3; decreased serum vitamin B12 

A 79-year-old woman comes to the physician because of nausea, dizziness, lightheadedness, and


fatigue for 5 days. She denies vomiting, diarrhea, constipation, or fever. She has a history of
hypertension, congestive heart failure, coronary artery disease, and emphysema. She is currently
taking atorvastatin, digoxin, lisinopril, carvedilol, and aspirin. She used to smoke one pack of
cigarettes daily for 40 years but quit 10 years ago. Her temperature is 37°C (98.6°F), pulse is
47/min, respirations are 20/min, blood pressure is 87/40 mm Hg. Oxygen saturation is 95% on
room air. On physical examination, she is pale and uncomfortable. Cardiovascular examination
shows an audible third heart sound with regular rate and rhythm. Chest examination shows a
prolonged expiratory phase, with mild crackles at the bases. There is 1+ pitting edema in the
lower extremities. ECG shows a third-degree heart block with a junctional rhythm at a rate of
45/min. Chest x-ray shows an enlarged cardiac silhouette and flattened diaphragms. Laboratory
studies show:
White blood cells 6,300/mm3
Hematocrit 45%
Platelets 225,000/mm3
Na+ 137 mEq/dL
K+ 2.1 mEq/dL
Cl −
103 mEq/dL
HCO3− 21 mEq/dL
Urea Nitrogen 30 mg/dL
Creatinine 2.2 mg/dL
AST 33 U/L
ALT 21 U/L
Alkaline Phosphatase 101 U/L
Which of the following is the best initial step in management?
Incorrect Answer ImageA.Administer dobutamine
Incorrect Answer ImageB.Administer high-dose steroids and bronchodilators
Correct Answer ImageC.Administer IV atropine
Incorrect Answer ImageD.Administer IV furosemide
Incorrect Answer ImageE.Perform transvenous pacing

A 64-year-old man comes to the emergency department with acute shortness of breath. He has a
past medical history of congestive heart failure, hypertension, and dyslipidemia. His medications
include metoprolol, furosemide, atorvastatin, aspirin, and enalapril. The family reports that for 1
week prior to admission he refused to take the furosemide because he was becoming incontinent
of his urine and was not always able to make it to the bathroom in time. His pulse is 110/minute,
respirations are 32/minute, blood pressure is 150/92 mm Hg. There are bilateral diffuse
pulmonary crackles, a cardiac S3, and 4+ edema to mid-shin. The chest X ray shows bilateral
pulmonary edema and an enlarged heart. The ECG shows only sinus tachycardia. Troponin
levels are normal. 
In the medical intensive care unit, the patient becomes increasingly hypoxic and is intubated for
respiratory support. He undergoes diuresis with IV furosemide, losing 5 kg weight in the next 48
hours. The respiratory technician now states that the patient has good oxygen saturation, and that
they have been attempting to wean the patient off the ventilator, but have failed so far. A chest x-
ray shows no evidence of pulmonary edema. Lab studies show: 
Blood:
WBC 6,000 mm3
Hemoglobin 13 g/dL
Hematocrit 38%
pH 7.55
PaCO2 55 mmHg
PaO2 100 mmHg
Serum:
Na+ 140 mEq/L
K+ 3.0 mEq/L
Cl-
90 mEq/L
HCO3- 40 mEq/L
BUN 30 mg/dL
Creatinine 1.0 mg/dL
Urinary chloride concentration 20 mEq/L
What is the best next step in the management of this patient?
Incorrect Answer ImageA.Add metolazone to furosemide
Correct Answer ImageB.Chloride and potassium repletion with potassium chloride
Incorrect Answer ImageC.Extubate the patient
Incorrect Answer ImageD.Increase the dose of furosemide
Incorrect Answer ImageE.Increase the FIO2 on the ventilator
Incorrect Answer ImageF.Wait and repeat the ABG in 2 hours

A 46-year-old woman comes to her physician and says that while applying makeup she noticed
a "small lump" move up and down in the lower part of her neck whenever she swallowed. On
examination, the physician confirms that she has a single, firm, 2-cm thyroid nodule in the right
lobe. There are no other abnormalities in the history or physical examination. Her pulse is
82/min and regular. Thyroid stimulating hormone (TSH) is within normal limits. Which of the
following is the most appropriate next step in management? 
Incorrect Answer ImageA.Biopsy via fine needle aspiration
Incorrect Answer ImageB.Clinical observation, repeating the TSH at least once a year
Incorrect Answer ImageC.Determination of T3 and T4 levels
Incorrect Answer ImageD.Radioactive iodine (RAI) thyroid scan
Correct Answer ImageE.Ultrasound of the thyroid gland

A 72-year-old woman comes to the emergency department because of a 1-day history of severe
left lower quadrant pain. She has had occasional left lower quadrant cramping after meals. She
has had no weight loss, change in stool caliber, or blood in her stool. She has a 15-year history of
constipation. Her temperature is 102.8ºF (39.4ºC). Physical examination shows tenderness to
palpation and guarding in the left lower quadrant. A CT scan of the abdomen shows an abscess
involving the sigmoid colon with a long segment of thickening of the colonic wall contiguous
with the abscess. Which of the following is the most likely cause of this patient's condition?
Incorrect Answer ImageA.Appendix epiploicae infarction
Incorrect Answer ImageB.Colon cancer with perforation
Incorrect Answer ImageC.Colonic ischemia
Correct Answer ImageD.Diverticulitis
Incorrect Answer ImageE.Perforated Crohn colitis

A 28-year-old woman comes to the physician because of a 2-year history of a lump in her right
upper arm. She has had no pain and the lump has not grown in size. She has a history of
hyperthyroidism. Current medications include atenolol and methimazole. Her mother has a
history of breast cancer. Physical examination shows a 3 cm, soft, subcutaneous mass on the
right upper lateral arm; the mass is mobile and nontender. Which of the following is the most
appropriate next step in management? 
Incorrect Answer ImageA.Chemotherapy
Correct Answer ImageB.Clinical observation
Incorrect Answer ImageC.Excision
Incorrect Answer ImageD.Intralesional corticosteroids
Incorrect Answer ImageE.Sentinel lymph node biopsy

A 35-year-old man comes to see his primary care physician for his annual appointment. He is
feeling fine and denies any symptoms. His past medical history is significant for
gastroesophageal reflux disease, for which he takes omeprazole. He works as an engineer and
does not smoke or drink alcohol. His vital signs and physical examination are normal. Complete
blood count and basic metabolic panel are within normal limits. His chest x-ray shows a 7 mm
solitary pulmonary nodule that has popcorn calcifications. There is no previous x-ray available
for comparison. Which of the following is the most appropriate next step in management? 
Incorrect Answer ImageA.Bronchoscopy with biopsy of the lesion
Incorrect Answer ImageB.Excise the lesion as soon as possible
Incorrect Answer ImageC.Needle biopsy of the lesion
Incorrect Answer ImageD.Positron emission tomographic (PET) scanning
Correct Answer ImageE.Serial CT scans every 3 months for 2 years

A 59-year-old woman comes to her health care provider because of recurrent episodes of
lightheadedness upon getting up in the morning and occasionally upon standing up from a chair.
She reports that on two occasions she has "passed out" soon after getting up from bed. She has a
history of rheumatic fever as a child and hypertension diagnosed 6 months ago. Her medications
include a beta-blocker and a thiazide diuretic. Her temperature is 37°C (98.6°F), pulse is 70/min
and regular, respirations are 14/min, and blood pressure is 130/80 mm Hg. A thorough physical
examination, including chest auscultation, is unremarkable. Which of the following is the most
likely cause of this patient's symptoms?
Correct Answer ImageA.Antihypertensive treatment
Incorrect Answer ImageB.Aortic stenosis
Incorrect Answer ImageC.Drop attacks
Incorrect Answer ImageD.Reflex syncope
Incorrect Answer ImageE.Vasovagal syncope

A 45-year-old man comes to the physician for a follow-up visit. He was hospitalized for an
esophageal variceal bleed and discharged 1 week ago. He takes no medications. He does not
drink alcohol and denies illicit drug use. Two of his paternal cousins "died of liver disease," and
his sister died of melanoma at age 47 years. Physical examination shows bronzed-appearing skin.
His sclerae are anicteric. Abdominal examination shows no abnormalities. There is no ascites or
peripheral edema. Which of the following is the most appropriate laboratory test in this patient? 
Incorrect Answer ImageA.Alpha1 antitrypsin genotype
Incorrect Answer ImageB.Alpha1 antitrypsin phenotype
Incorrect Answer ImageC.Serum copper
Correct Answer ImageD.Serum ferritin
Incorrect Answer ImageE.Serum nickel

A 34-year-old woman is brought to the emergency department in stable condition immediately


after being hit by an automobile while on her bicycle. She has had no weight loss, fever, chills,
night sweats, or cough. She denies muscle weakness or difficulty swallowing. She has a history
of depression and myasthenia gravis. Current medications include sertraline. Six months ago she
took oral prednisone for myasthenia gravis. She smoked one pack of cigarettes daily for 8 years,
but quit 5 years ago. Her maternal aunt died of breast cancer at age 68 years. There are bilateral
minor abrasions on the lower extremities, but otherwise, physical examination shows no
abnormalities. An x-ray film of the chest shows a 4 cm mediastinal mass. Which of the following
is the most likely diagnosis of this patient's mediastinal mass?
Incorrect Answer ImageA.Breast cancer
Incorrect Answer ImageB.Lung cancer
Incorrect Answer ImageC.Lymphoma
Incorrect Answer ImageD.Teratoma
Correct Answer ImageE.Thymoma

A 33-year-old woman comes to the physician because of several painless vulvar genital ulcers
that have been recurrent over the past several months. Currently, she has 2 ulcers that she
discovered a few days ago. She denies fevers, chills, muscle aches, or fatigue. She has a history
of syphilis. Past surgical history is significant for 2 cesarean deliveries. She takes no medications
and has no known drug allergies. Pelvic examination demonstrates 2 ulcerations on the labia
majora. The ulcerations are nontender and have well-defined bases with exuberant granulation
tissue. A scraping of one of the lesions reveals many mononuclear cells containing numerous
encapsulated bacilli. Which of the following is the most likely cause of these findings?
Incorrect Answer ImageA.Haemophilus ducreyi
Incorrect Answer ImageB.Human papillomavirus
Correct Answer ImageC.Klebsiella granulomatis
Incorrect Answer ImageD.Neisseria gonorrhoeae
Incorrect Answer ImageE.Trichomonas vaginalis

A 35-year-old man with a 19-year history of type 1 diabetes comes to the physician because of
progressive early satiety, and a 1-week history of abdominal bloating, nausea, and recurrent
episodes of vomiting after a large meal. He has had no fever, fatigue, or weight loss. His
temperature is 37.1°C (98.8°F), blood pressure is 115/75 mm Hg, and pulse is 80/min.
Abdominal exam shows no abnormalities. There are 2 healed ulcers on the ventral surface of his
left foot. Which of the following is the most appropriate next step in management? 
Correct Answer ImageA.Endoscopy
Incorrect Answer ImageB.Gastric scintigraphy
Incorrect Answer ImageC.Increase insulin dose
Incorrect Answer ImageD.IV erythromycin
Incorrect Answer ImageE.Octreotide

A 59-year-old Caucasian man comes to the outpatient medicine clinic with a complaint of
palpitations. The patient says that for the past two weeks, his heart has been “acting funny” and
sometimes “skips a beat”. At other times, he feels his heart racing. The patient has a past medical
history of hypertension managed by metoprolol 50 mg P.O. twice daily. He also has COPD
managed with albuterol and an ipratropium inhaler used on demand. Because of increased stress
at work, the patient has been drinking 4-6 beers a day for the past couple of months, and has
gained 20 lbs in the last six months. On physical examination, the patient is in no acute distress,
and in general, looks well. His temperature is 37.1ºC (98.8ºF), pulse 65/min, respirations are
14/min, blood pressure is 135/78 mm Hg, and pulse oximeter is 93% on room air. A 12-lead
electrocardiogram shows a normal sinus rhythm. What is the next best step in diagnosing this
patient?
Incorrect Answer ImageA.Inform the patient to return for follow-up in one month
Incorrect Answer ImageB.Order a 10-minute treadmill stress test in the outpatient clinic
Correct Answer ImageC.Order a 72-hour Holter monitor
Incorrect Answer ImageD.Order a transesophageal echocardiogram
Incorrect Answer ImageE.Order a transthoracic echocardiogram
Incorrect Answer ImageF.Order cardiac enzymes

A 55-year-old man with a history of mitral stenosis secondary to rheumatic heart disease
undergoes elective mechanical mitral valve replacement. Postoperatively he is started on a
heparin drip, with a plan to start warfarin once he is clinically stable. However, on hospital day
5, he develops pain and swelling in the left lower extremity. His toes in the left lower extremity
show evidence of ischemia. Arterial Doppler reveals thrombosis. Further testing shows a low
platelet count of 40,000/mm3. Heparin-induced antibodies are positive. Anticardiolipin
antibodies and lupus anticoagulant are negative. Heparin is discontinued. Which of the following
is the most appropriate next step in management?
Correct Answer ImageA.Argatroban
Incorrect Answer ImageB.Aspirin
Incorrect Answer ImageC.Clopidogrel
Incorrect Answer ImageD.Dalteparin and warfarin
Incorrect Answer ImageE.Enoxaparin and warfarin
Incorrect Answer ImageF.Warfarin

A 48-year-old man is brought to the emergency department by his brother after an episode of
sudden staring and lip-smacking that started 1 hour ago and lasted for 3 minutes. His brother
states that he has been somnolent and confused since the episode. The patient does not remember
the episode. He has had headaches for the past 6 months. He does not report fevers, fatigue, night
sweats, or weight loss. On examination, he appears drowsy but is oriented. Neurological
examination is unremarkable. His temperature is 37.1°C (98.8°F). A single axial CT image of the
head is shown below. There is a well circumscribed, dural-based mass in the right middle cranial
fossa that displaces the tip of the temporal lobe. The adjacent calvarial bone is mildly thickened.
Which of the following is the most likely diagnosis?
Incorrect Answer ImageA.Craniopharyngioma
Incorrect Answer ImageB.Glioblastoma multiforme
Correct Answer ImageC.Meningioma
Incorrect Answer ImageD.Paget disease
Incorrect Answer ImageE.Schwannoma

A 38-year-old man with a history of genital herpes comes to the physician because of right knee
pain and swelling. He reports that the symptoms started earlier in the day and it is extremely
painful to move or even touch his knee. He denies any trauma to the affected knee. He also
reports recent unprotected sex with a prostitute. His temperature is 39°C (102.2°F), blood
pressure 125/90 mm Hg, pulse 108/min and regular, and respirations 15/min. Physical
examination shows a swollen and tender right knee with a palpable effusion. The patient is
tachycardic but the examination is otherwise normal. An arthrocentesis is performed showing
purulent joint fluid which is sent to the laboratory for further analysis. Which of the following
findings is most likely? 
Correct Answer ImageA.Cell count with 75,000 WBC
Incorrect Answer ImageB.Culture positive for gram-positive cocci in clusters
Incorrect Answer ImageC.Gram stain revealing gram-positive diplococci
Incorrect Answer ImageD.Negative birefringent crystals
Incorrect Answer ImageE.Rhomboid-shaped crystals

A 79-year-old woman is brought to the emergency department by her daughter because of


lethargy and confusion. She has a 15-year history of hyperlipidemia. Current medications include
atorvastatin and vitamin D. Her temperature is 39.0°C (102.2°F), blood pressure 100/70 mm Hg,
pulse 123/min, and respirations 17/min. Examination shows photophobia and nuchal rigidity.
Mucous membranes are moist. The lungs are clear to auscultation. There is no edema.
Laboratory studies show: 
Na+ 114 mEq/L
K+ 3.5 mEq/L
Cl−
82 mEq/L
HCO3− 23 mEq/L
BUN 17 mg/dL
Creatinine 1.1 mg/dL
Glucose 116 mg/dL
Albumin 3.7 g/dL
INR 1.02
Hemoglobin 12 g/dL
White blood cell 27,000/mm3
Platelets 270,000/mm3
Hematocrit 31%
Urine sodium 42 mEq/dL
Plasma osmolality 270 mOsmol/kg
Urine output 10-30 mL/hour
Which of the following is the most likely cause of the hyponatremia? 
Incorrect Answer ImageA.Adrenal insufficiency
Incorrect Answer ImageB.Cirrhosis
Incorrect Answer ImageC.Psychogenic polydipsia
Incorrect Answer ImageD.Sepsis
Correct Answer ImageE.Syndrome of inappropriate antidiuretic hormone

A 40-year-old man is brought to the emergency room via ambulance because of abdominal pain
of 4 hours' duration. He describes severe, dull epigastric pain shortly after eating his evening
meal. He also feels very nauseous. He appears pale and is sweating profusely. Cardiovascular
examination shows muffled heart sounds with no murmurs, rubs, or gallops. An
electrocardiogram reveals ST-segment elevation in leads II, III, and aVF. Which of the following
findings is most specific for a diagnosis of right-sided infarction?
Incorrect Answer ImageA.Jugulovenous distension with clear lungs
Incorrect Answer ImageB.ST-segment elevation in leads II, III, and aVF
Correct Answer ImageC.ST-segment elevation in right-sided V4
Incorrect Answer ImageD.Wide Q waves in leads II, III, and aVF
Incorrect Answer ImageE.Tachycardia with hypotension

A 45-year-old woman comes to the physician because of a 4-day history of progressive fever,
flank pain, and blood in her urine. She has a history of hypertension controlled with diet and
exercise. Her temperature is 37.9ºC (100.2ºF), pulse is 105/min, respirations are 20/min, and
blood pressure is 150/88 mm Hg. Examination shows mild costovertebral angle tenderness
bilaterally. The remainder of the examination shows no abnormalities. Laboratory studies show:
Hemoglobin  11.0 g/dL 
Leukocyte count  12,500/mm3 
Platelet count  165,000/mm3 
Serum studies show:
Na+  137 mEq/L 
Cl– 98 mEq/L
K + 
4.8 mEq/L 
Blood urea nitrogen (BUN)  32 mg/dL 
Creatinine  1.5 mg/dL 
Glucose  95 mg/dL 
Calcium  8.9 mg/dL 
Urinalysis shows 85 leukocytes/HPF and 980 erythrocytes/HPF. A CT scan of the abdomen is
shown. Which of the following is the most likely diagnosis?
Incorrect Answer ImageA.Cystitis
Incorrect Answer ImageB.Glomerulonephritis
Incorrect Answer ImageC.Nephrolithiasis
Correct Answer ImageD.Polycystic kidney disease
Incorrect Answer ImageE.Pyelonephritis

A 60-year-old woman with a history of Cushing syndrome comes to the physician because of
pain and swelling in her right calf that is exacerbated by movement for 2 days. She has had no
trauma or prior injuries to her right lower extremity. She takes no medications. Vital signs are
normal. Examination shows an overweight patient with abdominal striae. Lungs are clear and
there are no murmurs, rubs, or gallops on cardiac auscultation. The right calf is asymmetrically
erythematous and edematous relative to the left leg. Radiographs of the right foot and leg show
no abnormalities. Which of the following is the most appropriate next step in diagnosis?
Incorrect Answer ImageA.Arterial duplex ultrasound of the right lower extremity
Incorrect Answer ImageB.Contrast-enhanced venography
Incorrect Answer ImageC.MRI of the right lower extremity
Incorrect Answer ImageD.No further tests, with warm compresses and elevation recommended
Correct Answer ImageE.Venous duplex ultrasound of the right lower extremity

A 30-year-old woman comes to the physician because of a 2-day history of severe watery
diarrhea. She has just returned from a camping trip in Mexico. She denies blood or mucus in the
stool. She reports having six to eight episodes of watery stools associated with lightheadedness.
She denies fever, chills, shakes, nausea, and vomiting. She reports consuming fresh fruit while in
Mexico. Which of the following is the most likely pathogen responsible for this patient's
symptoms?
Incorrect Answer ImageA.Bacillus cereus
Incorrect Answer ImageB.Enteroinvasive E. coli
Correct Answer ImageC.Enterotoxigenic E. coli
Incorrect Answer ImageD.Rotavirus
Incorrect Answer ImageE.Shigella

A 16-year-old girl comes to the physician because of a severe sunburn after mowing the lawn 1
day ago. She states that she has never had a sunburn like this before. She does not sunbathe or
use indoor tanning devices. She has a history of asthma and acne. Current medications include
inhaled fluticasone, inhaled albuterol, oral doxycycline, and topical benzoyl peroxide. Skin
examination shows deep red erythema on the malar cheeks, dorsal forearms and hands, and the
chest. The remainder of the examination shows no abnormalities. Which of the following is the
most likely cause of this patient's condition?
Incorrect Answer ImageA.Antibodies directed against nuclear antigens
Incorrect Answer ImageB.Complement-mediated vascular inflammation
Incorrect Answer ImageC.Immune-mediated reaction to ultraviolet light
Incorrect Answer ImageD.Interaction of ultraviolet light with benzoyl peroxide
Correct Answer ImageE.Interaction of ultraviolet light with doxycycline

A 33-year-old man comes to the physician for a physical examination and to have a chest x-ray
performed. It is required for his new job at an asbestos removal company. He is feeling well and
has no complaints. He takes no medications. He does not smoke cigarettes or use drugs. He has
no history of previous potentially toxic exposures. His temperature is 37.0ºC (98.6ºF), blood
pressure is 100/70 mm Hg, pulse is 63/min, and respirations are 18/min. His lungs are clear to
auscultation. He has no clubbing of his fingers. The remainder of the physical examination is
normal. Chest x-ray shows an 8 mm nodule in the left lower lobe with large, coarse, dense
calcifications. Which of the following is the most appropriate management regarding evaluation
of this mass?
Incorrect Answer ImageA.Bronchoscopy with biopsy
Incorrect Answer ImageB.Complete excision
Incorrect Answer ImageC.Needle biopsy
Incorrect Answer ImageD.No further workup is indicated
Incorrect Answer ImageE.PET scan
Correct Answer ImageF.Serial CT scans

A 55-year-old man with no known cardiac history comes to the emergency department because
of crushing substernal chest pressure that began 20 minutes prior to his arrival. He took sildenafil
earlier in the evening prior to sexual intercourse. His past medical history is significant for
bilateral inguinal hernia repairs 6 weeks ago, remote peptic ulcer disease, and prior cocaine and
heroin addiction. His blood pressure is 150/75 mm Hg with a pulse of 100/min. On examination,
he is diaphoretic and appears anxious. An electrocardiogram (ECG) obtained while he had severe
chest pain shows tall positive T waves and 1 mm elevation of the ST segments in leads V2
through V5. A serum troponin drawn on arrival is negative. Which of the following is the most
likely diagnosis?
Incorrect Answer ImageA.Aortic dissection
Correct Answer ImageB.Myocardial infarction (MI)
Incorrect Answer ImageC.Pulmonary embolus (PE)
Incorrect Answer ImageD.Spontaneous pneumothorax
Incorrect Answer ImageE.Stable angina

A 31-year-old man comes to the physician because of a 3-week history of progressive fatigue,
low-grade fevers, and myalgias and a one-week history of pruritus and jaundice. He is sexually
active with men and women and he uses condoms inconsistently. His medical history is
unremarkable and he takes no medications. He is ill-appearing. His temperature is 39.3ºC
(102.7ºF), pulse is 114/min, and blood pressure is 116/60 mm Hg. His sclera are icteric and there
are multiple excoriations over the skin. The liver edge is smooth, tender, and palpable 5 cm
below the right costal margin. The spleen tip is palpable 4 cm below the left costal margin. There
is no shifting dullness. Rectal examination shows brown stool that is negative for occult blood.
There is no peripheral edema. Which of the following laboratory findings predicts a higher
mortality over the next 3 months?
Incorrect Answer ImageA.Alanine aminotransferase (ALT) of 13,420 U/L
Incorrect Answer ImageB.Alkaline phosphatase (AP) of 282 U/L
Incorrect Answer ImageC.Aspartate aminotransferase (AST) of 983 U/L
Correct Answer ImageD.Prothrombin time of 19.6 seconds
Incorrect Answer ImageE.White blood cell count of 18,400/mm3

A 69-year-old woman is brought to her primary care physician for her routine yearly
examination. This year, her daughter drove her to the appointment. When asked about this, the
daughter whispers to the physician that her mother has gotten lost while driving to the
supermarket on two occasions over the past 6 months. It becomes clear during the history intake
that the patient does not remember the physician‘s name and seems to have some word-finding
difficulties. The daughter also mentions to the physician that she has recently begun managing
her mother’s checking account and other finances, due to the mother's forgetfulness. The patient
has osteoarthritis of the knees, and a history of “a treated, very serious sexually transmitted
disease many years ago,” but is otherwise healthy. Vital signs are within normal limits, and
physical examination is unremarkable. During the interview, the physician notes that the patient
shows slow thinking and impaired attention. Which of the following is the most appropriate next
step for evaluating this patient’s memory decline? 
Incorrect Answer ImageA.Computed tomography (CT) scan of the brain
Incorrect Answer ImageB.ELISA for HIV
Incorrect Answer ImageC.Lumbar puncture
Correct Answer ImageD.MMSE (Mini-Mental State Examination)
Incorrect Answer ImageE.RPR (Rapid Plasma Reagin) test

A previously healthy 18-year-old college freshman residing in a dormitory comes to the


emergency department because of progressive malaise, nausea, and vomiting for 1 day. He states
that bright lights hurt his eyes. He is alert and oriented. His temperature is 38.2ºC (100.8ºF),
pulse is 110/min, and blood pressure is 106/78 mm Hg. Physical examination shows neck
stiffness without focal neurologic deficits. There is no papilledema. Which of the following is the
most appropriate initial diagnostic test?
Incorrect Answer ImageA.Complete blood count with differential
Incorrect Answer ImageB.CT scan of the head
Incorrect Answer ImageC.Culture of cerebrospinal fluid
Incorrect Answer ImageD.MRI scan of the brain
Correct Answer ImageE.White blood cell count with differential of cerebrospinal fluid

A 62-year-old man comes to the physician for an annual checkup. He has felt well and has no
complaints. He denies fever, weight loss, fatigue, night sweats, diarrhea, or constipation. He has
noticed no bleeding or bruising. His past medical history is notable for hypertension, for which
he takes hydrochlorothiazide. He also takes supplemental vitamin C. He denies alcohol or
tobacco use. His typical diet includes steak approximately three times per week. His weight has
not changed from his last visit. His pulse is 77 beats/min and blood pressure is 133/83 mm Hg.
Head and neck examination demonstrates no lymphadenopathy or carotid bruits. Chest is clear to
auscultation, and he has no abnormal heart sounds. His abdomen is soft, without
hepatosplenomegaly. Peripheral examination reveals no clubbing, cyanosis, or edema. Rectal
exam reveals a minimally enlarged, smooth prostate. The stool is brown and is guaiac-positive.
His last colonoscopy was 2 years ago and was unremarkable, thus a false-positive is suspected.
What is the next step to exclude a colonic source of bleeding?
Incorrect Answer ImageA.Fecal DNA testing
Correct Answer ImageB.Fecal immunochemical testing (FIT)
Incorrect Answer ImageC.Flexible sigmoidoscopy
Incorrect Answer ImageD.Repeat colonoscopy
Incorrect Answer ImageE.Repeat fecal occult blood testing in 2 weeks

A 64-year-old woman comes to her physician for a follow-up examination because of


hypertension and congestive heart failure. She was recently hospitalized for pulmonary edema,
and an echocardiogram at that time showed an EF of 35-40% with a globally hypokinetic left
ventricle. She states that she uses 2 pillows to sleep at night and has occasional paroxysmal
nocturnal dyspnea. She is currently taking furosemide and carvedilol. Her blood pressure is
150/80 mm Hg and pulse is 80/min and regular. Physical examination shows clear lungs and
there are no extra heart sounds. The PMI is laterally displaced. There is no peripheral edema.
Which of the following is the most appropriate next step in management?
Correct Answer ImageA.Add ACE inhibitor to her regimen
Incorrect Answer ImageB.Add amlodipine to her regimen
Incorrect Answer ImageC.Add hydralazine to her regimen
Incorrect Answer ImageD.Decrease the dose of her beta-blocker
Incorrect Answer ImageE.Increase the dose of her loop diuretic

A 24-year-old medical student comes to the student health clinic because of a 2-month history of
epigastric pain that started 1 week before taking Step 2 of the USMLE. The pain becomes more
severe 30 minutes after meals, and she has been awakened from sleep by pain several times
weekly. Ranitidine has improved her symptoms transiently. She has a history of asthma and
seasonal allergies. Current medications include albuterol and loratadine. Which of the following
is the most likely cause of this patient's symptoms?
Incorrect Answer ImageA.Diffuse gastric mucosal vasoconstriction
Incorrect Answer ImageB.Gastrin-producing tumor
Correct Answer ImageC.Organism colonizing the gastric antrum
Incorrect Answer ImageD.Organism invading the mucosa
Incorrect Answer ImageE.Reduced gastrin release

A 57-year-old woman with a history of rheumatic fever as a child comes to the physician
because of a 6-month history of slowly progressive dyspnea on exertion and orthopnea. Her
blood pressure is 110/60 mm Hg, temperature is 37ºC (98.6ºF), pulse is 93/min and irregular, and
respirations are 18/min. Physical examination shows a localized mid-diastolic murmur near the
apex. A loud opening snap is heard after S2. The rhythm appears irregularly irregular. Which of
the following is most likely to be found on physical examination? 
Incorrect Answer ImageA.Decreased S1 intensity
Incorrect Answer ImageB.De Musset sign
Incorrect Answer ImageC.Increased P2 intensity
Correct Answer ImageD.Increased S1 intensity
Incorrect Answer ImageE.Large c-v wave
Incorrect Answer ImageF.Palpable left ventricular heave

A 28-year-old man comes to his physician because of a 6-month history of persistent vague and
dull headaches. He denies fever, neck stiffness, nausea, vomiting, or changes in vision. He has
no history of significant medical illnesses. Current medications include ibuprofen as needed for
the headaches. He denies tobacco or alcohol use. His temperature is 37.1°C (98.8°F), pulse is
70/min, respirations are 12/min, and blood pressure is 155/90 mm Hg. His previous records from
a year ago show a blood pressure of 107/64 mm Hg. Cardiac examination shows normal S1 and
S2 with no added heart sounds or murmurs. The rest of the physical examination, including
abdominal and peripheral examination, is unremarkable. A repeat blood pressure reading, one
week later, is 150/70 mm Hg. Laboratory studies show:
Na+ 147 mEq/L 
K +
3.0 mEq/L 
Cl− 107 mEq/L 
HCO3− 30 mEq/L 
Glucose 92 mg/dL
Blood urea nitrogen (BUN)  13 mg/dL 
Creatinine  0.8 mg/dL 
Which of the following is the most appropriate next step in diagnosis? 
Incorrect Answer ImageA.24-hour urine for metanephrine and catecholamine
Incorrect Answer ImageB.Insulin-like growth factor-1 (IGF-1) levels
Incorrect Answer ImageC.Random serum cortisol level
Incorrect Answer ImageD.Serum aldosterone levels
Incorrect Answer ImageE.Serum renin levels
Correct Answer ImageF.Serum renin levels and aldosterone levels

A 30-year-old man presents to his physician with 3 months of worsening headache, fatigue, and
increased urination. He has no significant past medical history and takes no medications. On
examination, his temperature is 37.0°C (98.6°F), pulse is 86/min, respirations are 9/min, and
blood pressure is 190/100 mm Hg. The physical examination is otherwise normal. Laboratory
studies show:
Sodium 148 mEq/L
Potassium 2.5 mEq/L
Chloride 100 mEq/L
Bicarbonate 39 mEq/L
Glucose 82 mg/dL
BUN 20 mg/dL
Creatinine 1.0 mg/dL
Plasma renin activity 0.5 ng/mL/h
Serum aldosterone 23 ng/dL
Morning serum cortisol 15 micrograms/dL
A urine electrolyte test shows a urine chloride of 28 mEq/L. Which of the following is the most
likely diagnosis?
Incorrect Answer ImageA.ACTH-secreting pituitary adenoma

Incorrect Answer ImageB.Diuretic abuse

Correct Answer ImageC.Primary hyperaldosteronism

Incorrect Answer ImageD.Renal artery stenosis


Incorrect Answer ImageE.Type 1 (distal) renal tubular acidosis (RTA)

A 67-year-old woman comes to the physician because of shortness of breath for the past two
months. She reports progressively worsening dyspnea while climbing the stairs in her house, and
lately while walking in the grocery store. She denies chest pain and dyspnea at rest. She has a
history of myocardial infarction 7 years ago. She has been smoking one pack of cigarettes per
day for the past 40 years and admits to drinking three glasses of wine per day and binge drinking
on the weekend. Body mass index is 40 kg/m2. Her temperature is 37°C (98.6°F), pulse is
110/min, respirations are 18/min, and blood pressure is 160/90 mm Hg. Oxygen saturation is
94% on room air. She is in no apparent distress. Cardiovascular examination shows normal heart
sounds with no murmurs or gallops. Chest auscultation shows end-inspiratory crackles. There is
no jugular venous distension or peripheral edema. A transthoracic echocardiogram shows an
ejection fraction of 55% and grade III diastolic left ventricular dysfunction in the presence of
normal left ventricular (LV) volume. Which of the following is the most likely cause of her
symptoms?
Incorrect Answer ImageA.Alcohol
Incorrect Answer ImageB.Drugs
Correct Answer ImageC.Hypertension
Incorrect Answer ImageD.Ischemic cardiomyopathy
Incorrect Answer ImageE.Obstructive lung disease

A 69-year-old man is brought to the emergency department with shortness of breath. The patient
has a known history of COPD and was hospitalized 3 times in the last 2 months with COPD
exacerbations. Today the patient complains of worsening shortness of breath and a productive
cough with yellow-green sputum. The patient is on home oxygen, 2 L/min by way of nasal
cannula. He also has a history of hypertension, coronary artery disease, and GERD. The patient
currently takes ipratropium, hydrochlorothiazide, lisinopril, aspirin, nitroglycerin, and
omeprazole. He has been a lifelong smoker with a 90 pack-year history. Vital signs are
temperature 37.4°C (99.4°F), blood pressure 148/86 mm Hg, pulse 106/min, and respirations
24/min. Physical examination shows decreased breath sounds and bilateral wheezing.
Cardiovascular examination shows a loud second heart sound. Abdomen is soft, nontender, and
nondistended. Liver is slightly enlarged. Examination of extremities shows 2+ lower extremity
edema. Arterial blood gas analysis is as follows:
pH  7.33
pCO2 58 mm Hg
pO2 54 mm Hg
HCO3 –
30 mEq/L.
The patient is admitted to the hospital. In addition to his regular medications, he is placed on
methylprednisolone and azithromycin. Which of the following treatment modalities decreases
morbidity and mortality in patients with COPD? 
Incorrect Answer ImageA.Antibiotics
Incorrect Answer ImageB.Anticholinergic medications
Incorrect Answer ImageC.Corticosteroids
Incorrect Answer ImageD.Lung volume reduction surgery
Correct Answer ImageE.Oxygen therapy

A 51-year-old man comes to the emergency department because of a 2-day history of fever,
nausea, vomiting, and severe abdominal pain radiating to the right shoulder. His past medical
history is unremarkable and he takes no medications. His temperature is 38.3ºC (101ºF), pulse is
90/min, and blood pressure is 130/70 mm Hg. Examination shows tenderness in the right upper
quadrant with abrupt cessation of inspiration on deep palpation of his right upper quadrant. The
sclera are anicteric. Abdominal ultrasound confirms the diagnosis. Which of the following is the
most appropriate definitive treatment in management?
Correct Answer ImageA.Cholecystectomy within 48 hours
Incorrect Answer ImageB.Immediate cholecystectomy
Incorrect Answer ImageC.IV antibiotics and observation
Incorrect Answer ImageD.IV fluids and observation
Incorrect Answer ImageE.Percutaneous drainage now

A 66-year-old man is brought to the emergency department because of acute onset of pleuritic
chest pain and dyspnea. He was collecting his baggage at the airport when he developed pain and
shortness of breath. He had a cast on his left leg secondary to a recent fracture occurring after a
skiing injury 8 days earlier. Because of his injury, he has spent the majority of his day lying on
the sofa. His pulse is 122/min and blood pressure is 90/60 mm Hg. A ventilation/perfusion scan
shows segmental mismatch in the right lobe. Which of the following management strategies is
most likely to reduce the risk for a recurrent thromboembolic event?
Incorrect Answer ImageA.Begin heparin and continue until the patient is ambulatory and no
evidence of clot; no further treatment once ambulatory
Incorrect Answer ImageB.Initiate treatment with subcutaneous low-molecular weight heparin for 12
weeks
Incorrect Answer ImageC.Place an inferior vena cava filter immediately, followed by thrombolytic
therapy if clot persists
Correct Answer ImageD.Start heparin and warfarin; once INR is therapeutic, stop heparin and
continue warfarin for 6 months after discharge
Incorrect Answer ImageE.Start heparin and warfarin; once INR is therapeutic, stop heparin and
continue warfarin for 6 weeks after discharge

A 65-year-old woman comes to the physician because of a two day history of back pain that
started after lifting a bag of groceries. She denies a history of urinary incontinence. She has type
2 diabetes mellitus controlled with diet and pulmonary thromboembolism diagonosed 5 years
ago. She had a total abdominal hysterectomy with bilateral salpingo-oophorectomy 20 years ago
for heavy vaginal bleeding. She takes no medications and smokes one pack of cigarettes daily for
35 years. She drinks 3 to 4 glasses of wine on social occasions. Examination shows tenderness
over T12 and L1. Laboratory studies show: 
Hemoglobin 13.0 g/dL
Hematocrit 39%
WBC 6,000 mg/dL
Platelets 200,000 mm3
Calcium 9.8 mg/dL
Phosphorus 4.0 mg/dL
Alkaline phosphatase 30 U/L
TSH 3.0 µU/mL
Which of the following is the most appropriate measure to prevent further progression of this
condition? 
Incorrect Answer ImageA.High-dose dexamethasone injection
Incorrect Answer ImageB.Hormonal replacement therapy
Correct Answer ImageC.Lifestyle modification and supplementation with calcium and vitamin D
Incorrect Answer ImageD.Surgical decompression of the spine
Incorrect Answer ImageE.Vitamin D supplementation and continuous monitoring of the patient's
calcium levels

A 62-year-old woman comes to the physician because of weakness and fatigue. She has had dry
mouth, dizziness when she stands up quickly, and difficulty lifting herself from a seated position
for 4 months. Her weakness has been worse in the morning, and her symptoms have improved
with activity. She has had progressive cough and mild chest pain. She has had a 13.6-kg (30-lb)
weight loss over the past 3 months. She has smoked two packs of cigarettes daily for 45 years.
She has a history of chronic bronchitis, hypertension, and hyperlipidemia. Current medications
include albuterol, hydrochlorothiazide, and pravastatin. Examination shows normal muscle tone
and 3/5 muscle strength in the proximal muscles. Which of the following is the most likely cause
of this patient's weakness? 
Correct Answer ImageA.Antibody-mediated destruction of presynaptic calcium channels
Incorrect Answer ImageB.Immune-mediated nicotinic acetylcholine receptor blockade
Incorrect Answer ImageC.Metabolic abnormalities and muscle wasting from cachexia
Incorrect Answer ImageD.Metastatic central nervous system involvement from small-cell cancer
Incorrect Answer ImageE.Occult tumor production of a parathyroid hormone-related protein

A 49-year-old man is being evaluated in the emergency department for a recent fall and blow to
his head. A CT scan is normal, and his injuries involve only a small laceration over the right eye.
His past medical history is significant for chronic gastritis and osteoarthritis, for which he
sometimes takes acetaminophen. Laboratory studies show: 
RBC count 4.1 × 106 million/mm3
Hematocrit 39%
Hemoglobin 12 g/dL
Leukocyte count 7,500/mm3
Mean corpuscular volume 110 μm3
Platelet count 140,000/mm3
Peripheral smear Shows hypersegmented neutrophils
Anti-IF antibodies Negative
Alkaline phosphate 50 U/L
LDH 150 U/L
AST 150 U/L
ALT 66 U/L
Total bilirubin 1 mg/dL
The above findings are consistent with which of the following diagnoses? 
Correct Answer ImageA.Chronic alcohol abuse
Incorrect Answer ImageB.Drug-induced gastritis
Incorrect Answer ImageC.Peptic ulcer disease

Incorrect Answer ImageD.Pernicious anemia

Incorrect Answer ImageE.Viral hepatitis

A 75-year-old man comes to the emergency department because of bright red blood in his stools.
He has been bleeding for the past day and is lightheaded. There is no prior history of blood in his
stools or hemorrhoids. His past medical history is significant for hypertension and coronary
artery disease treated with lisinopril and atorvastatin. His temperature is 37ºC (98.6ºF), blood
pressure is 90/55 mm Hg, and pulse is 110/min. Head, neck, cardiovascular, and lung
examinations are normal. There is no tenderness to palpation of the abdomen. There is blood in
the rectal vault on digital rectal examination. Laboratory data show a hematocrit of 32%. He is
immediately given intravenous fluid and 2 units of packed red blood cells. CT scan of the
abdomen is normal. Mesenteric angiography demonstrates a dilated and slow-filling vein in the
ascending colonic wall. Which of the following is true about the lesion responsible for this
patient's presentation?
Correct Answer ImageA.Endoscopic ablation is the next line of treatment
Incorrect Answer ImageB.Intra-arterial vasopressin has an equal rate of re-bleeding to embolization
Incorrect Answer ImageC.It accounts for 0.5% of lower GI bleeds
Incorrect Answer ImageD.It is a congenital lesion
Incorrect Answer ImageE.Most occur in the small bowel

A 55-year-old man is brought to the ER due to progressive substernal chest pain for the past hour
that has not responded to sublingual nitroglycerin. He reports no previous history of chest pain.
He has a history of type II diabetes mellitus and hypercholesterolemia for the past 20 years. Two
of his brothers sustained myocardial infarction at the age of 50 years. His body mass index
(BMI) is 27. His vitals are temp 36.7°C (98°F), pulse 110/min, respirations 18/min and BP
138/90. On physical examination, he has no added heart sounds or murmurs and his lungs are
clear. EKG done at the time of admission shows ST-segment depressions in leads I, aVL, and
V1-V4. Cardiac troponin I is 0.02 ng/mL. After initial medical stabilization, he undergoes
cardiac catheterization, which shows 70% occlusion in the right and left main coronary arteries.
His left ventricular ejection fraction is 55%. Which of the following is the next best step in
management?
Incorrect Answer ImageA.Angioplasty and stenting of all affected vessels
Incorrect Answer ImageB.Continued medical management until occlusion exceeds 90% or ejection
fraction becomes less than 35%
Correct Answer ImageC.Coronary artery bypass grafting (CABG) using the internal mammary
artery
Incorrect Answer ImageD.CABG using saphenous vein grafts
Incorrect Answer ImageE.Intensive medical therapy to bring his ejection fraction to more than 55%
and allow surgical coronary revascularization

A 48-year-old man comes to the physician because of a 2-month history of easy bruising and
weight gain. He has had a 10-kg (22-lb) weight gain over the past 2 months. He has a history of
childhood asthma. He takes no medications. He smoked one pack of cigarettes daily for 5 years
but quit 20 years ago. He drinks 3 to 5 beers on the weekends. His temperature is 37.2ºC
(99.0ºF), pulse is 80/min, respirations are 16/min, and blood pressure is 165/95 mm Hg.
Examination shows numerous linear, purple-colored skin markings over the abdomen. Urinalysis
shows significant glycosuria. Which of the following is the most appropriate next step in
diagnosis?
Incorrect Answer ImageA.Baseline plasma ACTH measurement
Incorrect Answer ImageB.CT scan of the chest and abdomen
Correct Answer ImageC.Dexamethasone suppression test
Incorrect Answer ImageD.Measurement of hemoglobin A1c level
Incorrect Answer ImageE.MRI of the head

A 43-year-old man comes to the health clinic for increasing fatigue and ankle swelling. He is
being followed for hypertension and recently diagnosed type 2 diabetes. He is currently taking
lisinopril for his hypertension, and his diabetes has been diet-controlled, with a hemoglobin
A1c of 7% 6 months ago. He smokes and occasionally drinks alcohol. He was using intravenous
methamphetamine until 12 months ago but now uses no recreational drugs. His temperature is
37.0ºC (98.6°F), and his blood pressure is 135/75 mm Hg. Oxygen saturation is 94% in room air.
Funduscopic examination is normal. The heart, lungs, and abdomen are normal. There is 2+
pretibial pitting edema. Serum studies show normal electrolytes, BUN, and creatinine; the serum
albumin is 2.1 mg/dL. The urinalysis shows 4+ protein but is otherwise normal, without cells or
casts. A 24-hour urine collection shows 4.5 g of protein. Which of the following serum studies
would be most useful now in diagnosing the cause of the patient’s proteinuria? 
Incorrect Answer ImageA.Anti-glomerular basement antibody (anti-GBM) titer

Incorrect Answer ImageB.Anti-streptolysin O titer

Correct Answer ImageC.HIV antibody assay

Incorrect Answer ImageD.Serum hemoglobin A1c assay


Incorrect Answer ImageE.Toxicology screen

A 73-year-old man comes to the emergency department because of abdominal pain. He describes
a dull, aching, constant pain in his mid-umbilical region. The pain has persisted over the past few
days with increasing intensity, and it is not relieved by changes in position or eating. The patient
has a past medical history significant for hypertension and coronary artery disease. He had a
myocardial infarction 3 years ago. In addition, he has moderate peripheral vascular disease with
a prior femoral-popliteal bypass graft on the left. His pulse is 86/min and blood pressure is
180/100 mm Hg. Cardiac examination shows a loud S4 gallop. Abdominal examination shows a
pulsatile mass in his abdomen and venous stasis changes bilaterally on his lower extremities.
Which of the following is the most appropriate initial diagnostic test at this time?
Correct Answer ImageA.Abdominal ultrasound (U/S)
Incorrect Answer ImageB.Aortic angiography
Incorrect Answer ImageC.CT abdomen without IV contrast
Incorrect Answer ImageD.CT abdomen with IV contrast
Incorrect Answer ImageE.CT abdomen with and without IV contrast

A 36-year-old man is brought to the emergency department 20 minutes after being knocked
unconscious while swimming in the ocean. He received basic life support with rescue breaths
and regained consciousness after he was pulled from the ocean. On arrival, he is alert and in
moderate respiratory distress. Which of the following is the most appropriate next step in
management? 
Incorrect Answer ImageA.Abdominal thrusts
Correct Answer ImageB.Continuous positive airway pressure
Incorrect Answer ImageC.Prophylactic antibiotics
Incorrect Answer ImageD.Steroid administration
Incorrect Answer ImageE.Tracheostomy

A 26-year-old woman comes to the physician for follow-up after undergoing a colonoscopy
secondary to intermittent bloody diarrhea, tenesmus, and rectal urgency. She is able to eat
normally and has not lost weight. She has two to three stools per day. Her colonoscopy results
show mild colonic inflammation and friability involving the distal 30 cm of colon and rectum.
Biopsy results are consistent with ulcerative colitis. Her laboratory studies, including blood
counts and erythrocyte sedimentation rate, are within the normal range. Stool cultures have been
negative. Which of the following is the most appropriate initial management?
Correct Answer ImageA.Aminosalicylate enema
Incorrect Answer ImageB.Azathioprine
Incorrect Answer ImageC.Corticosteroid enema
Incorrect Answer ImageD.Oral aminosalicylate
Incorrect Answer ImageE.Pulse steroid followed by oral taper

A 65-year-old man comes to the physician because of a 4-month history of red, swollen, and
tender skin lesions that come and go on his legs, abdomen, chest, and arms. He has a history of
hypertension and hypercholesterolemia treated with hydrochlorothiazide, metoprolol, and
lovastatin. Examination shows bitemporal wasting. There are firm, erythematous, subcutaneous
structures on the abdomen and right leg. The lesions are tender and cord-like. Which of the
following is the most likely diagnosis? 
Incorrect Answer ImageA.Angiosarcoma
Incorrect Answer ImageB.Carcinoid tumor
Incorrect Answer ImageC.Lung adenocarcinoma
Correct Answer ImageD.Pancreatic carcinoma
Incorrect Answer ImageE.Sarcoidosis

A 16-year-old girl is brought to the physician because of difficulty tasting food for 4 weeks. She
has also had difficulty chewing and swallowing and has a burning sensation in her mouth
whenever she eats spicy foods or drinks carbonated beverages. Aside from dry eyes treated with
artificial teardrops, her medical history is unremarkable. The patient clears her throat numerous
times, and her voice sounds hoarse. Her temperature is 37°C (98.6°F) and blood pressure is
114/72 mm Hg. Examination shows dry, red eyes and dry mucosal surfaces in the oral cavity,
along with dental caries. Which of the following is the most likely cause of this patient's
symptoms? 
Incorrect Answer ImageA.Impingement of chorda tympani
Incorrect Answer ImageB.Interruption of sympathetic nerve fibers
Correct Answer ImageC.Lymphocytic infiltration of salivary and lacrimal gland
Incorrect Answer ImageD.Recurrent laryngeal nerve compression
Incorrect Answer ImageE.Sialolithiasis

A 62-year-old woman with a history of cirrhosis is brought to the emergency department by her
neighbor because of increasing lethargy for 1 week. She has had multiple esophageal variceal
bleeds and prior admissions for hepatic encephalopathy. She has a history of a duodenal ulcer,
hypertension, and gout. She drinks 12 to 15 beers daily. Physical examination shows a lethargic
woman in no distress with abdominal distention consistent with ascites. She is not oriented to the
place or date. Her temperature is 37.1ºC (98.7ºF), blood pressure is 128/82 mmHg, pulse is
96/min, and respirations are 18/min. Her mucous membranes are dry. Neurologic examination
shows asterixis (tremor of the hands resembling bird flapping). She has recently undergone
paracentesis for her tense ascites. Which of the following treatment medications is most likely to
have caused her altered mental state?
Incorrect Answer ImageA.Colchicine
Incorrect Answer ImageB.Enalapril
Correct Answer ImageC.Furosemide
Incorrect Answer ImageD.Metoprolol
Incorrect Answer ImageE.Omeprazole

A 36-year-old woman comes to the physician because recurrent postprandial epigastric pain for
the past 2 weeks. The pain radiates to the substernal area. She gives a history of a chronic cough
that is worse at night. There has been no dysphagia, vomiting, night sweats, nocturia, or weight
loss. In addition, there is no past medical history and she takes only hydrochlorothiazide for
essential hypertension. The physical examination is normal and a stool test for occult blood is
negative. A resting ECG is normal at this time. Which of the following will confirm the most
likely diagnosis? 
Correct Answer ImageA.24-hour pH monitoring
Incorrect Answer ImageB.Barium swallow
Incorrect Answer ImageC.Endoscopy
Incorrect Answer ImageD.Stress test
Incorrect Answer ImageE.Ultrasound of the right upper quadrant

A 54-year-old man with type 2 diabetes mellitus comes to the emergency department because of
dyspnea, nausea, and vague chest discomfort. Further evaluation shows elevated cardiac
enzymes and ST-segment changes suggestive of ischemia. He undergoes coronary
catheterization which shows three-vessel disease and coronary artery bypass grafting is planned.
Although the surgery goes well, low cardiac output requires the placement of an intra-aortic
balloon pump (IABP). Over the next 2 days, the frequency of the synchronous IABP
counterpulsations is slowly reduced and the pump is removed. Which of the following is one of
the mechanisms by which the IABP most likely helped this patient?
Incorrect Answer ImageA.A compensatory increase in heart rate from balloon pulsations
Incorrect Answer ImageB.Improving forward flow during diastole and mean arterial pressures
Correct Answer ImageC.Increasing coronary artery perfusion from balloon inflation
Incorrect Answer ImageD.Increasing systolic blood pressure from balloon inflation
Incorrect Answer ImageE.Reducing diastolic blood pressure from balloon deflation

A previously well 18-year-old woman comes to her physician because of a 6-month history of
progressive weakness and fatigue. She takes no medications. She is sexually active with one
partner, has normal menstrual periods, and has never been pregnant. She weighs 53 kg (116 lb)
and is 152 cm (5 ft) tall, for a BMI of 22.9 kg/m2. Her blood pressure is 92/60 mm Hg and pulse
is 110/min. Physical examination shows some abrasions on the fingers but otherwise is normal.
A complete blood count is normal. Serum studies show: 
Sodium 134 mEq/L
Potassium 2.2 mEq/L
Chloride 80 mEq/L
HCO3 44 mEq/L
Creatinine 1.0 mg/dL
Glucose 97 mg/dL
Calcium 9.8 mg/dL
Which of the following diagnoses is most likely?
Incorrect Answer ImageA.Chronic diarrhea
Incorrect Answer ImageB.Cushing syndrome
Incorrect Answer ImageC.Licorice ingestion
Incorrect Answer ImageD.Primary hyperaldosteronism
Correct Answer ImageE.Vomiting

A 66-year-old man with a history of hypertension, diabetes mellitus, hypercholesterolemia,


coronary artery disease, and chronic renal insufficiency comes to the physician for evaluation of
increased blood pressure. He states that his blood pressure has been stable for several years, but
has been running high for the last few months. His medications have remained the same for the
last few years and include aspirin, metoprolol, clonidine, amlodipine, chlorthalidone, isosorbide
mononitrate, and hydralazine. His blood pressure is 200/90 mm Hg, pulse is 98/min, and
respirations are 12/min. There is a right-sided carotid bruit. Lungs are clear to auscultation.
Abdominal examination is shown in the video. Which of the following is most appropriate to
confirm the diagnosis?
Incorrect Answer ImageA.Aldosterone-to-renin ratio
Incorrect Answer ImageB.Captopril renal scan
Incorrect Answer ImageC.Computed tomography angiography
Incorrect Answer ImageD.Continued medical management
Incorrect Answer ImageE.Coronary angiography
Correct Answer ImageF.Duplex Doppler ultrasonography
Incorrect Answer ImageG.Magnetic resonance angiography
Incorrect Answer ImageH.Thyroid-stimulating hormone (TSH) levels
Incorrect Answer ImageI.Treadmill stress test

A 65-year-old woman is brought to the emergency department by her daughter because of


dizziness and palpitations 2 hours after having a syncopal episode. She has a history of coronary
artery disease, hypertension, paroxysmal atrial fibrillation, hyperlipidemia, and type 2 diabetes
mellitus. Current medications include nitroglycerin, isosorbide dinitrate, lisinopril, atorvastatin,
quinidine, and metformin. Her temperature is 36.6ºC (97.8ºF), blood pressure is 130/70 mm Hg,
pulse is 100/min, and respirations are 20/min. The lungs are clear to auscultation. Cardiac
examination shows irregular rhythm; no murmurs or rubs are heard. The remainder of the
physical examination shows no abnormalities. An ECG is shown. Which of the following is the
most appropriate next step in management?
Correct Answer ImageA.Administer IV bolus of MgSO4
Incorrect Answer ImageB.Administer IV lidocaine
Incorrect Answer ImageC.Apply 360 J of defibrillating current
Incorrect Answer ImageD.Discontinue quinidine and observe
Incorrect Answer ImageE.Increase the quinidine dose
Incorrect Answer ImageF.Place a temporary pacemaker

A 64-year-old Caucasian man with a 20 pack-year smoking history comes to the physician with a
report of "coughing up blood." He also states that he has been increasingly fatigued and weak for
the last 2 months and had noted a weight loss of approximately 8 pounds. A chest radiograph
demonstrates a large, right-sided hilar mass with left-sided hilar lymphadenopathy.
Histopathology of the patient's hilar mass demonstrates sheets of small round cells with nuclear
molding, granular chromatin, scant cytoplasm and foci of necrosis, and rosette formation. Which
of the following findings may be observed in a patient who has this neoplasm?
Incorrect Answer ImageA.Excessive PTHrP production
Incorrect Answer ImageB.Hypernatremia
Incorrect Answer ImageC.Hypocortisolism
Correct Answer ImageD.Lambert-Eaton syndrome
Incorrect Answer ImageE.Pseudohypoparathyroidism

A 32-year-old man comes to the physician because of a 2-week history of severe pruritus. He has
a 7-year history of ulcerative colitis treated with oral sulfasalazine and cortisone enemas.
Examination shows diffuse excoriations on the extremities and trunk. The remainder of the
examination is within normal limits. Laboratory studies show: 
Hemoglobin 9.2 g/dL
Total iron binding capacity  375 ug/dL
Alkaline phosphatase  322 U/L
Serum studies show:
Iron 50 ug/dL
Ferritin  15 ng/mL
Aspartate aminotransferase (AST, GOT)  32 U/L
Alanine aminotransferase (ALT, GPT)  20 U/L
Na+  137 mEq/L
Cl -
101 mEq/L
K+  3.8 mEq/L
HCO3 -
26 mEq/L
Urea nitrogen (BUN)  17 mg/dL
Creatinine  0.7 mg/dL
Which of the following diagnostic tests is most likely to confirm the diagnosis of this patient's
condition? 
Incorrect Answer ImageA.Antimitochondrial antibodies
Correct Answer ImageB.Endoscopic retrograde cholangiopancreatography (ERCP)
Incorrect Answer ImageC.Liver biopsy
Incorrect Answer ImageD.Perinuclear antineutrophil cytoplasmic antibodies (P-ANCA)
Incorrect Answer ImageE.Ultrasound of the abdomen

A 42-year-old woman comes to the emergency department by ambulance accompanied by her


husband. She is unconscious. A quick history from the husband indicates that for the past few
days she has been very tired and has not gone to work. Today she complained of a continued
headache, and gradually became confused and did not understand what he was saying to her. He
also noticed some unusual "purple blotches" on her arms and legs. Because of concern about her
airway and breathing, she is intubated and placed on a ventilator. The most obvious initial
finding on examination is scattered purpura and petechiae and she appears very pale. Laboratory
studies show:
Hematocrit 22%
Reticulocytes Marked increase
Peripheral smear Many nucleated RBCs,
schistocytes
Platelets 3,000/µL
Lactate dehydrogenase, LDH 800 units/L
Bilirubin 6.2/0.6 mg/dL
PT, PTT, fibrinogen Normal 
BUN 20 mg/dL
Creatinine 1.4 mg/dL
Urine Hemoglobin present
Which of the following is the most appropriate next step in the management of this patient? 
Incorrect Answer ImageA.Administer an intravenous infusion of platelets
Incorrect Answer ImageB.Begin intravenous corticosteroid therapy
Incorrect Answer ImageC.Begin intravenous cyclophosphamide therapy
Correct Answer ImageD.Begin large-volume plasmapheresis
Incorrect Answer ImageE.Transfuse with whole blood
A 64-year-old man with a history of degenerative joint disease comes to the physician because of
back pain of 4 months' duration. The pain localizes to his lower back and radiates to his buttocks
and thighs. The pain starts when he is walking and improves when he stops and stoops forward.
He has also had numbness and weakness in his lower extremities. The patient has undergone 2
months of physical therapy and high-dose ibuprofen without improvement of his symptoms. He
has a history of hypertension, hypercholesterolemia, and coronary artery disease (CAD). Current
medications include aspirin, metoprolol, amlodipine, lisinopril, simvastatin, and ibuprofen. His
blood pressure is 130/70 mm Hg, pulse is 90/min, and respirations are 14/min. Musculoskeletal
examination shows that the pain is elicited by extension of the spine. There is bilateral muscle
atrophy in the lower limbs. An MRI scan of the spine shows compression of the lumbar nerve
roots. Which of the following is the most appropriate next step in management?
Incorrect Answer ImageA.Bone scan
Incorrect Answer ImageB.Epidural steroid injection
Incorrect Answer ImageC.Increase the frequency of ibuprofen
Incorrect Answer ImageD.Repeat MRI scan of the spine
Correct Answer ImageE.Surgical decompression

A 32-year-old man comes to the physician because of difficulty losing weight. He has gained 15
kg (33 lbs) over the past 12 years. His medical history is unremarkable, and he takes no
medications. He works as an accountant. He does not smoke cigarettes, and he drinks 2 to 3
beers on the weekends. He weighs 100 kg (220 lbs) and is 178 cm (68 in) tall. His body mass
index is 31.6 kg/m2. His blood pressure is 125/80 mm Hg, pulse 65/min, and respirations 16/min.
Examination shows no other abnormalities. Which of the following is the most appropriate next
step in management? 
Incorrect Answer ImageA.Change to a ketogenic diet before embarking on an exercise program
Incorrect Answer ImageB.Check thyroid function before deciding on the best weight-loss program
Incorrect Answer ImageC.Increase his exercise to 5 days per week
Incorrect Answer ImageD.Evaluate cortisol levels
Correct Answer ImageE.Start a program of regular exercise followed by a low-calorie diet

A 42-year-old man comes to the physician because of persistent diarrhea and foul-smelling
breath. He noticed the onset of diarrhea approximately one week ago and is associated with
feculent eructation. He also complains of epigastric pain for the past year. He has not had any
change in his dietary habits and denies eating at any fast-food restaurants. He rarely drinks milk
or milk products and denies consumption over the past week. He denies any recent travel or
camping trips. His past medical history is notable for severe lower back pain treated with
approximately 2,500 mg of ibuprofen per day. He was prescribed ranitidine for his epigastric
pain, which has provided some relief. He does not drink or smoke. His abdominal examination is
notable for mild epigastric tenderness. There is no distension or organomegaly and bowel sounds
are normal. Rectal examination is normal and his stool does not contain any occult blood.
Laboratory data is shown below:
Serum white blood cells 7,500/mm3
Stool ova and parasites No organisms detected
Stool white blood cells None detected
Stool culture Negative
Which of the following is the most likely explanation for this patient's symptoms?
Incorrect Answer ImageA.Acute bacterial diarrhea
Correct Answer ImageB.Gastrocolic fistula
Incorrect Answer ImageC.Intestinal giardiasis
Incorrect Answer ImageD.Lactose intolerance
Incorrect Answer ImageE.Large-bowel obstruction

A 70-year-old white male is brought to the emergency department by paramedics with a chief
complaint of substernal chest pain. He describes the pain as crushing 8/10 in intensity, without
radiation. He has never experienced this pain before. The patient reports that the pain began
almost half an hour ago after he had been in an argument with his wife. His past medical history
includes hypertension, peptic ulcer disease, and ischemic stroke 2 months ago with no residual
neurologic deficits. He recently underwent a laparoscopic cholecystectomy 8 days ago without
complications. His medications are metoprolol 50 mg/day, lisinopril 20 mg/day, aspirin
81mg/day, and omeprazole 40 mg/day for peptic ulcer disease. The patient appears anxious and
diaphoretic in mild distress. His temperature is 37.6ºC (99.7ºF), pulse is 110/min, respirations are
18/min, and blood pressure is 180/105 mm Hg. Chest x-ray shows mild cardiomegaly. ECG
shows sinus tachycardia with 2 mm ST elevation in leads V1-V3 with left ventricular
hypertrophy. Cardiac enzymes are drawn and are pending. The patient is given fluids and 325
mg of aspirin. The current hospital is 3 hours away from a center with the capability for
percutaneous coronary intervention. Which of the following is an absolute contraindication to
initiating thrombolytic therapy in this patient?
Incorrect Answer ImageA.Age (70)
Incorrect Answer ImageB.BP of 180/105
Incorrect Answer ImageC.Cholecystectomy 8 days prior
Correct Answer ImageD.Ischemic stroke 2 months prior
Incorrect Answer ImageE.Peptic ulcer disease

A 44-year-old woman comes to the physician for a palpable nodule in the right lobe of her
thyroid gland. The nodule measures 2 cm and is firm. The rest of the thyroid gland cannot be felt
and is not tender. The patient also describes palpitations, heat intolerance, and weight loss
despite her ravenous appetite. She is thin, fidgety, and constantly moving, with moist skin and a
pulse of 105/min. She has no exophthalmos or pretibial edema. Her TSH is reported as much
lower than normal, and she has elevated levels of free T4. Which of the following is the most
appropriate next step in diagnosis? 
Incorrect Answer ImageA.Fine-needle aspiration (FNA) of the thyroid
Incorrect Answer ImageB.Measure T3
Incorrect Answer ImageC.MRI scan of the pituitary gland
Incorrect Answer ImageD.Needle core biopsy of the thyroid mass
Correct Answer ImageE.Radionuclide thyroid scan

A 55-year-old man consults a physician because of weakness, cramps, and paresthesias in both
legs. He has also experienced polyuria and nocturia. He has a history of hypertension beginning
at 40 years of age which is currently untreated. He takes no medications, and does not consume
tobacco products, alcohol, or dietary supplements. There is no family history of hypertension or
stroke. Vital signs are: temperature 37°C (98.6°F), pulse 65/min, respirations 15/min, and blood
pressure 140/100 mm Hg. There is weakness of hip flexion bilaterally. The physical examination
is otherwise normal. CBC is normal. Fasting serum chemistry results show:
Glucose 116 mg/dL
BUN 28 mg/dL
Creatinine 1.5 mg/dL
Sodium 136 mEq/L
Potassium 2.4 mEq/L
Chloride 110 mEq/L
Plasma aldosterone 64 ng/dL
Plasma renin activity 1.0 ng/ml/hr
Noncontrast CT shows enlarged adrenal glands bilaterally, without nodules. Which of the
following is the most appropriate pharmacotherapy to treat the most likely cause of hypertension
in this patient?
Incorrect Answer ImageA.Captopril
Correct Answer ImageB.Eplerenone
Incorrect Answer ImageC.Furosemide
Incorrect Answer ImageD.Hydrochlorothiazide
Incorrect Answer ImageE.Potassium
Incorrect Answer ImageF.Propranolol

An 82-year-old woman is admitted to the hospital because of right lower lobe pneumonia. Her
medical history is significant for dilated cardiomyopathy secondary to long-standing
hypertension and coronary artery disease. While in the hospital, she reports to the nurse she feels
uncomfortable and lightheaded. Telemetry shows her heart rate is between 160-190 and
irregular. An ECG shows an irregularly irregular rhythm with a rate of 180. Her blood pressure is
now 80/40 mm Hg. Which of the following is the most appropriate next step in management?
Correct Answer ImageA.Administer a 120-joule shock in synchronous mode
Incorrect Answer ImageB.Administer a bolus of IV metoprolol
Incorrect Answer ImageC.Begin anticoagulation with warfarin
Incorrect Answer ImageD.Continue to monitor the ECG
Incorrect Answer ImageE.Give a loading dose of IV amiodarone
Incorrect Answer ImageF.Shock with 150 joules in asynchronous mode

A 56-year-old man with a history of severe chronic obstructive pulmonary disease presents to the
emergency room with two days of increasing shortness of breath, fevers, and chills. He takes
nebulized albuterol and corticosteroids daily which have not helped his symptoms. He has a 50
pack-year smoking history and quit 2 years ago. His temperature is 38.7°C (101.7°F), pulse is
115/min, and oxygen saturation is 81% on 4 L of oxygen via nasal cannula. He is in distress with
retractions of his supraclavicular muscles. Lungs show poor air movement and diffuse wheezing.
A chest radiograph demonstrates a consolidation in his right middle lobe and gram stain of the
sputum shows gram-positive organisms in chains. He is intubated due to worsening hypoxemia.
Three days later he is still intubated in the intensive care unit. His family wishes to know
whether he is ready to be extubated. Which of the following parameters would best predict a
successful wean from ventilation?
Incorrect Answer ImageA.Arterial partial pressure of oxygen of 55 mm Hg
Incorrect Answer ImageB.Negative inspiratory pressure of 5 cm H2O
Incorrect Answer ImageC.Peak inspiratory pressure of 35 cm H2O
Correct Answer ImageD.Respiratory rate (breaths/min) / tidal volume (in L) ratio of 50
Incorrect Answer ImageE.Respiratory rate (breaths/min) / tidal volume (in L) ratio of 115

A 52-year-old man undergoes a prolonged coronary artery bypass graft procedure for blockages
in his left anterior descending and circumflex arteries. The procedure is successful and the
immediate postoperative period is uneventful. Several days later during morning rounds, the
patient is found to have scleral icterus. He denies pain, nausea, or any other symptoms. He is
afebrile and normotensive. On physical examination, he has a regular rate and rhythm, bibasilar
crackles, and no abdominal tenderness. There is no ascites. He denies a history of liver disease.
Laboratory data are:
AST 150 IU/L
ALT 130 IU/L
Alkaline phosphatase 400 IU/L
Total bilirubin 6.1 mg/dL
An ultrasound of the right upper quadrant is unremarkable. Which of the following is the most
appropriate next step in the management of this patient?
Incorrect Answer ImageA.Administration of intravenous antibiotics
Incorrect Answer ImageB.Administration of ursodeoxycholic acid
Incorrect Answer ImageC.Endoscopic retrograde cholangiopancreatography
Incorrect Answer ImageD.Liver biopsy
Correct Answer ImageE.No further action is necessary

A 56-year-old woman with a history of chronic obstructive pulmonary disease is hospitalized for
treatment of a newly diagnosed metastatic laryngeal cancer. Because of the large tumor burden
and poor lung function, surgery is deferred and she is currently being treated with chemotherapy
and radiation therapy for palliation. While in the hospital she develops a fever. Her temperature
is 39ºC (102.2ºF), pulse is 113/min, respirations are 18/min, and blood pressure is 100/60 mm
Hg. She is in no apparent distress. Her heart is tachycardic and regular without murmurs, lungs
are clear bilaterally, abdomen is benign, and the extremities are without edema or cords. Her skin
is without obvious lesions. Laboratory studies show: 
Hemoglobin 8.0 mg/dL
Leukocyte count 800/mm3
Platelets 17,000/mm3
Hematocrit 26%
Neutrophils 46%
Lymphocytes 32%
Bands 12%
After obtaining blood, urine, and sputum cultures, which of the following is the most appropriate
next step in management? 
Correct Answer ImageA.Cefepime and vancomycin
Incorrect Answer ImageB.Cefepime, vancomycin, and amphotericin B
Incorrect Answer ImageC.Cefepime, vancomycin, and voriconazole
Incorrect Answer ImageD.No treatment while awaiting culture results
Incorrect Answer ImageE.Vancomycin only
A 68-year-old man is admitted to the hospital for delirium associated with a urinary tract
infection. Upon adequate treatment of the infection, the patient's mental status improves
significantly, though he remains partly disoriented. He also shows impairment in short-term
memory, difficulties in naming simple objects, and impaired concentration. His family members
confirm an 8-month history of gradual progressive decline in cognitive abilities, which they
attribute to old age. However, the man is no longer able to manage his finances and has gotten
lost while driving to the grocery store on two occasions. Prior to his discharge from the hospital,
the nursing staff reports that he continues to drip urine, although his infection has resolved. The
nurses also notice that he walks slowly, with small steps, and appears to have trouble with
balance. Which of the following will best confirm the diagnosis of this patient’s condition? 
Incorrect Answer ImageA.Brain biopsy
Correct Answer ImageB.CT scan followed by lumbar puncture
Incorrect Answer ImageC.Mini-mental status exam
Incorrect Answer ImageD.No testing is required, as the diagnosis is based on clinical observation
Incorrect Answer ImageE.Urine culture

A 53-year-old woman comes to her physician for epigastric pain. She has noted this ongoing
symptom for the past year or so, and her current antacid regimen fails to relieve the pain. She
denies vomiting, although she experiences occasional nausea. Her past medical history is
generally unremarkable. She denies melena, hematochezia, weight loss, fatigue, or malaise. She
has not had any surgeries. Vital signs are within normal limits. Head, neck, chest, and
cardiovascular examinations are normal. There is mild tenderness to palpation of her
epigastrium, with no focal pain over her right upper quadrant. There is no ascites or
hepatosplenomegaly. Peripheral examination is normal. She is scheduled for a double contrast
upper gastrointestinal examination, which demonstrates thickened rugal folds in the antrum and a
smooth-margined ulcer in the same location. Which of the following is the most appropriate next
step in management? 
Incorrect Answer ImageA.Immunosuppressive medication
Incorrect Answer ImageB.Surgical resection
Correct Answer ImageC.Treatment with a proton pump inhibitor (PPI), clarithromycin, and
amoxicillin
Incorrect Answer ImageD.Workup for Zollinger-Ellison syndrome

A 36-year-old woman is brought to the emergency department after a head-on motor vehicle
collision. She has right-sided chest pain that has been progressive since the accident. Her pulse is
78/min, blood pressure is 126/74 mm Hg, and oxygen saturation is 98% on 1 L of oxygen by
face mask. Physical examination of the chest wall shows tenderness over the right fifth and sixth
ribs. Chest radiograph shows nondisplaced fractures of the lateral aspects of the right fifth and
sixth ribs and an irregular, focal opacification of the right lung near the rib injuries. Follow-up
chest CT imaging shows the opacity as a peripheral focal consolidation near the rib fractures.
The mediastinum appears unremarkable. ECG shows normal sinus rhythm. The patient was
admitted to the hospital for observation. Which of the following is the most likely diagnosis?
Incorrect Answer ImageA.Blunt aortic injury (BAI)
Incorrect Answer ImageB.Flail chest
Incorrect Answer ImageC.Hypoxic respiratory failure
Correct Answer ImageD.Pulmonary contusion
Incorrect Answer ImageE.Pulmonary embolism

A 62-year-old man with a 2-month history of renal cell carcinoma is brought to the emergency
department by his family because of confusion, weakness, constipation, nausea, vomiting, and
increased urine output for 3 days. His temperature is 36.8ºC (98.2ºF), pulse is 90/min, and blood
pressure is 138/90 mm Hg. Deep tendon reflexes are decreased. Laboratory studies show a
calcium level of 17 mg/dL and a phosphorus level of 1.2 mg/dL. A 24-hour urine phosphorus
level is 2.7 g. Which of the following proteins is most likely produced in excess by this patient's
tumor?
Incorrect Answer ImageA.Calcipotriene
Incorrect Answer ImageB.Calcitonin
Incorrect Answer ImageC.Calcitriol
Incorrect Answer ImageD.Parathyroid hormone (PTH)
Correct Answer ImageE.Parathyroid hormone-related peptide (PTHrP)

A 26-year-old woman comes to the physician for an annual visit. She states that she has been
feeling well over the past year except for a few flares of systemic lupus erythematosus (SLE).
She had SLE for the past 5 years and tested positive for antiphospholipid antibodies. Her past
medical history is significant for depression,for which she takes fluoxetine, and occasional
tension headaches, which she is able to control with acetaminophen. She has no prior surgery or
drug allergies. Her mother has ovarian cancer and 2 maternal aunts have endometrial cancer. She
lives with her husband and works as a hairstylist. Her physical examination including breast and
pelvic examination is normal. She states that she and her husband use condoms for contraception
but they do not like this method. She wants to know if she can "go on the pill." Which of the
following represents an absolute contraindication to the use of the combined oral contraceptive
pill in this patient?
Incorrect Answer ImageA.Depression
Incorrect Answer ImageB.Family history of endometrial cancer
Incorrect Answer ImageC.Family history of ovarian cancer
Correct Answer ImageD.Systemic lupus erythematosus (SLE)
Incorrect Answer ImageE.Tension headaches

A 39-year-old man comes to the physician because of new-onset wheezing. He is otherwise


healthy and runs approximately 3 miles per day. He denies any nocturnal wheezing or cough. He
has a history of chronic heartburn and takes ranitidine. He also has a history of hypertension,
which has been difficult to control over the past several months, with a combination of
propranolol, enalapril, and hydrochlorothiazide. He also uses over-the-counter decongestants for
occasional allergic rhinitis. He appears comfortable. His pulse is 68/min, respirations are 18/min,
and his blood pressure is 134/88 mm Hg. Physical examination shows soft expiratory wheezes
throughout both lung fields. Which of the following medications is most likely contributing to
this patient's wheezing?
Incorrect Answer ImageA.Enalapril
Incorrect Answer ImageB.Hydrochlorothiazide
Correct Answer ImageC.Propranolol
Incorrect Answer ImageD.Pseudoephedrine
Incorrect Answer ImageE.Ranitidine
An 80-year-old woman is brought to the emergency department because of pain in her right hip
after falling. She has a history of hypertension, type 2 diabetes mellitus, and chronic kidney
disease. Current medications include amlodipine, glyburide, and simvastatin. Radiograph of the
hips shows a right-sided hip fracture, for which the patient undergoes open reduction and internal
fixation. One day later, she is started on warfarin and physical therapy. She is unable to do the
physical therapy because of pain. On day 4 of hospitalization, she is lethargic and incoherent. CT
scan of the head shows no intracranial abnormalities. Laboratory studies show: 
INR 2.2
Calcium 12.0 mg/dL
Albumin 4.0 g/dL
PTH level 9 pg/mL (normal, 10–60 pg/mL)
Which of the following is the most likely etiology of this patient's hypercalcemia? 
Incorrect Answer ImageA.Glyburide
Correct Answer ImageB.Immobilization
Incorrect Answer ImageC.Primary hyperparathyroidism
Incorrect Answer ImageD.Secondary hyperparathyroidism
Incorrect Answer ImageE.Simvastatin
Incorrect Answer ImageF.Warfarin

A previously healthy 35-year-old woman comes to the physician because of sudden paralysis of
the lower limbs and dizziness for one day. She also reports that three months ago she had left eye
pain and blurry vision which resolved after one week. Her temperature is 37ºC (98.6ºF), blood
pressure is 130/70 mm Hg, pulse is 80/min, and respirations are 20/min. Neurologic examination
shows an awake, alert, oriented female with total loss of strength, reflexes, and sensation in her
lower extremities. Her upper extremity strength and sensation are intact. Physical examination
also shows a distended bladder. MRI studies show well-demarcated periventricular areas of T2
hyperintensity. Which of the following is the most likely diagnosis?
Incorrect Answer ImageA.Cytomegalovirus encephalitis
Incorrect Answer ImageB.Guillain-Barré syndrome
Correct Answer ImageC.Multiple sclerosis
Incorrect Answer ImageD.Progressive multifocal leukoencephalopathy
Incorrect Answer ImageE.Spinal cord compression

A 45-year-old woman with a history of asymptomatic ventricular couplets is brought to the


emergency department because of a syncopal episode. She was recently started on quinidine. An
ECG shows transient wide-complex tachycardia and prolongation of the QT interval, with waves
of alternating morphology. Laboratory studies show: 
WBC  9,300/mm3
Hb  11.7 g/dL
Hct  36%
Na +
144 mEq/L
K+ 3.2 mEq/L
Cl −
98 mEq/L
Mg2+ 2.0 mEq/L
Ca 2+
8.8 mg/dL
Creatinine  1.5 mg/dL
She is admitted to the hospital. Which of the following is the most appropriate next step in
patient care?
Incorrect Answer ImageA.Administer calcium gluconate
Incorrect Answer ImageB.Administer IV fluids
Correct Answer ImageC.Administer IV potassium
Incorrect Answer ImageD.Administer lidocaine
Incorrect Answer ImageE.Apply 360 J of defibrillating current

A 52-year-old man with hepatitis C and cirrhosis is brought to the emergency department by his
wife because of a 3-day history of abdominal discomfort and 1 day of confusion. The patient
started receiving furosemide 1 week ago for lower extremity and abdominal swelling. Other
current medications include propranolol for esophageal varices. He appears drowsy but is
arousable. His temperature is 37ºC (98.6ºF), pulse is 100/min, respirations are 20/min, and blood
pressure is 110/60 mm Hg. Neurologic examination shows asterixis. The abdomen is tense and
distended with diffuse tenderness and shifting dullness. There is no rebound tenderness or
guarding. There is lower extremity edema. Plasma ammonia level is elevated. He is admitted to
the hospital and receives oral lactulose. Which of the following is the most appropriate next step
in management?
Incorrect Answer ImageA.CT scan of the head
Incorrect Answer ImageB.Echocardiogram
Incorrect Answer ImageC.Lorazepam for the patient's confusion
Correct Answer ImageD.Paracentesis
Incorrect Answer ImageE.Spironolactone

A 67-year-old man comes to the physician because of difficulty breathing. He reports that his
symptoms started 2 months ago and have been worsening lately. Initially he was short of breath
only with exertion, but now he has frequent bouts of coughing and dyspnea even at rest. He is a
current 30 pack year smoker. On physical examination he has distant breath sounds in both lung
fields with soft expiratory wheezes. Chest x-ray shows hyperinflation of both lung fields.
Pulmonary function test shows FEV1/FVC ratio of 60% predicted, FEV1 of 70% predicted,
increased residual volume and total lung capacity, along with decreased DLCO. Arterial blood gas
shows a pH 7.36, pCO2 41 mm Hg, and pO2 63 mm Hg. Which of the following is the best long-
term management of this patient? 
Incorrect Answer ImageA.Inhaled budesonide
Correct Answer ImageB.Inhaled ipratropium bromide + inhaled albuterol
Incorrect Answer ImageC.Inhaled ipratropium bromide + theophylline
Incorrect Answer ImageD.Long term oxygen therapy
Incorrect Answer ImageE.Oral prednisone

A 53-year-old woman is brought to the emergency department because of an increasingly


anxious and jittery state. She is able to talk, though she is disoriented and easily distracted. Her
main complaints are vague abdominal pain and constipation for the last week. While talking to
you, she dozes off and has to be repeatedly awakened. From speaking to her friends, you
determine that her past medical history is significant for depression and breast cancer for which
she is receiving external beam radiation, paclitaxel, and fluoxetine. Vital signs are temperature
37.0°C (98.6°F), blood pressure 115/80 mm Hg supine, 95/70 mm Hg standing, pulse 120/min,
and respirations 24/min. Physical examination shows dry mucous membranes and surgical
evidence of a left radical mastectomy, but is otherwise unremarkable. Serum chemistries and
toxicology studies are sent to the laboratory and are pending. Electrocardiogram shows sinus
tachycardia with shortened rate-adjusted QT segments. Which of the following is the most likely
cause of this patient's symptoms? 
Incorrect Answer ImageA.Drug toxicity
Correct Answer ImageB.Hypercalcemia
Incorrect Answer ImageC.Hyperkalemia
Incorrect Answer ImageD.Hypocalcemia
Incorrect Answer ImageE.Hypokalemia

A 78-year-old man is brought to the emergency department from a nursing home because of
recurrent pneumonia. He has had several episodes of pneumonia over the past few months. When
questioned, he states that he regurgitates his food at least once a day, and he feels that it gets
"stuck" in his upper throat. His past medical history is notable for coronary artery disease,
diabetes, and hypertension. On physical examination, his temperature is 38.2ºC (100.8ºF) and
blood pressure is 150/85 mm Hg. Head and neck examination shows halitosis. His lungs show
bronchial breath sounds over his right anterior chest. He has no murmurs, rubs, or gallops.
Abdominal and peripheral examinations are normal. A pharyngeal barium swallow is performed
and shows contrast entering a pouch posterior to the hypopharynx. Which of the following
statements is true regarding this lesion? 
Incorrect Answer ImageA.It is associated with an abnormality of esophageal smooth muscle
Incorrect Answer ImageB.It is known as a Killian-Jamieson diverticulum
Incorrect Answer ImageC.It is located below the upper esophageal sphincter
Correct Answer ImageD.It represents a false diverticulum
Incorrect Answer ImageE.Ten percent develop squamous cell carcinoma within the lesion

A 40-year-old man comes to his primary care physician for his annual examination. He is 5 ft 10
in tall and weighs 190 lb. He has mild hypertension, which is being treated effectively with
hydrochlorothiazide. He has a 20 pack-year smoking history. His laboratory studies show a total
cholesterol level of 200 mg/dL, LDL level of 160 mg/dL, and an HDL level of 35 mg/dL. Which
of the following is the most important modifiable factor in this patient to reduce the morbidity
and mortality related to cardiovascular disease?
Correct Answer ImageA.Cigarette smoking
Incorrect Answer ImageB.Hypercholesterolemia
Incorrect Answer ImageC.Improved cardiac care if diagnosed with coronary artery disease
Incorrect Answer ImageD.More aggressive treatment of hypertension
Incorrect Answer ImageE.Weight reduction

A 47-year-old female smoker comes to the physician because of easy bruising, cutaneous striae
and excessive hair growth. Her blood pressure is 150/90 mm Hg. She has elevated levels of
cortisol and ACTH, which remain high after high dose dexamethasone suppression test. After
confirmation of the diagnosis, which of the following is the preferred treatment?
Incorrect Answer ImageA.Adrenalectomy
Incorrect Answer ImageB.Mitotane
Incorrect Answer ImageC.Palliative care
Correct Answer ImageD.Radiation and chemotherapy
Incorrect Answer ImageE.Trans-sphenoidal hypophysectomy

A 65-year-old Caucasian man is brought to the emergency room after he collapsed while
shopping at a local mall. His wife, who was with him when he collapsed, states that he has a
history of hypertension, hypercholesterolemia, and chronic renal insufficiency. He has been
taking his medications regularly and has never before had such an episode. On arrival, the patient
is found to be unresponsive, with shallow breathing. He is orally intubated. His pulse is 110/min,
respirations are 16/min, blood pressure is 180/100 mm Hg, and temperature 37.2ºC (98.9ºF).
Ventilator settings are assist control with rate 12, tidal volume 500 mL, and PEEP 5. Eye
examination shows that the right-sided pupil is dilated and nonreactive to light. The left pupil is
normal and reactive to light. Lungs are clear to auscultation. Cardiac examination shows no rub
or gallop. Abdomen is soft, nontender, and nondistended. Bowel sounds are present and active.
Extremities show no pitting edema. The patient withdraws his extremities on painful stimulation.
Lab tests show:
Sodium  142 mEq/L
Potassium  4.4 mEq/L
Chloride  101 mEq/L
Bicarbonate  27 mEq/L
BUN  60 mg/dL
Creatinine  2.6 mg/dL
Calcium  9.2 mg/dL
What is the most likely diagnosis? 
Incorrect Answer ImageA.Anticholinergic agent overdose
Incorrect Answer ImageB.Hypoglycemia
Incorrect Answer ImageC.Opiate overdose
Incorrect Answer ImageD.Pontine hemorrhage
Correct Answer ImageE.Uncal herniation

A 78-year-old woman is brought to the emergency department with shortness of breath. For the
past 3 hours she has experienced dyspnea at rest and has been coughing up sputum. She denies
chest pain. There is no past history of tuberculosis. The patient is in distress with a respiratory
rate of 32/min, pulse is 140/min, and blood pressure is 150/85 mm Hg. While being examined,
the patient coughs up pink frothy sputum. There is no chest-wall tenderness. Auscultation shows
fine rales throughout the chest. There are no murmurs or rubs. Which of the following is the
most appropriate next step in management?
Incorrect Answer ImageA.Administer an IV loop diuretic, nitrates, and morphine
Incorrect Answer ImageB.Administer dobutamine
Incorrect Answer ImageC.Administer dopamine
Incorrect Answer ImageD.Administer hydralazine
Incorrect Answer ImageE.Administer verapamil
Incorrect Answer ImageF.Echocardiography
Correct Answer ImageG.Electrocardiography
Incorrect Answer ImageH.MUGA scan
Incorrect Answer ImageI.Synchronized cardioversion
A 67-year-old man comes to the physician because of a 9-month history of burning epigastric
pain. Current medications include omeprazole. He has had no nausea, vomiting, odynophagia, or
dysphagia. Upper endoscopy shows abnormal-appearing gastric mucosa. A biopsy specimen of
the stomach shows neoplastic lymphocytes forming lymphoid follicles and sheets as well as
gram-negative rods; there is no tissue invasion. Immunohistochemical examination of the tissue
shows positivity for CD19, CD20, and CD22, and negativity for CD5, CD10, and CD23. What is
the most appropriate next step in management? 
Correct Answer ImageA.Antibiotics
Incorrect Answer ImageB.Antivirals
Incorrect Answer ImageC.Chemotherapy
Incorrect Answer ImageD.Radiation therapy
Incorrect Answer ImageE.Surgery

A 24-year-old woman comes to the emergency department because of shortness of breath and
wheezing. She reports a history of mild episodic asthma and usually has these symptoms 1 or 2
times a week. She also reports having a couple of nighttime symptoms over the past six months.
The current symptoms began 5 hours ago and have become gradually more severe. She takes no
medications. Her respirations are 28/min. Peak expiratory flow rate is 120 L/min. Oxygen
saturation on room air is 94%. Chest examination shows loud expiratory wheezing bilaterally.
The expiratory phase is mildly prolonged and air movement is fair. She is given a short-acting
beta-adrenergic agonist which resolves the symptoms within 10 minutes. FEV1 after stabilization
is 85% of predicted. Which of the following is the most appropriate long-term management for
this patient?
Correct Answer ImageA.Albuterol inhaler as needed
Incorrect Answer ImageB.Inhaled corticosteroid 1x a day
Incorrect Answer ImageC.Oral prednisone 1x a day
Incorrect Answer ImageD.Oral zafirlukast 1x a day
Incorrect Answer ImageE.Salmeterol inhaler 1x a day

A 34-year-old woman with HIV comes to the physician with her partner because of a 4-day
history of malaise and arthralgias. Her partner reports that the patient has had episodes of
confusion and tingling sensations in her arms and legs for the past 2 hours. The partner is
concerned because the patient used heroin 3 days ago and is still bleeding from the puncture site.
Current medications include zidovudine, didanosine, indinavir, and trimethoprim-
sulfamethoxazole. The patient is oriented to person, but not place and time. Her temperature is
39.1ºC (102.3ºF), pulse is 110/min, respirations are 20/min, and blood pressure is 150/90 mm
Hg. Examination shows scleral icterus. There are non-palpable purpuric spots on the chest and
upper and lower extremities. The lungs are clear to auscultation. Abdominal examination shows
diffuse mild tenderness. The spleen tip is palpated 4 cm below the left costal margin. Laboratory
studies show: 
Hemoglobin  7.1 gm/dL
Platelets  50,000/mm3
WBC count 4,600/mm3
CD4  230/uL
Peripheral smear  Fragmented RBCs
Bleeding time  11 minutes (normal, 2–7 minutes)
Urine protein  500 mg/dL
RBC  50/HPF
Peripheral smear shows red blood cell fragmentation. Which of the following findings is most
likely in this patient? 
Incorrect Answer ImageA.Normal creatinine levels
Incorrect Answer ImageB.Normal indirect bilirubin levels
Incorrect Answer ImageC.Normal lactate dehydrogenase levels
Correct Answer ImageD.Normal prothrobmin time and activated partial thromboplastin time levels
Incorrect Answer ImageE.Normal reticulocyte count
Incorrect Answer ImageF.Positive cryptococcal antigen on lumbar puncture

A 36-year-old man comes to the emergency department because of blurred vision, headache,
shortness of breath, and dizziness that occurred soon after drinking 20 shots of vodka. He was
recently discharged from an inpatient alcohol rehabilitation facility to an Alcoholics Anonymous
12-step program. He has a history of peptic ulcer disease with positive Helicobacter
pylori testing, for which he is undergoing treatment with lansoprazole, metronidazole, and
clarithromycin. Other medications include clonidine and naltrexone. He is allergic to penicillin,
which causes throat swelling. His blood pressure is 134/86 mm Hg, pulse 110/min, and
respirations 20/min. On physical examination the man appears flushed. The pupils are equal,
round, and reactive to light and accommodation. The lung fields are clear and the abdomen is
soft, nontender, and nondistended. Which of the following pharmacotherapies is likely to be the
cause of this patient's symptoms? 
Incorrect Answer ImageA.Clarithromycin
Incorrect Answer ImageB.Clonidine
Incorrect Answer ImageC.Lansoprazole
Correct Answer ImageD.Metronidazole
Incorrect Answer ImageE.Naltrexone

A 20-year-old man attending college comes to the student health clinic because of chest pain for
8 days. The pain is substernal and seems mildly worse with activity, although he has been so
fatigued the last week that he has barely exerted himself. There is some radiation of the pain to
the neck and back. A deep breath worsens the pain, as does recumbency. The pain is improved
by sitting upright and leaning forward. The patient has never had any pain like this before. He
has generally been healthy aside from an upper respiratory infection about 2 weeks ago. There
are no diseases that run in the family, and he takes no medications or illicit substances. His
temperature is 37.0ºC (98.6ºF), pulse is 110/min, respirations are 24/min, and blood pressure
115/75 mm Hg. He has clear lung fields and a normal jugular venous pressure. Cardiac
auscultation shows a scratchy sound heard best at the left lower sternal border. Which of the
following is most likely to confirm the diagnosis? 
Incorrect Answer ImageA.Amylase and lipase levels
Incorrect Answer ImageB.Chest radiograph
Incorrect Answer ImageC.Creatine kinase and troponin level
Incorrect Answer ImageD.Echocardiogram
Correct Answer ImageE.Electrocardiogram
A 28-year-old woman with epilepsy is brought to the emergency department by her brother
because of a 30-minute history of convulsions. Her temperature is 37ºC (98.6ºF), blood pressure
is 108/88 mm Hg, pulse is 102/min, and respirations are 28/min. Her oxygen saturation is 98%
on room air. Intravenous glucose, thiamine, and lorazepam are administered. Stat serum
chemistries are sent, as well as a urine toxicology screen and blood alcohol level. She continues
to seize despite lorazepam. Which of the following is the most appropriate next step in
management?
Incorrect Answer ImageA.Carbamazepine
Incorrect Answer ImageB.Diazepam
Correct Answer ImageC.Levetiracetam
Incorrect Answer ImageD.Midazolam
Incorrect Answer ImageE.Phenobarbital
Incorrect Answer ImageF.Rapid sequence intubation

A 26-year-old woman comes to the physician because of a 3-week history of burning,


nonradiating epigastric and chest pain that worsens with eating. She denies shortness of breath,
nausea, vomiting, or palpitations. She experiences a dry cough at night. She smokes 10 cigarettes
daily and drinks 2 to 3 beers daily. She does not use drugs. She is in no distress. Her weight is 87
kg (192 lb), height 168 cm (66 in), and body mass index is 31 kg/m2. Cardiac examination shows
a normal S1 and S2 with no murmurs. The abdomen is soft, nontender and nondistended. Which
of the following is the most appropriate next step in management?
Incorrect Answer ImageA.Famotidine
Correct Answer ImageB.Lifestyle modification
Incorrect Answer ImageC.Metoclopramide
Incorrect Answer ImageD.Nissen fundoplication
Incorrect Answer ImageE.Pantoprazole

A 34-year-old African American woman comes to the physician because of dyspnea on exertion
and a nonproductive cough. She has previously been in good health, except for a history of
mononucleosis while in college. She does not smoke or drink. She has been monogamous with
her husband since they were married 16 years ago. On physical examination, her temperature is
38.3ºC (100.9ºF), blood pressure is 132/80 mm Hg, pulse is 74/min, and respirations are 22/min.
There are purple areas of swelling on her nose and cheeks. The pharynx is clear. There are
scattered cervical lymph nodes and clubbing of the peripheral fingernail beds. Bilateral
inspiratory crackles are heard on a lung examination. There is a regular S1 and S2. The liver
span is 13 cm in the midclavicular line, and the spleen tip is palpable. A chest x-ray shows
bilateral hilar lymphadenopathy. Which of the following is the most appropriate next step in the
evaluation of this patient?
Correct Answer ImageA.Cervical lymph node biopsy
Incorrect Answer ImageB.HIV antibodies
Incorrect Answer ImageC.Liver biopsy
Incorrect Answer ImageD.Splenectomy
Incorrect Answer ImageE.Transthoracic hilar lymph node biopsy

A 55-year-old man is brought to his physician because of a feeling of pressure and fullness on
the left side of his abdomen. He has also had a gradual onset of fatigue. His physician finds that
his spleen is 6 cm below the left costal margin and firm. In addition, the liver edge is palpable
below the right costal margin; there is no adenopathy. A complete blood count shows a
hematocrit of 27% and a white blood cell count of 3,800/µL with 2,200 neutrophils/µL and 50
monocytes/µL. The lymphocyte count is normal. Platelet count is 60,000/µL. A bone marrow
aspirate is unable to be obtained and a subsequent bone marrow biopsy shows cells that express
the CD11c and CD22 antigens on immunophenotyping. Which of the following is the most
likely diagnosis?
Incorrect Answer ImageA.Aplastic anemia
Incorrect Answer ImageB.Fanconi anemia
Correct Answer ImageC.Hairy cell leukemia
Incorrect Answer ImageD.Non-Hodgkin lymphoma
Incorrect Answer ImageE.Waldenstrom macroglobulinemia

A 58-year-old woman is brought to the emergency department because of severe abdominal


cramping for the past 6 days. She says that her pain is epigastric and periumbilical, and the pain
increases after eating and is relieved when the stomach is empty. She has lost 30 lbs over the past
3 months and has also experienced fatigue and mild intermittent fevers over the same period. She
has a history of hypertension and is currently taking hydrochlorothiazide. Physical examination
shows a diffusely tender abdomen that is worse with deep palpation. Bowel sounds are present.
There is a palpable purpuric rash on the lower extremities bilaterally and necrotic skin changes in
the distal upper extremities. There is swelling and tenderness of the metacarpophalangeal and
proximal interphalangeal joints of both hands. Peripheral neuropathy is also present. Laboratory
studies show an erythrocyte sedimentation rate of 120 mm/hr and serum creatinine level of 2.4
mg/dL. An x-ray of the chest shows no abnormalities. Which of the following is the most likely
diagnosis? 
Incorrect Answer ImageA.Churg-Strauss vasculitis
Incorrect Answer ImageB.Giant cell arteritis
Incorrect Answer ImageC.Henoch-Schönlein purpura
Correct Answer ImageD.Polyarteritis nodosa
Incorrect Answer ImageE.Polymyalgia rheumatica
Incorrect Answer ImageF.Wegener granulomatosis

A 62-year-old man comes to the physician because of progressively worsening dyspnea on


exertion and peripheral edema. He has a history of childhood rheumatic heart disease. Over the
past 3 years, he has also had paroxysmal nocturnal dyspnea and orthopnea. He currently takes
digoxin, furosemide, and enalapril. Physical examination shows a loud, blowing decrescendo
diastolic murmur. There are also bounding peripheral pulses. Which of the following additional
physical findings is most likely to be seen in this patient? 
Incorrect Answer ImageA.Decreased heart rate <60/min
Incorrect Answer ImageB.Greater than a 10 mm Hg decrease in systolic blood pressure with
inspiration
Incorrect Answer ImageC.Increased heart rate >100/min
Incorrect Answer ImageD.Isolated increase in systolic blood pressure
Correct Answer ImageE.Large difference between systolic and diastolic pressures
A 48-year-old man comes to the physician because of a 4-month history of progressive weakness
that is worst in his legs. There are no alleviating or exacerbating factors. He has a history of
hypertension, hypercholesterolemia, gout, and asthma. Current medications include simvastatin,
colchicine, metoprolol, and prednisone. He has had no changes in his medication for 2 years.
Examination shows muscle atrophy of his quadriceps. Muscle strength is 4/5 in the proximal
lower extremities, 5/5 in the distal lower extremities, and 5/5 in the upper extremities.
Laboratory studies show CK 80 mU/mL (normal, 25–145 mU/mL). An EMG shows no
abnormalities. Which of the following is the most appropriate next step in management?
Incorrect Answer ImageA.Acetylcholine receptor antibody assay
Incorrect Answer ImageB.Discontinue the simvastatin
Incorrect Answer ImageC.Increase the dose of prednisone
Incorrect Answer ImageD.Muscle biopsy
Correct Answer ImageE.Taper the dose of prednisone

A 42-year-old man comes to the emergency department after vomiting "bright red blood." He
was watching TV when he suddenly felt nauseated and had to rush to the bathroom, where the
event occurred. He estimates approximately a cupful of blood was vomited. His past medical
history is unremarkable. He denies symptoms of heartburn or dyspepsia. He drinks alcohol
socially. He has never used IV drugs. On physical examination, his temperature is 97.0ºF
(36.1ºC), blood pressure is 110/65 mm Hg and pulse is 92/min. Examination of his oral cavity
confirms blood. Cardiopulmonary examination is normal. There is mild tenderness to palpation
of his epigastrium. He does not have ascites or hepatosplenomegaly. Skin examination shows no
palmar erythema or spider angiomata. There is no clubbing, cyanosis, or edema peripherally. On
endoscopy, blood is confirmed in the gastric lumen. There is no evidence for ulcers, gastric wall
erosion, or esophageal or gastric varices. Which of the following is the most likely etiology for
the patient's presentation?
Correct Answer ImageA.Dieulafoy lesion
Incorrect Answer ImageB.Esophageal varices
Incorrect Answer ImageC.Gastric adenocarcinoma
Incorrect Answer ImageD.Mallory-Weiss tear
Incorrect Answer ImageE.Menetrier disease

ReKap

An acute upper gastrointestinal bleed with a normal endoscopy should make the clinician suspect a
Dieulafoy lesion, which represents an intermittently bleeding vessel that retracts under the
mucosa during non-bleeding periods.

A 55-year-old African American woman comes to the physician because of increasing shortness
of breath with exercise and a dry cough that have been ongoing for the last 4 months. She has
been healthy all her life and is concerned about her limited ability to exercise regularly. She
denies any fever, chills, shakes, nausea, vomiting, diarrhea, chest pain, palpitations, hemoptysis,
or weight loss. She has worked at a bank all her life and is now the branch manager. She does not
take any medications and has no known drug allergies. She has smoked 2 cigarettes/day for the
last 15 years and drinks on the weekends. Her oxygen saturation is 93% on room air. Lungs have
fine crackles at the bases to auscultation. Her heart rate and rhythm are regular. Examination of
the extremities shows clubbing. Chest x-ray shows diffuse linear opacities. Pulmonary function
tests show a restrictive pattern. She has a decreased diffusion capacity. Which of the following is
the most likely diagnosis? 
Incorrect Answer ImageA.Acute interstitial pneumonia
Incorrect Answer ImageB.Asbestosis
Incorrect Answer ImageC.Emphysema
Correct Answer ImageD.Idiopathic pulmonary fibrosis
Incorrect Answer ImageE.Sarcoidosis

A 66-year-old man comes to the physician because of a hacking cough, shortness of breath, and
pleuritic chest pain. Although he states that he a chronic cough, he believes it has become worse
over the past 6 weeks. He has not had any fevers, chills, or night sweats, but has had an
unintentional 4.5 kg (10-lb) weight loss over the last month. When questioned, he admits to a
nearly 80-pack-year history of smoking. His temperature is 37.0ºC (98.6ºF), pulse is 88/min,
respirations are 24/min, and blood pressure is 160/95 mm Hg. Examination shows a man
appearing older than his stated age, with extremity and temporal wasting. On lung auscultation,
there are end-expiratory wheezes. No clubbing of the digits, distal cyanosis, or edema is noted.
Laboratory studies show:
Sodium 123 mEq/L
Potassium 4.1 mEq/L
Chloride 102 mEq/L
Bicarbonate 25 mEq/L
Glucose 139 mg/dL
Creatinine 1.0 mg/dL
Blood urea nitrogen 10 mg/dL
Chest radiograph shows a 2.5-cm right lobe peribronchial mass and mediastinal
lymphadenopathy. Fiberoptic bronchoscopy with biopsy shows lung cancer. Which of the
following is the most likely histologic type of lung cancer in this patient?
Incorrect Answer ImageA.Adenocarcinoma of the lung
Incorrect Answer ImageB.Large-cell carcinoma
Incorrect Answer ImageC.Metastatic cancer
Correct Answer ImageD.Small-cell carcinoma
Incorrect Answer ImageE.Squamous cell carcinoma

A 28-year-old Caucasian man presents to the clinic because of back pain. The pain is
intermittent, dull, and bilateral. The patient states he has had some relief with the use of over-the-
counter ibuprofen. The patient also asks that you evaluate his right testicle. He states that it has
been feeling heavy for the last few months. On examination, the testicles are nontender, and a
firm nodule is palpable on the lateral aspect of the right testicle. Ultrasound examination shows a
homogeneous hypoechoic mass in the right testicle. Which of the following tumor markers is
most likely elevated in this patient's serum? 
Incorrect Answer ImageA.CA-125
Incorrect Answer ImageB.CA 19-9
Correct Answer ImageC.hCG
Incorrect Answer ImageD.PSA
Incorrect Answer ImageE.S-100
A 45-year-old African American man comes to the physician for evaluation of his high blood
pressure. He has had two previous documented readings, with systolic pressure within 140-145
mm Hg and diastolic pressure within 86-90 mm Hg. The patient has not had any significant
illness and he takes no medications. His family history is negative for cardiovascular disease or
diabetes. He denies any episodes of palpitations, sweating, or tremor. He exercises regularly,
does not smoke, and consumes three or four alcoholic drinks daily on weekdays and up to six
drinks on weekends. His blood pressure is 142/88 mm Hg, which was confirmed again after
resting comfortably for 5 minutes. His height is 180 cm (71 in), weight is 75 kg (165 lb), and
BMI is 23.1 kg/m2. Physical examination shows no abnormalities. Which of the following is the
most appropriate next step in management?
Incorrect Answer ImageA.Antihypertensive treatment with a beta-blocker
Incorrect Answer ImageB.Antihypertensive treatment with a diuretic
Incorrect Answer ImageC.Antihypertensive treatment with an ACE inhibitor
Correct Answer ImageD.Restriction of alcohol consumption
Incorrect Answer ImageE.Weight reduction only

A 74-year-old, right-handed man comes to see the physician because of progressive gait
dysfunction. He and his family report a 6- to 7-month history of gradually deteriorating, wide-
based gait and increasing unsteadiness. He has gotten to the point of requiring assistance while
walking. His family also reports that he has had urinary incontinence over the same time period.
His past medical history is significant for several episodes of genital herpes and one episode of
meningitis as a child. He has had multiple sexual partners. On physical examination, he is awake
and alert, but oriented only to person. Strength and sensation are intact. When asked to walk
across the room, he is not able to walk in a straight line and shows a lack of balance. Which of
the following diagnostic tests would provide the most useful information regarding the patient's
diagnosis?
Incorrect Answer ImageA.Electroencephalogram
Incorrect Answer ImageB.Lumbar puncture
Correct Answer ImageC.MRI of the brain
Incorrect Answer ImageD.Rapid plasma reagin
Incorrect Answer ImageE.Vitamin B12 level

A 22-year-old woman comes to the physician because of a 3 month history of mild jaundice,
low-grade fever, arthralgia, malaise, and amenorrhea. Her medical history is unremarkable.
Current medications include oral contraceptives. She does not drink alcohol or smoke cigarettes.
Laboratory studies show:
HAV Negative
HBV Negative
Anti-HCV Positive; by enzyme immunoassay
HCV Negative, by recombinant immunoblot
assay (RIBA)
ALT 345 U/L
AST 285
Antinuclear antibody 1:640
(ANA)
Smooth-muscle 1:320
antibodies (SMA)
Biopsy specimen of the liver shows lymphocytic portal and periportal inflammation with early
bridging necrosis. Which of the following is the most likely diagnosis? 
Correct Answer ImageA.Autoimmune hepatitis
Incorrect Answer ImageB.Chronic hepatitis C
Incorrect Answer ImageC.Hepatic adenoma
Incorrect Answer ImageD.Nonalcoholic steatohepatitis
Incorrect Answer ImageE.Primary biliary cirrhosis

A 42-year-old woman with type 2 diabetes mellitus and hypertension comes to the physician
because of shortness of breath for several months. Her last hemoglobin A1c was 6.5% and she has
no associated diabetic retinopathy, proteinuria, or neuropathy. Her medications include lisinopril
and metformin. Her temperature is 37.0°C (98.6°F), her pulse is 87/min, respirations are 13/min,
blood pressure is 137/62 mm Hg. Pulse oximetry on room air shows an oxygen saturation of
98%. Physical examination shows a mildly displaced point of maximal impulse. The lungs are
clear to auscultation bilaterally with no crackles or dullness to percussion. There is no peripheral
edema. Stress echocardiogram shows no evidence of coronary artery disease, with no systolic or
diastolic dysfunction. Chest x-ray shows no abnormalities. Pulmonary function tests with
spirometry, lung volumes, and diffusion capacity are normal. Which of the following lung
volumes is most likely to have been measured directly in this patient?
Correct Answer ImageA.Expiratory reserve volume (ERV)
Incorrect Answer ImageB.Functional residual capacity (FRC)
Incorrect Answer ImageC.Residual volume (RV)
Incorrect Answer ImageD.Residual volume (RV) / Total lung capacity (TLC)
Incorrect Answer ImageE.Total lung capacity (TLC)

A 63-year-old African American man comes to the physician because of progressive fatigue. He
has noticed that during the past few months, he has been short of breath after walking only a
block. Prior to these past 2 months, he was capable of walking many blocks. He has not noticed
any chest pain, paroxysmal nocturnal dyspnea, lower-extremity swelling, or abdominal
distension. At rest, he has no difficulty breathing. His past medical history includes non-ulcer
dyspepsia and acid reflux disease. He takes ranitidine orally as needed. He is a retired lawyer and
denies smoking or alcohol intake. On physical examination, his blood pressure is 125/72 mm Hg,
pulse is 98 beats/min, and oxygenation is 95% on room air. Head and neck examination shows
conjunctival pallor. No lymphadenopathy or carotid bruits are appreciated. Cardiac examination
shows a 2/6 early systolic murmur and an active precordium. Lung and abdominal examinations
are normal. Extremity examination shows no clubbing, cyanosis, or edema. Laboratory studies
show:
White blood cells 5,600/mm3
Hematocrit 26%
Hemoglobin 9 mg/dL
Mean red blood cell volume 77 fl
Platelets 245,000/mm3
Which of the following is the next step in the management of this patient? 
Incorrect Answer ImageA.CT abdomen and pelvis
Correct Answer ImageB.Fecal occult blood testing
Incorrect Answer ImageC.Intramuscular iron dextran with follow-up in 6 weeks
Incorrect Answer ImageD.Oral ferrous sulfate with follow-up in 6 weeks
Incorrect Answer ImageE.Screening colonoscopy

A 40-year-old man is brought to the emergency department by his friends. They report he has
taken a lot of pills in a suicide attempt. They do not have any of the bottles of medication with
them. His temperature is 39.3ºC (103.0ºF), blood pressure 90/55 mm Hg, pulse 110/min and
irregular, and respirations 22/min. On physical examination the patient is oriented to person and
place but not time. His skin is flushed and dry. Neurologic examination shows dilated pupils.
Electrocardiogram shows widened QRS complexes, greater than 100 msec duration. Laboratory
studies show AST 26 IU/L and ALT 18 IU/L. Arterial blood gas shows pH 7.41. Which of the
following substances is the most likely cause of intoxication? 
Incorrect Answer ImageA.Acetaminophen
Incorrect Answer ImageB.Alprazolam
Incorrect Answer ImageC.Clonidine
Correct Answer ImageD.Desipramine
Incorrect Answer ImageE.Ethanol

A 52-year-old woman with a history of rheumatic heart disease comes to the physician for a
follow-up examination. She reports her symptoms of shortness of breath, difficulty climbing the
stairs, and episodes of waking up short of breath have worsened despite treatment with
hydrochlorothiazide. She has had a cough productive of bloody sputum. Examination shows no
jugular venous distension or pedal edema. Cardiac exam shows normal S1 and S2 and grade 3/6
pansystolic murmur at the apex that increases with expiration. ECG shows no abnormalities.
Echocardiography shows severe mitral regurgitation. She undergoes mitral valve replacement
surgery with a mechanical valve. Which of the following is the most appropriate INR range for
this patient? 
Incorrect Answer ImageA.0 to 1.0
Incorrect Answer ImageB.1.5 to 2.5
Correct Answer ImageC.2.5 to 3.5
Incorrect Answer ImageD.3.5 to 4.5
Incorrect Answer ImageE.4.5 to 5.5

A 39-year-old man comes to the physician because of progressive difficulty walking for 1 week.
He had an episode of right arm weakness 1 year ago that lasted 2 weeks, and he had 10 days of
blurry vision and eye pain 6 years ago. He does not drink alcohol. He has smoked one pack of
cigarettes daily for 19 years. His temperature is 37.1°C (98.8°F), blood pressure is 128/84 mm
Hg, and pulse is 74/min. Examination is shown (see media file). Neurologic examination shows
poor balance, decreased sensation to light touch in the upper extremities, and intentional tremor.
Flexion of the neck produces an electric shock-like pain down the patient's back. There is
nystagmus of both eyes. An MRI of the brain shows multiple T2 hyperintense lesions in the
periventricular white matter and corpus callosum. Which of the following is the most appropriate
next step in management? 
Incorrect Answer ImageA.Azathioprine and methotrexate
Incorrect Answer ImageB.Interferon-beta
Correct Answer ImageC.Intravenous corticosteroids
Incorrect Answer ImageD.Intravenous immunoglobulin (IVIG)
Incorrect Answer ImageE.Plasma exchange therapy

A 61-year-old man comes to the physician because of episodic pain in the upper thighs while
walking. He states that the pain occurs after walking 6 blocks or so, and that it is relieved after he
sits and rests for a few minutes. He describes the pain as dull and aching, and radiating to his
buttocks. He also feels slightly weak when he walks this distance. He has noticed difficulty
obtaining and maintaining an erection. He denies shortness of breath or lower extremity edema.
He has not noticed constipation or difficulty urinating. He has a history of hypertension and is
currently taking hydrochlorothiazide and lisinopril. He has smoked 1 pack of cigarettes daily for
the past 30 years, and he drinks alcohol socially. His pulse is 83/min and his blood pressure is
145/82 mm Hg. Physical examination shows a right carotid bruit. The lungs are clear but there is
a fourth heart sound. Femoral pulses are diminished bilaterally. There is no clubbing or cyanosis
but the lower extremities are cool to palpation. There is also decreased hair growth below the
knees. Neurologic examination shows normal reflexes throughout and 5/5 strength of the lower
extremity muscle groups. He denies shooting pains down his legs with passive hip flexion.
Which of the following is the most appropriate diagnostic test to confirm the diagnosis?
Incorrect Answer ImageA.Angiography
Correct Answer ImageB.Ankle-brachial index
Incorrect Answer ImageC.CT angiogram
Incorrect Answer ImageD.Doppler ultrasonography of the femoral veins bilaterally
Incorrect Answer ImageE.MRI of the lumbar spine

A 34-year-old man comes to the physician because of progressive difficulty swallowing over the
past 4 months. He has had the sensation of meat being stuck in his mid-chest immediately after
eating and frequent episodes of burning chest pain. Over the past 10 days he has had difficulty
swallowing soft foods. He has a 5-year history of gastroesophageal reflux disease. Current
medications include ranitidine, magnesium hydroxide and omeprazole. He has smoked one pack
of cigarettes daily for 15 years and he does not drink alcohol. Physical examination is normal.
Which of the following is the most likely explanation for this patient's symptoms?
Incorrect Answer ImageA.Diffuse esophageal spasm
Incorrect Answer ImageB.Esophageal squamous carcinoma
Incorrect Answer ImageC.Lower esophageal web
Correct Answer ImageD.Peptic esophageal stricture
Incorrect Answer ImageE.Scleroderma

A 65-year-old woman with slowly progressive dementia is brought to the physician by her son.
He says that he once found her talking to an invisible person. Mental status examination shows
severe deficits in attention, visuospatial skills, and verbal fluency, while short-term memory is
only mildly impaired. Physical examination shows jerking movements of the arm with
application of resistance. Gait is difficult for the patient to initiate. MRI of the brain shows mild
diffuse cortical atrophy. Which of the following is the most likely diagnosis?
Incorrect Answer ImageA.Alzheimer-type dementia
Incorrect Answer ImageB.Creutzfeldt-Jakob disease
Incorrect Answer ImageC.Dementia associated with motor neuron disease
Correct Answer ImageD.Dementia with Lewy bodies
Incorrect Answer ImageE.Multi-infarct dementia

A 25-year-old woman, gravida 1, para 0, at 24 weeks' gestation comes to the emergency


department because of worsening dyspnea and orthopnea over the past 10 days. Treatment with
IV furosemide is administered, but there is no improvement in her symptoms. She has a history
of rheumatic heart disease and mitral stenosis. She is currently receiving digoxin. An
echocardiogram from 1 year ago showed a mitral valve area of 1 cm2. Her blood pressure is
100/75 mm Hg and her pulse is 135/min. Physical examination shows jugular venous distention
at 15 cm of H2O and bilateral crackles on auscultation of the chest. Which of the following is the
next best step in management?
Incorrect Answer ImageA.Emergent valve replacement
Incorrect Answer ImageB.Intramuscular betamethasone
Incorrect Answer ImageC.Oral lisinopril
Correct Answer ImageD.Percutaneous balloon valvuloplasty
Incorrect Answer ImageE.Urgent cardioversion
Incorrect Answer ImageF.Urgent cesarean section

A 44-year-old woman comes to her physician's office because of recurrent epigastric and right
upper quadrant abdominal pain occasionally associated with nausea and vomiting. Each episode
lasts for at least 30 minutes. There is no history of heartburn and she does not take aspirin or
other medications. Her past medical history is otherwise unremarkable. There is a family history
of gallstones. Her temperature is 36.7°C (98°F), blood pressure is 110/70 mm Hg, pulse is
88/min, and respirations are 13/min. Her head and neck examinations are normal. Chest is clear
to auscultation. Heart shows normal S1 and S2 with no murmurs, rubs, or gallops. Palpation of
her right upper quadrant shows no focal areas of tenderness. Laboratory data show normal
bilirubin, liver function tests, and lipase levels. A right upper quadrant ultrasound shows the
absence of gallbladder wall thickening, pericholecystic fluid, or cholelithiasis. The common bile
duct is moderately dilated to 8 mm in diameter without stones or strictures. Which of the
following is the most appropriate next step in the evaluation of this patient? 
Incorrect Answer ImageA.Cholecystectomy
Incorrect Answer ImageB.Computed tomography (CT) of the abdomen
Correct Answer ImageC.Endoscopic retrograde cholangiopancreatography (ERCP) with manometry
Incorrect Answer ImageD.Endoscopic ultrasonography
Incorrect Answer ImageE.Magnetic resonance cholangiopancreatography (MRCP)

A 30-year-old woman comes to the emergency department because of progressive weakness for
6 days and difficulty breathing for 1 day. She has had droopy eyelids and difficulty swallowing
and chewing at the end of her meals. She states she had dysuria and urinary frequency that
resolved 8 days ago. She has had fluctuating weakness and fatigability for 3 years. The weakness
has been worse at night and has improved with rest. She is in moderate respiratory distress. Her
blood pressure is 116/74 mm Hg, pulse is 80/min, and respirations are 24/min. Her oxygen
saturation is 96% on room air. Examination shows 3/5 muscle strength bilaterally in the upper
extremities; reflexes are normal. The patient is admitted for respiratory distress. Twelve hours
later, she requires intubation and is transferred to the intensive care unit. Which of the following
is the most appropriate next step in management?
Incorrect Answer ImageA.Antitoxin and antibiotic treatment
Incorrect Answer ImageB.Edrophonium challenge
Incorrect Answer ImageC.Intravenous methylprednisolone
Correct Answer ImageD.Plasmapheresis
Incorrect Answer ImageE.Surgical thymectomy

A 60-year-old man who is walking down the street suddenly clutches his chest and falls to the
ground. A nearby physician comes to the man's side. The man appears diaphoretic and is
unresponsive. Which of the following is the most appropriate next step in management? 
Incorrect Answer ImageA.Begin chest compressions
Correct Answer ImageB.Call for help
Incorrect Answer ImageC.Check for a pulse
Incorrect Answer ImageD.Open patient's airway
Incorrect Answer ImageE.Perform 2 rescue breaths

A 37-year-old woman with a 6-month history of severe heartburn and difficulty swallowing


comes to the physician for a follow-up examination. Her symptoms have been persistent despite
treatment trials with antacids, ranitidine, and omeprazole. She has a history of hypertension and
Raynaud disease. Physical examination shows multiple telangiectasias over the face and taut skin
over the dorsum of both hands. Esophageal manometry is most likely to show which of the
following? 
Correct Answer ImageA.Decreased esophageal peristalsis and decreased lower esophageal sphincter
(LES) pressure
Incorrect Answer ImageB.Decreased esophageal peristalsis and increased LES pressure
Incorrect Answer ImageC.Increased esophageal peristalsis and decreased LES pressure
Incorrect Answer ImageD.Increased esophageal peristalsis and increased LES pressure
Incorrect Answer ImageE.Normal esophageal peristalsis and normal LES pressure

A 42-year-old man comes to the physician because of progressive left-sided weakness for 1
month. He has had blurry vision and difficulty walking. His medical history is unremarkable, and
he takes no medications. He has smoked one pack of cigarettes daily and drinks 8 to 10 beers
daily. He used IV drugs for 8 years, but he quit 6 months ago. His temperature is 37.1°C
(98.8°F), blood pressure is 128/82 mm Hg, and pulse is 82/min. Neurologic examination shows
right-sided hemianopsia, and muscle strength is 3/5 on the left upper and lower extremities with
brisk reflexes on the left side. There is no papilledema. Enzyme-linked immunosorbent assay
(ELISA) for human immunodeficiency virus is positive. CD4+ T lymphocyte count is 40/mm3.
There is cervical lymphadenopathy. A CT scan of the head shows numerous patchy
nonenhancing hypodensities in the subcortical white matter. Which of the following is the most
likely cause of this patient's symptoms? 
Incorrect Answer ImageA.Bilateral internal carotid artery stenosis
Correct Answer ImageB.Demyelination (PML)
Incorrect Answer ImageC.HIV infection of the white matter
Incorrect Answer ImageD.Infection with a protozoa
Incorrect Answer ImageE.Malignancy
A 21-year-old previously healthy woman who is a professional dancer comes to the physician
because of several episodes of near loss of consciousness during a performance. She has no
family history of coronary artery disease. She does not smoke or drink. Her blood pressure is
142/88 mm Hg and her pulse is 84/min and regular. Physical examination shows a brisk carotid
upstroke with a double impulse palpable. There is a loud S4 and a harsh systolic murmur heard
along the left sternal border. The murmur is accentuated during the Valsalva maneuver. An ECG
shows severe left ventricular hypertrophy. Which of the following drugs is most appropriate for
this patient? 
Incorrect Answer ImageA.Captopril
Incorrect Answer ImageB.Digoxin
Incorrect Answer ImageC.Felodipine
Incorrect Answer ImageD.Furosemide
Correct Answer ImageE.Metoprolol

A 45-year-old man comes to the physician because of a 3 month history of progressive


dysphagia for both solids and liquids. He denies pain, fever, or weight loss. He has no medical
history and is not on any medications. His blood pressure is 110/60 mm Hg, pulse is 70/min, and
respirations are 16/min. The lungs are clear to auscultation and there is a normal S1 and S2 and a
grade 2/6 systolic ejection murmur heard at the second right intercostal space. The liver edge and
spleen tip are not palpable. A test of the stool for occult blood is negative. Which of the
following is the most appropriate next step in management?
Incorrect Answer ImageA.24-hour pH monitoring
Incorrect Answer ImageB.Acid perfusion test (Bernstein test)
Correct Answer ImageC.Barium study
Incorrect Answer ImageD.Endoscopy
Incorrect Answer ImageE.Motility study

A 48-year-old man with a history of hypertension and diabetes mellitus is brought to the
emergency room by paramedics. Approximately 6 hours ago he started getting a headache that
he described as "the worst headache of my life." The headache started abruptly. A few minutes
later he passed out and had tonic-clonic seizures that were witnessed by his friends. The friends
called paramedics, and the man was brought to the emergency room. He was postictal on arrival
at the emergency room. His mentation improved over the next hour but he continued to report a
splitting headache. He also reported nausea. Physical examination in the emergency room is
significant for blood pressure 180/100 mm Hg and some neck rigidity. There is no papilledema
on fundoscopic examination. Neurologic examination shows no neurologic deficits. CT scan of
the head shows blood in the subarachnoid space. The patient is admitted to the intensive care
unit. Stat CBC, blood chemistry profile, and coagulation profile are ordered. Which of the
following additional tests should be ordered at this time?
Incorrect Answer ImageA.Carotid Doppler
Correct Answer ImageB.Digital subtraction angiography
Incorrect Answer ImageC.Echocardiography
Incorrect Answer ImageD.Lumbar puncture
Incorrect Answer ImageE.MRI of the brain
A 30-year-old previously healthy woman is brought to the physician because of episodic
dizziness and light-headedness. She states that for the past 3 months she has been feeling
increasingly tired and weak. She reports that she gets short of breath easily and her
breathlessness has also progressively worsened over the past 3 months. She has missed several
days of work because of her symptoms. She has no history of any major medical illnesses and
takes no medications. Physical examination shows cyanosis of the toes and fingers. There is also
bilateral pedal edema extending to the ankles and a fluid wave is detected during abdominal
examination. The liver is palpated 3 cm below the right costal margin. A chest x-ray shows no
infiltrates but has decreased pulmonary vascular markings bilaterally. An echocardiogram shows
an enlarged right atrium and ventricle with flattening of the septum. Tricuspid regurgitation is
also present. Which of the following is the most likely diagnosis?
Incorrect Answer ImageA.Acute myocarditis
Incorrect Answer ImageB.Constrictive pericarditis
Incorrect Answer ImageC.Dilated cardiomyopathy
Correct Answer ImageD.Primary pulmonary hypertension
Incorrect Answer ImageE.Restrictive cardiomyopathy

A 60-year-old man comes to the physician for a routine health maintenance examination. He is
asymptomatic, with no risk factors for colon cancer. There is no history of weight loss, diarrhea,
or tenesmus. There is no significant past medical history. A review of recent laboratory studies
shows two stool specimens that are negative for occult blood, and one other stool positive for
occult blood. Examination of the abdomen shows no abnormalities. A digital rectal examination
shows normal anal sphincter tone and a normal prostate. Anoscopy shows multiple internal
hemorrhoids. Which of the following is the appropriate next step in the management?
Incorrect Answer ImageA.Anti-hemorrhoidal suppositories
Incorrect Answer ImageB.High-fiber diet
Incorrect Answer ImageC.Observation
Incorrect Answer ImageD.Referral for surgical treatment
Correct Answer ImageE.Total colonoscopy

A 55-year-old woman comes to the physician because of numbness, tingling, and burning pain in
both hands for the past 5 months. Her symptoms wake her up at night and are triggered by
carrying heavy objects. The symptoms are relieved by shaking her hands. Her medical history is
unremarkable. She takes no medications. She is a waitress and carries large trays of food on a
daily basis. There is no atrophy of the thenar muscles. Her symptoms are reproduced by wrist
flexion and direct tapping over the ventral side of the wrists. The suspected diagnosis is
subsequently confirmed with nerve conduction studies and EMG. Which of the following is the
most appropriate next step in management?
Incorrect Answer ImageA.Endoscopic carpal tunnel release
Incorrect Answer ImageB.MRI scan of the wrist
Incorrect Answer ImageC.Nonsteroidal anti-inflammatory drugs
Correct Answer ImageD.Splints to keep her wrists in the neutral position
Incorrect Answer ImageE.Steroid injections into the carpal tunnels

A 67-year-old man comes to his primary care practitioner because of a 3-month history of
gradually worsening constipation and blood in his stool. Rectal examination shows bright red
blood mixed with brown stool and a hard, indurated mass. A colonoscopy shows a mass 7 cm
from the anal canal, and biopsy confirms the presence of adenocarcinoma. A surgical resection is
scheduled; during a pre-op workup, his pulse is 54/min. The electrocardiogram shows
consecutively conducted beats with the same PR interval followed by a blocked sinus P wave.
PR interval in the first beat after the block is similar to the PR interval before the block. A pause
encompassing the blocked P wave is exactly twice the cycle length. Which of the following is
the most appropriate next step in management?
Incorrect Answer ImageA.Atropine
Incorrect Answer ImageB.Measurement of cardiac enzymes
Incorrect Answer ImageC.Metoprolol
Incorrect Answer ImageD.Surgical resection of the mass with no cardiac intervention
Correct Answer ImageE.Temporary pacemaker insertion prior to surgery

A 49-year-old woman comes to the clinic because of nausea, vomiting, and periodic fever after
returning from India 2–3 weeks ago. She explains that she had visited distant relatives in rural
India and had “lived like a local.” She ate the local food, submerged herself in the Ganges River,
and spent time working with orphaned children as part of her stay. The only medication she took
during the trip was doxycycline twice daily. She had no vaccinations before leaving the country.
She reports mild to moderate diarrhea, joint pain, and muscle pain occurring shortly after
returning from India. Her temperature is 38.0ºC (100.4ºF), blood pressure is 130/80 mm Hg,
pulse is 98/min, and respirations are 22/min. Physical examination shows a thin woman in no
apparent distress with marked scleral icterus and a palpable, mildly tender liver that extends 2 cm
below the costal margin. Laboratory studies including blood and stool cultures, multiple
infectious serologies, and thin and thick blood smears, are unremarkable. Additional laboratory
studies show:
Leukocyte count 12,600/mm3
Neutrophils 60%
Eosinophils 2%
Monocytes 3%
Lymphocytes 39%
Aspartate aminotransferase (AST) 4,500 U/L
Alanine aminotransferase (ALT) 4,224 U/L
Alkaline phosphatase 245 U/L
Bilirubin 6.7 mg/dL
Hepatitis panel
Anti-HAV IgG Negative
Anti-HAV IgM Positive
Hepatitis B surface antigen Negative
Hepatitis B surface antibody Positive
Anti-Hepatitis B core antibody Negative
Hepatitis C antibody screen Negative
Stool toxin for C. Difficile Pending
Which of the following is the most appropriate next step in management?
Correct Answer ImageA.Evaluate close contacts for hepatitis A immunoglobulin or vaccination
Incorrect Answer ImageB.Evaluate close contacts once the patient's acute infection has subsided
Incorrect Answer ImageC.Hepatitis vaccination with combined A and B vaccine and
immunoglobulin
Incorrect Answer ImageD.No treatment or screening is required; schedule a follow-up visit with
liver function tests
Incorrect Answer ImageE.Start the patient on metronidazole to treat the pseudomembranous colitis

A previously healthy 55-year-old man comes to the physician because of persistent headache
bilaterally on the temporal and occipital/nuchal areas, which he describes as a squeezing,
bandlike pain, over the past 2 months. He developed a sudden severe headache this morning
followed by a seizure. He has smoked one pack of cigarettes daily for 30 years and has a family
history of hypercholesterolemia and coronary artery disease. Neurologic examination shows
decreased sensation and strength in the right half of the body. His blood pressure is 140/90 mm
Hg and pulse is 85/min. A CT of the head is obtained, which shows edema involving both
hemispheres and crossing the corpus callosum. Subsequent brain MRI is performed with
intravenous contrast and shows patchy enhancement extending from the left parietal lobe across
the corpus callosum into the right parietal lobe, as shown. Laboratory studies are normal. Which
of the following is the most likely diagnosis?
Incorrect Answer ImageA.Cerebral infarction
Incorrect Answer ImageB.Cerebral toxoplasmosis
Incorrect Answer ImageC.Demyelinating disease
Incorrect Answer ImageD.Intracerebral hemorrhage due to hypertension
Correct Answer ImageE.Glioblastoma

A 65-year-old man comes to see his primary care physician because of pain in the legs, primarily
in the calves with walking. The pain is usually relieved by rest. The patient's medical history is
significant for hypertension and hyperlipidemia. His medications are hydrochlorothiazide and
atorvastatin. He is a retired college professor. He does not drink alcohol and he quit smoking 6
months ago. His temperature is 36.5ºC (97.7ºF), pulse is 82/min, respirations are 18/min, blood
pressure is 130/74 mm Hg, and oxygen saturation 98% in room air. Physical examination shows
no apparent distress. The lungs are clear to auscultation. Heart sounds are normal with no
murmurs. Abdomen is soft, nontender, and nondistended. Examination of the extremities reveals
decreased femoral pulses and absent pedal pulses. Which of the following is the most appropriate
next test in management?
Correct Answer ImageA.Ankle-brachial index
Incorrect Answer ImageB.Contrast angiography
Incorrect Answer ImageC.CT angiography
Incorrect Answer ImageD.Duplex ultrasonography
Incorrect Answer ImageE.Magnetic resonance angiography (MRA)

A 29-year-old man comes to his primary care doctor for evaluation of a two years history of
intermittent yellow eyes. He has no other symptoms and feels otherwise well. He denies recent
flu-like symptoms, malaise, myalgias, abdominal pain, weight loss, nausea/vomiting, or
hemorrhoids. He had intermittent unprotected sex with different partners. He drinks alcohol
twice weekly and smokes 1 pack of cigarettes per week. His family history is notable only for
hypertension and coronary artery disease. He denies intravenous drug use and has never received
any blood transfusions. On physical examination, he is well developed and well nourished. Vital
signs are normal. There is scleral icterus. Skin is normal, and there is no lymphadenopathy or
gynecomastia. Cardiovascular pulmonary, and abdominal exams are normal. Laboratory studies
show:
White blood cells 8,500/mm3
Hematocrit 44%
Platelets 325,000/mm3
Urea nitrogen 17 mg/dL
Creatinine 0.8 mg/dL
Glucose 104 mg/dL
AST 33 U/L
ALT 25 U/L
Alkaline phosphatase 65 U/L
Total bilirubin 3.2 mg/dL
Conjugated bilirubin 0.3 mg/dL
Albumin 4.1 g/dL
Hepatitis B surface antibody positive
Anti-hepatitis A IgM negative
Anti-hepatitis C antibody negative
Peripheral blood smear no abnormal cells
LDH 85 U/L
Haptoglobin 102 mg/dL
Which of the following is the most likely explanation for the patient's jaundice?
Incorrect Answer ImageA.Acute hepatitis B infection
Correct Answer ImageB.Gilbert syndrome
Incorrect Answer ImageC.Hemolysis
Incorrect Answer ImageD.Mass in pancreatic head
Incorrect Answer ImageE.Primary biliary cirrhosis

A 44-year-old woman comes to the physician because of a 1-month history of a severe headache
that is not relieved by over-the-counter analgesics. The headache is located in the center of her
head. The headache has been constant and is worse in the mornings. She denies fever, weakness,
abnormal sensations, seizures, or loss of consciousness. She has a 15-year history of tension
headaches that are relieved by acetaminophen, and she has a history of breast cancer. She had a
modified radical mastectomy and several courses of chemotherapy 2 years ago. Vital signs are
normal. Physical examination shows no focal neurologic deficit. Which of the following is the
most appropriate next step in management? 
Incorrect Answer ImageA.Administration of dexamethasone
Incorrect Answer ImageB.CT of the head without IV contrast
Correct Answer ImageC.MRI of the brain with IV contrast
Incorrect Answer ImageD.Propranolol
Incorrect Answer ImageE.Sumatriptan

A 26-year-old first-year resident comes to his physician for evaluation. He recently went to the
emergency department with palpitations, and an atrial dysrhythmia was diagnosed. The decision
was made to start an intravenous infusion of diltiazem, which lead to unstable ventricular
tachycardia that required cardioversion. An ECG is shown. Which of the following is the most
likely diagnosis?
Incorrect Answer ImageA.Atrial fibrillation
Correct Answer ImageB.Pre-excitation syndrome
Incorrect Answer ImageC.Premature ventricular contraction
Incorrect Answer ImageD.Supraventricular tachycardia
Incorrect Answer ImageE.Ventricular tachycardia

A 48-year-old woman reports symptoms of pruritus and fatigue for at least the last 6 months. Her
symptoms have been gradual in onset and are not related to any environmental or occupational
exposures. Review of systems is positive for arthralgias and dry mouth and eyes requiring
regular use of normal saline eye drops. Her past medical history, family medical history, and
social history are unremarkable. Her temperature is 37ºC (98.6ºF), blood pressure is 128/88 mm
Hg, pulse is 88/min, and respirations are 20/min. Abdominal examination shows marked
hepatomegaly without splenomegaly or tenderness to palpation. Skin examination shows diffuse
patchy areas of hyperpigmentation as well as linear areas of excoriation that appear to be from
scratching. Laboratory studies show: 
Sodium 139 mEq/L
Potassium 4.4 mEq/L
Chloride 101 mEq/L
Bicarbonate 24 mEq/L 
BUN 12 mg/dL
Creatinine 0.7 mg/dL
Glucose 96 mg/dL
Albumin 3.1 g/dL
AST (SGOT) 58 U/L
ALT (SGPT) 81 U/L 
Alkaline phosphatase 858 U/L
Bilirubin 2.0 mg/dL
GGT 82 U/L
Protein, total 7.9 g/dL
Which of the following is the most appropriate management to delay progression of this patient's
condition? 
Incorrect Answer ImageA.Azathioprine
Incorrect Answer ImageB.Cholestyramine
Incorrect Answer ImageC.Liver transplantation
Incorrect Answer ImageD.Methotrexate
Correct Answer ImageE.Ursodeoxycholic acid

A 72-year-old man is brought to the emergency department because of progressive weakness,


headaches, difficulty with memory, and an inability to perform simple activities of daily living
for 3 days. He states that he no longer knows how to dress himself or brush his hair, and he has
had difficulty using his left hand. He has a history of coronary artery disease, a transient
ischemic attack 2 years ago, and atrial fibrillation. Current medications include warfarin. His
temperature is 36.7°C (98°F), blood pressure is 145/90 mm Hg, and pulse is 85/min. Neurologic
examination shows left-sided neglect. Muscle strength is 2/5 in the left upper extremity, 3/5 in
the left lower extremity, and normal for the right upper and lower extremities. There is a left-
sided homonymous hemianopsia. Sensation is normal. His INR is 1.2. Which of the following is
the most likely location of the patient's lesion? 
Incorrect Answer ImageA.Left parietal lobe
Incorrect Answer ImageB.Right internal capsule
Incorrect Answer ImageC.Right mid pons
Incorrect Answer ImageD.Right occipital cortex
Correct Answer ImageE.Right parietal lobe

A 65-year-old man with a history of hypertension is brought to the emergency department


because of left-sided chest pain radiating to the left arm for the past 2 hours. He has a family
history of coronary artery disease. He appears pale and diaphoretic. His pulse is 95/min and his
blood pressure is 120/90 mm Hg. Physical examination shows crackles at the bases of both lungs
and an S4 is heard on auscultation of the chest. ECG shows a 2 mm ST-elevation across the
precordium. He is treated with thrombolytics, aspirin, and metoprolol. The patient is transferred
to intensive care for further evaluation. His cardiac enzymes and troponins are markedly
elevated. An echocardiogram shows an ejection fraction of 30% and a large anterior wall motion
abnormality. Which of the following drugs is most appropriate for this patient after he is
medically stabilized?
Incorrect Answer ImageA.Amiodarone
Incorrect Answer ImageB.Diltiazem
Incorrect Answer ImageC.Lidocaine
Correct Answer ImageD.Lisinopril
Incorrect Answer ImageE.Procainamide

A 55-year-old man comes to the physician for a routine health maintenance examination. He has
a 4-year history of burning epigastric abdominal pain and chest discomfort that is worse at night
and associated with consuming large meals and drinking alcohol or coffee. He denies weight
loss, difficulty swallowing, pain with swallowing, or nocturnal cough. He weighs 90 kg (198 lb)
and is 173 cm (68 in) tall. His blood pressure is 135/89 mm Hg, pulse is 90/min, and respirations
are 15/min. The remainder of the examination shows no abnormalities. He undergoes a screening
upper endoscopy and colonoscopy. Which of the following is the most likely pathologic finding
on upper endoscopy?
Incorrect Answer ImageA.Conversion of columnar to squamous epithelium
Incorrect Answer ImageB.Dysplastic esophageal epithelium
Incorrect Answer ImageC.Fungating lesion in the mid-esophagus
Correct Answer ImageD.Metaplastic conversion of distal esophagus
Incorrect Answer ImageE.Pale, glossy discoloration of the cardia

A 48-year-old man comes to the emergency department because of burns on his hands after
spilling coffee on himself. He has had shaking of his hands for several years but until recently
this has not caused him significant difficulties. He has had no weakness, loss of consciousness,
or change in vision. He takes no medications. He has had a recent promotion at work. His father's
hands also shook. Cardiac examination shows a normal S1 and S2; no murmurs are heard. When
asked to reach out to grab a pen, his hand begins to shake. There is no tremor at rest. Muscle
strength is 5/5 in all extremities. Sensation is normal. Which of the following is the most
appropriate next step in management?
Incorrect Answer ImageA.Carbidopa-levodopa
Incorrect Answer ImageB.Lorazepam
Incorrect Answer ImageC.MRI of the brain
Incorrect Answer ImageD.Pramipexole
Correct Answer ImageE.Propranolol

A 68-year-old man comes to the physician because of progressive dyspnea over the past year. He
states that he has difficulty with exercise. He must sleep on two to three pillows at night, and he
has noticed swelling in his feet. He denies chest pain. He occasionally has a dry cough but has
not had fevers. He has a history of a heart murmur. Physical examination shows mild jugular
venous distension. Crackles are heard on auscultation of the chest that do not clear with coughing
at the bilateral lung bases. A 3/6 holosystolic murmur is best heard at the apex that radiates to the
left axilla. The abdomen is slightly distended with a positive fluid wave. Mild hepatomegaly is
also seen. There is 2+ pitting edema, with palpable pulses and no cyanosis. An echocardiogram
is ordered. Which of the following will most likely increase this patient's cardiac murmur? 
Incorrect Answer ImageA.Administration of amyl nitrate
Correct Answer ImageB.Hand-grip
Incorrect Answer ImageC.Inspiration
Incorrect Answer ImageD.Standing upright
Incorrect Answer ImageE.Valsalva maneuver

A 24-year-old man comes to the physician because of a 4-month history of abdominal pain and
postprandial diarrhea associated with a 4-kg (9-lb) weight loss. He reports having severe pain
with defecation over the past 2 months. His medical history is unremarkable and he takes no
medications. He has not traveled outside the United States. His temperature is 38.0ºC (100.4ºF).
Abdominal examination shows right lower quadrant tenderness and mild voluntary guarding.
Rectal examination shows a 1.5-cm anal fissure and brown stool that is positive for occult blood.
Which of the following is the most likely cause of this patient's symptoms?
Incorrect Answer ImageA.Gram-negative bacillus
Incorrect Answer ImageB.Mucosal ulceration, without transmural involvement, in the ascending
colon
Incorrect Answer ImageC.Niacin deficiency
Incorrect Answer ImageD.Toxin-producing organism
Correct Answer ImageE.Transmural inflammation in the region of the terminal ileum

A 42-year-old man comes to the physician because of lower back pain for 3 days. He does not
recall any local trauma or strenuous physical activity which may have precipitated the pain. The
pain does not radiate to the buttocks or legs. He has no history of any major medical illnesses
and takes no medications. The patient reports ongoing intravenous drug abuse. His temperature
is 39.4ºC (102.9ºF), pulse is 102/min, respirations are 16/min, and blood pressure is 108/65 mm
Hg. There is mild tenderness along the spine, and straight leg raise testing is normal. Muscle
strength is 5/5 in the lower extremities, and sensation is also normal. Which of the following is
the diagnostic study of choice for this patient’s condition?
Incorrect Answer ImageA.Complete blood count
Incorrect Answer ImageB.Erythrocyte sedimentation rate
Incorrect Answer ImageC.Lumbar puncture with cerebrospinal fluid analysis
Correct Answer ImageD.MRI scan of the lumbosacral spine
Incorrect Answer ImageE.Plain radiographs of the lumbosacral spine

A 40-year-old woman is brought to the emergency department by her fiancé 30 minutes after he
found her lying unresponsive on the floor of their bedroom. An empty bottle of the woman’s
antidepressants was found by her side. Her prescription had been filled earlier that morning. The
patient has a history of hypertension, atrial fibrillation, diabetes, and asthma. In addition to her
antidepressant, she is also currently taking lisinopril, amiodarone, glyburide, prednisone, and
albuterol. She has no known drug allergies. Her temperature is 37.8ºC (100ºF), pulse is 110/min,
respirations are 22/min, and blood pressure is 100/60 mm Hg. Oxygen saturation is 98% on room
air. On physical examination, she is confused, somnolent and has shallow expirations. Her pupils
are dilated and her eyes open to speech only. Her mucous membranes are dry. Abdominal
examination shows decreased bowel sounds. Deep tendon reflexes are normal in all extremities.
A gastric lavage is performed and multiple pill fragments are found. Which of the following
abnormalities is most likely to be seen on ECG?
Incorrect Answer ImageA.Left deviation of the QRS axis
Incorrect Answer ImageB.Shortening of the QT interval
Incorrect Answer ImageC.Sinus bradycardia
Incorrect Answer ImageD.ST-segment elevation
Correct Answer ImageE.Widening of the QRS complex

A 22-year-old female comes to her primary care physician because of diarrhea, bright red blood
in the stool, and abdominal cramping for several weeks. She states that she feels like she is
straining to have a bowel movement but is unable to completely empty her bowels. She has had
episodes like this before over the last year but never sought medical attention because the pain
associated with those earlier episodes was not as severe. She has an uncle with a similar
condition. On examination, her abdomen is mildly tender in both lower quadrants, and non-
distended. She has tender, erythematous nodules on her lower extremities. Colonoscopy shows
edematous, erythematous, and friable bowel mucosa beginning in the rectum and extending
proximally to the descending colon with no areas of sparing. The terminal ileum appears normal.
Histopathology shows mononuclear inflammation isolated to the mucosa and submucosa. Which
of the following is the best treatment option for this patient?
Incorrect Answer ImageA.Corticosteroids
Incorrect Answer ImageB.Infliximab
Incorrect Answer ImageC.Oral azathioprine
Incorrect Answer ImageD.Oral ciprofloxacin and metronidazole
Correct Answer ImageE.Oral mesalazine and mesalazine suppositories

The longtime primary care physician of an 85 year-old-woman is asked to help mediate care
between the family and the hospital's medical service. The patient sustained an anoxic brain
injury during an in-hospital cardiac arrest 1 week ago in which she had a pulseless period that
lasted at least 5 minutes. On physical examination, her vital signs are normal with the assistance
of continuous mechanical ventilation. Pupillary and corneal reflexes are present bilaterally.
There is episodic decorticate rigidity, but no purposeful movement present. An
electroencephalogram (EEG) suggests severe, diffuse cortical damage. The patient's husband
asks the physician if the patient is brain-dead. Which of the following is the most appropriate
response? 
Incorrect Answer ImageA.It is too early to predict brain death by the legal definition
Incorrect Answer ImageB.The decision on brain death must await the completion of an (MRI)
Incorrect Answer ImageC.The diagnosis of brain death can legally be made only by a neurologist
Correct Answer ImageD.The presence of brainstem function and posturing rules out brain death
Incorrect Answer ImageE.The suggestion of severe cortical damage by the EEG implies brain death

A 19-year-old college student comes to the physician because of recurrent chest pain. The pain
has been present for the past few years; it comes on with exercise and is relieved with rest. The
pain is described as being 6/10 in intensity and is located substernally, radiating to the left arm
and jaw. It is associated with diaphoresis and dyspnea. He denies experiencing palpitations,
nausea, or vomiting with the pain. His past medical history is unremarkable, and he does not take
any medications. He denies smoking, consuming alcohol, or using illicit drugs. His father is 57
years old, mother is 51 years old, and he has two younger male siblings ages 12 and 15; all are in
good health. His blood pressure is 115/69 mm Hg, pulse is 68/min, and oxygen saturation is 97%
on room air. His BMI is 20 kg/m2. There are no carotid bruits or jugular venous distension. The
lungs are clear to auscultation. Cardiovascular examination shows a normal S1 and S2 without
murmurs, rubs, or gallops. The point of maximal impulse is focal and nondisplaced. Abdominal
and neurological examinations show no abnormalities. There is no clubbing, cyanosis, or edema.
Laboratory studies show:
Leukocyte count 6,700/mm3
Hematocrit 43%
Platelet count 233,000/mm3
Sodium 139 mEq/L
Potassium 4.3 mEq/L
Chloride 105 mEq/L
Bicarbonate 24 mEq/L
Blood urea nitrogen 19 mg/dL
Creatinine 0.7 mg/dL
HDL 45 mg/dL
LDL 131 mg/dL
Triglycerides 150 mg/dL
TSH 3.1 µU/L
Urine toxicology negative
A chest radiograph is normal. An echocardiogram shows a normal left ventricle, valves, and
aorta. Which of the following is the most likely diagnosis? 
Correct Answer ImageA.Anomalous coronary artery origin
Incorrect Answer ImageB.Cocaine abuse
Incorrect Answer ImageC.Hypertrophic cardiomyopathy
Incorrect Answer ImageD.Kawasaki disease
Incorrect Answer ImageE.Premature atherosclerosis

A 42-year-old alcoholic man comes to the emergency department with severe abdominal pain
that began 3 hours ago. He is well known to the medical staff because of 3 prior hospitalizations
for acute pancreatitis. He states that this pain differs from prior episodes in that it started
suddenly and then became constant, generalized, and increased in severity. He lies motionless on
the stretcher. Abdominal examination shows tenderness, rigidity, and rebound in all four
abdominal quadrants. Bowel sounds are absent. Initial studies are significant for a serum amylase
level of 1,200 U/L and free air under both diaphragms on an upright plain radiograph of the
abdomen. Which of the following is the most likely diagnosis?
Incorrect Answer ImageA.Acute edematous pancreatitis
Incorrect Answer ImageB.Acute hemorrhagic pancreatitis
Incorrect Answer ImageC.Gangrenous acute cholecystitis
Correct Answer ImageD.Perforated viscus
Incorrect Answer ImageE.Ruptured pancreatic pseudocyst

A 45-year-old woman comes to the physician because of a 4-month history of headaches and
changes in her vision. The headaches have been off and on, occur randomly through the day, and
improve with acetaminophen. Her medical history is notable for polycystic ovarian syndrome,
obesity, and diabetes mellitus. Her family history is notable for multiple sclerosis and, thus, she
is concerned that she has it. Examination demonstrates normal central vision but difficulty with
bilateral peripheral vision. Fundoscopic examination is normal without papilledema. Motor,
sensation, and reflex exams are normal. Labs are notable for elevated glucose, which she says
has been more difficult to control over the past few months. Which of the following is the
diagnostic study of choice?
Incorrect Answer ImageA.Computerized tomography (CT) scan
Incorrect Answer ImageB.CSF analysis via lumbar puncture
Correct Answer ImageC.Magnetic resonance imaging (MRI)
Incorrect Answer ImageD.Serum prolactin measurement
Incorrect Answer ImageE.Thyroid function tests

A 60-year-old man is admitted to the hospital because of shortness of breath, dizziness, and chest
pain radiating to his left arm for 90 minutes. He has a history of type 2 diabetes mellitus and
hypertension. Current medications include hydrochlorothiazide and metformin. He drinks 5 to 8
beers daily. He has smoked one pack of cigarettes daily for 38 years. His pulse is 35/min and
blood pressure is 85/60 mm Hg. Examination shows no jugular venous distension. The lungs are
clear to auscultation. Laboratory studies show elevated troponin I and creatine kinase. ECG
shows ST elevation in leads II, III, and aVF. Right-sided ECG shows ST elevations in V4 and
V5. He is administered aspirin, clopidogrel, metoprolol and heparin. Which of the following is
the most appropriate initial step in management?
Incorrect Answer ImageA.Administer dobutamine
Correct Answer ImageB.Administer IV fluids
Incorrect Answer ImageC.Cardiac catheterization and angioplasty
Incorrect Answer ImageD.Insert a temporary pacemaker wire
Incorrect Answer ImageE.Stop the metoprolol

A 42-year-old woman comes to the physician because of epigastric abdominal pain. She has
noticed this symptom for the past two weeks. The pain occurs sporadically 1 to 3 hours after
meals and is not related to any specific types of food. She denies weight loss, hematemesis, or
melena. Her past medical history is significant for heartburn treated with over the counter
antacids. Her temperature is 37ºC (98.6ºF), pulse is 70/min, and blood pressure is 110/70 mm
Hg. Physical examination shows a soft abdomen with tenderness to deep palpation of the
epigastrium. Head, neck, cardiovascular, and chest examinations are normal. There is no
hepatosplenomegaly, ascites, or right upper quadrant tenderness on deep inspiration. The pulses
are normal without peripheral edema. A double contrast upper gastrointestinal barium study is
ordered, and it demonstrates a round ulcer with smooth margins in the duodenal bulb. Which of
the following is the most appropriate next step in management? 
Incorrect Answer ImageA.Biopsy of the ulcer
Incorrect Answer ImageB.Esophageal pH monitoring
Incorrect Answer ImageC.Oral pantoprazole
Incorrect Answer ImageD.Oral ranitidine
Correct Answer ImageE.Urea breath test

A 65-year-old man comes to the physician because of a 20-minute episode of left arm and left
leg weakness that occurred 1 day ago. He has a history of hypertension and
hypercholesterolemia. Current medications include losartan and simvastatin. Examination shows
a carotid bruit on the right. Neurologic examination shows no abnormalities. Carotid duplex
ultrasound shows 75% stenosis of the right internal carotid artery. The patient undergoes right
carotid endarterectomy. Three hours later, examination is shown (see media clip). Which of the
following nerves is most likely affected? 
Incorrect Answer ImageA.Left hypoglossal nerve
Incorrect Answer ImageB.Left marginal mandibular nerve
Correct Answer ImageC.Right hypoglossal nerve
Incorrect Answer ImageD.Right marginal mandibular nerve
Incorrect Answer ImageE.Right recurrent laryngeal nerve

A 58-year-old man is brought to the emergency department via ambulance because of shortness
of breath. For the past 3 hours, he has experienced dyspnea at rest and has been coughing up
sputum. He denies chest pain. His medical history is significant for stable angina. He is a 25
pack-year smoker and does not drink alcohol. His temperature is 37.2ºC (99ºF), pulse is 124/min,
and blood pressure is 101/75 mm Hg. The patient appears in distress and is breathing 60%
oxygen through a facemask. He is unable to complete full sentences. While examining the
patient, he coughs up blood tinged sputum. There is no chest wall tenderness; however,
auscultation shows fine crackles throughout the chest. There are no murmurs or rubs. An
electrocardiogram demonstrates signs of left ventricular hypertrophy. The patient does not
improve with intravenous furosemide, sublingual glyceryl trinitrate, and morphine. Which of the
following is the most appropriate next step in management?
Correct Answer ImageA.Dobutamine
Incorrect Answer ImageB.Dopamine
Incorrect Answer ImageC.Echocardiography
Incorrect Answer ImageD.Hydralazine
Incorrect Answer ImageE.MUGA scan
Incorrect Answer ImageF.Synchronized cardioversion
Incorrect Answer ImageG.Verapamil
A 55-year-old woman with a long-standing history of atrial fibrillation secondary to mitral
regurgitation comes to the emergency department with a painful right foot. The patient reports
that over the past few hours her foot has become more painful and now is completely numb. She
describes the pain as burning and states that it is not relieved by any intervention. She takes
warfarin, atenolol, digoxin, and aspirin. The patient also has a history of heparin allergy,
identified when she was treated for a previous deep venous thrombosis. On physical
examination, her pulse is irregularly irregular. Her lungs are clear, and she has a loud 3/6
holosystolic murmur heard best at the apex. Her right foot is gray and cool to the touch and has
poor capillary refill. Dorsalis pedis and posterior tibial pulses are absent on the right. Her
prothrombin time is 14.4 seconds (INR 1.4). Which of the following is the most appropriate
course of action?
Correct Answer ImageA.Emergently transfer to operating room for embolectomy
Incorrect Answer ImageB.Duplex ultrasonography of the right leg and foot
Incorrect Answer ImageC.Intravenous thrombolysis
Incorrect Answer ImageD.Start IV heparin
Incorrect Answer ImageE.Start subcutaneous low molecular-weight enoxaparin

A 52-year-old man is brought to the emergency department because of acute onset substernal
chest pain. The pain began 30 minutes ago and is associated with mild dyspnea, nausea, and
lightheadedness. He has a history of hypertension and type 2 diabetes mellitus. He has smoked
one pack of cigarettes daily for the past 20 years. He appears to be in moderate distress, is
diaphoretic, and is clutching his chest. His temperature is 37.0°C (98.6°F), blood pressure is
100/50 mm Hg, pulse is 95/min, and respirations are 20/min. Physical examination shows jugular
venous pressure of 11 cm, which increases with inspiration. A normal S1-S2 with an additional
gallop is heard over the left lower sternal border. Lung sounds are clear and the lower extremities
are cool to touch. An ECG is shown. While in the examination room, the patient develops a
headache and becomes increasingly diaphoretic. His blood pressure is 90/48 mm Hg. Which of
the following should be the first step in management?

Incorrect Answer ImageA.IV beta blockers


Correct Answer ImageB.IV fluid administration
Incorrect Answer ImageC.IV nitrates
Incorrect Answer ImageD.IV positive inotropes
Incorrect Answer ImageE.Percutaneous catheterization

A 71-year-old Caucasian man comes to the emergency department because of blurry vision and
blood-tinged urine. The symptoms began when he ran out of his medications 3 days ago and he
has "not had the time" to refill the prescriptions. He has a history of hypertension and is currently
taking a beta-blocker, an ACE inhibitor, and a calcium-channel blocker. His blood pressure is
200/110 mm Hg in both arms. Funduscopic examination shows flame hemorrhages and
papilledema. There is also an abdominal bruit. Urinalysis shows 3+ RBC/hpf and some
dysmorphic red blood cell casts. Which of the following is the next best step in management?
Incorrect Answer ImageA.Admit the patient to the intensive care unit for treatment
Incorrect Answer ImageB.Give the patient his usual daily doses of BP medications and observe him
Correct Answer ImageC.Give the patient IV medication to reduce his BP
Incorrect Answer ImageD.Order a bedside abdominal ultrasound to rule out abdominal aortic
aneurysm
Incorrect Answer ImageE.Order an ECG and observe the patient

A 69-year-old woman is brought to the emergency room because of chest pain. She was at the
store when she suddenly felt a "painful" sensation in her chest that was worse with breathing.
She has a past medical history of hypertension, diabetes, abdominal aortic aneurysm, and
multiple prior episodes of venous thromboembolism including pulmonary embolism and deep
vein thrombosis. Her medications include lisinopril, metformin, and warfarin. The patient is a 40
pack-year smoker and does not drink alcohol. On physical examination she appears in distress.
She is pale and sweating profusely. Her pulse is 110/min, respirations are 20/min, and blood
pressure is 110/70 mm Hg. She is breathing 100% oxygen through a face mask. There is no
jugular venous distension. Heart sounds are muffled with a regular rate and rhythm. Breath
sounds are clear to auscultation. An arterial blood gas shows pH 7.51, pCO2 28 mm Hg, and
pO2 94 mm Hg on 100% O2. Her INR is 1.2. Which of the following will lead to the most likely
diagnosis in this patient?
Incorrect Answer ImageA.Abdominal ultrasound
Correct Answer ImageB.CT pulmonary angiogram of the chest
Incorrect Answer ImageC.Echocardiography
Incorrect Answer ImageD.ECG
Incorrect Answer ImageE.Pacemaker placement

A 57-year-old man comes to the physician because of shortness of breath for the past three
months. He reports progressively worsening dyspnea while climbing the stairs in his house. He
states that he has been using more pillows at night to help with his sleep. He denies chest pain
and dyspnea at rest. Current medications include metoprolol and furosemide. He does not drink
alcohol and denies illicit drug use. His temperature is 37°C (98.6°F), pulse is 58/min,
respirations are 16/min, and blood pressure is 150/90 mm Hg. Oxygen saturation is 98% in room
air. Cardiovascular examination shows a regular heart rate and rhythm with an audible S4. There
are no murmurs or rubs. There is jugular venous distension. Chest auscultation shows end-
inspiratory crackles at the bases. He has lower extremity edema. Echocardiogram shows
increased left ventricular (LV) filling pressures, normal LV volume, and a normal ejection
fraction. Which of the following is the next best step in management?
Incorrect Answer ImageA.Add digoxin
Correct Answer ImageB.Add lisinopril
Incorrect Answer ImageC.Add verapamil
Incorrect Answer ImageD.Reduce the dose of metoprolol
Incorrect Answer ImageE.Transesophageal echocardiogram

A 19-year-old college sophomore comes to the emergency department because she feels very
weak and is having muscle cramping and fasciculations. She has been seeing a counselor for
anxiety and obsessive-compulsive disorder. When asked if she takes medications, she reluctantly
admits to chronic use of laxatives and diuretics to control her weight. Blood chemistry laboratory
values show the following: 
Sodium 136 mEq/L
K +
2.5 mEq/L
Cl− 95 mEq/L
HCO3 −
29 mEq/L
Mg2+ 1.6 mEq/L
pH 7.48
Electrocardiography is most likely to show which of the following? 
Correct Answer ImageA.Increased U-wave amplitude
Incorrect Answer ImageB.Prolongation of the P wave
Incorrect Answer ImageC.Shortening of the QT interval
Incorrect Answer ImageD.Tall, symmetric, peaked T waves
Incorrect Answer ImageE.Widening of the QRS complex

A 30-year-old man comes to the physician for a routine health maintenance examination. He
reports feeling fine. He has no history of any major medical illnesses and he takes no
medications. He only drinks alcohol socially. His temperature is 37.0℃ (98.6℉), pulse is
75/min, blood pressure is 160/70 mm Hg and respirations are 16/min. Physical examination
shows diminished pulses in the lower extremities, including the femoral, popliteal and dorsalis
pedis arteries as compared to the brachial artery. There is a prominent radio-femoral delay. The
rest of the physical examination is unremarkable. A CT scan of the chest with contrast shows a
coarctation of the aorta just above the ligamentum arteriosum. Which of the following conditions
is most likely to be associated with this patient's condition?
Correct Answer ImageA.Aortic stenosis
Incorrect Answer ImageB.Mitral regurgitation
Incorrect Answer ImageC.Mitral stenosis
Incorrect Answer ImageD.Pulmonic stenosis
Incorrect Answer ImageE.Tricuspid regurgitation

A 71-year-old man comes to his physician for follow-up after a recent emergency department
visit. He was admitted to the hospital 5 days ago because of palpitations and dyspnea. He has a
history of congestive heart failure and hypertension. He is currently taking a thiazide diuretic and
an ACE inhibitor. He appears well today and in no distress. His temperature is 36.8°C (98.2°F),
pulse is 100/min and irregularly irregular, respirations are 18/min, and blood pressure is 130/80
mm Hg. Physical examination shows scant bibasilar crackles. A grade 2/6 holosystolic murmur
is also heard, which is loudest at the apex. Jugular venous pressure is 10 cm above the right
atrium. ECG is shown. Which of the following is the most appropriate initial step in
management?
Incorrect Answer ImageA.DC Cardioversion
Incorrect Answer ImageB.Discontinue the ACE inhibitor
Incorrect Answer ImageC.Initiate amiodarone therapy
Correct Answer ImageD.Initiate beta-blocker therapy
Incorrect Answer ImageE.Initiate digoxin therapy
Incorrect Answer ImageF.Initiate furosemide therapy

An 82-year-old woman comes to the physician because of a right-sided throbbing headache


which has occurred intermittently for 2 weeks. She has also had intermittent jaw pain with
chewing and fatigue. She has a 25-year history of hypertension treated with hydrochlorothiazide.
She is in no distress. Her temperature is 37.1ºC (98.8ºF), blood pressure is 125/83 mm Hg, pulse
is 92/min, and respirations are 20/min. Examination shows a weak, tender, but palpable right-
sided temporal artery pulse, as well as tenderness to palpation on the right side of the face.
Which of the following is the most appropriate next step?
Incorrect Answer ImageA.Biopsy of the temporal artery
Incorrect Answer ImageB.Complete blood count
Incorrect Answer ImageC.Magnetic resonance angiography (MRA) of carotid circulation
Correct Answer ImageD.Measure erythrocyte sedimentation rate
Incorrect Answer ImageE.Trial of non-steroidal anti-inflammatory medications (NSAIDs)

A 53-year-old man comes to the physician for a health maintenance examination. He has no
complaints other than occasional headaches. His past medical history is significant only for
slightly elevated systolic blood pressures. The patient does not take any medications and denies
tobacco use although he does drink 3 glasses of wine daily. He works as an office manager and
sleeps 6–7 hours per night. His temperature is 36.8ºC (98.3ºF), pulse is 78/min, respirations are
16/min, and blood pressure is 154/92 mm Hg. His weight is 107 kg (236 lbs), height 185 cm (6 ft
2 in), and body mass index 30.3 kg/m2. Physical examination is unremarkable. The patient states
that he prefers lifestyle modification in lieu of initiating pharmacologic therapy. Which of the
following recommendations would be the most effective in reducing the patient's blood pressure?
Correct Answer ImageA.Adopt the DASH diet
Incorrect Answer ImageB.Exercise routine initiation
Incorrect Answer ImageC.Initiate continuous positive airway pressure at night
Incorrect Answer ImageD.Moderation of alcohol consumption
Incorrect Answer ImageE.Weight reduction

A 62-year-old woman with a history of borderline hypertension comes to the physician because
of a syncopal episode. She states that for the last few months she has been noticing some
generalized weakness. She has also had episodes in which she "nearly fainted." She says that she
passed out after climbing a flight of stairs. She regained consciousness after a minute or so and
there was no evidence of seizures or bowel/bladder incontinence. On physical examination, her
blood pressure is 110/70 mm Hg. There is no orthostasis. Lungs are clear to auscultation. Cardiac
examination shows a systolic murmur best heard at the right second intercostal space, radiating
to the carotids. There are no neurologic deficits. A CT scan of the head shows no abnormalities.
Bilateral carotid Doppler shows 10% stenosis on the right side and 20% stenosis on the left. An
echocardiogram shows the aortic valve area to be 0.9 cm2 and the gradient across the valve is 60
mm Hg. Which of the following is the most appropriate treatment for this patient? 
Correct Answer ImageA.Aortic valve replacement
Incorrect Answer ImageB.Balloon valvuloplasty
Incorrect Answer ImageC.Furosemide
Incorrect Answer ImageD.Left-sided carotid endarterectomy
Incorrect Answer ImageE.No therapy
Incorrect Answer ImageF.Right-sided carotid endarterectomy

A 42-year-old man with a history of IV drug use comes to a community outreach clinic
complaining of fever, malaise, and cough for the last 3 days. Physical examination shows a
cardiac murmur, which the patient states he has been unaware of. Given his recent use of IV
heroin and his chronic symptoms, the man is admitted to the hospital for blood cultures.
Treatment with IV vancomycin and gentamicin is initiated. 3 sets of blood cultures grow
methicillin-resistant Staphylococcus aureus, and a chest radiograph shows multiple bilateral
nodular densities. Treatment with IV vancomycin is continued. Cardiac auscultation is most
likely to show which of the following?
Incorrect Answer ImageA.Continuous machinery murmur throughout the cardiac cycle
Incorrect Answer ImageB.Decrescendo diastolic murmur heard best at mid-precordium
Correct Answer ImageC.Faint murmur that increases in intensity with inspiration
Incorrect Answer ImageD.Holosystolic murmur with radiation of the murmur to the axilla
Incorrect Answer ImageE.Opening snap and mid-diastolic rumbling murmur

A 60-year-old man comes to the physician because of recurrent stable angina. He is currently
receiving verapamil, metoprolol, sublingual nitroglycerin, daily low-dose aspirin, and
atorvastatin. He states that he has been taking all of his medications as instructed but still gets
symptoms of chest pressure when walking up 1 flight of stairs. His heart rate is 60/min and blood
pressure is 125/80 mm Hg. The physician explains the benefits of long-term management. Which
of the following is the most appropriate next step in management?
Incorrect Answer ImageA.Perform cardiac scintigraphy with thallium-201
Correct Answer ImageB.Perform coronary angiography
Incorrect Answer ImageC.Start him on oral fosinopril
Incorrect Answer ImageD.Start him on oral warfarin
Incorrect Answer ImageE.Start him on thrombolytic therapy

A 72-year-old woman is brought to the emergency room by ambulance because of chest pain.
Two hours ago she felt a dull pain at rest that radiated to the jaw. She also felt a distinct chest
tightness and shortness of breath. She has had similar pain before, mostly with exertion. She used
to be able to climb the stairs to her bedroom with little problem, but over the past 2 weeks it has
been impossible and she has been sleeping on the couch downstairs. She feels that this episode is
most likely due to her "heartburn." She has a past medical history of gastroesophageal reflux, for
which she is taking omeprazole. Her temperature is 37.2ºC (98.9ºF), pulse is 110/min, and blood
pressure is 152/96 mm Hg. She is clutching her chest in pain and sweating profusely. Neck
examination shows no abnormalities. Cardiac examination shows distant heart sounds that are
regular in rate and rhythm. There are clear breath sounds bilaterally. There is no ST-segment
elevation and/or left bundle branch block on electrocardiogram. Which of the following steps in
management will have the greatest reduction in mortality?
Incorrect Answer ImageA.100% oxygen, morphine, and nitrates
Incorrect Answer ImageB.Abciximab
Correct Answer ImageC.Aspirin
Incorrect Answer ImageD.Clopidogrel
Incorrect Answer ImageE.Streptokinase
Incorrect Answer ImageF.Tissue-plasminogen activator

A 58-year-old man is brought to the emergency department because of progressive shortness of


breath and lethargy for 3 days. He has a history of end-stage renal disease, hypertension, and
type 2 diabetes mellitus. He receives hemodialysis 3 times weekly, and he missed two
hemodialysis appointments over the past week. He does not drink alcohol or use illicit drugs. His
pulse is 110/min, blood pressure is 160/90 mm Hg, and oxygen saturation is 91% in room air.
Jugular venous pressure is 10 cm H2O. Breath sounds are decreased over the lower third of the
lungs bilaterally. There is 2+ pedal edema and mild generalized anasarca. During the
examination, he suddenly becomes unresponsive and pulseless. Which of the following is the
most likely explanation for this patient's acute findings?
Incorrect Answer ImageA.Acute congestive heart failure
Correct Answer ImageB.Cardiac arrhythmia caused by hyperkalemia
Incorrect Answer ImageC.Hemorrhagic stroke
Incorrect Answer ImageD.Hyperosmolar nonketotic coma
Incorrect Answer ImageE.Uremic pericarditis

A 41-year-old man is brought to the emergency department because of chest pain and shortness
of breath. An ECG shows normal sinus rhythm with no ST-T segment changes. He has no
history of any major medical illnesses and takes no medications. Laboratory studies show
negative cardiac enzymes. He is admitted to the intensive care unit, but 6 hours later reports
severe dizziness. His blood pressure is 70/40 mm Hg. A rhythm strip is shown. Which of the
following is the most appropriate next step in patient care?
Incorrect Answer ImageA.IV digoxin
Incorrect Answer ImageB.Start IV diltiazem drip
Incorrect Answer ImageC.Start IV heparin drip
Incorrect Answer ImageD.Start warfarin
Correct Answer ImageE.Synchronized cardioversion
Incorrect Answer ImageF.Unsynchronized cardioversion

A 58-year-old farmer with a history of rheumatic fever comes to the physician for a routine
physical examination. He states that he is selling his farm, retiring, and changing his health
insurance policy, which requires a complete history and physical. He has no history of any major
medical illness and takes no medications. He appears plethoric. Physical examination shows an
irregularly irregular pulse. Palpation of the precordium shows a tapping apex beat. Breath sounds
are clear bilaterally. Auscultation of the chest with the patient in the left lateral position shows a
diastolic murmur heard best with the stethoscope bell on expiration. An ECG shows atrial
fibrillation. Which of the following is the most appropriate initial diagnostic test? 
Incorrect Answer ImageA.Dobutamine echocardiography
Incorrect Answer ImageB.Left heart catheterization and angiography
Incorrect Answer ImageC.MUGA scan
Incorrect Answer ImageD.Stress test
Incorrect Answer ImageE.Transesophageal echocardiography
Correct Answer ImageF.Transthoracic echocardiography

A 69-year-old woman comes to the emergency department because of acute onset left-sided
chest pain associated with shortness of breath and diaphoresis. She has a history of type 2
diabetes mellitus, hypertension, and hypercholesterolemia. ECG shows ST elevation in leads V1
through V4. She is promptly given appropriate medical treatment and undergoes cardiac
catheterization and angioplasty with stenting of the proximal left anterior descending artery.
Twelve hours later, the patient reports increasing lightheadedness and dizziness. New ECG is
shown. Which of the following is the most appropriate next step in management?
Incorrect Answer ImageA.Amiodarone IV
Incorrect Answer ImageB.Defibrillation with 200 Joules (J)
Incorrect Answer ImageC.Glucagon IV
Incorrect Answer ImageD.Placement of a permanent pacemaker
Correct Answer ImageE.Transcutaneous pacing
Incorrect Answer ImageF.Transvenous pacing

A 40-year-old man comes to the physician because of a 4-month history of shooting pain in his
feet. He has had bluish discoloration of his feet. He has also had extreme cold intolerance and his
legs have become pale when elevated. He has no polyuria or polydipsia, fatigue, weight loss,
chest pain, or swelling. His medical history is unremarkable. He has smoked 2 packs of
cigarettes daily for 20 years. His pulse is 77/min and his blood pressure is 129/74 mm Hg. His
oxygen saturation is 96% in room air. Examination of the head and neck shows no abnormalities.
There is no jugular venous distension, lymphadenopathy, or carotid bruits. The lungs are clear to
auscultation. Cardiac examination shows a normal S1 and S2. No murmurs are heard. There is no
abdominal tenderness. His brachial, radial, femoral, and popliteal pulses are 2+. His dorsalis
pedis and posterior tibial pulses are decreased. There is a small ulcer on the right foot.
Laboratory studies show:
Na+ 141 mEq/L
K+ 4.1 mEq/L
Cl −
105 mEq/L
HCO3− 25 mEq/L
Blood Urea Nitrogen  17 mg/dL
Creatinine  0.9 mg/dL
Glucose  87 mg/dL
Hemoglobin A1c 5.5%
Angiography of the lower extremities shows a tortuous pattern of peripheral arterioles with
segmental involvement. The proximal arteries are spared. Which of the following is the most
likely diagnosis? 
Incorrect Answer ImageA.Acute limb ischemia
Incorrect Answer ImageB.Berger disease
Incorrect Answer ImageC.Peripheral artery disease
Incorrect Answer ImageD.Raynaud phenomenon
Correct Answer ImageE.Thromboangiitis obliterans

A 70-year-old man is brought to the emergency department because of exertional angina and
dyspnea for the past 3 months. Physical examination shows no abnormalities. An ECG shows
ST-segment elevation in leads I, aVL, and V5-V6. A coronary angiogram shows patchy disease
of the left anterior descending artery, the circumflex artery, and the right coronary artery.
Echocardiogram shows moderate to severe aortic stenosis. Which of the following is the most
appropriate next step in management?
Correct Answer ImageA.Aortic valve replacement and coronary artery bypass graft
Incorrect Answer ImageB.Coronary artery bypass graft with delayed aortic valve replacement in 2
months
Incorrect Answer ImageC.Percutaneous coronary intervention with angioplasty and stenting, if
necessary
Incorrect Answer ImageD.Schedule coronary artery bypass grafting
Incorrect Answer ImageE.Schedule percutaneous balloon valvuloplasty plus coronary angioplasty

A 45-year-old man suddenly loses consciousness and falls to the ground at work. His coworkers
immediately call 911. When paramedics arrive, he has a weak pulse. His friends report that he
complained of feeling dizzy and sweaty a few minutes before he collapsed. He was previously
healthy and had not been on any medications that they can recall. Examination shows an
unconscious man with no evidence of trauma. His airway is clear and he is breathing. An ECG
monitor shows a wide complex tachycardia at a rate of 200/min. Which of the following is the
most appropriate next step in management? 
Incorrect Answer ImageA.Administer a bolus of IV lidocaine
Incorrect Answer ImageB.Administer tPA
Incorrect Answer ImageC.Obtain complete vital signs
Incorrect Answer ImageD.Perform asynchronous cardioversion
Correct Answer ImageE.Perform synchronous cardioversion

A 61-year-old man who is a dentist comes to the physician because of chest pain with exertion.
He says that he has been in excellent health his whole life and does not smoke or drink. In
addition, he jogs 2 miles daily. Both of his parents lived into their nineties and died of "natural
causes," and his three siblings are alive and well. His pulse is 74/min, respirations are 16/min,
and blood pressure is 160/94 mm Hg. Physical examination shows a regular S1 and S2, and an
S4 is heard. There is a systolic crescendo-decrescendo murmur at the right second intercostal
space radiating to the carotid arteries. An ECG shows left ventricular hypertrophy. An
echocardiogram is performed, but the patient has poor acoustic windows and none of the
chambers or valves could be adequately visualized. Which of the following is most appropriate
to confirm the most likely diagnosis?
Incorrect Answer ImageA.Cardiac catheterization
Incorrect Answer ImageB.Exercise stress test
Incorrect Answer ImageC.Pulmonary artery capillary wedge pressure measurement
Incorrect Answer ImageD.Thallium stress test
Correct Answer ImageE.Transesophageal echocardiogram

A 56-year-old woman is brought to the emergency department because of shortness of breath.


She had been driving home from work when she suddenly began to have difficulty breathing.
She says she began to cough up sputum but did not have chest pain. Her medical history is
significant for type 2 diabetes mellitus, panic disorder, and stable angina. She has smoked one
pack of cigarettes daily for the past 25 years. Her temperature is 37.4ºC (99.3ºF), pulse is
124/min, and blood pressure is 152/94 mm Hg. She appears to be in distress and is unable to
complete full sentences. Physical examination shows no chest wall tenderness. There is jugular
venous distention. Fine crackles and rales are heard on auscultation of the chest. There are no
murmurs or rubs. An ECG shows no abnormalities. Treatment with IV furosemide, sublingual
nitroglycerin, and morphine is begun, but do not relieve her symptoms. IV dobutamine is begun
and 10 minutes later, her blood pressure is 75/60 mm Hg. Which of the following is the most
appropriate next step in management?
Correct Answer ImageA.Dopamine
Incorrect Answer ImageB.Echocardiography
Incorrect Answer ImageC.Hydralazine
Incorrect Answer ImageD.MUGA scan
Incorrect Answer ImageE.Synchronized cardioversion
Incorrect Answer ImageF.Verapamil

A 56-year-old man is brought to the emergency department because of chest pain and shortness
of breath with exertion. He has no history of any major medical illnesses and takes no
medications. An exercise stress test shows findings suggestive of coronary artery disease, and the
patient undergoes a cardiac catheterization. There is diffuse coronary artery disease but stent
placement is not deemed possible. The patient is medically managed and remains in the hospital
for the following 3 days. His pulse and blood pressure readings are shown. 
Day 1: blood pressure 146/96 mm Hg, pulse 80/min
Day 2: blood pressure 150/90 mm Hg, pulse 86/min
Day 3: blood pressure 140/96 mm Hg, pulse 73/min
Which of the following drugs is most appropriate for this patient? 
Incorrect Answer ImageA.Diltiazem
Incorrect Answer ImageB.Hydrochlorothiazide
Incorrect Answer ImageC.Lisinopril
Correct Answer ImageD.Metoprolol
Incorrect Answer ImageE.Nifedipine

A 25-year-old man sustained multiple stab wounds to the abdomen when he was mugged while
jogging in the park. He was dumped by the attackers behind thick vegetation and was not found
until the next morning. Exploratory surgery reveals multiple small bowel and colonic lacerations,
all of which are repaired. In the postoperative period, the patient has persistent hypotension
despite multiple fluid boluses. His central venous pressure is 12 mm Hg. A pulmonary artery
catheter is placed and reveals a high cardiac output and low peripheral resistance. Which of the
following is the most likely diagnosis?
Incorrect Answer ImageA.Adrenal insufficiency
Incorrect Answer ImageB.Cardiogenic shock
Incorrect Answer ImageC.Hypovolemic shock
Incorrect Answer ImageD.Neurogenic shock
Correct Answer ImageE.Septic shock

A 49-year-old man with a history of hypertension and hyperlipidemia comes to the physician
because of a 2-day history of a severe headache and blurry vision. He also has blood in his urine.
Current medications include atenolol, hydrochlorothiazide, and lovastatin. His temperature is
37.0ºC (98.6ºF), pulse is 83/min, respirations are 10/min, and blood pressure is 196/140 mm Hg.
Physical examination shows no abnormalities. Which of the following is the most appropriate
next step in management? 
Incorrect Answer ImageA.Administer IV hydralazine and admit the patient to the hospital
Correct Answer ImageB.Administer IV nitroprusside and admit the patient to the hospital
Incorrect Answer ImageC.Administer oral clonidine and recheck blood pressure in 30 to 60 minutes
Incorrect Answer ImageD.Administer oral felodipine and recheck blood pressure in 24 to 48 hours
Incorrect Answer ImageE.Increase the doses of his current medications and recheck blood pressure
in 1 week
Incorrect Answer ImageF.Schedule an outpatient CT scan of the head and recheck blood pressure in
1 week

A 34-year-old woman with HIV and active Pneumocystis pneumonia is brought to the hospital


and admitted for further management. Her last CD4 count was 44 cells/mm3(normal 400-1,400
cells/mm3). She has been doing reasonably well since admission with a stable course on
appropriate antibiotics. On day 2 of admission, her temperature is 38.3°C (101°F) orally, pulse is
110/min, and blood pressure is 80/40 mm Hg. Her extremities are warm. Mental status
examination is normal. The remainder of her physical examination is unchanged. Which of the
following is the most appropriate next step in management?
Correct Answer ImageA.Administer intravenous fluids
Incorrect Answer ImageB.Blood transfusion
Incorrect Answer ImageC.Emergent echocardiogram
Incorrect Answer ImageD.Empiric stress dose steroids
Incorrect Answer ImageE.Intravenous pressor support

A 62-year-old man with a history of diabetes mellitus and hypercholesterolemia is brought to the
emergency department because of retrosternal chest pain for the past 90 minutes. He states that
the severity of the pain is 9/10 on the pain scale. The pain radiates to his left arm and is not
relieved by nitroglycerin. An ECG shows ST-segment elevation in leads V1 through V5.
Physical examination shows no abnormalities. His temperature is 36.8°C (98.2°F), blood
pressure is 130/87 mm Hg, pulse is 90/min, and respirations are 18/min. Which of the following
drugs is the most effective in reducing mortality associated with this patient’s condition?
Incorrect Answer ImageA.ACE inhibitor
Correct Answer ImageB.Aspirin
Incorrect Answer ImageC.Beta-blocker
Incorrect Answer ImageD.Calcium channel blockers
Incorrect Answer ImageE.Morphine

A 70-year-old woman comes to the physician because of worsening exertional chest discomfort
and shortness of breath. She has a history of asthma, diabetes mellitus, and a myocardial
infarction 2 years ago. She is currently taking sublingual nitroglycerin, aspirin, metformin, and
atorvastatin. Her pulse is 75/min and her blood pressure is 140/95 mm Hg. Physical examination
shows no abnormalities. Basic laboratory studies are unremarkable. A stress test reveals
reversible ischemia in the inferior wall. A coronary angiogram shows a 70% stenosis in the right
coronary artery. Which of the following is the most appropriate treatment for this patient? 
Incorrect Answer ImageA.Amlodipine
Incorrect Answer ImageB.Diltiazem
Incorrect Answer ImageC.Hydrochlorothiazide
Incorrect Answer ImageD.Isosorbide dinitrate
Correct Answer ImageE.Metoprolol

A 34-year-old man comes to the emergency department with chest pain. He states that he was
dancing at a nightclub when he started to experience a gradually worsening, dull chest pain. He
continued dancing, but when the pain became severe he asked his friend to drive him to the
nearest hospital. He suspects that somebody may have spiked his drink. His vital signs are
temperature 38.2ºC (100.7ºF), pulse 160/min, blood pressure 166/98 mm Hg. On physical
examination, the patient is sweating profusely and is in a great deal of pain. He is currently
breathing 100% oxygen through a face mask. He appears extremely agitated, proclaiming, "Get
these bugs off me!" An electrocardiogram shows ST-segment depression. Which of the
following is the most appropriate next step in management?
Correct Answer ImageA.Aspirin, nitrates, and IV diazepam
Incorrect Answer ImageB.Aspirin, nitrates, and IV esmolol
Incorrect Answer ImageC.Coronary angiogram
Incorrect Answer ImageD.Coronary artery bypass grafting
Incorrect Answer ImageE.Dobutamine echocardiography
Incorrect Answer ImageF.Percutaneous coronary intervention
Incorrect Answer ImageG.Stress echocardiography
Incorrect Answer ImageH.Stress thallium

A 59-year-old man who is employed as a ranch hand comes to the emergency department
because of chest pain. Over the past 8 months, he has noticed a dull, central chest pain that
radiates to his left arm and jaw while walking. The pain subsides after about 2 minutes of rest but
quickly returns upon walking again. This is severely affecting his work, and he is concerned that
he will lose his job because of poor productivity. His vital signs are temperature 37.0ºC (98.6ºF),
pulse is 74/min, and blood pressure is 135/82 mm Hg. Stress test shows ST-segment depression
in leads I, aVL, V4, V5, and V6. Aspirin, nitrates, and metoprolol are initiated. A 12-hour fasting
serum LDL cholesterol concentration is 140 mg/dL. He is also started on atorvastatin and
advised to follow a low-fat diet. Two months later the patient returns and is still experiencing
chest pain during exercise. However, he states that his productivity at the farm has increased. His
resting pulse is 58/min. An echocardiogram shows an ejection fraction of 55%. What is the next
best step in the management of this patient?
Incorrect Answer ImageA.Administer eptifibatide and then perform a percutaneous transluminal
coronary angioplasty
Incorrect Answer ImageB.Administer tirofiban
Correct Answer ImageC.Coronary angiography
Incorrect Answer ImageD.Coronary artery bypass graft (CABG)
Incorrect Answer ImageE.Lisinopril

A 47-year-old African American man with a history of hypertension comes to the physician for a
routine examination. He is currently taking hydrochlorothiazide and has no other complaints. His
temperature is 37.0ºC (98.6ºF), blood pressure is 160/90 mm Hg, pulse is 83/min, and
respirations are 10/min. Physical examination shows no abnormalities and laboratory studies are
shown.
Na+ 142 mEq/L
K +
3.9 mEq/L
Cl− 109 mEq/L
HCO3 −
22 mEq/L
Glucose  96 mg/dL
BUN  13 mg/dL
Creatinine  0.6 mg/dL
Which of the following is the most appropriate next step in management? 
Correct Answer ImageA.Add a second antihypertensive medication
Incorrect Answer ImageB.Schedule for a renal duplex doppler ultrasound
Incorrect Answer ImageC.Send a 24-hour urine collection for cortisol
Incorrect Answer ImageD.Send a 24-hour urine collection for vanillylmandelic acid, metanephrine,
and catecholamine
Incorrect Answer ImageE.Send urine and serum aldosterone levels

A physician is called to see a 69-year-old woman who underwent cardiac catheterization via the
right femoral artery earlier that morning. She is now complaining of a cool right foot.
Examination shows a pulsatile mass over her right groin with loss of her distal pulses, and a bruit
is heard on auscultation over the point at which the right femoral artery was entered. Which of
the following is the most likely diagnosis?
Incorrect Answer ImageA.Cholesterol emboli syndrome
Incorrect Answer ImageB.Femoral aneurysm
Incorrect Answer ImageC.Femoral hernia
Correct Answer ImageD.Femoral pseudoaneurysm
Incorrect Answer ImageE.Retroperitoneal hematoma

A 48-year-old woman comes to her primary care physician complaining of shortness of breath.
She has a history of shortness of breath for the past several years, and it has worsened over the
last 6 months. There is associated orthopnea and leg swelling. Her exercise tolerance is limited
by shortness of breath to about half a block of walking. She has no past medical history and does
not use alcohol or recreational drugs. Cardiac examination shows an increased S1 with an
opening snap and a soft diastolic murmur at the apex. There is also a systolic murmur over the
anterior left sternal border. The electrocardiogram shows sinus tachycardia with right bundle
branch block. A chest x-ray shows pulmonary edema and straightening of the left heart border.
What is the most likely diagnosis?
Incorrect Answer ImageA.Aortic insufficiency
Incorrect Answer ImageB.Aortic stenosis
Incorrect Answer ImageC.Hypertrophic obstructive cardiomyopathy
Correct Answer ImageD.Mitral stenosis
Incorrect Answer ImageE.Mitral valve prolapse

A 64-year-old woman comes to the emergency department because of intermittent chest


discomfort. She reports substernal pressure that occasionally radiates to her left arm and lasts
approximately 10 minutes. She began noticing it when she walked distances greater than a half
mile. She has a history of hypertension, dyslipidemia, and gout. She is currently taking a baby
aspirin daily. She has smoked one half pack of cigarettes daily for the past 45 years. Physical
examination shows a delayed carotid upstroke and a systolic ejection murmur heard best at the
second intercostal space at the right sternal border. A soft S2 without splitting and an S4 are also
present. If the patient is instructed to perform a hand-grip maneuver, which of the following
effects is most likely to be seen? 
Incorrect Answer ImageA.Cause the S2 to split
Correct Answer ImageB.Decrease the intensity of murmur
Incorrect Answer ImageC.Increase the intensity of murmur
Incorrect Answer ImageD.No change
Incorrect Answer ImageE.Systolic thrill palpable over the femoral artery

A 69-year-old man comes to the clinic for his 6-month follow-up visit. He reports that he was
feeling fine until 6 weeks ago when he started having palpitations at different times of the day.
He also reports feeling fatigued and short of breath. He has a history of a myocardial infarction
one year ago and diverticulosis with recurrent GI bleeding. His current medications include
aspirin, hydrochlorothiazide, atorvastatin, and lisinopril. An ECG shows inconclusive findings,
so he was put on a Holter monitor for 5 days, which showed bradycardia alternating with brief
episodes of atrial flutter. His pulse is 50/min, respirations are 15/min, and blood pressure is
110/75 mm Hg. Which of the following is the most definitive treatment for the management of
this patient? 
Incorrect Answer ImageA.Atropine
Incorrect Answer ImageB.Heparin
Incorrect Answer ImageC.Isoproterenol
Correct Answer ImageD.Ventricular pacemaker
Incorrect Answer ImageE.Warfarin

A 55-year-old woman comes to the physician because of chest pain for the past 6 months. She
reports left-sided chest pressure that lasts for 2 to 3 minutes on exertion, which is relieved with
rest. There is no radiation of the chest pressure and she denies shortness of breath, nausea, or
palpitations. She states that these symptoms started approximately 6 months ago and have
remained stable since that time. She has very limited exercise tolerance because of arthritic knee
pain. She has a history of hypertension and asthma. She is currently taking a calcium channel
blocker and a thiazide diuretic. She used to smoke two packs of cigarettes daily for the previous
30 years, but quit 12 years ago. She denies any family history of heart disease. Her temperature
is 37.0°C (98.6°F), pulse is 88/min, respirations are 15/min, and blood pressure is 140/85 mm
Hg. Physical examination shows no abnormalities. Which of the following is the next best step in
evaluating this patient for coronary artery disease? 
Incorrect Answer ImageA.Adenosine stress test
Incorrect Answer ImageB.Cardiac catheterization
Correct Answer ImageC.Dobutamine stress test
Incorrect Answer ImageD.Exercise echocardiogram stress test
Incorrect Answer ImageE.Exercise electrocardiogram stress test

A 59-year-old woman comes to the physician for a follow-up evaluation of her hypertension. She
has attempted weight loss and dietary restriction for the past 6 months, but it has been
unsuccessful and in fact she has had a 4.5 kg (10-lb) weight loss. She has a history of intermittent
migraines (for which she takes acetaminophen), Raynaud phenomenon, and asthma. She
currently uses a rescue inhaler once or twice a week. Her blood pressure is 155/85 mm Hg.
Which of the following drugs is most appropriate for this patient's hypertension?
Incorrect Answer ImageA.ACE inhibitor
Incorrect Answer ImageB.Alpha-blocker
Incorrect Answer ImageC.Beta-blocker
Correct Answer ImageD.Calcium-channel blocker
Incorrect Answer ImageE.Loop diuretics
A 55-year-old man comes to the emergency department because of the sudden onset of shortness
of breath and severe chest pain that started 20 minutes ago. He reports no sweating, nausea, or
vomiting. His past medical history is significant for a myocardial infarction two years ago.
Current medications include aspirin, atenolol, lisinopril, hydrochlorothiazide, and lovastatin. His
temperature is 36.7ºC (98.0ºF), pulse is 95/min, respirations are 18/min, and blood pressure is
110/60 mm Hg. His oxygen saturation is 98% on room air. During the examination, the patient
becomes unresponsive. An ECG is shown. Which of the following is the most appropriate next
step in management?
Incorrect Answer ImageA.Administer 1 mg of epinephrine
Incorrect Answer ImageB.Administer a bolus of IV amiodarone
Incorrect Answer ImageC.Administer tPA
Incorrect Answer ImageD.Angioplasty
Incorrect Answer ImageE.DC cardioversion
Correct Answer ImageF.Defibrillate with 200 Joules (J)

A 42-year-old woman comes to the physician because of a 4-month history of worsening


shortness of breath. There is no past medical history. On physical examination her temperature is
37.1ºC (98.8ºF), blood pressure is 138/86 mm Hg, and pulse is 84/min. Lungs are clear. S1 is
normal but there is a fixed split of S2. There is a midsystolic ejection murmur over the left
sternal border. No edema is noted. Chest x-ray shows a prominent pulmonary artery, increased
pulmonary congestion, and cardiomegaly. Electrocardiogram indicates a right bundle branch
block. Which of the following is the most likely diagnosis?
Incorrect Answer ImageA.Aortic stenosis
Correct Answer ImageB.Atrial septal defect
Incorrect Answer ImageC.Hypertrophic obstructive cardiomyopathy
Incorrect Answer ImageD.Mitral stenosis
Incorrect Answer ImageE.Primary pulmonary hypertension
Incorrect Answer ImageF.Ventricular septal defect

 38-year-old woman comes to the physician because of dyspnea on exertion and chest pain for 3
weeks. She has been getting fatigued easily for the last few months but only recently has noticed
shortness of breath at rest or with minimal exertion. She has not had similar symptoms before.
She has no significant medical problems and does not see a physician regularly. She denies any
history of hypertension, diabetes, or heart disease. Oxygen saturation is 95% on room air.
Physical examination shows a loud second heart sound and trace pedal edema. A 12-lead ECG
shows no abnormalities. A nuclear stress test does not show any evidence of ischemia. A 2-D
echocardiogram shows moderate dilatation of the right ventricle with a normal left ventricular
ejection fraction. She has normal pulmonary function tests and a normal CT scan of the chest.
The patient undergoes a right heart catheterization, which shows a mean pulmonary artery
pressure of 56 mm Hg. A diagnosis of group 1 idiopathic pulmonary hypertension is made.
Which of the following is the most appropriate next step in management?
Incorrect Answer ImageA.Beta-blocker
Incorrect Answer ImageB.Calcium channel blocker
Correct Answer ImageC.IV prostacyclin
Incorrect Answer ImageD.No treatment
Incorrect Answer ImageE.Oxygen at 2 L/min
Several months after sustaining a crushing injury to his arm, a patient complains of constant,
burning, agonizing pain that does not respond to analgesic medications. The pain is aggravated
by the slightest stimulation, such as rubbing against the shirtsleeves. The arm is cold, cyanotic,
and moist, but not swollen. The radial pulse and neurologic examinations are normal. Which of
the following is most appropriate to provide diagnostic confirmation of the problem and
treatment? 
Incorrect Answer ImageA.Angiogram and subclavian vein bypass
Incorrect Answer ImageB.Cervical spine x-rays and cervical rib resection
Incorrect Answer ImageC.Doppler studies and arterial reconstruction
Incorrect Answer ImageD.Doppler studies and fasciotomy
Correct Answer ImageE.Sympathetic block and surgical sympathectomy

A 48-year-old woman presents to the office after referral by her gynecologist for the workup of
anemia following several months of fatigue. She has a history of type 1 diabetes mellitus.
Review of systems is positive for dyspepsia and lower-extremity paresthesia. Medications
include insulin and lisinopril. Examination is significant for reduced vibration sense in the lower
extremities. Hematocrit is 28% and hemoglobin 9 g/dL. Serum B12 level is decreased. Peripheral
blood smear is shown. Which of the following is the best next step in management?
Correct Answer ImageA.Anti-intrinsic factor antibodies level
Incorrect Answer ImageB.Intramuscular B12 replacement
Incorrect Answer ImageC.Methylmalonic acid (MMA) level
Incorrect Answer ImageD.Oral B12 replacement
Incorrect Answer ImageE.Schilling test

A 72-year-old man is brought to the physician by his son because of progressive difficulty with
memory for 5 months. The son says that his father cannot remember where he puts things and
often forgets to feed his dogs. One week ago, he walked to the nearby grocery store and could
not find his way home. He has forgotten the names of people he has known for several years. He
is able to recall the names of his family members and details about his upbringing and career. He
has a history of osteoarthritis treated with ibuprofen. His blood pressure is 130/80 mm Hg, pulse
is 82/min, and respirations are 16/min. Neurologic examination shows normal sensation, muscle
strength, and reflexes. A CT of the head shows no abnormalities. Serum thyroid stimulating
hormone levels, vitamin B12 levels, and rapid plasma reagin are all within normal limits. Which
of the following is the most appropriate pharmacotherapy?
Incorrect Answer ImageA.Diazepam
Correct Answer ImageB.Donepezil
Incorrect Answer ImageC.Memantine
Incorrect Answer ImageD.Tacrine
Incorrect Answer ImageE.Vitamin A
A 25-year-old woman comes to the physician because of scaling skin since childhood. She has a
history of atopy with her mother and brother having similar symptoms. Physical examination
shows a generalized fine scale over the body with keratin plugs in the orifices of hair follicles
over the back and extensor surfaces. The palms and soles show hyperlinearity. Which of the
following is the most likely diagnosis? 
Incorrect Answer ImageA.Epidermolytic hyperkeratosis
Correct Answer ImageB.Ichthyosis vulgaris
Incorrect Answer ImageC.Lamellar ichthyosis
Incorrect Answer ImageD.X-linked ichthyosis
Incorrect Answer ImageE.Xeroderma

A 60-year-old woman presents to the office with diarrhea and weight loss of approximately 30
lbs in the last 3 months. She describes her stools as bulky and foul-smelling. Review of systems
shows a very poor appetite in the last few months with occasional nausea. She denies abdominal
pain, blood in the stools, and dark-colored stools. Her history is significant for hypertension,
diabetes mellitus, hypercholesterolemia, coronary artery disease, and end stage renal disease.
Physical examination shows an undernourished female. Her pulse is 89/min, respirations are
14/min, and blood pressure is 110/72 mm Hg. She is afebrile. Lungs are clear to auscultation.
Cardiovascular examination is normal. Abdomen is soft, nontender and nondistended.
Extremities show no edema. She is hospitalized for further evaluation. CT of the abdomen
reveals a 2-cm mass at the head of the pancreas. The liver shows multiple areas of
hypoattenuation. Which of the following is the most appropriate next step in management?
Correct Answer ImageA.Chemotherapy
Incorrect Answer ImageB.Radiotherapy
Incorrect Answer ImageC.Supportive care
Incorrect Answer ImageD.Whipple procedure (pancreaticoduodenectomy)
Incorrect Answer ImageE.Whipple procedure with partial hepatectomy

A 50-year-old man is brought to the hospital after sustaining an acute non-ST segment elevation
myocardial infarction (NSTEMI). He is started on an aspirin, statin, and heparin continuous
infusion. Eight hours after this event, his pulse is 45/min and his blood pressure is 70/50 mm Hg.
An electrocardiogram reveals sinus rhythm. Which of the following is the most appropriate
intervention?
Correct Answer ImageA.Administer atropine IV
Incorrect Answer ImageB.Administer a beta-blocker
Incorrect Answer ImageC.Administer dobutamine
Incorrect Answer ImageD.Insert a transvenous pacemaker
Incorrect Answer ImageE.Perform cardiac catheterization

A 28-year-old man comes to the physician because of diffuse irritability, muscle cramping, and
fatigue for 4 months. He has had no weight loss, fevers, chills, rashes or joint pain. He takes no
medications. His medical history is unremarkable. He does not smoke or drink alcohol. His
temperature is 37ºC (98.6ºF) and blood pressure is 122/78 mm Hg. Oxygen saturation is 100% in
room air. Laboratory studies show:
Na+ 138 mEq/L 
Cl -
105 mEq/L 
K+ 4.7 mEq/L 
HCO3 -
24 mEq/L 
Urea nitrogen (BUN)  9 mg/dL 
Creatinine  0.8 mg/dL 
Calcium  7.7 mg/dL 
Phosphate  5.2 mg/dL 
Parathyroid hormone (PTH)  2 pg/mL (normal, 10–55 pg/mL)
Albumin  3.5 g/dL 
Which of the following is the most likely cause of this patient's condition? 
Incorrect Answer ImageA.Excess vitamin D
Correct Answer ImageB.Hypoparathyroidism
Incorrect Answer ImageC.Pseudohypoparathyroidism
Incorrect Answer ImageD.Secondary hyperparathyroidism
Incorrect Answer ImageE.Vitamin D deficiency

An 80-year-old woman is brought to the physician's office accompanied by a friend. The patient
reports severe fatigue, loss of weight and appetite, and a burning sensation in her feet. She lives
alone and has one meal a day delivered by a local food bank. The friend was visiting and noticed
the woman had some memory deficits and appeared very fatigued. Review of systems is
remarkable for difficulty sleeping and urinary incontinence. Examination shows not only
diminished memory and concentration, but an ataxic gait and diminished vibratory sensation in
her lower extremities. A complete blood count shows: 
Hemoglobin 8.8 g/dL
Leukocyte count  4,600/mm3
Mean corpuscular volume 128 µm3
Platelets 220,000/mm3
A blood smear most likely will reveal which of the following hematologic abnormalities? 
Incorrect Answer ImageA.Dark blue, coarse cytoplasmic granules in the cytoplasm of neutrophils
Incorrect Answer ImageB.Giant cytoplasmic granules within the cytoplasm of neutrophils
Correct Answer ImageC.Multilobed neutrophil nuclei with five or more segments
Incorrect Answer ImageD.Occasional neutrophils with bilobed nuclei and reduced granulation
Incorrect Answer ImageE.Predominance of neutrophils with a band-shaped nucleus

A 31-year-old woman comes to the physician because of progressive blurring of her vision for 5
days. She has pain when moving her right eye. She has never experienced these symptoms
before, and she does not wear glasses or contact lenses. She takes no medications. Her
temperature is 37°C (98.6°F), blood pressure is 132/84 mm Hg, and pulse is 75/min. Eye
examination shows that visual acuity is 20/80 in the right eye and 20/20 in the left eye. Her color
perception is diminished. There is an afferent pupillary defect in the right eye. Muscle strength is
5/5 and sensation is normal. Which of the following is the most likely fundoscopic finding in this
patient?
Correct Answer ImageA.A normal fundoscopic examination
Incorrect Answer ImageB.Cotton wool spots
Incorrect Answer ImageC.Optic disc swelling
Incorrect Answer ImageD.Peripapillary hemorrhages
Incorrect Answer ImageE.Retinal exudates

A 61-year-old man is receiving experimental chemotherapeutic infusions through a peripherally


inserted central catheter (PICC) line for the treatment of a low-grade lymphoma. Three weeks
into his treatment, he comes to urgent care because of worsening shortness of breath as well as
lower extremity swelling. Review of systems is positive for increasing fatigue, dyspnea on
exertion, nocturia, and 4-pillow orthopnea. Additionally, he reports a week of fevers and chills,
which he has attributed to his chemotherapy. His temperature is 38.9ºC (102.0ºF). Cardiac
examination shows an S3, and there is an 11 cm jugular venous column and peripheral edema.
The patient is admitted for a workup of suspected new-onset congestive heart failure.
Echocardiogram shows a vegetation on the aortic valve with severe regurgitation and poor left
ventricular function with an ejection fraction of 30-35%. The PICC line is removed and the
patient is admitted to a monitored bed. Empiric antibiotic treatment is started for endocarditis
treatment. Which of the following is the most appropriate intervention?
Incorrect Answer ImageA.Addition of empiric IV nafcillin
Incorrect Answer ImageB.Anticoagulation to reduce the risk of septic emboli
Incorrect Answer ImageC.Intra-aortic balloon pump placement
Incorrect Answer ImageD.No additional intervention but close cardiac monitoring
Correct Answer ImageE.Surgical consult for aortic valve replacement

A 72-year-old man was brought to the emergency department by his wife because of the sudden
onset of severe mid-abdominal pain which started 5 hours ago. He has a history of rheumatic
fever 55 years ago, acute cholecystitis, and a gastric ulcer. He underwent open cholecystectomy
22 years ago. Current medications include ranitidine and a multivitamin. He appears in moderate
distress and complains of increasing pain with any movement. His temperature is 38.7°C
(101.6°F), pulse is 104/min, and blood pressure is 160/90 mm Hg. His sclerae are anicteric and
his mucous membranes are dry. Abdominal examination reveals decreased bowel sounds, diffuse
tenderness, and involuntary guarding to mild palpation. Rectal examination shows brown stool
that is negative for occult blood. A radiograph of the abdomen shows free air underneath the left
hemidiaphragm. Which of the following is the most appropriate next step in management?
Incorrect Answer ImageA.Abdominal CT scan
Correct Answer ImageB.Emergent laparotomy
Incorrect Answer ImageC.High-dose oral omeprazole
Incorrect Answer ImageD.Histamine-2 receptor antagonist
Incorrect Answer ImageE.Observation after placement of a nasogastric tube
 54-year-old man comes to the physician because of a paroxysmal cough productive of frank
blood for 2 months. The coughing episodes occur 3 times per week. He has had mild shortness of
breath. He has had progressive eye pain for 1 week. He has hypertension controlled with
hydrochlorothiazide. His temperature is 36.8°C (98.2°F) and blood pressure is 126/85 mm Hg.
Examination shows tenderness with percussion over the maxillary sinuses, dark tissue, and loss
of normal nasal mucosa. Cardiovascular examination shows no abnormalities. Bilateral coarse
breath sounds are heard on auscultation. Ophthalmic examination with a slit-lamp shows cells in
his anterior chamber. A radiograph of the chest shows multiple cavitary nodules bilaterally
without hilar adenopathy. Which of the following diagnostic tests is most likely to be abnormal? 
Incorrect Answer ImageA.Direct laryngoscopy/pharyngoscopy
Incorrect Answer ImageB.Echocardiogram
Incorrect Answer ImageC.Purified protein derivative skin test for M. tuberculosis
Incorrect Answer ImageD.Serum angiotensin-converting enzyme level
Correct Answer ImageE.Urinalysis

A previously healthy 30-year-old woman comes to the emergency department because of


nosebleeds and heavy bleeding during menstrual periods. At present, she has a nosebleed that she
cannot stop. After questioning, she states that she has taken no medications in the past several
months and other than the bleeding, she has had no other signs of illness. After the posterior
nares are packed to stop the bleeding, she is examined. Scattered petechiae are found; there is no
hepatosplenomegaly or lymphadenopathy. There are no other positive physical findings. Her
platelet count is 20,000/µL, hematocrit is 31%, and megathrombocytes are seen on the peripheral
smear. PT, PTT, and bleeding time are normal. Which of the following is the most appropriate
next step in management? 
Incorrect Answer ImageA.Admit for intravenous immunoglobulin infusion
Incorrect Answer ImageB.Admit for platelet transfusion
Incorrect Answer ImageC.Perform a bone marrow aspiration
Incorrect Answer ImageD.Perform a splenectomy
Correct Answer ImageE.Start on oral prednisone

A previously healthy 27-year-old man comes to the physician because of a cough with whitish
sputum production for the past 3 days. The cough has been keeping him up at night and it is
affecting his job performance. He has no prior history of respiratory disease. His temperature is
37.0ºC (98.6ºF), pulse is 70/min, respirations are 18/min, and blood pressure is 130/80 mm Hg.
Physical examination is unremarkable. Which of the following is the most appropriate next step
in management?
Incorrect Answer ImageA.Admit to the hospital for medical management
Incorrect Answer ImageB.Perform a chest radiograph
Incorrect Answer ImageC.Perform a sputum Gram stain and culture
Correct Answer ImageD.Reassure and follow up if cough persists more than 10 days
Incorrect Answer ImageE.Send home with antibiotic therapy
Incorrect Answer ImageF.Send home with cough suppressant
Incorrect Answer ImageG.Send home with mucolytic agent
A 52-year-old man comes to the emergency department because of progressive cough that has
been productive of frank blood for 2 weeks. He has had a 2-month history of fatigue, fever,
shortness of breath, and a 5.4-kg (12-lb) weight loss. He also has a history of chronic sinus
infection. He does not smoke or use illicit drugs. He lives in a house with his wife and 2 children
and he works as a secretary. He denies recent travel. His blood pressure is 115/70 mm Hg.
Bilateral basilar rales are heard on auscultation of the lung. Laboratory studies show:
Hematocrit  33%
Hemoglobin  12 g/dL
WBC  12,000/mm3
Platelets  155,000/mm3
Erythrocyte sedimentation rate  110 mm/h
Creatinine  2.5 mg/dL
Urinalysis shows microscopic hematuria and red blood cell casts. Three sputum samples are
negative for acid-fast bacilli. A lung biopsy specimen shows necrotizing granulomas. Which of
the following is the most likely diagnosis? 
Incorrect Answer ImageA.Goodpasture syndrome
Correct Answer ImageB.Granulomatosis with polyangiitis
Incorrect Answer ImageC.Polyarteritis nodosa
Incorrect Answer ImageD.Sarcoidosis
Incorrect Answer ImageE.Tuberculosis

A 24-year-old woman who underwent successful bone marrow transplantation for leukemia
approximately 6 months earlier comes to the emergency department because of lower chest pain
and hematemesis occurring after a bout of recurrent, non-bloody vomiting. Her history is
significant for mucositis occuring during her bone marrow transplantation. Other than leukemia,
she has no other significant past medical history. She has otherwise been doing well since her
transplantation. She is currently not taking any medications. Vital signs are within normal limits.
Head, neck, cardiac, and chest examinations are normal. There is no significant abdominal
tenderness, masses, or hepatosplenomegaly. Her pulses are normal and there is no lower
extremity edema. Laboratory data are obtained and shown below.
White blood cells 7800/mm3
Hct 39%
Platelets 256,000/mm3
An upper gastrointestinal double contrast barium examination is performed and shows no
abnormalities of the esophagus or stomach. Which of the following is the most likely etiology for
her symptoms? 
Incorrect Answer ImageA.Cytomegalovirus esophagitis
Incorrect Answer ImageB.Esophageal adenocarcinoma
Incorrect Answer ImageC.Gastritis
Incorrect Answer ImageD.Herpes simplex-induced esophageal ulceration
Correct Answer ImageE.Mallory-Weiss tear
A 38-year-old man comes to the clinic because of pain in his right eye. The patient also reports
blurred vision, photophobia, and tearing for the last 2 days. The patient does not have a history of
medical problems and does not take any medications. Ophthalmologic examination of the lids,
lashes, and lacrimal ducts shows no abnormalities. The conjunctival examination demonstrates
360-degree perilimbal injection, which increases in intensity as it approaches the limbus. Visual
acuity is 20/200 in the right eye and 20/20 in the left eye. Extraocular movements are normal. On
the pupillary examination, the patient has photophobia when the light is directed into the right
eye. Slit-lamp exam shows inflammatory cells floating in the aqueous humor. Intraocular
pressure is normal bilaterally. Which of the following is the most appropriate next step in
management?
Incorrect Answer ImageA.Antibiotic drops
Correct Answer ImageB.Immediate ophthalmology referral
Incorrect Answer ImageC.No specific treatment is necessary
Incorrect Answer ImageD.Routine ophthalmology referral
Incorrect Answer ImageE.Topical corticosteroids

A 45-year-old man comes to the emergency department because of shaking, palpitations,


sweating, and anxiety for 15 minutes. He ate lunch 5 hours ago. He has had no chest pain or
shortness of breath. His medical history is unremarkable and he takes no medications. He works
in a hospital as a nurse. His temperature is 36.5°C (97.7°F), blood pressure is 105/65 mm Hg,
pulse is 104/min, and respirations are 14/min. His blood glucose is 35 mg/dL, and both serum
insulin and C-peptide levels are elevated. Which of the following is the most likely cause of this
patient's symptoms? 
Incorrect Answer ImageA.Autoimmune disease involving the pancreas
Incorrect Answer ImageB.Factitious insulin administration
Incorrect Answer ImageC.Increased insulin resistance
Correct Answer ImageD.Neuroendocrine tumor
Incorrect Answer ImageE.Tumor of alpha cells in islet of pancreas

A 58-year-old man who has a history of nephrolithiasis comes to his physician for recurrent
kidney stones. He first had a kidney stone 10 years ago, presenting with left-sided flank pain,
hematuria, and vomiting. An IVP done at that time revealed a 1 cm stone in the left ureter. He
underwent surgical lithotomy. Since that time he has had multiple stones on both sides. On two
occasions, he had extracorporeal shock wave lithotripsy (ESWL). Several times he has passed
small stones. Approximately 1 year ago, he had a stone analysis done that revealed the stone
composition to be calcium oxalate. He currently has no symptoms. The patient has well
controlled hypertension for which he takes amlodipine and hydrochlorothiazide. He takes
ibuprofen as needed for renal colic. His father also had a history of recurrent nephrolithiasis.
Vital signs and physical examination are normal. 
Serum studies reveal:
Sodium 136 mEq/L
Potassium 4.5 mEq/L
Chloride 100 mEq/L
Magnesium 2.0 mEq/L
Calcium 9.0 mg/dL
Bicarbonate 26 mEq/L
Uric acid 7.6 mg/dL
BUN 16 mg/dL
Creatinine 1.0 mg/dL
Urinalysis shows a urine pH of 5.5. There are no crystals or casts seen. Which of the following is
the best next step in management? 
Correct Answer ImageA.24-hour urine for calcium, citrate, oxalate, phosphate, uric acid, and
creatinine
Incorrect Answer ImageB.Creatinine clearance
Incorrect Answer ImageC.Discontinue amlodipine
Incorrect Answer ImageD.Discontinue hydrochlorothiazide
Incorrect Answer ImageE.Serum intact PTH level

A 42-year-old healthy woman comes to the physician for a routine physical examination. She has
a history of heartburn and is currently taking ranitidine. Her blood pressure is 136/79 mm Hg and
her pulse is 78/min. Physical examination shows no abnormalities. Two weeks later, she returns
for a repeat blood pressure check, which shows a blood pressure of 135/77 mm Hg. Which of the
following is the most appropriate next step in management?
Correct Answer ImageA.Encourage dietary modification and exercise
Incorrect Answer ImageB.Initiate both hydrochlorothiazide and lisinopril
Incorrect Answer ImageC.Initiate hydrochlorothiazide
Incorrect Answer ImageD.Initiate lisinopril
Incorrect Answer ImageE.No further management is required

A 52-year-old woman with a history of hypertension, type 1 diabetes mellitus and diabetic
nephropathy comes for a follow-up visit. She has a 3-month history of a worsening rash on her
right leg and denies fever or trauma to the area. Current medications include lisinopril,
hydrochlorothiazide, and insulin. Her temperature is 36.5°C (97.7°F), and pulse is 74/min, and
blood pressure is 148/90 mm Hg. Skin examination shows a 6 cm atrophic patch with
erythematous borders and a yellow-colored center. Hemoglobin A1c is 11.2%. Which of the
following is the most likely diagnosis?
Incorrect Answer ImageA.Acanthosis nigricans
Incorrect Answer ImageB.Erythema nodosum
Correct Answer ImageC.Necrobiosis lipoidica
Incorrect Answer ImageD.Pyoderma gangrenosum
Incorrect Answer ImageE.Scleroderma

A 65-year-old woman presents to her physician for a follow up of chronic facial pain. She
reports a one year history of intermittent facial pain that lasts for about a minute. The pain
worsens on yawning and brushing her teeth. She also reports occasional facial spasms. She
denies aura, visual changes, and nausea. She has been on carbamazepine, baclofen and other
antiepileptic medications over this year, none of which have provided relief. Which of the
following is the most appropriate next step in the management?
Incorrect Answer ImageA.Electroencephalogram
Incorrect Answer ImageB.“Pulsed” prednisone with prolonged taper
Incorrect Answer ImageC.Referral to dentistry for imaging and evaluation
Correct Answer ImageD.Surgery for decompression
Incorrect Answer ImageE.Trial of sumatriptan, ergotamine, and caffeine

A 22-year-old woman comes to the physician because of a 1-year history of pain and tingling in
her hands. When she goes outside during the winter her fingers turn white and become painful.
She has had malaise and has felt "achy" in her muscles and joints throughout her body. For the
past 1 month, she has had a sharp pain in her chest when she takes a deep breath. Serum studies
are positive for anti-U1-ribonucleoprotein autoantibodies, rheumatoid factor, and antinuclear
antibody. A radiograph of the chest is shown. Which of the following is the most likely
diagnosis?
Incorrect Answer ImageA.CREST syndrome
Incorrect Answer ImageB.Drug-induced lupus
Correct Answer ImageC.Mixed connective-tissue disease
Incorrect Answer ImageD.Rheumatoid arthritis
Incorrect Answer ImageE.Systemic sclerosis

A 44-year-old woman comes to the physician for a routine health maintenance examination. She
has had fatigue and an increase in the frequency of urination. She weighs 100 kg (220 lb) and is
170 cm (66 in) tall; her body mass index is 34.6 kg/m2. Her temperature is 36.7°C (98°F), pulse
is 86/min, and blood pressure is 118/80 mm Hg. On physical examination, the shown cutaneous
features are found. The remainder of the examination is unremarkable. Which of the following
findings is most likely in this patient?
Incorrect Answer ImageA.Coral-red fluorescence
Correct Answer ImageB.Hemoglobin A1c of 7.9
Incorrect Answer ImageC.Low-density lipoprotein cholesterol level of 230
Incorrect Answer ImageD.Positive carcinoembryonic antigen
Incorrect Answer ImageE.Presence of pseudohyphae on skin scrapings

A 75-year-old man is brought to the emergency department by his daughter because of a


progressive fever, confusion, and insomnia for 3 days. Recently he has become combative and
has reported unusual smells. He has a history of hypertension treated with hydrochlorothiazide
and lisinopril. Prior to 3 days ago, he frequently participated in his local bridge club and had no
difficulty with memory. On examination he is disoriented and mildly combative. His temperature
is 38.5ºC (101.4ºF), blood pressure is 118/73 mm Hg, pulse 98/min, and respirations 17/min.
Oxygen saturation is 96% on room air. Head and neck examination shows mild nuchal rigidity.
Neurologic examination shows difficulty following commands. A CT scan of the brain shows no
abnormalities. Lumbar puncture is performed and cerebrospinal fluid analysis is shown:
WBC 100/mm3, 68% lymphocytes 
RBC  200/mm3
Glucose  50 mg/dL
Protein  38 mg/dL
Which of the following is the most appropriate next step in management? 
Incorrect Answer ImageA.Electroencephalography
Correct Answer ImageB.IV acyclovir
Incorrect Answer ImageC.IV ceftriaxone, vancomycin, and ampicillin
Incorrect Answer ImageD.Magnetic resonance imaging
Incorrect Answer ImageE.Repeat collection of CSF in 3 tubes for analysis

A 67-year-old man comes to the emergency department because of the sudden onset of shortness
of breath and a sharp pain on his left side with inspiration that started while reclining on the
couch 5 hours ago. There is no radiation of the pain. He has had no nausea, vomiting, or
heartburn symptoms. He has had no fevers or sputum production. He has a history of
hypertension treated with lisinopril and hydrochlorothiazide and a history of pancreatic cancer.
He does not smoke cigarettes or drink alcohol. His father had a myocardial infarction at age 52
years. His pulse is 110/min, respirations are 22/min, and his oxygen saturation is 87% on room
air. Crackles are heard in the left lung . An ECG shows sinus tachycardia. A CT scan of the chest
shows a pulmonary embolism. The patient is started on intravenous heparin. Two days later,
examination is shown (see media clip). Which of the following is the most appropriate next step
in management? 
Incorrect Answer ImageA.Anticoagulation with aspirin only
Correct Answer ImageB.CT scan of the brain
Incorrect Answer ImageC.Doppler ultrasonography of the lower extremities
Incorrect Answer ImageD.IVC filter placement
Incorrect Answer ImageE.Switch anticoagulation to a direct thrombin inhibitor

A 41-year-old man comes to the clinic because of a chronic cough over the past 4 months, which
has now been accompanied by hemoptysis. He denies smoking or any past medical history. On
physical examination, his head and neck examination are normal. His lungs have diffuse bilateral
rales. Cardiac examination is normal. Laboratory findings show the following:
Blood chemistry:
Na+ 142 mEq/L
K+ 4.3 mEq/L
Cl −
110 mEq/L
HCO3− 24 mEq/L
BUN  39 mg/dL
Creatinine 2.9 mg/dL
Urinalysis:
Microscopic hematuria
4+ proteinuria
Which of the following serologic blood tests would be most helpful in confirming the suspected
diagnosis? 
Correct Answer ImageA.Anti-glomerular basement membrane antibodies
Incorrect Answer ImageB.Anti-mitochondrial antibodies
Incorrect Answer ImageC.Anti-neutrophilic antibodies
Incorrect Answer ImageD.Anti-parietal cell antibodies
Incorrect Answer ImageE.Anti-smooth muscle antibodies

A 71-year-old woman comes to the emergency department because of difficulty breathing. She is
found to have a supraglottic mass. After emergent tracheostomy and surgical resection, the mass
is identified as a squamous cell carcinoma. After an extensive workup including panendoscopy
and computed tomography (CT) scan, it is staged as T3N0M0. Adjuvant chemotherapy is
planned and the patient is transferred to the medical oncology service for treatment with cisplatin
and 5-fluorouracil. Shortly after chemotherapy is started, the patient is experiencing severe
nausea and vomiting which does not respond to promethazine and is most likely related to
chemotherapy. Which of the following is the most appropriate treatment at this time?
Incorrect Answer ImageA.Diphenhydramine
Incorrect Answer ImageB.Dronabinol
Incorrect Answer ImageC.Lorazepam
Incorrect Answer ImageD.Metoclopramide
Correct Answer ImageE.Palonosetron, dexamethasone, and fosaprepitant

A 70-year-old woman is brought to the emergency department because of a 2-day history of


productive cough with yellow-green sputum, shortness of breath, and fever. She has a history of
hypertension, coronary artery disease, osteoarthritis, and gastroesophageal reflux disease. She
has no recent hospitalizations, lives with her husband, and performs most activities of daily
living by herself. Current medications include nitroglycerin, aspirin, atenolol, and omeprazole.
She is in mild respiratory distress. Her temperature is 39.8ºC (103.6ºF), pulse is 120/min,
respirations are 32/min, blood pressure is 128/66 mm Hg, and oxygen saturation is 94% on
oxygen 2 L/min via nasal cannula. Decreased breath sounds and crackles are heard on
auscultation bilaterally. Leukocyte count is 14,100/mm3. Radiograph of the chest is shown.
Which of the following is the most appropriate pharmacotherapy?
Incorrect Answer ImageA.Aspirin
Correct Answer ImageB.Azithromycin and ceftriaxone
Incorrect Answer ImageC.Cefepime and ciprofloxacin
Incorrect Answer ImageD.Cefepime and vancomycin
Incorrect Answer ImageE.Furosemide

A 32-year-old woman who is a school teacher comes to the physician because of skin edema,
tightness, and induration. She also says she has episodes of skin discoloration in her hands that is
usually triggered by exposure to cold or emotional stress. These symptoms appeared several
weeks ago and have become progressively worse. She also experienced generalized weakness,
dyspnea on exertion, mild arthralgias, morning muscle stiffness, and occasional difficulty with
swallowing. There is no history of any major medical illnesses and she takes no medications. Her
temperature is 37.0ºC (98.6ºF), blood pressure is 150/88 mm Hg, pulse is 86/min, and
respirations are 20/min. Physical examination shows decreased breath sounds bilaterally and a
loud S2. The skin is indurated with telangiectasias on the face.
Laboratory studies show: 
WBC 12,500/mm3
Hemoglobin 10.7 g/dL
Hematocrit 31%
Platelets 265,000/mm3
Erythrocyte sedimentation rate 56 mm/h
Comprehensive metabolic panel reveals:
Na+ 137 mmol/L
K +
4.8 mmol/L
Cl− 98 mmol/L
BUN 39 mg/dL
Creatinine 1.6 mg/dL
Glucose 98 mg/dL
Calcium 8.9 mg/dL
Urinalysis shows 3 RBC/hpf, mild proteinuria, and red blood cell casts. Further studies show the
presence of antinuclear and anti topoisomerase-I antibodies. Which of the following drugs is
most appropriate for this patient's hypertension? 
Incorrect Answer ImageA.Atenolol
Incorrect Answer ImageB.Bosentan
Correct Answer ImageC.Captopril
Incorrect Answer ImageD.Hydrochlorothiazide
Incorrect Answer ImageE.Nifedipine

An 83-year-old woman consults a physician because she is "feeling so tired all the time." In-
office hematocrit is 35%. Peripheral blood smear shows many macrocytic red blood cells. On
questioning, the woman, whose finances are limited, admits that she has been living on a strictly
limited diet consisting mainly of bread. She has been drinking a powdered orange juice
substitute. She has not been taking vitamin pills because she feels she cannot afford them. A
nutritional deficiency of which of the following is the most likely cause of this patient's anemia? 
Correct Answer ImageA.Folate
Incorrect Answer ImageB.Iron
Incorrect Answer ImageC.Vitamin B12
Incorrect Answer ImageD.Vitamin C
Incorrect Answer ImageE.Vitamin K

A 34-year-old man comes to the physician because of recurrent episodes of dizziness and ringing
in his right ear for 3 months. The episodes of dizziness occur several times per week, usually
lasting between 30 minutes and 4 hours. He denies headaches, fever, weight loss, weakness,
numbness, or tingling. He has not had any loss of consciousness or seizures. His medical history
is unremarkable and he takes no medications. His temperature is 37.1ºC (98.8ºF), blood pressure
is 132/82 mm Hg, pulse is 78/min, and respirations are 12/min. Examination shows mild hearing
loss of the right ear. Which of the following is the most likely diagnosis? 
Incorrect Answer ImageA.Benign paroxysmal positional vertigo
Incorrect Answer ImageB.Labyrinthitis
Correct Answer ImageC.Ménière disease
Incorrect Answer ImageD.Migraine-associated dizziness
Incorrect Answer ImageE.Vestibular schwannoma

A 34-year-old woman comes to the emergency department with acute right upper quadrant
abdominal pain. The pain came on suddenly while she was at home watching a movie with her
husband. The patient reports that she never experienced this type of pain before. She also reports
feeling lightheaded and denies nausea or vomiting. Her past medical history is unremarkable.
She takes oral contraceptives. On physical examination, her temperature is 36.5°C (97.7°F),
blood pressure is 88/50 mm Hg, and pulse is 110/min. Head, neck, chest, and cardiovascular
examinations are normal. Palpation of her abdomen shows exquisite tenderness and guarding in
the right upper quadrant. Fluid resuscitation is initiated and a computed tomography of the
abdomen shows a heterogeneous subcapsular liver lesion with marked hemoperitoneum. Which
of the following statements is the most accurate regarding this patient's current diagnosis?
Incorrect Answer ImageA.After stabilizing the patient, a biopsy should be obtained
Incorrect Answer ImageB.Malignant degeneration occurs in 0.5% of these lesions
Correct Answer ImageC.The majority of these lesions are cold on a sulfur colloid scan
Incorrect Answer ImageD.There is no relationship to oral contraceptive use
Incorrect Answer ImageE.Ultrasound is highly specific for this lesion

A 71-year-old woman is brought to the emergency department with fatigue, diaphoresis, and
shortness of breath. The patient has a long history of coronary artery disease (CAD),
hyperlipidemia, hypertension, and diabetes mellitus. The patient is currently taking aspirin,
atenolol, captopril, simvastatin, and glyburide. She has been a heavy smoker for 40 years. Her
temperature is 36.5ºC (97.7ºF), pulse is 96/min, respirations are 25/min, blood pressure is 100/65
mm Hg, and SaO2 is 91% on room air. She is in distress. Examination shows distended neck
veins, bibasilar crackles, and a regular pulse and rhythm with an S3 gallop. Examination of the
extremities shows 2+ lower extremity edema. Brain natriuretic peptide (BNP) is elevated. Chest
radiograph shows a large heart with increased vascular markings. She is seated upright and given
oxygen, morphine, nitrates, and furosemide, but she continues to be short of breath. Which of the
following is the most appropriate next step in the management of this patient?
Incorrect Answer ImageA.Albuterol
Incorrect Answer ImageB.Digoxin
Correct Answer ImageC.Dobutamine
Incorrect Answer ImageD.Endotracheal intubation
Incorrect Answer ImageE.Spironolactone

A 27-year-old woman comes to the physician because of a 3-month history of amenorrhea,


decreased libido, and fatigue. She has had no visual changes or headache. She is sexually active
and consistently uses condoms. Her temperature is 37ºC (98.6ºF), blood pressure is 100/70 mm
Hg, pulse is 73/min, and respirations are 13/min. She is 165 cm (65 in) tall and weighs 56 kg
(123 lb); her body mass index is 20.5. Examination shows a milky white discharge from both
breasts. The remainder of the physical examination shows no abnormalities. Laboratory studies
show: 
Hemoglobin 13.9 g/dL
Thyroid-stimulating hormone 2.3 mU/L
Thyroxine 8 mcg/dL (n, 4.5–11.2 mcg/dL)
Urea nitrogen (BUN) 12 mg/dL
Creatinine 0.7 mg/dL
Urine hCG negative
MRI of the head shows an enlarged pituitary gland. Which of the following is the most
appropriate next step in management? 
Correct Answer ImageA.Cabergoline
Incorrect Answer ImageB.Octreotide
Incorrect Answer ImageC.Oral contraceptive pill
Incorrect Answer ImageD.Radiation therapy
Incorrect Answer ImageE.Transsphenoidal surgery

A 48-year-old woman comes to the clinic today because of claudication for the last 6 weeks. Her
past medical history is significant for atrial fibrillation and benign positional vertigo diagnosed 9
years ago. Her medications are atenolol, warfarin, and meclizine as needed. Physical
examination shows normal heart and lung examination with bilateral 1+ ankle edema and
diminished pulses. An ankle- brachial index (ABI) of 0.8 is calculated, and the decision is made
to suspend atenolol. Instead, the patient is started on amiodarone. Which of the following is the
most appropriate next step in the management of this patient? 
Incorrect Answer ImageA.Angiography
Correct Answer ImageB.Decrease the warfarin dose
Incorrect Answer ImageC.No additional changes are needed
Incorrect Answer ImageD.Obtain renal function test
Incorrect Answer ImageE.Refer for surgical consult

A 38-year-old woman comes to the physician because of a variety of abdominal issues. Over the
past 2 years, she has had near-daily lower abdominal pain and diarrhea, though occasionally she
suffers from periods of prolonged constipation. The pain is described as crampy in nature, is
present mainly in both lower abdominal quadrants, and is relieved with defecation. She reports
feeling bloated with abdominal distension. She denies any weight loss, fevers, nausea, vomiting,
or fecal incontinence, but does note that her stool occasionally shows large amounts of mucus. In
the past, she has tried a lactose-free diet but has not had any improvement in symptoms. Previous
evaluation has included normal stool cultures, hemoglobin, and normal flexible sigmoidoscopy.
Today, vital signs and physical examination, including rectal examination, are normal. Which of
the following is the most appropriate next step in management?
Incorrect Answer ImageA.Helicobacter pylori testing with treatment if positive
Incorrect Answer ImageB.Psychiatric consultation and likely psychotherapy
Correct Answer ImageC.Reassurance, supplemental fiber, stress reduction
Incorrect Answer ImageD.Serotonin antagonists
Incorrect Answer ImageE.Urgent colonoscopy to evaluate for right-sided lesions

A 62-year-old man with a history of long-standing hypertension, diabetes mellitus and


hypercholesterolemia presents to the emergency department with retrosternal chest pain for the
last few hours. He also reports shortness of breath that started about the same time as his chest
pain. His initial ECG and cardiac enzymes are within normal limits. A diagnosis of unstable
angina is made and he is given oxygen (by nasal cannula), morphine, aspirin and beta-blockers.
He is started on a heparin drip and admitted to the telemetry unit and his symptoms subside. On
his fourth hospital day, he develops pain in his right lower extremity. Physical examination
reveals absent dorsalis pedis and posterior tibial artery pulses on the right foot. Physical
examination shows absent dorsalis pedis and posterior tibial artery pulses on the right side and a
cold foot. A stat lower-extremity Doppler ultrasound examination confirms the diagnosis of
acute thrombosis of the popliteal artery. Laboratory evaluation shows: 
Hemoglobin 14 g/dL
Hematocrit  42%
WBC  5,300/mm3
Platelets  60,000/mm3
PT  18 sec
PTT  50 sec
Which of the following is the most likely diagnosis?
Incorrect Answer ImageA.Aspirin-induced thrombosis
Incorrect Answer ImageB.Factor V Leiden
Incorrect Answer ImageC.Idiopathic thrombocytopenic purpura
Incorrect Answer ImageD.Protein C deficiency
Incorrect Answer ImageE.Protein S deficiency
Incorrect Answer ImageF.Type I heparin-induced thrombocytopenia
Correct Answer ImageG.Type II heparin-induced thrombocytopenia

 53-year-old man with a history of chronic obstructive pulmonary disease comes to the physician
because of a productive cough for 3 weeks. He has had no fevers, chills, night sweats, or weight
loss. Current medications include albuterol and ipratropium bromide. He has smoked 2 packs of
cigarettes daily for 35 years. He works as an accountant. His temperature is 36.7ºC (98ºF), pulse
is 82/min, blood pressure is 128/78 mm Hg, and oxygen saturation is 93% on room air.
Decreased breath sounds in the right upper lobe and diffuse wheezing bilaterally are heard on
auscultation. The remainder of the examination shows no abnormalities. Laboratory studies are
within normal limits. A radiograph of the chest shows a nodule in the right upper lobe. A CT
scan of the chest is shown. Which of the following is the most appropriate next step in
management?
Correct Answer ImageA.Bronchoscopy
Incorrect Answer ImageB.CT-guided lung biopsy
Incorrect Answer ImageC.Pharmacologic/antibiotic therapy
Incorrect Answer ImageD.Pulmonary function testing
Incorrect Answer ImageE.Serial CT images every 3 months over the next 12 months
A 62-year-old woman with a 1-year history of dermatomyositis comes for a routine health
maintenance examination. She is concerned about the association of ovarian cancer with
dermatomyositis, and asks to be screened for ovarian cancer. She denies fever, fatigue,
weakness, weight loss, abdominal pain, or bloating. Current medications include methotrexate.
She does not smoke. Which of the following is the most sensitive screening modality for ovarian
cancer in this patient? 
Incorrect Answer ImageA.Bimanual examination followed by CA-125 measurement
Incorrect Answer ImageB.Bimanual examination followed by transvaginal ultrasound
Incorrect Answer ImageC.CA-125 measurement and BRCA mutation assessment
Incorrect Answer ImageD.CA-125 measurement and epithelial membrane antigen (EMA) detection
Correct Answer ImageE.CA-125 measurement followed by transvaginal ultrasound

A 43-year-old man with a history of type 1 diabetes mellitus comes to the emergency department
because of episodes of palpitations, agitation, and diffuse sweating for 4 months. He has had no
headaches. Current medications include lisinopril, insulin glargine, and insulin lispro. He is
anxious-appearing. His blood pressure is 155/95 mm Hg and pulse is 105/min and irregularly
irregular. Examination shows a wide-eyed stare on downward gaze. A grade 2/6 early systolic
murmur is heard over the left sternal border. Deep tendon reflexes are brisk. Laboratory studies
show a fasting glucose level of 72 mg/dL, and his TSH level is undetectable. Which of the
following is the most appropriate next step in management?
Incorrect Answer ImageA.Alpha-adrenergic blockade
Correct Answer ImageB.Beta-adrenergic blockade
Incorrect Answer ImageC.Decrease dose of insulin glargine
Incorrect Answer ImageD.Hydralazine
Incorrect Answer ImageE.Radioiodine thyroid ablation

A 46-year-old man with a history of hypertension and hypercholesterolemia comes to the


physician for a routine follow-up. The patient's job involves a lot of traveling, and he admits to
occasionally forgetting to take his medications with him when he travels. He reports several
episodes of chest pain in the past few months. The pain is sharp in nature, mainly over his lower
chest and epigastrium, and tends to come on with walking. He believes these episodes are due to
indigestion and has been taking antacids. There is a family history of heart disease, and his father
died of a heart attack at age 48. On physical examination, his pulse is 86/min and his blood
pressure is 150/80 mm Hg. His lungs are clear to auscultation. Cardiac auscultation shows
normal rate and rhythm, without murmurs, rubs, or gallops. There is no pedal edema. He is sent
for an exercise stress test. Five minutes into the test he develops ST depression of 3 mm in leads
V1-V5. The ST-segment depression is >0.12 seconds in duration, and the stress test is stopped.
Which of the following is the most appropriate next step in management?
Correct Answer ImageA.Coronary angiography
Incorrect Answer ImageB.Dobutamine stress test
Incorrect Answer ImageC.Echocardiogram
Incorrect Answer ImageD.Holter monitor
Incorrect Answer ImageE.Repeat stress test in 2 weeks
 74-year-old man with a history of hepatitis C infection comes to the emergency department after
vomiting large amounts of bright red blood followed by rectal bleeding 40 minutes later. He has
a history of myocardial infarction and underwent coronary artery bypass graft surgery 16 years
ago. During his surgery, he received multiple blood transfusions. His temperature is 37.2ºC
(98.9ºF), blood pressure is 96/56 mm Hg, pulse is 112/min, and respirations are 26/min. Mucous
membranes are dry. Cardiopulmonary examination shows no abnormalities. There is no
hepatosplenomegaly, ascites, or peripheral edema. Neurologic examination is normal. Which of
the following is the most appropriate next step in management? 
Incorrect Answer ImageA.Colonoscopy
Incorrect Answer ImageB.Electrocardiogram
Incorrect Answer ImageC.Evaluation for emergency liver transplant
Correct Answer ImageD.Placement of 2 large bore IV catheters
Incorrect Answer ImageE.Upper gastrointestinal endoscopy

A 44-year-old woman comes to the physician because of a mole on her right buttock that bleeds
after minor trauma. She has had 2 nevi excised 4 and 9 years ago, respectively, that were not
malignant. Her mother has type 2 diabetes mellitus and her father has hypertension and
hyperlipidemia. Physical examination is shown. Excisional biopsy is performed. There is a small
region of ulceration in the center of the lesion. Histological immunoassay is positive for S-100.
Which of the following is the single most important prognostic factor when determining tumor
staging?
Incorrect Answer ImageA.Degree of cellular atypia
Correct Answer ImageB.Depth of melanocyte invasion
Incorrect Answer ImageC.Presence or absence of ulceration
Incorrect Answer ImageD.Quality of inflammatory infiltrate
Incorrect Answer ImageE.Sentinel lymph node involvement

A 72-year-old woman comes to the emergency department because of a 3-day history of severe
shortness of breath, fevers, chills, and left-sided chest pain that is worse when she inspires. She is
in mild respiratory distress. Her temperature is 39.0ºC (102.2ºF), pulse is 133/min, respirations
are 24/min, and blood pressure is 100/60 mm Hg. Her oxygen saturation is 90% in room air.
Breath sounds are decreased at the left base and there is increased dullness to percussion on the
left. Cardiac examination shows a normal S1 and S2; no murmurs are heard. There is no
peripheral edema. A radiograph of the chest is shown. Thoracentesis yields 250 mL of yellowish,
red-tinged mucopurulent fluid. Which of the following is the most appropriate next step in
management?
Incorrect Answer ImageA.Arrange for thoracoscopic drainage
Incorrect Answer ImageB.Chest tube placement and start piperacillin-tazobactam
Correct Answer ImageC.CT scan of the chest
Incorrect Answer ImageD.Start ampicillin and gentamicin
Incorrect Answer ImageE.Start imipenem
A 71-year-old man comes to the physician because of increasing calf pain when walking uphill.
The symptoms have gradually increased over the past 2 months. He has a history of a transient
ischemic attack 6 months ago. He is currently taking diltiazem, hydrochlorothiazide, metoprolol,
isosorbide dinitrate, and a low-dose aspirin. His pulse is 66/min and his blood pressure is 152/90
mm Hg. Physical examination shows a right carotid bruit. The lower extremities are mildly cool
and have diminished pulses at the dorsalis pedis. An arterial ultrasound shows mild narrowing of
the popliteal arteries of the lower extremities. His current medications are continued and an ACE
inhibitor is added to help control his blood pressure. Which of the following is most appropriate
next step in the management of his leg pain?
Correct Answer ImageA.Cilostazol trial
Incorrect Answer ImageB.Decrease diltiazem dose
Incorrect Answer ImageC.Decrease hydrochlorothiazide
Incorrect Answer ImageD.Discontinue metoprolol
Incorrect Answer ImageE.Increase isosorbide dinitrate dose

A 42-year-old man has recently completed chemotherapy for treatment of a supraglottic


squamous cell carcinoma staged at T2N0M0. Although he tolerated most of his chemotherapy
regimen well, the last days of his cycle included high-dose steroids, and he became increasingly
agitated and disoriented. He was believed to be suffering from steroid-induced psychosis. After
completing his cycle of chemotherapy, he voluntarily entered an inpatient psychiatric treatment
facility because the steroid-induced mood swings combined with his pre-existing depression
seemed to necessitate close psychiatric monitoring. On admission to the psychiatry unit,
temperature is 37.0ºC (98.6ºF), pulse is 72/min, respirations are 20/min, and blood pressure is
128/80 mm Hg. Physical examination shows a pale, thin young man who seems anxious and
worried but is otherwise normal. Laboratory studies show: 
Hematocrit 31%
Leukocytes 900/mm3
Platelets 105,000/mm3
The differential slide review shows 40% neutrophils, 50% lymphocytes and 10% monocytes.
When the results are reviewed, the psychiatry team requests a medical consult. You review the
patient and his records. Which of the following is an appropriate treatment for this man's reduced
absolute neutrophil count? 
Incorrect Answer ImageA.Intramuscular pegfilgrastim
Incorrect Answer ImageB.Intravenous cefepime
Incorrect Answer ImageC.Intravenous imipenem
Correct Answer ImageD.Observation
Incorrect Answer ImageE.Transfusion

A 40-year-old man comes to the physician because of involuntary movements for 4 months. He
has had involuntary face grimacing and awkward movements of his upper extremities. He also
reports several years of depressed mood and irritability. He has a history of hypertension and
osteoarthritis. Current medications include hydrochlorothiazide and naproxen. His father had
involuntary movements and died at age 48 years. Neurologic examination shows involuntary
choreiform movements; sensation is normal and deep tendon reflexes are brisk. On recall testing,
he can remember two out of three objects after 3 minutes with distraction. Laboratory studies are
within normal limits. An MRI scan is shown. Which of the following is the most likely
diagnosis?
Incorrect Answer ImageA.Early-onset Alzheimer disease
Correct Answer ImageB.Huntington disease
Incorrect Answer ImageC.Lacunar infarction
Incorrect Answer ImageD.Multi-infarct dementia
Incorrect Answer ImageE.Pick disease

A 22-year-old man comes to the emergency department because of a 3-day history of progressive
weakness and tingling that started in his legs and has spread to his arms. He has had shortness of
breath for the past 2 hours. He has a history of human immunodeficiency virus (HIV); he has had
no prior opportunistic infection. His temperature is 36.8°C (98.2°F), blood pressure 115/75 mm
Hg, pulse 80/min, and respirations 30/min. Muscle strength is 2/5 bilaterally in the lower
extremities and 3/5 in the upper extremities. Deep tendon reflexes are absent in the lower
extremities. His CD4+ T lymphocyte count is 750 cells/mm3. Lumbar puncture is performed.
Cerebrospinal fluid (CSF) analysis is most likely to show which of the following? 
Incorrect Answer ImageA.Increased opening pressure
Correct Answer ImageB.Increased total protein
Incorrect Answer ImageC.Increased white blood cells
Incorrect Answer ImageD.Low glucose
Incorrect Answer ImageE.Presence of red blood cells

A 44-year-old man with AIDS comes to the emergency department because of a 2-week history
of dry cough, fever, and shortness of breath with exertion. He takes no medications, denies any
prior allergies to medications, or history of prior opportunistic infections. His temperature is
38.6ºC (101.5ºF), pulse is 110/min, respirations are 26/min, and blood pressure is 120/65 mm
Hg. Physical examination shows crackles bilaterally in the lower lobes. A radiograph of the chest
shows diffuse bilateral interstitial and alveolar infiltrates. His CD4+ T lymphocyte count is
120/mm3. Arterial blood gas (ABG) analysis shows a pH of 7.38, pCO2 of 35 mm Hg, and
PaO2 of 75 mm Hg on room air. Which of the following is the most appropriate
pharmacotherapy?
Incorrect Answer ImageA.IV azithromycin and trimethoprim-sulfamethoxazole (TMP-SMX)
Incorrect Answer ImageB.IV pentamidine and highly active antiretroviral therapy
Incorrect Answer ImageC.IV TMP-SMX and oral prednisone
Incorrect Answer ImageD.Oral dapsone and highly active antiretroviral therapy
Incorrect Answer ImageE.Oral TMP-SMX
Correct Answer ImageF.Oral TMP-SMX and highly active antiretroviral therapy

A 56-year-old woman comes to the emergency department with two days of right flank pain,
fever, dysuria, and occasional hematuria. She has Type 2 diabetes mellitus well controlled with
metformin (HbA1c 6.5% last month) and hypertension well controlled with lisinopril. She had a
left nephrectomy in childhood for ureteral reflux, but has no history of underlying renal disease
in the right kidney (microalbumin absent on urine testing 3 months ago). Her temperature is
39.4ºC (103ºF), blood pressure is 90/50 mm Hg, and pulse is 110/minute. She appears ill and
flushed, and has marked right costovertebral angle tenderness. The rest of the physical
examination is normal. Laboratory studies show:
Blood:
Hemoglobin 12.2 g/dL
Leukocyte count 22,000/mm3
Serum:
Urea nitrogen 16 mg/dL
Creatinine 1.2 mg/dL
Sodium 135 mEq/L
Potassium 3.8 mEq/L
Urine:
120 white blood cells/hpf
100 red blood cells/hpf
Many bacteria
2-3 white blood cell casts
CT scan of the abdomen shows an absent left kidney and air in the right renal parenchyma.
Intravenous saline and broad spectrum antibiotics are started. Which of the following is the most
appropriate next step in management? 
Incorrect Answer ImageA.Continued antibiotics and hydration alone
Incorrect Answer ImageB.Intravenous pyelogram
Correct Answer ImageC.Percutaneous nephrostomy
Incorrect Answer ImageD.Nephrectomy
Incorrect Answer ImageE.Ureteroscopy

A 30-year-old man was admitted to the intensive care unit 6 days ago following a motor vehicle
accident. He sustained a right femur fracture and respiratory difficulties caused by pulmonary
contusion. His condition has improved and he is recovering in the medical ward with no acute
condition. Physical examination shows right lung base rales. Abdominal examination reveals
hepatosplenomegaly. Laboratory workup reveals a hemoglobin of 9 g/dL, a leukocyte count of
55,000/mm3with 95% neutrophils and bands, and a platelet count of 160,000/mm3. No
myelocytes or metamyelocytes are noted. The leukocyte alkaline phosphatase (LAP) score is
high. Which of the following is the most likely cause of this patient's elevated leukocyte count?
Incorrect Answer ImageA.Agnogenic myeloid metaplasia
Incorrect Answer ImageB.Chronic lymphocytic leukemia
Incorrect Answer ImageC.Chronic myelogenous leukemia
Correct Answer ImageD.Leukemoid reaction
Incorrect Answer ImageE.Polycythemia vera

A 22-year-old woman comes to the emergency department because of progressive fever,


nonproductive cough, and shortness of breath for 5 days. She has taken clarithromycin for the
past 2 days; however, her symptoms have worsened. Her medical history is unremarkable. She
does not smoke or drink alcohol. She is in moderate respiratory distress. Her temperature is
38.8ºC (103.6ºF), blood pressure is 125/70 mm Hg, pulse is 130/min, respirations are 32/min,
and oxygen saturation is 92% on 4 to 5 L/min oxygen by nasal cannula. Decreased breath sounds
and crackles are heard on auscultation bilaterally. Radiograph of the chest and CT scan of the
chest are shown. Bronchoalveolar lavage shows numerous eosinophils (45%). Which of the
following is the most appropriate pharmacotherapy?
Incorrect Answer ImageA.Azithromycin
Correct Answer ImageB.Corticosteroids
Incorrect Answer ImageC.Doxycycline
Incorrect Answer ImageD.Meropenem
Incorrect Answer ImageE.Vancomycin

A 65-year-old woman comes to the physician because of a 3-month history of leg pain. She has
had painful cramps of the calves and thighs while walking down a flight of stairs. The symptoms
are not related to the distance walked. The pain resolved after about 10 minutes of rest. Her pain
is relieved by sitting and leaning forward. She has a 20-year history of hypertension controlled
with hydrochlorothiazide. She does not use tobacco or alcohol. Her blood pressure is 150/85 mm
Hg. There is mild tenderness over the lumbosacral spine. The remainder of the examination
shows no abnormalities. Which of the following is the most likely diagnosis? 
Incorrect Answer ImageA.Lumbago (mechanical back strain)

Incorrect Answer ImageB.Lumbar disc herniation


Correct Answer ImageC.Lumbar spinal stenosis
Incorrect Answer ImageD.Osteoarthritis
Incorrect Answer ImageE.Peripheral arterial disease

A 32-year-old woman comes to the physician because of bumps that developed on her earlobes
over the past 3 months. She states that they have been itchy and enlarging. She had her ears
pierced at these sites 6 months ago. Her past medical history is unremarkable. Physical
examination of her ears shows bilateral rubbery, firm, skin-colored nodules on her posterior
earlobes adjacent to the sites of the piercing. They are somewhat mobile and are tender to
palpation. Examination of her back and shoulders is shown. Which of the following is the most
appropriate next step in the management of the lesions on her ears?
Incorrect Answer ImageA.Compressive earrings
Incorrect Answer ImageB.Immediate surgical excision
Correct Answer ImageC.Intralesional corticosteroid injection
Incorrect Answer ImageD.Systemic corticosteroids
Incorrect Answer ImageE.Topical corticosteroids

A 78-year-old woman with known metastatic breast cancer is brought to the emergency
department because of altered mental status and diffuse bone pain. The bone pain has been
progressively worse over the past week, and her mental status began deteriorating this morning,
according to family. She was diagnosed with breast cancer 2 years earlier and underwent a
radical mastectomy. She was found to have metastatic disease to her liver 6 months prior. She
has been well until this past week. She does not have any other significant past medical history.
On physical examination, she appears lethargic and confused. She is normotensive and afebrile
and oxygen saturation is 97% on room air. Head and neck examination shows no papilledema or
jugular venous distension. Lungs are clear to auscultation and no abnormal heart sounds are
heard. Abdominal examination shows hepatomegaly but no tenderness or rebound is elicited.
Peripheral examination is normal. Chest radiograph shows multiple lucencies in the humeral
heads and ribs. CT scan of the brain shows no abnormalities. Laboratory data are shown below.
White blood cells 5,800/mm3
Hematocrit 34%
Platelets 140,000/mm3
Sodium 137 mEq/L
Potassium 4.5 mEq/L
Chloride 104 mEq/L
Bicarbonate 24 mEq/L
Urea nitrogen 14 mg/dL
Creatinine 1.0 mg/dL
Calcium 12.5 mg/dL
Albumin 2.7 g/dL
Which of the following is the most appropriate treatment for this patient's hypercalcemia? 
Incorrect Answer ImageA.Furosemide only
Incorrect Answer ImageB.Hydrochlorothiazide only
Incorrect Answer ImageC.IV furosemide, then IV normal saline
Correct Answer ImageD.IV normal saline plus furosemide as needed
Incorrect Answer ImageE.IV normal saline plus hydrochlorothiazide as needed

A 45-year-old man comes to the hospital because of sudden onset of palpitations. He has a
history of paroxysmal supraventricular tachycardia with approximately 3 episodes per year and a
history of asthma. He is currently taking inhaled beta-agonists. His pulse is 160/min and his
blood pressure is 115/75 mm Hg. He appears anxious. Physical examination shows no carotid
bruits, and there is no jugular venous distention. The chest is clear, and there is no clubbing,
cyanosis, or edema. An ECG shows a narrow complex tachycardia. A carotid massage is
performed, which fails to break the rhythm. Treatment with 6 mg of IV adenosine breaks the
rhythm. Two minutes later, his pulse is 140/min and he is in a narrow complex tachycardia
again. He is administered a second dose of adenosine, which does not break the rhythm. Three
minutes later, his pulse is 140/min and blood pressure is 110/72 mm Hg. Which of the following
is the most appropriate next step in management?
Correct Answer ImageA.IV diltiazem
Incorrect Answer ImageB.IV metoprolol
Incorrect Answer ImageC.IV procainamide
Incorrect Answer ImageD.Synchronized cardioversion
Incorrect Answer ImageE.Unsynchronized cardioversion
 62-year-old man is brought to the emergency department because he has been seizing for the last
hour without regaining consciousness. He has a history of small-cell lung cancer. He has no
history of hypertension, diabetes, thyroid disease, or congestive heart failure. His temperature is
37.4ºC (99.3ºF), blood pressure is 130/90 mm Hg, pulse is 86/min, and respirations are 14/min.
Physical examination including funduscopic examination shows no abnormalities. Laboratory
studies show: 
Na+ 112 mEq/L 
K+ 4.0 mEq/L 
Urea nitrogen  10 mg/dL 
Creatinine  0.8 mg/dL 
Head CT scan shows no abnormalities. Radiograph of the chest shows a right perihilar mass
unchanged from a radiograph 1 month ago. Which of the following is the most appropriate next
step in management? 
Incorrect Answer ImageA.Conivaptan
Incorrect Answer ImageB.Demeclocycline
Incorrect Answer ImageC.Fluid restriction
Correct Answer ImageD.NaCl 3%
Incorrect Answer ImageE.NaCl 0.9% and furosemide

A 22-year-old patient comes to the physician with her sister who has noticed a greenish tinge to
her eyes. She has a history of depression treated with sertraline and was diagnosed with adult
attention deficit hyperactivity disorder (ADHD) 5 years ago. Neurologic examination shows
bilateral dysmetria and a mild hand tremor at rest. Slit lamp examination shows a brown-green
pigment surrounding the iris. 
Laboratory studies:
Alkaline phosphatase 75 U/L
Serum studies:
Aspartate aminotransferase (AST, GOT)  90 U/L
Alanine aminotransferase (ALT, GPT) 73 U/L
Ceruloplasmin 15 mg/dL
[normal >20 mg/dL ] 
Which of the following is most likely to establish the diagnosis in this patient? 
Incorrect Answer ImageA.Genetic testing
Incorrect Answer ImageB.Liver biopsy
Incorrect Answer ImageC.Measurement of total serum copper
Incorrect Answer ImageD.Measurement of urinary copper excretion
Correct Answer ImageE.No further testing is needed to establish diagnosis

A 45-year-old man comes to the emergency department because of a two week history of left leg
pain. The pain began in his lower back after he was swinging a sledgehammer at a concrete
block during a home renovation. Several days later, he experienced a burning sensation
"shooting" down the side of his leg. He smokes one pack of cigarettes daily. He does not use
illicit substances. His temperature is 37°C (98.6°F). The pain is reproduced when the left lower
leg is passively extended with the patient in a seated position. An MRI scan of the lower back
shows herniation of the L3–L4 intervertebral disk. Which of the following physical examination
findings is most likely to be seen in this patient? 
Incorrect Answer ImageA.Decreased bulbocavernosus reflex
Incorrect Answer ImageB.Depressed ankle-jerk reflex
Correct Answer ImageC.Depressed knee-jerk reflex
Incorrect Answer ImageD.Reduced anal sphincter tone
Incorrect Answer ImageE.Saddle anesthesia

A 42-year-old African-American woman with a history of systemic lupus erythematosus (SLE)


comes to the clinic with one month of progressive new-onset fatigue, causing her to be
increasingly sedentary. She can now barely sit up by herself from the sofa. She denies muscle
weakness or pain. Her lupus has been stable and managed with hydroxychloroquine for the past
ten years. Her last flare two years ago, complicated by kidney injury, required the use of
corticosteroids. Vital signs are within normal limits. There is scleral icterus. Abdominal
examination shows hepatosplenomegaly. The remainder of the physical examination is normal.
The urine is bilirubin positive with no cells or casts. Serum and blood studies show:
WBC 6,000/mcL
Hematocrit 40%
Creatinine 1.1 mg/dL
Alanine aminotransferase (ALT) 342 U/L
Alkaline phosphatase 1,000 U/L
Aspartate aminotransferase (AST) 246 U/L
Bilirubin, total 5.0 mg/dL
Hepatitis A, B, C serology negative
Antinuclear antibodies positive
Anti-mitochondrial antibodies negative
Anti-Smith antibodies negative
Anti-smooth muscle antibodies positive
Cytoplasmic antineutrophil cytoplasmic antibodies negative
(c-ANCA)
Perinuclear antineutrophil cytoplasmic antibodies negative
(p-ANCA)
Which of the following diagnoses is most likely?
Correct Answer ImageA.Autoimmune hepatitis
Incorrect Answer ImageB.Hydroxychloroquine toxicity
Incorrect Answer ImageC.Polyarteritis nodosa
Incorrect Answer ImageD.Primary biliary cirrhosis
Incorrect Answer ImageE.Primary sclerosing cholangitis

A 63-year-old man comes to his primary care physician with shortness of breath and cough for
the previous 3 months. The patient, who has a 25 pack/year history of smoking, reports that his
cough is worse in the morning. Vital signs are temperature smoking, reports that his cough is
worse in the morning. Vital signs are temperature 36.7°C (98.1°F), blood pressure 134/72 mm
Hg, pulse 104/min, respirations 24/min, and oxygen saturation 92% on room air. Physical
examination shows decreased breath sounds and occasional wheezing bilaterally. Percussion
shows increased tympany throughout the lungs. Cardiovascular examination shows tachycardia
and a loud second heart sound. The rest of the physical examination is unremarkable. Chest x-ray
is ordered. Which of the following findings on chest x-ray would be expected? 
Incorrect Answer ImageA.Bilateral diffuse reticular or reticulonodular infiltrates predominately at
the periphery
Incorrect Answer ImageB.Diffuse reticulonodular infiltrates in the lower lungs, bilateral pleural
thickening and diaphragmatic calcifications with sparing of the costophrenic angles
Correct Answer ImageC.Flattening of the diaphragm, increased retrosternal air space, and a long,
narrow heart shadow
Incorrect Answer ImageD.Interstitial disease, bronchial thickening, and patchy bilateral alveolar
infiltrates
Incorrect Answer ImageE.Multiple small nodules that are more prominent in the upper lung fields
and calcification of the hilar lymph nodes

A 62-year-old man returns for a follow-up appointment with his physician. He was recently
treated with amphotericin B for Aspergillus pneumonia however, he still reports shortness of
breath and generalized weakness. He has a history of hypertension treated with
hydrochlorothiazide and lisinopril and esophageal reflux treated with omeprazole. His
temperature is 36.9°C (98.4°F), pulse is 104/min, blood pressure is 135/75 mm Hg, respirations
are 24/min, and oxygen saturation is 94% on room air. Physical examination shows an elderly
male in no acute distress. The heart and lungs are normal. Chest x-ray shows improvement and
resolution of the right upper lobe infiltrate. Laboratory studies show:
pH 7.36
PaCO2 26 mm Hg
PaO2 84 mm Hg
HCO3– 14 mEq/L
Na +
137 mEq/L
K+ 3.1 mEq/L
Cl–
114 mEq/L
Creatinine 1.5 mg/dL
BUN 28 mg/dL
Glucose 92 mg/dL
Ca 2+
9.2 mg/dL
Urinalysis shows pH 7.0 and specific gravity 1.020 and is negative for glucose, ketones, RBCs,
and WBCs. Which of the following is the most likely cause of acidemia in this patient? 
Incorrect Answer ImageA.Hypoventilation
Incorrect Answer ImageB.Lisinopril
Incorrect Answer ImageC.Omeprazole
Correct Answer ImageD.Renal tubular acidosis, type 1
Incorrect Answer ImageE.Renal tubular acidosis, type 2
Incorrect Answer ImageF.Renal tubular acidosis, type 4
A 62-year-old man is brought to the emergency department because of crushing chest pain and
diaphoresis. An ECG shows Q waves, and troponin and CK-MB fraction are elevated. His blood
pressure is 106/62 mm Hg and his pulse is 100/min. Cardiopulmonary examination shows an S4
and an S3, and crackles are heard over less than one third of the lung fields that clear with
coughing. The peripheral pulses are palpable and symmetric with adequate peripheral perfusion
and no edema. Which of the following is the most appropriate next step in patient care? 
Correct Answer ImageA.Administer 325 mg of a chewable aspirin
Incorrect Answer ImageB.Begin therapy with streptokinase
Incorrect Answer ImageC.Insert a pulmonary artery catheter
Incorrect Answer ImageD.Perform intra-aortic balloon counterpulsation
Incorrect Answer ImageE.Perform technetium-99m pyrophosphate scintigraphy

A 42-year-old woman comes to the physician because of a 4-month history of intermittent


abdominal cramps and diarrhea. She has had episodes of skin flushing lasting from 2–15
minutes, most pronounced in the head and neck area. She denies nausea, vomiting, constipation,
or blood in her stools. Her medical history is unremarkable and she takes no medications. A
grade 3/6 murmur is heard along the mid left sternal border. The lungs are clear to auscultation.
Abdominal examination shows no abnormalities. Which of the following is most likely to
establish the diagnosis? 
Correct Answer ImageA.5-hydroxyindoleacetic acid (5-HIAA) excretion in the urine
Incorrect Answer ImageB.Colonoscopy
Incorrect Answer ImageC.Echocardiogram
Incorrect Answer ImageD.Gastroscopy
Incorrect Answer ImageE.Pelvic ultrasound

A 56-year-old woman is brought to the hospital because of chest pain and shortness of breath.
She is allergic to heparin. Ultrasound shows a thrombus in the right superficial femoral vein.
Ventilation-perfusion scan shows a perfusion defect in the right lung. She is administered
supplemental oxygen and a loading dose of warfarin. After 3 days, she develops pain and redness
in her left breast. Physical examination shows erythema and tenderness of the left breast with a
central area of scar formation. A biopsy specimen of the lesion shows extensive thrombosis with
microvascular injury and fibrin deposits in the postcapillary venules and small veins. Loss of
function of which of the following is the most likely cause of this patient's skin lesion? 
Incorrect Answer ImageA.Factor VII
Incorrect Answer ImageB.Factor IX
Incorrect Answer ImageC.Platelets
Correct Answer ImageD.Protein C
Incorrect Answer ImageE.Von Willebrand factor

A 56-year-old man is admitted to the intensive care unit for IV antibiotics because of fever,
severe shortness of breath, and cough. His shortness of breath worsened and he was intubated
and placed on mechanical ventilation. One day later, his temperature is 37.4ºC (99.3ºF), pulse is
115/min, and blood pressure is 124/80 mm Hg. His oxygen saturation is 70% on 100% oxygen
by mechanical ventilation. A radiograph of the chest taken immediately following intubation and
a film taken during the hypoxia are shown. Which of the following is the most likely cause of
this patient's acute deterioration?
Incorrect Answer ImageA.Acute respiratory distress syndrome (ARDS)
Correct Answer ImageB.Airway obstruction
Incorrect Answer ImageC.Aspiration
Incorrect Answer ImageD.Hemothorax
Incorrect Answer ImageE.Pulmonary edema
Incorrect Answer ImageF.Spontaneous pneumothorax

A 41-year-old man comes to the emergency department with a 2-week history of fever, anorexia,
weight loss, and fatigue. He is otherwise healthy and has not seen a physician recently, but did
recently have his teeth cleaned. He is on no medications and has no allergies. He drinks alcohol
only occasionally and denies intravenous drug use. On physical examination he appears ill, with
a temperature of 38.9ºC (102ºF) and a few petechiae in both eyes. Cardiac examination shows a
3/6 pansystolic murmur heard best along the apex with radiation to the axilla, and a pericardial
rub. Blood is drawn and sent to the laboratory for culture. Which of the following is the most
sensitive diagnostic test to confirm the likely diagnosis? 
Incorrect Answer ImageA.Cardiac MRI
Incorrect Answer ImageB.Electrocardiogram
Incorrect Answer ImageC.Stress test
Correct Answer ImageD.Transesophageal echocardiogram (TEE)
Incorrect Answer ImageE.Transthoracic echocardiogram (TTE)

A 65-year-old man comes to the office for a routine health maintenance examination. He
recently recovered from an episode of pneumonia for which he was treated with levofloxacin for
5 days. He states that besides intermittent episodes of constipation, for which he takes docusate,
he has not had any other major medical problems. He denies fevers, cough, shortness of breath,
or a history of abnormal bleeding. His temperature is 37.1ºC (98.7ºF), pulse is 82/min,
respirations are 16/min, and blood pressure is 120/80 mm Hg. There is bilateral adenopathy in
the cervical and supraclavicular chain. Lymph nodes are hard and nontender; the largest is
approximately 4 cm. There is bilateral inguinal adenopathy. The rest of the physical examination
shows no abnormalities, including a rectal exam which shows no fecal occult blood. Laboratory
studies show:
Hemoglobin  14 g/dL
Platelets  170,000/mm3
Leukocyte count 130,000/mm3
Granulocytes  15%
Lymphocytes (mature)  80%
Monocytes  5%
Peripheral smear  Smudge cells
CD19  Positive
Which of the following is the most appropriate intervention? 
Incorrect Answer ImageA.Bone marrow transplant
Incorrect Answer ImageB.Chlorambucil
Incorrect Answer ImageC.Chlorambucil + prednisone
Incorrect Answer ImageD.Fludarabine
Correct Answer ImageE.No therapy is needed

A 50-year-old man comes to the physician because of a 6-month history of seizures. He has had
no aura or focal onset of seizure activity. He has had a total of three seizures, with generalized
tonic-clonic jerking activity lasting 1 to 2 minutes accompanied by urinary incontinence. He has
had confusion for up to 15 minutes after each seizure. He takes no medications. He started
drinking 15 to 18 beers twice weekly since his divorce 7 months ago. His temperature is 37.2°C
(99.0°F), pulse is 82/min, respirations are 13/min, and blood pressure is 142/87 mm Hg.
Neurologic examination shows no abnormalities. Which of the following is most likely to
explain the etiology of this patient's seizures?
Incorrect Answer ImageA.The seizure characteristics
Incorrect Answer ImageB.The patient's birth history
Incorrect Answer ImageC.The patient's profession
Correct Answer ImageD.The relationship of alcohol use to the seizures
Incorrect Answer ImageE.The time of day the seizures occur

A 65-year-old man with a history of multiple myeloma comes to the physician because of
progressive weakness of both legs. This has been going on for the past 6 months. He also has
diffuse back pain and recently noticed numbness in both legs. The patient denies urinary or fecal
incontinence. Current medications include melphalan, thalidomide, and prednisone. He has
smoked one pack of cigarettes daily for 50 years. His sister and mother have a history of
osteoporosis. Physical exam demonstrates tenderness to palpation of the spine, loss of
proprioception and temperature sensation in the lower extremities, and decreased strength in both
the flexors and extensors of the lower extremities. Which of the following is the most likely
cause of his weakness?
Incorrect Answer ImageA.Infiltration of peripheral nerves by amyloid
Correct Answer ImageB.Spinal cord compression by tumor
Incorrect Answer ImageC.Steroid-induced avascular necrosis of the femoral head
Incorrect Answer ImageD.Thalidomide-induced neurotoxicity
Incorrect Answer ImageE.Thalidomide-induced phocomelia

An AIDS patient develops symptoms suggestive of a severe, persistent pneumonia with cough,
fever, chills, chest pain, weakness, and weight loss. The patient does not respond to penicillin
therapy, but goes on to develop very severe headaches. The presence of focal neurologic
abnormalities leads the clinician to order a CT scan of the head. This demonstrates several brain
abscesses. Biopsy of one of these lesions demonstrates beaded, branching, filamentous gram-
positive bacteria that are weakly acid fast. Which of the following is the most likely causative
organism? 
Incorrect Answer ImageA.Actinomyces
Incorrect Answer ImageB.Aspergillus
Incorrect Answer ImageC.Burkholderia
Incorrect Answer ImageD.Francisella
Correct Answer ImageE.Nocardia

A 66-year-old man comes to his physician with urinary frequency and urgency. He has a 30
pack-year cigarette smoking history and quit 2 months ago. Digital rectal examination is normal.
There is no lymphadenopathy, no hepatomegaly, and no lower extremity lymphedema.
Urinalysis shows 4+ microscopic hematuria and is otherwise normal. Serum BUN is 11 mg/dL
and creatinine is 1.2 mg/dL. Which of the following is the best next step in management? 
Correct Answer ImageA.Cystoscopy
Incorrect Answer ImageB.Intravenous pyelography
Incorrect Answer ImageC.Pelvic magnetic resonance imaging
Incorrect Answer ImageD.Renal ultrasonography
Incorrect Answer ImageE.Voiding cystourethrogram

A 65-year-old man comes to the physician because of weakness and fatigue for the past 2
months. He is usually very energetic and maintains an active lifestyle. However, he often feels
more tired in the afternoons now and frequently has to cancel his outings and activities. His wife
is quite concerned and persuades him to go see his physician. The patient’s past medical history
is significant for prostatic hyperplasia, hypertension, and hypercholesterolemia. He does not
smoke tobacco, consume alcohol or illicit drugs, and is unsure of his family history. His
medications include terazosin, atorvastatin, and hydrochlorothiazide. On physical examination,
his pulse is 94/min, his blood pressure is 120/80 mm Hg, respirations are 14/min, and pulse
oximetry is 97% on room air. He appears pale and has brittle nails on his toes. A fecal occult
blood test is positive. Peripheral blood smear shows erythrocytes that are hypochromic. Which of
the following is the most appropriate next step in the diagnostic workup of this patient?
Incorrect Answer ImageA.Anti-parietal cell antibody testing
Incorrect Answer ImageB.Bleeding scan
Correct Answer ImageC.Colonoscopy
Incorrect Answer ImageD.Upper GI endoscopy
Incorrect Answer ImageE.Upper GI series

A 64-year-old man is brought to his physician for evaluation. His past medical history is
significant for severe emphysema and coronary artery disease. He received angioplasty with
stenting 2 years ago. His current medications include ipratropium metered-dose inhaler, home
oxygen therapy, aspirin, nifedipine, amiodarone, and isosorbide dinitrate. He smokes 1 or 2
cigarettes a day sporadically on weekends. A younger brother died of lung disease at age 50.
Physical examination shows bilateral expiratory wheezing at the bases and normal S1 and S2.
Abdominal exam is unremarkable, and extremity exam shows no clubbing, cyanosis, or edema.
Laboratory findings are shown below:
Hemoglobin 14 g/dL
Hematocrit 52%
Leukocyte count 4,300
Hemoglobin 14 g/dL
Platelet count 200,000
Na 135 mEq/L
K 4.0 mEq/L
CL 110 mEq/L
HCO3 28 mEq/L
BUN 20 mg/dL
Creatinine 1.0 mg/dL
AST 55 U/L
ALT 45 U/L
Alkaline phosphatase 140 U/L
PT 22 seconds
PTT 32 seconds
Which of the following is the most likely diagnosis? 
Correct Answer ImageA.Alpha-1-antitrypsin deficiency
Incorrect Answer ImageB.Amiodarone toxicity
Incorrect Answer ImageC.Primary hemochromatosis
Incorrect Answer ImageD.Primary sclerosing cholangitis
Incorrect Answer ImageE.Secondary hemochromatosis
Incorrect Answer ImageF.Wilson disease

A 32-year-old man comes to the physician because of a four day history of lower back pain. He
developed the pain a few hours after he rearranged his furniture. There is no radiation to the
buttocks or legs. His medical history is unremarkable. Temperature is 37ºC (98.6ºF), blood
pressure is 110/72 mm Hg, and pulse is 88/min. There is mild tenderness over the lumbosacral
spine. The straight leg raise test is negative. Neurologic examination shows no abnormalities.
Which of the following is the most appropriate next step in management? 
Incorrect Answer ImageA.Bed rest
Correct Answer ImageB.Continue regular activity with NSAIDs
Incorrect Answer ImageC.Muscle relaxants
Incorrect Answer ImageD.Referral to a neurosurgeon
Incorrect Answer ImageE.Steroid injection

A 32-year-old woman comes to the physician because of a 1-year history of skin changes on her
face and hands. She has had redness, itching, and progressive thickening of the skin of her
fingers. She also has pain and tingling of her fingers in the cold. She has had pain and stomach
discomfort after meals. Examination of her hand is shown. The skin is firm and smooth.
Laboratory studies show a positive antinuclear antibody and positive anti-topoisomerase
antibody. Which of the following clinical findings is most likely in this patient?
Correct Answer ImageA.Bibasilar pulmonary fibrosis
Incorrect Answer ImageB.GI hypermotility
Incorrect Answer ImageC.Photo-induced malar erythema
Incorrect Answer ImageD.Proximal muscle weakness
Incorrect Answer ImageE.Normal ECG
A 24-year-old man comes to the physician with a painful right knee. He bruised the knee one
hour ago while playing basketball and noticed immediate swelling and pain around the joint. An
opponent's knee hit the patient's knee directly. He does not recall hearing any snap or having any
awkward movements. He does not have a significant past medical history, nor does he take any
medications. On physical examination several ecchymoses in various stages are observed on his
arms. He states that he bruises easily. Knee examination demonstrates a moderate effusion,
without pain with any passive or active movement, and with a negative anterior and posterior
drawer test. The patient states that this is the third time this has happened. Laboratory data
show: 
WBC 6,800/mm3
Hct 43%
Platelets 245,000/mm3
PT 15 sec
PTT 42 sec
Bleeding time 14 min
Fibrinogen 220 mg/dL
Which of the following is the most likely etiology of the patient's coagulopathy? 
Incorrect Answer ImageA.Acquired factor inhibitor
Incorrect Answer ImageB.Factor VIII deficiency
Incorrect Answer ImageC.Factor IX deficiency
Incorrect Answer ImageD.Occult aspirin use
Correct Answer ImageE.Von Willebrand factor deficiency

A 56-year-old man is brought to the emergency department because of severe shortness of


breath, fever, and cough. He receives IV ceftriaxone and azithromycin. His respiratory function
worsens and he is intubated and placed on controlled mechanical ventilation. He is admitted to
the hospital and two days later, his oxygen saturation decreases to 70% on 100% oxygen by
mechanical ventilation. Decreased breath sounds on the left side are heard on auscultation and
there is dullness to percussion over the left lung fields. Radiograph of the chest is shown. Which
of the following is the most appropriate next step in management?
A.Aggressive diuresis
B.Bronchoscopy
C.Chest tube placement
D.Prednisone
E.Thoracocentesis

 56-year-old man is brought to the emergency department because of severe shortness of breath,
fever, and cough. He receives IV ceftriaxone and azithromycin. His respiratory function worsens
and he is intubated and placed on controlled mechanical ventilation. He is admitted to the
hospital and two days later, his oxygen saturation decreases to 70% on 100% oxygen by
mechanical ventilation. Decreased breath sounds on the left side are heard on auscultation and
there is dullness to percussion over the left lung fields. Radiograph of the chest is shown. Which
of the following is the most appropriate next step in management?
Incorrect Answer ImageA.Aggressive diuresis
Correct Answer ImageB.Bronchoscopy
Incorrect Answer ImageC.Chest tube placement
Incorrect Answer ImageD.Prednisone
Incorrect Answer ImageE.Thoracocentesis

A 62-year-old man is undergoing evaluation for urinary hesitancy and a weak urinary stream. He
states that he sleeps poorly because of the need to urinate 3 to 4 times per night. He has a history
of stage I hypertension that persists despite diet and exercise. His blood pressure is 152/94 mm
Hg. Physical examination shows an enlarged, symmetric, and non-nodular prostate. Which of the
following is the most appropriate pharmacotherapy?
Incorrect Answer ImageA.Atenolol
Correct Answer ImageB.Doxazosin
Incorrect Answer ImageC.Felodipine
Incorrect Answer ImageD.Finasteride
Incorrect Answer ImageE.Hydrochlorothiazide

A 54-year-old man is admitted to the hospital because of jaundice and right upper quadrant
abdominal tenderness. His temperature is 38ºC (100.4ºF). Physical examination shows
generalized muscle wasting. The liver is palpable 4 cm below the right costal margin and the
spleen is palpable 3 cm below the left costal margin. Ultrasound of the abdomen shows a small
amount of ascitic fluid. Laboratory studies show: 
Serum albumin 2.5 g/dL
Globulin 3.8 g/dL
Bilirubin:
Total 3.5 mg/dL
Direct 1.7 mg/dL
AST 300 U/L
ALT 120 U/L
Amylase 100 U/L
Alkaline phosphatase 100 U/L
Hematologic hemoglobin 10 g/dL
Mean corpuscular volume 100 µm
Leukocyte count 4,000/mm3
Segmented neutrophils 66%
Bands 7%
Prothrombin time 18 sec
Which of the following is the most likely diagnosis? 
Incorrect Answer ImageA.Acute pancreatitis
Correct Answer ImageB.Alcoholic hepatitis
Incorrect Answer ImageC.Cholecystitis
Incorrect Answer ImageD.Duodenal ulcer
Incorrect Answer ImageE.Viral hepatitis

A 50-year-old woman comes to the clinic for an annual health maintenance examination. She
reports feeling well. Her past medical history is significant for a long-standing history of severe
rheumatoid arthritis, for which she takes infliximab and ibuprofen. Her temperature is 37ºC
(98.6ºF), pulse is 80/min, respirations are 12/min, and blood pressure is 117/80 mm Hg. Her
oxygen saturation is 98% on room air. Physical examination shows a swan-neck deformity in
both hands. The rest of the physical examination is unremarkable. Laboratory studies show:
Hemoglobin  10 g/dL
Hematocrit  30%
WBC  8,000/mm3
Platelets  200,000/mm3
Sodium  135 mEq/L
Potassium  3.5 mEq/L
Chloride  105 mEq/L
HCO3 22 mEq/L
BUN 15 mg/dL
Creatinine  1.0 mg/dL
ESR 30 mm/h
Which of the following erythrocyte findings is most likely to be seen on peripheral blood smear? 
Incorrect Answer ImageA.Macrocytosis
Correct Answer ImageB.Microcytosis
Incorrect Answer ImageC.Spherocytosis
Incorrect Answer ImageD.Target cells
Incorrect Answer ImageE.Teardrop cells

A 63-year-old man comes in with acute thigh pain after slipping on his kitchen floor. He was
able to brace his fall with his arm but still found that his left thigh hit the ground followed by a
"snap." He has been experiencing thigh pain for the last year, but has not spoken with a physician
about his symptoms. He denies weight loss or loss of appetite. His past medical history is notable
for diabetes mellitus and hypertension, both of which have been well controlled. On physical
examination he is normotensive and afebrile. Head and neck examination shows decreased
hearing in both ears. Cardiac examination is notable for a 2/6 midsystolic crescendo-decrescendo
murmur. Pulmonary and abdominal examinations are unremarkable. Palpation of his thighs
elicits extreme pain and palpation of the non-injured thigh elicits a dull ache. Radiograph of the
left femur shows a midshaft fracture as well as an expanded deformed cortical contour.
Radiograph of the other extremity shows an expanded bowed femur. Bone scan shows focal
areas of intense uptake in both thighs. Laboratory data are:
Na+ 140 mEq/L
K +
4.8 mEq /L
Cl− 105 mEq/L
HCO3 −
24 mEq/L
Urea nitrogen  13 mg/dL
Creatinine  1.0 mg/dL
Calcium  9.2 mg /dL
Phosphate  2.9 mg/dL
AST  32 IU/L
ALT  25 IU/L
Alkaline phosphatase  400 IU/L
GGT 15 IU/L (normal: 9–50 IU/L)
ESR  5 mm/h
Other than a referral to orthopedic surgery, which of the following therapies should be offered to
this patient? 
Correct Answer ImageA.Bisphosphonate therapy
Incorrect Answer ImageB.Calcium supplementation
Incorrect Answer ImageC.Doxorubicin
Incorrect Answer ImageD.High-dose corticosteroids
Incorrect Answer ImageE.Hydrochlorothiazide

A 32-year-old man comes to his physician's office because of right upper quadrant pain, fatigue,
and fevers for the past week. Past medical history is remarkable only for travel to Mexico 1
month ago, during which he experienced several days of diarrhea. His temperature is 38.3°C
(101.0°F), blood pressure is 120/80 mm Hg, pulse is 86/min, and respirations are 13/min. Head
and neck examination is normal. Heart shows a normal S1 and S2 without murmurs, rubs, or
gallops. Chest auscultation shows crackles at the right lung base. On abdominal examination
there is diffuse right upper quadrant tenderness to palpation. No shifting dullness was found on
percussion. Peripheral pulses are normal. Laboratory data are shown below:
WBC count 15,000/mm3
Hgb 12 g/dL
Hct 36%
Platelet count 450,000/mm3
AST 150 IU/L
ALT 130 U/L
Alkaline phosphatase 280 U/L
Total bilirubin 1.0 mg/dL
Computed tomography of the abdomen shows a hypodense lesion in the right hepatic lobe as
well as a mass-like density in the cecum. Which of the following is most likely accurate
regarding this patient's current condition? 
Incorrect Answer ImageA.Imaging reliably differentiates this mass from a pyogenic liver abscess
Correct Answer ImageB.Men are more commonly affected than women
Incorrect Answer ImageC.Microscopic examination of the stool has a sensitivity of 75% for
detection of the causative organism
Incorrect Answer ImageD.Serologic testing plays little role in the diagnosis
Incorrect Answer ImageE.The majority of these patients have eosinophilia

A 55-year-old man who was involved in a motor vehicle accident was brought into the
emergency department by paramedics. They reported that the patient had not been wearing a
seatbelt and was found slumped on the steering wheel, which was deformed. The patient is weak
but conscious, and states that he feels pain when he breathes. His temperature is 37ºC (98.6ºF),
blood pressure is 130/80 mm Hg, pulse is 110/min, and respirations are 22/min. Oxygen
saturation is 95% on room air. Physical examination shows ecchymosis across the chest, and the
chest wall is tender to palpation. Pulmonary examination shows decreased air entry bilaterally
with normal resonance. Cardiac examination shows regular rate and rhythm; no murmurs are
appreciated. Several minutes after the initial examination, the patient's blood pressure increases
to 145/95 mm Hg, respirations increase to 26/min, and O2saturation decreases to 91% on room
air.
 Which of the following is the most appropriate next step in management? 
Incorrect Answer ImageA.Chest CT scan
Correct Answer ImageB.Give analgesia and supplemental oxygen
Incorrect Answer ImageC.Intubate and place on mechanical ventilator
Incorrect Answer ImageD.Perform needle thoracocentesis
Incorrect Answer ImageE.Start on empiric antibiotics

A 55-year-old man who was involved in a motor vehicle accident was brought into the
emergency department by paramedics. They reported that the patient had not been wearing a
seatbelt and was found slumped on the steering wheel, which was deformed. The patient is weak
but conscious, and states that he feels pain when he breathes. His temperature is 37ºC (98.6ºF),
blood pressure is 130/80 mm Hg, pulse is 110/min, and respirations are 22/min. Oxygen
saturation is 95% on room air. Physical examination shows ecchymosis across the chest, and the
chest wall is tender to palpation. Pulmonary examination shows decreased air entry bilaterally
with normal resonance. Cardiac examination shows regular rate and rhythm; no murmurs are
appreciated. Several minutes after the initial examination, the patient's blood pressure increases
to 145/95 mm Hg, respirations increase to 26/min, and O2saturation decreases to 91% on room
air.
The patient responds to supplemental oxygenation and analgesics. Repeat vital signs show blood
pressure is 134/86 mm Hg, pulse is 110/min, respirations are 22/min, and pulse oximetry is 97%
on 2 L of oxygen via nasal cannula. The patient continues to have persistent midsternal chest
pain. CT scan of the chest shows a mildly displaced sternal fracture and bilateral pulmonary
contusions, but is otherwise normal. ECG shows sinus tachycardia, right bundle branch block,
and occasional premature ventricular contractions (PVCs) without other significant
abnormalities. No prior ECG is available for review. Which of the following is the most
appropriate next step in management?
Correct Answer ImageA.Admit for cardiac monitoring and echocardiography
Incorrect Answer ImageB.Admit patient to surgical ward for analgesia and oxygen therapy
Incorrect Answer ImageC.Obtain arterial blood gas
Incorrect Answer ImageD.Obtain cardiac enzymes
Incorrect Answer ImageE.Transesophageal echocardiogram

A 45-year-old woman who regularly wears high-heeled, pointed-toe shoes complains of pain in
the forefoot after prolonged standing or walking. Occasionally she also experiences numbness, a
burning sensation, and tingling in the area. Physical examination shows no obvious deformities
and a very tender spot in the third interspace, between the third and fourth toes. There is no
redness, limitation of motion, or sign of inflammation. Which of the following is the most likely
diagnosis? 
Incorrect Answer ImageA.Gout
Incorrect Answer ImageB.Hallux rigidus
Incorrect Answer ImageC.Metatarsophalangeal articulation pain
Correct Answer ImageD.Morton neuroma
Incorrect Answer ImageE.Plantar fasciitis

A 44-year-old man comes to the physician because of decreased manual dexterity and weakness
in his arms and legs for 6 months. The weakness first started in his left arm and then involved the
right arm and legs. The weakness does not improve with rest. He denies difficulty with speech,
swallowing, and bladder or bowel incontinence. He also denies tingling or numbness in his hands
or feet. His medical history is unremarkable. His mother has hypertension and his father has type
2 diabetes mellitus. His temperature is 37°C (98.6°F), blood pressure is 134/84 mm Hg, and
pulse is 78/min. Examination is shown (click media file). There is bilateral spasticity and wasting
in the upper and lower extremities. Muscle strength is 3/5 in all extremities. Deep tendon
reflexes are brisk and Babinski sign is present bilaterally. Sensation is normal. Which of the
following is the most likely diagnosis? 
Correct Answer ImageA.Amyotrophic lateral sclerosis
Incorrect Answer ImageB.Hemorrhagic stroke
Incorrect Answer ImageC.Multiple sclerosis
Incorrect Answer ImageD.Myasthenia gravis
Incorrect Answer ImageE.Polymyositis

An 18-year-old man who is a college football player comes to the physician for a routine
physical examination. He states that he feels well and has no specific physical complaints.
Physical examination shows clear lungs bilaterally. Heart sounds are regular in rate and rhythm,
with a fourth heart sound. A harsh crescendo-decrescendo systolic murmur is heard at the apex.
The murmur becomes louder when the patient is asked to perform the Valsalva maneuver, and it
gets better when the patient is asked to squat. The carotid pulse is brisk in upstroke and bifid.
Which of the following is the most common presentation of this patient's condition? 
Incorrect Answer ImageA.Angina
Correct Answer ImageB.Dyspnea
Incorrect Answer ImageC.Palpitations
Incorrect Answer ImageD.Sudden death
Incorrect Answer ImageE.Syncope

A 63-year-old woman comes to the emergency department because of a 3-month history of


nausea and severe epigastric abdominal pain radiating to the back, worse with eating, and
associated with nausea and loose, oily foul smelling stools. She has a history of hypertension,
hyperlipidemia, and two episodes of acute pancreatitis occurring three and seven years ago. She
takes no medications. Her temperature is 38.3ºC (100.9ºF), pulse is 143/min, respirations are
16/min, and blood pressure is 160/90 mm Hg. Examination reveals epigastric tenderness, mild
abdominal distention, and hypoactive bowel sounds. A radiograph film of the abdomen shows
scattered pancreatic calcifications. Laboratory studies show:
Albumin 2.8 g/dL
Bilirubin total 1.1 mg/dL
Serum studies show:
Amylase 45 U/L
Lipase 10 U/L
Aspartate aminotransferase (AST, GOT)  98 IU/L
Alanine aminotransferase (ALT, GPT)  45 IU/L
Which of the following is the most likely underlying cause of this patient's symptoms? 
Correct Answer ImageA.Alcohol
Incorrect Answer ImageB.Elevated triglycerides
Incorrect Answer ImageC.Gallstones
Incorrect Answer ImageD.Hypercalcemia
Incorrect Answer ImageE.Idiopathic

A 19-year-old college student comes to the student health center for a runny nose. She has had
these symptoms for the last 2 years and would like treatment. She states that the symptoms occur
around the spring season and tend to abate in the middle of the summer. There have been bouts
of sneezing and intractable rhinorrhea. Her past medical history is unremarkable and she does
not take any medications. She denies smoking and occasionally has alcohol at parties. On
physical examination, she is normotensive and afebrile. Her nasal mucosa is boggy, pale, and
wet. There is no lymphadenopathy. Her lungs are clear to auscultation. The remainder of the
examination is unremarkable. Which of the following is the most appropriate pharmacotherapy
for initial treatment?
Correct Answer ImageA.Intranasal corticosteroids
Incorrect Answer ImageB.Intranasal cromolyn
Incorrect Answer ImageC.Intranasal montelukast
Incorrect Answer ImageD.Oral corticosteroids
Incorrect Answer ImageE.Oral loratadine
Incorrect Answer ImageF.Oral pseudoephedrine

A 40-year-old man with a history of HIV comes to the emergency department because of a 2-
week history of a diffuse, dull headache and low-grade fever. He denies nausea, vomiting,
photophobia, motor weakness, numbness, or tingling. Current medications are zidovudine,
lamivudine, efavirenz, and trimethoprim-sulfamethoxazole. His temperature is 37.6ºC (99.6ºF),
pulse is 76/min, respirations are 16/min, and blood pressure is 118/72 mm Hg. On physical
examination, his pupils are equal, round, and reactive to light. Fundoscopic examination shows
no papilledema. There is no neck rigidity or lymphadenopathy. Lungs are clear to auscultation.
Cardiac examination shows normal S1 and S2 without murmurs. A lumbar puncture is
performed, and CSF analysis is shown:
Opening pressure 28 mm Hg
Leukocyte count 20 cells/mm3
Lymphocytes 73%
Protein 100 mg/dL
Glucose 20 mg/dL
CSF microscopy shows encapsulated organisms. CT scan of the head shows no abnormalities.
Which of the following is the most appropriate pharmacotherapy? 
Incorrect Answer ImageA.Albendazole plus dexamethasone
Correct Answer ImageB.Amphotericin B plus flucytosine
Incorrect Answer ImageC.Ceftazidime plus ampicillin
Incorrect Answer ImageD.Pyrimethamine plus sulfadiazine
Incorrect Answer ImageE.Supportive treatment

A 74-year-old man visits his physician because of numerous extremely itchy blisters on his
chest, arms, and legs. He states that the blisters rarely rupture with trauma. He has had no fever.
Examination is shown. There are lesions on the abdomen, back, proximal upper extremities, and
lower extremities. Oral and genital examinations show no abnormalities. A biopsy specimen of
the skin shows subepidermal blistering with numerous eosinophils, and direct
immunofluorescence shows a linear band of IgG and C3 at the basement membrane. Which of
the following is the most likely diagnosis?
Correct Answer ImageA.Bullous pemphigoid
Incorrect Answer ImageB.Cicatricial pemphigoid
Incorrect Answer ImageC.Epidermolysis bullosa acquisita
Incorrect Answer ImageD.Junctional epidermolysis bullosa
Incorrect Answer ImageE.Paraneoplastic pemphigus
Incorrect Answer ImageF.Pemphigus vulgaris

A 21-year-old woman comes to the emergency department because of gross hematuria. Three
days ago, she noticed fever, cough, and a sore throat. Today she noted that her urine turned the
toilet water bright red. She has no history of kidney disease but she remembers being told that
she had microscopic hematuria on a college physical examination last year. She takes no
medications, does not smoke, and uses no recreational drugs. On examination, she is not in
distress. Her temperature is 37ºC (98.6ºF), pulse rate is 88/min, respirations are 12/min, and
blood pressure is 110/72 mm Hg. The skin is normal. Pharynx shows mild redness with no
exudates. She has no lymphadenopathy. Examination of the lungs, heart, and abdomen is normal.
Urinalysis shows pH 5.0, specific gravity 1.010, 1+ protein, 4+ blood, 20–50 RBC/hpf, no RBC
casts, and 2–5 WBC/hpf. Serum studies show normal BUN, creatinine, C3 and C4 levels.
Antinuclear antibody test is negative. Which of the following is the most likely cause of
hematuria in this patient?
Incorrect Answer ImageA.Acute interstitial nephritis
Correct Answer ImageB.IgA nephropathy
Incorrect Answer ImageC.Lupus nephritis

Incorrect Answer ImageD.Membranoproliferative glomerulonephritis

Incorrect Answer ImageE.Poststreptococcal glomerulonephritis


A 35-year-old woman comes to the physician because of an itchy patch of skin near her right
knee that developed after being bitten by mosquitoes in this area 6 months ago. The mosquito
bites resolved, but the area continued to be pruritic. She reports scratching the area periodically.
Physical examination shows a well defined, 6 cm diameter patch of skin that is dry, scaling,
hyperpigmented, and thickened. There is a ring of discrete brown papules at the periphery of the
lesion. Which of the following is the most likely diagnosis?
Correct Answer ImageA.Lichen simplex chronicus
Incorrect Answer ImageB.Pompholyx
Incorrect Answer ImageC.Psoriasis
Incorrect Answer ImageD.Seborrheic dermatitis
Incorrect Answer ImageE.Stasis dermatitis

A 65-year-old man presents to his physician because of back pain for the past 4 months. The pain
radiates to his buttocks and to the back of his legs bilaterally. The pain is worse with walking or
standing; it is relieved by sitting or walking uphill. He denies nocturnal pain and changes in
intensity of the pain. His past medical history is significant for an 8-year history of hypertension,
for which he takes amlodipine and lisinopril. He underwent surgical repair for an inguinal hernia
3 years ago. His family history is significant for hypertension in his brother and type 1 diabetes
mellitus in his mother. He denies cigarette and illicit drug use. He drinks alcohol on social
occasions. His vital signs are as follows: temperature is 37°C (98.6°F), pulse is 80/min,
respirations are 12/min and blood pressure is 142/84 mm Hg. On physical examination, the back
pain is elicited by extension of the spine. Neurological examination shows loss of balance while
standing with eyes closed. Which of the following is the most likely diagnosis?
Incorrect Answer ImageA.Cauda equina syndrome
Incorrect Answer ImageB.Lumbar disk herniation
Correct Answer ImageC.Lumbar spinal stenosis
Incorrect Answer ImageD.Metastatic disease
Incorrect Answer ImageE.Vertebral compression fracture

A 39-year-old woman completed her last cycle of postoperative adjuvant chemotherapy for
breast cancer 6 months ago. She now comes to the physician because of constant back pain for
about 3 weeks. She is tender to palpation over two well-circumscribed areas in the thoracic and
lumbar spine. Which of the following is the most appropriate next step in management?
Incorrect Answer ImageA.Back brace
Incorrect Answer ImageB.CT of the spine
Correct Answer ImageC.Radionuclide bone scan
Incorrect Answer ImageD.Radiotherapy of the tender spots
Incorrect Answer ImageE.X-ray films of the affected areas

A 33-year-old woman comes to the clinic for a medical evaluation required for her new job. She
reports that she has been healthy all of her life. In the last 3 weeks, however, she has felt
palpitations that are accompanied by weakness and mild dizziness on occasion. During the
physical examination, she has an episode of palpitations. Her pulse is 178/min, respirations are
16/min, and blood pressure is 115/70 mm Hg. An ECG taken that moment shows short PR
intervals with wide QRS complexes and a slurred upstroke in the initial deflection. Which of the
following is the most appropriate initial management for this patient's current condition? 
Incorrect Answer ImageA.Adenosine
Incorrect Answer ImageB.Amiodarone
Incorrect Answer ImageC.Metoprolol
Correct Answer ImageD.Procainamide
Incorrect Answer ImageE.Radiofrequency catheter ablation
Incorrect Answer ImageF.Synchronized cardioversion

A 65-year-old woman comes to the physician for a routine maintenance examination. She had a
fever with muscle aches and a nonproductive cough that resolved 1 week ago. Her medical
history is otherwise unremarkable. She takes no medications. She does not smoke cigarettes and
she drinks 1 or 2 glasses of wine at social occasions. Her temperature is 37.1°C (98.8°F), blood
pressure 124/82 mm Hg, and pulse is 76/min. The lungs are clear to auscultation. Cardiac
examination shows normal S1 and S2; no murmurs are heard. Which of the following medical
interventions best represents a primary preventive measure for this patient? 
Incorrect Answer ImageA.Isolation of disease contacts
Incorrect Answer ImageB.Mammography
Correct Answer ImageC.Routine immunization
Incorrect Answer ImageD.Screening for visual acuity
Incorrect Answer ImageE.Testing of stool for occult blood

A 33-year-old man comes to the physician because of a 6-month history of headache. He has had
excessive sweating and "skin oiliness." Examination shows prominent supraorbital ridges,
macroglossia, and large hands and feet. Serum insulin-like growth factor I (IGF-I) level is
increased. After 100 g of glucose is given, serum growth hormone is 4 ng/mL (normal ≤1
ng/mL). MRI of the head shows a 1.5-cm sellar mass with a slight suprasellar extension. Which
of the following is the most appropriate next step in management? 
Incorrect Answer ImageA.Cabergoline
Incorrect Answer ImageB.Octreotide
Incorrect Answer ImageC.Pegvisomant
Incorrect Answer ImageD.Radiation therapy
Correct Answer ImageE.Transsphenoidal surgery

A 74-year-old woman presents to the emergency department because of 48 hours of fever and
worsening shortness of breath. She has a chronic productive cough that produces copious dark
sputum. Her past medical history is significant for chronic obstructive pulmonary disease
(COPD) for the past 25 years, along with dyslipidemia, hypertension, and congestive heart
failure. Her current medications include simvastatin, lisinopril, metoprolol, and an ipratropium
inhaler. Her temperature is 38.6ºC (101.4ºF), pulse is 80/min, respirations are 20/min, and blood
pressure is 135/85 mm Hg. On physical examination, she has rhonchi and increased fremitus in
the posterior mid-lung field. A Gram stain of the sputum reveals many epithelial cells and
multiple gram-positive and gram-negative organisms, although no neutrophils are seen. Which of
the following is the most likely organism causing the symptoms? 
Incorrect Answer ImageA.Escherichia coli 
Correct Answer ImageB.Haemophilus influenzae 
Incorrect Answer ImageC.Klebsiella pneumoniae 
Incorrect Answer ImageD.Mycobacterium tuberculosis 
Incorrect Answer ImageE.Mycoplasma pneumoniae

A 32-year-old woman comes to the physician for a routine physical examination. She denies
chest pain, shortness of breath, or any other significant symptoms. She has no history of any
major medical illnesses and takes no medications. Her temperature is 36.4ºC (97.5ºF), pulse is
72/min, respirations are 16/min, and blood pressure is 120/70 mm Hg. Her weight is 65 kg (143
lb), height is 170 cm (5 ft 8 in), and BMI is 22.5 kg/m2. Physical examination shows no
abnormalities. Laboratory studies are within normal limits. An ECG shows occasional premature
atrial contractions. Echocardiogram shows normal left ventricle ejection fraction and no valvular
abnormalities. Which of the following is the most appropriate next step in patient care?
Incorrect Answer ImageA.Digoxin
Incorrect Answer ImageB.Diltiazem
Incorrect Answer ImageC.Dofetilide
Incorrect Answer ImageD.Flecainide
Incorrect Answer ImageE.Metoprolol
Correct Answer ImageF.Reassurance

A 54-year-old lawyer presents to the emergency department with intermittent, chronic diarrhea
for the past several years that is unresponsive to medication. He has associated fatigue, shortness
of breath and unintentional 1-kg (2.2-lb) weight loss. Symptoms have been gradually worsening
over the last 2 months. There is no past medical history. He is a nonsmoker and reports drinking
a glass of wine every night. He takes no medications. On physical examination, his pulse is
108/min and his blood pressure is 115/75 mm Hg. Conjunctival pallor is identified with
neurologic examination showing loss of vibratory and position sense.
Laboratory studies: 
Hematocrit  26%
WBC  3,700/mm3
Platelets  110,000/mm3
MCV  116 µm3
Reticulocyte count  0.5%
Serum lactate dehydrogenase  650 U/L
Serum total bilirubin  2 mg/dL
Which one of the following is the most likely cause of this patient's macrocytic anemia? 
Incorrect Answer ImageA.Alcoholic liver disease
Incorrect Answer ImageB.Folate deficiency
Incorrect Answer ImageC.Hemolytic anemia
Incorrect Answer ImageD.Myelodysplastic syndrome
Correct Answer ImageE.Vitamin B12 deficiency

A 72-year-old man comes to the emergency department because of increased nausea and itching
for the past few days. He also has decreased oral intake. He denies fever, chills, chest pain, or
shortness of breath. He was diagnosed recently with non-Hodgkin lymphoma and he completed
his first cycle of chemotherapy 10 days ago. As part of his workup for lymphoma, he underwent
several CT scans with contrast, which showed abdominal lymphadenopathy. His vital signs are
stable. Physical examination is unremarkable. Laboratory studies show:
Na+ 137 mEq/L
K +
4.5 mEq/L
Cl– 100 mEq/L
Ca 2+
5.0 mg/dL
HCO3– 24 mEq/L
BUN 56 mg/dL
Creatinine 6.0 mg/dL
Uric acid 14/mg/dL
Phosphate 8.0 mg/dL
Which of the following is the most likely cause of these laboratory findings? 
Incorrect Answer ImageA.Dehydration
Incorrect Answer ImageB.Interstitial nephritis
Incorrect Answer ImageC.Renal infarct
Correct Answer ImageD.Tumor lysis syndrome
Incorrect Answer ImageE.Ureteral obstruction

A 46-year-old woman comes to the emergency department because of right upper quadrant pain
and fever. She has had these symptoms for the past several days and they have become acutely
worse in the past several hours. Her past medical history is significant for gallstones. Her
temperature is 38.6°C (101.4°F), pulse is 105/min and blood pressure is 100/60 mm Hg. She has
mild scleral icterus. Her lungs are clear to auscultation, and there are no heart murmurs.
Palpation of the right upper quadrant shows diffuse tenderness. There is no ascites. Peripheral
examination is normal. Results of liver function tests are:
AST 130 IU/L
ALT 125 IU/L
Alkaline phosphatase 250 IU/L
Total bilirubin 3.1 mg/dL
Ultrasonography shows dilated intra- and extrahepatic ducts and a stone within the distal
common bile duct. Which of the following is the most appropriate therapy for this patient?
Incorrect Answer ImageA.Emergent sphincterotomy
Incorrect Answer ImageB.Intravenous hydration and ceftriaxone
Incorrect Answer ImageC.Intravenous hydration, ceftriaxone, and metronidazole
Incorrect Answer ImageD.Intravenous hydration, ceftriaxone, and sphincterotomy within the next 6
hours
Correct Answer ImageE.Intravenous hydration, ceftriaxone, metronidazole, and sphincterotomy
once stabilized
A 44-year-old man comes to the emergency department 4 hours after the onset of left lower back
pain that began shortly after he lifted a heavy box out of his car and carried it into his home. He
has had no leg weakness, bladder or bowel incontinence. He has a history of hypertension that is
controlled with hydrochlorothiazide. His temperature is 36.9°C (98.4°F), blood pressure is
138/85 mm Hg, and pulse is 85/min. The pain is reproduced when the patient leans to the right.
There is no tenderness to palpation and no deformities in the lower back or pelvis. The straight
leg raising test does not produce symptoms. Neurologic examination shows no abnormalities.
Which of the following is the most appropriate next step in management? 
Incorrect Answer ImageA.CT scan of the spine
Incorrect Answer ImageB.MRI scan of the spine
Correct Answer ImageC.Physical therapy
Incorrect Answer ImageD.Radiograph of the spine
Incorrect Answer ImageE.Ultrasound of the paraspinal region

A 75-year-old man is brought to the physician by his daughter because of progressive


forgetfulness and disorientation for 2 years. Last week, on two occasions, he left the house and
the neighbors found him in the street. He has been forgetful and has gotten confused when
dressing himself. The family has started to take care of his bills because of his confusion. His
medical history is unremarkable, and he takes no medications. Laboratory studies including
thyroid function tests, serum glucose levels, and metabolic panel are all within normal limits.
Which of the following neurotransmitter abnormalities is most likely to be seen in this patient's
condition? 
Correct Answer ImageA.Decreased acetylcholine
Incorrect Answer ImageB.Decreased dopamine
Incorrect Answer ImageC.Decreased norepinephrine
Incorrect Answer ImageD.Decreased serotonin
Incorrect Answer ImageE.Increased acetylcholine
Incorrect Answer ImageF.Increased dopamine
Incorrect Answer ImageG.Increased norepinephrine
Incorrect Answer ImageH.Increased serotonin

A 56-year-old woman with a history of atrial fibrillation is brought to the emergency department
via ambulance because of chest pain. One hour previously she had been enjoying the opera when
she felt a sudden, dull pain in her chest and left arm. She initially thought that she was emotional
as a result of the performance, but the pain became progressively more severe to the point that
she had to leave the concert. Current medications include digoxin. Her vital signs are
temperature 37.7ºC (99.9ºF), pulse 132/min, and blood pressure 105/70 mm Hg. On physical
examination, she appears panicked and in great discomfort. She is breathing through a face mask
set at 100% O2 and appears pale and drenched in sweat. There is no jugular venous distension.
Her heart sounds are muffled with no murmurs or rubs. Cardiac enzymes are pending. Initial
ECG demonstrates deviation of the J-point and nonspecific ST changes. What is the next best
step in management?
Incorrect Answer ImageA.Administer alteplase
Incorrect Answer ImageB.Coronary artery bypass grafting
Correct Answer ImageC.Echocardiogram
Incorrect Answer ImageD.Percutaneous coronary intervention
Incorrect Answer ImageE.Repeat electrocardiogram

A 24-year-old Caucasian woman comes to the physician because of a 6-month history of crampy
right lower abdominal pain that is worse with eating. She has had 4 loose bowel movements
daily and aching in her knees and ankles. She has had a 7 kg (15 lb) weight loss over the past 6
months. She denies fevers, chills, or night sweats. Physical examination shows 2 oral ulcers on
the inner lower lip that are covered by a gray exudate and surrounded by an erythematous halo.
Abdominal examination shows tenderness in the right lower quadrant; there are no masses or
hepatosplenomegaly. Rectal examination shows brown stool that is guaiac-positive. Which of the
following is the most appropriate diagnostic test for this patient's condition?
Incorrect Answer ImageA.Abdominal CT scan
Incorrect Answer ImageB.Abdominal ultrasound
Incorrect Answer ImageC.Blood test for perinuclear antineutrophil antibodies (pANCA) and Anti-
Saccharomyces Cerevisiae antibody (ASCA)
Correct Answer ImageD.Colonoscopy
Incorrect Answer ImageE.Sigmoidoscopy
Incorrect Answer ImageF.Upper gastrointestinal series

A 40-year-old woman comes to the physician because of a 1-year history of progressive


shortness of breath and a nonproductive cough. She has had no fevers or weight loss. Her
medical history is unremarkable. She has never smoked cigarettes. Her blood pressure is 124/84
mm Hg, pulse is 76/min, and respirations are 16/min. Crackles are heard in both mid lung zones
on auscultation. The remainder of the examination shows no abnormalities. Her serum calcium
level is 11.5 mg/dL. Angiotensin-converting enzyme (ACE) level is 100 ug/L (normal, <40
ug/L). A chest x-ray shows bilateral hilar fullness. A transbronchial lung biopsy shows multiple
discrete granulomas without necrosis. Which of the following is the most likely explanation for
this patient's hypercalcemia?
Incorrect Answer ImageA.Altered calcium sensing and a blunted parathyroid hormone (PTH)
feedback loop
Incorrect Answer ImageB.Exogenous production of PTH
Incorrect Answer ImageC.Granulomatous secretion of PTH-related hormone
Correct Answer ImageD.Increased production of 1,25-dihydroxycholecalciferol
Incorrect Answer ImageE.Lytic bone lesions and increased IL-6 production

A 29-year-old man comes to the physician because of what he describes as "funky toenails." He
is a professional surfer and states that he occasionally gets athlete's foot from the constant
moisture, which he treats with over-the-counter clotrimazole cream. Aside from his feet, he
states that he is in good health. He has had no major illnesses and his review of symptoms is
unremarkable. Both of his parents are still alive and, aside from hypertension, no diseases run in
his family. His temperature is 37.0°C (98.6°F), pulse is 70/min, respirations are 20/min, and
blood pressure is 129/73 mm Hg. Foot examination shows 2+ dorsalis pedis and posterior tibial
pulses. There is marked skin changes. Scaling, maceration, erythema, and superficial skin
erosions are present in the web spaces between the toes. Diffuse erythema, a fine white scaling of
the plantar and lateral foot, and areas of hyperkeratosis are also present. The nails appear
thickened and mildly dystrophic with subungual hyperkeratosis and areas of onycholysis. You
prescribe a combination clotrimazole-betamethasone cream. Which of the following is an
additional treatment that will help this patient? 
Incorrect Answer ImageA.Diabetes screening and treatment
Incorrect Answer ImageB.Fluconazole
Incorrect Answer ImageC.Griseofulvin
Incorrect Answer ImageD.Ketoconazole cream
Correct Answer ImageE.Terbinafine

A 58-year-old man with cirrhosis due to hepatitis C comes to his physician with 5 days of
worsening fatigue. He has been admitted three times in the past 4 months for variceal bleeding
and has chronic ascites that has been refractory to high-dose furosemide and spironolactone. He
notes that over the past 48 hours he has had a declining urinary output. His blood pressure is
110/70 mm Hg and pulse is 100/minute. On physical examination, he is jaundiced with scleral
icterus. Cardiopulmonary examinations are normal. He has tense ascites and a liver span of 7 cm
in the midclavicular line. His neurologic and mental status examinations are normal. Laboratory
studies show:
Serum
Sodium 131 mEq/L
Potassium 4.7 mEq/L
Chloride 100 mEq/L
Bicarbonate 22 mEq/L
Creatinine 3.1 mg/dL
Urea nitrogen 34 mg/dL
Albumin 2.1 mg/dL
Blood
WBC 4,600/ mm3
Hgb 9.4 g/dL
Hct 29%
A urinalysis shows no protein, blood, cells, or casts. The urine sodium concentration is <10
mEq/L. He is given 2 liters of 0.9% normal saline. His ascites worsens, and there is no
significant change in the laboratory values. Paracentesis makes him more comfortable. Which of
the following is the best next step in management?
Incorrect Answer ImageA.Blood transfusion
Incorrect Answer ImageB.Hemodialysis
Incorrect Answer ImageC.Kidney transplantation evaluation
Correct Answer ImageD.Liver transplantation evaluation
Incorrect Answer ImageE.Mesocaval shunt
A 71-year-old man comes to the physician because of a 2 day history of a painful rash. He has
had no fevers, chills, or weight loss. His past medical history is unremarkable. He lives with his
cousin who has AIDS. His temperature is 37.2ºC (99.0ºF). Physical examination is shown. There
is no lymphadenopathy. Laboratory studies show:
Hemoglobin  14 g/dL 
Mean corpuscular volume  90 fL 
Leukocyte count  11,500/mm3
Platelet count  350,000/mm3
Which of the following is the most appropriate pharmacotherapy? 
Incorrect Answer ImageA.Amitriptyline and gabapentin
Incorrect Answer ImageB.Dexamethasone
Incorrect Answer ImageC.Emtricitabine, tenofovir, and efavirenz
Incorrect Answer ImageD.Ganciclovir
Correct Answer ImageE.Valacyclovir

A 17-year-old boy is brought to the emergency department immediately after his parents
witnessed a seizure. His parents report that he has had bizarre behavior and has been talking
about strange smells. His past medical history is unremarkable, and he takes no medications. He
does not use illicit drugs or alcohol. His temperature is 38.2ºC (100.7ºF), blood pressure 120/80
mm Hg, pulse 95/min, and respirations 17/min. He is confused but follows commands.
Neurologic examination shows no focal abnormalities. CT scan of the brain shows hemorrhage
in the frontotemporal lobes. Which of the following is the most appropriate next step in
management? 
Correct Answer ImageA.Administration of IV acyclovir
Incorrect Answer ImageB.Lumbar puncture
Incorrect Answer ImageC.MRI scan of the brain
Incorrect Answer ImageD.PCR of CSF for the suspected microorganism
Incorrect Answer ImageE.Serologic diagnosis for the suspected microorganism
Incorrect Answer ImageF.Supportive management

A 57-year-old man with a history of hypertension and hyperlipidemia is brought to the


emergency department because of crushing substernal chest pain, diaphoresis, and nausea for the
past 3 hours. He denies palpitations or shortness of breath and he has had no previous episodes of
angina or myocardial infarction. His blood pressure is 82/60 mm Hg, temperature is 37.0°C
(98.6°F), pulse is 103/min, and respirations are 20/min. Physical examination shows jugular
venous distention, no murmurs, and clear lungs. An ECG is shown. A right-sided ECG is also
performed and shows ST elevation in V4R. Which of the following treatments is most
appropriate for this patient?
Incorrect Answer ImageA.Dopamine
Incorrect Answer ImageB.Furosemide
Correct Answer ImageC.IV fluids
Incorrect Answer ImageD.Metoprolol
Incorrect Answer ImageE.Nitroglycerin
A 42-year-old man with a history of lung cancer comes to the physician because of a 1-day
history of progressive nausea, vomiting, and lethargy. He takes opioid analgesics as needed for
pain, but has refused chemotherapy. He appears somnolent. His temperature is 37°C (98.6°F),
pulse is 80/min, respirations are 16/min, and blood pressure is 132/80 mm Hg. The lungs are
clear to auscultation. Cardiac examination shows a normal S1 and S2; no murmurs are heard.
Pulse oximetry shows 98% oxygen saturation on room air. Serum studies show: 
Calcium 13.4 mg/dL
Creatinine 1 mg/dL
Urea nitrogen (BUN) 18 mg/dL
Which of the following is the most appropriate next step in management?
Incorrect Answer ImageA.Calcitonin
Incorrect Answer ImageB.Etidronate
Incorrect Answer ImageC.Hydrochlorothiazide
Correct Answer ImageD.Intravenous saline
Incorrect Answer ImageE.Prednisone

A 60-year-old man comes to the physician for a routine health maintenance examination. He has
no significant medical history. Prior to his visit, he had a cholesterol panel tested, as advised by
his physician. The test shows an elevated LDL-cholesterol of 160 mg/dL and low HDL-
cholesterol of 38 mg/dL. Given these results, the patient is started on a heart-healthy diet and
exercise plan. Six months later, there has been mild improvement in the patient's weight and a
normalization of his HDL-cholesterol level, although his LDL-cholesterol still remains elevated
at 152 mg/dL. He has a normal blood pressure and has never smoked. Which of the following is
the most appropriate next step at this time?
Correct Answer ImageA.Atorvastatin
Incorrect Answer ImageB.Cholestyramine
Incorrect Answer ImageC.Continue diet and exercise
Incorrect Answer ImageD.Niacin
Incorrect Answer ImageE.Recheck lipid profile in 1 year

An 82-year-old woman is brought to the emergency department from the local nursing home
because of a five-day history of severe constipation and a one-day history of lower abdominal
discomfort and distention. She has a history of hypertension treated with hydrochlorothiazide
and bilateral knee osteoarthritis treated with acetaminophen and codeine. She has been bedridden
secondary to severe osteoarthritis. Her temperature is 37.1°C (98.8ºF) and blood pressure is
148/90 mm Hg. Abdominal examination shows middle and lower abdominal distention with
mild tenderness. Bowel sounds are normal. Rectal examination shows hard stool in the vault.
Which of the following is the most appropriate next step in management?
Incorrect Answer ImageA.Barium enema
Incorrect Answer ImageB.Colonoscopy
Incorrect Answer ImageC.Flexible sigmoidoscopy
Correct Answer ImageD.Manual disimpaction
Incorrect Answer ImageE.Nasogastric tube
A 19-year-old college student comes to student health services because of headaches for 4
months. Each headache lasts a full day, and she has had four to five headaches per month.
During these events, she has seen black spots and zigzag patterns in her left field of vision and
has felt numb in her left hand. The visual symptoms have been followed by throbbing headaches
on the left side of the head. Over-the-counter medications have provided minimal relief, and she
has needed to lie in a cool, quiet, dark room for any significant relief. Her temperature is 36.9°C
(98.4°F), blood pressure is 114/72 mm Hg, and pulse is 76/min. Examination shows no
abnormalities. Which of the following is the most appropriate pharmacotherapy in this patient?
Incorrect Answer ImageA.Ergotamine
Incorrect Answer ImageB.Ibuprofen
Incorrect Answer ImageC.Metoclopramide
Correct Answer ImageD.Propranolol
Incorrect Answer ImageE.Sumatriptan

A 59-year-old man who lives in the southeastern United States is taken to the emergency
department by his wife in the middle of the summer because of the sudden onset of severe right
flank pain accompanied by nausea and vomiting. He tells the physician that he has been jogging
outside recently in hot weather and has been on a low-carbohydrate, high-protein diet to lose
weight. By the time he is seen by the physician, the pain has now moved downward toward the
right testis, then abates. The physical examination is normal. The urinalysis shows numerous
RBCs. Serum studies are normal, including a normal calcium and uric acid level. A CT scan
shows multiple stones in the right kidney. He is discharged with the recommendation to increase
his hydration. However, in the next two months he develops similar symptoms three times, each
time resolving spontaneously. Which of the following is the best next step in management?
Incorrect Answer ImageA.Extracorporeal shock wave lithotripsy
Incorrect Answer ImageB.Parathyroid hormone assay

Correct Answer ImageC.Strain urine for stone analysis


Incorrect Answer ImageD.Treatment with allopurinol
Incorrect Answer ImageE.Treatment with thiazide diuretics

A 36-year-old Caucasian woman with a history of hypertension and diet-controlled diabetes


mellitus comes to the emergency department with abdominal pain of 2 days duration. Her pain is
mainly in the epigastric region and radiates to the back. She has some nausea but no vomiting.
She denies any diarrhea. During the past 2 years, she has had 3 episodes of dull upper quadrant
pain that exacerbated with fatty food. The episodes resolved spontaneously. She is on lisinopril
for hypertension. She drinks 1 to 2 beers every weekend. Physical examination reveals a young
female in moderate distress. Her temperature is 37.2ºC (99ºF), pulse is 110/min, respirations are
16/min, and blood pressure is 110/70 mm Hg. Lungs are clear to auscultation. Cardiovascular
examination reveals no murmur, rub, or gallop. Abdominal examination shows tenderness in the
epigastric region with no rebound, guarding, or rigidity. Laboratory studies show:
Hemoglobin 12.6 grams/dL
WBC 13000/ mm3, 18% bands 
Platelets 370,000/ mm3
Amylase 1900 U/L
Lipase 1600 U/L
Calcium 8.2 mg/dL
Triglycerides 800 mg/dL (Normal 35–160 mg/dL)
Aspartate aminotransferase (AST) 80 U/L
Alanine aminotransferase (ALT) 150 U/L
Which of the following is the most likely cause of her pancreatitis? 
Incorrect Answer ImageA.Alcohol abuse
Incorrect Answer ImageB.Ampullary stricture
Incorrect Answer ImageC.Drug-induced pancreatitis
Correct Answer ImageD.Gallstone pancreatitis
Incorrect Answer ImageE.Hypertriglyceridemia-induced pancreatitis

A 68-year-old man is brought to the emergency department immediately following a motor


vehicle accident. His past medical history is unremarkable and he takes no medications. His
pulse is 85/min and his blood pressure is 133/85 mm Hg. Oxygen saturation is 97% in room air.
He is alert and oriented. Examination shows a bruise over the right temple with tenderness to
palpation over the area. The remainder of the examination shows no abnormalities. Oxygen
saturation is 97% on room air. CT scan of the head shows soft-tissue swelling in the right
temporal region with no underlying skull fracture or parenchymal injury. However, there are two
lytic lesions in the skull. Serum protein electrophoresis shows an IgA gamma globulin spike.
Which of the following findings is diagnostic of this patient's condition? 
Incorrect Answer ImageA.Amyloidosis and increased urine Bence-Jones proteins
Incorrect Answer ImageB.Hypercalcemia and hyperviscosity syndrome
Incorrect Answer ImageC.Hyperviscosity syndrome and rouleaux formation on peripheral smear
Correct Answer ImageD.Increased serum IgA and clonal plasma cells from bone marrow biopsy
Incorrect Answer ImageE.Rouleaux formation of red blood cells and increased serum beta-2
microglobulin

A 58-year-old woman comes to the emergency department because of recurring, persistent


palpitations for the past month. The current episode seemed to last longer, and has been going on
for 1 hour. She denies chest pain, although she does feel anxious and slightly short of breath. She
has no history of any major illness and takes no medications. Her pulse is 110/min and blood
pressure is 110/71 mm Hg. Physical examination shows no abnormalities except an irregularly
irregular heart rhythm. ECG shows atrial fibrillation. Laboratory studies show: 
White blood cells  6,700/mm3
Hct  43% 
Platelets  225,000/mm3
Sodium  138 mEq/L
Potassium  4.2 mEq/L
Chloride  105 mEq/L
Bicarbonate  24 mEq/L
Urea nitrogen  14 mg/dL
Creatinine  0.7 mg/dL
Creatinine kinase  35 U/L
TSH  0.25 mU/L (0.5–5.0 mU/L) 
Free T4  6 ng/dL (0.7–1.9 ng/dL) 
Treatment with IV metoprolol reduces her pulse to 80/min, though she remains in atrial
fibrillation. She is transferred to telemetry and treatment with oral propranolol is started. Which
of the following is the most appropriate next step in management? 
Incorrect Answer ImageA.Amiodarone
Correct Answer ImageB.Methimazole
Incorrect Answer ImageC.Radioactive iodine ablation
Incorrect Answer ImageD.Subtotal thyroidectomy
Incorrect Answer ImageE.Synchronized cardioversion

A 50-year-old man comes to the physician because of a mass on the back of his left hand that
developed rapidly over the last 3 weeks. Examination shows a round, firm, flesh-colored, 1 cm
nodule with well-demarcated, sharply rising edges and a large central crater on the dorsal surface
of the left hand. A keratotic plug is present in the central crater. Which of the following is the
most likely diagnosis? 
Correct Answer ImageA.Keratoacanthoma
Incorrect Answer ImageB.Lipoma
Incorrect Answer ImageC.Malignant melanoma
Incorrect Answer ImageD.Pyogenic granuloma
Incorrect Answer ImageE.Seborrheic keratosis

A 34-year-old woman comes to the physician because of chronic constipation that has been
getting worse during the past 2 years. She has one bowel movement every week and has had
lower abdominal fullness and distension. She often feels the urge to have a bowel movement but
feels like it “gets stuck”, and she sometimes has to put a finger in her vagina to help pass stool.
She has occasional urinary incontinence. She has no blood in her stool, diarrhea, nausea,
vomiting, fever, fatigue, or weight loss. She delivered 5 children vaginally and had gestational
diabetes 2 years ago. Her sister has irritable bowel syndrome and her maternal uncle died of
colon cancer at age 75 years. The abdomen is nontender and bowel sounds are normal. Her
hemoglobin is 13.8 g/dL. Which of the following is the most likely cause of this patient's
constipation?
Incorrect Answer ImageA.Colon cancer
Incorrect Answer ImageB.Colonic inertia
Incorrect Answer ImageC.Diverticular stricture
Incorrect Answer ImageD.Irritable bowel syndrome
Correct Answer ImageE.Rectocele

A 57-year-old diabetic man with a past medical history of gastroesophageal reflux disease comes
to the emergency department because of pain. Four hours ago he had been sitting down eating
his daily meal with his family when he suddenly felt severe, epigastric pain with associated
nausea. His temperature is 37.8ºC (100ºF), pulse is 124/min, and blood pressure is 159/95 mm
Hg. The patient is sweating excessively. There is no chest tenderness and breath sounds are clear
bilaterally. There are muffled heart sounds with a regular rate. ECG shows ST-segment
elevations in leads II, III, and aVF. Twenty-four hours after percutaneous coronary intervention,
the patient's blood pressure drops to 98/62 mm Hg and pulse to 44/min. What is the most likely
diagnosis?
Incorrect Answer ImageA.Cardiac tamponade
Incorrect Answer ImageB.Congestive heart failure
Incorrect Answer ImageC.Free wall rupture
Incorrect Answer ImageD.Papillary muscle rupture
Incorrect Answer ImageE.Right ventricular infarct
Correct Answer ImageF.Third-degree atrioventricular block
Incorrect Answer ImageG.Ventricular septal rupture

A 40-year-old woman with a 9-year history of scleroderma comes to the physician because of
shortness of breath and a dry cough for 4 months, weakness, dyspnea with minimal exertion,
arthralgias, and difficulty with swallowing. Her temperature is 36.8ºC (98.2ºF), blood pressure is
135/75 mm Hg, pulse is 112/min, and respirations are 26/min. Physical examination shows
bilateral basilar crackles on auscultation of the chest. Cardiac examination shows a normal S1
and S2 without murmurs. A chest radiograph shows reticulonodular interstitial markings at the
lung bases. Pulmonary function tests show a DLco of 55%. Which of the following is the most
appropriate next step in diagnosis? 
Incorrect Answer ImageA.Arterial blood gases
Incorrect Answer ImageB.Echocardiogram
Incorrect Answer ImageC.Electrocardiogram
Correct Answer ImageD.High-resolution computed tomography
Incorrect Answer ImageE.Pulmonary angiogram

A 31-year-old man is brought to the emergency department by his brother because of altered
mental status, nausea, and vomiting. The brother reports that the patient recently lost his job and
has been upset lately. His past medical history is significant for type 1 diabetes mellitus, for
which he takes insulin. His temperature is 37.4°C (99.3°F), pulse is 118/min, respirations are
34/min, and blood pressure is 92/56 mm Hg. His oxygen saturation is 98% on room air. On
physical examination, he is somnolent. General examination shows dry oral mucosa and
decreased skin turgor. Abdominal examination shows generalized tenderness to palpation.
Laboratory studies show:
Na+ 134 mEq/L 
Cl- 103 mEq/L 
K + 
4.0 mEq/L 
HCO3- 8 mEq/L
Glucose 855 mg/dL 
pH 7.25
PaO2 98 mm Hg
PaCO2 20 mm Hg
Which of the following is the most appropriate next step in management? 
Incorrect Answer ImageA.Administer subcutaneous insulin
Incorrect Answer ImageB.IV bicarbonate infusion with dextrose
Correct Answer ImageC.IV fluids
Incorrect Answer ImageD.IV glucagon infusion
Incorrect Answer ImageE.IV insulin infusion with dextrose

A 55-year-old man complains of shortness of breath, wheezing, and worsening dyspnea on


exertion. He reports a past history of smoking >3 packs of cigarettes a day for the past 40 years.
He has never been intubated but has been hospitalized in the past for treatment with
bronchodilators. Pulmonary examination reveals decreased breath sounds bilaterally. Chest
radiograph shows flattening of the diaphragm and an increased anterior-posterior chest diameter.
He undergoes pulmonary function testing. Which of the following patterns will be seen on his
pulmonary function tests?
FVC FEV1/FVC Ratio TLC RV DLCO

B. ↓ ↓  ↑  ↓  N 

C. ↓  N or ↓ ↑  ↑  N 

D. ↑ ↑ ↓ ↓ ↓

E. ↓ ↓ ↓ ↓ ↓

A. ↓ ↓  ↑  ↑  ↓ 
Correct Answer ImageA.A
Incorrect Answer ImageB.B
Incorrect Answer ImageC.C
Incorrect Answer ImageD.D
Incorrect Answer ImageE.E

A 65-year-old man comes to the physician because of progressive weakness and fatigue for 6
months. He has had no weight loss. His medical history is unremarkable and he takes no
medications. His temperature is 37.0°C (98.6°F), blood pressure is 120/80 mm Hg, pulse is
70/min, and respirations are 14/min. The patient speaks with a soft and monotonous tone of
voice. He walks with a rigid posture and limited arm swing. There is a fine tremor of the fingers
that manifests at rest and resolves with movement. There is increased muscle tone when his arms
are passively flexed. Sensation and muscle strength are normal. Laboratory studies are within
normal limits. Which of the following is the most appropriate pharmacotherapy for this patient? 
Incorrect Answer ImageA.Cholinesterase inhibitor
Incorrect Answer ImageB.D1 and D4 dopamine receptors antagonist
Incorrect Answer ImageC.D2 dopamine receptors antagonist
Correct Answer ImageD.Dopamine precursor
Incorrect Answer ImageE.Serotonin 5-HT1 receptors agonist

A 36-year-old woman comes to the physician because of gradually worsening exertional dyspnea
and generalized weakness. She also has had chest discomfort and had a syncopal episode
recently. She has no history of any major medical illnesses and takes no medications. Her
temperature is 36.8ºC (98.2ºF), blood pressure is 130/70 mm Hg, pulse is 110/min, and
respirations are 22/min. Physical examination shows an increased pulmonic component of S2, as
well as wide splitting of S2. Lungs are clear to auscultation. There is hepatomegaly with palpable
pulsations of the liver and abnormal abdominal-jugular reflux. Lower extremities have 2+ pitting
edema. A chest x-ray film shows enlarged central pulmonary arteries and right heart dilatation.
An ECG shows right axis deviation and right ventricular hypertrophy. Pulmonary function
testing shows DLCO of 54% without significant restrictive ventilatory abnormalities. Which of
the following is the most likely diagnosis? 
Incorrect Answer ImageA.Dilated cardiomyopathy
Incorrect Answer ImageB.Hypertrophic cardiomyopathy
Incorrect Answer ImageC.Interstitial lung disease
Incorrect Answer ImageD.Obstructive lung disease
Correct Answer ImageE.Pulmonary hypertension

A 36-year-old man is brought to the emergency department after his wife found him
unresponsive on the sofa. She reports that there was an empty bottle of acetaminophen next to
him. Serum levels of acetaminophen are found to be in the toxic range. His temperature is 37.0ºC
(98.6ºF), blood pressure is 135/80 mm Hg, pulse is 100/min, and respirations are 12/min and
labored. He is intubated and admitted to the intensive care unit. Forty-eight hours later his
clinical status is much improved except for the markedly elevated liver enzymes and increasing
prothrombin time. Depletion of which metabolic substance is most likely causing these
findings? 
Incorrect Answer ImageA.Alcohol dehydrogenase
Incorrect Answer ImageB.Catalase
Correct Answer ImageC.Glutathione
Incorrect Answer ImageD.Glycogen
Incorrect Answer ImageE.Vitamin K

A 35-year-old man comes to the physician because of a 2-week history of daily headaches that
begin in the morning and last for the whole day. He says that the pain feels like a "squeezing
tightness" around his head and is particularly intense in the back of his neck. He has had no
fever, nausea, vomiting, seizures, or visual changes. He does not drink alcohol or smoke
cigarettes. He states that he eats a well-balanced diet and that he drinks 4 cups of coffee daily. He
works as an investment consultant in a brokerage firm. Physical examination shows no
abnormalities. Which of the following is the most appropriate next step in management? 
Incorrect Answer ImageA.CT scan of the head
Correct Answer ImageB.Ibuprofen trial
Incorrect Answer ImageC.Lumbar puncture
Incorrect Answer ImageD.Oxygen by nasal cannula
Incorrect Answer ImageE.Sumatriptan trial

A 48-year-old man comes to his physician for his yearly physical. The patient is asymptomatic
and has been in generally good health over the last year. He has no chronic medical illnesses and
takes no medications. He does not smoke or drink and he exercises four times per week. His
temperature is 37.2ºC (99.0ºF), pulse is 65/min, respirations are 14/min, and blood pressure is
129/78. His heart is in normal sinus rhythm and regular. Lungs are clear to auscultation. There is
no palpable hepatosplenomegaly. There is no urethral discharge. Rectal examination shows a
smooth, small prostate with guaiac-negative stool. There is no peripheral cyanosis, clubbing,
edema, or extremity weakness. Serum electrolytes, BUN, creatinine, glucose, and the complete
blood count are within normal limits. Urinalysis shows:
pH 6.6
Specific gravity 1.020
Protein 3+
Ketones Negative
Glucose Negative
Blood 1+ 
Leukocytes Negative 
Urine microscopy shows 20-25 RBC/hpf, with 20% dysmorphic red blood cells (acanthocytes);
0-1 WBC/hpf; and occasional red blood cell casts.
In a random (“spot”) urine sample, the total protein-creatinine ratio is 5.2 g/g, and the albumin-
creatinine ratio is 3.6 g/g.
Urine culture is negative for growth. A repeat urinalysis returns similar findings as the original.
Which of the following tests will most likely determine the cause of his hematuria?
Incorrect Answer ImageA.CT scan of the abdomen and pelvis
Incorrect Answer ImageB.Cystoscopy with bladder biopsy
Incorrect Answer ImageC.Intravenous pyelogram
Correct Answer ImageD.Percutaneous renal biopsy
Incorrect Answer ImageE.Renal ultrasound

A 55-year-old woman comes to the gastroenterologist for a screening colonoscopy. She has a
long history of constipation, but it has not changed recently. She has not noted any weight loss,
bloody stools, or fatigue. Her family history is unremarkable. Her temperature is 36.5ºC (97.8ºF),
pulse is 78/min, and blood pressure is 125/80 mm Hg. Her abdominal examination is benign.
Colonoscopy shows no polyps or masses, but there are several foci of deep pigmentation of the
mucosa (see image). One of these is sampled, and the pathology report states that there are
pigmented macrophages in the submucosa and lamina propria. Which of the following most
likely led to this appearance on colonoscopy? 
Correct Answer ImageA.Chronic laxative use
Incorrect Answer ImageB.Chronic lower gastrointestinal bleeding
Incorrect Answer ImageC.Chronic upper gastrointestinal bleeding
Incorrect Answer ImageD.Metastatic melanoma
Incorrect Answer ImageE.Underlying vascular malformations
A 24-year-old Asian woman comes to the clinic for a routine follow-up appointment. One month
ago, she required surgery for appendicitis. At that time, she was found to be anemic and was
referred to her primary care provider for further evaluation. She denies any fatigue, shortness of
breath, palpitations, heavy menstrual periods, or other symptoms. Past medical history is
unremarkable, though family history reveals multiple family members with anemia. Temperature
is 37.0ºC (98.6ºF), pulse is 78/min, respirations are 14/min, and blood pressure is 128/80 mm
Hg. Physical examination is unremarkable. Laboratory studies show hemoglobin 11.0 g/dL,
mean corpuscular volume (MCV) 60 µm3, and platelets 250,000/mm3. Peripheral blood smear
reveals a homogenous population of hypochromic and microcytic cells. Serum chemistries are
normal. Which of the following is the most likely cause of this patient's anemia? 
Correct Answer ImageA.Decreased globin chain synthesis
Incorrect Answer ImageB.Decreased heme synthesis
Incorrect Answer ImageC.Extravascular hemolysis by the spleen
Incorrect Answer ImageD.Iron blockade by macrophages
Incorrect Answer ImageE.Reduced stores of iron in the bone marrow

A 58-year-old man is brought to the emergency department by emergency medical services


because of chest pain and dyspnea. He states that he was at home with his family watching a
football game 40 minutes ago when he suddenly experienced "excruciating" pain that radiates to
his jaw. He was previously seen in an outpatient clinic for similar symptoms and was prescribed
glyceryl trinitrate sublingual spray, which alleviated his symptoms. However, during this
episode, his symptoms did not improve. He has a 45-pack-year smoking history. A review of his
medical records shows a previously normal resting electrocardiogram. His temperature is 37.5ºC
(99.5ºF), pulse is 126/min, and blood pressure is 100/60 mm Hg. On physical examination, the
patient appears pale and seems to be in discomfort with profuse sweating. His heart sounds are
distant and regular in rate and rhythm. There is no jugular venous distension. An
electrocardiogram shows QRS complexes of 0.16 seconds and notching of the R waves in leads
V5, V6, I, and AVL, most prominent in lead V6. There are no obvious ST-segment
elevations. Which of the following is the most appropriate next step in management?
Correct Answer ImageA.Aspirin
Incorrect Answer ImageB.Coronary angiography
Incorrect Answer ImageC.Coronary artery bypass grafting
Incorrect Answer ImageD.Echocardiography
Incorrect Answer ImageE.IV diazepam
Incorrect Answer ImageF.Percutaneous coronary intervention
Incorrect Answer ImageG.Pericardiocentesis

A 24-year-old female graduate student comes to the student health center before starting class.
Her medical history is unremarkable. Current medications include birth control pills and a
multivitamin. She traveled to Central America 2 years ago. She is sexually active and uses
condoms inconsistently. She does not use illicit drugs. Laboratory studies show:
HBsAg Nonreactive
Anti-HBs Positive
Anti-HAV (IgG) Positive
Anti-HAV (IgM) Nonreactive
Anti-HBc (IgG) Positive
Anti-HBc (IgM) Nonreactive
Anti-HCV  Negative
Which of the following is the most likely diagnosis? 
Incorrect Answer ImageA.Acute hepatitis A infection
Incorrect Answer ImageB.Acute hepatitis B infection
Incorrect Answer ImageC.Acute hepatitis C infection
Incorrect Answer ImageD.Past hepatitis B immunization
Correct Answer ImageE.Past hepatitis B infection

A 32-year-old woman comes to the physician because of a 2-month history of an itchy skin rash.
She first noticed a large pink patch on her right flank. Ten days later, many smaller reddish, scaly
spots appeared on her stomach, back, and shoulders. She experienced relief of the pruritis with
the use of a topical steroid cream. Her past medical history is unremarkable. Physical
examination shows a 3 cm, mildly erythematous, oval-shaped patch on the right flank. There are
multiple 0.5 to 1 cm erythematous macules on the trunk and back with scales at the periphery of
the lesions. The longer axis of the lesions are aligned with the cutaneous cleavage lines. Which
of the following is the most likely diagnosis?
Correct Answer ImageA.Pityriasis rosea
Incorrect Answer ImageB.Psoriasis vulgaris
Incorrect Answer ImageC.Seborrheic dermatitis
Incorrect Answer ImageD.Tinea versicolor
Incorrect Answer ImageE.Varicella

A 55-year-old woman of South Asian descent comes to the physician's office because of vague
right upper quadrant pain that has been ongoing for several months. She describes the pain as
dull and not associated with food intake. She denies fever, although she has noticed a 5 to 10 kg
(11 to 22 lb) weight loss. Her past medical history is remarkable for gallstones. She does not take
any medications. She immigrated to the United States approximately 10 years ago. She denies a
history of intravenous drug use, and she has never received a blood transfusion. Her temperature
is 36.7°C (98°F), pulse is 86/min, and blood pressure is 120/72 mm Hg. Her head, neck,
pulmonary, and cardiovascular examinations are within normal limits. Palpation of the right
upper quadrant shows moderate tenderness, with no rebound or guarding. Computed tomography
of the abdomen shows a mass arising from the gallbladder wall and invading the liver
parenchyma, as well as several stones within the gallbladder lumen. Which of the following
organisms has been implicated in the development of this lesion? 
Incorrect Answer ImageA.Ascaris lumbricoides
Incorrect Answer ImageB.Clonorchis sinensis
Incorrect Answer ImageC.Escherichia coli
Incorrect Answer ImageD.Fasciola hepatica
Incorrect Answer ImageE.Klebsiella pneumoniae
Correct Answer ImageF.Salmonella typhi

A 55-year-old man comes to the emergency department because of recent palpitations and
fatigue. He reports that his cough has gotten worse over the last few days and his palpitations are
worse when he is winded. He has a history of COPD and diabetes mellitus (DM) treated with
ipratropium, albuterol, and metformin respectively. He does not drink alcohol, but continues to
smoke one pack of cigarettes per day. His temperature is 36.4ºC (97.5ºF), his pulse is 118/min,
his respirations are 18/min, and his blood pressure is 100/70 mm Hg. His BMI is 28.5 kg/m2.
Examination shows decreased breath sounds, scattered rhonchi, no wheezing, and
hyperresonance to percussion. His cardiac examination shows an apical impulse with an irregular
pulse and distant heart sounds. Lower extremities reveal 1+ pedal edema. A 12-lead ECG is done
in the clinic, showing varying P-R intervals and multiple discrete P waves that have different
morphologies in leads I, II, and V1. Which of the following would be the most appropriate next
step in the management of this patient?
Incorrect Answer ImageA.Albuterol nebulizations
Incorrect Answer ImageB.Echocardiogram
Incorrect Answer ImageC.Metoprolol
Correct Answer ImageD.O2 saturation
Incorrect Answer ImageE.Synchronized cardioversion
Incorrect Answer ImageF.Verapamil

A 65-year-old woman with metastatic cancer is admitted to the hospital because of a 2 week
history of severe left thigh pain over the hip. She has been having trouble walking or bearing
weight because of the pain. The pain is worse at night especially when lying on her left side. She
has had no fever, morning stiffness, or trauma. Serum alkaline phosphatase level is 400 U/L.
Plain x-ray shows a lytic lesion in the proximal femur. This patient most likely has a malignancy
of which of the following? 
Correct Answer ImageA.Breast
Incorrect Answer ImageB.Colon
Incorrect Answer ImageC.Melanocytes
Incorrect Answer ImageD.Pancreas
Incorrect Answer ImageE.Thyroid

A 32-year-old woman comes to her physician because of irregularities in her menstrual cycle for
the past 3 months. She had her period a week ago after two months. Previously, her menses were
regular and would occur every 30 days. She also reports a white-colored discharge from her
breasts bilaterally. Her past medical history is significant for hypertension. She denies alcohol
and illicit drug use. She is married and has no children. Laboratory studies show:
Thyroid-stimulating hormone  3.0 μU/mL
Follicle-stimulating hormone  3.8 mU/mL 
Luteinizing hormone  4 U/L 
Prolactin  158 mcg/L
Urine human chorionic gonadotropin  Negative
MRI scan of the head shows a 1.5 cm adenoma in the pituitary gland. Which of the following is
the most appropriate next step in management? 
Incorrect Answer ImageA.Chemotherapy with cyclophosphamide
Correct Answer ImageB.Dopamine agonist
Incorrect Answer ImageC.Dopamine antagonist
Incorrect Answer ImageD.No treatment required
Incorrect Answer ImageE.Radiation treatment
Incorrect Answer ImageF.Transsphenoidal surgical resection of the adenoma

A 68-year-old man with a history of chronic obstructive pulmonary disease (COPD) comes to the
emergency department because of shortness of breath. He states that he has developed
progressively severe shortness of breath over the past 2 weeks, and complains of chest pain that
is worse on inspiration and while coughing. On physical examination, the patient appears
dyspneic and respirations are 18/min. Heart sounds are distant. Chest wall movement is reduced
on the right. The right side of the chest is dull to percussion. There are absent breath sounds on
the right lung field with decreased vocal resonance. A chest x-ray shows an obliterated
costophrenic angle on the right side with mediastinal shift. Which of the following is the most
appropriate next step in management?
Incorrect Answer ImageA.Arterial blood gases
Incorrect Answer ImageB.Bronchoscopy
Incorrect Answer ImageC.Pleurodesis
Correct Answer ImageD.Thoracentesis
Incorrect Answer ImageE.Tube thoracostomy

A 36-year-old man develops rapid mental status deterioration two days after sustaining a femoral
fracture in a skiing accident. Physical examination shows multiple petechiae in the anterior chest
and abdomen. On the third day, the patient lapses into a coma and dies. Postmortem examination
of the brain shows numerous petechial hemorrhages in the corpus callosum and centrum
semiovale. Which of the following is the most likely diagnosis? 
Incorrect Answer ImageA.Diffuse axonal injury
Correct Answer ImageB.Fat embolism
Incorrect Answer ImageC.Septic embolism
Incorrect Answer ImageD.Systemic thromboembolism
Incorrect Answer ImageE.Watershed infarction

A 58-year-old woman comes to the physician for a follow-up examination for her type 2 diabetes
mellitus and hypertension. A review of her records shows that she was switched from
hydrochlorothiazide to enalapril 6 months ago because of a documented allergic reaction. A few
days after beginning treatment with enalapril, she developed swelling of the lower lip. Enalapril
was promptly discontinued. At this time, which of the following is the most appropriate
pharmacotherapy for this patient's hypertension?
Correct Answer ImageA.Candesartan
Incorrect Answer ImageB.Captopril
Incorrect Answer ImageC.Hydralazine
Incorrect Answer ImageD.Hydrochlorothiazide
Incorrect Answer ImageE.Metoprolol

A 48-year-old male comes to the physician because of weight loss, fatigue, and a rash for 1 year.
He denies nausea, vomiting, dysphagia, hematochezia, melena, diarrhea, or constipation. He
complains of a cough productive of blood-tinged sputum and blood in his urine. He does not
smoke cigarettes. He recently visited rural India for 2 weeks. He has a history of asthma. Current
medications include an albuterol inhaler as needed for shortness of breath. His temperature is
37.8°C (100°F), blood pressure is 122/68 mm Hg, pulse is 89/min, and respirations are 15/min.
His oxygen saturation is 98% on room air. The mucous membranes are dry. Scattered wheezes
and coarse breath sounds are heard on auscultation. Cardiac examination shows a normal S1 and
S2; no murmurs are heard. There is no abdominal tenderness. There is a raised, palpable, papular
rash that does not blanch with pressure along his shins. Muscle strength is 3/5 with dorsiflexion
of the left foot and 2/5 with extension of the right knee. Babinski sign is absent. CT scan of the
chest shows thickened bronchi. Laboratory studies show: 
White blood cells  12,100/mm3
Neutrophils  75%
Lymphocytes  8%
Eosinophils  10%
Hematocrit  31%
Platelets 490,000/mm3
Na+ 134 mEq/L
K+ 5.3 mEq/L 
Cl −
100 mEq/L
HCO3− 20 mEq/L
Urea nitrogen  35 mg/dL
Creatinine  2.2 mg/dL
Glucose  103 mg/dL
AST  25 IU/L
ALT  31 IU/L
Alkaline phosphatase  90 IU/L
Antinuclear antibody  Positive in 1:128 titer 
C3 90 mg/dL (normal 88–252
mg/dL)
C4  20 mg/dL (normal 12–72
mg/dL)
Antineutrophil cytoplasmic antibody Positive in 1:256 titer
(ANCA) 
Purified protein derivative (PPD)  Nonreactive
Urinalysis  Red blood cell casts
Which of the following is the most likely diagnosis? 
Correct Answer ImageA.Churg-Strauss syndrome
Incorrect Answer ImageB.Disseminated aspergillosis
Incorrect Answer ImageC.Disseminated tuberculosis
Incorrect Answer ImageD.Occult carcinoma
Incorrect Answer ImageE.Wegener granulomatosis

A 48-year-old man with a history of hypertension comes to the emergency department because
of a severe headache and nausea for 12 hours. The pain is in the back of his head and extends to
the base of his neck. He had a headache 1 day ago that resolved. He denies fever, chills, or
muscle aches. Current medications include hydrochlorothiazide. He has smoked one pack of
cigarettes daily for 28 years. The patient appears in moderate distress. His temperature is 36.9°C
(98.8°F), blood pressure is 168/105 mm Hg, pulse is 72/min, and respirations are 20/min.
Examination shows nuchal rigidity, but there are no focal neurologic deficits. A non-contrast CT
of the head is normal. Which of the following is the most appropriate next step in management?
Incorrect Answer ImageA.CT of the head with contrast
Incorrect Answer ImageB.Empiric treatment with antibiotics
Correct Answer ImageC.Lumbar puncture with cerebrospinal fluid analysis
Incorrect Answer ImageD.Magnetic resonance imaging
Incorrect Answer ImageE.Repeat CT scan of the head without contrast in 48 hours

A 55-year-old woman comes to the emergency department with hematuria and severe flank pain.
An X ray shows multiple stones in the right kidney and ureter, and a renal sonogram shows
moderate right hydronephrosis. A nephrostomy tube is placed to relieve the obstruction. Analysis
of a passed stone shows calcium oxalate. Her serum electrolytes, calcium, and phosphorus are
normal. She comes to the clinic a week after the nephrostomy tube is removed. On physical
examination, she is afebrile and normotensive, and no focal findings are identified. An analysis
of her urine shows elevated calcium, with normal oxalate, phosphate, and uric acid. Which of the
following is the best next step in medical management?
Incorrect Answer ImageA.Administer ciprofloxacin
Incorrect Answer ImageB.Administer furosemide
Correct Answer ImageC.Administer hydrochlorothiazide
Incorrect Answer ImageD.Decrease her dietary calcium intake
Incorrect Answer ImageE.Sestamibi scan of the parathyroid glands

A 23-year-old Caucasian woman comes to the physician because of mild jaundice and fatigue.
She reports having dark urine and light-colored bowel movements. These symptoms have
occurred intermittently for at least the last 6 months and seem to have become worse recently.
Her past medical history is significant for idiopathic thrombocytopenic purpura as a child and
Hashimoto’s thyroiditis. Her temperature is 37ºC (98.6ºF), blood pressure 132/78 mm Hg, pulse
78/min, and respirations 20/min. Physical examination shows mild hepatosplenomegaly but no
other signs or stigmata of liver disease. Laboratory studies show: 
Blood/Serum Chemistries
Bicarbonate 24 mEq/L
BUN 12 mg/dL
Chloride 101 mEq/L
Creatinine 0.7 mg/dL
Glucose 96 mg/dL
Potassium 4.4 mEq/L
Sodium 139 mEq/L
Liver Function Tests
Albumin 3.1 g/dL
AST/SGOT 58 U/L
ALT/SGPT 81 U/L
Alkaline phosphatase 858 U/L
Bilirubin 2.0 mg/dL
GGT 82 U/L
Total protein 9.8 g/dL
Viral hepatitis serologies are negative. An antinuclear antibody (ANA) assay is positive with a
titer of 1:360 in a speckled pattern. Antimitochondrial antibodies (AMA) are negative. Anti-
smooth muscle antibodies (ASMA) are positive. Which of the following is the most appropriate
treatment for this patient? 
Incorrect Answer ImageA.Clofibrate
Incorrect Answer ImageB.Methotrexate
Incorrect Answer ImageC.Penicillamine
Correct Answer ImageD.Prednisone
Incorrect Answer ImageE.Ursodeoxycholic acid

A 43-year-old man presents to the physician because of progressively severe fatigue that has
developed over a period of approximately 3 months. On physical examination his skin and
mucous membranes show a waxy pallor. In-office hematocrit is 18% and peripheral blood smear
shows cells of normal morphology in all cell lines, but band-form neutrophils and reticulocytes
are nearly absent. On specific questioning about possible toxic exposures, the patient reports
using benzene frequently to clean his hands after working on the old house he recently bought
and is remodeling. Which of the following specimens would most likely be seen on bone marrow
biopsy? 
Incorrect Answer ImageA.Abundant megakaryocytes
Incorrect Answer ImageB.Bone marrow fibrosis
Incorrect Answer ImageC.Hypersegmented neutrophils
Correct Answer ImageD.Predominance of fat and stroma
Incorrect Answer ImageE.Several ringed sideroblasts

A 46-year-old woman comes to the physician for a routine examination. She states that her blood
pressure was checked a week ago at a health fair and it was 190/120 mmHg. She has no history
of any major medical illnesses and takes no medications. She has smoked one pack of cigarettes
daily for the past 10 years but does not drink alcohol. Her temperature is 36.8°C (98.3°F), blood
pressure is 196/118 mm Hg, pulse is 80/min, and respirations are 16/min. Ophthalmoscopic
examination shows arteriovenous nicking, but no hemorrhage. Physical examination shows a
mid-abdominal bruit. Laboratory studies are shown:
Na+ 140 mEq/L
K +
3.0 mEq/L
Cl− 100 mEq/L
HCO3 −
31 mEq/L
pH 7.5
PCO2 33 mm Hg
PO2 100 mm Hg
Urinalysis shows no abnormalities. Which of the following is the most likely cause of her
hypertension? 
Incorrect Answer ImageA.Glomerulonephritis
Incorrect Answer ImageB.Smoking
Incorrect Answer ImageC.Pheochromocytoma
Incorrect Answer ImageD.Renovascular hypertension caused by atherosclerosis
Correct Answer ImageE.Renovascular hypertension caused by fibromuscular dysplasia

A 39-year-old white man with no significant past medical history complains of fever and
bilateral knee and shoulder pain for the past several days. He reports that he was on a camping
expedition in Connecticut 2 weeks ago, where he had an unprotected sexual encounter with a
woman he met on the trip. He does recall a tick bite and says that he removed the tick
immediately. His temperature is 38.8ºC (101.8ºF), respirations are 22/min, and blood pressure is
120/60 mm Hg. Physical examination reveals warm, erythematous knees, pain on flexion and
extension of his knee and shoulder joints, and hemorrhagic pustules on his right foot and both
hands. Which of the following is the most likely explanation for this patient's knee pain?
Correct Answer ImageA.Gonococcal infection
Incorrect Answer ImageB.Gout
Incorrect Answer ImageC.Lyme disease
Incorrect Answer ImageD.Reactive arthritis
Incorrect Answer ImageE.Secondary syphilis

A 33-year-old man comes to the physician because of a 6-day history of a rash. He says that he
had similar lesions in the exact same location 2 years ago. He was diagnosed with a urinary tract
infection and an asthma exacerbation 1 week ago. He has a history of asthma, irritable bowel
syndrome, and atopic dermatitis. Current medications include albuterol, codeine, fluticasone
inhaler, trimethoprim-sulfamethoxazole, fexofenadine, and acetaminophen. His brother has
atopy. He is in no distress. Skin findings include two well-defined edematous plaques on his
arms. Which of the following medications is most likely the cause of the lesions?
Incorrect Answer ImageA.Acetaminophen
Incorrect Answer ImageB.Albuterol
Incorrect Answer ImageC.Codeine
Incorrect Answer ImageD.Fexofenadine
Incorrect Answer ImageE.Fluticasone inhaler
Correct Answer ImageF.Trimethoprim-sulfamethoxazole

A 47-year-old man with peptic ulcer disease comes to the physician because of persistent
epigastric abdominal pain. He was diagnosed with a duodenal ulcer and Helicobacter
pylori infection on endoscopy 2 months ago. He has decreased the size of his meals and has
stopped drinking coffee and alcohol. He completed a 2-week course of amoxicillin,
clarithromycin, metronidazole, and omeprazole. There has been no improvement in his
symptoms. Which of the following is the most appropriate next step in management?
Incorrect Answer ImageA.Bismuth, metronidazole, and tetracycline 4 times daily with omeprazole
twice daily for 10–14 days
Incorrect Answer ImageB.Check serum gastrin levels
Correct Answer ImageC.Perform a urea breath test
Incorrect Answer ImageD.Repeat omeprazole, amoxicillin, and clarithromycin twice daily for 10–14
days
Incorrect Answer ImageE.Treatment with omeprazole, levofloxacin, and amoxicillin twice daily for
14 days

A 77-year-old man comes to the physician because of decreasing exercise tolerance. Just 1 year
earlier, he was able to play doubles tennis for 2 hours. Over the past few months, however, he
has had progressive dyspnea on exertion and now can walk only two blocks on level ground
before becoming short-winded. He has also been awakening from sleep with shortness of breath
and requires 3 pillows to sleep comfortably. He has a history of rheumatic fever as a teenager.
His pulse is 92/min, respirations are 18/min, and blood pressure is 168/60 mmHg. Physical
examination shows jugular venous distention while lying supine. Bibasilar crackles are heard on
auscultation of the chest and extend one quarter up both posterior lung fields. S1 and S2 are
normal. A grade 2/6 blowing diastolic murmur is heard at the aortic area. An S3 is audible. The
liver edge is mildly tender, and there is moderate lower extremity edema extending to both
knees. Which of the following drugs is most appropriate for the underlying pathology of this
patient?
Correct Answer ImageA.Captopril
Incorrect Answer ImageB.Digoxin
Incorrect Answer ImageC.Furosemide
Incorrect Answer ImageD.Isosorbide dinitrate
Incorrect Answer ImageE.Warfarin

An 85-year-old woman with a history of hypertension and hyperlipidemia comes to the physician
for a routine health maintenance examination. She has had difficulty dressing herself and
combing her hair. She has bilateral shoulder and hip pain that lasts for approximately one hour
each morning. She denies fevers, weight loss, weakness, headaches, or visual disturbances.
Current medications include hydrochlorothiazide and pravastatin. Which of the following is the
most appropriate next step in management? 
Incorrect Answer ImageA.Bilateral shoulder MRI scan
Correct Answer ImageB.Complete physical examination
Incorrect Answer ImageC.Electromyography
Incorrect Answer ImageD.Measurement of erythrocyte sedimentation rate
Incorrect Answer ImageE.Measurement of serum rheumatoid factor
A 72-year-old man is admitted to the hospital with a 6-month history of headache and new-onset,
right-sided hemiparesis which lasted approximately 30 minutes. A stat CT of the brain fails to
reveal any acute pathology. He is given aspirin and admitted to the ward. His past medical
history includes itchiness and episodes of feeling warm and turning bright red; it also includes
asthma controlled with albuterol inhaler. He has smoked a pack of cigarettes daily for the past 40
years. His current blood pressure is 137/84 mm Hg, pulse is 98/min, and his temperature is 37ºC
(98.6ºF). Examination shows a flushed appearance and grade 2/6 systolic murmur at the lower
sternal border. The tip of the spleen is palpable 2 cm below the left costal margin. Laboratory
workup shows: 
Hematocrit 60%
Hemoglobin 18 g/dL
Leukocyte count 16,000/mm3
Platelet count 500,000/mm3
Serum erythropoietin level 2 mU/mL (normal: 5–20 mU/mL)
Which of the following is the most appropriate first step in management? 
Incorrect Answer ImageA.Add clopidogrel to current therapy
Incorrect Answer ImageB.Bone marrow biopsy
Incorrect Answer ImageC.Counseling on smoking cessation
Incorrect Answer ImageD.Phlebotomy with target hematocrit of 45%
Correct Answer ImageE.Phlebotomy with target hematocrit of 45% and hydroxyurea

A 23-year-old woman is brought to the emergency department by her mother after she had a
"spell." The patient's mother says that her daughter’s left hand began to jerk, and then the jerking
moved up to include her arm and face on the left side. The event lasted for 2 minutes. The patient
says that she smelled cookies before the jerking. She was unable to move her left arm, then
gradually gained her strength back. The mother states that they were having a conversation
during the episode. The patient was not incontinent and did not bite her tongue. Her medical
history is otherwise unremarkable and she takes no medications. Her temperature is 37.2°C
(99°F), blood pressure is 114/72 mmHg, and pulse is 90/min. Muscle strength is 4/5 in the left
upper extremity. Which of the following is the most likely diagnosis?
Incorrect Answer ImageA.Complex partial seizure
Incorrect Answer ImageB.Migraine headache with aura
Incorrect Answer ImageC.Motor tic
Correct Answer ImageD.Simple partial seizure
Incorrect Answer ImageE.Transient ischemic attack

A 51-year-old man comes to the physician because of a 1-month history of severe fatigue, leg
swelling, and jaundice. He has a 20-year history of hepatitis C. He had two previous hospital
admissions, two and five months ago. Physical examination shows scleral icterus and bitemporal
wasting. The liver is palpable 3 cm below the right costal margin, and the spleen is palpable 2 cm
below the left costal margin. There is abdominal ascites with shifting dullness and lower
extremity edema extending to the mid-calf. Laboratory studies showed an albumin level 2.1
g/dL, total bilirubin level 12.1 mg/dL, international normalization ratio (INR) of 2.4, and a serum
creatinine of 2.2 mg/dl. The calculated Model for End Stage Liver Disease (MELD) score is 33,
which is associated with a 3 month mortality of 52.6%. Which of the following is the most
appropriate long-term management of this patient?
Correct Answer ImageA.Evaluation for liver transplantation
Incorrect Answer ImageB.Interferon-alpha (IFN-alpha) or ribavirin monotherapy
Incorrect Answer ImageC.Ledipasvir/sofosbuvir (Harvoni)
Incorrect Answer ImageD.Mesocaval shunt
Incorrect Answer ImageE.Paracentesis weekly or monthly

A 63-year-old woman comes to the physician because of progressive shortness of breath for 2
weeks. She denies lower extremity edema, orthopnea, or paroxysmal nocturnal dyspnea. Her past
medical history is unremarkable and she does not take any medications. She is afebrile. Her
blood pressure is 128/74 mm Hg and oxygen saturation is 93% on room air. Head and neck
examination shows no lymphadenopathy, carotid bruits, or jugular venous distension.
Auscultation of the chest shows decreased left-sided breath sounds. Cardiac examination is
normal and the abdomen is soft and nontender without hepatosplenomegaly. Her peripheral
examination shows no clubbing, cyanosis, or edema. Chest x-ray shows a large left-sided pleural
effusion. The decision is made to perform a thoracentesis. Which of the following represents
proper technique for minimizing complications with this procedure?
Correct Answer ImageA.Insert the needle in the midaxillary line just above the rib and remove 1.0 L
of fluid slowly to improve her shortness of breath

Incorrect Answer ImageB.Insert the needle in the midaxillary line just above the rib and remove 3.0
L of fluid as rapidly as possible to improve her shortness of breath

Incorrect Answer ImageC.Insert the needle in the midaxillary line just below the rib and remove 1.0
L of fluid slowly to improve her shortness of breath

Incorrect Answer ImageD.Insert the needle in the midaxillary line just below the rib and remove 3.0
L of fluid as rapidly as possible to improve her shortness of breath

Incorrect Answer ImageE.Insert the needle in the midclavicular line just above the rib and remove
1.0 L of fluid slowly to improve her shortness of breath

Incorrect Answer ImageF.Insert the needle in the midclavicular line just above the rib and remove
3.0 L of fluid as rapidly as possible to improve her shortness of breath

Incorrect Answer ImageG.Insert the needle in the midclavicular line just below the rib and remove
1.0 L of fluid slowly to improve her shortness of breath

Incorrect Answer ImageH.Insert the needle in the midclavicular line just below the rib and remove
3.0 L of fluid as rapidly as possible to improve her shortness of breath
A 58-year-old woman is admitted to the hospital because of a 2-month history of weight loss and
abdominal bloating. She has had a 7 kg (15 lb) weight loss over the past 2 months. The liver is
tender, firm, and palpable 2 cm below the right costal margin. Paracentesis shows hemorrhagic
ascitic fluid. CT scan of the liver shows a large, intrahepatic multifocal tumor. A biopsy
specimen of the liver shows hepatocytes with moderate variation in cell and nuclear size and
shape; the cells are arranged in a trabecular pattern, have minimal connective tissue, and are
invading vascular channels. Which of the following is the most likely predisposing factor to
account for these findings? 
Incorrect Answer ImageA.Aflatoxin exposure
Correct Answer ImageB.Cirrhosis
Incorrect Answer ImageC.Hemochromatosis
Incorrect Answer ImageD.Oral contraceptives
Incorrect Answer ImageE.Tyrosinemia

A 60-year-old man comes to the physician reporting extremely severe, sharp, shooting pain on
the right side of his face. He describes an episode as being "like a bolt of electricity" brought
about by touching a specific area. This lasts about 60 seconds and occurs many times during the
day. He does not recall any nighttime episodes of pain. Neurologic examination shows no
abnormality. The affected area could not be examined because of the patient's fear of the pain it
might cause. Gadolinium-enhanced MRI shows no cranial nerve abnormalities. Which of the
following is the most appropriate next step in management? 
Incorrect Answer ImageA.Aspirin
Correct Answer ImageB.Carbamazepine
Incorrect Answer ImageC.Gabapentin
Incorrect Answer ImageD.Lamotrigine
Incorrect Answer ImageE.NSAIDs
Incorrect Answer ImageF.Stereotactic ablation
Incorrect Answer ImageG.Surgical decompression of trigeminal nerve

A 48-year-old man comes to the emergency department because of a painful and swollen right
knee for 2 days. He has a history of hypertension and osteoarthritis. Current medications include
hydrochlorothiazide and ibuprofen. Ibuprofen has not relieved his pain. He has smoked a pack of
cigarettes daily for 30 years and drinks 3 beers daily. He is not currently sexually active. His
temperature is 38.6°C (101.5°F), pulse is 106/min, respirations are 16/min, and blood pressure is
138/76 mm Hg. Examination of the right knee shows swelling, tenderness, warmth, and
erythema; range of motion is limited. A radiograph of the right knee shows no abnormalities.
Joint aspiration with synovial fluid analysis shows leukocyte count 96,000/mm3 with 80%
segmented neutrophils. Which of the following is the most appropria

A 47-year-old man who has a history of bipolar disorder and esophageal reflux comes to the
physician because of left flank pain and gross hematuria. The pain started a few hours ago. It is
sharp and extends from the left flank to the left groin. The patient states that he had similar pain
approximately 4 months ago. The vital signs and physical examination are normal except for left
flank tenderness. He is hospitalized and given analgesic medication. The following day, he
passes a kidney stone, with immediate relief of his pain. A subsequent CT scan shows a 1.2-cm
stone in the left distal ureter, together with left-sided hydronephrosis. The patient undergoes
ureteroscopic stone removal. The stone is then sent for analysis, which shows a calcium oxalate
stone. The patient is discharged after 4 days following a complete recovery. Six weeks later he
undergoes a 24-hour urine collection that shows: 
Calcium 424 mg/d (normal, <300 mg/d)
Uric acid 550 mg/d (normal, <800 mg/d)
Oxalate 32 mg/d (normal, <45 mg/d)
Citrate 600 mg/d (normal, > 320 mg/d)
Serum calcium, uric acid, and intact PTH levels are normal. In addition to increasing his fluid
intake and starting a low protein, low sodium diet, which of the following additional therapies
would be most appropriate to prevent further stone formation? 
Incorrect Answer ImageA.Allopurinol
Incorrect Answer ImageB.Furosemide
Correct Answer ImageC.Hydrochlorothiazide
Incorrect Answer ImageD.Potassium citrate
Incorrect Answer ImageE.Restrict dietary calcium
Incorrect Answer ImageF.Restrict dietary purineste pharmacotherapy? 
Incorrect Answer ImageA.Cefotaxime
Incorrect Answer ImageB.Gentamicin
Incorrect Answer ImageC.Penicillin
Correct Answer ImageD.Vancomycin and cefotaxime
Incorrect Answer ImageE.Vancomycin

A 59-year-old woman comes to the clinic because of an episode of left upper arm weakness that
resolved after 6 hours. Over the last month she has experienced increased fatigability, dyspnea
on exertion, palpitations, and a low-grade fever. Her past medical history is negative for
significant illness. Her temperature is 37.3ºC (99.0ºF), pulse is 89/min, respirations are 14/min,
and blood pressure is 115/70 mm Hg. Physical examination shows clear lungs with a regular
pulse and a mid-diastolic rumbling murmur on the fifth intercostal space at the midclavicular
line. She is sent for an echocardiography. Which of the following findings would this study most
likely show?
Incorrect Answer ImageA.Atrial thrombus
Incorrect Answer ImageB.Congenital heart defect
Correct Answer ImageC.Intracardiac lesion
Incorrect Answer ImageD.Left ventricular wall aneurysm
Incorrect Answer ImageE.Vegetative lesions on the leaflets
A 72-year-old man is brought to the emergency department because of heart palpitations and a
headache. His past medical history includes essential hypertension. He states that he normally
takes a variety of antihypertensive drugs but he ran out of "one or two of them" last night. His
blood pressure is 210/120 mm Hg. Ophthalmoscopic examination shows arteriovenous (AV)
nicking and papilledema. Urinalysis shows proteinuria. A review of his records shows that his
systolic blood pressure readings in the past have not been greater than 175 mm Hg. Which of the
following drugs, if stopped abruptly, is most likely to cause the findings seen in this patient?
Incorrect Answer ImageA.Atenolol
Correct Answer ImageB.Clonidine
Incorrect Answer ImageC.Felodipine
Incorrect Answer ImageD.Hydrochlorothiazide
Incorrect Answer ImageE.Lisinopril

A 73-year-old man comes to the clinic because of a rash that started on the right side of his
forehead 2 days ago. He tells you that the area is painful and itchy with the pain occurring 3 days
prior to the rash. He also reports malaise for several days as well as tender swellings in front of
his right ear and below his jaw. He denies any changes in his vision. On physical examination, he
is afebrile with the rash shown below. There is no conjunctival injection. Extraocular motion and
pupillary constriction are normal. Visual acuity is 20/20 with visual fields full to confrontation.
There are two tender, 8-mm pre-auricular lymph nodes and two 12-mm submandibular nodes.
The rest of his physical examination is normal. Which of the following is the most appropriate
next step in establishing a diagnosis?
Incorrect Answer ImageA.Biopsy of the lesions with microscopic analysis
Incorrect Answer ImageB.Direct fluorescent antibody for HSV
Correct Answer ImageC.No further tests are necessary to confirm the diagnosis
Incorrect Answer ImageD.Serum titers of anti-varicella IgG
Incorrect Answer ImageE.Tzanck prep of the lesions looking for multinucleated cells

A 61-year-old woman is brought to the emergency department by family after being found
unconscious and cold to the touch. She has a history of hypothyroidism. Current medications
include levothyroxine. Her blood pressure is 70/40 mm Hg, pulse is 40/min, respirations are
8/min, temperature is 34.4°C (94°F), and oxygen saturation is 82% on room air. She is
unresponsive to sternal rub. Physical examination shows thin, cool, and moist skin. There is
diffuse non-pitting edema in the upper and lower extremities. Which of the following is the most
appropriate next step in her management? 
Incorrect Answer ImageA.Cardiac defibrillation
Correct Answer ImageB.Endotracheal intubation and mechanical ventilation
Incorrect Answer ImageC.Intravenous hypertonic saline
Incorrect Answer ImageD.Intravenous pressor support
Incorrect Answer ImageE.Intravenous thyroxine

A 22-year-old woman comes to the clinic complaining of a "bout of the flu." She reports being
previously healthy. She enjoys hiking and camping and had led a camping trip in Vermont 2
weeks ago. Shortly after returning from her trip, she developed diffuse arthralgias, myalgias, and
chills. A review of symptoms reveals that she also has suffered from headaches, a low-grade
fever, and a rash. The woman wore mosquito repellent and cannot recall being bitten by a tick or
flea. However, she does report that several members of her group did suffer tick bites. Physical
examination is remarkable for an intensely erythematous, 2 cm lesion surrounded by several
concentric red rings. The erythema is most intense centrally. Which of the following is the most
likely vector of her disease? 
Incorrect Answer ImageA.Babesia microti
Incorrect Answer ImageB.Borrelia burgdorferi
Incorrect Answer ImageC.Dermacentor variabilis
Incorrect Answer ImageD.Ehrlichia chaffeensis
Correct Answer ImageE.Ixodes scapularis

An asymptomatic 73-year-old man with a history of type 2 diabetes mellitus is seen because of
worsening urinary frequency and difficulty initiating urination. He has had no pain with urination
or blood in his urine, but voids in small volumes 10 times/day. He occasionally has nocturnal and
urinary incontinence. He has a 10-year history of hypertension. Current medications include
metformin, pioglitazone, and lisinopril. His pulse is 88/min and blood pressure is 138/88 mm Hg.
Rectal examination shows an enlarged prostate with a hard nodule in the right lobe of the
prostate. Laboratory studies show hemoglobin A1c is 7.3% (normal, ⩽6%) and prostate-specific
antigen is 110 ng/mL (normal, <4 ng/mL). Which of the following imaging tests is most likely to
detect metastatic prostate cancer?
Correct Answer ImageA.Bone scan
Incorrect Answer ImageB.CT scan of the abdomen
Incorrect Answer ImageC.CT scan of the chest
Incorrect Answer ImageD.CT scan of the pelvis
Incorrect Answer ImageE.MRI scan of the brain

A 73-year-old man is brought to the emergency department by his family because of 1 week of
increasing confusion and diffuse abdominal pain. There is no headache, weakness, fever, or
change in bowel or urination habits. His family endorses a 6.8 kg (15 lb) weight loss over the
past 2 months. His medical history is significant for emphysema, for which he takes albuterol as
needed. He has a 30 pack-year smoking history, but he does not drink. On physical examination,
his vital signs are normal. He converses normally but is not oriented to date. He has dry mucous
membranes with decreased skin turgor, a prolonged expiratory phase, and mild, diffuse
abdominal tenderness. The remainder of the physical examination is normal. Laboratory studies
show: 
Blood
WBCs 6,700/mm3
Hct 46%
Platelets 265,000/mm3
Serum
Sodium 139 mEq/L
Potassium 4.2 mEq/L
Chloride 105 mEq/L
Bicarbonate 24 mEq/L
Urea nitrogen 10 mg/dL
Creatinine 1.1 mg/dL
Glucose 110 mg/dL
Calcium 12.9 mg/dL
Phosphate 3.1 mg/dL
Parathyroid hormone level 2 pg/mL (normal, 10–65 pg/mL)
Urine
Calcium 400 mg/24 hours (normal 100–300)
Which of the following is the most appropriate next step in diagnosis? 
Correct Answer ImageA.Chest radiograph
Incorrect Answer ImageB.CT scan of the head
Incorrect Answer ImageC.Sestamibi nuclear scan
Incorrect Answer ImageD.Serum calcitonin
Incorrect Answer ImageE.Serum cholecalciferol

A 78-year-old man is brought to the emergency department by his family because of progressive
confusion and headache since falling and hitting his head on the icy pavement yesterday. He has
had no seizures, fever, chills, weakness, or incontinence. The patient’s past medical history
includes hypertension, hyperlipidemia, benign prostatic hyperplasia, and diabetes mellitus.
Current medications include atorvastatin, tamsulosin and metformin. His temperature is 37.1°C
(98.8°F), pulse is 80/min, respirations are 14/min, and blood pressure is 120/80 mm Hg. On
physical examination, he seems confused and is only oriented to person. There is no nuchal
rigidity or photophobia. Neurologic examination shows a normal-appearing gait. All cranial
nerves are grossly intact. Muscle strength is 4/5 in all extremities. Sensation is normal. A CT
scan of the head is most likely to show which of the following?
Incorrect Answer ImageA.Biconvex, hyperdense mass between suture lines
Incorrect Answer ImageB.Enlarged ventricles
Correct Answer ImageC.Hyperdense, crescentic mass crossing suture lines
Incorrect Answer ImageD.Mass effect with surrounding edema
Incorrect Answer ImageE.Ring-enhancing lesions

A 68-year-old man presents to the clinic complaining of recurrent epigastric and substernal chest
pain. Over the past 10 years he has been treated periodically for gastroesophageal reflux disease
(GERD) with ranitidine and over-the-counter antacids, which have transiently helped his
symptoms. At times the pain has been so severe that he went to the emergency department, and
at each of those visits his cardiac evaluation was negative. He denies dysphagia or bloody stools.
His laboratory studies including hemoglobin are normal. Which of the following is the best next
step at this time?
Incorrect Answer ImageA.Barium swallow
Incorrect Answer ImageB.CT scan of the neck and chest
Correct Answer ImageC.Endoscopy with biopsies
Incorrect Answer ImageD.pH monitoring
Incorrect Answer ImageE.Start omeprazole and re-evaluate after 4 weeks

A 75-year-old man comes to the office because of severe dyspnea that has been ongoing for the
past year. He denies a cough, chest pain, hemoptysis, and weight loss. He has a significant
smoking history of 2 packs per day for 50 years. He has no other medical issues. He has no
allergies and takes no medications. He worked as a stone engraver until he retired 10 years ago.
Vital signs are stable. Pulmonary examination shows end expiratory crackles bilaterally. He has
clubbing of his fingers bilaterally. Chest x-ray shows multiple nodules. Which of the following is
the most likely diagnosis? 
Incorrect Answer ImageA.Adenocarcinoma
Incorrect Answer ImageB.Asbestosis
Incorrect Answer ImageC.Idiopathic pulmonary fibrosis
Correct Answer ImageD.Silicosis
Incorrect Answer ImageE.Tuberculosis

A 25-year-old man presents to his physician's office because people have told him over the past
two weeks that his eyes appear "yellow." He denies any other symptoms, including weight loss,
malaise, or abdominal pain. He drinks alcohol occasionally with meals. His past medical history
is unremarkable and he takes no medications. His temperature is 36.7°C (98.0°F), blood pressure
is 120/80 mm Hg, pulse is 86/min, and respirations are 13/min. His sclera are icteric. Head and
neck examinations are normal. Cardiopulmonary examination shows no abnormalities.
Abdominal examination and extremities are normal. The following lab data was obtained:
WBC 8,000/mm3
Hb 13 g/dL
Hct 39%
Platelets 450,000/mm3
ALT 17 IU/L
AST 12 IU/L
Alkaline phosphatase 56 IU/L
Total bilirubin 4.0 mg/dL
Conjugated bilirubin 3.5 mg/dL
Lactate dehydrogenase 60 U/L
A urinary examination is positive for bilirubin and a 500% greater-than-normal urinary
coproporphyrin. Which of the following conditions is the most likely diagnosis? 
Incorrect Answer ImageA.Biliary obstruction
Incorrect Answer ImageB.Dubin-Johnson syndrome
Incorrect Answer ImageC.Gilbert syndrome
Correct Answer ImageD.Rotor syndrome
Incorrect Answer ImageE.Viral hepatitis

 38-year-old Caucasian man is brought to the physician by his mother. The mother states that the
patient has become increasingly paranoid and irritable over the last 2 years. He suspects her of
poisoning his food. While he had many friends when he was young, he now has few friends and
rarely socializes. On physical examination, the physician notes that the man shows random,
uncontrollable, jerky movements. The patient scores 24 on the Mini-Mental Status Examination
(MMSE). Which of the following is most likely to be seen on neuroimaging in this patient? 
Correct Answer ImageA.Caudate nucleus atrophy
Incorrect Answer ImageB.Loss of dopaminergic neurons in the substantia nigra
Incorrect Answer ImageC.Multiple infarcts in the dominant hemisphere
Incorrect Answer ImageD.Non-dominant parietal lobe lesion
Incorrect Answer ImageE.Ventricular enlargement and reduction in cortical volume

A 43-year-old woman comes to the physician because of musculoskeletal pain for 8 months. She
has also had severe fatigue and depression. Because of her fatigue she has poor job performance
as an electrical engineer. She is married with one child. She does not smoke and drinks two to
three glasses of wine socially. Complete blood count, basic metabolic panel, erythrocyte
sedimentation rate, thyroid function tests, and muscle enzymes are normal. Physical examination
shows diffuse muscle tenderness. The patient reports already being in an aerobic exercise class,
but continues to have persistent pain. Which of the following is the most appropriate next step in
management? 
Correct Answer ImageA.Amitriptyline
Incorrect Answer ImageB.Fluoxetine
Incorrect Answer ImageC.Ibuprofen
Incorrect Answer ImageD.Prednisone
Incorrect Answer ImageE.Tramadol

An 82-year-old man is brought to the emergency department by ambulance with eight hours of
crushing chest pain radiating to his left arm associated with dyspnea, nausea, and sweating. His
temperature is 37.7°C (99.9°F), pulse is 128/min, and blood pressure is 168/92 mm Hg. His chest
pain resolves with aspirin, intravenous heparin, beta-blockers, intravenous nitrates, and
eptifibatide. Twelve hours after admission to the coronary care unit his blood pressure drops to
103/62 mm Hg. There is jugular venous distention. Cardiopulmonary examination shows
scattered rales and wheezes; there are no murmurs, rubs, or gallops. Which of the following is
the most likely diagnosis?
Incorrect Answer ImageA.Cardiac tamponade
Correct Answer ImageB.Congestive heart failure
Incorrect Answer ImageC.Free wall rupture
Incorrect Answer ImageD.Papillary muscle rupture
Incorrect Answer ImageE.Right ventricular infarct

A 51-year-old man presents to his physician for increasing ankle swelling. He has no chest pain,
dyspnea, or urinary complaints. He has noticed intermittent fatigue and mild fevers for 3 months.
He denies smoking or alcohol consumption. He used intravenous heroin 30 years ago. His
temperature is 37.7ºC (100.0ºF), pulse is 85 beats/min, and blood pressure is 145/95 mm Hg. The
optic fundus is normal; there is no lymphadenopathy or scleral icterus. Heart and lung
examinations are normal, as are abdominal, rectal, and scrotal examinations. Extremities show
2+ pitting edema. Laboratory data are shown below. 
White blood 13,000/mm3
cells 
Hematocrit  36%
Platelets  450,000/mm3
Sodium  140 mEq/L
Potassium  3.8 mEq/L
Chloride  105 mEq/L
Bicarbonate  24 mEq/L
Blood urea 15 mg/dL
nitrogen 
Creatinine  1.4 mg/dL
Albumin  2.1 g/dL
Cholesterol 320 mg/dL
Fasting glucose  91 mg/dL
Urinalysis trace blood, 4+ protein, 1–2 WBCs/high power
field, 1–2 RBCs/HPF, no casts 
A quantitation of his proteinuria yields 4 grams over in 24 hours. Which of the following is the
test most likely to yield diagnostic information?
Incorrect Answer ImageA.ANCA serologies

Incorrect Answer ImageB.Anti-glomerular basement membrane antibodies

Incorrect Answer ImageC.Anti-streptolysin O titer

Incorrect Answer ImageD.Erythrocyte sedimentation rate

Incorrect Answer ImageE.Hemoglobin A1c

Correct Answer ImageF.Hepatitis B surface antigen

A 59-year-old man who is homeless is brought to the emergency department because of


confusion and weakness. It is a bitterly cold night and there is snow on the ground. His
temperature is 32.0ºC (89.6ºF), blood pressure is 80/40 mm Hg, pulse is 53/min, and respirations
are 10/min. An electrocardiogram would most likely show which of the following? 
Incorrect Answer ImageA.Delta wave
Incorrect Answer ImageB.Depressed ST segments
Incorrect Answer ImageC.Diffuse broadening of T waves
Correct Answer ImageD.Osborne waves
Incorrect Answer ImageE.U waves

A 55-year-old woman comes to the physician because of a 6-month history of a raised skin
lesion on her right cheek. She has a long history of sun exposure. She denies fever, recent
trauma, or bug bites. There is a past medical history of hypertension treated with lisinopril.
Physical examination shows a 2 cm patch that is rough with an adherent, yellow-brown scale.
There is no tenderness or bleeding. Which of the following is the most likely diagnosis?
Correct Answer ImageA.Actinic keratosis
Incorrect Answer ImageB.Basal cell carcinoma
Incorrect Answer ImageC.Bowenoid papulosis
Incorrect Answer ImageD.Keratoacanthoma
Incorrect Answer ImageE.Malignant melanoma
A 45-year-old man with a 4-year history of cirrhosis is admitted to the hospital after vomiting
bright red blood for 1 hour. He appears to be in severe distress. His temperature is 37.2ºC
(99.0ºF), pulse is 135/min, respirations are 28/min, and blood pressure is 80/45 mm Hg. He
receives 3 units of packed red blood cells. Laboratory studies performed on the third day of
admission show:
TSH (third-generation test) 0.7 mIU/mL
Thyroxine (T4) 3.8 μg/dL
Triiodothyronine (T3) 30 ng/dL
Examination of the thyroid gland is normal. Which of the following is the most likely diagnosis? 
Correct Answer ImageA.Euthyroid sick syndrome
Incorrect Answer ImageB.Graves' disease
Incorrect Answer ImageC.Hashimoto's disease
Incorrect Answer ImageD.Medullary carcinoma of the thyroid
Incorrect Answer ImageE.Silent lymphocytic thyroiditis

A 45-year-old man with a 4-year history of cirrhosis is admitted to the hospital after vomiting
bright red blood for 1 hour. He appears to be in severe distress. His temperature is 37.2ºC
(99.0ºF), pulse is 135/min, respirations are 28/min, and blood pressure is 80/45 mm Hg. He
receives 3 units of packed red blood cells. Laboratory studies performed on the third day of
admission show:
TSH (third-generation test) 0.7 mIU/mL
Thyroxine (T4) 3.8 μg/dL
Triiodothyronine (T3) 30 ng/dL
Examination of the thyroid gland is normal. Which of the following is the most likely diagnosis? 
Correct Answer ImageA.Euthyroid sick syndrome
Incorrect Answer ImageB.Graves' disease
Incorrect Answer ImageC.Hashimoto's disease
Incorrect Answer ImageD.Medullary carcinoma of the thyroid
Incorrect Answer ImageE.Silent lymphocytic thyroiditis

A 62-year-old woman with metastatic colorectal cancer comes to the physician because of
progressive pain in her left hip for 2 months. Despite aggressive surgery and adjuvant
chemotherapy, the disease has spread throughout her pelvis and eroded into her left acetabulum.
Current medications include ibuprofen, acetaminophen, oxycodone, and gabapentin. She is going
to discontinue oxycodone and start taking fentanyl. She is extremely worried about her need for
increasing doses of narcotics because she has heard that they have awful side effects, and she
fears becoming "hooked." Which of the following clinically significant side effects is she most
likely to experience? 
Incorrect Answer ImageA.Addiction
Correct Answer ImageB.Constipation
Incorrect Answer ImageC.Nausea and vomiting
Incorrect Answer ImageD.Respiratory depression
Incorrect Answer ImageE.Sedation

A 55-year-old male comes to the emergency department because of acute onset of severe
abdominal pain. The patient states that he was sitting and watching TV when all of a sudden he
felt severe diffuse abdominal pain. He also reports having two loose bloody bowel movements.
His past medical history includes atrial fibrillation, hypertension, and dyslipidemia. Medications
include warfarin and metoprolol. His blood pressure is 90/50 mm Hg, pulse is 120/min,
respirations are 20/min, and temperature is 37.8°C (100°F). Abdominal examination shows
hypoactive bowel sounds and severe diffuse tenderness but his abdomen is soft without rebound
or guarding. Laboratory studies show: 
ABG:
pH 7.30
PaCO2 30 mm Hg
PaO2 90 mm Hg
CBC:
WBC  15,000 mm3
Neutrophils  95%
Lymphocytes  5%
Hemoglobin  12 g/dL
Hematocrit  36%
Platelets  200,000 mm3
Electrolytes:
Na+ 140 mEq/L
K+ 3.0 mEq/L
Cl -
110 mEq/L
HCO3- 14 mEq/L
BUN  25 mg/dL
Creatinine  0.5 mg/dL
Which of the following is the most appropriate next step in management?
Incorrect Answer ImageA.Emergent colonoscopy
Correct Answer ImageB.Intravenous fluids and broad-spectrum antibiotics
Incorrect Answer ImageC.Mesenteric angiography
Incorrect Answer ImageD.Order a CT scan with contrast
Incorrect Answer ImageE.Surgical embolectomy

A 55-year-old man is brought to the emergency department because of altered mental status.
According to the patient's daughter, he had reported a headache for the last 2 days. Last night, he
was febrile and had a few episodes of vomiting. This morning he was difficult to arouse. He has
a past medical history of hypertension, diabetes mellitus, chronic renal insufficiency, and
coronary artery disease. Medications include aspirin, clopidogrel, metoprolol, glyburide, and
lisinopril. His temperature is 38.4ºC (101.1ºF), blood pressure 190/106 mm Hg, pulse 102/min,
and respirations 16/min. Physical examination shows a patient who is confused and somnolent.
The patient moves all 4 extremities but is not cooperative for a complete neurologic examination.
The remainder of the examination is unremarkable. Head CT is normal. Lumbar puncture shows:
Opening pressure 350 mm H2O
Appearance  turbid
White blood cell count 1,500/microL
Neutrophil predominance (85%), 10 red blood cells/field
Glucose 20 mg/dL
Protein 300 mg/dL
Which of the following is the most appropriate therapy? 
Incorrect Answer ImageA.Ampicillin plus gentamicin
Incorrect Answer ImageB.Ampicillin plus cefotaxime
Incorrect Answer ImageC.Cefotaxime plus vancomycin
Incorrect Answer ImageD.Ceftazidime plus vancomycin plus ampicillin
Correct Answer ImageE.Ceftriaxone plus vancomycin plus ampicillin plus dexamethasone

A 47-year-old alcoholic man arrives at the hospital with unremitting abdominal pain. He was
brought in by his wife after the pain became unbearable. He has had bouts of abdominal pain in
the past, but none as severe as the current one. He drinks approximately 6 alcoholic beverages
per day. On physical examination, he is afebrile and his blood pressure is 105/62 mm Hg. His
pulse is 103 beats/min. Head and neck examination reveals no scleral icterus. His chest is clear.
His jugular veins are flat. No murmurs are auscultated. Diffuse abdominal tenderness is
appreciated. No clubbing, cyanosis, or edema are noted. Computed tomography of the abdomen
demonstrates peripancreatic fluid, pancreatic edema, and necrosis of approximately one-third of
the pancreas. The patient is resuscitated aggressively and is started on antibiotics. The patient
fails to improve significantly over the next 2 days. Which of the following is the next step in the
management of this patient? 
Incorrect Answer ImageA.Change antibiotic regimen
Incorrect Answer ImageB.Continue conservative management
Correct Answer ImageC.Pancreatic aspiration
Incorrect Answer ImageD.Repeat computed tomography
Incorrect Answer ImageE.Surgical debridement

A 65-year-old man is hospitalized for an acute exacerbation of chronic bronchitis. His past
medical history is only significant for chronic obstructive pulmonary disorder (COPD). His last
pneumococcal vaccination was 5 years ago. At the time of admission, the patient reports
dyspnea, a productive cough with green-tinged sputum, and pleuritic chest pain. Pulse oximetry
on room air shows an oxygen saturation of 80%. Arterial blood gas analysis shows pH 7.35,
pO2 51 mm Hg, and pCO2 58 mm Hg. The patient is treated with oxygen therapy, nebulized
ipratropium and albuterol treatments, and azithromycin. Over the course of 5 days, the patient's
respiratory status improves. His cough and sputum production diminishes and he is weaned from
supplemental oxygen. At the time of discharge, his oxygen saturation is 90% on room air, and
arterial blood gas shows pH 7.37, pCO2 50 mm Hg, and pO2 70 mm Hg. Which of the following
is the most appropriate treatment at this time?
Incorrect Answer ImageA.Continued antibiotics
Incorrect Answer ImageB.Home oxygen therapy
Incorrect Answer ImageC.Inhaled steroids
Incorrect Answer ImageD.Leukotriene inhibition
Correct Answer ImageE.Pneumococcal vaccine

A 22-year-old man comes to his physician because of intermittent abdominal pain that has been
present for the past year. He says that the pain is occasionally associated with eating. He denies
nausea or vomiting. His past medical history is unremarkable. He is not taking any medications.
His temperature 37.5°C (99.5°F), blood pressure is 126/80 mm Hg, pulse is 86/min, and
respirations are 14/min. On physical examination, the patient appears to be in no distress and is
holding his arm against his abdomen. Chest and heart examinations are normal. Mild tenderness
is elicited while palpating the right upper quadrant. There is no rebound, guarding, or ascites.
Peripheral examination is normal. Liver function tests are normal. A right upper quadrant
ultrasound shows a fusiform dilatation of the common bile duct, and subsequent
cholangiography confirms the diagnosis. Which of the following is the most appropriate next
step in the management of this patient?
Incorrect Answer ImageA.Endoscopic retrograde cholangiopancreatography
Incorrect Answer ImageB.Intravenous antibiotic therapy
Incorrect Answer ImageC.No further action is necessary
Incorrect Answer ImageD.Percutaneous choledochocystostomy
Correct Answer ImageE.Surgical excision

A 58-year-old woman comes to the emergency department because of double vision and
difficulty walking for 4 days. She has a history of seizure disorder but has had no seizure activity
for 1 year since starting treatment with an antiepileptic medication. She smoked two packs of
cigarettes daily for 10 years, but she quit 4 years ago. She appears confused and has slurred
speech. Her temperature is 36.8ºC (98.2ºF), blood pressure is 110/75 mm Hg, and pulse is
84/min. There is mild overgrowth of the gingiva. Neurologic examination shows marked
nystagmus and truncal ataxia. Which of the following medications is most likely causing her
symptoms? 
Incorrect Answer ImageA.Carbamazepine
Incorrect Answer ImageB.Gabapentin
Incorrect Answer ImageC.Phenobarbital
Correct Answer ImageD.Phenytoin
Incorrect Answer ImageE.Valproate

A 60-year-old man with a 4-month history of right shoulder pain comes for a follow-up visit.
The pain occasionally wakes him from sleep. He worked for 40 years as a contractor, lifting
heavy objects over his head. Examination shows tenderness below the acromion with painful
active range of motion. The right arm is passively abducted to 90 degrees and the patient is asked
to hold his arm in this position. When the arm is released, the arm drops to the patient's side. The
pain has been relieved by intra-articular injections of anesthetic and corticosteroid. The tendon
for which of the following muscles is most likely torn in this patient?
Incorrect Answer ImageA.Infraspinatus
Incorrect Answer ImageB.Long head of the biceps brachii
Incorrect Answer ImageC.Subscapularis
Correct Answer ImageD.Supraspinatus
Incorrect Answer ImageE.Teres major

 64-year-old obese woman comes to the physician because of chest pain. She states that she has
been experiencing progressively worsening, dull, retrosternal chest pain exacerbated by climbing
stairs and walking more than 2 blocks to the grocery store. She denies dyspnea or syncope. She
has smoked one pack of cigarettes daily for the past 25 years. She appears comfortable at rest.
Physical examination shows a feeble, slow-rising carotid pulse, a systolic thrill, and a forceful
apex beat. Breath sounds are clear bilaterally. Which of the following is the most accurate
diagnostic test? 
Incorrect Answer ImageA.Chest x-ray
Incorrect Answer ImageB.Dobutamine echocardiography
Incorrect Answer ImageC.Electrocardiography
Correct Answer ImageD.Left heart catheterization and angiography
Incorrect Answer ImageE.MUGA scan
Incorrect Answer ImageF.Stress test
Incorrect Answer ImageG.Transthoracic echocardiography

A 32-year-old woman with a history of depression is found to have microscopic hematuria


during a routine physical examination. The patient has not noticed any gross hematuria. She
denies any dysuria, flank pain, or urinary urgency or frequency. The patient has not had any
blood or urine tests before this physical examination. She states that she has generally been
healthy. She was diagnosed with depression approximately 1 year ago, which is moderately well
controlled with fluoxetine. She has smoked 1 pack of cigarettes per day for 10 years. Her
temperature is 37.1°C, pulse is 90/min, respirations are 16/min, and blood pressure is 150/90 mm
Hg. There is no skin rash. HEENT examination is normal. Her neck is supple with no JVD,
bruits, or lymphadenopathy. Cardiopulmonary examination is normal. Abdomen is soft,
nontender, and nondistended. Bowel sounds are normal. Extremities show no edema.
Urinalysis shows:
Specific gravity 1.015
pH 6.0
Blood 2+
Protein 3+
Ketones Negative
Glucose Negative
RBCs 15-20/HPF
WBCs 0/HPF
Epithelial cells Negative
Crystals Negative
Serum studies show:
Sodium 140 mEq/L
Potassium 3.9 mEq/L
Chloride 104 mEq/L
Bicarbonate 26 mEq/L
BUN 18 mg/dL
Creatinine 0.8 mg/dL
 What is the most likely etiology of the patient's hematuria?
Incorrect Answer ImageA.Bladder cancer
Incorrect Answer ImageB.Fluoxetine-induced hematuria
Correct Answer ImageC.Glomerulopathy
Incorrect Answer ImageD.Hemorrhagic cystitis
Incorrect Answer ImageE.Nephrolithiasis
Incorrect Answer ImageF.Urinary tract infection

A 62-year-old man is brought to the emergency department because of chronic chest pain. He
states that for the past few months he has been getting chest "pressure" localized to the substernal
region and radiating to the left arm on occasion. The pain occurs with exertion but never at rest.
He also says that when he gets the pain it usually lasts about 5 minutes but goes away with rest.
He denies chest pain at this time. He reports that his exercise tolerance is moderate and he gets
dyspnea on exertion after a few blocks of walking. He has no history of major illness and takes
no medications, but his family history is significant for a myocardial infarction in his father at
age 54. Physical examination shows no chest wall tenderness, but a carotid bruit is heard and the
dorsalis pedis pulses are decreased. ECG shows normal sinus rhythm with no ST-segment
changes. In addition to ascertaining other coronary risk factors, which of the following is the
most appropriate diagnostic test at this time?
Incorrect Answer ImageA.Cardiac biomarkers
Incorrect Answer ImageB.Coronary angiography
Incorrect Answer ImageC.Dipyridamole thallium stress test
Incorrect Answer ImageD.Exercise stress echocardiography
Correct Answer ImageE.Exercise stress test

 78-year-old man comes to the physician for a routine health maintenance examination. He has a
history of hyperlipidemia treated with pravastatin. He is retired from his job as a miner. Physical
examination shows numerous small keratotic papules on the palms and soles; there are more
lesions over the thenar and hypothenar eminences. There is a hyperkeratotic plaque on the
abdomen. The remainder of the examination shows no abnormalities. A biopsy specimen shows
invasive squamous cell carcinoma (SCC) arising from SCC in situ. Chronic exposure to which of
the following is most likely responsible for this patient's condition? 
Correct Answer ImageA.Arsenic
Incorrect Answer ImageB.Chromic acid
Incorrect Answer ImageC.Coal dust
Incorrect Answer ImageD.Para-phenylenediamine
Incorrect Answer ImageE.Silica

A 62-year-old woman comes to the physician because of lethargy, weakness, and depression for
4 months. She started taking paroxetine 3 months ago and has had no change in her mood. Her
past medical history is otherwise unremarkable. Her pulse is 80/min, respirations are 12/min, and
her blood pressure is 100/60 mm Hg. The lungs are clear to auscultation. Cardiac examination
shows a normal S1 and S2; no murmurs are heard. There is no pedal edema. Laboratory studies
show: 
Na+ 134 mEq/L
K +
5.5 mEq/L
Cl− 101 mEq/L
HCO3− 22 mEq/L
BUN  20 mg/dL
Creatinine  0.9 mg/dL
Glucose  82 mg/dL
Cortisol (A.M.)  4 µg/dL (normal, 5–23 µg/dL)
30-minute cosyntropin stimulation test 8 µg/dL
60-minute cosyntropin stimulation test  8 µg/dL
Plasma adrenocorticotropin:  75 pg/mL (normal, 9–52 pg/mL
Which of the following is the most likely diagnosis? 
Incorrect Answer ImageA.Adrenal crisis
Correct Answer ImageB.Primary adrenal insufficiency
Incorrect Answer ImageC.Secondary adrenal insufficiency
Incorrect Answer ImageD.Secondary hyperaldosteronism
Incorrect Answer ImageE.Tertiary adrenal insufficiency

A 35-year-old man is brought to the emergency department because of a 3-day history of nausea
and vomiting, rash, and progressive confusion. He has no past medical history and does not take
any medications. He is a nonsmoker and doesn't drink alcohol. His temperature is 38.5ºC
(101.3ºF), pulse is 98/min, and blood pressure is 110/75 mm Hg. He is alert and oriented to
person only. It is difficult to engage him in focused conversation. There are scattered petechiae
and ecchymosis over the chest and both lower extremities. There is no neck stiffness or
localizing neurological signs. Laboratory studies show:
Hematocrit  21%
Hemoglobin  7.2 mg/dL
Leukocyte count  9,000/mm3
Platelets 13,000/mm3
Mean corpuscular volume 89 µm3
Reticulocyte count 5% 
Creatinine  1.8 mg/dL
Aspartate aminotransferase (AST)  24 U/L
Alanine aminotransferase (ALT) 25 U/l
Lactate dehydrogenase 648 U/L
Total bilirubin  2.2 mg/dL
Prothrombin time  12 seconds
Partial thromboplastin time, activated 30 seconds
Peripheral smear shows schistocytes and decreased platelets. Which of the following is the most
likely diagnosis? 
Incorrect Answer ImageA.Disseminated intravascular hemolysis
Incorrect Answer ImageB.HELLP syndrome
Incorrect Answer ImageC.Hemolytic uremic syndrome
Incorrect Answer ImageD.Idiopathic thrombocytopenic purpura
Correct Answer ImageE.Thrombotic thrombocytopenic purpura

A 60-year-old woman comes to the physician because of increasing fatigue over the past several
months. She reports that her regular daily activities lead to pain and stiffness especially when
washing her hair. She also has trouble finding a comfortable position at rest. She has lost 12
pounds over the past 3 months, but prior to this time she has always been healthy and is not
taking any medications. Physical examination shows tenderness of the proximal muscles and
limitation of shoulder and hip movements. Which of the following is the most likely diagnosis?
Incorrect Answer ImageA.Depression
Incorrect Answer ImageB.Hypokalemia
Correct Answer ImageC.Polymyalgia rheumatica
Incorrect Answer ImageD.Polymyositis
Incorrect Answer ImageE.Temporal arteritis

A 22-year-old man comes to the emergency department because of vomiting and abdominal pain
for the past day. He reports feeling thirsty and drinking a lot of water lately. He also reports
increased urination for the past two weeks. He denies contact with ill people. He has no history
of significant medical illnesses and does not take any medications. He does not smoke, drink
alcohol or use illicit drugs. His temperature is 36.6°C (97.9°F), pulse is 115/min, respirations are
28/min, and blood pressure is 112/68 mm Hg. Oxygen saturation is 98% on room air. Physical
examination shows dry skin with decreased turgor. Cardiovascular examination shows normal
heart sounds, no added heart sounds or murmurs. The lungs are clear to auscultation. Abdominal
examination shows generalized tenderness to palpation. Laboratory studies show:
pH 7.28
PaCO2 26 mm Hg
PaO2 95 mm Hg
Na+ 135 mEq/L
Cl -
95 mEq/L 
K+ 5.6 mEq/L 
HCO3 -
18 mEq/L
Which of the following tests is required for diagnosis? 
Incorrect Answer ImageA.Calcium
Incorrect Answer ImageB.Creatinine
Correct Answer ImageC.Glucose
Incorrect Answer ImageD.Hemoglobin and hematocrit
Incorrect Answer ImageE.Lactic acid
Incorrect Answer ImageF.Liver function tests (LFTs)
Incorrect Answer ImageG.White blood cell (WBC) count
A 37-year-old man comes to the physician because of flank pain, painful urination, and blood in
his urine for 1 week. He has a history of hypertension treated with diet and exercise. He takes no
medications. His brother had a renal transplant at the age of 43 years. His temperature is 36.7ºC
(98.0ºF), pulse is 90/min, respirations are 20/min, and blood pressure is 152/88 mm Hg.
Examination shows bilateral costovertebral angle tenderness. Laboratory studies show:
Hemoglobin  11.3 g/dL 
Leukocyte count  10,500/mm3 
Platelet count  265,000/mm3
INR 1.2
Serum studies show:
BUN  18 mg/dL 
Creatinine  1.0 mg/dL 
Calcium  8.9 mg/dL 
Urinalysis shows 15 to 20 leukocytes/HPF and 200 erythrocytes/HPF. A CT scan of the
abdomen is shown. Which of the following is the most appropriate initial pharmacotherapy for
this patient's hypertension?
Incorrect Answer ImageA.Amlodipine
Incorrect Answer ImageB.Clonidine
Incorrect Answer ImageC.Hydrochlorothiazide
Correct Answer ImageD.Lisinopril
Incorrect Answer ImageE.Spironolactone

A 65-year-old man is scheduled for an elective cholecystectomy. He is on coumadin therapy for


a mechanical aortic valve and is admitted 5 days prior to surgery for intravenous heparin
bridging. Coumadin is held and intravenous heparin is initiated. Laboratory studies from the day
of admission and four days later are shown below:
Initial hospitalization:
Leukocyte count 6,040/mm3
Hematocrit 43%
Platelet count 235,000/mm3
INR 2.7
Partial thromboplastin time 41 seconds
Hospital day 4:
Leukocyte count 7,200/mm3
Hematocrit 41%
Platelet count 90,000/mm3
INR 1.2
Partial thromboplastin time 66 seconds
His intravenous heparin is immediately stopped. Which of the following is the next step in the
management? 
Correct Answer ImageA.Intravenous bivalirudin
Incorrect Answer ImageB.No further intervention is required
Incorrect Answer ImageC.Platelet transfusion
Incorrect Answer ImageD.Reinitiation of oral Coumadin
Incorrect Answer ImageE.Subcutaneous low-molecular-weight heparin
A 55-year-old, HIV-positive man comes to the physician because of a painless fungating mass
growing out of the anus which he is able to feel when he wipes himself after having a bowel
movement. For the past 6 months, he has noticed blood on the toilet paper and occasionally
noticed blood coating the outside of the stools. The patient has lost an unspecified amount of
weight and appears emaciated. On physical examination, the mass is easily visible, is fixed to
surrounding tissues, and appears to grow out of the anal canal. It measures 3.5 cm in diameter.
He also has hard, bilaterally enlarged inguinal lymph nodes. Which of the following is the most
likely diagnosis? 
Incorrect Answer ImageA.Adenocarcinoma of the rectum
Incorrect Answer ImageB.Condyloma acuminata of the anus
Incorrect Answer ImageC.External hemorrhoids
Incorrect Answer ImageD.Rectal prolapse
Correct Answer ImageE.Squamous cell carcinoma of the anus

A 22-year-old man is brought to the emergency department after being pinned by a steel beam,
which had fallen on his legs and lower torso. He is alert and oriented. His blood pressure is
105/75 mm Hg and pulse 95/min. Examination shows bilaterally crushed lower extremities.
Laboratory studies show: 
Sodium 143 mEq/L (135–145 mEq/L)
Potassium 5.2 mEq/L (3.5–5.2 mEq/L)
Bicarbonate 20 mEq/L (20–29 mEq/L)
Chloride 98 mEq/L (96–106 mEq/L)
Urea nitrogen 37 mg/dL (7–20 mg/dL)
Creatinine 1.1 mg/dL (0.8–1.4 mg/dL)
Creatine phosphokinase 15,300 U/L (52–200 U/L)
Aspartate aminotransferase 112 IU/L (<40 IU/L)
Alanine aminotransferase 99 IU/L (<40 IU/L)
Hematocrit 38% (41–50%)
Arterial blood gas on 40% oxygen shows pH 7.33, PCO2 37 mmHg, and PO2 80 mmHg. Which
of the following is the most appropriate next step in management?
Incorrect Answer ImageA.12-lead electrocardiogram
Incorrect Answer ImageB.Echocardiogram
Incorrect Answer ImageC.Fractionated creatine kinase (CK) levels
Correct Answer ImageD.IV crystalloid and bicarbonate
Incorrect Answer ImageE.Right upper quadrant ultrasound

A 45-year-old man comes to the emergency department because of a headache and discharge
from his left ear for 2 days. Two days ago, he suffered a head injury in a motor vehicle accident.
He denies losing consciousness during this accident, and no evaluation was done at that time. His
medical history is unremarkable. He is in no distress. His temperature is 36.7ºC (98.0ºF), blood
pressure is 128/64 mm Hg, pulse is 82/min, and respirations are 16/min. Examination shows a
clear, watery discharge from the left ear. The remainder of the examination shows no
abnormalities. Which of the following is the most appropriate diagnostic test for this patient's ear
discharge? 
Correct Answer ImageA.Beta-2-transferrin
Incorrect Answer ImageB.Glucose level
Incorrect Answer ImageC.Protein analysis
Incorrect Answer ImageD.Salt content
Incorrect Answer ImageE.Specific gravity

A 17-year-old boy comes to the physician because of a 4-year history of facial lesions. He says
that some lesions begin as painful "bumps," whereas others look like "pimples" and large pores.
His medical history is unremarkable, and he takes no medications. He drinks occasionally on
weekends. He denies illicit drug use. He is sexually active and he uses condoms consistently. His
temperature is 36.9°C (98.4°F). Physical examination is shown. There are open and closed
comedones, numerous deep inflammatory nodules, and ice pick-like scarring. Which of the
following is the most appropriate pharmacotherapy for this patient's condition?
Incorrect Answer ImageA.Oral acitretin
Correct Answer ImageB.Oral isotretinoin
Incorrect Answer ImageC.Oral minocycline
Incorrect Answer ImageD.Oral prednisone
Incorrect Answer ImageE.Oral spironolactone
Incorrect Answer ImageF.Topical tazarotene/benzoyl peroxide

A 70-year-old man with an 88 pack-year smoking history and hypertension comes to the
physician because of slowly progressive shortness of breath over the last 1 to 2 years. Physical
examination shows a cachectic man who is in mild respiratory distress using accessory muscles
and "pursed lip" breathing. He is not cyanotic. His breath sounds are decreased without wheezing
or rhonchi. His heart sounds are distant. He has no edema. A chest x-ray shows hyperinflation of
lung fields with a small heart, flattened diaphragm, and no infiltrates or edema. Pulmonary
function testing is most likely to show which of the following findings?
Incorrect Answer ImageA.Bronchodilator response of approximately 15-20%
Correct Answer ImageB.Decreased diffusion capacity of the lung for carbon monoxide
Incorrect Answer ImageC.Decreased residual volume
Incorrect Answer ImageD.Decreased total lung capacity
Incorrect Answer ImageE.Increased FEV1/FVC ratio

A 71-year-old man is brought to the emergency department because of an episode of bright red
blood per rectum. The patient states that he passed a grossly bloody bowel movement a few
hours ago. The bowel movement was associated with some cramping and lower abdominal pain.
He has a history of coronary artery disease and a myocardial infarction 3 years ago. He is poorly
compliant with his beta-blocker and diuretic therapy. His blood pressure readings have run
approximately 140/85 mm Hg at home. While the patient is in the emergency department, he has
another episode of large-volume, bright red blood per rectum. His pulse is 120/min and blood
pressure is 100/60 mm Hg while he is supine, and his pulse is 142/min and blood pressure is
75/55 mm Hg when he stands up. His hematocrit is 23%. He suddenly develops substernal chest
tightness that radiates to his left shoulder. An ECG is shown. Which of the following
interventions is the most appropriate next step in management?
Incorrect Answer ImageA.Administration of aspirin by mouth
Incorrect Answer ImageB.Administration of a beta-blocking agent
Incorrect Answer ImageC.Administration of nitroglycerin sublingually
Incorrect Answer ImageD.Administration of nitroglycerin patch
Correct Answer ImageE.Blood transfusion

A 46-year-old chocolate-factory worker comes to the physician because of heartburn. The pain
occurs more frequently after large meals and at night when he is lying in bed. He noticed severe
pain with swallowing for the last 2 days, which prompted him to visit his physician. He also
reports a bitter taste in the back of the throat and a persistent nonproductive cough. Antacids and
over-the-counter ranitidine did not relieve his symptoms. His physical examination is remarkable
only for moderate obesity. Which of the following is the most appropriate next step in diagnosis?
Incorrect Answer ImageA.Acid perfusion test
Correct Answer ImageB.Endoscopy
Incorrect Answer ImageC.Esophageal manometry
Incorrect Answer ImageD.pH testing
Incorrect Answer ImageE.Proton pump inhibitor, and review in 4 weeks

A 37-year-old man comes to the physician for a routine physical examination. He has a history
of cardiac surgery in early childhood. Physical examination shows a systolic thrill palpated at the
left sternal border, and a soft diastolic murmur is heard in the base of the heart that increases
with inspiration and decreases with the Valsalva maneuver. There is also an extra sound heard
after systole and a widely split second heart sound. Which of the following is the most likely
diagnosis? 
Incorrect Answer ImageA.Aortic regurgitation
Incorrect Answer ImageB.Aortic stenosis
Incorrect Answer ImageC.Mitral regurgitation
Incorrect Answer ImageD.Mitral stenosis
Correct Answer ImageE.Pulmonary regurgitation
Incorrect Answer ImageF.Pulmonary stenosis

A 34-year-old woman comes to the physician because of a 6-month history of ringing in her ears,
a feeling of spinning, and progressive hearing loss in her left ear. The spinning sensation is
episodic and lasts 30 to 60 minutes. During these episodes, she has had nausea and vomiting. She
denies fever, chills, cough, or pain in the ears or throat. Her blood pressure is 130/70 mm Hg,
pulse is 62/min, and respirations are 15/min. Examination shows left-sided hearing loss. An MRI
scan of the head shows no abnormalities. Which of the following is the most likely diagnosis?
Incorrect Answer ImageA.Vestibular schwannoma
Correct Answer ImageB.Ménière disease
Incorrect Answer ImageC.Migraine
Incorrect Answer ImageD.Multiple sclerosis
Incorrect Answer ImageE.Benign positional vertigo

A 35-year-old woman comes to the physician because of a one month history of right wrist pain.
The pain is worse with activity and is relieved by rest. She reports difficulty gripping objects
with her right hand. She gave birth to her first child 6 months ago. Physical examination shows
tenderness to palpation of the radial styloid. There is pain with resistance to thumb abduction and
extension. Tinel test is negative. Inflammation of which of the following structures is the most
likely cause of this patient's symptoms? 
Incorrect Answer ImageA.Abductor pollicis brevis tendon
Incorrect Answer ImageB.Brachioradialis tendon
Incorrect Answer ImageC.Deep branch of the radial nerve
Correct Answer ImageD.Extensor pollicis brevis tendon
Incorrect Answer ImageE.Flexor carpi radialis tendon

A 59-year-old man comes to his primary care physician with left foot pain for the past 6 months.
He reports chronic, progressive pain, numbness, and paresthesias in his left foot, but he has not
sought medical advice until now. His past medical history is significant for insulin-dependent
diabetes mellitus, hypertension, and smoking one pack of cigarettes per day for 25 years. Current
medications include amlodipine, lisinopril, and a basal-bolus regimen of insulin. His most recent
Hemoglobin A1c was 8.7%. His temperature is 37°C (98.6°F), pulse is 85/min, respirations are
22/min, and blood pressure is 164/88 mm Hg. Physical examination shows lower extremities that
are warm and well-perfused. A 2+ palpable dorsalis pedis and posterior tibial pulse are noted. He
has bilateral loss of fine sensory detection in the lower extremities. There is also a clean, 2 x 2
cm area of skin induration on the distal aspect of the dorsum of the left foot. It is warm, tender,
and erythematous, but there is no fluctuance, skin breakdown, or discharge. Which of the
following is the most important intervention in the management of this patient's current
condition?
Incorrect Answer ImageA.Angiogram
Correct Answer ImageB.Blood glucose control
Incorrect Answer ImageC.Electrocardiogram
Incorrect Answer ImageD.Ophthalmologic evaluation
Incorrect Answer ImageE.Smoking cessation

A 45-year-old man comes to the emergency department because of diarrhea and severe lethargy.
For the past 2 weeks he has been having voluminous episodes of watery diarrhea multiple times
a day associated with nonbilious vomiting. Two weeks ago he spent time with his nieces and
nephews, whom he later found out all had some kind of gastrointestinal “bug.” He has been
having “a difficult time keeping anything down,” and on the advice of his primary care physician
he finally decided to come for further evaluation. His speech is slurred. While supine, his blood
pressure is 90/60 mm Hg and pulse is 80/min; while standing, blood pressure is 70/50 mm Hg
and pulse is 120/min. The patient appears weak and malnourished. His skin has diminished skin
turgor, mucous membranes are dry upon observation, and his eyes appear sunken. Laboratory
studies show:
Complete blood count
WBC 8,000 mm3
Hemoglobin 18 g/dL
Hematocrit 55%
Platelets 250,000 mm3
Serum
Na+ 124 mEq/L
K+ 3.3 mEq/L
Cl -
88 mEq/L
HCO3- 30 mEq/L
BUN 30 mg/dL
Creatinine 1.0 mg/dL
Glucose 70 mg/dL
What is the most appropriate management for this patient's current condition? 
Incorrect Answer ImageA.Intravenous dextrose in water (D5W)
Incorrect Answer ImageB.Intravenous 0.45% saline solution
Correct Answer ImageC.Intravenous 0.9% saline solution
Incorrect Answer ImageD.Intravenous 3% saline solution
Incorrect Answer ImageE.Oral replacement electrolytes

A 76-year-old man comes to the physician for a routine health maintenance examination. He has
been feeling well since his last visit 6 months ago. He lives in an assisted living community and
requires minimal assistance with activities of daily living. His past medical history is significant
for gout, hypertension, and osteoarthritis which are well-controlled with medications. He is up to
date with his health care screening examinations and vaccinations. Vital signs and physical
examination are within normal limits. Laboratory studies show:
Serum chemistry:
Bicarbonate  25 mEq/L
Blood urea nitrogen (BUN)  14 mg/dL
Calcium 8.9 mg/dL
Chloride  103 mEq/L
Creatinine  0.9 mEq/L
Glucose  108 mg/dL
Potassium  3.9 mEq/L
Sodium  140 mEq/L
Complete blood count:
Hematocrit  39%
Leukocytes  5,000/mm3
Platelets  189,000/mm3
Liver function tests:
Albumin 3.4 g/dL
Alkaline phosphatase 102.0 U/L
ALT (SGPT) 41.0 U/L
AST (SGOT) 30.0 U/L
Bilirubin, total 0.4 mg/dL
Total protein  10.0 g/dL
To evaluate the increased plasma protein levels, serum and urine protein electrophoresis tests are
performed. The urine electrophoresis is negative. The serum protein electrophoresis confirms the
presence of elevated gamma globulin levels and detects a monoclonal pattern identified as IgG
kappa-chain M protein. A bone marrow biopsy is performed and shows 8% plasma cells.
Radiographs of the axial skeleton are normal. Which of the following is the most appropriate
next step in the management of this patient?
Incorrect Answer ImageA.Immunoglobulin levels including IgA and IgG
Incorrect Answer ImageB.No additional testing
Incorrect Answer ImageC.Order CT chest, abdomen, and pelvis
Incorrect Answer ImageD.Order nuclear bone scan
Correct Answer ImageE.Repeat serum electrophoresis in 6 months

A 22-year-old African immigrant comes to the emergency department because of acute


abdominal pain. The pain is described as sharp and worse in the left upper quadrant. He has had
frequent episodes of abdominal pain over the last 24 hours with each episode gradually resolving
within 3 hours. He has a history of sickle cell anemia with recurrent episodes of bone pain. He is
currently not taking any medications. His temperature is 36.7°C (98°F), pulse is 102/min, blood
pressure is 120/72 mm Hg, and respirations are 18/min. Head and neck examination shows pale
mucosa. Chest is clear to auscultation. Heart is tachycardic and auscultation shows a loud
systolic murmur throughout the precordium. The spleen is not palpable. Laboratory data showed:
WBC 5,700/mm3
Hct 25%
Hb 8 g/dL
Platelets 250,000/mm3
Creatinine 1.1 mg/dL
Total bilirubin 4.9 mg/dL
Conjugated bilirubin 1.0 mg/dL
Chest x-ray shows no consolidation. The vertebrae are noted to be H-shaped. An ultrasound of
his abdomen is performed, showing a shrunken spleen and several gallstones without evidence of
cholecystitis. Which of the following is the most likely composition of these gallstones? 
Correct Answer ImageA.Black pigment
Incorrect Answer ImageB.Brown pigment
Incorrect Answer ImageC.Calcium oxalate
Incorrect Answer ImageD.Cholesterol
Incorrect Answer ImageE.Uric acid

A 17-year-old boy is brought to the emergency department after being found unconscious near a
bottle of antifreeze. The father reports that the boy has been drinking alcohol excessively. Over
the past few weeks, the boy has acted depressed and has stopped spending time with his friends.
On arrival, the boy withdraws from painful stimuli and speaks with incomprehensible sounds.
Which of the following will confirm the diagnosis of antifreeze ingestion? 
Incorrect Answer ImageA.Evaluate for hyperemia of his optic discs
Correct Answer ImageB.Examine his urine under a microscope
Incorrect Answer ImageC.Examine the patient's breath for a fruity odor
Incorrect Answer ImageD.Obtain an electrocardiogram
Incorrect Answer ImageE.Request a serum osmolarity measurement

A 33-year-old man is brought to the emergency department 30 minutes after his family witnessed
him having a seizure. He was eating dinner when he got a blank look on his face. His arm started
shaking, and he then fell to the ground and had generalized convulsions. He was not arousable
for several minutes after the episode. His medical history is unremarkable and he takes no
medications. He emigrated from Mexico 4 years ago. He is a vegetarian. He does not drink
alcohol or use illicit drugs. In the emergency room, he is alert and oriented. His temperature is
36.8°C (98.2°F) and blood pressure 118/76 mm Hg. Head and neck examination shows no
abnormalities. Lungs are clear to auscultation. Cardiac examination shows a normal S1 and S2;
no murmurs are heard. Laboratory studies show: 
Hematocrit  44%
Platelets  189,000/mm3
Leukocyte count  6,700/mm3
Serum studies show:
Na+  138 mEq/L 
Cl –
105 mEq/L 
K+  3.9 mEq/L 
HCO3 -
24 mEq/L 
Blood urea nitrogen (BUN)  11 mg/dL 
Creatinine  0.8 mg/dL 
Which of the following is the most appropriate next step in diagnosis? 
Correct Answer ImageA.Computed tomography of the brain
Incorrect Answer ImageB.Electroencephalography
Incorrect Answer ImageC.Lumbar puncture
Incorrect Answer ImageD.Serum glucose
Incorrect Answer ImageE.Urine toxicology screen

An 18-year-old man comes to the physician because of a painful finger for 5 days. He has a
history of asthma and occasional cold sores. Current medications include albuterol and
fluticasone. He works part-time as a carpenter and frequently injures his hands and fingers.
Examination shows grouped vesicles on an erythematous base on the distal phalanx of the
patient's right index finger. There is exquisite tenderness to palpation. Which of the following is
the most appropriate next step in management? 
Incorrect Answer ImageA.Admission for intravenous antibiotics
Incorrect Answer ImageB.Incision and drainage
Incorrect Answer ImageC.Magnetic resonance imaging of the hand
Correct Answer ImageD.Observation and analgesics
Incorrect Answer ImageE.Oral antibiotics and observation
A 55-year-old man with a history of chronic obstructive pulmonary disease and alcoholism
comes to the physician because of worsening cough with yellow-green sputum production and
occasional blood for 3 days. He also has had fatigue, shortness of breath with exertion, and right-
sided chest pain with deep breaths. He smoked 1 pack of cigarettes daily for 35 years, but he quit
3 days ago. He drinks 2 glasses of vodka daily. He appears flushed and is sweating. His
temperature is 38.4ºC (101.0ºF), pulse is 116/min, respirations are 18/min, and blood pressure is
146/96 mm Hg. Chest wall movements are decreased. Percussion over the right side of the chest
is dull. Crackles are heard in the right lung field with increased vocal resonance. Which of the
following is the most likely causal organism?
Incorrect Answer ImageA.Haemophilus influenzae
Incorrect Answer ImageB.Klebsiella pneumoniae
Incorrect Answer ImageC.Moraxella catarrhalis
Incorrect Answer ImageD.Mycoplasma pneumoniae
Correct Answer ImageE.Streptococcus pneumoniae

A 47-year-old African-American woman is brought to the emergency department for shortness


of breath, chest pain with respirations, and a nonproductive cough that has been going on for the
last 3 to 4 months. Her medical history is significant for influenza treated with amantadine 1
week ago, ventricular tachycardia treated with an implantable cardioverter-defibrillator (ICD)
and amiodarone, and use of OCP since she began menopause one year ago. She drinks 2 glasses
of wine a day and smoked cigarettes for 20 years but quit 10 years ago. She reports that she has
lost weight, even though her appetite is good. Her temperature is 37.6ºC (99.6ºF), pulse is
98/min, respirations are 20/min, and blood pressure is 120/60 mm Hg. Physical examination
shows lungs with diffuse crackles, decreased air movement, and a pleural rub. Cardiac
examination shows an apical impulse, and no murmurs or gallops are appreciated. There is a
purplish mottling of her lower extremities noted. Laboratory studies are taken and shown below:
Hb 12 g/dL
Hct 35%
WBC 12,000/mm3
Neutrophils 68%
Bands 3%
Creatinine 1.1 mg/dL
Sodium 136 mEq/L
Potassium 5.0 mEq/L
Calcium 9.6 mg/dL
Amiodarone levels 1.6 µg/mL (normal levels, 1–3 µg/mL) 
A chest radiograph shows patchy alveolar infiltrates. Pulmonary function tests are taken and
show a normal FEV1/FVC ratio and a decreased FEV1. Which of the following is the most likely
diagnosis? 
Incorrect Answer ImageA.Acute bronchitis
Correct Answer ImageB.Amiodarone pulmonary toxicity
Incorrect Answer ImageC.Chronic bronchitis
Incorrect Answer ImageD.Pulmonary embolism
Incorrect Answer ImageE.Sarcoidosis
A 52-year-old man comes to the physician because of "yellow" eyes. He has noticed this
symptom progressing over the past month with some increased itchiness over his body and a 4.5
kg (10 lb) weight loss. His past medical history is notable for hypertension and hyperlipidemia
treated with metoprolol and atorvastatin, respectively. He does not smoke or drink alcohol. Vitals
are within normal limits. He has icteric sclera and appears cachectic. His lungs are clear to
auscultation and on cardiac examination there was a regular rate and rhythm. There is diffuse
right upper quadrant tenderness on palpation of the abdomen. Laboratory data are: 
WBC 8,000/mm3
Hb 11 g/dL
Hct 33%
Platelets 500,000 mm3
Sodium 135 mEq/L
Potassium 4.2 mEq/L
Creatinine 1.1 mg/dL
Bilirubin 11.1 mg/dL
Conjugated bilirubin 9.2 mg/dL
Abdominal ultrasound shows dilation of the right and left hepatic ducts with a normal
gallbladder and a normal common bile duct. Which of the following statements is true for this
lesion?
Incorrect Answer ImageA.Approximately 20% of patients with HCV cirrhosis will develop this
disease
Incorrect Answer ImageB.Chronic pancreatitis is a common risk factor
Incorrect Answer ImageC.Entamoeba histolytica is associated with this disease

Incorrect Answer ImageD.Primary sclerosing cholangitis (PSC) is not associated with this disease

Correct Answer ImageE.There is an association with recurrent pyogenic cholangitis (RPC)

A 63-year-old woman comes to the emergency department because of severe mid-abdominal


pain. She reports that the pain has increased in intensity over the past few days. There has been
no associated nausea or vomiting, no change in bowel habits, and no relief afforded by position
changes. She is postmenopausal and does not take hormone replacement therapy. She has a 30-
year history of hypertension, and has been noncompliant with her therapy of calcium channel
blocker and thiazide diuretic. On examination her abdomen is obese but there is a suggestion of a
non-tender, pulsatile mass in the epigastric region. The remainder of the physical examination is
normal. Which of the following is the most appropriate next step in management? 
Correct Answer ImageA.Abdominal CT scan with contrast
Incorrect Answer ImageB.Abdominal CT scan without contrast
Incorrect Answer ImageC.Abdominal ultrasound
Incorrect Answer ImageD.Angiography
Incorrect Answer ImageE.Immediate surgical intervention
A 47-year-old woman comes to the physician because of a 2-month history of poor sleep, muscle
aches, fatigue, headaches, and a poor mood. She has had fatigue, headaches, and a poor mood.
She takes no medications. Her temperature is 36.7ºC (98ºF), blood pressure is 125/80 mm Hg,
pulse is 70/min, and respirations are 14/min. Examination shows point tenderness bilaterally at
the lower sternocleidomastoid muscle, trapezius muscle, near the lateral epicondyle, greater
trochanter, upper outer quadrant of the buttock, and medial fat pad of the knee. Laboratory
studies show: 
Hemoglobin 14 g/dL
Leukocyte count 6,000/mm3
Platelet count 270,000/mm3
Erythrocyte sedimentation rate is normal. Which of the following is the most appropriate next
step in management? 
Incorrect Answer ImageA.Brain MRI scan
Incorrect Answer ImageB.Electroencephalography
Incorrect Answer ImageC.Muscle biopsy
Incorrect Answer ImageD.Nerve conduction studies
Correct Answer ImageE.Obtain widespread pain index and symptom severity scale

A 34-year-old man comes to the physician because of severe intermittent left flank pain and
hematuria since yesterday. He has a history of medullary thyroid carcinoma that was resected at
10 years of age. He does not take any medications besides levothyroxine. Family history is
significant for thyroid cancer and kidney stones in his mother and brother. Vital signs are
normal. Physical examination shows mild tenderness in the left flank region without guarding or
rigidity. Laboratory studies show a calcium level of 12.8 mg/dL. Albumin levels are 4.0 g/dL.
CT scan of the abdomen shows a 0.8 mm stone in the urethra. Which of the following screening
tests should be performed in this patient?
Incorrect Answer ImageA.Colonoscopy
Incorrect Answer ImageB.Serum α-fetoprotein
Incorrect Answer ImageC.Serum pancreatic peptides
Incorrect Answer ImageD.Serum prolactin
Correct Answer ImageE.Urinary metanephrines

A 29-year-old woman with a 3-year history of multiple sclerosis comes to the physician because
of urinary incontinence for 6 months. Despite a trial of fluid restriction to 1,800 mL per day and
scheduled voiding, she continues to have episodes of urinary leakage and a sensation of a full
bladder. Current medications include methylprednisolone and interferon. Examination shows 3/5
muscle strength in the left upper extremity. Pelvic examination shows no abnormalities.
Urodynamic studies show a hypotonic bladder and high postvoid residual volume. Which of the
following is the most appropriate pharmacotherapy? 
Correct Answer ImageA.Alpha 1-receptor blocker

Incorrect Answer ImageB.Glatiramer acetate


Incorrect Answer ImageC.Imipramine
Incorrect Answer ImageD.Levofloxacin
Incorrect Answer ImageE.Mirabegron

A 72-year-old man is admitted to the hospital because of a 6-month history of headache after a
transient ischemic attack. He has had flushing, sweating, and generalized itching after a hot
shower. He has smoked one pack of cigarettes daily for 40 years. His temperature is 37.0ºC
(98.6ºF), pulse is 98/min, and blood pressure is 137/84 mm Hg. Examination shows a red, ruddy
complexion. A grade 2/6 systolic murmur is heard at the left lower sternal border. The spleen is
palpated 2 cm below the left costal margin. Laboratory studies show:
Hct 60%
Hb 18 g/dL
Leukocyte count 16,000/mm3
Platelet count 700,000/mm3
Serum erythropoietin level is 2 mU/mL (normal, 0–19 mU/mL). In addition to aspirin and
scheduled phlebotomies, which of the following is the most appropriate pharmacotherapy? 
Incorrect Answer ImageA.Anagrelide
Incorrect Answer ImageB.Chlorambucil
Correct Answer ImageC.Hydroxyurea
Incorrect Answer ImageD.Interferon alpha
Incorrect Answer ImageE.Stem cell transplantation

A 38-year-old man comes to the physician because of a 6-month history of weight loss, fatigue,
and black tarry stools. His father died of colon cancer at age 42 years. His paternal uncle is 70
years old and in good health. The patient's wife, age 32 years, is healthy, as are their twin 10-
year-old children. His hemoglobin is 8.9 g/dL with a mean corpuscular volume of 72 fL.
Colonoscopy shows at least 200 polyps. A biopsy specimen of 4 polyps shows early malignant
degeneration without invasion. Which of the following is the most appropriate initial colon
cancer screening strategy for the patient's family members?
Incorrect Answer ImageA.Annual fecal occult blood testing for the 2 children and the patient's uncle
Incorrect Answer ImageB.Barium enema starting at age 12 years for the children, repeat every 3
years
Incorrect Answer ImageC.Colonoscopy or flexible sigmoidoscopy every 3 years for the children and
wife
Correct Answer ImageD.Genetic screening of the patient, followed by genetic testing of the 2
children
Incorrect Answer ImageE.Immediate colonoscopy of the uncle, clinical observation of the rest of the
family

An 81-year-old man is brought to the hospital after being found on his living room floor. He had
last been in contact with his family 3 days ago. He is obtunded, but is breathing on his own and
is hemodynamically stable. His physical examination is significant for a temperature of 38.0ºC
(100.7ºF) and a tender, tense right calf, thigh, and buttock. His laboratory studies show: 
Potassium 5.9 mEq/L
Serum Creatinine 3.5 mg/dL
Lactic Acid 2.6 mmol/L
Serum Creatine Kinase 7,200 U/L
White Blood Cells 17,000/mcL
BUN 88 mg/dL
Incorrect Answer ImageA.Emergency fasciotomy
Incorrect Answer ImageB.IV mannitol
Incorrect Answer ImageC.IV sodium bicarbonate
Incorrect Answer ImageD.Oral Kayexalate
Correct Answer ImageE.Urgent hemodialysis
Incorrect Answer ImageF.Vascular surgical consult for creation of arteriovenous fistula

A 45-year-old man comes to the emergency department because of a 1-day history of progressive
blurry vision, difficulty speaking, and difficulty swallowing. He also has nausea, constipation,
and dry eyes. He reports that two people in his office have had similar symptoms. He denies
recent history of travel. His medical history is unremarkable, and he does not take any
medications. He is in no distress. His temperature is 36.9°C (98.4°F), blood pressure is 128/84
mm Hg, and pulse is 74/min. Cranial nerve examination shows dysarthria, dilated unreactive
pupils, and bilateral ptosis. Muscle strength is 3/5 in the shoulders and arms and 5/5 in the lower
extremities. Sensation is normal. Which of the following is the most likely diagnosis?
Incorrect Answer ImageA.Amyotrophic lateral sclerosis
Correct Answer ImageB.Botulism
Incorrect Answer ImageC.Guillain-Barré syndrome
Incorrect Answer ImageD.Myasthenia gravis
Incorrect Answer ImageE.Organophosphate poisoning

A 55-year-old man with a 20-pack-year history of smoking comes to the physician because of
moderate shortness of breath for 3 weeks. He also has a long history of chronic cough, which
recently became associated with hemoptysis. Vital signs are within normal limits. Physical
examination shows no jugular venous distention. Heart sounds are normal. Chest x-ray shows a
pleural effusion on the right side without evidence of masses. A diagnostic thoracentesis
produces bloody pleural fluid with no particular odor. The pleural fluid has a hematocrit of 10%,
a protein content of about 60% of the serum protein value, a glucose of 50 mg/dL, and a
differential cell count of 60% lymphocytes. Which of the following is the most appropriate next
step in management?
Incorrect Answer ImageA.Bronchoscopy with bronchoalveolar lavage
Incorrect Answer ImageB.Determine amylase levels in the pleural fluid
Incorrect Answer ImageC.MRI of the chest
Incorrect Answer ImageD.Pleural biopsy
Correct Answer ImageE.Send cytology of the pleural fluid
A 63-year-old nulliparous woman comes to the physician for a routine physical examination. She
says that she has no current physical complaints and never feels the need to visit her physician,
stating that the last time she saw a physician was during a childhood illness. Physical
examination shows a tapping apex beat. Lungs are clear bilaterally. Auscultation of the chest
with the patient in the left lateral position shows a diastolic murmur heard best with the
stethoscope bell on expiration. ECG shows no abnormalities. Echocardiogram shows mitral
stenosis with a valvular area of 3.0 cm2 (normal 4–6 cm2). Which of the following is the next best
step in management? 
Incorrect Answer ImageA.Balloon valvulotomy
Incorrect Answer ImageB.Furosemide
Incorrect Answer ImageC.Metoprolol
Incorrect Answer ImageD.Valve replacement
Correct Answer ImageE.Yearly follow-up

A 65-year-old woman comes to the physician because of intractable diarrhea for the past 2
months. Her symptoms started 2 years ago and have been severe over the past 2 weeks, during
which she has noted 10 to 12 watery stools per day. She has not noticed any blood in her stools
or any fevers. She denies recent travel or any deviation from her regular diet. Her past medical
history is remarkable only for type 1 diabetes. Her only medication is insulin. She does not
smoke or drink alcohol. Her temperature is 37.0°C (98.6°F), blood pressure is 90/55 mm Hg,
pulse is 108/min, and she is oxygenating 98% on room air. She appears fatigued and profoundly
dehydrated. She has orthostatic hypotension. Her lungs are clear to auscultation, and she has no
abnormal heart sounds. Abdominal examination shows mild tenderness without rebound or
guarding. No hepatosplenomegaly is identified. The remainder of her physical examination is
normal. Laboratory values are: 
White blood cells 7,800/mm3
Hematocrit 33%
Platelets 190,000/mm3
Sodium 133 mEq/L
Chloride 99 mEq/L
Bicarbonate 20 mEq/L
Urea nitrogen 17 mg/dL
Creatinine 0.9 mg/dL
Stool ova and parasites No organisms identified
Stool culture Normal flora
Which of the following is the most appropriate next step in management? 
Incorrect Answer ImageA.Add fiber to the diet and prescribe amitriptyline
Correct Answer ImageB.Measure serum anti-tissue transglutaminase antibodies
Incorrect Answer ImageC.Measure serum anti-gliadin antibody
Incorrect Answer ImageD.Small bowel biopsy
Incorrect Answer ImageE.Start lactose and gluten-free diet

A 57-year-old woman comes to her physician for advice regarding hypercholesterolemia


diagnosed on a free screening at work. She has been postmenopausal for the past 5 years and has
not been on hormone replacement therapy. She exercises twice a week for 30 minutes. She has a
family history of coronary artery disease (father, age 62). Her physical examination shows
normal vital signs and a normal exam. Which of the following, if present, would be the best
indication to start the patient on HMG-CoA reductase (statin) therapy? 
Incorrect Answer ImageA.HDL cholesterol 25 mg/dL
Incorrect Answer ImageB.LDL cholesterol 160 mg/dL
Incorrect Answer ImageC.Smoking history
Incorrect Answer ImageD.Total cholesterol 200 mg/dL
Incorrect Answer ImageE.Triglycerides 200 mg/dL

Correct Answer ImageF.Well-controlled diabetes mellitus

A 27-year-old brick layer comes to the emergency department with a swollen left knee. The pain
began 36 hours ago and has limited his ability to work. He has had a fever and shaking chills
over the past 24 hours. His temperature is 38.9°C (101.9°F) and pulse is 104/min. The rest of the
physical examination is unremarkable except for a swollen, erythematous knee with an obvious
effusion. There is limited range of motion. An arthrocentesis reveals 90,000/mm3 WBC and 82%
neutrophils. A Gram stain reveals many neutrophils but no organisms are seen. Polarizing
microscopy reveals no crystals. Based on the most likely etiology, which of the following is the
most appropriate initial step in therapy? 
Correct Answer ImageA.Ceftriaxone and vancomycin
Incorrect Answer ImageB.Ceftriaxone
Incorrect Answer ImageC.Indomethacin
Incorrect Answer ImageD.Nafcillin
Incorrect Answer ImageE.Vancomycin

A 56-year-old African American woman with hyperlipidemia and hypertension comes to the
physician for a routine health maintenance examination. She has been feeling well since her last
visit 6 months ago. She does not exercise regularly but eats a well balanced, healthy diet. She has
had no blurry vision, polyuria or polydipsia. She has had an unintentional 5 kg (11 lb) weight
gain since her last visit. Current medications include amlodipine, atorvastatin, and lisinopril. She
does not smoke, drink alcohol, or use recreational drugs. Family history is positive for type 2
diabetes mellitus in her father and sister. Her blood pressure is 128/84 mm Hg. Body mass index
(BMI) is 26 kg/m2 .Which of the following is the most appropriate screening test for this
patient?
Incorrect Answer ImageA.C-peptide level
Correct Answer ImageB.Fasting plasma glucose level
Incorrect Answer ImageC.Hemoglobin A1c level
Incorrect Answer ImageD.Oral glucose tolerance testing (OGTT)
Incorrect Answer ImageE.Urine glucose measurement

 40-year-old woman comes to the physician because of fever and blood in her urine for 4 days.
She has had no dysuria, urgency, recent infections, or rash. She has a history of asthma
controlled with albuterol. Her mother has type 2 diabetes mellitus and her father died of a
ruptured cerebral aneurysm at age 61 years. Her temperature is 38.2ºC (100.7ºF), pulse is
91/min, respirations are 20/min, and blood pressure is 152/88 mm Hg. Examination shows mild
costovertebral angle tenderness on the right side. Laboratory studies show:
Hematocrit  32% 
Leukocyte count  9,500/mm3
Platelet count  165,000/mm3
Serum studies show:
Na+ 137 mEq/L 
Cl– 103 mEq/L
K+ 4.8 mEq/L 
Blood urea nitrogen (BUN)  32 mg/dL 
Creatinine  1.6 mg/dL 
Glucose  98 mg/dL 
Calcium  8.9 mg/dL 
Urinalysis shows 77 leukocytes/HPF and 710 erythrocytes/HPF; there are no casts. A CT scan of
the abdomen is shown. This patient is at greatest risk for which of the following conditions?
Correct Answer ImageA.Cardiac valve disease
Incorrect Answer ImageB.Ectopia lentis
Incorrect Answer ImageC.Liver cirrhosis
Incorrect Answer ImageD.Pancreatic insufficiency
Incorrect Answer ImageE.Panniculitis

A 43-year-old woman comes to her physician because of fatigue and poor exercise tolerance.
She reports that, during this period, her visits to the gym have decreased in frequency. She is
"exhausted" throughout the day and can barely stay awake until 10 P.M. She also reports joint
stiffness in the morning, which subsides as the day progresses. Her menstrual periods are regular.
She has no history of significant medical illnesses. She takes oral contraceptive pills. She denies
cigarette and alcohol use. Her temperature is 37ºC (98.6ºF), pulse is 80/min, respirations are
16/min, and blood pressure is 125/85 mm Hg. Physical examination shows swollen,
erythematous wrists and metacarpophalangeal and proximal interphalangeal joints. Laboratory
studies show:
Hemoglobin (Hgb) 10 g/dL
Hematocrit (Hct) 31%
Mean corpuscular volume (MCV) 74 μm3
Reticulocyte count 0.5%
Erythrocyte sedimentation rate 50 mm/h
(ESR)
Ferritin, serum 250 ng/dL
Iron 20 μg/dL
Total iron binding capacity (TIBC) 150 μg/dL (N= 240–450
mcg/dL)
Urinalysis Normal
Which of the following is the most appropriate next step in the management of her anemia?
Incorrect Answer ImageA.Bone marrow biopsy
Incorrect Answer ImageB.Discontinue the oral contraceptive pills
Incorrect Answer ImageC.Erythropoietin injections
Incorrect Answer ImageD.Order hemoglobin electrophoresis
Incorrect Answer ImageE.Prescribe oral iron therapy
Correct Answer ImageF.Prescribe oral methotrexate

A 47-year-old man comes to the emergency department because of abdominal pain for 2 weeks.
He takes no medications. He drinks 8 to 10 beers daily. He does not smoke or use illicit drugs.
His temperature is 36.8ºC (98.2ºF), blood pressure is 128/66 mm Hg, pulse is 90/min,
respirations are 20/min, and oxygen saturation is 98% on room air. The lungs are clear to
auscultation. Cardiac examination shows a normal S1 and S2; no murmurs are heard. The
abdomen is distended with tenderness in the right upper quadrant and shifting dullness. There is
2+ peripheral edema bilaterally. Laboratory studies show: 
Hematocrit 34% 
Leukocyte count 6,500/mm3
Platelet count 51,000/mm3
Total protein  5.7 g/dL 
Albumin  2.7 g/dL 
Direct bilirubin  0.8 mg/dL 
Alkaline phosphatase  210 U/L 
Serum:
Na+ 130 mEq/L 
Cl −
98 mEq/L
K+ 4.8 mEq/L 
Blood urea nitrogen (BUN)  18 mg/dL 
Creatinine  1.0 mg/dL 
Glucose  98 mg/dL 
Calcium  8.9 mg/dL 
Aspartate aminotransferase (AST, GOT)  170 U/L 
Alanine aminotransferase (ALT, GPT)  85 U/L 
Amylase  45 U/L 
A CT scan of the abdomen is shown. Which of the following is the most appropriate next step in
the management of this patient?
Incorrect Answer ImageA.Cholecystectomy
Incorrect Answer ImageB.IV antibiotics
Incorrect Answer ImageC.Nasogastric tube
Incorrect Answer ImageD.Pain control and strain his urine
Correct Answer ImageE.Substance abuse counseling

A 66-year-old man comes to the emergency department because of leg weakness and dyspnea.
He has end stage renal disease due to diabetic and hypertensive nephropathy and has been
receiving chronic hemodialysis treatments for the past year. His medications include lisinopril
and nifedipine. He has not adhered to prescribed dietary sodium and potassium restrictions, and
has missed his last two dialysis treatments. His temperature is 37°C (98.6°F), pulse is 90/min,
respiratory rate is 20/min, and blood pressure 186/82 mm Hg. The estimated jugular venous
pressure is 12 cm H2O. There are bibasilar lung crackles, an S3 cardiac gallop sound, 1+ pitting
pretibial edema, and decreased strength in both legs. Laboratory studies show:
Hemoglobin  8.8 g/dL 
Leukocyte count  8,000/mm3
Platelet count  186,000/mm3
Serum studies show:
Na+ 139 mEq/L 
Cl -
100 mEq/L 
K+ 7.9 mEq/L 
HCO3 -
14 mEq/L 
Urea nitrogen (BUN)  156 mg/dL 
Creatinine 7.2 mg/dL
Glucose 112 mg/dL
ECG shows normal sinus rhythm with tall, tented/peaked T waves and a wide QRS complex.
The patient receives one ampule of calcium gluconate, one ampule of 50% dextrose with 10 units
of regular insulin, one ampule of sodium bicarbonate intravenously, as well as an inhaled beta-2-
adrenergic agonist. Which of the following is the most appropriate next step in management of
the patient's potassium abnormality?
Incorrect Answer ImageA.Administer 120 mg of furosemide IV
Incorrect Answer ImageB.Administer patiromer
Correct Answer ImageC.Emergent hemodialysis
Incorrect Answer ImageD.Send the patient home and arrange transportation for outpatient dialysis
Incorrect Answer ImageE.Stop lisinopril and monitor potassium levels

A 43-year-old woman comes to the physician because of irregular menses, lethargy, and feeling
down for the past 4 months. Her last menstrual period was 7 weeks ago. She has also had a 9-kg
(20-lb) unintentional weight gain. She was diagnosed with type 2 diabetes mellitus 6 months ago
which is being managed by lifestyle modifications. Examination shows predominant fat
deposited around her abdomen and trunk with proximal muscle wasting. There is facial edema
without hirsutism or acne. Pelvic examination is unremarkable. Urine pregnancy test is negative.
Which of the following is the most appropriate next step in diagnosis?
Incorrect Answer ImageA.24-hour urine catecholamine and metanephrine levels
Correct Answer ImageB.24-hour urine free cortisol levels
Incorrect Answer ImageC.Adrenocorticotropic hormone (cosyntropin) stimulation test
Incorrect Answer ImageD.Luteinizing hormone and follicle-stimulating hormone levels
Incorrect Answer ImageE.Plasma renin level
Incorrect Answer ImageF.Thyroid-stimulating hormone level
Incorrect Answer ImageG.Urinary sodium level

A 48-year-old alcoholic man admitted after being found stuporous in the park. The patient’s
mental status has improved with restarting his lactulose and discharge is planned for tomorrow.
His temperature is 37.0ºC (98.6ºF), blood pressure is 148/80 mm Hg, pulse is 62/min, and
respirations are 20/min. The patient’s mental status is improved and he does not seem to be in
any distress. Examination shows temporal wasting, numerous chest wall spider angiomas, mild
gynecomastia, and marked hepatosplenomegaly. The physical findings of shifting dullness and
2+ pitting edema bilaterally are also present. The nurse is concerned because the patient’s total
calcium level is 7.0 mg/dL, and she wants to know what to do. Which of the following is the
most appropriate management at this time?
Incorrect Answer ImageA.Administer calcium carbonate, 1–2 g/day
Incorrect Answer ImageB.Administer calcium gluconate, 200 mg (2 ampules) stat
Incorrect Answer ImageC.Administer vitamin D supplementation
Correct Answer ImageD.Check albumin level
Incorrect Answer ImageE.Check magnesium level

A 7-year-old child is brought to the emergency department in an ambulance after she was hit by
a car while crossing the street in front of her home. She was thrown 30 feet away from the site of
impact and was found unconscious at the scene. Physical examination shows a depressed level of
consciousness. She is immediately intubated and admitted in the pediatric intensive care unit.
Two days later, she remains unconscious and continues to require full ventilatory support. The
intensive care team is concerned about the possibility of brain death. Which of the following
physical examination findings is most compatible with a diagnosis of brain death?
Incorrect Answer ImageA.Core temperature 31ºC (87.8ºF)
Incorrect Answer ImageB.Irregularly shaped pupils
Incorrect Answer ImageC.Present cough reflex
Incorrect Answer ImageD.Present gag reflex
Correct Answer ImageE.Pupils unresponsive to light

A 68-year-old man comes to his physician because of a headache, dizziness, blurred vision, and
fatigue. On examination, he appears plethoric; ophthalmoscopic examination shows engorged
retinal veins. His spleen is palpable 3 cm below the left costal margin. Complete blood count
shows a hematocrit of 56% with normal red blood cell morphology, increased red cell mass,
white blood cell count of 16,000/µL with increased basophils and eosinophils, and platelets at
700,000/µL. Which of the following is the major cause of morbidity and death in this illness? 
Incorrect Answer ImageA.Bleeding
Incorrect Answer ImageB.Heart failure
Incorrect Answer ImageC.Opportunistic infection
Incorrect Answer ImageD.Renal failure
Correct Answer ImageE.Thrombosis

A 63-year-old woman comes to the emergency department with chest pain, nausea, shortness of
breath, and sudden onset of dysphagia. The pain radiates to her left jaw and back. It is not
increased or relieved by changing body position. She denies fever, chills, and hemoptysis. She
was diagnosed 7 years ago with hypertension treated with hydrochlorothiazide. Her temperature
is 36.5ºC (97.7ºF), her pulse is 115/min, her respirations are 26/min, and her blood pressure is
160/92 mm Hg. Physical examination shows an elderly woman in severe distress from pain.
Examination shows an apical impulse with a regular rhythm, no rubs, and asymmetric pulses on
the upper extremities. Breath sounds are diminished on the left. ECG shows left ventricular
hypertrophy but no ST segment alterations. A chest radiograph shows a hazy aortic knob with a
mediastinal width of 9 cm. Which of the following is the most appropriate next step in the
management of this patient?
Correct Answer ImageA.Labetalol
Incorrect Answer ImageB.Nitroprusside
Incorrect Answer ImageC.Send for CT angiogram of the chest
Incorrect Answer ImageD.Send for CT scan of the chest without contrast
Incorrect Answer ImageE.Transesophageal echocardiography (TEE)
Incorrect Answer ImageF.Transthoracic echocardiogram

A 56-year-old man is admitted to the hospital because of respiratory distress. An endotracheal


tube is placed for mechanical ventilation at a tidal volume of 900 mL, a respiratory rate of
12/min, and a fraction of inspired oxygen of 50%. The positive end-expiratory pressure is 10 cm
of water. Current medications include subcutaneous heparin and aspirin. Two days later, he
develops tachycardia and has a blood pressure of 80/50 mm Hg. Cardiac examination shows
multiple premature contractions. His arterial blood gas shows a pO2 of 40 mm Hg. Which of the
following is the most likely cause of the patient's condition? 
Incorrect Answer ImageA.Bronchial secretions
Incorrect Answer ImageB.Cardiac arrhythmia
Incorrect Answer ImageC.Myocardial infarction
Correct Answer ImageD.Pneumothorax
Incorrect Answer ImageE.Pulmonary embolus

A 54-year-old woman comes to the physician because of shortness of breath. She has had
progressive dyspnea over the past 8 months and has had difficulty climbing the stairs at her
home. She uses one pillow to sleep at night. She appears cachectic with plethoric facies. She
begins to cough up bloody sputum in the examination room. Physical examination shows a loud
mid-diastolic rumble heard best at the apex on expiration when she lies in the left lateral
position. An ECG shows no abnormalities. An echocardiography shows a mitral valve area of
1.1 cm2 (normal 4–6 cm2). Which of the following is the next best step in management? 
Correct Answer ImageA.Balloon valvotomy
Incorrect Answer ImageB.Enalapril
Incorrect Answer ImageC.Metoprolol
Incorrect Answer ImageD.Valve replacement
Incorrect Answer ImageE.Warfarin

A 63-year-old woman is brought to the emergency department because of focal left lower
quadrant abdominal pain. Her past medical history is remarkable for type 2 diabetes treated with
insulin. On physical examination, she is found to be febrile with a temperature of 101°F
(38.3°C). Her blood pressure is 120/80 mm Hg, pulse is 102/min, and respirations are 14/min.
Head, neck, chest, and cardiovascular examinations are unremarkable. Palpation of the left lower
quadrant produces rebound pain. Computed tomography of the abdomen shows acute
diverticulitis, without perforation or abscess formation, and diffuse fatty infiltration of the liver.
Which of the following is of prognostic importance for this latter finding?
Incorrect Answer ImageA.Alkaline phosphatase level
Incorrect Answer ImageB.Aminotransferase levels
Incorrect Answer ImageC.Hemoglobin A1C

A 63-year-old man comes with 48 hours of mid-abdominal colicky pain, nausea, anorexia, and
vomiting. He has a distant history of deep venous thrombosis and pulmonary embolism
previously treated with anticoagulants. His temperature is 37ºC (98.6ºF), pulse is 102/min, and
blood pressure is 95/60 mm Hg. On physical examination the abdomen is distended and
tympanitic. Abdominal x-ray films show multiple distended loops of small bowel and distension
of the right colon, up to the middle of the transverse colon. Stool is positive for occult blood.
Which of the following is the most appropriate next step in the diagnosis? 
Incorrect Answer ImageA.Colonoscopy
Correct Answer ImageB.CT scan of the abdomen
Incorrect Answer ImageC.Doppler ultrasound
Incorrect Answer ImageD.Exploratory laparotomy
Incorrect Answer ImageE.Upper GI Endoscopy

A 54-year-old obese man comes to the physician for a routine physical examination. He was
diagnosed with type 2 diabetes 1 year earlier. He was started on a regimen of diet and exercise
alone for three months. At that time, his hemoglobin A1c was 8% and he was prescribed
metformin. Three months later, exenatide was added because of continued poor glycemic
control. His other medications are propranolol and lisinopril for hypertension, which he has
taken for the past 5 years. Two weeks before the current visit, naproxen was added for severe
knee pain caused by osteoarthritis. His blood pressure is 154/92 mm Hg, and he has a soft fourth
heart sound. The remainder of the physical examination is normal. At the current visit, the serum
creatinine is 2.8 mg/dL and potassium 5.5 meq/L. One year and three months earlier, the serum
creatinine was 1.6 mg/dL and the potassium was 4.7 meq/L. Which of his medications is most
likely responsible for the increase in creatinine and potassium?
Incorrect Answer ImageA.Exenatide
Incorrect Answer ImageB.Lisinopril
Incorrect Answer ImageC.Metformin
Correct Answer ImageD.Naproxen
Incorrect Answer ImageE.Propranolol

A 30-year-old woman comes to the physician because of fatigue, loss of appetite, cold
intolerance, and constipation for 1 year. She has also felt depression and a loss of sex drive. She
has had a decrease in the growth of her pubic hair for 2 years. Her menstrual cycles have become
irregular for 1 year and she has not had a period in the past 2 months. She has a history of acne
and a difficult labor and delivery 2 years ago. Her blood pressure is 100/65 mm Hg. Examination
shows dry, pale skin and thin, brittle hair. Which of the following is the most likely diagnosis? 
Incorrect Answer ImageA.Pituitary adenoma
Correct Answer ImageB.Pituitary infarction
Incorrect Answer ImageC.Postpartum depression
Incorrect Answer ImageD.Primary adrenal insufficiency
Incorrect Answer ImageE.Primary hypothyroidism

A 39-year-old woman comes to the physician because of a 1-week history of a rash on her legs.
She has a 17-year history of systemic lupus erythematosus. She had 2 episodes of deep vein
thrombosis 4 years ago. She has had 4 miscarriages. Current medications include warfarin and
hydroxychloroquine. Her temperature is 37.2°C (99°F), blood pressure 145/90 mm Hg, and pulse
80/min. Examination shows a violaceous, net-like discoloration of her lower extremities. Platelet
count is 25,000/mm3. Which of the following laboratory abnormalities is most likely in this
patient? 
Incorrect Answer ImageA.Anti-collagen VII antibodies
Incorrect Answer ImageB.Anti-U1RNP antibodies
Incorrect Answer ImageC.Cold agglutinins
Incorrect Answer ImageD.False-negative RPR
Correct Answer ImageE.Prolonged aPTT

A previously healthy 65-year-old man is brought to the physician because of increasingly severe
memory disturbances for 10 months. He has a history of prostate enlargement, for which he is
taking finasteride. Recently, he thinks the medication is no longer working as he has dribbled
continuously for the past few months. His temperature is 36.7ºC (98ºF), blood pressure is 110/80
mm Hg, pulse is 60/min, and respirations are 14/min. Mini-Mental Status examination shows
mild-to-moderate short-term memory deficits. Papilledema is absent on funduscopic
examination, but the physician notices that the patient cannot walk straight and walks with a
broad base. Laboratory studies are normal. Which of the following is the most appropriate next
step in management?
Incorrect Answer ImageA.Lumbar puncture
Correct Answer ImageB.MRI of the brain
Incorrect Answer ImageC.Prostate biopsy
Incorrect Answer ImageD.Suprapubic catheterization
Incorrect Answer ImageE.Treatment with donepezil

A 28-year-old woman comes to her physician because she has had episodes of passing reddish-
brown urine in the morning after getting out of bed. She also reports several small, painful
nodules on her legs, which preceded the problem with her urine. The physician verifies the
presence of these lesions, which are purpuric and painful to palpation. There are no other
positive examination findings. CBC shows hematocrit 29% with reticulocyte count 2.5%. No
diagnostic features of this anemia are seen on the peripheral smear. The following findings are
also present: increased serum LDH, low-normal white-cell and platelet counts, hemosiderin in
the urine, and a decreased leukocyte alkaline phosphatase. The diagnosis may be confirmed by
which of the following findings? 
Incorrect Answer ImageA.Cresyl violet staining of the peripheral smear to find Heinz bodies
Incorrect Answer ImageB.Finding a dry tap on attempted bone-marrow aspiration
Correct Answer ImageC.Flow cytometry demonstrating deficiency of CD59 on red blood cells
Incorrect Answer ImageD.Increased osmotic fragility of red blood cells
Incorrect Answer ImageE.Presence of a monoclonal IgM spike in the beta-globulin region on serum
protein electrophoresis

A 65-year-old woman was brought to the emergency department (ED) with chest pain and
shortness of breath. She has a long history of coronary artery disease (CAD), hypertension,
hyperlipidemia, diabetes mellitus complicated by diabetic retinopathy and nephropathy, anemia
of chronic disease, and lower gastrointestinal bleeding 6 months ago from diverticulosis.
Medications include atenolol, lisinopril, simvastatin, glyburide, and erythropoietin. Soon after
arriving in the ED she loses consciousness and has a non-palpable pulse. She is intubated without
complications and CPR is initiated. After 12 minutes of advanced cardiopulmonary resuscitation
the patient regains sinus rhythm. Her vital signs are temperature 36.5ºC (97.7ºF), blood pressure
is 100/70 mm Hg, and pulse is 65/min. Auscultation of the lungs shows few bibasilar crackles.
Heart sounds are distant without murmurs, rubs, or gallops. Rectal examination shows decreased
rectal tone with brown stool in the rectum, negative for occult blood. Her initial laboratory
workup shows:
WBC 7,300/mm3
Hemoglobin 11.0 g/dL
Hematocrit 31%
Platelets 230,000/mm3
Sodium 134 mmol/L
Potassium 4.9 mmol/L
Calcium 8.5 mg/dL
Creatinine 2.1 mg/dL
BUN 35 mg/dL
Glucose 180 mg/dL
Hemoglobin A1C 7.2%
CK-MB 4.1 ng/mL (normal, 0-3 ng/mL)
Troponin 0.6 ng/mL (normal, <0.4 ng/mL)
Administration of a thrombolytic agent is considered. Which of the following represent a relative
contraindication for thrombolytic therapy in this patient? 
Incorrect Answer ImageA.Age
Incorrect Answer ImageB.Anemia
Incorrect Answer ImageC.Diverticulosis
Incorrect Answer ImageD.Hypotension
Correct Answer ImageE.Prolonged CPR

A 35-year-old woman comes to the emergency department because of shortness of breath since
awakening this morning. She denies any cough, chest pain, or fever, and denies ever feeling this
way before. She has a significant past medical history of a deep venous thrombosis 2 years ago.
She takes oral contraceptives. She states that she smokes one pack of cigarettes per day. Her
temperature is 36.7°C (98.0°F), pulse is 110/min, respirations are 24/min, blood pressure is
110/60 mm Hg, and oxygen saturation is 95% on room air. A ventilation-perfusion scan is
reported as "high probability" of a pulmonary embolism. Which of the following is the most
likely electrocardiographic finding?
Incorrect Answer ImageA.Atrial fibrillation
Incorrect Answer ImageB.Right axis deviation
Incorrect Answer ImageC.Right bundle branch block
Incorrect Answer ImageD.S wave in lead I, Q wave in lead III, inverted T wave in lead III
Correct Answer ImageE.Sinus tachycardia

A 63-year-old woman is brought to the hospital because of an acute inferior myocardial


infarction. She is oliguric and has a blood pressure of 80/55 mm Hg. A Swan-Ganz catheter is
placed, which shows an increased right atrial pressure of 11 mm Hg, normal pulmonary artery
pressure of 22/4 mm Hg, and a diminished pulmonary capillary wedge of 4 mm Hg. Which of
the following is the most appropriate next step in management?
Incorrect Answer ImageA.Balloon angioplasty
Incorrect Answer ImageB.Digoxin
Correct Answer ImageC.Fluids
Incorrect Answer ImageD.Intra-aortic balloon counterpulsation
Incorrect Answer ImageE.Vasopressors

A 64-year-old man comes to the physician because of a 3-day history of dark, foul-smelling
stools and weakness. He reports burning abdominal and chest pain that is partially relieved with
eating. He denies vomiting, hematemesis, diarrhea, or constipation. He has a history of hepatitis
C infection and cirrhosis. He drinks 4 to 5 beers daily. His blood pressure is 110/78 mm Hg
when sitting and 95/60 mm Hg when standing, and his pulse is 90/min when sitting and 110/min
when standing. Examination shows spider angiomata and palmar erythema. The spleen tip is
palpated 3 cm below the left costal margin. Rectal examination shows fecal occult blood. His
hemoglobin is 8.6 g/dL. Which of the following is the most likely cause of this patient's
bleeding?
Incorrect Answer ImageA.Diverticular disease
Correct Answer ImageB.Duodenal ulcer
Incorrect Answer ImageC.Esophageal varices
Incorrect Answer ImageD.Gastric ulcer
Incorrect Answer ImageE.Portal hypertensive gastropathy

A 32-year-old woman comes to the emergency with 3 hours of severe right abdominal pain,
which just stopped in the ED waiting area. She first noticed it after lifting boxes, and felt it was a
muscle strain. The pain was sharp, radiated to her groin, and "comes and goes." When the pain
was most severe, she was unable to catch her breath. She denies any fever, chills, or prior similar
episodes. She does complain of urinary urgency and frequency but denies dysuria or hematuria.
Her last bowel movement was yesterday and was normal, and her last menstrual period was
normal, two weeks ago. She is comfortable. Her temperature is 37.1ºC (98.8ºF), pulse is 75/min,
and blood pressure is 125/85 mm Hg. Heart and lung examinations are normal. Abdominal
examination reveals mild middle right-sided abdominal tenderness without rebound. The pelvic
exam is normal. A pregnancy test is negative. A urinalysis shows many red blood cells, but no
crystals or casts. Which of the following is the most likely complication of the patient’s disease? 
Incorrect Answer ImageA.Blood pressure falls
Incorrect Answer ImageB.Hematocrit falls
Correct Answer ImageC.Recurrent abdominal pain
Incorrect Answer ImageD.Serum creatinine rises
Incorrect Answer ImageE.Temperature rises

A 35-year-old man is brought to the emergency department by his wife because of progressive
stupor and confusion for the past 3 days. His wife reports that for the past few days, she has
noticed excessive thirst, urination, and progressive confusion in her husband. His past medical
history is significant for type 1 diabetes mellitus, for which he has been taking insulin. He was
recently diagnosed with a left lower lobe pneumonia, which was treated with antibiotics one
week ago. His temperature is 37.1°C (98.8°F), pulse is 120/min, respirations are 28/min, and
blood pressure is 85/55 mm Hg. Oxygen saturation is 98% on room air. On physical
examination, he is minimally arousable and breathing rapidly. His mucous membranes are dry,
and skin turgor is poor. Abdominal examination is unremarkable. Laboratory data shows:
White blood cells 20,200/mm3
Hematocrit 48%
Platelets 459,000/mm3
Sodium 143 mEq/L
Potassium 4.9 mEq/L
Chloride 108 mEq/L
Bicarbonate 15 mEq/L 
Urea nitrogen 25 mg/dL
Creatinine 1.1 mg/dL
Glucose 525 mg/dL
Urinalysis positive for ketones
He is given 10 units of IV insulin along with 1 liter of 0.9% normal saline and potassium
replacement. Following that, he is started on an insulin infusion. Which of the following should
guide conversion from IV to subcutaneous insulin?
Correct Answer ImageA.Anion gap
Incorrect Answer ImageB.Level of consciousness
Incorrect Answer ImageC.Serum glucose level <250 mg/dL
Incorrect Answer ImageD.Serum ketone level
Incorrect Answer ImageE.Urine ketone level

A 22-year-old African American man comes to the physician because of fatigue and mild
jaundice. He is preparing to go on a youth mission to rural Guatemala, and he saw his primary
doctor yesterday for vaccinations and other travel medications. This morning he noted that he
developed severe abdominal pain after starting prophylactic medication, but he does not recall
the medication’s names. He also became progressively weak and dizzy after this episode. When
he went to the bathroom, his urine had become darker than usual. He denies any other medical
problems and denies any recent substance use. His temperature is 38.3ºC (101.0ºF), blood
pressure is 105/45 mm Hg, pulse is 130/min, and respirations are 22/min. Physical examination
shows a fit man with some increased work of breathing. Sclerae and mucous membranes are
noted to be slightly icteric. His skin is also slightly jaundiced. There is a 3/6 systolic flow
murmur, an enlarged liver, and a palpable spleen tip with diffuse abdominal tenderness. Which
of the following represents the most likely diagnosis?
Incorrect Answer ImageA.Autosomal dominant
Incorrect Answer ImageB.Autosomal recessive
Incorrect Answer ImageC.Spontaneous mutation
Incorrect Answer ImageD.X-linked dominant
Correct Answer ImageE.X-linked recessive

A 36-year-old man comes to the physician because of progressive loss of sensation and tingling
in his arms for 4 months. He has had no motor weakness or bowel/bladder dysfunction. He
reports some mild swallowing difficulty. He has a history of gastroesophageal reflux treated with
pantoprazole. He has smoked one pack of cigarettes daily for 8 years. He does not drink alcohol,
and he does not use illicit drugs. His temperature is 36.9°C (98.4°F), blood pressure is 124/76
mm Hg, and pulse is 68/min. Examination shows a supple neck. Neurologic examination shows
decreased pain and temperature sensation of the upper extremities with normal vibration and
position sense. Muscle strength is 4/5 throughout. Deep tendon reflexes are slightly hyperactive.
Tone is increased in the lower extremities. An MRI of the brain is most likely to show which of
the following abnormalities?
Correct Answer ImageA.Arnold-Chiari malformation
Incorrect Answer ImageB.Cerebellar atrophy
Incorrect Answer ImageC.Cyst in the third ventricle
Incorrect Answer ImageD.Parasagittal meningioma
Incorrect Answer ImageE.Vestibular schwannoma

An otherwise healthy 42-year-old man presents to the emergency department with a nosebleed.
He states that the bleeding started 30 minutes ago and has not stopped since. He has no history of
nosebleeds. He recently had an ankle injury and has been taking ibuprofen 600 mg every 4–6
hours for the last 4 days. The patient denies any fever, chills, dizziness, or weakness. His sister-
in-law has von Willebrand disease. He does not smoke, drink, or take illegal drugs. His
temperature is 37.0º C (98.6º F), pulse is 72/min, respirations are 13/min, and blood pressure is
120/80 mm Hg. On physical examination, the patient has continuous bleeding from the left
nares. Pressure is applied and the bleeding improves slightly. Laboratory tests are sent while
anterior packing is performed and return as follows: 
Hemoglobin 14 g/dL
Hematocrit 42%
WBC count 5,000/mm3
Platelets 200,000/mm3
PT 14 sec
PTT 32 sec
INR 1.0 (normal: 0.8–1.2)
Bleeding time 10 min
Which of the following is the most likely cause of this patient's bleeding? 
Incorrect Answer ImageA.Decreased prostaglandin D2
Incorrect Answer ImageB.Decreased prostaglandin E2
Correct Answer ImageC.Decreased thromboxane A2
Incorrect Answer ImageD.Increased prostaglandin E2
Incorrect Answer ImageE.Inherited defect of platelet aggregation

A 48-year-old avid runner and mountain climber comes to the physician to discuss proper
vaccination before leaving for Africa for a cross-training event. He mentions that he has always
been healthy and does not take any regular medications except for multivitamins. He
occasionally gets lightheaded at the end of his runs but has never passed out. He does not have
any symptoms currently and feels well. His temperature is 37.0ºC (98.6ºF), pulse is 54/min,
respirations are 12/min, and blood pressure is 105/60 mm Hg. Laboratory studies including
electrolytes, TSH, and free thyroxine are within normal limits. An ECG taken in the office is
shown. Which of the following is the most appropriate next step in management?
Incorrect Answer ImageA.Administer atropine
Incorrect Answer ImageB.Administer dopamine
Incorrect Answer ImageC.Administer proper vaccination
Incorrect Answer ImageD.Place the patient on a 24-hour Holter monitor
Correct Answer ImageE.Transvenous pacemaker

A 67-year-old man comes to the office because of worsening shortness of breath and cough. The
patient was discharged from the hospital recently after being admitted with an exacerbation of
COPD. He is a lifelong smoker, with an 80-pack-year smoking history. He also has a history of
hypertension and coronary artery disease. The patient currently takes ipratropium, captopril,
aspirin, and nitroglycerin. His vitals are temperature 37.0°C (98.6°F), blood pressure is 145/85
mm Hg, pulse is 98/min, and respirations are 22/min. Physical examination shows decreased
breath sounds and occasional wheezing bilaterally. Cardiovascular exam shows normal heart
sounds. Abdomen is soft, nontender, and nondistended. There is no clubbing, cyanosis, or
edema. Arterial blood gas analysis shows pH 7.35, pCO2 55 mm Hg, pO2 67 mm Hg, and
HCO3– 30 mEq/L. Chest x-ray shows flattening of diaphragm. The patient underwent pulmonary
function testing, which showed the presence of obstructive lung disease. Which of the following
is the best prognostic indicator in this patient?
Incorrect Answer ImageA.Degree of hypercapnia
Incorrect Answer ImageB.Degree of hypoxemia
Incorrect Answer ImageC.Duration of smoking in pack-years
Correct Answer ImageD.Forced expiratory vital capacity in the first second (FEV1)
Incorrect Answer ImageE.Frequency of exacerbations

A 66-year-old stand-up comedian is brought to the emergency department because of chest pain.
Two hours ago, he had been on stage as part of an improvisation act when halfway through a
scene, he suddenly experienced a dull, tightening pain that radiated to his left arm. Embarrassed,
he had to walk offstage, and one of the crew called 911. He states that he has had similar pain
before, but not as severe. He also reports shortness of breath during this episode. He currently
takes no medication. He is a 45 pack-year smoker and drinks 30 units of alcohol a week. His
temperature is 37.6ºC (99.6ºF), pulse is 120/min, and blood pressure is 165/96 mm Hg. He is
currently breathing 100% oxygen through a face mask. An electrocardiogram shows the absence
of ST-segment elevation and the patient is started on aspirin, atorvastatin, enoxaparin,
metoprolol, nitrates, and morphine. CK-MB and troponins are markedly elevated. He continues
to endorse symptoms of chest pressure and repeat ECG shows worsening ST depressions. Which
of the following is the most appropriate next step in management?
Incorrect Answer ImageA.Administer IV diazepam
Incorrect Answer ImageB.Administer tissue plasminogen activator
Incorrect Answer ImageC.Coronary artery bypass grafting
Correct Answer ImageD.Coronary angiogram
Incorrect Answer ImageE.Prescribe nifedipine
Incorrect Answer ImageF.Prescribe enalapril

A 21-year-old man comes to the physician because of a 2-month history of frequent episodes of
abdominal cramping and loose stools. He has had 2 to 3 episodes daily. The stools have become
increasingly bloody during the last month. He has had the sensation of rectal fullness but has
been unable to pass any fecal matter. He has not traveled outside of the country and has no sick
contacts. His temperature is 36.9°C (98.4°F), blood pressure is 118/75 mm Hg, and pulse is
66/min. Sigmoidoscopy shows inflammation in a circumferential pattern from the anal verge to
the mid-sigmoid colon, where a transition to normal mucosa is seen. Which of the following is
the most appropriate pharmacotherapy?
Incorrect Answer ImageA.IV hydrocortisone
Incorrect Answer ImageB.Metronidazole
Incorrect Answer ImageC.Oral azathioprine
Incorrect Answer ImageD.Oral prednisone
Correct Answer ImageE.Rectal mesalamine enema

A 63-year-old man comes to the physician for a routine medical examination. He has no
symptoms. He takes metformin for diabetes mellitus but does not check his glucose at home. He
has a distant history of chest pains after eating but has had none in 3 years. He has a history of
hypertension but has not taken his antihypertensive medication in 6 months. He has a 20 pack-
year smoking history, but quit smoking 1 year ago. He drinks three beers per day. Heart rate is
84/min and blood pressure is 159/79 mm Hg. On examination, a fourth heart sound is heard.
There are crackles at his lung bases and 1+ pitting edema at the ankles. Serum studies show:
Na+ 136 mEq/L
K +
5.6 mEq/L
Cl– 108 mEq/L
HCO3 –
18 mEq/L
Creatinine 1.7 mg/dL
Blood urea nitrogen 19 mg/dL
What is the most likely explanation for this patient's serum abnormalities?
Incorrect Answer ImageA.Gastrointestinal bicarbonate loss
Incorrect Answer ImageB.Type 1 renal tubular acidosis
Incorrect Answer ImageC.Type 2 renal tubular acidosis
Correct Answer ImageD.Type 4 renal tubular acidosis
Incorrect Answer ImageE.Uremia

A 68-year-old woman comes to the emergency department because of nausea, vomiting, and
abdominal pain for 24 hours. She denies fever, chills, or rigors. She was recently diagnosed with
multiple myeloma and she has a history of rheumatoid arthritis, hypertension, and
hypercholesterolemia. She has had no chemotherapy yet. Her temperature is 36.7°C (98°F),
pulse is 110/min, respirations are 16/min, and blood pressure is 90/60 mm Hg. Examination
shows dry oral mucosa and decreased skin turgor. The lungs are clear to auscultation. Cardiac
examination shows normal S1 and S2; no murmurs are heard. There is no peripheral edema.
Laboratory studies show: 
Hemoglobin 16.8 g/dL
Leukocyte count  8,000/mm3 
Platelet count  170,000/mm3
Na+ 134 mEq/L
Cl- 101 mEq/L
K + 
4.5 mEq/L
HCO3- 25 mEq/L
Urea nitrogen (BUN)  60 mg/dL
Creatinine  1.4 mg/dL
Glucose 80 mg/dL
Calcium 13.2 mg/dL
Albumin 4.2 g/dL
Which of the following is the most appropriate next step in management of this patient's
hypercalcemia? 
Incorrect Answer ImageA.Bisphosphonate
Incorrect Answer ImageB.Calcitonin
Incorrect Answer ImageC.Hydrochlorothiazide
Incorrect Answer ImageD.IV furosemide
Correct Answer ImageE.IV normal saline
Incorrect Answer ImageF.Oral calcium carbonate
Incorrect Answer ImageG.Melphalan immediately
Incorrect Answer ImageH.Prednisone immediately

A previously healthy 19-year-old woman is admitted to the hospital burn unit after being brought
to the emergency department 45 minutes after being rescued from a house fire. She weighs 50 kg
(110 lb). Her temperature is 37.0ºC (98.6ºF), pulse is 108/min, respirations are 28/min, and blood
pressure is 89/50 mm Hg. Examination shows severe burns covering 60% of her body surface
area. She is treated with aggressive IV hydration with lactated Ringers, wound debridement,
decompressive escharotomy, and topical silver sulfadiazine. A Foley catheter is placed and urine
output over each of the last 3 hours has been 50 mL/h, 55 mL/h, and 55 mL/h. ECG shows
markedly peaked T waves and a widened, almost wavelike QRS complex. Which of the
following is the most likely cause of this patient's findings?
Incorrect Answer ImageA.Medications
Incorrect Answer ImageB.Metabolic acidosis
Incorrect Answer ImageC.Sepsis
Incorrect Answer ImageD.Severe dehydration
Correct Answer ImageE.Tissue catabolism

 27-year-old man comes to the emergency department because of a 1-week history of progressive
weakness and tingling that began in his feet and spread to his thighs and upper limbs. He has
been unsteady on his feet because of the weakness. He had diarrhea 4 weeks ago. His medical
history is otherwise unremarkable, and he takes no medications. He is alert and oriented. Muscle
strength is 3/5 bilaterally in the lower extremities. There is areflexia. Which of the following is
the most appropriate diagnostic test in this patient?
Correct Answer ImageA.Cerebrospinal fluid analysis
Incorrect Answer ImageB.Magnetic resonance image of the spine
Incorrect Answer ImageC.Muscle biopsy
Incorrect Answer ImageD.Nerve conduction studies
Incorrect Answer ImageE.Serum titers for acetylcholine receptor antibodies

A 65-year-old man is admitted to the hospital for worsening headache, light-headedness, blurry
vision and fatigue over the last 3 months. He also reports weakness and generalized itching. On
examination, he is in no apparent discomfort. His temperature is 37ºC (98.6ºF), pulse is 98/min,
and blood pressure is 137/84 mm Hg. There is facial plethora and diffuse linear scratch marks
that do not appear to have secondary infection. His spleen is 4 cm below the left costal margin.
Laboratory studies show the following:
Hematocrit 62%
Hemoglobin 20 g/dL
WBC count 18,000/mm3
Platelet count 550,000/mm3
Serum erythropoietin 1 mU/mL (normal: 5–20 mU/mL)
Leukocyte alkaline phosphate 90 U/L
Which of the following is the most likely diagnosis?
Incorrect Answer ImageA.Chronic myeloid leukemia
Incorrect Answer ImageB.Myelofibrosis
Incorrect Answer ImageC.Myelodysplastic syndromes
Correct Answer ImageD.Primary polycythemia
Incorrect Answer ImageE.Secondary polycythemia

A 70-year-old man is brought to the emergency department after being found unresponsive at
home. On arrival, his temperature is 39.5°C (103°F), pulse is 120/min, respirations are 30/min,
and blood pressure is 75/45 mm Hg. Bilateral rales are heard on auscultation. Cardiac
examination shows inaudible heart sounds. An ECG shows sinus tachycardia. His urine output is
10 mL/h. An x-ray film of the chest is shown. A right heart catheterization shows a right atrial
pressure of 6 mm Hg, pulmonary capillary wedge pressure of 25 mm Hg, and a cardiac output of
2.5 L/min. He is administered antibiotics. Which of the following is the most likely cause of the
hypotension?
Incorrect Answer ImageA.Gastrointestinal bleeding
Incorrect Answer ImageB.Gram-negative sepsis
Correct Answer ImageC.Left ventricular dysfunction
Incorrect Answer ImageD.Pericardial tamponade
Incorrect Answer ImageE.Pulmonary embolus

A 40-year-old female was brought to the clinic because of difficulty breathing. The patient is a
known asthmatic and was treated successfully with albuterol in each episode until this one. She
feels fine between episodes and does not have any other medical problems. The patient’s
daughter reports that her mother has allergies to cats. Her temperature is 36.7ºC (98.4ºF), pulse is
98/min, respirations are 22/min, and blood pressure 120/60 mm Hg. Physical examination of the
lungs demonstrates (click media file). The rest of her physical examination is unremarkable.
Pulmonary function testing is ordered. Which of the following sets of pulmonary function tests is
consistent with bronchial asthma?
A. 51 82 114 74

B. 55 85 102 95

C. 60 74 72 62

D. 62 76 73 74

E. 96 89 93 88

Forced Forced vital Total lung Carbon


expiratory capacity, % capacity, % monoxide
volume in first of predicted of predicted diffusing
second, % of (FVC) (TLC) capacity, % of
predicted predicted
(FEV1) (DLCO)
Incorrect Answer ImageA.A
Correct Answer ImageB.B
Incorrect Answer ImageC.C
Incorrect Answer ImageD.D
Incorrect Answer ImageE.E

A 56-year-old woman comes to the physician for a physical examination required for health
insurance. She has a history of ischemic heart disease. She states that she feels relatively well
since her percutaneous coronary intervention 4 years ago. Since that time, she has stopped
smoking cigarettes, avoids eating red meat and foods high in saturated fat, and runs 3 miles on a
treadmill 3 days a week. For the past 1 month, however, she has gotten severely short of breath
within the first 5 minutes of running. She used to smoke 2 packs of cigarettes daily for 35 years
and drink 2 glasses of red wine a night. Her pulse is 94/min and blood pressure is 145/95 mm
Hg. Cardiopulmonary exam shows minimal crackles at the bases. There is a grade 2/6
pansystolic murmur, heard best at the apex and radiating to the axilla. ECG shows Q waves in
the inferior and lateral leads. Echocardiogram confirms your diagnosis. Which of the following
is the most appropriate initial step in management?
Correct Answer ImageA.Enalapril and furosemide
Incorrect Answer ImageB.Furosemide
Incorrect Answer ImageC.Left heart catheterization and coronary angiography
Incorrect Answer ImageD.Mitral valve replacement
Incorrect Answer ImageE.Mitral valve repair

A 42-year-old woman with long-standing ulcerative colitis well controlled with sulfasalazine
comes to the clinic because of a two month history of worsening yellow eyes, progressive
fatigue, and itchiness. Over the past two months she reports the inability to perform her usual
activities due to fatigue. Her skin has become progressively more pruritic, for which she has self-
medicated with over-the-counter corticosteroids and antihistamine ointments with minimal relief.
There is a past medical history of hypothyroidism and Raynaud's syndrome diagnosed 5 years
ago. She was hospitalized last year with fever and jaundice treated with antibiotics. Vital signs
are within normal limits. Skin examination shows multiple excoriations and jaundice. There is
scleral icterus. Abdominal examination shows marked hepatosplenomegaly. The urine is dark
and bilirubin is positive. Serum studies show: 
Alanine aminotransferase (ALT) 42 U/L
Alkaline phosphatase 1,200 U/L
Aspartate aminotransferase (AST) 42 U/L
Bilirubin, total 4.0 mg/dL
Antinuclear antibodies positive
Anti-mitochondrial antibodies  negative
Anti-Smith antibodies negative
Anti-smooth muscle antibodies positive
Cytoplasmic antineutrophil cytoplasmic antibodies negative
(c-ANCA)
Perinuclear antineutrophil cytoplasmic antibodies positive
(p-ANCA)
Which of the following diagnoses is most likely?
Incorrect Answer ImageA.Autoimmune hepatitis
Incorrect Answer ImageB.Polyarteritis nodosa
Incorrect Answer ImageC.Primary biliary cirrhosis
Correct Answer ImageD.Primary sclerosing cholangitis
Incorrect Answer ImageE.Systemic lupus erythematosus
A 65-year-old man comes to the clinic complaining of general weakness and shortness of breath.
His past medical history is significant for hypertension, coronary artery disease, hyperlipidemia,
and diabetes mellitus type 2 complicated by diabetic nephropathy and diabetic neuropathy.
Medications include aspirin, nitroglycerin, hydrochlorothiazide, and NPH insulin. His
temperature is 36.7°C (98.0°F), pulse is 104/min, respirations are 20/min, blood pressure is
144/84 mm Hg, and oxygen saturation is 96% in room air. Physical exam shows an elderly male
in no acute distress. The heart, lung, and abdominal exams are normal. He has trace pedal edema
and decreased sensation in the feet bilaterally. The rest of the physical exam is unremarkable.
Laboratory analysis shows: 
ABG
pH 7.35
PaCO2 36 mm Hg
PaO2 94 mm Hg
Serum
Na+ 136 mEq/L
K+ 5.6 mEq/L
Cl –
110 mEq/L
HCO3– 16 mEq/L
BUN 28 mg/dL
Creatinine 1.5 mg/dL
Glucose 116 mg/dL
Ca 2+
9.5 mg/dL
Lactic acid 1.7 mEq/L (normal, 0.5–2.0 mEq/L)
Urinalysis shows pH 4.7 and specific gravity 1.018, and is negative for glucose, ketones, RBCs,
and WBCs. Which of the following is the most likely cause of acidemia in this patient? 
Correct Answer ImageA.Hyporeninemic hypoaldosteronism
Incorrect Answer ImageB.Ketoacidosis
Incorrect Answer ImageC.Lactic acidosis
Incorrect Answer ImageD.Reduced kidney proximal tubular bicarbonate reabsorption
Incorrect Answer ImageE.Reduced urinary acidification (inability to maximally lower urine pH)

A 34-year-old woman comes to her physician because of a 2-month history of anxiety, insomnia,
and irritability. She reports an unintentional 5 kg (11 lb) weight loss over this period. She denies
heat intolerance, dysphagia, changes in vision, fever, and gastrointestinal disturbances. Her past
medical history is significant for premenstrual dysphoric disorder (PMDD), for which she takes
fluoxetine. She has no family history of medical illnesses. She denies alcohol and illicit drug use.
She is divorced and currently unemployed. Physical examination is shown. Laboratory studies
show a thyroid-stimulating hormone level (TSH) of 0.05 µU/mL. Radioactive iodine uptake
(RAIU) shows diffuse, homogeneously increased iodine uptake throughout the thyroid gland.
Which of the following is the most appropriate definitive treatment for this patient's condition?
Incorrect Answer ImageA.Buspirone
Incorrect Answer ImageB.Lorazepam
Incorrect Answer ImageC.Propylthiouracil
Correct Answer ImageD.Radioactive iodine ablation
Incorrect Answer ImageE.Surgery

A 33-year-old man comes to the physician because of stabbing headaches behind his left eye
which have occurred several times per day for the past 2 weeks and last 15 to 60 minutes. He
denies photophobia, nausea, or seizures. In the past, these episodes of headaches have lasted up
to 6 weeks. He appears agitated. His temperature is 37.3ºC (99.2ºF), blood pressure is 145/90
mmHg, pulse is 98/min, and respirations are 17/min. Examination shows tearing of the left eye.
The left pupil measures 2 mm, the right pupil is 4 mm, and the left eyelid is slightly drooped.
Visual acuity is normal. Which of the following is the most appropriate next step in
management?
Incorrect Answer ImageA.MRI brain
Correct Answer ImageB.Oxygen
Incorrect Answer ImageC.STAT CT head
Incorrect Answer ImageD.Sumatriptan
Incorrect Answer ImageE.Verapamil

A 34-year-old woman, gravida 2 para 2, comes to the physician 6 weeks after the delivery of a
healthy infant girl. She developed gestational diabetes mellitus (GDM) during the 2nd trimester
of her pregnancy which was managed with insulin. She had a normal vaginal delivery without
complications. She is currently breastfeeding her child and takes no medications. The results of a
75-g, 2-hour oral glucose tolerance test (OGTT) performed today are shown:
Fasting glucose 150 mg/dL
2-h plasma glucose 210 mg/dL
Which of the following is the most likely diagnosis in this patient? 
Incorrect Answer ImageA.Impaired fasting glucose
Incorrect Answer ImageB.Impaired glucose tolerance
Incorrect Answer ImageC.Persistent gestational diabetes
Incorrect Answer ImageD.Resolved gestational diabetes
Correct Answer ImageE.Type 2 diabetes mellitus

A 78-year-old man with a history of diabetes presents to his physician because of chest pain and
shortness of breath. Over the past 6 months, he has been experiencing dull chest pain and
dyspnea every time he walks uphill to his home. The pain and dyspnea are relieved by rest. He
denies chest pain at rest. He is a 55-pack-year smoker. Vital signs are temperature 37.0°C
(98.6°F), pulse 60/min, and blood pressure 135/82 mm Hg. On physical examination the patient
is comfortable. There is no jugular venous distension. Cardiac examination shows normal heart
sounds, regular rate and rhythm, and no murmurs, rubs, or gallops. Breath sounds are clear
bilaterally. Current medications include NPH insulin and captopril. The patient undergoes a
stress test, which shows ST-segment depression of 1 mm in leads III and aVF. He is sent home
on aspirin, nitrates, metoprolol, and atorvastatin. He is also advised on diet modification. Despite
this treatment regimen, he continues to complain of symptoms. The patient undergoes coronary
angiography, which shows significant disease in the proximal left circumflex coronary artery and
the right coronary artery. Which of the following is the best next step in management?
Correct Answer ImageA.Coronary artery bypass grafting
Incorrect Answer ImageB.Dobutamine echocardiography
Incorrect Answer ImageC.Percutaneous transluminal coronary angioplasty with stent insertion
Incorrect Answer ImageD.Stress echocardiography
Incorrect Answer ImageE.
Stress thallium scan

A 19-year-old college freshman comes to the university health clinic because of recurrent
episodes of wheezing during basketball practices. He has been a starting shooting guard for this
team and has symptoms of shortness of breath shortly after beginning practice and during games,
accompanied by a nonproductive cough and chest tightness. He denies any rest symptoms. They
occur regardless of whether the practices are held indoors or outdoors. His physical examination
is within normal limits. Which of the following cells are most likely to mediate his symptoms?
Incorrect Answer ImageA.Eosinophils
Incorrect Answer ImageB.Lymphocytes
Correct Answer ImageC.Mast cells
Incorrect Answer ImageD.Monocytes
Incorrect Answer ImageE.Neutrophils

A 33-year-old woman comes to the physician's office because of progressive fatigue, mild
shortness of breath during activity, and pale skin for 1 year. She states that her mood is good and
her appetite has been unchanged. She has had no hematemesis, hemoptysis, melena,
hematochezia, or hematuria. She has normal menstruation. She denies fevers or weight loss. Her
past medical history is unremarkable and she takes no medications. She does not smoke or drink
alcohol. Her temperature is 37.2°C (99°F), pulse is 90/min, and blood pressure is 124/74 mm Hg.
Examination shows pale mucous membranes. No lymphadenopathy is appreciated. The lungs are
clear to auscultation and a grade 2/6 early systolic murmur is heard over the left sternal border.
There is no hepatosplenomegaly, clubbing, or cyanosis of the extremities. Laboratory studies
show: 
WBC  6,700/mm3
Hematocrit  29%
Platelets  225,000/mm3
Blood smear  Microcytosis, target red blood cells
Which of the following is the most appropriate diagnostic test? 
Incorrect Answer ImageA.Colonoscopy
Correct Answer ImageB.Hemoglobin electrophoresis
Incorrect Answer ImageC.Reticulocyte count
Incorrect Answer ImageD.Serum ferritin
Incorrect Answer ImageE.Serum lead level

A 63-year-old man in apparent good health is convinced after reading in the news that he will
live longer if he takes one aspirin tablet every day. After approximately 3 weeks of taking a daily
regular aspirin, he began to notice bright red blood on the toilet paper after his bowel movements
when he had to strain. He denies a history of hemorrhoids or family history of colon cancer.
Anoscopy and digital rectal exam show external and internal hemorrhoids, neither of which are
bleeding at the time of the examination. Which of the following is the most appropriate next step
in management?
Incorrect Answer ImageA.Flexible sigmoidoscopy
Incorrect Answer ImageB.Rubber-band ligation of the internal hemorrhoids
Incorrect Answer ImageC.Stool softeners
Correct Answer ImageD.Total colonoscopy
Incorrect Answer ImageE.Upper endoscopy

A 26-year-old man is brought to the emergency department because he was muttering


incoherently at his job and lying in a field. He has had urinary and bowel incontinence. He
appears diaphoretic, drooling, and confused. His blood pressure is 85/50 mm Hg, pulse is
50/min, respirations are 35/min, and oxygen saturation is 96% on room air. Neurologic
examination shows 1-mm pupils; muscle strength is 4/5 in all extremities. Laboratory studies
show: 
pH  7.2
pO2 94 mm Hg
pCO2 52 mm Hg
HCO3− 18 mg/dL
Plasma osmolality 282 mosmol/kg 
Which of the following is the most appropriate pharmacotherapy for this patient? 
Incorrect Answer ImageA.Atropine
Correct Answer ImageB.Atropine and pralidoxime (2-PAM)
Incorrect Answer ImageC.Benztropine and 2-PAM
Incorrect Answer ImageD.Fomepizole
Incorrect Answer ImageE.Malathion
Incorrect Answer ImageF.Physostigmine
Incorrect Answer ImageG.Physostigmine and 2-PAM
Incorrect Answer ImageH.Pralidoxime

A 35-year-old man comes to the emergency department because of fever and chest pain. He
reports having sharp chest pain with inspiration, a small amount of hemoptysis, and chills. The
emergency staff members know the man well, as he has been admitted numerous times for
alcohol withdrawal, pancreatitis, cocaine-induced psychosis, and disseminated gonorrhea. In
addition to alcohol and cocaine, the man has a long history of IV heroin abuse. His temperature
is 38.3ºC (101.0ºF), blood pressure is 158/78 mm Hg, pulse is 110/min, and respirations are
24/min. Examination of the chest shows scattered crackles but no dullness to percussion or
egophony. Cardiac auscultation is remarkable for a 2/6 systolic murmur, best heard at the lower
left sternal border. The murmur increases slightly with inspiration. The patient's arms are notable
for injection-site trauma, and small, painless nodules are present on his hands. Routine laboratory
studies, electrocardiogram, blood cultures, and chest radiograph are ordered in the emergency
department. Which of the following is most likely to be a radiographic finding in this patient?
Incorrect Answer ImageA.Bilateral patchy interstitial infiltrates
Incorrect Answer ImageB.Enlarged cardiac silhouette
Incorrect Answer ImageC.Focal lobar consolidation
Correct Answer ImageD.Multiple bilateral nodular densities
Incorrect Answer ImageE.Peribronchial cuffing and Kerley B lines

A 66-year-old wheelchair-bound woman with a history of multiple sclerosis comes to the


physician because of dysuria, frequent urination, intermittent gross hematuria, and chronic low
back pain. The pain is constant, non-radiating, and located over her flanks bilaterally. In the past
12 months, she has had three urinary tract infections and one hospitalization for acute
pyelonephritis of the left kidney. She also has arthritis, hypertension controlled with an ACE
inhibitor, and a hysterectomy performed 25 years ago for fibroids. On physical examination, she
is in no distress. Her temperature is 36.7°C (98°F), pulse is 85/minute, respirations are
12/minute, and blood pressure is 130/70 mm Hg. Heart and lung examinations are within normal
limits. Her abdomen is soft and nondistended without guarding and there is mild bilateral
costovertebral angle tenderness. Pelvic and rectal examinations are normal. Urine studies show:
pH 8.0
Specific gravity 1.010
Proteins 1+
RBC 30–40/hpf
WBC 75–90/hpf
Nitrite Positive
Leukocyte esterase Positive
A plain abdominal x-ray (KUB) shows large calcifications in the renal pelvis and calyces
bilaterally. Which of the following is the most likely composition of these calcifications?
Incorrect Answer ImageA.Calcium oxalate dihydrate
Incorrect Answer ImageB.Calcium oxalate monohydrate
Incorrect Answer ImageC.Cystine
Correct Answer ImageD.Magnesium-ammonium-phosphate
Incorrect Answer ImageE.Uric acid

A 65-year-old man comes to the physician because of difficulty lifting his arms. He sustained
fractures to his left tibia and fibula in a motor vehicle accident 6 weeks ago and has required the
use of crutches since that time. He has had no other weakness, seizures, loss of consciousness,
back pain, or episodes of incontinence. Examination is shown (see media clip). Which of the
following nerves is most likely affected? 
Incorrect Answer ImageA.Axillary nerve
Incorrect Answer ImageB.Brachial plexus
Incorrect Answer ImageC.Intercostobrachial nerve
Correct Answer ImageD.Long thoracic nerve
Incorrect Answer ImageE.Radial nerve

A 28-year-old man comes to the physician because of inability to conceive with his wife for 3
years. His medical history is unremarkable and he takes no medications. His wife has a child
from a previous relationship. His height is 185 cm (73 in) and his BMI is 20 kg/m2. His
temperature is 36.9ºC (98.4ºF), blood pressure is 128/82 mm Hg, pulse is 86/min, and
respirations are 14/min. Examination shows mild, nontender gynecomastia and small testes
bilaterally. The remainder of the examination shows no abnormalities. Which of the following
laboratory results are most consistent with this patient's condition? 
Incorrect Answer ImageA.Decreased testosterone, decreased follicle-stimulating hormone (FSH),
and decreased luteinizing hormone (LH)
Correct Answer ImageB.Decreased testosterone, increased FSH, and increased LH
Incorrect Answer ImageC.Decreased testosterone, increased FSH, and normal LH
Incorrect Answer ImageD.Decreased testosterone, normal FSH, and decreased LH
Incorrect Answer ImageE.Normal testosterone, increased FSH, and increased LH

A 61-year-old man is brought to the emergency department because of chest pain for 2 hours. He
has a long-standing history of coronary artery disease and hypertension and had a coronary
bypass procedure 6 years ago. The patient has chronic stable angina that is usually precipitated
by activity and relieved by rest. His medications include aspirin, captopril, and metoprolol.
About 3 weeks ago, his physician prescribed sildenafil, and he has been using the drug with
success. His last sexual encounter was 4 days ago. This morning, he developed chest discomfort.
His temperature is 36.1ºC (97.0ºF), pulse is 100/min, respirations are 22/min, and blood pressure
is 220/120 mm Hg. Ocular examination shows papilledema. An electrocardiogram shows
nonspecific changes. Which of the following is the most appropriate next step in management? 
Correct Answer ImageA.Intravenous labetalol
Incorrect Answer ImageB.Intravenous morphine
Incorrect Answer ImageC.Oral clonidine
Incorrect Answer ImageD.Oral metoprolol
Incorrect Answer ImageE.Sublingual nitroglycerin

A 72-year-old man with a 25-year history of emphysema secondary to alpha-1 antitrypsin


deficiency comes to his physician after he develops the acute onset of fevers, rigors, and a cough
productive of green sputum. His symptoms gradually worsened over the past 36 hours, which
caused him to seek medical attention in the emergency department. He has been taking a
beclomethasone inhaler twice daily, albuterol nebulizer treatment 4 times daily, and
azithromycin for a recent bronchitis. On physical examination he is 183 cm (6 ft) tall and weighs
80 kg (176 lb). His temperature is 38.3°C (100.9°F), blood pressure is 162/92 mm Hg, pulse is
94/min, and respirations are 32/min. Lung examination shows diffuse bilateral coarse rhonchi.
He uses his sternocleidomastoid muscles during inspiration. Arterial blood gas shows pH 7.20,
pCO2 60 mm Hg, and pO2 52 mm Hg. Over the next 2 hours he becomes increasingly tachypneic
and his pCO2 rises to 74 mm Hg. The decision is made for intubation and mechanical ventilation.
Which of the following settings would be the most appropriate setting for tidal volume during
mechanical ventilation?
Incorrect Answer ImageA.300 mL/breath
Correct Answer ImageB.500 mL/breath
Incorrect Answer ImageC.700 mL/breath
Incorrect Answer ImageD.850 mL/breath
Incorrect Answer ImageE.1,000 mL/breath

A 60-year-old man comes to the physician for an annual physical examination. He complains of
occasional fatigue but denies any other symptoms. He denies any loss of weight or loss of
appetite. Temperature is 36.9ºC (98.6ºF), pulse is 90/min, respirations are 16/min, and blood
pressure is 120/80 mm Hg. His lungs are clear to auscultation, and cardiovascular examination is
within normal limits. Extremities show no pitting edema. Laboratory workup shows: 
Hgb 15.6 g/dL
WBC 6,000/mm3
Platelets 70,000/mm3
Peripheral smear shows low platelet count but is otherwise within normal limits. The patient's
platelet count was within normal limits 1 year ago (it was 280,000/mm3). Liver function tests,
serum, and urine protein electrophoresis are within normal limits. The patient undergoes a bone-
marrow biopsy which shows normal myeloid and erythroid precursors. Megakaryocytes are
present in increased numbers. Which of the following is the most appropriate next step in
management of the patient's thrombocytopenia? 
Incorrect Answer ImageA.Corticosteroids by mouth
Incorrect Answer ImageB.IV immunoglobulin
Incorrect Answer ImageC.Platelet transfusion
Incorrect Answer ImageD.Rituximab
Incorrect Answer ImageE.Steroids
Correct Answer ImageF.Watchful management

A 19-year-old man undergoes colonoscopy because of a family history of multiple polyps in his
siblings. His brother underwent total proctocolectomy at age 23 years, and his sister underwent a
total proctocolectomy at age 29 years. Colonoscopy shows 150 to 200 small colonic polyps
within the rectosigmoid; biopsy specimens of five of the polyps found four of them to be benign
adenomas with one showing high grade dysplasia. Which of the following is the most
appropriate next step in management? 
Incorrect Answer ImageA.Administer 5-fluorouracil and leucovorin
Incorrect Answer ImageB.Lymph node biopsy
Incorrect Answer ImageC.Resection of the sigmoid colon
Incorrect Answer ImageD.Schedule a colonoscopy in 1 year
Correct Answer ImageE.Total proctocolectomy

An 89-year-old woman is seen in follow-up for symptoms of feeling weak, faint, and “dizzy”
when she sits up over the past 2 days. She feels well when lying down. On standing, she feels
unsteady and lightheaded and fears falling and hurting herself. She states that the room is not
moving around her, and the floor is steady and secure. Yesterday, an ECG was done and showed
a sinus rhythm at a rate of 59/min. This is the same pattern that she has had for the past 5 years.
Her hemoglobin was 10.2 g/dL and she wore a Holter monitor for the past 24 hours that showed
no arrhythmias.
Pertinent recent history indicates that approximately 6 weeks ago she took a lot of aspirin
because of a cold, after which she had a melanotic stool. Two weeks later, laboratory studies
showed a hemoglobin of 9.8 g/dL, and a repeated stool guaiac test was negative for occult GI
bleeding. Two weeks ago, she was placed on thiazide diuretics because of ankle edema and
advised to limit fluid and sodium intake. Physical examination shows she is alert and oriented.
Her pulse is 60/min and blood pressure is 110/70 mm Hg when lying flat; pulse is 65/min and
blood pressure is 90/50 mm Hg when standing. Which of the following would be the next best
step in management?
Correct Answer ImageA.Discontinue thiazide and fluid restriction
Incorrect Answer ImageB.ENT consultation
Incorrect Answer ImageC.Gastroscopy and colonoscopy
Incorrect Answer ImageD.Pacemaker insertion
Incorrect Answer ImageE.Schedule a CT of her head
Incorrect Answer ImageF.Transfuse her up to a hemoglobin of 14

A 19-year-old college basketball player comes to the physician for a routine health maintenance
examination. He states that he feels well and has no specific physical complaints. A late systolic
murmur is heard on auscultation of the chest. Which of the following is the most likely finding?
Incorrect Answer ImageA.Aortic regurgitation
Incorrect Answer ImageB.Bicuspid aortic valve
Incorrect Answer ImageC.Mitral stenosis
Correct Answer ImageD.Mitral valve prolapse
Incorrect Answer ImageE.Pulmonary valve stenosis

A 39-year-old man is brought to the emergency department because of a witnessed seizure.


Although he is mildly confused, he reports a 1-week history of fatigue, chronic diarrhea and
progressive left-sided weakness. He also reports intermittent night sweats, occasional rigors, and
a loss of appetite. His previous records show a CD4+ lymphocyte count of 138 cells/mm3 and a
HIV viral load of 650,000 copies/mm3. His vital signs are as follows: temperature 37.5°C
(99.5°F), pulse 110/min, respirations 16/min and blood pressure 130/80 mm Hg. Physical
examination shows a thin, cachectic man with a mild left hemiparesis. MRI shows a ring-
enhancing lesion. He is started on a combination of pyrimethamine, folinic acid, and sulfadiazine
for presumed central nervous system (CNS) toxoplasmosis. After 10 days of inpatient treatment,
his neurologic examination worsens and he continues to complain of weakness and night sweats.
Which of the following is the most appropriate next step in management?
Incorrect Answer ImageA.Addition of liposomal amphotericin B to therapy
Correct Answer ImageB.Brain biopsy of intracranial mass
Incorrect Answer ImageC.Lumbar puncture with CSF analysis
Incorrect Answer ImageD.Neurosurgical consultation for resection
Incorrect Answer ImageE.Therapy change to clindamycin

A 28-year-old woman comes to her physician because of a 3-month history of increased


urination and thirst. She reports an increase in the frequency of urination to more than 20 times a
day. She denies blurry vision, painful urination, or blood in her urine. Her past medical history is
significant for bipolar disorder and hypercholesterolemia. Current medications include lithium
and atorvastatin. Her temperature is 37.2ºC (99ºF), pulse is 76/min, respirations are 12/min, and
blood pressure is 115/75 mm Hg. Physical examination is unremarkable. Laboratory studies
show a serum sodium level of 132 mEq/L. Urine osmolality is 160 mOsm/kg. After water
deprivation, the urine osmolality increases to 510 mOsm/kg. Urinalysis shows 2 to 3
erythrocytes/HPF and 1 to 2 leukocytes/HPF. Which of the following is the most appropriate
next step in management?
Incorrect Answer ImageA.Amiloride
Incorrect Answer ImageB.Diagnostic intranasal desmopressin
Correct Answer ImageC.Fluid restriction and management of bipolar disorder
Incorrect Answer ImageD.Insulin
Incorrect Answer ImageE.Therapeutic intranasal desmopressin
Incorrect Answer ImageF.Trimethoprim/sulfamethoxazole

A 70-year-old man is brought to the emergency department because of chest pain for 30 minutes.
Electrocardiogram indicates an acute myocardial infarction, and he is given aspirin and tissue
plasminogen activator (tPA) and subsequently started on a heparin drip. Vital signs are within
normal limits upon hospital admission, and a beta-blocker, ACE inhibitor, and statin are also
started. Six days after hospitalization, the patient becomes confused and drowsy and complains
of mild dyspnea. His heart rate is 120/min, respirations are 25/min, and blood pressure is now
65/50 mm Hg. Pulse oximetry is 80% on room air. He is transferred to the intensive care unit,
where he is intubated and started on normal saline IV fluids. An electrocardiogram shows sinus
tachycardia with low voltage. A pulmonary artery catheter is placed, and the following readings
are obtained: 
Right atrial pressure 18 mm Hg (normal 2–10 mm Hg)
Right ventricular pressure  30/18 mm Hg (normal 15–30/0–5
mm Hg)
Pulmonary capillary wedge 17 mm Hg (normal 5–11 mm Hg)
pressure
Which of the following is the most appropriate next step in management? 
Correct Answer ImageA.Arrange for pericardiocentesis
Incorrect Answer ImageB.Continue current treatment
Incorrect Answer ImageC.Continue IV fluids
Incorrect Answer ImageD.Start diuretics
Incorrect Answer ImageE.Start dopamine

A 34-year-old man comes to the emergency department because of 4 hours of progressive "chest
tightness" and increasing shortness of breath. These symptoms are not relieved with metered-
dose albuterol inhalers. He reports that these symptoms are consistent with previous asthma
exacerbations but this one seems more severe. He had an upper respiratory infection
approximately 3 days before the onset of these symptoms. His temperature is 38.0°C (100.7°F),
blood pressure is 130/70 mm Hg, pulse is 103/min, and respirations are 23/min. He is a young
man in significant respiratory distress, using his accessory muscles. Physical examination shows
bilateral wheezing and poor air movement with a prolonged expiratory phase. An arterial blood
gas test shows pH 7.55, PCO2 21 mm Hg, and PO2 60 mm Hg.
He is given high-flow nebulizers and IV steroids. An hour later his respiratory rate is 14/min.
Repeat ABG shows pH 7.46, PCO2 34 mm Hg, and PO2 64 mm Hg. Which of the following is
the most correct statement about his condition at this time?
Incorrect Answer ImageA.Acutely, a compensatory increase in bicarbonate level is expected
Incorrect Answer ImageB.Carbon dioxide retention is commonly seen early in the course of asthma
Incorrect Answer ImageC.Inhaled steroids would have been equally effective in the acute setting
Incorrect Answer ImageD.Normalization of the repeat arterial blood gas test is likely caused by
rapid onset of IV steroids
Correct Answer ImageE.The normalization in arterial blood gases suggests impending respiratory
failure

A 28-year-old woman comes to the physician because of progressive growth of a mass below her
left knee for 6 months. She denies pain or trauma to the area. Her medical history is
unremarkable. Physical examination shows a hard, round, nontender mass on the posterior aspect
of her lower leg. CT scan of the left leg shows a 3 cm pedunculated mass attached to the
gastrocnemius muscle. An incisional biopsy is consistent with low-grade fibrosarcoma. Which of
the following is the most appropriate next step in management?
Incorrect Answer ImageA.Close clinical observation
Incorrect Answer ImageB.Combination of radiation and chemotherapy
Incorrect Answer ImageC.External beam radiation
Incorrect Answer ImageD.Neoadjuvant chemotherapy
Correct Answer ImageE.Surgical excision of the mass

A 44-year-old woman comes to the emergency department 8 hours after experiencing the sudden
onset of severe, progressive right upper quadrant abdominal pain radiating to her back. Before
the pain started, she ate lunch at a fast-food restaurant. She reports having two prior episodes, the
first which occurred during her last pregnancy 3 years ago and the second, two months ago. Her
temperature is 38.1ºC (100.5ºF). Abdominal examination shows right upper quadrant tenderness
and cessation of inspiration with deep palpation. Rectal examination shows brown stool negative
for occult blood. Her leukocyte count is 12,900/mm3, hematocrit is 39%, and total bilirubin is 2.1
mg/dL. A right upper quadrant ultrasound is obtained and interpreted as a “poor quality image.”
Which of the following is the most appropriate next step in management? 
Incorrect Answer ImageA.CT scan of the abdomen and pelvis

Incorrect Answer ImageB.Endoscopic retrograde cholangiopancreatography (ERCP)

Correct Answer ImageC.Hepatobiliary iminodiacetic acid (HIDA) scan

Incorrect Answer ImageD.Magnetic resonance cholangiopancreatography (MRCP)

Incorrect Answer ImageE.No further testing is required since this condition is diagnosed clinically
A 63-year-old woman arrives in her physician's office because of profuse, watery diarrhea. She
has had this symptom for the past several weeks, and despite using loperamide and changing her
diet, the diarrhea has persisted. She denies recent travel or camping trips. Even when she does
not eat, the diarrhea is present. Additionally, she complains of sporadic facial flushing. Her past
medical history is only notable for hypertension, for which she has been taking
hydrochlorothiazide for many years now. On physical examination, she appears dehydrated and
has poor skin turgor. Other than dry mucous membranes, her examination is unremarkable. She
is sent for a small bowel follow-through barium examination that demonstrates a non-obstructive
filling defect in her distal jejunum, which on a CT-scan represents a tumor protruding into the
bowel lumen. Given her history, which of the following most likely coexists with her small
bowel tumor? 
Incorrect Answer ImageA.Aortic valve stenosis
Incorrect Answer ImageB.Hamartomatous polyps
Correct Answer ImageC.Hepatic metastases
Incorrect Answer ImageD.Medullary thyroid tumor
Incorrect Answer ImageE.Mesenteric lymph node metastases
Incorrect Answer ImageF.Pituitary tumor

A 66-year-old woman comes to the emergency department with palpitations, shortness of breath,
and lightheadedness. She has a history of hypertension for which she takes a diuretic. Her pulse
is 152/min, respirations are 16/min, and blood pressure is 105/65 mm Hg. Physical examination
shows an elderly woman in moderate distress; auscultation of her neck shows bilateral bruits and
cardiac examination shows distant heart sounds with no rubs or gallops. An immediate ECG is
taken and shown. Which of the following is the most appropriate next step in the management of
this patient?
Incorrect Answer ImageA.Adenosine
Incorrect Answer ImageB.Carotid massage
Correct Answer ImageC.Forcefully exhaling against a closed mouth and nose
Incorrect Answer ImageD.Synchronized cardioversion
Incorrect Answer ImageE.Verapamil

An 18-year-old college student is brought to the emergency department by concerned friends


after he was unable to get out of bed. He reports a 3-day history of progressive weakness and
fatigue. His symptoms initially began in his feet, with tingling and burning, and then progressed
to weakness that ascended his legs until they were too weak to support him. He also reports
difficulty taking deep breaths. His medical history is unremarkable. His temperature is 37.0ºC
(98.6ºF), blood pressure is 130/88 mm Hg, pulse is 100/min, and respirations are 36/min.
Physical examination shows a young man in mild distress who is having difficulty completing
sentences and clearing secretions. Cranial nerve examination shows the patient has difficulty
raising his head against gravity. Upper extremity strength is 3/5 bilaterally and deep tendon
reflexes are normal. Lower extremity strength is 1/5 bilaterally and lower extremity deep tendon
reflexes are absent bilaterally. Cardiac and pulmonary examinations are normal. Which of the
following is the most appropriate next step in management?
Correct Answer ImageA.Elective intubation
Incorrect Answer ImageB.High-dose glucocorticoids
Incorrect Answer ImageC.Intravenous immunoglobulin
Incorrect Answer ImageD.Low-dose glucocorticoids
Incorrect Answer ImageE.Nerve conduction studies

A 58-year-old woman comes to the clinic because of a painless lump in her neck and excessive
sweating for 4 months. She has felt warm and flushed, even when others have felt comfortable.
She has had frequent stools, insomnia, and heart palpitations. She takes no medications. Her
temperature is 38ºC (100.4ºF), blood pressure is 138/88 mm Hg, pulse is 121/min, and
respirations are 20/min. Examination shows a thin woman who has a resting tremor. There is a
palpable, nontender, 0.5-cm nodule in the left lobe of the thyroid gland. Laboratory studies
show: 
Thyroid-stimulating hormone (TSH) 0.1 μU/mL
Thyroxine (T4)  13.2 μg/dL
A radioactive iodine (RAI) uptake thyroid scan is performed. Which of the following findings is
most likely to be seen in this patient?
Incorrect Answer ImageA.Increased radioactive iodine (RAI) uptake throughout the thyroid
Incorrect Answer ImageB.Increased RAI uptake throughout the non-nodular tissue of the thyroid
Incorrect Answer ImageC.Low RAI uptake in the nodular tissue of the thyroid
Incorrect Answer ImageD.Normal RAI scan, no abnormalities noted
Correct Answer ImageE.Reduced RAI uptake in the non-nodular tissue
Incorrect Answer ImageF.Reduced RAI uptake throughout the thyroid

A 74-year-old man is brought to the emergency department because of chest pain with dyspnea,
nausea, and diaphoresis. His blood pressure is 145/93 mm Hg and pulse is 98/min. Physical
examination shows jugular venous distension and basilar crackles. No abnormal heart sounds are
heard but peripheral pulses are faint. X-ray of the chest shows mild interstitial pulmonary edema.
ECG shows new inferior ST-segment elevations. The closest hospital with percutaneous
interventional capability is more than 2 hours away. Which of the following is the most
appropriate pharmacotherapy?
Incorrect Answer ImageA.Alteplase
Incorrect Answer ImageB.Aspirin and amlodipine
Correct Answer ImageC.Aspirin, heparin, and alteplase
Incorrect Answer ImageD.Heparin and alteplase
Incorrect Answer ImageE.Heparin and aspirin

A 67-year-old woman is brought to the emergency department because of a 3-day history of


progressive fever, shortness of breath, a productive cough with yellow-green sputum, and mild
bilateral chest pain. She has a history of hypertension, coronary artery disease, and
hyperlipidemia. Current medications include nitroglycerin, aspirin, atenolol, and atorvastatin.
She is a retired schoolteacher, and she does not smoke or drink alcohol. Her temperature is
39.9ºC (103.8ºF), pulse is 128/min, respirations are 32/min, blood pressure is 83/59 mm Hg, and
oxygen saturation is 89% on oxygen 2 L/min via nasal cannula. She is in moderate respiratory
distress. Crackles and decreased breath sounds are heard at both lung bases. Cardiac examination
shows normal S1 and S2; no murmurs are heard. Laboratory studies show: 
Hemoglobin  13.2 g/dL
Leukocyte count  19,800/mm3
Segmented neutrophils  86%
Platelet count  105,000/mm3
Serum studies show: 
Na+ 139 mEq/L
Cl–  103 mEq/L
K +
4.2 mEq/L
HCO3–  25 mEq/L
Blood urea nitrogen (BUN)  32 mg/dL
Creatinine  1.74 mg/dL 
A radiograph of the chest is shown. Which of the following is the most appropriate next step in
management?
Incorrect Answer ImageA.Admit her to a general medicine floor and start therapy for congestive
heart failure secondary to acute pericarditis
Incorrect Answer ImageB.Admit her to a general medicine floor and start IV ceftriaxone and
azithromycin
Correct Answer ImageC.Admit her to the intensive care unit and start IV ceftriaxone and
azithromycin
Incorrect Answer ImageD.Observe her in the emergency department and start IV ceftriaxone and
azithromycin
Incorrect Answer ImageE.Outpatient treatment with oral moxifloxacin and reevaluation in 24 hours

A 25-year-old woman is admitted to the hospital for treatment of non-Hodgkin lymphoma. She is
found by CT scan to have advanced disease which is confirmed with PET scan and
mediastinoscopy with biopsy. After much consultation, she is started on an experimental
chemotherapeutic protocol. She initially tolerates the protocol well and reports no major adverse
events. However, by the fourth day of her treatment, her urine output is 30 cc/hour and her
laboratory results show the following: 
Hemoglobin 10.5 g/dL
Hematocrit 32%
Leukocyte count 4,000/mm3
Platelets 140,000/mm3
Sodium 137 mEq/L
Potassium 5.1 mEq/L
Chloride 100 mEq/L
Bicarbonate 22 mEq/L
BUN 65 mg/dL
Creatinine 4.5 mg/dL
Despite aggressive fluid administration for 2 hours with several fluid boluses, her laboratory
values are essentially unchanged. Urine output is now 26 cc/hour. Which of the following is
most likely responsible for these findings? 
Incorrect Answer ImageA.Bleomycin
Correct Answer ImageB.Cisplatin
Incorrect Answer ImageC.Cytarabine
Incorrect Answer ImageD.Doxorubicin
Incorrect Answer ImageE.Vincristine

A 67-year-old male is hospitalized with his first episode of acute pancreatitis. He is managed
conservatively, has a gradual recovery, and is discharged from the hospital after 1 week. Six
weeks after his discharge he comes to the office for a follow-up visit. He reports increased
abdominal pain, fatigue, and feels overall unwell. In addition, he has noticed some abdominal
bloating and is concerned about it. He denies any nausea, vomiting, or diarrhea. His pulse is
80/min, respirations are 12/min, and blood pressure is 120/70 mm Hg. He is afebrile. Abdominal
examination reveals fullness in the epigastric region. The remainder of the physical examination
is within normal limits. A CT scan reveals the presence of a low, attenuated, homogenous 6-cm
cystic lesion in the body of the pancreas. Which of the following is the most appropriate next
step in management? 
Incorrect Answer ImageA.Check Carbohydrate Antigen 19-9 (CA 19-9)
Incorrect Answer ImageB.Endoscopic ultrasound (EUS) with biopsy
Incorrect Answer ImageC.Oral pancrelipase
Incorrect Answer ImageD.Percutaneous drainage of the cyst
Correct Answer ImageE.Treat symptomatically and repeat CT scan in 4 weeks

A 34-year-old man is brought to the emergency department because of severe abdominal pain
and difficulty breathing for the last 30 minutes after eating dolphin-fish. He has developed skin
flushing, generalized pruritus, and hives. His blood pressure is 90/50 mm Hg and pulse is
115/min. His medical history is unremarkable, and he takes no medications. He does not drink
alcohol or use illicit drugs. Examination shows facial swelling and generalized urticaria. There
are mild expiratory wheezes in all lung fields on auscultation. Which of the following is the most
likely explanation for these findings? 
Incorrect Answer ImageA.Ciguatera poisoning
Incorrect Answer ImageB.Paraquat poisoning
Incorrect Answer ImageC.Saxitoxin poisoning
Correct Answer ImageD.Scombroid poisoning
Incorrect Answer ImageE.Tetrodotoxin poisoning

A 72-year-old woman is brought to the emergency department because of epigastric abdominal


pain, nausea, and profuse sweating. Vital signs are within normal limits. She appears pale and
sweaty, and cardiovascular examination shows normal heart sounds without murmurs or rubs.
An electrocardiogram shows ST-segment elevation in leads II, III, and aVF. She undergoes
coronary angiography and percutaneous coronary intervention. Two hours after the procedure,
her blood pressure suddenly falls to 88/56 mm Hg, pulse is 128/min, respirations are 20/min, and
oxygen saturation is 94% in room air. Her lungs are clear to auscultation. There is notable
jugular venous distension. A focused cardiac ultrasound reveals no fluid in the pericardial sac.
Which of the following is the next best step in management?
Incorrect Answer ImageA.Atropine
Incorrect Answer ImageB.Emergency coronary artery bypass graft (CABG) surgery
Correct Answer ImageC.Intravenous fluids
Incorrect Answer ImageD.Pericardiocentesis
Incorrect Answer ImageE.Pericardiocentesis with pericardial window

A 68-year-old man is brought to the physician by his wife because of gait abnormalities and
forgetfulness for 2 years. Two years ago, he was hospitalized after a fall, and his walking has
been impaired since then, though slowly improved. This was the first time his wife noticed any
difficulties with memory. Two months ago, there was a sudden deterioration in cognitive
function, particularly with his language abilities. He has a 25-year history of hypertension
controlled with hydrochlorothiazide. His blood pressure is 134/84 mm Hg and pulse is 78/min.
Cardiac examination shows a normal S1 and S2; no murmurs are heard. Neurologic examination
shows a wide-based, unsteady gait. Mini-mental status examination shows a score of 20/30. His
hemoglobin level is 15.4 g/dL. Which of the following is the most likely cause of the patient's
condition?
Incorrect Answer ImageA.Alzheimer dementia
Incorrect Answer ImageB.Cerebellar tumor
Correct Answer ImageC.Multi-infarct (vascular) dementia
Incorrect Answer ImageD.Parkinson disease
Incorrect Answer ImageE.Vitamin B12 deficiency

A 55-year-old man comes to the emergency department because of a 6-month history of a red
eruption on his skin and sores around his mouth. He has had diarrhea, increased thirst, urination,
and hunger. He has had a 14-kg (31-lb) weight loss. His temperature is 36.8ºC (98.2ºF), pulse is
76/min, and blood pressure is 134/86 mm Hg. Physical examination shows erythematous annular
plaques with central bullae and erosions in the perioral and perineal regions, groin, lower
abdomen, and proximal lower extremities. Laboratory studies show:
Hemoglobin 9.8 g/dL
Mean corpuscular volume 92 fL
Serum glucose 231 mg/dL
Which of the following tumors is the most likely cause of this patient's condition? 
Incorrect Answer ImageA.Gastrinoma
Correct Answer ImageB.Glucagonoma
Incorrect Answer ImageC.Insulinoma
Incorrect Answer ImageD.Somatostatinoma
Incorrect Answer ImageE.VIPoma
A 45-year-old woman is brought to the emergency department by her daughter because of
confusion and a headache for 2 days. The daughter reports that the patient ran out of her
medications 2 weeks ago and did not refill her prescriptions. She has a history of hypertension
and gastroesophageal reflux disease. Current medications include metoprolol and famotidine.
She has no allergies to medications. Her temperature is 36.6ºC (97.9ºF), blood pressure is
250/120 mm Hg, and pulse is 85/min. Lungs are clear to auscultation. Cardiac examination
shows an S1, S2, and a loud S4; no murmurs are heard. ECG shows left axis deviation and
increased amplitude of the QRS complexes. Which of the following is the most appropriate next
step in management?
Incorrect Answer ImageA.Administer IV furosemide
Incorrect Answer ImageB.Administer IV hydralazine
Incorrect Answer ImageC.Administer IV lorazepam
Correct Answer ImageD.Administer IV nitroprusside
Incorrect Answer ImageE.Order CT scan of the brain with contrast
Incorrect Answer ImageF.Order CT scan of the brain without contrast

A 60-year-old man comes to the emergency department with shortness of breath and dull, left-
sided chest pain. He has had a low-grade fever for the past 3 days. He was treated recently for
pneumonia with antibiotics but says that he never quite returned to baseline. He has no other
medical issues and has no allergies. He denies alcohol abuse or drug use. His temperature is
37.8°C (100.0°F), pulse is 70/min, and blood pressure is 120/80 mm Hg. Physical examination
shows decreased breath sounds on the left and decreased tactile fremitus. Left decubitus chest x-
ray is consistent with a large left-sided pleural effusion >15 mm with layering. Thoracentesis
shows turbid fluid with WBC count 70,000, RBC count 20,000, LDH 1,500 IU/L, serum LDH
600 IU/L (normal 50-150 IU/L), and pleural fluid pH 7.1. Repeat chest x-ray shows pneumonia
in the left upper lobe. Gram stain of the fluid fails to reveal organisms. Cultures of the pleural
fluid are sent to the laboratory. Which of the following is the most appropriate initial
management?
Incorrect Answer ImageA.Antibiotic coverage and observation
Correct Answer ImageB.Chest tube insertion
Incorrect Answer ImageC.Pleural biopsy
Incorrect Answer ImageD.Pleurodesis
Incorrect Answer ImageE.Video-assisted thorascopic surgery (VATS)

A 60-year-old woman comes to the physician because she has been having daily temperature
elevations to 100ºF (37.8ºC). In addition, she reports severe lethargy for the past 3 weeks and
night sweats for the past one week. On examination, she has sternal tenderness on palpation and
her spleen is palpated 4 cm below the left costal margin. CBC shows a white blood cell count of
70,000 with a predominance of mature myeloid cells. 1% blasts seen. Hematocrit is 38% and
platelet count is 400,000/µL. Which of the following is the most appropriate next step in
management? 
Incorrect Answer ImageA.Admit to the hospital for cultures and broad-spectrum IV antibiotics
Incorrect Answer ImageB.Look for smudge cells on the peripheral smear
Incorrect Answer ImageC.Perform CT scan of the thorax and abdomen
Correct Answer ImageD.Perform FISH on peripheral blood to find the BCR-ABL1 fusion gene
Incorrect Answer ImageE.Start treatment with imatinib mesylate

A 29-year-old man comes to the emergency department because of nausea and vomiting blood
for 2 hours. He has had 2 episodes of upper gastrointestinal bleeding during the last year. He has
a history of bleeding from a duodenal ulcer. Current medications include pantoprazole. He
denies the use of aspirin or nonsteroidal anti-inflammatory drugs. He does not drink alcohol. His
blood pressure is 130/75 mm Hg. Endoscopy shows one ulcer in the distal duodenum and one in
the jejunum. Which of the following is the most likely cause of his symptoms? 
Incorrect Answer ImageA.Autoimmune destruction of the parietal cells
Correct Answer ImageB.Autonomous production of gastrin
Incorrect Answer ImageC.Bacterial colonization of antral mucosa
Incorrect Answer ImageD.Impaired production of the mucus bicarbonate layer in the stomach
Incorrect Answer ImageE.Noncompliance with medication

A 48-year-old man comes to his physician for a routine health maintenance examination. He has
had a 5-kg (11-lb) weight gain over the past year. He has a 4-year history of variably controlled
hypertension, for which he takes hydrochlorothiazide. His mother has type 2 diabetes mellitus
and his father had a myocardial infarction at age 53. He weighs 108 kg (238 lb) and is 165 cm
(65 in) tall (BMI 39.7). His blood pressure is 150/88 mm Hg. His waist circumference is 109 cm
(43 in). Today his fasting serum glucose level is 115 mg/dL, and one month ago it was 123
mg/dL. Lipid testing is most likely to show which of the following in this patient? 
Incorrect Answer ImageA.Elevated ratio of HDL cholesterol to LDL cholesterol
Correct Answer ImageB.Elevated ratio of triglyceride to HDL cholesterol
Incorrect Answer ImageC.Increased total cholesterol level and normal triglyceride level
Incorrect Answer ImageD.Increased triglyceride level and increased HDL cholesterol level
Incorrect Answer ImageE.Increased triglyceride level and low LDL cholesterol level

A previously healthy 26-year-old man comes to the physician because of progressive left ankle
pain for 5 days. He has been taking ibuprofen without relief. He sprained his ankle 3 months ago
while jumping into a shallow rock pool. He has had pain and discharge on urination for 2 weeks.
He is sexually active with multiple partners and does not consistently use condoms. His
temperature is 37.1ºC (98.8ºF) and pulse is 80/min. Examination shows a tender left ankle with a
slight effusion and marked tenderness at the insertion of the Achilles tendon. Genitourinary
examination shows multiple superficial ulcers on the glans penis; there is no discharge.
Arthrocentesis of the joint shows a white blood cell count of 45,000/mm3 and 70% neutrophils
with no crystals. He is prescribed sulfasalazine. Which of the following is the most appropriate
additional pharmacotherapy for this patient? 
Incorrect Answer ImageA.Allopurinol
Correct Answer ImageB.Doxycycline
Incorrect Answer ImageC.Highly active antiretroviral therapy
Incorrect Answer ImageD.Penicillamine
Incorrect Answer ImageE.Penicillin
A 25-year-old man comes to the emergency department with progressively worsening low back
pain for 3 weeks. He has had intermittent episodes of night sweats, fevers, and chills. He also has
difficulty walking and difficulty controlling his bowel movements. He has no history of major
medical illness and takes no medications. He has a history of intravenous drug abuse. His
temperature is 38.9ºC (102.0ºF), pulse is 120/min, respirations are 20/min, and blood pressure is
100/60 mm Hg. There is tenderness over the L4 vertebral body. Leukocyte count is 32,000
cells/mm3 and erythrocyte sedimentation rate is 100 mm/h. Which of the following is the most
likely causative organism of this patient’s condition?
Incorrect Answer ImageA.Influenza virus
Incorrect Answer ImageB.Streptococcus viridans 
Incorrect Answer ImageC.Streptococcus pneumoniae 
Incorrect Answer ImageD.Klebsiella pneumoniae

Correct Answer ImageE.Staphylococcus aureus 

A 59-year-old man comes in for an office visit because of a 3 year history of intractable
coughing and sputum production. He denies hemoptysis, fevers, night sweats, weight loss, or
acid reflux. His past medical history is notable for hypertension, for which he takes
hydrochlorothiazide and metoprolol. He has smoked 1 pack of cigarettes per day for nearly 20
years. He does not use alcohol. His temperature is 36.8ºC (98.2ºF), blood pressure is 135/76 mm
Hg, and pulse is 90/min. Cardiopulmonary examination shows diffuse wheezes and a prolonged
expiratory phase. There are no murmurs, rubs, or gallops. Abdominal examination is
unremarkable. He has 1+ pitting edema, but no clubbing or cyanosis. Pulmonary function tests
show an FEV1/FVC of 0.65. Arterial blood gas values are pH 7.34, partial pressure of arterial
oxygen 70 mm Hg, and partial pressure of carbon dioxide 50 mm Hg. Which of the following
therapies will prolong this man's survival? 
Incorrect Answer ImageA.Cessation of beta blockade
Incorrect Answer ImageB.Home oxygen
Incorrect Answer ImageC.Inhaled bronchodilators
Incorrect Answer ImageD.Inhaled corticosteroids
Correct Answer ImageE.Smoking cessation

A 51-year-old man undergoes a double-contrast barium enema for a routine colon cancer
screening examination. There is a 3-mm polyp in the ascending colon that is rounded and
smooth. The polyp meets the radiographic criteria for being a benign sessile polyp. No other
lesions are seen within the colon. Which of the following is the most appropriate next step in
management? 
Incorrect Answer ImageA.Colonoscopy in 1 year
Correct Answer ImageB.Colonoscopy now
Incorrect Answer ImageC.CT abdomen and pelvis
Incorrect Answer ImageD.Repeat the barium enema in 1 year
Incorrect Answer ImageE.Sigmoidoscopy now
A 25-year-old man with a recent syncopal episode is brought to the hospital because of
congestive heart failure. His blood pressure is 160/100 mm Hg and pulse is 90/min. A grade 2/6
harsh systolic ejection murmur is heard on auscultation of the chest. Echocardiogram shows a
severely thickened ventricular septum and systolic anterior motion of the mitral valve. Which of
the following is most likely to be seen in this patient?
Correct Answer ImageA.Decreased murmur with hand grip
Incorrect Answer ImageB.Decreased murmur with Valsalva
Incorrect Answer ImageC.Delayed carotid upstroke
Incorrect Answer ImageD.Increased murmur with squatting
Incorrect Answer ImageE.Murmur radiating to carotid arteries

A 67-year-old Caucasian male patient comes to the physician because of weakness, fatigue, and
paresthesias. He is asking the physician to prescribe him something that could increase his
appetite since he has lost almost 12 pounds in the last 4 weeks. He has also had two episodes of
nasal bleeding, blurred vision, and headaches during the past week. Past history is significant for
hypertension and diabetes for which he is taking hydrochlorothiazide and glimepiride,
respectively. His temperature is 39.4ºC (101.1ºF), pulse is 94/min, respirations are 16/min, and
blood pressure is 130/80 mm Hg. The patient appears confused. Head examination shows pale
conjunctiva. Cardiac examination shows regular rate and rhythm with a normal S1 and S2 and
displaced apical impulse. Lungs are clear to auscultation. The liver and spleen are palpated 4 and
3 cm below the right and left costal margins, respectively. The patient has impaired vibration and
proprioception in the lower extremities, bilaterally. Laboratory studies show:
Hemoglobin  9 g/dL
Platelets 100,000/mm3
White blood cells  4,500/mm3
MCV  88 um3
MCHC  32%
Peripheral smear  Plasmacytoid lymphocytes and rouleaux
formation
Bone marrow biopsy  >10% infiltration by small lymphocytes
showing plasma cell differentiation 
Creatinine  1.1 mg/dL
BUN  18 mg/dL
Serum protein Monoclonal spike (IgM)
electrophoresis
Which of the following is the most likely diagnosis?
Incorrect Answer ImageA.Chronic lymphocytic leukemia
Incorrect Answer ImageB.Marginal zone lymphoma
Incorrect Answer ImageC.Monoclonal gammopathy of undetermined significance
Incorrect Answer ImageD.Multiple myeloma
Correct Answer ImageE.Waldenström macroglobulinemia
A 61-year-old man is brought to the emergency department because of a 6-hour history of
burning abdominal pain that radiates to his back. He had nausea exacerbated by eating or
drinking. He denies vomiting, diarrhea, constipation, or blood in his stools. He has a history of
type 2 diabetes mellitus, hypertension, hypercholesterolemia, hypertriglyceridemia, peptic ulcer
disease, and gout. He underwent a below-the-left-knee amputation because of a diabetic foot
ulcer 2 years ago and a vagotomy and antrectomy (Billroth I procedure) 10 years ago. His blood
pressure is 198/110 mmHg and pulse 110/min. Abdominal examination shows diffuse tenderness
with no rebound or guarding. Laboratory studies show:
Serum amylase 220 U/L
Serum lipase 250 U/L
Serum glucose 291 mg/dL
Blood alcohol Undetectable
Urine amphetamines Positive
The plasma portion of the spun blood sample is cloudy and straw colored. Ultrasound shows a
diffusely enlarged, hypoechoic pancreas; the gallbladder is normal-appearing, and a sonographic
Murphy sign is negative. Which of the following is the most likely underlying cause of this
patient's condition? 
Incorrect Answer ImageA.Cocaine abuse
Incorrect Answer ImageB.Elevated LDL cholesterol
Incorrect Answer ImageC.Hypertensive crisis
Correct Answer ImageD.Hypertriglyceridemia
Incorrect Answer ImageE.Perforated peptic ulcer

A 26-year-old man comes to the physician because of progressive neck and shoulder pain and
decreased sensation in both arms for 3 months after he fell off a ladder. Recently he has had
balance problems while walking and severe weakness in his upper and lower extremities. He has
a 10-year history of type 1 diabetes mellitus controlled with insulin. He had one episode of
diabetic ketoacidosis 8 years ago. He drinks two to three drinks of alcohol on weekends and he
does not use illicit drugs. His temperature is 37.1°C (98.8°F), blood pressure is 110/70 mm Hg,
pulse is 88/min, and respirations are 12/min. Neurologic examination shows decreased pain and
temperature sensation of the upper extremities, shoulders, and upper torso. Touch and vibration
sensation are normal. There is wasting and weakness of the hand muscles. Muscle strength is 4/5
in the lower extremities; there is spasticity. Deep tendon reflexes of the lower extremities are
brisk. There is decreased position and vibration sensation in the lower limbs. Hemoglobin A1C is
6.7%. A CT scan of the head shows no abnormalities. Which of the following is the most likely
diagnosis? 
Incorrect Answer ImageA.Diabetic neuropathy
Incorrect Answer ImageB.Multiple sclerosis
Incorrect Answer ImageC.Spinal cord transection
Incorrect Answer ImageD.Subacute combined degeneration of the spinal cord
Incorrect Answer ImageE.Syringobulbia
Correct Answer ImageF.Syringomyelia
A 44-year-old waitress with a known history of asthma comes to the emergency department with
severe shortness of breath, cough, and wheezing. On arrival, she presents with (click media file).
Her pulse is 104/min, respirations are 32/min, and blood pressure is 154/84 mm Hg. With
inspiration, her blood pressure falls to 112/70 mm Hg. On lung examination, there are loud,
high-pitched wheezes and a prolonged expiratory phase. Which of the following physical
examination findings is most significant in evaluating this patient?
Incorrect Answer ImageA.Hypertension
Incorrect Answer ImageB.Loud wheezing
Incorrect Answer ImageC.Prolonged expiratory phase
Correct Answer ImageD.Pulsus paradoxus
Incorrect Answer ImageE.Tachycardia

A 58-year-old Asian man followed for chronic stable hepatitis caused by HCV infection presents
with right upper quadrant pain and an acute increase in abdominal girth. Abdominal examination
shows diffuse tenderness worse in the right upper quadrant and a fluid wave. Paracentesis shows
blood-tinged fluid, and laboratory examination is significant for a significantly for an elevated
alpha-fetoprotein of 435 ng/mL. Which of the following are potential risk factors for the
condition affecting this patient? 
Correct Answer ImageA.Hemochromatosis and nonalcoholic fatty liver disease
Incorrect Answer ImageB.High-fat, low-fiber diet and ulcerative colitis
Incorrect Answer ImageC.H. pylori infection and pernicious anemia
Incorrect Answer ImageD.Obesity and biliary stasis
Incorrect Answer ImageE.Smoking and CDKN2A mutations

A 19-year-old woman with a history of panic disorder and anxiety comes to the emergency
department because of shortness of breath. She does not have chest pain, palpitations,
diaphoresis, or light-headedness. She has recently stopped all of her outpatient medications
because she believed that she no longer needed them. She appears anxious and is in no acute
distress. Her temperature is 37.0ºC (98.6ºF), blood pressure is 120/70 mmHg, pulse is 50/min,
respirations are 50/min, and blood pressure is 120/70 mm Hg. Her oxygen saturation is 99% in
room air. Examination shows no other abnormalities. An ECG is shown. She is administered
lorazepam and her symptoms resolve. Which of the following is the most appropriate next step in
management?
Incorrect Answer ImageA.Administer amiodarone
Incorrect Answer ImageB.Anticoagulation
Incorrect Answer ImageC.Cardioversion
Incorrect Answer ImageD.Electrophysiologic study
Incorrect Answer ImageE.Pacemaker placement
Correct Answer ImageF.Reassurance

An 18-year-old army recruit goes on a 24-hour forced march. When he urinates 2 hours after the
end of the march, his urine is red-tinged. He goes to the infirmary where a dipstick test of urine
is positive for hemoglobin. Blood is drawn and shows a hematocrit of 35%, and it is commented
that the serum is red-tinged. Creatine kinase (CPK) serum levels are normal. Peripheral smear
shows schistocytes and RBC fragments. This patient's anemia is most likely due to which of the
following? 
Incorrect Answer ImageA.Autoimmune attack on red cells
Incorrect Answer ImageB.Inadequate hemoglobin synthesis
Incorrect Answer ImageC.Hemolytic uremic syndrome
Incorrect Answer ImageD.Rhabdomyolysis
Correct Answer ImageE.Traumatic hemolysis

A 32-year-old man comes to the physician because of perianal itching, bleeding, and pain for 4
months. Over-the-counter anesthetic and astringent suppositories and increased dietary fiber
have provided no relief. Examination is shown. Rectal examination shows no masses and no
blood in the stool. Which of the following is the most likely cause of this patient's condition? 
Incorrect Answer ImageA.Fistula in ano
Incorrect Answer ImageB.Paget’s disease of the anus
Incorrect Answer ImageC.Rectal prolapse
Incorrect Answer ImageD.Thrombosed external hemorrhoids
Correct Answer ImageE.Viral infection

A 70-year-old man comes to the emergency department because of a 3-day history of right
temporal headache, fever, and profound malaise. During the past few days, he has experienced
pain in the temporal region while combing his hair. His medical history is unremarkable. His
temperature is 39.5ºC (103.1ºF), pulse is 98/min, respirations are 24/min, and blood pressure is
130/80 mm Hg. On physical examination, he appears acutely ill. Neurologic examination of
cranial nerves shows them to be intact, and he has 5/5 strength in all extremities with normal
sensation. Fundoscopic examination shows optic discs with sharp margins, no papilledema, and
normal venous pulsations. Examination with an eye chart shows diminished visual acuity on the
right. Laboratory studies show: 
Hematocrit 39.0%
Hemoglobin 10.9 g/dL
Leukocytes 8,800/μL (neutrophils 68%)
Erythrocyte sedimentation rate (ESR) 92 mm/h
C-reactive protein (CRP) 55 mg/L (n < 3 mg/L)
Which of the following is the most appropriate next step in management? 
Incorrect Answer ImageA.Acetazolamide
Incorrect Answer ImageB.Carbamazepine
Correct Answer ImageC.High-dose (60 mg/day) prednisone treatment
Incorrect Answer ImageD.Low-dose (10 mg/day) prednisone treatment
Incorrect Answer ImageE.Temporal artery biopsy

An 82-year-old woman is admitted to the hospital with altered mental status and a depressed
level of consciousness. She has a past medical history of Alzheimer's disease and osteoporosis.
Home medications include donepezil, vitamin D, calcium, and alendronate. Over the next 24
hours, blood and urine cultures are positive for gram-negative rods and IV antibiotics are started.
On day 3, she becomes tachypneic and reports diffuse chest pain. Her blood pressure is 88/40
mm Hg, pulse is 120/min, respirations are 36/min, temperature is 38ºC (100.3ºF), and oxygen
saturation is 80% on room air. Auscultation of the chest shows diffuse crackles. Heart sounds are
normal. Chest radiograph shows diffuse bilateral alveolar infiltrates and a normal cardiac
silhouette. The patient is transferred to the intensive care unit, where she is intubated. Arterial
blood gas shows pO2 of 60 on 100% oxygen. Hemodynamic parameters obtained in the ICU
show central venous pressure 15 cm H2O (normal 6), pulmonary capillary wedge pressure 8 mm
Hg, and ejection fraction 59%. Which of the following is the most likely diagnosis? 
Correct Answer ImageA.Acute respiratory distress syndrome
Incorrect Answer ImageB.Cardiogenic shock and pulmonary edema
Incorrect Answer ImageC.Community-acquired pneumonia
Incorrect Answer ImageD.Hospital-acquired pneumonia
Incorrect Answer ImageE.Ventilator-acquired pneumonia

A 58-year-old man comes to his physician because of a yellow discoloration of his mouth and
eyes. He has been drinking a liter of vodka daily for the past 30 years and was told by a
physician 20 years ago that he had liver disease. He recently lost his job as a mail carrier because
he has been forgetting where to deliver the mail. Current medications include propranolol,
prescribed during a past emergency room visit for “bloody vomit”. His blood pressure is 100/60
mm Hg, pulse is 60/min, respirations are 12/min, and temperature is 38.4ºC (101.1ºF). Physical
examination shows generalized jaundice, multiple areas of spider angiomata across his chest and
face, palmar erythema, and bilateral gynecomastia. Abdomen shows distension that is dull to
percussion, with a fluid wave and shifting dullness. Stool is heme negative. Neurologic exam
shows a flapping tremor. Blood and serum studies show:
WBC 15,000 mm3
Hemoglobin 14 g/dL
Hematocrit 42%
Platelets 100,000 mm3
Prothrombin time 16 seconds
Sodium 130 mEq/L
Potassium 3.0 mEq/L
Chloride 94 mEq/L
Bicarbonate 30 mEq/L
BUN 10 mg/dL
Creatinine 1.4 mg/dL
AST 30 U/L
ALT 30 U/L
Which of the following is the most appropriate next step in management?
Incorrect Answer ImageA.Abdominal ultrasonography
Incorrect Answer ImageB.Evaluation for liver transplantation
Incorrect Answer ImageC.Gastroesophageal endoscopy
Incorrect Answer ImageD.Increase propranolol
Correct Answer ImageE.Paracentesis

A 35-year-old woman comes to the emergency department because of chest pressure for 30
minutes. She has had similar episodes intermittently over the last 5 years, usually when sleeping.
Over the last year, the symptoms have been more frequent and severe and occasionally
associated with a severe migraine headache. The pain is midsternal and is described as pressure
that extends like a "band around her chest." The emergency department physician is initially
dubious that the pain is cardiac in origin because the woman has no coronary disease risk factors.
Electrocardiogram, however, shows 2 mm ST-segment elevations and inverted T-waves in leads
II, III, and aVF plus 1 mm ST-segment depressions in leads V1 through V5. The patient is given
aspirin, nitroglycerin, morphine, and oxygen, and by the time the cardiologist arrives at the
emergency department, the patient's electrocardiogram has returned to normal. Which of the
following is the most likely explanation for these findings?
Incorrect Answer ImageA.Diffuse intimal thickening with focal areas of atherosclerotic narrowing
Incorrect Answer ImageB.Intermittent thrombus formation and lysis in the left anterior descending
artery
Incorrect Answer ImageC.Intermittent thrombus formation and lysis in the right coronary artery
Incorrect Answer ImageD.Plaque rupture and thrombus formation in the left anterior descending
artery
Correct Answer ImageE.Spasm with transiently increased coronary vascular tone in the right
coronary artery

A 25-year-old Caucasian man with a history of depression and anxiety disorder is brought to the
emergency department by his roommate in an unconscious state. Upon arriving at home that
evening, the roommate found the patient lying on the floor, not breathing "very well." The
roommate called paramedics and the patient was intubated in the field before being brought to
the emergency department. According to the roommate, the patient's depression had been getting
worse for the last few weeks. The patient had stopped taking his medications and refused to keep
his appointments with his psychiatrist. On examination the patient is comatose, unresponsive,
and orally intubated. His pulse is 100/min, respirations are 8/min, and blood pressure is 120/70
mm Hg. He is afebrile. His pupils are constricted. Lungs are clear to auscultation. Cardiovascular
examination shows no rub or gallop. Abdomen is soft, nontender, and nondistended. Bowel
sounds are decreased. Extremities show no pitting edema. He does withdraw all 4 extremities on
painful stimuli. Blood chemistry, including blood glucose, is within normal limits. CT scan of
the head is normal. What is the most likely etiology of this patient's neurologic status? 
Incorrect Answer ImageA.Amitriptyline overdose
Incorrect Answer ImageB.Barbiturate overdose
Correct Answer ImageC.Opiate ingestion
Incorrect Answer ImageD.Serotonin syndrome
Incorrect Answer ImageE.Tyramine interaction
Incorrect Answer ImageF.Wernicke's encephalopathy
A 48-year-old woman with a history of partial epilepsy is brought to the emergency department
20 minutes after having a witnessed seizure at the bus stop. Her friend states that the patient's left
hand started jerking, then she fell to the ground and her whole body jerked for 2 minutes. She
had urinated during this seizure. Her seizures had been well controlled with phenytoin prior to
this episode. Current medications include phenytoin. She has a history of alcohol abuse and has
been sober for 2 months. She is alert and in no distress. Examination shows no abnormalities.
Which of the following diagnostic tests is the most appropriate to determine the cause of this
patient's breakthrough seizure?
Incorrect Answer ImageA.Alcohol level
Incorrect Answer ImageB.Complete blood count (CBC)
Incorrect Answer ImageC.Electroencephalogram
Incorrect Answer ImageD.Head CT scan
Correct Answer ImageE.Phenytoin level

A 63-year-old man comes to the physician for a follow-up visit. He has had a cough productive
of sputum for the past 3 months and had a similar episode 1 year ago. He has had no fever or
chills. Today he is "feeling better." He has a history of hypercholesterolemia. He has smoked 3
packs of cigarettes daily for 50 years. He is in no acute distress. His temperature is 37.0ºC
(98.6ºF), pulse is 90/min, respirations are 18/min, and blood pressure is 135/85 mm Hg. Chest
examination shows decreased breath sounds, bilateral basilar wheezes, and distant heart sounds.
The anteroposterior diameter of the chest is increased. Pulmonary function testing shows: 
Total lung capacity  100%
FEV1 80%
Forced vital capacity  70%
FEV1/FVC  68%
Peak flow  74%
FEV25–75% 74%
Fifteen minutes after the administration of an albuterol inhaler, FEV1 is 85%. Which of the
following is the most appropriate next step in management? 
Incorrect Answer ImageA.Perform a chest radiograph
Incorrect Answer ImageB.Perform a methacholine challenge test
Incorrect Answer ImageC.Start a salmeterol inhaler
Incorrect Answer ImageD.Start azithromycin
Correct Answer ImageE.Vaccinate against influenza and Streptococcus pneumoniae

A 28-year-old man with a history of primary sclerosing cholangitis comes to the physician
because of a 3-month history of foul-smelling, greasy diarrhea. He has had difficulty seeing at
night, progressive jaundice, and a 5.5 kg (12 lb) weight loss. He has a 15-year history of
ulcerative colitis that has been in remission for the past year. His temperature is 37°C (98.6°F).
Examination shows scleral icterus and scattered excoriations over the skin. Which of the
following is the most likely cause of this patient's recent symptoms? 
Incorrect Answer ImageA.Bacterial overgrowth
Correct Answer ImageB.Bile salt deficiency
Incorrect Answer ImageC.Celiac sprue
Incorrect Answer ImageD.Pancreatic insufficiency
Incorrect Answer ImageE.Whipple disease

A 69-year-old woman comes to the physician because of exertional chest pain for the past 18
months. She says that her chest pain always occurs after she walks 2 blocks at a fast pace. She
does not have the pain any other time. She has a history of non-insulin-dependent diabetes and
hypertension, and has smoked one pack of cigarettes daily for the past 35 years. Vital signs are
within normal limits. Physical examination shows no abnormalities. An exercise stress test
shows ST-segment depressions in leads V1, V2, and V3 after 4 minutes of exercise that is
reversible with rest. A cardiac catheterization shows 80% stenosis of the proximal left main
coronary artery. Which of the following is the most appropriate treatment for this patient? 
Incorrect Answer ImageA.Immediately perform percutaneous balloon angioplasty
Incorrect Answer ImageB.Prescribe sublingual nitroglycerin as needed for pain and a daily aspirin
Incorrect Answer ImageC.Re-examine her in 6 months and ask her to stop exercising until then
Correct Answer ImageD.Send her for coronary artery bypass grafting
Incorrect Answer ImageE.Start a beta-blocker and aspirin, and if symptoms persist, schedule
coronary artery bypass grafting

A 30-year-old man with a 12-year history of epilepsy is brought to the emergency department
with multiple full-body convulsions over 1 hour with persistent altered mental status between
convulsions. Current medications include lamotrigine and carbamazepine. Examination shows
repetitive tonic jerking of all extremities. He is administered a high dose of IV benzodiazepines
and his seizures stop. Which of the following is the most appropriate next step in management?
Incorrect Answer ImageA.Electroconvulsive therapy
Correct Answer ImageB.Intravenous levetiracetam
Incorrect Answer ImageC.Intravenous propofol
Incorrect Answer ImageD.Oral carbamazepine
Incorrect Answer ImageE.Oral ethosuximide

A 24-year-old graduate student comes to the university health clinic because of complaints of
intermittent nonproductive cough and wheezing. She usually has these symptoms around the
time of her menses and has no other pulmonary symptoms between episodes. The symptoms
usually last 3 or 4 days and gradually resolve spontaneously. She has no other past medical
history and smokes only at dinner parties, approximately once every few months. She takes
ibuprofen for treatment of premenstrual cramping. She appears comfortable but is mildly
tachypneic. Physical examination shows high-pitched expiratory wheezes in both lung fields and
a moderately impaired air movement; the flow is measured at 90 L/min (normal, >250 L/min). A
chest x-ray is normal. Which of the following mechanisms best explains this patient's
bronchospasm? 
Incorrect Answer ImageA.Decreased leukotriene production
Incorrect Answer ImageB.Increased cyclooxygenase expression
Correct Answer ImageC.Increased leukotriene production
Incorrect Answer ImageD.Increased production of prostaglandins
Incorrect Answer ImageE.Increased prostacyclin production

A 42-year-old, HIV-positive woman comes to the emergency room because of abdominal pain
and fever. She has noticed increasing pain in her right upper quadrant. Her husband has noticed
that her eyes appear "yellow." Her past medical history is unremarkable. She does not smoke,
consume alcohol, or take medications. Her temperature is 38.4°C (101.2°F), pulse is 102/min,
and blood pressure is 110/72 mm Hg. She is obese with icteric sclera. Palpation of the abdomen
shows mild to moderate right upper quadrant pain. The remainder of the physical examination is
normal. An ultrasound of her right upper quadrant is limited due to her body habitus but shows
dilated intrahepatic ducts. Laboratory data are: 
WBC 13,100/mm3
Hct 42%
Hb 14 g/dL
Platelets 255,000/mm3
Na 138 mEq/L
K 4.2 mEq/L
Cl 105 mEq/L
HCO3 22 mEq/L
Creatinine 1.1 mg/dL
Total bilirubin 5.3 mg/dL
Conjugated bilirubin 4.3 mg/dL
ALT 92 U/L
AST 88 U/L
Alkaline phosphatase 250 IU/L
After antibiotics are started, the patient undergoes an ERCP, which shows a smooth tapering of
the common hepatic duct and an abrupt termination of the cystic duct with a large filling defect
and nonvisualization of the gallbladder. Which of the following is responsible for this patient's
presentation? 
Incorrect Answer ImageA.AIDS cholangiopathy
Incorrect Answer ImageB.Hemolysis
Correct Answer ImageC.Mirizzi syndrome
Incorrect Answer ImageD.Pancreatic head neoplasm
Incorrect Answer ImageE.Sphincter of Oddi dysfunction

A 63-year-old man comes to the physician because of episodic chest pain. He states he has had
chest pain on two or three occasions in the past 6 months. The pain has always been short-lived,
and he has not required any nitroglycerin to relieve the pain. He has a history of type 2 diabetes
mellitus and coronary artery disease. He had a myocardial infarction (MI) 17 years ago, and he
underwent a 4-vessel coronary artery bypass graft 2 years ago. He is currently taking metoprolol,
irbesartan, isosorbide dinitrate, and aspirin. His pulse is 62/min and his blood pressure is 100/60
mm Hg. Physical examination shows no abnormalities and a thallium stress test is performed.
The infarct on the inferior wall does not take up radiotracer on either the stress or resting images.
There is a second area adjacent to the old infarct that fails to take up radiotracer on stress images
but takes up radiotracer on the rest images. Which of the following is the next best step in patient
care?
Incorrect Answer ImageA.Add simvastatin to his medications
Correct Answer ImageB.Coronary angiography
Incorrect Answer ImageC.Coronary artery bypass grafting
Incorrect Answer ImageD.Increase the dosage of isosorbide dinitrate
Incorrect Answer ImageE.Increase the dosage of metoprolol
Incorrect Answer ImageF.No further management is warranted

A 67-year-old man with a 1-month history of pancreatic cancer comes to the emergency
department because of shortness of breath and sharp pain on his left side with inspiration for 4
hours. There is no radiation of the pain. He has no nausea, vomiting, or heartburn symptoms. He
denies fevers or cough. He has a history of hypertension treated with lisinopril and
hydrochlorothiazide. He does not smoke or drink alcohol. His pulse is 110/min and his
respirations are 22/min. His oxygen saturation is 87% on room air. Crackles are heard in the left
lung field. ECG shows sinus tachycardia. A CT scan of the chest shows a pulmonary embolus.
The patient is started on intravenous heparin. Two days later, he develops right-sided weakness.
A CT scan of the brain is shown. The heparin is stopped. Which of the following is the most
appropriate next step in management?
Incorrect Answer ImageA.Anticoagulation with aspirin only
Incorrect Answer ImageB.Doppler ultrasonography of the lower extremities
Correct Answer ImageC.IVC filter placement
Incorrect Answer ImageD.No further management is required
Incorrect Answer ImageE.Reinitiate anticoagulation in 1 week

A 36-year-old man comes to the emergency department because of a 3-hour history of passing
large amounts of bright red blood from his rectum. He had a similar episode of rectal bleeding 3
years ago. He has had frequent nosebleeds since childhood. His brother and father have also had
gastrointestinal bleeding episodes. His temperature is 38.0ºC (100.4ºF), blood pressure is 90/50
mm Hg, pulse is 143/min, and respirations are 16/min. He appears to be in moderate distress.
Examination shows telangiectasias of the buccal mucosa and nasal turbinates. Cardiac
examination shows a normal S1 and S2 without murmurs. The lungs are clear to auscultation.
The abdomen is mildly tender with absent bowel sounds. There is no peripheral edema. Which of
the following is the most likely diagnosis?
Incorrect Answer ImageA.Familial adenomatous polyposis syndrome
Incorrect Answer ImageB.Hereditary nonpolyposis colorectal cancer syndrome
Correct Answer ImageC.Osler-Weber-Rendu syndrome
Incorrect Answer ImageD.Peutz-Jeghers syndrome
Incorrect Answer ImageE.Ulcerative colitis

A 62-year-old man has a 2-week history of substernal heaviness and shortness of breath when
walking up the stairs to his apartment. He describes dizziness and difficulty breathing on
exertion for the past 6 months that is relieved by rest. His medications include simvastatin for
hypercholesterolemia and an albuterol inhaler for mild intermittent asthma. On physical
examination, he is found to have a grade 2/6 midsystolic murmur located at the right second
intercostal space with radiation to the neck, delayed and diminished carotid upstrokes, and
paradoxical splitting of the S2 heart sound. There is no peripheral edema and there are equal
pulses in all extremities. Temperature is 37.2ºC (98.9ºF), blood pressure is 130/95 mm Hg
bilaterally, pulse is 80/min, and respirations are 18/min. Chest x-ray shows an increased
cardiothoracic ratio, an accentuated left ventricular shadow, and mild blunting of the
costodiaphragmatic recesses on lateral view. Electrocardiogram shows normal rate and rhythm
with left ventricular strain pattern, and echocardiography shows a calcific bicuspid aortic valve,
transvalvular gradient of 55 mmHg, and aortic valve area of 0.8 cm2. Which of the following is
the most effective management step to reduce mortality?
Correct Answer ImageA.Aortic valve replacement
Incorrect Answer ImageB.Endocarditis prophylaxis
Incorrect Answer ImageC.Exercise stress test
Incorrect Answer ImageD.Furosemide therapy
Incorrect Answer ImageE.Increased physical activity
Incorrect Answer ImageF.Metoprolol therapy
Incorrect Answer ImageG.Percutaneous balloon valvuloplasty

A 69-year-old man is brought to the emergency department by his wife because of confusion and
strange behavior for 2 days. His wife reports that he has been mumbling and has not gotten out
of bed. Before the onset of his confusion, he met friends for playing cards daily. He has a history
of depression and Parkinson disease. Current medications include fluoxetine and benztropine. He
is in no distress. His temperature is 37.8°C (100°F), blood pressure is 130/85 mm Hg, and pulse
is 82/min. He is able to say his name but unable to give the date or his location. Examination
shows dilated, non-reactive pupils. The lungs are clear to auscultation. Cardiac examination
shows a normal S1 and S2; no murmurs are heard. Muscle strength is 5/5. Deep tendon reflexes
are normal. Laboratory studies show: 
Na+ 134 mEq/L 
Cl –
102 mEq/L
K+ 4.6 mEq/L 
HCO3 –
26 mEq/L 
Blood urea nitrogen (BUN)  16 mg/dL
Creatinine 0.8 mg/dL
Ca+ 9.2 mg/dL
A CT scan of the head shows mild cerebral atrophy but is otherwise unremarkable. Which of the
following is the most likely cause of this patient's delirium? 
Correct Answer ImageA.Anticholinergic agent overdose
Incorrect Answer ImageB.Hemorrhagic stroke
Incorrect Answer ImageC.Hypoglycemia
Incorrect Answer ImageD.Natural disease progression
Incorrect Answer ImageE.Opiate overdose
A 64-year-old man with a 30-year history of gastroesophageal reflux disease (GERD) comes to
the emergency department because of 6 episodes of hematemesis occuring within the last 10
hours. Current medications are antacids. He does not drink alcohol, smoke cigarettes, or use
illicit drugs. His blood pressure is 115/75 mm Hg and pulse 80/min. Hemoglobin is 7.1 g/dL and
serum immunoassay for H. pylori is negative. Urea breath test is
negative. Esophagogastroduodenoscopy (EGD) shows a gastric ulcer with a bleeding vessel. The
bleeding is controlled with hemoclips. Which of the following is the most appropriate next step
in management? 
Incorrect Answer ImageA.Amoxicillin, clarithromycin, and omeprazole
Correct Answer ImageB.Biopsy of the ulcer margins
Incorrect Answer ImageC.Bismuth subsalicylate, metronidazole, omeprazole, and               
tetracycline
Incorrect Answer ImageD.Omeprazole
Incorrect Answer ImageE.Truncal vagotomy and pyloroplasty

A 63-year-old woman is brought to the emergency department by ambulance because of chest


pain. She states that the pain began during her morning walk. It started as a dull pressure over her
breastbone and then radiated to her left arm. Over the next few minutes, it escalated in intensity
and was not relieved by rest. At that point, she called 911 and was brought to the ED. Her past
medical history is significant for hypertension and hyperlipidemia. Her medications include
atenolol and simvastatin daily. On physical examination, her pulse is 60/min and her blood
pressure is 190/100 mm Hg. Which of the following is the most appropriate agent to lower this
patient's blood pressure?
Incorrect Answer ImageA.IV furosemide
Correct Answer ImageB.IV nitroglycerin
Incorrect Answer ImageC.Oral hydrochlorothiazide
Incorrect Answer ImageD.Oral metoprolol
Incorrect Answer ImageE.Sublingual nitroglycerin

A 35-year-old man comes to the physician because of an 8-month history of pain in his right arm
that began 1 month after he fractured his humerus. He states that when the pain first started, the
skin on his right arm felt sweaty and warm. He has recently also developed a constant burning
and aching sensation, and he has severe pain when the skin of his right arm is touched lightly. He
has had no fever, weight loss, generalized weakness, changes in vision, or episodes of
incontinence. Examination shows that the skin of the right forearm is cool, shiny, and cyanotic.
There is an early flexion contracture of the fingers and the wrist of the right hand. Which of the
following conditions best explains this patient's symptoms?
Correct Answer ImageA.Complex regional pain syndrome
Incorrect Answer ImageB.Dupuytren contracture
Incorrect Answer ImageC.Erb-Duchenne palsy
Incorrect Answer ImageD.Horner syndrome
Incorrect Answer ImageE.Klumpke palsy
A 39-year-old man comes to the emergency department because of a 2-day history of severe
epigastric pain that radiates to the back, nausea, and vomiting. He reports that he had 3 similar
episodes during the past year that required hospitalization. He drinks 16 to 20 beers daily. He has
smoked 2 packs of cigarettes daily for 20 years. His temperature is 38.4ºC (101ºF), blood
pressure is 114/70 mm Hg, and pulse is 110/min. Laboratory studies show: 
Hemoglobin 12.4 g/dL 
Na+ 141 mEq/L
Cl- 90 mEq/L
K + 
3.7 mEq/L 
HCO- 39 mEq/L
Urea nitrogen (BUN)  25 mg/dL
Creatinine 1.1 mg/dL 
Amylase 300 U/L 
Lipase 275 U/L
Which of the following complications is most likely in this patient? 
Correct Answer ImageA.Alveolar-capillary membrane destruction due to circulating phospholipase
Incorrect Answer ImageB.Development of a carcinoma as a result of pancreatic duct injury
Incorrect Answer ImageC.Hypercalcemia as a result of stimulation of parathyroid secretion
Incorrect Answer ImageD.Hypertension due to altered intravascular volume
Incorrect Answer ImageE.Hypoglycemia due to impaired insulin production

A 68-year-old man is brought to the emergency department by ambulance after being involved in
a car collision. He had been feeling unwell the past few days with symptoms of chest pain and
cough. At the accident site he received oxygen and a rigid cervical collar. A large-bore IV was
placed and normal saline was started. His Glasgow Coma Score (GCS) on arrival to the
emergency department was 10. He reported significant abdominal, back, and chest pain. He has a
history of hypertension, diabetes mellitus, and hypercholesterolemia. Vital signs on arrival were:
temperature 38.5ºC (101.3ºF), pulse 75/min, respirations 21/min, and blood pressure 110/76 mm
Hg. The cardiac monitor is placed showing ST elevation in leads II, III, and AVF. His physical
exam shows a supple neck and bruised anterior chest with breath and heart sounds diminished.
His epigastric area is tender and he has normal bowel sounds. He begins shouting that his chest,
abdomen, and back are even more painful and he is asking for something to relieve the pain. His
blood pressure drops to 76/55 mm Hg, with an O2 saturation of 90%. A Swan-Ganz catheter is
placed and the following readings are obtained:
Right atrial pressure  17 mm Hg (normal, 1–7 mm Hg)
Right ventricular pressure  15/10 mm Hg (normal, 15–30/0–5 mm
Hg)
Pulmonary capillary wedge 3 mm Hg (normal, 4–12 mm Hg)
pressure (PCWP) 
Cardiac output  3.0 L/min (normal, 3.5–5.5 L/min)
Systemic venous resistance 1,100 dynes/sec/cm5 (normal 770–1,500
(SVR)  dynes/sec/cm5) 
Which of the following is the most likely cause of this patient's hemodynamic instability? 
Correct Answer ImageA.Cardiogenic shock
Incorrect Answer ImageB.Cardiac tamponade
Incorrect Answer ImageC.Hypovolemic shock
Incorrect Answer ImageD.Neurogenic shock
Incorrect Answer ImageE.Sepsis

A 73-year-old man is brought to the emergency department because of altered speech for 2
hours. He has a history of coronary artery disease, a transient ischemic attack 2 years ago, and
atrial fibrillation. The patient's wife states that he is right-handed. Current medications include
warfarin, which he takes inconsistently. He is fully conscious, but is not able to answer
orientation questions. His temperature is 37ºC (98.6ºF), blood pressure is 140/90 mm Hg, pulse
is 84/min and irregularly irregular, and respirations are 21/min. Examination is shown (see media
file). Muscle strength is 5/5 in all extremities. Sensation is normal. Which of the following
arteries is most likely affected? 
Incorrect Answer ImageA.Basilar artery
Incorrect Answer ImageB.Left anterior cerebral artery
Correct Answer ImageC.Left middle cerebral artery
Incorrect Answer ImageD.Posterior cerebral artery
Incorrect Answer ImageE.Right anterior cerebral artery
Incorrect Answer ImageF.Right middle cerebral artery

A 59-year-old man comes to the physician complaining of weight loss, jaundice, pruritus, and
vague right upper quadrant abdominal pain that have been present and progressing for the past 3
months. He has had ulcerative colitis for at least 30 years. For the past 10 years he has also been
known to have sclerosing cholangitis. Physical examination shows hepatomegaly. Laboratory
studies show a bilirubin level of 16 mg/dL with direct (conjugated) bilirubin of 13 mg/dL,
minimally elevated transaminase levels, and an alkaline phosphatase level 10 times the upper
limit of normal. Ultrasound was limited due to overlying bowel gas. Abdominal CT scan shows
ductal dilation in both hepatic lobes and a small gallbladder. The right and left hepatic ducts
could not be visualized. This patient's obstruction is most likely located in which of the
following structures?
Incorrect Answer ImageA.Ampulla of Vater
Incorrect Answer ImageB.Common bile duct
Correct Answer ImageC.Confluence of hepatic ducts
Incorrect Answer ImageD.Cystic duct
Incorrect Answer ImageE.Head of the pancreas

A 34-year-old woman is brought to the emergency department by her husband because of the
sudden onset of palpitations while resting. She also complains of shortness of breath and has
mild chest pressure. She is currently taking a multivitamin supplement and oral contraceptives.
Her pulse is 153/min, respirations are 19/min, and blood pressure is 110/71 mm Hg. She appears
anxious. Physical examination shows no carotid bruits or jugular venous distention. Lungs are
clear to auscultation. No abnormal heart sounds are heard. Her abdomen is soft and nontender,
and there is no clubbing, cyanosis, or edema. An ECG shows a narrow complex tachycardia at a
rate of 155/min. A carotid massage is performed. Which of the following mechanisms best
explains how the carotid massage will help control this patient's condition?
Incorrect Answer ImageA.Increase conduction in the Purkinje fiber system
Correct Answer ImageB.Increase in AV conduction delay
Incorrect Answer ImageC.Increase in AV conduction velocity transmission
Incorrect Answer ImageD.Modulating atrial myocyte electrical conduction
Incorrect Answer ImageE.Reducing the refractory time of SA nodal action potentials

A 36-year-old woman presents to the emergency department disoriented and confused. She has a
staring look. She is unable to coordinate muscle activity during voluntary movement. When she
speaks, she breaks each of the words into syllables. Her blood pressure is 125/88 mm Hg, pulse
88/min, and respirations 16/min. Her glucose level is 35. She reports a history of smoking one
pack of cigarettes per day and drinking 6 to 10 beers per night. Physical examination is limited
due to the patient’s altered mental status, but there are no focal neurologic deficits. Her
extraocular eye muscle examination is grossly abnormal. Which of the following IV substances
should most likely be administered first?
Incorrect Answer ImageA.Glucose
Incorrect Answer ImageB.Haloperidol
Incorrect Answer ImageC.Lorazepam
Correct Answer ImageD.Thiamine
Incorrect Answer ImageE.Valproic acid

A 53-year-old woman comes to the physician because of a 6-month history of fatigue and
generalized pruritus. She reports a 2-kg (4-lb) weight loss. There is a history of sinusitis and
recurrent oral and vaginal candida infections. She is sexually active with her husband. She does
not smoke or use illicit drugs, but she drinks 2 to 3 beers daily. Current medications include baby
aspirin, multivitamins, and artificial tears. Her temperature is 36.7°C (98°F). The sclerae are
mildly icteric. There are excoriations noted on all four extremities, trunk, and back. The liver is
enlarged and non-tender. Laboratory studies show: 
Alkaline phosphatase  260 U/L
Bilirubin, total  3.1 mg/dL
Alanine aminotransferase (ALT)  42 U/L
Aminotransferase (AST)  35 U/L
Which of the following is the most likely diagnosis? 
Incorrect Answer ImageA.Alcoholic hepatitis
Incorrect Answer ImageB.Autoimmune hepatitis
Incorrect Answer ImageC.Hepatocellular carcinoma
Correct Answer ImageD.Primary biliary cirrhosis
Incorrect Answer ImageE.Primary sclerosing cholangitis

A 55-year-old man comes to the emergency department because of progressive and severe lower
back pain for the past 2 weeks and urinary and bowel incontinence since yesterday. He has had
no fever, chills, shortness of breath, or recent trauma. He has a history of small cell lung
carcinoma (SCLC) treated with chemotherapy 3 years ago. He takes no medications. His brother
has a history of ankylosing spondylitis. His temperature is 36.8°C (98.2°F), pulse is 80/min and
respirations are 22/min. On physical examination, muscle strength is 4/5 bilaterally in the lower
extremities. Which of the following additional findings is most likely in this patient?
Correct Answer ImageA.Diminished anal sphincter tone
Incorrect Answer ImageB.Hypotension
Incorrect Answer ImageC.Lower extremity hyperreflexia
Incorrect Answer ImageD.Point tenderness in the sacral region
Incorrect Answer ImageE.Positive Babinski sign

A 38-year-old woman comes to the clinic for a routine checkup. She has no current complaints
about her health, but wishes to make sure all of her health care maintenance is "up-to-date." Her
past medical history is significant for ulcerative colitis diagnosed at the age of 29, an
appendectomy, and borderline hypertension. She states that her ulcerative colitis is well
controlled, with 3 to 4 moderately formed bowel movements per day. She rarely sees any blood
in her stool, eats normally, and has no abdominal pain. Her review of systems is otherwise
unremarkable, as is her family medical history. Her medications include sulfasalazine and folic
acid supplements. She exercises regularly. Regarding her preventive health care history, she
states that she has recently had an influenza vaccine and she performs self-breast examinations
but has never had a mammogram, a colonoscopy, or had her lipid level checked. She is curious
to know if she needs these evaluations. Her temperature is 37.0ºC (98.6ºF), blood pressure is
138/81 mm Hg, pulse is 73/min, and respirations are 20/min. Physical examination is within
normal limits. Which of the following is the most appropriate next step in management?
Correct Answer ImageA.Colonoscopy with biopsy
Incorrect Answer ImageB.Electrocardiogram (ECG)
Incorrect Answer ImageC.Flexible sigmoidoscopy
Incorrect Answer ImageD.Lipid level
Incorrect Answer ImageE.Mammogram

A 65-year-old woman with a history of myocardial infarction 8 months ago is brought to the
emergency department by ambulance with shortness of breath. For the past 2 hours, she has
experienced dyspnea at rest and has been coughing up some reddish sputum. She denies chest
pain. She is a 35-pack-year smoker and drinks 20 ounces of alcohol a week. She appears to be in
distress and is breathing 60% oxygen through a face mask. Her blood pressure is 80/60 mm Hg.
She is unable to complete full sentences. During the exam, she coughs up sputum, which is
bloodstained. There is evidence of jugular venous distention and pedal edema. There is no chest
wall tenderness; however, fine crepitations are heard throughout the chest on auscultation. There
are no murmurs or rubs. Her rhythm strip is shown. What is the best next step in management?
Incorrect Answer ImageA.Dobutamine
Incorrect Answer ImageB.Dopamine
Incorrect Answer ImageC.Echocardiography
Incorrect Answer ImageD.Hydralazine
Incorrect Answer ImageE.IV loop diuretic, nitrates, and morphine
Incorrect Answer ImageF.MUGA scan
Correct Answer ImageG.Synchronized cardioversion
Incorrect Answer ImageH.Verapamil

A 71-year-old man is brought to the physician by his granddaughter because she is concerned
that he has been "acting funny" for the last 9 months. The granddaughter has noticed that he has
been confused and has had difficulty remembering where he is or with whom he is talking. The
patient has been paranoid, accusing the housekeeper and longtime friends of trying to steal from
him. He has a history of hypertension, osteoarthritis, and mild depression that has been
adequately controlled with medication. There have been no recent major life stressors. Current
medications include acetaminophen, aspirin, hydrochlorothiazide, atenolol, and fluoxetine. His
temperature is 37.0ºC (98.6ºF), pulse is 68/min, respirations are 20/min, and blood pressure is
142/80 mm Hg. Neurologic examination is normal. A mini-mental status examination shows a
score of 19/30. Laboratory studies show:
Hemoglobin 13.8 mg/dL
Leukocyte count 7,000/mm3
Platelet count 276,000/mm3
Serum:
Na+ 138 mEq/L
Cl -
104 mEq/L
K+ 4.4 mEq/L
HCO3 -
23 mEq/L
Urea nitrogen (BUN) 18 mg/dL
Glucose  110 mg/dL
Creatinine 0.9 mg/dL
Calcium 9.1 mg/L
Vitamin B12 750 pg/mL (normal,200-900)
Rapid plasma reagin is negative. A CT scan of the head shows diffuse atrophy but no masses.
Which of the following would be the most appropriate next step to confirm the diagnosis before
starting treatment? 
Incorrect Answer ImageA.Apolipoprotein E level
Incorrect Answer ImageB.Brain biopsy
Incorrect Answer ImageC.Lumbar puncture
Incorrect Answer ImageD.Magnetic resonance imaging (MRI) of the brain
Correct Answer ImageE.No additional tests

A 22-year-old man with Crohn's disease is admitted to the hospital because of a 3-day history of
abdominal distention, nausea, and vomiting. He has not passed flatus or had bowel movements
for the past 2 days. Prior to the onset of these symptoms, he denies abdominal pain, diarrhea,
bloody stools, or fevers for approximately 2 years. He takes no medications. His temperature is
37.4°C (99.3°F). Abdominal examination reveals diffuse mild tenderness to palpation; bowel
sounds are absent. Rectal examination shows no stool. Which of the following is the most likely
cause of this patient's symptoms? 
Correct Answer ImageA.Fibrosis and a stricture in the terminal ileum
Incorrect Answer ImageB.Fistula from the ileum to the sigmoid
Incorrect Answer ImageC.Gastric outlet obstruction
Incorrect Answer ImageD.Obstructing cecal carcinoma
Incorrect Answer ImageE.Obstructing ileal carcinoid

A previously healthy 23-year-old man comes to the physician because of chest pain. He says he
was playing basketball when his "heart felt funny like it was beating out of place," and he
experienced central chest pain. He denies recreational drug use and admits to drinking alcohol
during "weekend parties." Cardiac examination shows a fourth heart sound. A harsh crescendo-
decrescendo systolic murmur is heard at the apex. The murmur becomes worse when the patient
is asked to stand from a sitting position and improves when the patient is asked to squat. His
carotid pulse is brisk in upstroke and bifid. Treatment with metoprolol and verapamil is begun.
Three weeks later, he returns to the physician because his condition has not improved. He
continues to have chest pain and shortness of breath. ECG from 2 weeks ago shows nonsustained
ventricular tachycardia. His current ECG shows no abnormalities. An echocardiogram shows an
ejection fraction of 70%. Which of the following is the next best step in patient care?
Incorrect Answer ImageA.Digoxin
Incorrect Answer ImageB.Enalapril
Correct Answer ImageC.Myectomy
Incorrect Answer ImageD.Spironolactone
Incorrect Answer ImageE.Warfarin

A 17-year-old girl is brought to the emergency department because of headache, vomiting, and
progressive weakness in her legs and right arm for 6 days. She also has had bowel and bladder
incontinence. One week ago, she had a cough, sneezing, and runny nose. Her medical history is
unremarkable, and she takes no medications. She appears lethargic. Her temperature is 38.2ºC
(100.8ºF), blood pressure 110/72 mm Hg, and pulse 77/min. Neurologic examination shows
right-sided facial weakness. Muscle strength is 4/5 in the right upper extremity and 2/5 in the
lower extremities. There is poor rectal tone. An MRI of the brain and spine shows hyperintense
lesions throughout the white matter and spinal cord. Which of the following is the most likely
diagnosis? 
Correct Answer ImageA.Acute disseminated encephalomyelitis
Incorrect Answer ImageB.Guillain-Barré syndrome
Incorrect Answer ImageC.Polymyositis
Incorrect Answer ImageD.Progressive multifocal leukoencephalopathy
Incorrect Answer ImageE.Transverse myelitis

A 43-year-old man comes to the physician for a follow-up examination 2 months after he
underwent upper endoscopy because of difficulty swallowing solid foods. Endoscopy showed a
smooth, tapered stricture of the distal esophagus that was dilated with an endoscopic balloon
dilator. A biopsy specimen of the esophagus showed chronic inflammation and fibrosis. His
symptoms of difficulty swallowing that he had prior to the dilation have resolved. He smoked 2
packs of cigarettes daily for 19 years and quit 2 years ago. He rarely drinks alcohol. Which of the
following is the most appropriate next step in management? 
Incorrect Answer ImageA.Famotidine
Correct Answer ImageB.Lansoprazole
Incorrect Answer ImageC.Magnesium hydroxide
Incorrect Answer ImageD.Metoclopramide
Incorrect Answer ImageE.No medication is necessary

A 64-year-old man is undergoing coronary angiography after a stress echocardiogram showed a


region of reversible ischemia in the left anterior descending (LAD) artery distribution. Coronary
catheterization is performed, and a 70% stenosis is detected in the mid-LAD, across which a
stent is placed. The patient tolerates the procedure well and is sent to the observational unit. Over
the next few hours, he develops fever, fatigue, and myalgias. His temperature is 38.1°C
(100.6°F), blood pressure is 130/80 mm Hg, pulse is 110/min, and respirations are 12/min.
Oxygen saturation is 98% on room air. Physical examination shows cyanosis of the big toes and
a lacy erythematous rash over the lower extremities. Cardiovascular and respiratory
examinations are normal. Laboratory studies show:
White blood cells  6,700/mm3
Neutrophils  70%
Lymphocytes  20%
Eosinophils  7%
Hematocrit  42%
Platelets  357,000/mm3
Na +
139 mEq/L
K +
5.1 mEq/L
Cl− 105 mEq/L
HCO3 −
22 mEq/L
Urea nitrogen  14 mg/dL
Creatinine  3.0 mg/dL 
Which of the following is the most likely cause of the patient's condition? 
Correct Answer ImageA.Cholesterol embolization syndrome (CES)
Incorrect Answer ImageB.Contrast-induced nephropathy
Incorrect Answer ImageC.Immune complex deposition nephropathy
Incorrect Answer ImageD.Intravascular volume depletion
Incorrect Answer ImageE.Poor cardiac output

A 44-year-old man presents to his physician (click media file) because of progressive weakness
in his extremities and one side of his face for the past 1 year. He reports difficulty swallowing
and slurred speech recently. He denies bowel or bladder incontinence, pain, difficulty with
memory, or numbness. His vital signs are: temperature 37.1°C (98.8°F), pulse 75/min,
respirations 16/min and blood pressure 132/88 mm Hg. Oxygen saturation is 99% on room air.
Physical examination shows tongue fasciculations. Muscle strength is 3/5 in the upper and lower
extremities. Deep tendon reflexes are brisk. Sensation is normal. Electromyogram shows
widespread axonal disease. Which of the following is the most appropriate counseling about the
patient's prognosis?
Incorrect Answer ImageA.Bladder and bowel dysfunction is a common complication
Incorrect Answer ImageB.Dementia and mental status changes will become more pronounced
Incorrect Answer ImageC.His children each have a 50% chance of developing the disease
Correct Answer ImageD.Respiratory failure and death within 3 to 5 years is common
Incorrect Answer ImageE.Sensory deficits and neuropathic pain are late findings

A 35-year-old Caucasian woman comes to her primary care physician with a two month history
of fatigue and itching, worst when she wears wool sweaters. She has tried taking oral
diphenhydramine for her pruritus with minimal improvement. She denies weight loss, fevers, or
night sweats. She has no significant history of past illness, transfusions, or use of intravenous
drugs. She denies taking any other medications or recent travel. Both her parents are alive and
healthy. Her temperature is 36.6ºC (98ºF), blood pressure is 110/65 mm Hg, and pulse is 75/min.
Her head and neck examination show no scleral icterus or thyroid abnormalities. Her chest and
cardiac examinations are normal. Abdominal examination shows an enlarged liver and mild right
upper quadrant tenderness to palpation without distension. Skin examination shows several
xanthelasma along her lower eyelids and erythema in the regions of excoriation. An ultrasound
of her right upper quadrant is normal. Serum studies show:
Aspartate aminotransferase (AST) 131 U/L
Alanine aminotransferase (ALT) 120 U/L
Alkaline phosphatase 750 U/L
Total cholesterol 630 mg/dL
Total bilirubin 2.0 mg/dL
Gamma globulins Normal
Which of the following serum studies will most likely be elevated? 
Incorrect Answer ImageA.Antimicrosomal antibody titer
Correct Answer ImageB.Antimitochondrial antibody titer
Incorrect Answer ImageC.Anti-Sm antibody titer
Incorrect Answer ImageD.Antismooth muscle antibodies
Incorrect Answer ImageE.Hepatitis C antibody profile

A 69-year-old man with a 25-year history of hypertension comes to the emergency department
because of progressive shortness of breath for 1 week. He admits to dietary indiscretion on
several occasions during the past 2 weeks, including multiple foods high in salt. Current
medications include hydrochlorothiazide, nifedipine, and aspirin. His temperature is 37.0°C
(98.6°F), pulse 113/min, and respirations 28/min, blood pressure 180/60 mm Hg. Oxygen
saturation is 91% on 6 L nasal cannula. Physical examination shows he is in moderate respiratory
distress and appears anxious and diaphoretic. There is jugular venous distension (JVD) to the
angle of the jaw. The heart rate is tachycardic and regular. Bilateral diffuse crackles are heard on
auscultation. The point of maximal impulse is laterally displaced; an S3 is present. There is 2+
pedal edema. ECG shows sinus tachycardia with left ventricular hypertrophy. Echocardiogram is
performed and is notable for a left ventricular ejection fraction (LVEF) of 30%. X-ray film of the
chest is shown. After treatment of the acute exacerbation, which of the following medications
will most likely decrease this patient's risk of mortality?
Incorrect Answer ImageA.Acebutolol
Incorrect Answer ImageB.Amlodipine
Correct Answer ImageC.Carvedilol
Incorrect Answer ImageD.Digoxin
Incorrect Answer ImageE.Diltiazem
Incorrect Answer ImageF.Furosemide
Incorrect Answer ImageG.Hydralazine
Incorrect Answer ImageH.Nitroglycerin
Incorrect Answer ImageI.Triamterene

A 28-year-old man is brought to the emergency department after being involved in a bar room
fight. During the altercation, he was punched in the left eye. He transiently lost consciousness
but awoke on his own. He felt pain over his eye, and the soft tissue around his eye swelled
considerably over the next 30 minutes. His past medical history is unremarkable. He is afebrile
and normotensive. There is marked soft-tissue swelling around the orbit. Visual acuity is intact,
as is the afferent pupillary reflex. CT scan of the orbits is obtained and shown. Which of the
following is the most likely complication of this injury?
Incorrect Answer ImageA.Anterior mandibular tooth numbness
Correct Answer ImageB.Anterior maxillary tooth numbness
Incorrect Answer ImageC.Blindness
Incorrect Answer ImageD.Lateral rectus muscle entrapment
Incorrect Answer ImageE.Trismus

A 63-year-old man comes to the physician after the pharmacist tells him that "his medications
are wrong." He has a history of cirrhosis and portal hypertension caused by alcohol abuse. There
is no history of gastrointestinal bleeding. He recently retired from his job as an airline pilot. He
smokes one pack of cigarettes daily. He no longer drinks alcohol. Co-administration of which of
the following is most likely to lead to an adverse drug effect? 
Incorrect Answer ImageA.Lactulose and oral potassium
Incorrect Answer ImageB.Metoprolol and isosorbide dinitrate

Incorrect Answer ImageC.Prednisone and inhaled albuterol

Incorrect Answer ImageD.Spironolactone and hydrochlorothiazide


Correct Answer ImageE.Spironolactone and oral potassium

A 23-year-old man sustains multiple stab wounds to his left chest. At the emergency department,
his pulse is 110/min, blood pressure is 70/45 mm Hg, and central venous pressure 4 mmHg. He
is diaphoretic, anxious, and notably pale. It is quickly determined in the initial assessment that he
has a left hemopneumothorax, and a chest tube is inserted. A total of 380 mL of blood is
recovered initially, and another 120 mL is suctioned during the next hour. He is also
administered 2 liters of lactated Ringer's, followed by 2 units of blood. Reassessment at the end
of the first hour shows that his lung is expanded. However, his central venous pressure has
increased to 22 mmHg and his neck veins have become distended. Vital signs are notable for a
pulse of 115/min and blood pressure of 85/70 mmHg.He remains diaphoretic and in moderate
distress. Which of the following is the most likely diagnosis?
Incorrect Answer ImageA.Continued bleeding
Incorrect Answer ImageB.Intrinsic cardiogenic shock
Correct Answer ImageC.Pericardial tamponade
Incorrect Answer ImageD.Tension pneumothorax
Incorrect Answer ImageE.Vasodilatory (distributive) shock

A 27-year-old woman is brought to the emergency department after suddenly losing


consciousness, falling to the ground, and shaking for approximately 2 minutes. During this
episode she lost control of her bladder. She has never had an episode like this before. She has
had progressive weakness of the right side of her body for the past 2 weeks and a headache and
nausea for the last 5 days. She has had no recent fevers, chills, or sick contacts. She emigrated
from Honduras and is employed as a migrant farm worker. Muscle strength is 4/5 in the right
upper extremity, 3/5 in the right lower extremity, and 5/5 on the left side. A Babinski sign is
present on the right. Her leukocyte count is 7,200/mm3. She is HIV seronegative. Lumbar
puncture and cerebrospinal fluid analysis shows elevated protein and a leukocyte count of
110/mm3 with 12% eosinophils. Gram stain of the cerebrospinal fluid shows no organisms. An
MRI scan of the brain is shown. Which of the following is the most appropriate next step in
management?
Correct Answer ImageA.Albendazole and dexamethasone
Incorrect Answer ImageB.Amphotericin B plus flucytosine
Incorrect Answer ImageC.Mefloquine and symptomatic treatment
Incorrect Answer ImageD.Pyrimethamine plus sulfadiazine
Incorrect Answer ImageE.Surgical resection and shunt placement

A 57-year-old man comes to the physician for follow-up after a sigmoidoscopy for rectal
bleeding. He had a colonoscopy 5 years ago and was told to follow up in 10 years. However, he
had some recent blood per rectum and sigmoidoscopy was performed. It showed internal
hemorrhoids and a 4 mm polyp in the mid rectum that was removed. Histologic examination
showed it to be a tubular adenoma. He has had no change in bowel habits, change in stool
caliber, or unintended weight loss. His medical history is unremarkable. Three consecutive fecal
occult blood tests are negative. Which of the following is the most appropriate next step in
management?
Incorrect Answer ImageA.A bowel regimen with increased fluids, fiber, and stool softeners
Incorrect Answer ImageB.Plan a repeat colonoscopy in 1 year
Incorrect Answer ImageC.Repeat sigmoidoscopy in 6 weeks to assure complete removal of the
polyp
Correct Answer ImageD.Schedule a colonoscopy now
Incorrect Answer ImageE.Treat his hemorrhoids and reevaluate in 6 weeks

An 80-year-old woman is brought to the emergency department because of dyspnea on exertion.


Her blood pressure is 100/70 mm Hg and pulse is 75/min. Physical examination shows distended
jugular veins and distant heart sounds. There is a third heart sound. There is hepatomegaly and
2+ pedal edema. ECG shows occasional premature ventricular contractions and low voltage.
Chest x-ray shows clear lung fields with a dilated cardiac silhouette. An echocardiogram shows
significant left ventricular wall thickening with a speckled pattern, along with severe dilation of
the left and right atria. Which of the following is the most likely diagnosis?
Incorrect Answer ImageA.Alcoholic cardiomyopathy
Correct Answer ImageB.Amyloidosis
Incorrect Answer ImageC.Hemochromatosis
Incorrect Answer ImageD.Tuberculosis
Incorrect Answer ImageE.Viral myocarditis

A 42-year-old man comes to the physician because of 3 episodes of seizures that were witnessed
by his roommate in the last 2 months. He has had no fever, chills, rigors, photophobia, vomiting,
or neck stiffness. He has a history of hypertension and type 2 diabetes mellitus. Current
medications include lisinopril, metformin, and glipizide. He emigrated from India 1 year ago. He
does not smoke, drink alcohol, or use drugs. His temperature is 37.1°C (98.8°F), blood pressure
is 150/90 mm Hg, pulse 82/min, and respirations 14/min. Neurologic examination shows no
abnormalities. There is no lymphadenopathy. Lungs are clear to auscultation. Cardiac
examination shows normal S1and S2; no murmurs are heard. CT scan of the head shows multiple
calcified non-enhancing hypodense lesions that are <1 cm in size; there are 4 calcified smaller
lesions. The patient is started on phenytoin. Which of the following is the most appropriate next
step in management?
Correct Answer ImageA.Albendazole
Incorrect Answer ImageB.CNS radiation treatment
Incorrect Answer ImageC.Metronidazole
Incorrect Answer ImageD.Stop glipizide
Incorrect Answer ImageE.Valproic acid

A 72-year-old man with longstanding COPD and diabetes mellitus is admitted to the hospital for
worsening dyspnea requiring intubation and sent to the intensive care unit (ICU). On hospital
day 14, he winced upon palpation of his right upper quadrant. He has remained intubated for 2
weeks and has had several unsuccessful attempts at weaning. Total parenteral nutrition was
initiated on hospital day 7 because of severe gastroparesis and high gastric residual volumes
during attempted enteral feeding. His temperature, blood pressure and pulse are normal. Cardiac
examination is normal and lungs show coarse breath sounds, wheezes, and a long expiratory
phase. Abdominal examination shows decreased bowel sounds without distension or ascites.
Ultrasound of the abdomen was significant for gallbladder thickening and pericholecystic fluid
without gallstones. Laboratory studies are shown below:
White blood cells 16,000/mm3
Hematocrit 41%
Platelets 235,000/mm3
Sodium 139 mEq/L
Potassium 4.5 mEq/L
Chloride 105 mEq/L
Bicarbonate 24 mEq/L
Urea nitrogen 17 mg/dL
Creatinine 1.6 mg/dL
Glucose 190 mg/dL
AST 98 U/L
ALT 105 U/L
Alkaline phosphatase 260 U/L
Total bilirubin 3.7 mg/dL
Conjugated bilirubin 2.8 mg/dL
Lipase 40 U/L
What is the most likely diagnosis?
Correct Answer ImageA.Acalculous cholecystitis
Incorrect Answer ImageB.Acute pancreatitis
Incorrect Answer ImageC.Acute calculous cholecystitis
Incorrect Answer ImageD.Chronic cholecystitis
Incorrect Answer ImageE.Ischemic hepatitis

A 64-year-old man comes to the emergency department via ambulance because of epigastric
pain. He was outside playing golf when he suddenly felt extreme "crushing" epigastric pain
along with shortness of breath and profuse sweating. His golfing partner immediately called 911.
The patient has a long-standing history of hypertension and type II diabetes mellitus. His
medications include metformin, amlodipine, and hydrochlorothiazide. His last HbA1c was 9%.
His temperature is 37.7°C (99.9°F), pulse is 126/min, respirations are 24/min, and blood pressure
is 166/94 mm Hg. Oxygen saturation is 99% in room air. The patient appears moribund and is
sweating excessively. Cardiopulmonary examination shows normal JVP, regular rate and rhythm
without extra heart sounds or murmurs, and clear lungs to auscultation. An ECG shows T-wave
inversion and ST-segment elevation of greater than 1 mm in leads II, III, and aVF. Coronary
angiography will most likely show disease in which of the following vessels?
Incorrect Answer ImageA.Left circumflex artery
Incorrect Answer ImageB.Left anterior descending artery
Incorrect Answer ImageC.Left main coronary artery
Incorrect Answer ImageD.Posterior descending artery
Correct Answer ImageE.Right coronary artery

n 80-year-old woman comes to the emergency department because of the sudden onset of lower
extremity weakness that caused her to collapse while getting out of bed this morning. She denies
any history of trauma or back pain. She has no history of significant medical illnesses and is not
on any medication. Her temperature is 37.1°C (98.8°F), pulse is 80/min, respirations are 14/min,
and blood pressure is 140/90 mm Hg. On physical examination, she is alert and oriented to time,
place and person. Her sensation and strength in the upper extremities are intact. Lower extremity
examination shows weakness bilaterally with loss of pain and temperature sensation. Vibration
and proprioception are intact. Deep tendon reflexes are absent bilaterally. Abdominal
examination shows a distended bladder. Which of the following is the most likely diagnosis?
Incorrect Answer ImageA.Anterior cerebral artery occlusion
Correct Answer ImageB.Anterior spinal artery occlusion
Incorrect Answer ImageC.Cauda equina syndrome
Incorrect Answer ImageD.Guillain-Barré syndrome
Incorrect Answer ImageE.Thoracic spinal cord compression

A 66-year-old man comes to the emergency department with nausea, vomiting, fever, and right
upper quadrant pain. He has had these symptoms for the past several days. His past medical
history is notable for diabetes and coronary artery disease. His last hemoglobin A1c taken 2 weeks
ago was 8.2%. His medications include insulin, atorvastatin, metoprolol, and lisinopril. His
temperature is 37.8°C (100.1°F), pulse is 98/min, and blood pressure is 122/75 mm Hg. Head
and neck examination is normal. Chest is clear to auscultation. Heart shows a normal S1 and S2
with no murmurs, rubs, or gallops. Palpation of the right upper quadrant causes severe abdominal
pain. There are decreased deep tendon reflexes. Laboratory studies show a white blood cell count
of 15,200 cells/mm3. An ultrasound of the right upper quadrant shows poor visualization of the
gallbladder because of marked reflection from the gallbladder wall. Which of the following is the
most appropriate treatment for this patient's current condition? 
Incorrect Answer ImageA.Cholecystectomy and intravenous ciprofloxacin
Correct Answer ImageB.Cholecystectomy and intravenous ciprofloxacin and metronidazole
Incorrect Answer ImageC.Cholecystectomy and intravenous metronidazole
Incorrect Answer ImageD.HIDA scan (cholescintigraphy)
Incorrect Answer ImageE.Intravenous ciprofloxacin, metronidazole and observation.

 62-year-old man is hospitalized in the intensive care unit and is currently recovering from an
aortic valve replacement for infectious endocarditis complicated by perivalvular abscess. While
recovering from his operation, he suffers sudden shortness of breath, pleuritic chest pain, and
tachycardia. An electrocardiogram shows right ventricular strain and sinus tachycardia but is
otherwise unremarkable. A stat CT angiogram confirms the diagnosis of pulmonary embolism,
showing a right subsegmental blood clot. In addition to his recent valve replacement, the patient
has a history of a recent hemorrhagic stroke believed to be related to septic emboli, hypertension,
and diabetes. Which of the following is the most appropriate management of this patient?
Correct Answer ImageA.Inferior vena cava filter
Incorrect Answer ImageB.Low molecular-weight heparin
Incorrect Answer ImageC.Thrombolytic therapy
Incorrect Answer ImageD.Unfractionated heparin
Incorrect Answer ImageE.Unfractionated heparin and warfarin

A 29-year-old woman comes to the neurology clinic complaining of two weeks of unsteadiness.
She says that the symptoms have come on gradually and are worsening progressively. She has
never had these symptoms before. The patient denies any visual difficulty or weakness. A
presumptive diagnosis of multiple sclerosis is discussed; however, neuroimaging studies reveal a
well-circumscribed cystic mass with a mural nodule in the right cerebellar hemisphere.
Significant hematologic values include:
Hematocrit 55%
Red blood cells 6 × 106/mm3
The patient undergoes surgical excision of the cerebellar tumor. Further clinical investigations
lead to discovery of a vascular tumor in the right retina. Which of the following syndromes is
most likely responsible for this clinical presentation? 
Incorrect Answer ImageA.Li-Fraumeni syndrome
Incorrect Answer ImageB.Neurofibromatosis type 1
Incorrect Answer ImageC.Neurofibromatosis type 2
Incorrect Answer ImageD.Tuberous sclerosis
Correct Answer ImageE.von Hippel-Lindau syndrome

A 47-year-old man with ulcerative colitis comes to the emergency department with severe
abdominal pain. Over the previous 2 weeks, he has suffered from numerous bloody, voluminous
bowel movements (10-12/day), spiking fevers, and a diffuse, crampy abdominal pain. He has
been taking increasing amounts of loperamide for his diarrhea without relief. Additionally,
approximately 1 week ago the patient began taking oral steroids left over from a previous flare of
ulcerative colitis, but with no improvement in symptoms. Past medical history is significant for a
long, difficult course of ulcerative colitis with multiple flares requiring hospitalization. Over the
last few years, however, his ulcerative colitis has been controlled with sulfasalazine and 6-
mercaptopurine. His temperature is 39.0ºC (102.2ºF), blood pressure is 108/60 mm Hg, pulse is
112/min, and respirations are 26/min. Examination shows a middle-aged man in moderate
distress. His abdomen is diffusely tender with rebound pain and hypoactive bowel sounds.
Laboratory studies show: 
Hematologic:
Hematocrit 32%
Leukocytes 19,200/mm3 (92% neutrophils) 
Platelets 155,000/mm3
Sedimentation rate 45 mm/hr
Blood chemistry:
Bicarbonate 19 mEq/L
Chloride 88 mEq/L
BUN 32 mg/dL
Creatinine 1.2 mg/dL
Potassium 3.3 mEq/L
Sodium 133 mEq/L
Glucose 98 mg/dL
A stat abdominal radiograph shows a large, dilated colon that fills most of the abdomen. No free
air is evident on upright or decubitus films. Broad-spectrum antibiotics, intravenous fluids, and
intravenous steroids are initiated. The patient is made NPO, a nasogastric tube is placed, and
antidiarrheal medications are discontinued. The patient is admitted to the hospital for treatment
of an ulcerative colitis flare. He continues to have bloody diarrhea, spiking fever, and worsening
abdominal pain. A repeat abdominal radiograph 1 day later is unchanged, still showing a large
colon and no free air. At this time, which of the following is the most appropriate treatment?
Incorrect Answer ImageA.6-Mercaptopurine
Incorrect Answer ImageB.Azathioprine
Correct Answer ImageC.Colectomy
Incorrect Answer ImageD.Colonoscopy with decompression
Incorrect Answer ImageE.Cyclosporine
Incorrect Answer ImageF.Infliximab

A 68-year-old woman comes to the emergency department because of chest pain. Over the past 8
months, she has been experiencing dull chest pain every time she walks her grandchildren to
school. Initially, the pain did not bother her and she "walked through the pain," but now the pain
has become progressively worse over the past 6 months and comes on at shorter and shorter
distances. Lately, she has to stop and rest for 5 minutes so that the pain subsides. She denies
chest pain at rest. The patient undergoes a stress test which is abnormal. She is subsequently
prescribed aspirin, nitrates, and atenolol. Her serum LDL cholesterol is 140 mg/dL, and
atorvastatin is added; she is then advised to start a diet low in saturated fats and high in fiber and
to continue exercising on a regular basis. Despite this treatment regimen, she continues to
complain of symptoms. She undergoes coronary angiography, which reveals patchy disease in
the distal part of the left anterior descending artery and the distal part of the circumflex artery.
What is the most appropriate next step in management?
Incorrect Answer ImageA.Coronary artery bypass grafting
Incorrect Answer ImageB.Dobutamine echocardiography
Correct Answer ImageC.Percutaneous transluminal coronary angioplasty with stent insertion
Incorrect Answer ImageD.Prescribe enalapril
Incorrect Answer ImageE.Stress echocardiography
Incorrect Answer ImageF.Stress thallium scan

A 39-year-old woman with a 15-year history of scleroderma and hypertension comes to the
physician because of difficulty swallowing solid food for 6 months. She notes that food "sticks"
in her lower chest. She denies any problems swallowing liquids. She has had no fevers or weight
loss. Current medications include erythromycin, ranitidine, topical glucocorticoids, and
lisinopril. Which of the following is the most likely cause of this patient's symptoms? 
Incorrect Answer ImageA.Development of a squamous carcinoma in the upper third of the
esophagus
Incorrect Answer ImageB.Recent return of peristaltic activity in the body of the esophagus
Incorrect Answer ImageC.Reverse peristalsis in the body of the esophagus
Incorrect Answer ImageD.Reverse peristalsis of the lower esophagus
Correct Answer ImageE.Scarring at the lower esophagus because of chronic acid reflux

A 27-year-old man comes in for an annual physical examination. He is asymptomatic. Cardiac


examination shows there is a systolic ejection murmur at the left lower sternal border. The
murmur becomes louder with standing and a Valsalva maneuver, but the intensity of the murmur
is reduced with squatting. An electrocardiogram shows marked left ventricular hypertrophy.
Which of the following is the most appropriate next step in management? 
Incorrect Answer ImageA.Antiarrhythmic drugs
Correct Answer ImageB.Avoid competitive exercise
Incorrect Answer ImageC.Referral for an automatic implantable cardiac defibrillator
Incorrect Answer ImageD.Referral for surgical myectomy
Incorrect Answer ImageE.Start beta-blockers

A 35-year-old man with no significant medical history comes to the emergency department
because of severe chest pain. The pain is worse with ingestion of cold or hot liquids. There is no
history of weight loss and no oral lesions. The remainder of the physical examination is normal.
Cardiac examination shows no abnormalities. Which of the following is most appropriate to
confirm the diagnosis?
Incorrect Answer ImageA.24-hour pH monitoring
Incorrect Answer ImageB.Barium swallow
Incorrect Answer ImageC.Endoscopy
Correct Answer ImageD.Manometry
Incorrect Answer ImageE.Trial of PPI

A 72-year-old man comes to the emergency department because of the acute onset of chest pain
that began 3 hours ago while he was reclining on his sofa. The pain is sharp, left-sided, and
radiates to his left arm. He has a history of hypertension and diabetes mellitus and is currently
taking hydrochlorothiazide and metformin. He denies alcohol consumption, although he has a
long history of smoking. His pulse is 88/min and his blood pressure is 135/82 mm Hg. Physical
examination shows carotid bruits without jugular venous distension. The chest is clear to
auscultation, and no abnormal heart sounds are heard. Abdominal and peripheral examinations
are benign. A lateral chest radiograph is shown. Further evaluation rules out myocardial
infarction, and his pain is relieved with sublingual nitroglycerin. Which of the following is the
most appropriate next step in management?
Correct Answer ImageA.Contrast-enhanced computed tomography of the chest
Incorrect Answer ImageB.Initiate beta-blocker therapy
Incorrect Answer ImageC.Magnetic resonance imaging of the thoracic spine
Incorrect Answer ImageD.Open lung biopsy
Incorrect Answer ImageE.Percutaneous lung biopsy

A 31-year-old man comes to the physician because of a 9-month history of difficulty swallowing
solids and liquids. He has been waking in the middle of the night coughing. He has had a 7 kg
(15 lb) weight loss. His medical history is unremarkable and he takes no medications.
Esophagogram shows a dilated esophagus with a smoothly tapered distal esophagus. Which of
the following manometry findings is most likely in this patient? 
Incorrect Answer ImageA.Decreased peristalsis and decreased resting lower esophageal sphincter
(LES) pressure
Correct Answer ImageB.Decreased peristalsis and elevated resting LES pressure
Incorrect Answer ImageC.Increased peristalsis and normal resting LES pressure
Incorrect Answer ImageD.Increased number of simultaneous contractions with elevated resting LES
pressure
Incorrect Answer ImageE.Normal peristalsis and elevated resting LES pressure
A 63-year-old man is admitted to the hospital because of progressive shortness of breath, fatigue,
and hypotension for 7 months. He is in moderate respiratory distress. His temperature is 37.0ºC
(98.6ºF), pulse is 118/min, respirations are 22/min, and blood pressure is 68/42 mm Hg. His
oxygen saturation is 90% on 2 liters of oxygen via nasal cannula. Examination shows jugular
venous distension to the angle of the jaw that increases during inspiration. Cardiac examination
shows distant heart sounds, an early diastolic sound, and weak peripheral pulses. Lungs are clear
on auscultation. Abdominal examination shows ascites and hepatomegaly. There is 3+ pedal
edema. A radiograph of the chest shows pericardial calcifications. Echocardiography shows
atrial septal shifting to the left with inspiration, and dilated inferior and superior vena cava. A
tentative diagnosis of constrictive pericarditis is made. Pulmonary artery catheterization is most
likely to show which of the following?
Incorrect Answer ImageA.Right atrial (RA) pressure 12 mm Hg, right ventricular (RV) pressure
21/11 mm Hg, pulmonary artery (PA) pressure 20/10 mm Hg, pulmonary capillary wedge
pressure (PCWP) 8 mm Hg
Incorrect Answer ImageB.RA pressure 18 mm Hg, RV pressure 28/18 mm Hg, PA pressure 40/23
mm Hg, PCWP 24 mm Hg
Correct Answer ImageC.RA pressure 20 mm Hg, RV pressure 32/20 mm Hg, PA pressure 30/20 mm
Hg, PCWP 21 mm Hg
Incorrect Answer ImageD.RA pressure 20 mm Hg, RV pressure 40/20 mm Hg, PA pressure 75/30
mm Hg, PCWP 28 mm Hg
Incorrect Answer ImageE.RA pressure 22 mm Hg, RV pressure 45/25 mm Hg, PA pressure 80/35
mm Hg, PCWP 10 mm Hg

A 42-year-old woman with a 15-year history of heartburn comes to the physician because of
difficulty swallowing large bites of food for 4 months. She has had no difficulty with soft foods
or liquids. She has had no fever, loss of appetite, fatigue, or weight loss. She takes no
medications. Her temperature is 37°C (98.6°F), blood pressure is 130/85 mm Hg, and pulse is
70/min. Cardiac examination shows a normal S1 and S2 without murmurs. Abdominal
examination is normal. Which of the following is the most likely explanation for her symptoms? 
Incorrect Answer ImageA.Adenocarcinoma in the lower third of the esophagus
Incorrect Answer ImageB.Barrett esophagus in the distal esophagus
Correct Answer ImageC.Fibrosis and narrowing at the distal esophagus
Incorrect Answer ImageD.Schatzki ring in the distal esophagus
Incorrect Answer ImageE.Squamous carcinoma in the middle third of the esophagus

A 67-year-old white man comes to the emergency department with a chief complaint of general
malaise for the past 2 days. He reports feeling nauseated during this time and vomited twice thus
far. He denies any sick contacts or recent travel. Review of systems is positive for drowsiness,
fatigue, headache, and dizziness. The patient admits to some “color change” in vision, nausea,
vomiting, palpitations, and shortness of breath. The patient has a past medical history of non-
insulin-dependent diabetes mellitus, congestive heart failure, and hyperlipidemia. His
medications include glipizide 20 mg, metoprolol, lisinopril, furosemide, digoxin, and
atorvastatin.On examination, the patient is in mild distress and appears slightly nervous and
diaphoretic. His temperature is 37.5ºC (99.5ºF), pulse is 58/min, respirations are 18/min, and
blood pressure 105/75 mm Hg. A 12-lead ECG appears as shown. The patient is instructed to
suspend all of his medications for the moment. Which of the following would be the most
appropriate step in the management of this patient?
Incorrect Answer ImageA.Cardiac enzymes
Incorrect Answer ImageB.Cardioversion
Incorrect Answer ImageC.Echocardiogram
Correct Answer ImageD.Electrolyte panel

A 70-year-old Asian man comes to the physician because of weight loss and "heartburn"
symptoms. The patient states that he has slowly been losing weight for approximately the last
year. He reports pain in his upper central abdomen and heartburn for the past two years. Physical
examination shows a thin-appearing man with temporal wasting, a small palpable mass in the
periumbilical region, and a positive fecal occult blood test. Upper endoscopy shows a friable
mass in the gastric fundus and biopsy demonstrates gastric adenocarcinoma. Which of the
following is a risk factor for this patient's disease?
Incorrect Answer ImageA.Aflatoxin exposure
Incorrect Answer ImageB.Arsenic exposure
Incorrect Answer ImageC.Dermatomyositis
Correct Answer ImageD.Pernicious anemia
Incorrect Answer ImageE.Radon exposure

A 67-year-old white woman comes to the outpatient clinic for her yearly physical. She reports
being in good health and requests a refill on her blood pressure medicine. The patient has a past
medical history of hypertension controlled with metoprolol and lisinopril and diabetes mellitus
managed with 20 units of subcutaneous insulin in the morning and in the evening. The patient
reports her glucose ranges from 100 to 160 mg/dL, which she monitors closely. The patient is in
no apparent distress. Her vital signs are temperature 37.0ºC (98.7ºF), pulse is 89/min,
respirations are 14/min, and blood pressure is 135/78 mm Hg. Chest auscultation showed (click
media file) with radiation to the carotids. The lungs are clear to auscultation bilaterally, and no
rales or rhonchi are appreciated. The patient's HbA1c is 6.5%. Her chemistry panel is within
normal limits. Which of the following is the most appropriate next step in the management of
this patient?
Incorrect Answer ImageA.Inform patient she must take prophylactic antibiotics
Incorrect Answer ImageB.Monitor patient closely
Incorrect Answer ImageC.Order chest radiograph
Incorrect Answer ImageD.Order ECG
Correct Answer ImageE.Order transthoracic echocardiography
Incorrect Answer ImageF.Schedule catheterization of the heart

A 49-year-old man with a long history of alcohol abuse is brought to the physician by his wife
because of gradually increasing confusion over the past two weeks. Initially, he felt drowsy
throughout the day and had difficulty sleeping at night. Since then, he has become confused and
occasionally disoriented as to time and day. He can recognize his wife and neighbors but is
unable to maintain casual conversations. He denies fever, chills, and abdominal pain. Six months
ago, he was admitted to the hospital with an upper gastrointestinal bleed secondary to esophageal
varices. His past medical history is significant for hypertension. He is currently taking enalapril,
spironolactone, and propranolol. On physical examination, he is lethargic and disheveled. His
temperature is 36.9ºC (98.4ºF), blood pressure is 112/64 mm Hg, pulse is 62/min, and
respirations are 18/min. Abdominal examination shows a firm liver edge with liver span
extending 7 cm below the right costal margin in the midclavicular line. There is no shifting
dullness and a spleen tip is not palpable. On a mental status examination, he recognizes the
physician but cannot name the date, the reason for this visit, or his home address. Neurologic
examination is nonfocal and asterixis is present. Which of the following is the most appropriate
management at this time?
Correct Answer ImageA.Lactulose only
Incorrect Answer ImageB.Lactulose plus neomycin
Incorrect Answer ImageC.Neomycin only
Incorrect Answer ImageD.Oral branched chain amino acid (BCAA) supplements
Incorrect Answer ImageE.Restriction of protein intake

A 57-year-old man comes to the outpatient medicine clinic for a regular visit. He has been in
good health and denies any new complaints other than a mild tremor of his hand which doesn't
resolve with activity. He is married and has one teenage child and works as an attorney. His past
medical history is remarkable for essential hypertension controlled with diet. His blood pressure
today is 170/92 mm Hg, and his pulse is 82/min. Physical examination shows a prominent,
laterally displaced apical impulse on heart exam, and a tremor of both upper extremities with
movement. The reflexes are normal bilaterally and there is no associated rigidity or bradykinesia.
Which of the following is the most appropriate therapy for this patient?
Incorrect Answer ImageA.α-1 receptor antagonist
Incorrect Answer ImageB.ACE inhibitors
Correct Answer ImageC.Beta blockers
Incorrect Answer ImageD.Calcium channel blockers
Incorrect Answer ImageE.Thiazide diuretic

A 63-year-old man comes to the emergency department with sudden onset of chest pain,
diaphoresis, and shortness of breath. The pain radiates to his left jaw and back and is not
increased or relieved by changing body position. He denies fever or chills. He was diagnosed 4
years ago with hypertension and is being treated with hydrochlorothiazide. His vital signs are
temperature 36.5ºC (97.7ºF), pulse is 110/min, respirations are 22/min, and blood pressure is
185/90 mm Hg. Physical examination shows an elderly man in moderate distress. Examination
shows an apical impulse with a regular rhythm, no rubs, and asymmetric pulses on the upper
extremities. Breath sounds are diminished on the left. A chest radiograph shows a hazy aortic
knob with a mediastinal width of 9 cm. Which of the following findings can be associated with
this patient's current disease?
Incorrect Answer ImageA.Crescendo-decrescendo murmur
Correct Answer ImageB.Decrescendo murmur
Incorrect Answer ImageC.Dullness to percussion
Incorrect Answer ImageD.Hemoptysis
Incorrect Answer ImageE.Loud P2

A 33-year-old man comes to the physician because of painful ulcers on his penis that developed
from pustules over the last 4 days. He has had no fevers, myalgias, or fatigue. He was treated
with ceftriaxone for penile discharge 3 months ago. He takes no medications and has no known
drug allergies. External examination shows bilateral soft and tender ulcers with irregular borders
and a ring of erythema surrounding them on the penis, measuring approximately 1 cm in
diameter. The ulcers are associated with bilaterally tender and enlarged lymph nodes. Which of
the following is the most likely cause of these findings?
 Incorrect Answer ImageA.Chlamydia trachomatis 
 Correct Answer ImageB.Haemophilus ducreyi 
 Incorrect Answer ImageC.Herpes simplex virus
 Incorrect Answer ImageD.Klebsiella granulomatis
 Incorrect Answer ImageE.Treponema pallidum

Diagnostic
A previously healthy 2-year-old boy is brought to the emergency department because of a
temperature of 40℃ (104℉). While being comforted by his mother, his whole body stiffens,
followed by rhythmic contractions of his extremities and unresponsiveness lasting approximately
3 minutes. Ten minutes later he is alert, behaving normally, and without focal neurologic
findings or meningeal signs. Which of the following would be the most appropriate next step in
management?
 Incorrect Answer ImageA.Administer phenobarbital
 Incorrect Answer ImageB.Complete an electroencephalogram
 Correct Answer ImageC.Lower his temperature
 Incorrect Answer ImageD.Order a CT scan of the head
 Incorrect Answer ImageE.Perform a lumbar puncture

A 25-year-old African American man with sickle cell anemia comes to the emergency
department in a sickle cell crisis. He reports dyspnea and severe pain upon inspiration. His
temperature is 40°C (104°F), pulse is 120/min, and respirations are 34/min. His PO2 on room air
is 45 mm Hg. Chest x-ray shows a diffuse hazy infiltrate at the right lung base. The emergency
department resident gives the patient oxygen and fluids, administers analgesia, draws blood
cultures, and starts him on antibiotics. Which of the following is the most appropriate next step
in management?
 Incorrect Answer ImageA.Administer hydroxyurea
 Incorrect Answer ImageB.Initiate heparin therapy
 Incorrect Answer ImageC.Obtain MRI of the chest
 Correct Answer ImageD.Perform an exchange transfusion
 Incorrect Answer ImageE.Transfuse fresh frozen plasma

Three days after sustaining a myocardial infarction, a 68-year-old man develops of severe
abdominal pain. His temperature is 39℃ (102.2℉), pulse is 80/min, and respirations are 30/min
and shallow. Physical examination shows a soft abdomen with diffuse tenderness on palpation.
Rectal examination is guaiac positive. The patient has a newly elevated white blood cell count of
15,000/mm3. Which of the following is the most appropriate next step in management?
 Incorrect Answer ImageA.Abdominal radiographs
 Incorrect Answer ImageB.Antibiotic prophylaxis
 Correct Answer ImageC.Emergency mesenteric arteriogram
 Incorrect Answer ImageD.Endoscopic examination
 Incorrect Answer ImageE.Prophylaxis for stress ulcer

A 76-year-old man with a long history of medical problems is diagnosed with advanced liver
cancer. The physician reviews with the patient the few available treatment options, and the
patient selects a combination drug therapy currently in phase two clinical trials. This
combination therapy is a recent innovation and is not available outside of the clinical trial. When
contacting the clinical trial administrator, the physician is informed that the clinical trial is
nearing the completion of enrollment and will very soon stop accepting new patients. "We can
probably fit your patient in," says the administrator, "but only if he is enrolled in the next 24
hours." Admission to the clinical trial requires a "certificate of need" signed by two physicians.
To meet this criterion, the physician contacts a colleague and asks him to sign the certificate so
that the necessary paperwork can be completed within the 24-hour period. "I'll email you the
form, and you can sign it and fax it back to me," the physician says. "Let me know if you have
any questions." At this point, which of the following is the best response by the physician's
colleague? 
 Incorrect Answer ImageA.Agree to sign the form and schedule a time to more fully
examine the patient in the coming week
 Incorrect Answer ImageB.Ask to talk with the patient to be sure that he has fully
considered his options before proceeding
 Incorrect Answer ImageC.Call the clinical trial administrator and discuss the criteria for
inclusion in the study before proceeding further
 Incorrect Answer ImageD.Call the patient's physician and have him provide a full
briefing on the patient's medical history before proceeding
 Correct Answer ImageE.Refuse to sign the form under these circumstances and discuss
other available options with the patient's physician
 Incorrect Answer ImageF.Sign the form and send it back as quickly as possible to ensure
that the patient can be enrolled for the treatment

A 20-year-old man comes to his physician describing "yellow-colored eyes" for one day. For the
past week, he has had nausea, vomiting, malaise, joint pain, and headache. The patient denies
any history of intravenous drug use, blood transfusion, or sexual activity. He recently returned
from vacationing in an Indian community, which had an outbreak of a similar illness. His
temperature is 37.8ºC (100ºF), and there is right upper quadrant abdominal tenderness. Serum
bilirubin is 3.8 mg/dL, aspartate aminotransferase (AST) is 2830 U/L, and alanine
aminotransferase (ALT) is 2237 U/L. Which of the following is the most likely diagnosis? 
 Correct Answer ImageA.Hepatitis A
 Incorrect Answer ImageB.Hepatitis B
 Incorrect Answer ImageC.Hepatitis C
 Incorrect Answer ImageD.Hepatitis D
 Incorrect Answer ImageE.Hepatitis due to Epstein-Barr virus (EBV)

A 46-year-old woman comes to the emergency department because of fever and shaking chills
for one day. She recently underwent a modified radical mastectomy for infiltrating ductal
carcinoma of her right breast. She has an indwelling venous Hickman catheter through which she
has received one cycle of cyclophosphamide, methotrexate, and fluorouracil. Her temperature is
38.9°C (102°F) and there is erythema around the catheter site. Her leukocyte count is
12,000/mm3 with an 88% neutrophil predominance. Which of the following is the most likely
causal organism?
 Incorrect Answer ImageA.Bacteroides fragilis
 Incorrect Answer ImageB.Klebsiella  species
 Incorrect Answer ImageC.Pseudomonas aeruginosa
 Correct Answer ImageD.Staphylococcus epidermidis
 Incorrect Answer ImageE.Streptococcus pyogenes

 37-week gestational age female newborn, whose mother had an uncomplicated pregnancy, dies
24 hours after spontaneous vaginal delivery. Apgar scores were 2 and 6 at 1 and 5 minutes,
respectively. The neonate was found to have bilateral cleft lip and palate, microphthalmia, and
postaxial polydactyly. Autopsy shows holoprosencephaly and multiple cardiac anomalies. Which
of the following karyotypes would most likely be seen on examination of the patient’s
chromosomes?
 Correct Answer ImageA.Trisomy 13
 Incorrect Answer ImageB.Trisomy 21
 Incorrect Answer ImageC.45,XO
 Incorrect Answer ImageD.46,XY
 Incorrect Answer ImageE.47,XXX

A 14-year-old boy has pain in his proximal left tibia. The pain began 3 weeks earlier and was not
associated with any traumatic event. The pain is described as an intermittent dull ache during the
day. At night the pain is worse and sharp and occasionally awakens him from sleep. It is
substantially relieved by over-the-counter anti-inflammatory medications. Physical examination
shows point tenderness over the proximal tibia 2 cm below the insertion of the patellar tendon.
There is no erythema, swelling, or abnormal warmth over the area of tenderness. Radiographs of
the region reveal a 3 mm sharply demarcated, round, radiolucent area within the cortex of the
proximal tibia, surrounded by a dense reactive zone. Which of the following is the most likely
diagnosis?
 Incorrect Answer ImageA.Chondroblastoma
 Incorrect Answer ImageB.Enchondroma
 Incorrect Answer ImageC.Osgood-Schlatter disease
 Incorrect Answer ImageD.Osteochondroma
 Correct Answer ImageE.Osteoid osteoma

A 40-year-old woman, gravida 3, para 2, at 16 weeks' gestation has a grade 1 cervical


intraepithelial neoplastic lesion and low-grade squamous intraepithelial lesion on Pap smear.
Speculum examination reveals a closed cervical os without lesions. HPV/cytology cotesting is
performed 12 months later and shows that she is HPV positive. Which of the following is the
most appropriate next step in management?
 Incorrect Answer ImageA.Cervical conization
 Correct Answer ImageB.Colposcopy with indicated biopsies
 Incorrect Answer ImageC.CT scan of the pelvis
 Incorrect Answer ImageD.Laser ablation of the cervix
 Incorrect Answer ImageE.Repeat Pap smear in 6 months
A 60-year-old obese woman with a history of diabetes mellitus comes to the physician because
of a single episode of vaginal bleeding. She underwent menopause at age 52. She is sexually
active with a single partner and has a history of infertility. Which of the following is the most
appropriate next step in management? 
 Correct Answer ImageA.Endometrial biopsy
 Incorrect Answer ImageB.Fractional dilatation and curettage
 Incorrect Answer ImageC.No treatment required
 Incorrect Answer ImageD.Order coagulation studies
 Incorrect Answer ImageE.Progesterone therapy

A 34-year-old intravenous drug user comes to the emergency department because of increasing
difficulty breathing, generalized weakness, and nonproductive cough for 3 days. His temperature
is 38.3°C (101°F). The lungs are clear to auscultation. Chest x-ray shows diffuse interstitial
infiltrates bilaterally. Arterial blood gas analysis while the patient is breathing room air shows:
pH 7.44
PO2 55 mm Hg
PCO2 28 mm Hg
The patient is started on supplemental oxygen. Which of the following is the most appropriate
next step in management?
 Incorrect Answer ImageA.Administer amphotericin
 Incorrect Answer ImageB.Administer cefuroxime
 Incorrect Answer ImageC.Administer isoniazid, rifampin, pyrazinamide, and ethambutol
 Incorrect Answer ImageD.Administer penicillin
 Correct Answer ImageE.Administer trimethoprim-sulfamethoxazole

A 5-year-old boy is brought to the emergency department because of high fever and trouble
breathing. He began complaining of a worsening sore throat 5 hours ago and is now refusing not
only food but also fluids. His temperature is 39.6°C (103.2°F), pulse is 100/min, and respirations
are 42/min. The boy appears to be frightened, sitting forward in his father's lap with his neck
extended, and is drooling. Inspiratory stridor is audible. Which of the following is the most
appropriate next step in management?
 Incorrect Answer ImageA.Arterial blood gas analysis
 Incorrect Answer ImageB.Complete physical examination
 Incorrect Answer ImageC.Focused examination of the oropharynx
 Incorrect Answer ImageD.Immediate administration of aerosolized racemic epinephrine
 Correct Answer ImageE.Immediate placement of an artificial airway

A 27-year-old man with a history of alcohol use disorder and multiple admissions to psychiatric
hospitals comes to the emergency department because of auditory hallucinations. The man is
very agitated and delusional and reports that he is hearing a voice telling him that he is bad.
Haloperidol is administered intramuscularly. At the same time, an anticholinergic is
administered. Which of the following is the most likely reason for administering the
anticholinergic?
 Incorrect Answer ImageA.Prevention of seizures
 Incorrect Answer ImageB.Prevention of urinary retention
 Incorrect Answer ImageC.Prophylaxis against tardive dyskinesia
 Correct Answer ImageD.Reduction of akathisia
 Incorrect Answer ImageE.Reduction of heart rate

A 24-year-old woman with no significant past medical history comes to the physician for a
routine health maintenance examination. She has had noticeable weight loss despite an above
normal appetite over the past few weeks. She also notes occasional palpitations at rest as well as
increased nervousness and tremor. She takes no medications. Her sister had a "thyroid problem"
and was put on medication, which seemed to help. Physical examination shows a thin, anxious
woman with a less than normal BMI. Her pulse is 100/min and she has smooth "velvety" skin.
Her thyroid is mildly enlarged though not painful. Which of the following is the most likely
diagnosis?
 Incorrect Answer ImageA.Acute thyroiditis (suppurative)
 Correct Answer ImageB.Graves disease
 Incorrect Answer ImageC.Hashimoto thyroiditis
 Incorrect Answer ImageD.Papillary carcinoma of the thyroid
 Incorrect Answer ImageE.Subacute thyroiditis

A 55-year-old man with a history of coronary artery disease is admitted to the cardiac telemetry
unit with a diagnosis of unstable angina. He is treated with aspirin, heparin, and metoprolol. Five
days later, while still on these medications, he reports the sudden onset of pain in his left great
toe. Physical examination shows the toe is cold, pale, and painful. His platelet count is
30,000/mm3 and partial thromboplastin time is 60 seconds. Which of the following is the most
appropriate next step in management?
 Incorrect Answer ImageA.Administer colchicine
 Incorrect Answer ImageB.Discontinue aspirin
 Correct Answer ImageC.Discontinue heparin
 Incorrect Answer ImageD.Increase the metoprolol dose
 Incorrect Answer ImageE.Transfuse platelets

A 16-year-old girl was a restrained driver involved in a high-speed motor vehicle collision. She
was found unconscious at the scene. Primary trauma survey shows ecchymoses over her left
forehead, seat belt marks over her chest and abdomen, and no obvious bony deformities of her
extremities. A cross-table cervical spine film, chest x-ray film, and pelvic x-ray film are normal.
While undergoing a CT scan of the head, her blood pressure drops to 60/30 mm Hg with a pulse
of 130/min. Which of the following is the most likely cause of her hypotension?
 Correct Answer ImageA.Intra-abdominal injury
 Incorrect Answer ImageB.Pneumothorax
 Incorrect Answer ImageC.Proximal femur fracture
 Incorrect Answer ImageD.Spinal shock
 Incorrect Answer ImageE.Uncal herniation syndrome

A 40-year-old woman comes to the physician because of discharge from one of her breasts. She
has no past medical history. She is not lactating and has no family history of breast disease.
Physical examination shows a moderate amount of serosanguinous discharge expressed from the
left nipple. Cytologic examination of the fluid is negative for abnormal cells. Which of the
following is the most appropriate next step in management?
 Incorrect Answer ImageA.Advise the patient to return in 6 months for reevaluation
 Correct Answer ImageB.Obtain a mammogram
 Incorrect Answer ImageC.Perform a chest x-ray
 Incorrect Answer ImageD.Perform an excisional biopsy of the retroareolar area
 Incorrect Answer ImageE.Perform a needle biopsy of the retroareolar area

A 63-year-old man with hypertension comes to the emergency department because of weakness
and intermittent palpitations for 3 days. Two weeks ago, his physician prescribed a new blood
pressure medication, but the man is unable to recall which one. Physical examination shows no
abnormalities. An ECG shows evidence of an old inferior myocardial infarction and peaked T-
waves. Laboratory studies show a serum potassium of 6.6 mg/dL. Which of the following
medications is most likely responsible for his hyperkalemia?
 Incorrect Answer ImageA.Atenolol
 Correct Answer ImageB.Captopril
 Incorrect Answer ImageC.Furosemide
 Incorrect Answer ImageD.Hydrochlorothiazide
 Incorrect Answer ImageE.Verapamil

A 62-year-old widowed woman is referred to a psychiatrist. Since her husband died of colon
cancer over a year ago, the woman has had a 6.8 kg (15 lb) weight loss although she has not been
dieting. She reports constipation and rectal pain. A recent evaluation by a gastroenterologist,
including a colonoscopy and endoscopy, was unremarkable. The patient also reports persistent
fatigue, early morning awakening, and intermittent thoughts of ending her life so she can join her
husband. Which of the following is the most likely diagnosis?
 Incorrect Answer ImageA.Adjustment disorder with depressed mood
 Incorrect Answer ImageB.Bereavement
 Incorrect Answer ImageC.Conversion disorder
 Incorrect Answer ImageD.Generalized anxiety disorder
 Correct Answer ImageE.Major depressive disorder

An 85-year-old man comes to the emergency department because of the sudden onset of severe
mid-abdominal pain, diarrhea, and vomiting. He has no significant past medical history. His
temperature is 38.3℃ (101℉), blood pressure is 124/68 mm Hg, and pulse is 104/min. His
abdomen is moderately distended but without localized tenderness or peritoneal signs. His
leukocyte count is 26,000/mm3, hemoglobin is 14 g/dL, and hematocrit is 43%. ECG shows atrial
fibrillation with a ventricular rate of 94/min and no ischemic changes. Which of the following is
the most appropriate next step in management?
 Incorrect Answer ImageA.Anticoagulation with heparin
 Incorrect Answer ImageB.Check serum lactate level
 Correct Answer ImageC.CT angiogram of the abdomen
 Incorrect Answer ImageD.Stool culture
 Incorrect Answer ImageE.Upper endoscopy

A 2-week-old male infant born at 30 weeks gestation is in the neonatal intensive care unit. He
has been extubated after being treated for respiratory distress syndrome and received
indomethacin to successfully close a patent ductus arteriosus. He has been on slowly advancing
nasogastric feedings, which had to be discontinued due to abdominal distension and blood in the
stool. An abdominal radiograph shows pneumatosis intestinalis. Which of the following is the
most likely diagnosis?
 Incorrect Answer ImageA.Duodenal atresia
 Incorrect Answer ImageB.Meconium ileus
 Correct Answer ImageC.Necrotizing enterocolitis
 Incorrect Answer ImageD.Situs inversus
 Incorrect Answer ImageE.Volvulus

A 33-year-old man with type 1 diabetes comes to the physician because of fever, chills, and
cough for 4 days. He reports that he was well until 5 days ago when he developed fatigue,
generalized myalgia, sore throat, ear pain, and diarrhea followed over the past few days with a
nonproductive, dry cough and daily fevers. Laboratory studies show:
Leukocyte count 8,500/mm3
Sodium (Na+) 140 mEq/L
Alanine aminotransferase (ALT) 18 U/L
Aspartate aminotransferase (AST) 15 U/L
Alkaline phosphatase 50 U/L
Cold agglutinins Present
Chest x-ray shows diffuse interstitial infiltrates. Which of the following is the most likely causal
organism?
 Incorrect Answer ImageA.Klebsiella pneumoniae
 Incorrect Answer ImageB.Legionella pneumophila
 Correct Answer ImageC.Mycoplasma pneumoniae
 Incorrect Answer ImageD.Pneumocystis jirovecii
 Incorrect Answer ImageE.Streptococcus pneumoniae

A 30-year-old woman, gravida 3, para 1, aborta 2, comes to the emergency department in active
labor. She received no prenatal care and did not receive medical attention for her two prior
miscarriages. She vaginally delivers a 3,230-g (7-lb 2-oz) term male infant. The amniotic fluid
and umbilical cord are yellow in color. The neonate’s vital signs immediately after delivery are
within normal limits. Physical examination of the neonate's head, eyes, ears, nose, and throat
shows no abnormalities. The lungs are clear to auscultation. There is a grade 3/6 murmur audible
throughout the cardiac cycle that does not radiate, and the liver is palpable 4 cm below the right
costal margin. Analysis of the umbilical cord blood shows a hemoglobin of 11 g/dL and bilirubin
of 3.5 g/dL. Eight hours later, the neonate appears yellow and is tachypneic and tachycardic.
Which of the following is the most likely Rh antigen status of the mother and the neonate?
 Incorrect Answer ImageA.Rh-positive mother and Rh-positive neonate
 Incorrect Answer ImageB.Rh-positive mother and Rh-negative neonate
 Incorrect Answer ImageC.Rh-negative mother and Rh-negative neonate
 Correct Answer ImageD.Rh-negative mother and Rh-positive neonate
 Incorrect Answer ImageE.Rh-negative father and Rh-positive neonate

A 22-year-old woman, gravida 3, para 2, is diagnosed with a failed intrauterine pregnancy at 11


weeks' gestation. The cervix is closed and ultrasound identifies retained products of conception
in the uterus. Removal of the products of conception by suction and curettage is indicated to
prevent which of the following complications? 
 Incorrect Answer ImageA.Amniotic fluid embolism
 Incorrect Answer ImageB.Diabetes mellitus
 Correct Answer ImageC.Disseminated intravascular coagulation
 Incorrect Answer ImageD.Renal failure
 Incorrect Answer ImageE.Rh-D alloimmunization

A 69-year old man is found to have occult blood in his stool on a routine physical examination.
Colonoscopy shows a mass in his right colon and biopsy indicates an adenocarcinoma.
Abdominal CT scan shows no evidence of hepatic metastasis and lymph nodes do not appear
enlarged. The mass is removed at surgery and local lymph nodes are negative for neoplasm.
Which of the following serologic markers would be most appropriate to monitor disease
recurrence and maintain surveillance for metastasis?
 Incorrect Answer ImageA.α-fetoprotein
 Incorrect Answer ImageB.β-human chorionic gonadotropin
 Correct Answer ImageC.Carcinoembryonic antigen (CEA)
 Incorrect Answer ImageD.Carbohydrate Antigen 19-9 (CA 19-9)
 Incorrect Answer ImageE.Carbohydrate Antigen 125 (CA 125)

A 72-year-old woman comes to the physician because of new-onset blurred vision in her right
eye for 1 day. For the past month, she has had headaches, fever, malaise, fatigue, and stiffness in
her neck and shoulders. Her temperature is 38.7°C (101.6°F). Neurologic and funduscopic
examinations show no abnormalities. Laboratory studies show a leukocyte count of 6500/mm3, a
hematocrit of 32%, and an erythrocyte sedimentation rate of 110 mm/hour. Which of the
following is the most appropriate next step in diagnosis?
 Incorrect Answer ImageA.Bone marrow aspirate
 Incorrect Answer ImageB.Cerebral angiography
 Incorrect Answer ImageC.Lumbar puncture
 Incorrect Answer ImageD.Serum creatine kinase levels
 Correct Answer ImageE.Temporal artery biopsy

A 72-year-old woman comes to the physician because of new-onset blurred vision in her right
eye for 1 day. For the past month, she has had headaches, fever, malaise, fatigue, and stiffness in
her neck and shoulders. Her temperature is 38.7°C (101.6°F). Neurologic and funduscopic
examinations show no abnormalities. Laboratory studies show a leukocyte count of 6500/mm3, a
hematocrit of 32%, and an erythrocyte sedimentation rate of 110 mm/hour. Which of the
following is the most appropriate next step in diagnosis?
 Incorrect Answer ImageA.Bone marrow aspirate
 Incorrect Answer ImageB.Cerebral angiography
 Incorrect Answer ImageC.Lumbar puncture
 Incorrect Answer ImageD.Serum creatine kinase levels
 Correct Answer ImageE.Temporal artery biopsy

A 68-year-old man with an extensive history of tobacco use is admitted to the hospital because of
worsening cough, dyspnea, and fever for 1 week. Pulmonary function studies performed 1 year
ago revealed a total lung capacity of 135% predicted normal, a vital capacity of 52% predicted,
an FEV1 of 35% predicted, and an FEV1/FVC ratio of 40% predicted. Chest x-ray shows
hyperexpanded lung fields bilaterally and a dense infiltrate in the left lower lobe. Which of the
following is the most likely causal organism? 
 Incorrect Answer ImageA.Haemophilus influenzae
 Incorrect Answer ImageB.Legionella pneumoniae
 Incorrect Answer ImageC.Mycoplasma pneumoniae
 Incorrect Answer ImageD.Staphylococcus aureus
 Correct Answer ImageE.Streptococcus pneumoniae

A 68-year-old man with an extensive history of tobacco use is admitted to the hospital because of
worsening cough, dyspnea, and fever for 1 week. Pulmonary function studies performed 1 year
ago revealed a total lung capacity of 135% predicted normal, a vital capacity of 52% predicted,
an FEV1 of 35% predicted, and an FEV1/FVC ratio of 40% predicted. Chest x-ray shows
hyperexpanded lung fields bilaterally and a dense infiltrate in the left lower lobe. Which of the
following is the most likely causal organism? 
 Incorrect Answer ImageA.Haemophilus influenzae
 Incorrect Answer ImageB.Legionella pneumoniae
 Incorrect Answer ImageC.Mycoplasma pneumoniae
 Incorrect Answer ImageD.Staphylococcus aureus
 Correct Answer ImageE.Streptococcus pneumoniae

Thirty minutes after circumcision, a newborn boy has significant bleeding around the surgical
area. Physical examination shows large hematomas at the sites where he received his vitamin K
injection and first hepatitis B immunization; no other abnormalities are found. The mother and
father have no significant past medical history, but the boy's maternal uncle has a "bleeding
disorder" and had knee surgery as a teenager. Which of the following laboratory studies would
most likely be abnormal in the neonate?
 Incorrect Answer ImageA.Bleeding time
 Correct Answer ImageB.Partial thromboplastin time
 Incorrect Answer ImageC.Platelet count
 Incorrect Answer ImageD.Prothrombin time
 Incorrect Answer ImageE.Thrombin time

An 8-year-old boy is brought to the physician for a routine health maintenance examination. The
family has recently moved to the area and this is his first visit to the office. He has always been
in good health and is up to date with all of his immunizations. The mother reports that over the
past month, she has seen a number of bruises at various locations and he seems to be tired most
of the time. He is at the 5th percentile for height and at the 20th percentile for weight. His thumbs
are both hypoplastic; the other digits appear normal. The mother states that she has been told that
this is a birth defect. There are several round, smooth, flat, hyperpigmented areas of various sizes
on his trunk and extremities. He has scattered bruises and many petechiae, mostly on his
extremities. Which of the following is the most likely diagnosis?
 Incorrect Answer ImageA.Cri du chat syndrome
 Incorrect Answer ImageB.Down syndrome
 Correct Answer ImageC.Fanconi anemia
 Incorrect Answer ImageD.Fetal alcohol syndrome
 Incorrect Answer ImageE.Trisomy 18

A 36-year-old woman comes to the physician because of palpitations, fatigue, and dyspnea on
exertion for 1 week. She emigrated from Puerto Rico to the United States 14 years ago. She was
last seen 4 years ago when she was at 28 weeks’ gestation in her third pregnancy. She had
symptoms of dyspnea on exertion and an episode of hemoptysis at the time. Physical
examination at that time showed rales at both lung bases and a mid-diastolic murmur at the apex
preceded by an opening snap. Physical examination now shows an irregularly irregular pulse and
rales at both lung bases in addition to the previous cardiac findings. Which of the following
findings is most likely to be seen on an electrocardiogram?
 Correct Answer ImageA.Atrial fibrillation
 Incorrect Answer ImageB.Bigeminy
 Incorrect Answer ImageC.Premature atrial contractions
 Incorrect Answer ImageD.Premature ventricular contractions
 Incorrect Answer ImageE.Second-degree heart block

A 9-year-old boy is brought to the emergency department because of loss of consciousness 7


hours after being hit on the right side of the head with a baseball bat. His parents state that after
the accident, he was complaining of a headache and nausea. He vomited twice prior to “passing
out”. In the emergency department, his temperature is 36.7ºC (98.0ºF), pulse is 60/min,
respirations are 8/minute, and blood pressure is 140/82 mm Hg. He cannot be aroused. There is
right-sided pupillary dilatation and blurring of the optic disc margins with decreased venous
pulsations bilaterally. Which of the following complications has most likely occurred?
 Incorrect Answer ImageA.Cingulate herniation
 Incorrect Answer ImageB.Horner syndrome
 Incorrect Answer ImageC.Subdural hematoma
 Incorrect Answer ImageD.Tonsillar herniation
 Correct Answer ImageE.Uncal (transtentorial) herniation

A 48-year-old woman comes to the physician because of amenorrhea. She states that she has not
had a normal menstrual period for 2 months, although she has had some mild spotting recently.
Before that, she had been having heavy periods irregularly. She has depression, for which she
takes fluoxetine. Physical examination, including pelvic examination, is unremarkable. Which of
the following is the most appropriate next step in management? 
 Incorrect Answer ImageA.Check serum follicle-stimulating hormone (FSH)
 Correct Answer ImageB.Check urine human chorionic gonadotropin (hCG)
 Incorrect Answer ImageC.Perform an endometrial biopsy
 Incorrect Answer ImageD.Prescribe hormone replacement therapy
 Incorrect Answer ImageE.Prescribe medroxyprogesterone acetate

A 21-year-old man is brought to the emergency department by his parents after they find him
lying unresponsive on his bedroom floor. He has a history of intravenous drug abuse. His
temperature is 37.2ºC (99.0ºF), pulse is 52/min, and blood pressure is 110/70 mm Hg. He makes
only a single, shallow respiratory effort during 15 seconds of observation. Further examination
shows that he has constricted pupils, and he does not respond to noxious stimuli. Manual bag-
valve-mask ventilation with 100% oxygen is initiated. Which of the following is the most
appropriate next step in management?
 Incorrect Answer ImageA.Acidify the urine
 Incorrect Answer ImageB.Administer intravenous epinephrine
 Correct Answer ImageC.Administer naloxone
 Incorrect Answer ImageD.Alkalinize the urine
 Incorrect Answer ImageE.Insert a chest tube

A 3-year-old girl is undergoing evaluation for a 1-month history of intermittent fevers, fatigue,
malaise, and weight loss. For the past several days she has told her parents that her arms and legs
have been hurting. Her temperature is 38°C (100.4°F), pulse is 120/min, respirations are
20/minute, and blood pressure is 100/60 mm Hg. Her skin is pale and there are scattered
petechiae present. Peripheral blood smear shows a number of large round white blood cells with
dark-staining nuclei that almost fill the cytoplasm. Further cytogenetic and molecular studies are
performed. Based on the most likely diagnosis, which of the following translocations would be
considered a favorable prognostic factor?
 Incorrect Answer ImageA.t(1;19)
 Incorrect Answer ImageB.t(2;8)
 Incorrect Answer ImageC.t(8;14)
 Incorrect Answer ImageD.t(8;22)
 Correct Answer ImageE.t(12;21)

A 4-year-old boy is brought to the hospital because of respiratory distress. He has had several
previous hospital admissions for pneumonia. Past medical history is significant for the failure to
pass meconium at birth. His temperature is 38.4°C (101.1°F) and respirations are 35/min.
Physical examination shows a thin, malnourished boy who is at the 20th percentile for height and
below the 5th percentile for weight. Rales are heard over the left lower lobe and percussion over
this region is dull. Sputum culture grows Pseudomonas aeruginosa.  Which of the following
would most likely confirm the diagnosis?
 Incorrect Answer ImageA.Bronchoscopy
 Incorrect Answer ImageB.CT scan of the thorax
 Incorrect Answer ImageC.D-xylose absorption test
 Incorrect Answer ImageD.Pulmonary function tests
 Correct Answer ImageE.Sweat chloride test

A 48-year-old female is brought to the emergency department because of bizarre behavior. She is
unable to give a history and appears frightened and confused. The resident on call remembers
seeing her in the ER previously but is unable to recall specific details about her or her diagnosis.
On physical examination, the patient’s temperature is 37.3°C (99.1°F), blood pressure is 124/80
mm Hg, and pulse is 115/min. Her oral mucosa is dry and neurological examination is
unremarkable. Her speech is rapid and her thoughts appear inconsistent. Laboratory studies
show:
Sodium 153 mEq/L
Potassium 4.1 mEq/L
Chloride 119 mEq/L
Bicarbonate 24 mEq/L
Which of the following psychiatric medications is most likely responsible for these
abnormalities? 
 Incorrect Answer ImageA.Carbamazepine
 Incorrect Answer ImageB.Chlorpromazine
 Incorrect Answer ImageC.Imipramine
 Correct Answer ImageD.Lithium
 Incorrect Answer ImageE.Lorazepam

A 57-year-old man comes to the emergency department because of severe abdominal pain and
vomiting. The patient reports that over the past 6 hours he has had a dramatic progression of
initially mild midepigastric pain. In the past 2 hours, he began vomiting black thick material. He
has had no other symptoms and reports no food intake in the past 48 hours. While sitting in the
triage area of the emergency department he still has severe epigastric discomfort and continues to
vomit bilious, nonbloody material. The patient is escorted back to the examination area. Which
of the following is the most appropriate next step in management?
 Correct Answer ImageA.Assess his temperature, blood pressure, and pulse
 Incorrect Answer ImageB.Order a right upper-quadrant ultrasound
 Incorrect Answer ImageC.Order an abdominal CT scan
 Incorrect Answer ImageD.Initiate broad-spectrum antibiotic therapy
 Incorrect Answer ImageE.Obtain an endoscopic retrograde cholangiopancreatography
(ERCP)

A 1-month-old infant is brought to the physician for a well-child examination. He was born at 39
weeks’ gestation via uncomplicated, spontaneous vaginal delivery. During routine prenatal
screening for HIV infection, the mother had a positive enzyme-linked immunosorbent assay
(ELISA). A confirmatory Western blot assay was also positive. The mother is well and has a
CD4+ lymphocyte count of 610/mm3. Which of the following tests would best determine if the
child is infected with HIV?
 Incorrect Answer ImageA.ELISA
 Incorrect Answer ImageB.Northern blot
 Correct Answer ImageC.Polymerase chain reaction
 Incorrect Answer ImageD.Southern blot
 Incorrect Answer ImageE.Western blot

A 38-year-old woman comes to the physician because of weight gain and edema. She is an active
intravenous heroin user and has a medical history significant for endocarditis, which was treated
3 years ago. She reports that an HIV test performed 4 months ago was negative and that she has
been otherwise healthy. Her temperature is 37°C (98.6°F), blood pressure is 155/90 mm Hg, and
pulse is 80/min. Examination shows crackles at the bases of both lungs and extensive edema
involving her legs and trunk. Her urine is strongly positive for protein, and the urinary sediment
shows red cells and red cell casts. Her serum creatinine is 3.4 mg/dL and albumin is 2.9 g/dL.
Which of the following is the most likely diagnosis? 
 Correct Answer ImageA.Focal and segmental glomerulosclerosis
 Incorrect Answer ImageB.Interstitial nephritis
 Incorrect Answer ImageC.Nephrogenic diabetes insipidus
 Incorrect Answer ImageD.Obstructive uropathy
 Incorrect Answer ImageE.Prerenal azotemia

A 26-year-old female athlete comes to the emergency department after she awakens in the
middle of the night shaking, sweating, and gasping for breath. The patient tells the physician "I
felt like I was going to die." The patients states that the same thing happened to her during the
night one week ago. An ECG is normal. Her blood pressure is 120/80 mm Hg, and her pulse is
90/min. The rest of the physical examination is unremarkable. Which of the following is the
most likely diagnosis?
 Incorrect Answer ImageA.Generalized anxiety disorder
 Correct Answer ImageB.Panic disorder
 Incorrect Answer ImageC.Posttraumatic stress disorder
 Incorrect Answer ImageD.Premenstrual dysphoric disorder
 Incorrect Answer ImageE.Social anxiety disorder
A 34-year-old woman, gravida 2, para 1, at 26 weeks' gestation has painful contractions every 5
minutes and a cervical os dilated to 3 cm. Intravenous fluids, terbutaline, and magnesium sulfate
are administered. Eight hours later, the patient develops a headache and flushing. Her respiratory
rate is 8/min and deep tendon reflexes are diminished symmetrically. Which of the following is
the most appropriate next step in management?
 Incorrect Answer ImageA.Anticholinergic agent
 Incorrect Answer ImageB.Beta-blocker
 Correct Answer ImageC.Calcium gluconate
 Incorrect Answer ImageD.Methergine
 Incorrect Answer ImageE.Phosphate

Prior to its approval by the Food and Drug Administration, clinical trials were conducted to
examine the efficacy and side effects associated with a new psychiatric medication. Preliminary
work suggested that in some patients, the drug might lower hemoglobin levels to a dangerous
degree. To test for this side effect directly, two groups of 50 patients each were recruited and
randomly assigned to receive either the new psychiatric drug or a placebo over a 1-month period.
At the end of the month, hemoglobin levels were assessed for both groups and compared. Which
of the following describes the statistical test most likely used in this comparison?
 Incorrect Answer ImageA.Analysis of variance
 Incorrect Answer ImageB.Chi-square analysis
 Incorrect Answer ImageC.Number needed to treat
 Incorrect Answer ImageD.Odds ratio
 Correct Answer ImageE.Relative Risk

A previously healthy 15-year-old boy is brought to the physician because of a lump in his right
breast, which he found 1 week ago. There is no pain or discharge. He has had no other recent
problems and takes no medications or illicit drugs. Physical examination shows a firm,
nontender, 2-cm mass under the right nipple which is concentric with the areola. The other breast
is normal. Genital development is at Tanner stage 4. Which of the following is the most
appropriate next step in management?
 Incorrect Answer ImageA.Obtain a mammogram
 Correct Answer ImageB.Reassurance and observation
 Incorrect Answer ImageC.Refer to a plastic surgeon
 Incorrect Answer ImageD.Refer for genetic evaluation
 Incorrect Answer ImageE.Testosterone injection

A 3-year-old boy is brought to the emergency department after sustaining a closed head injury
from falling off his bicycle. He has been a healthy child with no prior history of trauma. His vital
signs are within normal limits. He has superficial scrapes to the right temporal scalp with
minimal bleeding. He opens and closes his eyes spontaneously, his best verbal response is
oriented and he obeys commands with respect to his motor response. The remainder of his
physical examination is unremarkable. Which of the following findings, if present, would signify
an increased risk of post-traumatic epilepsy?
 Correct Answer ImageA.Acute intracranial hemorrhage
 Incorrect Answer ImageB.Brief loss of consciousness
 Incorrect Answer ImageC.Linear skull fracture
 Incorrect Answer ImageD.Post-traumatic vomiting
 Incorrect Answer ImageE.Retrograde amnesia

A 21-year-old woman has a single, painful labial ulcer which began as a papule approximately
10 days ago. On examination, the ulcer is deep, purulent, and very tender. There is also tender
unilateral inguinal adenopathy. Which of the following is the most likely diagnosis?
 Correct Answer ImageA.Chancroid
 Incorrect Answer ImageB.Granuloma inguinale
 Incorrect Answer ImageC.Herpes simplex virus
 Incorrect Answer ImageD.Human papillomavirus
 Incorrect Answer ImageE.Lymphogranuloma venereum
 Incorrect Answer ImageF.Pelvic inflammatory disease
 Incorrect Answer ImageG.Primary syphilis
 Incorrect Answer ImageH.Secondary syphilis
 Incorrect Answer ImageI.Tertiary syphilis

A 28-year-old woman comes to the emergency department because of fever, headache, and neck
stiffness for 3 days. She reports that she previously had painless lesions on her genitalia
approximately 2 months ago, but they have since resolved. She is sexually active but does not
consistently use condoms. She is lethargic but arousable, with photophobia and decreased neck
range-of-motion. There are pink, round, flat lesions on her abdomen and thighs, which she states
do not itch. Which of the following is the most likely diagnosis?
 Incorrect Answer ImageA.Chancroid
 Incorrect Answer ImageB.Granuloma inguinale
 Incorrect Answer ImageC.Herpes simplex virus
 Incorrect Answer ImageD.Human papilloma virus
 Incorrect Answer ImageE.Lymphogranuloma venereum
 Incorrect Answer ImageF.Pelvic inflammatory disease
 Incorrect Answer ImageG.Primary syphilis
 Correct Answer ImageH.Secondary syphilis
 Incorrect Answer ImageI.Tertiary syphilis

A 38-year old homosexual man complains of 5 days rectal pain, constipation, and urgency, often
with small amounts of actual stool and occasional blood. He notes two episodes of incontinence.
Examination is unremarkable except for rectal tenderness. Stool is positive for occult blood.
Select the most appropriate diagnosis. 
 Incorrect Answer ImageA.Chancroid
 Incorrect Answer ImageB.Granuloma inguinale
 Correct Answer ImageC.Herpes simplex virus
 Incorrect Answer ImageD.Human papilloma virus
 Incorrect Answer ImageE.Lymphogranuloma venereum
 Incorrect Answer ImageF.Pelvic inflammatory disease
 Incorrect Answer ImageG.Primary syphilis
 Incorrect Answer ImageH.Secondary syphilis
 Incorrect Answer ImageI.Tertiary syphilis

A 58-year-old woman with stage III endometrial carcinoma has 2 days of anuria. Which of the
following is the most likely diagnosis?
 Correct Answer ImageA.Acute obstructive uropathy
 Incorrect Answer ImageB.Amyloidosis
 Incorrect Answer ImageC.Focal glomerular sclerosis
 Incorrect Answer ImageD.Hepatorenal syndrome
 Incorrect Answer ImageE.Interstitial nephritis
 Incorrect Answer ImageF.Kimmelstiel-Wilson glomerulosclerosis
 Incorrect Answer ImageG.Membranoproliferative glomerulonephritis
 Incorrect Answer ImageH.Membranous glomerulonephritis
 Incorrect Answer ImageI.Minimal change disease

A 68-year-old man with multiple myeloma has increasing fatigue and edema. Serum creatinine is
3.7 mg/dL and urinalysis shows 3+ proteinuria. Ultrasound shows enlargement of both kidneys.
Which of the following is the most likely diagnosis?
 Incorrect Answer ImageA.Acute obstructive uropathy
 Correct Answer ImageB.Amyloidosis
 Incorrect Answer ImageC.Focal segmental glomerulosclerosis
 Incorrect Answer ImageD.Hepatorenal syndrome
 Incorrect Answer ImageE.Hypertensive nephrosclerosis
 Incorrect Answer ImageF.Interstitial nephritis
 Incorrect Answer ImageG.Membranoproliferative glomerulonephritis
 Incorrect Answer ImageH.Membranous glomerulopathy
 Incorrect Answer ImageI.Minimal change disease

An 18-year-old woman slips and falls under a moving car, and her right leg becomes entrapped
in a wheel. Extrication is prolonged. She initially appears stable, however, she quickly
deteriorates en route to the hospital. On arrival at the emergency department, she is hypotensive
and tachycardic. She is given 2 L of Ringer's lactate over 20 minutes; this is rapidly followed by
several units of packed red cells. Laboratory studies are then sent which show a serum potassium
concentration of 6.1 mEq/L. Which of the following is most likely to account for her elevated
serum potassium concentration?
 Correct Answer ImageA.Crush injury of the extremity
 Incorrect Answer ImageB.Norepinephrine released from shock and trauma
 Incorrect Answer ImageC.Occult intra-abdominal hemorrhage
 Incorrect Answer ImageD.Potassium content in the Ringer's lactate
 Incorrect Answer ImageE.Renal tubular damage from rapid transfusion

A previously healthy 24-year-old male college student comes to the emergency department
because of fever and persistent nonproductive cough for 1 week. He reports having a headache
and sore throat prior to the onset of the fever and cough. Leukocyte count is 7,500/mm3 and a
chest x-ray shows diffuse bilateral interstitial infiltrates. Which of the following is the most
appropriate next step in management?
 Incorrect Answer ImageA.Amantadine
 Incorrect Answer ImageB.Ceftriaxone
 Correct Answer ImageC.Doxycycline
 Incorrect Answer ImageD.Penicillin G
 Incorrect Answer ImageE.Vancomycin

A 21-year-old nulligravid woman comes to the physician because of a whitish vaginal discharge
associated with burning and severe itching for 3 days. She has been taking oral contraceptives for
the past 4 years. Her temperature is 37.3℃ (99.1℉), pulse is 76/min, respirations are 23/min,
and blood pressure is 128/82 mm Hg. Pelvic examination shows a thick, white, odorless vaginal
discharge with vulvar erythema. Which of the following is the most likely diagnosis?
 Incorrect Answer ImageA.Bacterial vaginosis
 Correct Answer ImageB.Candida vaginitis
 Incorrect Answer ImageC.Chlamydia  cervicitis
 Incorrect Answer ImageD.Gonococcal cervicitis
 Incorrect Answer ImageE.Trichomonas vaginitis

A 43-year-old woman is brought to the emergency department because of fever and confusion.
Her husband states that she has had a fever to 101.5°F for one week, has become increasingly
confused, and had a spontaneous nosebleed. Her temperature is 38°C (100.5°F) and pulse is
110/min. She is disoriented to time and place, and diffuse petechiae are noted over her trunk.
Laboratory results show:
Hemoglobin 7 gm/dL
Platelets 20,000/mm3
Prothrombin time 12 seconds
Creatinine 3.5 mg/dL
Lactate dehydrogenase (LDH) 900 U/L
Peripheral smear shows schistocytes and polychromasia. Which of the following is the most
likely diagnosis?
 Incorrect Answer ImageA.Autoimmune hemolytic anemia
 Incorrect Answer ImageB.Bernard-Soulier syndrome
 Incorrect Answer ImageC.Disseminated intravascular coagulation
 Incorrect Answer ImageD.Immune thrombocytopenic purpura
 Correct Answer ImageE.Thrombotic thrombocytopenic purpura
A previously healthy 48-year-old woman comes to the physician because of a productive cough
for 4 weeks. She denies fever but notes a 10-pound weight loss. She has never smoked or used
illicit drugs. She drinks alcohol rarely. Physical examination shows no abnormalities. Chest x-
ray shows a 4-cm, spiculated mass in the right lower lung. CT scan shows hilar and mediastinal
lymphadenopathy; no abdominal or pelvic masses are seen. Biopsy of the lung lesion would
most likely show which of the following?
 Correct Answer ImageA.Adenocarcinoma
 Incorrect Answer ImageB.Large cell carcinoma
 Incorrect Answer ImageC.Sarcoidosis
 Incorrect Answer ImageD.Small cell carcinoma
 Incorrect Answer ImageE.Squamous cell carcinoma

A 23-year-old woman, gravida 1, para 0, at 6 weeks' gestation comes to the physician for an
initial prenatal evaluation. Her past medical history is significant for an allergic reaction to
penicillin, resulting in generalized urticaria and laryngeal edema. VDRL is positive on routine
screening. Fluorescent treponemal antibody (FTA) test is also positive. Which of the following is
the most appropriate pharmacotherapy?
 Incorrect Answer ImageA.Doxycycline
 Correct Answer ImageB.Penicillin desensitization followed by penicillin
 Incorrect Answer ImageC.Streptomycin
 Incorrect Answer ImageD.Tetracycline
 Incorrect Answer ImageE.Vancomycin

A 49-year-old woman comes to the physician for a routine health maintenance examination in
July. She has no significant medical or surgical history. She received her childhood
immunizations according to the CDC schedule but has received no immunizations as an adult.
Physical examination shows no abnormalities. Which of the following immunizations is
indicated at this time?
 Incorrect Answer ImageA.Influenza vaccine
 Incorrect Answer ImageB.Measles-mumps-rubella vaccine
 Incorrect Answer ImageC.Pneumococcal vaccine
 Correct Answer ImageD.Tetanus-diphtheria-acellular pertussis (Tdap) vaccine
 Incorrect Answer ImageE.Varicella vaccine

A 2-day-old girl suddenly becomes cyanotic. She was the product of an uncomplicated full-term
pregnancy and her mother had no significant prior medical history. A chest radiograph shows
increased pulmonary vascular markings. Which of the following is the most likely cause of the
cyanosis? 
 Incorrect Answer ImageA.Atrial septal defect
 Incorrect Answer ImageB.Patent ductus arteriosus
 Correct Answer ImageC.Transposition of the great vessels
 Incorrect Answer ImageD.Toxoplasmosis
 Incorrect Answer ImageE.Ventricular septal defect

The National Diabetes Association has proposed lowering the blood glucose level that is most
commonly used to diagnose diabetes. If followed, this recommendation is most likely to affect
the most commonly used screening test procedure by increasing which of the following?
 Incorrect Answer ImageA.Number of false negative test results
 Incorrect Answer ImageB.Number of true negative test results
 Incorrect Answer ImageC.Positive predictive value of the test
 Correct Answer ImageD.Resulting prevalence of diabetes
 Incorrect Answer ImageE.Specificity of the test

A 9-month-old girl is brought to the physician by her mother. The mother states that over the last
few weeks her daughter has been having episodes of fever, coughing, and sweating. Physical
examination and laboratory studies are unremarkable. When the physician asks the mother how
she takes the child’s temperature, the mother requests a thermometer to prove that the child has a
fever. A medical student then notices that the mother goes to the restroom with the thermometer
before returning it to the doctor. Which of the following is the most likely diagnosis for the
mother?
 Correct Answer ImageA.Factitious disorder imposed on another
 Incorrect Answer ImageB.Histrionic personality disorder
 Incorrect Answer ImageC.Illness anxiety disorder
 Incorrect Answer ImageD.Major depressive disorder
 Incorrect Answer ImageE.Somatic symptom disorder

A 9-month-old girl is brought to the physician by her mother. The mother states that over the last
few weeks her daughter has been having episodes of fever, coughing, and sweating. Physical
examination and laboratory studies are unremarkable. When the physician asks the mother how
she takes the child’s temperature, the mother requests a thermometer to prove that the child has a
fever. A medical student then notices that the mother goes to the restroom with the thermometer
before returning it to the doctor. Which of the following is the most likely diagnosis for the
mother?
 Correct Answer ImageA.Factitious disorder imposed on another
 Incorrect Answer ImageB.Histrionic personality disorder
 Incorrect Answer ImageC.Illness anxiety disorder
 Incorrect Answer ImageD.Major depressive disorder
 Incorrect Answer ImageE.Somatic symptom disorder

A 34-year-old man is transferred to an intensive care unit after receiving burns over most of his
body during a house fire. On the second day after admission, he begins to vomit blood.
Endoscopic examination demonstrates multiple punctate mucosal lesions, some of which are still
bleeding. Which of the following is the most likely diagnosis? 
 Correct Answer ImageA.Acute erosive gastritis
 Incorrect Answer ImageB.Chronic erosive gastritis
 Incorrect Answer ImageC.Dieulafoy’s lesion
 Incorrect Answer ImageD.Nonerosive gastritis
 Incorrect Answer ImageE.Pernicious anemia

A 25-year-old man comes to the physician because of pain and swelling in his neck for 5 days.
He also describes episodes of palpitations and sweating. He has no history of thyroid disease or
irradiation to the neck. His temperature is 38.0°C (100.4°F). Physical examination shows a
diffusely enlarged and tender thyroid gland. There is no erythema or warmth of the overlying
skin. Laboratory studies show an erythrocyte sedimentation rate of 60 mm/h. Thyroid function
tests show:
Thyroxine(T4)  Slightly increased
Free T4 Slightly increased
Triiodothyronine (T3)  Slightly increased
Thyroid-stimulating hormone Slightly decreased
A radioactive iodine uptake test shows decreased uptake. Which of the following is the most
likely diagnosis?
 Incorrect Answer ImageA.Acute pyogenic thyroiditis
 Incorrect Answer ImageB.Graves disease
 Incorrect Answer ImageC.Hashimoto thyroiditis
 Incorrect Answer ImageD.Hemorrhagic thyroid cyst
 Correct Answer ImageE.Subacute thyroiditis

A 19-year-old woman, gravida 1, para 0, at 40 weeks' gestation is admitted to the labor and
delivery floor with contractions every 5 minutes. There have been no prenatal complications.
The membranes are intact, and the cervix is 2 cm dilated and 50% effaced. Clusters of clear,
fluid-filled vesicles are seen on the labia majora. Which of the following would best reduce the
risk of neonatal infection?
 Incorrect Answer ImageA.Administration of intravenous acyclovir during labor and
vaginal delivery
 Incorrect Answer ImageB.Administration of intravenous ganciclovir during labor and
vaginal delivery
 Incorrect Answer ImageC.Administration of varicella-zoster immune globulin to the
neonate immediately after vaginal delivery
 Correct Answer ImageD.Delivery of the fetus by cesarean section
 Incorrect Answer ImageE.Forceps delivery

A 24-year-old man comes to the physician because of headaches and recurrent stomach
discomfort that have been present for three weeks and have been keeping him up at night. When
answering the doctor’s questions, the patient looks around the room and avoids eye contact with
the physician. During the physical examination, the patient at first recoils, but then seems more
relaxed as cervical lymph nodes are examined. Physical examination shows an elevated heart
rate and rapid breathing but is otherwise unremarkable. The physician next inquires about any
recent events or changes in the patient's life. In response, the patient gets angry and says, "You
just think it's all in my head. Maybe it's time I found another doctor who can really help me!" At
this point, which of the following is the physician's best reply?
 Incorrect Answer ImageA."Actually, I haven't decided what the issue is yet. I just wanted
to explore some possibilities."
 Incorrect Answer ImageB."I meant no disrespect by the question. Sometimes we get
stress reactions that manifest as physical symptoms."
 Incorrect Answer ImageC."I'm sorry that you feel that way. I'll provide you with the
name of another physician whom I hope you will like better."
 Incorrect Answer ImageD."I'm sorry that you think I'm not listening. Please tell me what
you think is the source of your symptoms."
 Correct Answer ImageE."I'm sorry to have upset you. Tell me a bit about what you think
is the reason you have been feeling the way you have lately."
 Incorrect Answer ImageF."Look, you seem a bit anxious today, and I'm just trying to get
a handle on why that might be the case."
 Incorrect Answer ImageG."No, I don't think this is just in your head, but I do need to
explore all possibilities."
 Incorrect Answer ImageH."Try to calm down and let's talk a bit about what's going on
here."

 59-year-old woman has nephrotic syndrome associated with idiopathic membranous


glomerulopathy. She loses 8 g of protein per day in her urine and her serum albumin is 2.1 g/dL.
Because of her condition, she is at increased risk for which of the following? 
 Incorrect Answer ImageA.Hypercalcemia
 Correct Answer ImageB.Hypercholesterolemia
 Incorrect Answer ImageC.Hyperthyroidism
 Incorrect Answer ImageD.Malignant hypertension
 Incorrect Answer ImageE.Oral and vaginal infections with Candida albicans

 47-year-old man comes to the emergency department because of shortness of breath for 1 week.
His temperature is 37.2°C (99°F), respirations are 26/min, and blood pressure is 148/60 mm Hg.
There is no jugular venous distension. His lungs are clear to auscultation bilaterally. Abdominal
examination shows a 14-cm liver, shifting dullness, and a palpable spleen tip. There are spider
angiomata on his chest, palmar erythema, and contractures of the third and fourth fingers
bilaterally. Mild lower extremity edema is present. An ECG shows sinus tachycardia. A
complete blood count shows:
Hematocrit 26%
Hemoglobin 8.5 g/dL
Mean corpuscular hemoglobin 36.7 pg/cell
Mean corpuscular hemoglobin concentration 33%
Mean corpuscular volume 111 µm3
Which of the following is the most likely cause of the patient's anemia? 
 Incorrect Answer ImageA.Hemolysis
 Incorrect Answer ImageB.Inability to absorb iron efficiently
 Incorrect Answer ImageC.Infiltration of the bone marrow by plasma cells
 Correct Answer ImageD.Nutritional deficiency
 Incorrect Answer ImageE.Splenic destruction

A 73-year-old woman has jaw pain while eating and pain over the scalp when brushing her hair.
She also notes recent weight loss and depression. She appears thin and has tenderness over both
temporal regions. Laboratory studies show a hemoglobin concentration of 9.7 g/dL and an
erythrocyte sedimentation rate of 150 mm/h. Which of the following is the most appropriate
pharmacotherapy?
 Incorrect Answer ImageA.Antidepressants
 Correct Answer ImageB.Corticosteroids
 Incorrect Answer ImageC.Disease-modifying antirheumatic drugs (DMARDs)
 Incorrect Answer ImageD.Narcotics
 Incorrect Answer ImageE.Nonsteroidal antiinflammatories (NSAIDs)

A 28-year-old man was involved in a motor vehicle accident. The emergency medical technician
reports that the patient was not wearing a seatbelt and the steering wheel was bent forward in the
car. His blood pressure is 80/60 mm Hg and his pulse is 120/min and weak. His neck veins are
markedly distended and his trachea is midline. Heart sounds are distant and the lungs are clear
bilaterally. The abdomen is soft and nontender. Which of the following is the most likely
diagnosis?
 Incorrect Answer ImageA.Aortic dissection
 Incorrect Answer ImageB.Myocardial contusion
 Correct Answer ImageC.Pericardial tamponade
 Incorrect Answer ImageD.Tension pneumothorax
 Incorrect Answer ImageE.Transected aorta

A recently published study compared the occurrence of acute asthma exacerbations for 100
children on a new medication compared with 100 children taking more standard asthma
prevention medications. The children from both groups were followed over a 3-month period and
any asthma exacerbations were noted. The results of the study are shown below. 
Asthma Exacerbation No Asthma Exacerbation Totals

New 20 80 100

Old 40 60 100

Totals 60 140 200


The report on the study included the statement that the results were significant at the level of p
<0.05 and that the researchers rejected the null hypothesis. Based on these results, if an asthmatic
child is placed on the new medication, which of the following is the chance that the child will
suffer an acute asthma exacerbation within the next three months? 
 Incorrect Answer ImageA.5%
 Incorrect Answer ImageB.10%
 Correct Answer ImageC.20%
 Incorrect Answer ImageD.33%
 Incorrect Answer ImageE.95%

A previously healthy 9-year-old boy is brought to the physician by his mother because of a rash
that she first noted four days ago. Two days earlier, he had a temperature of 37.9°C (100.2°F)
and began complaining of abdominal pain and pain in his knees and ankles. Physical
examination shows a non-blanching erythematous macular rash over his buttocks and lower
extremities with a few scattered palpable purpura. Passive flexion of his knees and ankles is
painful but there is no swelling, warmth, or erythema of the joints. He has diffuse abdominal
tenderness with palpation. Stool studies are positive for blood. Urinalysis shows 2+ protein and
urine microscopic examination shows numerous red blood cells. Which of the following is the
most likely diagnosis?
 Incorrect Answer ImageA.Angioneurotic edema
 Correct Answer ImageB.Henoch-Schönlein purpura
 Incorrect Answer ImageC.Kawasaki disease
 Incorrect Answer ImageD.Polyarteritis nodosa
 Incorrect Answer ImageE.Wegener granulomatosis

A 34-year-old woman who has had several years of intermittent joint and muscle aches presents
with complaints of dyspnea on exertion and a cough. On physical examination, she has an
erythematous rash bilaterally over her cheeks and urticarial lesions on her extremities. Lung
sounds are diminished on the right side. There is mild joint tenderness in her hands and wrists
but no edema or decreased range of motion. A chest x-ray film indicates a large right pleural
effusion. Her erythrocyte sedimentation rate is 65 mm/h and her platelet count is 31,000/mm3.
Which of the following is the most likely diagnosis?
 Incorrect Answer ImageA.Polymyositis
 Incorrect Answer ImageB.Rheumatoid arthritis
 Incorrect Answer ImageC.Sarcoidosis
 Incorrect Answer ImageD.Scleroderma
 Correct Answer ImageE.Systemic lupus erythematosus

A 20-year-old white man in a highly agitated state is brought to the emergency department by his
college roommate. He is extremely anxious, appears frightened, and his speech is dysarthric and
slurred. He is extremely labile emotionally, and has been thrashing about and inadvertently
causing lacerations on himself, requiring restraint. His blood pressure is 185/100 mm Hg and his
pulse is 115/min. On examination, he seems to have horizontal nystagmus in both eyes. Which of
the following is the most likely diagnosis? 
 Incorrect Answer ImageA.Alcohol withdrawal
 Incorrect Answer ImageB.Lithium intoxication
 Incorrect Answer ImageC.Neuroleptic malignant syndrome
 Incorrect Answer ImageD.Paranoid schizophrenia
 Correct Answer ImageE.Phencyclidine intoxication

A 23-year-old woman comes in for routine gynecologic care. She is currently not sexually active.
Pelvic examination is unremarkable. Pap smear shows dysplastic cells consistent with cervical
intraepithelial neoplasia (CIN) class I. Which of the following is the most appropriate next step? 
 Incorrect Answer ImageA.Cervical conization
 Correct Answer ImageB.Colposcopy
 Incorrect Answer ImageC.CT scan of the pelvis
 Incorrect Answer ImageD.Hysterectomy
 Incorrect Answer ImageE.Ultrasound of the pelvis

An 11-year-old girl has had a long history of bloody diarrhea, right lower quadrant pain, non-
destructive arthritis, and weight loss. She is brought to the hospital because her mother noticed a
foul smell to her urine. Urinalysis demonstrates fecal material within the urine. Which of the
following dermatologic lesions is most commonly associated with this disorder? 
 Incorrect Answer ImageA.Erythema chronicum migrans
 Incorrect Answer ImageB.Erythema marginatum
 Correct Answer ImageC.Erythema nodosum
 Incorrect Answer ImageD.Erysipelas
 Incorrect Answer ImageE.Tinea versicolor

A 13-year-old boy is brought to the physician because of left knee pain that has become
progressively worse for 2 weeks. There is no history of prior trauma. Physical examination
shows no point tenderness, erythema, or swelling over the left knee. The boy walks with his right
leg externally rotated, which is causing him to limp. Voluntary range of motion testing of the left
t hip shows obligate external rotation and decrease in internal rotation. Flexion of the left hip
produces external rotation. The right knee and hip are normal. Which of the following is the most
likely diagnosis?
 Incorrect Answer ImageA.Fracture of the acetabulum
 Incorrect Answer ImageB.Infectious arthritis of the hip
 Incorrect Answer ImageC.Legg-Calvé-Perthes disease
 Correct Answer ImageD.Slipped capital femoral epiphysis
 Incorrect Answer ImageE.Subluxation of the knee joint
Simulacros
A 43-year-old white woman comes to the emergency department because of a 1-day history of
increasingly severe pain localized to the right upper quadrant and radiating to the right lower
scapula. She has also been experiencing nausea and vomiting. The woman has had similar, but
milder, episodes of pain in the past, which had resolved spontaneously in a few days. Her
temperature is 38.3°C (101.0°F). Physical examination reveals involuntary guarding of
abdominal muscles on the right. The gallbladder is palpable. Which of the following is the most
appropriate next step in diagnosis? 
 Incorrect Answer ImageA.CT scan
 Incorrect Answer ImageB.Endoscopic retrograde cholangiography
 Incorrect Answer ImageC.Esophagogastroduodenoscopy
 Incorrect Answer ImageD.MRI scan
 Correct Answer ImageE.Ultrasound

A 22-year-old woman comes to the physician for an annual health maintenance examination. She
currently feels well. Her menstrual cycle is regular. She has no medical problems but does smoke
one pack of cigarettes per day. She is sexually active with multiple partners and uses condoms
inconsistently. Physical examination, including pelvic examination is unremarkable. A Pap
smear shows a high-grade squamous intraepithelial lesion (HSIL). Which of the following is the
most appropriate next step in management?
 Incorrect Answer ImageA.Perform a cone biopsy
 Incorrect Answer ImageB.Perform a hysterectomy
 Correct Answer ImageC.Perform colposcopy
 Incorrect Answer ImageD.Repeat pap smear in 1 year
 Incorrect Answer ImageE.Repeat pap smear in 6 months

A 43-year-old woman comes to the emergency department because of dizziness, tremor,


diaphoresis, and shortness of breath. She indicates to the physician that she has come into the
hospital with similar complaints twice in the past several weeks. On those occasions, as well as
currently, her physical examination, routine laboratory studies, electrocardiogram, and cardiac
enzymes have all been unremarkable. The patient states that these episodes are starting to
concern her greatly and she is worried about going out of her house alone now. Which of the
following is the most likely diagnosis? 
 Incorrect Answer ImageA.Factitious disorder
 Incorrect Answer ImageB.Generalized anxiety disorder
 Correct Answer ImageC.Panic disorder
 Incorrect Answer ImageD.Schizophrenia
 Incorrect Answer ImageE.Social anxiety disorder

A 28-year-old man is brought to the emergency department by his family because of a two-day
history of confusion. Over the past eight hours, he has been sitting in a chair with a stiff posture,
not speaking when questioned. He was released from the hospital two weeks prior after a third
admission for schizophrenia, catatonic type, and he has been taking haloperidol 10 mg QHS. On
physical examination, the patient seems confused and sleepy. His temperature is 39.6°C
(103.2°F), blood pressure is 145/100 mm Hg, pulse is 109, and respirations are 22/min.
Neurological examination shows rigidity in his extremities. Laboratory studies show creatine
phosphokinase to be five times the upper limit of normal. Which of the following is the most
likely diagnosis?
 Incorrect Answer ImageA.Catatonia
 Incorrect Answer ImageB.Neuroleptic-induced acute dystonia
 Incorrect Answer ImageC.Neuroleptic-induced parkinsonism
 Correct Answer ImageD.Neuroleptic malignant syndrome
 Incorrect Answer ImageE.Schizophrenia with catatonia

A 35-year-old man comes to the physician because of daily severe, periorbital, right-sided
headaches over the past 6 weeks. The pain often awakens him from sleep and is so excruciating
that he wants to bang his head against the wall. He reports nasal "stuffiness" and nausea. He has
had similar episodes on 2 previous occasions. Physical examination shows right-sided ptosis,
lacrimation, and reddening of the right eye. Laboratory studies show: 
Hemoglobin 16 g/dL
Hematocrit 49%
Leukocyte count 6,000/mm3
Erythrocyte sedimentation rate 5 mm/hr
Which of the following is the most likely diagnosis? 
 Incorrect Answer ImageA.Classic migraine
 Correct Answer ImageB.Cluster headache
 Incorrect Answer ImageC.Common migraine
 Incorrect Answer ImageD.Temporal arteritis
 Incorrect Answer ImageE.Tension headache

A 5-month-old undervaccinated boy is brought to the physician because of frequent watery


stools and vomiting present for the past 2 days. The child has been in daycare since age 3 months
and has had several episodes of fever and runny nose in the meantime, but this is the first time he
has had diarrhea. It started approximately 48 hours earlier with the passage of 6 to 7 loose,
watery stools with a foul smell. He had also vomited a couple of times and has had fever since
the beginning of the symptoms. The parents had called the daycare to see if other children were
affected, but learned that no one else had similar symptoms. They had not traveled anywhere
since the child was born. On physical examination, the child is in moderate distress, with a
temperature of 38.1°C (100.6°F), a pulse of 120/min, and respirations of 26/min. He has dry
mucosa and his anterior fontanelle is sunken. Skin turgor is also diminished. Palpation of the
abdomen does not reveal tenderness. On auscultation, increased peristalsis can be heard. The
stool is guaiac-positive for occult blood but shows no visible gross blood on examination. He is
given intravenous rehydration in the office and sent home on supportive treatment with lots of
fluids and antipyretics as needed. Two days later, the mother calls the office to say that all the
symptoms have completely resolved and the child is feeling well and behaving normally. Which
of the following was the most likely cause of this patient's illness? 
 Incorrect Answer ImageA.Clostridium difficile
 Incorrect Answer ImageB.Clostridium perfringens
 Incorrect Answer ImageC.Enterotoxigenic Escherichia coli
 Correct Answer ImageD.Rotavirus
 Incorrect Answer ImageE.Staphylococcus aureus

A 22-year-old woman, gravida 1, para 0, at 18 weeks' gestation comes to the physician for a
prenatal visit. Her estimated date of delivery is based on her last menstrual period. The
pregnancy has been complicated by severe nausea and vomiting in the first trimester that has
since resolved. She has no medical problems and has never had surgery. She takes a daily
prenatal vitamin and is allergic to penicillin. At this visit, she is informed that the maternal serum
alpha-fetoprotein (MSAFP) test that had been performed at her previous visit came back
elevated. She is referred for an ultrasound. Which of the following is the most likely cause of this
patient's elevated MSAFP?
 Incorrect Answer ImageA.Gastroschisis
 Correct Answer ImageB.Incorrect dating
 Incorrect Answer ImageC.Neural tube defect
 Incorrect Answer ImageD.Preterm labor
 Incorrect Answer ImageE.Twin pregnancy

A 44-year-old man has a body mass index (BMI) of 45 kg/m2, and he suffers from type 2
diabetes mellitus, hypertension, sleep apnea, severe arthritis of both knees, and depression. He
has attempted dietary control under medical supervision but has been unable to maintain any
significant weight loss. He also has tried hypnosis and counseling without success. Which of the
following options will provide him with the best option for long-term weight loss? 
 Incorrect Answer ImageA.Gastric banding
 Incorrect Answer ImageB.Jejunoileal bypass
 Correct Answer ImageC.Laparoscopic gastric pouch with Roux-en-Y bypass
 Incorrect Answer ImageD.Liquid diet
 Incorrect Answer ImageE.Thyroxine and appetite suppressant therapy

A 54-year-old woman is referred to a specialist because of extreme muscle weakness. She went
to her primary care physician, explaining that she was so weak that she could not walk from one
side of her bed to the other when trying to make up her bed in the mornings. The physician
verified that although she did not appear to be short of breath, she could not get up from a
squatting position without helping herself with both arms. When she walks into the specialist's
office, an instant presumptive diagnosis of Cushing is made because of her typical Cushingoid
appearance. A 24-hour urinary collection for free cortisol shows a sixfold elevation from normal.
Which of the following studies would be most helpful to determine if she has an adrenal
adenoma rather than a pituitary adenoma? 
 Incorrect Answer ImageA.Morning and evening serum cortisol levels
 Incorrect Answer ImageB.Overnight low-dose (1 mg) dexamethasone suppression test
 Incorrect Answer ImageC.Pituitary MRI
 Correct Answer ImageD.Serum corticotropin level
 Incorrect Answer ImageE.Serum potassium level

A 13-year-old boy is brought to the physician for a health maintenance examination. He has a
history of asthma since infancy and was hospitalized several times. During the last
hospitalization 2 years ago, he required mechanical ventilation following a respiratory arrest.
After that episode, he uses inhaled albuterol two to three times a day because of shortness of
breath or wheezing. He has not seen a physician in more than 8 months because he feels that the
asthma control is good. Upon further questioning, he states that he wakes up once or twice every
night due to coughing. Examination today shows a well-nourished adolescent in no distress.
Upper airway examination is normal. Chest examination shows pectus deformity and bilateral
rhonchi. There is no cardiac murmur and the abdominal examination is normal. The physician
explains to the parents that the control of symptoms is not adequate and that it seems that there
should be a modification in the drug treatment of the boy's asthma. In addition to a long-acting
beta2-agonist, which of the following is the most appropriate drug to treat this patient's asthma in
the long term?
 Incorrect Answer ImageA.Cromolyn inhaler
 Correct Answer ImageB.Fluticasone inhaler
 Incorrect Answer ImageC.Ipratropium nebulizer
 Incorrect Answer ImageD.Montelukast
 Incorrect Answer ImageE.Prednisone by oral route

A 69-year-old man with a history of coronary artery disease and congestive heart failure comes
to the physician to inquire about the possibility of receiving an implantable cardiac defibrillator
(ICD). He has heard that they can improve survival but wants a professional opinion before
proceeding with this invasive intervention. The physician recalls reading a recent meta-analysis
of randomized controlled trials of ICDs that studied their use in secondary prevention of sudden
cardiac death. The following table compares one-year, all-cause mortality in patients randomized
to ICD or medical therapy. 
Deaths/Total Subjects

Trial A 24/507 55/509

Trial B 13/99 64/189

Trial C 30/328 43/331

ICD Group Medical Therapy Group


Because the trials all used the same endpoint, were of similar quality and design, and studied the
same types of patients, the physician analyzes their data in aggregate. The patient should be told
that, according to the three combined trials, to prevent one death over the course of 1 year in
patients similar to him, which of the following is the number of ICDs that would need to be
placed? 
 Incorrect Answer ImageA.1
 Incorrect Answer ImageB.7.2
 Incorrect Answer ImageC.8.5
 Correct Answer ImageD.12
 Incorrect Answer ImageE.934

A 20-year-old woman is brought to the emergency department after having what is described as
a grand mal seizure after taking some pills. Her brother, who witnessed the seizure, states the
patient has made suicide attempts in the past. The brother is concerned that the patient may have
tried to overdose on one of their father's many medications. Apparently the patient told her
brother that she had swallowed the contents of one of the pill bottles. He doesn't know what his
sister took, but says that their father takes "heart pills", "some pill for depression," and a
"medication for his thyroid gland." The patient is extremely confused and disoriented and only
sobs when asked questions. Her temperature is 39.7°C (103.5°F), blood pressure is 98/48 mm
Hg, pulse is 118/min, and respirations are 24/min. Examination shows 8-mm pupils, equal and
mildly reactive to light, tachycardia, and hypoactive bowel sounds. Her skin is warm and
flushed, though there is no diaphoresis, and she has hyperactive reflexes. A Foley catheter is
placed, resulting in the immediate collection of 800 ml of clear yellow urine. An
electrocardiogram shows a prolonged QT interval and a slightly widened QRS complex. A stat
abdominal film shows an ileus gas pattern. Laboratory studies, including a toxicology screen, are
pending. This patient most likely is suffering from which of the following?
 Correct Answer ImageA.Anticholinergic poisoning
 Incorrect Answer ImageB.Digitalis overdose
 Incorrect Answer ImageC.Lithium toxicity
 Incorrect Answer ImageD.Serotonin syndrome
 Incorrect Answer ImageE.Thyroid storm

A 26-year-old man is involved in a serious motorcycle accident and is thrown more than 50 feet,
finally colliding with a nearby parked car. Emergency medical services bring him immediately to
the hospital, where he is found to be conscious but disoriented and in a great deal of pain.
Skeletal survey shows fractures of multiple ribs, the vertebral body at T11, his right femur, and
multiple pelvic fractures. He is stabilized and transferred to the intensive care unit (ICU), where
he develops hypoxemic respiratory failure and requires mechanical ventilation. On the second
day in the ICU his serum creatinine increases to 5.8 mg/dL from its initial level of 0.9 mg/dL.
Continuous hemodialysis is started and the patient remains stable. One day later the nurse notes
that the patient has begun to bleed from around intravenous lines and from traumatic wounds that
were sutured previously. Nasogastric lavage yields frank blood. Which of the following is the
most likely etiology of his bleeding? 
 Incorrect Answer ImageA.Hemolytic-uremic syndrome (HUS)
 Incorrect Answer ImageB.Over-anticoagulation with heparin
 Correct Answer ImageC.Tissue factor or cytokine activation of the coagulation pathway
 Incorrect Answer ImageD.Uremic platelet dysfunction
 Incorrect Answer ImageE.von Willebrand's disease

A 67-year-old woman comes to the physician for her semiannual examination. At her last visit,
she was diagnosed with hypertension after her blood pressure was found to be above 175/95 mm
Hg on three consecutive visits. She has been treated for hypertension for the past 8 months. She
is compliant with her beta-blocker therapy, but is concerned that her home blood pressure
monitor often shows her blood pressure to be "high". Her home blood pressure log reveals that
her daily blood pressures average 150/85 mm Hg. She was originally started on a low dose of
beta-blocker. Which of the following is the most appropriate next step in management? 
 Incorrect Answer ImageA.Add an agent from another class to her therapy
 Incorrect Answer ImageB.Discontinue the beta-blocker and start therapy with another
agent
 Incorrect Answer ImageC.Do nothing; her blood pressure is very well controlled
 Correct Answer ImageD.Increase the dose of her beta-blocker
 Incorrect Answer ImageE.Schedule a blood test and renal angiogram to rule out causes of
secondary hypertension

An otherwise healthy 31-year-old construction worker has an irregularly shaped, 8-mm, black-
and-blue eroded nodule on his left forearm. He initially attributed the lesion to an injury he
sustained while working on a scaffold several months ago and did not seek treatment. However,
the lesion has repeatedly scabbed over and grown larger. Although painless, the patient is
concerned that the area is infected and requests antibiotics. Which of the following is the most
appropriate next step in management?
 Incorrect Answer ImageA.Cryotherapy with liquid nitrogen to the affected area
 Incorrect Answer ImageB.Erythromycin 500 mg by mouth 3 times a day
 Correct Answer ImageC.Excisional biopsy of the affected area for pathology
 Incorrect Answer ImageD.Hydrocortisone 1% cream applied to the area twice a day
 Incorrect Answer ImageE.Observe and have the patient return for follow-up in 2 months

A 24-year-old man comes to the physician because of shortness of breath and episodes of near
syncope. He reports that when he exercises, he often feels “faint” and has trouble catching his
breath. He denies orthopnea or any significant dyspnea with usual activity. He has no past
medical history, takes no medications regularly, and has no relevant family history, although he
does report his maternal uncle died suddenly while playing basketball. Physical examination
shows a harsh systolic ejection murmur heard best at the left lower sternal border that increases
with Valsalva. A forceful apical impulse is palpated in the fifth intercostal space. Which of the
following is the most likely diagnosis? 
 Incorrect Answer ImageA.Benign flow murmur
 Incorrect Answer ImageB.Dilated cardiomyopathy
 Correct Answer ImageC.Hypertrophic cardiomyopathy
 Incorrect Answer ImageD.Restrictive cardiomyopathy
 Incorrect Answer ImageE.Ventricular septal defect

A 27-year-old woman comes to the physician for an annual gynecologic examination and a refill
of the combination oral contraceptive she has been using for the past 10 years. The patient has no
chronic medical problems and has had no previous surgeries. She has smoked a half a pack of
cigarettes daily for the past 10 years, has a glass of wine with dinner once a week, and does not
use illicit drugs. She has had 9 lifetime sexual partners, all men, and is sexually active with one
male partner currently. Vital signs are within normal limits. A speculum examination and
bimanual pelvic examination are normal. A Papanicolaou test shows a high-grade squamous
intraepithelial lesion. Which of the following is the most appropriate next step in management?
 Correct Answer ImageA.Colposcopy of the cervix
 Incorrect Answer ImageB.Loop electrosurgical excision procedure
 Incorrect Answer ImageC.Human papillomavirus testing
 Incorrect Answer ImageD.Human papillomavirus vaccination
 Incorrect Answer ImageE.Repeat Papanicolaou test in 1 year

A 14-year-old girl is brought to the emergency department because of right lower quadrant pain,
low-grade fever, and anorexia for 10 hours. She has diffuse abdominal pain to deep palpation.
She is taken to the operating room and during the procedure, the patient's appendix is found to be
normal in appearance. Upon further evaluation, a section of the ileum is found to be edematous,
red, and mildly thickened, and several lymph nodes are enlarged. The rest of the abdomen is
unremarkable. Which of the following is the most appropriate next step in management? 
 Correct Answer ImageA.Appendectomy
 Incorrect Answer ImageB.Closure of the abdomen
 Incorrect Answer ImageC.Diverting ileostomy
 Incorrect Answer ImageD.Removal of abnormal ileum
 Incorrect Answer ImageE.Reduction of a telescoped loop of bowel
 Incorrect Answer ImageF.Transverse colectomy

A 24-year-old woman, gravida 1, para 0, at 8 weeks’ gestation is brought to the emergency


department because of the sudden onset of lower abdominal pain and syncope. After a 4-year
history of infertility, pregnancy was achieved following clomiphene-induced ovulation. The
patient has had only a single prenatal visit, 1 week ago. Prenatal laboratory studies obtained at
that visit show:
Hemoglobin 12.0 g/dL
Blood type O
Rh negative
Lewis antibody positive
Rubella antibody negative
Rapid plasma reagin negative
Hepatitis B antigen negative
Pap smear low-grade squamous
intraepithelial lesion
Vital signs are within normal limits, although orthostatic changes are noted with positional
adjustment. The mucous membranes are pale. The abdomen is slightly protuberant, bowel
sounds are decreased, and there is tenderness to palpation throughout the lower abdomen. The
pelvic examination is difficult to perform due to voluntary guarding. A culdocentesis retrieves 20
mL of unclotted blood. A diagnosis of hemoperitoneum presumed secondary to a ruptured
ectopic pregnancy is made. Intravenous fluid resuscitation is begun, the patient's condition
stabilizes, and she is taken to surgery where left salpingectomy is performed. Which of the
following is the most appropriate next step in management?
 Correct Answer ImageA.Administration of anti-D immune globulin
 Incorrect Answer ImageB.Cryosurgery of the uterine cervix
 Incorrect Answer ImageC.Hepatitis B vaccination
 Incorrect Answer ImageD.In vitro fertilization and embryo transfer
 Incorrect Answer ImageE.Repeat anti-Lewis titer

A 36-year-old woman comes into the hospital in labor at 34 weeks' gestation. She has had an
unremarkable prenatal course but awoke on the morning of admission to find her membranes
ruptured. Her labor lasts 24 hours and results in the delivery of a 4-lb, 8-oz baby girl with
APGAR scores of 8 and 10, at 1 and 5 minutes respectively. The baby appears to be doing well
for the first 3 hours of life but then becomes lethargic and hypotonic. Her leukocyte count is
29,000/mm3, hematocrit is 51%, and platelets are 251,000/mm3. Chest x-ray is clear. Lumbar
puncture reveals a glucose of 30 mg/dL, and 40 WBC with 90% polymorphonuclear cells. Which
of the following is the most likely cause of the newborn's condition? 
 Incorrect Answer ImageA.Escherichia coli
 Incorrect Answer ImageB.Group A hemolytic streptococcus
 Correct Answer ImageC.Group B streptococcus
 Incorrect Answer ImageD.Haemophilus influenzae
 Incorrect Answer ImageE.Herpes simplex virus
 Incorrect Answer ImageF.Toxoplasmosis

A 36-year-old woman comes into the hospital in labor at 34 weeks' gestation. She has had an
unremarkable prenatal course but awoke on the morning of admission to find her membranes
ruptured. Her labor lasts 24 hours and results in the delivery of a 4-lb, 8-oz baby girl with
APGAR scores of 8 and 10, at 1 and 5 minutes respectively. The baby appears to be doing well
for the first 3 hours of life but then becomes lethargic and hypotonic. Her leukocyte count is
29,000/mm3, hematocrit is 51%, and platelets are 251,000/mm3. Chest x-ray is clear. Lumbar
puncture reveals a glucose of 30 mg/dL, and 40 WBC with 90% polymorphonuclear cells. The
infection affecting this baby could have been most effectively prevented by which of the
following interventions?
 Incorrect Answer ImageA.Eliciting a history of genital herpes from the mother
 Correct Answer ImageB.Intrapartum ampicillin
 Incorrect Answer ImageC.Prenatal treatment of the mother with penicillin
 Incorrect Answer ImageD.Testing the mother for toxoplasmosis and treating her as
indicated
 Incorrect Answer ImageE.Vaccination of the mother against H. influenzae

A 34-year-old woman, gravida 3, para 1, at 26 weeks' gestation comes to the physician for a
routine prenatal visit. She has no complaints at this time. She has gained 2.2 kg (5 lb) since her
last prenatal visit 4 weeks ago. Her blood pressure is 100/60 mm Hg. On physical examination
her fundus measures 30 cm and the fetal heart rate is 140/min. Urine dip shows trace protein, 1+
glucose, and trace leukocytes. She schedules an ultrasound for the following week. Which of the
following is the most appropriate next step in management? 
 Correct Answer ImageA.Administer a 50-g glucose load and measure serum glucose in
1-hour
 Incorrect Answer ImageB.Advise her to have a triple screen
 Incorrect Answer ImageC.Perform a 3-hour glucose tolerance test
 Incorrect Answer ImageD.Refer her for a non-stress test
 Incorrect Answer ImageE.Send urinalysis and urine culture

A 51-year-old HIV-positive man has multiple dark red plaques on his shins, abdomen, and hard
palate. The lesions began as flat marks that looked like bruises but have persisted and are now
elevated. The lesions are asymptomatic and have been present for about 3–4 months. Current
antiretroviral treatment regimen is efavirenz, tenofovir and emtricitabine. On physical
examination, he is afebrile and has normal vital signs. The remainder of the examination is
normal, except for the skin and mucous membranes. On his hard palate is a 2 cm x 1 cm dusky
red plaque. On his shins and anterior abdomen are about 15 discrete 1 to 3 cm dark red infiltrated
plaques without ulceration or surrounding skin changes. The histopathologic evaluation
demonstrates a vascular sarcomatous pattern. Which of the following agents is most likely
associated with these lesions?
 Incorrect Answer ImageA.Cytomegalovirus (CMV)
 Incorrect Answer ImageB.Epstein-Barr virus (EBV)
 Incorrect Answer ImageC.Human herpesvirus-6 (HHV-6)
 Incorrect Answer ImageD.Human herpesvirus-7 (HHV-7)
 Correct Answer ImageE.Human herpesvirus-8 (HHV-8)

A 34-year-old man comes to the physician because of fatigue. He reports lifelong difficulty in
concentrating and feeling down but denies ever being suicidal. He has previously been evaluated
by a psychiatrist who prescribed fluoxetine for possible depression, but it did not seem to help.
On more careful questioning, it is clear that the patient has excessive daytime sleepiness. He has
difficulty staying awake at his job as a production coordinator of an electronics firm and notes
that he used to fall asleep during classes in school and college. He has driven off the road twice
in the last year and now does not drive for fear of a similar incident. He thinks he obtains enough
sleep (six to seven hours on the weekdays, eight hours on the weekends), and is refreshed on
awakening. He admits to vivid dreaming. These dreams are often at the onset of sleep, during
which he notes a peculiar detached and "levitation-like" sensation. He regularly naps for 15
minutes in the midafternoon and feels alert for the next two to three hours. He notes almost daily
episodes of transient (5-10 second) sensations of paralysis on awakening from nocturnal sleep or
naps. His body goes peculiarly weak at the knees when he has a good laugh and he has to support
himself. During these episodes, he also notes a numb and "odd" sensation in the jaw area when
he tries to speak. He does not smoke regularly, but admits to experimenting with various drugs
during his college days. Physical examination and laboratory studies are unremarkable and body
mass index is 24 kg/m2. Which of the following is the most likely diagnosis?
 Incorrect Answer ImageA.Insomnia
 Incorrect Answer ImageB.Major depressive disorder
 Correct Answer ImageC.Narcolepsy
 Incorrect Answer ImageD.Obstructive sleep apnea
 Incorrect Answer ImageE.Stimulant use disorder

A 19-year-old woman, who was the restrained front-seat passenger in an automobile accident, is
being evaluated in the emergency department. It is reported that the driver did not survive the
accident, which involved a head-on collision between two vehicles. The patient received 2 L of
crystalloid during transportation and is receiving 100% oxygen via facemask. She has extensive
facial trauma and suspected severe internal injuries secondary to the massive impact. She lost
consciousness after the collision but regained consciousness shortly after the paramedic team
arrived. Which of the following is the first priority in the stabilization of this patient? 
 Incorrect Answer ImageA.Continue aggressive fluid replacement
 Incorrect Answer ImageB.Do an extensive neurologic examination
 Correct Answer ImageC.Establish that the patient has an airway
 Incorrect Answer ImageD.Gain intravenous access in both upper extremities
 Incorrect Answer ImageE.Place a Foley catheter

A 74-year old woman with a history of coronary artery disease, hypertension and hyperlipidemia
is brought to the emergency department because of severe left-sided chest pain, which is
unrelieved by nitroglycerin. The patient reports that the pain is very similar to when she
experienced a heart attack 5 years ago. In the emergency department, her temperature is 37.3°C
(99°F), pulse is 106/min, respirations are 20/min, and blood pressure 95/60 mm Hg. In general,
the patient appears to be in mild distress secondary to pain. Physical examination is significant
for crackles at the lung bases bilaterally. Heart rate is regular and heart sounds are distant with no
murmurs or rubs. Examination of extremities shows mild ankle edema, no cyanosis or clubbing.
An electrocardiogram (ECG) shows 2-mm ST-segment elevation in leads V2–V5. The patient is
given supplemental oxygen and morphine for pain however, several minutes later the patient
begins to complain of worsening shortness of breath. Within minutes, the patient becomes
unresponsive and telemetry shows the rhythm below (Figure 1). 30 seconds later the patient still
remains unresponsive. Telemetry is consistent with asystole, and is shown below (Figure 2).
Which of the following is the most appropriate next step in the management of this patient?
 Incorrect Answer ImageA.Amiodarone
 Incorrect Answer ImageB.Defibrillation, 200 J
 Incorrect Answer ImageC.Defibrillation, 360 J
 Correct Answer ImageD.Epinephrine
 Incorrect Answer ImageE.Lidocaine
 Incorrect Answer ImageF.Procainamide

A 39-year-old florist has numerous painless lesions on her right arm that have been present for
the past 3 weeks. She seems to recall having an initial lesion on her right index finger which
spontaneously healed about a month ago. She has otherwise been in her normal state of health
and denies any cough, fever, or night sweats. On physical examination, there are 10 discrete,
erythematous, 1 cm nodules in a linear pattern on her right dorsal arm from her wrist to her upper
arm. She is treated with a supersaturated solution of potassium iodide, and the lesions clear.
Which of the following is the most likely diagnosis?
 Incorrect Answer ImageA.Aspergillosis
 Incorrect Answer ImageB.Blastomycosis
 Incorrect Answer ImageC.Coccidioidomycosis
 Incorrect Answer ImageD.Histoplasmosis
 Correct Answer ImageE.Sporotrichosis

Paramedics bring a 33-year-old man to the hospital after being found with an altered mental
status at a nearby fast-food establishment. The paramedics report that the patient was yelling at
customers and screaming that his food had been poisoned. They found the patient cowering in
the corner of the bathroom, picking imaginary insects from his skin. The patient is jittery and
wishes to leave but eventually agrees to a medical evaluation. He reports being well and healthy
all of his life and takes no current medications. He denies tobacco use, drinks socially, and
denies use of illicit drugs. His temperature is 39.7°C (103.5°F), blood pressure is 180/105 mm
Hg, pulse is 110/min, respirations are 20/min, and oxygen saturation is 99% on room air. The
patient is disheveled and anxious and smells of tobacco and alcohol. He is alert and oriented and
has a normal neurologic examination. His skin is warm and flushed with needle track marks in
the left antecubital fossa. Pupils are 8 mm, equal, sluggishly reactive to light, with a normal
funduscopic examination. Routine laboratory studies including a complete blood count, serum
chemistries, and urinalysis are normal. A urine toxicology screen is pending. Which of the
following is the most appropriate treatment for this patient's likely condition?
 Incorrect Answer ImageA.Flumazenil
 Correct Answer ImageB.Lorazepam
 Incorrect Answer ImageC.Metoprolol
 Incorrect Answer ImageD.Naloxone
 Incorrect Answer ImageE.Nitroprusside

A 35-year-old fair-skinned man is found on physical examination to have a 1.8 cm pigmented


lesion of the skin overlying the tip of his right scapula. The lesion has irregular borders, several
hues of bluish and black pigmentation, and an asymmetrical shape. There is no ulceration. The
rest of the physical examination is normal. Full-thickness biopsy reveals melanoma with a depth
of 0.65 mm. Which of the following is the most appropriate management?
 Correct Answer ImageA.Local excision with a 1 cm surgical margin
 Incorrect Answer ImageB.Local excision with a 1 cm surgical margin and sentinel lymph
node biopsy
 Incorrect Answer ImageC.Local excision with a 2 cm surgical margin
 Incorrect Answer ImageD.Local excision with a 2 cm surgical margin and sentinel lymph
node biopsy
 Incorrect Answer ImageE.Local excision with a 4 cm surgical margin

A 42-year-old woman comes to the physician because of pain in her side that began suddenly
one day ago. She began to feel nauseated and had two episodes of emesis shortly after the pain
began. She describes the pain as wave-like, located in the right flank area and radiating to the
groin. She denies ever having pain like this before. Her past medical history is remarkable only
for three normal pregnancies and her only medication is calcium supplements. Her temperature is
37.0°C (98.6°F), blood pressure is 145/80 mm Hg, and pulse is 108/min. She appears to be in
mild distress. Head, neck, lung, and cardiac examinations are normal. There is marked
tenderness on percussion of the right costovertebral angle. Urinalysis shows 2+ blood but is
otherwise negative. Laboratory studies show:
Sodium 141 mEq/dL (Normal 136-145 mEq/dL)
Potassium 4.4 mEq/dL (Normal 3.5-5.0 mEq/dL)
Chloride 108 mEq/dL (Normal 90-110 mEq/dL)
Bicarbonate 23 mEq/dL (Normal 22-26 mEq/dL)
Blood urea nitrogen 14 mg/dL (Normal 5-20 mg/dL)
Creatinine 0.8 mg/dL (Normal 0.6-1.2 mg/dL)
Calcium 11.9 mg/dL (Normal 8.5-10.5 mg/dL)
Albumin 3.8 g/dL (Normal 3.2-4.5 g/dL)
Further questioning and examination would most likely show which of the following? 
 Incorrect Answer ImageA.Chronic diarrhea
 Incorrect Answer ImageB.Exertional dyspnea
 Incorrect Answer ImageC.Swollen, painful joints in the fingers and toes
 Incorrect Answer ImageD.Tetany in arm muscles when blood pressure measurement is
attempted
 Correct Answer ImageE.Vague periumbilical abdominal pain

An 80-year-old nursing home resident is brought to the physician because of a new-onset skin
eruption that started several weeks earlier. She reports severe, constant pruritus. She was treated
multiple times for scabies without relief. The nursing home staff reports that the rash changes on
a daily basis producing different patterns every time they examine her. No other nursing home
patients or family members of the patient are affected. Her past medical history is significant for
diabetes mellitus, hypertension, thyroidectomy for thyrotoxicosis, and pulmonary emboli
associated with deep venous thrombosis. Current medications include nifedipine, metformin,
warfarin, levothyroxine, and a daily multivitamin. Vital signs are normal. On physical
examination, the patient does not appear to be in distress. Her skin has a striking appearance with
undulating wavy bands of slightly elevated erythema over the entire body. There is a trailing
scale on all the lesions, which are concentric in appearance, giving the impression of a wood
grain pattern. This skin eruption is typically associated with a paraneoplastic phenomenon
heralding malignancy in which of the following locations?
 Incorrect Answer ImageA.Endocrine pancreas
 Incorrect Answer ImageB.Larynx
 Correct Answer ImageC.Lung
 Incorrect Answer ImageD.Retroperitoneal fat
 Incorrect Answer ImageE.Thyroid

A 54-year-old woman crashes her car against a telephone pole at high speed. On arrival at the
emergency department, she is in moderate respiratory distress. She has multiple bruises over the
chest and multiple sites of point tenderness over the ribs. X-rays show multiple rib fractures on
both sides, but both lungs are expanded and seem normal. There is no widening of the
mediastinum. On closer observation, it is noted that a segment of the chest wall on the left side
caves in when she inhales and bulges out when she exhales. Which of the following is the most
appropriate management for her condition?
 Incorrect Answer ImageA.External fixation of rib fractures
 Incorrect Answer ImageB.Internal fixation of rib fractures
 Incorrect Answer ImageC.Surgical repair of transected aorta
 Incorrect Answer ImageD.Tight bandaging of the chest wall
 Correct Answer ImageE.Treatment of pulmonary contusion

A 23-year-old woman comes to the physician because of chronic pelvic pain due to
endometriosis that was diagnosed with laparoscopy 3 years ago. She has tried oral contraceptives
(OCPs), medroxyprogesterone acetate, and danazol, all of which have failed to provide her with
much relief. She has no other medical problems. Other than the laparoscopy, she has had no
surgeries. She takes no medications currently. She is allergic to penicillin. Vital signs are normal.
Physical examination shows no uterine or adnexal masses but moderate tenderness in the pelvis.
The patient agrees to start taking leuprolide. Which of the following side effects is this patient
most likely to experience?
 Incorrect Answer ImageA.Hip fracture
 Correct Answer ImageB.Hot flashes
 Incorrect Answer ImageC.Increased skin elasticity
 Incorrect Answer ImageD.Multiple pregnancy
 Incorrect Answer ImageE.Vaginal discharge

A 33-year-old woman, gravida 3, para 2, at 34 weeks' gestation comes to the physician for a
prenatal visit. She has had no bleeding, loss of fluid, or contractions. She states that the baby is
moving very little. Her pregnancy is complicated by her having hepatitis B and hepatitis C. She
is also Rh negative. Her past obstetric history is significant for a classic cesarean delivery 5 years
ago, followed by a low transverse cesarean delivery 3 years ago. Because of the decreased fetal
movement, a nonstress test is performed and appears to be nonreactive. Further testing is planned
with either a biophysical profile (BPP) or a contraction stress test (CST). Which of the following
elements of this patient's history represents a contraindication to a contraction stress test (CST)?
 Incorrect Answer ImageA.Hepatitis B
 Incorrect Answer ImageB.Hepatitis C
 Correct Answer ImageC.History of classic cesarean delivery
 Incorrect Answer ImageD.History of low transverse cesarean delivery
 Incorrect Answer ImageE.Rh negative

A 37-year-old woman comes to the office because of frequent genital herpes outbreaks. A review
of her record shows that she was first diagnosed with herpes simplex virus-2 (HSV-2) infection 3
years ago. She has had 8 to 10 outbreaks per year over the last few years and these outbreaks are
causing her significant physical and emotional distress. She has no other medical problems. She
takes acyclovir during the episodes, but no other medications. She has no drug allergies. Physical
examination demonstrates some scarring on the labia majora, but no active lesions. The patient is
counseled regarding taking acyclovir on a daily basis as suppressive therapy. Counseling to this
patient should include which of the following? 
 Incorrect Answer ImageA.Daily acyclovir can be taken for no longer than one year
 Incorrect Answer ImageB.Daily acyclovir can be taken for no longer than six months
 Incorrect Answer ImageC.Suppressive therapy eliminates subclinical viral shedding
 Incorrect Answer ImageD.Suppressive therapy reduces recurrences by about 33%
 Correct Answer ImageE.Suppressive therapy reduces recurrences by about 75%

A 4-week-old infant is brought to the physician by her 19-year-old mother because of coughing
and rapid breathing for 6 days. The infant had been healthy and feeding well prior to the onset of
symptoms, although the mother noticed the infant’s eyes were red for several days before she
began to cough. Her temperature is 37°C (98.6°F). She has a short, staccato cough. She has
bilateral mucopurulent conjunctivitis, moderate nasal congestion, mild tachypnea, and rales
diffusely throughout both lung fields. Which of the following organisms is the most likely cause
of her illness? 
 Incorrect Answer ImageA.Adenovirus
 Incorrect Answer ImageB.Bordetella pertussis
 Correct Answer ImageC. Chlamydia trachomatis
 Incorrect Answer ImageD. Pneumocystis carinii
 Incorrect Answer ImageE.Respiratory syncytial virus

A 16-year-old boy is brought to the clinic by his father who says that the boy has been
increasingly aggressive and has been stumbling and tripping around the house for several weeks.
The father is especially worried about the teen’s recent and uncharacteristically violent behavior.
The patient's temperature is 38.0°C (100.4°F), blood pressure is 140/90 mm Hg, pulse is 90/min,
and respirations are 22/min. He has slightly dilated pupils and nystagmus. As the doctor is
examining him, the teen has a seizure, making the remainder of the physical examination
impossible. Which of the following is the most likely cause of these findings?
 Incorrect Answer ImageA.Cocaine withdrawal
 Incorrect Answer ImageB.Heroin
 Incorrect Answer ImageC.Marijuana
 Incorrect Answer ImageD.Morphine
 Correct Answer ImageE.Phencyclidine (PCP)

A 35-year-old man comes to the emergency department because of shortness of breath. His
symptoms have worsened over the past 3 days and are associated with a cough productive of
yellow sputum. His past medical history is significant for asthma. His only medication is an
albuterol inhaler. He has a 20-pack-year history of tobacco use and currently smokes two packs
per day. On physical examination, his temperature is 37.8°C (100°F), blood pressure is 160/87
mm Hg, pulse is 69/min, and respirations are 24/min. His lung examination is significant for
diminished breath sounds with diffuse wheezing. His chest radiograph is significant for
hyperexpansion with no evidence of consolidation. Pulse oximetry shows 90% oxygen saturation
on room air. Which of the following is the most appropriate next step in therapy?
 Correct Answer ImageA.Albuterol nebulizer
 Incorrect Answer ImageB.Helium-oxygen mixture
 Incorrect Answer ImageC.Ipratropium nebulizer
 Incorrect Answer ImageD.IV beta adrenergic blocker
 Incorrect Answer ImageE.Racemic epinephrine
A 43-year-old man is brought to the emergency room because of excruciating abdominal
pain that began at 8:23 PM (he looked at his watch when the pain "hit him"). On physical
examination approximately 30 minutes after the onset of symptoms, he has a rigid abdomen, lies
motionless on the examination table, has no bowel sounds, and is obviously in great pain, which
he describes as constant and encompassing his entire abdomen. There is tenderness to palpation
in all four quadrants of the abdomen, involuntary guarding, and rebound tenderness. Abdominal
x-ray shows free air under the diaphragm. Which of the following is the most likely cause of this
patient’s pathology?
 Incorrect Answer ImageA.Acute abdomen, the nature of which cannot yet be defined
 Incorrect Answer ImageB.Acute inflammatory process affecting an intra-abdominal
viscera
 Incorrect Answer ImageC.Acute obstruction of an intra-abdominal viscera
 Incorrect Answer ImageD.Ischemic process affecting intra-abdominal organs
 Correct Answer ImageE.Perforation of the gastrointestinal tract

A 52-year-old, obese African American woman is admitted to the hospital for an elective gastric
bypass operation for weight reduction. On post-op day 1, the patient acutely develops tachypnea,
tachycardia, and an oxygen requirement of 4 L. She complains of left-sided pleuritic chest pain.
Which of the following is the most appropriate diagnostic study? 
 Incorrect Answer ImageA.Chest x-ray
 Incorrect Answer ImageB.CT scan
 Incorrect Answer ImageC.ECG with rhythm strip
 Incorrect Answer ImageD.MRI
 Correct Answer ImageE.Ventilation-perfusion scan

A 25-year-old woman, gravida 1, para 0, comes to the physician for her first prenatal visit. Her
last menstrual period was 7 weeks ago. She has had some nausea and vomiting but otherwise
feels well. Past medical and surgical history are unremarkable. Her family history is significant
for cystic fibrosis with an affected aunt. Her husband has an affected cousin. Physical
examination shows no abnormalities. Given her family history, she is concerned about the risks
of having a child with cystic fibrosis, and she inquires about cystic fibrosis screening. Which of
the following is the most appropriate response?
 Correct Answer ImageA.Screening is available and appropriate
 Incorrect Answer ImageB.Screening is inappropriate in her case
 Incorrect Answer ImageC.Screening is mandatory
 Incorrect Answer ImageD.Screening is not available in utero
 Incorrect Answer ImageE.Screening is unnecessary: she has a 1 in 4 chance of having an
affected child

A 14-year-old boy is brought in by his mother after the police caught him stealing a neighbor's
bicycle. In addition, the boy is doing poorly in school, and he is suspected of being involved in a
recent arson in the adjacent neighborhood. Neighbors report that they have often seen him
lighting fires and burning trash in vacant lots, and they suspect him in two other burglaries that
occurred in the past year. The boy has a history of fighting and truancy at school since the age of
10. Which of the following is the most likely diagnosis? 
 Incorrect Answer ImageA.Antisocial personality disorder
 Incorrect Answer ImageB.Attention deficit/hyperactivity disorder
 Correct Answer ImageC.Conduct disorder
 Incorrect Answer ImageD.Oppositional defiant disorder
 Incorrect Answer ImageE.Pyromania

A 24-year-old woman is admitted to the hospital for a broken femur. The patient was in a motor
vehicle accident 20 hours ago and was brought to the hospital by EMS. On the scene, she was
found belted in her car in the driver's seat, and her only documented injury was the leg fracture.
She had no loss of consciousness or altered mental status. On arrival at the hospital, radiographs
confirm a closed fracture of her femur. She is stabilized overnight and scheduled for surgery the
next day. For which of the following major complications is this patient most at risk? 
 Incorrect Answer ImageA.Air embolism
 Incorrect Answer ImageB.Cerebrovascular hemorrhage
 Correct Answer ImageC.Fat embolism
 Incorrect Answer ImageD.Osteomyelitis
 Incorrect Answer ImageE.Permanent disability

A 49-year-old waitress presents with painful swelling of the left knee. The symptoms began over
the past 48 hours and have limited her ability to work. She has developed a fever over the past 24
hours but denies rigors, cough, rash, or headaches. On physical examination, her temperature is
38.3°C (100.9°F), blood pressure is 116/72 mm Hg, pulse is 96/min, and respirations are 16/min.
There is a palpable effusion around the left knee capsule in association with erythema and
warmth. Which of the following would be the most appropriate next step in management? 
 Incorrect Answer ImageA.Complete blood count
 Incorrect Answer ImageB.Left knee x-ray film
 Correct Answer ImageC.Left knee arthrocentesis
 Incorrect Answer ImageD.Left knee MRI
 Incorrect Answer ImageE.Left knee arthroscopy

A previously rare respiratory infection caused by an inhaled pathogen is suddenly discovered in a


group of children who attend daycare and their siblings. The infection is spread rapidly through
the daycare and the siblings' schools. Prevalence increases over 5 years, from 3 to 31% in three
years, peaking at 62% by five years. Time trends for this infection are monitored by means of a
standard screening test. As prevalence increases, which of the following would be the likely
impact on screening-test performance measures? 
 Incorrect Answer ImageA.Negative predictive value will remain unchanged
 Correct Answer ImageB.Positive predictive value will increase
 Incorrect Answer ImageC.Sensitivity will increase
 Incorrect Answer ImageD.Specificity will decrease
 Incorrect Answer ImageE.The impact cannot be determined

A 30-year-old woman, gravida 1, para 0, at 31 weeks' gestation comes to the physician because
of vaginal bleeding. She states that she had an episode of vaginal bleeding with bright red blood
this morning. The amount was a little less than her typical menses. First-trimester
ultrasonography has shown a dichorionic-diamniotic twin pregnancy. She has no significant
medical or surgical history. Her temperature is 37°C (98.6°F), pulse is 88/min, blood pressure is
100/60 mm Hg, and respirations are 12/min. Speculum examination shows approximately 10 cc
of old blood in the vagina. An ultrasound is performed that shows two appropriately grown for
gestational age twins with a marginal placenta previa. The fetal heart rate tracings are both
reactive and the patient is not having contractions. Which of the following is the most
appropriate next step in management?
 Correct Answer ImageA.Administration of corticosteroids
 Incorrect Answer ImageB.Administration of magnesium sulfate
 Incorrect Answer ImageC.Induction of labor
 Incorrect Answer ImageD.Immediate cesarean delivery of both twins
 Incorrect Answer ImageE.Immediate cesarean delivery of the lower twin only

A 52-year-old woman undergoes a routine screening mammogram that shows several areas of
increased density with microcalcifications in the left breast. She denies any active symptoms and
has no significant past medical history. Her family history is negative for any medical illnesses,
including cancer. She denies alcohol and illicit drug use. Physical examination, including a
bilateral breast examination, is unremarkable. She undergoes radiologically-guided core biopsies
that show ductal carcinoma in situ (DCIS) in separate samples from three quadrants of the left
breast. Which of the following is the most appropriate course of action?
 Incorrect Answer ImageA.Continued clinical observation until palpable masses develop
 Incorrect Answer ImageB.Lumpectomy of the affected areas, plus axillary sampling
 Incorrect Answer ImageC.Lumpectomy of the affected areas, followed by radiotherapy
 Incorrect Answer ImageD.Modified radical mastectomy with axillary sampling
 Correct Answer ImageE.Simple, total mastectomy

A 23-year-old man has a 2-cm, hard, right supraclavicular mass that he first noticed 1 week ago.
The rest of the history and physical examination are noncontributory. He is referred to a surgeon
to have a biopsy, and by the time he is seen 1 week later, the mass has nearly doubled in size.
Incisional biopsy is performed, and frozen section is suggestive of metastatic testicular cancer.
Physical examination of the testicles is repeated after surgery and again found to be
unremarkable. Sonogram of the scrotal contents is likewise negative. Blood is sent for laboratory
work. By the next week the final pathology report is most compatible with (but not diagnostic of)
testicular cancer, the laboratory reports show very elevated levels of alpha-fetoprotein and beta-
human chorionic gonadotropin, and the remaining supraclavicular mass has again doubled in
size. Which of the following is the most likely source of this man's problem? 
 Incorrect Answer ImageA.Adrenal glands
 Incorrect Answer ImageB.Head and neck mucosa
 Incorrect Answer ImageC.Lungs
 Incorrect Answer ImageD.Pituitary gland
 Correct Answer ImageE.Retroperitoneal tissue

An 18-hour-old newborn is jaundiced. The baby was born to a 28-year-old mother who also has
a 3-year-old daughter. The pregnancy was uneventful and the infant was delivered by vaginal
delivery with the help of forceps. The Apgar scores were 7 and 10 at 1 and 5 minutes,
respectively. The baby has been breast-fed from birth and he is able to latch on and suck well. He
has passed urine twice since birth but no meconium. Upon examination, the infant is jaundiced
up to the nipples. He appears alert and has normal tone and Moro reflex. There is no cardiac
murmur and lungs are clear to auscultation. The liver and spleen are not enlarged. Laboratory
studies show: 
Serum bilirubin Total 11 mg/dL, direct 0.5 mg/dL
Mother's blood group O, Rh negative
Infant's blood group A, Rh negative
Infant's blood count
Hemoglobin 15 g/dL
Hematocrit 45%
Reticulocytes 8%
White cell count 13,000/mm3
Platelets 325,000/mm3
Which of the following is the most likely explanation for the infant's jaundice? 
 Incorrect Answer ImageA.Breastfeeding jaundice
 Correct Answer ImageB.Hemolytic jaundice
 Incorrect Answer ImageC.Physiologic jaundice
 Incorrect Answer ImageD.Polycythemia
 Incorrect Answer ImageE.Sepsis

A 79-year-old man comes to the emergency department after a fall. He had been bathing himself
when he slipped from a seated position and landed on his right side. On arrival he is in acute
distress and is diaphoretic. His blood pressure is 158/90 mm Hg, pulse is 110/min, and
respirations are 24/min. He is lying with his right leg internally rotated in extension.
Radiographic examination confirms a complete fracture through the greater trochanter of the
right femur, and he is taken for surgical complete hip replacement. He reports having a normal
puberty, with acne and shaving by age 14 years. He is 178 cm (70 in) tall, weighs 75 kg (165 lb),
and has had no recent changes in his height or weight. His libido has declined steadily over the
last several decades, but he and his wife still have intercourse approximately once per week. He
was active around his house and was able to live independently with minimal assistance. He
denies a history of smoking and has been an occasional drinker all his life. His family history is
remarkable for his mother having multiple vertebral compression fractures in her mid-seventies.
Laboratory studies show: 
Serum calcium 9.8 mg/dL (8.5–10.5 mg/dL)
Serum free testosterone 280 ng/dL (200–1,000
ng/dL)
Serum thyroid-stimulating 3.5 mU/mL (0.5–5.0
hormone mU/mL)
Now that his fracture has been surgically reduced, which of the following is the most important
and appropriate initial therapy? 
 Incorrect Answer ImageA.Estrogen supplementation
 Correct Answer ImageB.Oral bisphosphonate therapy
 Incorrect Answer ImageC.Oral calcium and vitamin D supplementation
 Incorrect Answer ImageD.Program of weight bearing exercise

 Incorrect Answer ImageE.Transdermal testosterone supplementation


While driving home from the hospital, a physician sees an elderly woman lying in the street. She
appears semiconscious and has blood coming out of the corner of her mouth. Apparently she is a
victim of a hit and run motor vehicle accident, and the physician stops to help. The patient is
drifting in and out of consciousness and is able to moan and gurgle only occasionally. While the
physician is calling for help on a cellular phone, the patient loses consciousness completely. The
physician then kneels at the patient's side and plans to manage her airway. Which of the
following is the most appropriate technique to manage this patient's airway? 
 Correct Answer ImageA.Direct jaw thrust without head tilt to bring the tongue forward
 Incorrect Answer ImageB.Head tilt and chin lift to remove a glottic obstruction
 Incorrect Answer ImageC.Insertion of a nasopharyngeal airway to maintain the airway
 Incorrect Answer ImageD.Provide two breaths with an available mouth-to-pocket face
mask
 Incorrect Answer ImageE.Slowly turn the patient to the left lateral decubitus position

A 43-year-old woman comes to the clinic because of lower extremity swelling that has increased
over the last several months. She notes thickened skin over her face, neck and trunk, along with
episodes of blisters on the back of her hands and feet occurring after minor trauma. The skin is
first red, then forms large, tense bullae which become hemorrhagic and spontaneously drain.
Over time, the skin has become tough and thick. She also notes a history of sporadic, self-limited
episodes of red-orange colored urine, particularly after alcohol consumption. She denies
lethargy, fevers, menstrual irregularity, muscle pains, or swelling. Her past medical history is
unremarkable except for two pregnancies 20 years earlier that were complicated by massive
hemorrhages requiring multiple blood transfusions. Her only medications are oral contraceptive
pills that she has taken for years. On physical examination, she is in no distress, and her vital
signs are normal. Cardiac and pulmonary examinations are normal, and there is no abdominal
organomegaly. The skin over her hands, shins, and feet is thick and indurated but without
erythema or other lesions. This woman's condition is commonly associated with which of the
following?
 Incorrect Answer ImageA.Autoimmune thyroid disease
 Incorrect Answer ImageB.Episodic rhabdomyolysis
 Incorrect Answer ImageC.Esophageal dysmotility
 Correct Answer ImageD.Hepatitis C disease
 Incorrect Answer ImageE.Progressive renal failure

A 47-year-old obese woman with systemic lupus erythematosus comes to the emergency
department because of fever and vague upper abdominal pain for 24 hours. She reports nausea
with vomiting and epigastric discomfort. She is finishing up a 14-day course of oral prednisone
for a lupus flare, and she has been taking ibuprofen for joint pains. Vital signs are: temperature
38.5°C (101.5°F) and pulse 105/min. On physical examination, there is mild tenderness to
palpation in the right upper quadrant with decreased bowel sounds. Pertinent laboratory findings
are: leukocyte count of 14,500/mm3, serum amylase of 195 U/L, liver function tests are normal.
CT scan of the abdomen shows some fluid and air around the second portion of the duodenum,
tracking into Morrison’s pouch. There are inflammatory changes surrounding the head of the
pancreas and the gallbladder. Which of the following is the most likely diagnosis? 
 Correct Answer ImageA.A perforated duodenal ulcer
 Incorrect Answer ImageB.Cholecystitis with gallbladder perforation
 Incorrect Answer ImageC.Cholangitis
 Incorrect Answer ImageD.Lupus gastritis
 Incorrect Answer ImageE.Pancreatitis

A 6-year-old boy with no past medical history is brought to the physician by his parents, who are
divorced. The patient's father is concerned that the child is "hyperactive" and "won't calm down"
every time the patient comes to see him for weekend visits. He reports that the patient
consistently has difficulty sitting still, cannot complete homework, and does not listen or pay
attention to him. The mother notices "no such problems" when the patient is with her. Both
parents are concerned about his academic future. The patient appears calm in the office and
physical examination is unremarkable. Which of the following is the most appropriate next step
in management?
 Incorrect Answer ImageA.Begin a trial of antidepressants
 Correct Answer ImageB.Obtain information from the patient's teacher regarding his
behavior in school
 Incorrect Answer ImageC.Order an electroencephalogram (EEG)
 Incorrect Answer ImageD.Recommend that the school test him for placement in special
education
 Incorrect Answer ImageE.Prescribe a psychostimulant

A 72-year-old woman comes to the physician for a routine health maintenance visit. She has a
long-standing history of gastroesophageal reflux disease, which has been treated successfully
with omeprazole. She also has a psychiatric history that is not well delineated. Physical
examination shows an elderly woman who appears disheveled. Upon further questioning, she
reports being tired all the time, having a decreased appetite, and feelings of worthlessness and
extreme guilt over the last month. The patient denies suicidal thoughts, but reports she has little
to live for since her husband died 6 years ago. She also reports that 2 months ago she had
numerous racing thoughts, kept starting numerous projects around the house, and only slept for
10 hours over a period of 8 days. She denies delusions or hallucinations. Which of the following
is the most likely diagnosis?
 Correct Answer ImageA.Bipolar I disorder
 Incorrect Answer ImageB.Depression
 Incorrect Answer ImageC.Generalized anxiety disorder
 Incorrect Answer ImageD.Mania
 Incorrect Answer ImageE.Schizophrenia

A 21-year old white male comes to the physician for evaluation of episodes of shortness of
breath, which have increased over the last 2 months. The patient reports that these episodes
happened before, but he was not concerned since he has had several bouts of "a cold". Now he
complains of significant dyspnea when walking up stairs or playing sports. He also complains of
episodes of a dry, non-productive cough. The patient's past medical history is significant for
upper respiratory tract infections in childhood and gastroesophageal reflux disease. The patient is
a college student, denies drinking alcohol, and he has never smoked. Review of systems is
negative for fever, chills or weight loss. His temperature is 37.0°C (98.6°F), blood pressure is
122/74 mm Hg, pulse is 104/min and respirations are 24/min. Physical examination shows a
well-developed young white male in no acute distress. A diffuse wheeze is heard on lung
auscultation. Chest x-ray shows hyperinflated lungs and a flattened diaphragm.
Electrocardiogram shows sinus tachycardia with no acute ST-segment changes. The patient is
given albuterol in the physician’s office and his shortness of breath is relieved completely.
Obstructive lung disease is suspected and pulmonary function testing is ordered. Which of the
following results will confirm a diagnosis of obstructive lung disease?
% FEV1; % FVC; % TLC; % DLco
 Incorrect Answer ImageA.88; 83; 94; 95
 Correct Answer ImageB.54; 80; 118; 92
 Incorrect Answer ImageC.53; 85; 71; 69
 Incorrect Answer ImageD.95; 102; 96; 98
 Incorrect Answer ImageE.59; 62; 71; 60

A 52-year-old woman comes to her primary care physician because of difficulty falling asleep
which has become progressively worse over the past three years. Her symptoms begin while
sitting in bed to read at around 10 PM. She describes a discomfort in her calves and feet and a
“crawling” sensation running up and down her legs. She notes that there is an urge to move her
legs, which results in momentary relief from the discomfort. Within 10 to 15 seconds the
symptoms return. Rubbing the feet, putting them in warm water, or walking offers relief from her
discomfort. The sensations are so severe that she is unable to fall asleep until after 1 AM and has
found herself feeling fatigued in the morning. Her husband also noted that she kicks her legs
vigorously while sleeping. The woman was recently surprised to learn that her mother and
another sister also had a similar problem. Physical examination and laboratory studies are
normal. Which of the following is the most likely diagnosis? 
 Incorrect Answer ImageA.Akathisia
 Incorrect Answer ImageB.L-5 radiculopathy
 Incorrect Answer ImageC.Nocturnal cramps
 Incorrect Answer ImageD.Painful peripheral neuropathy
 Correct Answer ImageE.Restless legs syndrome

A 71-year-old man comes to the emergency department because of increasing shortness of breath
for 1 day. He has a 50 pack-year smoking history, long-standing hypertension, and was
diagnosed with congestive heart failure a year ago. He reports that he has been well over the past
year, but yesterday began to develop worsening shortness of breath and severe dyspnea with
minimal exertion. He denies any chest pain at rest. His oxygen saturation on 3L nasal cannula is
90% and his respiratory rate is 22/min. On examination, his lungs have bibasilar rales, an S3 is
appreciated, and his apical impulse is weak in the 6th intercostal space. His jugular venous
pressure is 12 cm, and he has 2+ pitting edema of his lower extremities. He has been compliant
with his furosemide and digoxin therapy. Which of the following is the most appropriate next
step in management?
 Correct Answer ImageA.Admit the patient to the hospital for further evaluation and start
IV diuretics
 Incorrect Answer ImageB.Advise the patient to see his primary care physician the next
morning
 Incorrect Answer ImageC.Increase the dose of digoxin and send the patient home with
follow-up
 Incorrect Answer ImageD.Increase the dose of furosemide and send the patient home
with follow-up
 Incorrect Answer ImageE.Observe the patient in the emergency room after giving a dose
of oral furosemide

The following vignette applies to the next two items. 


A 58-year white man with a history of end-stage liver disease secondary to hepatitis C is
admitted to the hospital with decreased urine output and generalized weakness for the last 4 days.
His past medical history is significant for end-stage liver disease secondary to hepatitis C
diagnosed 2 years ago. The patient takes lactulose, furosemide, and spironolactone for his
ascites. His vitals are temperature 37.0, heart rate 96/min, blood pressure 125/70 and respirations
22/min. Physical examination is significant for icteric sclera with dry and jaundiced skin. Breath
sounds are decreased at the bases with occasional crackles. Heart sounds are distant. The
abdomen is distended, slightly tender, without guarding or rebound tenderness, and bowel
sounds are present in all four quadrants. Ascites is present, but it is not significantly different
from his last examination 2 weeks ago. Examination of extremities reveals 3+ pitting edema, no
cyanosis or clubbing. Laboratory tests reveal: 
WBC count 9,000/mm3
Hemoglobin 11.6 gm/dL
Hematocrit 38%
Platelets 87,000/ml
Sodium 128 mmol/L
Potassium 4.9 mmol/L
Chloride 98 mmol/L
Carbon dioxide 20 mmol/L
BUN 80 mg/dL
Creatinine 3.6 mg/dL
Glucose 110 mg/dL
Calcium 7.8 mg/dL
Total bilirubin 14.6 mg/dL
Alkaline phosphatase 240 U/L
AST 578 U/L
ALT 660 U/L
Baseline creatinine was 1.2 mg/dL and BUN was 26-30 mg/dL
Urinalysis is bland with no white blood cells or protein. Urine sodium concentration was 8
meq/L. Renal ultrasound did not show any significant abnormalities. The patient is given
intravenous albumin along with cessation of his diuretics for 48 hours. However, his condition
does not improve. His urine output remains low and creatinine measured on the following day
was 3.7 mg/dL. On repeat examination, his ascites and lower extremity edema appear worse than
previously. Which of the following is the most likely diagnosis?
 Incorrect Answer ImageA.Acute glomerulonephritis
 Incorrect Answer ImageB.Acute renal failure secondary to dehydration
 Incorrect Answer ImageC.Hepatopulmonary syndrome
 Correct Answer ImageD.Hepatorenal syndrome
 Incorrect Answer ImageE.Pyelonephritis
A 58-year white man with a history of end-stage liver disease secondary to hepatitis C is
admitted to the hospital with decreased urine output and generalized weakness for the last 4 days.
His past medical history is significant for end-stage liver disease secondary to hepatitis C
diagnosed 2 years ago. The patient takes lactulose, furosemide, and spironolactone for his
ascites. His vitals are temperature 37.0, heart rate 96/min, blood pressure 125/70 and respirations
22/min. Physical examination is significant for icteric sclera with dry and jaundiced skin. Breath
sounds are decreased at the bases with occasional crackles. Heart sounds are distant. The
abdomen is distended, slightly tender, without guarding or rebound tenderness, and bowel
sounds are present in all four quadrants. Ascites is present, but it is not significantly different
from his last examination 2 weeks ago. Examination of extremities reveals 3+ pitting edema, no
cyanosis or clubbing. Laboratory tests reveal: 
WBC count 9,000/mm3
Hemoglobin 11.6 gm/dL
Hematocrit 38%
Platelets 87,000/ml
Sodium 128 mmol/L
Potassium 4.9 mmol/L
Chloride 98 mmol/L
Carbon dioxide 20 mmol/L
BUN 80 mg/dL
Creatinine 3.6 mg/dL
Glucose 110 mg/dL
Calcium 7.8 mg/dL
Total bilirubin 14.6 mg/dL
Alkaline phosphatase 240 U/L
AST 578 U/L
ALT 660 U/L
Baseline creatinine was 1.2 mg/dL and BUN was 26-30 mg/dL
Urinalysis is bland with no white blood cells or protein. Urine sodium concentration was 8
meq/L. Renal ultrasound did not show any significant abnormalities. The patient is given
intravenous albumin along with cessation of his diuretics for 48 hours. However, his condition
does not improve. His urine output remains low and creatinine measured on the following day
was 3.7 mg/dL. On repeat examination, his ascites and lower extremity edema appear worse than
previously. Which of the following is the most appropriate next step in the management?
 Incorrect Answer ImageA.Clonidine
 Correct Answer ImageB.Combination of midodrine and octreotide
 Incorrect Answer ImageC.Continue normal saline IV
 Incorrect Answer ImageD.Hemodialysis
 Incorrect Answer ImageE.Midodrine
 Incorrect Answer ImageF.Octreotide

The following vignette also applies to the next item.


A 42-year-old woman is playing tennis with a much better player. She tries to stop an oncoming
ball, but sustains an injury that prevents her from continuing the game. In the emergency
department the patient is in pain, holding the right hand with the left. Diffuse swelling is noted in
the base of the hypothenar area of the right hand. There is an underlying bluish discoloration in
the same area. During the examination, she states that the butt of the tennis racquet struck her
wrist when the oncoming ball met her racquet. Further examination reveals normal range of
movement of the fingers, although the pain increases when she tries to make a fist. No rotational
abnormality is noted in the metacarpals. The base of the hypothenar region is tender to palpation.
Which of the following is the most likely diagnosis? 
 Incorrect Answer ImageA.Acute carpal tunnel syndrome
 Incorrect Answer ImageB.Colles fracture
 Correct Answer ImageC.Fracture of the hamate
 Incorrect Answer ImageD.Fracture of the scaphoid bone
 Incorrect Answer ImageE.Hematoma of the thenar eminence

A 42-year-old woman is playing tennis with a much better player. She tries to stop an oncoming
ball, but sustains an injury that prevents her from continuing the game. In the emergency
department the patient is in pain, holding the right hand with the left. Diffuse swelling is noted in
the base of the hypothenar area of the right hand. There is an underlying bluish discoloration in
the same area. During the examination, she states that the butt of the tennis racquet struck her
wrist when the oncoming ball met her racquet. Further examination reveals normal range of
movement of the fingers, although the pain increases when she tries to make a fist. No rotational
abnormality is noted in the metacarpals. The base of the hypothenar region is tender to palpation.
The patient is concerned about the possibility of additional injury to the hand. Which of the
following nerves would be most likely affected? 
 Incorrect Answer ImageA.Axillary nerve
 Incorrect Answer ImageB.Median nerve
 Incorrect Answer ImageC.Musculocutaneous nerve
 Incorrect Answer ImageD.Radial nerve
 Correct Answer ImageE.Ulnar nerve

A previously healthy 34-year-old woman comes to the physician because of fever and cough
with greenish sputum production for two days. She denies dyspnea. Her temperature is 38.3°C
(101.0°F), pulse is 88/min, and respirations are 18/min. There is no accessory muscle use or
conversational dyspnea, nor are there bronchial breath sounds, rales, or egophony. Chest x-ray
reveals a right lower lung consolidation. Leukocyte count is 13,000/mm3. Which of the following
is the most appropriate treatment for this patient? 
 Incorrect Answer ImageA.Amoxicillin
 Incorrect Answer ImageB.Ampicillin-sulbactam
 Correct Answer ImageC.Azithromycin
 Incorrect Answer ImageD.Ceftriaxone
 Incorrect Answer ImageE.Erythromycin plus ceftriaxone

A 52-year-old woman comes to the physician complaining of involuntary loss of urine when she
coughs and sneezes. The loss of urine occurs both during the day and when she is asleep at night.
The urge to void can be triggered by hearing a toilet flush or by touching a cold door handle. She
cannot suppress the urge to void. Her last menstrual period was 3 weeks ago. Past medical
history is significant for a tubal sterilization. Pelvic examination reveals a mild cystocele and
rectocele. Neurologic examination is unremarkable. Which of the following is the most likely
diagnosis? 
 Incorrect Answer ImageA.Bypass incontinence
 Incorrect Answer ImageB.Detrusor areflexia
 Correct Answer ImageC.Detrusor hyperreflexia
 Incorrect Answer ImageD.Genuine stress incontinence
 Incorrect Answer ImageE.Irritation incontinence

A 5-year-old boy has fever and rhinorrhea for three days. He has no past medical history and no
other complaints. His temperature is 38.8°C (101.8°F), pulse is 90/min, and respirations are
22/min. There is clear mucus in the anterior nares and the right tympanic membrane is
erythematous and bulging and has decreased mobility and no light reflex. The oropharynx, lungs,
and abdomen are normal. Which of the following is the most likely causal organism? 
 Incorrect Answer ImageA.Adenovirus
 Incorrect Answer ImageB.Respiratory syncytial virus
 Incorrect Answer ImageC.Staphylococcus aureus
 Correct Answer ImageD.Streptococcus pneumoniae
 Incorrect Answer ImageE.Streptococcus pyogenes

A 58-year-old man comes to his physician for a routine follow-up. His medical history is
significant for hypertension, coronary artery disease with a myocardial infarction three years ago,
and an aortofemoral bypass six years ago. He takes aspirin, metoprolol, and isosorbide dinitrate.
He smokes one pack of cigarettes per day and does not drink alcohol. A recent fasting lipid
profile showed: 
Total cholesterol 230 mg/dL
LDL cholesterol 147 mg/dL (recommended <100 mg/dL)
HDL cholesterol 27 mg/dL (recommended >35 mg/dL)
Triglycerides 277 mg/dL
Which of the following is the most appropriate initial step in management of this lipid
abnormality? 
 Incorrect Answer ImageA.Prescribe only a cholesterol-lowering diet
 Correct Answer ImageB.Prescribe a cholesterol-lowering diet and a
hydroxymethylglutaryl coenzyme A (HMG-CoA) reductase inhibitor
 Incorrect Answer ImageC.Prescribe fibric acid
 Incorrect Answer ImageD.Recommend one to two ounces of red wine per day
 Incorrect Answer ImageE.Repeat a fasting lipid profile in three to six months

A 2-day-old infant, who was non-cyanotic at birth, is noted to have a systolic murmur along his
left sternal border. The remainder of the physical examination is normal. An electrocardiogram
and chest x-ray are normal. Which of the following is the most likely diagnosis? 
 Incorrect Answer ImageA.Patent ductus arteriosus
 Incorrect Answer ImageB.Tetralogy of Fallot
 Incorrect Answer ImageC.Transposition of the great vessels
 Incorrect Answer ImageD.Tricuspid atresia
 Correct Answer ImageE.Ventricular septal defect

A 31-year-old woman comes to her primary care physician because of dyspnea and general
weakness. The patient says that she experienced significant shortness of breath with moderate
activity and she had one syncopal episode a few days ago. The patient's past medical history is
unremarkable. Her temperature is 36.8°C (98.2°F), pulse is 110/min, respirations are 22/min, and
blood pressure is 125/70 mm Hg. Physical examination shows an increased pulmonic component
of S2 and paradoxical splitting of S2. Lung examination is normal. Abdominal examination
reveals hepatomegaly with palpable pulsations of the liver and an abnormal abdomino-jugular
reflux. Electrocardiogram (ECG) shows right axis deviation and right ventricular hypertrophy.
Which of the following is an expected finding on chest x-ray? 
 Incorrect Answer ImageA.Bilateral infiltrates
 Correct Answer ImageB.Enlarged central pulmonary arteries
 Incorrect Answer ImageC.Left ventricular hypertrophy
 Incorrect Answer ImageD.Normal chest x-ray
 Incorrect Answer ImageE.Pleural effusions

A 50-year-old man comes to the clinic for a routine visit. He has a long history of poorly
controlled diabetes and hypertension, commonly misses clinic appointments, and is often
noncompliant with his medications. Known diabetic complications in this patient include
gastroparesis and early nephropathy. He has come to see you to discuss problems getting and
maintaining an erection, and he is requesting a pill he saw advertised on television. When asked,
he admits that he rarely gets spontaneous erections, but states that he still desires to have sex.
Physical examination, including a diabetic foot examination, is remarkable for slightly reduced
sensation in the soles of the patient's feet and a moderate left-sided varicocele. Which of the
following is the most likely cause of this patient's erectile dysfunction? 
 Correct Answer ImageA.Autonomic neuropathy
 Incorrect Answer ImageB.Peripheral neuropathy
 Incorrect Answer ImageC.Presence of a varicocele
 Incorrect Answer ImageD.Reduced testosterone levels
 Incorrect Answer ImageE.Vascular insufficiency

A 28-year-old woman seeks obstetrical consultation for the first time in her pregnancy when she
is 1 week away from her due date of delivery. A sonogram done at that time reveals that the baby
has a massive sacrococcygeal tumor almost as big as the baby herself. A cesarean section is
recommended and carried out. The mass is covered by normal-appearing skin, and the little girl
moves her lower extremities normally. The anus is displaced anteriorly, but it is normal and has
normal tone. CT scan shows that the mass is mostly posterior to the coccyx, with only a small
internal component that extends into the pelvis, in front of the sacrum. Biopsy of the mass
confirms a diagnosis of sacrococcygeal teratoma. The areas sampled have no evidence of
malignancy. Which of the following is the recommended course of action? 
 Incorrect Answer ImageA.Observation, expecting involution
 Correct Answer ImageB.Prompt surgical removal
 Incorrect Answer ImageC.Radiation therapy
 Incorrect Answer ImageD.Surgical removal between the ages of 2 and 6 months
 Incorrect Answer ImageE.Systemic chemotherapy

A 75-year-old man comes to the physician because of concern over difficulties with his speech
and memory that have occurred over the past 6 months. He is accompanied by his wife. He
speaks slowly while describing his current condition, often pausing to think or restart his
sentence. He states that he often loses his train of thought in mid-sentence. He reports that he
frequently gets lost while driving around his neighborhood where he has lived for the past four
decades. Although he is fairly certain that he knows his way home, he often winds up in the
wrong destination. During the interview, his wife steps out of the room briefly. At this point, the
patient confides that he is worried that she is not really his wife but is an identical-appearing
impostor. Physical examination shows a resting tremor, bradykinesia, and mild postural
instability. The patient has difficulty paying attention to the examination and seems unable to
concentrate on following simple commands. Which of the following is the most likely diagnosis?
 Incorrect Answer ImageA.Alzheimer's disease
 Correct Answer ImageB.Dementia with Lewy Bodies
 Incorrect Answer ImageC.Frontotemporal dementia
 Incorrect Answer ImageD.Multi-infarct or vascular dementia
 Incorrect Answer ImageE.Parkinson's disease

A 47-year-old man comes to the emergency department because of a tender lesion on the dorsum
of his left hand for the past 3 days. He initially felt some relief after applying warm compresses
over the lesion, but it is now progressing. He also reports low-grade fever, chills, malaise, and a
headache. He has had multiple similar but less exuberant skin lesions develop on various body
sites in the past year. His past medical history is significant for diabetes mellitus and
hypertension. Current medications include furosemide, metformin, and aspirin. He has no
allergies to medications. His temperature is 38.7°C (101.7°F), pulse is 95/min, respirations are
22/min, and blood pressure is 140/90 mm Hg. On physical examination, there is an 8 x 6 cm
tender, erythematous, and edematous plaque with multiple yellow-white, agminated pustules on
the dorsum of the left hand. Palpation of the lesion shows fluctuation and oozing pus from
multiple sites of the plaque. Which of the following microorganisms is the most likely cause of
this patient's disease?
 Incorrect Answer ImageA.Group A beta-hemolytic streptococcus (GAS)
 Incorrect Answer ImageB.Herpes simplex virus
 Incorrect Answer ImageC.Human papillomavirus
 Incorrect Answer ImageD.Pseudomonas aeruginosa
 Correct Answer ImageE.Staphylococcus aureus

A 25-year-old man has an exploratory laparotomy for a gunshot wound of the abdomen. At the
time of surgery, 2 segments of injured small bowel are resected and anastomosed. The patient
makes an uneventful recovery, regains bowel function on the fourth postoperative day, and is
discharged from the hospital on the fifth day. At approximately 10 days postoperatively, he
begins to spike temperatures up to 39.5°C (103.1°F) every day, and he also develops
leukocytosis. His wound seems to be healing well and is not red or tender. What is the most
likely cause of his fever?
 Incorrect Answer ImageA.Deep venous thrombosis (DVT)
 Correct Answer ImageB.Intra-abdominal abscess
 Incorrect Answer ImageC.Pulmonary atelectasis
 Incorrect Answer ImageD.Urinary tract infection (UTI)
 Incorrect Answer ImageE.Wound infection
A 27-year-old woman comes to the physician for an annual examination. She states that she has
been feeling well over the past year. She has a normal menstrual cycle every 29 days. She has no
medical problems and has never had surgery. She takes a multivitamin daily, but no medications.
She is allergic to sulfa drugs. Her family history is significant for her father dying of a
myocardial infarction at the age of 70 years. Her mother and sisters are alive and well. Physical
examination, including breast and pelvic examination, is normal, but the urine dipstick detects
hematuria. A repeat urine dipstick test performed 2 weeks later was also positive for hematuria.
Which of the following is the most appropriate next step in management?
 Incorrect Answer ImageA.Combined hysteroscopy/laparoscopy
 Correct Answer ImageB.Microscopic examination of the urine
 Incorrect Answer ImageC.Referral to a nephrologist
 Incorrect Answer ImageD.Referral to a urologist
 Incorrect Answer ImageE.Repeat urine dipstick in 2 weeks

A 24-year-old woman comes to the physician complaining of a vaginal discharge for 4 days. Her
last menstrual period began 10 days ago. She is sexually active with one partner and they use
condoms for birth control. She has no chronic medical conditions and takes no medications. She
has no known drug allergies. She has never been pregnant. General physical examination is
unremarkable. On sterile speculum examination there is a foamy, malodorous discharge. Wet
prep microscopic examination reveals multiple motile organisms. She is advised to notify her
sexual partners about her condition. Which of the following is the most appropriate
pharmacotherapy? 
 Incorrect Answer ImageA.Ampicillin
 Incorrect Answer ImageB.Doxycyline
 Incorrect Answer ImageC.Ciproflaxacin
 Incorrect Answer ImageD.Macrodantin
 Correct Answer ImageE.Metronidazole

A 23-year-old woman comes to the physician for an annual visit. She states that she has been
feeling well over the past year. She has menses approximately every 30 days. She is sexually
active with one partner and they intermittently use condoms for birth control. She has no medical
problems. She had a laparoscopic left ovarian cystectomy at the age of 18 and no other surgeries.
Examination, including breast and pelvic examinations, are normal. The patient has heard about
using a "vaginal ring" for contraception and she wants more information about this type of birth
control. Counseling of this patient should include which of the following? 
 Incorrect Answer ImageA.There is no "vaginal ring" form of contraception
 Incorrect Answer ImageB.The "vaginal ring" can be used in women with a history of
DVT
 Correct Answer ImageC.The "vaginal ring" is more than 98% effective when used
correctly
 Incorrect Answer ImageD.The "vaginal ring" protects against HIV transmission
 Incorrect Answer ImageE.The "vaginal ring" should be used only by breastfeeding
women

A 6-month-old girl is brought to the emergency department by her mother because of breathing
difficulty that started earlier that day. The child had been ill with a cold and cough for the
previous 2 days and was running a high fever that responded partially to over-the-counter
medication. That morning the mother noticed an increase in the cough with loud wheezing and
very shallow breathing. The girl refused to eat because she could not catch her breath in between
and became very fussy and irritable. When her fever reached 39 C (102.2 F) and would not
respond to the medication she had given her earlier, the mother came to the hospital. There is no
significant past medical history and the family history is positive for hay fever. The father tends
to refrain from smoking cigarettes when their daughter is in the room. On physical examination,
the patient is restless and in severe respiratory distress with respirations of 65/min, intercostal
and subcostal retractions, wheezing, and rales. Arterial blood gases show a PO2 of 60 mm Hg.
Chest radiography shows hyperinflation of the lungs with air trapping and atelectasis. Which of
the following is the most appropriate next step in management? 
 Incorrect Answer ImageA.High-dose inhaled steroids and discharge
 Correct Answer ImageB.Hospitalization for further treatment and possible mechanical
ventilation
 Incorrect Answer ImageC.Inhaled ribavirin and observation in the emergency department
 Incorrect Answer ImageD.Intravenous antibiotics in the emergency department followed
by oral antibiotics and discharge
 Incorrect Answer ImageE.Intravenous immune globulin

A 37-year-old man lives alone and has no close friends. He works during the night shift at the
post office and has little interaction with others. He has not engaged in sexual activity since he
was 18 years old, but he does not feel much desire. He maintains a close relationship with his
sister but does not seek out relationships with others. People have told him that he seems
"detached" and that he has difficulty experiencing or expressing emotions. Which of the
following is the most likely diagnosis? 
 Incorrect Answer ImageA.Antisocial personality disorder
 Incorrect Answer ImageB.Avoidant personality disorder
 Incorrect Answer ImageC.Paranoid personality disorder
 Correct Answer ImageD.Schizoid personality disorder
 Incorrect Answer ImageE.Social anxiety disorder

A 12-year-old boy is brought to the physician because of a pattern of behavior that has been
worsening over the past year. His mother states that he has been bullying other boys at school,
staying out late without permission, setting small fires in abandoned lots, and physically abusing
neighborhood cats. The boy does not appear apologetic and offers the story that his classmates
must be framing him. During the examination, a speculum from the examination room drawer
falls out from underneath his shirt. Which of the following is the most likely diagnosis?
 Incorrect Answer ImageA.Antisocial personality disorder
 Incorrect Answer ImageB.Attention deficit hyperactivity disorder (ADHD)
 Correct Answer ImageC.Conduct disorder
 Incorrect Answer ImageD.Oppositional defiant disorder (ODD)
 Incorrect Answer ImageE.Tourette syndrome

A 4-year-old girl was diagnosed with left-sided otitis media about 10 days ago and was
prescribed oral amoxicillin for 7 days. She has since developed bloody diarrhea with mucus,
crampy abdominal pain, and fever. Her temperature is 39.4°C (102.9°F), pulse is 88/min, and
respirations are 16/min. Physical examination shows normal bowel sounds and her abdomen is
diffusely tender to palpation. Which of the following is the most appropriate initial step in
diagnosis?
 Incorrect Answer ImageA.Barium enema
 Correct Answer ImageB.Evaluation of stool for Clostridium difficile toxins
 Incorrect Answer ImageC.Evaluation of stool for rotavirus
 Incorrect Answer ImageD.Stool examination for ova and parasites
 Incorrect Answer ImageE.Stool Hemoccult test

A 69-year-old man is brought to the emergency department by his children. They state that he
has become increasingly confused over the past week and is unsteady on his feet. They report
that he does not see a physician for any medical conditions and does not take any medications.
He has had alcohol use disorder for the past 20 years. He is unable to coordinate muscle activity
during voluntary movement, leading to unsteadiness when walking. He has bilateral paresis of
the lateral gaze and vertical and horizontal nystagmus. Which of the following is the most
appropriate next step in management? 
 Incorrect Answer ImageA.Administration of intravenous dextrose
 Incorrect Answer ImageB.Administration of intravenous mannitol
 Correct Answer ImageC.Administration of intravenous thiamine
 Incorrect Answer ImageD.CT of the brain with contrast
 Incorrect Answer ImageE.Lumbar puncture and examination of the cerebrospinal fluid
(CSF)

A 64-year-old man with generalized abdominal pain and jaundice is diagnosed with pancreatic
adenocarcinoma. In the days following the diagnosis, the patient states to his doctors that "I don't
have cancer - all I need is a Vitamin B12 shot and I will feel better fast". The patient is oriented at
all times, scoring 29/30 on a mini-mental state examination, but refuses all chemotherapy
medications, and adamantly refuses to be seen by the hospital oncologist, because "he is a cancer
doctor and I don't have cancer." Which of the following is the most likely diagnosis?
 Incorrect Answer ImageA.Acute stress disorder
 Correct Answer ImageB.Adjustment disorder
 Incorrect Answer ImageC.Alzheimer dementia
 Incorrect Answer ImageD.Delusional disorder
 Incorrect Answer ImageE.Major depressive disorder

A 23-year-old woman seeks help for exquisite pain with defecation and blood streaks on the
outside of her stools, which she has been having for several weeks. Because of the pain, she has
avoided having bowel movements, and when she finally did, the stools were hard and even more
painful. When seen, she has no fever or leukocytosis. Physical examination is limited because of
her pain, but on inspection, there is no evidence of a thrombosed external hemorrhoid or perianal
abscess, just a skin tag in the posterior midline. Digital rectal examination or even distracting the
buttocks is impossible due to her pain. Which of the following is the most appropriate next step?
 Incorrect Answer ImageA.Examination under anesthesia
 Incorrect Answer ImageB.Excision of the skin tag
 Incorrect Answer ImageC.Flexible sigmoidoscopy
 Correct Answer ImageD.Topical nitroglycerin
 Incorrect Answer ImageE.Topical steroid cream

A 68-year-old woman comes to the physician because of a painful lump in her vagina. She states
that the lump has been there for a few months, but has recently begun to cause her pain. She has
hypertension, for which she takes a diuretic, but no other medical problems. Examination shows
a 4 cm cystic mass near the patient's introitus by the right labia. The mass is mildly tender. The
remainder of the pelvic examination is normal. Which of the following is the most appropriate
next step in management? 
 Incorrect Answer ImageA.Antibiotics by mouth

 Incorrect Answer ImageB.Aspiration of the cyst


 Incorrect Answer ImageC.Cold compresses

 Correct Answer ImageD.Drainage and biopsy of the cyst


 Incorrect Answer ImageE.Ultrasound guided drainage

A 6-year-old boy is brought to the physician because of fever for 3 days. He has no significant
past medical history and takes no medications. His younger brother has had similar upper
respiratory symptoms the prior week and the parents just want to make sure that nothing serious
is going on. On physical examination, the patient is in mild distress with a temperature of 39.0ºC
(102.2ºF), pulse is 110/min, and respirations are 26/min. Pharyngeal mucosa is erythematous and
there is a clear nasal discharge. There is a soft, vibratory systolic heart murmur grade 2/6, best
heard at the left lower sternal border. The murmur was not present at the last routine physical
examination. Which of the following is the most likely diagnosis?
 Correct Answer ImageA.Innocent heart murmur
 Incorrect Answer ImageB.Patent ductus arteriosus murmur
 Incorrect Answer ImageC.Pulmonary flow murmur
 Incorrect Answer ImageD.Venous hum
 Incorrect Answer ImageE.Ventricular septal defect murmur

A 31-year-old woman, gravida 4, para 4, comes to the physician for placement of an intrauterine
device (IUD). Her youngest child is 1 year old. She thinks that she is done childbearing, but is
not certain. She is in a monogamous relationship with her husband of 10 years. Previously, she
has used other forms of contraception, including the diaphragm and depot medroxyprogesterone
acetate, but was not completely satisfied with these methods. Her past medical history is
unremarkable. She takes no medications. Which of the following medications should be used as
antibiotic prophylaxis for her IUD insertion?
 Incorrect Answer ImageA.Azithromycin
 Incorrect Answer ImageB.Cefazolin
 Incorrect Answer ImageC.Doxycycline
 Correct Answer ImageD.No antibiotic prophylaxis
 Incorrect Answer ImageE.Vancomycin

A 62-year-old man with a past history of passing several ureteral stones comes to the emergency
department complaining of right flank pain that began 2 hours ago. The pain had a sudden onset,
is colicky, radiates to the inner thigh and scrotum, and is accompanied by nausea and vomiting.
On physical examination, he looks dehydrated and is obviously in great pain, but physical
findings in the abdomen and flank are not impressive. He is a little sore, but deep palpation does
not elicit muscle guarding or tenderness. There is no rebound pain. He has red blood cells in the
urine and his serum creatinine is 2.7 mg/dL. Plain abdominal films are normal. Which of the
following is the most appropriate next step in diagnosis?
 Correct Answer ImageA.CT scan of the abdomen
 Incorrect Answer ImageB.Intravenous pyelogram (IVP)
 Incorrect Answer ImageC.MRI of the abdomen
 Incorrect Answer ImageD.Retrograde cystourethrogram
 Incorrect Answer ImageE.Retrograde urethrogram

A 23-year-old woman from Hawaii comes to the physician for an annual health maintenance
examination. She states that she has been in good health over the past year. She takes a
combination oral contraceptive pill. She has three sexual partners and they use condoms
intermittently. She has no medical problems and no known drug allergies. Physical examination,
including breast and pelvic examination, is normal. A test for gonorrhea performed during the
visit returns as positive. Which of the following is the most appropriate pharmacotherapy for this
patient?
 Correct Answer ImageA.Ceftriaxone and azithromycin
 Incorrect Answer ImageB.Ciprofloxacin and azithromycin
 Incorrect Answer ImageC.Ciprofloxacin and doxycycline
 Incorrect Answer ImageD.Ofloxacin and azithromycin
 Incorrect Answer ImageE.Ofloxacin and doxycycline

A 72-year-old man who lives alone calls 911 saying that he has severe chest pain. Except for
indicating that he did not sustain any trauma, he cannot give a coherent history when picked up
by emergency medical technicians. On arrival at the emergency department, he is cold and
diaphoretic with a blood pressure of 80/65 mm Hg. He also has an irregular, feeble pulse of
130/min. His neck and forehead veins are distended and he is short of breath. Which of the
following is the most likely diagnosis? 
 Incorrect Answer ImageA.Extrinsic cardiogenic shock
 Incorrect Answer ImageB.Hypoglycemic shock
 Incorrect Answer ImageC.Hypovolemic shock
 Correct Answer ImageD.Intrinsic cardiogenic shock
 Incorrect Answer ImageE.Vasomotor shock

A 19-year-old man is admitted to the hospital with high fever, chills, and leg pain of 1 day’s
duration. He has a medical history of acute myeloid leukemia diagnosed 6 months ago.
Approximately 2 weeks ago, he underwent treatment with cytarabine (ara-C). He has no other
medical problems and no known allergies to medications. Family history is unremarkable. On
admission, his temperature is 39.5°C (103.1°F), pulse is 100/min, and respirations are 24/min.
Petechiae are present on the dorsum of the right foot and the fourth toe. Within hours, there are
multiple violaceous red plaques extending from the fourth right toe onto the dorsal foot, anterior
shin, and medial thigh and groin. The distribution follows a lymphangitic pattern. Palpation of
the lesions shows exquisite tenderness, and palpable, enlarged right inguinal lymph nodes are
noted. Inspection of the interdigital spaces on both feet is significant for macerated thick scale
and fissures. A complete blood count shows:
Leukocytes 0.3 x 109/L
Red blood cells 3.6 x 1012/L
Platelets 18 x 106/L
Which of the following microorganisms most likely will grow from blood cultures? 
 Incorrect Answer ImageA.Beta-hemolytic group A Streptococcus
 Incorrect Answer ImageB.Epidermophyton floccosum
 Incorrect Answer ImageC.Pasteurella multocida
 Correct Answer ImageD.Pseudomonas aeruginosa
 Incorrect Answer ImageE.Trichophyton mentagrophytes

A 44-year-old man comes to the emergency department because of severe back pain and flu-like
symptoms including severe headache that is worse when he coughs, myalgias, shaking chills, and
drowsiness. Over the past 4 days, he noted left leg weakness and severe lumbar back pain
radiating down his left leg. His past medical history is significant for diabetes mellitus and
hypertension, neither of which is well controlled. Although he denies any alcohol, drug, or
tobacco use, the patient smells of alcohol and tobacco. Vital signs include a temperature of
39.4°C (103.0°F), blood pressure 168/95 mm Hg, pulse 110/min, and respirations 24/min.
Physical examination is remarkable for tenderness over his lumbar vertebrae, left-sided lower
extremity weakness, foot drop, and a positive left Babinski sign. Placing an intravenous line is
extremely difficult as most of the patient's veins are scarred and sclerosed and appear to have
multiple puncture marks on the overlying skin. Which of the following is the most appropriate
diagnostic test?
 Incorrect Answer ImageA.Contrast enhanced CT scan of the brain and spine
 Incorrect Answer ImageB.Contrast enhanced myelography
 Incorrect Answer ImageC.CT scan of the spine and brain without contrast
 Incorrect Answer ImageD.Lumbar puncture and CSF analysis
 Correct Answer ImageE.MRI of the spine

An 18-month-old male infant is noted by his parents to have anorexia, weight loss, and right
upper abdominal discomfort. When he is seen by his pediatrician, he is found to be jaundiced
with a total bilirubin of 12 mg/dL. There is a palpable mass arising from his liver, and CT scan
confirms the presence of a large, single, right-sided liver mass. The infant also has elevated
levels of alpha-fetoprotein. Which of the following is the most likely diagnosis?
 Incorrect Answer ImageA.Hemangioma
 Correct Answer ImageB.Hepatoblastoma
 Incorrect Answer ImageC.Hepatocellular carcinoma
 Incorrect Answer ImageD.Metastatic cancer to the liver
 Incorrect Answer ImageE.Rhabdomyosarcoma

A 63-year-old man is admitted to the intensive care unit in cardiogenic shock from a large,
anteroseptal myocardial infarction. At admission he is confused, feels cold and clammy, and is in
acute renal failure. He is placed initially on an intra-aortic balloon pump and a dopamine drip
and is sent for emergent cardiac catheterization. The next day the patient improves to the point
where the intra-aortic balloon pump is weaned. He reports a dull, right upper quadrant pain.
Additionally, the patient appears to have icteric sclera, a finding not noticed on admission Liver
function tests are obtained. Which of the following is an expected pattern for his liver function
tests?
 Incorrect Answer ImageA.AST level less than the ALT level, with both greater than
1,000 U/L
 Incorrect Answer ImageB.AST more than twice the ALT level, ALT level less than 300
U/L
 Correct Answer ImageC.Extremely high aminotransferase levels, which gradually
normalize
 Incorrect Answer ImageD.Near normal INR, glucose level, and total bilirubin level
 Incorrect Answer ImageE.Total protein level that is more than 2 to 3 times the albumin
level

A 6-month-old boy is receiving ranitidine and cisapride for gastroesophageal reflux. Five days
after beginning therapy with erythromycin for otitis media, he is found dead in his crib. Which of
the following is the most likely cause of death? 
 Incorrect Answer ImageA.Anaphylaxis
 Incorrect Answer ImageB.Aspiration pneumonia
 Incorrect Answer ImageC.Bacterial sepsis
 Incorrect Answer ImageD.Sudden infant death syndrome
 Correct Answer ImageE.Ventricular arrhythmia

A 45-year-old woman comes in to the clinic with right upper quadrant colicky pain that
developed yesterday after dinner. The pain is severe and it radiates to the right shoulder. The
patient also complains of nausea and vomiting. The patient did not have similar episodes in the
past and otherwise she is healthy. Her vitals are temperature 38.1°C (100.6°F), pulse 102/min,
blood pressure 128/68 mm Hg, respirations 18/min. Physical exam showed normal
cardiovascular and pulmonary findings. Abdomen is tender in right upper quadrant, Murphy's
sign is positive, no rebound tenderness or guarding, bowel sounds are present. Complete blood
count showed elevated white blood cell count. Liver function tests revealed mild elevation of
serum bilirubin and alkaline phosphatase. Which of the following is the best test to confirm acute
cystic obstruction. 
 Incorrect Answer ImageA.Abdominal ultrasound
 Incorrect Answer ImageB.CT scan of the abdomen
 Incorrect Answer ImageC.Endoscopic retrograde cholangiopancreatography (ERCP)
 Correct Answer ImageD.HIDA (hepatobiliary iminodiacetic acid) scan
 Incorrect Answer ImageE.KUB

A 36-year-old man comes to the physician because he "always feels kind of down." He is "a poor
sleeper" and is "always pretty tired." He states, "I never got where I wanted in life because I was
never confident enough." He has never made a suicide attempt and denies any suicidal intent,
ideation, or plan. The patient goes on to add, "Things never get that bad for me, but they're never,
ever good." He has had these symptoms at least since his late adolescence. Physical examination
is unremarkable. Which of the following is the most likely diagnosis?
 Incorrect Answer ImageA.Bipolar I disorder
 Incorrect Answer ImageB.Cyclothymic disorder
 Correct Answer ImageC.Persistent depressive disorder
 Incorrect Answer ImageD.Schizoid personality disorder
 Incorrect Answer ImageE.Schizophreniform disorder

A 17-year-old sprinter comes to the high school health office because of shortness of breath after
a warm-up exercise. He reports tightness in his chest and difficulty catching his breath. Similar
episodes have occurred twice in the last year and each time has resolved slowly with rest. He has
a history of seasonal allergies to trees and grass. He occasionally uses ibuprofen for knee pain
after practice but has not used any in the last two weeks. On physical examination, his
respirations are 24/min but does not appear to be in distress. Lungs show bilateral expiratory
wheezes, with a prolonged expiratory phase. On cardiac examination, there is a regular S1 and
S2 at a rate of 78/min, and no murmurs are heard. An electrocardiogram shows normal sinus
rhythm with a non-deviated electrical axis. Which of the following is the most appropriate
medication at this time?
 Incorrect Answer ImageA.Cromolyn inhaler
 Incorrect Answer ImageB.Cromolyn nasal spray
 Correct Answer ImageC.Inhaled albuterol
 Incorrect Answer ImageD.Inhaled beclomethasone
 Incorrect Answer ImageE.Subcutaneous epinephrine injection

A 69-year-old man with hyperlipidemia and hypertension comes to the emergency department
because of chest pain. His chest pain began abruptly during the night and woke him from his
sleep. He describes the substernal pain as 10/10 in severity, "tearing" in nature, and radiating to
his back and left scapula. The pain is not relieved by rest, but is associated with profuse
diaphoresis and nausea. Which of the following is the most likely diagnosis?
 Correct Answer ImageA.Aortic dissection
 Incorrect Answer ImageB.Cardiac ischemia
 Incorrect Answer ImageC.Esophageal rupture
 Incorrect Answer ImageD.Myocardial infarction
 Incorrect Answer ImageE.Perforated duodenal ulcer

A 37-year-old man with a 7-year history of treatment-resistant schizophrenia is brought to the


emergency department after suddenly falling to the floor and shaking vigorously. After 10
minutes, the patient recovers with no memory of the incident and a severe headache. He reports
that he has no history of falling unexpectedly or having seizures, and knows of no one in his
family who does. He is currently taking clozapine for schizophrenia, trazodone for sleep,
ibuprofen or aspirin for headaches, and erythromycin for a recent respiratory tract infection.
Which of the following medications is most likely responsible for this incident?
 Incorrect Answer ImageA.Aspirin
 Correct Answer ImageB.Clozapine
 Incorrect Answer ImageC.Erythromycin
 Incorrect Answer ImageD.Ibuprofen
 Incorrect Answer ImageE.Trazodone

The following vignette applies to the next three items. 


A 28-year-old jogger comes to a sudden stop while running to avoid being hit by a teenager on
roller blades. In doing so, he falls and is unable to get up without pain. When the paramedics
arrive at the scene, they notice that he is in considerable pain, unable to walk, and bleeding from
a wound on the forehead just above the left eyebrow. The patient did not lose consciousness after
the fall. In the emergency department, he complains of headache and pain in the right knee, and
he vomits. His vital signs are: pulse 88/min, blood pressure 120/90 mm Hg, respirations 16/min,
and temperature 37°C (98.6°F). There is a 1-cm laceration over the left eyebrow. The pupils are
equal, round, and reactive to light and accommodation. The cranial nerves are grossly intact. The
deep tendon reflexes are normal, and Babinski sign is negative. Bruises are noted over both
palms, but no deformity is noted in the wrists. Movement of the wrists is normal and pain free.
The range of motion in the elbows and the shoulders are normal and pain free. The left foot,
ankle, knee, and hip joint are normal and the range of motion pain free. The right foot, ankle, and
hip joint are normal as well. The right knee is grossly and uniformly swollen, and while it is
examined, the patient states that at the time of coming to a sudden stop, he felt "something
popping in my knee, before it swelled up." Range of motion cannot be tested due to the pain and
swelling. The quadriceps expansion is intact, and straight leg raise cannot be attempted due to
the pain. Contraction of the quadriceps muscle is painful. Which of the following is the most
likely diagnosis? 
 Incorrect Answer ImageA.Acute inflammatory reaction
 Incorrect Answer ImageB.Dislocation of the patella
 Correct Answer ImageC.Hemarthrosis
 Incorrect Answer ImageD.Ruptured Baker cyst
 Incorrect Answer ImageE.Tear of the medial meniscus

A 28-year-old jogger comes to a sudden stop while running to avoid being hit by a teenager on
roller blades. In doing so, he falls and is unable to get up without pain. When the paramedics
arrive at the scene, they notice that he is in considerable pain, unable to walk, and bleeding from
a wound on the forehead just above the left eyebrow. The patient did not lose consciousness after
the fall. In the emergency department, he complains of headache and pain in the right knee, and
he vomits. His vital signs are: pulse 88/min, blood pressure 120/90 mm Hg, respirations 16/min,
and temperature 37°C (98.6°SF). There is a 1-cm laceration over the left eyebrow. The pupils are
equal, round, and reactive to light and accommodation. The cranial nerves are grossly intact. The
deep tendon reflexes are normal, and Babinski sign is negative. Bruises are noted over both
palms, but no deformity is noted in the wrists. Movement of the wrists is normal and pain free.
The range of motion in the elbows and the shoulders are normal and pain free. The left foot,
ankle, knee, and hip joint are normal and the range of motion pain free. The right foot, ankle, and
hip joint are normal as well. The right knee is grossly and uniformly swollen, and while it is
examined, the patient states that at the time of coming to a sudden stop, he felt "something
popping in my knee, before it swelled up." Range of motion cannot be tested due to the pain and
swelling. The quadriceps expansion is intact, and straight leg raise cannot be attempted due to
the pain. Contraction of the quadriceps muscle is painful. Arthroscopy is most likely to show
which of the following findings? 
 Correct Answer ImageA.Tear of the anterior cruciate ligament
 Incorrect Answer ImageB.Tear of the lateral meniscus
 Incorrect Answer ImageC.Tear of the medial meniscus
 Incorrect Answer ImageD.Tear of the posterior cruciate ligament
 Incorrect Answer ImageE.Tear of the tendon of the popliteus muscle

A 28-year-old jogger comes to a sudden stop while running to avoid being hit by a teenager on
roller blades. In doing so, he falls and is unable to get up without pain. When the paramedics
arrive at the scene, they notice that he is in considerable pain, unable to walk, and bleeding from
a wound on the forehead just above the left eyebrow. The patient did not lose consciousness after
the fall. In the emergency department, he complains of headache and pain in the right knee, and
he vomits. His vital signs are: pulse 88/min, blood pressure 120/90 mm Hg, respirations 16/min,
and temperature 37°C (98.6°F). There is a 1-cm laceration over the left eyebrow. The pupils are
equal, round, and reactive to light and accommodation. The cranial nerves are grossly intact. The
deep tendon reflexes are normal, and Babinski sign is negative. Bruises are noted over both
palms, but no deformity is noted in the wrists. Movement of the wrists is normal and pain free.
The range of motion in the elbows and the shoulders are normal and pain free. The left foot,
ankle, knee, and hip joint are normal and the range of motion pain free. The right foot, ankle, and
hip joint are normal as well. The right knee is grossly and uniformly swollen, and while it is
examined, the patient states that at the time of coming to a sudden stop, he felt "something
popping in my knee, before it swelled up." Range of motion cannot be tested due to the pain and
swelling. The quadriceps expansion is intact, and straight leg raise cannot be attempted due to
the pain. Contraction of the quadriceps muscle is painful. Which of the following is the most
appropriate treatment? 
 Incorrect Answer ImageA.Closed reduction
 Incorrect Answer ImageB.Leaving it alone
 Incorrect Answer ImageC.Manipulation
 Incorrect Answer ImageD.Surgical excision to prevent recurrence
 Correct Answer ImageE.Surgical repair

A 47-year-old woman comes to the emergency department because of stomach pain. She states
that she has "multiple conditions," including gastroesophageal reflux disease and erosive
gastritis. She also states that she has osteoarthritis, migraine headaches, chronic sinusitis, and
dysmenorrhea. She feels she may have temporal lobe epilepsy as she sometimes "shakes." She
hands over a folder containing 10 years of her laboratory studies. These, along with her current
physical examination and laboratory studies, are unremarkable. When the patient is told about
the current negative results she seems puzzled but relieved, she thanks the doctor and leaves for
home. Which of the following is the most likely diagnosis?
 Incorrect Answer ImageA.Delusional disorder, somatic type
 Incorrect Answer ImageB.Histrionic personality disorder
 Incorrect Answer ImageC.Illness anxiety disorder
 Incorrect Answer ImageD.Malingering
 Correct Answer ImageE.Somatic symptom disorder

A 47-year-old woman comes to the emergency department because of stomach pain. She states
that she has "multiple conditions," including gastroesophageal reflux disease and erosive
gastritis. She also states that she has osteoarthritis, migraine headaches, chronic sinusitis, and
dysmenorrhea. She feels she may have temporal lobe epilepsy as she sometimes "shakes." She
hands over a folder containing 10 years of her laboratory studies. These, along with her current
physical examination and laboratory studies, are unremarkable. When the patient is told about
the current negative results she seems puzzled but relieved, she thanks the doctor and leaves for
home. Which of the following is the most likely diagnosis?
 Incorrect Answer ImageA.Delusional disorder, somatic type
 Incorrect Answer ImageB.Histrionic personality disorder
 Incorrect Answer ImageC.Illness anxiety disorder
 Incorrect Answer ImageD.Malingering
 Correct Answer ImageE.Somatic symptom disorder

A 24-year-old woman is brought to the emergency department because of severe left-sided chest
pressure, shortness of breath, and palpitations for 30 minutes. She appears acutely short of
breath. Her blood pressure is 140/90 mm Hg, pulse is 104/min, and respirations are 26/min. Her
lungs are clear to auscultation. An arterial blood gas on room air shows a pH of 7.50, a PCO2 of
28 mm Hg, and a PO2 of 110 mm Hg. Which of the following represents this patient's acid/base
status? 
 Incorrect Answer ImageA.She has a metabolic acidosis and a respiratory alkalosis
 Incorrect Answer ImageB.She has a metabolic alkalosis and a respiratory acidosis
 Incorrect Answer ImageC.She has a metabolic alkalosis and a respiratory alkalosis
 Incorrect Answer ImageD.She has a metabolic alkalosis
 Correct Answer ImageE.She has a respiratory alkalosis

A 49-year-old African American woman with hypertension and hyperlipidemia is evaluated in


the emergency department with complaints of overwhelming episodes of severe pounding
headache, profuse sweating, and palpitations. The patient reports a sense of impending doom
during these episodes. She is tachycardic and hypertensive. Which of the following is the best
imaging test for diagnosing and localizing the cause of this patient's symptoms? 
 Incorrect Answer ImageA.Abdominal aortic angiography
 Incorrect Answer ImageB.CT scan
 Correct Answer ImageC.I-metaiodobenzylguanidine scan
 Incorrect Answer ImageD.Nephrotomography
 Incorrect Answer ImageE.Sonography

A 35-year-old blonde woman comes to the clinic for a routine physical examination. She has a
peeling sunburn and is active in several outdoor sports. Her physical examination is remarkable
for a 1 x 1 cm raised black-and-red-colored lesion on her right shoulder. The borders are
irregular, and she says that it has recently started to itch. Which of the following is the most
appropriate next step in management? 
 Incorrect Answer ImageA.Administration of 1% hydrocortisone cream
 Incorrect Answer ImageB.Shave biopsy
 Correct Answer ImageC.Wide excision biopsy
 Incorrect Answer ImageD.Followup in one month
 Incorrect Answer ImageE.No followup is required

A 5-year-old girl is brought to the physician by her parents with a fever and a rash of 3 days’
duration. She was healthy until a 5 days ago, when she had a flu-like viral illness with a sore
throat and headache. Two days later she started breaking out with a very itchy skin rash on the
face and trunk. She has been getting new breakouts of the rash daily, and the itch is disrupting
her sleep. Otherwise, she does not have any significant past medical history, and the only
medication her parents gave her was over-the-counter cold syrup that helped a little with the
fever and discomfort. She is in mild acute distress with a temperature of 38.1℃ (100.6℉) and a
pulse of 100/min. On her oral mucosa there is a vesicular exanthem, mainly on the palate. On the
face and trunk there are 30 to 40 vesicles, fresh and umbilicated pustules, and crusted papules,
most on a faint erythematous base. Many of the lesions are excoriated. Which of the following is
most likely to develop as a result of this current illness?
 Incorrect Answer ImageA.Erythema multiforme
 Correct Answer ImageB.Herpes zoster
 Incorrect Answer ImageC.Pneumonia
 Incorrect Answer ImageD.Recurrent orolabial herpes
 Incorrect Answer ImageE.Rheumatic fever

A 71-year-old Argentinian woman is brought to the hospital for evaluation of difficult to control
hypertension. She has been treated for hypertension for the past three years by her primary care
physician. Her medical history is notable for hyperlipidemia. She has a documented intolerance
to ACE-inhibitors manifested by a rapid decline in her renal function. She also has had two
episodes of acute pulmonary edema in the past. She has a 50-pack-year history of smoking but
quit 3 years ago after her diagnosis. She has also altered her diet to avoid high salt foods and
claims she is compliant with her medications. At her last office visit, labs were notable for a
mildly elevated creatinine of 1.2 mg/dL and urinalysis showed microscopic hematuria. Her
primary care physician admitted her to the hospital for further evaluation. Her physical
examination is remarkable for a blood pressure of 180/100 mm Hg, a prominent apical pulse, an
abdominal bruit to the right of midline, and 2+ dorsalis pedis and radial pulses. Laboratory
studies show: 
Hematocrit 38%
Leukocyte count 5,800/mm3
Sodium 144 mEq/L
Potassium 3.6 mEq/L
Bicarbonate 28 mEq/L
BUN 22 mg/dL
Creatinine 1.3 mg/dL
Which of the following is the most likely cause of her hypertension? 
 Incorrect Answer ImageA.Aldosterone secreting tumor
 Incorrect Answer ImageB.Coarctation of the aorta
 Incorrect Answer ImageC.Essential hypertension
 Incorrect Answer ImageD.Pheochromocytoma
 Correct Answer ImageE.Renal artery stenosis

A 72-year-old man is scheduled to undergo surgical repair of an umbilical hernia. He sustained a


myocardial infarction (MI) 8 years ago and had coronary angiography with PTCA to two
coronary lesions at that time. He has a history of congestive heart failure, but his latest
echocardiogram shows a well-compensated left ventricle with only mildly depressed systolic
function. He also has hypertension and hyperlipidemia, both of which have been well controlled
for the last 2 years. Today he notes mild, stable, general fatigue but is otherwise without
complaint. He is able to walk six blocks without stopping to rest, and his anginal pain is
unchanged in character. His medications are simvastatin, aspirin, lisinopril, furosemide twice
daily, and spironolactone. Physical examination reveals a well appearing elderly man with a
blood pressure of 124/73 mm Hg and a pulse of 92/min. Cardiac examination is unremarkable
and his jugular venous pulse is approximately 2 cm above the sternal angle. His lungs are clear to
auscultation, his liver is not enlarged, and there is no extremity edema. The umbilical hernia in
question is reducible, nontender, and has active bowel sounds, but is quite protuberant and
bothersome to the patient. Which of the following is the most appropriate preoperative
diagnostics and therapeutics for this patient? 
 Incorrect Answer ImageA.Continue current medical regimen, hold antihypertensive
medications on the day of surgery
 Incorrect Answer ImageB.Empiric perioperative beta-blockade with target heart rate of
50-60/min
 Correct Answer ImageC.Noninvasive stress cardiac imaging and coronary
revascularization if warranted, plus empiric perioperative beta-blockade with target heart rate of
50-60/min
 Incorrect Answer ImageD.Perioperative nitroglycerin infusion as needed to maintain
blood pressure at less than 120/80 mm Hg
 Incorrect Answer ImageE.Screening exercise EKG, perioperative beta-blockade only if
found to have ischemic changes

A 29-year-old woman comes to the physician because of severe headaches and burning pain in
her hands and feet. Originally she attributed the headaches to stress. The patient and her husband
are planning their first child and she is appropriately anxious about the changes this event will
bring to their relationship. The headaches have continued, however, and now are associated with
intermittent episodes of tinnitus. She has never had these symptoms in the past and reports being
healthy all of her life. She exercises regularly, does not drink or smoke tobacco, and does not
know of any diseases that run in the family. When asked specifically, she denies any recent
episodes of weakness, visual changes, altered mental status, bleeding, heavy menses, or easy
bruising. A complete review of systems fails to reveal any abnormalities. Physical examination is
unremarkable. She is a healthy young woman in no apparent distress and without focal
neurologic signs. Laboratory studies show: 
Hematologic
Hematocrit 39%
Leukocytes 9,000/mm3 (normal differential)
Platelets 992,000/mm3
Mean corpuscle volume 88 µm3
A review of the peripheral blood smear shows markedly increased numbers of platelets,
numerous giant platelets, and platelet clumps. The erythrocytes and neutrophils appear to have
normal morphology. Which of the following is the most appropriate treatment for this patient's
condition? 
 Incorrect Answer ImageA.Anagrelide therapy
 Correct Answer ImageB.Daily low-dose aspirin
 Incorrect Answer ImageC.Ferrous sulfate, oral
 Incorrect Answer ImageD.No treatment, follow clinically
 Incorrect Answer ImageE.Phlebotomy
A 46-year-old man comes to the clinic because of poor energy level, decreased libido, and
continuing loss of body hair over the past 3 years. He complains of "just feeling deflated," and
his wife has complained of his loss of interest in sex. He is unable to attain an erection sufficient
to have sexual intercourse, and he no longer has erections on awakening in the morning. He no
longer masturbates and has not had an orgasm in more than 1 year. His weight has not changed
and he denies heat or cold intolerance. He denies any headaches or changes in his vision.
Regarding his pubertal development, he started shaving at age 13 years, has always been a
normal height, and his voice began to change at approximately age 14 years. On physical
examination he is an obese man who seems lethargic. Extraocular movements are intact and
visual fields are full to confrontation. There is no thyromegaly or thyroid nodule. Lung, cardiac,
and abdominal examination are unremarkable. He has large, fatty breasts without palpable
mammary tissue or enlarged areolae. Genital examination shows a normal phallus and atrophic,
bilaterally descended testes. He has a male pattern escutcheon but overall sparse body hair.
Laboratory studies show:
Serum testosterone 46 ng/mL (normal 300-1,200
ng/mL)
Serum leuteinizing hormone 6 mU/mL (normal 5-15
mU/mL)
Serum follicle-stimulating 4 mU/mL (normal 3-15
hormone mU/mL)
Serum thyroid-stimulating 1.3 µU/mL (normal 0.5-5.0
hormone µU/mL)
Serum prolactin 12 ng/ML (normal <15 ng/mL)
An MRI of the brain is normal. Which of the following is the most likely cause of this patient's
hypogonadism?
 Incorrect Answer ImageA.Congenital deficiency of gonadotropin-releasing hormone
 Incorrect Answer ImageB.Excessive estrogen production by adipose tissue
 Correct Answer ImageC.Loss of gonadotropin-releasing hormone-producing neurons
from the hypothalamus
 Incorrect Answer ImageD.Pituitary adenoma suppressing secretion of luteinizing and
follicle-stimulating hormones
 Incorrect Answer ImageE.Primary testicular failure

A 3-week-old infant is brought to the emergency department because of fever and poor feeding.
The infant was born at full term via normal vaginal delivery and has been formula fed since
birth. He was well until early this morning, when he started to refuse the bottle. He was noticed
to be febrile and less active during the day. Upon examination the baby appears to be lethargic
with a temperature of 38.7 C (101.6 F). His extremities are warm and there is no respiratory
distress. The chest and abdominal examination is normal and there is no cardiac murmur. The
anterior fontanelle feels full and the infant does not cry when stimulated. A lumbar puncture (LP)
is performed. Cerebrospinal fluid examination shows: 
At this time, which of the following statements is most correct about his condition? 
 Incorrect Answer ImageA.A humoral immunodeficiency is present
 Correct Answer ImageB.This is an infection that was acquired in the perinatal period
 Incorrect Answer ImageC.His mother is a nasal carrier of the infecting organism
 Incorrect Answer ImageD.There is a spinal defect with communicating dermal sinus
 Incorrect Answer ImageE.Skin contamination during the LP is the source of the gram-
positive organisms in the cerebrospinal fluid

A 55-year-old man has a 3-cm mass in the lower part of his neck, just to the right of the midline.
The mass moves up and down with swallowing, and was first noticed a couple of weeks ago. The
patient has no clinical signs of thyroid dysfunction, and he has normal levels of thyroid-
stimulating hormone and T4. A fine-needle aspiration of the mass is read as definitively
malignant, but the pathologist is inclined toward a diagnosis of adenocarcinoma rather than one
of the common thyroid tumors. A thyroidectomy is performed, and when the fresh specimen is
sectioned, a bright yellow tumor is identified. The frozen section shows that it is composed of
clear cells. Which of the following is the most likely diagnosis? 
 Incorrect Answer ImageA.Anaplastic thyroid cancer
 Incorrect Answer ImageB.Benign follicular adenoma
 Incorrect Answer ImageC.Metastatic melanoma
 Correct Answer ImageD.Metastatic renal cell carcinoma
 Incorrect Answer ImageE.Sarcoma
A 17-year-old boy is brought to the emergency department by four police officers. They report
that he was found outside a concert and friends said he "was wacky on drugs, maybe coke." The
friends told the officers that the boy had attacked a stranger and said, "You know what he is; I
have to take him out." The friends dispersed after giving the previously stated information.
Police report that the victim of the attack was taken by ambulance to a nearby hospital and
appeared to have a broken arm. On the way to the hospital, the boy had been uncooperative and
assaultive and bit one of the officers. The boy is grunting and pacing and spits at the officers
when they try to redirect him. The nurse in triage does not attempt to take vital signs because he
is afraid. He warns the physician to be careful when approaching the patient. The patient is a
large, diaphoretic male with nystagmus, straining against his handcuffs. When he sees the
physician, he says "Come a little closer, Doc, I got a secret for you." The patient denies drug use
and then lets out an open-mouthed scream, "Devils, I will take you out," and lunges for the
nearest staff member. About this time the patient's mother calls the emergency department and
says, "If you tie him down or give him anything, I will sue every one of you." The mother says
she will arrive at the hospital in about 2 hours. Police wrestle the patient to the floor and report
that they cannot safely keep him face down on the floor, and he is therefore placed in physical
restraints. Which of the following is the most appropriate next step in management? 
 Incorrect Answer ImageA.Call for advice from the hospital's Risk Management
 Incorrect Answer ImageB.Call the courthouse to appoint the patient a lawyer
 Incorrect Answer ImageC.Call the patient's father to consent for emergency medication
 Correct Answer ImageD.Force an injection of a benzodiazepine
 Incorrect Answer ImageE.Promise to let the patient out of restraints if he takes
haloperidol
 Incorrect Answer ImageF.Release the patient from restraints until parental consent is
obtained

A 33-year-old woman, gravida 2, para 1, at 40 weeks' gestation is in the second stage of labor
and pushing. Her prenatal course was complicated by gestational diabetes detected by a 28-week
glucose-loading test. The patient has been treated with insulin. She has no other medical
problems and has never had surgery. As the fetus is being delivered, the head emerges and is
quickly retracted back into the perineum. Gentle traction on the head fails to result in delivery of
the anterior shoulder of the baby. The provider and the patient are the only two people in the
room. Which of the following is the most appropriate next step in management?
 Incorrect Answer ImageA.Apply fundal pressure
 Incorrect Answer ImageB.Apply suprapubic pressure
 Correct Answer ImageC.Call for help
 Incorrect Answer ImageD.Perform an episiotomy
 Incorrect Answer ImageE.Perform the Zavanelli maneuver

A 13-year-old boy is brought to the physician because of recent-onset enuresis. He has had these
accidents happen 4 or 5 times during both the day and at night over the previous 3 weeks. The
parents had also noticed that he was drinking unusually large amounts of water and his appetite
also increased. He has lost 5 pounds in the past 2 months despite this increased appetite. He had
been in good health previously. He is in 8th grade and is a good student, physically active, and
attending karate class 3 times a week. On physical examination, the patient is in no acute
distress. Vital signs are within normal limits. He is in the 50th percentile for height and the 40th
percentile for weight. Inspection of the mouth shows dry but otherwise normal-appearing
mucosa. Auscultation of the chest and palpation of the abdomen are unremarkable. Laboratory
studies show elevated glucose at 270 mg/dL (normal 74–106 mg/dL) and bicarbonate 21 mEq/L
(normal 20–29 mEq/L). Complete blood count is normal. Urine glucose is 4+ mmol/L and
ketones in the urine. A point-of-care HbA1c is 11.5 g/dL. Which of the following is the most
appropriate treatment approach?
 Incorrect Answer ImageA.Diet
 Incorrect Answer ImageB.Diet and exercise
 Incorrect Answer ImageC.Insulin
 Incorrect Answer ImageD.Insulin and exercise
 Correct Answer ImageE.Insulin, diet, and exercise

A 58-year-old woman is postoperative day 4 after a colon cancer resection. She develops sudden,
sharp, left-sided chest pain, which increases with inspiration. She is noted to be hypoxemic, and
an arterial blood gas shows a pH of 7.54, a pCO2 of 20 mm Hg, and a pO2 of 53 mm Hg. A
ventilation-perfusion scan is consistent with a massive left-sided pulmonary embolism to the left
mainstem pulmonary artery. Despite 100% oxygen administered via a tight-fitting face mask, she
is unable to raise her oxygen saturation above 90%. She is intubated, and positive end-expiratory
pressure (PEEP) is added. Two hours later, she suddenly becomes hypotensive, and her
oxygenation is further impaired. Breath sounds are absent in the right hemithorax, and there are
distended neck veins as well as tracheal deviation to the left. Which of the following would be
the most appropriate next step in management?
 Incorrect Answer ImageA.Initiate therapy with IV streptokinase
 Incorrect Answer ImageB.Obtain a repeat ventilation-perfusion scan
 Incorrect Answer ImageC.Obtain pulmonary angiography
 Correct Answer ImageD.Perform a needle thoracostomy in the right hemithorax
 Incorrect Answer ImageE.Begin broad-spectrum antibiotics
A 50-year-old man is brought to the emergency department because of lightheadedness. He has a
history of lung cancer, which was diagnosed a month ago and found to be widely metastatic to
the bone and pericardium. On physical examination, his blood pressure is 70/40 mm Hg and
pulse is 100/min. Heart sounds are distant and soft. ECG shows low voltage, and electrical
alternans is present. Chest radiograph shows that the cardiac silhouette has a "water bottle"
appearance. Which of the following is the most appropriate intervention?
 Incorrect Answer ImageA.Beta-blockers
 Incorrect Answer ImageB.Cardiac catheterization
 Incorrect Answer ImageC.Nonsteroidal anti-inflammatory drugs
 Correct Answer ImageD.Pericardiocentesis
 Incorrect Answer ImageE.Steroids

A 55-year-old man is currently hospitalized for evaluation of unstable angina. The on-call
physician is notified by the nurse that he has developed severe lower abdominal pain and
suprapubic pressure. The patient underwent left heart catheterization 12 hours ago, during which
he had a transient episode of hypotension and bradycardia that was successfully treated with
atropine and epinephrine. He has not urinated since the procedure. The nurse notes that he is
tachycardic, with a regular pulse of 110/min. A review of systems taken on admission shows
nocturia and a weak urinary stream. Based on this history, which of the following is the most
likely etiology of his abdominal pain?
 Incorrect Answer ImageA.Cholesterol emboli syndrome
 Incorrect Answer ImageB.Contrast-induced nephropathy
 Incorrect Answer ImageC.Nephrolithiasis
 Incorrect Answer ImageD.Pyelonephritis
 Correct Answer ImageE.Urinary retention

A 30-year-old man presents to his physician because of progressive muscle weakness for the past
3 days. He reports increased weakness in his legs while climbing the stairs. Two weeks ago, he
had an upper respiratory infection, which resolved within a few days without treatment. His vital
signs are as follows: temperature 37.5°C (99.5°C), pulse 80/min, respirations 28/min and blood
pressure 132/77 mm Hg. Oxygen saturation is 96% on room air. Physical examination shows
symmetric upper and lower extremity weakness. Muscle strength is 3/5 in the lower limbs and
4/5 in the upper limb. Deep tendon reflexes are absent in the lower limb and positive in the upper
limb. Sensation is intact in all extremities. Lumbar puncture shows the following CSF values:
Opening pressure Normal
Cells 5 cells/mm3 (mostly lymphocytes)
Protein 80 mg/dL
Glucose 50 mg/dL
Which of the following is the most likely diagnosis? 
 Incorrect Answer ImageA.Botulism
 Correct Answer ImageB.Guillain-Barrè syndrome
 Incorrect Answer ImageC.Myasthenia gravis
 Incorrect Answer ImageD.Poliomyelitis
 Incorrect Answer ImageE.Polymyositis
A 4-year-old girl with sickle cell disease is brought to the emergency department with a
temperature of 39.6 C (103.2 F). Other than irritability, the physical examination is
unremarkable. Laboratory evaluations reveal a white blood cell count of 18,200/mm3 (normal
5,000-10,000), with 88% polymorphonuclear neutrophils, 10% lymphocytes, and 2% monocytes,
and a hemoglobin of 7.6 g/dL (normal 10-15 in this age population). Two sets of blood cultures
are obtained and are sent to the laboratory. Which of the following is the most appropriate next
step in management? 
 Incorrect Answer ImageA.Observe the child pending blood culture results
 Incorrect Answer ImageB.Administer amoxicillin orally
 Incorrect Answer ImageC.Administer ceftazidime and gentamicin intravenously
 Correct Answer ImageD.Administer ceftriaxone (Rocephine) intravenously
 Incorrect Answer ImageE.Administer vancomycin and gentamicin intravenously

A 12-year boy is brought to the emergency department by his parents because of progressive
weakness for several days. He has otherwise been healthy and has met all developmental
milestones. His immunizations are up to date. He reports that he had an upper respiratory
infection 2 weeks ago. On physical examination, his vital signs are labile with a pulse range of
60-100/min and systolic blood pressure ranging from 80-120 mm Hg. He is alert and oriented.
Neurologic examination shows bilateral weakness in the upper and lower extremities. Deep
tendon reflexes are absent and sensation is intact. Which of the following is the most likely
diagnosis?
 Correct Answer ImageA.Guillain-Barrè syndrome
 Incorrect Answer ImageB.Myasthenia gravis
 Incorrect Answer ImageC.Polymyositis
 Incorrect Answer ImageD.Transverse myelitis
 Incorrect Answer ImageE.Viral encephalitis

A 60-year-old man comes to the physician because of a reddish, scaling lesion on his lower lip
that has been growing for the past year. It is painless, and he is here now because his wife made
him come. He has not seen a physician for the past 4 years. He works as a farmer and is exposed
to the sun. A photograph of the lesion is shown. Which of the following is the most appropriate
next step in management? 
 Incorrect Answer ImageA.Acyclovir
 Incorrect Answer ImageB.Bacitracin ointment
 Incorrect Answer ImageC.Photodynamic therapy
 Incorrect Answer ImageD.Retinoid therapy
 Correct Answer ImageE.Schedule a biopsy of the lesion

A 34-year-old bus driver comes to the physician with complaints of difficulty swallowing. The
symptoms began gradually, approximately 9 months ago, and have prevented him from chewing
solids or drinking liquids comfortably. He finds himself awakened at night with cough and
occasional morning regurgitation of recognizable food from the night before. He has learned to
reduce his oral intake and has lost 6 pounds over the past 2 months. He does not smoke or drink
and has no family history of esophageal or other gastrointestinal illnesses. His physical
examination is unremarkable. Which of the following is the most likely diagnosis? 
 Correct Answer ImageA.Achalasia
 Incorrect Answer ImageB.Esophageal adenocarcinoma
 Incorrect Answer ImageC.Lower esophageal web
 Incorrect Answer ImageD.Peptic stricture
 Incorrect Answer ImageE.Progressive systemic sclerosis

A 14-year-old girl is brought to the physician by her parents because of a multitude of symptoms
that have developed over the previous several months. They first noticed that she had become
nervous and hyperactive, could not sit in one place for longer than a few minutes, and would
burst into tears at every little thing that she disliked. They thought she was going through some
"phase" and initially ignored it, but then the teacher called them to discuss her school
performance. She was always an excellent student, but her grades had significantly dropped
lately. She cannot seem to concentrate in class and her handwriting has become poor. In addition,
she has trouble sleeping and keeps getting out of bed to eat all night. Despite that, she seems to
have significantly lost weight and they wonder if she may be doing something they are not aware
of. She dropped out of her gymnastics class because it had become too exhausting for her. Her
medical history is unremarkable and she takes no medication. On physical examination, the
patient seems to be uncomfortable and jittery. She keeps shifting her weight on the chair and
complaining about how hot it is in the office. Her temperature is 36.7°C (98.0°F), pulse is
120/min, respirations are 18/min, and blood pressure is 135/65 mm Hg. Inspection of her scalp
shows diffuse thinning and brittle hair. Her skin is smooth and clammy to the touch, whereas her
palms are sweaty. Extension of both arms shows a moderately severe tremor. Palpation of the
neck confirms the presence of a diffuse, nontender, symmetric enlargement of the thyroid gland.
Auscultation of the heart is remarkable for tachycardia. Results of serum hormone levels are
shown below:
Thyroxine, total (T4)  25 µg/dL
Thyroxine, free 40 ng/dL
Thyroxine, total (T3)  320 ng/dL
Thyroid-stimulating hormone 0.05 µg/dL
Which of the following is the most appropriate pharmacotherapy? 
 Incorrect Answer ImageA.Iodide
 Incorrect Answer ImageB.Propranolol
 Correct Answer ImageC.Propylthiouracil
 Incorrect Answer ImageD.Radioablation with iodine
 Incorrect Answer ImageE.Surgery

A 42-year-old woman, gravida 2, para 1, at 39 weeks' gestation is brought to the operating room
for an elective repeat cesarean delivery and a tubal ligation. Her prenatal course was
uncomplicated. She has no medical problems. She had a cesarean delivery 4 years ago for a
breech fetus and she did not wish to attempt vaginal birth after cesarean (VBAC). After the
delivery of a healthy fetus, attempts are made to deliver the placenta. The placenta is delivered
fragmented with parts of the placenta still densely adherent to the uterus. The patient is bleeding
heavily from these adherent placental portions. Which of the following is the most appropriate
next step in management? 
 Incorrect Answer ImageA.Administer methylergonovine
 Incorrect Answer ImageB.Administer prostaglandin-F-2-alpha
 Correct Answer ImageC.Perform a hysterectomy
 Incorrect Answer ImageD.Perform a tubal ligation
 Incorrect Answer ImageE.Perform uterine artery embolization

A 3-year-old boy has history of nasal congestion and greenish, foul-smelling drainage. His
mother reports that this has been going on for approximately 3 to 4 weeks. Before that, he has
had occasional upper respiratory tract symptoms, such as stuffy nose and discharge, but no other
similar episodes. Further questioning reveals that the nasal drainage is primarily from the left
side. There is no associated bleeding. The child is otherwise healthy, without significant other
past medical history. On physical examination there is obvious nasal congestion and mouth
breathing. Examination of the anterior nasal cavity reveals purulent rhinorrhea on the left. There
is some congestion on the right, but no drainage. The remainder of the head and neck
examination is within normal limits. Which of the following is the most appropriate next step in
management? 
 Incorrect Answer ImageA.Order a CT scan of the sinuses to rule out sinusitis
 Incorrect Answer ImageB.Order plain radiographs of the sinuses to rule out sinusitis
 Incorrect Answer ImageC.Order a sweat chloride test to rule out cystic fibrosis
 Correct Answer ImageD.Perform an endoscopic examination for possible nasal foreign
body
 Incorrect Answer ImageE.Prescribe amoxicillin for 10 days for possible sinusitis

A 76-year-old man with diabetes, on dialysis for chronic renal failure, has had two right lower
extremity arterial bypasses in the past 6 months, trying to salvage his limb from advanced
peripheral vascular disease. The procedures fail, and dry gangrene of the foot develops. Surgical
amputation is required. Pertinent findings before this planned procedure include: a history of
three myocardial infarctions, the last one 9 months ago, with an ejection fraction (EF) of 21%; a
serum cholesterol of 240 mg/dL; fasting blood sugars fluctuating between 135 and 198 mg/dL;
hemoglobin of 5 mmol/L; and serum sodium concentration of 142 mEq/L. To minimize the
operative risk, which of the following is the most appropriate management?
 Incorrect Answer ImageA.Keep him on a low sodium diet until the time of surgery
 Incorrect Answer ImageB.Lower his cholesterol to less than 200 mg/dL
 Incorrect Answer ImageC.Perform the procedure with blood sugars maintained at 95
-120mg/dL
 Correct Answer ImageD.Transfusion to a hemoglobin of 7-8 mmol/L
 Incorrect Answer ImageE.Wait to do his surgery until he is 12 months past the last infarct

An 11-year old girl is brought to the physician for a well-child examination. The mother states
that her daughter has gained an excessive amount of weight in the last year. Although her school
performance has remained average, she feels tired and no longer participates in sporting
activities. Her eating habits have not changed and there appears to be no excess of high-calorie
food items. Her weight is at the 95th percentile, while her height is at the 10th to 25th percentile.
A growth chart shows that her growth has slowed considerably. Two years ago her height was in
the 50th to 75th percentile. Examination shows a slightly obese girl with dry skin. A firm,
lobular goiter is present. No lymphadenopathy is identified and there is no exophthalmia. The
rest of the examination is unremarkable and there are no secondary sexual characteristics. Which
of the following is the most likely explanation for her condition?
 Incorrect Answer ImageA.Grave disease
 Correct Answer ImageB.Hashimoto thyroiditis
 Incorrect Answer ImageC.Pituitary tumor
 Incorrect Answer ImageD.Puberty goiter
 Incorrect Answer ImageE.Thyroid neoplasm

An 82-year-old woman comes to the physician for a routine visit. She feels generally well but
states that her exercise tolerance has diminished lately. She can no longer walk briskly to church
each morning, but rather must stop to rest multiple times along the way. She denies any bleeding
or recent trauma, and her diet and appetite have been normal. She does not smoke and has a glass
of wine with dinner most nights. Her weight is stable. She has a past medical history of coronary
artery disease, diabetes mellitus type 2, gastroesophageal reflux disease, urinary incontinence,
and peripheral vascular disease. Her current medications include aspirin, atenolol, simvastatin,
metformin, and lansoprazole. Physical examination shows an elderly but well developed woman
who is pale and listless. Her peripheral pulses are all diminished and her extremities are cool to
palpation. Sensation is decreased in the distal soles of her feet bilaterally, but this has been
relatively stable over the past 2 years. A mini mental status examination score is 28 out of 30.
Laboratory studies show: 
Leukocyte count 2,900/mm3
Hemoglobin 9.2 g/dL
Hematocrit 29%
Platelets 95,000/mm3
MCV 108.2
RDW 15.3
Examination of the peripheral blood smear shows numerous neutrophils with Barr bodies and
others with bilobed nuclei. Giant platelets also can be seen. Further laboratory studies show: 
Serum total bilirubin 0.8 (0.1–1.0
mg/dL mg/dL)
Serum alanine aminotransferase 22 U/L (5–35 IU/L)
Serum aspartate aminotransferase 30 U/L (5–40 IU/L)
Serum gamma-glutamyl 27 U/L (8–38 U/L)
transpeptidase (GGT)
Over the long term, which of the following is the most likely course of this patient's condition? 
 Incorrect Answer ImageA.Improve with vitamin B12 and folate supplementation
 Incorrect Answer ImageB.Lead to irreversible neurologic deficits
 Correct Answer ImageC.Progress to a life-threatening leukemia
 Incorrect Answer ImageD.Resolve when she abstains from alcohol
 Incorrect Answer ImageE.Resolve with removal of one or more of her medications

A 45-year-old man presents to his physician because of swelling and pain in both knees. He
denies recent trauma or rash. He has no fever, chills, or shakes. He denies drug use or prolonged
immobilization. He is physically active. He has a history of diabetes mellitus and hypertension.
His medications include glyburide 2.5 mg daily and metoprolol 25 mg twice daily. He has no
allergies. Vital signs are normal. Physical examination of the heart, lungs, abdomen, and
extremities is normal. He has bilateral knee swelling with pain on movement without joint line
tenderness. Arthrocentesis shows irregular rhomboid crystals. These crystals exhibit weakly
positive birefringence under polarized light. Which of the following is the most likely diagnosis?
 Correct Answer ImageA.Calcium pyrophosphate deposition disease
 Incorrect Answer ImageB.Gout
 Incorrect Answer ImageC.Osteoarthritis
 Incorrect Answer ImageD.Rheumatoid arthritis
 Incorrect Answer ImageE.Septic arthritis

A 32-year-old man returns to the clinic after the successful excision of a large protruding mass
from his left mandible. Pathology reported that the surgical margins were clean and
histopathology was consistent with an osteoma. The patient is concerned about his risk for
recurrent tumors. He currently feels well, exercises regularly, and does not smoke or consume
excessive amounts of alcohol. His past medical history is remarkable for supernumerary teeth
and multiple dental caries. He sees a dermatologist for treatment of acne and sebaceous and
epidermoid cysts. Family history includes a father and grandfather who died of colon cancer in
their forties and a grandmother who died of breast cancer at age 68 years. Head and neck
examination shows a well-healed surgical wound. Funduscopic examination shows multiple
bilateral, small, pigmented, ovoid lesions. Visual acuity is 20/25 OD and 20/20 OS. There are
multiple generalized lipomas. Which of the following is the most appropriate intervention to
reduce this patient's risk for future malignancy?
 Incorrect Answer ImageA.Colon cancer screening starting early, at age 40 years
 Correct Answer ImageB.Colonoscopy now, followed by early colectomy
 Incorrect Answer ImageC.Regular ophthalmologic and funduscopic examination
 Incorrect Answer ImageD.Routine skull, mandible, and jaw radiographs
 Incorrect Answer ImageE.Upper endoscopy with small bowel followthrough

The following vignette is the same for the next two items.
A 13-year-old boy is apprehended after beating and robbing an elderly woman. While in police
custody, he says that he is suicidal and is brought to the emergency department. The patient lives
with his mother and younger brother. The patient's father, from whom he is estranged, is
physically abusive and shows evidence of alcohol use disorder. The patient's mother is called and
she states that the patient has had a history of behavioral and legal problems since the age of 9.
He has many arrests for crimes such as shoplifting, vandalism, and trespassing. Earlier this week,
the patient was expelled from school after severely beating another student. On mental status
examination, the patient has a bright affect, is conversant, and is actually quite charming. Which
of the following is the most likely diagnosis?
 Incorrect Answer ImageA.Attention deficit hyperactivity disorder (ADHD)
 Correct Answer ImageB.Conduct disorder
 Incorrect Answer ImageC.Major depressive disorder
 Incorrect Answer ImageD.Oppositional defiant disorder
 Incorrect Answer ImageE.Tourette disorder

A 22-year-old primigravid woman is admitted to the hospital in active labor. She does not report
having any associated vaginal discharge, bleeding, or leakage of fluid. She has had no prenatal
care. Based on the date of her last menstrual period, gestational age is around 31 weeks. Her vital
signs are within normal limits. Abdominal examination shows a fundal height of 29 cm. She is
having regular contractions every two to three minutes that last almost 40-50 seconds each. Her
membranes are intact and her cervix is dilated to 6 cm. A soft fetal part is palpated at the cervical
os, suggestive of breech presentation. Blood is drawn for routine prenatal laboratory studies and
swabs for vulvovaginal cultures are taken. Which of the following is the next best step in her
management?
 Incorrect Answer ImageA.Allow vaginal delivery
 Incorrect Answer ImageB.Begin tocolytic therapy
 Correct Answer ImageC.Obtain an obstetrical ultrasound examination
 Incorrect Answer ImageD.Perform an external cephalic version
 Incorrect Answer ImageE.Schedule an emergency cesarean delivery

A 55-year-old man comes to the physician because of discomfort and heaviness in his groin for
the past month. He reports that the discomfort is intermittent and typically occurs during
defecation. He has had no associated nausea, vomiting, diarrhea, dysuria or hematuria, and he
has no history of trauma. His past surgical history is significant for an open cholecystectomy
performed 20 years ago. He smokes 3 packs of cigarettes per day and has had a productive cough
with yellow sputum for the past year. On physical examination, his temperature is 37°C (98.6°F),
pulse is 78/min, respirations are 18/min, and blood pressure is 130/90 mm Hg. Scattered
bibasilar rales are audible on chest auscultation. A protruding mass is identified in the right groin
region lateral to the pubic tubercle which becomes prominent when the patient is standing and
disappears when he lies supine. Which of the following is the most likely diagnosis?
 Incorrect Answer ImageA.Femoral hernia
 Incorrect Answer ImageB.Incisional hernia
 Correct Answer ImageC.Inguinal hernia
 Incorrect Answer ImageD.Testicular torsion
 Incorrect Answer ImageE.Umbilical hernia

While being interviewed for admission to the Marine Corps by a sergeant, an 18-year-old
applicant seems guarded and not forthcoming. The sergeant flags the applicant's record for
neuropsychological testing to determine if there is reason to believe that the man has a
psychiatric illness, particularly, a psychotic disorder. Which of the following psychological tests
would be most effective in identifying a psychotic disorder in this applicant?
 Incorrect Answer ImageA.Bender Gestalt
 Correct Answer ImageB.Rorschach
 Incorrect Answer ImageC.Structured Clinical Interview for DSM-5 Dissociative
Disorders (SCID-D)
 Incorrect Answer ImageD.Stanford-Binet
 Incorrect Answer ImageE.Wechsler Adult Intelligence Scale (WAIS)

While skiing, a 32-year-old man goes over a 4-foot drop-off and lands on his right knee. He is in
significant pain and is unable to get up. Emergency medical services immobilize his right lower
limb in a back-splint and transport him to the nearest emergency department via helicopter. In
the emergency department, his temperature is 37.0°C (98.6°F), pulse is 88/min, respirations are
20/min, and blood pressure is 140/90 mm Hg. His right knee is grossly swollen, and movements
are not possible without eliciting pain. No shortening of the extremity is noted. The left knee is
normal. The rest of the physical examination is unremarkable. Hemarthrosis of the right knee is
suspected. Which of the following is the most appropriate examination maneuver to confirm this
diagnosis?
 Incorrect Answer ImageA.Displacing the patella anteriorly
 Incorrect Answer ImageB.Displacing the patella laterally
 Incorrect Answer ImageC.Displacing the patella medially
 Correct Answer ImageD.Displacing the patella posteriorly
 Incorrect Answer ImageE.Testing the knee jerk reflex

A 34-year-old woman is struck while trying to cross the street on her way to work. She is
catapulted 10 feet into the air before hitting the ground. She does not lose consciousness during
this entire event. When the paramedics arrive, she reports left-sided pain. She is placed into a
cervical collar and brought to the nearby trauma center. At the trauma center, her pulse is
88/min, and blood pressure is 118/71 mm Hg. Oxygen saturation is 98% on 2 L of nasal cannula
supplemental oxygen. There is a small laceration on the left cheek, but otherwise no facial
trauma. Heart sounds are regular, but there are diminished breath sounds over the left lung field.
Her abdomen is nondistended, but there is an abrasion over the left side with tenderness in the
left upper quadrant; however, no peritoneal signs are present. Lower extremity examination
shows a left ankle deformity. A complete workup with radiologic imaging is performed. Injuries
include a left pneumothorax, small splenic laceration without active bleeding, and left ankle
fracture. Which of the following is the most appropriate management of her splenic injury?
 Incorrect Answer ImageA.Angiographic embolization
 Incorrect Answer ImageB.Emergent splenectomy
 Incorrect Answer ImageC.Give immunization for encapsulated organisms
 Correct Answer ImageD.Observation with serial abdominal examinations and complete
blood counts
 Incorrect Answer ImageE.Perform diagnostic peritoneal lavage

The following vignette applies to the next two items.


A 21-year-old student comes to emergency department with severe generalized weakness, nausea
and vomiting. The patient says that in the last several days he has increased thirst despite
increased fluid intake. He also reports increased urination and 10-pound weight loss. Prior to this
episode the patient did not have any significant medical problems. He does not smoke or drink
alcohol. His vitals are temperature 36.8°C (98.2°F), pulse 108/min, blood pressure 100/65 mm
Hg, respiration 22/min, weight 63 kg, and height 175 cm. Physical examination reveals a well-
developed young man in mild distress. Oral mucosa is dry. The patient has fruity odor and
breathes deeply. Breath sounds are clear bilaterally. Heart sounds are normal with no murmurs or
gallops. Abdomen is soft, slightly tender on palpation with normoactive bowel sounds.
Neurological examination is unremarkable. Laboratory tests reveal:
Sodium 131 mmol/L
Potassium 3.3 mmol/L
Chloride 88 mmol/L
Carbon dioxide 12 mmol/L
Blood urea nitrogen 22 mg/dL
Creatinine 1.3 mg/dL
Glucose 760 mg/dL
Calcium 9.2 mg/mL
Urine is positive for ketones
Arterial blood gases revealed pH 7.15, pO2- 91 mm Hg, pCO2- 14 mm Hg and HCO3- 17 mm Hg.
Diabetic ketoacidosis is the initial working diagnosis. Which of the following is the initial step in
the management of this patient? 
 Correct Answer ImageA.Intravenous fluids
 Incorrect Answer ImageB.Intravenous insulin
 Incorrect Answer ImageC.Intravenous potassium
 Incorrect Answer ImageD.Intravenous sodium bicarbonate
 Incorrect Answer ImageE.Subcutaneous insulin

A 21-year-old student comes to emergency department with severe generalized weakness, nausea
and vomiting. The patient says that in the last several days he has increased thirst despite
increased fluid intake. He also reports increased urination and 10-pound weight loss. Prior to this
episode the patient did not have any significant medical problems. He does not smoke or drink
alcohol. His vitals are temperature 36.8°C, pulse 108/min, blood pressure 100/65 mm Hg,
respiration 22/min, weight 63 kg, and height 175 cm. Physical examination reveals a well-
developed young man in mild distress. Oral mucosa is dry. The patient has fruity odor and
breathes deeply. Breath sounds are clear bilaterally. Heart sounds are normal with no murmurs or
gallops. Abdomen is soft, slightly tender on palpation with normoactive bowel sounds.
Neurological examination is unremarkable. Laboratory tests reveal:
Sodium 131 mmol/L
Potassium 3.3 mmol/L
Chloride 88 mmol/L
Carbon dioxide 12 mmol/L
Blood urea nitrogen 22 mg/dL
Creatinine 1.3 mg/dL
Glucose 760 mg/dL
Calcium 9.2 mg/mL
Urine is positive for ketones
Arterial blood gases revealed pH 7.15, pO2- 91 mm Hg, pCO2- 14 mm Hg and HCO3- 17 mm Hg.
Diabetic ketoacidosis is the initial working diagnosis. The appropriate treatment is initiated and
the patient is transferred to the intensive care unit for further management. On the second day it
the patient's condition appears to be improving, however his ketone level starts rising. Which of
the following is the most appropriate next step in management? 
 Correct Answer ImageA.Do nothing
 Incorrect Answer ImageB.Increase dose of insulin
 Incorrect Answer ImageC.Increase rate of intravenous fluids administration
 Incorrect Answer ImageD.Intravenous sodium bicarbonate
 Incorrect Answer ImageE.Re-check ketone level to rule out possible lab error

The following vignette applies to the next two items. 


A 9-year-old boy with no significant past medical history is brought to the clinic by his mother
because of "behavior problems." They have a note from his teacher complaining of the child's
immaturity and disruptiveness. Specifically, the note states that he suddenly blurts out
obscenities several times during the school day. Additionally, he shakes his desk and jerks his
head, which is disruptive to other students. The mother has noticed the same types of behavior
but does not feel that it is intentional. Which of the following is the most likely diagnosis? 
 Incorrect Answer ImageA.Attention deficit hyperactivity disorder (ADHD)
 Incorrect Answer ImageB.Conduct disorder
 Incorrect Answer ImageC.Learning disorder not otherwise specified
 Incorrect Answer ImageD.Schizophrenia
 Correct Answer ImageE.Tourette disorder

A 9-year-old boy with no significant past medical history is brought to the clinic by his mother
because of "behavior problems." They have a note from his teacher complaining of the child's
immaturity and disruptiveness. Specifically, the note states that he suddenly blurts out
obscenities several times during the school day. Additionally, he shakes his desk and jerks his
head, which is disruptive to other students. The mother has noticed the same types of behavior
but does not feel that it is intentional. This patient most likely has some symptoms that are
included in the criteria for which of the following disorders? 
 Incorrect Answer ImageA.Conduct disorder
 Incorrect Answer ImageB.Mental retardation
 Correct Answer ImageC.Obsessive-compulsive disorder
 Incorrect Answer ImageD.Posttraumatic stress disorder from sexual abuse
 Incorrect Answer ImageE.Schizophrenia, paranoid type

An international group of physicians is volunteering to provide care for the medically


underserved countries in Africa. A group is examining preschool children in a small
impoverished village 50 miles away from the closest urban settlement. The first child examined
is a 4-year-old girl. She is apathetic and lethargic, and cries the moment anybody attempts to
examine her. She is below the 5th percentile for both height and weight and has a protruding
abdomen. Her arms and legs are very thin and there is moderate pedal edema. Her eyes are
sunken and her cheeks have lost most of the buccal fat. She has an eczematous rash around her
mouth and on her extremities. Her hair is sparse and demonstrates the flag sign-alternating bands
with and without pigment along the length of the hair shaft. Which of the following nutritional
deficiencies is the most likely cause of this clinical presentation?
 Incorrect Answer ImageA.Niacin
 Correct Answer ImageB.Protein
 Incorrect Answer ImageC.Total caloric intake
 Incorrect Answer ImageD.Vitamin A
 Incorrect Answer ImageE.Vitamin B2
 Incorrect Answer ImageF.Vitamin K

A 59-year-old woman is admitted to the hospital for operative arthroscopy of the left knee. She
has hypertension for 10 years and a total abdominal hysterectomy with bilateral salpingo-
oophorectomy 15 years ago for uterine fibroids and ovarian cysts. Physical examination is
normal. Routine laboratory studies show: 
Sodium 140 mEq/L
Potassium 4.0 mEq/L
Chloride 105 mEq/L
BUN 20 mg/dL
Creatinine 0.8 mg/dL
Calcium 11.8 mg/dL
 Correct Answer ImageA.Adenoma of the parathyroid gland
 Incorrect Answer ImageB.Carcinoma of the colon
 Incorrect Answer ImageC.Carcinoma of the parathyroid gland
 Incorrect Answer ImageD.Hypoparathyroidism
 Incorrect Answer ImageE.Multiple myeloma

A 54-year-old man with a history of hypertension comes to his primary care physician for a
routine health maintenance examination. Physical examination shows that the patient has a
goiter. The rest of the physical examination is unremarkable. A number of laboratory studies are
performed, including thyroid function tests, and all the results are normal. Which of the
following is the most appropriate management for his diffuse goiter?
 Correct Answer ImageA.Observation
 Incorrect Answer ImageB.Propylthiouracil
 Incorrect Answer ImageC.Radio-iodine therapy
 Incorrect Answer ImageD.Suppressive doses of thyroid hormone (thyroxine)
 Incorrect Answer ImageE.Subtotal thyroidectomy

A 71-year-old man has had recurring headaches throughout the day for the past week. He has
chronic stable exertional angina, which is well controlled with isosorbide dinitrate and
propranolol. He had a pneumococcal vaccination five years ago and received an annual influenza
vaccination. He is a retired carpenter, he is married, and has three children. His blood pressure is
184/102 mm Hg in both arms and pulse is 86/min. Funduscopic examination is normal. Carotid
artery upstrokes are brisk and symmetric and a left carotid artery bruit is present. Lungs are clear
and cardiac examination shows a regular heart rhythm without murmurs, rubs, or gallops.
Abdominal examination shows no hepatosplenomegaly, masses, or bruits. On rectal examination,
he has guaiac-negative stool and his prostate is mildly and symmetrically enlarged. There is no
lower extremity edema. For evaluation of his hypertension, he undergoes a nuclear perfusion
scan which shows diminished uptake in the left kidney. Further laboratory examination would
most likely show which of the following? 
 Correct Answer ImageA.High renin, high angiotensin II, and high aldosterone
 Incorrect Answer ImageB.High renin, high angiotensin II, and low aldosterone
 Incorrect Answer ImageC.Low renin, high angiotensin II, and low aldosterone
 Incorrect Answer ImageD.Low renin, low angiotensin II, and low aldosterone
 Incorrect Answer ImageE.Low renin, low angiotensin II, and high aldosterone

A 6-year-old boy is brought to the physician because of a 3-day history of productive cough,
fever, and chills. The coughing makes it difficult to catch his breath. His mother is concerned
because this is the third infection the child has had this year. Although always a poor eater, he
has been particularly fussy with the onset of this illness and has lost several pounds. Several
episodes of foul-smelling diarrhea have also occurred. Past medical history is significant for
recurrent respiratory infections. The child denies ear pain, pleuritic chest pain, nausea, and
vomiting. Examination shows a thin, young male who is alert and oriented. His temperature is
38.1°C (101.6°F) and respirations are 34/min. Physical examination shows clubbing of the digits,
bilateral nasal polyps, increased anteroposterior chest diameter, hyperresonance to percussion,
and apical crackles. Copious amounts of thick sputum are coughed up by the patient. The rest of
the physical examination is unremarkable. A chest x-ray is obtained, which is shown above.
Which of the following is the most likely causative organism of this patient's infection?
 Incorrect Answer ImageA. Haemophilus influenzae
 Incorrect Answer ImageB. Mycoplasma pneumoniae
 Correct Answer ImageC. Pseudomonas aeruginosa
 Incorrect Answer ImageD.Respiratory syncytial virus
 Incorrect Answer ImageE.  Streptococcus pneumonia

A 25-year-old medical student experiences fatigue and weight loss in his first year of medical
school and becomes convinced that he has leukemia. He comes to the student health center for
the third time asking for "confirmatory testing" and requests that the physician discuss the matter
with his regular internist in his hometown. Physical examination, serum chemistries, and
complete blood count with differential performed at his initial visit 1 month ago showed no
abnormalities. A review of systems is otherwise negative. The student has no paranoid ideations,
ideas of reference, or perceptual disturbances. The student's internist at home reveals that a
complete physical examination and laboratory studies that he conducted 6 months ago because of
similar concerns indicate no evidence of any physical illness. Which of the following is the most
likely diagnosis at this time?
 Incorrect Answer ImageA.Body dysmorphic disorder
 Incorrect Answer ImageB.Conversion disorder
 Correct Answer ImageC.Illness anxiety disorder
 Incorrect Answer ImageD.Malingering
 Incorrect Answer ImageE.Somatic symptom disorder

A 69-year-old man is brought to a psychiatrist by his daughter because he has been acting
"strange." The patient tells the doctor that he has been "depressed." The daughter tells the doctor
that the patient has reported “a bad smell” in his apartment but that she and others could not
detect a bad odor there. She also notes that the patient has called her from places that he “did not
know how he got to.” He occasionally describes "out-of-body experiences," during which he
feels he can observe himself from outside. His daughter was especially concerned when she saw
him rubbing his stomach and protruding his tongue repetitively. The psychiatrist suspects a brain
lesion. A lesion in which of the following locations is most likely to be responsible for these
symptoms?
 Incorrect Answer ImageA.Cerebellum
 Incorrect Answer ImageB.Frontal lobe
 Incorrect Answer ImageC.Occipital lobe
 Incorrect Answer ImageD.Parietal lobe
 Correct Answer ImageE.Temporal lobe

A 17-year-old boy is brought to the hospital by ambulance following a motor vehicle collision.
He was a restrained front-seat passenger. The patient smells of alcohol and he is combative and
confused. He has multiple facial lacerations and a fractured nose. Copious amounts of bloody
fluid are draining from his nostrils. Fluid from his nose dripping onto the bed sheet shows a
“halo” pattern: there is a central circle of dark blood, surrounded by a concentric circle of lighter
pink fluid. Which of the following is the most appropriate next diagnostic study?
 Correct Answer ImageA.CT scan of the head and neck
 Incorrect Answer ImageB.Fiberoptic examination of the nose and nasopharynx
 Incorrect Answer ImageC.Magnetic resonance cisternogram
 Incorrect Answer ImageD.Sinus x-rays
 Incorrect Answer ImageE.Skull x-rays

A 42-year-old man with a long history of HIV/AIDS comes to the clinic because of weakness
and confusion. For the last 2 to 3 weeks he reports "feeling out of it," having difficulty paying
attention or even concentrating on previously simple to perform tasks. Most concerning,
however, is the patient's weakness. He reports progressive weakness and a sense of heaviness
over the left side of his body. The patient is considering hospice care, as his CD4 count has
diminished over the last year and, despite multiple courses of triple drug regimens, his viral load
has increased. However, he is currently willing to undergo diagnostic tests if a treatment is
available. Examination shows a thin, cachectic appearing man with a left hemiparesis. An MRI
shows multifocal superficial, subcortical white-matter lesions located near the grey-white matter
junction. There is no mass effect and, following gadolinium administration, none of the lesions
enhance. Which of the following is the most appropriate management?
 Incorrect Answer ImageA.Administer comfort care treatment only
 Incorrect Answer ImageB.Dexamethasone and mannitol administration
 Incorrect Answer ImageC.Empiric treatment trial for toxoplasmosis
 Correct Answer ImageD.Lumbar puncture with cerebrospinal fluid (CSF) PCR analysis
 Incorrect Answer ImageE.Stereotactic brain biopsy of a lesion

A 16-year-old boy comes to the clinic for a routine new patient visit. When discussing his past
medical history, he says that, as a child, he was diagnosed with Peutz-Jeghers syndrome.
Additionally, as a young boy, he had a testicular mass removed that was found to be a Sertoli cell
tumor. Since then, he has had no problems and has figured that he is cured. As such, he has not
had routine follow up, and only now comes to see a physician as a college requirement for a
recent physical examination. A full review of systems fails to reveal any abnormalities. Which of
the following is an expected finding given this patient's condition? 
 Incorrect Answer ImageA.Adenomatous polyps in the colon and small intestine
 Incorrect Answer ImageB.Bilateral ovoid pigmented lesions on funduscopic examination
 Incorrect Answer ImageC.Blurred optic disc margins on funduscopic examination
 Correct Answer ImageD.Hyperpigmentation of the mucocutaneous tissues
 Incorrect Answer ImageE.Hyperpigmented, velvety lesions in the axilla and groin

An 8-year-old girl is brought to the physician because her teacher has reported that her school
performance has steadily declined in the past 2 months. The teacher invited the parents to a
conference and told them that their daughter has increasingly frequent staring spells, after which
she returns to normal activity but is unaware of what had transpired immediately before that.
This has resulted in both a decline in the child's school performance and in the girl feeling an
increasing sense of frustration. The parents did notice these spells at home, too, but thought that
she was just caught up in her thoughts. However, on direct questioning, they confirmed that this
has happened when they were out walking as well. The child would suddenly stop in her steps
and then continue after a few seconds as if nothing had happened. This never lasted more than 5
to 10 seconds. When questioned, the girl denied knowing she had done that. Her past medical
history is not significant and there is no family history of similar disorders. On physical
examination, the child is in no acute distress. Her vital signs are within normal limits, as is the
physical examination. An electroencephalogram shows regular and symmetric 3-Hz spike-and-
slow-wave complexes provoked by hyperventilation. Which of the following is the most likely
diagnosis?
 Correct Answer ImageA.Absence seizures
 Incorrect Answer ImageB.Complex partial seizures
 Incorrect Answer ImageC.Psychogenic seizures
 Incorrect Answer ImageD.Reflex epilepsy
 Incorrect Answer ImageE.Shuddering attacks

A 7-year-old child falls on his outstretched hand, with the arm extended. His father, who
witnessed the accident, describes his elbow breaking by hyperextension. He comes in holding his
arm close to his body and complaining of pain at the elbow, which on physical examination is
grossly deformed and minimally swollen. X-rays show a supracondylar fracture of the humerus
without displacement of the distal fragment. Which of the following is correct concerning this
type of fracture? 
 Incorrect Answer ImageA.Hidden mirror image injury at the wrist
 Correct Answer ImageB.High incidence of neurovascular injuries
 Incorrect Answer ImageC.High incidence of nonunion with functional impairment after
casting
 Incorrect Answer ImageD.Near universal need for open reduction and internal fixation
 Incorrect Answer ImageE.Very likely interference with future growth

A 23-year-old man has been involved in a severe automobile accident. He sustained a closed
head injury, multiple extremity fractures, and a pelvic fracture. He was first attended to at a
peripheral community hospital and was subsequently transferred to a level-one trauma care unit.
On arrival, he is hypotensive, with a blood pressure of 72/59 mm Hg and a feeble pulse of
124/min. He is in a deep coma with bilateral dilated and fixed pupils. Ultrasound performed in
the emergency department is negative for intra-abdominal bleeding, and he receives a rapid
infusion of 2 L Ringer's lactate. This increases his blood pressure to 98/64 mm Hg and allows
him to undergo CT scanning in the emergency department. The scans show severe diffuse
blurring of the grey-white matter interface with no intracranial hematomas or deviation of the
midline structures, no cervical spine fractures, no intra-abdominal bleeding, and a contained
pelvic hematoma. The neurosurgeons advise that fluid restriction be implemented to minimize
increased intracranial pressure, whereas the general surgeons believe that further fluid infusion is
needed to increase his systemic blood pressure. A reasonable compromise to achieve both of
these goals would be to continue his fluid resuscitation with the use of which of the following?
 Incorrect Answer ImageA.3% saline
 Correct Answer ImageB.7.5% saline
 Incorrect Answer ImageC.Normal saline
 Incorrect Answer ImageD.Ringer's lactate
 Incorrect Answer ImageE.Type-specific blood

A 22-year-old woman comes to the physician for birth control counseling. She has been on an
oral contraceptive pill for the past 2 years. However, she is not happy with this method because
she frequently forgets to take her pills. She reports that her friends have been using the “monthly
shot” as a form of contraception and she would like to know more about this method. She reports
regular menstrual cycles with the last period three weeks ago. She has no history of significant
medical illnesses. Current medications include acetaminophen for headache. She denies alcohol
and illicit drug use. Her temperature is 37.1°C (98.8°F), pulse is 80/min, respirations are 12/min,
and blood pressure is 120/80 mm Hg. Physical examination is unremarkable. She is given an
injection of depot medroxyprogesterone acetate. Which of the following recommendations
should be given to the patient?
 Incorrect Answer ImageA.Back-up contraception is not required
 Correct Answer ImageB.Back-up contraception is required for 2 weeks
 Incorrect Answer ImageC.Duration of the use is not limited
 Incorrect Answer ImageD.Duration of the use should be limited to 1 year
 Incorrect Answer ImageE.Repeat injection in one month
 Incorrect Answer ImageF.Repeat injection in 4 months

A 16-year-old girl comes to her physician because of a sore throat and fever. The symptoms
started 2 days earlier with painful swallowing and a fever of 38.1°C (100.6°F). Since yesterday
morning she has had six loose stools and toward the end of the day began experiencing itchy
eyes and a gritty feeling, as if she had sand in her eyes that she could not wash out despite
multiple attempts. She has just returned from summer camp where several other girls who shared
the same cabin had similar symptoms. Apart from the current illness, she is in good health and
has no medical problems. She does not take any medication except a daily multivitamin. She is
an athletic and well-developed girl. Her temperature is 37.9°C (100.2°F), pulse of 90/min, and
respirations of 14/min. She has a follicular conjunctivitis and diffuse erythema of the
nasopharynx with mild edema of the tonsils and mucous exudate. The cervical lymph nodes are
palpable and tender. Which of the following is the most likely cause of this patient's current
illness?
 Correct Answer ImageA.Adenovirus
 Incorrect Answer ImageB.Cytomegalovirus
 Incorrect Answer ImageC.Epstein-Barr virus
 Incorrect Answer ImageD.Human immunodeficiency virus
 Incorrect Answer ImageE.Influenza virus type A

A 34-year-old woman, gravida 3, para 2, at 39 weeks’ gestation comes to the labor and delivery
ward with a gush of blood from the vagina, abdominal pain, and irregular, painful contractions.
Her prenatal course was significant for her being Rh-negative and antibody negative. Her
temperature is 37°C (98.6°F), pulse is 110/minute, blood pressure is 110/70 mm Hg, and
respirations are 12/minute. Abdominal examination shows a tender abdomen and cervical
examination shows the cervix to be closed and long with a significant amount of blood in the
vagina. The fetal heart rate is 170/min with moderate to severe variable decelerations with
contractions. The diagnosis of placental abruption is made, an emergent cesarean delivery is
performed which is uneventful. The infant’s blood type is A positive. Which of the following is
the most appropriate next step in management?
 Incorrect Answer ImageA.Apt test
 Incorrect Answer ImageB.Broad-spectrum antibiotics
 Correct Answer ImageC.Kleihauer-Betke test
 Incorrect Answer ImageD.RhoGAM (300 µg) administration
 Incorrect Answer ImageE.Serum IgG level
 Incorrect Answer ImageF.Urinalysis

A 7-year-old boy passes a large, bloody bowel movement. He is hemodynamically stable, and he
has a hemoglobin of 14 g/dL. Nasogastric aspiration yields clear, greenish fluid. Physical
examination, including anoscopy, is unremarkable. Which of the following is the most
appropriate next diagnostic test? 
 Incorrect Answer ImageA.Celiac arteriogram
 Incorrect Answer ImageB.Colonoscopy
 Correct Answer ImageC.Radioactively labeled technetium scan
 Incorrect Answer ImageD.Radioactively tagged red cell study
 Incorrect Answer ImageE.Upper gastrointestinal endoscopy

 A 17-year-old girl is brought to the Emergency Department with seizure-like activity for more
than 5 minutes. She is immediately given lorazepam, which stops the seizure. She is placed on a
cardiac monitor, and a wide complex tachycardia is noted. The ventricular tachycardia reverts to
sinus tachycardia after defibrillation is performed. Her finger stick serum glucose is 98,
temperature is 38.5°C (101.3°F), pulse is 190/min, respirations are 26/min, and blood pressure is
150/95 mm Hg. Physical examination reveals a lethargic, pale teen with dilated and reactive
pupils, dry mucous membranes, shallow respirations, diaphoresis, and brisk deep tendon
reflexes. She has been living in a homeless shelter for the past few months. Urine and blood
toxicology studies are done as part of routine care and return with abnormal results. The urine
toxicology screen is most likely to detect which of the following substances?
 Incorrect Answer ImageA.Barbiturates
 Correct Answer ImageB.Cocaine
 Incorrect Answer ImageC.Heroin
 Incorrect Answer ImageD.Marijuana
 Incorrect Answer ImageE.Phencyclidine

A 65-year-old man comes to the physician because of increasing urinary frequency and dribbling
at night for 3 months. He has no past medical history and is on no medications. On physical
examination, a digital rectal exam shows a normal-sized prostate with no masses. The prostate-
specific antigen (PSA) level is 15.4 ng/mL. Ultrasonography shows a small hypoechoic area on
the prostate measuring 6 x 8 mm in the right lobe. Which of the following is the most appropriate
next step in management?
 Incorrect Answer ImageA.Administer leuprolide
 Correct Answer ImageB.Biopsy of the prostate lesion
 Incorrect Answer ImageC.Perform bone scan
 Incorrect Answer ImageD.Perform CT scan of the abdomen and pelvis
 Incorrect Answer ImageE.Repeat PSA in 3 months

A pedestrian is hit by a car and is unconscious. Within a few minutes, he starts to move around
and moan. When the ambulance arrives, he is moving all four extremities spontaneously and
mumbling that his neck hurts. Shortly thereafter, he lapses again into unconsciousness and is
unable to be aroused. In the emergency department, it is noted that his left pupil is fixed and
dilated, and he has clear fluid dripping from the left ear. The trauma team intubates him nasally
over a fiberoptic bronchoscope and does a quick initial survey that shows no other obvious
injuries. He is hemodynamically stable. Which of the following is the most appropriate next step
in management?
 Incorrect Answer ImageA.Antibiotics and high dose corticosteroids
 Incorrect Answer ImageB.Cervical spine and skull x-ray films
 Correct Answer ImageC.CT scan of the head, extended to include the cervical spine
 Incorrect Answer ImageD.Emergency ear surgery to stop the leak of cerebrospinal fluid
 Incorrect Answer ImageE.Otoscopic examination and laboratory studies of the fluid

A 4-week-old boy is brought to the physician by his mother because of a 1 day history of labored
breathing. His birth was uneventful and immunizations are up to date. His mother reports that the
patient developed conjunctivitis on the fourth day of life. On physical examination, he is
breathing rapidly at 40 breaths per minute and is afebrile. His chest examination shows bilateral
inspiratory crackles and a slight wheeze. On chest x-ray, bilateral pneumonia is evident. The
leukocyte count is elevated at 15,000 with 40% eosinophils. Which of the following is the most
likely pathogen causing the patient's symptoms?
 Incorrect Answer ImageA.Ascaris lumbricoides
 Correct Answer ImageB.Chlamydia trachomatis
 Incorrect Answer ImageC.Mycoplasma pneumoniae
 Incorrect Answer ImageD.Pneumocystis carinii
 Incorrect Answer ImageE.Varicella zoster virus

A 13-year-old boy is brought to the physician because of right knee pain that has persisted for
the past 2 months despite the use of over-the-counter analgesics. His mother states that he has
been limping since the pain started. The pain is of insidious onset, but its intensity sometimes
suddenly increases. On physical examination, the boy is obese; his weight is higher than the 90th
percentile. Flexion of the right hip causes marked outward rotation and reproduces the pain.
Internal rotation of the right hip is limited, and the right leg is slightly shortened compared with
the left leg. Which of the following is the most likely diagnosis? 
 Incorrect Answer ImageA.Juvenile rheumatoid arthritis
 Incorrect Answer ImageB.Legg-Calve-Perthes disease
 Incorrect Answer ImageC.Septic arthritis
 Correct Answer ImageD.Slipped capital femoral epiphysis
 Incorrect Answer ImageE.Toxic synovitis

A 37-year-old woman presents with a swollen and tender left wrist. The symptoms began
approximately 1 day earlier and have become increasingly severe during that time. She had a
fever and shaking chills last night. She has no past medical history and she takes over the counter
ibuprofen for treatment of occasional premenstrual cramps. On physical examination, her wrist
has a decreased range of motion and is tender to palpation. The joint space has an effusion and is
tender, erythematous, and warm to touch. Which of the following is the most appropriate next
step in the management? 
 Incorrect Answer ImageA.Initiating colchicine
 Incorrect Answer ImageB.Initiating indomethacin therapy
 Incorrect Answer ImageC.Initiating IV antibiotics
 Incorrect Answer ImageD.Obtaining a wrist x-ray film
 Correct Answer ImageE.Perform arthrocentesis

A 32-year-old woman comes to the physician for a follow-up evaluation. She was seen originally
2 weeks ago for irregular menses that had been worsening and excess body hair. She is obese but
has no other medical problems. She has never had surgery. She is allergic to penicillin. On
examination, the patient is obese with a BMI of 29 kg/m2. Her vital signs are normal. She has
hair on her face, lower abdomen, and thighs, and significant acne on her face. The remainder of
the physical examination is normal. The diagnostic evaluation from her prior visit, including
laboratory studies, is consistent with polycystic ovary syndrome (PCOS). Which of the following
should be performed on this patient at this point?
 Incorrect Answer ImageA.Echocardiogram
 Incorrect Answer ImageB.Electrocardiogram
 Incorrect Answer ImageC.Fasting glucose level
 Correct Answer ImageD.Glucose challenge test
 Incorrect Answer ImageE.Treadmill testing

A 19-year-old woman, gravida 1, para 1, comes to the physician because of dull abdominal pain,
fever, and malodorous vaginal discharge since yesterday. She had a cesarean delivery at 35
weeks' gestation 2 days ago. The cesarean delivery was performed because she was in advanced
preterm labor with premature preterm rupture of membranes. Her pregnancy was complicated by
mild anemia and preterm labor at 33 weeks', for which she was given corticosteroids. She
became homeless during the pregnancy. Her temperature is 38.5°C (101.3°F). Physical
examination shows uterine tenderness and a malodorous vaginal discharge but is otherwise
normal. She is diagnosed with endometritis and started on intravenous (IV) antibiotics. Which of
the following is the greatest risk factor for the development of postpartum endometritis in this
patient?
 Incorrect Answer ImageA.Anemia
 Correct Answer ImageB.Cesarean delivery
 Incorrect Answer ImageC.Corticosteroids
 Incorrect Answer ImageD.Low socioeconomic status
 Incorrect Answer ImageE.Young maternal age

A 7-year-old, otherwise healthy girl fails a routine school hearing screening test. A follow-up
audiogram reveals a left-sided profound sensorineural hearing loss and normal hearing on the
right. There is no history of a prior audiogram or newborn screening exams for comparison. The
child reports having always had better hearing on the right. Her birth history is unremarkable.
She was the product of a spontaneous term delivery with no history of perinatal infections or
complications. There is no known family history of hearing loss or deafness. At this point,
recommendations should include which of the following? 
 Incorrect Answer ImageA.Cochlear implantation
 Incorrect Answer ImageB.A hearing aid for the left ear
 Correct Answer ImageC.Preferential seating in school and hearing protection for the right
ear
 Incorrect Answer ImageD.Speech therapy
 Incorrect Answer ImageE.Treatment with corticosteroids and an antiviral medication

An 82-year-old man is brought to the emergency department after he was found unconscious in
his apartment when a door-to-door search was made by the authorities during a severe heatwave.
The authorities report that his room had closed windows and no air conditioning. Air temperature
measured by the rescuers at that time was 42.2°C (108°F). On arrival at the emergency
department, his temperature is 43°C (109.4°F), pulse is 110/min, respirations are 20/min, and
blood pressure is 89/72 mm Hg. On physical examination, his skin is hot and dry. He has a
decreased response to physical stimuli and his pupils are dilated bilaterally. Laboratory data
shows a PaCO2 of 29 mm Hg and HCO3- of 24 mEq/L. He is promptly started on intravenous
fluid resuscitation. Which of the following complications is most likely to develop in this
patient?
 Incorrect Answer ImageA.Adrenal insufficiency
 Correct Answer ImageB.Disseminated intravascular coagulation (DIC)
 Incorrect Answer ImageC.Metabolic alkalosis
 Incorrect Answer ImageD.Respiratory acidosis
 Incorrect Answer ImageE.Severe hypoglycemia

A 35-year-old pathology laboratory technician fears that she is going to "bring some sort of bug
home" from work. She is concerned that she is contaminated, even though she takes all the
necessary precautions. She feels that she is unable to clean herself thoroughly and spends at least
half an hour washing her hands at work and continues the same ritual when she gets home. She
has to take a new clean towel each time she dries her hands and throws it right away into a
separate bag for "her own" dirty laundry. Her husband tries to reassure her that her fear is
exaggerated. She says that if she does not do these things she gets anxious and distressed, filled
with thoughts of "giving illnesses" to her children. She follows certain washing rituals to
decrease her distress. She is aware of how irrational this is, yet she is unable to stop it. Which of
the following neurotransmitters is most likely abnormal in this condition? 
 Incorrect Answer ImageA.Acetylcholine
 Incorrect Answer ImageB.Dopamine
 Incorrect Answer ImageC.Gamma-aminobutyric acid (GABA)
 Incorrect Answer ImageD.Norepinephrine
 Correct Answer ImageE.Serotonin

A 79-year-old man is admitted to the hospital with cough, fevers, and increasing shortness of
breath. He had been well until the day before admission. His past medical history is remarkable
for well-controlled hypertension, mild anemia, and three bouts of pneumonia (each involving
different pulmonary lobes) over the last 2 years. His pneumonia and influenza vaccinations are
up to date and his only medications are lisinopril, hydrochlorothiazide, and a multivitamin. He
has never smoked cigarettes or used illicit drugs and he drinks alcohol only occasionally. He
seems pale and lethargic, with a temperature of 38.4°C (101.1°F) and a pulse of 105/min. His
oxygen saturation is 90% on room air and 97% on 3 L/min O2 by nasal cannula. There are
scattered rales and bronchophony at the left lung base. A chest radiograph shows a left lower
lobe consolidation. Laboratory studies show: 
Leukocyte 22,000/mm3 (Normal 4,500–11,000/mm3)
count
Hemoglobin 12.2 g/dL (Normal 13.5–16 g/dL)
Hematocrit 37.3% (Normal 42–50%)
Platelets 91,000/mm3 (Normal 150,000–400,000
cells/mm3)
MCV 107.1 fL (Normal 80–95 fL)
RDW 14.2 (Normal 11–14.5)
Which of the following is the most likely underlying factor contributing to his current infection? 
 Incorrect Answer ImageA.Aspiration
 Correct Answer ImageB.Impaired immune function
 Incorrect Answer ImageC.Nutritional deficiency
 Incorrect Answer ImageD.Occult lung malignancy
 Incorrect Answer ImageE.Underlying chronic obstructive pulmonary disease (COPD)

Two days after admission to the hospital, a 57-year-old man reports symptoms of chest pain and
shortness of breath. He is currently post-op day 2 from a pancreaticoduodenectomy for cancer of
the head of the pancreas. Physical examination of his chest is unremarkable. He does not have
jugular venous distention. His electrocardiogram shows ST depression and T-wave flattening in
the anterolateral leads. He has elevated levels of troponin. His blood gases show PCO2 of 43 mm
Hg and PO2 of 87 mm Hg. Chest x-ray is unremarkable. Which of the following is the most
appropriate management at this time?
 Incorrect Answer ImageA.Deployment of inferior vena cava filter
 Incorrect Answer ImageB.Heparin drip to be followed by warfarin
 Incorrect Answer ImageC.Intravenous thrombolytic therapy
 Incorrect Answer ImageD.Send the patient for CT pulmonary angiogram of the chest
 Correct Answer ImageE.Start aspirin and atorvastatin for presumed myocardial infarction

A 69-year-old man is being evaluated for dementia. Over the last several months, he has
developed severe memory deficits as well as labile and bizarre mood. Physical examination
shows hypotonia, an intention tremor, bradykinesia, and hyperreflexia. Additionally, his pupils
are nonreactive to light but have a minimal accommodation reflex. Laboratory studies show a
normal thyroid stimulating hormone level, normal vitamin B12 and folate levels, a normal
erythrocyte sedimentation rate, and a reactive rapid plasma reagin (RPR) and fluorescent
treponemal antibody absorption assay (FTA-ABS). An electrolyte panel and complete blood
count are unremarkable. Which of the following is the most appropriate next step in this patient's
management?
 Incorrect Answer ImageA.Begin donepezil and vitamin E, discuss prognosis
 Incorrect Answer ImageB.Give ceftriaxone and azithromycin now
 Incorrect Answer ImageC.Give subcutaneous penicillin G now, follow closely
 Incorrect Answer ImageD.Order MRI of the brain
 Correct Answer ImageE.Perform lumbar puncture and cerebrospinal fluid (CSF) analysis
A 32-year-old woman tentatively diagnosed with schizophrenia has been treated successfully for
one month with haloperidol. One hour ago, she suddenly became delirious. Her temperature is
40.0°C (104.0°F), blood pressure is 180/90 mm Hg, pulse is 90/min, and respirations are 18/min.
She has a coarse tremor of her upper extremities and diffuse muscular rigidity. In addition to
hydration, which of the following therapies is indicated? 
 Incorrect Answer ImageA.Amiodarone
 Correct Answer ImageB.Dantrolene
 Incorrect Answer ImageC.Midazolam
 Incorrect Answer ImageD.Phenytoin
 Incorrect Answer ImageE.Vortioxetine

A 51-year-old man was recently diagnosed with chronic myelocytic leukemia and has received
chemotherapy for the past 3 months. He comes to the emergency department accompanied by his
wife and three daughters who indicate that he has become progressively lethargic over the past 3
days. The patient rapidly progresses into a coma and dies soon after. Laboratory studies sent at
the time of presentation show: 
Sodium 140 mEq/L
Potassium 5.2 mEq/L
Chloride 98 mEq/L
Bicarbonate 26 mEq/L
Glucose 90 mg/dL
Hemoglobin 14.4 g/dL
Hematocrit 41%
Platelets 210,000/mm3
Leukocyte count 500/mm3
Autopsy reveals the patient developed a mass effect, resulting in brain herniation and death as
shown in the photograph above. Which of the following is the most likely diagnosis? 
 Incorrect Answer ImageA.Arbovirus encephalitis
 Correct Answer ImageB.Aspergillosis
 Incorrect Answer ImageC.Brain metastasis
 Incorrect Answer ImageD.Cryptococcal meningitis
 Incorrect Answer ImageE.Hydrocephalus

A 22-month-old boy has had a fever for the past week. He is irritable but consolable. His
temperature is 40°C (104°F), pulse is 120/min, and blood pressure is 90/70 mm Hg. He has
nonpurulent conjunctivitis, erythema of the posterior pharynx, and dry, cracked lips. The tongue
is erythematous with prominent papillae. There is no nuchal rigidity. Cervical adenopathy is
present. The hands and feet are edematous. There is a maculopapular rash covering the trunk and
the extremities. Which of the following is the most appropriate therapy? 
 Incorrect Answer ImageA.Antistaphylococcal antibiotic
 Incorrect Answer ImageB.Antistreptococcal antibiotic
 Correct Answer ImageC.Intravenous immune globulin
 Incorrect Answer ImageD.Symptomatic treatment
 Incorrect Answer ImageE.Systemic corticosteroid
A healthy 22-year-old woman comes to the physician in May with a recurrent itchy rash located
on her upper trunk. She says she was previously prescribed a topical medication that resolved her
rash. On physical examination, she is afebrile and has normal vital signs. Scattered on her upper
trunk are about 30 discrete, 2 to 3 cm oval, scaly, salmon-colored macules. There are no other
cutaneous changes, and the remainder of the examination is normal. A potassium hydroxide
preparation of the lesion demonstrates a "spaghetti and meatball" pattern of fungal elements.
Which of the following is the most likely diagnosis?
 Incorrect Answer ImageA.Candidiasis
 Incorrect Answer ImageB.Pityriasis rosea
 Incorrect Answer ImageC.Sporotrichosis
 Incorrect Answer ImageD.Tinea corporis
 Correct Answer ImageE.Tinea versicolor

A 56-year old man is admitted to the emergency room with dizziness and lightheadedness. The
patient also reports palpitations, shortness of breath, nausea and vomiting. The patient explains
that his shortness of breath has worsened over the last couple of weeks and has taken extra doses
of theophylline. He has a history of coronary artery disease, hypertension, hyperlipidemia and
chronic obstructive lung disease. The patient currently takes aspirin, nitroglycerin, lisinopril,
lipitor, ipratropium and theophylline for these conditions. He denies drinking alcohol, but admits
smoking 2 packs per day for the last 35 years. His vitals are temperature 36.8°C, pulse 166/min,
blood pressure 140/85 mm Hg, respirations 22/min, oxygen saturation 95% on room air. In
general, the patient appears to be in mild distress. On physical examination, lung auscultation
reveals diffuse wheezing bilaterally and a tachycardic heart rate. Heart sounds are distant, with
no murmurs or rubs. Extremities are without peripheral edema, cyanosis or clubbing. Complete
blood count and basic metabolic panel did not reveal any significant abnormalities and an
electrocardiogram (ECG) is performed and shown below.
His respiratory medication is believed to be the cause of his ECG findings and is discontinued
immediately. Which of the following is the most appropriate next step in management of this
patient?
 Incorrect Answer ImageA.Cardioversion
 Incorrect Answer ImageB.Defibrillation
 Incorrect Answer ImageC.Diazepam
 Incorrect Answer ImageD.Epinephrine
 Incorrect Answer ImageE.Hemodialysis
 Incorrect Answer ImageF.Hemoperfusion
 Correct Answer ImageG.Metoprolol

A 66-year-old woman comes to the physician because of nausea, abdominal cramping and loss
of appetite. Yet, she states that her abdomen is getting larger and she is gaining weight.
Menopause was 14 years ago and she took oral estrogen replacement for a year after menopause.
She underwent a hysterectomy at the age of 45 years because of massive uterine leiomyomas.
Her vital signs are within normal limits. Physical examination shows a distended abdomen with
dullness to percussion which shifts when she rolls onto her side. Pelvic examination shows a
large pelvic mass. Which of the following laboratory parameters is most likely to be elevated?
 Incorrect Answer ImageA.α-fetoprotein
 Incorrect Answer ImageB.Alkaline phosphatase
 Incorrect Answer ImageC.β-hCG
 Incorrect Answer ImageD.CA 19-9
 Correct Answer ImageE.CA-125
 Incorrect Answer ImageF.Creatine phosphokinase
 Incorrect Answer ImageG.Lactate dehydrogenase

The following vignette applies to the next two items. 


A 3-week old male infant is brought to the pediatrician because of vomiting after every feeding.
He is normally breastfed and his emesis is described as nonbilious. He appears hungry again
after vomiting. He was born at 35 weeks’ gestation to a primigravid 21-year old woman via
uncomplicated vaginal delivery and has been healthy until now. On physical examination, the
abdomen is soft to palpation; there is no tenderness, guarding, or rigidity, and bowel sounds are
present. There is a nontender, firm, oval-shaped mass in the right upper quadrant. The anus is
patent, and no genital abnormality is present. Which of the following is the most likely
diagnosis?
 Incorrect Answer ImageA.Duodenal atresia
 Incorrect Answer ImageB.Hepatic abscess
 Correct Answer ImageC.Hypertrophy of the pylorus
 Incorrect Answer ImageD.Intussusception
 Incorrect Answer ImageE.Mesenteric cyst
 Incorrect Answer ImageF.Volvulus of the stomach

A 3-week old male infant is brought to the pediatrician because of vomiting after every feeding.
He is normally breastfed and his emesis is described as nonbilious. He appears hungry again
after vomiting. He was born at 35 weeks’ gestation to a primigravid 21-year old woman via
uncomplicated vaginal delivery and has been healthy until now. On physical examination, the
abdomen is soft to palpation; there is no tenderness, guarding, or rigidity, and bowel sounds are
present. There is a nontender, firm, oval-shaped mass in the right upper quadrant. The anus is
patent, and no genital abnormality is present. Which of the following is the most appropriate
treatment for this patient’s condition?
 Correct Answer ImageA.Dividing the hypertrophied fibers
 Incorrect Answer ImageB.Laparotomy and derotation of the stomach
 Incorrect Answer ImageC.Reassurance and observation
 Incorrect Answer ImageD.Reducing the telescoped segments of bowel
 Incorrect Answer ImageE.Ultrasound-guided percutaneous aspiration

The following vignette applies to the next two items.


A 54-year-old woman with end-stage liver disease (ESLD) is admitted to the hospital because of
confusion and disorientation a few hours after she ate turkey for Thanksgiving Day’s dinner. The
patient was diagnosed with hepatitis C several years ago, and her condition has progressively
worsened in the last 10 months. She has had 2 prior episodes of spontaneous bacterial peritonitis
(SBP) and one episode of upper gastrointestinal (GI) bleeding secondary to esophageal varices.
The patient has also developed renal insufficiency and is currently undergoing hemodialysis 3
times a week. She was placed on the liver transplant list 6 months ago. On admission to the
hospital, her temperature is 37.2ºC (99.0ºF), pulse is 96/min, respirations are 16/min, and blood
pressure is 125/75 mm Hg. Her skin and mucous membranes are jaundiced. Lungs are clear to
auscultation bilaterally. Heart sounds are normal, and there are no audible murmurs. Her
abdomen is soft but distended, slightly tender on palpation, and shifting dullness is positive.
Lower extremity edema is present bilaterally. Neurological examination shows asterixis and
hyperactive deep tendon reflexes. Laboratory studies show:
White blood cell count 9,800/ml
Hemoglobin 9.0 g/dL
Hematocrit 29%
Platelets 58,000/ml
Sodium 130 mmol/L
Potassium 3.6 mmol/L
Chloride 98 mmol/L
Carbon dioxide 19 mmol/L
Blood urea nitrogen 58 mg/dL
Creatinine 3.0 mg/dL
Glucose 102 mg/dL
Calcium 7.8 mg/mL
Total protein 5.9 g/dL
Albumin 2.1 g/dL
Total bilirubin 16 g/dL
Direct bilirubin 10.5 g/dL
Alkaline phosphatase 256 U/L
AST 187 U/L
ALT 175 U/L
Ammonia 68 mcmol/L
Prothrombin time (PT) 30.3 sec
PT INT 2.9
Liver transplantation is being considered at this time for definitive management of this patient's
condition. Liver transplant allocation for patients with end stage liver disease is dependent on
which of the following?
 Incorrect Answer ImageA.Degree of ascites and encephalopathy
 Incorrect Answer ImageB.Degree of the liver enzymes' elevation
 Incorrect Answer ImageC.History of upper gastrointestinal bleeding
 Incorrect Answer ImageD.History of spontaneous bacterial peritonitis
 Incorrect Answer ImageE.Presence of renal failure that requires hemodialysis
 Correct Answer ImageF.Serum bilirubin, creatinine, and INR values
 Incorrect Answer ImageG.Transplant waiting time

A 39-year-old woman with type 1 diabetes comes to the emergency department complaining of
lethargy, increased thirst, and increased frequency of urination. Physical examination shows
pallor, a blood pressure of 78/50 mm Hg, pulse of 124/min, and tachypnea. Laboratory studies
show: sodium 131 mEq/L, potassium 5.8 mEq/L, serum pH 7.19, blood glucose 560 mg/dL, and
serum osmolarity 334 mOsmol/kg. An electrocardiogram shows normal sinus rhythm. Which of
the following is the most appropriate initial therapy for this patient? 
 Incorrect Answer ImageA.Intravenous hydration with dextrose 5%, intravenous
bicarbonate, and calcium chloride
 Incorrect Answer ImageB.Intravenous hydration with dextrose 5% in water, along with
an insulin bolus followed by an insulin infusion
 Correct Answer ImageC.Intravenous hydration with 0.9% NaCl and an insulin bolus
followed by an insulin infusion
 Incorrect Answer ImageD.Intravenous insulin bolus followed by an insulin infusion and
bicarbonate
 Incorrect Answer ImageE.Intravenous insulin bolus followed by an insulin infusion and
oral sodium polystyrene sulfonate

A 29-year-old man with a history of grand mal seizures is found unconscious at home. He has
been poorly compliant with his regimen of phenobarbital and valproic acid. He has no other past
medical history except for alcoholic hepatitis. He is unconscious and unarousable. The
paramedics arrive and find that his blood pressure is 112/68 mm Hg, pulse is 94/min, respirations
are 12/min, and he is afebrile. He withdraws to painful stimuli in all four extremities. He is
incontinent of urine. The paramedics administer intravenous naloxone, thiamine, and 50%
glucose without any patient response. Which of the following is the most likely diagnosis? 
 Correct Answer ImageA.Grand mal seizure
 Incorrect Answer ImageB.Hypoglycemic coma
 Incorrect Answer ImageC.Phenobarbital overdose
 Incorrect Answer ImageD.Spontaneous subarachnoid hemorrhage
 Incorrect Answer ImageE.Wernicke encephalopathy

A 26-year-old woman, gravida 3, para 2, at 32 weeks’ gestation comes to the physician for a
routine prenatal visit. She had two uncomplicated vaginal deliveries 5 and 8 years ago. She has a
history of systemic lupus erythematosus (SLE). Her last flare was one year ago, causing a malar
rash, arthralgias and pleuritic chest pain. This flare was successfully treated with a short course
of prednisone. She is not taking any medications currently and does not have any similar
symptoms right now. The physical examination shows no abnormalities. A fetal ultrasound is
normal. She is concerned about the effects of SLE on her unborn child. For which of the
following is the fetus at increased risk of developing? 
 Incorrect Answer ImageA.Choanal atresia
 Correct Answer ImageB.Heart block
 Incorrect Answer ImageC.Neural tube defects
 Incorrect Answer ImageD.Retinoblastoma
 Incorrect Answer ImageE.Sweet syndrome
 Incorrect Answer ImageF.Tricuspid valve atresia

A 48-year-old Caucasian man is admitted to the hospital for total thyroidectomy. The patient
does well immediately postoperatively, except for mild nausea and disorientation thought to be
attributable to the anesthetic. Approximately 24 hours after the procedure, the patient begins to
complain of a generalized "tingling" sensation and severe muscle cramps. Which of the
following is the most appropriate next step in management? 
 Incorrect Answer ImageA.Administration of a benzodiazepine
 Incorrect Answer ImageB.Administration of oxygen via nasal cannula
 Incorrect Answer ImageC.Intravenous infusion of an antiemetic
 Correct Answer ImageD.Intravenous infusion of calcium gluconate
 Incorrect Answer ImageE.Neurologic consultation

An 82-year old Caucasian male is admitted to the hospital with severe aspiration pneumonia. The
patient is appropriately treated in the intensive care unit with intubation and ventilatory support.
His condition improves and he is successfully extubated on the 6thday after admission. On the
8th day after admission, the patient was transferred to the general medicine floor where he was
diagnosed with severe malnutrition. Nutritional consultation provided recommendations and
appropriate nutritional support was initiated. Which of the following is the most sensitive
indicator to evaluate the effectiveness of nutritional interventions?
 Incorrect Answer ImageA.Albumin
 Incorrect Answer ImageB.Blood urea nitrogen
 Correct Answer ImageC.Prealbumin
 Incorrect Answer ImageD.Total protein
 Incorrect Answer ImageE.Transferrin

A previously healthy 32-year-old woman seeks treatment for "depression" that has not resolved
on its own one month after her father's death from stomach cancer. She continues to have poor
sleep, poor appetite, feelings of guilt about her father's death, and thoughts that she should have
died with her father. She has kept in close contact with her friends who have told her that she has
started to walk and talk like her father. Which of the following is the most likely diagnosis?
 Incorrect Answer ImageA.Acute stress disorder
 Incorrect Answer ImageB.Adjustment disorder
 Correct Answer ImageC.Bereavement
 Incorrect Answer ImageD.Major depressive disorder
 Incorrect Answer ImageE.Post-traumatic stress disorder

A 70-year-old woman develops nausea, colicky abdominal pain, vomiting, and progressive
abdominal distention over a period of 4 days. She has not had a bowel movement or passed any
flatus for 3 days. She also gives a history of occasional episodes of right upper quadrant
abdominal pain for which she never received medical treatment. She is the mother of six
children, and she has otherwise been in good health all her life. She has never had abdominal
surgery. Physical examination shows a dehydrated, mildly obese woman with abdominal
distention and high-pitched bowel sounds that coincide with the colicky pain. She has no
palpable hernias. X-ray of the abdomen shows scant gas in the colon, dilated loops of small
bowel with air-fluid levels, and air in the biliary tree. Which of the following is the most likely
diagnosis? 
 Incorrect Answer ImageA.Cancer of the gallbladder
 Incorrect Answer ImageB.Emphysematous cholecystitis
 Correct Answer ImageC.Gallstone ileus
 Incorrect Answer ImageD.Intussusception
 Incorrect Answer ImageE.Small bowel carcinoma

A 74-year-old man comes to the physician because of a 2-year history of worsening leg pain. He
describes pain in both of his legs as an aching, throbbing pain that occurs with walking and is
relieved with rest. The pain never seems to occur at rest or while sleeping but has been steadily
getting worse. The pain is mainly in his left calf and always occurs when walking more than a
block. His past medical history includes coronary artery disease with a myocardial infarction at
age 65, diabetes mellitus type 2 diagnosed at age 62, hypertension, and tobacco use. Physical
examination shows diminished left dorsalis pedis and posterior tibialis pulses. The skin of his left
foot and ankle is shiny, with loss of hair and dystrophic nails. Which of the following is the most
appropriate initial step in diagnosis?
 Correct Answer ImageA.Ankle-brachial index
 Incorrect Answer ImageB.Exercise treadmill test
 Incorrect Answer ImageC.Magnetic resonance angiography
 Incorrect Answer ImageD.Segmental limb pressures
 Incorrect Answer ImageE.Segmental plethysmography

A 52-year-old man with diabetes comes to the emergency department of a large academic
medical center complaining of difficulty breathing, severe nausea and vomiting, and mild
abdominal discomfort. He is concerned that something he ate made him ill. About 1 hour ago
during breakfast he suddenly felt ill with these symptoms and decided to come straight to the
emergency department. In addition to his diabetes, he has a past medical history that includes
hypertension, hyperlipidemia, and tobacco use. Physical examination is unremarkable aside from
some mild chest wall and epigastric tenderness. A workup in the emergency department includes
a set of cardiac enzymes that reveal a normal troponin and creatinine kinase level with a
moderately elevated myoglobin level. An electrocardiogram reveals a widened QRS complex in
a left bundle pattern, not present on an electrocardiogram from 1 year earlier, and a normal sinus
rhythm of 68 beats/min. Which of the following is the most appropriate next step in the
management? 
 Incorrect Answer ImageA.Admit to intermediate cardiac care unit for cardiac monitoring
and serial enzymes
 Incorrect Answer ImageB.Give intravenous thrombolytics, transfer to intensive care unit
for monitoring
 Incorrect Answer ImageC.Hold patient until next set of enzymes are available, discharge
if within normal limits
 Correct Answer ImageD.Send patient to catheterization laboratory for percutaneous
coronary intervention
 Incorrect Answer ImageE.Transfer patient to medicine ward for further workup of
atypical chest pain

A 3-year-old girl is brought to the physician for evaluation of a facial birthmark. The lesion was
present at birth, light pink in color, and affected the left side of the face from the upper eyelid to
the lip angle. It has not changed in size over time but has assumed a more purple-red color. The
child has no history of seizures and has reached all her developmental milestones at the
appropriate ages. The family history is unremarkable for cutaneous or neurologic disorders. On
physical examination, the patient is in no acute distress. Her vital signs are within normal limits.
There is a homogeneously violaceous, well demarcated, and slightly thickened plaque that
extends over the V1 branch of the trigeminal area on the right and ends abruptly in the midline.
Ophthalmologic examination shows no abnormalities. A CT of the head shows intracranial
calcifications. Which of the following complications is most likely to develop in this patient? 
 Incorrect Answer ImageA.Contralateral hemiparesis
 Correct Answer ImageB.Convulsions
 Incorrect Answer ImageC.Glaucoma
 Incorrect Answer ImageD.Intellectual disability
 Incorrect Answer ImageE.Iris angiomas

Because of the presence of a heart murmur, an otherwise asymptomatic 3-month-old boy has an
echocardiogram performed. The study shows a small, restrictive ventricular septal defect located
low in the muscular septum. The murmur had not been present at the time of birth, but was
recognized at a well-child visit 3 months later. Further management should include which of the
following therapies? 
 Incorrect Answer ImageA.Cardiac catheterization
 Incorrect Answer ImageB.Elective surgical repair before the age of 2 years
 Correct Answer ImageC.Prophylaxis for subacute bacterial endocarditis, and continued
observation
 Incorrect Answer ImageD.Radiologic closure by deployment of a button
 Incorrect Answer ImageE.Therapy with indomethacin

A 63-year-old woman complains of difficulty swallowing that has been present for
approximately 6 months. When asked for details, she opens her blouse, points to an obviously
enlarged thyroid gland, and says that that is the point where the food has trouble "getting
through." She adds that she has had that goiter for at least 25 years, but it has been increasing
steadily and slowly in size. When a review of systems is conducted, she gives a positive answer
to almost everything that she is asked, but the picture that emerges is one of chronic fatigue,
constipation, slow mental processes, slow voice, and cold intolerance. Palpation of the neck
shows a freely movable thyroid, without nodules, approximately the size of a human fist. There
is no adenopathy. The laboratory studies show low levels of T4 and high levels of thyroid
stimulating hormone (TSH). She is a native of Cape Cod, Massachusetts, and has lived there all
her life. Which of the following is the most likely diagnosis? 
 Incorrect Answer ImageA.Chronic iodine deficiency
 Incorrect Answer ImageB.Graves disease
 Correct Answer ImageC.Hashimoto thyroiditis
 Incorrect Answer ImageD.Medullary cancer
 Incorrect Answer ImageE.Papillary cancer

A 39-year-old woman comes to the physician for a routine health maintenance examination. She
states that she has been feeling well over the past year except for occasional back pain and
depression. She states that she is very interested in complementary and alternative medicine
(CAM) and that she uses St. John's wort and some Native American approaches. Other than
depression and back pain, she has no other medical problems. Past surgical history is significant
for two cesarean deliveries. She takes no other medications and has no known drug allergies.
Physical examination, including breast and pelvic examination, is normal. Which of the
following should be included in the counseling of this patient regarding her use of
complementary and alternative medicine? 
 Incorrect Answer ImageA.Encourage her to increase her use of CAM
 Incorrect Answer ImageB.Explain that use of CAM is hazardous to her health
 Incorrect Answer ImageC.Ignore her use of CAM
 Incorrect Answer ImageD.Refuse to treat her unless she stops the use of CAM
 Correct Answer ImageE.Take a full history regarding her use of CAM

A 14-month-old boy has a persistent rash that has been refractory to topical hydrocortisone 0.5%
cream. The child is often ill with colds and ear infections, and bruises easily. The rash, though, is
making him very uncomfortable because of the itch and he keeps on waking up at night
scratching. The scratches then become infected and a new cycle of distress develops. There is no
family history of atopic disease and their 4-year-old daughter has no skin problems or allergies.
The patient is currently on oral amoxicillin for a recent ear infection but takes no other
medication. On examination, there are multiple ill-defined, confluent, scaly, and crusted
erythematous patches on the cheeks and scalp, retroauricularly, and on the extensor aspects of
the extremities. The dorsal hands show areas of moderate lichenification. His skin is overall very
dry and lightly scaly with many excoriations. The arms and shins have bruises in various stages
and petechiae. Laboratory studies are most likely to show which of the following findings?
 Incorrect Answer ImageA.Elevated IgM level
 Incorrect Answer ImageB.Low IgA level
 Incorrect Answer ImageC.Low IgE level
 Incorrect Answer ImageD.Lymphopenia
 Correct Answer ImageE.Thrombocytopenia

A 39-year-old businessman with no prior medical problems is rushed to the emergency


department following the sudden onset of dizziness, shortness of breath, and palpitations. His
blood pressure on admission is 190/110 mm Hg, his pulse is 124/min, and he is diaphoretic. His
wife says that his behavior has changed over the past couple of months since he became chief
executive officer (CEO) of his company. She notes that at times, he seems energetic, euphoric, or
irritable; then he seems "To be down" for no reason. He just returned from one of his many
business meetings at which he ”Spent more money than ever before.” The patient is smiling
inappropriately and denies any alcohol or drug use. Which of the following will most likely be
found on a urine drug screen?
 Correct Answer ImageA.Cocaine
 Incorrect Answer ImageB.Heroin
 Incorrect Answer ImageC.Nicotine
 Incorrect Answer ImageD.Organic inhalants
 Incorrect Answer ImageE.Phencyclidine

A 62-year-old man reports an episode of gross, painless hematuria. There is no history of trauma,
and further questioning determines that he had total hematuria, rather than initial or terminal
hematuria. The man does not smoke and has had no other symptoms referable to the urinary
tract. He has no known allergies. Physical examination, including rectal examination, is
unremarkable. His serum creatinine is 0.8 mg/dL, and, except for the presence of many red cells,
his urinalysis is normal and shows no red cell casts. His hematocrit is 46%. Which of the
following is the most appropriate initial step in the workup? 
 Incorrect Answer ImageA.Coagulation studies and urinary cultures
 Correct Answer ImageB.Multi-phasic CT urography and cystoscopy
 Incorrect Answer ImageC.PSA determination and prostatic biopsies
 Incorrect Answer ImageD.Retrograde cystogram and pyelograms
 Incorrect Answer ImageE.Sonogram and non-contrast CT scan of both kidneys
A 6-year-old boy is brought to the pediatrician because of a 3-day history of skin lesions. On
physical examination, he has multiple yellow, crusted erosions below the nares and on the
cheeks, chin, and upper extremities. The rest of the examination is normal. Which of the
following is the most appropriate treatment for this condition? 
 Incorrect Answer ImageA.Oral acyclovir
 Incorrect Answer ImageB.Oral amoxicillin
 Correct Answer ImageC.Oral cephalexin
 Incorrect Answer ImageD.Topical ketoconazole
 Incorrect Answer ImageE.Topical 2% hydrocortisone

A 54-year-old African American man, with a history of smoking and alcohol dependence comes
to the physician because of progressive dysphagia that began 3 months ago. He first noticed
difficulty swallowing meat; it then progressed to other solid foods, then to soft foods, and now to
liquids as well. He locates the place where the food "sticks" at the lower end of the sternum. He
has lost 30 pounds. Which of the following is the most appropriate first step in diagnosis? 
 Correct Answer ImageA.Barium swallow study
 Incorrect Answer ImageB.Gastrografin swallow study
 Incorrect Answer ImageC.Esophageal manometry
 Incorrect Answer ImageD.Esophageal pH monitoring
 Incorrect Answer ImageE.Esophagoscopy

A 42-year-old, unemployed laboratory technician is admitted to the hospital for nausea,


vomiting, and abdominal pain. She tells the physician that she had been diagnosed in the past
with disseminated lupus erythematosus and that she had Hodgkin disease. She seems worried
that an extensive medical workup failed to confirm any of the previous diagnoses or find a cause
of her actual symptoms. On examination, the only physical findings are scars and some abscesses
on her thighs. She explains that she had been intentionally injured by the nurses in a previous
hospital. Which of the following is the most likely diagnosis? 
 Correct Answer ImageA.Factitious disorder
 Incorrect Answer ImageB.Ganser syndrome
 Incorrect Answer ImageC.Hypochondriasis
 Incorrect Answer ImageD.Malingering
 Incorrect Answer ImageE.Somatization disorder

A 78-year-old male presents to the emergency department after a rear end automobile collision.
He is in no distress and his vitals are within normal limits and stable. He reports that during the
accident he hyperextended his neck, and now is having difficulty moving his arms and states he
has a burning pain in both arms. Physical examination is significant for 2/5 strength throughout
bilateral upper extremities and 5/5 strength in the bilateral lower extremities. Which of the
following is the most likely diagnosis?
 Incorrect Answer ImageA.Anterior cord syndrome
 Correct Answer ImageB.Central cord syndrome
 Incorrect Answer ImageC.Complex regional pain syndrome
 Incorrect Answer ImageD.Posterior cord syndrome
 Incorrect Answer ImageE.Spinal cord hemisection
A 27-year-old male prisoner with a self-described history of physical abuse is brought to the
emergency department by prison staff. The patient states that he has severe leg pain after falling
out of his bunk bed and that he is unable to walk. Neurologic examination shows normal deep
tendon reflexes, but the patient has decreased sensation to pain and pinprick. The following
morning, the patient is seen walking but complains that he is unable to pass urine, and he is told
by the nurse that he will have to have a catheter inserted. As his physician walks in unexpectedly
on rounds, the patient is seen sneaking back into bed from the direction of the restroom. Which
of the following is the most likely diagnosis? 
 Incorrect Answer ImageA.Antisocial personality disorder
 Incorrect Answer ImageB.Conversion disorder
 Incorrect Answer ImageC.Drug dependence
 Correct Answer ImageD.Factitious disorder
 Incorrect Answer ImageE.Schizophrenia

A 29-year-old woman is brought to the hospital by her husband. She has not slept in several days
and cleans the house, drinks wine, and listens to loud music in the middle of the night. She spent
$2,000 on a shopping spree over 4 days and states that she quit her job, is changing careers and is
starting a private business. In the interview room, she talks incessantly, giggles with the nurse,
and unbuttons her blouse to show her newly bought underwear. This behavior has lasted for 8
days. She has always been cheerful, and has had short periods of time when she was more
energetic, but never like this. She denies the use of street drugs, and her urine drug screen is
negative. Physical examination is unremarkable. Which of the following is the most likely
diagnosis?
 Correct Answer ImageA.Bipolar I disorder
 Incorrect Answer ImageB.Bipolar II disorder
 Incorrect Answer ImageC.Borderline personality disorder
 Incorrect Answer ImageD.Cyclothymic disorder
 Incorrect Answer ImageE.Schizophrenia

A 10-week-old infant has a growing lesion on his right eyelid. He was born at 35 weeks'
gestation after an uneventful pregnancy and had Apgar scores of 8 and 9 at 1 and 5 minutes,
respectively. At the time of birth, there was a red patch over his right eyebrow and upper eyelid
with no other abnormalities on physical examination. The mother reports that the skin lesion
began thickening about 2 weeks ago and is steadily growing in size. It has started interfering
with complete opening of the upper eyelid. On examination, the patient has a spongy, nontender,
erythematous plaque extending from the top of the right eyebrow to the free margin of the right
eyelid and from the medial third of the upper eyelid to the lateral canthus. When the patient
opens his eye and looks straight ahead, the upper eyelid covers half of the cornea and sclera. The
lateral upward gaze leads to complete obstruction of sight. Ophthalmologic examination shows
no abnormalities. The physician explains to the mother that the child needs to be treated with
high-dose systemic corticosteroids with the goal of arresting growth and inciting involution of
the lesion. The mother is appalled by the possible side effects of this treatment and does not wish
to subject her son to it. In an attempt to explain the possible consequences of nonintervention at
this stage of the child's development, the physician tries to make it clear that there may be
permanent sequelae to the patient's eyesight. This child is at greatest risk for which of the
following complications if the lesion continues to obstruct vision in his right eye?
 Correct Answer ImageA.Amblyopia
 Incorrect Answer ImageB.Proptosis
 Incorrect Answer ImageC.Pseudostrabismus
 Incorrect Answer ImageD.Retinopathy of prematurity
 Incorrect Answer ImageE.Strabismus

A 33-year-old woman, gravida 2, para 1, at 35 weeks' gestation comes to labor and delivery
because of leakage of fluid from her vagina. She states that she felt the leakage earlier in the
night after coughing and had a second episode later. She says that the fetus is moving well and
that she denies contractions or bleeding from the vagina. This pregnancy has been
uncomplicated. Her obstetric history is significant for a normal vaginal delivery following an
uncomplicated pregnancy 3 years ago. Speculum examination shows a normal appearing vagina
with no pool of fluid. A pH test is performed that reveals a normal vaginal pH. A slide is made of
a swab from the vagina and no ferning is seen. Sterile vaginal examination shows the cervix is 1
cm dilated, 25% effaced, and the vertex is at 0 station. An abdominal ultrasound is performed
that shows a normal amniotic fluid volume. The fetal heart rate is 130/min with moderate
variability. The patient has a few rare contractions. Which of the following is the most
appropriate next step in management?
 Correct Answer ImageA.Discharge home with instructions, precautions, and follow-up
 Incorrect Answer ImageB.Initiate induction of labor for ruptured membranes
 Incorrect Answer ImageC.Perform cesarean delivery for nonreassuring fetal testing
 Incorrect Answer ImageD.Start magnesium sulfate for tocolysis
 Incorrect Answer ImageE.Start penicillin for Group B Streptococcus prophylaxis

A 67-year-old man was found to have an abdominal aortic aneurysm on physical examination 6
months ago. At that time, an ultrasound indicated the size of the aneurysm to be 4.0 cm. He
returns for a scheduled follow-up visit. His medical history is remarkable for long-standing
hypertension, mild chronic obstructive pulmonary disease (COPD), and cigarette smoking. He is
completely asymptomatic. Ultrasound at this time shows the aneurysm to measure 5.0 cm. CT
scan indicates that the aneurysm extends from the origin of the renal arteries into both common
iliac arteries. Which of the following is the most appropriate next step in management?
 Incorrect Answer ImageA.Arteriogram
 Incorrect Answer ImageB.Continued observation
 Correct Answer ImageC.Elective surgical repair
 Incorrect Answer ImageD.Endoluminal stent
 Incorrect Answer ImageE.Smoking cessation

A 3-month-old baby is brought to the physician by his mother for a well-baby visit. She proudly
reports that people remark that the baby has unusually beautiful eyes, which are huge and always
shiny. Inspection reveals that indeed the baby's eyes are much larger than normal, the corneas are
large and appear edematous, the baby has constant tearing, and he seems to avoid the light.
Based upon these finding, the physician should suspect that the baby has which of the following
conditions? 
 Incorrect Answer ImageA.Congenital cataracts
 Correct Answer ImageB.Congenital glaucoma
 Incorrect Answer ImageC.Gonococcal conjunctivitis
 Incorrect Answer ImageD.Keratitis sicca
 Incorrect Answer ImageE.Orbital cellulitis

A 7-year-old girl who attends second grade is brought to the emergency department by social
services accompanied by the school nurse who provides a history. The nurse says that the child
has been very withdrawn lately and has shown repeated signs of poor attention and regression in
the class. The nurse decided to call social services when the girl complained to her teacher of
abdominal pain and a physical examination showed bloodstained discharge on her underwear.
The mother arrives at the emergency department and relates that the child is not ill and does not
take any medication. The mother shares that the girl is the eldest of three children from the
mother's first marriage. Her past medical history is unremarkable. The mother denies any
problems at home. She has been living with her second husband for 2 years now. On physical
examination, the patient is in no acute distress and her vital signs are within normal limits. Her
breasts are in Tanner I stage of development and she does not have any pubic or axillary hair.
Inspection of the genitals shows several moist, pink, verrucous papules on the introitus of the
vagina and a thin, blood-tinged leukorrhea. The hymenal opening is 12 mm in diameter. Which
of the following is the most likely diagnosis?
 Incorrect Answer ImageA.Lichen sclerosus et atrophicus
 Incorrect Answer ImageB.Menarche
 Incorrect Answer ImageC.Sarcoma botryoides
 Correct Answer ImageD.Sexual abuse
 Incorrect Answer ImageE.Vulvar hemangioma

A 62-year-old man with long-standing type 2 diabetes mellitus comes to the physician for a
routine health maintenance examination. He reports that his home blood glucose measurements
have improved since his last visit 6 months ago despite no changes in compliance or to his
current insulin and metformin doses. His medical history is significant for diabetic retinopathy,
depression, stable hepatitis C, gastroparesis, and chronic renal insufficiency. Physical
examination is unchanged from the patient's last visit. His most recent hemoglobin A1c level is
6.8%. Previously, these levels have ranged from 8 to 11%. Which of the following is the most
likely cause for this patient's improved glycemic control?
 Incorrect Answer ImageA.Change in the patient's diet
 Incorrect Answer ImageB.Declining hepatic function
 Incorrect Answer ImageC.Improved insulin sensitivity
 Incorrect Answer ImageD.Inaccurate home glucometer
 Correct Answer ImageE.Worsening renal disease

A 28-year-old man comes to the clinic because of testicular pain and swelling. For the past few
months, he has noticed a dull, aching pain in his left testicle. He notices that it is slightly swollen
at the end of the day, and less so in the morning. The pain is partially relieved with NSAIDs and
by lying down. Physical examination shows a large left scrotal mass that is soft and mobile and
appears to have discrete, palpable cords within it. The mass is partially reducible with
recumbency, does not transilluminate, and increases in size with a Valsalva maneuver. Which of
the following is the most likely possible long-term complication of this condition? 
 Incorrect Answer ImageA.Bowel obstruction
 Incorrect Answer ImageB.Hypogonadotropic hypogonadism
 Incorrect Answer ImageC.Malignancy of the testicle
 Correct Answer ImageD.Reduced sperm count and infertility
 Incorrect Answer ImageE.Testicular torsion and necrosis

A 25-year-old African American woman comes to the office because of tender lesions on her
shins that appeared several weeks earlier. She first thought she had injured herself but now notes
the appearance of new lesions. Her past medical history is significant for asthma diagnosed 6
months ago. Current medications include a steroid inhaler. On physical examination, she is in no
acute distress. Her vital signs are normal. On the anterior shins, there are multiple tender, deep-
seated, violaceous red nodules up to 5 cm in diameter. The older lesions have a bruised
appearance, displaying shades of yellow, green, and brown. There are no palpable popliteal or
inguinal lymph nodes. Examination of the oral mucosa is unremarkable. Inspection of the eyes
shows enlarged lacrimal glands. The remainder of the physical examination is normal. A
complete blood count with differential, urinalysis, chest and abdominal radiographs, and
serologic tests for connective tissue disorders are pending. Which of the following findings is
most likely present in this patient?
 Incorrect Answer ImageA.Air-fluid levels on abdominal radiographs
 Incorrect Answer ImageB.Anti-Smith antibodies
 Correct Answer ImageC.Hilar lymphadenopathy on chest x-ray
 Incorrect Answer ImageD.Increased reticulocyte count
 Incorrect Answer ImageE.Maltese crosses in the urine sediment

An 8-year-old boy is brought to the physician by his mother because of fever, headache, and a
tender, enlarged mass in the right armpit of several days' duration. The child has been in good
health and has no significant past medical history. He goes to second grade and spends a lot of
his free time outdoors playing with friends. The mother remembers that a few weeks ago he
brought home a kitten that he and his friends had found on the street, and she is afraid he might
have caught some disease from it. On physical examination, the child is in mild distress. His
temperature is 38.1°C (100.6°F), pulse is 90/min, and respirations are 20/min. On the right dorsal
wrist, there are several erythematous papules arranged in a linear fashion, and the boy confirms
that he had been scratched by the kitten on the hands and arms several times while they were
playing. In the right axilla, there is a tender mass 5 cm in diameter with erythema of the
overlying skin. Which of the following is the most likely causative organism of this patient's
disease?
 Correct Answer ImageA. Bartonella henselae
 Incorrect Answer ImageB.Borrelia burgdorferi
 Incorrect Answer ImageC.Erysipelothrix rhusiopathiae
 Incorrect Answer ImageD.Pasteurella multocida
 Incorrect Answer ImageE.Streptococcus pyogenes

A 46-year-old man had a right hemicolectomy 4 days ago because of colon cancer. His
nasogastric tube was removed 24 hours ago when he began passing flatus. This afternoon he
began having multiple episodes of watery, green-colored, foul-smelling diarrhea associated with
abdominal cramping. He has been receiving 1 g of cefazolin every 6 hours since his surgery. On
examination, he is in no distress. His temperature is 38.2°C (100.8°F), pulse is 89/min, and blood
pressure is 119/74 mm Hg. His staple line is clean and dry, but his abdomen is slightly distended
compared with that seen on morning rounds. There is mild tenderness to palpation diffusely, but
no rebound tenderness. Which of the following is the most appropriate step in management? 
 Incorrect Answer ImageA.Begin antidiarrheal medication
 Correct Answer ImageB.Discontinue the cefazolin
 Incorrect Answer ImageC.Obtain stool sample and await results before moving on to next
step
 Incorrect Answer ImageD.Replace the nasogastric tube
 Incorrect Answer ImageE.Schedule a colonoscopy

A 26-year-old woman, gravida 3, para 2, aborta 1, comes to the physician because she has not
had a menstrual period in 6 months. She reports having two uncomplicated deliveries 3 and 5
years ago. She had an incomplete spontaneous abortion 8 months ago and underwent dilatation
and curettage; she also had a tubal ligation at that time. She does not report any significant
weight gain over the past year. She has no history of major medical illness and takes no
medications. She is 150 cm (4 ft 11 in) tall and weighs 50 kg (110 lb); BMI is 22 kg/m2. Her vital
signs are within normal limits. Pelvic examination is unremarkable. A urine pregnancy test and
serum beta-HCG levels are negative. Serum TSH is 2.3 μU/mL, serum prolactin is 13 ng/mL,
and serum FSH is 15 mIU/mL. Which of the following is the most appropriate next step in
management?
 Incorrect Answer ImageA.Obtain serum estradiol levels
 Correct Answer ImageB.Order a progesterone challenge test
 Incorrect Answer ImageC.Sample the endometrium
 Incorrect Answer ImageD.Schedule a hysterosalpingogram
 Incorrect Answer ImageE.Schedule a pelvic ultrasound

A 60-year-old man had a deep venous thrombosis in his right leg 4 months ago. At the time, a
hematologic evaluation for a hypercoagulable state was negative. He is now admitted to the
hospital with a deep venous thrombosis in his left leg. His serum CA 19-9 level is 25,000 U/mL
(reference range: 0-37 U/mL). Which of the following studies would most likely confirm the
underlying diagnosis? 
 Correct Answer ImageA.Abdominal CT scan
 Incorrect Answer ImageB.Bilateral venograms
 Incorrect Answer ImageC.Bone marrow biopsy
 Incorrect Answer ImageD.Chest CT scan
 Incorrect Answer ImageE.Echocardiogram
 Incorrect Answer ImageF.Ventilation-perfusion scan

A 65-year-old man comes to the physician because he has had abdominal pain, general
weakness, and fatigue that have progressively worsened over the past several months. On further
questioning, he notes that there has been a change in bowel habits that has been characterized by
thin-caliber stools and occasional streaking with blood. He denies marked bleeding per rectum,
melena, nausea, or vomiting. He also endorses unintentional weight loss in the same time period
such that he now has to wear belts with his pants. His past medical history is significant for
hypertension and benign prostatic hyperplasia, which are managed with terazosin. Physical
examination shows a pale, thin man in no acute distress with normal vital signs. There is no
abdominal pain, guarding, or rebound tenderness noted on palpation. There are no masses
palpated, no hepatosplenomegaly, and bowel sounds are present in all four quadrants. Rectal
examination shows no masses, but stool is heme positive on guaiac test. The rest of the physical
examination is unremarkable. His hematocrit is 32%, and hemoglobin is 9 g/dL. He undergoes a
colonoscopy, and a 14-cm segment of his sigmoid colon is shown in the photograph above.
Which of the following is the most likely diagnosis?
 Correct Answer ImageA.Adenocarcinoma of the colon
 Incorrect Answer ImageB.Crohn disease
 Incorrect Answer ImageC.Diverticulitis with perforation
 Incorrect Answer ImageD.Intussusception
 Incorrect Answer ImageE.Ulcerative colitis

A 29-year-old man comes to the physician because of vague lower back pain, mild dysuria, and a
sensation of incomplete urination for the past 2 weeks. He has seen blood in his urine during the
past 2 days which prompted him to seek medical advice. He has no past medical history and
reports a monogamous sexual relationship for the past 7 years. His temperature is 37.2°C
(99.0°F), pulse is 80/min, respirations are 12/min, and blood pressure is 130/80 mm Hg. There is
no costovertebral angle tenderness. The abdomen is soft, and the bladder edge is palpated 1 cm
below the umbilicus. There is no urethral discharge, and the testes are normal. An adequate
digital rectal examination cannot be performed because of tenderness. Gram stain of the urine
shows multiple polymorphonuclear leukocytes and intracellular gram-negative rods. Which of
the following is the most likely diagnosis?
 Incorrect Answer ImageA.Epididymitis
 Correct Answer ImageB.Prostatitis
 Incorrect Answer ImageC.Pyelonephritis
 Incorrect Answer ImageD.Ureteral obstruction
 Incorrect Answer ImageE.Urethritis
 Incorrect Answer ImageF.Urinary tract infection

The following vignette applies to the next two items. 


A 58-year-old man with a history of depression experiences a manic episode and is admitted to
the psychiatry service. He is started on lithium and his symptoms remit. He is sent home with a
prescription for lithium but two months later, the patient comes to the emergency department
because of confusion and lethargy. On examination, he is arousable but is not able to converse or
follow directions. Neurologic examination is otherwise nonfocal. His blood pressure is 110/60
mm Hg and pulse is 100/min. His mucous membranes are dry. The remainder of the examination
is normal. Laboratory studies show a serum sodium of 158 mEq/L, and a serum creatinine of 1.5
mg/dL. Which of the following is the most likely cause of these findings?
 Incorrect Answer ImageA.Central nervous system mediated diabetes insipidus
 Incorrect Answer ImageB.Interstitial nephritis
 Correct Answer ImageC.Nephrogenic diabetes insipidus
 Incorrect Answer ImageD.Nephrotic syndrome
 Incorrect Answer ImageE.Syndrome of inappropriate antidiuretic hormone
A 58-year-old man with a history of depression experiences a manic episode and is admitted to
the psychiatry service. He is started on lithium and his symptoms remit. He is sent home with a
prescription for lithium but two months later, the patient comes to the emergency department
because of confusion and lethargy. On examination, he is arousable but is not able to converse or
follow directions. Neurologic examination is otherwise nonfocal. His blood pressure is 110/60
mm Hg and pulse is 100/min. His mucous membranes are dry. The remainder of the examination
is normal. Laboratory studies show a serum sodium of 158 mEq/L, and a serum creatinine of 1.5
mg/dL. Which of the following is the most appropriate alternative therapy for his bipolar
disorder?
 Correct Answer ImageA.Carbamazepine
 Incorrect Answer ImageB.Diazepam
 Incorrect Answer ImageC.Fluoxetine
 Incorrect Answer ImageD.Haloperidol
 Incorrect Answer ImageE.Phenobarbital

The following vignette applies to the next two items. 


A 28-year-old white woman with no past psychiatric or substance abuse history just delivered
her first child with no complications. The patient, when seen on rounds, seems bizarre today. Her
husband has been making attempts for her to interact with her baby, but the patient states that she
is uninterested. When asked why she chooses not to bond with her baby, the patient replies,
"That baby is not mine and it looks like it is possessed by the devil." The patient's physical
examination and routine postpartum laboratory studies are all within normal limits. Which of the
following is the most likely diagnosis? 
 Incorrect Answer ImageA.Bipolar disorder, type I
 Incorrect Answer ImageB.Cyclothymic disorder
 Incorrect Answer ImageC.Major depressive disorder, with atypical features
 Incorrect Answer ImageD.Postpartum blues
 Correct Answer ImageE.Postpartum psychosis

A 28-year-old white woman with no past psychiatric or substance abuse history just delivered
her first child with no complications. The patient, when seen on rounds, seems bizarre today. Her
husband has been making attempts for her to interact with her baby, but the patient states that she
is uninterested. When asked why she chooses not to bond with her baby, the patient replies,
"That baby is not mine and it looks like it is possessed by the devil." The patient's physical
examination and routine postpartum laboratory studies are all within normal limits. Which of the
following is the most appropriate next step in management? 
 Incorrect Answer ImageA.Discharge her with social services followup
 Incorrect Answer ImageB.Have the patient's discharge be contingent on interaction with
baby
 Correct Answer ImageC.Obtain an immediate psychiatric consultation and evaluation for
transfer to the inpatient psychiatry unit
 Incorrect Answer ImageD.Prescribe an antidepressant before discharge
 Incorrect Answer ImageE.Schedule a followup appointment with outpatient psychiatry
after discharge
A 77-year-old man comes to the physician for six months of urinary incontinence, mild
forgetfulness, and difficulties with balance. His imbalance led to a wrist fracture last week after
falling at home. He has a history of treated tuberculosis 20 years earlier and gonorrhea while in
the military 40 years earlier. He also has hypertension and diabetes, for which he takes
propranolol, hydrochlorothiazide, and glipizide. Which of the following is the most likely cause
of his current symptoms?
 Incorrect Answer ImageA.Alzheimer’s disease
 Incorrect Answer ImageB.Chronic meningitis
 Correct Answer ImageC.Normal pressure hydrocephalus
 Incorrect Answer ImageD.Polypharmacy
 Incorrect Answer ImageE.Pseudodementia

A 4-week-old girl is brought to the physician for a well-child examination. She has been
developing well until a few days earlier when she started fussing over meals and refusing to
breastfeed. She would start nursing normally, then after a few minutes begin to cry and refuse to
continue. The parents thought that she might not be getting enough milk so they started
supplementing with formula, but that does not seem to make the feeding any easier. Otherwise,
they do not have any particular concerns. On physical examination, the infant is in no acute
distress. She watches her mother as she speaks and smiles when she looks at her. Vital signs are
within normal limits. On her oral mucosa, especially covering the tongue and palate, there are
diffuse white, adherent plaques. An inflamed base and pinpoint hemorrhages appear when the
plaques are scraped with a tongue depressor. Which of the following is the most appropriate
treatment?
 Incorrect Answer ImageA.Oral ketoconazole
 Incorrect Answer ImageB.Oral terbinafine
 Incorrect Answer ImageC.Topical gentian violet
 Correct Answer ImageD.Topical nystatin
 Incorrect Answer ImageE.Topical tretinoin

A 17-year-old girl has cystic facial acne. She recently tried topical benzoyl peroxide-clindamycin
and oral erythromycin. Her doctor recommends starting a trial of oral isotretinoin. Besides
pregnancy testing, this patient should be monitored for which of the following conditions?
 Incorrect Answer ImageA.Diabetes insipidus
 Incorrect Answer ImageB.Diabetes mellitus
 Incorrect Answer ImageC.Hyperphosphatemia
 Correct Answer ImageD.Hypertriglyceridemia
 Incorrect Answer ImageE.Thrombocytopenia

A 37-year-old sexually active woman comes to the physician because of a vaginal discharge. She
is in a monogamous heterosexual relationship and receives intramuscular depo-
medroxyprogesterone injections every 3 months for contraception. On pelvic examination, the
vaginal discharge is gray and homogeneous. The vaginal pH is 5.5. The vaginal epithelium does
not appear erythematous or edematous. A fishy odor is noted when 10% KOH is added to the
discharge on a wet mount. Which of the following is the most appropriate pharmacotherapy?
 Incorrect Answer ImageA.Amoxicillin
 Incorrect Answer ImageB.Ceftriaxone
 Incorrect Answer ImageC.Clotrimazole
 Correct Answer ImageD.Metronidazole
 Incorrect Answer ImageE.Penicillin
 Incorrect Answer ImageF.Trimethoprim-sulfamethoxazole

A 30-year-old African American woman comes to the physician with an 8-month history of
increasing fatigue, shortness of breath, and occasional cough. She is sexually active and takes no
medication except for oral contraceptives and occasional aspirin for joint pain. Examination
reveals painful red lesions on her shins, which she says appeared approximately one month ago.
Bilateral fullness at the hilar region is noted on chest x-ray. Laboratory studies show a leukocyte
count of 3,000/mm3, with 6% eosinophils, a calcium of 12 mg/dL, and an erythrocyte
sedimentation rate of 50 mm/h. Which of the following is the most likely diagnosis? 
 Incorrect Answer ImageA.Tuberculosis
 Incorrect Answer ImageB.Hodgkin disease
 Correct Answer ImageC.Sarcoidosis
 Incorrect Answer ImageD.Histoplasmosis
 Incorrect Answer ImageE.Pneumocystis carinii pneumonia

A 54-year-old man has pain and swelling in his right leg for one day. Sixteen hours ago, several
bullae formed on the leg, and now they are larger and appear necrotic. His temperature is 40.0°C
(104.0°F), blood pressure is 105/70 mm Hg, heart rate is 105/min, and respirations are 22/min.
There is tense edema up to the knee on his right leg with patches of anesthesia on the ankle and
foot. The dorsalis pedis and posterior tibial pulses are palpable, but the leg is cool to the touch.
Which of the following is the most appropriate next step in management?
 Incorrect Answer ImageA.Angiography
 Incorrect Answer ImageB.CT scan
 Incorrect Answer ImageC.MRI
 Correct Answer ImageD.Surgical exploration
 Incorrect Answer ImageE.Ultrasound

A 45-year-old man comes to the emergency department because he has had three episodes of
melena over the past 3 hours. He feels lightheaded. He has been taking 10 800-mg tablets of
ibuprofen daily for back pain. He has had no similar prior episodes and takes no other
medications. His blood pressure is 90/50 mm Hg and pulse is 120/min. Rectal examination
reveals black stool. His hematocrit is 40%. Which of the following is the most appropriate initial
management? 
 Correct Answer ImageA.Administer a fluid bolus
 Incorrect Answer ImageB.Administer vasopressors
 Incorrect Answer ImageC.Discontinue NSAIDs
 Incorrect Answer ImageD.Perform endoscopy
 Incorrect Answer ImageE.Transfuse blood

A 55-year-old man with prior alcoholism has had constant epigastric and mid-back pain for
several years. He no longer drinks, but he has a history of numerous episodes of acute
pancreatitis, and for the past several years has had diabetes and steatorrhea. He takes pancreatic
enzymes and insulin that effectively control his steatorrhea and diabetes, but his pain is
worsening. At this time, his serum and urinary amylases and lipases are all normal. An
abdominal CT scan shows a large, calcified pancreas, with dilated areas in the pancreatic duct.
An ERCP shows the pancreatic duct with intermittent dilation and strictures in a “chain of lakes”
type pattern. What is the best option for long-term pain relief in this patient? 
 Incorrect Answer ImageA.Increase his pancreatic enzyme replacement dose
 Incorrect Answer ImageB.Perform a celiac ganglion block
 Correct Answer ImageC.Side-to-side pancreaticojejunostomy
 Incorrect Answer ImageD.Stent the pancreatic duct
 Incorrect Answer ImageE.Whipple procedure (pancreaticoduodenectomy)

A 52-year-old man is brought to the emergency department complaining of nausea. He is


obviously drunk, as he smells of alcohol. The patient is well known to the emergency department
staff, as he often appears in the emergency department for one condition or another. The patient
has been evaluated for liver disease in the past and has been found to be hepatitis B and hepatitis
C seronegative, but he does have evidence of alcoholic steatosis and hepatitis confirmed by liver
biopsy. He is now complaining of severe nausea and dull but mild right upper quadrant pain that
has persisted for the last 2 days. He reports his last drink as a being a fifth of vodka earlier in the
morning, which he admits is his usual "breakfast." Vital signs are: temperature 38.0 C (100.4 F),
blood pressure 155/92 mm Hg, pulse 98/min, and respirations 20/min. Physical examination
reveals a thin man in mild distress. He has icteric sclera and dry mucous membranes. Although
lung and cardiac examinations are unremarkable, abdominal examination reveals a tender,
palpable liver 9 cm below the costal margin. Laboratory studies show: 
Which of the following is the key factor in determining whether this patient would benefit from
corticosteroids? 
 Incorrect Answer ImageA.Bilirubin level
 Incorrect Answer ImageB.Clinical jaundice
 Correct Answer ImageC.Discriminant function
 Incorrect Answer ImageD.MELD score
 Incorrect Answer ImageE.Prothrombin time

A prison psychiatrist is evaluating a 20-year-old man who comes from a middle class
background. He is incarcerated for an assault involving serious injury. The assault stemmed from
a dispute over a parking space. The patient had been enrolled in college and was a good student.
He does not drink or use any illicit drugs. He admits to having a problem "controlling his
temper" that has often led to repeated fights and verbal confrontations with strangers. Which of
the following is an appropriate treatment for his condition? 
 Incorrect Answer ImageA.Bupropion
 Incorrect Answer ImageB.Chloral hydrate
 Incorrect Answer ImageC.Diphenhydramine
 Incorrect Answer ImageD.Hydroxyzine
 Correct Answer ImageE.Propranolol

A 7-year-old girl undergoes a hepatectomy and orthotopic cadaveric hepatic transplant to treat
end-stage liver disease secondary to biliary atresia. She received a left lateral hepatic segment
from an adult donor and is recovering well until the third postoperative day when she develops
worsening abdominal pain. Her temperature is 37.1°C (98.8°C), pulse is 110/min, respiratory
rate is 24/min, and blood pressure is 130/78 mm Hg. Oxygen saturation is 99% on room air. On
physical examination, she is uncomfortable, but otherwise her cardiovascular and respiratory
examinations are normal. Abdominal examination shows a clean, midline incision with
abdominal distention and generalized tenderness to palpation. Laboratory studies show:
Hematocrit 45%
White blood cells 6,500/mm3
Platelets 250,000/mm3
Na+ 140 mEq/dL
K+ 4.0 mEq/dL
Cl–
103 mEq/dL
HCO3– 24 mEq/dL
Urea nitrogen 12 mEq/dL
Creatinine 1 mg/dL
Bilirubin, serum 1.5 mg/dL
Bilirubin, direct 0.5 mg/dL
ALT 1,080 U/L
AST 1,070 U/L
ALP 300 U/L
Prothrombin time 11 sec
INR 1.0
Which of the following is the most appropriate next step in management?
 Incorrect Answer ImageA.Doppler studies to rule out hepatic artery thrombosis
 Correct Answer ImageB.Doppler studies to rule out portal vein thrombosis
 Incorrect Answer ImageC.Liver biopsy to investigate for acute rejection
 Incorrect Answer ImageD.Liver biopsy to investigate for hyperacute rejection
 Incorrect Answer ImageE.Ultrasound to rule out biliary obstruction

A 62-year-old man undergoes a total laryngectomy for laryngeal cancer. He tolerates the
procedure without difficulty and is admitted postoperatively to the otolaryngology floor. He has
been receiving intravenous morphine as needed and an intravenous antibiotic every 8 hours. On
postoperative day 1, he has no complaints except for some moderate discomfort. He has a
temperature of 38.8°C (101.8°F) but otherwise, his vital signs are stable. He has been able to
void postoperatively and his urine output is within normal limits. A neck drain has produced 30
ccs of serosanguinous fluid overnight. Physical examination shows the patient to be awake and
alert. His heart sounds are regular. Lung examination shows slightly diminished breath sounds,
but no crackles, bilaterally. Neck examination shows a widely patent stoma and an incision with
sutures and drain in place. There is no erythema, fluctuance, or drainage at the incision site. He
has been out of bed one time but has not begun routine ambulation. Which of the following is the
most likely cause of this patient's fever?
 Correct Answer ImageA.Atelectasis
 Incorrect Answer ImageB.Deep venous thrombosis
 Incorrect Answer ImageC.Intravenous morphine
 Incorrect Answer ImageD.Surgical wound infection
 Incorrect Answer ImageE.Urinary tract infection
A 24-year-old woman, gravida 1, para 0, at 38 weeks' gestation is admitted to the hospital for
induction of labor. At a routine prenatal visit, she was found to have a blood pressure of 160/100
mm Hg and 3+ proteinuria. A follow-up blood pressure of 162/104 mm Hg was obtained and she
was diagnosed with preeclampsia and sent to the hospital for induction of labor. Prior to this
diagnosis, her prenatal course was uncomplicated and she has no medical or surgical history.
Physical examination shows the cervix is long, thick, and closed. The fetal heart rate is 135/min
and reactive, and no contractions are seen on monitoring. Misoprostol is placed intravaginally for
cervical ripening. One hour later, the patient is noted to be contracting every 1–2 minutes and the
fetal pulse decreases to 80/min for 3 minutes. Which of the following is the most likely
diagnosis?
 Incorrect Answer ImageA.Placental abruption
 Incorrect Answer ImageB.Prelabor rupture of membranes
 Incorrect Answer ImageC.Umbilical cord prolapse
 Correct Answer ImageD.Uterine hyperstimulation
 Incorrect Answer ImageE.Uterine rupture

A 44-year-old woman is brought to the emergency department because of abdominal pain and
vaginal spotting. She has been spotting since last night and her pain began earlier in the day with
a sudden onset of right lower quadrant pain. She has a medical history significant for multiple
episodes of pelvic inflammatory disease (PID). She has never had surgery. Physical examination
shows right lower quadrant tenderness to palpation with rebound tenderness. Speculum
examination shows scant blood in the vagina. Laboratory evaluation shows a serum β-hCG level
of 2,100 mU/mL. Transvaginal ultrasound shows a right adnexal mass with significant free fluid
in the abdomen and an empty uterus. The decision is made to bring the patient to the operating
room. Persistent trophoblastic tissue is most likely to result from which of the following surgical
management strategies?
 Incorrect Answer ImageA.Salpingectomy
 Incorrect Answer ImageB.Salpingo-oophorectomy
 Correct Answer ImageC.Salpingostomy
 Incorrect Answer ImageD.Supracervical hysterectomy, bilateral salpingo-oophorectomy
 Incorrect Answer ImageE.Total abdominal hysterectomy, bilateral salpingo-
oophorectomy

A 43-year-old man reports that he had a 9-lb weight loss over the past nine months. The
symptoms are accompanied by difficulty swallowing both solids and liquids. He has woken on
several occasions at approximately 4 AM regurgitating partially digested foods. An upper
gastrointestinal series is performed and reveals a widely dilated esophagus with a smoothly
tapering distal esophagus. There appears to be partially digested food present in the esophagus.
Which of the following is the most likely cause of this patient's symptoms?
 Correct Answer ImageA.Achalasia
 Incorrect Answer ImageB.Diffuse esophageal spasm
 Incorrect Answer ImageC.Esophageal squamous cancer
 Incorrect Answer ImageD.Peptic stricture
 Incorrect Answer ImageE.Scleroderma
A 59-year-old man comes to the physician because of shortness of breath. The patient has been
coming to this physician for many years for treatment of osteoarthritis. He has had 5 days of
fever, chills, and productive cough. He has not been hospitalized recently and lives at home with
his wife. The patient reports slowly worsening and progressive shortness of breath even with
minimal exertion. There is no associated chest pain or pressure. He routinely takes only a
nonsteroidal anti-inflammatory agent for pain. He has a long smoking history of greater than 150
pack years. The patient undergoes pulmonary function testing. Which of the following are the
most representative results for this patient with moderate chronic obstructive pulmonary disease?
 Incorrect Answer ImageA.FEV1 2.1 L, FEV1/FVC 0.9, residual volume 2.0 L, total lung
capacity 7 L
 Correct Answer ImageB.FEV1 0.9 L, FEV1/FVC 0.7, residual volume 3.5 L, total lung
capacity 9 L
 Incorrect Answer ImageC.FEV1 2.6 L, FEV1/FVC 0.8, residual volume 1.3 L, total lung
capacity 5 L
 Incorrect Answer ImageD.FEV1 1.3 L, FEV1/FVC 0.9, residual volume 0.9 L, total lung
capacity 9 L
 Incorrect Answer ImageE.FEV1 0.9 L, FEV1/FVC 0.9, residual volume 1.0 L, total lung
capacity 12 L

A 6-month-old previously healthy child is brought to the physician because of sudden onset of
lethargy, constipation, generalized weakness, and poor feeding. He has been meeting all
development milestones and his immunization schedule is up to date. On further questioning, his
parents mention that his diet was recently advanced to a homemade formula of evaporated milk
and honey. On physical examination, he has stable vital signs, his lungs are clear to auscultation,
he has normal skin turgor and full fontanelles. Which of the following is the most likely
explanation for the patient's presentation?
 Correct Answer ImageA.Botulism
 Incorrect Answer ImageB.Hirschsprung disease
 Incorrect Answer ImageC.Hypernatremia
 Incorrect Answer ImageD.Hyponatremia
 Incorrect Answer ImageE.Hypothyroidism

A 12-year-old girl comes to the physician for an annual examination. She has been in good
health for the past year and has no complaints. She began having menses this year and, after a
few irregular cycles, is now having a monthly period. Past medical history is significant for
multiple episodes of otitis media as a child. She takes no medications and has no known drug
allergies. Past surgical history is unremarkable. Vital signs are within normal limits and physical
examination is unremarkable. Which of the following is the most appropriate next step in
management?
 Correct Answer ImageA.Human Papillomavirus (HPV) immunization
 Incorrect Answer ImageB.Immunization against HIV
 Incorrect Answer ImageC.Japanese encephalitis virus immunization
 Incorrect Answer ImageD.Rabies virus immunization
 Incorrect Answer ImageE.Salmonella typhi immunization
A 7-year-old boy is referred by his school for psychiatric evaluation. The teachers have noticed
that, in the past year, he has been unable to sustain attention in class, and has been fidgeting and
talking to his peers during class. He seems unable to wait for others to finish speaking and keeps
interrupting and blurting out answers before questions are completed. At home, his parents state
that he is forgetful, loses things easily and often gets injured while playing. Which of the
following is the most likely diagnosis? 
 Correct Answer ImageA.Attention-deficit/hyperactivity disorder (ADHD)
 Incorrect Answer ImageB.Bipolar disorder
 Incorrect Answer ImageC.Conduct disorder
 Incorrect Answer ImageD.Posttraumatic stress disorder
 Incorrect Answer ImageE.Rett syndrome

A 25-year-old man is brought to the emergency department because he was involved in a high-
speed motor vehicle collision. He was an unrestrained, front-seat passenger and has signs of
moderate respiratory distress. His pulse is 110/min, respirations are 26/min, and blood pressure is
120/80 mm Hg. Physical examination shows no breath sounds in the left hemithorax. A chest x-
ray film shows a collapsed left lung and multiple air-fluid levels filling the left pleural cavity. A
nasogastric tube that had been placed prior to taking the film is seen on the radiograph as
reaching the upper abdomen and then curling up into the left chest. Which of the following is the
most likely diagnosis?
 Incorrect Answer ImageA.Blow out of pulmonary blebs
 Incorrect Answer ImageB.Esophageal rupture or perforation
 Correct Answer ImageC.Left diaphragmatic rupture
 Incorrect Answer ImageD.Left hemopneumothorax
 Incorrect Answer ImageE.Major injury to the tracheobronchial tree

A 48-year-old man comes to a primary care physician because of a one-week history of


symptoms consistent with pneumonia. Since this is the patient's first visit to the clinic, the
physician gathers a full history for a new patient assessment. The patient has no significant past
medical, surgical, or psychiatric history. Family history is significant for a brother and an uncle
with schizophrenia. Social history reveals that the patient lives alone, has minimal contact with
family, and describes no real social activities or friends. When questioned about this, he states,
"I've never been much interested in my family or being around people". The patient has worked
delivering newspapers for the past 15 years. He has not dated since having one girlfriend in the
11th grade. During the interview, though he seems emotionally detached, he denies depressive
symptoms or psychotic symptoms. Which of the following is the most appropriate psychiatric
diagnosis?
 Incorrect Answer ImageA.Avoidant personality disorder
 Incorrect Answer ImageB.Other psychotic disorder
 Correct Answer ImageC.Schizoid personality disorder
 Incorrect Answer ImageD.Schizophrenia
 Incorrect Answer ImageE.Social anxiety disorder

A 22-year-old man comes to the emergency department because of dyspnea, palpitations, and a
headache. These symptoms came on soon after he took trimethoprim-sulfamethoxazole for a
urinary tract infection. Laboratory studies show a normochromic, normocytic anemia. A
peripheral blood smear reveals Heinz bodies. Which of the following is the most likely cause of
this patient's anemia? 
 Incorrect Answer ImageA.Lead poisoning
 Incorrect Answer ImageB.Folate deficiency
 Correct Answer ImageC.Glucose-6-phosphate dehydrogenase deficiency
 Incorrect Answer ImageD.Hereditary spherocytosis
 Incorrect Answer ImageE.Occult blood loss

A young mother notices that her 4-week-old son's right foot is not as flexible as the left one, and
tends to assume an awkward position at rest. This was present when the baby was born but she
did not make much of it, as she believed it would resolve with time. She subsequently found out
about two family members on her husband's side that had similar problems at birth and required
surgical treatment at an early age. Physical examination shows a thriving male infant. He weighs
5.1 kg (11 lb 4 oz) and is 60 cm (23.6 in) in length. His vital signs are normal and he is in no
acute distress. His right foot is found to be slightly smaller than the left one, medially rotated,
and very stiff. There is also medial rotation of the heel. Which of the following is the most likely
diagnosis? 
 Incorrect Answer ImageA.Femoral anteversion
 Incorrect Answer ImageB.Metatarsus adductus
 Incorrect Answer ImageC.Pes planus
 Correct Answer ImageD.Talus equinovarus
 Incorrect Answer ImageE.Tibial torsion

A 25-year-old woman, gravida 2, para 2, comes to the physician for a postpartum visit. Six
weeks ago she had a normal vaginal delivery of a 3.4 kg (7 lb 7 oz) boy. Her antepartum course
was uncomplicated. She has no complaints and states that she is feeling well. She has no medical
problems and has never had surgery. Physical examination is entirely normal. She wants to know
how long it is recommended that she wait until trying to conceive again. She wants to know if
there is any period of waiting that is associated with improved outcomes in the next pregnancy.
Which of the following is the most appropriate advice to give this patient? 
 Incorrect Answer ImageA."There is no association between pregnancy interval and
outcomes."
 Incorrect Answer ImageB."Two children are enough and you should undergo
sterilization."
 Correct Answer ImageC."Waiting 18 months is associated with better outcomes."
 Incorrect Answer ImageD."You should try to conceive in the next 6 months."
 Incorrect Answer ImageE."You should wait at least 60 months before trying to conceive
again."

A 72-year-old man comes to the physician because he can feel a mass behind his right knee.
Physical examination discloses the presence of a 3.5-cm pulsatile mass in his right popliteal area
and a 3-cm pulsatile mass in his left popliteal area. Except for significant obesity with a large
girth, the rest of his physical examination is non-contributory. Which of the following is the most
appropriate next step in management?
 Correct Answer ImageA.Abdominal CT scan
 Incorrect Answer ImageB.Arteriogram
 Incorrect Answer ImageC.Continued observation
 Incorrect Answer ImageD.Elective surgical repair of both popliteal artery aneurysms
 Incorrect Answer ImageE.Long-term course of intravenous antibiotics

A 70-year-old man is brought to the emergency department at 6:20 PM. The family indicates that
at 6 PM this evening he suddenly developed paralysis of his left leg and weakness of his left arm.
He was fully conscious and able to speak, and he has remained that way. In response to specific
questioning, he indicates that he did not experience a headache either before or during this
episode. Neurologic examination reveals flaccid paralysis of his left lower extremity and paresis
of his left upper extremity. The brief clinical examination is completed by 6:35 PM. He indicates
that both of those deficits are neither better nor worse now than they were when he first
experienced them. Which of the following is the most appropriate next step in management?
 Incorrect Answer ImageA.Anticoagulation with heparin
 Incorrect Answer ImageB.Arteriogram of the neck vessels
 Incorrect Answer ImageC.Duplex ultrasound of the carotid vessels
 Incorrect Answer ImageD.Infusion of thrombolytic agents
 Correct Answer ImageE.Noncontrast CT scan of the head

A 23-year-old woman and her 3-year-old son are brought to the emergency department at 2 a.m.
by paramedics after being involved in a motor vehicle accident. The mother was driving and
apparently swerved off the side of the road and hit a guardrail on the local highway. She did not
have the seat belt fastened and sustained a head laceration and blunt chest trauma, while the
child, who was seated in the back, did not appear to be injured. The mother appears unkempt,
with long, tangled hair and soiled clothing. She is confused and has trouble giving information
but the paramedics report that they managed to understand that the child has a hearing problem.
The nurse reports the child is 85 cm (33.5 in) tall and weighs 11 kg (24.2 lb). He is very active,
but has trouble standing without help and does not react accordingly to verbal instruction. When
handed the otoscope he grabs it weakly with both hands but then drops it on the floor. On
examination, the child has an abnormally small head. He has short palpebral fissures, a short,
upturned nose and maxillary hypoplasia. He has a smooth philtrum and a thin upper lip. There is
a linear vertical scar parallel to the philtrum on the left side of the upper lip that indicates he had
a cleft lip at birth. Which of the following agents is most likely responsible for these
malformations if used by the mother during her pregnancy?
 Correct Answer ImageA.Alcohol
 Incorrect Answer ImageB.Hydantoin
 Incorrect Answer ImageC.Retinoic acid
 Incorrect Answer ImageD.Trimethadione
 Incorrect Answer ImageE.Valproate

A 36-year-old homeless man comes to the physician for a follow-up after being discharged for
severe pneumonia and altered mental status 2 weeks ago. During his admission, he was
diagnosed with diabetes based on his elevated blood glucose levels and mild ketonuria post-
recovery. He was started on metformin upon being discharged. Today, the patient reports feeling
tired, easily confused, and having occasional blurred vision. He also reports increased hunger,
thirst, and urination. Past medical history is unremarkable, and he does not report any previous
hospitalizations. Family history is significant for type 1 diabetes mellitus in his maternal uncle.
Vital signs are normal. Physical examination shows a thin man in no apparent distress and is
otherwise unremarkable. A dipstick urinalysis shows mild ketones and moderate glucose. Which
of the following can help differentiate type 1 from type 2 diabetes in this patient?
 Correct Answer ImageA.Anti-glutamic acid decarboxylase (GAD) antibodies
 Incorrect Answer ImageB.Anti-insulin receptor antibodies
 Incorrect Answer ImageC.Glucokinase gene testing
 Incorrect Answer ImageD.Serum C-peptide concentrations
 Incorrect Answer ImageE.Serum Insulin levels

A 43-year-old man comes to the physician because of an uncontrollable urge to move his legs.
He reports feeling a periodic ‘creepy-crawly’ sensation in his left leg and foot which relieves
with the movement of the limb. He denies numbness and pain in his left leg and foot. His past
medical history is significant for sleep apnea, diagnosed 3 years ago. His family history is
unremarkable for any neurologic disease. His temperature is 37.1°C (98.8°F), pulse is 80/min,
respirations are 12/min, and blood pressure is 130/80 mm Hg. Physical examination is
unremarkable. Which of the following is the most appropriate next step in management?
 Correct Answer ImageA.Complete blood count and iron panel
 Incorrect Answer ImageB.Empiric trial of oral quinine at bedtime
 Incorrect Answer ImageC.Full electrolyte and chemistry panel
 Incorrect Answer ImageD.Genetic analysis for trinucleotide repeats
 Incorrect Answer ImageE.Sleep study with an apnea-hypopnea index

A 25-year-old man comes to the physician for a new patient evaluation and is found to have a
continuous, rumbling murmur at the second left intercostal space, radiating to the back. He
denies any complaints of fever, chills, shakes, shortness of breath, dyspnea on exertion, or chest
pain. He has no history of intravenous drug use. He denies any history of rheumatic fever as a
child. Which of the following is the most likely diagnosis? 
 Incorrect Answer ImageA.Aortic regurgitation
 Incorrect Answer ImageB.Aortic stenosis
 Incorrect Answer ImageC.Hypertrophic obstructive cardiomyopathy
 Incorrect Answer ImageD.Mitral regurgitation
 Correct Answer ImageE.Patent ductus arteriosus

A 29-year-old woman comes to the physician to discuss the results of her Pap smear. Her report
indicates that the smear was adequate, and classified as showing atypical glandular cells (AGS).
She reports feeling well. Her menstrual cycle is regular and her last menstrual period was two
weeks ago. Past medical history is significant for vulvar warts and multiple episodes of pelvic
inflammatory disease (PID). She is currently divorced, works as a bartender, and has a steady
relationship with her boyfriend. She has never used any contraception. At age 23, she was
incarcerated for four years after being arrested for prostitution and possession of controlled
substances. She smokes two packs of cigarettes daily and drinks alcoholic beverages socially but
does not use any illicit drugs. She is not taking any medications currently and is allergic to
penicillin. Vital signs are normal. Pelvic examination shows a small amount of yellow dried
discharge on the fourchette, inflamed vagina with a moderate amount of gray exudate, and
nulliparous, pink cervix with mucoid discharge in the os. There is no cervical motion tenderness
and adnexal tenderness or mass. The uterus is anteverted, anteflexed, and normal in size, shape,
and position. Microscopic evaluation of vaginal secretions shows many clue cells, but otherwise
is negative for common vaginal pathogens. Which of the following is the next best step in
management? 
 Incorrect Answer ImageA.Cervical conization
 Correct Answer ImageB.Culture the endocervical secretions for gonorrhea and
chlamydia; order serologic studies for syphilis, HIV, and hepatitis B
 Incorrect Answer ImageC.Prepare a Gram stain of the cervical secretions; treat if positive
for gonorrhea
 Incorrect Answer ImageD.Repeat the Pap smear
 Incorrect Answer ImageE.Schedule a colposcopy with biopsy, perform an endocervical
curettage and endometrial sampling

A 58-year-old man comes to the emergency department because of severe chest pain for the last
40 minutes. The pain began at work during a stressful meeting. He had a non-Q wave myocardial
infarction three years ago. He has a 10 pack/year tobacco history and a family history of
coronary disease. He is diaphoretic, and an electrocardiogram shows ST segment depressions in
V1–V6. Sublingual nitroglycerin is administered and relieves the pain in approximately 20
minutes. He is admitted to the hospital for further evaluation and therapy. Which of the
following is the most appropriate management? 
 Incorrect Answer ImageA.Electrolytes, blood urea nitrogen, and creatinine
 Correct Answer ImageB.Serial creatine phosphokinase with MB isoforms
 Incorrect Answer ImageC.Single creatine phosphokinase with MB isoform
 Incorrect Answer ImageD.Single lactate dehydrogenase and aspartate transaminase
 Incorrect Answer ImageE.Single troponin I

A 22-year-old woman, gravida 1, para 0, at 20 weeks' gestation is undergoing a sonographic fetal


survey to identify fetal anomalies. She has had type 1 diabetes mellitus since age 11 and has been
giving herself twice-daily subcutaneous insulin injections. Her attention to blood glucose control
has been sporadic at best. She has a glucose meter at home but uses it infrequently. Her
hemoglobin A1c level on her first prenatal visit 8 weeks ago was 14%. A fetal anomaly is found
on the sonogram. Which of the following is the most likely anomaly found?
 Incorrect Answer ImageA.Duodenal atresia
 Incorrect Answer ImageB.Radial aplasia
 Incorrect Answer ImageC.Sacral agenesis
 Correct Answer ImageD.Transposition of the great vessels
 Incorrect Answer ImageE.Unilateral renal agenesis

An 18-year-old mother brings her 5-week-old first born son to the office because of extensive
redness and scaling that have developed over the infant's face and scalp in the past 14 days. The
pregnancy was uneventful and the baby was born vaginally at full term, with Apgar scores of 9
and 9 at 1 and 5 minutes, respectively. There is no family history of skin disease. On physical
examination, the scalp is diffusely pink with abundant adherent epithelial debris, especially thick
and scaly on the parietal scalp and vertex. The eyebrows and glabella are erythematous and with
scant greasy yellow scale. The rest of the physical examination is normal. Which of the
following is the most appropriate prophylactic measure to prevent this disorder? 
 Incorrect Answer ImageA.Application of topical antifungal gel to the scalp daily for the
first month of life
 Incorrect Answer ImageB.Application of topical corticosteroid cream to the scalp daily
for the first 6 weeks of life
 Incorrect Answer ImageC.Avoiding washing of the scalp and face with anything but
clean water
 Correct Answer ImageD.Daily application of baby oil and gentle washing with baby
shampoo
 Incorrect Answer ImageE.There is really nothing that can be done to prevent this from
developing

A 71-year-old white man has a 10-year history of an asymptomatic, slowly enlarging “bump” on
his nose. He has worked as a farmer his entire life. He has a history of type 2 diabetes mellitus
controlled with diet and exercise. Physical examination shows a 7-mm pearly, telangiectatic,
dome-shaped papule on his nose. Which of the following is the most likely diagnosis? 
 Incorrect Answer ImageA.Acne rosacea
 Incorrect Answer ImageB.Actinic keratosis
 Correct Answer ImageC.Basal cell carcinoma
 Incorrect Answer ImageD.Seborrheic keratosis
 Incorrect Answer ImageE.Squamous cell carcinoma

A 17-year-old young woman comes to her physician because of general weakness, malaise and
decreased appetite for the last 3 months. The patient also reports generalized, mild skin itching.
The patient does not have any significant past medical problems and denies any recent travel or
contacts with sick people. Her temperature is 37.2°C (98.9°F), pulse is 76/min, blood pressure is
130/75 mm Hg, and respirations are 16/min. Examination of her eyes, ears, nose, throat, and
neck is normal, except presence of mild scleral and cutaneous jaundice. Lungs are clear to
auscultation. Heart has a regular rate and rhythm with no murmurs. Abdomen is soft, non-tender,
mildly distended with normoactive bowel sounds. Rectal examination shows normal rectal tone
and guaiac negative brown stool. Liver function testing shows an elevation of liver enzymes 10
times above normal values. Serology for hepatitis A, B and C is negative. Serological testing is
positive for the presence of anti-smooth muscle antibodies. Which of the following is the most
appropriate next step in management for this patient? 
 Incorrect Answer ImageA.Alpha interferon
 Incorrect Answer ImageB.Lamivudine
 Incorrect Answer ImageC.Liver transplant
 Incorrect Answer ImageD.No treatment necessary
 Correct Answer ImageE.Prednisone and azathioprine

A 32-year-old woman is brought to the emergency department after a motorcycle accident. She
opens her eyes in response to verbal commands and responds to questions appropriately. Her
abdomen is distended, and she complains of pain in her right upper quadrant. X-rays show a left
femoral fracture and multiple pelvic fractures, but no rib or cervical spine fractures. Laparotomy
reveals a nonbleeding laceration of the liver, which is summarily drained. Postoperatively, the
patient is stable for 48 hours, and then becomes anxious and develops progressively shallow
respirations and tachypnea. Her temperature is 38.2°C (100.8°F). She denies chest pain. She is
neurologically intact. Which of the following is the next step in the diagnosis? 
 Correct Answer ImageA.Arterial blood gas
 Incorrect Answer ImageB.Chest radiography
 Incorrect Answer ImageC.CT scan of the chest
 Incorrect Answer ImageD.Electrocardiograph
 Incorrect Answer ImageE.Ultrasound of the lower extremities

A 32-year-old woman is brought to the emergency department after a motorcycle accident. She
opens her eyes in response to verbal commands and responds to questions appropriately. Her
abdomen is distended, and she complains of pain in her right upper quadrant. X-rays show a left
femoral fracture and multiple pelvic fractures, but no rib or cervical spine fractures. Laparotomy
reveals a nonbleeding laceration of the liver, which is summarily drained. Postoperatively, the
patient is stable for 48 hours, and then becomes anxious and develops progressively more
shallow respirations and tachypnea. Her temperature is 38.2°C (100.8°F). She denies chest pain.
She is neurologically intact. Which of the following is the most appropriate next step in
management? 
 Incorrect Answer ImageA.Digitalization
 Incorrect Answer ImageB.Exploratory laparotomy
 Incorrect Answer ImageC.Heparinization
 Incorrect Answer ImageD.Insertion of a chest tube
 Correct Answer ImageE.Intubation and ventilation

A 15-year-old girl comes to the emergency department because of the sudden onset of watery
diarrhea tinged with blood. The girl was previously healthy and she has not been exposed to any
uncooked meat or eaten any unusual foods. Her only medications are topical benzoyl peroxide
and oral clindamycin for acne vulgaris. Her temperature is 38.1°C (100.5°F). Physical
examination shows a slightly distended abdomen that is diffusely tender. Which of the following
is the most likely diagnosis?
 Incorrect Answer ImageA.Gastroenteritis
 Incorrect Answer ImageB.Irritable bowel syndrome
 Correct Answer ImageC.Pseudomembranous colitis
 Incorrect Answer ImageD.Salmonella infection
 Incorrect Answer ImageE.Ulcerative colitis

A 72-year old man was brought to the emergency department because of a recent onset of
blurred vision, headache and accompanying nausea and vomiting. The patient also reports
lightheadedness and general weakness for the last few hours. The patient has hypertension,
coronary artery disease and hyperlipidemia. However, he denies chest pain and shortness of
breath at this time. The patient usually takes nitroglycerin, aspirin, digoxin, lasix and lipitor. He
recently started taking lisinopril as well. He does not have any allergies. His vitals are
temperature 37.0° C (98.6° F), pulse 46/min, blood pressure 85/60 mm Hg, and respirations
20/min. Physical examination is significant mild ankle edema but no cyanosis or clubbing. Skin
is pale and dry with slightly decreased turgor. On neurological examination the patient appears to
be confused but has no focal findings. Laboratory studies and an electrocardiogram (ECG) are
shown.
Sodium 136 mmol/L
Potassium 3.3 mmol/L
Chloride 98 mmol/L
Carbon dioxide 25 mmol/L
Blood urea nitrogen 28 mg/dL
Creatinine 1.5 mg/dL
Glucose 102 mg/dL
Calcium 8.8 mg/mL
Which of the following is the most appropriate next step in management? 
Digoxina intoxication
 Incorrect Answer ImageA.Activated charcoal PO
 Incorrect Answer ImageB.Digoxin-specific antibody Fab fragments
 Incorrect Answer ImageC.Electrical pacing
 Incorrect Answer ImageD.Lidocaine IV
 Incorrect Answer ImageE.Order cardiac enzymes
 Incorrect Answer ImageF.Order plasma digoxin level
 Correct Answer ImageG.Oxygen, cardiac monitoring, and IV access
 Incorrect Answer ImageH.Potassium chloride IV

A 39-year-old stockbroker comes to the emergency department complaining of sudden and


severe chest pain that began 40 minutes ago and is radiating to the jaw and left shoulder. He
denies any previous episodes and has been treated for hypertension with lisinopril. The patient is
diaphoretic and appears moderately uncomfortable. His temperature is 37.2°C (99.0℉), pulse is
62/min, respirations are 16/min, and blood pressure is 110/70 mm Hg. There is no jugular venous
distension, and his lungs are clear. His heart rhythm is regular, with a grade 1/6 systolic murmur.
There is no peripheral edema. Laboratory studies show a leukocyte count of 11,800/mm3,
troponin of 2.0 ng/mL, and hemoglobin of 15 g/dL. Electrocardiography demonstrates 3 mm ST-
segment depression in leads II, III, and aVF. Which of the following arteries is most likely
affected?
 Incorrect Answer ImageA.Left anterior descending artery
 Incorrect Answer ImageB.Left circumflex artery
 Incorrect Answer ImageC.Left main coronary artery
 Correct Answer ImageD.Right coronary artery

A 62-year-old woman is being treated for depression with psychotic features. She has received
treatment with amitriptyline, thioridazine, and benztropine. After several days of treatment,
during which she seems to be slowly improving, she suddenly becomes agitated and shouts at
frightening images she says are “coming out of the wallpaper patterns”. She cannot identify the
time of the day or the day of the week. She is febrile and flushed, her mouth is dry, and her lips
are cracked. Which of the following is the most appropriate therapy? 
 Incorrect Answer ImageA.Bromocriptine
 Incorrect Answer ImageB.Methohexital
 Incorrect Answer ImageC.Phentolamine
 Correct Answer ImageD.Physostigmine
 Incorrect Answer ImageE.Succinylcholine

A 35-year-old man is involved in a freeway accident in which he was the driver. Many
automobiles telescoped into one another. He was wearing a seat belt at the time of impact. In the
emergency department, the patient is conscious and oriented to time, person, and place. He
seems to be remarkably free of symptoms and wants to go home. Vital signs are: blood pressure
110/60 mm Hg, pulse 100/min, and respirations 22/min. On clinical examination, there was no
alcoholic odor in his breath. His pupils are equal and reacting to light and accommodation, and
no cranial nerve abnormality is found. The trachea is deviated slightly to the right. No
tenderness, bruises, or masses are found in the neck. He had a swelling over the right clavicle,
which was nontender. On further questioning, he states that this is an old fracture of the clavicle,
which occurred a few years previously while playing football. There is a small bruise over the
sternum and tenderness over the same area. Chest expansion is equal, and air entry in both lungs
normal. The point of maximal impulse (PMI) is in the left 5th intercostal space, a little lateral to
the mid-clavicular line. Heart sounds are normal. No added sounds are noted. Examination of the
abdomen reveals no bruises, tenderness, guarding, or rigidity. The liver and spleen are not
palpable, and bowel sounds are present. Examination of the extremities reveals no fractures or
abnormalities. Radiology of the chest reveals a widened mediastinum. Which of the following is
the most appropriate next step in the diagnosis? 
 Incorrect Answer ImageA.Angiography of the thoracic aorta
 Incorrect Answer ImageB.Computerized tomography
 Incorrect Answer ImageC.Magnetic resonance imaging
 Incorrect Answer ImageD.Radionuclide angiography
 Correct Answer ImageE.Transesophageal echo

A 45-year-old woman, gravida 2, para 2, two years post tubal ligation, comes to the physician
because of a sudden onset of right- lower-quadrant pain. For the last year, her menses have come
at irregular intervals, and her last normal period was three months ago. She has no other
gastrointestinal or genitourinary complaints, and her surgical history is only positive for the
sterilization operation. Vital signs are within normal limits. Pertinent physical findings are
limited to the abdomen and pelvis. Physical examination of the abdomen shows a well-healed
small midline scar between the pubis and umbilicus. There is no palpable organopathy, but there
is diffuse tenderness in both lower abdominal quadrants. Pelvic examination shows the vulva and
vagina within normal limits; a small amount of dark brown discharge in the vagina coming from
a closed cervical os; the uterus is normal in size, shape, and position; diffuse tenderness in both
adnexal regions, and voluntary guarding prevents a complete examination. Hemoglobin is 12
g/dL, and a urinalysis is negative for glucose, albumin, acetone, and formed elements. Pelvic
ultrasound examination shows a normal uterus with thin endometrium, a complex mass on the
right measuring 5 cm in length, and a small amount of fluid in the cul-de-sac. Which of the
following is the most likely diagnosis?
 Incorrect Answer ImageA.Acute appendicitis
 Incorrect Answer ImageB.Adnexal torsion
 Correct Answer ImageC.Ectopic pregnancy
 Incorrect Answer ImageD.Tubo-ovarian abscess
 Incorrect Answer ImageE.Ureteral calculus

A 22-year-old man is hit repeatedly over the head with a baseball bat in a bar-room brawl. The
assailant escapes. When the paramedics arrive they find that the man is unresponsive, without
spontaneous respiration, and pulseless. Which of the following is necessary to make a diagnosis
of brain death in this man? 
 Incorrect Answer ImageA.Demonstration of asystole on electrocardiography
 Correct Answer ImageB.Failure of the vestibulo-ocular reflex to elicit eye movements
 Incorrect Answer ImageC.He may have a respiratory rate of 4 per minute
 Incorrect Answer ImageD.If he has a history of recent alcohol use an alcohol level must
be obtained
 Incorrect Answer ImageE.Transient nystagmus with bilateral ear irrigation

A 79-year-old woman with osteoarthritis comes to the physician because of fatigue, arthralgias,
and constipation for 3 months. She smokes one pack of cigarettes per day. She does not take any
medications. Her blood pressure is 96/78 mm Hg and pulse is 60/min. She weighs 60 kg and is
160 cm tall. She has thinning of the outer half of the eyebrows and delayed return of deep tendon
reflexes. The remainder of the physical examination is normal. Laboratory studies show: 
Hemoglobin 14.0 g/dL
Total cholesterol 240 mg/dL
Sodium 130 mEq/L
Creatine kinase 700 U/L
CK-MM 680 U/L
CK-MB 20 U/L
CK-BB undetectable
Which of the following is the most likely cause of the patient's elevated creatine kinase? 
 Incorrect Answer ImageA.Excessive exercise
 Correct Answer ImageB.Hypothyroidism
 Incorrect Answer ImageC.Intracerebral hemorrhage
 Incorrect Answer ImageD.Myocardial infarction
 Incorrect Answer ImageE.Polymyositis

An 18-month-old boy is brought to the office because of a new onset rash that is worsening. The
boy had just gotten over an upper respiratory tract infection a week earlier when "little red
bumps" started showing up first on his feet and then spreading slowly up his legs all the way to
his buttocks. In addition, he has had colicky cramps for the past 24 hours that have kept them all
up for the night. The boy was in good health and had no medical problems before this episode.
His immunizations are up to date and he has been developing appropriately. There are no known
allergies to medications and he is not taking any medications at this time. The upper respiratory
tract infection was treated symptomatically with over-the-counter cold and cough medicine. On
physical examination the patient is not in any distress. On the lower extremities and buttocks
there are multiple nonblanchable skin lesions as shown in the photograph. Palpation of the
abdomen reveals diffuse mild tenderness. His stool is positive for occult blood.Which of the
following is the most likely cause of this patient's skin lesions? 
 Correct Answer ImageA.Beta-hemolytic Streptococcus group A
 Incorrect Answer ImageB.Coxsackie A16
 Incorrect Answer ImageC.Haemophilus influenzae
 Incorrect Answer ImageD.Herpes simplex virus
 Incorrect Answer ImageE.Staphylococcus aureus

A 76-year-old man with a past medical history of hypertension and cigarette smoking comes to
the physician because of masses on both sides of his face. One mass is in front of the right
earlobe, and another mass on the left is located slightly below the angle of the mandible. The
masses have been present for many years and are asymptomatic, freely movable, and about 2 cm
in diameter. The patient has intact function of both facial nerves and no enlarged cervical nodes.
Which of the following is the most likely diagnosis? 
 Incorrect Answer ImageA.Chronic parotitis
 Incorrect Answer ImageB.Lymphoma
 Incorrect Answer ImageC.Parotid cylindromas
 Incorrect Answer ImageD.Pleomorphic adenomas
 Correct Answer ImageE.Warthin tumors

A 29-year-old woman comes to the clinic for evaluation of excessive body hair. Since the onset
of puberty she has had increasing amounts of hair on her upper lip, chin, abdomen, back, and
chest. She takes no medications and has had two normal pregnancies. She is currently
asymptomatic. Her mother and most of her aunts also have a similar pattern of excessive hair. On
further questioning and examination, which of the following findings would favor most strongly
the diagnosis of idiopathic hirsutism? 
 Incorrect Answer ImageA.Clitoromegaly
 Incorrect Answer ImageB.Frontal balding
 Incorrect Answer ImageC.Obesity
 Correct Answer ImageD.Regular menses
 Incorrect Answer ImageE.Severe acne

A 24-year-old woman is brought to the physician by her parents because of problems with
"social skills." The parents are concerned because the patient constantly needs help making
decisions and can never assert her own opinion, even regarding such unimportant issues like
clothing and cosmetics. She feels extremely uncomfortable when alone for more than a brief
period and is intensely afraid of being left to care for herself. She often feels helpless and fears
confrontations or disagreements with her parents and coworkers. This often has led to significant
problems at work because of her difficulty in initiating projects or doing things on her own.
Which of the following is the most likely diagnosis? 
 Incorrect Answer ImageA.Borderline personality disorder
 Correct Answer ImageB.Dependent personality disorder
 Incorrect Answer ImageC.Histrionic personality disorder
 Incorrect Answer ImageD.Panic disorder with agoraphobia
 Incorrect Answer ImageE.Schizophreniform disorder
A 72-year-old man has surgical pinning of an intertrochanteric fracture of the right hip. His
history is significant for a broken left hip 3 years ago, at which time he had a postoperative
pulmonary embolus. This time he is placed on heparin on the first postoperative day, at which
time he is also started on warfarin. His platelet count was 375,000/mm3 at the time of surgery,
and it becomes 79,000/mm3 on the fifth postoperative day, while he is still receiving both
anticoagulant drugs (heparin and warfarin). Which of the following is the most appropriate
management? 
 Incorrect Answer ImageA.Continue present therapy
 Incorrect Answer ImageB.Continue present therapy and add platelet transfusions
 Incorrect Answer ImageC.Discontinue both anticoagulant drugs
 Incorrect Answer ImageD.Discontinue both anticoagulant drugs and administer
protamine
 Correct Answer ImageE.Discontinue heparin and continue warfarin

A 6-month-old boy is brought to the physician because of a rash that has been worsening over
the past several weeks. The infant was adopted at age 4 months from an orphanage in Ukraine
with no significant past medical history or family history of skin disease. The rash was not
present when the adoptive parents went to pick him up, but developed about 2 weeks after arrival
in the United States. The mother tells you that the rash is itchy, especially at nighttime. However,
he sleeps and eats well and is developing normally. Apart from the parents, there is a
grandmother and a 6-year-old adopted daughter living in the same household and both have
recently had extremely itchy rashes in the web spaces of their hands. On physical examination,
the patient is in no acute distress but is attempting to rub and scratch his abdomen while in his
mother's arms. Vital signs are within normal limits and the weight and length are both in the 70th
percentile. Inspection of the skin reveals multiple small erythematous papules and pustules with
multiple excoriations on the trunk, arms, and legs. The conjunctivae and oral mucosa are
unremarkable and the remainder of the examination is within normal limits. Which of the
following is the most appropriate treatment?
 Incorrect Answer ImageA.Oral cephalexin for 7 days
 Incorrect Answer ImageB.Oral trimethoprim-sulfamethoxazole for 3 days
 Incorrect Answer ImageC.Topical fluocinonide for up to 2 weeks
 Correct Answer ImageD.Topical permethrin for the entire family, applied on day 1 and
day 7 of treatment
 Incorrect Answer ImageE.Topical sulfur ointment, applied daily for 6 days

A 28-year-old woman, gravida 1, para 0, at 36 weeks' gestation is sent to the labor and delivery
department because of blood pressures averaging 180/120 mm Hg, 3+ proteinuria on urine
dipstick, and contractions. Her pregnancy had been uncomplicated. She is started on a
magnesium sulfate infusion with a standard dosing protocol, and induction of labor is begun with
oxytocin. Four hours later, she is in active labor with cervical dilation to 6 cm. The fetal heart
rate is 140/min and reactive. Her blood pressure remains elevated at 170/120 mm Hg. Which of
the following is the most appropriate next step in management?
 Correct Answer ImageA.Administer hydralazine
 Incorrect Answer ImageB.Decrease the oxytocin infusion
 Incorrect Answer ImageC.Increase the magnesium infusion
 Incorrect Answer ImageD.Increase the oxytocin infusion
 Incorrect Answer ImageE.Perform a cesarean delivery

A 27-year-old woman comes to the office because of infertility. She was diagnosed with
endometriosis 6 years ago. She has been unable to conceive for the past 2 years. A thorough
infertility evaluation suggests that her infertility is caused by her endometriosis. She has no other
medical problems. She had a diagnostic laparoscopy that demonstrated endometriosis 6 years
ago. She takes no medications and is allergic to penicillin. Physical examination shows findings
consistent with endometriosis. Which of the following pharmacologic treatments is most likely
to improve this patient's chances of conception? 
 Incorrect Answer ImageA.Combination oral contraceptive pill
 Incorrect Answer ImageB.Danazol
 Incorrect Answer ImageC.Estrogen
 Incorrect Answer ImageD.Medroxyprogesterone
 Correct Answer ImageE.No pharmacologic treatment improves fertility in patients with
endometriosis

A 27-year-old woman comes to the office because of infertility. She was diagnosed with
endometriosis 6 years ago. She has been unable to conceive for the past 2 years. A thorough
infertility evaluation suggests that her infertility is caused by her endometriosis. She has no other
medical problems. She had a diagnostic laparoscopy that demonstrated endometriosis 6 years
ago. She takes no medications and is allergic to penicillin. Physical examination shows findings
consistent with endometriosis. Which of the following pharmacologic treatments is most likely
to improve this patient's chances of conception? 
 Incorrect Answer ImageA.Combination oral contraceptive pill
 Incorrect Answer ImageB.Danazol
 Incorrect Answer ImageC.Estrogen
 Incorrect Answer ImageD.Medroxyprogesterone
 Correct Answer ImageE.No pharmacologic treatment improves fertility in patients with
endometriosis

A 4-month-old infant boy has gained only 10 ounces since birth. He has failed to gain weight
with multiple formula preparations. His stools have been loose and fatty. An older sister had
similar symptoms and has been repeatedly hospitalized for failure to maintain weight and height,
along with recurrent pulmonary infections. Which of the following is the most likely cause of
this patient's gastrointestinal symptoms? 
 Incorrect Answer ImageA.Achlorhydria
 Incorrect Answer ImageB.Bacterial overgrowth
 Incorrect Answer ImageC.Colonic inertia
 Incorrect Answer ImageD.Gastric hypersecretion
 Correct Answer ImageE.Pancreatic exocrine insufficiency

A 76-year-old man presents to the clinic for his semiannual examination. The patient is well
known to the clinic and has been having semiannual examinations for the past 3 years to follow
up his chronic obstructive pulmonary disease (COPD). The patient is a long-time smoker, with a
200+ pack year history. He is status post a right upper wedge resection 2 years ago for
adenocarcinoma, and at that time had severe obstructive disease. A recent chest CT showed
apical bullae and severe emphysematous changes. The patient has moderate dyspnea on exertion
and often has shortness of breath with minimal activity. On this visit, the patient relates that he is
even more short of breath at rest and is almost unable to perform any physical activity as a result.
Which of the following would most strongly suggest the need to initiate home oxygen therapy? 
 Incorrect Answer ImageA.Exercise-induced oxygen desaturations to less than 92%
 Incorrect Answer ImageB.Resting arterial PaO2 showing an alveolar arterial gradient of
more than 12 mm Hg
 Correct Answer ImageC.Resting arterial PaO2 of less than 55 mm Hg
 Incorrect Answer ImageD.Resting PaO2 of greater than 40 mm Hg
 Incorrect Answer ImageE.Room air oxygen saturation of less than 92%

A 7-day-old boy who is the product of an uncomplicated gestation is brought to the physician
because of hypospadias. The baby is otherwise healthy, and is urinating without any difficulty.
On physical examination, vital signs are stable, lungs are clear and the heart is beating at a
regular rate. The only abnormal physical finding is the hypospadias. Urinalysis is negative for
infection. Which of the following is the most appropriate next step? 
 Incorrect Answer ImageA.Measuring serum creatinine level
 Correct Answer ImageB.Schedule a renal ultrasound
 Incorrect Answer ImageC.Obtain an intravenous pyelogram
 Incorrect Answer ImageD.Cystography
 Incorrect Answer ImageE.Performing a circumcision

A 53-year-old woman is admitted for depression with psychotic features. During the
examination, the psychiatrist asks her whether she has ever had problems with her heart. The
patient responds, "What are you saying? I don't have a heart anymore; there is nothing left inside
me." Which of the following is the patient most likely exhibiting? 
 Incorrect Answer ImageA.Cenesthetic hallucination
 Incorrect Answer ImageB.Depersonalization
 Correct Answer ImageC.Nihilistic delusion
 Incorrect Answer ImageD.Overvalued idea
 Incorrect Answer ImageE.Verbigeration

A 4-year-old girl is brought to the physician because of a crusted honey-colored erythema


resulting from rupture of tiny vesicles and pustules. Her temperature is 37.7 C (102 F). Skin
lesions are distributed over the face and extremities. Physical examination reveals enlargement
of lymph nodes in the cervical and axillary regions. Which of the following is the most frequent
pathogen of this skin infection? 
 Incorrect Answer ImageA.Human herpesvirus 7
 Correct Answer ImageB.Staphylococcus aureus
 Incorrect Answer ImageC.Streptococcus pyogenes
 Incorrect Answer ImageD.Propionibacterium acnes
 Incorrect Answer ImageE.Trychophyton fungi

After a minor but distressing automobile accident, a patient is unable to move one leg. Physical
examination shows no obvious injury that might have caused the paralysis. His reflexes are
intact. A CT of the spine shows no back injury. The patient is reassured by the negative findings
and the paralysis resolves over a 2-week period. Which of the following is the most likely
diagnosis?
 Incorrect Answer ImageA.Body dysmorphic disorder
 Correct Answer ImageB.Conversion disorder
 Incorrect Answer ImageC.Factitious disorder
 Incorrect Answer ImageD.Malingering
 Incorrect Answer ImageE.Somatic symptom disorder

An 18-year-old college student comes to the student health clinic because of a 2-week history of
fever, chills, and a sore throat. His temperature is 38.3°C (100.9°F), pulse is 70/min, respirations
are 18/min, and blood pressure is 110/70 mm Hg. Physical examination shows marked
pharyngeal hyperemia, tonsillar exudates, cervical lymphadenopathy, and splenomegaly.
Amoxicillin therapy is begun, and the patient is sent back to his dormitory. Two days later, he
returns because of a maculopapular rash. Which of the following is most likely to confirm the
diagnosis?
 Incorrect Answer ImageA.Chest x-ray
 Correct Answer ImageB.Heterophile antibody test
 Incorrect Answer ImageC.Lateral x-ray film of the neck
 Incorrect Answer ImageD.Rapid Streptococcus test
 Incorrect Answer ImageE.Varicella virus antibody immunofluorescence

A 15-month-old boy is brought to the emergency department by his parents because he suddenly
turned blue and had difficulty breathing while he was playing in the backyard of their home. The
parents have noticed several times in the previous 2–3 months that he had developed a bluish
discoloration around the lips but did not think much of it, as it was wintertime. Recently,
however, the boy had increasing fatigability and would stop in the middle of playing to catch his
breath by sitting down or squatting. This morning when he suddenly turned blue and started
behaving in a very flustered manner they rushed him to the emergency department. The parents
deny any other significant medical conditions or allergies to medication. They have a 4-year-old
daughter who is in good health. On physical examination, the patient is in the fifth percentile for
height and weight. His lips and fingertips are bluish in color and he frequently stops to catch his
breath while playing in the examination room. There is a hint of clubbing of his fingers. Vital
signs are within normal limits, but a complete blood count shows a hematocrit of 66%. A chest
radiograph shows a boot-shaped heart with an uptilted apex and clear lung fields. On
auscultation, there is a harsh systolic ejection murmur and a single S2 is heard. Which of the
following is the most likely diagnosis? 
 Incorrect Answer ImageA.Endocardial cushion defect
 Incorrect Answer ImageB.Myocarditis
 Incorrect Answer ImageC.Patent ductus arteriosus
 Correct Answer ImageD.Tetralogy of Fallot
 Incorrect Answer ImageE.Transposition of the great vessels

A 29-year-old woman, gravida 1, para 0, at 37 weeks' gestation comes to the physician for a
prenatal visit. She is HIV-positive and was diagnosed 3 years ago. She has no other medical
problems. She has never had surgery. She is currently taking zidovudine. She has no known drug
allergies. An HIV RNA level that was checked recently has come back at 2,000 copies/ml.
Which of the following is the most appropriate management regarding mode and timing of
delivery? 
 Incorrect Answer ImageA.Perform membrane stripping at this visit
 Correct Answer ImageB.Recommend cesarean delivery at 38 weeks
 Incorrect Answer ImageC.Recommend cesarean delivery at 40 weeks
 Incorrect Answer ImageD.Recommend induction of labor at 38 weeks
 Incorrect Answer ImageE.Recommend induction of labor now

A 44-year-old obese woman has been having episodes of right upper quadrant abdominal pain
brought on by the ingestion of fatty foods. Cholelithiasis is diagnosed by sonogram. The study
also shows a common duct of 3.5 mm in diameter. She has no history of jaundice, acholia, or
choluria, and her liver function tests are normal. She undergoes a laparoscopic cholecystectomy,
during the course of which a large arterial bleeder is encountered and repeatedly clipped until the
bleeding is controlled. No intraoperative cholangiogram is done. A Penrose drain is left in the
operative area and is brought out through a separate stab incision. There is no significant
drainage and the drain is removed the following day. In the postoperative period she becomes
progressively jaundiced. By the seventh postoperative day she has a serum bilirubin level of 13
mg/dL, 11 mg/dL of which is direct, conjugated bilirubin, and an alkaline phosphatase level of
435 U/L. Transaminases and serum amylase are normal. An HIDA scan shows uptake of the
radioactive material by the liver, with no radioactivity detected in the common duct or
duodenum. Which of the following is the most likely diagnosis? 
 Correct Answer ImageA.Inadvertent clipping of the common duct
 Incorrect Answer ImageB.Inadvertent clipping of the right hepatic artery
 Incorrect Answer ImageC.Hepatitis
 Incorrect Answer ImageD.Retained common duct stone
 Incorrect Answer ImageE.Transected common duct

A healthy 19-year-old soldier suffers third-degree burns over 90% of his body surface area when
a gasoline depot in a remote country is hit by enemy fire. He is resuscitated in the field and is
eventually airlifted to a sophisticated burn center in the United States. Diagnostic studies show
that 48 hours postburn, he has a basal metabolic rate twice what would have been expected for
his size, as well as a similarly elevated cardiac output. His burns have been covered with silver
sulfadiazine, and there is no reason to believe that they are infected. Which of the following is
the most likely reason for his abnormal physiologic parameters?
 Incorrect Answer ImageA.Endogenous blocking of the effect of catecholamines
 Incorrect Answer ImageB.Evaporative heat loss
 Correct Answer ImageC.High circulating levels of cytokines and catecholamines
 Incorrect Answer ImageD.High circulating levels of thyroxine and melatonin
 Incorrect Answer ImageE.Radiating heat loss

A 5-year-old girl is brought to the physician because her parents have observed bright red blood
in her stools intermittently for the past 3 months. She has no pain in association with bowel
movements, and the parents report that the stools are soft in consistency. She is otherwise an
active child with a good appetite and has not lost any weight. No bruising or bleeding from any
other site has been seen. Examination shows a well-appearing girl who is slightly pale.
Abdominal examination shows no distension or tenderness and no masses are felt. Rectal
examination shows normal tone, no fissures or tears, but blood is present on the examiner's
finger following digital examination. Complete blood count shows: 
Hemoglobin 9.4 g/dL
MCV 68 fL
Leukocyte count 7.4 x 109/L
Platelets 350 x 109/L
Which of the following is the most appropriate next step in management?
 Incorrect Answer ImageA.Abdominal ultrasound
 Correct Answer ImageB.Colonoscopy
 Incorrect Answer ImageC.Ferrous sulfate
 Incorrect Answer ImageD.Rectal swab culture for gonococcus and chlamydia
 Incorrect Answer ImageE.Stool for ova and parasites

A 66-year-old woman with a history of chronic obstructive pulmonary disease (COPD) comes to
the emergency department because of shortness of breath and palpitations for 36 hours. She has
used her ipratropium/albuterol inhaler multiple times but with no relief. She was in her usual
state of health until two days ago. She denies any recent chest pressure, pleuritic pain,
diaphoresis, or lower extremity edema. Her past medical history includes COPD, tobacco use,
hypertension, and diastolic heart failure. Her temperature is 37.3°C (99.1°F), blood pressure is
138/82 mm Hg, pulse is 138/min, and respirations are 22/min. Physical examination shows
diffuse expiratory wheezing and irregularly irregular rhythm. The rest of her examination is
unremarkable. An electrocardiogram shows an irregularly irregular rhythm with no apparent P
waves. As part of her management in the emergency department, the patient is started on an
intravenous diltiazem drip. Unfortunately, there is a mistake in the concentration of the
diltiazem. Instead of a 10-mg intravenous loading dose, the patient receives 100 mg. The
diltiazem is stopped immediately. Shortly after the bolus, she becomes significantly drowsy and
her blood pressure decreases to 81/59 mm Hg. Which of the following is the most appropriate
next step in the management of this patient?
 Correct Answer ImageA.Calcium gluconate
 Incorrect Answer ImageB.D5 water, 1-L bolus
 Incorrect Answer ImageC.Emergent hemodialysis
 Incorrect Answer ImageD.Glucagon
 Incorrect Answer ImageE.Lavage and charcoal

A 42-year-old man comes to the clinic complaining of weakness and fasciculations. He describes
a crampy pain in his legs, occasional jerky spasticity, and fasciculations in the muscle of his left
arm and left face. He is agitated and depressed about his condition, as his father passed away in
middle age from Lou Gehrig disease. Neurologic examination reveals proximal muscle
weakness, muscle spasticity, and hyperactive reflexes. A left-sided Babinski sign is present. The
patient is referred for electrodiagnostic studies to confirm the suspected diagnosis.
Electromyography studies show denervation and prominent fasciculations consistent with
anterior horn cell dysfunction. In addition to medical and psychosocial supportive care, which of
the following is a medical intervention that has been shown to improve symptoms and prolong a
patient's life until tracheostomy? 
 Incorrect Answer ImageA.Bromocriptine
 Incorrect Answer ImageB.Haloperidol
 Incorrect Answer ImageC.Penicillamine
 Incorrect Answer ImageD.Pyridostigmine
 Correct Answer ImageE.Riluzole

A 37-year-old man reports a poor energy level, worsening over the past year. He “feels
exhausted,” and his wife has complained of his loss of interest in sex. He cannot attain an
erection, and for this reason, has not had sex in 6 months. He no longer has erections on
awakening in the morning. His weight has not changed and he denies heat or cold intolerance.
On further questioning, he notes that he has had more frequent headaches and has had trouble
with driving recently, especially with changing lanes. On physical examination, he is a well-
developed, well-nourished man who seems lethargic. Extraocular movements are intact, but
visual field examination shows a bitemporal defect. His thyroid gland is nonpalpable. Lung,
cardiac, and abdominal examination are unremarkable. There is mild gynecomastia. Genital
examination shows a normal phallus and soft, atrophic, bilaterally descended testes. He has a
male pattern of pubic hair but his overall body hair is sparse. Laboratory studies show: 
Serum total testosterone 110 ng/mL (200–1,000 ng/mL)
Serum luteinizing hormone 2 mU/mL (5–15 mU/mL)
Serum follicle-stimulating hormone 1 mU/mL (3–15 mU/mL)
Further laboratory studies most likely would show which of the following?
 Incorrect Answer ImageA.Depressed levels of gonadotropin-releasing hormone
 Incorrect Answer ImageB.Elevated serum estrogen levels
 Correct Answer ImageC.High levels of serum prolactin
 Incorrect Answer ImageD.Low levels of sex-steroid binding globulin
 Incorrect Answer ImageE.Normal levels of serum free testosterone

A 6-week-old female infant is brought to the physician for a well-child visit. The mother reports
that the child has been feeding well with formula. She also describes some difficulty with diaper
changing as her left leg doesn't abduct as readily as the right, and occasionally she hears a
clicking sound. The child appears well, is gaining weight appropriately, smiles, and responds to
environmental stimuli in an age-appropriate manner. Her temperature is 37.0°C (98.6°F), pulse is
160/min, and respirations are 35/min. Cardiac and lung examinations are normal. The abdomen
is soft, non-tender, and non-distended with normal bowel sounds. Examination of the extremities
shows asymmetric thigh skin folds and uneven knee levels when the infant's feet are placed
together while supine. Which of the following is likely to be found on further evaluation of the
left hip?
 Incorrect Answer ImageA.Avascular necrosis of the femoral head on bone scan
 Incorrect Answer ImageB.Dislocation of the femoral head on hip radiograph
 Correct Answer ImageC.Dislocation of the femoral head on hip ultrasound
 Incorrect Answer ImageD.Femoral epiphysis slipped with respect to the femur on frog-
leg lateral x-ray
 Incorrect Answer ImageE.Fracture of the femoral head on hip radiograph

A 28-year-old man is evaluated in the emergency department because of acute-onset abdominal


and flank pain. He denies trauma, although he has been training for an upcoming marathon. The
pain is sharp, colicky, and radiates to his groin. He has nausea but denies vomiting. His urine is
dark in color and dipstick positive for blood. Which of the following is the most likely
diagnosis? 
 Incorrect Answer ImageA.Aortic dissection
 Incorrect Answer ImageB.Infarction of left kidney
 Incorrect Answer ImageC.Ischemic colitis
 Incorrect Answer ImageD.Rhabdomyolysis
 Correct Answer ImageE.Ureterolithiasis

A 29-year-old man with insulin-dependent diabetes comes to the physician for an annual
physical examination. He has been on a stable insulin regimen over the last year, and claims that
he has been strictly compliant with diet and frequent measurement of his fingerstick glucose.
One year ago he had mild proliferative retinopathy, and six months ago he developed a small
heel ulcer that healed after leg elevation and antibiotics. He is on 22 units of insulin glargine
nightly and 8 units of insulin lispro three times per day with meals. His blood pressure is 142/96
mm Hg, pulse is 68/min, and respirations are 16/min. His heart and lungs are normal. Abdominal
examination shows no hepatosplenomegaly, masses, or tenderness. The extremities have no
evidence of cellulitis or ulceration. Laboratory studies show:
Sodium 142 mEq/L
Potassium 4.9 mEq/L
Bicarbonate 22 mEq/L
Chloride 106 mEq/L
BUN 19 mg/dL
Creatinine 1.7 mg/dL
Which of the following findings would be most indicative of worsening renal function? 
 Incorrect Answer ImageA.Fasting blood glucose of 242 mg/dL
 Incorrect Answer ImageB.Hard exudates on retinal examination
 Incorrect Answer ImageC.Red blood cells in the urine
 Incorrect Answer ImageD.Systolic blood pressure of 150 mm Hg daily over the next
month
 Correct Answer ImageE.Urine collection over 24 hours showing 2.6 g of protein

A 13-year-old girl recently started menstruating and reports heavy periods. She uses two pads
per hour, which she feels is excessive. She also has had trouble with nosebleeds and bleeding
gums while brushing her teeth for a number of years. A maternal aunt and her father have a
similar bleeding problem. Physical examination shows no abnormalities. Laboratory studies
show iron deficiency anemia, as well as prolonged PTT. Platelet count and PT are normal.
Which of the following is the most likely diagnosis?
 Incorrect Answer ImageA.Deficiency of vitamin K
 Incorrect Answer ImageB.Disseminated intravascular coagulation
 Incorrect Answer ImageC.Hemophilia A
 Incorrect Answer ImageD.Thrombotic thrombocytopenic purpura
 Correct Answer ImageE.Von Willebrand disease

A 13-year-old boy has been making grunting sounds, squinting his eyes, and grimacing nearly
every day for the past two years. His mother had not sought treatment because she believed that
he was purposely producing these symptoms to annoy her. However, she is now concerned
because he is doing significantly worse at school than in the past. The boy’s teacher has also
noticed the symptoms and has told the mother of her concern. The boy has no known illnesses
and takes no medication. Which of the following is the most likely diagnosis?
 Incorrect Answer ImageA.Chronic vocal tic disorder
 Incorrect Answer ImageB.Huntington disease
 Incorrect Answer ImageC.Oppositional defiant disorder
 Correct Answer ImageD.Tourette syndrome
 Incorrect Answer ImageE.Transient tic disorder
 Incorrect Answer ImageF.Wilson disease

A 39-year-old man has difficulty controlling his blood pressure. He reports compliance with his
medications and follows a low-salt diet. He jogs 4 miles 3 times weekly. He is married with two
healthy children and works full-time as a church administrator. He does not smoke or drink. His
father died suddenly at age 39 and he has a brother and sister on dialysis. There is no family
history of diabetes. His medications are propranolol 40 mg four times daily and
hydrochlorothiazide 50 mg daily. His blood pressure is 156/96 mm Hg, and pulse is 66/min.
Cardiac examination shows an S4 without murmurs. Abdomen shows no hepatosplenomegaly or
tenderness. There is fullness at the costovertebral angle bilaterally. Both testes are descended and
without masses. On rectal examination, his prostate is smooth, symmetric, and not enlarged.
Pulses are full and symmetric distally. Laboratory studies show:
Sodium 140 mEq/L
Potassium 4.9 mEq/L
Chloride 110 mEq/L
Bicarbonate 22 mEq/L
BUN 16 mg/dL
Creatinine 1.4 mg/dL
Glucose 101 mg/dL
Urinalysis shows trace protein, no glucose, and heme pigment positive. Microscopic examination
reveals many red blood cells without casts. Which of the following is the most likely diagnosis? 
 Correct Answer ImageA.Adult polycystic kidney disease
 Incorrect Answer ImageB.Bilateral hydronephrosis
 Incorrect Answer ImageC.Chronic glomerulonephritis
 Incorrect Answer ImageD.Hereditary solitary kidney syndrome
 Incorrect Answer ImageE.Hypernephroma

The following vignette also applies to the next item. 


Seven months after the death of his father, a 27-year-old graduate student with no prior
psychiatric history experiences irritability, crying, difficulty in concentrating, and trouble falling
asleep. When he's with his friends, he frequently talks about his father and how much he misses
him. Once a social person, he has been withdrawn and rarely goes outside his home. He feels
excessive guilt and wishes he had spent more time with his father prior to his death. Outside of
an unexplained 10-lb weight loss (his original body weight was 150 lb), there are no significant
findings on physical examination. Which of the following is the most likely diagnosis?
 Incorrect Answer ImageA.Bereavement
 Incorrect Answer ImageB.Dependent personality disorder
 Correct Answer ImageC.Major depressive disorder
 Incorrect Answer ImageD.Persistent depressive disorder
 Incorrect Answer ImageE.Posttraumatic stress disorder

Seven months after the death of his father, a 27-year-old graduate student with no prior
psychiatric history experiences irritability, crying, difficulty in concentrating, and trouble falling
asleep. When he's with his friends, he frequently talks about his father and how much he misses
him. Once a social person, he has been withdrawn and rarely goes outside his home. He feels
excessive guilt and wishes he had spent more time with his father prior to his death. Outside of
an unexplained 10-lb weight loss (his original body weight was 150 lb), there are no significant
findings on physical examination. Which of the following is the most appropriate single
intervention for improvement in this patient’s symptoms in one to two months?
 Correct Answer ImageA.Antidepressant medication
 Incorrect Answer ImageB.Antipsychotic medication
 Incorrect Answer ImageC.Long-term psychodynamic psychotherapy
 Incorrect Answer ImageD.Support from family and friends
 Incorrect Answer ImageE.Supportive psychotherapy

A 23-year-old woman comes to the physician for an annual examination. She has no complaints.
She has been sexually active for the past 2 years, and uses the oral contraceptive pill for
contraception. She has depression, for which she takes fluoxetine, but she takes no other
medications and has no allergies to medications. Family history is negative for cancer and
cardiac disease. Examination is unremarkable. Which of the following screening tests should this
patient most likely have? 
 Incorrect Answer ImageA.Colonoscopy
 Incorrect Answer ImageB.Mammogram
 Correct Answer ImageC.Pap smear
 Incorrect Answer ImageD.Pelvic ultrasound
 Incorrect Answer ImageE.Sigmoidoscopy

A 2,700 g newborn boy is born to a healthy 27-year-old woman at 41 weeks of gestation via
uncomplicated vaginal delivery. At 1 minute after birth, the baby's body is pink, but the hands
and feet are bluish in color. His heart rate is 110/min, and nasopharyngeal suction results in a
grimace followed by a slow and irregular respiratory effort. Movements are active and with good
tone. At 5 minutes, his fingers and toes are still bluish, but the heart rate is 140/min;
nasopharyngeal probing leads to a cough and strong crying, and the respiratory effort has
become regular. Body tone remains good, with increased activity. Which of the following Apgar
scores are most appropriate for this baby boy at 1 and 5 minutes, respectively?
 Incorrect Answer ImageA.5 and 7
 Incorrect Answer ImageB.6 and 8
 Correct Answer ImageC.7 and 9
 Incorrect Answer ImageD.8 and 10
 Incorrect Answer ImageE.9 and 10

A 24-year-old man has a 2-year history of recurrent oral and genital ulcers, erythema nodosum,
and large joint polyarthritis. He now comes to the physician for the management of recurrent
visual blurring. Examination, including slit-lamp evaluation, shows evidence of iridocyclitis,
hypopyon, and small retinal infarcts. Steroids are begun, but ocular inflammation continues.
Which of the following is the optimal drug therapy for steroid-resistant ocular inflammation in
this condition? 
 Incorrect Answer ImageA.Colchicine
 Correct Answer ImageB.Cyclosporine
 Incorrect Answer ImageC.Intravenous high-dose immune globulin
 Incorrect Answer ImageD.Thalidomide

A mother brings her 1-year-old son to the physician for a well-child visit. He is doing well, has
an unremarkable medical history, and does not take any medication. Immunizations are up-to-
date. The child does not appear to be in any distress. On physical examination, the patient is in
the 75th percentile for height and 85th percentile for weight. Vital signs are within normal limits.
On inspection of the oral cavity, you note that the maxillary central and lateral incisors are
almost completely destroyed by caries and the remaining stumps are discolored with brown-to-
black cavitations. The cuspids and molars are intact. The rest of the physical examination is
within normal limits. Which of the following is the most likely cause of this patient's disorder? 
 Incorrect Answer ImageA.Excessive fluoride intake
 Incorrect Answer ImageB.Improper diet
 Incorrect Answer ImageC.Lack of fluoride supplementation
 Correct Answer ImageD.Repeated bottle feedings with sweetened drinks
 Incorrect Answer ImageE.Thumb sucking

A 19-year-old healthy man has just been exposed to a patient who is known to have measles
while he was away at college. He denies any symptomatic changes or the development of a rash.
He is not known to have received the MMR vaccinations during childhood, and his rubeola
antibody level is undetectable. Which of the following is the most appropriate advice to give
about the period of infectivity?
 Incorrect Answer ImageA.Infectious period ends 1 week after salivary gland swelling
 Incorrect Answer ImageB.Infectious period ends when the lesions have crusted over
 Correct Answer ImageC.Infectious period extends from onset of symptoms to 4 days
after rash
 Incorrect Answer ImageD.Infectious period extends from onset of symptoms to onset of
rash
 Incorrect Answer ImageE.Infectious period extends to 3 weeks after symptoms develop

An 81-year-old man is brought to the hospital for the management of acute pulmonary edema
due to an acute on chronic systolic heart failure exacerbation. He has had two myocardial
infarctions in the past two years and has an ejection fraction of 18%. He is admitted to the
coronary care unit, and his heart failure improves slowly over the next two days. While in the
coronary care unit his renal function deteriorates, and his BUN and creatinine rise from 32
mg/dL and 1.9 mg/dL, to 60 mg/dL and 2.4 mg/dL, respectively. His extremities are cool to
touch. A right heart catheterization shows normal filling pressures with a cardiac index of 1.6
L/min/m2. Which of the following medications is most likely to increase his renal perfusion?
 Incorrect Answer ImageA.Aspirin
 Incorrect Answer ImageB.Isosorbide dinitrate
 Correct Answer ImageC.Low-dose dobutamine
 Incorrect Answer ImageD.Metoprolol
 Incorrect Answer ImageE.Verapamil

A 74-year-old man with hypertension is admitted to the hospital because of dehydration and a
loss of 7 kg (15.4 lb) from his usual weight of 70 kg (155 lb) Medical evaluation is
unremarkable. The patient states, "I often forget to eat and drink since my wife died." The wife's
death occurred 3 months earlier. He is tearful at times during his hospitalization and speaks little
to the nurses, physicians, and other patients. On the day of discharge, when asked how he is
doing, he replies, "I'm going home but there is nothing to go home to." He admits to feeling sad
when asked about his mood. Which of the following is the most appropriate next step in
management?
 Correct Answer ImageA.Ask about thoughts regarding suicide
 Incorrect Answer ImageB.Call a family member to take the patient home
 Incorrect Answer ImageC.Obtain a neuroimaging study before discharge
 Incorrect Answer ImageD.Prescribe an antidepressant
 Incorrect Answer ImageE.Recommend a psychiatric consultation after discharge

A 13-year-old boy is apprehended after beating and robbing an elderly woman. While in police
custody, he says that he is suicidal and is brought to the emergency department. The patient lives
with his mother and younger brother. The patient's father, from whom he is estranged, is
physically abusive and shows evidence of alcohol use disorder. The patient's mother is called and
she states that the patient has had a history of behavioral and legal problems since the age of 9.
He has many arrests for crimes such as shoplifting, vandalism, and trespassing. Earlier this week,
the patient was expelled from school after severely beating another student. On mental status
examination, the patient has a bright affect, is conversant, and is actually quite charming.
Patients with this condition are most likely to develop which of the following disorders in
adulthood?
 Correct Answer ImageA.Antisocial personality disorder
 Incorrect Answer ImageB.Bipolar disorder
 Incorrect Answer ImageC.Eating disorder not otherwise specified
 Incorrect Answer ImageD.Schizotypal personality disorder
 Incorrect Answer ImageE.Social anxiety disorder

A 12-year-old boy is brought to the office because of a skin lesion that developed on his chin
several days earlier and is spreading. He had just returned from summer sports camp where he
spent 2 weeks. He remembers sustaining a minor injury to that area 2 days before the lesion
appeared. The patient denies any discomfort or pain but admits to scratching the lesion
occasionally. The scabs come off easily, but soon thereafter new ones form from the seeping of a
straw-colored liquid. He is in excellent health and has no history of medical problems or recent
medication use. He thinks that some other children had similar skin lesions while in camp. On
physical examination a single skin lesion is present on the chin as shown in the photograph.
Inspection of the oral mucosa and teeth is unremarkable. Submandibular lymph nodes are barely
palpable and nontender. The remainder of the physical examination is within normal limits.
Which of the following is the most appropriate treatment?  This patient has the typical clinical
picture of impetigo contagiosa, a staphylococcal, streptococca
 Incorrect Answer ImageA.No treatment is necessary
 Correct Answer ImageB.Oral amoxicillin and topical mupirocin
 Incorrect Answer ImageC.Oral terbinafine
 Incorrect Answer ImageD.Topical ketoconazole
 Incorrect Answer ImageE.Topical mupirocin

A 34-year-old woman has a 4.5 cm pulsatile mass on the left side of her neck. The mass has been
present for 3 or 4 years and has slowly enlarged to its present size. The mass is located below
and slightly anterior to the angle of the mandible. On palpation, the mass is pulsatile, and it can
be displaced laterally and medially, but not superiorly or inferiorly. A CT scan of the neck shows
the mass to be located in the area of the carotid bifurcation. MRI shows a “salt and pepper”
appearance of the mass. An extensive laboratory workup demonstrates slightly elevated amounts
of catecholamines in the urine, but the patient is normotensive and denies palpitations. Which of
the following is the most likely diagnosis?
 Incorrect Answer ImageA.Arteriosclerotic aneurysm of the carotid artery
 Incorrect Answer ImageB.Branchial cleft cyst
 Correct Answer ImageC.Carotid body tumor
 Incorrect Answer ImageD.Congenital arteriovenous malformation
 Incorrect Answer ImageE.Dissecting aneurysm of the carotid artery

A 40-year-old African American man comes to the emergency department because of a 2-day
history of increased shortness of breath and cough. He reports chest pain that increases with
exertion and is partially relieved with rest. He denies hemoptysis or wheezing. He has a long-
standing history of hypertension, which is poorly controlled with atenolol 50 mg once a day. His
temperature is 37.0°C (98.6°F), blood pressure is 232/130 mm Hg in the right arm and 226/125
in the left, pulse is 77/min, and oxygen saturation is 92% on 5 liters nasal O2. There is an S4,
bibasilar inspiratory rales, a jugular venous pressure of 15 cm, and 2+ pitting edema to the upper
shins. Electrocardiogram shows a normal rate and rhythm with evidence of left atrial
enlargement. The S-wave has a voltage of 9 mV in V1 and 12 mV in V3, and the R-wave
measures 29 mV in V5. The R-wave in aVL has a voltage of 19 mV. There are depressed ST-
segments in V5 and V6 with T-wave inversions in a "hockey stick" pattern. Echocardiogram
shows an ejection fraction of 20-25%, normal right ventricular size and function, pulmonary
hypertension, and no valvular abnormalities. Serum chemistries show a blood urea nitrogen level
of 38 mg/dL and a creatinine of 1.5 mg/dL. Urinalysis shows 2+ proteinuria and 1+ blood.
Which of the following is the most likely primary cause of his acute shortness of breath?
 Incorrect Answer ImageA.Acute myocardial ischemia
 Incorrect Answer ImageB.Decompensated pulmonary hypertension
 Correct Answer ImageC.Heart failure exacerbation from hypertensive cardiomyopathy
 Incorrect Answer ImageD.Progressive renal failure
 Incorrect Answer ImageE.Pulmonary embolism
 Incorrect Answer ImageF.Viral myocarditis

A 52-year-old woman comes to the office because of a painful skin condition on her fingers that
developed over the previous 2 months and seems to be worsening. She is in good overall health
and has no other complaints. Her past medical history is remarkable for well-controlled
hypertension. She takes losartan and hydrochlorothiazide daily, along with a multivitamin. The
family history is unremarkable. The patient is retired and spends a lot of her time gardening,
where she always wears gloves. She had planted many new plants and weeded out remote parts
of her garden in the previous 2–3 months. On physical examination, she is in no acute distress
and her vital signs are within normal limits. Inspection of the hands shows that the fingertips of
her thumbs, first, and second fingers are dry, scaly, and with deep fissures that have a
hemorrhagic crust at their base. The remainder of her hands is unaffected. Her nails appear
normal except for notches and cracks of the distal free edge consistent with repeated trauma.
Which of the following plants is the most likely cause of this patient's hand dermatitis?
 Incorrect Answer ImageA.Celery
 Incorrect Answer ImageB.Mango
 Incorrect Answer ImageC.Primrose
 Incorrect Answer ImageD.Poison ivy
 Correct Answer ImageE.Tulips

A 48-year-old man has a transhiatal esophagectomy performed for a T1 squamous cell


carcinoma at the distal end of the esophagus. Reconstruction was done by pulling the stomach up
to the neck and anastomosing it to the cervical esophagus. A pyloroplasty also was done. A drain
was left in place next to the cervical anastomosis and brought out through the neck incision. A
right chest tube also was placed because the right pleural space was entered during the dissection
around the hiatus. A feeding catheter jejunostomy was placed at the time of surgery. The first 5
postoperative days are uneventful, but on the sixth day, a large amount of white, milky fluid
begins to drain out of the chest tube. By the eighth postoperative day, the drainage reaches 300
mL per day. A sample of the fluid is sent to the laboratory, which shows very high levels of
triglycerides. The patient is still NPO, but he has been receiving 3 liters per day of Ensure, a
nutritional replacement fluid, by way of the jejunostomy catheter since the third postoperative
day. At this time, which of the following is the most appropriate management?
 Incorrect Answer ImageA.Barium studies to determine the exact location of the fistula
 Correct Answer ImageB.Change jejunostomy feedings to a fat-free solution
 Incorrect Answer ImageC.Immediate reoperation
 Incorrect Answer ImageD.Remove the chest tube
 Incorrect Answer ImageE.Stop feedings and initiate parenteral nutrition

A 13-year-old African-American girl is brought to the physician because of pronounced scaling


of the scalp that started about 2 months earlier. They tried using hair moisturizers and anti-
dandruff shampoo, but it only slightly helped the problem. They have no pets. The patient's past
medical history is unremarkable. No other family members, including two younger sisters, have
any similar problems with their scalp. On physical examination, the patient is in no acute
distress. Vital signs are within normal limits and she is in the 70th percentile for weight and 65th
percentile for height. In the scalp, there is diffuse, yellowish-gray scaling along with scattered
perifollicular pale red papules. Many hairs are broken off at a length of 1-2 centimeters. Discrete,
non-tender lymph nodes are palpable bilaterally on the posterior neck. Wood's lamp examination
of the scalp is negative. Which of the following microorganisms is the most likely etiologic
agent?
 Incorrect Answer ImageA.Epidermophyton floccosum
 Incorrect Answer ImageB.Microsporum canis
 Incorrect Answer ImageC.Trichophyton mentagrophytes
 Incorrect Answer ImageD.Trichophyton schoenleinii
 Correct Answer ImageE.Trichophyton tonsurans

A 22-year-old woman, gravida 1, para 0, at 38 weeks' gestation is hospitalized for a deep venous
thrombosis (DVT). She was admitted 2 days previously with left lower extremity edema and was
found to have a DVT on ultrasound. Intravenous heparin therapy was initiated. She also was
found to have a urinary tract infection and was started on nitrofurantoin. She now goes into labor
with contractions every 2 minutes. Her cervix changes from closed to 3 cm dilated. Which of the
following is the most appropriate next step in management? 
 Incorrect Answer ImageA.Place inferior vena caval filter
 Incorrect Answer ImageB.Start magnesium sulfate
 Incorrect Answer ImageC.Start oxytocin
 Correct Answer ImageD.Stop heparin
 Incorrect Answer ImageE.Stop nitrofurantoin

A 33-year-old woman comes to the emergency department because of left lower quadrant
abdominal pain. She states that the pain has been coming and going over the past 3 days, but now
it is constant and excruciating. Past medical history is significant for a recent deep venous
thrombosis for which she is taking daily low molecular weight heparin (LMWH) injections. She
has never had surgery before. On physical examination, she is curled up on the stretcher moaning
in pain despite recent treatment with morphine. Her abdomen is severely tender with rebound
and guarding. Bimanual examination shows a left lower quadrant adnexal mass that is extremely
tender. Pelvic ultrasound shows an enlarged, 9-cm left ovarian cyst with no demonstrable blood
flow to the left ovary. Which is the most appropriate management?
 Incorrect Answer ImageA.Intravenous antibiotics and observation
 Incorrect Answer ImageB.Laparoscopy under epidural anesthesia
 Correct Answer ImageC.Laparoscopy under general anesthesia
 Incorrect Answer ImageD.Laparoscopy under spinal anesthesia
 Incorrect Answer ImageE.Laparotomy under spinal anesthesia

A 45-year-old chronic alcoholic presented to the emergency department 5 years ago with 24
hours of epigastric pain radiating to his back, nausea, and vomiting. He gradually recovered from
this acute episode. Over the next 5 years, he is repeatedly admitted for similar symptoms. He
then presents with gradual onset of weight loss, midabdominal pain radiating to his back, and
steatorrhea. Which is of the following has most likely occurred? 
 Incorrect Answer ImageA.Cholangiocarcinoma
 Correct Answer ImageB.Exocrine insufficiency of the pancreas
 Incorrect Answer ImageC.Gastric outlet obstruction
 Incorrect Answer ImageD.Pancreatic adenocarcinoma
 Incorrect Answer ImageE.Scarring of the entire length of the common bile duct

A 50-year-old female politician who has given speeches successfully in the past, has a sudden
onset of "extreme, paralyzing fear" before walking to a podium to give a speech. She now
becomes so anxious about giving speeches that her professional activities are being negatively
affected. She states that she is terrified of stuttering and "making a fool" of herself in front of
unfamiliar people. Which of the following is the most likely diagnosis?
 Incorrect Answer ImageA.Agoraphobia
 Incorrect Answer ImageB.Generalized anxiety disorder (GAD)
 Incorrect Answer ImageC.Panic disorder
 Incorrect Answer ImageD.Posttraumatic stress disorder (PTSD)
 Correct Answer ImageE.Social anxiety disorder

A 68-year-old man is admitted to the medical service for chest pain. The patient has an 80 pack-
year smoking history and is known to have an elevated total cholesterol but unknown LDL and
HDL components. He is a known insulin-dependent diabetic with a recent hemoglobin A1c
fraction of 8.3%. He has a history of chronic, stable angina precipitated by exertion and relieved
by rest. During the examination, he is free from chest pain. His blood pressure is 160/90 mm Hg,
pulse is 90/min, and respirations are 22/min. He is mildly diaphoretic. On physical examination,
he has an S3 gallop, bibasilar course rales, and an abdominal bruit. A chest radiograph shows
mild pulmonary edema. On ECG obtained on arrival to the floor shows ST segment depressions
in leads V3, V4, V5 and V6. Which of the following is the most likely diagnosis? 
 Incorrect Answer ImageA.Costochondritis
 Incorrect Answer ImageB.Pulmonary embolus
 Incorrect Answer ImageC.Musculoskeletal chest pain syndrome
 Correct Answer ImageD.Myocardial ischemia
 Incorrect Answer ImageE.Myocardial infarction

A 22-year-old woman of Polynesian descent is taken to the emergency department because of


severe abdominal pain. The pain, which began about 3 hours earlier, is periumbilical. She
vomited shortly after the pain started, but the nausea mostly subsided by the time she arrived at
the hospital. On physical examination, she appears acutely ill and is perspiring. Her temperature
is 38.1°C (100.6°F), blood pressure is 110/80 mm Hg, pulse is 110/min and regular and
respirations are 22/min. The chest is clear to auscultation. On abdominal examination, there is
tenderness in the right lower quadrant; this pain is also elicited when pressure is applied to the
non-tender left lower quadrant. The remainder of the physical examination is unremarkable. A
stat complete blood count demonstrates an erythrocyte count of 4.3 million/mm3, a leukocyte
count of 22,000/mm3 with 60% segmented neutrophils and 20% band forms, and a platelet count
of 300,000/mm3. Which of the following is the most appropriate next step in management? 
 Incorrect Answer ImageA.Barium enema
 Incorrect Answer ImageB.CT scan of abdomen
 Incorrect Answer ImageC.Esophagogastroduodenoscopy
 Correct Answer ImageD.Laparotomy
 Incorrect Answer ImageE.Ultrasound of abdomen and pelvis

 24-year-old graduate student and teaching assistant comes to the student health center after
being instructed by his college dean to seek counseling. He explains he does not need
counseling, but rather that the problem is that his work is superior to that of all his peers and
faculty supervisors which creates conflict within his department. According to the collateral
history obtained from the student’s instructors, the man cannot tolerate feedback and only
associates himself with people who praise him. He has had to be reprimanded for treating his
undergraduate students without empathy or understanding in demanding that their work be
submitted in two different forms for each assignment. Given this history, which of the following
is the most likely diagnosis?
 Incorrect Answer ImageA.Antisocial personality disorder
 Incorrect Answer ImageB.Borderline personality disorder
 Correct Answer ImageC.Narcissistic personality disorder
 Incorrect Answer ImageD.Paranoid personality disorder
 Incorrect Answer ImageE.Schizotypal personality disorder

 32-year-old woman is admitted to the hospital because of nausea, vomiting, and abdominal pain.
The medical team has so far found no reason for these symptoms and a psychiatrist is asked to
assess the patient for depression. The psychiatrist learns that she has had many medical problems
and, over the past 10 years, has been in the hospital more than 10 times. She reports having had
excessive menstrual bleeding and a miscarriage, as well as difficulty becoming aroused during
sexual intercourse. She cannot tolerate high-fiber foods and has heartburn and bloating. Two
years ago, she had a lump in her throat and had difficulty swallowing, but nothing was found
during evaluation. In the past couple of years, she has had frequent migraine headaches and
blurred vision and has seen several neurologists. She also hurt her back and now has chronic
back pain. Which of the following is the most likely diagnosis?
 Incorrect Answer ImageA.Conversion disorder
 Incorrect Answer ImageB.Generalized anxiety disorder
 Incorrect Answer ImageC.Hypochondriasis
 Incorrect Answer ImageD.Pain disorder
 Correct Answer ImageE.Somatization disorder

A 40-year-old woman is referred for neurologic consultation because of weakness and spasticity
of the upper limbs for the past three years. She has a long history of psychiatric problems,
characterized by depression and poorly defined somatic complaints such as a recurrent sensation
of electric shocks down the spine. Four years ago, the patient experienced transient loss of visual
acuity, which was interpreted at that time as a psychosomatic manifestation. On examination, she
appears moderately anxious, but with no abnormalities in orientation, memory, or judgment.
Neurologic examination shows spastic paraparesis and mild loss of proprioceptive sensation in
both arms. The neurologist arranges for a lumbar puncture. This shows increased protein with
oligoclonal bands in the CSF. Which of the following is the most likely diagnosis? 
 Incorrect Answer ImageA.Acute disseminated encephalomyelitis
 Incorrect Answer ImageB.Lyme disease
 Correct Answer ImageC.Multiple sclerosis
 Incorrect Answer ImageD.Paraneoplastic syndrome
 Incorrect Answer ImageE.Spinocerebellar syndrome

A 15-month-old boy is admitted to the hospital because of a high fever and pulmonary
consolidation on a chest radiograph. Ever since he was 6 months old he has been in and out of
the hospital with recurrent infections of the skin and pneumonia. There has hardly been a month
that he was not on antibiotics. The mother had to quit her job to be able to take care of him
because he never could go to daycare for more than a day at a time. This latest episode started a
couple of days ago when painful, red, hot lesions developed on his arm and buttock accompanied
by a cough and high fever. The family history is significant for discoid lupus erythematosus in
the mother and an older sister with recurrent aphthous stomatitis. On physical examination, the
child is in moderate distress with a temperature of 39.4°C (102.9°F), pulse is 115/min, and
respirations are 28/min and shallow. Several tender, erythematous, and edematous nodules are
present on the right arm and both glutei. A chest x-ray shows left lower lobe consolidation.
Histopathologic examination of the skin lesions shows suppurative granulomatous inflammation
of the dermis and subcutis. Sputum cultures grow Candida albicans. Which of the following is
the most appropriate diagnostic study for his underlying condition?
 Incorrect Answer ImageA.Absolute neutrophil count
 Incorrect Answer ImageB.Blood culture
 Incorrect Answer ImageC.Bone marrow aspirate
 Incorrect Answer ImageD.Lung biopsy
 Correct Answer ImageE.Nitroblue tetrazolium test

A 15-year-old homeless girl comes to the emergency department because of a several-week


history of vaginal discharge, pelvic pain, and "spotting" between her periods. She denies fever,
emesis, diarrhea, or dysuria. She is sexually active and admits to using condoms inconsistently.
Her last menstrual period ended 1 week ago. Vital signs are: temperature: 37°C (98.6°F), pulse
80/min, blood pressure 110/70 mm Hg, and respirations 18/min. Urine pregnancy test is
negative. Pelvic examination shows a friable cervix and mucopurulent discharge at the cervical
os. There is no cervical motion tenderness. Cultures and a Pap test are obtained. Which of the
following is the most appropriate next step in management? 
 Incorrect Answer ImageA.Counsel the patient to use condoms and have her follow up in
the gynecology clinic in 3 days
 Incorrect Answer ImageB.Give the patient a prescription for a 7-day, twice-a-day course
of 100 mg of doxycycline
 Incorrect Answer ImageC.Administer one IM injection of 2.4 million units of benzathine
penicillin G
 Correct Answer ImageD.Observe the patient take one dose of 1 g of azithromycin and
400 mg of cefixime while in the emergency department
 Incorrect Answer ImageE.Obtain parental consent before initiating therapy

A 29-year-old woman, gravida 2, para 1, at 34 weeks' gestation comes to the physician for a
prenatal visit. She states that the fetus is moving well and she denies bleeding, loss of fluid, or
contractions. Her medical history consists of diabetes that has been controlled with insulin during
the pregnancy. Her past surgical history is significant for a transfundal myomectomy with
removal of a 10-cm fibroid 3 years ago. She has no known drug allergies. Which of the
following is the most appropriate delivery plan for this patient?
 Correct Answer ImageA.Cesarean delivery after documented fetal lung maturity
 Incorrect Answer ImageB.Cesarean delivery at 35 weeks' gestation
 Incorrect Answer ImageC.Cesarean delivery at 41 weeks' gestation
 Incorrect Answer ImageD.Induction of labor at 38 weeks' gestation
 Incorrect Answer ImageE.Induction of labor at 41 weeks' gestation

A 4-month-old girl is brought to the physician for a routine well-child visit. The parents explain
that she has been developing well, and they are eager to start her on solid food because she
seems to be so advanced. They are a little concerned about excessive scaling in the scalp lately
and noticed that sometimes when she wakes up, her eyelids seem covered with secretions. She
does not have any other medical problems and is being breastfed. The family history is
unremarkable. On physical examination, the patient appears to be a content infant, sitting calmly
in her mother's lap. Vital signs are normal and her weight and length are both in the 90th
percentile for age. On the vertex of her scalp there is a patch of pale erythema covered by a thick,
sticky, yellow scale. Her eyelids and eyebrows are slightly erythematous with the scant sticky,
yellowish scale. There are no palpable lymph nodes in the head and region. The examination is
otherwise within normal limits. Which of the following statements is the most appropriate
regarding this patient's skin condition?
 Incorrect Answer ImageA.Immunizations should be withheld until the condition resolves
 Correct Answer ImageB.It would be helpful to gently wash the scalp and eyelids with a
mild solution of baby shampoo and a soft toothbrush
 Incorrect Answer ImageC.The findings are normal and no treatment is indicated
 Incorrect Answer ImageD.The patient most likely has psoriasis and a biopsy is indicated
 Incorrect Answer ImageE.This is a bacterial infection and the patient needs topical
antibiotics

During a bank robbery, an innocent bystander is shot repeatedly in the upper abdomen. Because
of a brief ensuing confrontation between the robber and the police, the ambulance crew cannot
reach the patient until approximately 10 minutes later. They find the patient conscious and
anxiously asking for a blanket and a drink of water. He is pale and perspiring heavily. His blood
pressure is 85/60 mm Hg, and he has a barely perceptible pulse at a rate of 110/min. A fully
staffed trauma center is 2 miles away from the site of the shooting. Which of the following is the
most appropriate management for the ambulance crew to perform? 
 Incorrect Answer ImageA.Endotracheal intubation
 Incorrect Answer ImageB.Establish intraosseous access
 Incorrect Answer ImageC.Place a central venous line
 Incorrect Answer ImageD.Place the patient in pneumatic trousers before moving him
 Correct Answer ImageE.Transport to the hospital

A 63-year-old man comes to the physician for a routine health maintenance examination. He was
diagnosed with type 2 diabetes mellitus (T2DM) two months ago which is being managed with
lifestyle modifications. In addition to this, he is obese and has hypertension. Current medications
include aspirin, orlistat, and lisinopril. Vital signs are normal. BMI is 31 kg/m2. Physical
examination is unremarkable. His most recent hemoglobin A1c level is 8.3%. After much
discussion, the patient agrees that it is time to start medical treatment for his diabetes. However,
he is concerned about gaining weight, which he has heard is a common side effect of antidiabetic
drugs. Which of the following is the most appropriate medication for this patient?
 Incorrect Answer ImageA.Glyburide
 Incorrect Answer ImageB.Insulin
 Correct Answer ImageC.Metformin
 Incorrect Answer ImageD.Nateglinide
 Incorrect Answer ImageE.Rosiglitazone

A 28-year-old woman comes to the clinic for a preoperative checkup. For the last few years she
has suffered from fluctuating weakness and fatigability and has frequent episodes of diplopia and
dysarthria. Her weakness has been diffuse, affecting her arms and legs and her face and eyes.
One recent exacerbation of symptoms resulted in hospitalization and close monitoring for
possible elective intubation, which was avoided with the use of noninvasive bilevel positive
pressure (BiPAP) ventilation. After a prolonged workup she was diagnosed with myasthenia
gravis, received a short course of steroids, and was scheduled for a thymectomy. She currently
feels well and a review of symptoms fails to reveal any evidence of current disease. Aside from
myasthenia gravis, her past medical history is unremarkable. Physical examination is
unremarkable, and laboratory studies, with the exception of the presence of antiacetylcholine
receptor antibodies, are normal. Which of the following is the most appropriate preoperative
management? 
 Incorrect Answer ImageA.Check preoperative thyroid hormone levels
 Correct Answer ImageB.CT scan or MRI of chest before operation
 Incorrect Answer ImageC.Defer operation given lack of symptoms
 Incorrect Answer ImageD.No recommendations, continue as planned
 Incorrect Answer ImageE.Testing for antithymocyte antibodies

A 45-year-old man was brought to the hospital because of chest pain for the past 3 hours. He is
mildly agitated and admits that he ingested cocaine before the pain started. He has a long-
standing history of cocaine abuse. He has no other medical issues, takes no medications, and has
no allergies. His heart rate is 125/min and blood pressure is 150/90 mm Hg. His lungs are clear
and heart has a regular rhythm. An electrocardiogram shows normal sinus rhythm with non-
specific T wave changes. Initial cardiac enzymes are normal. Which of the following is the most
appropriate management? 
 Incorrect Answer ImageA.Administer a beta-blocker
 Correct Answer ImageB.Administer a calcium channel blocker
 Incorrect Answer ImageC.Obtain a head CT scan
 Incorrect Answer ImageD.Place him in a quiet room and observe
 Incorrect Answer ImageE.Start unfractionated heparin drip

A 65-year-old man comes to the emergency department because of sudden onset severe
substernal chest pain radiating to the neck and back. Dyspnea and stridor are also present. He
denies nausea, vomiting, or sweating. The patient's wife states that he was relaxing after dinner
and watching television when the symptoms began. He has a history of hypertension and
coronary artery disease with a prior coronary artery bypass graft (CABG) surgery. His pulse is
118/min, respirations are 32/min, and blood pressure is 70/40 mmHg. He appears in significant
distress. A chest x-ray is shown above. Which of the following is the most likely diagnosis?
 Incorrect Answer ImageA.Dissecting abdominal aortic aneurysm
 Correct Answer ImageB.Dissecting thoracic aortic aneurysm
 Incorrect Answer ImageC.Myocardial infarction
 Incorrect Answer ImageD.Pericardial tamponade
 Incorrect Answer ImageE.Ruptured aortic valve leaflet

A 71-year-old man is brought to the emergency department. While eating dinner earlier, he noted
that he was clumsy while cutting his steak with his right hand. As he arose from the kitchen
table, he found it difficult to support his weight on his right leg. He has a history of hypertension
treated with metoprolol for the last eight years with moderate success. On physical examination,
he is alert and fully oriented. His temperature is 37°C (98.6°F), blood pressure is 148/94 mm Hg,
pulse is 62/min, and respirations are 14/min. His fundi are normal and his pharynx is clear. He
has a regular heart rhythm without any murmurs, rubs, or gallops. His lungs are clear.
Neurologic examination shows diminished strength and sensation in the right-upper and right-
lower extremities. Hemianopia of the right field of both eyes is noted on visual field
examination. An electrocardiogram shows normal sinus rhythm at a rate of 62/min with normal
intervals and a physiologic axis. Which of the following is the most likely cause of his
symptoms?
 Incorrect Answer ImageA.Basilar artery occlusion
 Incorrect Answer ImageB.Lacunar infarction
 Incorrect Answer ImageC.Occlusion of the right internal carotid artery
 Correct Answer ImageD.Occlusion of the left internal carotid artery
 Incorrect Answer ImageE.Left ventricular thrombus

A 27-year-old man with a history of chewing tobacco since the age of 12 comes to the physician
because of a lump on his tongue. Physical examination shows the presence of a red, fleshy,
friable 2 cm mass on the left lateral border of the tongue. It is non-tender. Which of the following
is the best step in management of this patient? 
 Incorrect Answer ImageA.Acyclovir
 Incorrect Answer ImageB.HIV testing
 Incorrect Answer ImageC.Oral swish and swallow nystatin
 Correct Answer ImageD.Punch biopsy of the lesion
 Incorrect Answer ImageE.Treat with broad-spectrum antibiotics

 healthy 19-year-old woman comes to the physician for a routine health maintenance
examination. She has questions about a skin lesion on her anterior chest. It has been present since
birth and has not changed significantly over time. Examination shows an oval, raised, pigmented
lesion measuring 2.5 cm in the greatest dimension as shown in the photograph. The borders are
smooth, and there is no color variegation. Which of the following is the most appropriate next
step in management? 
 Correct Answer ImageA.Advise her to report any changes
 Incorrect Answer ImageB.Cryotherapy of the lesion
 Incorrect Answer ImageC.Excisional biopsy of the lesion
 Incorrect Answer ImageD.Shave biopsy of the lesion
 Incorrect Answer ImageE.Topical hydrocortisone cream to the lesion twice a day for 7
days

A 35-year-old homeless alcoholic is brought to the emergency department after he was found
intoxicated in the street. Physical examination shows a scaling erythematous rash in linear
formation primarily in the webs between his fingers. There are similar lesions on his scrotum.
There are no pustules present. When directly questioned about these lesions, he says that these
areas have been intensely pruritic, primarily at night, for the last few weeks. Which of the
following is the most likely cause of this patient's skin lesions? 
 Incorrect Answer ImageA.Bacterial infection
 Incorrect Answer ImageB.Herpetic whitlow
 Incorrect Answer ImageC.Psoriasis
 Correct Answer ImageD.Scabies
 Incorrect Answer ImageE.Syphilis

A 45-year-old Caucasian woman comes to the physician because of fever, nausea, vomiting and
dysuria over the past two days with right upper quadrant abdominal pain radiating to the scapula.
She took an over the counter medication for fever, but could not recall the name. She recently
returned from a business trip to Africa. She has been unable to lose weight (currently 65 lbs
above her ideal body weight) despite trying several different weight loss diets. She has allergic
rhinitis currently managed with the use of over-the-counter medications. She smokes one pack of
cigarettes daily, and drinks one can of beer every night. She is single, works as a marketing
executive, and is not sexually active. Temperature is 39.5°C (103.1°F), blood pressure is 130/94
mm Hg, pulse is 90/min, and respirations are 16/min. Physical examination shows tenderness in
the right quadrant with guarding. The spleen and liver are not palpable and there is no
costovertebral angle tenderness. Laboratory studies show:
Hemoglobin 15 g/dL
Hematocrit 45%
Leukocyte count 19,700/mm3
Segmented neutrophils 65%
Band forms 5%
Eosinophils 1%
Basophils 0%
Lymphocytes 28%
Monocytes 1%
Platelet count 370,000/mm3
Creatinine 1.3 mg/dL
BUN 22 mg/dL
Glucose 74 mg/dL
Amylase 140 U/L
Which of the following is the most likely diagnosis? 
 Incorrect Answer ImageA.Acute appendicitis
 Correct Answer ImageB.Acute cholecystitis
 Incorrect Answer ImageC.Acute pancreatitis
 Incorrect Answer ImageD.Perforated duodenal ulcer
 Incorrect Answer ImageE.Renal calculus

A 37-year-old man comes to a psychiatrist because of a depressed mood, irritability, "tics", and
difficulty at work. On brief neurologic examination, the psychiatrist notes that the "tics" are
involuntary movements of the head, arms, and trunk that the patient gracefully completes with
voluntary movements such as combing his hair. When asked, the patient reports that his father
died in his fifties, with what he was told was dementia. Which of the following is the most likely
mode of transmission of this? 
 Correct Answer ImageA.Autosomal-dominant transmission
 Incorrect Answer ImageB.Autosomal-recessive transmission
 Incorrect Answer ImageC.It is a bacterial infection
 Incorrect Answer ImageD.It is a viral infection
 Incorrect Answer ImageE.Sex-linked transmission

The following vignette applies to the next two items.


A 36-year old woman comes to her primary care physician with chief complaint of absence of
menses for last 6 months and white discharge from her breasts. The patient also complains of hot
flashes and vaginal dryness. The patient says that she otherwise is healthy and did not have any
significant medical problems in the past. She is not allergic any to medications and does not take
any medications. She is divorced and lives alone. She is not currently sexually active and denies
possibility of being pregnant. Her vital signs are normal and physical examination is
unremarkable except small amount of white discharge from her breasts. Urine HCG test was
negative. Plasma prolactin level was found to be significantly elevated - 850 ng/ml. Prolactinoma
was suspected and MRI was ordered. MRI revealed presence of microadenoma, 1 mm in
diameter. Which of the following sets of laboratory tests is consistent with diagnosis of
prolactinoma? 
Gonadotropin-releasing Luteinizing Follicle-stimulating
hormone hormone hormone
 Incorrect Answer ImageA.Decreased;Elevated;Elevated
 Correct Answer ImageB.Decreased;Decreased;Decreased
 Incorrect Answer ImageC.Normal;Elevated;Elevated
 Incorrect Answer ImageD.Elevated;Decreased;Decreased
 Incorrect Answer ImageE.Elevated;Normal;Normal

A 36-year old woman visits her primary care physician with amenorrhea for the last 6 months
and a white discharge from her breasts. The patient also complains of hot flashes and vaginal
dryness. The patient says that she otherwise is healthy and did not have any significant medical
problems in the past. She does not take any medications and has no allergies. She is divorced and
lives alone. She is not currently sexually active and denies possibility of being pregnant. Her
vital signs are normal and physical examination is unremarkable except small amount of white
discharge from her breasts. Urine HCG test was negative. Plasma prolactin level was found to be
significantly elevated - 850 ng/ml. Prolactinoma was suspected and MRI was ordered. MRI
revealed presence of microadenoma, 1 mm in diameter. Which of the following is most
appropriate treatment for this condition? 
 Incorrect Answer ImageA.Amitriptyline
 Correct Answer ImageB.Carbegoline
 Incorrect Answer ImageC.Haloperidol
 Incorrect Answer ImageD.Metoclopramide
 Incorrect Answer ImageE.Radiotherapy
 Incorrect Answer ImageF.Transsphenoidal surgery

A 71-year-old moderately obese man who underwent right hemicolectomy for colon cancer 6
days prior complains of "leaking from his wound." He has been unable to tolerate oral fluids and
therefore remains in the hospital. His gown is wet and there is pinkish fluid draining from the
wound. The superior staples have been dislodged and there is a gap in the incision. The gap is
probed with a cotton-tipped swab. It is difficult to remove the swab from the incision and when it
is finally dislodged, it is noted that it was caught on a piece of bowel that is located in the
superficial abdominal fat. Which of the following is the most appropriate next step? 
 Incorrect Answer ImageA.Order a CT scan
 Incorrect Answer ImageB.Pack the wound with gauze and reevaluate in the morning
 Correct Answer ImageC.Prepare for an emergent laparotomy
 Incorrect Answer ImageD.Remove all the staples
 Incorrect Answer ImageE.Take the patient to the treatment room for examination with
better lighting

A 26-year-old woman, gravida 3, para 0, aborta 3, comes to the physician because of a history of
three spontaneous abortions at about 6 weeks gestation over the past 2 years. Menarche was at
age 13. Menstrual periods are regular at 28-day intervals, lasting 5 days. She has no known past
medical conditions. A male cause of infertility has been ruled out. Physical examination is within
normal limits. Which of the following is the most likely cause of recurrent pregnancy loss in this
patient?
 Incorrect Answer ImageA.Antiphospholipid syndrome
 Incorrect Answer ImageB.Congenital uterine anomaly
 Incorrect Answer ImageC.Chromosomal abnormality
 Correct Answer ImageD.Idiopathic
 Incorrect Answer ImageE.Incompetent cervix
 Incorrect Answer ImageF.Pelvic inflammatory disease
 Incorrect Answer ImageG.Submucous myoma

A 5-year-old boy has a sore throat and rash on his hands and feet of 3 days duration. The child
reports mild tenderness where the lesions are present. He is refusing to eat and drink. A cousin
visited last week during spring break and had similar skin lesions that were resolving. The
patient's family has a 6-month-old at home and is worried that the illness may be contagious and
more deleterious to the infant. On physical examination, the patient does not seem to be in acute
distress. His vital signs are within normal limits. Pinpoint vesicles on an erythematous base are
diffusely present on the oral mucosa. The palms, soles, and proximal forearms are studded with
tender violaceous papules, some with a central vesicle. He has lesions present on the buttocks
and peri-anally as well. Which of the following is the most likely diagnosis?
 Incorrect Answer ImageA.Erythema multiforme
 Correct Answer ImageB.Hand, foot, and mouth disease
 Incorrect Answer ImageC.Herpes simplex virus infection
 Incorrect Answer ImageD.Human papillomavirus infection
 Incorrect Answer ImageE.Papular acrodermatitis of childhood (Gianotti-Crosti
syndrome)

A 41-year-old with two children complains of lower abdominal pain and vaginal bleeding. She
has a positive home pregnancy test after 6 weeks of amenorrhea. On examination, her
temperature is 38.3 C (101.0 F). Examination of the abdomen reveals bowel sounds to be
present. The abdomen is soft; the liver, kidney, and spleen are not palpable; there is some
tenderness to deep palpation in both lower quadrants. The vulva and vagina are normal. There is
a small amount of malodorous sanguinopurulent discharge from the cervix. The cervical os is
closed. The uterus is top limits normal size and is tender to motion. The adnexa are tender to
palpation, and there is a 5-cm left adnexal mass. Which of the following is the most likely
diagnosis? 
 Incorrect Answer ImageA.Ectopic pregnancy
 Incorrect Answer ImageB.Incomplete abortion
 Incorrect Answer ImageC.Missed abortion
 Incorrect Answer ImageD.Pyometra
 Correct Answer ImageE.Septic abortion

A 19-year-old unkempt woman comes unaccompanied to the emergency department in advanced


labor. This is her first pregnancy and she has not had any prenatal care. She is a new patient and
is reluctant to provide information about herself, except that she does not know who the father of
the child is and has no relatives that can be reached in case of an emergency. She is transferred to
the obstetric department and, several hours later, she delivers a baby boy that is small for
gestational age and has a disproportionately large head. He weighs 1,900 g (4 lb 3 oz) and is 35
cm (13.8 in) long. Apgar scores are 4 and 6 at 1 and 5 minutes, respectively. On the second day,
the infant develops coarse jitters and a high-pitched cry. He is hyperactive, restless, and feeding
poorly. His body temperature is 38.2°C (100.8°F). In addition, he becomes tachypneic and
develops diarrhea. Which of the following is likely to be positive in the meconium screen of this
neonate?
 Incorrect Answer ImageA.Glucose
 Correct Answer ImageB.Heroin
 Incorrect Answer ImageC.Homogentisic acid
 Incorrect Answer ImageD.Marijuana
 Incorrect Answer ImageE.Phenobarbital

An infant boy is born full-term to a 26-year-old mother prenatally diagnosed with


polyhydramnios. Soon after birth, he is noted to have some respiratory compromise, with
increased oral secretions. He is also noted to have abdominal distention that is becoming more
pronounced with each breath he takes. An attempt is made to pass a feeding tube into the
oropharynx, but resistance is encountered. An x-ray shows that the distal tip of the tube is coiled
in the thorax. Abdominal film shows the bowel to contain gas. The neonatologist feels that the
infant is too critical to undergo extensive surgery at this time. Which of the following is the next
most important step in management? 
 Incorrect Answer ImageA.Echocardiography to evaluate for cardiac anomalies
 Incorrect Answer ImageB.Emergent bronchoscopy and simultaneous esophagoscopy
 Correct Answer ImageC.Placement of a gastrostomy tube
 Incorrect Answer ImageD.Schedule esophagostomy
 Incorrect Answer ImageE.Ultrasonography to evaluate for renal anomalies

A 39-year-old woman with insulin- dependent diabetes has a foot ulcer that has not healed with
local wound care. Over the past 10 days she has had increasing pain in this area and fevers to
38.3°C (101.0°F). Physical examination shows a deep ulcer at the left heel with tenderness to
palpation in the area. There is mild erythema around the ulcer. Which of the following is the
most appropriate long-term management? 
 Incorrect Answer ImageA.Amputation, since antibiotics are unlikely to be effective
 Incorrect Answer ImageB.Intraarticular injection of antibiotics to treat her osteomyelitis
 Incorrect Answer ImageC.Incision and drainage of ulcer with wound care
 Incorrect Answer ImageD.Long-term immobilization to prevent weight bearing
 Correct Answer ImageE.Treatment with intravenous antibiotics for possible osteomyelitis
if diagnostic studies are positive

A 41-year-old woman has a 5-month history of headaches. She is seeking medical attention now,
as she has recently been experiencing small amounts of epistaxis and streaky hemoptysis. She
also has a low-grade fever and arthralgias, but no obvious arthritis. Physical examination shows
maxillary sinus tenderness, and inspiratory crackles heard over the left scapular region. Her
leukocyte count is 12,400/mm3 and serum creatinine is 2.2 mg/dL. Urine examination shows 10-
12 red blood cells per high-power field. Which of the following is the most appropriate
diagnostic study at this time? 
 Incorrect Answer ImageA.Anti-double-stranded DNA antibody
 Incorrect Answer ImageB.Anti-GBM antibody
 Correct Answer ImageC.Anti-proteinase-3 antibody
 Incorrect Answer ImageD.C3 complement
 Incorrect Answer ImageE.Rheumatoid factor
A 61-year-old woman has a breast mass that is clinically and mammographically highly
suspicious for cancer. A fine needle aspiration confirms the diagnosis. In the preoperative
evaluation she is found to have liver and bone metastasis. A lumpectomy is done for removal of
the mass, and the pathologist reports the presence of a 2-cm infiltrating ductal carcinoma in the
specimen, with clear margins all around. Which of the following is appropriate further
management? 
 Incorrect Answer ImageA.Antiestrogens, like tamoxifen
 Incorrect Answer ImageB.Conversion to complete mastectomy
 Incorrect Answer ImageC.Local radiation to both breasts
 Incorrect Answer ImageD.Sampling of axillary lymph nodes
 Correct Answer ImageE.Systemic chemotherapy

A 74-year-old man is brought to the physician by his family members who report that over the
last 6 months to a year, the patient "just hasn't been acting like himself." Although he has
experienced mild memory impairment for the last 3 years at least, things have reportedly become
much worse in recent months. Now the patient has become extremely irritable and has episodes
of significant confusion. A few days ago, he was found wandering in his neighborhood shopping
mall parking lot, apparently having forgotten how to get home. The patient's son is concerned
about Alzheimer’s disease as the patient's father suffered from the condition in the later years of
his life. If this patient does have Alzheimer’s-type dementia, which of the following is the
earliest finding likely to be found on neuropsychiatric examination?
 Incorrect Answer ImageA.Delusions and hallucinations
 Incorrect Answer ImageB.Difficulties in planning and reasoning
 Incorrect Answer ImageC.Poorly controlled behavior
 Correct Answer ImageD.Prominent memory impairment
 Incorrect Answer ImageE.Subtle visuospatial impairment

During a high-school football game, a 17-year-old player is hit from the side and feels his knee
give way. A local physician, watching his own son in the game, goes to the sideline and
examines the player. A Lachman test is normal (patient has a firm endpoint and similar results on
the uninjured knee). The patient's knee does open up medially to a valgus stress significantly
when contrasted with the uninjured knee. The patient is able to walk with only a minimal limp.
The coach asks if the boy can play in the second half of the game. What should the physician tell
the coach about the player? 
 Incorrect Answer ImageA.He may have sprained his anterior cruciate ligament and may
play in the second half wearing a knee brace
 Incorrect Answer ImageB.He may have sprained his medial collateral ligament and
should be able to play right away
 Incorrect Answer ImageC.He may have torn his anterior cruciate ligament and should not
play in the second half
 Correct Answer ImageD.He may have torn his medial collateral ligament and should not
play in the second half
 Incorrect Answer ImageE.He may have torn his medial meniscus and should not play in
the second half
A 3-hour-old neonate develops worsening cyanosis. The infant was delivered at term vaginally
after an uncomplicated pregnancy. Apgar scores were 6 and 7 at 1 and 5 minutes, respectively.
Physical examination shows tachypnea, subcostal retractions, a normal first heart sound, and a
loud and single second heart sound. A chest X-ray is performed that shows an “egg on-a-string”
appearance of the cardiomediastinal silhouette. Echocardiogram is diagnostic for transposition of
the great vessels with a patent ductus arteriosus, an atrial septal defect, and a small ventricular
septal defect. While preparations are being made for surgical correction of the anomaly,
administration of which of the following would be the next best step in the management of this
patient?
 Incorrect Answer ImageA.Bosentan
 Incorrect Answer ImageB.Indomethacin
 Correct Answer ImageC.Prostaglandins
 Incorrect Answer ImageD.Vasoconstrictor
 Incorrect Answer ImageE.Vasodilator

An 18-year-old woman attempts suicide by drinking an entire bottle of liquid drain cleaner.
Shortly after doing so, she calls 911 and requests help. She is first transported to a small
community hospital, from which she is referred to a major medical center. She arrives at this
center 2 hours after the caustic ingestion. She is found to have black exudates in her mouth and
throat, and she reports severe chest and abdominal pain. Other findings include impressive
subcutaneous emphysema in the lower neck and upper chest and a rigid abdomen with guarding
and rebound in all quadrants. Her temperature is 39.0°C (102.2°F), pulse is 110/min, and blood
pressure 80/60 mm Hg. After resuscitative measures are initiated, which of the following is the
most appropriate next step in management?
 Incorrect Answer ImageA.Antibiotics and further observation
 Incorrect Answer ImageB.Barium swallow
 Incorrect Answer ImageC.Induced vomiting
 Incorrect Answer ImageD.Surgical repair of esophageal and gastric perforations
 Correct Answer ImageE.Surgical resection of dead esophagus and stomach

A 29-year-old woman, gravida 3, para 2, at 39 weeks' gestation comes to the physician for a
prenatal visit. Her pregnancy dating is by an 8-week ultrasound. Her pregnancy has been
complicated by gestational diabetes requiring insulin for blood sugar control. A 36-week group
B Streptococcus (GBS) culture was positive. Her past obstetric history is significant for two
normal vaginal deliveries. She has no complaints of loss of fluid, vaginal bleeding, or
contractions. She states that the fetus is moving well. Her fundal height is 42 cm and her cervix
is long, thick, and closed. She is sent for an ultrasound that an estimated fetal weight of 4,900 g.
Which of the following is the most appropriate next step in management?
 Incorrect Answer ImageA.Perform amniocentesis for fetal lung maturity
 Incorrect Answer ImageB.Perform repeat GBS testing
 Correct Answer ImageC.Recommend cesarean delivery
 Incorrect Answer ImageD.Recommend induction of labor
 Incorrect Answer ImageE.Start magnesium sulfate

A male infant is born at 40 weeks' gestation to a 25-year-old primigravid woman via primary
Cesarean section because of breech presentation. Apgar scores are 9 and 9 at 1 and 5 minutes,
respectively. Physical examination is normal. Infant and mother are scheduled to be discharged
home on hospital day 4. Before discharge, a nurse performs a heel stick on the infant to collect a
blood specimen for the Newborn Screening Program. Which of the following metabolic
disorders is routinely tested for in newborn screening in all states?
 Incorrect Answer ImageA.Hartnup disease
 Incorrect Answer ImageB.Homocystinuria
 Incorrect Answer ImageC.Lesch-Nyhan syndrome
 Incorrect Answer ImageD.Oculocerebrorenal syndrome
 Correct Answer ImageE.Phenylketonuria

A 52-year-old carpenter presents to his physician with swelling in his right knee, which began 2
days earlier. He denies any history of arthritis or trauma in that region. Until the swelling began,
he had been jogging approximately 2 miles daily. Over the past 48 hours, his knee has become
swollen and painful to weight bearing. His temperature 38.2°C (100.7°F), and the knee has a
tender effusion, which is erythematous and warm. There is a limited range of motion. Which of
the following would be the most relevant in this patient's history? 
 Incorrect Answer ImageA.Family history of rheumatoid arthritis
 Incorrect Answer ImageB.History of a bacterial gastroenteritis
 Incorrect Answer ImageC.History of hepatitis B exposure
 Incorrect Answer ImageD.History of a recent upper respiratory tract infection
 Correct Answer ImageE.Unprotected sex with a prostitute

 70-year-old man presents with a complaint of intense pain from a red and swollen right toe. He
has had no prior events such as this. He cannot recall trauma to the toe. An x-ray film of the right
foot is negative for a fracture. He is taking no medications, and denies a history of alcohol abuse.
He is given a diagnosis of gout and his serum is drawn for analysis. Which of the following
findings would be most helpful in making the diagnosis of gout in this patient? 
 Incorrect Answer ImageA.Associated right ankle effusion
 Incorrect Answer ImageB.Painless elbow nodule
 Correct Answer ImageC.Serum uric acid elevation
 Incorrect Answer ImageD.Response to colchicine
 Incorrect Answer ImageE.X-ray film showing first metatarsophalangeal joint erosion

An infant is delivered at full term via spontaneous vaginal delivery to a 29-year-old primigravid
mother who had limited prenatal care. At delivery, the infant is noted to have subcostal
retractions and cyanosis despite good respiratory effort. The abdomen is scaphoid. On bag and
mask ventilation, auscultation of the lungs shows decreased breath sounds on the left, with heart
sounds louder on the right. Which of the following is the most likely diagnosis?
 Incorrect Answer ImageA.Dextrocardia with situs inversus
 Correct Answer ImageB.Diaphragmatic hernia
 Incorrect Answer ImageC.Pneumonia
 Incorrect Answer ImageD.Pulmonary hypoplasia
 Incorrect Answer ImageE.Spontaneous pneumothorax

During a campaign appearance, a political candidate is shot point-blank in the right chest with a .
22 caliber revolver. The entrance wound is well above the nipple line, just under the third rib, at
the level of the anterior axillary line. His motorcade brings him to the emergency department, but
he makes it a point to walk in, holding his right chest with a bloody hand and waving for the
news media. A chest x-ray shows a hemothorax on the right, and the bullet is seen to be
embedded in the right paraspinous muscles. A chest tube is placed in the right pleural cavity, and
650 mL blood is recovered. Over the ensuing 4 hours, he continues to drain between 250 and 350
mL blood per hour. He is hemodynamically stable, and has received a total of 4 units of packed
red blood cells (PRBC). Which of the following is the most appropriate next step in
management?
 Incorrect Answer ImageA.Arteriography with embolization of the bleeding vessels
 Incorrect Answer ImageB.Continued observation and appropriate blood replacement
 Correct Answer ImageC.Thoracotomy and ligation of bleeding vessels
 Incorrect Answer ImageD.Thoracotomy, ligation of bleeding vessels, and removal of the
bullet
 Incorrect Answer ImageE.Transfusion of fresh frozen plasma (FFP) to correct the
coagulopathy

A 54-year-old woman comes to the emergency department with severe left-sided flank pain that
is referred to the left labia majora. She is very uncomfortable lying still on the stretcher and is
continuously shifting, trying to find a comfortable position. An intravenous pyelogram shows a
7-mm ureteral stone at the ureteropelvic junction. She has a normal coagulation profile. Which of
the following would most likely be the best therapy in this case? 
 Incorrect Answer ImageA.Plenty of fluids and analgesics and await spontaneous passage
 Correct Answer ImageB.Extracorporeal shock wave lithotripsy (ESWL)
 Incorrect Answer ImageC.Endoscopic retrograde basket extraction
 Incorrect Answer ImageD.Endoscopic retrograde laser vaporization of the stone
 Incorrect Answer ImageE.Open surgical removal

A 36-year-old woman comes to the psychiatrist for an initial visit after relocating from her home
several states away. She has been taking lithium for bipolar disorder and has been stable for
several years after two manic episodes in her early 30s. Prior to moving, she was seen by a
psychiatrist every two months and had blood drawn for routine monitoring related to her lithium
treatment every six months. It has been about eight months since her last psychiatric
appointment. Which of the following is the most appropriate laboratory studies to order at this
time?
 Incorrect Answer ImageA.Liver transaminases
 Incorrect Answer ImageB.Platelet count
 Correct Answer ImageC.Thyroid-stimulating hormone (TSH)
 Incorrect Answer ImageD.Urinalysis
 Incorrect Answer ImageE.Vitamin B12 level

A 30-year-old woman is brought to the emergency department after a syncopal episode. Two
months ago she had an episode of fever, chills, and generalized weakness during a visit to Cape
Cod. She has a history of cocaine abuse and she last used 4 months ago. She uses oral
contraceptives. Her blood pressure is 110/65 mm Hg and pulse is 43/min. Serum studies and
chest x-ray are normal. An electrocardiogram shows complete heart block, but there is no
evidence of ischemia or prior infarct. Which of the following is the most likely cause of this
patient's complete heart block?
 Correct Answer ImageA.Infection with Borrelia burgdorferi
 Incorrect Answer ImageB.Infection with HIV
 Incorrect Answer ImageC.Infection with Ixodes dammini
 Incorrect Answer ImageD.Myocardial infarction from cocaine use
 Incorrect Answer ImageE.Myocardial infarction from a coronary thrombus

A 1-year-old boy is brought to the physician because of a few days of "cold symptoms." He has
malaise, a low-grade fever, and coryza. There is no history of significant medical problems and
the child's immunizations are up to date. He is sent home with a recommendation of oral
acetaminophen and generous fluids. Two days later he begins to have difficulty breathing and a
bark-like cough. He is in moderate distress with a temperature of 38.1°C (100.6°F), a barking
cough, and intermittent inspiratory stridor. Nasal flaring and intercostal retractions are present at
rest. Which of the following organisms is the most likely cause of his condition? 
 Incorrect Answer ImageA.Cytomegalovirus
 Incorrect Answer ImageB.Epstein-Barr virus
 Incorrect Answer ImageC.Human herpesvirus 6
 Incorrect Answer ImageD.Measles virus
 Correct Answer ImageE.Parainfluenza virus

A 12-year-old girl with no significant medical history is brought to the physician by her mother
for a periodic health maintenance examination. The girl has had odorless, white vaginal
discharge for the past 3 months. She denies dysuria or vaginal pruritis. She has not yet reached
menarche and denies any type of sexual intercourse. Physical examination shows a non-
erythematous vulva with no rashes or lesions apparent, and normal external genitalia Tanner
stage 2, with scant, thin, white, odorless vaginal discharge at the introitus. Evaluation of the
vaginal discharge shows leukocytes, epithelial cells, scant lactobacilli, and a pH of 4.0. Which of
the following is the most appropriate next step in the management? 
 Incorrect Answer ImageA.Call child protection services to report a suspicion for sexual
abuse in this case
 Incorrect Answer ImageB.Make the diagnosis of bacterial vaginosis and treat her with a
single dose of metronidazole
 Incorrect Answer ImageC.Perform a speculum examination to rule out a vaginal foreign
body
 Correct Answer ImageD.Reassure the patient and her mother that this is physiologic
leukorrhea and is a normal finding
 Incorrect Answer ImageE.Recommend a steroid contraception with progestins to
decrease the amount of discharge

n 18-year-old girl comes to the emergency department because of dysuria. She had previously
called her internist, who prescribed ciprofloxacin over the phone. She has been taking the
antibiotics for 3 days but is feeling worse. She had a new sexual partner 10 days ago and does
not use condoms. She had a low-grade fever, muscle aches, and a general "out-of-sorts" feeling
before developing the dysuria. Her temperature is 37.7°C (99.9°F), blood pressure is 110/70 mm
Hg, pulse is 88/min, and respirations are 12/min. Physical examination is unremarkable.
Examination of the vulva shows multiple painful ulcers, ranging from 1 to 5 mm in diameter.
Which of the following is the most likely diagnosis?
 Incorrect Answer ImageA.Chancroid
 Incorrect Answer ImageB.Donovanosis
 Incorrect Answer ImageC.Lymphogranuloma venereum
 Correct Answer ImageD.Primary herpes simplex
 Incorrect Answer ImageE.Primary syphilis

There is a serious motor vehicle accident involving three cars, and one of the drivers in the
accident is being brought to the local emergency department because the weather is too severe to
airlift the patient to a trauma center. Emergency medical services inform the emergency
department that the victim is a 21-year-old man who was found ejected from his vehicle with no
obvious external injuries but is unable to ambulate. He is able to speak with emergency medical
services initially, complaining of severe pelvic and low back pain. However, during transfer, he
loses consciousness and develops unstable vital signs with profound hypotension, despite
receiving 2 liters of normal saline intravenously. The paramedics establish an airway. They
inform the physician that the patient's pelvis feels unstable, and there is significant widening. As
the patient arrives in the emergency department, his blood pressure continues to drop and is now
60/40 mm Hg. A chest radiograph shows that no mediastinal widening and no pneumothorax are
present. Which of the following is the most appropriate next step in management?
 Incorrect Answer ImageA.Contact the trauma hospital to arrange helicopter transport
once weather permits
 Incorrect Answer ImageB.Insert a large-bore needle into the left fifth intercostal space
 Incorrect Answer ImageC.Order a head CT without contrast
 Incorrect Answer ImageD.Perform an abdominal ultrasound
 Correct Answer ImageE.Wrap the patient's pelvis tightly with a sheet

A 64-year-old woman has progressive jaundice that she first noticed 2 weeks ago. She has a total
bilirubin of 12 mg/dL, with 8 mg/dL direct (conjugated) and 4 mg/dL indirect (unconjugated),
and minimally elevated transaminases. Her alkaline phosphatase is approximately 10 times the
upper limit of normal. She is otherwise asymptomatic, but is found to be anemic and to have
positive (4+) occult blood in the stool. Sonogram shows dilated intrahepatic and extrahepatic
biliary ducts and a very large, distended, thin-walled gallbladder without stones. Which of the
following is the most likely diagnosis? 
 Correct Answer ImageA.Adenocarcinoma of the ampulla of Vater
 Incorrect Answer ImageB.Adenocarcinoma of the gallbladder
 Incorrect Answer ImageC.Adenocarcinoma of the head of the pancreas
 Incorrect Answer ImageD.Cholangiocarcinoma
 Incorrect Answer ImageE.Choledocholithiasis

A 62-year-old woman comes to the emergency department because of shortness of breath. For
the past week she has had difficulty breathing, particularly when she lies down. Additionally, she
often gets winded after walking one or two blocks, whereas previously she could walk miles
without problems. She also has noticed that her legs have become mildly swollen over the past
few weeks. She denies any chest pain and has never had heart trouble in the past. Her past
medical history is significant for a long history of poorly controlled hypertension. Cardiac
examination shows a prominent but nondisplaced point of maximal impulse and an S4, with a
jugular venous pressure of 10 cm. Bilateral crackles are heard on lung examination, and 1+
pitting edema is present in both lower extremities. A portable chest radiograph shows
cephalization of the pulmonary vasculature and small, bilateral effusions. The cardiac silhouette
is borderline enlarged. An electrocardiogram is remarkable only for high voltages, particularly in
the precordial leads. A transthoracic echocardiogram is performed that shows an ejection fraction
of 70% with moderate left ventricular hypertrophy. Which of the following is an additional likely
finding on echocardiogram?
 Incorrect Answer ImageA.Accelerated early diastolic filling with a corresponding
increased E wave
 Incorrect Answer ImageB.Dyssynchrony in ventricular contractions with abnormal septal
motion
 Correct Answer ImageC.Increased late diastolic filling and reduced relaxation of the
ventricle
 Incorrect Answer ImageD.Large dilated left atrium with relatively preserved left
ventricular size
 Incorrect Answer ImageE.Mild to moderate tricuspid and mitral valve regurgitant flow

Paramedics bring a 24-year-old man to the hospital after a witnessed seizure at a nearby bar. The
paramedics describe symptoms consistent with a grand mal seizure. By the time of arrival to the
emergency department, the patient is awake and alert, and states that he has never had a seizure
before and reports being well and healthy all of his life. He does not take any medications. He
denies tobacco use, states he "drinks socially," and denies any use of illicit drugs. Vital signs are:
temperature 39.7°C (103.5°F), blood pressure 190/110 mm Hg, pulse 115/min, respirations
22/min, and oxygen saturation 99% while breathing room air. The patient seems generally
healthy, though he is disheveled and anxious and smells of tobacco and alcohol. He is
appropriately alert and oriented and has a normal neurologic examination. His skin is warm and
flush, with needle track marks in the left antecubital fossa. Pupils are 8 mm, equal, and
sluggishly reactive to light, though funduscopic examination is normal. The rest of the
examination, with the exception of tachycardia, is unremarkable. Routine laboratory studies,
including a complete blood count, full set of chemistries, and a urinalysis, are within normal
limits. Which of the following is the most likely cause of a seizure in this patient?
 Incorrect Answer ImageA.Anticholinergic poisoning
 Incorrect Answer ImageB.Bacterial meningitis
 Correct Answer ImageC.Cocaine intoxication
 Incorrect Answer ImageD.Delirium tremens
 Incorrect Answer ImageE.Heroin overdose

A 9-month-old girl is brought to the emergency department by her parents because of a seizure
witnessed by the child's grandmother. She has been ill with high fever for the past 3 days. They
had tried bringing the fever down with over-the-counter cold drops and frequent cool baths,
without much success. Today while playing in her bed the child suddenly developed a
generalized seizure that lasted approximately 5 seconds. She has not had any medical problems
before this and is not taking any medication. Immunizations are up to date and her last well-child
visit was within normal limits. There is no family history of seizures. On physical examination
the child is in moderate distress with a temperature of 39.7°C (103.4°F), pulse is 140/min, and
respirations are 30/min. Inspection of the skin, oral mucosa, and conjunctivas is unremarkable.
There is generalized lymphadenopathy. An ophthalmologic examination does not show any
abnormalities. Appropriate blood work is performed and the patient is admitted for observation.
The following morning a diffuse rash is noted over the entire body as shown in the photograph.
The child is now afebrile and in a good mood. Which of the following was the most likely cause
of this patient's illness?
 Incorrect Answer ImageA.Cytomegalovirus
 Incorrect Answer ImageB.Epstein-Barr virus
 Correct Answer ImageC.Human herpesvirus 6
 Incorrect Answer ImageD.Human herpesvirus 8
 Incorrect Answer ImageE.Parvovirus B19

A 3-year-old girl is brought to the emergency department with status asthmaticus. She received
five consecutive albuterol nebulizer treatments before arriving at the hospital and now her pulse
is 70/min with wide QRS complexes and absent T waves. At this time, which of the following is
the most appropriate therapy?
 Incorrect Answer ImageA.Intravenous adenosine bolus
 Incorrect Answer ImageB.Intravenous digoxin bolus
 Correct Answer ImageC.Intravenous potassium chloride infusion
 Incorrect Answer ImageD.Intravenous procainamide infusion
 Incorrect Answer ImageE.Intravenous verapamil bolus

A 66-year-old woman is admitted to the hospital for increasing shortness of breath at rest and
with exertion. Over the past several days she has also noticed onset of lower extremity edema.
She has diabetes mellitus treated with a sulfonylurea and a 90 pack-year smoking history. She
reports that she has "high cholesterol" and that she has been on a thiazide diuretic since her last
hospitalization. Laboratory studies show a total cholesterol of 289 mg/dL with an LDL of 110
mg/dL, a hemoglobin A1 of 9.3%, and a urine dipstick showing 2+ albumin. Which of the
following is the most appropriate medical therapy for this patient? 
 Incorrect Answer ImageA.A thiazide diuretic and a sulfonylurea
 Incorrect Answer ImageB.A thiazide diuretic, digoxin, and a sulfonylurea
 Incorrect Answer ImageC.A thiazide diuretic, ACE inhibitor, an HMG-CoA reductase
inhibitor, and a sulfonylurea
 Incorrect Answer ImageD.A loop diuretic, an ACE inhibitor, an HMG-CoA reductase
inhibitor, and a sulfonylurea
 Correct Answer ImageE.A loop diuretic, an ACE inhibitor, an HMG-CoA reductase
inhibitor, and insulin

A 29-year-old woman comes to the emergency department because of dizziness and chest pain.
She has recently been taking nonsteroidal antiinflammatory agents for some joint pain after a
sports injury. She reports that she was well until 5 days ago when she thought she had "the flu,"
with a cough and some rhinorrhea. Three days ago, she began to note chest pain that was
exacerbated by lying down. She sought care today after she tried to stand up at work and
experienced extreme lightheadedness. Her blood pressure is 105/60 mm Hg sitting and 80/40
mm Hg standing. There is a 12 mmHg fall in systolic blood pressure during inspiration. She has
elevated neck veins, quiet but normal heart sounds, and cool extremities. Which of the following
is the most likely diagnosis?
 Incorrect Answer ImageA.Dehydration
 Incorrect Answer ImageB.Musculoskeletal injury of chest wall
 Incorrect Answer ImageC.Pericardial effusion without tamponade physiology
 Correct Answer ImageD.Pericardial tamponade
 Incorrect Answer ImageE.Pneumonia

An 11-year-old girl has just recovered from a sore throat that was successfully treated with
penicillin for the previous 10 days. Suddenly, she starts developing numerous drop-size (2-5 mm
in diameter) erythematous papules on her trunk and extremities that are covered with a fine
silvery scale. She is up-to-date on her immunizations and is usually an active and healthy child.
Which of the following is the most likely diagnosis?
 Incorrect Answer ImageA.Erythema multiforme
 Incorrect Answer ImageB.Erythema nodosum
 Correct Answer ImageC.Guttate psoriasis
 Incorrect Answer ImageD.Lichen planus
 Incorrect Answer ImageE.Pityriasis rosea

A healthy 43-year-old Caucasian man has had an erection for the past 6 hours that "does not
want to go away." He has dysuria and some discomfort within the penis but denies any trauma
associated with intercourse. He has never experienced anything like this before. He denies using
any medication or illicit drugs. On physical examination, the corpora cavernosa is rigid and
tender, whereas the glans penis and corpus spongiosum are soft. There is no curvature associated
with the erection, and there are no palpable abnormalities along the length of the penile shaft.
Testicles are descended and normal to palpation. Rectal examination shows good sphincter tone
and a normal sized, smooth prostate. Urinalysis and blood counts are all within normal limits.
Which of the following is the most likely diagnosis? 
 Incorrect Answer ImageA.Hypospadias
 Incorrect Answer ImageB.Paraphimosis
 Incorrect Answer ImageC.Penile chordee
 Incorrect Answer ImageD.Peyronie disease
 Correct Answer ImageE.Priapism

A 40-year-old woman has a growth on her right arm. It has enlarged slowly over the past several
months, and the patient is concerned it may be skin cancer. She denies itching, bleeding, or
scabbing of the area. She vaguely recalls banging the arm several months ago while gardening.
She uses sunblock when outside for prolonged periods of time, and there is no family history of
skin cancer. On physical examination, the lesion measures 5 mm in diameter. It is slightly
elevated and firm with an irregular surface as indicated in the picture above. The lesion dimples
downward when compressed laterally. The remainder of the skin and general physical
examination are unremarkable. Which of the following would most likely be present on
histological examination? 
 Incorrect Answer ImageA.Abnormal appearing keratinocytes
 Incorrect Answer ImageB.Abnormal melanocytes with large, erratic nuclei extending
deep into the dermis
 Incorrect Answer ImageC.Basal cells with large, round nuclei and local invasion of
surrounding tissue
 Correct Answer ImageD.Fibroblasts, collagen, histiocytes, and capillaries
 Incorrect Answer ImageE.Uniform intradermal proliferation of squamous cells

A 69-year-old man with hyperlipidemia and hypertension comes to the emergency department
because of chest pain. His chest pain began abruptly during the night and woke him from his
sleep. He describes the substernal pain as a 10/10 in severity, "stabbing" in nature, and radiating
to his back and left scapula. The pain is not relieved by rest, but is associated with profuse
diaphoresis and nausea. His blood pressure is 190/100 mm Hg, with equal and symmetric pulses
in his dorsalis pedis arteries. A CT scan with contrast of his chest shows acute aortic dissection
extending from the descending aorta to his iliac bifurcation. Which of the following is the most
appropriate next step in management? 
 Correct Answer ImageA.Begin nitroprusside and beta-blocker therapy to reduce his
blood pressure
 Incorrect Answer ImageB.Begin unfractionated intravenous heparin
 Incorrect Answer ImageC.Get an angiogram of his lower extremity vascular tree to
evaluate further the extent of the dissection
 Incorrect Answer ImageD.Get an ultrasound of his abdomen to rule out an aneurysm with
the dissection
 Incorrect Answer ImageE.Refer the patient for immediate surgical intervention
The following vignette applies this item and the next item.
A 14-year-old boy is riding his bicycle when he hits a pothole and falls. He seems all right and
goes home. Now, one hour later, he comes to the emergency department complaining of
abdominal pain. He had attacks of spasmodic abdominal pain initially, but the pain has now
become constant. The pain is mainly in the epigastric area and does not radiate elsewhere. A
month before he was treated for gastroenteritis, which he had contracted after a school trip to a
national park. He is not currently on any medications and does not suffer from allergies. He did
not lose consciousness at the time of injury, nor did he injure his head. He said that he had been
wearing a helmet at the time. While in the emergency department, he vomits coffee-ground
material twice. Vital signs are: pulse 100/min, blood pressure 100/60 mm Hg, respirations
20/min, and temperature 39 C (102.2 F). On examination, the child is conscious and in moderate
distress, clutching his upper abdominal region. Examination of the head, ears, eyes, nose, and
throat is unremarkable. The pupils are equal and reactive to light and accommodation, and the
cranial nerves are grossly intact. The trachea is centrally positioned, and no masses or tenderness
are noted in the neck. Chest expansion is equal, and no chest bruises are noted. Air entry is equal
in both lung fields, and no additional sounds are present. The point of maximum impulse is in the
left fifth intercostal space, and the heart sounds are normal. There is a soft systolic murmur over
the pulmonary area. Examination of the abdomen reveals a small bruise over the epigastric area
with local tenderness. Some guarding is also noted here. Rigidity is absent. The liver and spleen
are not enlarged, and bowel sounds are diminished. While the physician is examining the
patient's abdomen, the patient reveals that the handlebar of the bicycle struck his abdomen as he
fell. Examination of the extremities reveals no fractures or abnormalities. Chest x-ray is normal.
An abdominal x-ray shows retroperitoneal air. Laboratory studies show: 
Which of the following is the most likely diagnosis? 
 Incorrect Answer ImageA.Cholecystitis
 Incorrect Answer ImageB.Pancreatic injury
 Correct Answer ImageC.Ruptured duodenum
 Incorrect Answer ImageD.Ruptured spleen
 Incorrect Answer ImageE.Soft tissue injury of the epigastrium

A 14-year-old boy is riding his bicycle when he hits a pothole and falls. He seems all right and
goes home. Now, one hour later, he comes to the emergency department complaining of
abdominal pain. He had attacks of spasmodic abdominal pain initially, but the pain has now
become constant. The pain is mainly in the epigastric area and does not radiate elsewhere. A
month before he was treated for gastroenteritis, which he had contracted after a school trip to a
national park. He is not currently on any medications and does not suffer from allergies. He did
not lose consciousness at the time of injury, nor did he injure his head. He said that he had been
wearing a helmet at the time. While in the emergency department, he vomits coffee-ground
material twice. Vital signs are: pulse 100/min, blood pressure 100/60 mm Hg, respirations
20/min, and temperature 39°C (102.2°F). On examination, the child is conscious and in moderate
distress, clutching his upper abdominal region. Examination of the head, ears, eyes, nose, and
throat is unremarkable. The pupils are equal and reactive to light and accommodation, and the
cranial nerves are grossly intact. The trachea is centrally positioned, and no masses or tenderness
are noted in the neck. Chest expansion is equal, and no chest bruises are noted. Air entry is equal
in both lung fields, and no additional sounds are present. The point of maximum impulse is in the
left fifth intercostal space, and the heart sounds are normal. There is a soft systolic murmur over
the pulmonary area. Examination of the abdomen reveals a small bruise over the epigastric area
with local tenderness. Some guarding is also noted here. Rigidity is absent. The liver and spleen
are not enlarged, and bowel sounds are diminished. While the physician is examining the
patient's abdomen, the patient reveals that the handlebar of the bicycle struck his abdomen as he
fell. Examination of the extremities reveals no fractures or abnormalities. Chest x-ray is normal.
An abdominal x-ray shows retroperitoneal air. Laboratory studies show: 
Hemoglobin 14 g/dL
Hematocrit 42%
Leukocyte count 18,300/mm3
Serum creatinine 1.4 mg/dL
BUN 22 mg/dL
Serum glucose 90 mg/dL
Serum amylase 700 U/L
Which of the following is the most appropriate step in management? 
 Correct Answer ImageA.Immediate laparotomy
 Incorrect Answer ImageB.Intravenous fluids, analgesics, and nasogastric tube suction
 Incorrect Answer ImageC.Laparoscopic cholecystectomy
 Incorrect Answer ImageD.Observation
 Incorrect Answer ImageE.Roux-en-Y procedure

The following vignette applies to the next two items.


A 65-year-old woman is brought to the emergency department after a fall. She was walking
down the street when she slipped, fell, and tried to save herself. Thereafter, she developed
excruciating pain in the wrist. She is now holding the right hand with the left. The wrist is
diffusely swollen, discolored on the dorsal aspect, and deviated slightly to the radial side. The
lower end of the forearm is tender, especially on the radial side. The range of movement of the
fingers is normal but painful due to the proximal injury. The elbow and shoulder on the right
show no abnormality. The left wrist is normal. She has no other injuries. Which of the following
is the most likely diagnosis? 
 Correct Answer ImageA.Colles fracture
 Incorrect Answer ImageB.Dislocation of the lower end of the radius
 Incorrect Answer ImageC.Radial ligament sprain
 Incorrect Answer ImageD.Scaphoid fracture
 Incorrect Answer ImageE.Smith fracture

A 65-year-old woman is brought to the emergency department after a fall. She was walking
down the street when she slipped, fell, and tried to save herself. Thereafter, she developed
excruciating pain in the wrist. She is now holding the right hand with the left. The wrist is
diffusely swollen, discolored on the dorsal aspect, and deviated slightly to the radial side. The
lower end of the forearm is tender, especially on the radial side. The range of movement of the
fingers is normal but painful due to the proximal injury. The elbow and shoulder on the right
show no abnormality. The left wrist is normal. She has no other injuries. Based on the history
and findings, it is most important to exclude possibility of trauma to which of the following
nerves 
 Incorrect Answer ImageA.Axillary nerve
 Correct Answer ImageB.Median nerve
 Incorrect Answer ImageC.Musculocutaneous nerve
 Incorrect Answer ImageD.Radial nerve
 Incorrect Answer ImageE.Ulnar nerve

A 65-year-old woman suffered the loss of her husband two years ago. Before the loss, the couple
had been happily married for 20 years. The patient became very upset for a period of 6 months
and cried often. She also had a decreased appetite during this period. Over the next 6 months, the
patient's symptoms resolved. She now comes to the physician for her annual examination and
says she has short periods of longing for the deceased. It is worse on special occasions. She
denies loss of appetite, suicidal thoughts, homicidal thoughts, guilt, decreased sleep, and any
prior psychiatric history. She volunteers her time twice a week at the hospital and is very busy in
many senior clubs. Which of the following is the most likely diagnosis? 
 Incorrect Answer ImageA.Bipolar I disorder
 Incorrect Answer ImageB.Depression, major
 Incorrect Answer ImageC.Dysthymia
 Correct Answer ImageD.Normal grief

A 51-year-old woman comes to the physician because of abdominal pain, diarrhea, sweating,
flushing, and palpitations that begin about 30 minutes after eating a full meal. She recently
underwent surgery for a duodenal ulcer, during which a Billroth II anastomosis was performed.
Which of the following is the most appropriate management at this time?
 Incorrect Answer ImageA.Beta-blocker therapy
 Incorrect Answer ImageB.Conversion of the Billroth II anastomosis to a Billroth I
anastomosis
 Incorrect Answer ImageC.Octreotide scan
 Incorrect Answer ImageD.Somatostatin treatment
 Correct Answer ImageE.Trial of a low-carbohydrate diet

A mother brings her 3-year-old son to the physician because his sleep has become increasingly
restless and he cannot seem to stop itching around his anal area. She has tried using diaper-rash
creams and giving him cool oatmeal baths before going to bed without relief. The boy goes to
daycare where they spend a lot of time outdoors on the playground. He particularly likes to play
in the sandbox with other children. On physical examination, the boy is a well-nourished and in
no acute distress. The only abnormality found is multiple perianal excoriations. Which of the
following is the most appropriate next step in diagnosis?
 Correct Answer ImageA.Cellophane tape test
 Incorrect Answer ImageB.CBC with differential
 Incorrect Answer ImageC.No tests are necessary since this is psychogenic pruritus
 Incorrect Answer ImageD.Skin biopsy
 Incorrect Answer ImageE.Tzanck smear

A 7-year-old boy has had severe headaches for the past several weeks. On examination, his testes
are enlarged and some pubic hair is noted. Spinal fluid cytology is positive for human chorionic
gonadotropin. Which of the following is the most likely diagnosis? 
 Incorrect Answer ImageA.Acoustic neuroma
 Incorrect Answer ImageB.Glioblastoma
 Incorrect Answer ImageC.Meningioma
 Correct Answer ImageD.Pinealoma
 Incorrect Answer ImageE.Pituitary adenoma

A 27-year-old pregnant woman is rushed to the emergency department by paramedics. She had
been in a car accident and sustained blunt trauma to the chest and abdomen, which caused her to
go into premature labor at 32 weeks' gestation. She was hemodynamically stable and delivered a
baby boy without further complications. Apgar scores are 4 and 6 at 1 and 5 minutes,
respectively. Shortly thereafter, he develops tachypnea, nasal flaring, and grunting. The lips and
body are blue in color. After intubation, chest radiography shows a hazy, ground-glass
appearance of the lungs. Which of the following is the most likely diagnosis?
 Incorrect Answer ImageA.Diaphragmatic hernia
 Incorrect Answer ImageB.Meconium aspiration syndrome
 Incorrect Answer ImageC.Persistent fetal circulation
 Correct Answer ImageD.Respiratory distress syndrome
 Incorrect Answer ImageE.Transient tachypnea of the newborn

While sledding, a 6-year-old boy hits a small post and is injured. The parents bring the child
home because he is complaining of left shoulder pain. While eating dinner, the boy passes out,
and the parents call 911. When he arrives at the emergency department, he is unconscious and
appears to be in shock. A quick physical examination shows a contusion over the left 8th and 9th
ribs with bilateral breath sounds throughout all lung fields. No other abnormalities are noted. His
temperature is 37.2°C (99.0°F), pulse is 160/min, respirations are 26/min, and blood pressure is
70/50 mm Hg. Which of the following is the most likely diagnosis?
 Incorrect Answer ImageA.Diaphragmatic rupture
 Incorrect Answer ImageB.Left shoulder separation
 Incorrect Answer ImageC.Lung contusion
 Correct Answer ImageD.Ruptured spleen
 Incorrect Answer ImageE.Tension Pneumothorax

A 69-year-old man has a 5-day history of constipation, vomiting, and crampy abdominal pain.
He reports anorexia and says that he has been passing gas up until the day prior. His past medical
history is significant for a history of hypertension and a right colectomy 6 years ago for colon
cancer. He follows up regularly with his physicians. His blood pressure is 158/94 mm Hg, and
pulse is 112/min. Physical examination shows a softly distended, mildly tender abdomen, with
no obvious hernias. Digital rectal examination is unremarkable. The abdominal x-ray (KUB)
shows dilated loops of small bowel with air-fluid levels and no obvious gas in the colon. Which
of the following is the most appropriate next step in the management of this patient? 
 Incorrect Answer ImageA.Administer laxatives
 Incorrect Answer ImageB.Barium enema
 Incorrect Answer ImageC.CT scan of the abdomen
 Incorrect Answer ImageD.Exploratory laparotomy
 Incorrect Answer ImageE.Nasogastric lavage
 Correct Answer ImageF.Placement of a nasogastric tube (NGT)

A previously healthy 31-year-old woman comes to the physician because of headaches,


dizziness, episodes of confusion, epistaxis, bleeding gums, and fevers for the past two days. She
appears acutely ill and pale, with a temperature of 39.5°C (103.1°F), petechiae, and dysphasia.
Laboratory studies show:
Hemoglobin 8.0 g/dL
Hematocrit 24%
Leukocyte count 3,200/mm3
Neutrophils 60%
Bands 19%
Lymphocytes 15%
Eosinophils 2%
Platelet count 25,000/mm3
Red blood smear many schistocytes
Sodium 134 mEq/L
Potassium 5.0 mEq/L
Chloride 108 mEq/L
Bicarbonate 18 mEq/L
Blood urea nitrogen 24 mg/dL
Creatinine 1.5 mg/dL
Bilirubin, total 4.2 mg/dL
Bilirubin, direct 1.0 mg/dL
Lactate dehydrogenase 184 U/L
Prothrombin time 15 seconds
Partial thromboplastin time 30 seconds
Which of the following is the most appropriate treatment? 
 Incorrect Answer ImageA.Desmopressin acetate
 Incorrect Answer ImageB.Dipyridamole
 Correct Answer ImageC.Plasmapheresis
 Incorrect Answer ImageD.Platelet transfusion
 Incorrect Answer ImageE.Splenectomy

A 52-year-old man is found to have a single, palpable thyroid nodule during a routine health
maintenance examination. Ultrasound is performed and shows a solitary, dominant nodule with
suspicious features that is 4.8 cm in greatest diameter. There are no ultrasonographically
suspicious lymph nodes on either side of the neck. Fine-needle aspiration is performed and
cytology is read as suspicious for malignancy. At the time of surgery, the affected lobe and
thyroid isthmus are removed and sent for frozen section. The pathologist reports papillary stalks
with concentric layers of calcium (psammoma bodies) and makes a diagnosis of papillary thyroid
cancer. Which of the following is the most appropriate next step in management?
 Incorrect Answer ImageA.Closure of the neck without further resection
 Incorrect Answer ImageB.Resection of all tissue within a 1.0 cm radius of the ipsilateral
thyroid bed
 Correct Answer ImageC.Total thyroidectomy
 Incorrect Answer ImageD.Total thyroidectomy and bilateral modified radical neck
dissections
 Incorrect Answer ImageE.Total thyroidectomy and bilateral radical neck dissections

An 83-year-old man is being evaluated for dementia. Over the past 5 years, he has developed
memory impairment, changes in personality, and language difficulties. The symptoms have
worsened in a stepwise fashion, in which his mental status is stable for a while and then suddenly
worsens. His past medical history includes a long history of poorly controlled diabetes,
hypertension, coronary artery disease, and hyperlipidemia. A full dementia workup reveals
diffuse white matter changes on MRI and small, discrete areas of infarction. Laboratory studies
are unremarkable. In addition to optimizing management of this patient's multiple medical
problems, which of the following is an appropriate treatment likely to delay the progression of
his disease?
 Incorrect Answer ImageA.Carbidopa/levodopa
 Correct Answer ImageB.Clopidogrel
 Incorrect Answer ImageC.Dipyridamole
 Incorrect Answer ImageD.Donepezil
 Incorrect Answer ImageE.Vitamin E

Two days after undergoing a mastectomy for breast cancer, a 42-year-old woman is informed
that her most recent studies show that she has lung metastases. She initially takes the bad news
very calmly and is able to discuss treatment options with her physician. The following day she
asks her husband not to come to visit her. When the nurse comes to check her vital signs, the
patient becomes angry and tells her to stop pretending that she cares for the patients because she
surely has other, better things to think about. She refuses to take her medication and yells that
she has the right to decide what she will take and how she should be treated. The same behavior
continues when the physician comes for rounds. The physician understands what the real
problem is but tries to come up with a statement that could help the patient. Which of the
following statements is an example of empathic validation? 
 Incorrect Answer ImageA."Being like an angry child will make things worse for you."
 Incorrect Answer ImageB."It seems that hurting others is your way of feeling better about
yourself."
 Incorrect Answer ImageC."It seems that you want to go home."
 Correct Answer ImageD."Life must seem awful right now. It is difficult to feel helpless
and depend on others for help."
 Incorrect Answer ImageE."The nurse may be distracted today, I agree, but that is not a
reason to yell."

A 62-year-old man with insulin-dependent diabetes develops constant right upper quadrant
abdominal pain and fever, and he does not seek medical help until he becomes nearly comatose
several days later. At the time of hospital admission, he has a serum glucose of 850 mg/mL, a
blood pH of 7.0, and a serum potassium level of 5.4 mEq/L. His urinary output is approximately
15–20 ml per hour, and he looks profoundly dehydrated. He is tender over the right upper
quadrant of the abdomen, and a sonogram confirms the presence of acute cholecystitis. He
receives broad-spectrum antibiotics, 2 liters of normal saline, and an intravenous drip of insulin.
Laboratory studies then show a serum glucose of 350 mg/dL, pH of 7.2, and serum potassium of
5.1 mEq/L. After another liter of saline and continued insulin drip, his glucose is measured at
210 mg/dL, pH is 7.4, and serum potassium is reported as 3.0 mEq/L. His urinary output has
increased to 110 mL per hour. Which of the following is the most appropriate next step in
management? 
 Incorrect Answer ImageA.Bring to the operating room for open cholecystectomy
 Incorrect Answer ImageB.Diuretic administration
 Incorrect Answer ImageC.Increase the insulin dose
 Correct Answer ImageD.Initiate intravenous potassium
 Incorrect Answer ImageE.Proceed with percutaneous cholecystostomy

A 59-year-old diabetic man has a 2-day history of right upper quadrant abdominal pain, nausea,
and vomiting. He had experienced similar episodes in the past, but they all had spontaneous
resolution, and he never sought medical help. This time, the pain began as a colicky pain
radiating to the back and to the right shoulder, but after a few hours, it became constant and
unrelenting. He has a temperature of 38.9°C (102°F). There is pain to deep palpation, muscle
guarding, and rebound tenderness in the right upper quadrant of the abdomen. He has a leukocyte
count of 14,000/mm3, blood glucose of 250 mg/dL, bilirubin of 2.3 mg/dL, and normal alkaline
phosphatase. Intravenous fluids, insulin drip, and intravenous antibiotics are started, and a
sonogram of the right upper quadrant is performed. The study shows gallstones in the
gallbladder, a thick gallbladder wall with air in the wall, and pericholecystic fluid. There is mild
dilation of the intrahepatic bile ducts, but the common bile duct is not dilated. Three hours after
the start of therapy, his pain is worse and the physical findings are more impressive. Which of
the following is the most appropriate management at this time?
 Incorrect Answer ImageA.Admission to ICU with continued medical management
 Correct Answer ImageB.Cholecystectomy
 Incorrect Answer ImageC.ERCP and drainage of the common duct
 Incorrect Answer ImageD.Percutaneous cholecystostomy
 Incorrect Answer ImageE.Surgical decompression of the common duct by T-tube
A 28-year-old woman, gravida 1, para 0, at 41 weeks' gestation comes to the emergency
department because of contractions. Her prenatal course has been unremarkable. She had a
laparoscopic right ovarian cystectomy at the age of 24, and has no other past medical or surgical
history. She takes prenatal vitamins and has no known drug allergies. Physical examination
shows the cervix 4-cm dilated and 60% effaced. Two hours later, she is continuing to contract
every 3 minutes, her cervix is 6-cm dilated and 100% effaced, and the fetal heart rate is 130/min
with accelerations and mild variable decelerations. An amniotomy is performed that reveals thick
green fluid. Which of the following is the most appropriate next step in management?
 Correct Answer ImageA.Amnioinfusion
 Incorrect Answer ImageB.Cesarean delivery
 Incorrect Answer ImageC.Forceps-assisted vaginal delivery
 Incorrect Answer ImageD.Oxytocin
 Incorrect Answer ImageE.Vacuum-assisted vaginal delivery

A 6-year-old boy is brought to the emergency department because of headaches. The parents
report that he has passed very little urine since yesterday and that his urine color was smoky
brown. He was previously well, except for a skin infection 4 weeks ago. His temperature is
37.2°C (99.0°F), pulse is 98/min, respirations are 22/min, and blood pressure is 148/102 mm Hg.
Examination shows periorbital edema, normal lungs and abdomen, normal cardiac rhythm, and a
third heart sound. After controlling his blood pressure with nifedipine, the patient is admitted for
management. Laboratory studies show: 
Complete blood count
Hemoglobin 11.4 g/dL
White cell 8.4 x 109/L
Differential: polymorphs 48%, lymphocytes 42%,
monocytes 8%
Platelets 280 x 109/L
Serum chemistry
Sodium 136 mEq/L
Potassium 4.3 mEq/L
Chloride 106 mEq/L
Bicarbonate 26 mEq/L
Calcium 7.1 mg/dL
BUN 28 mg/dL
Creatinine 2.1 mg/dL
C3 complement 28 mg/dL (normal 75-200)
Albumin 3.9 g/dL
Anti-DNase B titer 1,400 U (normal 240-480)
Urine
pH 6.5
Specific gravity 1.015
Glucose Negative
Protein Moderate
Blood Large
Red cell casts Present
Which of the following is the most appropriate treatment? 
 Incorrect Answer ImageA.Co-trimoxazole (Sulfamethoxazole and trimethoprim)
 Incorrect Answer ImageB.Intravenous immunoglobulin
 Incorrect Answer ImageC.Peritoneal dialysis
 Incorrect Answer ImageD.Prednisone
 Correct Answer ImageE.Salt restriction

A 34-year-old man has a swollen left knee of 2 days duration. He denies any known trauma to
that region and has no prior history of any musculoskeletal complaints. He is in otherwise
excellent health. He is homosexual and practices safe sex with a single partner. On physical
examination, his knee is swollen, tender to palpation, and erythematous and has a limited range
of motion. An arthrocentesis is performed. Which of the following is most suggestive of septic
arthritis in this patient? 
 Incorrect Answer ImageA.A complete blood cell count with 4,300 white blood cells/mm3
 Incorrect Answer ImageB.A joint fluid aspirate with a white blood cell count of 8,000
cells/mm3
 Incorrect Answer ImageC.A joint fluid aspirate with a white blood cell count of 16,000
cells/mm3
 Incorrect Answer ImageD.A joint fluid aspirate with a white blood cell count of 18,000
cells/mm3
 Correct Answer ImageE.A joint fluid aspirate with a white blood cell count of 93,000
cells/mm3

A 32-year-old man with schizophrenia is brought to the hospital by his sister. She relates that the
patient recently has become incoherent and has been locking himself in his room for days at a
time. His family is concerned because he has tried many antipsychotic agents and none of them
seem to have worked. During the examination, the patient stares at the wall and cannot seem to
find the words to respond to questions. Physical examination is unremarkable. The patient is
started on clozapine and scheduled for a visit to the clinic in 1 week. One week later, he arrives
at the clinic for his appointment. Which of the following is the most appropriate next step in
management?
 Correct Answer ImageA.Complete blood count (CBC)
 Incorrect Answer ImageB.Electrocardiogram (ECG)
 Incorrect Answer ImageC.Electroencephalogram (EEG)
 Incorrect Answer ImageD.Prolactin levels
 Incorrect Answer ImageE.Thyroid-stimulating hormone levels (TSH)

A 31-year-old homeless woman is brought to the emergency department after being found face
down on the sidewalk of a city street. The woman has a long history of admissions to the hospital
for alcohol-related issues, including seizures and hallucinations. A male passerby noted that after
she fell to the sidewalk, the woman had what he described as “convulsions” and then vomited.
The woman remained face down until the paramedics arrived about 10 minutes later. On
physical examination, the patient had dry mucous membranes, a jugular venous pressure of less
than 5 cm, and diffuse ecchymoses on her face and breasts. Which of the following vitamins is it
most important to administer prior to volume resuscitation with IV fluids containing glucose? 
 Correct Answer ImageA.Vitamin B1 (Thiamine)
 Incorrect Answer ImageB.Vitamin B3 (Niacin)
 Incorrect Answer ImageC.Vitamin B12 (Cobalamin)
 Incorrect Answer ImageD.Vitamin C
 Incorrect Answer ImageE.Vitamin K

A 62-year-old man comes to the physician because of perineal discomfort and intermittent
drainage and blood noted in his underwear. Four months ago, he had a perirectal abscess drained
surgically, but he says this is not nearly as painful. Physical examination shows a small bud of
granulation tissue 2 cm lateral to the anus, and a cord-like subcutaneous thickening going from
the opening toward the anal canal. Some serous drainage can be expressed from the granulation
tissue. Which of the following is the most likely diagnosis?
 Incorrect Answer ImageA.Anal fissure
 Incorrect Answer ImageB.Crohn disease
 Correct Answer ImageC.Fistula-in-ano
 Incorrect Answer ImageD.Herpes simplex infection
 Incorrect Answer ImageE.Squamous cell carcinoma of the anus

A 25-year-old man is stabbed in the right chest. He comes into the emergency department fully
awake and alert, and, in a normal tone of voice, he states that he feels short of breath. His vital
signs are normal and stable. On physical examination, he has no breath sounds at the right base,
and only faint breath sounds at the apex. He is dull to percussion over the right base. A chest x-
ray film confirms that he has a hemothorax on that side. Which of the following is the most
appropriate next step in management? 
 Incorrect Answer ImageA.Oxygen by mask, analgesics, and no specific intervention
 Incorrect Answer ImageB.Intubation and use of a respirator
 Incorrect Answer ImageC.Insertion of a chest tube in the right second intercostal space
 Correct Answer ImageD.Insertion of a chest tube at the right base
 Incorrect Answer ImageE.Exploratory thoracotomy

 30-year-old woman comes to the physician because of abdominal pains that she describes as
"the worst imaginable. " She has a history of physical complaints including nausea, bloating,
abdominal pain, dysmenorrhea, fatigue, fainting, and dysuria since adolescence. On physical
examination, there is no abdominal stiffness or masses. Tenderness is elicited, but the site varies
and is not reproducible, even a minute or two later. The patient notes that her life is stressful but
she does not seem concerned about any specific life events. She leaves the physician with the
impression that she may have an underlying personality disorder, possibly borderline or
antisocial. Which of the following is the most likely diagnosis?
 Incorrect Answer ImageA.Conversion disorder
 Incorrect Answer ImageB.Factitious disorder
 Incorrect Answer ImageC.Illness anxiety disorder
 Incorrect Answer ImageD.Malingering
 Correct Answer ImageE.Somatic symptom disorder

A 34-year-old man is brought to the emergency department by the police. The policemen say that
they apprehended the suspect after a woman complained that he was "rubbing his penis up
against her” on a crowded subway car. They brought him to the emergency department because
he is "obviously sick" to be doing something "so crazy." His history reveals other arrests for the
same behavior. The man states that he is homeless and unable to keep a job because of his
"constant need to touch unknown women." Which of the following is the most likely diagnosis?
 Incorrect Answer ImageA.Exhibitionistic disorder
 Correct Answer ImageB.Frotteuristic disorder
 Incorrect Answer ImageC.Pedophilic disorder
 Incorrect Answer ImageD.Sexual masochism disorder
 Incorrect Answer ImageE.Voyeuristic disorder

The father of a 4-year-old boy reports that a friend told him that his son seems to be “slow”
developmentally and that the father should take the child in for evaluation and early intervention.
Upon evaluation, the physician finds that the boy copies a cross and a square, hops and walks
downstairs using alternate feet, knows his colors, and tells stories. The father notes that the child
also helps in brushing his teeth and plays well with other children in group play. Physical and
neurological examination are normal. At the end of the examination the child asks, "Can I go out
to play now?" Which of the following is the most appropriate next step in management?
 Incorrect Answer ImageA.Draw blood for a complete blood count and electrolytes in
search for a cause of his delay
 Correct Answer ImageB.Reassure the father that his son is showing normal development
 Incorrect Answer ImageC.Refer the boy to a neurologist to evaluate his delays
 Incorrect Answer ImageD.Refer the boy to a physical therapist to help with his motor
delay
 Incorrect Answer ImageE.Refer the boy to a speech therapist to aid with his speech delay

A 29-year-old woman from India comes to the physician because of genital ulcers. She states
that the ulcers have been progressive over the past several months, but they are not painful. She
has no medical problems. She had a right salpingectomy at the age of 22, for an ectopic
pregnancy. She takes no medications and is allergic to penicillin. Examination shows multiple
highly vascular, "beefy red" ulcerations on the external genitalia. These ulcers bleed easily on
contact. There is no regional lymphadenopathy. Which of the following is the most likely
pathogen? 
 Correct Answer ImageA.Calymmatobacterium granulomatis
 Incorrect Answer ImageB. Chlamydia trachomatis 
 Incorrect Answer ImageC.Herpes Simplex Virus-1
 Incorrect Answer ImageD.Herpes Simplex Virus-2
 Incorrect Answer ImageE.Streptococcus agalactiae
A 40-year-old woman comes to the emergency department because of a fever and severe lower
abdominal pain for the past 3 days. She has no significant past medical history and her
gynecologic history is significant only for an intrauterine device (IUD) for contraception that
was placed 2 months ago. When questioned about her sexual history, she states that she has a
new sexual partner. Her temperature is 39°C (102.2°F), pulse is 102/min, respirations are
20/min, and blood pressure is 130/78 mm Hg. Physical examination shows rebound tenderness in
the lower abdomen without guarding; a purulent discharge coming from the cervix, and there is
cervical motion tenderness and bilateral adnexal tenderness. Which of the following is the most
appropriate next step in management?
 Incorrect Answer ImageA.Admit the patient and start intravenous gentamicin and
clindamycin therapy
 Correct Answer ImageB.Outpatient treatment with ceftriaxone and doxycycline and do
not remove the IUD
 Incorrect Answer ImageC.Perform a diagnostic laparoscopy
 Incorrect Answer ImageD.Remove the IUD and send the patient home with doxycycline
therapy
 Incorrect Answer ImageE.Remove the IUD and send the patient home with no antibiotic
therapy

A 27-year-old woman comes to the physician because of feelings of sadness, anger, and anxiety
in the days before her menstrual cycle. She also complains of difficulty working and
concentrating during these times. She has no medical problems. Her physical examination is
normal. A review of her symptom calendar demonstrates that her symptoms have occurred on a
monthly basis over the past four menstrual cycles and that the symptoms resolve completely
shortly after onset of menses. She has tried taking calcium and vitamin B6 and started an exercise
program, none of which has helped her symptoms. She is prescribed fluoxetine. Which of the
following side effects is this patient most likely to experience? 
 Incorrect Answer ImageA.Breast engorgement
 Incorrect Answer ImageB.Deafness
 Incorrect Answer ImageC.Diabetes
 Incorrect Answer ImageD.Paralysis
 Correct Answer ImageE.Sexual dysfunction

A 4-month-old boy is brought to the physician because of a "funny bobbing of the head" that
started happening over the past several days. He was delivered at full term after an uneventful
pregnancy, is up to date on immunizations, has been developing appropriately, and has had no
trouble with feeding or sleeping. The family history is significant for a seizure disorder in a
maternal uncle. When questioned about the episodes of peculiar movement, the mother describes
them as "bobbing" that would cause an energetic movement of the head toward the knees,
immediately followed by a return to an upright position. The individual movements would last
only a few seconds, but occur in clusters of 10 to 15, and clusters are occurring two to three
times per day. They usually happen upon awakening. On physical examination, the patient is a
well-developed infant in no acute distress. His weight and length are in the 50th percentile. The
neurologic examination is within normal limits but an electroencephalogram shows
hypsarrhythmia. Which of the following is most commonly associated with this disorder?
 Incorrect Answer ImageA.Hereditary congenital lymphedema
 Incorrect Answer ImageB.Hereditary hemorrhagic telangiectasia
 Incorrect Answer ImageC.Klippel-Trenaunay-Weber syndrome
 Incorrect Answer ImageD.Neurofibromatosis type I
 Correct Answer ImageE.Tuberous sclerosis complex

A 50-year-old nurse working in the emergency department reports feeling sweaty and dizzy and
suddenly passes out. She is awakened but seems confused and lethargic. A fingerstick blood
glucose test shows a glucose level of 32 mg/dL. The nurse is given an oral dextrose gel and
quickly improves. She does not understand what has happened, as she is not a diabetic patient
and is thus not taking insulin or oral hypoglycemic agents. For that matter, she does not take any
medications and has been generally healthy. She does note, however, that she occasionally
suffers panic attacks in which she feels anxious, suffering from a fast heart rate and palpitations
and feeling sweaty and hungry. Her symptoms often resolve with eating and rest. The patient is
evaluated for her syncopal episode. A 12-h fast is performed, which induces a hypoglycemic
response of 40 mg/dL. Insulin levels and C peptide levels are found to be elevated after fasting.
Which of the following is the most likely cause of this patient's hypoglycemia? 
 Incorrect Answer ImageA.Alcoholic ketosis
 Incorrect Answer ImageB.Glucagonoma
 Correct Answer ImageC.Insulinoma
 Incorrect Answer ImageD.Surreptitious insulin use
 Incorrect Answer ImageE.Undiagnosed diabetes mellitus

A 32-year-old woman is brought to the hospital after being found with an altered mental status.
Her sister, who accompanied her to the hospital, explains that the patient has been depressed
most of her life and has attempted suicide twice before. Today the patient was found intoxicated
and had apparently been binge drinking for the last 4 days. More concerning, however, were two
empty bottles of acetaminophen found near the patient. The patient's current vital signs are:
temperature 37.0°C (98.6°F), blood pressure 110/82 mm Hg, pulse 94/min, and respirations
28/min. The patient is obtunded and is oriented only to person. Abdominal examination reveals
tender hepatomegaly. Laboratory studies, including an acetaminophen level, are drawn and
currently pending. While waiting for studies to return, the patient is given a charcoal lavage and
her first dose of N-acetylcysteine. If this patient is indeed suffering from acetaminophen
overdose, which of the following is an expected finding of liver function tests? 
 Incorrect Answer ImageA.AST level less than the ALT level, with both greater than
1,000 U/L
 Incorrect Answer ImageB.AST level more than twice the ALT level, ALT level less than
500 U/L
 Correct Answer ImageC.Extremely high aminotransferase levels, greater than 2,000 to
3,000 U/L
 Incorrect Answer ImageD.Near normal alkaline phosphatase and gamma-
glutamyltransferase levels
 Incorrect Answer ImageE.Total protein level that is more than 2 to 3 times the albumin
level

A 54-year-old man comes to the emergency department because of recent onset persistent lower
pelvic pain and dysuria. He notes that he strains when urinating and feels that he cannot
completely empty his bladder. He has no prior history of urinary problems. His past medical
history is significant for hypertension, osteoarthritis, and chronic low back pain. His temperature
is 38.2°C (100.8°F), blood pressure is 165/90 mm Hg, and pulse is 94/min. Physical examination
shows a normal penis and testes without discharge from the urethra. There are no scrotal masses
and no tenderness on palpation of the genitalia. Digital rectal examination is painful with gentle
palpation of the prostate gland. The initial approach to this patient's complaint should include
which of the following?
 Incorrect Answer ImageA.Empiric therapy with ciprofloxacin
 Incorrect Answer ImageB.Postvoid residual urinary volume
 Correct Answer ImageC.Simple urine culture
 Incorrect Answer ImageD.Transrectal ultrasound
 Incorrect Answer ImageE.Vigorous prostatic massage with urine culture immediately
afterward

A 41-year-old woman is brought to the emergency department because her coworkers noticed
that she appeared agitated and was sweating profusely. The patient seems confused and does not
answer any of the physician's questions. Her coworker reports that the patient was diagnosed
with schizophrenia and has been taking haloperidol for many years. She has no known allergies.
Her temperature is 38°C (101°F), blood pressure is 150/100 mm Hg, pulse is 120/min, and
respirations are 24/min. Physical examination shows severe muscle rigidity. Leukocyte count is
12,500/mm3, and creatine phosphokinase level is 1,500 U/L. Which of the following is the most
likely diagnosis?
 Incorrect Answer ImageA.Neuroleptic-induced acute akathisia
 Incorrect Answer ImageB.Neuroleptic-induced acute dystonia
 Incorrect Answer ImageC.Neuroleptic-induced parkinsonism
 Incorrect Answer ImageD.Neuroleptic-induced tardive dyskinesia
 Correct Answer ImageE.Neuroleptic malignant syndrome

An infant is brought to the physician for a routine well-child examination. The parents state that
she is learning new things every day but is still waking up every 3 to 4 hours during the night and
breastfeeding. The mother states that she “is a little exhausted but is not bothered” as long as the
girl is developing well. The parents introduced solid foods into her diet about 6 weeks ago, and
she is tolerating them well. On physical examination, the infant appears to be in no distress. Her
vital signs are within normal limits, as well as weight and length. When the infant is laid down
on the examining table, she squirms, rolls over to a prone position, and attempts to crawl. She
extends to reach the examining light, grasps it in the physician's hand, and laughs when her
mother takes her into her arms. Once in her mother's lap, she sits with her back rounded and
leans forward on her hands. Which of the following is the most likely age of this child?
 Incorrect Answer ImageA.3 months
 Incorrect Answer ImageB.4 months
 Correct Answer ImageC.6 months
 Incorrect Answer ImageD.9 months
 Incorrect Answer ImageE.12 months

A 71-year-old woman has decreased urinary output over the last 48 hours. She denies any current
fevers, chills, or dysuria and any prior such symptoms. Nine months ago she was diagnosed with
a uterine tumor and was found to have metastatic disease fixed to the pelvic wall. She underwent
three courses of radiation therapy, as well as chemotherapy with platinum and doxorubicin. Her
temperature is 37°C (98.6°F), pulse is 84/min, respirations are 16/min, and blood pressure is
156/94 mm Hg. Head and neck examination is normal. No lymphadenopathy is present. Heart
and lung examinations are normal. On abdominal examination, there is a hard, nonmobile
infraumbilical mass. On pelvic examination, the uterus is immobile and extremely firm. There is
bilateral lower extremity edema to the mid-thighs. Laboratory studies show:
Leukocyte 6,900/mm3
Hemoglobin 9.8 g/dL
Hematocrit 29%
Platelets 111,000/mm3
Sodium 137 mEq/L
Potassium 3.9 mEq/L
Bicarbonate 22 mEq/L
BUN 24 mg/dL
Creatinine 2.3 mg/dL
Which of the following is the most likely cause of her renal insufficiency? 
 Correct Answer ImageA.Bilateral ureteral obstruction
 Incorrect Answer ImageB.Malignant Hypertension
 Incorrect Answer ImageC.Platinum-induced acute tubular necrosis
 Incorrect Answer ImageD.Platinum-induced glomerulonephritis
 Incorrect Answer ImageE.Unilateral ureteral obstruction

While in the shower, a 23-year-old otherwise healthy man finds a "lump" in his left testicle. He
comes to the physician concerned that he has cancer. Physical examination shows a solid,
nontender, and fixed mass in the left testicle. The remainder of the physical examination is
unremarkable. An ultrasound is performed. Which of the following is the most likely diagnosis? 
 Incorrect Answer ImageA.Choriocarcinoma
 Incorrect Answer ImageB.Fibroma
 Incorrect Answer ImageC.Leydig cell tumor
 Correct Answer ImageD.Seminoma
 Incorrect Answer ImageE.Teratocarcinoma

A previously healthy 42-year-old man suddenly develops sharp, severe abdominal pain that
radiates into his right groin and is associated with waves of nausea followed by vomiting and
perspiration. He reports frequent urination since the pain began but denies hematuria. He is
unable to get comfortable and is very restless. On physical examination he has mild right
costovertebral angle tenderness but no other significant findings. Which of the following is the
most likely diagnosis? 
 Incorrect Answer ImageA.Appendicitis
 Incorrect Answer ImageB.Herpes zoster
 Incorrect Answer ImageC.Right testicular torsion
 Correct Answer ImageD.Right ureteral calculus
 Incorrect Answer ImageE.Sigmoid diverticulitis

A 71-year-old retired schoolteacher is brought to the physician by his daughter for evaluation.
She is concerned because he appears to have become increasingly forgetful. He has lost his car
keys on three separate occasions over the last month. On another occasion, he left his apartment
door ajar after leaving for the day. She became particularly concerned when he was found
wandering lost in the community in which he lived for many years. He is alert and oriented to
place, person, and time. He can clearly relate his prior educational, social, and occupational
histories. Physical examination is unremarkable. Which of the following is the most appropriate
next step in management? 
 Incorrect Answer ImageA.Cerebrospinal fluid examination
 Incorrect Answer ImageB.CT scan of the head
 Incorrect Answer ImageC.Electroencephalogram
 Correct Answer ImageD.Mini mental state examination
 Incorrect Answer ImageE.Trial of antidepressant

A 55-year-old female with a kidney transplant secondary to end-stage renal disease comes to her
primary care physician. She complains of mild shortness of breath on exertion and generalized
weakness for the last 2 months. Vital signs, physical examination, and chest x-ray are normal. A
tuberculin test was performed as part of her diagnostic workup and showed 7 mm induration
after 72 hours. Sputum samples for acid-fast bacteria are negative. The patient denies any contact
with sick people and has no prior history of tuberculosis. Which of the following is the most
appropriate next step in management? 
 Incorrect Answer ImageA.Isoniazid for 1 month
 Incorrect Answer ImageB.Isoniazid for 3 months
 Correct Answer ImageC.Isoniazid for 9 months
 Incorrect Answer ImageD.No treatment is necessary, continue observation
 Incorrect Answer ImageE.Order sputum culture

The following vignette applies to this item and the next item. 
A 43-year-old woman comes to the physician today because of muscular pain. Over the past few
years, she has had headaches, shortness of breath, nasal congestion, difficulty swallowing, and
diffuse muscular pain. She has been to different doctors in the past few years and despite
extensive imaging and laboratory studies, no definitive etiology for her symptoms has been
found. As in the past, physical examination and laboratory studies are unremarkable during this
visit. Which of the following is the most likely diagnosis?
 Incorrect Answer ImageA.Conversion disorder
 Incorrect Answer ImageB.Illness anxiety disorder
 Incorrect Answer ImageC.Malingering
 Incorrect Answer ImageD.Obsessive-compulsive disorder
 Incorrect Answer ImageE.Panic disorder
 Correct Answer ImageF.Somatic symptom disorder

Over the past few years a 43-year-old woman has had nausea and poor appetite. There have been
no significant changes in her weight, and she continues to exercises regularly. She admits
relationship difficulties with her husband because of intense pain in intercourse. A systems
review also reveals the presence of chronic headaches, shortness of breath, nasal congestion,
difficulty swallowing, and diffuse muscular pain. She has been to eight different doctors in the
past few years and despite extensive imaging and laboratory studies, no definitive etiology for
her problems has been found. Emphasis on which of the following is most important in the
management of this patient? 
 Incorrect Answer ImageA.The chronic and unremitting nature of the symptoms
 Correct Answer ImageB.Identification of one primary physician and regular
communication between primary physician and consultants
 Incorrect Answer ImageC.Immediate patient awareness that her symptoms are all
psychologic in origin
 Incorrect Answer ImageD.The need for serial laboratory and imaging testing of the
patient
 Incorrect Answer ImageE.The patient's eventual need for disability coverage

A 74-year-old man with COPD comes to the emergency department with severe shortness of
breath. He has been treated with antibiotics over the last month because of persistent bronchitis
symptoms. He is extremely tachypneic and cyanotic. An arterial blood gas shows a pH of 7.27, a
PCO2 of 74 mm Hg, and a PaO2 of 50 mm Hg. Over the next hour, his breathing becomes
increasingly labored and he is intubated. A chest x-ray reveals proper placement of the
endotracheal tube and areas of consolidation in both the right middle lobe and the left lower lobe.
A left-sided pleural effusion is also present. Over the next four days, he is treated with
intravenous levofloxacin and gradually improves. At the time of hospital discharge, he is given
prescriptions for albuterol nebulizers, inhaled fluticasone/salmeterol, and a daily aspirin. A
repeat ABG prior to his discharge shows a pH of 7.35, a PCO2 of 57 mm Hg, and a PaO2 of 53
mm Hg. Which of the following factors in this patient's history indicates that the patient should
receive home oxygen therapy?
 Correct Answer ImageA.An arterial blood gas with a PO2 less than 55 mm Hg
 Incorrect Answer ImageB.The long history of COPD
 Incorrect Answer ImageC.The need for chronic beta-agonist treatment
 Incorrect Answer ImageD.The recent history of bilateral pneumonia
 Incorrect Answer ImageE.The recent need for intubation
A 65-year-old woman comes to her physician because of severe left lower abdominal pain and
nausea since the morning. She also had a fever for which she took a "couple of aspirins," which
seemed to help but had little effect in alleviating the pain. She admits experiencing previous
episodes of mild pain in the past, but the pain usually went away on its own. When prompted,
she endorses passing flatus. She takes multivitamins regularly and a herbal supplement for her
chronic constipation. Her past medical history is otherwise unremarkable, except for an
appendectomy at age 16. She is married and has 4 living children. She became menopausal at the
age of 53. Her temperature is 39.0°C (102.2°F), pulse is 100/min, respirations are 20/min, and
blood pressure is 100/60 mm Hg. She appears flushed. Abdominal examination shows tenderness
and guarding over the lower-left flank. A rectal examination is difficult to perform due to pain.
Which of the following is the most likely diagnosis?
 Incorrect Answer ImageA.Adhesions
 Incorrect Answer ImageB.Carcinoma of the sigmoid colon
 Correct Answer ImageC.Diverticulitis
 Incorrect Answer ImageD.Ischemic colitis
 Incorrect Answer ImageE.Twisted left ovarian cyst
 Incorrect Answer ImageF.Volvulus of the sigmoid colon

A 19-year-old college student comes to the clinic with a 1-month history of intermittent crampy
abdominal pain, as well as alternating diarrhea and constipation. The pain is relieved after a
bowel movement. The diarrhea is watery, and she has to rush to the bathroom at very frequent
intervals. When constipation occurs, it lasts for five to seven days, and her stools are pellet-like.
She has had no rectal bleeding or weight loss. She claims that she is under stress related to the
recent divorce of her parents, who were married for 22 years. Her past history is significant for a
hospitalization for pneumonia at age 14, and 2 episodes of urinary tract infections. She is
sexually active with her third boyfriend in three years. He uses condoms, and she uses pills for
contraception. She has no urinary symptoms at this time. Her temperature is 37.0°C (98.6°F).
Her oral cavity is moist. Her lungs sound clear, and heart sounds are normal. Her abdomen is
slightly tender in the right iliac fossa, and bowel sounds are present. A digital rectal examination
reveals hard, formed stools in the vault that are guaiac-negative. The best treatment for this
condition involves which of the following? 
 Correct Answer ImageA.Dietary modification and reassurance
 Incorrect Answer ImageB.Metronidazole
 Incorrect Answer ImageC.Prednisone
 Incorrect Answer ImageD.Proton pump inhibitors

 Incorrect Answer ImageE.Stool culture

A previously healthy 22-year-old college student who believes that aliens will soon invade the
world is convinced by his roommate to see a physician. The student insists that what he says is
true, and that his friend is insane to believe otherwise. He was also recently known to associate
with the leader of a cult known to believe firmly in this view. He tells his friend and the
physician that he is not crazy. His speech is organized, and he has no hallucinations. His
roommate is certain that he does not use any drugs. The student agrees to basic laboratory tests
and a physical examination, all of which are normal. Which of the following is the most likely
diagnosis? 
 Incorrect Answer ImageA.Brief psychotic disorder
 Incorrect Answer ImageB.Schizoaffective disorder
 Incorrect Answer ImageC.Schizophrenia, disorganized type
 Incorrect Answer ImageD.Schizophreniform disorder
 Correct Answer ImageE.Shared psychotic disorder

A mother brings her 8-month-old boy to the pediatrician because he appears "colicky." She states
that he appears happy and healthy until abdominal pain overtakes him. The pain is short-lived,
resolves on its own, and generates no problems until the next episode arises. These episodes
began yesterday. His temperature is 37.0°C (98.6°F). Physical examination shows a happy and
playful infant in no acute distress. There is a vague abdominal mass in the right upper quadrant,
and the right-lower quadrant appears "empty." His diaper has a small amount of stool mixed with
blood and mucus. Which of the following is the most appropriate next step in management?
 Incorrect Answer ImageA.Obtain plain films of the abdomen
 Incorrect Answer ImageB.Order a CT scan of the abdomen
 Incorrect Answer ImageC.Perform scrotal ultrasound
 Incorrect Answer ImageD.Plan for exploratory surgery
 Correct Answer ImageE.Send him for a barium enema

A 72-year-old man comes to the physician because of a non throbbing headache that has
persisted for three months. It involves the right side of the head, is moderately severe, is present
on awakening, and is not relieved by acetaminophen. Physical examination is unremarkable.
Fundoscopy is normal and there is no tenderness to palpation of the temporal arteries. The
temporomandibular joint moves without pain. Laboratory examination shows an erythrocyte
sedimentation rate of 106 mm, and complete blood count and chemistry are normal. Which of
the following is the most appropriate initial management strategy? 
 Incorrect Answer ImageA.Head MRI with and without contrast
 Incorrect Answer ImageB.NSAIDs and observation for the development of any visual
symptoms
 Correct Answer ImageC.Start glucocorticoids and arrange for an urgent temporal artery
biopsy
 Incorrect Answer ImageD.Start glucocorticoids and biopsy only if there is no response to
treatment
 Incorrect Answer ImageE.Visual field examination by a neuro- ophthalmologist

A 21-year-old white male visits his primary care physician for advice on insulin management.
The patient was diagnosed 6 years ago with diabetes mellitus type 1 and placed on regular and
NPH insulin 2x daily. He has been taking 8 units of regular insulin and 16 units of NPH insulin
in the morning, and then 4 units of regular insulin and 8 units of NPH in the evening. He has
good overall blood glucose control and feels well, but has recently noticed an asymptomatic rise
in the morning blood glucose level, ranging 150-180 mg/dL. Physical examination and vital
signs are within normal limits. The physician requests a 3 AM blood glucose level which the
patient reports to be 160 mg/dL. Which of the following is the most appropriate next step in
management? 
 Incorrect Answer ImageA.Increase dose of NPH and regular insulin in the evening
 Incorrect Answer ImageB.Increase dose of NPH and regular insulin in the morning
 Incorrect Answer ImageC.Increase dose of regular insulin in the evening
 Incorrect Answer ImageD.Increase dose of NPH insulin in the morning
 Correct Answer ImageE.Increase dose of NPH insulin in the evening

A 69-year-old retired police officer is seen in the clinic for a 3-year history of abnormal sleep
behavior. His wife provides the description of the behavior. Initially averaging once a week, the
episodes now occur almost on a nightly basis with varying degrees of severity. The onset,
usually occurring between 4 and 6 A.M., is abrupt, with wild screaming, kicking, and hitting, and
often culminates in his leaping out of bed and slamming into the wall. When he wakes up, he is
bewildered by the scene around him, but is fully conscious and alert, and remembers a distinct
and action-packed dream. Getting back to sleep is not difficult, and repeat episodes are rare. He
exercises regularly, has no daytime sleepiness, and sleeps at a regular time and for a regular
duration (11 P.M. to 6 A.M.). There is no abnormal daytime behavior or history suggestive of a
psychiatric disorder; and there is no history of spousal abuse. Physical examination is normal
except for healing bruises on the forehead. He states that his dreams have become more and
more action-packed and aggressive over the past two to three years. Which of the following is
the most likely diagnosis? 
 Incorrect Answer ImageA.Nightmares
 Incorrect Answer ImageB.Night terrors
 Incorrect Answer ImageC.Nocturnal seizures
 Correct Answer ImageD.Rapid eye movement (REM) behavior disorder
 Incorrect Answer ImageE.Sleepwalking

A 15-year-old girl has a 1-cm, round mass in the midline of the neck, at the level of the hyoid
bone. It has been present for at least 6 or 7 years, but did not become a problem until it got
infected and drained a few months ago. Examination at this time shows no residual
inflammation. There is a small scar where the spontaneous drainage took place. Beneath the skin
a 1-cm, round cystic mass can be felt; and when the tongue is pulled at the same time that the
mass is held, there seems to be a connection between the two. In addition to removing the
obvious palpable mass, adequate resection of this lesion requires which of the following
procedures? 
 Incorrect Answer ImageA.Combined intraoral and neck incisions
 Incorrect Answer ImageB.Frozen sections to verify clear margins
 Correct Answer ImageC.Removal of the central portion of the hyoid bone
 Incorrect Answer ImageD.Removal of submental lymph nodes
 Incorrect Answer ImageE.Resection of the thyroid gland in continuity
A 50-year-old woman comes to the physician because of a skin rash for the past 2 weeks. She
reports that she developed multiple small, erythematous, scaly papules on her shoulders,
forearms, neck, and upper torso after starting diltiazem for the treatment of hypertension and
minocycline for acne rosacea. Additionally, since starting the new medications, she has had low-
grade fevers and diffuse arthralgias. Physical examination, aside from the described rash, shows
hepatomegaly but is otherwise unremarkable. An antinuclear antibody (ANA) test is ordered,
which is positive. Based on these findings, the diagnosis of drug-induced lupus erythematosus is
considered. Which of the following diagnostic tests will most likely be abnormal in this patient?
 Incorrect Answer ImageA.Anti-DNA antibodies
 Correct Answer ImageB.Anti-histone antibodies
 Incorrect Answer ImageC.Anti-Smith antibodies
 Incorrect Answer ImageD.Complement level
 Incorrect Answer ImageE.Coombs' test

A 38-year-old alcohol and cocaine addict comes to the emergency department, stating he "needs
to be hospitalized." He is very vague and makes poor eye contact. He reports being "depressed
and suicidal." He has no place to go because he has been kicked out of the halfway house since
he relapsed on crack cocaine. He blames his ex-girlfriend who offered him some crack to smoke.
When the physician insists on more details about the symptoms, he gets angry and accuses the
physician of being cruel and arrogant. He threatens to leave the hospital and throw himself in
traffic. He gives permission for the physician to obtain data from the halfway house but then
cannot remember the name and phone number. Physical examination is unremarkable. During
his interview, his associations are tight and there is no evidence of psychotic symptoms. Urine
drug screen is negative. Considering these findings, which of the following conditions should be
ruled out first? 
 Incorrect Answer ImageA.Delusional disorder
 Incorrect Answer ImageB.Factitious disorder
 Correct Answer ImageC.Malingering
 Incorrect Answer ImageD.Schizophrenia
 Incorrect Answer ImageE.Substance-induced mood disorder

A 24-year-old man suffers blunt abdominal trauma in a car accident. He undergoes an


exploratory laparotomy and repair of multiple intra-abdominal injuries including small bowel
and colon. The operation lasts 6 hours, and he receives approximately 10 liters of Ringer's lactate
solution and 5 units of packed red blood cells. At the end of the operation, all the tissues are
grossly swollen, and abdominal wall closure is not possible. He has a mesh of absorbable
material stapled all around the open abdominal wall edges. Three weeks later, when granulation
tissue has completely covered the mesh, he has split thickness skin grafts placed over the
granulation tissue. The grafts take successfully. This modality of therapy may result in which of
the following conditions?
 Correct Answer ImageA.A large ventral hernia that will require subsequent repair
 Incorrect Answer ImageB.Paralytic ileus requiring prolonged parenteral nutrition
 Incorrect Answer ImageC.Permanent increase in the volume of extracellular fluid
 Incorrect Answer ImageD.Respiratory and renal failure
 Incorrect Answer ImageE.Strong abdominal wall requiring no further surgical revisions

A previously healthy 3-year-old girl is brought to the physician because of ear pain, irritability,
and poor appetite for 4 days. She is diagnosed with bilateral acute otitis media and prescribed a
10-day course of oral antibiotics. Three weeks later, she is brought back to the pediatrician's
office, and the mother reports that her daughter completed the entire course of the antibiotics.
The child's irritability and poor appetite have resolved. Physical examination shows the tympanic
membranes are dull and without movement on pneumatic otoscopy; she has persistent bilateral
middle-ear effusions. Which of the following is the next best step in management?
 Incorrect Answer ImageA.Prescribe another course of antibiotics
 Incorrect Answer ImageB.Prescribe a course of steroids
 Correct Answer ImageC.Reassurance and close follow-up
 Incorrect Answer ImageD.Recommend ear ventilation tube placement
 Incorrect Answer ImageE.Tympanocentesis with culture

A 28-year-old woman, gravida 1, para 0, at 41 weeks’ gestation comes to the labor and delivery
department because of contractions. She states that the contractions began this morning and have
been increasing in intensity and duration. Her prenatal course has been uncomplicated. Physical
examination shows the cervix is 4 cm dilated and 100% effaced; the vertex is at -2 station. The
fetal heart rate is 140/min with moderate variability. The patient is contracting every 2 to 3
minutes and breathing through her contractions. Which of the following is the most appropriate
next step in management?
 Incorrect Answer ImageA.Amnioinfusion
 Incorrect Answer ImageB.Artificial rupture of membranes
 Incorrect Answer ImageC.Cesarean delivery
 Correct Answer ImageD.Observation
 Incorrect Answer ImageE.Vacuum-assisted fetal lowering

A 2-month-old girl is brought to the physician because her parents are concerned about her large
head size. She was born at term to a primigravid mother after an unsupervised pregnancy by
cesarean section due to labor that failed to progress. The head circumference was 36 cm at birth.
The baby was well for the first month, but recently has been vomiting intermittently after feeds.
The mother describes some vomiting episodes as "forceful." At today's visit, the head
circumference is 44 cm (>98th percentile) with a normal head shape. The anterior fontanelle is
full and tense and the skull sutures are separated by 0.5 cm. There is no dimorphism or
congenital malformations. The baby appears alert but does not fixate her gaze or follow colored
objects. The tone on both sides is normal and she moves all four limbs. Examination of the spine
is normal. Lung and cardiac examinations are unremarkable and there is no hepatosplenomegaly.
Which of the following is the most appropriate next step in management?
 Incorrect Answer ImageA.Chromosomal analysis
 Correct Answer ImageB.CT scan of the head
 Incorrect Answer ImageC.Lumbar puncture
 Incorrect Answer ImageD.Measurement of mother's and father's head circumferences
 Incorrect Answer ImageE.TORCH infections titer

 44-year-old woman with a history of recurrent biliary colic presents with 18 hours of severe
right upper quadrant pain, fever, and jaundice. An abdominal ultrasound reveals a markedly
dilated common bile duct. Multiple gallstones are seen in the gallbladder. Which of the following
would best determine whether there is cystic duct obstruction? 
 Incorrect Answer ImageA.Abdominal CT scan
 Incorrect Answer ImageB.Abdominal MRI
 Incorrect Answer ImageC.Abdominal ultrasound with Doppler flow studies
 Incorrect Answer ImageD.Endoscopic retrograde cholangiopancreatography (ERCP)
 Correct Answer ImageE.HIDA scan

A 26-year-old man with schizophrenia comes to the emergency department because of a 2-hour
history of involuntary contractions of the muscles in his neck. He states that he was watching
television and "all of a sudden I turned my head and my neck locked". The patient notes that he
began taking haloperidol 3 days earlier. Physical examination shows no abnormalities except
torticollis. Which of the following is the most appropriate pharmacotherapy?
 Incorrect Answer ImageA.Amantadine
 Correct Answer ImageB.Benztropine
 Incorrect Answer ImageC.Bromocriptine
 Incorrect Answer ImageD.Clonidine
 Incorrect Answer ImageE.Propranolol

A 40-year-old woman is brought to the hospital after overdosing on alcohol and pills. In talking
to a psychiatrist, she denies any prior psychiatric problems but says that about a week ago her
apartment burned down. She was trying to get a job before that happened, but when she realized
that she had lost everything she had, and that moving in with her family would probably not be
possible, she decided to take her own life. Which of the following is the most likely diagnosis? 
 Incorrect Answer ImageA.Acute stress disorder
 Correct Answer ImageB.Adjustment disorder
 Incorrect Answer ImageC.Post traumatic stress disorder
 Incorrect Answer ImageD.Malingering
 Incorrect Answer ImageE.Major depressive disorder

A 28-year-old woman with a history of schizophrenia is brought to the hospital by a friend. The
woman had been an inpatient at a psychiatric hospital and, for several months after being
discharged, had been maintained on haloperidol decanoate intramuscular injections. The friend
tells the doctor that after the last injection two days ago, the woman has seemed "strange." She is
stiff, cannot swallow or speak clearly, and appears “shaky.”The friend is concerned that she has
some kind of infection because she has a fever. Her temperature is 38.7°C (101.7°F), blood
pressure is 157/104 mm Hg, pulse is 122/min, and respirations are 24/min. She has increased
tone in her neck and extremities, and appears tremulous, diaphoretic, and confused. Her
leukocyte count is 19,600/mm3 and the serum creatine phosphokinase is markedly elevated. A
workup for infection is negative. Which of the following is the most likely diagnosis?
 Incorrect Answer ImageA.Acute dystonic reaction
 Incorrect Answer ImageB.Lethal catatonia
 Incorrect Answer ImageC.Malignant hyperthermia
 Correct Answer ImageD.Neuroleptic malignant syndrome
 Incorrect Answer ImageE.Serotonin syndrome

A 76-year-old man is brought to the emergency department (ED) by ambulance for altered
mental status. The patient has a past medical history significant for multiple myeloma. He was
last seen by his primary care physician 4 weeks ago and was reported by family members to be
well at that time. Two days ago, the family noticed that the patient had increased confusion and
over the next 48 hours, slowly became obtunded and could no longer follow simple commands.
On physical examination, his pulse is 110/min and his blood pressure is 90/50 mm Hg. His
mucous membranes are dry and there is no jugular venous pulse evident. His lungs are clear and
he is alert only to person and cannot follow commands. His deep tendon reflexes are brisk.
Serum chemistries and toxicologies are sent. Which of the following is the most appropriate
initial step in management?
 Incorrect Answer ImageA.Administer IV bicarbonate
 Incorrect Answer ImageB.Administer IV pamidronate
 Incorrect Answer ImageC.Administer IV salmon calcitonin
 Incorrect Answer ImageD.Begin aggressive diuresis with furosemide
 Correct Answer ImageE.Begin aggressive intravenous hydration and diuresis

A 20-year-old college student comes to the emergency department because of the sudden onset
of excruciating left-sided flank pain radiating to the labia. She states that the flank pain is
associated with tenesmus, rectal pain, nausea, and vomiting. She had 2 episodes of hematuria.
She has no chronic medical conditions and has never had any similar episodes in the past. She
has recently started taking "megadoses" of vitamin A, vitamin D, vitamin E, pyridoxine, and
methylphenidate (her roommate's prescription) to "help with final examinations". Her
temperature is 37.8℃ (100℉), blood pressure is 130/80 mm Hg, pulse is 80/min, and
respirations are 26/min. Examination shows severe left costovertebral angle tenderness. An x-ray
film of the kidneys, ureter, and bladder (KUB) shows a radiopaque structure in the left ureter.
Which of the following is the most likely explanation for these findings?
 Incorrect Answer ImageA.Vitamin A excess
 Correct Answer ImageB.Vitamin D excess
 Incorrect Answer ImageC.Vitamin E excess
 Incorrect Answer ImageD.Methylphenidate
 Incorrect Answer ImageE.Pyridoxine excess

A patient with acute hepatitis B comes to the office complaining of severe fatigue, low-grade
fevers and weight loss. He was diagnosed 2 weeks earlier, and hepatitis B surface antigen was
positive. A physical examination reveals jaundice with diffuse skin excoriations. His liver and
spleen are both markedly enlarged and tender. Peripheral edema is present. Which of the
following would be the worst prognostic sign? 
 Incorrect Answer ImageA.Albumin of 3.1 g/dL
 Incorrect Answer ImageB.Bilirubin of 9.4 mg/dL
 Correct Answer ImageC.Prothrombin time of 19 seconds with an INR of 2.1
 Incorrect Answer ImageD.SGOT (AST) of 2 U/L
 Incorrect Answer ImageE.SGOT (AST) of 1200 U/L

A 2-week-old infant is brought to the physician because of vomiting. The infant was born at term
at home to a 23-year-old woman who did not receive any prenatal care. Physical examination
shows dehydration and ambiguous genitalia. Laboratory studies show hyponatremia. Additional
laboratory studies are most likely to show which of the following findings?
 Incorrect Answer ImageA.Decreased levels of plasma renin
 Incorrect Answer ImageB.Decreased levels of serum 17-hydroxyprogesterone
 Incorrect Answer ImageC.Elevated levels of aldosterone
 Incorrect Answer ImageD.Elevated levels of serum chloride
 Correct Answer ImageE.Elevated levels of serum potassium

A 22-year-old woman, gravida 2, para 0, at 9 weeks' gestation comes to the emergency


department because of vaginal bleeding and abdominal cramping. She states that her symptoms
started about 8 hours ago and have steadily worsened. Her first pregnancy resulted in a
spontaneous abortion at 8 weeks' gestation. She has no other medical or surgical history.
Examination shows normal vital signs. She has no abdominal tenderness. Speculum examination
shows tissue in the vagina, which is removed and seen to be an intact 9-week fetus with placental
tissue. Her cervix is closed, there is no active bleeding, and her uterus is firm and mildly
enlarged. An ultrasound shows no evidence of products of conception in the uterus. Which of the
following is the most appropriate management? 
 Correct Answer ImageA.Observation and discharge if stable
 Incorrect Answer ImageB.Hospital admission
 Incorrect Answer ImageC.Dilation and curettage
 Incorrect Answer ImageD.Laparoscopy
 Incorrect Answer ImageE.Laparotomy

A 25-year-old man is brought to the emergency department after being involved in a motor
vehicle crash. He has sustained major facial trauma with obvious mandible and midface fractures
in addition to multiple facial lacerations. The oral cavity contains blood coming from his
lacerations and fractures. His neck and chest have no obvious signs of trauma. While he is being
transported from the ambulance stretcher to the emergency room bed, his endotracheal tube,
which was placed by paramedics at the scene of the crash, becomes completely dislodged. Three
attempts at oral intubation are unsuccessful, as are attempts to perform bag-mask ventilation.
Placement of a laryngeal mask airway also fails to establish ventilation. Which of the following
is the best next step in obtaining a stable airway?
 Incorrect Answer ImageA.Attempt oral intubation again in the emergency room
 Incorrect Answer ImageB.Attempt oral intubation in the operating room
 Incorrect Answer ImageC.Attempt nasotracheal intubation via flexible bronchoscopy
 Correct Answer ImageD.Perform a cricothyrotomy
 Incorrect Answer ImageE.Perform a tracheotomy in the operating room

A 37-year-old man seeks help for a painful right ankle. He says that the day before, he was
playing basketball and tried to jump on his toes to reach the basket, when he heard a pop and felt
sharp acute pain in his right Achilles tendon. He fell to the ground and had to stop playing
because of the pain. He can still walk, but is limping, and he cannot stand tiptoe on the affected
side. The area is swollen, and therefore, it is not possible to actually feel the Achilles tendon by
direct palpation. Squeezing the calf does not provoke any plantar flexion. A blood pressure cuff
inflated around the mid calf shows no increase in pressure when the ankle is dorsiflexed. Which
of the following is the most likely diagnosis? 
 Incorrect Answer ImageA.Fractured tarsal navicular
 Incorrect Answer ImageB.Plantar fasciitis
 Correct Answer ImageC.Ruptured Achilles tendon
 Incorrect Answer ImageD.Sprained ankle
 Incorrect Answer ImageE.Sural nerve injury

A psychiatrist doing consultations for a large corporation is asked to speak to a 31-year-old


woman who has been with the company for 1 year. She is a competent worker, but her
interpersonal style and dress have caused some concern among her coworkers. She dresses
provocatively, with skirts that are inappropriately short and blouses that are tight and revealing.
She flirts with clients and makes sexually suggestive statements when she is not the center of
attention during meetings. When talking to the woman, the physician notes that she is visibly
hurt and upset because she describes her coworkers as her "best friends," even though her contact
with these people is limited to the work setting. Which of the following is the most likely
diagnosis? 
 Incorrect Answer ImageA.Borderline personality disorder
 Correct Answer ImageB.Histrionic personality disorder
 Incorrect Answer ImageC.Major depressive disorder
 Incorrect Answer ImageD.Panic disorder
 Incorrect Answer ImageE.Schizoid personality disorder

A 71-year-old woman is recovering from a severe case of community-acquired pneumonia that


required intubation and transfer to the intensive care unit. The patient is stabilized and her
oxygenation slowly improves postintubation. While reviewing her chart, the intern taking care of
her notices that in the past she has had a borderline elevated thyroid stimulating hormone (TSH)
level, and decides to check a full thyroid function panel. The results are as follows: 
Thyroid-stimulating hormone (TSH) 0.2 µU/L
Thyroxine (T4) 2.1 µg/dL
Triiodothyronine (T3) resin uptake 28%
Although the patient is unable to communicate because of her intubation and sedation, a careful
interview of the patient's family fails to reveal any evidence of symptoms consistent with
hypothyroidism. No glandular abnormalities are noted on thyroid examination. Regarding this
patient's abnormal thyroid function tests, which of the following is the most appropriate next step
in the management? 
 Incorrect Answer ImageA.Bedside ultrasound of the thyroid gland
 Incorrect Answer ImageB.MRI of the brain and the pituitary gland
 Correct Answer ImageC.No treatment, recheck TSH as an outpatient
 Incorrect Answer ImageD.Recheck thyroid function tests in 2 days
 Incorrect Answer ImageE.Start thyroxine (T4) replacement therapy now

A 28-year-old white man of German descent comes to the emergency department because of a
“bad sunburn” sustained over the weekend while camping. He denies a history of sunburn and
was surprised when he developed painful and swollen red patches on his hands and face after
being exposed to the sun for only a few hours during the past 2 days. He applied sunscreen with
a sun protection factor of 15 to all sun-exposed areas prior to sun exposure as he had done in the
past. He is otherwise in good health and has no medical problems. His current medications are
limited to acne treatment started approximately 6 months earlier including minocycline 100 mg
daily, topical benzoyl peroxide gel in the morning, and topical tretinoin cream at night. Family
history is unremarkable for any skin disorders. On physical examination, the patient is in mild
distress. Vital signs are normal. There are well-demarcated areas of erythema and edema on his
face, around his mouth and eyes, and on the dorsum of his hands. Examination of the remainder
of the skin shows several small depigmented patches around the umbilicus and on the glans
penis. The remainder of the physical examination is normal. Which of the following statements
is most correct about his condition?
 Incorrect Answer ImageA.Depigmented patches on the trunk and genitals are unrelated
 Incorrect Answer ImageB.Direct immunofluorescence of lesional skin would be positive
 Incorrect Answer ImageC.Future sun exposure is less likely to cause this type of skin
reaction
 Correct Answer ImageD.Lesional skin biopsy will be devoid of melanocytes
 Incorrect Answer ImageE.The patient sustained a phototoxic drug reaction

A 4-year-old girl is brought to the physician because of a fever and a rash. The mother states that
about a week ago the child started to feel sick, and about 4 days ago broke out in a rash on her
face and body. She has no significant past medical history and is up-to-date on all of her
immunizations. Her temperature is 38.6°C (101.5°F). There is bilateral erythema on the cheeks,
circumoral pallor, and a symmetric, lace-like rash on her trunk and extremities. Which of the
following is the most likely causal organism?
 Incorrect Answer ImageA.Adenovirus
 Incorrect Answer ImageB.Herpes virus
 Correct Answer ImageC.Parvovirus
 Incorrect Answer ImageD.Rubeola virus
A 33-year-old woman, gravida 3, para 2, at 18 weeks' gestation comes to the physician because
of a 9-mm ulcerated lesion on her left labium majora, which has been present for the past 7 days.
She is a business executive who recently returned from a trip to Australia. On her first prenatal
visit 6 weeks ago, urine nucleic acid amplification (NAAT) was positive for chlamydia. She is
afebrile. On examination, the lesion feels firm, has a rolled edge, and is painless. She also has
palpable but painless inguinal nodes. Which of the following is the most appropriate
pharmacotherapy?
 Incorrect Answer ImageA.Acyclovir
 Incorrect Answer ImageB.Ceftriaxone
 Correct Answer ImageC.Penicillin
 Incorrect Answer ImageD.Tetracycline
 Incorrect Answer ImageE.Valacyclovir

A 29-year-old diabetic man has been receiving maintenance peritoneal dialysis for the last two
years. He calls his physician one morning to report that his dialysate effluent has become cloudy.
He has taken his temperature at home and it is 38.1 C (100.6 F). He denies any abdominal pain
or other symptoms. He is instructed to come to the emergency department. Which of the
following is the most likely diagnostic finding? 
 Incorrect Answer ImageA.Perforated viscus
 Incorrect Answer ImageB.Pneumococcal peritonitis
 Incorrect Answer ImageC.Peritoneal white blood cell count of approximately 700/UL,
with 85% lymphocytes and 15% polymorphonuclear neutrophils
 Correct Answer ImageD.Peritoneal white blood cell count of approximately 620/UL,
with 92% polymorphonuclear neutrophils and 8% lymphocytes

A 32-year-old woman develops the lesion indicated in the photomicrograph above after
vacationing in Martha's Vineyard, Massachusetts 3 weeks earlier. The lesion initially began as a
small erythematous macule that has expanded slowly over the past several weeks. She reports
accompanying systemic symptoms of fever, chills, and myalgias and denies pain at the lesion
site. She has no allergies. Her temperature is 38.1°C (100.6°F). Which of the following is the
most appropriate next step in management?
 Correct Answer ImageA.Begin therapy with doxycycline
 Incorrect Answer ImageB.Biopsy the affected area for pathologic examination
 Incorrect Answer ImageC.Observe and follow-up in 2 months
 Incorrect Answer ImageD.Perform a lumbar puncture and await results of cerebrospinal
fluid examination
 Incorrect Answer ImageE.Take serial blood samples for culture

A 21-year-old college student arrives at the emergency department accompanied by her


roommate. The patient is vomiting, appears to be somewhat confused, and looks pale. Her
roommate remarks that she is a diabetic, and that she has been studying all week for final exams.
The roommate adds that the patient has had very little sleep all week, and has not been observed
making any insulin injections, something that she normally does twice each day. Which of the
following is the most likely diagnosis? 
 Incorrect Answer ImageA.Acute abdomen
 Incorrect Answer ImageB.Cocaine intoxication
 Correct Answer ImageC.Diabetic ketoacidosis
 Incorrect Answer ImageD.Ectopic pregnancy
 Incorrect Answer ImageE.Migraine headache

A 29-year-old man comes to the emergency department because of syncope and chest pain. The
patient has no past medical history and takes no medicines regularly. He reports that he was well
until 3 days ago, when he thought he had "the flu", with a cough and some rhinorrhea. Three
days ago, he began to experience chest pain, exacerbated by sitting forward. He sought care
today after he passed out while trying to stand up from his desk at work. His blood pressure is
90/50 mm Hg sitting, and he has elevated neck veins and quiet but normal heart sounds. Which
of the following is the most important diagnostic study? 
 Incorrect Answer ImageA.Aortic angiogram
 Incorrect Answer ImageB.Chest CT scan
 Incorrect Answer ImageC.Chest x-ray
 Correct Answer ImageD.Echocardiography
 Incorrect Answer ImageE.Electrocardiogram

A 19-year-old woman, gravida 1, para 0, at 33 weeks' gestation comes to the physician because
of a headache for the last 12 hours. She has tried mild analgesics without relief. Additionally, she
has pain in her mid-upper abdomen that is not relieved by over-the-counter antacids. She states
her weight has increased by 4.5 kg (10 lbs) over the past 7 days. Her blood pressure is 165/100
mm Hg, pulse is 100/min, and respirations are 20/min. Physical examination shows fundal height
is 32 cm; fetal heart rate is 145/min with moderate variability. She has 3+ pitting pretibial and
ankle edema. A complete blood count is most likely to show which of the following findings?
 Incorrect Answer ImageA.Hematocrit of 25%
 Correct Answer ImageB.Hemoglobin of 15 g/dL
 Incorrect Answer ImageC.Mean corpuscular volume (MCV) of 110 m3
 Incorrect Answer ImageD.Platelet count of 500,000/mm3
 Incorrect Answer ImageE.Red cell distribution width (RDW) of 17

A 33-year-old woman, gravida 2 para 0 aborta 1 at 8 weeks’ gestation comes to the emergency
department due to nausea, vomiting, and vaginal spotting. The patient has been able to only keep
down water for the past 12 hours. Her first pregnancy was a right-sided tubal ectopic pregnancy
treated with methotrexate. She takes a daily prenatal vitamin. She has no medical problems and
has had no surgeries. Vital signs are within normal limits. A bimanual pelvic examination shows
a 12-week size uterus. Speculum examination shows a closed cervix and scant blood in the
posterior cul-de-sac. An hCG level is 124,690 mIU/mL. A transvaginal pelvic ultrasound shows
a heterogeneous cystic mass in the uterus, no yolk sac, fetus, or fetal heartbeat, and no adnexal
masses. Which of the following is the most appropriate next step in management?
 Correct Answer ImageA.Dilation and curettage
 Incorrect Answer ImageB.Laparoscopic salpingectomy
 Incorrect Answer ImageC.Methotrexate therapy
 Incorrect Answer ImageD.Misoprostol therapy
 Incorrect Answer ImageE.Serial pelvic ultrasounds

The following vignette applies to the next three items. 


A 38-year-old white man is admitted to the hospital with severe abdominal pain, nausea and
vomiting for 2 days. The pain is dull and it is localized in epigastric area and radiates to the back.
The patient had several episodes of emesis without blood. The patient also admits to an alcoholic
binge 2 days ago. He has not experienced similar episodes in the past and does not have
significant medical problems. He does not take any medications. He is a construction worker. He
smokes 1 pack of cigarettes a day and drinks on average 6-8 beers a day. His vitals are
temperature 38.0°C (98.6°F), supine blood pressure 115/70 mm Hg pulse 100/min, standing
blood pressure 85/60 mm Hg and pulse 124/min, respirations 22/min, oxygen saturation 93% on
room air, weight 130 kg, and height 178 cm. Physical examination reveals an obese man in mild
to moderate distress secondary to pain. Breath sounds are decreased bilaterally. Heart rate is
tachycardic, normal S1/S2 with no murmurs or gallops. Abdomen is soft, slightly distended, and
diffusely tender on palpation, but no guarding or rebound tenderness. Bowel sounds are absent.
No hepatosplenomegaly noted. Rectal examination showed guaiac-negative stool. Laboratory
tests show:
Leukocyte count 16,000/mm3
Hemoglobin 15.6 gm/dL
Hematocrit 51%
Serum glucose 180 mg/dL
Calcium 7.0 mg/dL
Amylase 580 U/L
Lipase 1,400 U/L
The patient is diagnosed with acute pancreatitis. Which of the following is the most appropriate
next step in management? 
 Incorrect Answer ImageA.Abdominal ultrasound
 Incorrect Answer ImageB.CT scan of the abdomen
 Incorrect Answer ImageC.IV calcium
 Correct Answer ImageD.IV fluids
 Incorrect Answer ImageE.IV insulin
 Incorrect Answer ImageF.KUB
 Incorrect Answer ImageG.Oxygen administration
 Incorrect Answer ImageH.Surgery consult

A 38-year old white man is admitted to the hospital with severe abdominal pain, nausea and
vomiting for 2 days. The pain is dull and it is localized in epigastric area and radiates to the back.
The patient had several episodes of emesis without blood. The patient also admits to an alcoholic
binge 2 days ago. He has not experienced similar episodes in the past and does not have
significant medical problems. He does not take any medications. He is a construction worker. He
smokes 1 pack of cigarettes a day and drinks on average 6-8 beers a day. His vitals are
temperature 38.0 C (98.6 F), supine blood pressure 115/70 mm Hg pulse 100/min, standing
blood pressure 85/60 mm Hg and pulse 124/min, respirations 22/min, oxygen saturation 93% on
room air, weight 130 kg, and height 178 cm. Physical examination reveals an obese man in mild
to moderate distress secondary to pain. Breath sounds are decreased bilaterally. Heart rate is
tachycardic, normal S1/S2 with no murmurs or gallops. Abdomen is soft, slightly distended, and
diffusely tender on palpation, but no guarding or rebound tenderness. Bowel sounds are absent.
No hepatosplenomegaly noted. Rectal examination showed guaiac-negative stool. Laboratory
tests shows; 
The patient is diagnosed with acute pancreatitis. Which of the following is the best way to
monitor course of pancreatitis? 
 Incorrect Answer ImageA.Abdominal ultrasound
 Incorrect Answer ImageB.Blood glucose measurements
 Correct Answer ImageC.CT scan of the abdomen
 Incorrect Answer ImageD.Monitoring of white blood cell count
 Incorrect Answer ImageE.Serial amylase measurements
 Incorrect Answer ImageF.Serial lipase measurements

A 38-year old white man is admitted to the hospital with severe abdominal pain, nausea and
vomiting for 2 days. The pain is dull and it is localized in epigastric area and radiates to the back.
The patient had several episodes of emesis without blood. The patient also admits to an alcoholic
binge 2 days ago. He has not experienced similar episodes in the past and does not have
significant medical problems. He does not take any medications. He is a construction worker. He
smokes 1 pack of cigarettes a day and drinks on average 6-8 beers a day. His vitals are
temperature 38.0 C (98.6 F), supine blood pressure 115/70 mm Hg pulse 100/min, standing
blood pressure 85/60 mm Hg and pulse 124/min, respirations 22/min, oxygen saturation 93% on
room air, weight 130 kg, and height 178 cm. Physical examination reveals an obese man in mild
to moderate distress secondary to pain. Breath sounds are decreased bilaterally. Heart rate is
tachycardic, normal S1/S2 with no murmurs or gallops. Abdomen is soft, slightly distended, and
diffusely tender on palpation, but no guarding or rebound tenderness. Bowel sounds are absent.
No hepatosplenomegaly noted. Rectal examination showed guaiac-negative stool. Laboratory
tests shows; 
The patient is diagnosed with acute pancreatitis. Despite aggressive management the patient
develops necrotizing pancreatitis. Which of the following is the most appropriate
pharmacotherapy? 
 Incorrect Answer ImageA.Doxycycline
 Correct Answer ImageB.Imipenem
 Incorrect Answer ImageC.Oxacillin
 Incorrect Answer ImageD.Penicillin
 Incorrect Answer ImageE.Vancomycin

The following vignette applies to the next two items. 


A 27-year-old woman is brought to the emergency department from a nearby subway station
because of chest pain and shortness of breath. She explains that she "always feels weird getting
on the train or elevators." She normally tries to avoid these situations because of an incident 7
months ago when she was trapped on an elevator. She has had episodes of sudden chest pressure,
heart palpitations, and shortness of breath that last only a few minutes, but she typically fears for
her life. There have also been times when she "just felt like [she] was going crazy." She is a
nonsmoker and has no significant past medical history. Physical examination, cardiac workup,
urine toxicology, and metabolic workup are all negative. Which of the following is the most
likely diagnosis? 
 Incorrect Answer ImageA.Adjustment disorder with anxiety
 Incorrect Answer ImageB.Dysthymic disorder
 Incorrect Answer ImageC.Major depressive disorder
 Correct Answer ImageD.Panic disorder
 Incorrect Answer ImageE.Social phobia

A 27-year-old woman is brought to the emergency department from a nearby subway station
because of chest pain and shortness of breath. She explains that she "always feels weird getting
on the train or elevators." She normally tries to avoid these situations because of an incident 7
months ago when she was trapped on an elevator. She has had episodes of sudden chest pressure,
heart palpitations, and shortness of breath that last only a few minutes, but she typically fears for
her life. There have also been times when she "just felt like [she] was going crazy." She is a
nonsmoker and has no significant past medical history. Physical examination, cardiac workup,
urine toxicology, and metabolic workup are all negative. Which of the following is an
accompanying feature of this patient's illness? 
 Correct Answer ImageA.Agoraphobia
 Incorrect Answer ImageB.Delusions, somatic type
 Incorrect Answer ImageC.Flight of ideas
 Incorrect Answer ImageD.Obsessions
 Incorrect Answer ImageE.Tactile hallucinations

A 68-year-old man has a history of non-small cell lung carcinoma that was stage IIa at the time
of diagnosis. He underwent surgery and chemotherapy 3 years ago. He now complains of
dyspnea, orthopnea, and a nonproductive cough. He denies fever, chills, or chest pain. Vital signs
show a temperature of 37.0°C (98.6°F), blood pressure 90/48 mm Hg, pulse 100/min, and
respirations 20/min. There is an 18 mm Hg decline in systolic blood pressure during inspiration.
He has elevated neck veins, clear lung sounds, and distant heart sounds. Chest x-ray shows clear
lung fields and cardiomegaly. Electrocardiogram shows electrical alternans but no ischemic
changes. Which of the following is the most appropriate next step in management? 
 Incorrect Answer ImageA.Administer furosemide
 Incorrect Answer ImageB.Administer trimethoprim-sulfamethoxazole
 Incorrect Answer ImageC.Order a CT scan of the chest
 Correct Answer ImageD.Order an echocardiogram
 Incorrect Answer ImageE.Prepare him for bronchoscopy
A 23-year-old medical student comes to the school clinic because of daily "irresistible attacks of
sleep" that have occurred over the past 5 months. He reports that he has always been in good
health prior to this. He also describes vivid dreams that occur just as he is falling asleep. The
student reports a recent event in which he collapsed and was unable to move (although he was
awake) that occurred just before a stressful biochemistry examination. Physical examination and
laboratory studies are unremarkable. Which of the following is the most appropriate
pharmacotherapy?
 Incorrect Answer ImageA.Clozapine
 Incorrect Answer ImageB.Haloperidol
 Incorrect Answer ImageC.Lithium carbonate
 Correct Answer ImageD.Methylphenidate hydrochloride
 Incorrect Answer ImageE.Valproic acid

A previously healthy 20-year-old woman comes to the emergency department because of sudden
onset headache and joint pain. On further questioning, she states her throat is sore and she has
had vomiting and diarrhea for two days. She is in the third day of her menstrual period. On
examination, her temperature is 39.2°C (102.6°F), blood pressure 70/45 mm Hg, pulse is
120/min, and respirations are 25/min. She appears ill, and her skin has a diffuse rash that looks
like she has severe sunburn. Which of the following is the most likely etiology of her condition?
 Incorrect Answer ImageA.Acute hemolytic episode
 Incorrect Answer ImageB.Bacterial endotoxin
 Correct Answer ImageC.Bacterial exotoxin
 Incorrect Answer ImageD.IgE-autoantibody response
 Incorrect Answer ImageE.IgG-mediated allergic reaction

A 41-year-old piano teacher is brought to the emergency department by his wife because of
recent changes in his behavior. For the past 2-weeks he has had a diminished need for sleep, and
is restless during the daytime. He says he has more energy and confidence than ever and does not
understand his wife’s concerns. He has a low income but last week he bought $10,000 worth of
new furniture and electrical appliances on three credit cards. He states that he does not care about
paying the cards off. Medical history includes long-standing diabetes mellitus with recent
elevation of levels of BUN and creatinine. Which of the following is the most appropriate long-
term pharmacotherapy for this patient?
 Incorrect Answer ImageA.Clonidine
 Incorrect Answer ImageB.Clozapine
 Incorrect Answer ImageC.Lithium
 Correct Answer ImageD.Valproic acid
 Incorrect Answer ImageE.Verapamil

A 30-year-old woman, gravida 4, para 2, comes to the physician for a preconception visit. She is
concerned because one of her best friends recently had a child with spina bifida. She wants to
know if there is anything that she can do to prevent this. Which of the following is the most
appropriate therapy to reduce this patient’s risk of having a child with this birth defect?
 Incorrect Answer ImageA.Amniocentesis
 Incorrect Answer ImageB.Calcium
 Correct Answer ImageC.Folic acid
 Incorrect Answer ImageD.Iron
 Incorrect Answer ImageE.Prenatal vitamins 3 times daily

A new tumor marker used to screen patients for ovarian cancer is reported in the literature.
Serum levels of the marker are elevated in more than 90% of patients with ovarian cancer. In
addition, the marker is elevated in 10% of patients with gastrointestinal cancers and 4% of
patients with no identifiable neoplasms. Which of the following descriptions of the test's efficacy
is affected by the prevalence of ovarian cancer in the population that will be screened? 
 Incorrect Answer ImageA.Sensitivity
 Incorrect Answer ImageB.Specificity
 Correct Answer ImageC.Positive predictive value
 Incorrect Answer ImageD.Validity
 Incorrect Answer ImageE.Precision

A 60-year-old woman complains of vulvar itching for the past 18 months. Several practitioners
have prescribed various creams and salves without success during this time. Ten years ago she
had a cervical conization for carcinoma in situ; subsequent Pap smears have been normal. Her
last normal menstrual period was eight years ago. She has been taking combination
estrogen/progestin as hormone replacement therapy. She has no gastrointestinal or other
genitourinary complaints. She seldom has intercourse because of her perineal discomfort. No
emollients or marital aids are used. She has no known allergies. Past medical history and review
of systems are noncontributory. Her abdomen is soft; the liver, kidney, and spleen are not
palpable. No masses are palpable, but in the left inguinal region, there are two irregularly
enlarged, nontender lymph nodes about 2 cm in diameter. The vulva has a 2-cm punched out
lesion at the base of the left labia majora that bleeds easily. The vagina appears somewhat
atrophic. The cervix is clean, pink, and without abnormality. Pap smears are taken. Saline
preparation of vaginal secretions is negative for common vaginal pathogens. The remainder of
the pelvic examination is within normal limits. Which of the following is the most appropriate
management? 
 Incorrect Answer ImageA.Arrange for needle aspiration of the left inguinal adenopathy
 Correct Answer ImageB.Biopsy the lesion
 Incorrect Answer ImageC.Culture the lesion for aerobic and anaerobic bacteria, herpes
virus, and spirochetes
 Incorrect Answer ImageD.Send for medical records of her conization and previous
medications used to treat her condition, order symptomatic treatment until they return
 Incorrect Answer ImageE.Take a Pap smear of the vulvar lesion

A 43-year-old man with chronic hepatitis C, chronic pancreatitis, and hypertension is admitted to
the hospital for evaluation of abdominal pain. He has had multiple similar symptoms over the
past year. Review of symptoms is positive for moderate abdominal pain exacerbated by
coughing. Current medications include valsartan, hydrochlorothiazide, and pancreatic enzyme
replacement. He has no known drug allergies. Family history is unremarkable. Social history is
significant for promiscuous sexual relationships and a 30 pack-year smoking history. On
admission the patient is in mild distress. His temperature is 36.6°C (97.9°F), pulse is 70/min, and
respirations are 19/min. On physical examination the patient's skin and conjunctivae have a
yellow hue. Multiple punctiform and linear scars are present on the chest, abdomen, and
proximal extremities. Which of the following is the most appropriate next step in the
management of this patient's skin disease?
 Incorrect Answer ImageA.Blood culture
 Incorrect Answer ImageB.Potassium hydroxide examination of skin curettage
 Incorrect Answer ImageC.Skin biopsy
 Correct Answer ImageD.Symptomatic antipruritic treatment
 Incorrect Answer ImageE.Tzanck test

 60-year-old woman with a history of hypertension is being evaluated for headaches, a burning
sensation in her hands and feet, and recurrent episodes of blurred vision. Despite knowing better,
she smokes nearly two packs of cigarettes a day and has done so for the last 40 years. Vital signs
are: temperature 37.0°C (98.6°F), blood pressure 162/95 mm Hg, pulse 69/min, respirations
20/min with an oxygen saturation of 94% on room air. Physical examination reveals an obese
middle-aged woman with marked facial plethora. Funduscopic examination is remarkable for
venous engorgement but no evidence of papilledema or hemorrhages. Examination of the chest
reveals diffuse crackles in the bases, whereas cardiac examination is unremarkable. There is
obvious splenomegaly, with a palpable spleen tip 5 cm below the costal margin. Laboratory
studies show: 
Hematologic
Hematocrit 60%
Hemoglobin 19.5 g/dL
Leukocytes 19,000/mm3 (normal differential)
Platelets 510,000/mm3
Mean corpuscle volume 88 µm3
An erythropoietin level also is drawn and is in the low-normal range. Based on these results, the
patient is given a diagnosis of polycythemia vera. Before giving this patient this diagnosis, which
of the following is an additional appropriate diagnostic study? 
 Correct Answer ImageA.Carboxyhemoglobin and sleep study
 Incorrect Answer ImageB.Erythropoietin receptor analysis
 Incorrect Answer ImageC.No additional studies needed
 Incorrect Answer ImageD.Renal ultrasound and CT scan of abdomen
 Incorrect Answer ImageE.Serum iron studies

A 41-year-old man with a history of alcoholic cirrhosis comes to the emergency department with
gross hematemesis lasting approximately 1 hour. He is found to be orthostatic and tachycardic
despite receiving aggressive fluid resuscitation in transit to the emergency department. The
patient has a nasogastric tube placed that shows a bright red, bloody aspirate that fails to clear
with lavage. The patient is transfused emergently, given a bolus injection of octreotide, and sent
for emergent endoscopy. Endoscopy confirms the presence of bleeding esophageal varices that
are, after much difficulty, successfully banded. Over the next two days, the patient is observed
closely and found to have no evidence of recurrent bleeding. He is discharged with plans for
close followup including alcohol cessation treatment. Which of the following is an additional
approach that has shown to reduce the risk for recurrent variceal bleeding and prolong survival?
 Correct Answer ImageA.Beta blocker therapy with oral propranolol
 Incorrect Answer ImageB.Histamine H2 blocker with daily famotidine
 Incorrect Answer ImageC.No additional treatment, follow clinically
 Incorrect Answer ImageD.Oral octreotide or somatostatin treatment
 Incorrect Answer ImageE.Transjugular intrahepatic portosystemic shunt (TIPS)

A 32-year-old mother brings her 1-year-old son for a second opinion. He was diagnosed with
cerebral palsy at birth and she was not given any hope as far as treatment and prognosis. She had
a high-risk pregnancy because of a thyroid problem and previous pregnancy loss. The child was
small for gestational age at birth and had intracranial bleeding. He scored low on the Apgar scale
and was in the neonatal intensive care nursery for 2 months. During the first year of life he has
had three episodes of pneumonia, but has managed to receive most of his immunizations. The
family history is negative for neurologic disorders. On physical examination, the patient is in no
acute distress. His vital signs are within normal limits. He is in the 10th percentile for height and
weight. Neurologic examination reveals upper motor neuron deficits with increased muscle tone
in the legs more than in the arms. Deep tendon reflexes are exaggerated. There is a scissoring
gait pattern with flexed and adducted hips, knees flexed in valgus and ankles in equinus,
producing a tiptoe type of walking pattern. He also has moderate strabismus. Appropriate
counseling should include which of the following? 
 Incorrect Answer ImageA.If chorea and athetosis develop, they can be successfully
managed with medication
 Incorrect Answer ImageB.The child's diet will not require major modifications
 Incorrect Answer ImageC.The motor deficit will most likely worsen with age
 Incorrect Answer ImageD.There is no increased risk of seizures
 Correct Answer ImageE.Regular physical and occupational therapy are crucial

A 31-year-old woman comes to the physician because of a foul-smelling vaginal discharge. She
states that she first noticed it about one week ago and that her symptoms have persisted. She has
one sexual partner, her husband, and has never had a sexually transmitted disease or other
medical illness. She had an appendectomy at the age of 17. She takes no medications and has no
known drug allergies. Physical examination shows a homogeneous, white vaginal discharge
coating the vaginal walls. The discharge is malodorous and when treated with normal saline and
viewed under the microscope, vaginal epithelial cells coated with coccobacillary organisms are
present. Which of the following is the most appropriate treatment for the patient's sexual partner?
 Incorrect Answer ImageA.Oral clindamycin
 Incorrect Answer ImageB.Oral metronidazole
 Incorrect Answer ImageC.Topical clindamycin
 Incorrect Answer ImageD.Topical metronidazole
 Correct Answer ImageE.Treatment of sex partners is not recommended
A 66-year-old man comes to his physician for a surveillance colonoscopy that had been
scheduled previously. The physician realizes that the patient looks jaundiced, and rather than
pursuing the colonoscopy, a workup for the jaundice is started. The patient’s past medical history
is significant for alcohol abuse, and at the age of 39 years, the patient had severe peptic ulcer
disease for which he had a vagotomy and antrectomy. The surgical reconstruction anastomosed
the gastric remnant to a Roux-en-Y loop, and the duodenum was closed proximally beyond the
resected pylorus and drained downstream into the jejunum, 45 cm beyond the gastric
anastomosis. Laboratory studies show total bilirubin of 5.4 mg/dL, with 3.8 mg/dL direct
fraction, alkaline phosphatase of 345 U/L, and normal transaminases. The prothrombin time is 22
seconds, with a control of 14 seconds. Ultrasound of the upper abdomen shows dilated
intrahepatic and extrahepatic ducts and a very large, distended, thin-walled gallbladder. CT scan
confirms the presence of dilated ducts and dilated gallbladder but is otherwise nondiagnostic. To
further define the cause of the patient’s jaundice, which of the following is the next best
diagnostic study?
 Incorrect Answer ImageA.Coombs test
 Incorrect Answer ImageB.Endoscopic retrograde cholangiopancreatogram (ERCP)
 Correct Answer ImageC.Magnetic resonance cholangiopancreatogram (MRCP)
 Incorrect Answer ImageD.Percutaneous transhepatic cholangiogram (PTC)
 Incorrect Answer ImageE.Serology

A 34-year-old woman comes to the physician for birth control counseling. Six months ago she
and her husband of 5 years had their second child. In the past they have used condoms, but they
find difficulty with using them consistently. She thinks she may be done with childbearing,
although they are still considering having one more child. Her past medical history is significant
for depression, for which she is treated with fluoxetine. Her past surgical history is significant for
a laparoscopic left salpingectomy 4 years ago for an ectopic pregnancy. She also has had two
cesarean deliveries. She has no known drug allergies. Physical examination, including breast and
pelvic examination, is entirely normal. After a discussion of the risks and benefits of the various
birth control options, she states that she would like more information on the intrauterine device
(IUD). Which of the following is a relative contraindication to the placement of the intrauterine
device in this patient? 
 Incorrect Answer ImageA.Age
 Incorrect Answer ImageB.Desire for possible future fertility
 Incorrect Answer ImageC.History of depression
 Correct Answer ImageD.History of ectopic pregnancy
 Incorrect Answer ImageE.Previous cesarean delivery

A 16-year-old boy is brought to the emergency department by his parents because of an episode
of severe chest pain several hours earlier. The patient also has a flu-like illness that started a
week earlier. He reports fever, chills, abdominal discomfort, and feeling tired during the prior
week. He has had a low-grade fever for 5 to 6 days and a sore throat, but pharyngeal swab
cultures done at the primary care physician's office had been negative and he was taking over-
the-counter cold medication only. Physical examination shows a well developed and well
nourished young man in moderate distress. His temperature is 37.8°C (100.0°F), pulse is
120/min, and respirations are 28/min. A differential white blood cell count shows a normal
number of neutrophils and marginally elevated lymphocytes. An electrocardiogram shows low
voltage QRS complexes throughout the limb leads. Chest radiography shows increased
pulmonary markings and an enlarged heart silhouette. Which of the following is the most
appropriate next step in the management?
 Correct Answer ImageA.Admit to monitored floor for further evaluation and
management
 Incorrect Answer ImageB.Discharge from emergency department after stabilization of
vital signs
 Incorrect Answer ImageC.Overnight observation in the emergency department
 Incorrect Answer ImageD.Reassurance and discharge
 Incorrect Answer ImageE.Recommend symptomatic antipyretic therapy, bed rest, a low
salt diet, and discharge from emergency department

A 27-year-old woman, gravida 2, para 2, comes to the physician to have her staples removed
after an elective repeat cesarean delivery 5 days ago. Her pregnancy course was uncomplicated.
She states that she is doing well except that since the delivery she has noticed some episodes of
sadness and tearfulness. She is eating and sleeping normally and has no strange thoughts or
thoughts of hurting herself or others. She is busy with the duties of taking care of her newborn
but has no negative feelings towards him. She has no previous history of mental illness. Physical
examination shows no abnormalities. Which of the following is the most likely diagnosis?
 Correct Answer ImageA.Postpartum blues
 Incorrect Answer ImageB.Postpartum depression
 Incorrect Answer ImageC.Postpartum mania
 Incorrect Answer ImageD.Postpartum psychosis
 Incorrect Answer ImageE.Poststerilization depression

A vomiting infant is brought to the emergency room. The blood work results reveal a normal
blood count, but a hyponatremic, hypochloremic, metabolic alkalosis. Which of the following
would be consistent with these findings? 
 Incorrect Answer ImageA.Diabetes mellitus
 Correct Answer ImageB.Cystic fibrosis
 Incorrect Answer ImageC.Ethanol poisoning
 Incorrect Answer ImageD.Iron ingestion
 Incorrect Answer ImageE.Isoniazid ingestion

A patient has been coming to psychotherapy for several weeks. The psychiatrist has been dealing
with countertransference feelings, including problems coping with the patient's intense
devaluation of him as a therapist, as well as the patient's arrogant attitude and sense of
entitlement. The psychiatrist feels bored during these sessions which probably results from the
patient's obliviousness toward him. Which of the following is the most likely diagnosis in this
patient?
 Incorrect Answer ImageA.Antisocial personality disorder
 Incorrect Answer ImageB.Bipolar I disorder
 Incorrect Answer ImageC.Histrionic personality disorder
 Correct Answer ImageD.Narcissistic personality disorder
 Incorrect Answer ImageE.Schizophrenia

A 9-week-old female infant is brought to the physician because of a bright red plaque on her
forehead. The plaque is slightly raised, measures 1 cm in greatest diameter, and blanches
partially on pressure. No other similar cutaneous lesions are noted. The mother reports that the
lesion appeared in the second or third week of life and has been gradually increasing in size. The
infant is otherwise healthy and is developing normally. Which of the following is the most
appropriate next step in management?
 Correct Answer ImageA.Further diagnostic procedures or treatments are not necessary
 Incorrect Answer ImageB.MRI and/or CT studies
 Incorrect Answer ImageC.Referral to a dermatologist for biopsy
 Incorrect Answer ImageD.Surgical excision
 Incorrect Answer ImageE.Treatment with systemic propranolol

A 4-year-old boy is brought to the emergency department because of left leg pain, a high fever,
and refusal to walk since yesterday. His temperature is 39.8°C (103.6°F), blood pressure is
122/68 mm Hg, pulse is 102/min, and respirations are 20/min. His left femur is tender to
palpation 3 cm above the left knee, and there is marked soft tissue swelling. A plain radiograph
of his left leg is normal. A bone scan shows increased uptake around the metaphysis of the left
femur. Which of the following is the most likely pathogen?
 Incorrect Answer ImageA.Escherichia coli
 Incorrect Answer ImageB.Haemophilus influenzae
 Incorrect Answer ImageC.Salmonella
 Correct Answer ImageD.Staphylococcus aureus
 Incorrect Answer ImageE.Streptococcus pneumoniae

A middle-aged homeless man is brought to the emergency department because of very severe
pain in his forearm. He had passed out after drinking a bottle of cheap wine, and then slept on a
park bench for an indeterminate time, probably more than 12 hours. Shortly after he woke up and
began to walk, the pain began. There are no signs of trauma, but the muscles in his forearm are
very firm and tender to palpation, and passive motion of his fingers and wrist elicits excruciating
pain. Pulses at the wrist are normal. Which of the following is the most appropriate next step in
management? 
 Incorrect Answer ImageA.Analgesics and observation
 Incorrect Answer ImageB.Immobilization in a sling
 Incorrect Answer ImageC.Immobilization in a plaster cast
 Incorrect Answer ImageD.Emergency embolectomy
 Correct Answer ImageE.Emergency fasciotomy

A 4-month-old girl is brought to the physician because of frequent crying, sleep disturbances,
and decreased appetite. She has been irritable since birth, especially after feeding when she
usually spits up or vomits. She also seems to get hiccups frequently. She has not been gaining
weight as expected, and the parents are concerned that something may be wrong. Last night, they
thought that she had stopped breathing for approximately 20 seconds. This also happened after
she had fallen asleep breastfeeding. The parents did not note any cyanosis during either of these
events. On physical examination, the infant does not seem to be in any acute distress. Her vital
signs are within normal limits and her weight and length are in the tenth percentile. Otherwise,
the physical examination is normal. Which of the following is the most appropriate next step in
management?
 Incorrect Answer ImageA.Inserting a nasogastric feeding tube
 Incorrect Answer ImageB.Nissen fundoplication
 Incorrect Answer ImageC.Prokinetics
 Incorrect Answer ImageD.Proton pump inhibitors
 Correct Answer ImageE.Thickening of feeds and elevating the head of the bed

A 54-year-old woman comes to the physician because of decreased interest in sexual activity
since menopause 3 years ago. She further mentions that her libido has decreased rapidly and is
now “less than zero”. She has been married for 26 years and states that she and her husband have
a happy marriage. Her past medical history is significant for hypothyroidism. She has never had
surgery. Current medications include levothyroxine. She is allergic to penicillin. Her physical
examination, including breast and pelvic examination, is normal. Which of the following should
be included in the counseling of this patient regarding postmenopausal decreased libido?
 Incorrect Answer ImageA.Androgens have never been shown to increase libido
 Incorrect Answer ImageB.Her condition is normal and requires no intervention
 Correct Answer ImageC.Sexual dysfunction is a complex disorder with multiple causes
 Incorrect Answer ImageD.Testosterone causes desirable lipoprotein alterations
 Incorrect Answer ImageE.Testosterone has very few side effects

A 7-year-old boy develops end-stage renal disease and is placed on chronic hemodialysis. The
family is advised that he also should be placed on the waiting list for a renal transplant, but they
are reluctant because they fear the unknown problems that might be faced after the procedure.
Which of the following is the most compelling argument why this patient should get a transplant
if at all possible? 
 Incorrect Answer ImageA.Children can do well with a transplant of only half a kidney,
rather than an entire organ
 Incorrect Answer ImageB.Children have no preformed antibodies that might threaten the
viability of a transplant
 Correct Answer ImageC.Growth and development are arrested by end-stage renal
disease, and resume when children get a new kidney
 Incorrect Answer ImageD.Mental retardation is a serious complication of chronic
hemodialysis
 Incorrect Answer ImageE.Rejection episodes need no immunosuppressive treatment in
children

A 22-year-old woman comes to the physician for a follow-up visit. Four weeks ago, she was
diagnosed with gonorrhea. At that time she was found to be negative for chlamydia. She received
cefixime 400 mg orally in a single dose. She currently feels well. She has two sexual partners
and uses condoms inconsistently. She does not smoke, drink alcohol, or use illegal drugs.
Physical examination shows normal vaginal discharge and no evidence of cervicitis. Assuming
that the patient has not been previously vaccinated, she should be vaccinated against which of the
following sexually transmitted diseases? 
 Incorrect Answer ImageA.Chlamydia trachomatis
 Incorrect Answer ImageB.Gonorrhea
 Incorrect Answer ImageC.Hepatitis A
 Correct Answer ImageD.Hepatitis B
 Incorrect Answer ImageE.Syphilis

A 9-month-old boy is brought to the emergency department because of intractable vomiting that
started several hours earlier. The mother reports that he has been feeding poorly in the last 2
weeks and has been very irritable and crying a lot. Also, his activity has been somewhat reduced
and he appears to be tired and cranky all the time. He had several episodes of vomiting in the
previous week, mostly in the morning, but this day, the vomiting just would not stop. The mother
denies respiratory stridor, apnea, difficulty swallowing, or changes in bowel and bladder
function. The mother denies having changed any feeding patterns recently. No other family
member has had any similar problems. On physical examination, the child is in mild distress
with a pulse of 140/min and respirations of 22/min. He is very fussy and cries loudly during most
of the examination. His head circumference is in the 98th percentile for age, and dilated scalp
veins are noted through a shiny scalp. On palpation, the cranial sutures are wide and the anterior
fontanelle is bulging in the upright position when the infant is not crying. Both ocular globes are
deviated downward with the upper lids retracted, and sclerae are visible above the iris, but there
is no nystagmus. There is increased tone of the lower extremities. Which of the following is the
most likely diagnosis?
 Incorrect Answer ImageA.Arnold-Chiari malformation type I
 Incorrect Answer ImageB.Arnold-Chiari malformation type II
 Incorrect Answer ImageC.Arrested hydrocephalus
 Incorrect Answer ImageD.Normal pressure hydrocephalus
 Correct Answer ImageE.Subacute hydrocephalus

A 19-year-old woman is brought to the emergency department after being involved in a bicycle
accident where she was hit by a car as it turned in front of her while crossing a crosswalk. She
suffered multiple lacerations which bled profusely and soon lost consciousness. Emergency
medical technicians were able to resuscitate her with fluids and controlled the bleeding
somewhat on arrival to the emergency department. She takes no medications. On physical
examination, she is tachycardic with a blood pressure of 100/83 mm Hg. There are no obvious
fractures, but she has numerous lacerations and abrasions on her arms, chest, and head that have
stopped bleeding. CT of the abdomen reveals no intraabdominal injuries, but there is a pelvic
fracture with a moderate pelvic hematoma. Her initial hematocrit is 20%, and over the course of
several hours, she receives a total of 6 units of packed red blood cells (PRBC’s). It is noted that
her lacerations have started to rebleed. Repeat CT shows the pelvic hematoma to be stable in
size. Routine coagulation studies show an International Normalized Ratio (INR) of 2.1 and an
aPTT of 49 sec. Which of the following is the most likely cause of the patient's excessive
bleeding? 
 Incorrect Answer ImageA.Congenital defect in platelet function
 Incorrect Answer ImageB.Congenital deficiency of a coagulation factor
 Incorrect Answer ImageC.Decreased endothelial expression of von Willebrand factor
 Incorrect Answer ImageD.Deficiency of vitamin K
 Correct Answer ImageE.Dilutional coagulopathy
 Incorrect Answer ImageF.Transfusion reaction

A 61-year-old healthy man has an episode of painless hematuria. Workup includes a CT scan
that shows the presence of a 5-cm tumor in his right kidney, indicative of renal cell carcinoma.
There is concern for tumor invasion into the renal vein and inferior vena cava (IVC). Prior to
surgical resection, which of the following should be performed? 
 Incorrect Answer ImageA.Cardiac stress test to establish the risk of surgery
 Correct Answer ImageB.CT scan of the chest to exclude metastatic disease
 Incorrect Answer ImageC.Evaluation of the function of the contralateral kidney
 Incorrect Answer ImageD.Percutaneous biopsy of the mass to establish a diagnosis
 Incorrect Answer ImageE.PET scan

An 18-year-old patient is admitted to the hospital with altered mental status. A quick intake
examination reveals a thin, obtunded young woman with a fruity scent to her breath, dry mucous
membranes, and orthostatic hypotension. Laboratory studies show ketonuria, glucosuria, and
severe hyperglycemia. The patient is rehydrated with normal saline and placed on an insulin drip.
Her electrolytes are monitored closely and over the next 24 hours the patient markedly improves.
When asked the next day, she recalls a 2-week history of increasing thirst, hunger, and urination;
these are all new symptoms. She does report having an annoying urinary tract infection at about
that time, to which she attributed her subsequent problems. As part of her workup, she is tested
for antibodies to glutamic acid decarboxylase (GAD), which are positive. Which of the following
is an important component of the pathogenesis of this patient's disease?
 Correct Answer ImageA.Association with HLA system and increased risk for
autoimmune disorders
 Incorrect Answer ImageB.Genetic mutations in hepatocyte nuclear factors and insulin-
promoter factor
 Incorrect Answer ImageC.Humoral autoimmune response to pancreatic and hepatic
insulin receptors
 Incorrect Answer ImageD.Peripheral insulin resistance and a defect in beta cell secretion
of insulin
 Incorrect Answer ImageE.Strong heritable, genetic basis, with almost 100% concordance
in monozygotic twins

A 34-year-old college-educated man tells his physician that he feels "kind of down." He explains
that when he was younger he was success-oriented, but failed in a number of pursuits. Now he
notes that he is unassertive and that when specific opportunities for professional advancement
occur, he does not take advantage of them. He adds that his belief is that, although qualified, he
"would not get a promotion anyway" and that he shouldn’t have bothered to complete a college
degree in the first place. In the past, he has had complaints of fatigue and poor sleep that had no
discernible organic cause. This patient displays which of the following characteristics?
 Incorrect Answer ImageA.Gender dysphoria
 Incorrect Answer ImageB.Ideas of reference
 Correct Answer ImageC.Learned helplessness
 Incorrect Answer ImageD.Magical thinking
 Incorrect Answer ImageE.Suicidal ideation

A 4-year-old African American girl is brought to the emergency department with a 6-hour
history of hip pain. The pain increased gradually, but now the patient is unable to walk and cries
loudly when her mother attempts to show the physician the leg. She is fully immunized,
including pneumococcal vaccine polyvalent and has no medical history other than "some sort of
blood problem." Physical examination shows a 4-year-old in severe distress. Her temperature is
38.7°C (101.6°F), respirations are 28/min, blood pressure is 110/72 mm Hg, and pulse is 78/min.
Hip examination shows a swollen, warm, erythematous hip painful to both passive and active
movement. Distal pulses and neurologic examination are within normal limits. Laboratory
studies show: 
Hemoglobin 9.8 g/dL
MCV 77 µm3
Reticulocyte count 7%
Leukocyte count 17,000/mm3
Segmented neutrophils 77%
Bands 12%
Monocytes 8%
Eosinophils 1%
Erythrocyte sedimentation rate 54 mm/hr
A hip x-ray shows changes consistent with osteomyelitis. Which of the following is the most
likely causative organism? 
 Incorrect Answer ImageA.Escherichia coli
 Incorrect Answer ImageB.Proteus mirabilis
 Incorrect Answer ImageC.Pseudomonas aeruginosa
 Correct Answer ImageD.Salmonella typhi
 Incorrect Answer ImageE.Streptococcus  pyogenes
A 24-year-old woman with no significant past medical history comes to the clinic in the spring
for a pre-employment physical examination. She has been hired as a receptionist at a law firm.
History and physical examination are unremarkable. Listed below is her immunization record:
Tetanus primary series During infancy
Tetanus Booster 15 years
Measles 15 months
Mumps 5 years
Rubella 5 years
Influenza 1 year ago
Hepatitis B Never
Pneumovax Never
What vaccination does the patient need at this time? 
 Incorrect Answer ImageA.Influenza
 Incorrect Answer ImageB.Measles
 Correct Answer ImageC.No vaccination required
 Incorrect Answer ImageD.Pneumococcal polysaccharide
 Incorrect Answer ImageE.Tetanus

A 71-year-old man is admitted to the hospital because of constipation for the last 3 days. He has
not been able to pass stools regularly in the last 6-8 months, and therefore occasionally uses
laxatives. At present he complains of nausea and mild, crampy pain in the left iliac fossa. He has
tried to consume more fiber in his diet, but says that he does not enjoy his food as much, adding
that it also causes bloating. He also complains of having seen bright red blood per rectum this
morning while he was straining to empty his bowels. His history is significant for coronary artery
disease, and he had a coronary artery bypass graft 3 or 4 years ago. He now has atrial fibrillation.
His current medications include low-dose aspirin, cephalexin for a sore throat that started two
days ago, warfarin, and enalapril. His immunizations are up-to-date, and his previous annual
checkup 6 months ago revealed occult blood per rectum. His prothrombin time has regularly
been maintained at a therapeutic value for warfarin. A flexible sigmoidoscopy is performed, but
cannot locate the site of bleeding. His blood pressure has dropped from 140/90 mm Hg on
admission to 100/70 mm Hg. Emergent surgery is planned. Which of the following is indicated
at this time? 
 Incorrect Answer ImageA.Desmopressin
 Incorrect Answer ImageB.Intravenous heparin
 Incorrect Answer ImageC.Platelet transfusion
 Incorrect Answer ImageD.Subcutaneous vitamin K
 Correct Answer ImageE.Transfusion of fresh frozen plasma

An 8-month-old boy is brought to the emergency department by his parents who state that he fell
off the couch yesterday. The boy is unresponsive and is without a pulse. He has bilateral retinal
hemorrhages and contusions on both arms. Resuscitative efforts are unsuccessful. Post mortem
examination shows small subdural hematomas and diffuse brain swelling. Which of the
following is the most likely cause of death?
 Incorrect Answer ImageA.Accidental head trauma
 Incorrect Answer ImageB.Intentional head trauma
 Correct Answer ImageC.Shaken baby syndrome
 Incorrect Answer ImageD.Sudden infant death syndrome
 Incorrect Answer ImageE.Unidentified birth injury

A 64-year-old man comes to the emergency department after 24 hours of repeated vomiting and
watery diarrhea. The symptoms began approximately 6 hours after eating at a local restaurant.
His wife had similar symptoms that resolved spontaneously. He has a history of stable exertional
angina and mild symptoms of congestive heart failure with an ejection fraction of 37%. There are
mild symptoms of dyspnea on exertion and orthopnea. His medications are digoxin, captopril,
furosemide, aspirin, and isosorbide dinitrate. His blood pressure is 102/64 mm Hg, and pulse is
94/min. His mucous membranes are dry, his neck veins are flat, his lungs are clear, and he has a
regular S1 and S2. Abdominal examination reveals mild mid-abdominal tenderness, and his brown
stool is guaiac-negative. U waves are seen in all leads of an electrocardiogram. Which of the
following medications is contributing to his electrocardiographic findings? 
 Incorrect Answer ImageA.Aspirin
 Incorrect Answer ImageB.Captopril
 Correct Answer ImageC.Furosemide
 Incorrect Answer ImageD.Isosorbide dinitrate
 Incorrect Answer ImageE.Propranolol

A 27-year-old plumber comes to the physician because of a persistent cough. When the doctor
tells the patient that he needs to have a chest x-ray, he refuses and says, “You are just trying to
get a ‘kickback’ from the radiologist.” During the interview, the doctor finds the patient
emotionally cold and determines that he has a long history of distrust and suspiciousness of
others. The patient finds threatening meanings in what people say, even if the remark is
complimentary. He feels that people deceive him and is reluctant to confide in others lest they
use that information to hurt him. He lacks close friends and often chooses solitary activities but
shows no evidence of a thought disorder. Which of the following is the most likely diagnosis?
 Incorrect Answer ImageA.Delusional disorder, persecutory type
 Correct Answer ImageB.Paranoid personality disorder
 Incorrect Answer ImageC.Schizoid personality disorder
 Incorrect Answer ImageD.Schizophrenia
 Incorrect Answer ImageE.Schizotypal personality disorder

The following vignette applies to the next two items. 


A 33-year-old man is sure that his wife is being unfaithful to him. He has hired 3 separate private
investigators to follow her for the 3 years that he has been married. Each of the investigators has
determined that there is no evidence that his wife has been having affairs. The patient refuses to
believe this and wants to hire a more competent private investigator to confirm his suspicions.
He denies any symptoms of depression, any perceptual disturbances, or belief that others are
conspiring against him. He has worked as a salesman for the past 10 years and makes a
comfortable living. His sister recommended that he see a physician before these "ideas" ruin his
marriage. Which of the following is the most likely diagnosis? 
 Correct Answer ImageA.Delusional disorder, jealous type
 Incorrect Answer ImageB.Persistent depressive disorder
 Incorrect Answer ImageC.Major depressive disorder
 Incorrect Answer ImageD.Schizophrenia
 Incorrect Answer ImageE.Delusional symptoms in partner of individual with delusional
disorder

A 33-year-old man is sure that his wife is being unfaithful to him. He has hired 3 separate private
investigators to follow her for the 3 years that he has been married. Each of the investigators has
determined that there is no evidence that his wife has been having affairs. The patient refuses to
believe this and wants to hire a more competent private investigator to confirm his suspicions.
He denies any symptoms of depression, any perceptual disturbances, or belief that others are
conspiring against him. He has worked as a salesman for the past 10 years and makes a
comfortable living. His sister recommended that he see a physician before these "ideas" ruin his
marriage. Which of the following statements best describes the functional prognosis for this
patient? 
 Incorrect Answer ImageA.He is at increased risk for developing an early-onset dementia
 Incorrect Answer ImageB.He is at increased risk for developing neuromuscular
symptoms
 Incorrect Answer ImageC.He will most likely experience a progressive deterioration in
function
 Incorrect Answer ImageD.He will most likely experience a sudden deterioration in
function
 Correct Answer ImageE.He will most likely experience little change in occupational
functioning

A 40-year-old actress is brought to the emergency department by her husband because of a


severe headache. The headache began during a shower and reached its maximal intensity in less
than 30 seconds. The pain is holocranial but most severe behind the eyes and at the top of the
neck. Since arriving at the hospital one hour ago, the patient has become drowsy and asked for
the overhead lights in her room to be dimmed. She has a history of hypertension that is treated
with diltiazem. Her temperature is 37.2°C (99.0°F), blood pressure is 152/96 mm Hg, pulse is
94/min, and respirations are 16/min. She appears uncomfortable and drowsy, but is easily
arousable. She is alert and oriented to place, time, and person. Funduscopic examination is
normal, but she complains that the bright light is very uncomfortable. She is unable to touch her
chin to her chest when asked. Heart and lung examination are normal. Her pulses are full and
symmetric. Neurologic examination is nonfocal. Which of the following is the most likely
clinical finding?
 Incorrect Answer ImageA.Carotid duplex scan revealing 95% occlusion of the right
internal carotid artery
 Incorrect Answer ImageB.CSF revealing 400 white blood cells with 98% neutrophils and
2% lymphocytes
 Incorrect Answer ImageC.CSF revealing low glucose
 Correct Answer ImageD.CT scan of the head revealing hyperintensity in the sylvian
fissure and cortical sulci
 Incorrect Answer ImageE.Temporal artery biopsy showing granulomatous changes in the
artery wall

A 55-year-old woman comes to the physician because of severe swelling and burning pain in her
right leg and foot for the past two weeks. She also reports excessive sweating in her right lower
limb. She underwent an open reduction and internal fixation of a right ankle fracture six weeks
ago. Her temperature is 37.1°C (98.6°F), pulse is 110/min, respirations are 14/min, and blood
pressure is 140/80 mm Hg. Physical examination shows a warm, erythematous, and tender right
lower extremity with nonpitting edema and hyperhidrosis. Physical examination of the left lower
limb is normal. Laboratory studies show a leukocyte count of 7,000/mm3 and an erythrocyte
sedimentation rate (ESR) of 18 mm/hr. X-ray of the right lower limb shows a healed spiral
fracture of the right lateral malleolus with global diffuse radiolucency throughout the limb.
Which of the following is the most appropriate initial step in management?
 Incorrect Answer ImageA.Diuretic therapy
 Incorrect Answer ImageB.Nonsteroidal anti-inflammatory (NSAID) therapy
 Incorrect Answer ImageC.Periarterial nerve blocks
 Correct Answer ImageD.Physical therapy
 Incorrect Answer ImageE.Steroid therapy

A 6-year-old boy is brought to the physician for a school entrance physical examination. He is
well developed and well nourished. His height is in the 75th percentile and weight is at the 40th
percentile. His temperature is 37.0°C (98.6°F), pulse is 100/min, and respirations are 16/min.
Blood pressure taken in the right arm is 115/75 mm Hg. Examination of the heart shows a
systolic ejection click heard best in the suprasternal notch, and a systolic murmur heard over the
left sternal border in the third intercostal space radiating to the left infrascapular area. The lungs
are clear to auscultation. There is a delay between the right and left radial pulse waves and the
left femoral pulse wave. The remainder of the physical examination is normal except for notably
long fingers. Which of the following is the most likely diagnosis?
 Incorrect Answer ImageA.Aortic stenosis
 Incorrect Answer ImageB.Juxtaductal coarctation of the aorta
 Correct Answer ImageC.Marfan syndrome
 Incorrect Answer ImageD.Preductal coarctation of the aorta
 Incorrect Answer ImageE.Turner syndrome

A 31-year-old woman, gravida 2, para 2, is referred to the gynecologist because of her inability
to become pregnant. She has been trying for the past 1 1/2 years to conceive. She has 2 children
from a previous partner. Her present partner has no children. She has regular periods every 28
days lasting 4 days. She has been taking her temperature each morning, and each month for the
last 6 months it has shown a pattern consistent with ovulation. She has had no surgeries or pelvic
infections. She reports that she and her partner are having intercourse at the appropriate time in
the cycle. She has a TSH of 3.4 µU/mL, a prolactin level of 13 ng/mL, and a luteal-phase
progesterone level of 11 ng/mL (Normal= 5 - 20 ng/mL). Which of the following is the most
appropriate next step in management? 
 Incorrect Answer ImageA.Arrange for a laparoscopy
 Correct Answer ImageB.Obtain a semen analysis
 Incorrect Answer ImageC.Order a serum FSH and LH
 Incorrect Answer ImageD.Schedule a hysterosalpingogram
 Incorrect Answer ImageE.Schedule a postcoital test

A 28-year-old primigravid woman visits an obstetrician for her initial prenatal visit. She is
approximately 7 weeks' gestation and has no known medical illnesses. One year ago she was
treated for gonorrhea with "a penicillin," following which, she had a severe allergic reaction
manifested by a rash and gastrointestinal upset. Her physical examination is within normal
limits. Prenatal laboratory tests are within normal limits, except that the cervical culture is
positive for Neisseria gonorrhoeae. Which of the following is the most appropriate therapy for
her current condition? 
 Incorrect Answer ImageA.Ampicillin
 Incorrect Answer ImageB.Chloramphenicol
 Correct Answer ImageC.Spectinomycin
 Incorrect Answer ImageD.Tetracycline
 Incorrect Answer ImageE.Ceftriaxone

A 44-year-old man is obsessed with his work and spends very little time at home with his wife
and children. His employees describe him as a "control freak"; he oversees every aspect of his
employees' work and spends so much time making decisions that the group’s productivity is
always lacking. When asked about the possibility of being more flexible in his approach to work,
he replies, "I just can't afford to. My employees aren't confident in what they're doing and they
would just make mistakes and slow everything down." Which of the following defense
mechanisms best explains this man’s statement about his employees?
 Incorrect Answer ImageA.Blocking
 Incorrect Answer ImageB.Denial
 Incorrect Answer ImageC.Isolation
 Correct Answer ImageD.Projection
 Incorrect Answer ImageE.Splitting

A 41-year-old man has had a history of seasonal bronchial asthma for several years. His
symptoms are usually well controlled with steroid inhalers in the spring and albuterol inhalers as
needed. The rest of the year, his symptoms are inactive. For the last year, his symptoms have
bothered him and have not been controlled by his usual regimen. He wheezes mostly at night and
also feels that his voice is more hoarse than usual. His other symptoms include substernal
burning on occasion, a sour taste in his mouth, and occasional nausea, especially when eating
fried foods. On physical examination he is overweight, he is overweight, his vital signs are
normal, and his lungs show bilateral, scattered, expiratory wheeze with good air entry.
Abdominal examination is normal. Which of the following is the most appropriate
pharmacotherapy to control his symptoms? 
 Incorrect Answer ImageA.Amoxicillin
 Incorrect Answer ImageB.Increased dose of albuterol
 Incorrect Answer ImageC.Ipratropium inhaler
 Correct Answer ImageD.Omeprazole
 Incorrect Answer ImageE.Prednisone

On a sunny Saturday morning, a local physician is an assistant coach for the elementary school
soccer team. All is well until the last minute of the first half when, in an attempt to score a goal,
two players end up in a head-on collision. As they fall to the ground, the physician runs across
the field to see if they are all right. One of them is a 12-year-old boy, and he gets up as if nothing
happened and walks over to help the other boy up. He has moderate discomfort in the area of his
vertex, where he was hit, but has no other complaints and denies loss of consciousness or
disturbance of vision and equilibrium. The second player is an 11-year-old boy who is still sitting
on the ground, unable to get up. He is a little confused and nauseated but awake, alert, and
oriented to person, place, and time. He clearly remembers hitting the other player in the head and
gently rubs the frontoparietal scalp where the collision occurred. In a couple of minutes he is
feeling much better and wants to continue with the game. Which of the following are the most
appropriate recommendations for these two boys to return to contact sports? 
 Incorrect Answer ImageA.Both boys, if asymptomatic, may return to contact sports
immediately
 Incorrect Answer ImageB.Both boys, if asymptomatic, may return to contact sports in 1
week
 Incorrect Answer ImageC.Both boys, if asymptomatic, may return to contact sports in 2
weeks
 Incorrect Answer ImageD.The 11-year-old boy should call it a season
 Correct Answer ImageE.The 12-year-old boy may return to play immediately, whereas
the 11-year-old boy, if asymptomatic, may return to contact sports in 20 minutes

A 45-year-old man is brought to the emergency department and is diagnosed with a non-ST
segment elevation myocardial infarction (NSTEMI). He undergoes appropriate treatment with
aspirin, clopidogrel, metoprolol, lisinopril, heparin drip, and atorvastatin in the emergency
department and is admitted to the intensive care unit. Forty-eight hours after admission he
develops a new murmur that is holosystolic. Right heart catheterization shows no oxygen step-up
between the right atrium and ventricle. The right atrial pressure is 12 mm Hg, right ventricular
pressure is 60/11 mm Hg, pulmonary artery pressure is 60/25 mm Hg, and the wedge pressure is
20 mm Hg with large V waves. Which of the following is the most likely cause of these
findings? 
 Incorrect Answer ImageA.Aortic dissection
 Correct Answer ImageB.Papillary muscle rupture
 Incorrect Answer ImageC.Pericardial tamponade
 Incorrect Answer ImageD.Ventricular free wall rupture
 Incorrect Answer ImageE.Ventricular septal rupture
A 59-year-old man with diabetes comes to the clinic at the urging of his wife. Approximately 5
days ago he suffered an episode of severe reflux-like symptoms associated with shortness of
breath. At the time he took some antacids, which provided minimal relief, and rested. He still felt
ill the next day, however, and had to stay home and rest. Since then, he has suffered from
hiccups and some mild dyspepsia that seem to be improving. He denies any recurrent episodes of
reflux-like symptoms or shortness of breath. In addition to diabetes, he has a history of chronic
renal insufficiency, hypertension, and smokes half a pack of cigarettes a day. His family history
includes a brother who died at age 52 years from a myocardial infarction (MI). An
electrocardiogram in the office shows a left bundle-branch pattern, present on old
electrocardiograms. Which of the following is the most appropriate serum marker to determine
whether this patient suffered a recent myocardial infarction? 
 Incorrect Answer ImageA.Creatinine kinase, total and MB
 Incorrect Answer ImageB.Lactate dehydrogenase
 Incorrect Answer ImageC.Myoglobin levels
 Correct Answer ImageD.Troponin I levels
 Incorrect Answer ImageE.Troponin T levels

A 47-year-old man comes to the physician because of a rapidly enlarging skin lesion on the
forehead. It is asymptomatic, but he is disturbed by its appearance. He has a history of liver
cirrhosis secondary to chronic hepatitis C and a liver transplant performed 3 years earlier.
Current medications include tacrolimus and prednisone. His family history is unremarkable.
Vital signs are within normal limits. On the forehead, two adjacent large nodules are present as
shown in the photograph. Inspection of the entire skin and oral mucosa does not reveal any other
similar lesions. There are palpable, rubbery-firm, non-tender lymph nodes present in the
retroauricular area. Which of the following viruses has been implicated in the pathogenesis of
this patient’s condition? 
 Incorrect Answer ImageA.Cytomegalovirus
 Correct Answer ImageB.Epstein-Barr virus
 Incorrect Answer ImageC.Herpes simplex virus type 1
 Incorrect Answer ImageD.Human T-cell lymphotropic virus-1
 Incorrect Answer ImageE.Human herpesvirus 8

A 57-year-old man is seen after passing a bloody bowel movement. He has also lost about 15
pounds in the past 2 months, which he attributes to decreased appetite. He otherwise has no
symptoms. Workup reveals a hemoglobin of 9.0 g/dL and colonoscopy shows a 4-cm, sessile,
fungating mass in his cecum. Biopsies return a diagnosis of adenocarcinoma. CT scan shows that
approximately 60% of his liver is replaced by metastatic disease. Liver function tests show
normal bilirubin, slightly elevated transaminases, and alkaline phosphatase approximately 5
times the upper limit of normal. Which of the following is the most appropriate management? 
 Incorrect Answer ImageA.Blood transfusion
 Incorrect Answer ImageB.Endoscopic resection of the cecal mass
 Incorrect Answer ImageC.Radiation therapy to the liver
 Incorrect Answer ImageD.Right hemicolectomy
 Correct Answer ImageE.Systemic chemotherapy with a 5-fluorouracil (5-FU) based
regimen

A 41-year-old woman undergoes an abdominal hysterectomy for a benign condition. The


patient's surgery was difficult due to extensive pelvic adhesions from 3 prior cesarean sections,
and there was significant blood loss requiring the transfusion of 2 units of packed red blood cells
postoperatively. Foley catheter was removed on postoperative day 1, and she was discharged
with a stable hemoglobin level and stable electrolytes. She is seen 2 weeks postoperatively, and
she describes persistent clear drainage from the vagina since she was discharged home. She has
been wearing a pad and is changing it every few hours. There are no fevers, dysuria, hematuria,
or change in bowel habits. She is afebrile and her vital signs are within normal limits. Her
abdominal incision is well healed and there is no incisional drainage or erythema. Pelvic
examination shows that her vaginal mucosa is not erythematous or friable. The vaginal cuff
incision is inspected and there is clear fluid noted to be pooling in the vaginal fornices. Rectal
examination does not show any masses or tenderness. Laboratory studies including complete
blood count with differential and urinalysis are all within normal limits and urine culture has no
growth. Which of the following is the most appropriate next step in management? 
 Incorrect Answer ImageA.Begin oral antibiotics
 Incorrect Answer ImageB.Examination under anesthesia and opening the vaginal cuff
 Correct Answer ImageC.Placement of a Foley catheter
 Incorrect Answer ImageD.Reassurance that this is normal and will resolve

 Incorrect Answer ImageE.Schedule a CT scan of the pelvis

A 39-year-old woman is brought to the emergency department because of suspected uterine


perforation during termination of her 7-week pregnancy. The physician performing the procedure
states that he may have perforated the uterus early in the procedure and, at one point, he believed
he saw bowel in the suction curette. The patient is in severe pain. Her temperature is 38.1°C
(100.6°F), pulse is 118/min, respirations are 24/min, and blood pressure is 95/60 mm Hg.
Physical examination shows marked, diffuse abdominal tenderness with guarding and rigidity.
Bimanual examination shows an extremely tender, retroverted uterus with adnexal tenderness.
Which of the following is the most appropriate next step in management? 
 Incorrect Answer ImageA.Colonoscopy
 Incorrect Answer ImageB.Dilation and curettage
 Incorrect Answer ImageC.Esophagogastroduodenoscopy (EGD)
 Incorrect Answer ImageD.Laparoscopy
 Correct Answer ImageE.Laparotomy

A 26-year-old woman comes to the physician because of a 1-year history of irregular menses.
Her past medical history is significant for polycystic ovarian syndrome (PCOS), which was
diagnosed 5 years ago. She became pregnant after undergoing ovulation induction with
clomiphene citrate 2 years ago. After her pregnancy, she was lost to follow-up until now. She is
not currently sexually active and does not wish to become pregnant for a few more years. Her
temperature is 37.1°C (98.8°F), pulse is 80/min, respirations are 12/min, and blood pressure is
120/80 mm Hg. Physical examination is unremarkable. Which of the following is the most
appropriate management of this patient's irregular menses?
 Incorrect Answer ImageA.Clomiphene citrate
 Correct Answer ImageB.Combination oral contraceptive pill
 Incorrect Answer ImageC.Insulin
 Incorrect Answer ImageD.No treatment is necessary for this condition
 Incorrect Answer ImageE.Thyroid hormone replacement

A 64-year-old woman initiates chemotherapy for metastatic breast cancer. She develops severe
nausea followed by hours of vomiting during her first chemotherapy infusion, which becomes
worse on subsequent infusions. Which of the following is the most effective in controlling her
nausea and vomiting? 
 Incorrect Answer ImageA.Bismuth
 Incorrect Answer ImageB.Morphine
 Incorrect Answer ImageC.Omeprazole
 Correct Answer ImageD.Ondansetron
 Incorrect Answer ImageE.Sertraline

A grandmother brings her 13-year-old grandson, who is in her custody, to the psychiatrist. The
boy has been talking back to his teachers and not complying with school rules. When asked to
stop, he gets angry and blames others, usually his classmates. In addition, his grades are
deteriorating. The grandmother has been struggling with behavior issues with him in the past
year and thinks he needs "structure and control." The boy says his grandmother does not let him
do anything, and he resents having to live with her. Physical examination is unremarkable and
psychological studies show no indication of a learning disorder. Which of the following is the
most likely diagnosis?
 Incorrect Answer ImageA.Adjustment disorder with disturbance of conduct
 Incorrect Answer ImageB.Attention-deficit/hyperactivity disorder (ADHD)
 Incorrect Answer ImageC.Conduct disorder
 Incorrect Answer ImageD.Cornelia de Lange syndrome
 Correct Answer ImageE.Oppositional defiant disorder

The mother of a 23-year-old man brings her son to a psychiatrist for a second opinion one week
after his discharge from the hospital. The young man had been hospitalized for one week due to
the new onset of auditory hallucinations and tangentiality of thoughts. During his hospitalization,
he was diagnosed with schizophreniform disorder and was treated with risperidone 2 mg PO
BID. A medical work-up for this first episode of psychosis was negative. In discussing course
and prognosis with the patient and his mother, which of the following features would be
considered a good prognostic indicator? 
 Incorrect Answer ImageA.Family history of schizophrenia
 Incorrect Answer ImageB.Lack of any known precipitating factors or acute stressors
 Correct Answer ImageC.Onset of psychotic symptoms within a month of the first change
in behavior
 Incorrect Answer ImageD.Poor premorbid educational and occupational performance
 Incorrect Answer ImageE.Withdrawn, autistic behavior and flat affect

A 50-year-old man is brought to his new primary care physician by his family, who report that he
has developed personality changes, impaired memory, and difficulty with speech over the past
10 months. His medical history is notable for intellectual disability. His medical chart indicates
that, at birth, he was diagnosed with trisomy 21 by genetic karyotyping. Physical examination
reveals epicanthal folds, a transverse palmar crease, and Brushfield spots on the iris. He is slow
to respond to questions and can recall only one out of three objects after 5 minutes. Which of the
following is the most likely cause of these new neurologic symptoms?
 Correct Answer ImageA.Alzheimer dementia
 Incorrect Answer ImageB.Hydrocephalus
 Incorrect Answer ImageC.Hypothyroidism
 Incorrect Answer ImageD.Multiple strokes
 Incorrect Answer ImageE.Prion infection

A 23-year-old man comes to the physician because he feels a disabling fear that his coworkers
judge him to be a bad employee. He minimizes conversations at work and avoids having to
present in meetings whenever possible so as to avoid the scrutiny of others. He says he has
always been shy but used to enjoy his coworkers and felt he did his job well. He, in fact, did his
job so well that he received an unwanted promotion last year. Since then, he feels it is just a
matter of time before he faces some kind of public humiliation. Which of the following is the
most likely diagnosis?
 Incorrect Answer ImageA.Antisocial personality disorder
 Incorrect Answer ImageB.Dependent personality disorder
 Incorrect Answer ImageC.Generalized anxiety disorder
 Incorrect Answer ImageD.Schizoid personality disorder
 Correct Answer ImageE.Social anxiety disorder

A 22-year-old gang member is brought to the emergency department with multiple gunshot
wounds to the chest and abdomen. He has labored breathing and is cyanotic, diaphoretic, cold,
and shivering. He is wide awake, and in a normal tone of voice, he tells everyone that he is going
to die. An initial survey reveals a blood pressure of 60/40 mm Hg. His pulse is 150/min and
barely perceptible. He is in obvious respiratory distress and has distended veins in his neck and
forehead. His trachea has deviated to the left, and the right side of his chest is hyper resonant to
percussion with no breath sounds. Which of the following is the most appropriate initial step in
management?
 Incorrect Answer ImageA.Emergent endotracheal intubation
 Incorrect Answer ImageB.Immediate chest x-ray films
 Incorrect Answer ImageC.Insert a 16-gauge needle in the fifth left intercostal space
 Correct Answer ImageD.Insert a 16-gauge needle in the second right intercostal space
 Incorrect Answer ImageE.Stat arterial blood gases

A 25-year-old male ski instructor comes to the physician because of fatigue for the past week.
He had previously been in excellent physical health and able to work 12 hours daily. He reports
diffuse myalgias and temperatures as high as 39.2°C (102.5°F) at home. He has also had
difficulty swallowing because of a sore throat. He drinks approximately 72 ounces of beer on
weekends and smokes one pack of cigarettes per day. On examination, his temperature is 38.7°C
(101.6°F), and his posterior pharynx is injected. He has bilateral anterior and posterior cervical
adenopathy, with lymph nodes measuring as large as 1 cm. They are firm, mobile, and mildly
tender. On abdominal examination, his liver span is 14 cm in the midclavicular line, and a spleen
tip is palpable. There is no ascites or peripheral edema. Which of the following is the most likely
cause of his symptoms?
 Incorrect Answer ImageA.Acute hepatitis B infection
 Correct Answer ImageB.Infectious mononucleosis
 Incorrect Answer ImageC.Non-Hodgkin lymphoma
 Incorrect Answer ImageD.Sarcoidosis
 Incorrect Answer ImageE.Streptococcal pharyngitis

A young mother brings her 6-month-old firstborn son for a well-child examination. She states
that the baby has been doing well and she has not noticed any abnormalities in his development.
The boy is breastfeeding and is increasing the number of solid foods he eats with each passing
day. He sleeps almost through the night and stopped having colic 2 months earlier. On physical
examination, the boy appears to be in no distress. He is in the 50th percentile for weight and
length and his vital signs are normal. On extension of the head, the boy flexes his arms and legs.
When a finger is put in his palm, he flexes his hand, elbow, and shoulder ipsilaterally. Tactile
stimulation of the cheek prompts him to turn his mouth to that side, and touching the dorsum of
his foot leads to a step-up movement. The physician takes the baby in her hands and simulates a
fall, to which he reacts by extending his arms. The mother asks about the newborn reflexes and
when they are supposed to disappear. Which of the following is likely to be the one reflex that
persists throughout life?
 Incorrect Answer ImageA.Extension of the head leads to flexion of the arms and legs
 Incorrect Answer ImageB.Finger in palm leads to flexion of the hand, elbow, and
shoulder ipsilaterally
 Correct Answer ImageC.Stimulation of a fall leads to extension of arms
 Incorrect Answer ImageD.Stimulation of the cheek prompts turning of mouth to that side
 Incorrect Answer ImageE.Stimulation of the dorsum of the foot leads to a step-up
movement

A 36-year-old woman, gravida 3, para 2, at 35 weeks' gestation comes to labor and delivery
because of contractions. She states that the contractions started this morning and have been
increasing in frequency and intensity. Her last pregnancy ended with an emergent classical
cesarean delivery at 28 weeks for an abruption. In a pregnancy before that, she had a normal
vaginal delivery after an uncomplicated prenatal course. Physical examination shows the cervix
6-cm dilated and 100% effaced, and the fetus is breech. The fetal heart rate is 140/min with
moderate variability, and the patient is contracting every 3 minutes. Which of the following is
the most appropriate next step in management?
 Incorrect Answer ImageA.Administration of corticosteroids
 Incorrect Answer ImageB.Administration of magnesium sulfate
 Incorrect Answer ImageC.Discharge of the patient home
 Correct Answer ImageD.Immediate cesarean delivery
 Incorrect Answer ImageE.Induction of labor with oxytocin

A 59-year-old man with a long history of smoking is referred for evaluation of hypertension of
recent onset that is refractory to medical management. On the physical examination, a faint right
flank bruit is heard and a pulsatile abdominal mass is detected. Pertinent laboratory values show
serum potassium of 4.5 mEq/L, serum creatinine of 1.2 mg/dL, elevated levels of plasma renin,
and negative urinary VMA. CT scan shows a 6.5-cm abdominal aortic aneurysm (AAA) arising
at the level of the renal arteries and extending down into both common iliac arteries. Cardiac
evaluation is negative for coronary artery disease. Surgical repair of the aneurysm is
recommended, but before the operation is performed, which of the following is the most
appropriate next step in management?
 Incorrect Answer ImageA.Aggressive medical treatment for the hypertension
 Correct Answer ImageB.Arteriogram
 Incorrect Answer ImageC.MRI of both adrenal glands
 Incorrect Answer ImageD.MRI of both kidneys
 Incorrect Answer ImageE.Spiral CT scan of the thoracic aorta

During the seventh month of her pregnancy, a very wealthy 34-year-old woman has a sonogram
performed to check on the progress of her unborn child. The study shows that the fetus has a
very large congenital diaphragmatic hernia on the left side. Detailed analysis of the images
allows the radiologist to predict that the infant will be born with severe respiratory distress. The
woman asks that, regardless of cost, every possible step be taken to give the infant the best
chance of survival. Which of the following is the most appropriate management? 
 Incorrect Answer ImageA.Deliver the infant by C-section so that immediate surgical
repair of the hernia can be done in an adjacent operating room
 Correct Answer ImageB.Deliver the infant within a medical center equipped with
extracorporeal membrane oxygenation
 Incorrect Answer ImageC.Induce labor right now so that surgery can be done on the
infant as soon as possible
 Incorrect Answer ImageD.Perform intrauterine surgery, to correct the defect before birth
 Incorrect Answer ImageE.Perform a lung transplant at the time of the infant's birth

A 3-year-old girl is brought to the emergency department by her mother because of intractable
vomiting and diarrhea that started that morning. The mother explains that the girl may have a
serious disease and asks that all necessary examinations and tests be performed to determine
what it is. She is very affectionate toward her child and does not leave her side at any time. She
demands to know exactly what is being done at all times and what she can do to help, as she is a
nurse practitioner who has devoted herself to being a mother and housewife since her daughter
was born. The patient's prior medical history is significant for multiple hospitalizations with
similar gastrointestinal complaints that have not been causally elucidated. The child has had
several different pediatricians because the family has moved frequently since her birth. The
family history is significant for an older sibling that died at age 8 months from sudden infant
death syndrome. On physical examination, the patient is a pale, frightened girl in moderate
distress, frequently gagging and vomiting a thin, watery liquid. Her temperature is 36.6°C
(97.9°F), pulse is 120/min, and respirations are 22/min. Her weight is in the 5th percentile and
her height is in the 30th percentile. No abnormalities are found on examination of the skin and
visible mucosa, and her abdomen is nontender to palpation. Basic laboratory studies are within
normal limits. After several hours of intravenous rehydration and observation in the emergency
department, the patient is stable and the physician informs the mother that her daughter will be
discharged with appropriate follow-up instructions. The mother is not satisfied, however, and
insists that additional tests be performed, such as gastroscopy and magnetic resonance imaging to
determine the cause of the child's illness. Which of the following is the most appropriate next
step in the management? 
 Incorrect Answer ImageA.Consult gastroenterology for a full workup
 Correct Answer ImageB.Contact the patient's pediatrician and request prior medical
records
 Incorrect Answer ImageC.Discharge the patient
 Incorrect Answer ImageD.Order magnetic resonance imaging of the abdomen
 Incorrect Answer ImageE.Request a psychiatric consult for the mother

A 68-year-old man is brought to the emergency department because of chills and a cough. He
has a history of prostate cancer metastatic to bone that was diagnosed 2 years ago. At that time
his PSA was 103 ng/mL. Bone scan showed positive radioisotope uptake throughout the axial
skeleton and the skull. He was started on goserelin, and four months later his back pain improved
significantly, requiring only occasional acetaminophen, and his PSA decreased to 4.0 ng/mL. He
was doing well until approximately 3 months ago when he started to have more pain in his back.
Oral morphine was added to his acetaminophen by his primary care physician. Three days ago
the patient developed a cough productive of brown sputum, profound fatigue, and shortness of
breath that became progressively worse. Physical examination shows a cachectic elderly man in
moderate distress with a temperature of 38.3°C (101.0°F), blood pressure of 110/60 mm Hg,
pulse of 110/min, respirations of 20/min, and oxygen saturation of 94% on room air. Chest
examination shows right mid-lung rhonchi and egophony. Chest x-ray shows a right lung
consolidation with air bronchograms. Laboratory studies show: 
Leukocyte count 3,500/mm3
Neutrophils 80%
Lymphocytes 13%
Monocytes 5%
Hemoglobin 7 g/dL
Hematocrit 21%
MCV 79
Platelets 105,000/mm3
Reticulocyte count 4.1%
LDH 207 U/L
PSA 423 ng/mL
Creatinine 1.1 mg/dL
Total bilirubin 1.2 mg/dL
Direct bilirubin 0.6 mg/dL
Peripheral smear is unremarkable. Which of the following best explains this patient's
hematologic abnormalities?
 Correct Answer ImageA.Bone marrow suppression caused by its infiltration by prostate
cancer
 Incorrect Answer ImageB.Disseminated intravascular coagulation caused by pneumonia
 Incorrect Answer ImageC.Evan syndrome
 Incorrect Answer ImageD.Goserelin-induced bone marrow suppression
 Incorrect Answer ImageE.Thrombotic thrombocytopenic purpura

A 71-year-old man has low back pain that began insidiously approximately 6 months ago. The
pain is constant, dull, aching, of moderate intensity, worse at night, and partially relieved with
over-the-counter analgesics. Past history includes a carotid endarterectomy 3 years ago and
lobectomy for cancer of the lung 8 years ago. He has had no further transient ischemic attacks
and has had no evidence of tumor recurrence. However, he still smokes 1 pack of cigarettes per
day. On physical examination, the only pertinent finding is a large, pulsatile mass deep in his
upper abdomen. Sonogram shows an infrarenal fusiform abdominal aortic aneurysm with a
maximum diameter of 7.5 cm. CT scan confirms the presence and size of the aneurysm and
shows it to be surrounded by a rind of inflammatory tissue. He has an elevated erythrocyte
sedimentation rate. Which of the following is the most appropriate management at this time?
 Incorrect Answer ImageA.Arteriogram
 Correct Answer ImageB.Elective surgical repair of the abdominal aortic aneurysm
 Incorrect Answer ImageC.Emergency surgical repair of the abdominal aortic aneurysm
 Incorrect Answer ImageD.Prolonged course of intravenous antibiotic therapy
 Incorrect Answer ImageE.Radionuclide bone scans

A 67-year-old woman is admitted to the medical intensive care unit because of an upper
gastrointestinal bleed. Upon admission the patient had adequate fluid resuscitation and since
admission vital signs have been stable. She has a history of hypertension and gastroesophageal
reflux disease (GERD) and denies previous episodes of bleeding. Which of the following is the
most appropriate diagnostic test to localize the source of bleeding?
 Incorrect Answer ImageA.Arteriography
 Incorrect Answer ImageB.Exploratory laparotomy
 Incorrect Answer ImageC.Nasogastric lavage
 Incorrect Answer ImageD.Radionucleotide scan
 Incorrect Answer ImageE.Upper gastrointestinal series
 Correct Answer ImageF.Upper gastrointestinal endoscopy
A 53-year-old Caucasian man comes to a new physician for an initial visit. He has been in good
health his entire life, but has started to worry about himself because one of his friends recently
suffered a heart attack. He works as a banker and has not seen a physician in 20 years. He does
not use tobacco or alcohol, and he exercises regularly. His review of systems is negative. He
weighs 168 pounds and is 6 ft tall. His blood pressure is 165/90 mm Hg, and pulse 92/min. This
blood pressure reading is confirmed a second time. His physical examination is remarkable only
for a prominent apical impulse. Electrocardiogram shows normal sinus rhythm with borderline
left ventricular hypertrophy. Which of the following is the most appropriate next step in the
management? 
 Incorrect Answer ImageA.See this patient again in 6 months for verification of blood
pressure with no intervention at this time
 Incorrect Answer ImageB.Initiate a lifestyle modification and re-evaluate the patient in 1
year
 Incorrect Answer ImageC.Initiate therapy with an ACE inhibitor
 Correct Answer ImageD.Initiate therapy with a beta-blocker
 Incorrect Answer ImageE.Initiate therapy with a calcium channel blocker

A previously healthy 14-month-old girl who immigrated as an infant from Nigeria has been
inconsolably crying for the past 6 hours. An older sibling has been home from school for the past
2 days with a fever and cough. On examination, the child's temperature is 38.2°C (100.8°F),
pulse is 170/min, and blood pressure is 100/60 mm Hg. She is well nourished, well developed,
crying, has mild scleral icterus, and notable symmetric, tender swelling of the fingers and toes.
Laboratory studies show a hemoglobin of 9 g/dL, a white blood cell count of 10,000, and 10%
reticulocytes. Which of the following would most likely be absent from a hemoglobin
electrophoresis?
 Correct Answer ImageA.Hemoglobin A
 Incorrect Answer ImageB.Hemoglobin A2
 Incorrect Answer ImageC.Hemoglobin F
 Incorrect Answer ImageD.Hemoglobin S

A 76-year-old white man is brought to the hospital with worsening shortness of breath. The
patient's past medical history is significant for long standing coronary artery disease,
hypertension and congestive heart failure. His temperature is 37.0°C (98.6°F), pulse is 106/min,
respirations are 20/min, blood pressure is 145/83 mm Hg, and oxygen saturation is 94% on 2 L
oxygen via nasal cannula. In the emergency room the patient is diagnosed with a congestive
heart failure exacerbation and is given oxygen and intravenous furosemide. He is in no acute
distress and physical examination reveals the presence of jugular venous distention, distant heart
sounds and an S3 heart sound. The patient has 2+ lower extremity pitting edema. Chest x-ray
shows bilateral pleural effusions, more significant on the left. A diagnostic thoracentesis is
performed. Which of the following sets of laboratory data is consistent with a pleural effusion
secondary to congestive heart failure?
Pleural Pleural Pleural Leukocyte
fluid/plasma fluid LDH fluid/plasma count
protein ratio U/L LDH ratio mm3

A. 0.4 240 0.4 18,000

B. 0.6 180 0.7 18,000

C. 0.6 220 0.5 2,300

D. 0.4 170 0.5 1,900

E. 0.7 240 0.7 120,000


 Incorrect Answer ImageA.A
 Incorrect Answer ImageB.B
 Incorrect Answer ImageC.C
 Correct Answer ImageD.D
 Incorrect Answer ImageE.E

A 50-year-old woman, gravida 3, para 3, has heavy menstrual periods for the past 6 months that
occur at 40- to 50-day intervals. Until that time menstruation had been regular, coming every 28
days. She is taking no medications. Vital signs are within normal limits, and pertinent findings
are limited to the pelvic examination. The external genitalia are normal, and there is a small
cystorectocele. The cervix has some dark blood in the external os, but is otherwise unremarkable.
The uterus is of normal size, retroverted, retroflexed, and bulky, but symmetrical. There are no
adnexal masses. A serum pregnancy test is negative. Which of the following is the next best step
in management? 
 Incorrect Answer ImageA.Arrange for a total vaginal hysterectomy, anterior and
posterior colporrhaphy, and bilateral salpingo-oophorectomy
 Incorrect Answer ImageB.Order a serum FSH, HPL, and estradiol
 Correct Answer ImageC.Plan for an endometrial aspiration
 Incorrect Answer ImageD.Prescribe cyclic progestin therapy
 Incorrect Answer ImageE.Schedule a dilation and curettage

A 42-year-old woman, gravida 3, para 3, comes to the physician because of urinary leakage. She
has been well until 3 months ago when she began working as an aide in a nursing home and
started to experience recurrent involuntary episodes of urine loss, especially during work. She
denies having these episodes at night. All of her children weighed more than 4 kg (9 lb) at birth.
During her last pregnancy, she developed gestational diabetes which she successfully managed
with diet. She is healthy and does not take any medications. Three years ago she had an
uncomplicated total abdominal hysterectomy for symptomatic uterine fibroids. She weighs 72.5
kg (160 lb) and is 147 cm (4 ft 10 in) tall. Physical examination shows a soft, nontender
abdomen and a well-healed Pfannenstiel incision with no costovertebral angle or suprapubic
tenderness. There is a first-degree cystocele present with urethral hypermobility. No pelvic
masses are present. Urinalysis is negative for albumin and nitrites but positive for glucose.
Which of the following is the most likely diagnosis?
 Incorrect Answer ImageA.Bypass incontinence

 Incorrect Answer ImageB.Functional incontinence


 Incorrect Answer ImageC.Mixed incontinence
 Incorrect Answer ImageD.Overflow incontinence
 Correct Answer ImageE.Stress incontinence
 Incorrect Answer ImageF.Urge incontinence

Two hours after delivery, a 2,950 g newborn female develops respiratory distress and cyanosis.
She was born at 38 weeks’ gestation via vaginal delivery and received Apgar scores of 8 and 8 at
1 and 5 minutes, respectively. Pulse oximetry shows an oxygen saturation of 82%. During
physical examination, she begins to cry, turns pink, and the oxygen saturation level rises to 98%.
After she calms down and ceases crying, she soon develops cyanosis again. Which of the
following is the most appropriate diagnostic study?
 Incorrect Answer ImageA.Chest x-ray
 Correct Answer ImageB.CT scan of the head and neck
 Incorrect Answer ImageC.Indirect laryngoscopy
 Incorrect Answer ImageD.Placing a mirror in front of the patient's mouth to detect
fogging
 Incorrect Answer ImageE.Rhinography

A 15-year-old boy is brought to the emergency department because of the sudden onset of
excruciating pain in his scrotum and left groin. He woke up from sleep due to the pain and has
had two episodes of vomiting. He denies any history of trauma and dysuria. He has recently
become sexually active and does not use barrier protection. His temperature is 37°C (98.6°F),
pulse is 120/min, respirations are 14/min, and blood pressure is 130/90 mm Hg. On physical
examination, he is in visible distress. Genital examination shows a swollen scrotum and an
elevated right testicle, which is exquisitely tender to palpation. Elevation of the testicle does not
relieve the pain. The cremasteric reflex is also absent. Which of the following is the most
appropriate next step in management?
 Incorrect Answer ImageA.Administer ceftriaxone and doxycycline
 Incorrect Answer ImageB.Blood culture for N. gonorrhea and Chlamydia
 Incorrect Answer ImageC.Observe in the emergency department and if no clinical
improvement is seen, order a testicular scan
 Incorrect Answer ImageD.Order an HIV test
 Incorrect Answer ImageE.Perform Doppler ultrasound
 Correct Answer ImageF.Prepare for immediate surgical exploration

The following vignette also applies to the next item. 


A 19-year-old woman has a painful, swollen right knee. There is no history of trauma. She notes
that her wrists and ankles were sore over the past week, but she did not have swelling of these
joints. Her temperature is 38.1°C (100.5°F), pulse is 72/min, and respirations are 16/min. The
neck is supple, with no lymphadenopathy. There are no tonsillar exudates. Cardiac examination
is normal. The lungs are clear to auscultation. The abdomen is soft, nontender, and nondistended,
with normal bowel sounds. The right knee joint is swollen, warm, erythematous, and tender.
There is a large joint effusion and a decreased range of passive and voluntary motion. X-ray of
the right knee reveals normal articular surfaces, an effusion, and significant soft tissue swelling.
Painless necrotic pustules are present on the right lower leg and sole of the foot. Which of the
following is the most appropriate next step in management? 
 Correct Answer ImageA.Arthrocentesis
 Incorrect Answer ImageB.Administration of intravenous antibiotics
 Incorrect Answer ImageC.Arthroscopy
 Incorrect Answer ImageD.Joint immobilization
 Incorrect Answer ImageE.Serology for chlamydia

A 19-year-old woman has a painful, swollen right knee. There is no history of trauma. She notes
that her wrists and ankles were sore over the past week, but she did not have swelling of these
joints. Her temperature is 38.1 C (100.5 F), pulse is 72/min, and respirations are 16/min. The
neck is supple, with no lymphadenopathy. There are no tonsillar exudates. Cardiac examination
is normal. The lungs are clear to auscultation. The abdomen is soft, nontender, and nondistended,
with normal bowel sounds. The right knee joint is swollen, warm, erythematous, and tender.
There is a large joint effusion and a decreased range of passive and voluntary motion. X-ray of
the right knee reveals normal articular surfaces, an effusion, and significant soft tissue swelling.
Painless necrotic pustules are present on the right lower leg and sole of the foot. Which of the
following is the most likely diagnosis? 
 Incorrect Answer ImageA.Gout
 Incorrect Answer ImageB.Pseudogout
 Correct Answer ImageC.Gonococcal arthritis
 Incorrect Answer ImageD.Staphylococcal arthritis
 Incorrect Answer ImageE.Panarthritis

The following vignette applies to the next two items.


A 28-year-old man is involved in a frontal collision with another vehicle. He was the driver of
the car, and he was wearing a seat belt at the time of the accident. In the emergency department,
the patient is conscious and oriented to time, person, and place. His vital signs are: blood
pressure 110/60 mm Hg, pulse 100/min, and respirations 22/min. His breath smells of alcohol.
The pupils are equal and reacting to light and accommodation. The cranial nerves are grossly
intact, except for nystagmus on the left. He has a 3-cm laceration of the scalp over the left frontal
area. His speech is slurred, and he has difficulty with the finger-to-nose test. Deep tendon
reflexes are normal, Babinski sign is negative, and there is no evidence of hemiparesis. There are
bruises over the right thigh, left forearm, and the left pectoral area. During the neurologic
examination, the patient keeps insisting that there is nothing wrong with him except for the
laceration in the head. The trachea is central. No bruises, masses, or tenderness are noted in the
neck. Chest expansion is equal, and air entry in both lungs is normal. The point of maximum
impulse is in the left 5th intercostal space in the mid-clavicular line. Heart sounds are normal,
and there are no additional sounds. A diagnosis of myocardial contusion is entertained.
Examination of the abdomen reveals no bruises, tenderness, guarding, or rigidity. The liver and
spleen are not palpable, and bowel sounds are present. This condition is most likely caused by
trauma to which of the following regions of the heart? 
 Incorrect Answer ImageA.Interventricular septum
 Incorrect Answer ImageB.Left atrium
 Incorrect Answer ImageC.Left atrium and left ventricle
 Incorrect Answer ImageD.Right atrium
 Correct Answer ImageE.Right atrium and right ventricle

A 28-year-old man is involved in a frontal collision with another vehicle. He was the driver of
the car, and he was wearing a seat belt at the time of the accident. In the emergency department,
the patient is conscious and oriented to time, person, and place. His vital signs are: blood
pressure 110/60 mm Hg, pulse 100/min, and respirations 22/min. His breath smells of alcohol.
The pupils are equal and reacting to light and accommodation. The cranial nerves are grossly
intact, except for nystagmus on the left. He has a 3-cm laceration of the scalp over the left frontal
area. His speech is slurred, and he has difficulty with the finger-to-nose test. Deep tendon
reflexes are normal, Babinski sign is negative, and there is no evidence of hemiparesis. There are
bruises over the right thigh, left forearm, and the left pectoral area. During the neurologic
examination, the patient keeps insisting that there is nothing wrong with him except for the
laceration in the head. The trachea is central. No bruises, masses, or tenderness are noted in the
neck. Chest expansion is equal, and air entry in both lungs is normal. The point of maximum
impulse is in the left 5th intercostal space in the mid-clavicular line. Heart sounds are normal,
and there are no additional sounds. A diagnosis of myocardial contusion is entertained.
Examination of the abdomen reveals no bruises, tenderness, guarding, or rigidity. The liver and
spleen are not palpable, and bowel sounds are present. Which of the following is the most
appropriate study to establish the diagnosis? 
 Incorrect Answer ImageA.Cardiac enzymes
 Incorrect Answer ImageB.Chest x-ray
 Incorrect Answer ImageC.Echocardiography
 Incorrect Answer ImageD.Magnetic resonance imaging
 Correct Answer ImageE.Radionuclide angiography

A 7-year-old boy is brought to the pediatrician by his mother because of "hair loss". The mother
reports that over the past several weeks there has been a patch of scaling hair loss on her son's
scalp that has grown progressively larger. The child has no other symptoms related to the hair
loss and the mother has not witnessed the child scratching his scalp or pulling his hair.
Examination of the scalp shows patchy areas of dandruff-like scaling with extensive hair loss and
minimal inflammation. The child is afebrile and calm throughout the examination. He has no
other areas of skin with abnormalities. Which of the following is the most likely diagnosis?
 Incorrect Answer ImageA.Alopecia areata
 Incorrect Answer ImageB.Aplasia cutis
 Incorrect Answer ImageC.Scabies
 Correct Answer ImageD.Tinea capitis
 Incorrect Answer ImageE.Trichotillomania

 32-year-old woman brings her 3-month-old firstborn son to the physician for a routine checkup.
The pregnancy was uneventful, and she delivered a healthy 3.2-kg, 46-cm-long infant at full term
with no complications. At birth, the baby boy had a discrete red patch present on the glabella and
on the nuchal area; both blanched readily with pressure. The baby has been thriving since birth
and the mother is very happy with his development. However, while the patch on the forehead
has completely disappeared, she is concerned about the red patch on the back of the neck. The
boy is alert and smiling during the office visit. His vital signs are normal and he is in no acute
distress. There is no sign of any vascular malformation on the face. On the midline of the nuchal
area, a well-demarcated pink-red macule approximately 1.3 cm in size is seen with the long axis
parallel to the spine. It readily blanches with pressure. Which of the following is the most likely
type of anomaly resulting in the skin lesion?
 Incorrect Answer ImageA.Arteriovenous fistula
 Correct Answer ImageB.Capillary malformation
 Incorrect Answer ImageC.Increased sensitivity of blood vessels to catecholamines
 Incorrect Answer ImageD.Phlebectasias
 Incorrect Answer ImageE.Venous malformation

A 71-year-old man with hypertension comes to the physician because of gradually increasing
episodes of "urination problems." Over the past month, he has had five to six episodes of
nocturia, a weak urinary stream about half the time, and a sensation of not emptying his bladder
almost all the time. He denies straining, urgency, or frequency of micturition. His only
medications are atenolol and aspirin. His pulse is 60/min and blood pressure is 150/95 mm Hg.
Prostate examination is normal, with an estimated volume <40 cc. Abdominal examination
shows normal sized bladder without enlargement. Urinalysis is negative for protein, erythrocytes,
and leukocytes. Laboratory studies show:
Glucose 90 mg/dL
Creatinine 1.0 mg/dL
Prostate-specific 5 µg/mL
antigen (0-6.5 µg/mL for patients over the age of
70 years)
Which of the following treatments is the most appropriate to alleviate the patient's symptoms? 
 Correct Answer ImageA.Doxazosin
 Incorrect Answer ImageB.Finasteride
 Incorrect Answer ImageC.Transurethral incision of the prostate
 Incorrect Answer ImageD.Transurethral resection of the prostate
 Incorrect Answer ImageE.Trimethoprim-sulfamethoxazole

A 67-year-old woman has an acute onset of severe back pain. On physical examination, she has
point tenderness at the T8 level of her spine. X-ray of the thoracic spine shows a compression of
the T8 vertebral body. Further evaluation shows a protein level of 10.5 gm/dL, albumin 4 gm/dL,
creatinine 2.6 mg/dL, and calcium 12 mg/dL. Which of the following studies will most likely
confirm the diagnosis? 
 Incorrect Answer ImageA.Bilateral mammography

 Incorrect Answer ImageB.Bone densitometry

 Incorrect Answer ImageC.Parathyroid hormone (PTH) level

 Incorrect Answer ImageD.Renal biopsy

 Correct Answer ImageE.Serum immune protein electrophoresis

A 71-year-old man undergoes an uneventful left inguinal hernia repair under spinal anesthesia.
Seven hours after surgery, the man has not urinated. He feels the urge to void, but when he
stands at the toilet, he is unable to initiate urination. He tries to strain like he usually has to while
voiding at home but says his incisional pain prevents him from doing so adequately. His
temperature is 37.0°C (98.6°F), pulse is 65/min, respirations are 20/min and blood pressure is
120/80 mm Hg. On physical examination, his wound dressing is clean and dry. His lower
abdomen is distended over the suprapubic area. There is a palpable mass in the suprapubic region
which is dull to percussion. On rectal examination, the prostate is uniformly enlarged and non-
tender. The rest of his physical examination is unremarkable. Which of the following is the most
appropriate next step in management?
 Incorrect Answer ImageA.Give a bolus of intravenous fluid
 Correct Answer ImageB.In-and-out catheterization
 Incorrect Answer ImageC.Order a bladder ultrasound
 Incorrect Answer ImageD.Place a suprapubic cystostomy catheter
 Incorrect Answer ImageE.Treatment with Tamsulosin

 21-year-old male college student agrees to be the chef for his fraternity's homecoming barbecue.
He consumes several cans of beer while cooking and becomes intoxicated. Shortly thereafter, his
right sleeve catches fire, and he suffers major burns to his right upper extremity. He is rushed to
the emergency department where the physician identifies third-degree burns to the right arm,
including a circumferential burn to his right forearm. Access is quickly obtained and fluid
resuscitation initiated. Two hours following his initial presentation, the patient complains of a
deep ache and burning sensation in his right forearm. There is a diminished radial pulse and poor
capillary refill of the affected extremity. Pulse oximetry on his right second finger shows an
oxygen saturation of 58%. Which of the following is the most appropriate next step in the
management of this patient?
 Incorrect Answer ImageA.Check carboxyhemoglobin levels
 Incorrect Answer ImageB.Debride wounds at bedside and plan for escharotomy in 24
hours
 Incorrect Answer ImageC.Draw blood-alcohol level
 Incorrect Answer ImageD.Perform escharotomy under general anesthesia
 Correct Answer ImageE.Perform escharotomy without general anesthesia

A 31-year-old man is brought to the emergency department after being involved in an


automobile accident as an unrestrained driver. He is conscious but in moderate distress and
reports severe retrosternal chest pain. His temperature is 37°C (98.6°F), pulse is 120/min,
respirations are 20/min, and blood pressure is 100/60 mm Hg. Physical examination shows
crepitus on palpation of the anterior chest wall but no bruises. The trachea is deviated slightly to
the right. The left hemithorax does not expand fully, is dull to percussion, and there are
diminished breath sounds on the left. The right hemithorax is normal. On cardiac examination,
the point of maximum impulse is in the left 5th intercostal space, midclavicular line. The heart
sounds are normal. The abdomen is soft, nontender, nondistended, and audible bowel sounds are
present in all four quadrants. Chest x-ray shows mediastinal air and a left pleural effusion. ECG
shows sinus tachycardia but is otherwise unremarkable. Laboratory studies show:
Hemoglobin 15 g/dL
Hematocrit 45%
Platelet count 350,000/mm3
Leukocyte count 17,500/mm3
Segmented neutrophils 75%
Band forms  5%
Eosinophils 2%
Basophils 1%
Lymphocytes 16%
Monocytes 1%
Serum sodium 150 mEq/L
Serum potassium 4.5 mEq/L
Serum chloride 100 mEq/L
Serum bicarbonate 22 mEq/L
Serum magnesium 1.5 mEq/L
Serum creatinine 1.2 mg/dL
BUN 16 mg/dL
Serum glucose 110 mg/dL
Serum amylase 180 U/L
Which of the following is the best test to confirm the diagnosis? 
 Correct Answer ImageA.Contrast esophagogram
 Incorrect Answer ImageB.CT scan of the chest
 Incorrect Answer ImageC.MRI of the chest
 Incorrect Answer ImageD.Transesophageal echocardiography
 Incorrect Answer ImageE.Ultrasound scan

A 4-year-old boy is brought to the pediatrician because he is growing up "differently" from his
older brother. The mother noticed that he does not speak as much as other children his age,
repeats what other people say, plays with the same train, pushing it around tirelessly, is not
interested in playing with children his age, and does not care to receive hugs or kisses from his
parents. These difficulties have been present since the age of two. On physical examination, he
has age-appropriate height, weight, and head circumference. Which of the following is the most
likely diagnosis? 
 Incorrect Answer ImageA.Asperger disorder
 Correct Answer ImageB.Autistic disorder
 Incorrect Answer ImageC.Rett disorder
 Incorrect Answer ImageD.Rumination disorder
 Incorrect Answer ImageE.Schizophrenia

A 54-year-old man is admitted to the hospital for fevers and increasing fatigue. Two weeks
earlier he began to experience fevers that waxed and waned but have become progressively more
intense. The fatigue also has worsened steadily and is now so severe as to keep him bed-bound
for the last 3 days. His past medical history is remarkable only for dyslipidemia and mild,
asymptomatic mitral valve prolapse. His only medications are aspirin and lovastatin. On physical
examination he seems lethargic but in no acute distress. His temperature is 38.2°C (100.8°F),
blood pressure is 125/82 mm Hg, and pulse is 112/min. Funduscopic examination is
unremarkable, there is no photophobia, and there is no nuchal rigidity. The lungs are clear to
auscultation and cardiac examination is unremarkable. There are no visible skin lesions.
Laboratory studies show: 
Leukocyte count 15,200/mm3 (Normal 4.4–11.0 cells/mm3)
Hemoglobin 13.3 g/dL (Normal adult male 14–18 g/dL)
Hematocrit 39.3% (Normal adult male 42–52%)
Platelets 460,000/mm3 (Normal 150–400,000/mm3)
MCV 82 fL (Normal 80–95 fL)
Iron studies show:
Serum iron 75 µg/dL (Normal 60–190 mcg/dL)
Serum TIBC 390 µg/dL (Normal 250–420 mcg/dL)
Serum ferritin 20 ng/mL (Normal 15–200 ng/mL)
A transesophageal echocardiogram (TEE) reveals mitral valve prolapse but otherwise normal
cardiac valves and normal systolic function. 
Multiple blood cultures grow Streptococcus bovis after 3 days. He is treated as an outpatient with
intravenous penicillin and his symptoms resolve. Appropriate further care for the cause of this
patient's acute illness should include which of the following? 
 Incorrect Answer ImageA.Erythropoietin therapy
 Correct Answer ImageB.Full colonoscopy
 Incorrect Answer ImageC.Oral iron supplementation
 Incorrect Answer ImageD.Serial echocardiogram
 Incorrect Answer ImageE.Surgical replacement of the mitral valve

A 21-year-old man comes to the physician because of concerns about his father's death. His
father, who was 59-years-old, recently passed away after a 15-year struggle with Huntington
disease. The son denies experiencing chorea, cognitive deficits, or neuropsychiatric symptoms.
He does report feeling sad about his father’s death. He reports feeling anxious and concerned
regarding his future risk of disease. Vital signs and physical examination are unremarkable.
Which of the following is the most appropriate management for this patient?
 Incorrect Answer ImageA.Advise that the disease becomes less severe from generation to
generation
 Incorrect Answer ImageB.Begin carbidopa/levodopa at the first sign of Huntington
symptoms
 Incorrect Answer ImageC.Reassure him that the risk of disease in the next 30 years is
miniscule
 Correct Answer ImageD.Recommend genetic counseling for the family, followed by
genetic testing
 Incorrect Answer ImageE.Schedule regular brain imaging studies to look for caudate
atrophy

A 65-year-old African American man comes to the emergency department because of lip
swelling that started that afternoon. He first noted a tingling sensation in the upper lip, followed
by slow and steady enlargement that continues to worsen. He tried using ice-packs to relieve the
swelling, but it did not help. He has no prior history of such an event. His past medical history is
significant for hypertension, gout, diabetes mellitus, and coronary artery disease. Current
medications include lisinopril, hydrochlorothiazide, allopurinol, metformin, and aspirin. He has
no known allergies to medications. On physical examination, the patient is in mild acute distress
and appears frightened. His pulse is 90/min, respirations are 20/minute, and blood pressure is
145/90 mm Hg. You quickly assess the airway and find that it is not compromised. Bilateral
breathing sounds are normal. Physical examination of the face is seen in the image. The oral
mucosa, palate, and tongue appear normal. His voice is crisp without any hoarseness. The rest of
the physical examination is within normal limits. Which of the following medications is the most
likely cause of this patient's lip swelling?
 Incorrect Answer ImageA.Allopurinol
 Incorrect Answer ImageB.Aspirin
 Incorrect Answer ImageC.Hydrochlorothiazide
 Correct Answer ImageD.Lisinopril
 Incorrect Answer ImageE.Metformin

A 75-year-old man has jaundice, weight loss, and anorexia. These symptoms began
approximately 3 months ago and have gradually become more impressive. In addition to the
jaundice and the signs of weight loss, physical examination is remarkable for a very large, tense,
but not tender, gallbladder that can be palpated in the right upper quadrant. Laboratory studies
show a total bilirubin of 27 mg/dL, with 21 mg/dL direct fraction (conjugated), and an alkaline
phosphatase 10 times the upper limit of normal. Sonogram of the right upper quadrant shows
dilated intrahepatic and extrahepatic biliary ducts and a very large, thin-walled, dilated
gallbladder without stones. Which of the following is the most appropriate next step in
management?
 Incorrect Answer ImageA.Check a serum CA 19-9
 Correct Answer ImageB.CT scan of the abdomen
 Incorrect Answer ImageC.Endoscopic retrograde cholangiopancreatogram (ERCP)
 Incorrect Answer ImageD.Laparoscopy
 Incorrect Answer ImageE.Upper gastrointestinal endoscopy
A 5-year-old boy is brought to the physician because his parents have noticed that one eye seems
to have become larger than the other over the past several weeks. They deny any trauma or injury
to that area. The child has no complaints and is in good health. He does not take any medication.
Family history is significant for neurofibromatosis in the maternal family. On physical
examination, the patient is in no acute distress. His vital signs are within normal limits.
Inspection of the head shows mild proptosis of the left eye. Fundoscopic examination confirms
the presence of papilledema in the left eye with normal visual acuity, while the right eye is
normal. CT of the orbits shows a diffuse tubular enlargement of the left optic nerve with kinking.
MRI shows mass-like enhancement of the enlarged left optic nerve. Which of the following is
the most appropriate next step in the management?
 Incorrect Answer ImageA.Biopsy of the tumor
 Incorrect Answer ImageB.Chemotherapy
 Incorrect Answer ImageC.Radiotherapy
 Correct Answer ImageD.Reevaluation in 6 months
 Incorrect Answer ImageE.Surgery

A 33-year-old woman, gravida 3, para 2, at 38 weeks' gestation comes to the hospital because of
contractions. She has had an uneventful prenatal course and has a history of two normal
spontaneous vaginal deliveries at term. She has no medical or surgical history. Physical
examination shows the cervix is 5 cm dilated, 100% effaced, and the vertex is at 0 station. She is
contracting every 2 minutes and the fetal heart rate is 140/min with moderate variability. Over
the next 3 hours, she progresses in labor and delivers a 3.6 kg (8 lb) male infant. Which of the
following is the most appropriate preventive management of postpartum hemorrhage for this
patient?
 Incorrect Answer ImageA.Administration of magnesium sulfate
 Correct Answer ImageB.Administration of oxytocin
 Incorrect Answer ImageC.Curettage of the uterus to remove the placenta
 Incorrect Answer ImageD.Immediate manual removal of the placenta
 Incorrect Answer ImageE.Transfusion of fresh frozen plasma

A 42-year-old woman comes to the physician because of heavy menstrual periods. Her periods
have been getting heavier over the past 3 years and they are now causing her great distress. The
patient has hypertension that is well-controlled with an ACE inhibitor. She has no other medical
problems and has never had surgery. She has smoked half a pack of cigarettes per day for the
past 20 years. Physical examination, including breast and pelvic examinations, is normal.
Laboratory studies show a hematocrit of 32%, which is down from her previous hematocrit of
38% 1 year ago. If the patient fails medical management, which of the following surgical
approaches is most likely to result in the highest patient satisfaction for addressing the problem
of heavy menstrual bleeding?
 Incorrect Answer ImageA.Diagnostic hysteroscopy
 Incorrect Answer ImageB.Diagnostic laparoscopy
 Incorrect Answer ImageC.Endometrial ablation
 Incorrect Answer ImageD.Endometrial resection
 Correct Answer ImageE.Hysterectomy

A 10-year-old boy is brought to the office by his parents because of fatigue, dizziness, and easy
tiring that started several months earlier. He had no significant medical problems before this and
has not been on any medication recently. Even now, the symptoms develop only after he engages
in strenuous physical activity. The parents noticed the problem only when the boy joined the
school football team and could not keep up with practice. On physical examination, the patient is
in no acute distress. He is 145 cm (4 ft 9 in) tall and weighs 40 kg (88 lb). His vital signs are
within normal limits and all his immunizations are up to date. Baseline laboratory studies are
unremarkable, but a chest x-ray shows a slightly enlarged right atrium and ventricle. An
electrocardiogram shows a right ventricular conduction delay in the anterior leads. Auscultation
of the heart shows a wide, fixed split of S2. In which of the following locations will a heart
murmur most likely be detected?
 Incorrect Answer ImageA.Apex of the heart
 Correct Answer ImageB.Left mid and upper sternal border
 Incorrect Answer ImageC.Lower left sternal border
 Incorrect Answer ImageD.Right sternal border
 Incorrect Answer ImageE.Upper mid-back

An inner-city family has been using a neighbor to care for their 3-year-old child while the
parents work. The neighbor is diagnosed with pulmonary tuberculosis. PPD test of the 3-year-old
is negative. Which of the following is indicated for the 3-year-old? 
 Incorrect Answer ImageA.Ethambutol chemoprophylaxis
 Correct Answer ImageB.Isoniazid chemoprophylaxis
 Incorrect Answer ImageC.No chemoprophylaxis
 Incorrect Answer ImageD.Rifapentine chemoprophylaxis
 Incorrect Answer ImageE.Streptomycin chemoprophylaxis

A previously healthy 7-year-old girl comes to the office with complaints of episodic abdominal
pain over the past several months. The pain is periumbilical and sharp but does not wake her
from sleep or interfere with play. She has no fever, joint complaints, or constipation or diarrhea.
Growth and development have been normal. The physical examination is within normal limits.
Which of the following is the most likely diagnosis? 
 Incorrect Answer ImageA.Acute appendicitis
 Incorrect Answer ImageB.Acute cholecystitis
 Incorrect Answer ImageC.Crohn disease
 Correct Answer ImageD.Functional abdominal pain
 Incorrect Answer ImageE.Irritable bowel syndrome

A 28-year-old man presents to his primary care provider for his yearly physical examination. He
currently smokes approximately one pack of cigarettes a day and is interested in quitting. He has
no significant past medical history and has no allergies. He is interested in smoking cessation
classes but would like additional help. Which of the following medications would be the most
appropriate agent for treating nicotine dependence in this patient? 
 Correct Answer ImageA.Bupropion (Wellbutrin, Zyban)
 Incorrect Answer ImageB.Buspirone (BuSpar)
 Incorrect Answer ImageC.Clonazepam (Klonopin)
 Incorrect Answer ImageD.Fluoxetine (Prozac)
 Incorrect Answer ImageE.Nefazodone (Serzone)

A 72-year-old man comes to the hospital because of right flank pain. He describes the pain as
colicky in nature and has lasted for the past 12 hours. He is found to have a 3 mm ureteral stone
impacted at the ureterovesical junction. He is given IV fluids and analgesics. His symptoms
slightly improve and he is told the stone should pass spontaneously. Later that day, he develops
shaking chills and spikes a fever. On examination, he appears ill in moderate distress. His
temperature is 40°C (104°F), blood pressure is 80/40 mm Hg, pulse is 118/min, and respirations
are 26/min. Laboratory results show an elevated white blood count and lactate. Which of the
following is the most appropriate next step in management?
 Incorrect Answer ImageA.Addition of IV antibiotics to the current therapeutic regimen
 Incorrect Answer ImageB.Crushing and extraction of the stone via cystoscopy
 Incorrect Answer ImageC.Extracorporeal shock wave lithotripsy and parenteral
antibiotics
 Incorrect Answer ImageD.Immediate insertion of a suprapubic catheter into the bladder
 Correct Answer ImageE.IV antibiotics and immediate decompression of the urinary tract
above the stone

A 57-year-old man comes to the emergency department because of excruciating pain in his right
first toe. He describes the pain as so severe that it woke him from a deep sleep. He has no
chronic medical conditions, does not take any medications, and denies any similar episodes in
the past. He admits to a few "drinking binges" over the past 2 weeks. His temperature is 38.1°C
(100.5°F), blood pressure is 130/90 mm Hg, and pulse is 80/min. Physical examination shows an
erythematous, warm, swollen, and exquisitely tender right great toe. The skin overlying the first
metatarsophalangeal joint is dark red, tense, and shiny. Synovial fluid analysis shows negatively
birefringent, needle-shaped crystals within polymorphonuclear leukocytes (PMNs). Laboratory
studies show: 
Serum
Leukocytes 16,000/mm3
Uric acid 15 mg/dL
Calcium 9 mg/dL
Which of the following is the most appropriate pharmacotherapy? 
 Incorrect Answer ImageA.Allopurinol
 Incorrect Answer ImageB.Ceftriaxone
 Correct Answer ImageC.Indomethacin
 Incorrect Answer ImageD.Probenecid
 Incorrect Answer ImageE.Sulfinpyrazone
An 11-year-old girl is brought to the physician because her tuberculin skin test was positive
during a routine screening at school. She does not have any symptoms and has been free of
serious illness in the previous year. The parents state that they had an elderly cousin from
Eastern Europe visit about 6 months earlier and they wonder if that could have been the source
of her exposure. During the time that their cousin was present several other relatives came to
visit, one of them a 16-year-old boy with lymphoma who is on immunosuppressive therapy. The
parents are concerned about whether anything should be done to protect themselves and other
family members from tuberculosis. The physician advises that a tuberculin skin test should be
performed in all contacts and appropriate treatment given to those who test positive. Which of
the following would be considered a positive tuberculin skin test?
 Correct Answer ImageA.5-mm induration in an immunosuppressed child with lymphoma
 Incorrect Answer ImageB.7-mm induration in a homeless person
 Incorrect Answer ImageC.8-mm induration in a hospital nurse
 Incorrect Answer ImageD.10-mm induration in a healthy young adult

A 43-year-old woman comes to the physician because of vaginal bleeding. She states that this
bleeding seems to occur every month or two. The bleeding is more than "spotting" but less than
her periods were before she had a hysterectomy 8 months ago. She is obese but has no other
medical problems. She had one ASCUS Pap smear 5 years ago, but all recent yearly Pap tests
have been normal. Past surgical history is significant for the supracervical hysterectomy 8
months ago. She takes no medications and has no drug allergies. Speculum examination shows a
normal vagina and cervix with no lesions or evidence of bleeding. A bimanual examination is
normal for a woman status-post hysterectomy but is limited by the patient's size. A urine
pregnancy test is negative. Which of the following is the most likely diagnosis?
 Incorrect Answer ImageA.Bacterial vaginosis
 Incorrect Answer ImageB.Cervical cancer
 Incorrect Answer ImageC.Ectopic pregnancy
 Correct Answer ImageD.Menstrual bleeding
 Incorrect Answer ImageE.Wound dehiscenc

A 23-year-old woman wants her newborn full-term infant boy circumcised. Before performing
the circumcision, administration of which of the following anesthetic agents is the correct
management?
 Correct Answer ImageA.1 ml of 1% lidocaine
 Incorrect Answer ImageB.1 ml of 1% lidocaine with epinephrine
 Incorrect Answer ImageC.10 ml of 10% lidocaine
 Incorrect Answer ImageD.10 ml of 10% lidocaine with epinephrine
 Incorrect Answer ImageE.Anesthesia is not necessary for circumcision

A 3-year-old girl is brought to the physician for a preschool health examination. The parents
deny any significant medical history except congenital aniridia with moderate photophobia. The
child is on no medication. The family history is significant for congenital aniridia in the father.
On physical examination, the child is in no acute distress. Her blood pressure is 135/85 mm Hg,
pulse is 85/min, and respirations are 18/min. Inspection of the eyes shows bilateral fixed, dilated
pupils. Palpation of the abdomen shows a firm mass in the left flank region that is nontender to
palpation and approximately 8 cm in greatest diameter. The rest of the physical examination is
normal. A complete blood count and differential shows no abnormality. Urinalysis shows
microscopic hematuria. Which of the following is the most appropriate next step in the
management?
 Incorrect Answer ImageA.Bone scan
 Incorrect Answer ImageB.Chest x-ray
 Incorrect Answer ImageC.Computerized tomography of the abdomen
 Incorrect Answer ImageD.Magnetic resonance imaging of the abdomen
 Correct Answer ImageE.Ultrasound of the abdomen

A 63-year-old man receives a subtotal gastrectomy for gastric adenocarcinoma with a Billroth 2
reconstruction. He subsequently develops a duodenal fistula due to a blowout of the duodenal
stump. For the past 10 days, his output through the drain site has been between 750 to 1,500 mL
per day of green fluid. His temperature is 37.1°C (98.8°F), pulse is 110/min, respiratory rate is
12/min, and blood pressure is 130/80 mm Hg. Physical examination is unremarkable. Laboratory
studies show:
Na+ 132 mEq/L
K+ 4.2 mEq/L
Cl -
104 mEq/L
HCO3- 15 mEq/L
pH 7.2
pCO2 35 mm Hg
pO2 96 mm Hg
Administration of which of the following is the next best step in management?
 Incorrect Answer ImageA.Ammonium chloride
 Incorrect Answer ImageB.Dextrose 5% in water
 Incorrect Answer ImageC.Normal saline
 Incorrect Answer ImageD.Potassium chloride
 Correct Answer ImageE.Sodium bicarbonate

A 70-year-old man comes to the physician because of a new-onset, progressively worsening,


itchy rash for the past 2 months. Otherwise, he reports feeling healthy. His last medical checkup
was 2 years ago. Past medical history is significant for seasonal allergies, mild gastritis, and
irritable bowel syndrome. Current medications include low dose aspirin and occasional antacids.
He quit smoking 20 years ago but occasionally drinks alcohol. On physical examination, the
patient is in no acute distress. Inspection of the skin shows multiple tan to brown greasy,
keratotic papules 3–10 mm in diameter on the face, chest, and back. Which of the following is
the most appropriate next step in the management of this patient?
 Incorrect Answer ImageA.Neurologic examination
 Incorrect Answer ImageB.Systemic antihistamine
 Correct Answer ImageC.Test stool for occult blood
 Incorrect Answer ImageD.Treatment of the lesions with liquid nitrogen
 Incorrect Answer ImageE.Tuberculin skin test

A 19-year-old male is brought to the physician by his parents because of their concern of a
tremor and worsening abnormal behavior. He has a history of behavioral disorders that began in
high school and a prior diagnosis of attention deficit hyperactivity disorder (ADHD). Over the
past year, he developed a tremor in both arms, often trips and falls while walking, occasionally
slurs his speech, and recently has started drooling. Particularly disturbing is that he has episodes
of inappropriate and uncontrollable grinning. He is concerned that his symptoms are getting
worse. His past medical history, in addition to his ADHD, is significant for a diagnosis of
"autoimmune liver disease" that was treated with corticosteroids. Ocular examination is
remarkable for a dark greenish-brown deposit around the cornea. Abdominal examination shows
marked hepatosplenomegaly while neurologic examination confirms a tremor, unstable gait, and
rigidity of the extremities. Which of the following is the most appropriate treatment for this
patient's condition?
 Correct Answer ImageA.Chelation therapy
 Incorrect Answer ImageB.Copper supplements
 Incorrect Answer ImageC.Dopaminergic agonists
 Incorrect Answer ImageD.Haloperidol
 Incorrect Answer ImageE.Plasma exchange

A 19-year-old woman is being evaluated for malaise, abdominal pain, nausea, and persistently
elevated liver function tests. Her problems have lasted for approximately 6 months and seem to
be getting progressively worse. Recent laboratory studies showed elevated AST and ALT levels,
INR of 2.5 and hypergammaglobulinemia. Evaluation including hepatitis B and C viruses,
antimitochondrial antibodies, a serum ceruloplasmin level, and an alpha-1 antitrypsin level are
negative. Antinuclear antibodies (ANA) and anti smooth muscle antibodies (SMA) are positive.
A liver biopsy is obtained and diagnosis confirmed. Based on these results, the patient is placed
on prednisone. Two weeks later she returns to the clinic complaining of difficulty sleeping,
spontaneous bouts of crying, a depressed mood, and the inability to think clearly. The prednisone
is stopped and the patient's depression improves. Which of the following is the most appropriate
therapy? 
 Correct Answer ImageA.Azathioprine
 Incorrect Answer ImageB.Liver transplant
 Incorrect Answer ImageC.Methylprednisolone
 Incorrect Answer ImageD.Sulfasalazine
 Incorrect Answer ImageE.Ursodiol

A 59-year-old man is admitted to the hospital and undergoes an uncomplicated subtotal


thyroidectomy. Shortly after the procedure, while in the post-anesthesia care unit, the patient
begins to complain of difficulty breathing. The patient's breathing becomes more labored, and he
develops stridor. On physical examination, his neck is tense and some blood is seeping from the
wound. Which of the following is the most appropriate initial step in management? 
 Incorrect Answer ImageA.Aspirate the hematoma using a large-bore needle
 Incorrect Answer ImageB.Intubate the patient with an endotracheal tube
 Correct Answer ImageC.Open the wound at the bedside to evacuate the hematoma
 Incorrect Answer ImageD.Perform an emergent tracheostomy
 Incorrect Answer ImageE.Return to the operating room to control the bleeding

A 67-year-old man with a history of hyperlipidemia and a 50 pack-year tobacco habit comes to
the emergency department complaining of substernal chest pain. The pain began one hour ago
while he was playing with his grandson. The pain is nonradiating and is associated with
shortness of breath, nausea, and vomiting. His blood pressure is 90/50 mm Hg, obtained in both
arms, pulse is 100/min, and oxygen saturation is 98% on room air. Jugular venous pulsations are
noted at 12 cm; he has clear lung fields and a right-sided S3. The rest of the examination is
unremarkable. An electrocardiogram shows 3 mm ST segment elevations in leads II, III, and
AVF. Which of the following is the most likely diagnosis? 
 Incorrect Answer ImageA.Acute myocarditis
 Incorrect Answer ImageB.Acute aortic dissection
 Incorrect Answer ImageC.Acute cholecystitis
 Correct Answer ImageD.Acute right ventricular infarction
 Incorrect Answer ImageE.Acute pulmonary embolism

An 18-year-old woman comes to the clinic saying that her period, which is normally regular, is
eight days overdue. It is the first time this has happened, and she is very worried that she may be
pregnant. She is currently sexually active with her third boyfriend in as many years. She has no
history of burning during urination or similar complaints in the past. She does, however, admit to
occasional heartburn. She drinks only on weekends, likes to watch her weight, and exercises
regularly at the gym. She smokes occasionally, but does not do drugs (although she has tried
marijuana on one or two occasions). She is an average student and lives with her grandparents.
On examination her chest is clear. No murmurs or foreign sounds are heard. Abdomen is soft
with bowel sounds and no guarding or rigidity. A urine pregnancy test is negative, and a serum
β-hCG level is also negative. She is reassured that she is not pregnant. What form of
contraception would be best in this situation? 
 Correct Answer ImageA.Condoms with nonoxynol-9 spermicide
 Incorrect Answer ImageB.Diaphragm
 Incorrect Answer ImageC.Norplant injection
 Incorrect Answer ImageD.Oral contraceptive pills
 Incorrect Answer ImageE.The rhythm method

A 27-year-old primigravid woman at 16 weeks' gestation has an increased appetite and heat
intolerance. She has gained 1 kg (2.2 lb) in the last 4 weeks. Her current weight is 66.2 kg (146
lb). Her pulse is 90/min. Uterine growth is consistent with menstrual history. Laboratory studies
show a thyroid-binding globulin slightly increased, T3 uptake decreased, and increased total
serum cholesterol. Which of the following is the most likely diagnosis?
 Incorrect Answer ImageA.Choriocarcinoma
 Incorrect Answer ImageB.Diabetes mellitus
 Incorrect Answer ImageC.Hyperthyroidism
 Incorrect Answer ImageD.Hypothyroidism
 Correct Answer ImageE.Normal pregnancy

A 9-year-old girl is brought to the emergency department with a displaced tibial fracture suffered
while playing soccer. She does not have any neurovascular complications and is placed in
skeletal traction. Later that evening, her temperature is 38.3°C (101.0°F) and she has severe pain
in the anterior leg with swelling. The pain is increased on passive plantarflexion of the ankle and
extension of the toes. Which of the following is the most appropriate next step in management?
 Incorrect Answer ImageA.Administer analgesic medication
 Incorrect Answer ImageB.Administer broad-spectrum antibiotics
 Incorrect Answer ImageC.Attempt closed reduction of the fracture under anesthesia
 Incorrect Answer ImageD.Attempt open reduction with internal fixation of the fracture
 Incorrect Answer ImageE.Increase weight on the traction apparatus
 Correct Answer ImageF.Perform a fasciotomy of the extensor compartment of the leg

A 50-year-old woman comes to the physician because of fatigue for the last several months. She
is getting increasingly forgetful and does not have much energy to carry out her daily activities.
Her appetite has decreased considerably, and she has noticed that her clothes feel tight. She feels
very sleepy during the daytime but denies insomnia at night. She feels that her hands and feet are
swollen and that she has some tingling sensation in her fingers. She also notes thinning of her
hair and occasional hair loss. Her past history is insignificant. She denies any recent stress in her
life. There is no history of bleeding from any site, and she is postmenopausal. Examination
shows an overweight middle-aged female with a pulse of 50/min and blood pressure within
normal limits. Her skin is dry and ichthyotic, and cardiac, pulmonary, and abdominal
examinations are within normal limits. Neurologic examination shows that her reflexes are
globally decreased. Treatment of this condition is most likely to include which of the following?
 Incorrect Answer ImageA.Amitriptyline
 Incorrect Answer ImageB.Continuous positive airway pressure
 Incorrect Answer ImageC.Fluoxetine
 Incorrect Answer ImageD.Iron supplementation
 Correct Answer ImageE.Levothyroxine

A 29-year-old schoolteacher has a 3-week history of facial swelling and lower extremity edema.
Over the past year, she has had recurrent episodes of arthralgias, requiring the frequent use of
nonsteroidal antiinflammatory drugs. She also has had recurrent oral ulcers unrelated to any
other symptoms. Approximately three weeks ago she began noting facial swelling around the
eyes upon awakening in the morning. One week ago she noted increased difficulty putting on her
shoes because of foot swelling. Her past medical history is significant for recurrent attacks of
renal colic involving the left kidney. Her last episode occurred 6 months ago and resolved at
home after 24 hours. Her only medications are ibuprofen taken on an as-needed basis for the
aching in her hands and knees. In her kindergarten classroom, four children have been out sick
over the last week because of upper respiratory tract infections. Her blood pressure is 158/102
mm Hg, pulse is 82/min, and respirations are 14/min. She is afebrile. She has faint areas of
erythema in both maxillary areas. An aphthous ulcer is seen on her upper buccal mucosa. Her
lungs are clear, and she has a regular heart rhythm. On abdominal examination, there are no
masses or hepatosplenomegaly. There is no costovertebral angle tenderness. Examination of the
extremities reveals moderate lower extremity edema to the mid-calf bilaterally. Her pulses are
full and symmetric. Laboratory studies show: 
Urinalysis reveals two white blood cells per high-power field, three red blood cells per high-
power field, and scattered red blood cell casts. Urine dipstick tests strongly positive for protein.
Which of the following is the most likely cause of her renal dysfunction? 
 Incorrect Answer ImageA.Acute interstitial nephritis
 Incorrect Answer ImageB.Acute tubular necrosis
 Correct Answer ImageC.Glomerulonephritis
 Incorrect Answer ImageD.Left urethral obstruction
 Incorrect Answer ImageE.Renal vasoconstriction

A 54-year old white woman with end-stage liver disease secondary hepatitis C is admitted to the
hospital with confusion and disorientation after she ate turkey on Thanksgiving dinner. The
patient was diagnosed with hepatitis C several years ago and her condition has progressively
worsened in last 10 months. She has had 2 prior episodes of spontaneous bacterial peritonitis and
one episode of upper gastrointestinal bleeding secondary to esophageal varices. The patient has
also developed renal insufficiency and is currently undergoing hemodialysis 3 times a week. The
patient was placed on liver transplant list 6 months ago. On admission to the hospital, her vitals
are temperature 37.2°C, heart rate 96/min, blood pressure 125/65 mm Hg, respirations 24. Her
skin and mucous membranes are jaundice. Lungs are clear to auscultation bilaterally. Heart
sounds are normal, without murmurs. Abdomen is soft, distended, and slightly tender on
palpation, ascites is present. Bowel sounds are normoactive in all four quadrants. Lower
extremity edema 3+ is present bilaterally. Neurological examination reveals asterixis and
hyperactive deep tendon reflexes. Laboratory studies are performed: 
White blood cell count 9,800/ml
Hemoglobin 9.0 g/dL
Hematocrit 29%
Platelets 58,000/ml
Sodium 130 mmol/L
Potassium 3.6 mmol/L
Chloride 98 mmol/L
Carbon dioxide 19 mmol/L
Blood urea nitrogen 58 mg/dL
Creatinine 3.0 mg/dL
Glucose 102 mg/dL
Calcium 7.8 mg/mL
Total protein 5.9 g/dL
Albumin 2.1 g/dL
Total bilirubin 16 g/dL
Direct bilirubin 10.5 g/dL
Alkaline phosphatase 256 U/L
AST 187 U/L
ALT 175 U/L
Ammonia 68 mcmol/L
Prothrombin time (PT) 30.3 sec
PT INT 2.9
Which of the following is the most appropriate next step in management for her current
condition? 
 Incorrect Answer ImageA.IV fluids
 Correct Answer ImageB.Lactulose
 Incorrect Answer ImageC.Nadolol
 Incorrect Answer ImageD.Neomycin
 Incorrect Answer ImageE.Potassium chloride
 Incorrect Answer ImageF.Transjugular intrahepatic portosystemic shunt (TIPS)
procedure
 Incorrect Answer ImageG.Urgent hemodialysis

The following vignette also applies to the next item. 


A 21-year-old college student is tired much of the time during the winter, sleeps excessively, and
experiences episodes of tearfulness that he cannot explain. He struggles academically at the
beginning of the winter semester and then is able to lift his grades "when there's more sun." The
patient denies any suicidal intent, ideation, or plan. Which of the following is the most likely
diagnosis? 
 Incorrect Answer ImageA.Bipolar disorder, type I
 Incorrect Answer ImageB.Cyclothymic disorder
 Incorrect Answer ImageC.Dysthymic disorder
 Correct Answer ImageD.Seasonal affective disorder
 Incorrect Answer ImageE.Social phobia

A 21-year-old college student tells the physician that over the past three winters he has been tired
much of the time, sleeps excessively, and experiences episodes of tearfulness that he cannot
explain. The student notes that while he struggles academically at the beginning of the winter
semester, his grades improve "when there's more sun." The patient denies suicidal intent,
ideation, or plan. Medical evaluation is unremarkable. Which of the following is the most
appropriate treatment for this patient's condition at this time?
 Incorrect Answer ImageA.Dextroamphetamine
 Incorrect Answer ImageB.Electroconvulsive therapy
 Incorrect Answer ImageC.Haloperidol
 Incorrect Answer ImageD.Methylphenidate
 Correct Answer ImageE.Phototherapy
A 53-year-old lawyer comes to the physician because she has not felt "up to par" over the past 2
months. She is married, has two grown sons, and has a successful career. In the past couple of
weeks, she has stopped taking care of her appearance and has frequently called in sick to work,
when she actually has been having difficulty getting out of bed. She states that she has lost her
appetite recently and her interest in sex with her husband has decreased considerably. She
recently told her husband that at times she wonders whether she should go on living. She denies
any history of drug or alcohol abuse, and a complete physical examination 3 months earlier
showed her to be in good health. Which of the following is the mostly likely diagnosis? 
 Incorrect Answer ImageA.Bipolar I disorder
 Incorrect Answer ImageB.Generalized anxiety disorder
 Correct Answer ImageC.Major depressive disorder
 Incorrect Answer ImageD.Panic disorder
 Incorrect Answer ImageE.Schizophrenia

A 7-year-old boy who recently emigrated from Haiti is brought to the physician because he has
had recurring episodes of high fever every third day for the past two weeks. He is also
complaining of headache, generalized achiness, and abdominal pain. His temperature is 40.0°C
(104.0°F), blood pressure is 90/50 mm Hg, and pulse is 140/min. He appears ill and jaundiced.
He has no nuchal rigidity. He has a large, tender spleen. Laboratory studies show:
Hemoglobin 7.5 g/dL
Hematocrit 22%
Leukocyte count 3,500/mm3
Segmented neutrophils 35%
Lymphocytes 20%
Monocytes 45%
Platelet count 350,000/mm3
Total bilirubin 10.0 mg/dL
Direct bilirubin 1.5 mg/dL
Which of the following is the most appropriate next step in diagnosis? 
 Incorrect Answer ImageA.Bacterial blood culture
 Correct Answer ImageB.Blood smear
 Incorrect Answer ImageC.Bone marrow aspiration
 Incorrect Answer ImageD.Liver biopsy
 Incorrect Answer ImageE.Viral serology

A 74-year-old woman comes to the physician because of pressure during urination that has been
worsening over the last couple of years. There is no associated dysuria, fevers, or chills. She has
been to many physicians who have all diagnosed her with urinary tract infections; however, the
prescribed antibiotics do not seem to "clear things up." Lately, she also has been having
discomfort with sexual intercourse. She denies any renal stones, difficulty with ambulation,
trauma to the area, or vaginal surgery. She has five grown children, all born vaginally. On
physical examination, her abdomen is soft and nontender, without organomegaly or palpable
masses. Pelvic examination shows atrophied and dry vaginal mucosa; the vaginal vault is
completely occupied by a smooth round mass that protrudes beyond the introitus. The mass is
nontender, soft, and easily reduced anteriorly so that the vaginal vault may be completely
examined. With the mass reduced, the urethral meatus is grossly normal in appearance. The
cervix is without lesions or tenderness. No adnexal masses are appreciated. With the mass
protruding, the patient is asked to cough, which causes further protrusion and expulsion of urine
from the meatus. When the mass is manually reduced and the patient is asked to cough, there is
no loss of urine. Urinalysis is normal. Which of the following is the most likely diagnosis?
 Correct Answer ImageA.Cystocele
 Incorrect Answer ImageB.Paraurethral cyst
 Incorrect Answer ImageC.Sarcoma botryoides of the vagina
 Incorrect Answer ImageD.Transitional cell carcinoma of the bladder
 Incorrect Answer ImageE.Urethral prolapse

A 19-year-old college student comes to the university health clinic because of a 2 week history
of shortness of breath. Two other students in his college dormitory have similar symptoms. He
finds that his shortness of breath has limited his ability to play on the lacrosse team. He has had a
nonproductive cough that persists throughout the day and keeps him awake at night. He felt
feverish over the last 48 hours. He denies smoking cigarettes and admits to smoking marijuana
occasionally on weekends. He has been monogamous with his girlfriend for the last two years
and reports regular condom use. His temperature is 37.8°C (100.0°F), blood pressure is 112/68
mm Hg, pulse is 67/min, and respirations are 22/min. His lungs have scattered rhonchi
throughout both lung fields with excellent air movement and no wheezing. He has a regular heart
rhythm. The remainder of his physical examination is normal. Laboratory studies show a
leukocyte count of 5400/mm3, hemoglobin of 14 g/dL, a hematocrit of 43%, and a platelet count
of 149,000/mm3. Serum electrolytes and liver function tests are normal. A chest x-ray shows a
faint bilateral interstitial infiltrate. There is no adenopathy or pleural effusion. Which of the
following findings would be most consistent with this patient's diagnosis? 
 Correct Answer ImageA.Autoimmune hemolysis
 Incorrect Answer ImageB.Autoimmune hepatitis
 Incorrect Answer ImageC.Glomerulonephritis
 Incorrect Answer ImageD.Pancreatic insufficiency
 Incorrect Answer ImageE.Serum sickness

A 67-year-old man with a 110-pack/year history of smoking comes to the physician because of
chest pain. For the past few months, he has been getting chest "pressure" localized to the
substernal region, radiating to the left arm on occasion. The pain occurs when walking up a hill
and is relieved by rest. His exercise tolerance is moderate, and he experiences dyspnea on
exertion after a few blocks of walking. He has no history of coronary disease, but his family
history is positive for his father having a myocardial infarction at age 52. He is quite concerned
about this pain given his family history. He is pain-free in the office. Physical examination
shows no chest wall tenderness to palpation. A carotid bruit is heard and he has decreased pulses
at the dorsalis pedis artery. In addition to ascertaining his other coronary risk factors, which of
the following is the most appropriate diagnostic intervention?
 Incorrect Answer ImageA.Obtain a resting electrocardiogram
 Incorrect Answer ImageB.Schedule the patient for a cardiac echocardiogram
 Correct Answer ImageC.Schedule the patient for an exercise treadmill stress test
 Incorrect Answer ImageD.Schedule the patient for immediate coronary angiography
 Incorrect Answer ImageE.Schedule the patient for non-urgent coronary angiography

A 64-year-old man comes to the emergency department because he has been "coughing up
blood" for the past eight hours. He has no prior history of lung disease, has never smoked, and
has no history of cardiac disease. His past medical history is significant for migraine headaches,
recurrent sinus infections requiring drainage, cholecystectomy, and appendectomy. His blood
pressure is 146/82 mm Hg, pulse is 84/min, respirations are 28/min, and his temperature is
37.9°C (100.2°F). He has tenderness over the right frontal sinus. There are scattered rhonchi
throughout both lung fields. His leukocyte count is 13,400/mm3, hemoglobin is 12 g/dL,
hematocrit is 35%, and platelet count is 392,000/mm3. His sodium is 142 mEq/L, potassium is
4.8 mEq/L, chloride is 110 mEq/L, bicarbonate is 20 mEq/L, BUN is 23 mg/dL, creatinine is 2.1
mg/dL, and glucose is 112 mg/dL. Urinalysis reveals two white blood cells per high power field,
trace protein, and countless red blood cells. Which of the following is the most likely diagnosis? 
 Incorrect Answer ImageA.Goodpasture syndrome
 Correct Answer ImageB.Wegener granulomatosis
 Incorrect Answer ImageC.Acute tubular necrosis
 Incorrect Answer ImageD.Membranous glomerulonephritis
 Incorrect Answer ImageE.IgA nephropathy

A 55-year-old woman with type 2 diabetes mellitus comes to her physician because of a 1-week
history of burning pain in her feet at night. She reports nonadherence to the diet and exercise that
was recommended by her physician. She has not been checking her blood sugars recently. She
denies numbness, weakness, and changes in vision. Her past medical history is significant for a
10-year history of type 2 diabetes mellitus. Current medications include glyburide 5 mg daily.
Her temperature is 37.1°C (98.8°F), pulse is 80/min, respirations are 12/min, and blood pressure
is 150/100 mm Hg. Fundoscopic examination shows microaneurysms. Neurological examination
shows a loss of ankle reflexes and decreased patellar reflexes. The rest of the physical
examination, including cardiac and abdominal examinations, is normal. Urinalysis shows
moderate proteinuria. 24-hour urinary protein excretion is 500 mg. Which of the following is the
most appropriate pharmacotherapy to reduce morbidity?
 Incorrect Answer ImageA.Atenolol
 Incorrect Answer ImageB.Dietary protein restriction
 Incorrect Answer ImageC.Diltiazem
 Incorrect Answer ImageD.Hydrochlorothiazide
 Correct Answer ImageE.Lisinopril

A 58-year-old man comes to the emergency department with a massive upper gastrointestinal
bleed (UGIB). He denies any alcohol abuse but admits to being a longtime smoker, and he has
recently been taking large amounts of ibuprofen for back pain. He is stabilized, and an upper
endoscopy shows an ulcer in the second portion of the duodenum with a visible vessel but no
active bleeding. Which anatomic structure is the most likely cause of his bleeding? 
 Correct Answer ImageA.Gastroduodenal artery
 Incorrect Answer ImageB.Pancreatic parenchyma
 Incorrect Answer ImageC.Portal vein
 Incorrect Answer ImageD.Splenic artery
 Incorrect Answer ImageE.Superior mesenteric artery (SMA)

A 69-year-old smoker with COPD, stable exertional angina, and hypertension comes to the
physician for a periodic health maintenance examination. Over the last week, he has had an
increasing productive cough of greenish sputum. Over the last 6 months he has required two
courses of antibiotics for bronchitis. His medications include verapamil, aspirin, captopril, an
albuterol inhaler, and hydrochlorothiazide. After completing a history and physical examination,
which of the following is the most appropriate long-term management? 
 Incorrect Answer ImageA.Annual influenza vaccination each summer
 Correct Answer ImageB.Annual influenza vaccination in October
 Incorrect Answer ImageC.Annual influenza vaccination in October and annual chest x-
ray
 Incorrect Answer ImageD.Annual influenza vaccination and annual sputum cytology
 Incorrect Answer ImageE.Annual influenza vaccination and pneumococcal vaccination

 69-year-old man undergoes an open sigmoid colectomy for colon cancer. Nine days
postoperatively, the lower pole of the incision begins to drain approximately 50-75 mL per day
of a brown fluid that looks and smells like feces. A CT scan shows a fistula between the colonic
anastomosis and the wound. There is no abscess or free fluid in the abdomen. The patient is
afebrile with a soft and nontender abdomen, and he has been tolerating oral food intake without
any problems. He has had a few bowel movements and is passing gas. The rest of the wound is
healing well. Which of the following is the next best step in management? 
 Incorrect Answer ImageA.Broad-spectrum antibiotics
 Incorrect Answer ImageB.Diverting ileostomy
 Incorrect Answer ImageC.Laparotomy and redo the anastomosis
 Incorrect Answer ImageD.NPO and parenteral nutrition(TPN)
 Correct Answer ImageE.Observation

A 50-year-old man comes to the emergency department because of chest pain. He is given two
sublingual nitroglycerin tabs, after which the chest pain resolves. Although there are no
electrocardiographic abnormalities and cardiac enzymes are within normal limits, the patient has
significant risk factors for coronary artery disease including diabetes, hypertension, and a 15
pack-year history of tobacco use. He is admitted for further evaluation and is planned to undergo
a pharmacologic nuclear stress test the next morning. The hospital routinely uses dipyridamole
nuclear perfusion studies to evaluate for coronary artery disease. Before the test, it is important
that this patient avoid ingesting which of the following?
 Incorrect Answer ImageA.Atenolol
 Correct Answer ImageB.Caffeine
 Incorrect Answer ImageC.Insulin
 Incorrect Answer ImageD.Nifedipine
 Incorrect Answer ImageE.Nitrates

A 17-year-old girl comes to the university health center with her boyfriend because of a
malodorous vaginal discharge of 5 days' duration. She also complains of vulvar itching and
discomfort during sexual intercourse. She has been in a steady sexual relationship with a single
male partner for over 6 months and uses oral contraceptive pills. Her partner does not use a
condom. He has no complaints and did not notice any discomfort or discharge. Her medical
history is otherwise insignificant and she takes no medication except the above. On physical
examination, there is a thin, bubbly, yellowish-green discharge in the vagina and vulvovaginal
erythema. The cervix is diffusely erythematous with pinpoint hemorrhages. There is no lower
abdominal tenderness with bimanual palpation. Application of 10% potassium hydroxide to a
vaginal swab sample releases a fishy odor. A saline wet mount shows the presence of
characteristic organisms and numerous polymorphonuclear neutrophils. Which of the following
is the most appropriate management? 
 Correct Answer ImageA.Both she and her sexual partner should be treated with a single
oral dose of 2 g of metronidazole
 Incorrect Answer ImageB.Both she and her sexual partner should be treated with topical
0.75% metronidazole cream until symptoms resolve
 Incorrect Answer ImageC.Both she and her sexual partner should be treated with a 10-
day oral course of metronidazole
 Incorrect Answer ImageD.Obtain parental consent before treatment is initiated
 Incorrect Answer ImageE.Explain that the sexual partner does not require treatment
 Incorrect Answer ImageF.Explain that this is a self-limited condition that does not
require treatment

A newborn infant develops respiratory distress within a few hours after birth. Physical
examination shows a very scaphoid abdomen, a prominent chest that is larger on the left than on
the right, and absent breath sounds on the left. X-rays demonstrate the presence of stomach and
bowel in the left chest. Pulse oximetry is monitored at the left foot and the right hand. When the
devices were first installed, they showed a saturation of 96% on both sides, but a couple of hours
later the readings show 93% on the right hand and 82% on the left foot. The infant has been
intubated already and is receiving oxygen. Which of the following would be the most helpful
additional therapy at this time? 
 Incorrect Answer ImageA.Chest tube placement on the left side
 Incorrect Answer ImageB.Indomethacin
 Correct Answer ImageC.Inhaled nitric oxide
 Incorrect Answer ImageD.Surgical closure of the ductus arteriosus
 Incorrect Answer ImageE.Surgical correction of the coarctation
A 32-year-old woman, gravida 5, para 4, at 29 weeks’ gestation is brought to the emergency
department after suddenly feeling faint and passing out while attending an arts and crafts class.
She is arousable and reports abdominal pain which is constant and does not feel like the labor
pains she experienced during her previous deliveries. She has no vaginal bleeding or discharge.
She has had routine prenatal care and her pregnancy has been uncomplicated up to this point.
She has never been on birth control pills and all her previous pregnancies have ended in normal
vaginal deliveries. Her blood pressure is 85/65 mm Hg and her heart rate is 120/min with a faint
pulse. A focused abdominal sonogram for trauma (FAST) scan shows a large amount of intra-
abdominal fluid. Which of the following is the most likely diagnosis? 
 Incorrect Answer ImageA.Ruptured abdominal aortic aneurysm
 Incorrect Answer ImageB.Ruptured ectopic pregnancy
 Incorrect Answer ImageC.Ruptured hepatic adenoma
 Correct Answer ImageD.Ruptured splenic artery aneurysm
 Incorrect Answer ImageE.Ruptured uterus

A 26-year-old woman, gravida 2, para 1, at 8 weeks' gestation comes to the physician for an
initial prenatal visit. She states that she has had some mild nausea but otherwise is feeling well.
Her past obstetric history is significant for a normal vaginal delivery following an uncomplicated
pregnancy 3 years ago. Her past medical history is significant for human immunodeficiency
virus (HIV) positivity, determined a few months ago. Her CD4 count is 500/mm3 and her HIV
RNA viral load is 1,000 copies/ml. She has never had surgery, takes no medications, and has no
known drug allergies. Her physical examination is normal for a woman at 8 weeks' gestation.
She is very concerned about how pregnancy will affect her disease and vice versa. Which of the
following should be included in the counseling of this patient? 
 Incorrect Answer ImageA.Birth defects are markedly increased in offspring of HIV-
positive women
 Incorrect Answer ImageB.Combination antiretroviral therapy may decrease preterm birth
rate
 Correct Answer ImageC.Drug therapy can significantly reduce maternal-fetal HIV
transmission
 Incorrect Answer ImageD.Pregnancy leads to rapid progression of HIV disease
 Incorrect Answer ImageE.Zidovudine leads to significant pregnancy complications

As a part of a study to identify risk factors associated with breast cancer, 500 women with breast
cancer and 500 women of the same age and ethnicity in which no cancer is detected upon
screening are asked detailed questions about past dietary patterns. Once the interviews are
complete, the resulting data are analyzed to examine past exposure to a high-fat diet in relation to
a present diagnosis of breast cancer. This study uses which of the following study designs? 
 Correct Answer ImageA.Case-control study
 Incorrect Answer ImageB.Case report
 Incorrect Answer ImageC.Case series report
 Incorrect Answer ImageD.Clinical trial
 Incorrect Answer ImageE.Cohort study
A 74-year-old woman comes to the emergency department complaining of very severe
abdominal pain, which began abruptly 8 hours ago. She describes the pain as "the worst I've ever
had." On questioning, she is unable to give a precise location but indicates that her entire mid-
abdomen is extremely painful. She has been followed for the past 10 years for symptoms of
congestive heart failure after she had an anterior wall myocardial infarction. She has remained
relatively well controlled with only occasional dyspnea on exertion. Her medications include
captopril, furosemide, digoxin, isosorbide dinitrate, and aspirin. She has not had any prior
surgery. On physical examination, she appears extremely uncomfortable. Her temperature is
38.9°C (101.9°F), blood pressure is 174/102 mm Hg, and pulse is 118/min and irregularly
irregular. On cardiac examination, there is a II/VI holosystolic murmur heard best at the apex
and radiating to the axilla. She has an irregularly irregular S1 and S2, and scattered bibasilar
rales. An abdominal examination reveals mild distention and no hepatosplenomegaly. The
abdomen is diffusely soft but very tender to palpation. A rectal examination reveals brown,
guaiac-positive stool. She has no audible bowel sounds. Which of the following is the most
likely diagnosis? 
 Incorrect Answer ImageA.Diverticulitis
 Incorrect Answer ImageB.Ischemic colitis
 Correct Answer ImageC.Mesenteric ischemia
 Incorrect Answer ImageD.Pancreatitis
 Incorrect Answer ImageE.Small bowel obstruction

A 54-year-old man presents to his physician with 2 days of cough and fever. His past medical
history is significant for mild asthma and peripheral vascular disease (PVD). He takes albuterol
metered dose inhalers as needed and has never been intubated for his asthma. His PVD manifests
as calf claudication and has been stable over the past few years. His other medications include
atenolol, lisinopril, and quinine. He has no drug allergies. He denies rigors, chills, nausea,
vomiting, or any pleuritic chest pain. On physical examination, he appears well with an
occasional cough. His temperature is 38.0°C (100.4°F), blood pressure is 150/84 mm Hg, pulse
is 90/min and regular, and respirations are 22/min and somewhat labored. His lungs have
bibasilar crackles and a questionable area of increased dullness near the right base. The rest of
the examination is unremarkable. Which of the following is the most appropriate intervention at
this time? 
 Incorrect Answer ImageA.No intervention is indicated
 Incorrect Answer ImageB.Prescribe penicillin and send the patient home
 Correct Answer ImageC.Obtain a chest radiograph
 Incorrect Answer ImageD.Obtain an arterial blood gas
 Incorrect Answer ImageE.Refer the patient to the hospital for admission

A 1-month-old male infant is brought to the emergency department by his mother, who states
that he has been having forceful vomiting for the past several days. She states that he vomits
every time she feeds him, and the situation seems to be getting worse, although he does not seem
to be in pain. She describes the vomitus as non-bilious, and he has had normal stools with no
blood in them. On physical examination, the infant appears to be mildly dehydrated, his
abdomen is soft, and there is a palpable 2 cm, firm, mobile mass in the right upper quadrant.
Which of the following is the most likely diagnosis?
 Incorrect Answer ImageA.Duodenal atresia
 Incorrect Answer ImageB.Hirschsprung disease
 Incorrect Answer ImageC.Intussusception
 Incorrect Answer ImageD.Midgut volvulus
 Correct Answer ImageE.Pyloric stenosis

A 16-year-old boy is brought into the emergency department by a friend. The teenager is semi-
comatose, with a pulse of 60/min and respirations of 6-8/min. His pupils are constricted. The
friend notes that the boy “took something.” Which of the following will most likely be positive
on a urine toxicology screen?
 Incorrect Answer ImageA.Amphetamine intoxication
 Incorrect Answer ImageB.Cocaine intoxication
 Incorrect Answer ImageC.Ethanol intoxication
 Correct Answer ImageD.Opioid intoxication
 Incorrect Answer ImageE.Tricyclic antidepressant intoxication

A 61-year-old man with an 80-pack-year smoking history comes to the physician because of
increasing dyspnea on exertion. He also has developed a productive cough, which has persisted
over most of the past 2 years. He denies any other medical problems and has never been
hospitalized for evaluation of his symptoms. On physical examination, his blood pressure is
132/74 mm Hg, pulse is 70/min, and respirations are 24/min. On lung examination, there are
loud expiratory rhonchi, with a prolonged expiratory phase in both lung fields. There is an
increased anteroposterior chest diameter. Heart sounds are regular but distant. A chest x-ray film
shows hyperinflation of both lung fields and a normal-sized heart. Which of the following would
most likely be expected on pulmonary function tests?
 Correct Answer ImageA.Decreased forced expiratory volume in 1 second (FEV1) to
forced vital capacity (FVC) ratio
 Incorrect Answer ImageB.Decreased residual volume
 Incorrect Answer ImageC.Decreased total lung capacity
 Incorrect Answer ImageD.Increased FEV1
 Incorrect Answer ImageE.Increased FVC

In the United States, approximately 60% of people who commit suicide have an affective
disorder. Approximately 25% of all suicides are alcohol-related. What is the probability that a
randomly selected suicide is neither alcohol-related nor in a person with an affective disorder?
 Incorrect Answer ImageA.5%
 Incorrect Answer ImageB.10%
 Incorrect Answer ImageC.15%
 Incorrect Answer ImageD.25%
 Correct Answer ImageE.30%
 Incorrect Answer ImageF.40%
 Incorrect Answer ImageG.45%
 Incorrect Answer ImageH.60%
 Incorrect Answer ImageI.75%

A 29-year-old woman, gravida 1, para 0, at 38 weeks' gestation comes to the labor and delivery
ward because of decreased fetal movement. She states that she has felt her fetus moving very
little over the past 2 days. She has no contractions, bleeding from the vagina, or leakage of fluid.
This pregnancy has been uncomplicated thus far. Her temperature is 37.0°C (98.6°F), blood
pressure is 158/94 mm Hg, pulse is 90/min, and respirations are 12/min. Physical examination
shows fundal height of 38 cm; the cervix is long, closed, posterior, and firm. The fetal heart rate
is 140/min with moderate variability. Urine protein is 3+ on dipstick evaluation. Laboratory
studies show:
Leukocytes 12,000/mm3
Hematocrit 39%
Platelets 98,000/mm3
Which of the following is the most likely diagnosis?
 Incorrect Answer ImageA.Gestational thrombocytopenia
 Incorrect Answer ImageB.Human immunodeficiency virus (HIV) infection
 Incorrect Answer ImageC.Immune thrombocytopenia purpura
 Incorrect Answer ImageD.May-Hegglin anomaly
 Correct Answer ImageE.Preeclampsia

A 17-year-old boy is the victim of a motorcycle crash, and after exploratory laparotomy, he
undergoes a splenectomy. Surgical drains are left in the operative area and are brought out
through a separate stab wound. Two days postoperatively, the drains are removed. On the fifth
postoperative day, when he is at home and on a regular diet, he develops persistent hiccups. Four
days after that, he notices fever. He had been instructed to return to the clinic on the fifteenth
postoperative day to receive a pneumococcal polysaccharide vaccine (PPSV23), but he shows up
on the twelfth day because his fever and hiccups have persisted. At that time he has a
temperature of 38.9°C (102.0°F), a leukocyte count of 14,000/mm3, and a left pleural effusion on
chest x-ray. Urinalysis is normal. The wound is healing well with no signs of infection. Which of
the following is the most likely diagnosis?
 Incorrect Answer ImageA.Atelectasis
 Correct Answer ImageB.Left subphrenic abscess
 Incorrect Answer ImageC.Overlooked duodenal injury
 Incorrect Answer ImageD.Postsplenectomy sepsis
 Incorrect Answer ImageE.Surgical injury to the phrenic nerve

A 5-week-old baby boy has a 5-day history of protracted, projectile, nonbilious vomiting.
Physical examination shows marked dehydration, a scaphoid abdomen, visible gastric peristaltic
waves, and a small, ovoid, 2-cm mass palpable in the right upper quadrant. Which of the
following is the most appropriate choice of intravenous fluid to rehydrate this patient? 
 Incorrect Answer ImageA.D5 1/4 normal saline, with 5 mEq of KCl per liter
 Correct Answer ImageB.D5 1/2 normal saline with 30 mEq of KCl per liter
 Incorrect Answer ImageC.D5W
 Incorrect Answer ImageD.D5W with 44 mEq of sodium bicarbonate per liter
 Incorrect Answer ImageE.Ringer's lactate

An 8-month-old boy is brought to the emergency department because of diarrhea. According to


his mother, the infant was well until last evening, but overnight had three loose, watery stools. In
the last 6 hours, there have been three more stools and he has vomited once. He has no fever and
appears alert and thirsty. His pulse is 105/min, respirations are 30/min, and blood pressure is
90/62 mm Hg. The anterior fontanelle is not sunken and the skin turgor is preserved. Capillary
refill is 1-2 seconds. Abdominal examination reveals no distension and good bowel sounds. He
has a wet diaper, and there is no rash in the diaper region. Laboratory studies show:
Hemoglobin 11.5 g/dL
Leukocyte count 14 x 109/L
Differential: polymorphs 28%, lymphocytes 62%, monocytes 8%
Platelets 480 x 109/L
Serum sodium 142 mEq/L
Serum potassium 4.3 mEq/L
Serum chloride 106 mEq/L
Serum bicarbonate 23 mEq/L
Blood urea nitrogen 8 mg/dL
Serum creatinine 0.3 mg/dL
Which of the following is the most appropriate next step in management?
 Incorrect Answer ImageA.Admit for intravenous hydration
 Incorrect Answer ImageB.Provide intravenous rehydration and then discharge home
 Incorrect Answer ImageC.Provide oral kaolin-pectin and discharge home
 Incorrect Answer ImageD.Recommend oral rehydration and co-trimoxazole at home
 Correct Answer ImageE.Recommend oral rehydration and observation at home

A 44-year-old man comes to the clinic complaining of finger stiffness in both of his hands. You
know the patient well, as you have spent many hours to assisting him with management of his
diabetes and tobacco and alcohol cessation. Unfortunately, over the years the patient has had
poorly controlled diabetes and continues to drink and smoke on a regular basis. Today the patient
complains of finger stiffness that is worse in his right hand though present in both hands, and a
feeling of "thickness" in his palms. Although there is no pain in his hands, his fourth and fifth
fingers are mildly flexed, requiring the patient to rub his palm and fingers to straighten them. An
examination of his hands shows some small, mildly tender nodules over the flexor tendon, a
mildly contracted hand at rest, and reduced flexibility of the metacarpal and proximal
interphalangeal joints of the fourth and fifth digits. No swelling, pain, or tenderness of the joints
is noted. Radiograph of the hands is unrevealing. The patient wants to know if this condition is
related to his job as a jackhammer operator, as he plans to sue his employer and collect state
disability payments. Which of the following is the most likely underlying cause of this patient's
condition? 
 Incorrect Answer ImageA.Chondrocalcinosis of the flexor tendons
 Correct Answer ImageB.Fibrosis in and around the palmar fascia
 Incorrect Answer ImageC.Median nerve entrapment from repetitive stress
 Incorrect Answer ImageD.Neuropathic osteoarthropathy related to diabetes
 Incorrect Answer ImageE.Repetitive stress induced tenosynovitis

A 64-year-old man with advanced pulmonary tuberculosis is undergoing a laparoscopic hernia


repair with CO2 insufflation. He is under general anesthesia with an endotracheal tube and
mechanical ventilation. One hour into the procedure, the anesthesiologist notes high airway
pressures which progressively increase. Simultaneously, his blood pressure drops to 60/30 mm
Hg and oxygen saturation declines to 85%. End-tidal CO2 (ETCO2) also begins to increase.
Which of the following is the most likely diagnosis? 
 Incorrect Answer ImageA.CO2 embolus
 Incorrect Answer ImageB.Decreased lung compliance due to tuberculosis and
pneumoperitoneum
 Incorrect Answer ImageC.Insufficient sedation
 Incorrect Answer ImageD.Myocardial infarction
 Correct Answer ImageE.Tension pneumothorax

A 34-year-old man comes to the physician complaining of the recent onset of a facial rash. The
rash is largely asymptomatic, but he dislikes the appearance of the lesions. He admits to drinking
excessive amounts of alcohol and using intravenous drugs. He is not taking prescription
medications and has no history of atopy. On examination, there is a scaly, erythematous macular
rash in the nasolabial creases and above the eyebrows. There is also excessive scaling and
erythema in the centrofacial area. A KOH evaluation of the area is negative. The patient admits
to a recent total body eruption, which was accompanied by low-grade fever, dry cough, and
malaise. These symptoms have since cleared. He is diagnosed with severe seborrheic dermatitis
on his face and appropriate therapy is prescribed. Which of the following is the most appropriate
next step in diagnosis? 
 Incorrect Answer ImageA.Lyme titer
 Correct Answer ImageB.HIV test
 Incorrect Answer ImageC.Total serum IgE
 Incorrect Answer ImageD.Rapid plasma reagin
 Incorrect Answer ImageE.Anti-Ro antibodies

A 64-year-old male is admitted to the intensive care unit with sepsis due to pneumonia. He is
intubated on admission due to respiratory failure. On physical examination, his temperature is
38.5°C (101.3°F), pulse is 110/min, and blood pressure is 80/60 mm Hg. Chest examination
shows bibasilar crackles on auscultation and sinus tachycardia with no rubs, gallops or murmurs.
There are no signs of pitting edema in his extremities. He is started on intravenous antibiotics.
Four hours later, the nurse notes that his urine output is only 10 cc’s. A Swan-Ganz catheter is
inserted and the following readings taken:
Right atrial pressure 3 mm Hg
Pulmonary artery pressure 15/6 mm Hg
Pulmonary Capillary Wedge Pressure (PCWP) 5 mm Hg
Cardiac Output 7.1 Liters/min
Cardiac Index 3.2
Based on this patient's clinical findings and catheter readings, which of the following is the most
appropriate next step in management?
 Incorrect Answer ImageA.Administer diuretic therapy
 Incorrect Answer ImageB.Administer dobutamine
 Incorrect Answer ImageC.Administer vasopressin
 Correct Answer ImageD.Begin intravenous fluids
 Incorrect Answer ImageE.No further intervention

A 75-year-old man comes to the physician for evaluation of tingling in both feet, which has
gradually worsened over the past few months. His past medical history is significant for
hypertension and osteoarthritis. He underwent a partial gastrectomy 15 years ago for a perforated
ulcer. His medications are hydrochlorothiazide and acetaminophen. His blood pressure is 136/80
mm Hg. He is alert and oriented, and neurological examination shows decreased vibration and
proprioception sense to the level of the ankles bilaterally. He has a positive Romberg sign. The
rest of the neurological examination is normal. Laboratory studies show: 
Hematocrit 39%
Hemoglobin 12 g/dL
Mean corpuscular volume 110 µm3
Platelet count 200,000/mm3
Glucose 100 mg/dL
Thyroid-stimulating hormone 1.0 µU/mL
Creatinine 1.0 mg/dL
Which of the following is the most likely explanation for these findings? 
 Incorrect Answer ImageA.Chronic inflammatory demyelinating polyneuropathy
 Incorrect Answer ImageB.Folate deficiency
 Incorrect Answer ImageC.Hypothyroidism
 Incorrect Answer ImageD.Meralgia paresthetica
 Correct Answer ImageE.Vitamin B12 deficiency

A 22-year-old woman comes to the physician for a second opinion about her "hideous nose." She
has kept her nose covered by a thick scarf until walking into the privacy of the examination
room. She begins to cry while she removes the scarf and explains that she is so ashamed of her
nose that for the past year she has been unable to attend social events and repeatedly turns down
requests by her friends to go out. She admits that she believes this would all get better if she got
a "nose job" and is seeking a referral to a well-known plastic surgeon. Examination shows the
patient’s nose is typical with no obvious deformity or asymmetry. The patient, who otherwise
shows intact reality testing, notes that she has gone to six other physicians who have all said that
there is nothing they can do for her. Which of the following is the most likely diagnosis?
 Incorrect Answer ImageA.Adjustment disorder with anxiety
 Correct Answer ImageB.Body dysmorphic disorder
 Incorrect Answer ImageC.Conversion disorder
 Incorrect Answer ImageD.Illness anxiety disorder
 Incorrect Answer ImageE.Major depressive disorder
 Incorrect Answer ImageF.Schizophrenia

A 36-year-old woman comes to the physician because of difficulty with swallowing. She also
has generalized weakness, dyspnea on exertion, mild arthralgias, and morning muscle stiffness.
The patient also reports episodes of skin discoloration in her hands which are usually triggered
by exposure to cold or emotional stress. The patient's past medical history is unremarkable. She
does not take any medications and does not have any allergies. She works as a hairstylist. Her
temperature is 36.8°C (98.2°F), pulse is 82/min, respirations are 18/min, and blood pressure is
138/78 mm Hg. Physical examination shows skin thickening and tightening of the hands and
telangiectasias on the face. The rest of the physical examination is unremarkable. Laboratory
studies show erythrocyte sedimentation rate (ESR) of 45 mm/h and the presence of anti-nuclear
and anti-topoisomerase-I antibodies. Which of the following parts of the gastrointestinal tract is
most commonly affected by this disease?
 Incorrect Answer ImageA.Duodenum
 Correct Answer ImageB.Esophagus
 Incorrect Answer ImageC.Ileum
 Incorrect Answer ImageD.Jejunum
 Incorrect Answer ImageE.Large intestine

A 29-year-old woman with a 17-year history of type 1 diabetes comes to the emergency
department complaining of a 4-day history of being "very thirsty and urinating much more than
usual." She admits that she has not taken any insulin lately as she ran out of syringes about a
week ago. She denies any fever, chills, or cough. She has had two bouts of emesis while in the
emergency department. Vital signs: temperature 37.0 C (98.6 F), blood pressure 89/47 mm Hg,
pulse 112/min, and respirations 29/min. Which of the following is the most appropriate sequence
of laboratory tests that should be used to confirm the diagnosis? 
 Incorrect Answer ImageA.A chemistry seven-panel (Na+, K+, Cl-, CO2, BUN, creatinine,
glucose) and an arterial blood gas
 Incorrect Answer ImageB.A chemistry seven-panel (Na+, K+, Cl-, CO2, BUN, creatinine,
glucose) and a serum ketone level
 Correct Answer ImageC.A chemistry seven-panel (Na+, K+, Cl-, CO2, BUN, creatinine,
glucose) and a urine analysis
 Incorrect Answer ImageD.A CBC with differential and a reticulocyte count
 Incorrect Answer ImageE.A urine analysis and a serum ketone level

A 59-year-old African-American woman comes to the physician because of "dirty" armpits and
neck. She states that she has had a slowly progressing discoloration of her axillae and the skin
around her neck for the past 2–3 years. She has a history of hypertension, which is well
controlled with a thiazide diuretic. She has felt easily fatigued recently and has lost about 11.3 kg
(25 lb) from her normal weight of 63.5 kg (140 lb). She is not diabetic. On examination, she has
velvety hyperpigmentation of both axillae and the skin around her neck. She appears frail and
gaunt. The physician is very much concerned about the possibility of internal malignancy. Given
her cutaneous findings, the most likely location of her malignancy is which of the following?
 Incorrect Answer ImageA.Bone marrow
 Incorrect Answer ImageB.Breast
 Incorrect Answer ImageC.Lung
 Incorrect Answer ImageD.Lymph nodes
 Correct Answer ImageE.Stomach

A morbidly obese 70-year-old woman with a 15-year history of non-insulin dependent diabetes,
hypertension, and hypercholesterolemia has a lesion between the toe webs of her right foot. It
has been present for at least several weeks and started as a small, painless area that she thought
would resolve on its own. She has soaked the foot at night in warm soapy water and has applied
topical antibiotics and sterile adhesive bandages to the area. She continues to carry out her daily
activities. The patient states that outside of the lesion's unpleasant appearance, "it really has not
bothered me at all." Her temperature is 37.0°C (98.6°F). There is a large, deep ulcerated area
between the third and fourth toe webs of the right foot as shown in the photograph above. It is
moist and foul smelling, and there is erythema of the surrounding digits and foot. There is no
tracking or lymphadenitis present. Which of the following findings would most likely be present
on neurological examination of her lower extremities? 
 Correct Answer ImageA.Bilateral diminished ankle jerk (Achilles) reflex
 Incorrect Answer ImageB.Bilateral increased knee jerk reflex
 Incorrect Answer ImageC.Bilateral increased perception of vibration in the lower
extremities
 Incorrect Answer ImageD.Unilateral decreased perception of pain in the right lower
extremity
 Incorrect Answer ImageE.Unilateral motor weakness on right leg raise

The following vignette applies to the next two items. 


A 35-year-old woman, gravida 3, para 2, at 40 weeks' gestation, comes to the emergency
department in labor. She has had no prenatal care. Over the past year she has been feeling tired
and unable to do the things she used to do. She has noted a white material in her mouth at times,
and was admitted to the hospital on one occasion with severe pneumonia 10 months ago. While
she was in the hospital she was diagnosed with HIV infection, but she has never received
antiretroviral therapy. Since then, she has had no recurrent illnesses. Which of the following is
considered appropriate intrapartum care? 
 Incorrect Answer ImageA.Intravenous zidovudine, nevirapine, and ritonavir
 Incorrect Answer ImageB.Oral nevirapine and zidovudine
 Incorrect Answer ImageC.Oral nelfinavir, zidovudine, lamivudine
 Incorrect Answer ImageD.No specific HIV care
 Correct Answer ImageE.Intravenous zidovudine alone

A 35-year-old woman, gravida 3, para 2, at 40 weeks' gestation, comes to the emergency


department in labor. She has had no prenatal care. Over the past year she has been feeling tired
and unable to do the things she used to do. She has noted a white material in her mouth at times,
and was admitted to the hospital on one occasion with severe pneumonia 10 months ago. While
she was in the hospital she was diagnosed with HIV infection, but she has never received
antiretroviral therapy. Since then, she has had no recurrent illnesses. Which of the following is
the most appropriate management of the newborn? 
 Incorrect Answer ImageA.No specific HIV care
 Incorrect Answer ImageB.No therapy until HIV status is determined
 Incorrect Answer ImageC.No therapy until resistance pattern of HIV is established in the
mother
 Incorrect Answer ImageD.Oral zidovudine for 6 months
 Correct Answer ImageE.Oral zidovudine for 6 weeks

A malnourished middle-aged homeless man is brought to the emergency department. He is


disoriented to person, place, and time and unable to walk without assistance. His temperature is
37.0°C (98.4°F), blood pressure is 134/80 mm Hg, pulse is 86/min, and respiratory respirations
are 18/min. Neurologic examination shows lateral nystagmus. Evaluation of strength and
sensation can not be performed. Which of the following is the most appropriate next step in
management? 
 Incorrect Answer ImageA.Administration of diazepam
 Incorrect Answer ImageB.Intravenous infusion of glucose
 Correct Answer ImageC.Intravenous infusion of thiamine
 Incorrect Answer ImageD.Neuroimaging studies
 Incorrect Answer ImageE.Toxicological screening

A 1-year-old girl is brought to the emergency room because of refusal to use her right arm and
crying when anyone else attempts to move her right arm. This began after she was lifted off the
floor by her arms while dancing with her mother. On physical examination, the patient is holding
the arm in a partially flexed position, close to her body, and cries with any manipulation. Distal
pulses are intact, and the child moves all digits. No deformities, swelling, or bony tenderness are
noted. Cardiovascular and respiratory examinations are within normal limits. Which of the
following is the most likely diagnosis?
 Incorrect Answer ImageA.Fractured displacement of the radial epiphysis
 Correct Answer ImageB.Radial head subluxation
 Incorrect Answer ImageC.Radial nerve injury
 Incorrect Answer ImageD.Shoulder separation
 Incorrect Answer ImageE.Supracondylar fracture of humerus

A 16-year-old boy is brought to the emergency department by paramedics. He is accompanied by


two friends who state that he had a seizure about an hour and a half earlier. They were having a
"jam session" in the garage at the home of a friend whose parents were away for the weekend. It
was very cold so they lit up an old stove they found in the basement. A few of the others
complained of headache and nausea, but since they were all smoking heavily and having a few
drinks, they did not think much of it. Once the patient began vomiting and went into a seizure,
they realized something was actually wrong. The two friends who brought him in said they also
felt headaches and became "sort of jittery", but that mostly resolved by the time they reached the
hospital. The paramedics report that the patient was unconscious when they arrived but came to
after they gave him oxygen. On physical examination, the patient is awake but feels weak and is
confused. He constantly feels nauseous and has uncoordinated movements. His pulse is 110/min
and respirations are 32/min. Examination of the ocular fundus shows retinal hemorrhage. Which
of the following is most likely to be present in the patient's urine? 
 Incorrect Answer ImageA.Glucose
 Incorrect Answer ImageB.Hemoglobin
 Incorrect Answer ImageC.Maltese crosses
 Correct Answer ImageD.Myoglobin
 Incorrect Answer ImageE.Red blood cell casts

A 61-year-old stockbroker is brought by his family to the psychiatric emergency department. He


has lost 15 pounds, has not slept well in three weeks, and his memory is poor. He is experiencing
problems with naming objects, tying his shoelaces, and balancing his checkbook. At times,
especially in the afternoon, he feels better. At night, he feels depressed and his memory worsens.
He is unable to go to work and fears his family is angry with him for being "lazy." His energy is
low, he is hopeless, and he finds little pleasure in activities. He reports that he lost a lot of money
in the stock market 3 weeks ago. Which of the following is the most likely diagnosis? 
 Incorrect Answer ImageA.Adjustment disorder with depressed mood
 Incorrect Answer ImageB.Dementia, Alzheimer type
 Incorrect Answer ImageC.Dissociative amnesia
 Incorrect Answer ImageD.Hypothyroidism
 Correct Answer ImageE.Major depressive disorder

A 26-year-old woman, gravida 3, para 3, has just delivered her third male child. During the
pregnancy, she had several urinary tract infections, all successfully treated with ampicillin.
Delivery was by cesarean section due to arrest of the active phase of labor. On day 1,
examination of the newborn shows a hypotonic, weak baby with absent stretch reflexes. The
baby has a weak cry and appears to have increased work of breathing. Further examination
shows fasciculations of the tongue but preservation of the extraocular muscles. Laboratory
studies show:
Hemoglobin 14.2 g/dL
Hematocrit 43.00%
Leukocyte 4,200/mm3
Sodium 140 mEq/L
Potassium 3.9 mEq/L
Chloride 100 mEq/L
Bicarbonate 25 mEq/L
BUN 16 mg/dL
Creatinine 1.2 mg/dL
Glucose 108 mg/dL
Which of the following is the most likely diagnosis? 
 Incorrect Answer ImageA.Becker muscular dystrophy
 Incorrect Answer ImageB.Infantile myasthenia gravis
 Incorrect Answer ImageC.Neonatal sepsis
 Incorrect Answer ImageD.Tay-Sachs disease
 Correct Answer ImageE.Werdnig Hoffman

A 27-year-old man comes to the physician because of scrotal discomfort. He states that for the
past 3 weeks he has been feeling a vague and heavy sensation on the right side of his scrotum.
He denies any dysuria, urethral discharge, testicular trauma, prior testicular surgery, fevers, or
chills. Physical examination shows that the right testicle is enlarged, and there is an irregular,
non-tender mass that seems to be arising from and obliterating the normal testicular architecture.
The mass is not reducible, does not transilluminate, and does not change with the Valsalva
maneuver. The spermatic cord can be palpated superior to the mass and is normal. Urinalysis is
unremarkable. Which of the following is the most appropriate management for this patient's
condition?
 Incorrect Answer ImageA.A 14-day course of antibiotics
 Incorrect Answer ImageB.Ligation of dilated pampiniform plexus
 Correct Answer ImageC.Radical orchiectomy
 Incorrect Answer ImageD.Removal of fluid surrounding the testicle
 Incorrect Answer ImageE.Repair of defect in the floor of the inguinal canal
 Incorrect Answer ImageF.Ultrasound-guided biopsy of the mass

A 32-year-old woman was diagnosed with schizophrenia 6 years ago. She has been hospitalized
multiple times for agitation and delusional thinking despite maintenance therapy with
antipsychotic medication. Over the past year, she has shown markedly decreased social
functioning and a flattened affect. A trial of clozapine is initiated. For which of the following
side effects must the patient be monitored routinely?
 Correct Answer ImageA.Agranulocytosis
 Incorrect Answer ImageB.Diabetes insipidus
 Incorrect Answer ImageC.Rebound hypertension
 Incorrect Answer ImageD.Rhabdomyolysis
 Incorrect Answer ImageE.Torsades de pointes

A 21-year-old college senior has a 3-day history of a painless ulcer on his penis. He says that he
was at a party about 3 weeks ago and had unprotected sexual intercourse. He has been in his
normal state of health since then. He denies any similar previous lesions and says he never saw
blisters in the area. The ulcer has been stable since it erupted. On examination, there is a single, 5
mm, nontender ulcer on the distal penile shaft. The ulcer has a clean base and firm, indurated
borders. Bilateral inguinal adenopathy is present. Which of the following studies will most likely
establish a diagnosis? 
 Incorrect Answer ImageA.Bacterial culture
 Incorrect Answer ImageB.Biopsy
 Correct Answer ImageC.Dark-field microscopy
 Incorrect Answer ImageD.Rapid plasma reagin (RPR)
 Incorrect Answer ImageE.Tzanck smear

A 48-year-old Hispanic man comes to the clinic asking about prostate cancer protection. He has
heard that finasteride can prevent prostate cancer and wants to know if he should take the
medication. He has heard that this medication can reduce the risk for prostate cancer by 25% in
men aged 55 years and older. In general he has been healthy and "just wishes to stay that way."
Although his diet is low fiber and high protein, he does exercise regularly and does not drink
alcohol or smoke cigarettes. He does report a distant episode of prostatitis when he was in the
Marines, but otherwise has had no genitourinary problems. Surgical history is remarkable for an
appendectomy as a child and an elective vasectomy 10 years ago. There is no family history of
prostate cancer that he is aware of, though his mother, sister, and aunt died of breast cancer at a
young age. Physical examination, including a prostate examination, is unremarkable. In deciding
whether preventive treatment is appropriate, you review the risk factors for prostate cancer with
this patient. Which of the following is the most significant risk factor for prostate cancer in this
patient? 
 Incorrect Answer ImageA.Diet and nutrition
 Incorrect Answer ImageB.Ethnicity
 Correct Answer ImageC.Family medical history
 Incorrect Answer ImageD.History of prostatitis
 Incorrect Answer ImageE.Surgical history

A 62-year-old obese man comes to the clinic because of anxiety, difficulty sleeping, and
episodes of pressured speech and racing thoughts. Additionally, he has noticed swelling in his
legs, something that has never occurred in the past. He states that his problems began a few
weeks ago when he went to the emergency room for treatment of a painful knee and toe.
Aspiration of an inflamed joint showed an aspirate containing needle-shaped, negatively
birefringent crystals. The patient was started on an oral medication for his presumed diagnosis,
after which his current symptoms began. His past medical history, aside from this recurrent
arthritis, includes hypertension, alcoholism, and chronic renal insufficiency, for which an
arteriovenous graft was recently created in anticipation of dialysis. Given this patient's history
and complaints of drug side effects, which of the following is the most likely treatment that was
given to this patient? 
 Incorrect Answer ImageA.Allopurinol
 Incorrect Answer ImageB.Colchicine
 Incorrect Answer ImageC.Indomethacin
 Correct Answer ImageD.Prednisone
 Incorrect Answer ImageE.Probenecid

A 41-year-old diabetic woman comes to the physician for a routine healthcare examination. She
is concerned about her elevated home blood glucose levels, particularly in the morning before
breakfast. Readings after meals and at bedtime range from 140–160 mg/dL. She has also checked
her glucose levels at 3 am for the past 2 weeks, and the readings did not show any episodes of
hypoglycemia. Her current insulin regimen includes a twice-daily injection of NPH and regular
insulin. She administers the injection before breakfast and before dinner. Which of the following
is the most appropriate modification in her current insulin regimen to treat her early morning
hyperglycemia?
 Correct Answer ImageA.Increase the evening dose of NPH insulin
 Incorrect Answer ImageB.Increase the evening dose of regular insulin
 Incorrect Answer ImageC.Increase the morning dose of NPH insulin
 Incorrect Answer ImageD.Increase the morning dose of regular insulin
 Incorrect Answer ImageE.Increase the morning dose, reduce the evening dose of
combined insulin

A 54-year-old woman had a successful renal transplant from a related donor several years ago.
She comes to her physician for follow-up because her renal function has steadily been
deteriorating. A biopsy of the graft is read as "chronic rejection," with endothelial proliferation
and fibrosis of the graft. The patient has been on a standard immunosuppressive regimen,
consisting of azathioprine and low-dose cyclosporine. Which of the following is the most
appropriate treatment for this patient?
 Incorrect Answer ImageA.Antilymphocyte globulin
 Incorrect Answer ImageB.High doses of cyclosporine
 Correct Answer ImageC.A new transplant
 Incorrect Answer ImageD.Steroid boluses
 Incorrect Answer ImageE.Tacrolimus

A 4-year old girl is brought to the physician by her parents for evaluation of a skin condition
present since infancy. Her mother states that the rash waxes and wanes, especially with the
seasons, and is more prominent during the winter months. It is very itchy and often wakes the
child up in the middle of the night. They have been using corticosteroid creams, lotions, and
ointments of various strengths for the past 3 years with no lasting effect. The mother is
concerned that they may be harming the skin. The child's past medical history is remarkable for
mild asthma and hay fever. Current medication includes cetirizine and topical fluocinonide
ointment as needed. The family history is also significant for asthma. On physical examination,
the patient is in no acute distress. She is in the 50th percentile for height and weight and her vital
signs are within normal limits. Inspection of the skin is remarkable for extensive lichenified,
crusty and scaly erythematous plaques on the neck antecubital fossa and popliteal regions. Her
eyelids are edematous and erythematous with prominent Dennie-Morgan folds. The skin of the
cheeks appears to be thin, translucent, and with multiple telangiectasias. On the arms, there are
also areas of thin, translucent skin with telangiectasias. You explain to the parents that the
atrophy and telangiectasias are the result of frequent topical steroid application and that they
should hold off further corticosteroid use. Which of the following topical medications may be
safely and effectively used for the treatment of this child's skin disease in view of the above
clinical findings?
 Incorrect Answer ImageA.Adapelene cream
 Incorrect Answer ImageB.Diflorasone diacetate ointment
 Incorrect Answer ImageC.Penciclovir cream
 Correct Answer ImageD.Tacrolimus ointment
 Incorrect Answer ImageE.Tretinoin cream

A 31-year-old woman, gravida 1, para 1, is 2 days postpartum following a vaginal delivery. Her
pregnancy was complicated by gestational diabetes that was treated with insulin. The patient also
had group B Streptococcus cystitis in the first trimester. This was treated with ampicillin at the
time of diagnosis and the patient received penicillin prophylaxis during labor and delivery. The
patient also was found to be hepatitis B surface antigen-positive during her prenatal course, and
the newborn received HBIG and hepatitis B vaccine within 12 hours of birth. She was diagnosed
with a urinary tract infection on postpartum day 1 and started on levofloxacin. With all of the
complications that she has had during the pregnancy, she wants to know if she can breastfeed her
infant. Which of the following is true regarding her ability to breastfeed?
 Incorrect Answer ImageA.Gestational diabetes does preclude breastfeeding
 Incorrect Answer ImageB.Group B Streptococcus cystitis does preclude breastfeeding
 Correct Answer ImageC.Hepatitis B infection does not preclude breastfeeding
 Incorrect Answer ImageD.Levofloxacin use does not preclude breastfeeding
 Incorrect Answer ImageE.There are no contraindications to breastfeeding

A 39-year-old woman at 12 weeks' gestation comes to the physician for a prenatal visit. She has
a history of rheumatoid arthritis, diagnosed 4 years ago. She has had a few episodes of worsening
of the disease since that time. Over the past year, however, she has been doing better. She now
complains of mild nausea and vomiting, but is otherwise doing well with no joint complaints.
She has no surgical history and no allergies. Physical examination is consistent with a patient at
12 weeks' gestation. Which of the following is the most appropriate next step in management?
 Incorrect Answer ImageA.Azathioprine
 Correct Answer ImageB.Expectant management
 Incorrect Answer ImageC.Gold compounds
 Incorrect Answer ImageD.Ibuprofen
 Incorrect Answer ImageE.Prednisone

A 4-year-old girl is brought to the physician by her mother because of skin lesions on the face
and arms. She recently noticed them, first around the eyes and on the sclera, and then on the
cheeks and in the elbow crease in increasing numbers. They do not appear to bother the patient.
The child's medical history is significant for a gait disorder that began when she started to walk
and progressively worsened, and for recurrent sinopulmonary and skin infections. She now has
an awkward, swaying gait with choreic and athetoid movements. On examination, there are
multiple nonblanching, dilated blood vessels present on the bulbar conjunctivas, on the nose and
cheeks, and in the antecubital fossae. The child is drooling and has erythematous, scaly patches
with thick purulent crusts on the corners of the mouth. Which of the following is the most likely
diagnosis? 
 Correct Answer ImageA.Ataxia-telangiectasia
 Incorrect Answer ImageB.Chronic granulomatous disease
 Incorrect Answer ImageC.IgA deficiency
 Incorrect Answer ImageD.Sex-linked agammaglobulinemia
 Incorrect Answer ImageE.Thymic hypoplasia

A 72-year-old man comes to the physician because of a 3-day history of right-sided chest pain.
He denies any shortness of breath, nausea, or vomiting. Physical examination shows a unilateral,
erythematous, maculopapular rash extending from the anterior chest wall around to the back in a
dermatomal pattern. The remainder of the physical examination is normal. In conversation, he
states that he is going to visit his grandchildren next week and that their mother "doesn't believe
in immunizations". His grandchildren are at increased risk from which of the following rashes?
 Incorrect Answer ImageA.Discrete maculopapular lesions that become confluent as they
spread from "head to toe"
 Incorrect Answer ImageB.Dome-shaped papules with central umbilication
 Incorrect Answer ImageC.Expanding annular lesion with central clearing
 Incorrect Answer ImageD."Slapped-cheek" appearance and a lacy reticular rash
 Correct Answer ImageE.Vesicles at various stages of evolution

A 5-year-old boy is brought to an emergency room because of a painful, swollen knee joint. The
boy had fallen while playing, and the joint had subsequently begun to swell. The mother reports
that the boy was known to have hemophilia B. Replacement of which of the following is
indicated? 
 Incorrect Answer ImageA.Factor C
 Incorrect Answer ImageB.Factor S
 Incorrect Answer ImageC.Factor VII
 Incorrect Answer ImageD.Factor VIII
 Correct Answer ImageE.Factor IX

You might also like